You are on page 1of 1066

QUICK REVIEW SERIES for

BDS 4th Year


Vol 1
SECOND EDITION
This page intentionally left blank
QUICK REVIEW SERIES for

BDS 4th Year


Vol 1
SECOND EDITION

J Jyotsna Rao
bds, mds, pgcoi (mahe), f isoi
Director, SRS Dental Exams Academy, Bengaluru
Ex-Professor, Department of Oral and Maxillofacial Surgery
The Oxford Dental College, Hospital and Research Centre
Bengaluru, INDIA
RELX India Pvt. Ltd.
Registered Office: 818, 8th Floor, Indraprakash Building, 21, Barakhamba Road, New Delhi 110001
Corporate Office: 14th Floor, Building No. 10B, DLF Cyber City, Phase II, Gurgaon-122 002, Haryana, India

Quick Review Series for BDS 4th Year, Volume 1, Rao J Jyotsna

Copyright © 2017 by RELX India Pvt. Ltd. (formerly known as Reed Elsevier India Ltd)
Copyright © 2015 by Reed Elsevier India Pvt. Ltd.
All rights reserved.

ISBN: 978-81-312-4879-9
e-Book ISBN: 978-81-312-4934-5

No part of this publication may be reproduced or transmitted in any form or by any means, electronic or me-
chanical, including photocopying, recording, or any information storage and retrieval system, without permis-
sion in writing from the publisher. Details on how to seek permission, further information about the Publisher’s
permissions policies and our arrangements with organizations such as the Copyright Clearance Center and the
Copyright Licensing Agency, can be found at our website: www.elsevier.com/permissions.

This book and the individual contributions contained in it are protected under copyright by the Publisher (other
than as may be noted herein).

Notice

Practitioners and researchers must always rely on their own experience and knowledge in evaluating and using
any information, methods, compounds or experiments described herein.  Because of rapid advances in the
medical sciences, in particular, independent verification of diagnoses and drug dosages should be made.  To
the fullest extent of the law, no responsibility is assumed by Elsevier, authors, editors or contributors in relation
to the adaptation or for any injury and/or damage to persons or property as a matter of products liability, neg-
ligence or otherwise, or from any use or operation of any methods, products, instructions, or ideas contained
in the material herein.

Manager, Content Strategy: Nimisha Goswami


Sr Project Manager—Education Solutions: Shabina Nasim
Managing Editor: Anand K Jha
Project Manager: Nayagi Athmanathan
Sr Production Executive: Ravinder Sharma
Sr Cover Designer: Milind Majgaonkar

Laser typeset by GW India


Printed in India by
Dedicated
to
SRI GURU RAGHAVENDRA SWAMI
(The eternal power who has showered his blessings on me to successfully complete the entire range
of Quick Review Series for BDS)
This page intentionally left blank
Foreword

Orthodontics
I am humbled to write the Foreword for the book entitled Quick Review Series for
BDS 4th Year: Orthodontics. It pleases me greatly to introduce all to the author,
Dr J Jyotsna Rao.
The author needs to be complimented on making a special effort to address the ex-
amination needs of the undergraduates, who I hope will welcome this publication.
With integrity, skill and passion this comprehensive orthodontic Quick Review book
covers the depth and the breadth of orthodontics admirably.
I do hope that students who study this book seriously will get a good grasp of the
subject matter which will enable them to have a very successful academic career.
You can prepare yourself for an exciting opportunity to learn, improve your abilities
and better performance in the examinations. This book QRS orthodontics for final BDS is eminently readable.
It is a treasure and will serve as a beacon of light to brightly illuminate the endless pathway towards your academic
excellence. This book has the potential to train the students and prepare them for the formidable challenge to face the
examinations.
I whole heartedly recommend this book to the students of final BDS who desire to perform their best in their aca-
demic life.
I congratulate the young author Dr J Jyotsna Rao for this venture and wish all the readers of this book all success.

Dr CP Rao mds
Ex-principal and Professor of Orthodontics, Government Dental College, Hyderabad
Ex-principal and Professor of Orthodontics, HKDET’s Dental College, Humnabad
Ex-professor of Orthodontics, Army College of Dental Sciences, Secunderabad
President, Indian Orthodontic Society (1975)
President, Indian Dental Association, AP State Branch (1985 & 1995)
Member, High Power Committee for Recommending Essentiality Certificate for
Establishment of Medical and Dental Colleges in the State of Andhra Pradesh

Paedodontics
I am extremely happy to pen a few words about this conscientiously written book. It is a common knowledge that books
play a major complementary and contributing role in any educational process, where they are envisioned to facilitate self-
learning beyond classroom exercises.
This book of Quick Review Series for BDS IV: Paedodontics, authored by Dr J Jyotsna Rao, is presented with such a
systemic approach that it demonstrates her consummate skill in preparing students for examinations. It is good to see that
she has shared her vast experience in academics with the students through this book.
While going through the pages of this book I found that the author has made a sincere attempt to present the subject of
paedodontics as per the syllabus of Dental Council of India to fulfil the long-term need of a concise quick review book
with best standards, simple language and required depth of explanation of the subject through questions and answers of
various university examinations.

vii
viii Foreword

Designing such a book is a challenging task, especially if it has to be concise and comprehensive in scope. Such a ver-
sion demands wise sifting, prudent pruning and meaningful condensing of the enormous and variegated knowledge base
of the subject.
This outstanding resource is perfect for those studying in BDS IV year. The easy-to-understand text material serves as
both preparatory tool at the start of study course providing road map of the subject to be learnt and at the course end help-
ing rapid review and recapitulation of what has been learnt.
I am confident that this book is undeniably appropriate for exam-going undergraduate students craving for a thorough
review of subjects in a short period of time.

Regards

Dr Murali Mohan
Principal, Professor and Head of the Department
Department of Conservative Dentistry and Endodontics
Government Dental College and Hospital
Vijayawada, Andhra Pradesh

Community Dentistry
I am extremely happy to pen a few words about this conscientiously written book. It is common knowledge that books
play a major complementary and contributing role in any educational process, where they are envisioned to facilitate
self-learning beyond classroom exercises.
This book of Quick Review Series for BDS 4th Year: Community Dentistry authored by Dr J JYOTSNA RAO is pre-
sented with such a systemic approach that it demonstrates her consummate skill in preparing students for examinations.
It is good to see that she has shared her vast experience in academics with the students through this book.
When going through the pages of this book, I found that the author has made sincere attempt to present the subject of
Preventive and Community Dentistry as per the syllabus of DCI to fulfill the long-term need of a concise quick review
book with best standards, simple language and required depth of explanation of the subject, through question and answers
of various university examinations.
Designing such a book is a challenging task, especially if it is to be concise and comprehensive in scope. Such a version
demands wise sifting, prudent pruning and meaningful condensing of the enormous and variegated knowledge base of
4th year BDS subjects.
This outstanding resource is perfect for those studying in final year BDS. The easy to understand text material, serves
as both preparatory tool at the start of study course providing road map of the subject to be learnt and at the end of the
course it helps in rapid review and recapitulation of what is learnt.
I am confident that this book is undeniably appropriate for exam-going UG students who are craving for thorough
review of subjects in a short period.

Regards
Dr Murali Mohan
Principal, Professor and Head
Department of Conservative Dentistry and Endodontics
Govt Dental College and Hospital
Vijayawada (A P)
Foreword ix

Periodontics
Examinations could be difficult and stressful even for the well-prepared. More so in a professional course with 8 volu-
minous subjects such as the final BDS, can make the most diligent and conscientious student nervous and all at sea. It is
not uncommon for students to develop a sudden feeling of emptiness as if they have not studied anything at all. The very
sight of a big textbook makes them all the more apprehensive. At these times a ready-reckoner which provides all the
essentials in a nutshell being precise and concise would be a big help. This is exactly what Dr Jyotsna Rao in her latest
book on Periodontics has endeavoured to do. It is not easy for a subject like Periodontics whose ambit is wide which
includes histology, pathology, microbiology, immunology, various disease conditions along with treatment applications
comprising of both non-surgical and surgical avenues to be condensed into a simple and comprehensible text. However,
Dr Jyotsna Rao who has vast experience in preparing MDS aspirants for various entrance examinations has been very
successful in this regard. This book based upon the questions which appeared in the past 20 years, in the University ex-
aminations have been answered in a simple and logical manner. Various topics have been listed in an orderly way without
an overlap. This makes it easy for the students to rapidly review the entire subject and also recollect whatever they had
studied during the final BDS.
Overall, Dr Jyotsna’s effort is commendable and this book should be useful not only for the students appearing for final
BDS examination but also for those preparing for MDS entrance examination.

Dr C D Dwarakanath
Professor and Head
Department of Periodontics
Director, Postgraduate Studies
Vishnu Dental College
Bhimavaram
This page intentionally left blank
Preface

I am overwhelmed by the positive response received from the students all over the country to the first edition of this book.
When the first edition of this book was published, it was released as eight subject wise individual books. In order to further
make it more convenient and comfortable for the students to prepare for exams, this second edition is made with tremen-
dous changes combining all eight final BDS subjects into two volumes with four subjects in each volume.
This volume of the book contains four subjects, i.e. Orthodontics, Paedodontics, Community Dentistry and Periodon-
tics. All along with upgraded change, I have taken care so that the basic format is maintained, which is previously so well
received.
I have restructured the contents of this book in such a manner that students will be benefitted by using comprehensive
and relevant information given in the book.
It is planned in a meticulous manner and I have endeavoured comprehensively to refer and include relevant information
from the standard textbooks. Though written in a question- and- answer format, this book is arranged in a logical sequence
for the purpose of better recapitulation.
Unlike previous edition here Short Essay and Short Note questions and answers are marked within Long Essays wher-
ever it was possible, so that students will have double advantage in preparing for exams by conserving time and energy.
This makes it easy for the students to quickly review the entire subject and also recollect whatever they had studied
during the academic year of final BDS.
This book is primarily intended for undergraduate students, but can also be used as a quick reference book by post-
graduate students to recollect the subject.
I hope this book will make an important contribution to the students in understanding the subject and excelling in their
examinations.
J Jyotsna Rao
drjjrao@gmail.com

xi
This page intentionally left blank
Acknowledgements

First of all I thank almighty for his blessings without which this work would not have been possible.
I would like to first thank my father Mr J Sudharshan Rao who is the key person behind all my successful endeavours.
I am thankful to my mother Mrs S Sujatha Laxmi for her unforgettable sacrifices and choicest blessings. My warmest
regards to my husband Mr K Vinayak Rao for his constant support to enhance my software skills.
My thanks and love to my son Master K Raghasai without whose co-operation this work would not have been possible.
I am thankful to my brother Mr J Jayakrishna for his valuable constructive suggestions.
My sincere thanks is due to Dr CP Rao, Dr Murali Mohan, and Dr CD Dwarakanath for writing Foreword for this book.
My sincere thanks is to Dr P Balreddy Principal, Professor and Head, Department of Oral and Maxillofacial Surgery,
Government Dental College and Hospital, Hyderabad, for his blessings. I wish to thank Dr BK Reddy, Ex-Principal,
Government Dental College , Hyderabad and Meghana Dental College, Nizamabad for his blessings and advice.
My sincere thanks to Dr Bhaskar Y, Dr P Chidambar, Dr Laxmikanth, and Mr Kiran (Librarian, Oxford Dental College,
Bangalore) and Narayana Swami for their invaluable support in collecting previous years’ question papers from various
universities.
I would like to specially thank Dr Parmar Adithi Kiritikumar and Dr Priyanka Das, Dr Saniyara Khanam and
Dr Mardidiam Lanong for their valuable contribution in preparing manuscript. I would like to extend my regards to
Dr Rajini and P Nethravathi for their help in correction of manuscripts.
Thanks to Elsevier India, especially Dr Lalit Singh, Mrs Nimisha Goswami, Mr Anand K Jha and all other team mem-
bers for their active contribution in publishing this book.
I would like to take this opportunity to thank all those people who, directly or indirectly were instrumental in success-
fully bringing out this book.
Last but not the least, I acknowledge all my friends and colleagues for their best wishes to boost my morale.

xiii
This page intentionally left blank
Contents

Foreword vii Topic 26 Management of Common 


Preface xi
Malocclusions 224
Acknowledgements xiii
Topic 27 Management of Class II Malocclusion 231
Part I Orthodontics Topic 28 Management of Class III Malocclusion 236
Topic 1 Introduction to Orthodontics 3 Topic 29 Management of Open Bite, Crossbite 
Topic 2 General Principles and Concepts  and Deep Bite 240
of Growth 7 Topic 30 Cleft Lip and Palate 250
Topic 3 Growth and Development of Cranial  Topic 31 Surgical Orthodontics 257
and Facial Structures 21 Topic 32 Retention and Relapse 262
Topic 4 Development of Dentition and  Topic 33 Genetics in Orthodontics 269
Occlusion 31 Topic 34 Lab Procedures 272
Topic 5 Functional Development 41 Topic 35 Materials Used in Orthodontics 275
Topic 6 Occlusion – Basic Concepts 45
Topic 7 Classification of Malocclusion 48 Part II Paedodontics
Topic 8 Aetiology of Malocclusion 58 Topic 1 Introduction to Paedodontics 283
Topic 9 Oral Habits 68 Topic 2 Examination, Diagnosis and 
Topic 10 Orthodontic Diagnosis 79 Radiographic Techniques 288
Topic 11 Cephalometrics 97 Topic 3 Theories of Child Development 300
Topic 12 Skeletal Maturity Indicators 115 Topic 4 Parent Counselling and Child 
Topic 13 Model Analysis 119 Behaviour 308
Topic 14 Biology and Mechanics of Tooth  Topic 5 Behavioural Science and Psychologic 
Movement 128 Management of Children’s Behaviour 318
Topic 15 Anchorage 138 Topic 6 Therapeutic Management 338
Topic 16 Age Factors in Orthodontics 145 Topic 7 Management of Handicapped Children347
Topic 17 Preventive Orthodontics 147 Topic 8 Management of Children with 
Topic 18 Interceptive Orthodontics 155 Systemic Diseases and HIV Infection 353
Topic 19 Methods of Space Gaining 161 Topic 9 Management of Children with 
Topic 20 Arch Expansion 166 Cleft Lip and Palate 362
Topic 21 Extractions 172 Topic 10 Growth and Development of the 
Topic 22 Orthodontic Appliances – General  Face and Dental Arches 364
Principles 176 Topic 11 Development and Morphology 
Topic 23 Removable Orthodontic Appliances 178 of Primary Teeth and Occlusion 373
Topic 24 Fixed Orthodontic Appliances 194 Topic 12 Acquired and Developmental 
Topic 25 Myofunctional and Orthopaedic  Disturbances of the Teeth and 
Appliances 205 Associated Oral Structures 391

xv
xvi Contents

Topic 13 Developing Malocclusion and  Topic 5 Epidemiology of Gingival and 


Its Management and Preventive  Periodontal Diseases 772
Measures 403 Topic 6 Periodontal Microbiology 780
Topic 14 Oral Habits 427 Topic 7 Dental Calculus, Iatrogenic and 
Topic 15 Gingival and Periodontal Diseases  Other Local Predisposing Aetiological 
in Children 438 Factors 789
Topic 16 Home Oral Hygiene for Children  Topic 8 Smoking and Periodontium 794
and Adolescents 447 Topic 9 Host Response: Basic Concepts 795
Topic 17 Dental Caries in Children and  Topic 10 Host–Microbial Interactions in 
Adolescents 458 Periodontal Diseases 800
Topic 18 Pit and Fissure Sealants 483 Topic 11 Trauma from Occlusion 802
Topic 19 Atraumatic Restorative Treatment 489 Topic 12 Influence of Systemic Diseases on the 
Topic 20 Fluorides and Oral Habits 492 Periodontium and Periodontal 
Topic 21 Paediatric Restorative Materials  Medicine 813
and Rubber Dam Application 507 Topic 13 Dental Implants 822
Topic 22 Restoration of Primary Carious Teeth 523 Topic 14 Defence Mechanisms of the Gingiva 826
Topic 23 Paediatric Endodontics 533 Topic 15 Gingival Inflammation and Clinical 
Topic 24 Traumatic Injuries of Anterior Teeth  Features of Gingivitis 834
and Management 547 Topic 16 Gingival Enlargements 843
Topic 25 Local Anaesthesia and Oral Surgery  Topic 17 Acute Gingival Infections 856
for the Child Patient 562 Topic 18 Desquamative Gingivitis 864
Topic 26 NSAIDs, Antimicrobial Drugs and  Topic 19 Gingival and Periodontal Diseases in 
Miscellaneous 568 Children and Young Adolescents 868
Topic 20 Periodontal Pocket 873
Part III Community Dentistry
Topic 21 Bone Loss in Periodontal Diseases 884
Topic 1 Introduction 575
Topic 22 Periodontitis: Chronic, Refractory 
Topic 2 Public Health 584
and Necrotizing Ulcerative 892
Topic 3 Epidemiology 603
Topic 23 Aggressive Periodontitis 897
Topic 4 Dental Public Health 628
Topic 24 Periodontal Abscess 904
Topic 5 Preventive Dentistry 649
Topic 25 Halitosis 908
Topic 6 Fluorides 671
Topic 26 Clinical Diagnosis and Advanced 
Topic 7 Indices Used in Preventive Dentistry 685
Diagnostic Methods 911
Topic 8 Health Statistics 710
Topic 27 Determination of Prognosis 926
Topic 9 Social Sciences 723
Topic 28 Periodontal Treatment Plan 931
Part IV Periodontics Topic 29 Periodontal Instrumentation 934
Topic 1 Gingiva 745 Topic 30 Principles of Periodontal 
Topic 2 Tooth-Supporting Structures  Instrumentation 944
(Periodontal Ligament, Alveolar  Topic 31 Sonic and Ultrasonic Instrumentation 950
Bone, Cementum) 757 Topic 32 General Principles and Concepts 
Topic 3 Age-Related Changes in the  of Growth 953
Periodontium 766 Topic 33 Plaque Control 956
Topic 4 Classification of Diseases of the  Topic 34 Chemotherapeutic Agents 973
Periodontium 769 Topic 35 Periodontal Splints 981
Contents xvii

Topic 36 General Principles of Periodontal  Topic 42 Endodontic Periodontal Lesions 


Surgery 984 and Their Management 1033
Topic 37 Gingival Surgical Procedures 997 Topic 43 Orthodontic Periodontal Inter-
Topic 38 Periodontal Flap Surgery 1006 relationship 1038
Topic 39 Resective Osseous Surgery 1010 Topic 44 Perioprosthodontics/Occlusal 
Topic 40 Regenerative Osseous Surgery 1015 Evaluation 1040
Topic 41 Furcation Involvement and Its  Topic 45 Supportive Periodontal Treatment 
Management 1028 (Maintenance Phase) 1044
This page intentionally left blank
Section I

Topic-Wise Solved Questions


of Previous Years

PART I ORTHODONTICS
Topic 1 Introduction to Orthodontics 3
Topic 2 General Principles and Concepts of Growth 7
Topic 3 Growth and Development of Cranial and Facial Structures 21
Topic 4 Development of Dentition and Occlusion 31
Topic 5 Functional Development 41
Topic 6 Occlusion: Basic Concepts 45
Topic 7 Classification of Malocclusion 48
Topic 8 Aetiology of Malocclusion 58
Topic 9 Oral Habits 68
Topic 10 Orthodontic Diagnosis 79
Topic 11 Cephalometrics 97
Topic 12 Skeletal Maturity Indicators 115
Topic 13 Model Analysis 119
Topic 14 Biology and Mechanics of Tooth Movement 128
Topic 15 Anchorage 138
Topic 16 Age Factors in Orthodontics 145
Topic 17 Preventive Orthodontics 147
Topic 18 Interceptive Orthodontics 155
Topic 19 Methods of Space Gaining 161
Topic 20 Arch Expansion 166
Topic 21 Extractions 172
Topic 22 Orthodontic Appliances: General Principles 176
Topic 23 Removable Orthodontic Appliances 178
Topic 24 Fixed Orthodontic Appliances 194
Topic 25 Myofunctional and Orthopaedic Appliances 205
Topic 26 Management of Common Malocclusions 224
Topic 27 Management of Class II Malocclusion 231
Topic 28 Management of Class III Malocclusion 236
Topic 29 Management of Open Bite, Crossbite and Deep Bite 240
Topic 30 Cleft Lip and Palate 250
Topic 31 Surgical Orthodontics 257
Topic 32 Retention and Relapse 262
Topic 33 Genetics in Orthodontics 269
Topic 34 Lab Procedures 272
Topic 35 Materials Used in Orthodontics 275
Section I

Topic-Wise Solved Questions


of Previous Years
Part I
Orthodontics

Topic 1
Introduction to Orthodontics
COMMONLY ASKED QUESTIONS
LONG ESSAYS:
1 . Define orthodontics. Describe aims and science of orthodontics.
2. What is orthodontia? Describe the various sequelae of malocclusion of teeth.
3. Describe briefly the aims, objectives, scope and limitations of orthodontic treatment. [Same as LE Q.1]

SHORT ESSAYS:
1. Objectives of orthodontics. [Ref LE Q.1]

SHORT NOTES:
1 . Aims of orthodontics. [Ref LE Q.1]
2. Jackson’s triad. [Ref LE Q.1]
3. Define interceptive and preventive orthodontics.
4. Define orthodontics. [Ref LE Q.1 and Q.2]
5. Adult orthodontics.
6. Contributions of E.H. Angle to orthodontics.
7. Aims and scope of orthodontics. [Same as SN Q.1]
8. Describe Andrew Jackson’s triad. [Same as SN Q.2]
9. Give the BSSO definition of orthodontics. [Same as SN Q.4]

3
4 Quick Review Series for BDS 4th Year, vol 1

SOLVED ANSWERS
LONG ESSAYS:
Q.1. Define orthodontics. Describe aims and science of they interfere with normal growth changes and pro-
orthodontics. duce further abnormality.
vi. It has been found (Gardiner, 1956) that at least 50%
Ans.
of all schoolchildren may benefit from orthodontic
treatment, and dental surgeons, in general family
{SN Q.4} practice, are often requested by parents to provide
such treatment.
l Orthodontics is the study of growth and development
of the masticatory apparatus and the prevention and Objectives of orthodontic treatment are as follows (Fig. 1.1):
treatment of abnormalities of this development. (SN Q.2 and SE Q.1)
l According to British Society of Orthodontics

(BSSO), ‘Orthodontics includes the study of growth


and development of the jaws and face particularly, {(The objectives of orthodontic treatment are briefly
and the body generally, as influencing the position of summarized by Jackson into the following three headings:
the teeth; the study of action and reaction of internal i. Functional efficiency
and external influences on the development and the ii. Structural balance
prevention and correction of arrested and perverted iii. Aesthetic harmony
development’. All the above three objectives put together are popularly
known as Jackson’s triad.
i. Establishing functional efficiency:
Aims of orthodontic treatment are as follows: Aesthetic harmony
i. All branches of dentistry have one common aim, the
establishment of as good an occlusion as possible, not
only in the functional but also in the aesthetic sense.
ii. The aim of orthodontics is to achieve a functional and
aesthetically harmonious occlusion by altering perma-
nently the positions of natural teeth.
This is accomplished by the careful stimulation of
alveolar bone tissue to alter its shape, and support the
teeth in a more favourable position.

{SN Q.1} Functional efficiency Structural balance


Fig. 1.1  Jackson’s Triad.
iii. The goal of modern orthodontics is to create the best
possible occlusal relationship within the framework
of acceptable facial aesthetics and stability of the Correction of malocclusion eliminates all the unfa-
occlusion. vourable sequelae of malocclusion and thereby
iv. It may be said that orthodontics seeks the following: restoring the functional efficiency of the masticatory
a. To intercept departures from normal development apparatus.
of the masticatory organs. ii. Restoring structural balance:
b. To restore conditions to normal development at Achieving structural balance between the hard and
the earliest when required. soft tissues maintains stability of the corrected maloc-
c. To establish as good an occlusion as possible in clusion.
both functional and aesthetic sense. Failure to achieve structural balance will lead to
relapse or loss of correction achieved.
iii. Aesthetic harmony:
v. It is important that in order to achieve the results, we The prime objective of orthodontic treatment is the
do not interfere with normal function more than is improvement of facial aesthetics and is the single
required. The use of appliances should be kept to the most common reason for the patients to approach an
minimum possible to attain the desired result lest orthodontist.)}
Section | I  Topic-Wise Solved Questions of Previous Years 5

Q.2. What is orthodontia? Describe the various sequelae l Many a time, functional aberrations will lead to
of malocclusion of teeth. unilateral crossbite, which, in turn, will cause
facial asymmetry.
Ans.
l Flattening of mandibular anteriors may be caused

due to increased deep bite and abnormal lip posture.


{SN Q.4} l Anterior occlusal interferences will cause pseudo-

class III.
l Orthodontics is the study of growth and development iv. Abnormal muscle function:
of the masticatory apparatus and the prevention and l Abnormal muscle activity could be contributing to
treatment of abnormalities of this development. malocclusion.
l In the case of lip trap, cushioning of lower lip behind the

proclined upper incisors will aggravate proclination.


The various problems or sequelae of malocclusion are as
v. Improper deglutition:
follows:
Many malocclusions result in abnormal functioning of
i. Psychological and social problems.
stomatognathic system like improper deglutition.
ii. Poor appearance.
vi. Mouth breathing:
iii. Interference with normal growth and development like
Malocclusions, such as increased overjet, can result in
crossbites causing asymmetry and influences of over-
mouth breathing, usually correction of increased over-
bite and overjet.
jet can make lip closure possible, establishing anterior
iv. Improper or abnormal muscle function like hyperac-
oral seal and making nasal breathing possible.
tive mentalis, hypoactive upper lip, increased buccina-
vii. Improper mastication:
tor pressures, tongue thrust and associated muscle
Malaligned teeth change the pattern of chewing,
habits like lip biting, nail biting, finger sucking, etc.
which can lead to temporomandibular joint (TMJ)
v. Abnormal deglutition.
problems, periodontal problems, etc.
vi. Mouth breathing.
viii. Speech defects:
vii. Improper mastication.
l Malocclusion affects the speech pattern of indi-
viii. Speech defects.
viduals.
ix. Increased caries activity.
l Effect of cleft lip: Speech problem in cleft lip pa-
x. Predisposition to periodontal disease.
tients are due to velopharyngeal incompetence,
xi. Temporomandibular joint problems.
naso-oral communication, abnormal tongue pos-
xii. Impacted and unerupted teeth leading to pathologies
ture and function and lip tissue inadequacy.
like cysts and damage to other teeth.
ix. Increased predilection to caries and periodontal
xiii. Risk of truma/accidents.
diseases:
xiv. Prosthetic rehabilitation complications.
l Irregular teeth make self-cleansing of oral cavity
i. Psychological and social problems: less effective and may lead to increased suscepti-
l Irregular and protruding teeth have a negative im- bility to caries and periodontal diseases.
pact on a patient’s psychology. l Loss of proper contact between teeth and abnormal
l Children with malocclusion become introvert and axial inclinations could lead to uneven distribution
their social behaviour is immature, e.g. introversion of functional stresses, which, in turn, can lead to
or self-consciousness and response to nicknames periodontal problems.
like ‘Bugs bunny’, ‘Buckteeth’, etc. x. Temporomandibular joint disorders:
ii. Poor appearance: Malocclusion causes temporomandibular joint prob-
l Poor appearance due to malocclusion forms hinder- lems like clicking, crepitus, pain and dysfunction.
ance to the child’s performance in school as well as xi. Malocclusion and trauma
in other extracurricular activities like games. One of the most common problems associated with
l Preventive or interceptive measures should be at- class II div 1 malocclusion is high risk of trauma to
tempted to correct malocclusion if it is detected in maxillary anterior teeth.
early stages of child development. xii. Impacted and unerupted tooth:
iii. Interference with growth and development: l Impacted teeth may interfere with eruption of suc-
l Perverted perioral muscle activity due to abnormal cessor or neighbouring tooth and may also cause
finger-sucking habit could cause morphological resorption of the roots of adjacent tooth.
and functional changes to dentition. l Possibility of development of pathologies like
l Common effect of abnormal perioral muscle activ- cysts due to impacted/unerupted tooth is most
ity is development of posterior crossbites. likely.
6 Quick Review Series for BDS 4th Year, vol 1

xiii. Prosthetic rehabilitation problems i. Orthodontic treatment of adults is known as adult ortho-
l Supraeruption of tooth into opposing edentulous dontics.
area, and tipping of teeth into adjacent edentulous ii. Orthodontic treatment for adults is broadly classified
area cause space problems for prosthetic rehabili- into two types:
tation. a. Adjunctive orthodontic treatment
b. Comprehensive orthodontic treatment
Q.3. Describe briefly the aims, objectives, scope and
limitations of orthodontic treatment.
(a) Adjunctive orthodontic treatment procedure is car-
Ans. ried out to facilitate other dental procedures to con-
trol disease and restore function.
[Same as LE Q.1]
Example: Uprighting of molars, forced eruption,
crossbite correction, diastema closure, etc.
SHORT ESSAYS: (b) Comprehensive orthodontic treatment is essential
treatment procedure carried out in children for cor-
Q.1. Objectives of orthodontics.
rection of malocclusion. Response to orthodontic
Ans. force is slightly slower in adults as compared to
children.
[Ref LE Q.1]
Q.6. Contributions of E.H. Angle to orthodontics.
SHORT NOTES: Ans.
Q.1. Aims of orthodontics. Edward H. Angle is considered as the ‘father of modern
Ans. orthodontics’ for his numerous contributions to the special-
ity of orthodontics, which are as follows:
[Ref LE Q.1] i. Publication of book on orthodontics in 1887
Q.2. Jackson’s triad. ii. Concept of occlusion in orthodontics
iii. Hypothesis of ‘Key of Occlusion’
Ans. iv. Classification of malocclusion
[Ref LE Q.1] v. Various appliances like:
l Angle’s E-arch
Q.3. Define interceptive and preventive orthodontics. l Pin and tube

l Ribbon arch appliance


Ans.
l Edge-wise appliance.
i. According to Graber, interceptive orthodontics is ‘that
phase of the science and art of orthodontics employed to Q.7. Aims and scope of orthodontics.
recognize and eliminate potential irregularities in the Ans.
developing dentofacial complex’.
ii. Graber defined preventive orthodontics as the action [Same as SN Q.1]
taken to preserve the integrity of what appears to be the
Q.8. Describe Andrew Jackson’s triad.
normal occlusion at a specific time.
Q.4. Define orthodontics. Ans.

Ans. [Same as SN Q.2]

[Ref LE Q.1 and Q.2] Q.9. Give the BSSO definition of orthodontics.

Q.5. Adult orthodontics. Ans.


Ans. [Same as SN Q.4]
Section | I  Topic-Wise Solved Questions of Previous Years 7

Topic 2
General Principles and Concepts of Growth
COMMONLY ASKED QUESTIONS
LONG ESSAYS:
1. Define growth and development. Mention the various theories of growth and write in detail functional matrix
hypothesis.
2. Define growth and enumerate various theories of bone growth.
3. Enumerate the various methods of measuring growth. Discuss the clinical importance of the knowledge of
growth and development in orthodontics.
4. Define growth. Discuss briefly clinical application of knowledge of growth and development in orthodontics.
5. Enumerate the various theories of growth. Explain in detail the functional matrix theory. [Same as LE Q.1]
6. Enumerate various theories of growth. Describe functional matrix theory of Moss. [Same as LE Q.1]
7. Enumerate theories of bone growth. [Same as LE Q.1]
8. What are the methods of measuring growth? Discuss the importance of the knowledge of growth and develop-
ment in orthodontics. [Same as LE Q.3]

SHORT ESSAYS:
1. Growth spurts.
2. Methods of gathering growth data.
3. Discuss drift and displacement with examples.
4. Functional matrix theory.
5. Neurotrophism.
6. Scammon’s curve. [Ref LE Q.4]
7. Safety valve mechanism.
8. Endochondral and intramembranous bone formation.
9. Growth site versus growth centre.
10. Expanding V principle. [Ref LE Q.2]
11. Growth spurts and two clinical importances. [Same as SE Q.1]
12. Pubertal growth spurts. [Same as SE Q.1]
13. Methods of studying growth. [Same as SE Q.2]
14. What are growth studies? [Same as SE Q.2]
15. Functional matrix theory of growth and development. [Same as SE Q.4]
16. Explain differential growth and Scammon’s growth curve. [Same as SE Q.6]
17. Cephalocaudal gradient of growth. [Same as SE Q.6]
18. Define and distinguish between ‘growth centre’ and ‘growth site’ with examples. [Same as SE Q.9]

SHORT NOTES:
1. Growth spurts. [Ref SE Q.1]
2. Growth sites. [Ref SE Q.9]
3. Growth curve.
4. Capsular matrix.
5. Functional matrix theory.
6. Methods of measuring/studying growth. [Ref LE Q.3]
7. Twin studies.
8. Neurotrophism.
9. Types of bone growth movements. [Ref SE Q.3]
10. Growth trends.
11. Growth centres.
8 Quick Review Series for BDS 4th Year, vol 1

1 2. Differential growth.
13. Methods of gathering growth data.
14. Enumerate the peak periods of postnatal growth. [Same as SN Q.1]
15. Enumerate various theories of growth. [Ref LE Q.1]
16. Growth spurts and two clinical importances. [Same as SN Q.1]
17. Prepubertal growth spurt. [Same as SN Q.1]
18. Scammon’s growth curves. [Same as SN Q.3]
19. Enumerate the various tissues for which Scammon’s growth curves are plotted. [Same as SN Q.3]
20. Cortical drift. [Same as SN Q.9]

SOLVED ANSWERS
LONG ESSAYS:
Q.1. Define growth and development. Mention the vari- l Soft tissues 1 skeletal elements related to single
ous theories of growth and write in detail functional function are known as functional cranial component.
matrix hypothesis.
Associated with one single
Ans. function totally all the

{SN Q.15}
i. Skeletal elements are ii. Soft tissues are known
Based on the expression of intrinsic genetic potential, known as skeletal unit. as functional matrix.
various theories of craniofacial growth are as follows:
i. Genetic theory by Brodie It was demonstrated that the origin, growth and
l
ii. Sutural dominance theory by Sicher maintenance of a skeletal unit depend almost ex-
iii. Cartilaginous theory by Scott clusively upon its functional matrix.
iv. Functional matrix theory by Melvin Moss 1968 – updated version of Melvin Moss hypothesis:
v. Von Limborgh’s theory l Functional matrix hypothesis claims that the ori-
Other concepts/theories related to craniofacial gin, form, position, growth and maintenance of all
growth are as follows: skeletal tissues and organs are always secondary,
i. Hunter and Enlow’s growth equivalent concept compensatory and necessary responses to chrono-
ii. Petrovic’s cybernetic theory logically and morphologically prior events or
processes that occur in specifically related non-
Functional matrix theory (Melvin Moss) skeletal tissues, organs or functional spaces.
l The functional matrix concept attempts to compre- l Each of the independent functions in craniofacial

hend the relation between form and function. region are carried out by functional cranial com-
l Functional matrix hypothesis was put forward by ponent (it consists of all tissues 1 organs 1
Melvin Moss based on the work of Van der Klaauw. spaces and skeletal parts necessary to carry out a
Simply stated, the theory is as follows: ‘There is no given function).
direct genetic influence on the size, shape or position
of skeletal tissues, only the initiation of ossification. Functional cranial component
All skeletogenic activities are primarily based upon is divided into
the functional matrices’.
According to original version of functional matrix
hypothesis: Functional matrix Skeletal unit
l Head is a composite structure n Operationally comprises comprises
consisting of number of relatively independent
functions. All the tissues, Skeletal tissues related to
For example: digestion, respiration, vision, organs and functioning specific function
olfaction and speech equilibrium and neural in- spaces as a whole matrix
tegration
l Each function is carried out by a group of soft Skeletal unit: Comprising bone, cartilage and tendi-
tissues, which are supported and protected by nous tissue. It is nothing but all skeletal tissues as-
related skeletal elements. sociated with a single function.
Section | I  Topic-Wise Solved Questions of Previous Years 9

Skeletal unit l The skeletal units are passively and second-


arily moved in space as their enveloping cap-
Microskeletal unit Macroskeletal unit
sule is expanded. This kind of translative
When a bone comprises several con- When adjoining por- growth is not brought about by deposition and
tiguous skeletal units, it is known as tions of a number of
resorption.
microskeletal unit. neighbouring bones
Examples: are united to function Clinical implications of functional matrix theory
i. Mandible has following microskel- as a single cranial l Orthodontic corrections of teeth are done either
etal units: alveolar, angular, condy- component, it is by intraoral and/or extraoral appliances. The force
lar, gonial, mental, coronoid and termed as macroskele- application by these appliances tends to alter the
basal. tal unit, e.g. entire
functional matrix.
ii. Maxilla has the following micro- endocranial surface of
l Alteration in periosteal functional matrix pro-
skeletal units: oribital, pneumatic, the calvarium.
palatal and basal. duces changes in microskeletal unit, i.e. alveolar
bone, while the alteration of capsular functional
Functional matrix matrix produces changes in macroskeletal unit,
It consists of muscles 1 glands 1 nerves1 ves- i.e. jaws.
sels 1 fat 1 teeth and functioning spaces. Q.2. Define growth and enumerate various theories of
bone growth.
Functional Matrix
Ans.
Growth has been defined by various clinicians in different
Periosteal matrix Capsular matrix ways, as follows.
According to:
It includes muscles + Example: Neurocranial J.S. Huxley: ‘The self-multiplication of living substance’.
blood vessels + nerves + capsule and orofacial Krogman: ‘Increase in size, change in proportion and pro-
glands etc. capsule. gressive complexity’.
Todd: ‘An increase in size’.
They act directly and They act indirectly and Meridith: ‘Entire series of sequential anatomic and physi-
actively upon their related passively on their related ological changes taking place from the beginning of prena-
skeletal units, bring about skeletal units producing a tal life to senility’.
transformation in their size secondary compensatory Moyers: ‘Quantitative aspect of biological development per
and shape by bone translation in space. unit of time’.
deposition and resorption. Moss: ‘Change in any morphological parameter which is
measurable’.
Based on the expression of intrinsic genetic potential,
Capsular Matrix various theories of craniofacial growth are as follows:
Neurocranial capsule Orofacial capsule i. Genetic theory by Brodie
ii. Sutural dominance theory by Sicher
Is made up of skin connective Surrounds and protects the
tissue, aponeurotic layer, loose oronasopharyngeal spaces
iii. Cartilaginous theory by Scott
connective layer, periosteum, which constitute the orofacial iv. Functional matrix theory by Melvin Moss
base of the skull and two lay- capsular matrix. The growth of v. Von Limborgh’s theory
ers of duramatter which sur- facial skull is influenced by Other concepts/theories related to craniofacial growth
rounds and protects neurocra- volume and patency of these are as follows:
nial capsular functional spaces.
matrix, which is the brain 1
i. Hunter and Enlow’s growth equivalent concept
leptomeninges 1 CSF. ii. Petrovic’s cybernetic theory
i. Genetic theory
l Each of the neurocranial capsule and orofacial l This is one of the earliest theories put forward by
capsule is an envelope which contains series of Brodie in 1941.
functional cranial components (i.e. skeletal l Brodie stated simply that all growth is controlled
units 1 functional matrix), which as a whole is by genes, and is planned.
sandwiched between two covering layers. This theory is more of an assumption and is not proved.
l The alterations in special position of skeletal l Primary genetic control determines only certain
units are brought about by the expansion of features and does not have complete influence
these capsules within which the respective over all growth.
bones arise and grow, and are maintained.
10 Quick Review Series for BDS 4th Year, vol 1

ii. Sicher’s sutural dominance theory/Sicher’s hypothesis/ l Nasal septum also shows innate growth poten-
Sutural theory tial on being transplanted to a different location.
l Sutural dominance theory was put forward by iv. Functional matrix theory (Melvin Moss)
Sicher. According to him, bone growth within the l The functional matrix concept attempts to compre-

various craniofacial units is the result of growth tak- hend the relation between form and function.
ing place in sutures. l Functional matrix hypothesis was put forward by

l According to Sicher, the growth of skull tissue is Melvin Moss based on the work of Van der Klaauw.
controlled by its own genetic potential. Simply stated, the theory is as follows: ‘There is no
According to him, all bone-forming elements, like car- direct genetic influence on the size, shape or posi-
tilage, sutures and periosteum, are growth centres. tion of skeletal tissues, only the initiation of ossifi-
l This theory is also known as the sutural dominance cation. All skeletogenic activity is primarily based
theory because proliferation of connective tissue upon the functional matrices’.
and its replacement by bone in the suture is consid- l Head is a composite structure. Each function is

ered to be the primary event. carried out by a group of soft tissues, which
l Growth taking place in the sutures which connect are supported and protected by related skeletal
maxillary complex to the cranium causes down- elements.
ward movement of the midface. l Soft tissues 1 skeletal elements related to single func-

Drawbacks of sutural theory: tion are known as functional cranial component.


l Any unusual pressure on suture initiates bone re-

sorption and not bone’s deposition, as bone is a Associated with one single
tension adapted tissue. function totally all the
l Sutures do not act as primary growth centres.

Growth in the sutural area is a secondary response


to functional needs. i. Skeletal elements are ii. Soft tissues are known
l Evidence in favour of secondary role of sutural known as skeletal unit. as functional.
growth is more.
l Based on various experimental studies, it was
l It was demonstrated that the origin growth and
shown that extirpation or removal of facial sutures maintenance of the skeletal unit depend almost ex-
had no effect on the growth of skeleton. clusively on its functional matrix.
iii. Cartilaginous theory 1968 – updated version of Melvin Moss’ hypothesis:
l Cartilaginous theory is also known as Scott’s hy-
l Functional matrix hypothesis claims that the origin,
pothesis/nasal septal theory and is put forward by form, position, growth and maintenance of all skel-
James Scott. etal tissues and organs are always secondary com-
l This theory is based on the principle that intrinsic
pensatory and necessary responses to chronologi-
growth-controlling factors are present in cartilage cally and morphologically prior events or processes
and periosteum. that occur in specifically related nonskeletal tis-
l According to Scott, cartilaginous sites throughout
sues, organs or functional spaces.
the skull are primary growth centres and growth of l Each of the independent functions in craniofacial
cartilage in nasal septum provides force that dis- region are carried out by functional cranial compo-
places maxilla downwards and forward. Nasal sep- nent, which consists of all tissues 1 organs 1
tum is considered to be the major contributor in spaces and skeletal parts necessary to carry out a
maxillary growth. given function.
l In mandible, condylar cartilage is considered to be
v. Von Limborgh’s theory
the growth centre present bilaterally with the horse- l In 1970, Von Limborgh put forward a multi-facto-
shoe-shape mandible in-between. rial theory.
Experimental evidences in favour of this theory are as l He explains the process of growth and development
follows: in a view that combines all three existing theories:
l Removal of nasalseptal cartilage in rats and rab-
functional matrix theory, sutural theory by Sicher
bits resulted in deficient snout of these animals. and genetic theory.
l Transplantation of a part of epiphyseal plate and
l According to Von Limborgh, five factors that con-
synchondroses to a different location results in trol the growth are as follows:
continued growth in the new location, which in- i. Intrinsic genetic factors – These are genetic
dicates innate growth potential of the cartilage. control of the skeletal units themselves.
Section | I  Topic-Wise Solved Questions of Previous Years 11

ii. Local epigenetic factors – Bone growth is de-


{SN Q.6}
termined by genetic control originating from
adjacent structures like brain and eyes. Various methods of measuring growth are as follows:
iii. General epigenetic factors – These are genetic A. Proffit’s classification
factors determining growth from distant
structures, e.g. sex hormones and growth Proffit’s classification
hormones.
iv. Local environmental factors – Nongenetic fac-
tors from local external environment, e.g. hab- Measurement approaches Experimental approaches
its, muscle force. i. Anthropometry i. Vital staining
v. General environmental factors – Nongenetic ii. Craniometry ii. Radioactive tracer
general environment factors, e.g. nutrition iii. Cephalometry iii. Autoradiography
iv. Arcial growth iv. Implant radiography
and O2.
v. Logarithmic spiral
Views expressed by Von limborgh can be summarized as vi. Finite element analysis
follows:
l Chondrocranial and desmocranial growth are con- B. Moyer’s classification is of the following types:
trolled by intrinsic genetic factors. i. Quantitative
l Cartilaginous parts of skull are considered as ii. Observations
growth centres. iii. Rating and ranking
l Sutural and periosteal growth is additionally iv. Opinions
governed by local nongenetic environmental
infamies.
l Sutural growth is controlled by influences origi-
The quantitative method of measuring growth is again of
nating from skull cartilages 1 other adjacent skull the following types:
structures.
l Periosteal growth to a large extent depends on

growth of adjacent structures.


Other concepts/theories related to craniofacial growth are Direct Indirect Combination
as follows: measurements measurements
i. Anthropometry i. Study casts i. Radiography
[SE Q.10] + implants
ii. Craniometry ii. Radiographs ii. Radiograph +
{Hunter and Enlow’s growth equivalent concept: metaphysics bands
According to Enlow’s expanding ‘V’ principle: iii. Vital staining iii. Photographs iii. Autoradiography
l Many facial bones or parts of bone have a ‘V’-
iv. Implant markers
shaped pattern of growth. In these bones the growth
movements and enlargement occur towards the wide v. Histochemistry
ends of the ‘V’ as a result of differential deposition
and selective resorption of bone.
l Bone deposition occurs on the inner side of the wide
Various methods of measuring growth in detail are as
end of ‘V’ and bone resorption on the outer surface. follows:
Deposition also takes place at the ends of two arms I. Craniometry and anthropometry
l Craniometry is the art of measuring of skulls so as
of the ‘V’ resulting in growth movement towards the
ends. to discover their specific differences. Precise mea-
l In a number of regions, such as the base of the
surements can be made with craniometry. It is a
mandible, ends of long bones, mandibular body, cross-sectional type of study.
l Site, amount and rate of growth cannot be elicited
palate etc., the ‘V’ pattern of the growth
occurs.} by craniometry but gives information about direc-
tion of growth to some extent.
l Anthropometry is the measurement of skeletal
Q.3. Enumerate the various methods of measuring
growth. Discuss the clinical importance of the knowl- dimensions on living individuals. It is a longitudi-
edge of growth and development in orthodontics. nal study.
l Anthropometry gives little information about the

Ans. amount of growth and to some extent the rate of


12 Quick Review Series for BDS 4th Year, vol 1

growth, whereas it gives relatively accurate infor- l Disadvantage of this method is that it is not a
mation about the direction of growth. longitudinal study; hence repeated data of the
Clinical uses same individual cannot be obtained.
l Cranial and facial index are two important in- III. Implant radiography
dices used in orthodontics. l Use of implant radiograph to study bone growth

l Index is the ratio of smaller to a larger linear was introduced by Professor Bjork in 1969. It is
measurement expressed by means of percentage. an experimental method for studying physical
bone growth.
Maximum cranial breadth Procedure:
Cranial index 10
Maxim
mum cranial length l It involves inserting small bits of biologically

l Maximum cranial breadth ‘is the measured inert metal alloys into growing bone, either
distance between the two most prominent mandible or maxilla.
l Very tiny metallic implants, 1.5 mm long and
points on the either side of the head’.
l Maximum cranial length ‘is the measured dis- 0.5 mm in diameter made of Tantalumare,
tance from glabella to opisthocranion, the most were used.
l Osseo-integrated implants serve as reference
prominent point of the occipital bone in the
midline’. points, and serial cephalometric radiographs
l The values of cranial index are more for are taken repeatedly over a period of time,
brachycephalic/short and round head types and and compared.
are around 80 to 85, and are less of around Preferable sites of implants in mandible and maxilla are as
70–75 for dolichocephalic/long narrow head follows:
types, while for mesocephalic/middle type, the i. Mandible
l Anterior aspect of symphysis in the midline be-
values are in between the above two types.
low roots.
Facial height l Two pins on the right side of body of mandible:
Facial index 100
Zygomatic breadtth one below first premolar and another below sec-
ond premolar or first molar.
l Facial height is the measured distance from
l External surface of ramus on the right side at the
nasion to gnathion whereas zygomatic breadth
level of occlusal surface of molars.
is the distance between two zygomas.
ii. Maxilla
l The values of facial index are more for leptopro-
l Hard palate behind deciduous canines.
sopic, i.e. high and narrow facial type 90–95;
l After eruption of maxillary incisors, below the
less for euryprosopic, i.e. broad and round fa-
anterior nasal spine.
cial type 80–85; and average in between these
l Bilaterally, one implant on the either side of zygo-
two values for mesoprosopic, i.e. middle type
matic process of maxilla.
around 85–90.
Junction of hard palate and alveolar process
II. Vital staining
medial to the first molar.
l Vital staining is an experimental method of
l Implant radiography gives very accurate informa-
measuring growth introduced by John Hunter in
tion about site, amount and direction of growth,
the eighteenth century.
while a relatively accurate information about the
Commonly used dyes for vital staining are as follows:
rate of growth.
l Alizarin S
l Drawbacks of this method are that it is a two-
l Radioactive tracers
dimensional study of three-dimensional process
l Fluorochrome
and radiation hazard.
l Tetracycline

l Trypan blue Q.4. Define growth. Discuss briefly clinical applica-


Procedure: tion of knowledge of growth and development in
l This technique involves injecting certain orthodontics.
dyes that stain mineralizing tissues and get
Ans.
incorporated in bones and teeth.
l Animals are sacrificed and tissues are studied l Growth has been described in so many terms.
histologically for the manner in which bone Todd defines growth as ‘increase in size’.
is laid down, site of growth, direction, dura- Krogman: increase in size, change in spatial proportion
tion and amount of growth at different sites over time
of the bone. Huxley: Self-multiplication of living tissues.
Section | I  Topic-Wise Solved Questions of Previous Years 13

Moss defines growth as any change in morphology At the time of birth, head constitutes 25%–30%
which is within measurable parameters. and there is increased growth of body and
Meredith defines growth as the entire series of anatomic limbs.
and physiological changes taking place between the In an adult, the head constitutes only 12%, while
beginning of prenatal life and the close of senility. limbs account to 50%. These changes in the pat-
Moyer defines growth as the biological process by tern of growth are because of cephalocaudal
which living matter gets larger. gradient.
l Development is defined in simple words as the ‘progres- l Cephalocaudal growth in face:
sion towards maturity’. At the time of birth, jaws and face are less devel-
l According to Melvin Moss, ‘Development can be con- oped compared to skull. Maxilla being closer to
sidered as a continuum of causally related events from head grows faster, and the growth is completed
the fertilization of ovum onwards’. before mandibular growth. Mandible being
Importance of knowledge of growth and development in away from the brain grows more and growth
orthodontics is as follows: completes later than maxilla.
Craniofacial growth is a complex phenomenon. A thor-
(b) Scammon’s cure of growth:
ough knowledge of the normal pattern of growth and
Major tissues of the human body are divided into
normal variations will help in identifying the problems
four types:
and also utilize the normal growth to advantage in treat-
i. Lymphoid tissue
ment.
ii. Neural tissue
Clinical implications of growth and development can be
iii. General tissue
studied under the following headings:
iv. Genital tissue
i. Growth pattern
These different tissues grow at different times at
ii. Differential growth
different rates
iii. Variability
i. Lymphoid tissue
iv. Timing variations
Proliferates rapidly in late childhood to almost
v. Safety valve mechanism
200% of adult size. Adaptation to protect child
i. Growth pattern from infection, by 18 years it undergoes invo-
l Definite arrangement of designs in definite propor- lution to reach adult size.
tional relationship is known as pattern. In relation to ii. Neural tissue
growth, pattern can be defined as proportional rela- Grows very rapidly and almost reaches adult
tionship over time. size by 6–7 years of age, after that a very little
l The patterns are the controlling or restricting mech- growth occurs in neural tissues.
anisms to preserve the integration of parts of the iii. General/visceral 1 (muscle, bone and other
body under varying conditions. Differential growth organs)
(cephalocaudal growth, Scammon’s growth) and They exhibit ‘S’-shaped curve of rapid growth
predictability are the contributors to pattern. up to 2–3 years of age followed by slow phase
ii. Differential growth of 3–10 years of age.
Followed by rapid phase of growth after 10th
[SE Q.6]
year, terminating by 18–20 years.
l {Throughout life, human body does not grow at the iv. Genital tissue (reproductive organs)
same rate, different organs grow at different rates to a Negligible growth until puberty. They grow
different amount and at different times, this is known as rapidly at puberty, reaching adult size, after
differential growth. which growth ceases.
l The concepts of differential growth are more clearly
Effect of Scammon’s growth in facial region:
understood by two important aspects of growth: Mandible follows somatic growth pattern. Long-time
(a) Cephalocaudal gradient of growth growth is seen until about 18–20 years in males.
(b) Scammon’s cure of growth Maxilla follows neural growth pattern and growth
(a) Cephalocaudal gradient of growth ceases earlier; hence skeletal problems of the maxilla
l An axis of increased growth gradient extending should be treated earlier to mandible.}
from head towards the feet is called ‘cephalocau- iii. Variability in growth
dal growth’. Variability in growth is the law of nature. No two
l In fetal life, head constitutes 50% of the total body individuals mimic alike, and no two individuals
length, while limbs are primitive (30%). grow in the same pattern.
14 Quick Review Series for BDS 4th Year, vol 1

The reasons for variability in growth are as follows: Sex differences:


a. Variation within normal range: evaluated by l Boys and girls exhibit variation between

Wetzel’s grid onset of menarche and rate of growth.


b. Variation due to other influences, which include: In girls, there is early onset of menarche, and
i. Heredity growth completes faster than boys of the same
ii. Nutrition age. In boys, there is delayed onset of puberty,
iii. Racial difference and growth occurs over a longer period.
iv. Climate Growth spurts:
v. Exercise l Growth does not take place uniformly at all

vi. Socioeconomic factors times. There seems to be periods when a sud-


vii. Psychological factors den acceleration in growth occurs. This sud-
viii. Size of the family den increase in growth is termed growth spurt.
ix. Hormonal changes l The growth spurts can be utilized for growth

c. Variation due to timing effects: modulation treatment.


i. Body build v. Safety valve mechanism
ii. Sex difference l Safety valve mechanism is a nature’s attempt
iii. Growth spurts to maintain proper occlusion. To compensate
a. Variation within normal range: for horizontal growth in mandible, the maxil-
l Is evaluated by Wetzel’s grid. lary intercanine width serves as a safety valve.
l The resultant curve obtained by plotting the l In mandible, the intercanine width is com-

height and weight of an individual against the pleted at 9 years of age in girls and at around
age over a period of time is compared with nor- 10 years of age in boys.
mal range. l In the maxilla, the intercanine width is com-

l Any unexpected growth pattern changes should pleted by 12 years of age in girls and at 18
be evaluated and investigated for growth abnor- years in boys.
mality. l The delay in growth of maxillary intercanine

b. Variation due to other influence areas: arch width serves as a ‘safety valve’ for puber-
l Heredity: On rate of growth and onset of men- tal growth spurts in mandible. Maxillary inter-
arche, there is genetic control. canine width adjusts to the mandibular denti-
l Nutrition: Certain parts of the body may be af- tion as it is brought forward, this is called
fected by malnutrition and they show retardation ‘safety valve mechanism’.
of growth.
Q.5. Enumerate the various theories of growth. Explain
l Racial differences: Differences in skeletal matu-
in detail the functional matrix theory.
rity are exhibited by different races.
l Climate and seasonal effects: People living in Ans.
cold places have more of fat or adipose tissue.
[Same as LE Q.1]
l Exercise: Increases muscle mass and physique.

c. Variation due to timing effects: Q.6. Enumerate various theories of growth. Describe
l Timing variations in growth is nothing but occur- functional matrix theory of Moss.
rence of the same events at different times for
Ans.
different individuals.
iv. Timing variations in growth are due to following [Same as LE Q.1]
reasons:
Q.7. Enumerate theories of bone growth.
i. Body build
ii. Sex difference Ans.
iii. Growth spurts
[Same as LE Q.1]
Body build:
Ectomorphic: Late maturing individuals, grow for Q.8. What are the methods of measuring growth? Dis-
a longer period. cuss the importance of the knowledge of growth and
Mesomorphic: Individuals exhibiting average development in orthodontics.
growth period.
Ans.
Endomorphic: Early maturing individuals where
growth completes much faster. [Same as LE Q.3]
Section | I  Topic-Wise Solved Questions of Previous Years 15

SHORT ESSAYS: Advantages:


l The specific developmental pattern of an individ-
Q.1. Growth spurts. ual can be studied and compared as the same
Ans. subjects are followed up over a long period.
l Developmental variations among individuals
{SN Q.1} within the sample can be studied.
Disadvantages:
Growth does not take place uniformly at all times. There l A long period of time is involved; it often takes
seems to be periods when a sudden acceleration in years or decades to complete a study.
growth occurs. This sudden increase in growth is termed l These studies require maintenance of laboratory
growth spurt. research personnel and data storage systems for a
Etiology: The physiological alteration in hormonal long period of time.
secretion. l These can be expensive.
The timing of growth spurts differs in boys and girls. l As these studies are performed over prolonged
They are as follows: periods of time, there is a risk of sample size re-
A. Just before birth duction due to change of place, or due to other
B. One year after birth unforeseen events.
C. Mixed dentition growth spurt (boys 8–11 years and . Cross-sectional studies
b
girls 7–9 years) l Cross-sectional studies are carried out by observa-
D. Prepubertal growth spurt (boys 14–16 years and girls tion and measurement made of different samples and
11–13 years) studied at different periods.
Clinical importance: Advantages:
Knowledge of growth spurts is essential for suc- l These studies are of short duration.
cessful treatment planning in orthodontics. l They are less expensive.
l Growth modulation by means of functional
l It is possible to repeat the study in case of any
and orthodontic appliances elicits better re- flaw.
sponse during growth spurts. c. Semi-longitudinal studies
l In these studies, to derive the advantages of gathering

growth data, it is possible to combine cross-sectional


l Surgical correction involving maxilla and mandible
and longitudinal methods.
should be carried out only after cessation of growth spurts.
l During pubertal growth spurts, there is change in growth Types of growth data:
direction from vertical to horizontal. The physical growth can be studied by a number of ways:
l Periods of maximum growth are suitable for arch ex- i. Opinion
pansion and rapid skeletal expansion procedures. l It is a clever guess of an experienced person
l Growth spurt period is the best time for interceptive and is the crudest means of studying growth.
orthodontic procedures. l This method of studying growth is not very sci-
entific and should be avoided when better meth-
Q.2. Methods of gathering growth data.
ods are available.
Ans. ii. Observations
A method of gathering growth-related information
The various growth studies are as follows:
by observation. These are useful in studying all or
a. Longitudinal studies
none of the phenomena such as the presence or ab-
b. Cross-sectional studies
sence of disease, e.g. presence or absence of a car-
c. Semi-longitudinal studies
ies and Class II molar relation.
a. Longitudinal studies iii. Ratings and rankings
l This type of study consists of the observations and Rating makes use of standard, conventionally ac-
measurements pertaining to growth made on a per- cepted scales for classification. Ranking involves
son or a group of persons at regular intervals over a the arrangement of data in an orderly sequence
prolonged period of time. based on the value.
l The longitudinal studies are long-term studies where Whenever it is difficult to quantify a particular
the same sample is studied by means of follow-up data, it is possible to adopt a method of rating and
examination. ranking.
16 Quick Review Series for BDS 4th Year, vol 1

Quantitative measurements Examples: Growth of the cranial base results in the forward
A scientific approach to study growth is based on ac- and downward displacement of maxilla.
curate measurements, which are of three types:
Q.4. Functional matrix theory.
i. Direct data: Direct data are obtained from mea-
surements that are taken on living persons or Ans.
cadavers by means of scales, measuring tapes or
l Functional matrix hypothesis was put forward by
calipers.
Melvin Moss based on the work of Van der Klaauw.
ii. Indirect data: Growth measurements derived
Simply stated, the theory is, ‘There is no direct genetic
from images, photographs, radiographs or den-
influence on the size, shape or position of skeletal tis-
tal casts of a person.
sues, only the initiation of ossification. All skeletogenic
iii. Derived data: The data derived by comparing two
activity is primarily based upon the functional matrices’.
measurements. These two sets of measurements
According to original version of functional matrix
can be of different time frames or of two different
hypothesis:
samples.
l Head is a composite structure operationally con-

Q.3. Discuss drift and displacement with examples. sisting of number of relatively independent func-
tions, e.g. digestion, respiration, vision and neural
Ans.
integration.
l Each function is carried out by a group of soft tis-

sues, which are supported and protected by re-


{SN Q.9} lated skeletal elements.
l Drift and displacement are two basic modes of Soft tissues 1 skeletal elements related to single
movements involved during growth. The overall pro- function are known as functional cranial component.
cess of craniofacial enlargement is a combination of
Functional cranial component
drift and displacement.
is divided into
Cortical drift
l Growth of most bones occurs due to interplay of

bone deposition and resorption. A combination of


bone deposition and resorption resulting in a Functional unit Skeletal unit
growth movement towards the depositing surface
is called cortical drift. Comprises all the tissues, Skeletal tissues related to
l If bone deposition and resorption on either side of
organs and functioning specific function
a bone are equal, then the thickness of the bone spaces as a whole matrix
remains constant.
l It was demonstrated that the origin, growth and
l If in case more bone is deposited on one side and
maintenance of skeletal unit depend almost exclu-
less bone resorbed on the opposite side, then the
sively on its functional matrix.
thickness of the bone increases.
Functional matrix:
l Drift occurs in all the regions of growing bones,
It consists of muscles 1 glands 1 nerves 1 vessels
producing generalized enlargement as well as re-
1 fat 1 teeth and functioning spaces.
location of parts.
Displacement Functional Matrix
The movement of whole bone as a unit is known as
displacement.
It can be of two types: primary or secondary Periosteal matrix Capsular matrix
a. Primary displacement
If a bone gets displaced as a result of its own It includes muscles + Example: Neurocranial
growth, it is known as primary displacement, blood vessels + nerves + capsule and orofacial
e.g. growth of the maxilla at tuberosity region glands etc. capsule.
results in pushing of maxilla against the cranial
base, resulting in the displacement of maxilla They act directly and They act indirectly and
in forward and downward directions. actively upon their related passively on their related
b. Secondary displacement skeletal units, bring about skeletal units producing a
If a displacement of bone occurs as a result of transformation in their size secondary compensatory
growth and enlargement of adjacent bone, it is and shape by bone translation in space.
known as secondary displacement. deposition and resorption.
Section | I  Topic-Wise Solved Questions of Previous Years 17

Capsular matrix has neurocranial capsule and oro- differentiation, without which further continuation of
facial capsule. Each of the neurocranial capsules myogenesis usually does not occur.
and orofacial capsules is an envelope which con- l Neurovisceral trophism:

tains series of functional cranial components (i.e. Visceral organs, like the salivary glands, fat tissue
skeletal units 1 functional matrix) which as a and other organs, are trophically regulated, at least
whole is sandwiched between two covering layers. in part.
l The alterations in special position of skeletal units
Q.6. Scammon’s curve.
is brought about by the expansion of these cap-
sules within which the respective bones arise, Ans.
grow and are maintained.
[Ref LE Q.4]
l The skeletal units are passively and secondarily

moved in space as their enveloping capsule is ex- Q.7. Safety valve mechanism.
panded. This kind of translative growth is not
Ans.
brought about by deposition and resorption.
Clinical implications of functional matrix theory: l Safety valve mechanism is a nature’s attempt to main-
l The force application by orthodontic appliances tain proper occlusion. To compensate for horizontal
tends to alter the functional matrix. growth in mandible, the maxillary intercanine width
l Alteration in periosteal functional matrix pro- serves as a safety valve.
duces changes in microskeletal unit. i.e. alveolar l In mandible, the intercanine width is completed by 9 years

bone, while the alteration in capsular functional of age in girls and at around 10 years of age in boys.
matrix produces changes in macroskeletal unit, l In the maxilla, the intercanine width is completed by the

i.e. jaws. age of 12 years in girls and at 18 years in boys.


l The delay in growth of maxillary intercanine arch width
Q.5. Neurotrophism.
serves as a ‘safety valve’ for pubertal growth spurts in
Ans. mandible. Maxillary intercanine width adjusts to the
mandibular dentition as it is brought forward, this is
l A nonimpulse transmitting neural function that involves
called ‘safety valve mechanism’.
a xoplasmic transport and provides for long-term inter-
action between neurons and innervated tissues and regu- Q.8. Endochondral and intramembranous bone formation.
lates homeostatically the morphological, compositional
Ans.
and functional integrity of those tissues is known as
neurotrophism. The process of bone formation occurs by two basic meth-
l Presently, the nature of neurotrophic substances and the ods, namely:
process of their introduction into the target tissue are i. Endochondral bone formation
unknown. ii. Intramembranous bone formation
l The various types of neurotrophic mechanisms are as i. Endochondral ossification/cartilaginous ossification/
follows: indirect ossification/endochondral bone formation
i. Neuroepithelial trophism l In this type of osteogenesis, the bone formation is

ii. Neurovisceral trophism preceded by the formation of a cartilaginous model,


iii. Neuromuscular trophism which is subsequently replaced by bone.
l Neuroepithelial trophism: Endochondral bone formation occurs as follows:
There is a neurotrophic control over epithelial mito- l At the site of bone formation, the condensation of

sis and synthesis. Certain neurotrophic substances mesenchymal cells takes place and some of them
released by nerve synapses control normal epithelial differentiate into chondroblasts and lay down hya-
growth. If this neurotrophic process is lacking or line cartilage.
deficient, abnormal epithelial growth, orofacial hy- l The cartilage is surrounded by a membrane called

poplasia and malformation occur. perichondrium, which is highly vascular and con-
Example: The presence of taste buds is dependent tains osteogenic cells.
upon an intact innervation. The nerves are not only l The intercellular substance surrounding the carti-

important for the sensation of taste but also for lage cells becomes calcified due to an enzyme
healthy growth of taste buds. If the taste buds are de- alkaline phosphatase secreted by cartilage cells.
innervated, they become atrophic. l The cartilage cells are deprived of nutrition, lead-

l Neuromuscular trophism: ing to their death. This results in the formation of


Embryonic myogenesis is independent of neural in- empty spaces, called primary areolae.
nervation and trophic control. The neural innervation l The blood vessels and osteogenic cells from

is established approximately at the myoblast stage of perichondrium invade the calcified cartilaginous
18 Quick Review Series for BDS 4th Year, vol 1

matrix, which is now reduced to bars or walls due


to eating away of calcified matrix. This leaves Growth site Growth centre
large empty spaces between the walls called sec- Sites of growth does not Centres of growth continue
ondary areolae. continue to grow when to grow when transplanted
l The osteogenic cells of perichondrium become os-
transplanted to another area to another area
teoblasts and arrange along the surface of these bars They markedly response to Their response to external
of calcified matrix and lay down osteoid, which external influences influences is less whereas
later becomes calcified to form lamella of bone. response to functional
needs is more
l The calcified matrix of cartilage acts as a support

for bone formation, and layers of osteoid are These are the places where They cause growth of the
exaggerated growth takes major part of the bone
secreted one upon other.
place but they do not cause
The entire process of endochondral ossification is growth of the whole bone
continuous and repetitive.
All growth sites are not growth All growth centres are
Importance of endochondral ossification
centres growth sites
l Cartilage behaves like a soft tissue, and growth

takes place by both interstitial growth and apposi- Theories of growth are not Various theories of growth
based on growth site are based on growth
tional growth.
centres
l Cartilage can grow in heavy pressure areas, as it

is a pressure-adapted tissue unlike bone, e.g. cra- Growth sites do not control The overall growth of the
the overall growth of the bone bone is controlled by
nial base.
growth centre
l Linear growth takes place allowing lengthening of

bones.
ii. Intramembranous bone formation
l Here, the formation of bone is not preceded by the

formation of a cartilaginous model. Instead, bone is Q.10. Expanding V principle.


laid down directly in a fibrous membrane. Ans.
The intramembranous bone is formed in the following
manner: [Ref LE Q.2]
l At the site of bone formation, mesenchymal cells Q.11. Growth spurts and two clinical importances.
become aggregated and some of them lay down
bundles of collagen fibres. Ans.
l Some mesenchymal cells enlarge and form osteo- [Same as SE Q.1]
blasts, which secrete a gelatinous matrix called
osteoid around collagen fibres. Q.12. Pubertal growth spurts.
l The osteoid is converted into bone lamella by depo- Ans.
sition of calcium salts in it, and now the osteoblasts
move away from the lamellae and a new layer of [Same as SE Q.1]
osteoid is secreted, which also gets calcified. Q.13. Methods of studying growth.
l Some of the osteoblasts get entrapped between

two lamellae. These are called osteocytes. Ans.

Q.9. Growth site versus growth centre. [Same as SE Q.2]

Ans. Q.14. What are growth studies?


Ans.
{SN Q.2} [Same as SE Q.2]
Differences between growth site and centre are as follows: Q.15. Functional matrix theory of growth and development.
Growth site Growth centre Ans.
It is any location or site where It is a location or place [Same as SE Q.4]
growth takes place where genetically con-
trolled growth takes place Q.16. Explain differential growth and Scammon’s growth
It is a region where periosteal These are the places of curve.
or sutural bone formation and ossification with tissue-
remodelling resorption adap- separating force
Ans.
tive to environment take place [Same as SE Q.6]
Section | I  Topic-Wise Solved Questions of Previous Years 19

Q.17. Cephalocaudal gradient of growth. l The skeletal units are moved passively and secondarily
in space as their enveloping capsule is expanded. This
Ans.
kind of translative growth is not brought about by depo-
[Same as SE Q.6] sition and resorption.
Q.18. Define and distinguish between ‘growth centre’ Q.5. Functional matrix theory.
and ‘growth site’ with examples.
Ans.
Ans.
l Functional matrix hypothesis was put forward by Melvin
[Same as SE Q.9] Moss based on the work of Van der Klaauw. Simply
stated, the theory is as follows: ‘There is no direct genetic
influence on the size, shape or position of skeletal tissues,
SHORT NOTES: only the initiation of ossification. All skeletogenic activity
Q.1. Growth spurts. is primarily based upon the functional matrices’.
l Functional cranial component is divided into functional
Ans.
matrix and skeletal unit.
[Ref SE Q.1] l Functional matrix comprises all tissues 1 organs 1

functioning spaces as a whole, while skeletal unit com-


Q.2. Growth sites.
prises skeletal tissues related to specific function matrix.
Ans. l It was demonstrated that the origin, growth and mainte-

nance of skeletal unit depend almost exclusively upon


[Ref SE Q.9]
its functional matrix.
Q.3. Growth curve.
Clinical implications of functional matrix theory:
Ans. l The force application by orthodontic appliances tends to
alter the functional matrix.
Scammon’s growth curve indicates that growth rates of
l Alteration of periosteal functional matrix produces
different tissues are different at different ages.
changes in microskeletal unit, while alteration in capsular
Example: Various tissues for which Scammon’s growth
functional matrix produces changes in macroskeletal unit.
curve is plotted are lymphoid, neural, general or visceral
and genital tissues. Q.6. Methods of measuring/studying growth.
l Lymphoid tissue proliferates rapidly in late childhood to
Ans.
almost 200% of adult size.
By 18 years, it undergoes involution to reach adult size. [Ref LE Q.3]
l Neural tissue grows very rapidly and almost reaches
Q.7. Twin studies.
adult size by 6–7 years of age and after that a very little
growth occurs in neural tissue. Ans.
l General/visceral tissue exhibits ‘S’-shaped curve, which l In twin study, twins are compared. Comparing monozy-
indicates rapid growth up to 2–3 years of age followed gotic twins with dizygotic twins is the best way to de-
by slow phase between 3 and 10 years of age, followed termine the extent of genetic effect on the malocclusion.
again by rapid phase of growth, occurring after 10th l The heritability of malocclusion can be determined by
year and terminating by 18–20 years. comparing the ordinary siblings, monozygotic twins
l Genital tissue shows negligible growth until puberty. and dizygotic twins.
They grow rapidly at puberty, reaching adult size, after l In case of monozygotic twins, any change in occlusion or
which growth ceases. features could be attributed to environmental factor as both
Q.4. Capsular matrix. of them have similar DNA, whereas in dizygotic twins, in-
terplay of genetic and environmental factors is responsible.
Ans.
Q.8. Neurotrophism.
l Capsular matrix is a component of functional matrix. It
has neurocranial capsule and orofacial capsule. Ans.
l Each of the neurocranial capsules and orofacial cap- l A nonimpulse transmitting neural function that involves
sules is an envelope containing series of functional axoplasmic transport and provides for long-term interaction
cranial components, and, as a whole, is sandwiched between neurons and innervated tissues that homeostatically
between two covering layers. regulates the morphological, compositional and functional
l Alterations at special positions of skeletal units is brought integrity of those tissues is known as neurotrophism.
about by the expansion of these capsules within which l Presently the nature of neurotrophic substances and the pro-
the respective bones arise, grow and are maintained. cess of their introduction into the target tissue is unknown.
20 Quick Review Series for BDS 4th Year, vol 1

l The various types of neurotrophic mechanisms are In an adult, the head constitutes only 12%, while
i. Neuroepithelial trophism limbs account to 50%. These changes in the pattern
ii. Neurovisceral trophism of growth are because of cephalocaudal gradient.
iii. Neuromuscular trophism l Scammon’s growth curve indicates that growth rate

of different tissues is different at different ages.


Q.9. Types of bone growth movements.
Example: The various tissues for which Scammon’s
Ans. growth curve is plotted are lymphoid, neural, gen-
eral or visceral and genital tissues.
[Ref SE Q.3]
Q.13. Methods of gathering growth data.
Q.10. Growth trends.
Ans.
Ans.
Various methods of gathering growth data are as follows:
Three types of growth trends enumerated by Tweed are as
a. Longitudinal studies
follows:
b. Cross-sectional studies
i. Type A: Maxilla and mandible grow in unison, both
c. Semi-longitudinal studies
downward and forward. ANB shows no change.
The physical growth can be studied by a number of ways:
ii. Type B: Maxilla grows more rapidly than mandible.
i. Opinion
ANB angle increases.
ii. Observations
iii. Type C: Mandible grows faster than maxilla. Decrease
iii. Ratings and rankings
in ANB angle.
Quantitative measurements:
These growth trends are helpful in planning retention for A scientific approach to study growth is based on ac-
individual orthodontic cases. curate measurements, which are of three types:
i. Direct data
Q.11. Growth centres.
ii. Indirect data
Ans. iii. Derived data
l Growth centre is a location or place where genetically Q.14. Enumerate the peak periods of postnatal growth.
controlled growth takes place. Ans.
l These are places of ossification with tissue-separating

force and they cause growth of the major part of the bone. [Same as SN Q.1]
l Centres of growth continue to grow when transplanted
Q.15. Enumerate various theories of growth.
to another area.
l Their response to external influence is less, whereas
Ans.
response to functional needs is more. [Ref LE Q.1]
l All growth centres are growth sites.

l The overall growth of the bone is controlled by a growth


Q.16. Growth spurts and two clinical importances.
centre. Ans.
Q.12. Differential growth. [Same as SN Q.1]
Ans. Q.17. Prepubertal growth spurt.
l Throughout life, human body does not grow at the same Ans.
rate, and different organs grow at different rates to a dif- [Same as SN Q.1]
ferent amount and at different times, this is known as
differential growth. Q.18. Scammon’s growth curves.
l The concept of differential growth is more clearly
Ans.
understood by two important aspects of growth:
a. Cephalocaudal gradient of growth [Same as SN Q.3]
b. Scammon’s cure of growth Q.19. Enumerate the various tissues for which Scammon’s
a. Cephalocaudal gradient of growth growth curves are plotted.
l An axis of increased growth gradient extending Ans.
from head towards the feet is called ‘cephalocau-
[Same as SN Q.3]
dal growth’.
l In fetal life, head constitutes 50% of total body Q.20. Cortical drift.
length, while limbs are primitive (30%).
Ans.
At the time of birth, head constitutes 25%–30%
and there is increased growth of body and limbs. [Same as SN Q.9]
Section | I  Topic-Wise Solved Questions of Previous Years 21

Topic 3
Growth and Development of Cranial
and Facial Structures
COMMONLY ASKED QUESTIONS
LONG ESSAYS:
1 . Describe in detail prenatal and postnatal growth of mandible.
2. Define growth and development. Discuss prenatal growth of maxilla.
3. Explain how maxilla increases in width, length and height.
4. Discuss prenatal and postnatal growth of mandible and its clinical implications in orthodontics. [Same as LE Q.1]
5. Define growth and development. Explain the postnatal growth of mandible. [Same as LE Q.1]
6. Explain in detail postnatal growth and development of maxilla. [Same as LE Q.3]

SHORT ESSAYS:
1 . Development of palate.
2. Describe the developmental defects of maxilla.
3. Postnatal growth and development of mandible. [Ref LE Q.1]
4. Spheno-occipital synchondrosis.
5. Sutural growth of maxilla.
6. Development of tongue.
7. Mechanism of bone growth.

SHORT NOTES:
1. Define synchondrosis.
2. Meckel’s cartilage.
3. Enumerate the mechanisms of bone growth.
4. Endochondral bone formation.
5. Development of palate.
6. Nasal septal cartilage.
7. Sutural growth of maxilla. [Ref LE Q.3]
8. Butler’s field theory.
9. How does infant mandible differ from adult mandible?
10. Enumerate types of synchondrosis. [Same as SN Q.1]

SOLVED ANSWERS
LONG ESSAYS:
Q.1. Describe in detail prenatal and postnatal growth of Prenatal growth phases are as follows:
mandible. i. Period of ovum (from fertilization to 14th day)
ii. Period of embryo (from 14th to 56h day)
Ans.
iii. Period of foetus (56th day to birth)
22 Quick Review Series for BDS 4th Year, vol 1

Prenatal growth of mandible:

1st structure to develop in primordium of lower jaw

Mandibular division of V nerve. (Function: presence of nerve is postulated as being necessary


to induce osteogenesis by production of neurotrophic factors.)

Followed by

Mesenchymal condensation forming the first arch (mandibular arch).

Mandible is derived from the ossification of osteogenic membrane


formed from ectomesenchymal condensation at around 36–38 days.

Resulting intramembranous bone lies lateral to Meckel’s cartilage of 1st arch


(mandibular arch).

At 6th week of IU life


In the region of bifurcation of inferior alveolar nerve into mental and incisive branches, a
single ossification centre for each half of the mandible arises.

During the 7th week of IU life


The bone begins to develop lateral to Meckel’s cartilage and continues until the
posterior aspect is covered by the bone.
Ossification stops at the point which will later become the mandibular ingual, from where
Meckel’s cartilage continues into middle ear and develops into auditory ossicles,
i.e. malleus and incus
The remaining part of the Meckel’s cartilage continues on its own to form the sphenomandibular
ligament and the spinous process of the sphenoid bone, which are remnants of it.

Between 8th and 12th weeks of IU life


There is marked acceleration in mandibular growth, as a result mandibular length increases,
and the external auditory meatus appears to move posteriorly.

Between 10th and 14th weeks of IU life


Secondary accessory cartilages appear to form the head of the condyle, part of the
coronoid process and mental protuberance. Soon the growing intramembranous ossification
fuses the coronoid process to the ramus.

The ossification of the ramus proceeds and the condyle is soon fused to the mandible at about 16 weeks.
Meckel’s cartilage does persist until as long as the 24th week of IU life before it disappears.
Section | I  Topic-Wise Solved Questions of Previous Years 23

[SE Q.3] l As the articulation of the condyle to the glenoid


fossa is constant, the anterior displacement
{Postnatal growth of mandible: causes displacement of the mandible anteriorly
l Among all the facial bones, the mandible undergoes the
as it grows posteriorly.
largest amount of postnatal growth, and exhibits the
l As the mandible grows anteriorly, the opening
largest variability in morphology.
of the mental foramen faces backwards so that
l Mandibular growth in the postnatal life shows the inte-
the neurovascular bundle leaves the foramen
gration of periosteal and capsular matrices of the func-
directed backwards.
tional matrix theory by Moss.
l There is corresponding surface remodelling at
l Capsular matrix involves the oropharyngeal functional
the anterior border with deposition in the pos-
spaces and the mandible grows according to the func-
terior surface of the symphysis and resorption
tional needs of a particular functional system. The pro-
in the superior part of the anterior surface and
cess of surface remodelling usually involves the activity
deposition in the inferior aspect.
of the periosteal matrix, i.e. muscle fibres.
Width:
Mandible at birth: l There is deposition in the lateral surface of the

l Mandible at birth is much smaller in size and varies in ramus and resorption on the lingual surface of
shape from the adult form. The infant mandible has a mandible below the mylohyoid ridge. In con-
short more or less horizontal ramus with obtuse gonial trast, the coronoid process undergoes apposition
angle. at the medial surface and resorption at the lateral
l The condyles are low and at the position of the occlusal surface. This expands the mandible like a V.
plane. The symphyseal suture has not ossified. l The condyle undergoes reduction of bone on

the lateral aspect of neck, and deposition cor-


Growth in the first year:
responding to the V principle makes the con-
l It involves growth at the symphyseal suture and lateral
dyle longer at the neck.
expansion in the anterior region to accommodate the
l Following the V principle, the inter-ramal dis-
erupting anterior teeth.
tance is efficiently increased by the growth of
l The mental foramen is directed at right angle to the
mandible, which helps the mandible to keep
surface of the corpus.
pace with the growth of the cranial base.
l There is increased bone deposition on the posterior sur-
l The mandible, which is often retrognathic in
face of the ramus of the mandible.
the newborn, assumes an orthognathic relation
l The infant mandible is suited for the suckling activity
with the maxilla during adulthood due to the
since the condyle and the glenoid fossa is flat, which
growth of the bone in length.
helps in the anteroposterior movement of the mandible.
l The condylar cartilage contributes little, if any,

Mandible in the adult/concept of V principle: to the growth and does not act as primary
The adult mandible differs from the mandible of an infant growth centre. In patients with ankylosis of the
in that TM joint, the mandible is found to grow to
l The ramus is longer and the gonial angle is less obtuse. normal length.
l The bone is larger on the whole and the condyle is well l The muscular processes of the mandible like

developed. angle, coronoid and condylar processes are


l All these changes take place in the growth of the man- under the influence of the periosteal matrix.
dible in the form of an expanding V. Height:
l Because of its horseshoe shape, it is easier to visualize l Alveolar process height increases well with

mandible as a V-shaped bone than the maxilla. eruption of teeth.


V principle of growth: l Bone deposition taking place in the lower bor-

According to this principle, growth of mandible in der of the mandible also contributes to increase
length, width and height is as follows: in the height of the mandible.}
Length: Rotation of mandible:
l The growth of the mandible in length antero- l Bjork used implants to study the growth

posteriorly is by the deposition of bone at the pattern of mandible and found that mandible
posterior border of the ramus and resorption at undergoes growth rotation. It was found that
the anterior surface, which helps to lengthen although mandible undergoes rotation, the
mandible so that the anterior part of the ramus effects seen are minimal due to external com-
is occupied by the posterior part of the body in pensation.
the future and accommodates the developing It was concluded that the growth of mandible is
permanent molars. largely influenced by the functional matrices, and the
24 Quick Review Series for BDS 4th Year, vol 1

condylar cartilage has little influence in its overall Stages of prenatal growth of craniofacial region are as
growth. follows:
Summary of mandibular growth: l The prenatal craniofacial growth develops in three

Length increases by: stages:


i. Surface apposition at posterior border of ramus and a. The period of the ovum from fertilization till
resorption at anterior border. second week.
ii. Deposition at bony chin. b. The period of embryo from 2nd to 8th week.
iii. Growth at condylar cartilage. c. The period of foetus from the 9th week till birth.
Height increases by: l The tissues of the face, both hard and soft tis-

i. Surface apposition at the alveolar border. sues, are of neural crest cell origin, also called
ii. Apposition at the lower border of mandible. ecto-mesenchymal origin.
iii. Growth at the condylar cartilage. l The neural crest cell gives rise to diverse struc-

Width increases by: tures both near the site of its origin and at the
i. Sutural growth up to 1st year postnatally. remote sites. In the head and neck region, the
ii. Later surface apposition at the outer surface. neural crest cells give rise to the facial processes,
Growth sites in mandible are as follows: the branchial arches and their cartilages, etc.
i. Mandibular condyle Frontonasal process:
ii. Posterior border of ramus l The head begins to take shape at around 21 days after
iii. Alveolar process conception. The migrating neural crest cells when
iv. Lower border of mandible encounter the lens placode, they form two streams:
v. Suture The anterior stream of cells forms the mesenchyme of
Q.2. Define growth and development. Discuss prenatal the frontonasal process and the posterior stream mi-
growth of maxilla. grates to form the structures of the branchial arches.
l Most of the development of the face takes place be-
Ans.
tween 3rd and 8th week of IUL. At around 4th week
Growth has been defined by various clinicians in different of IUL, the branchial arches begin to develop.
ways as follows: Branchial arches:
According to: l The branchial arches, developing during the late so-

J.S. Huxley – ‘The self-multiplication of living sub- mite period, are formed from the mesoderm of the
stance’. ventral foregut. There are five pairs of branchial
Krogman – ‘Increase in size, change in proportion and arches, the fifth being transitory.
progressive complexity’. l The first arch is the mandibular arch and the second

Todd – ‘An increase in size’. arch is the hyoid arch.


Meridith – ‘Entire series of sequential anatomic and The jaws of the face, i.e. maxilla and mandible, are de-
physiologic changes taking place from the beginning of rived from the first arch.
prenatal life to senility’. l Meanwhile, the frontonasal process of the forebrain
Moyers – ‘Quantitative aspect of biologic development just above the stomodeum develops bilateral thicken-
per unit of time’. ings called nasal placodes. In the middle, there is
Moss – ‘Change in any morphological parameter which invagination of placode to form nasal pits, on both
is measurable’. sides of them there are elevations, which are medial
l Development is defined in simple words as, ‘pro- and lateral nasal processes.
gression towards maturity’. Maxillary process:
l According to Melvin Moss, ‘Development can be l By around the 4th week of IUL, facial process arises
considered as a continuum of causally related from the first arch, which corresponds to mandibular
events from the fertilization of ovum onwards’. processes. Later, ventromedially two more swellings
Prenatal growth of maxilla: grow from the mandibular processes called the max-
l Maxillae, a pair of bones on either side of the middle illary processes.
third of the face, are formed by intramembranous l By about the 6th week, the processes of the face are

bone formation. easily discernible. The stomodeum is bound by the


l Due to its more cranial location, maxilla is ahead of frontonasal process above; the mandibular process
mandible in growth generally. below, and the sides being occupied by the maxillary
l The cephalocaudal gradient of growth of maxilla processes.
closely follows the neural growth curve in the Scam- l The maxillary process grows ventromedially towards

mon’s curves. the nasal processes.


Section | I  Topic-Wise Solved Questions of Previous Years 25

The maxillary process fuses with the lateral nasal pro- l A passive or secondary displacement of the naso-
cess and migrates medially to contact the inferolateral maxillary complex occurs in a downward and for-
side of the medial nasal process. ward direction as the cranial base grows. This is a
l The maxillary and the medial nasal processes are secondary type of displacement; the nasomaxillary
initially separated by the epithelial nasal fin, which complex is simply moved anteriorly as the middle
soon degenerates so that the mesenchyme of the two cranial fossa grows in that direction.
processes fuses. The maxillary and mandibular pro- l It is an important growth mechanism during the pri-
cesses fuse at the sides to form the cheek tissue. mary dentition period but becomes less important as
l The lateral nasal process forms the alae of the growth of cranial base slows down.
nose. The medial nasal process of both sides fuse to l Active translation takes place when the growth at the
form the globular process in the middle which gives tuberosity of the maxilla pushes the maxilla forward.
rise to the tip of the nose, columella, the philtrum, the A primary type of displacement is seen in a forward
labial tuberculum of the upper lip, the frenulum and direction which results in the whole maxilla being
the entire primary palate. carried anteriorly.
l The maxillary process forms the alveolus, which bears l The amount of this forward displacement equals the
teeth distal to the canines and the secondary palate. amount of posterior lengthening. This is a primary
l The area of fusion of the maxillary and mandibular type of displacement as the bone is displaced by its
processes forms the commissural corners of the own enlargement.
mouth. As the two processes grow towards the fellow
of the opposite side, the stomodeum is narrowed.
Ossification centres of maxilla:
l Maxilla develops by the intramembranous ossifica-
{SN Q.7}
tion. A primary ossification centre appears at about B. Growth at the sutures by connective tissue prolifera-
early 8th week at the termination of the infraorbital tion:
nerve just above the canine tooth lamina. There are l The maxillae articulate with the surrounding
two centres for each maxilla. bones of cranium and cranial base with the help of
l Secondary cartilages appear at the end of the 8th week the number of sutures like the zygomaticomaxil-
in the regions of the zygomatic and alveolar processes lary, frontomaxillary, pterygopalatine and zygo-
that ossify and fuse with the primary centre. maticotemporal sutures.
l Around 8th week, two ossification centres appear in l According to Sicher, growth at these paired paral-
the region of premaxilla on each side. The centres lel sutures will move the maxilla downward and
rapidly merge with the primary centres and are over- forward. It is only secondary and not a primary
shadowed by the growth of primary centres. mechanism.
l As growth of the surrounding soft tissue occurs,
Q.3. Explain how maxilla increases in width, length
the maxilla is carried downwards and forward,
and height?
leading to the opening up of space at sutural at-
Ans. tachments..

Postnatal growth of maxilla:


l The development and growth of maxilla is completed

early compared to the mandible. l New bone is now formed on the either side of the
l The growth of maxilla, especially in width, follows sutures, leading to the overall increase in size of
closely the neural growth curve more than the general bones on either side. Hence, a tension-related bone
growth curve in the Scammon’s curve. formation occurs at the sutures.
Growth of maxilla: C. Remodelling:
Growth of maxilla occurs by the following processes. l In addition to the growth occurring at the sutures,
A. Displacement or translation: simultaneous resorption and deposition move the
l Displacement or translation of a bone is the process surfaces of the maxilla while maintaining the integ-
by which specific local areas come to occupy new rity and basic shape of the bone.
actual positions in succession as the entire bone l Remodelling by bone deposition and resorption oc-
enlarges. It may be active or passive. curs to bring about:

l Maxilla is attached to the cranial base; hence, the (a) Increase in size

growth of the cranial base has a direct bearing on (b) Change in shape of bone
the nasomaxillary growth. (c) Change in functional relationship
26 Quick Review Series for BDS 4th Year, vol 1

l Maxillary growth matures first in width fol- bone deposition occurs on the endosteal sur-
lowed by the depth and the length. It would be face of the labial cortex and periosteal surface
easier to discuss the growth of maxilla in the of the lingual cortex, and the maxilla moves
same order. forward.
i. Maxillary width: l The zygomatic bone moves in a posterior di-

l The floor of the orbit faces superiorly, laterally rection to keep its relation with the maxilla.
and anteriorly. Growth proceeds in this direc- This happens by resorption in the anterior
tion by deposition and resorption on the lateral surface and deposition in the posterior sur-
surface of the orbital rim. face.
l Resorption occurs on the lateral surface of the iii. Maxillary height
orbital rim, leading to lateral movement of the l Bone resorption is seen on the floor of the

eye ball. To compensate, there is bone deposi- nasal cavity. To compensate, there is bone
tion on the medial rim of the orbit and on the deposition on the palatal side. Thus, a net
external surface of the lateral rim. downward shift occurs, leading to increase in
l Mid-palatal suture is active till 15 years, but it maxillary height.
cannot be generalized. Due to sutural growth, l As the teeth start erupting, bone deposition

there is bone fill in the mid-palatal area and occurs at the alveolar margins. This increases
resorption in the lateral aspect. vertical height of the maxilla and the depth of
l The entire wall of the sinus except the mesial the palate. This increase is seen as long as the
wall undergoes resorption. This results in in- teeth erupt. This contributes to early increase
crease in size of maxillary sinus. in the height of maxilla and accounts for about
l In the zygomatic process and the zygomatic 40% increase in the maxillary height.
arch, it is more complex. There is deposition
Maxillary growth can be summarized as follows:
on the posterior and lateral aspects and re-
l Length increases by – Sutural growth and the surface
sorption on the anterior and medial surfaces.
apposition at maxillary tuberosity.
Thus, the zygomatic bone moves in a poste-
l Width increases by – Growth at the median palatine
rior direction.
suture and apposition at zygomatic bone.
l The face enlarges in width by bone formation
l Height increases by – Sutural growth, surface appo-
on the lateral surface of the zygomatic arch and
sition and alveolar growth.
resorption on its medial surface.
l Growth sites in maxilla:
l The nasal part of the maxilla faces in an
i. Maxillary tuberosity
anterior, lateral and superior direction. The
ii. Sutures
growth proceeds in the same direction.
iii. Alveolar border
There is surface removal of bone from the
iv. Nasal septum
periosteum lining the inner aspect of the
v. Lateral walls
nasal cavity and deposition on the endosteal
surface. This allows the expansion of nasal Q.4. Discuss prenatal and postnatal growth of mandible
cavity. and its clinical implications in orthodontics.
ii. Anteroposterior/depth
Ans.
l Zygomatic bone moves posteriorly and later-

ally by deposition in the posterior and lateral [Same as LE Q.1]


surface and resorption in the medial surface.
Q.5. Define growth and development. Explain the post-
l Bone deposition occurs along the posterior
natal growth of mandible.
margin of the maxillary tuberosity. This causes
lengthening of the dental arch and enlargement Ans.
of the anteroposterior dimension of the entire
[Same as LE Q.1]
maxillary body. This helps to accommodate
the erupting molars. Q.6. Explain in detail postnatal growth and develop-
l The anterior nasal spine prominence increases ment of maxilla.
due to bone deposition. In addition, there is
Ans.
resorption from the periosteal surface of la-
bial cortex. As a compensatory mechanism, [Same as LE Q.3]
Section | I  Topic-Wise Solved Questions of Previous Years 27

SHORT ESSAYS: deposition on the palatal periosteal surface and re-


sorption on the nasal floor, the palate comes to oc-
Q.1. Development of palate. cupy a lower position.
Ans.
l When viewed in a cross section, the deposition of the
bone occurs along the whole of the periosteal surface
The palate is formed by contributions of the following: of the palate in such a way that the bone expands in
a. Maxillary process a lateral direction and also downwards.
b. Palatal shelves of the maxillary process
l The nasal floor due to resorption increases in volume
c. Frontonasal process and descends down from the level of infraorbital rim.
l The frontonasal process gives rise to the premaxil-

l In conjunction with the V principle, half of the external
lary region, while the palatal shelves form the rest of surface involved in this growth is depository and the
the palate. other half is resorptive, thus half of the bone tissue of
l As the palatal shelves grow medially, their union is
the palate is endosteal and the other half is periosteal.
prevented by the presence of tongue. Thus, initially
l The same concept is seen even in the anteroposterior
the developing palatal shelves grow vertically down- growth of the palate/midsagittal section of the palate.
wards towards the floor of the mouth. There is deposition on the palatal surface and resorp-
l During the 7th week of intrauterine life, withdrawal of
tion along the anterior surface/incisor area and supe-
tongue from palatal shelves aids in the transformation rior surface, expanding the palate like a V.
of their position from vertical to horizontal direction.
l By 8½ weeks of intrauterine life, two palatal shelves Q.2. Describe the developmental defects of maxilla.
are in close approximation with each other which are Ans.
initially covered by an epithelial lining. As they join,
the epithelial cells degenerate. The connective tis- The developmental defects of maxilla are as follows:
sues of the palatal shelves intermingle with each l The most prominent defect in the development of max-

other, resulting in their fusion. illa is the cleft lip, either unilateral or bilateral.
l The entire palate does not contact and fuse at the l The cleft lip can be complete or partial.

same time. Initially, contact occurs in the central re- l The most common is unilateral cleft lip (1 in 800 births).

gion of the secondary palate posterior to the premax- Bilateral cleft lip is rare and produces a protuberant, free
illa. From this point, closure occurs both anteriorly hanging middle part of the lip.
and posteriorly. l The cleft lip occurs due to failure of fusion of maxillary

l The mesial edges of the palatal processes fuse with the and medial nasal processes.
free lower end of nasal septum, and thus separate the l The failure of fusion of medial nasal processes produces

nasal cavities from each other and the oral cavity. the midline cleft, the ‘true hare lip’, which is exceed-
l Ossification of the palate occurs from the 8th week of ingly rare.
intrauterine life. The palate ossifies from a single cen- l Lateral facial cleft or oblique facial cleft is occasionally

tre derived from the maxilla. The most posterior part seen in the condition of failure of fusion of lateral nasal
of the palate does not ossify and it forms a soft palate. process with the maxillary process.
The mid-palatal suture ossifies by 12–14 years. l Cleft lip/palate and other facial clefts develop during the

l Postnatal growth of palate follows the concept of period of organ formation in the craniofacial develop-
expanding ‘V’ by Enlow. It is a more complex pro- ment. This approximates to about 28–55 days of IUL.
cess. It is one of the best examples of the expanding l Overfusion of maxillary and mandibular processes leads

V principle. to a small mouth, called microstomia.


l Growth at the suture by bone fill-in contributes more
Q.3. Postnatal growth and development of the mandible.
to the increase in width of the palate than remodel-
ling. The width of the palate also increases by the Ans.
growth of the alveolar process, which diverges out.
[Same as LE Q.1]
l Many bones or parts of the bone are in the form of V.

Bone deposition takes place on the inner side of V, Q.4. Spheno-occipital synchondrosis.
and resorption takes place on the outer surface.
Ans.
l If the outer surface of the expanding V is taken, then

the periosteal surface could be found to be lined with l ‘Synchondroses’ are defined as the bands of cartilage
osteoclasts, and endosteal surface is found to be present at the junction of various bones during the bone
lined with osteoblasts. formation stage.
l In a young child, the maxillary arch and the nasal l These synchondroses form important growth sites in the
floor are very close to the inferior orbital rim. By base of skull.
28 Quick Review Series for BDS 4th Year, vol 1

l Cranial base grows by cartilaginous growth in the syn- l Elongation of synchondroses in combination
chondroses which later gets calcified. with drift and remodelling contribute to cranial
Types of synchondroses: base lengthening.
They are classified into four subtypes:
Q.5. Sutural growth of maxilla.
i. Intersphenoidal synchondroses – occur at birth
ii. Intraoccipital synchondroses – occur at 3–5 years Ans.
iii. Spheno-occipital synchondroses – occur at 20 years
l The maxilla is connected to the cranium and cranial
iv. Sphenoethmoidal synchondroses – exact age is
base by a number of sutures. These sutures include:
not known
a. Frontonasal suture
Spheno-occipital Synchondrosis:
b. Frontomaxillary suture
l Spheno-occipital synchondroses are the cartilagi-
c. Zygomatico-temporal suture
nous junction between the sphenoid and the occipi-
d. Zygomatico-maxillary suture
tal bones and are active up to the age of 12–15 years
e. Pterygo-palatine suture
and become fused by the age of 20 years.
l These sutures are all oblique and more or less parallel to
l It is responsible for most of the lengthening of
each other. This allows the downward and forward repo-
cranial base between foramen magnum and sel-
sitioning of maxilla as growth occurs at these sutures.
laturcica and is the important growth site of cra-
l As growth of the surrounding soft tissue occurs, the
nial base during childhood.
maxilla is carried downwards and forward. This leads to
l Cranial base lengthening is contributed by elon-
the opening up of space at sutural attachments.
gation of synchondroses in combination with drift
l New bone is now formed on either side of the suture.
and remodelling.
Thus, the overall size of the bones on either side in-
l As endochondral bone growth occurs at the
creases. Hence, a tension-related bone formation occurs
spheno-occipital synchondrosis, the sphenoid and
at the sutures.
occipital bones move apart. At the same time, new
endochondral bone is laid down in the medullary Q.6. Development of tongue.
region and the cortical bone is formed in endos-
Ans.
teal and periosteal regions. Thus sphenoid and
occipital bones increase in length and width. i. At approximately 4 weeks, the tongue appears in em-
l Cartilage unlike bone is a pressure-adapted tissue bryo in the form of two lateral lingual swellings and
and can grow in heavy pressure areas, e.g. cranial one median swelling, the tuberculum impar. These
base. three swellings originate from the first pharyngeal arch
Clinical implications: (Fig. 3.1).
l Spheno-occipital synchondroses are responsi- ii. A second median swelling, the copula, or hypobran-
ble for most of the lengthening of cranial base chial eminence, is formed by mesoderm of the second,
between foramen magnum and sella turcica. third, and part of the fourth arch.
l It is the major contribution of endochondral iii. A third median swelling, formed by the posterior part
growth till 20 years. of the fourth arch.

Lateral lingual
swelling I Body of
tongue
Tuberculum impar I
Terminal
II sulcus

Copula (hypobranchial III Foramen cecum II


eminence) Palatine
tonsil
Epiglottal swelling
IV III
Laryngeal orifice Root of
IV tongue
Arytenoid swellings
V Epiglottis
V swelling

Fig. 3.1  Pharyngeal arches involved in the development of tongue.


Section | I  Topic-Wise Solved Questions of Previous Years 29

iv. As the lateral swellings increase in size, they overgrow the Displacement:
l
tuberculum impar and merge, forming the anterior two- It is the growth of the bone as a whole unit so
thirds or body of the tongue, sensory innervation to this that the bone is taken away from its articula-
area is by the mandibular branch of the trigeminal nerve. tion with other bones. It is otherwise called
v. The posterior part, or root, of the tongue originates from translation.
the second, third, and part of the fourth pharyngeal arch. l Displacement is of two kinds:
The sensory innervation to this part of the tongue sup- i. Primary or active:
plied by the glossopharyngeal nerve indicates that tissue Where the movement is due to the growth
of the third arch overgrows that of the second. of the bone itself, e.g. growth of maxilla at
vi. The extreme posterior part of the tongue is innervated tuberosity region
by the superior laryngeal nerve, reflecting its develop- ii. Secondary displacement:
ment from the fourth arch. It is the movement of one bone due to the
vii. Some of the tongue muscles probably differentiate in growth of other bones or translation within
situ, but most are derived from myoblasts originating the capsule. It is passive.
in occipital somites. Thus, tongue musculature is in- Example: Growth of maxilla due to growth
nervated by the hypoglossal nerve. at the spheno-occipital synchondrosis.
viii. The general sensory innervation of the tongue is easy
to understand:
l The body is supplied by the trigeminal nerve, the
SHORT NOTES:
nerve of the first arch. Q.1. Define synchondrosis.
l The root is supplied by the glossopharyngeal and

vagus nerves, the nerves of the third and fourth Ans.


arches, respectively. l ‘Synchondroses’ are defined as the bands of cartilages
l Special sensory innervation (taste) to the body of present at the junction of various bones during the bone
the tongue is provided by the chorda tympani formation stage.
branch of the facial nerve. l These synchondroses form important growth sites in the

Q.7. Mechanism of bone growth. base of the skull.


l Cranial base grows by cartilaginous growth in the syn-
Ans. chondroses which later gets calcified.
Mechanisms of bone growth: Types of synchondroses:
The growth of any bone follows certain basic processes These are classified into four subtypes:
like remodelling and growth movements caused by i. Intersphenoidal synchondroses – occur at birth
drifts and displacement. ii. Intraoccipital synchondroses – occur at
Remodelling: 3–5 years
l Remodelling is the differential growth activity
iii. Spheno-occipital synchondroses – occur at
necessary for bone shaping. It is not a uniform 20 years
process. The process of remodelling is differen- iv. Sphenoethmoidal synchondroses – exact age is
tial, e.g. if deposition takes place on the outer/ not known
periosteal surface of bone then resorption takes Q.2. Meckel’s cartilage.
place in the endosteal surface.
l Remodelling is the basic growth process provid-
Ans.
ing regional changes in the shape, dimensions and l The Meckel’s cartilage is the cartilage of the first bran-
proportions of bone. chial arch.
l It also performs regional adjustments in the bone l Meckel’s cartilage appears bilaterally as cartilaginous
to the changing functional demands. bars, the anterior aspect of these two cartilages ap-
Growth movements: proaches each other near the midline but do not fuse,
Two types of growth movements occur during the while posteriorly they terminate in a bulbous structure
enlargement of the cranial bones, they are drift and called malleus.
displacement. l Malleus and incus are derivatives of Meckel’s cartilage.
l Drift: l The part of the cartilage extending from the region of
It is the movement of the bone surface caused by the middle ear to mandible disappears but its sheath
deposition and resorption towards the depository forms the ‘anterior ligament of the malleus’ and the
surface. It is otherwise called transformation. ‘sphenomandibular ligament.’
30 Quick Review Series for BDS 4th Year, vol 1

Q.3. Enumerate the mechanisms of bone growth. (ii) The main part of the definitive palate is formed by
two shelf-like outgrowths from the maxillary promi-
Ans.
nences known as palatine shelves, which appear in
Mechanisms of bone growth: the 6th week of development and are directed
The growth of any bone follows certain basic processes obliquely downward on each side of the tongue.
like remodelling and growth movements. (iii) During 7th week, the palatine shelves ascend to at-
Remodelling: tain a horizontal position above the tongue, and fuse
l Remodelling is the differential growth activity neces- to form the secondary palate.
sary for bone shaping. It is not a uniform process. (iv) Anteriorly, the shelves fuse with the triangular pri-
l It is the basic growth process providing regional changes mary palate, and the incisive foramen is the midline
in the shape, dimensions and proportions of bone. landmark between the primary and secondary plates.
l It also performs regional adjustments in the bone to (v) At the same time as the palatine shelves fuse, the
the changing functional demands. nasal septum grows down and joins with the cephalic
Growth movements: aspect of the newly formed palate.
l There are two types of growth movements, namely
Q.6. Nasal septal cartilage.
(a) Drift
It is the movement of the bone surface caused by Ans.
the deposition and resorption towards the deposi-
tory surface. It is otherwise called transformation. l Nasal septum appears to be the primary factor in the
(b) Displacement displacement of nasomaxillary complex.
l During prenatal period, nasal septal cartilage lies behind
It is the growth of the bone as a whole unit so that
the bone is taken away from its articulation with the cranial base cartilages and is attached in front and
other bones. It is otherwise called translation. below to premaxillary bone as well as vomer, and pos-
l Displacement is of two kinds:
teriorly to mesethmoid cartilage.
l During postnatal development, perpendicular plate of
(i) Primary or active displacement
(ii) Secondary displacement ethmoid ossifies from mesethmoid cartilage at birth.
l The perpendicular plate of ethmoid fuses with cribri-
Q.4. Endochondral bone formation. form plate of ethmoid at 3 years of age. With this, the
Ans. cranial components are fused with the facial bony ele-
ments.
l Endochondral bone is formed in a cartilaginous model; l Anterior part of the nasal septum remains cartilaginous
osteoblasts invade cartilage and replace it. throughout life.
l The first step during endochondral bone formation is the l After the union of cranial and facial bony components,
differentiation of mesenchymal cells into chondrocytes. the nasal septum has a very little effect on facial growth.
These chondrocytes form a rough model which is
enclosed by perichondral cells. Q.7. Sutural growth of maxilla.
l Cartilage mass grows by both interstitial growth and
Ans.
apposition. Cartilage cells hypertrophy and their matrix
begins to get calcified. [Ref LE Q.3]
l The blood vessels penetrate the cartilage mass from the
Q.8. Butler’s field theory.
perichondrium during this time.
l These penetrating blood vessels carry undifferentiated Ans.
mesenchymal cells along with them, which get con-
l The human dentition is divided into four fields: incisor,
verted into osteoblasts.
canine, premolar and molar.
l Osteoblasts subsequently deposit bone and bone spicules are
l The most distal tooth in each field is more susceptible to
formed. Gradually the cartilage mass is replaced by bone.
changes or variations, which include absence of tooth,
l Endochondral bone formation is usually seen in the
and variation in size, shape and structure. This is called
areas where there are high levels of compression.
‘Butler’s field theory’.
Example: cranial base and movable joints
Example: Lateral incisors, second premolars and third
Q.5. Development of palate. molars are the most variable teeth in their group.
l Canine is the least variable tooth in the arch.
Ans.
l Butler’s field theory does not apply in lower anterior

(i) The primary palate is derived from the intermaxillary region, where mandibular central incisor is more com-
segment. monly missing than lateral incisor.
Section | I  Topic-Wise Solved Questions of Previous Years 31

Q.9. How does infant mandible differ from adult mandible? l The bone is larger on the whole and the condyle is
well developed.
Ans.
l All these changes take place in the growth of the

Infant mandible: mandible in the form of an expanding V.


The infant mandible has a short, more or less horizontal
ramus with obtuse gonial angle. The condyles are low Q.10. Enumerate types of synchondrosis.
and at a position of occlusal plane. Ans.
The adult mandible differs from the mandible of an infant in that:
l The ramus is longer and the gonial angle is less obtuse. [Same as SN Q.1]

Topic 4
Development of Dentition and Occlusion
COMMONLY ASKED QUESTIONS
LONG ESSAYS:
1 . Discuss the development of occlusion and its significance.
2. Describe the development of dentition and intra-arch tooth relationships from 6th year to 12th year of a child’s
life. [Same as LE Q.1]
3. Define normal occlusion. Describe normal occlusion in deciduous dentition and its further development till the
age of 12 years. [Same as LE Q.1]

SHORT ESSAYS:
1. Space of Nance.
2. Describe briefly the development of dentition and occlusion.
3. Incisal liability.
4. Anthropoid spaces.
5. Transient malocclusions.
6. Andrew’s keys to normal occlusion.
7. Ugly duckling stage.
8. Terminal planes. [Ref LE Q.1]
9. Curve of Spee.
10. Gum pads.
11. Leeway space of Nance. [Same as SE Q.1]
12. Physiological spaces. [Same as SE Q.4]
13. Write briefly about Roth’s keys of occlusion. [Same as SE Q.6]
14. Flush terminal plane. [Same as SE Q.8]

SHORT NOTES:
1. Gum pads.
2. Primate spaces.
3. Incisor liability. [Ref SE Q.3]
4. Ugly duckling stage. [Ref SE Q.7]
5. Leeway space. [Ref SE Q.1]
6. Distal flush terminal plane. [Ref LE Q.1]
7. Flush terminal plane. [Ref LE Q.1]
8. Theories of eruption.
9. Sequence of eruption of permanent teeth. [Ref LE Q.1]
10. Transient malocclusion. [Ref SE Q.5]
11. Andrew’s keys to normal occlusion. [Ref SE Q.6]
32 Quick Review Series for BDS 4th Year, vol 1

1 2. Enumerate the stages of eruption of normal occlusal development.


13. Centric relation.
14. Roth’s keys of functional occlusion. [Ref SE Q.6]
15. Premature loss of deciduous teeth.
16. Enumerate the stages of tooth development.
17. Leeway space of Nance. [Same as SN Q.5]
18. Transitional period. [Same as SN Q.10]
19. Enumerate few self-correcting malocclusions. [Same as SN Q.10]
20. Enumerate Andrew’s six keys to normal occlusion. [Same as SN Q.11]

SOLVED ANSWERS
LONG ESSAYS:
Q.1. Discuss the development of occlusion and its
significance.
Ans.

Predental period Deciduous dentition Mix dentition period Permanent dentition period
(Birth–6 months) period (6–2½ to 3½ years) (6–12 years) (starts after shedding of
last primary tooth)

3 phases

i. Molar relation
Gum pads and i.Spring of anterior teeth First transition Intertransition Second ii. Eruption sequence
their relationship ii.Primate/simion spaces. period period transition period
iii.Shallow overjet/overbite
iv. Vertical inclination of i. Emergence of
anterior teeth 6 6 Incisal liability
v. Deep bite (early stages) 6 6
vi. Flush terminal plane
ii. Replacement of Ugly duckling stage
incisors A with 1
Early mesial shift

Leeway space
iii. Replacement
of CDE Late mesial shift

Change of terminal planes

Occlusal development can be divided into the following


periods or stages:
Predental period:
l This period extends from birth to 6 months of age after

birth, i.e. 0–6 months. Lateral sulcus

l The neonate is without teeth for about 6 months of life.

The alveolar arches of an infant during this period are Dental groove
known as gum pads.
Features of gum pads are as follows Fig 4.1: Gingival groove
i. They are pink in colour and firm in consistency cov-
ered by dense layer of fibrous periosteum. Fig. 4.1  Maxillary gum pad.
Section | I  Topic-Wise Solved Questions of Previous Years 33

ii. They are horseshoe shaped and develop in the fol- anthropoid spaces. These spaces help in the place-
lowing two parts: ment of canine cusps of the opposing arch.
(a) Labiobuccal portion iii. Shallow overjet and overbite
(b) Lingual portion iv. Ovoid arch form
Dental groove: v. Almost vertical inclination of anterior teeth
These two portions are separated by a groove vi. Flush terminal plane:
called the dental groove. The mesiodistal relation between the distal surfaces
Transverse grooves: of the upper and lower second deciduous molars (E)
iii. The gum pads are divided into 10 segments by is called the terminal plane.
transverse grooves. Each segment consists of one A normal feature of deciduous dentition is a flush
developing deciduous tooth sac. terminal plane, where the distal surfaces of the up-
Lateral sulcus Fig 4.1: per and lower second deciduous molars are in the
iv. The transverse groove between the canine and first same plane.
deciduous molar segment is called the lateral sulcus. vii. Deep bite:
The lateral sulcus of mandibular arch is normally The deep bite occurs in the initial stages of devel-
more distal than that of maxillary arch. opment and is accentuated by the more upright de-
The lateral sulci are useful in judging the interarch ciduous incisors compared to their successors.
relationship of maxilla and mandible at a very early This deep bite is reduced later due to:
stage. (a) Eruption of deciduous molars
v. Upper and lower gum pads are almost similar to (b) Attrition of incisors
each other. (c) Forward movement of the mandible due to
Relationship of gum pads: growth
l When upper and lower gum pads are approximated,
The mixed dentition period:
there is a complete overjet all around, as the upper
This period ranges from 6 to 12 years of age.
gum pad is wider and longer than lower gum pad.
This period can be divided into three phases:
l Mandibular lateral sulci are posterior to maxillary
(a) First transitional period
ones.
(b) Intertransitional period
l Class II pattern is exhibited as maxillary gum pad
(c) Second transitional period
being more prominent.
l Anterior open bite: This infantile open bite is consid- (A) First transitional period:
ered normal. Contact occurs between upper and It is characterized by the following:
lower gum pads in first molar region and a space ex- (i) Emergence of first permanent molars
ists between them anteriorly, known as infantile open (ii) Exchange of deciduous incisors with permanent
bite, which helps in sucking. incisors.
The Deciduous Dentition Period:
The deciduous dentition period extends from 6 months (i) Emergence of first permanent molars:
to 6 years of postnatal life. It starts with the eruption of l Mandibular first molar is the first permanent tooth
deciduous mandibular central incisors and completes to erupt at around 6 years of age.
with second deciduous molars coming into occlusion. l The distal surface of the second deciduous molar,
The eruption of all primary teeth is completed by 2½ – i.e. (E) guides the first permanent molars into the
3½ years of age. dental arch.
The normal features of an ideal occlusion in the primary l The location and the relationship of first perma-
dentition are as follows: nent molars depend much on the distal surface
i. Spacing of anterior teeth: relationship between upper and lower second
Spaces existing between the deciduous teeth,
deciduous molars E
called physiological or developmental spaces, are E
important for normal development of permanent
(SN Q.6 and SE Q.8)
dentition.
ii. Primate/anthropoid/simian spaces: l {(The distal surface relationship between the up-
These physiological spaces are present invariably on per and lower second deciduous molars can be of
mesial side of maxillary canines and distal side of three types:
mandibular canines. a. Flush terminal plane (76%)
As these spaces are commonly seen in primates. b. Mesial step terminal plane (14%)
They are known as primate spaces, simian spaces or c. Distal step terminal plane (10%))}
34 Quick Review Series for BDS 4th Year, vol 1

a. Flush terminal plane: In this type of relationship:


l Distal surface of mandibular second decidu-
ous molar ) )
E E
is more mesial than that
6 E D of maxillary second deciduous molar

) )
E E .

l The permanent molars erupt directly into


6 E D the Angle’s class I occlusion.
If forward growth of mandible
Fig. 4.2  Flush terminal plane.
Persists Minimal

(SN Q.7 and SE Q.8)


l {(The distal surface of upper and lower second Leads to Establishes
deciduous molars is in one vertical plane. This
type of relationship is called flush or vertical ter-
Angle’s class III molar Angle’s class I molar
minal plane relationship, which is a normal fea- relationship relationship
ture of deciduous dentition.
l The erupting first permanent molars may also be

in a flush or end on relationship which shifts to {SN Q.6}


class relation by
c. Distal step terminal plane:
a. Early shift
b. Late shift)}
l Early shift:

Occurs during early mixed dentition period, 6 E D


where eruptive force of the first permanent molar
is sufficient to push the deciduous first and second
molars forward to close primate spaces and estab-
6 E D
lish class I molar relationship.
l Late shift:

Occurs in the late mixed dentition period. In chil-


dren lacking primate spaces, the erupting perma- Fig. 4.4  Distal step terminal plane.
nent first molars drift mesially, utilizing the Lee-
way space, when deciduous second molars
In this type of relationship, the distal surface of E E is
exfoliate.
b. Mesial step terminal plane: more distal to that of E E .

Erupting permanent molars assume Angle’s class II


occlusion here.}

6 E D
(ii) The exchange of incisors:
The deciduous incisors are replaced by permanent
incisors during first transition period.
6 E D The mesiodistal width of permanent incisors is
larger than deciduous teeth that they replace.
Incisal liability is the difference between the
Fig. 4.3  Mesial step terminal plane. amount of space needed for accommodation of
Section | I  Topic-Wise Solved Questions of Previous Years 35

the incisors and the amount of space available for


{SN Q.9}
them to occupy. It was described by Warren
Mayne in 1969. The Permanent Dentition Period:
This period extends from shedding of last primary
tooth and eruption of all permanent teeth.
Incisal liability The frequently seen eruption sequence of the perma-
nent dentition is as follows:
l Maxillary arch 6-1-2-4-3-5-7
7 mm in maxillary arch 5 mm in mandibular arch or
6-1-2-3-4-5-7
Utilization of inter-dental spaces seen l Mandibular arch 6-1-2-3-4-5-7
in primary dentition or
Incisal liability can 6-1-2-4-3-5-7)
Increase in inter-canine width
be overcome by

Q.2. Describe the development of dentition and intra-


Change in incisor inclination arch tooth relationships from 6th year to 12th year of a
child’s life.
Ans.
(B) Intertransitional period:
[Same as LE Q.1]
l It is relatively stable and no changes occur during

this phase to mixed dentition. Q.3. Define normal occlusion. Describe normal occlu-
l The maxillary and mandibular arches consist of sets sion in deciduous dentition and its further development
of deciduous and permanent teeth during this period. till the age of 12 years.
(C) Second transitional period: Ans.
i. Replacement of deciduous molars and canines by
[Same as LE Q.1]
premolars and permanent cuspids, respectively, is
a characteristic of this phase.
ii. The Leeway space of Nance is the excess space SHORT ESSAYS:
available after the exchange of deciduous molars
Q.1. Space of Nance.
and canines with permanent teeth. It is utilized for
mesial drift of mandibular molars to establish class Ans.
I molar relation.
iii. Ugly duckling stage (7–11 years of age): Also {SN Q.5}
known as Broadbent’s Phenomena, it is a tran-
i. The combined mesiodistal width of permanent ca-
sient or self-correcting malocclusion seen in
nines and premolars is usually less than deciduous
maxillary incisor region, particularly during
canines and molars which they replace. This surplus
eruption of permanent canines.
space is called the Leeway space of Nance.
During eruption of permanent canines, they im-
ii. The Leeway space is greater in mandibular arch
pinge on roots of lateral incisors displacing
compared to maxillary arch.
them mesially, which in turn results in trans-
In maxillary arch n 1.8 mm (0.9 mm per side of the
mission of force onto the roots of central inci-
arch)
sors, which also get displaced mesially.
In mandibular arch n 3.4 mm (1.7 mm on each size
A resultant distal divergence of crowns of two cen-
of arch)
tral incisors causes a midline diastema.
iii. This space is utilized for the establishment of class I
This situation has been described by Broadbent as
molar relation by facilitating mesial drift of man-
Ugly Duckling stage as children tend to look ugly
dibular molars.
during this phase.
36 Quick Review Series for BDS 4th Year, vol 1

Q.2. Describe briefly the development of dentition and


occlusion.
Ans.
Development of dentition and occlusion is as follows:
DEVELOPMENT OF DENTITION AND OCCLUSION

PREDENTAL PERIOD DECIDUOUS DENTITION MIX DENTITION PERIOD PERMANENT DENTITION PERIOD
(0–6 months birth) (Period: 6–2½ to 3½ years) (6 years–12 years) (Shedding of last primary tooth and
(6 months–6 years) eruption of all permanent teeth)
(complete deciduous teeth
3 phases
formation is 6 months –
2½–3½ years)
Eruption
GUMPADS: Sequence
First transition Inter-transition Second
Alveolar arches Maxilla:
period period transition period
without teeth, 6-1-2-4-3-5-7
horseshoe Mandible:
shape, pink 6-1-2-3-4-5-7
Normal Features: i. Emergence of Inter-transition i. Characterized
and firm i. Spacing of anterior teeth period by
consistency 6 6
(physiologic/development 6 6 CD Ereplacement
spaces) by 345
ii. Primate/anthrapoid/simion Exchange of A with 1.
ii. Location and Stable and ii. Leeway space
spaces located C↓ No change
relation of of Nance CDE-345:
↓C
iii. Shallow overjet and 6 6 1.8
overbite 6 6 3.4
iv. Ovoid arch form depends on iii. Ugly duckling
v. Vertical inclinaton of terminal plane stage (8–9 years)
anterior teeth relation of E’s. eruption of
vi. Flush terminal plane iii. 3 types of terminal planes
3 3
vii. Deepbite in early a. Flush terminal plane
stages b. Mesial terminal plane
c. Distal terminal plane
iv. Exchange of incisors
Incisal stability:
Maxillary -7mm
Mandibular-5mm

Q.3. Incisal liability. This space discrepancy is compensated by the following


three mechanisms:
Ans. i. Increased intercanine width:
During the period of permanent incisor eruption,
significant amount of increase in intercanine arch
{SN Q.3} width occurs. It is about 3 to 4 mm.
ii. Interdental spacing:
l Incisal liability was described by Warren Mayne in Spacing present in primary dentition helps in align-
1969. ment of incisors. The primate space present in the
l The mesiodistal width of permanent incisors is larger
upper arch mesial to primary canine is also used.
compared to primary incisors. iii. Labial eruption of incisor:
l For proper alignment of anterior teeth, the erupting
Deciduous incisors stand upright. The permanent
permanent incisors require more space. incisors, which replace them, are proclined labially,
l This difference between the amount of space needed
placing them in a wider arch.
for the incisors and the amount available for them is
called the ‘incisal liability’. Q.4. Anthropoid spaces.
l A favourable incisal liability exists when the primary Ans.
dentition is an open dentition, whereas an unfavour-
l Generalized spaces present in the deciduous dentition
able situation exists in closed dentition.
which plays important role in the normal development
l The incisal liability is about 7.6 mm in maxillary
of permanent dentition are called physiological spaces
arch and 6 mm in mandibular arch.
or developmental spaces
Section | I  Topic-Wise Solved Questions of Previous Years 37

l The anteroposterior growth of jaws is the reason for C. Mixed dentition


physiological spaces. Absence of spaces indicates de-
velopment of crowding in permanent dentition. Transient malocclusion Reason for correction
l Primary dentition in which interdental spaces are pres- i. Deep bite Physiological bite raisers at
ent is called open dentition or spaced dentition. 6 and 12 years with the
eruption of first and second
l For the normal development of permanent dentition,
permanent molars. The over-
presence of spaces in primary dentition is important. lying gingival pad of tissue
l Invariable spacing exists mesial to maxillary canines will act as bite raiser
and distal to mandibular canines. These physiological ii. Ugly duckling stage Eruption of maxillary canine
spaces are called primate spaces or simian spaces or
anthropoid spaces as they are seen commonly in pri- iii. Lower anterior crowding Increase in intercanine width
mates. These spaces help in the placement of canine iv. End on molar relationship Late mesial shift
cusps of opposing arch. v. Flush terminal plane Both late mesial shift and
l Absence of spaces in primary dentition is an indication differential jaw growth
that crowding of teeth may occur when larger perma-
nent teeth erupt.
l Primate spaces are used in early mesial shift.

Q.5. Transient malocclusions. Q.6. Andrew’s keys to normal occlusion.

Ans. Ans.

{SN Q.10} {SN Q.11}

l Transient malocclusions are also known as self- There are two keys of occlusion:
correcting malocclusions or transitional malocclusions. i. Andrew’s keys of static occlusion
l Transient malocclusions are those conditions which ii. Roth’s keys of functional occlusion
will look like malocclusion at some particular time. i. Andrew’s keys of static occlusion
But with normal growth, the condition gets corrected Key 1: Molar relation:
on its own without any treatment. l Mesiobuccal cusp of maxillary first molar rests

Transient malocclusions are observed in various in the mesiobuccal groove of mandibular first
stages of occlusal development as follows: molar.
l Distal surface of the distobuccal cusp of maxil-
A. Stage of predental jaw relation lary first molar should occlude with mesial
Existing transient surface of the mesiobuccal cusp of mandibular
malocclusions Correction mechanism second molar.
i. Retrognathic mandible Gets corrected with cephalo- l Mesiolingual cusp of the maxillary first molar

caudal growth and differential should occlude in the central fossa of mandibu-
growth of mandible lar first molar.
ii. Complete overlap of Transverse and sagittal growth Key 2: Crown angulations (tip):
maxillary gum pad of mandibular gum pad l The gingival portion of the long axis of each

iii. Anterior open bite Eruption of primary incisors crown should be distal to the incisal portion;
this is known as crown angulation.
iv. Infantile swallow With initiation of function at
l Measured by the inclination of long axis of the
about 18 months of age
crown to a line perpendicular to the occlusal plane.
B. Stage of primary dentition Key 3: Crown inclination (torque):
l The buccolingual inclination of the long axis
Type of transient
of the crown, not the long axis of the entire
malocclusion present Correction mechanism
tooth is known as crown inclination.
i. Deep bite Eruption of primary molars l Negative crown inclination or lingual crown

ii. Increased overjet Attrition of incisal edges inclination occurs in maxillary and mandibular
iii. Flush terminal plane More forward growth of mandible posteriors whereas positive or labial inclina-
tion is seen in maxillary incisors.
iv. Spacing Early mesial shift
Key 4: Rotation:
v. Edge to edge at Closes with eruption of perma- l The resulting angle between the line perpen-
about 6 years of age nent successors shedding of dicular to the occlusal plane and one tangent to
primary incisors and eruption of
the middle of the labial or buccal clinical
permanent incisors
crown is known as rotation.
38 Quick Review Series for BDS 4th Year, vol 1

l Absence of rotation. incisors gets closed automatically as the canine


l Arch should be devoid of any rotated tooth. comes into occlusion.
l A rotated molar occupies more mesiodistal l This stage is called ugly duckling stage because it

space, creating a situation unreceptive to nor- represents a metamorphosis from an unaesthetic


mal occlusion, while a rotated incisor occupies phase to an aesthetic phase.
less space.
Key 5: Interproximal contact:
Clinical significance:
l Proximal contacts should be tight and no spac-
l As a guideline, spontaneous closure of maxillary
ing should be present.
midline diastema up to 2 mm is more likely, while
Key 6: Curve of Spee:
total closure of a median diastema greater than 2 mm
l Deep curve of Spee results in crowding.
is unlikely.
l Flat curve of Spee is most receptive for normal
l During the ugly duckling stage, any attempt to close
occlusion.
the median diastema will be hazardous. Apex of lat-
l Reverse curve of Spee results in spacing.
eral incisors will be damaged and canine may be
deflected from its normal path of eruption.
{SN Q.14} Q.8. Terminal planes.
ii. Roth’s keys of functional occlusion Ans.
Key 1: Coincidence of intercuspal position (ICP) and
[Ref LE Q.1]
retruded contact position (RCP).
Key 2: Maximum and stable cusp to fossa contacts Q.9. Curve of Spee.
throughout the buccal segments.
Ans.
Key 3: Disclusion of posterior teeth in mandibular
protrusion by even contacts on incisors. l Curve of Spee refers to the anteroposterior curvature of
Key 4: Lateral movements of the mandible are occlusal surfaces beginning at the tip of lower cuspid, and
guided by the working side canines, with disclusion following the cusp tips of bicuspids and molars continu-
of all other teeth on both working and non-working ing as an arc through the condyle. If the curve is ex-
sides. tended, it would form a circle of about 4-inch diameter.
l The curve results from variations in axial alignment of

lower teeth. The long axis of each lower tooth is aligned


Q.7. Ugly duckling stage.
nearly parallel to its individual arc of closure around the
Ans. condylar axis. This requires a gradual progressive in-
creased mesial tilting of teeth towards molars which
creates the curve of Spee.
{SN Q.4}
l Deep curve of Spee results in crowding.
l Ugly duckling stage is also known as Broadbent’s l Flat curve of Spee is most receptive for normal occlusion.
phenomena or physiological median diastema. l Reverse curve of Spee results in spacing.
l Ugly duckling stage is a transient form of malocclu-
Q.10. Gum pads.
sion, wherein midline diastema is present between
maxillary central incisors. It is commonly seen be- Ans.
tween 7 and 11 years of age.
l Neonates are without teeth for about 6 months of life.
l During the eruption stages of canine, it will be im-
The alveolar arches in infants during this period are
pinging on the roots of lateral incisors, and the re-
known as gum pads.
sulting pressure causes the lateral incisor to erupt
The features of gum pads are as follows:
into oral cavity with divergence of crown distally.
i. They are pink in colour and firm in consistency
l Even after the lateral incisor erupts fully, this pres-
covered by dense layer of fibrous periosteum.
sure effect from the erupting canine persists and is
ii. They are horseshoe shaped and develop in two
also transmitted to the central incisors, which results
parts:
in the divergence of crowns and convergence of roots
(a) Labiobuccal portion
towards midline. This bilateral effect causes a tempo-
(b) Lingual portion
rary midline diastema.
Dental groove
l This temporary spacing that occurs between central
These two portions are separated by a groove called
incisors and sometimes between central and lateral
the dental groove.
Section | I  Topic-Wise Solved Questions of Previous Years 39

Q.14. Flush terminal plane.


Ans.
Lateral sulcus
[Same as SE Q.8]

Dental groove
SHORT NOTES:
Q.1. Gum pads.
Gingival groove
Ans.
Fig. 4.5  Maxillary gum pad. i. The alveolar arches at the time of birth are called gum
pads and are firm and pink.
Transverse grooves ii. Maxillary gum pads develop in two parts, namely
iii. The gum pads are divided into 10 segments by labiobuccal and lingual; these are demarcated by dental
transverse grooves. Each segment consists of groove. Labiobuccal part grows fast and is divided into
one developing deciduous tooth sac. 10 segments by transverse grooves, which correspond
Lateral sulcus to the deciduous tooth sac.
iv. Transverse groove between the canine and the iii. The groove between canine and deciduous first molar is
first deciduous molar segment is called the lat- called lateral sulcus.
eral sulcus. iv. Gingival groove demarcates palate from gum pads.
The lateral sulcus of mandibular arch is normally v. Lower gum pads are V-shaped, and are similar to maxil-
more distal than that of maxillary arch. lary gum pads, but the segments are less defined when
The lateral sulci are useful in judging the inter- compared with maxillary gum pad.
arch relationship of maxilla and mandible at vi. Gum pads relationship is arbitrary, as they do not have
every early stage. definite relationship. When upper and lower gum pads
v. Upper and lower gum pads are almost similar to are approximated, there is a complete overjet all around.
each other. Class II pattern is exhibited and anterior open bite, this
Relationship of gum pads infantile open bite is considered normal.
l When upper and lower gum pads are approxi-
Q.2. Primate spaces.
mated, there is a complete overjet all around, as
the upper gum pad is wider as well as longer than Ans.
the lower gum pad.
l Generalized spaces present in the deciduous dentition
l Mandibular lateral sulci are posterior to maxillary
which plays an important role in the normal develop-
ones.
ment of permanent dentition are called primate spaces,
l Class II pattern is exhibited as maxillary gum pad
physiological spaces or developmental spaces.
being more prominent.
l The anteroposterior growth of jaws is the reason for
l Anterior open bite: This infantile open bite is
physiological spaces. Absence of spaces indicates de-
considered normal. Contact occurs between upper
velopment of crowding in permanent dentition.
and lower gum pads in the first molar region and
l Invariably, spacing exists mesial to the maxillary ca-
a space exists between them anteriorly, known as
nines and distal to the mandibular canines. These physi-
infantile open bite, which helps in sucking.
ological spaces are called primate spaces or simian
Q.11. Leeway space of Nance. spaces or anthropoid spaces as they are seen commonly
in primates. These spaces help in the placement of ca-
Ans.
nine cusps of opposing arch.
[Same as SE Q.1] l Primate spaces are used in early mesial shift.

Q.12. Physiological spaces. Q.3. Incisor liability.


Ans. Ans.
[Same as SE Q.4] [Ref SE Q.3]
Q.13. Write briefly about Roth’s keys of occlusion. Q.4. Ugly duckling stage.
Ans. Ans.
[Same as SE Q.6] [Ref SE Q.7]
40 Quick Review Series for BDS 4th Year, vol 1

Q.5. Leeway space. Q.13. Centric relation.


Ans. Ans.
[Ref SE Q.1] l Centric relation is the relation of mandible to maxilla
when the mandibular condyles are in the most superior
Q.6. Distal flush terminal plane.
and retruded position in glenoid fossa with the articular
Ans. disc properly interposed.
l Centric relation is also called ligamentous position or
[Ref LE Q.1]
terminal hinge position.
Q.7. Flush terminal plane. l At centric relation position, both condyles are simulta-

neously seated most superiorly and far back in un-


Ans.
strained position in respective glenoid fossa.
[Ref LE Q.1]
Q.14. Roth’s keys of functional occlusion.
Q.8. Theories of eruption.
Ans.
Ans.
[Ref SE Q.6]
l The exact mechanism by which the tooth erupts is still
Q.15. Premature loss of deciduous teeth.
unclear. Cross-linking of maturational fibres of periodon-
tal ligament provides force for eruption. This seems to be Ans.
a contemporary view.
Premature or early loss of deciduous teeth causes:
Various theories of eruption are as follows:
i. Migration of adjacent teeth into the space that pre-
l Pulp theory
vents eruption of successors.
l Vascular theory
ii. Premature loss of deciduous teeth leads develop-
l Root elongation theory
ment of malocclusion.
l Alveolar bone growth theory
iii. Loss of deciduous second molar can cause marked
l Genetic theory
forward shift of permanent first molar, thereby
l Follicular theory
blocking eruption of second premolar, which gets
l Hammock ligament theory
impacted or deflected to abnormal position.
Q.9. Sequence of eruption of permanent teeth.
Q.16. Enumerate the stages of tooth development.
Ans.
Ans.
[Ref LE Q.1]
Development of tooth was divided into 10 stages by Nolla
Q.10. Transient malocclusion. as follows:
i. Stage 1 – presence of crypt
Ans.
ii. Stage 2 – initial calcification
[Ref SE Q.5] iii. Stage 3 – one-third of crown completed
iv. Stage 4 – two-thirds of crown completed
Q.11. Andrew’s keys to normal occlusion.
v. Stage 5 – crown almost completed
Ans. vi. Stage 6 – crown completed
vii. Stage 7 – one-third of root completed
[Ref SE Q.6]
viii. Stage 8 – two-thirds of crown completed
Q.12. Enumerate the stages of eruption of normal ix. Stage 9 – root almost completed with open apex
occlusal development. x. Stage 10 – apical end of the root completed
Ans. Q.17. Leeway space of Nance.
Occlusal development can be divided into the following Ans.
periods or stages:
[Same as SN Q.5]
i. Pedental period: birth – 6 months
ii. Deciduous dentition period: 6–2½ to 3½ years Q.18. Transitional period.
iii. Mix dentition period: 6–12 years
Ans.
iv. Permanent dentition period: starts after shedding of
last primary tooth [Same as SN Q.10]
Section | I  Topic-Wise Solved Questions of Previous Years 41

Q.19. Enumerate few self-correcting malocclusions. Q.20. Enumerate Andrew’s six keys to normal occlusion.
Ans. Ans.
[Same as SN Q.10] [Same as SN Q.11]

Topic 5
Functional Development
COMMONLY ASKED QUESTIONS
LONG ESSAYS:
1. Discuss in detail various functions of stomatognathic system.

SHORT ESSAYS:
1 . Trajectories of force.
2. Deglutition. [ Ref LE Q.1]
3. Buccinator mechanism.

SHORT NOTES:
1 . Trajectories of force in mandible. [Ref SE Q.1]
2. Wolff’s law of transformation of bone.
3. Infantile swallow. [Ref LE Q.1]
4. Buccinator mechanism.
5. Trajectories of facial skeleton.
6. Pterygoid response.
7. Discuss about Wolff’s law of transformations of bone. [Same as SN Q.2]

SOLVED ANSWERS
LONG ESSAYS:
Q.1. Discuss in detail various functions of stomato- a. Infant feeding:
gnathic system.
Ans. {SN Q.3}
l Salzmann defined stomatognathics as an approach to

the practice of orthodontics which takes into consider- l Infants consume food by suckling, an automatic
ation the interdependence of form and function of teeth, reflex.
jaw relationship, temporomandibular articulation, cra- l In infants, suckling and swallowing proceed to-

niofacial conformation and dental occlusion. gether as suckle–swallow, which is characterized


l The various functions of stomatognathic system are as by caving in of cheeks, bobbing of the hyoid
follows: bone, extended head with anterior mandibular
A. Mastication thrust and elongated tongue, with lips pursed
B. Deglutition around the nipple.
C. Respiration Moyers lists the features of infantile swallow as follows:
D. Speech l Infantile swallow is guided by the lips and tongue.

l Jaws are apart with the tongue placed between the


A. Mastication gum pads.
l Mastication can be grouped as follows:
l Mandible is positioned by muscles of facial
a. Infant feeding (before teeth eruption) expression.
b. Mature or adult mastication (after teeth eruption)
42 Quick Review Series for BDS 4th Year, vol 1

The various types of infant feeding are as follows: [SE Q.2]


i. Breastfeeding
{B.  Deglutition
Milk is directed to the pharynx by the
Stages of deglutition:
peristaltic movement of the tongue.
Fletcher had divided the deglutition pattern into four
ii. Bottle feeding
stages:
Milk is expelled into the mouth.
i. Preparatory phase:
iii. Spoon feeding
l This phase starts as soon as food is ingested.
Here lips are sealed around the edge of a
l The liquid or crushed food is placed in swallow
spoon to draw food into the mouth.
preparatory position and mouth is sealed by lips
Tongue passes the bolus of food to swal-
or tongue.
lowing between gum pads and erupting
ii. Oral phase
teeth.
l Soft palate moves upward and tongue drops
Mature swallowing patterns are observed usu-
downward and backward.
ally by 18 months of age.
l Larynx and hyoid move upward.
b. Adult mastication:
l Muscles of mastication play an active role in
Mastication is defined as the reduction of food
stabilization.
in size, changing its consistency, mixing it with
l A smooth path is created for the bolus, and solid
saliva and forming into a bolus suitable for
food is pushed by the tongue and liquid food
swallowing.
just flows down.
The features of adult swallowing are follows:
iii. Pharyngeal phase
l Teeth are together.
l Begins as the bolus of food passes through the
l Mandible is stabilized by the muscles of
fauces.
mastication.
l Nasopharynx is closed and the tongue and hyoid
The six stages of mastication as outlined by Mur-
bone move forward.
phy are as follows:
iv. Oesophageal phase
i. Preparatory phase:
l Food passes through the cricopharyngeal
l The ingested food is positioned by the
sphincter through oesophagus to the stomach.}
tongue.
C.  Speech and malocclusion
l Mandible moves towards the chewing
l Speech is a learned behaviour.
side.
l Lips, tongue and velopharyngeal structures modify
ii. Food contact phase:
the outgoing breath stream to produce different
l After food is ingested, there is a mo-
variations in speech.
mentary pause in chewing as the sen-
l Bilabial sounds are the first sound to be developed.
sory receptors study the consistency of
Examples of various sounds:
food.
Bilabial (lips) n p.b
iii. Crushing phase:
Labiodental n f,v
l Crushing of ingested food starts with
Linguodental n th
high velocity, and later slows down.
Linguoalveolar nt,d,5
iv. Tooth contact phase:
Linguopalatal n‘ch’ ‘sh’
l After 4–5 strokes of crushing phase,

tooth contact happens accompanied by


change in direction of chewing. SHORT ESSAYS:
v. Grinding phase: Q.1. Trajectories of force.
l During this phase, unilateral chewing
Ans.
of food particles takes place.
l During grinding, vigorous contact of l The trajectorial theory states that the lines of orientation
maxillary and mandibular molars oc- of the bony trabeculae follow the pathways of maximal
curs. pressure and tension.
vi. Centric occlusion: l Benninghoff did extensive study on dried craniofacial

l Movement of teeth comes to a halt. bones and said that stress trajectories or lines of orienta-
l After this stage, preparatory stage of tion of the bony trabeculae involved not only the cancel-
the next stroke of mastication begins. lous bone but also the compact bone.
Section | I  Topic-Wise Solved Questions of Previous Years 43

l These trajectories or functional lines are otherwise l Other reinforcing members are infraorbital, zygo-
called Benninghoff lines. matic buttress, hard palate, walls of orbit and
l The stress trajectories respond to the demands of func- lesser wings of sphenoid.
tional forces collectively as a unit and not as a single bone. II. Trajectories of mandible
Accordingly, the head is made up of only two functional
units:
(i) Craniofacial unit {SN Q.1}
(ii) Mandible Mandible is made of major and minor trajectories
i. Craniofacial unit i. Major trajectories:
l The trajectories extend in a fan-like fashion l Trabecular columns originate beneath the teeth

from the mid-palatal suture across the alveolar in the alveolar process and join together into a
bone through the maxilla and end at the base of common stress pillar or trajectory system.
the skull. l Mandibular canal and nerve are protected by

l The bones of the face are united with the cra- this concentration of trabeculae.
nial bones by these fan-like trajectories, which l High resistance to bending forces is provided

continue across the facial bones and do not by the thick cortical layer of trabeculae present
stop at the suture. along the lower border of the mandible.
I. Maxillary trajectories: ii. Minor trajectories:
Maxilla provides maximum strength with minimum material l These minor accessory stress trajectories are

because of the following trajectories. They are as follows: due to the effect of muscle attachments.
Example: Trajectories seen at the symphysis, gonial an-
Frontonasal/canine pillar
gle and one trabecular pattern are seen running down-
wards from the coronoid process into the ramus and
Vertical pillars Malar zygomatic pillar
body of the mandible.

Pterygoid pillar
Maxilla Q.2. Deglutition.
trajectories
Ans.
Horizontal reinforcing members: [Ref LE Q.1]
• Trajectories from hard palate,
orbital walls, zygomatic arches, palatal Q.3. Buccinator mechanism.
bones and lesser wing of sphenoid
Ans.
a. Vertical pillars: l Teeth and supporting structures of the jaw are under the
i. Frontonasal vertical pillar/buttress control of adjacent muscles.
l This pillar or buttress runs vertically along piri- l The balance between the muscles is responsible for the
form aperture and crest of the nasal bones and integrity of dental arches and the relation of teeth to the
ends in the frontal bone, transmitting pressures arches.
from the incisors, canines and first premolar. l Buccinator mechanism is a phenomenon where a con-
ii. Malar zygomatic vertical pillar/buttress: tinuous band of muscles encircle the dentoalveolar re-
l In the zygomatic area, it splits into three parts: gion from the buccal aspect and is firmly anchored at
one passes through the zygomatic arch and the pharyngeal tubercle of occipital bone.
other along the lateral border of orbit and the l It starts with the decussating fibres of orbicularis oris
last along the lower border of orbit; finally all joining the right and left fibres of the lip which consti-
three parts ends in base of the skull. tute the anterior component of buccinator mechanism.
l It transmits stress from the posterior teeth and l It then runs laterally and posteriorly around the corner of
also receives force of the masseter muscles. the mouth, joining other fibres of the buccinator muscle,
iii. Pterygoid vertical pillar/buttress: which gets inserted into the pterygomandibular raphe.
l Runs vertically from the chonchae of nasal l Here it mingles with the fibres of superior constrictor
cavity and posterior teeth ending in the mid- muscle and runs posteriorly and medially to get fixed to
dle portion of the base of the skull the pharyngeal tubercle.
b. Horizontal reinforcing members: l All these 13 muscles with elasticity and contractility act
l Supraorbital rim acts as a receptor of forces from like a rubber band tightly encircling the bone system,
canine and zygomatic pillar. i.e. mandible.
44 Quick Review Series for BDS 4th Year, vol 1

l Tongue acts opposing the buccinator mechanism from l All these 13 muscles with elasticity and contractility act
within exerting an outward force. like a rubber band tightly encircling the bone system,
l The dentition is in a constant state of dynamic equilibrium. i.e. mandible.
l There is a balance of forces between muscles that are l Tongue acts opposing the buccinator mechanism from

believed to influence the position and stability of the within exerting an outward force.
dentoalveolar complex.
Q.5. Trajectories of facial skeleton.
Clinical significance:
Ans.
l Malocclusion is caused due to any imbalance in bucci-

nator mechanism. l Benninghoff stated that stress trajectories or lines of


l Due to certain deleterious oral habits like thumb sucking, orientation of the bony trabeculae involve not only the
tongue thrusting, etc. the equilibrium between buccinator cancellous bone but also the compact bone.
mechanism and tongue is lost, causing constricted maxil- l These trajectories or functional lines are otherwise

lary arch, increased proclination and open bite etc. called Benninghoff lines.
l Maxilla provides maximum strength because of the

following trajectories:
SHORT NOTES:
Frontonasal/canine pillar
Q.1. Trajectories of force in mandible.
Ans. Vertical pillars Malar zygomatic pillar

[Same as SE Q.1] Maxilla Pterygoid pillar


trajectories
Q.2. Wolff’s law of transformation of bone.
Ans. Horizontal reinforcing members
• Trajectories from hard palate, orbital walls,
Wolff’s law of transformation of bone: zygomatic arches, palatal bones and lesser
l In the year 1870, Julius Wolff attributed the arrange- wing of sphenoid
ment of trabecular pattern of bone to functional forces.
l A change in the direction and magnitude of force Major trajectories
could produce a marked change in the internal archi- Mandibular
tecture and external form of the bone. This is called trajectories
‘Wolff’s law of transformation of bone’. Minor trajectories
l Increase in function leads to increase in density of

bone, while lack of function leads to decrease in l Major trabecular columns originate beneath the teeth in
trabecular pattern. the alveolar process and join together into a common
l Simply stated, stresses of tension or pressure on stress pillar or trajectory system.
bones stimulate changes within the bone. l The minor accessory stress trajectories are due to the

effect of muscle attachments, e.g. trajectories seen at the


Q.3. Infantile swallow.
symphysis, gonial angle etc.
Ans.
Q.6. Pterygoid response.
[Ref LE Q.1]
Ans.
Q.4. Buccinator mechanism.
l Pterygoid vertical pillar/buttress runs vertically from the
Ans. chonchae of nasal cavity and posterior teeth ending in
the middle portion of the base of the skull.
l Buccinator mechanism is a phenomenon where a con-
l It is one of the vertical pillars of maxillary trajectories.
tinuous band of muscles encircle the dentoalveolar re-
l The trajectorial theory states that the lines of orientation
gion from the buccal aspect and is firmly anchored at
of the bony trabeculae follow the pathways of maximal
the pharyngeal tubercle of occipital bone.
pressure and tension.
l It starts with the decussating fibres of orbicularis oris
l These trajectories or functional lines are otherwise
joining the right and left fibres of lip which constitute
called Benninghoff lines.
the anterior component of buccinator mechanism.
l The stress trajectories respond to the demands of func-
l It then runs laterally and posteriorly around the corner of
tional forces collectively as a unit and not as a single bone.
the mouth, joining other fibres of the buccinator muscle,
which gets inserted into the pterygomandibular raphe. Q.7. Discuss about Wolff’s law of transformations of bone.
l Here it mingles with the fibres of superior constrictor
Ans.
muscle and runs posteriorly and medially to get fixed to
the pharyngeal tubercle. [Same as SN Q.2]
Section | I  Topic-Wise Solved Questions of Previous Years 45

Topic 6
Occlusion – Basic Concepts
COMMONLY ASKED QUESTIONS
LONG ESSAYS:
1. What are six keys of normal occlusion? State how Ackerman profit system is an improvement over Angle’s
classification.

SHORT ESSAYS:
1 . What are the forces of occlusion?
2. Describe six keys to normal occlusion. [Ref LE Q.1]
3. Roth’s keys of functional occlusion.
4. Occlusal plane.

SHORT NOTES:
1 . Curve of Spee.
2. Overjet and overbite.
3. Normal occlusion concept. [Ref LE Q.1]
4. Centric relation and centric occlusion.
5. Enlist Andrew’s six keys to normal occlusion. [Ref LE Q.1]
6. Overjet. [Same as SN Q.2]
7. Features of normal occlusion. [Same as SN Q.3]
8. Describe six keys to normal occlusion. [Same as SN Q.5]

SOLVED ANSWERS
LONG ESSAYS:
Q.1. What are six keys of normal occlusion? State how i. Molar interarch relationship
Ackerman profit system is an improvement over Angle’s
classification.
Ans.

(SN Q.5 and SE Q.2)


{(Andrew in 1970 put forward the keys to normal occlusion.
The Andrew’s six keys to normal occlusion are as follows:
i. Molar interarch relationship
ii. Mesiodistal crown angulation
iii. Labiolingual crown inclination
iv. Absence of rotation
v. Tight contacts
vi. Curve of Spee)
46 Quick Review Series for BDS 4th Year, vol 1

Mesiobuccal cusp of 6 6
l should occlude in the
{SN Q.3}
groove between mesial and mesiobuccal cusp of
Normal occlusion:
. l Angle’s concept of normal occlusion is based on key
6 6
Mesiolingual cusp of 6 6
l should occlude in and line of occlusion.

central fossa of 6 6 . Key of occlusion:


l Angle considered maxillary first molar as the key of

Crown of 6 6 must be angulated so that distal mar-


l
occlusion, as it is most constant at its position, and he
related it to key ridge position.
ginal ridge occludes with mesial marginal ridge of l According to Angle, the mesiobuccal cusp of upper

. first molar rests in the mesiobuccal groove of man-


7 7 dibular first molar in normal occlusion, i.e. class I
ii. Mesiodistal crown angulaton molar relation.
Line of occlusion:
l There are two lines of occlusion, namely maxillary

and mandibular.
l The maxillary line of occlusion is a smooth curve

that passes through central fossa of upper molars and


along the cingulum of upper canines and incisors.
l Mandibular line of occlusion runs along the buccal

cusps of posteriors and incisal edges of anteriors.


According to Angle, in normal occlusion, there
should be full complement of teeth present.
G Lines of occlusion are intact in both maxillary and
D
mandibular arches and molars in class I relation.

M SHORT ESSAYS:
Q.1. What are the forces of occlusion?
For occlusion to be considered normal, the gingival part Ans.
of the long axis of crown must be distal to the occlusal
The forces of occlusion are of three types:
part of the line. Different teeth exhibit different crown
a. Forward or anterior force
angulation.
i. Forward force is also called anterior component
iii. Labiolingual crown inclination
force. It comes into effect after the eruption of first
l The crown inclination is determined from a mesial
permanent molar into occlusion.
or distal view.
ii. This force is produced due to relationship of long
l If gingival area of the crown is more lingually placed
axis of teeth to the occlusal surfaces and action of
than the occlusal area, it is called 1ve crown inclination.
muscles like buccinator and masseter on the teeth.
l If gingival area of the crown is more labially or buc-
b. Distal and lingual force
cally placed than the occlusal area, it is called –ve
i. Distal and lingual forces are produced by circumoral
crown inclination.
muscles and buccinators.
iv. Absence of rotation
ii. These forces act on incisors and help to keep canines
l Normal occlusion is characterized by the absence of
in place.
any rotation.
c. Anterior resultant force
l Rotated posterior teeth occupy more space in arch.
i. This force is the anterior resultant of the two forces
l Rotated anterior incisors occupy less space in arch.
which act in the opposite direction.
v. Tight contacts
ii. In spite of these forces, teeth have inherent disposi-
To consider an occlusion as normal, there should be
tion to drift mesially.
tight contact between adjacent teeth.
vi. Curve of Spee Q.2. Describe six keys to normal occlusion.
A normal occlusion plane according to Andrew
Ans.
should be flat, with the curve of Spee not exceeding
1.5 mm.} [Ref LE Q.1]
Section | I  Topic-Wise Solved Questions of Previous Years 47

Q.3. Roth’s keys of functional occlusion. SHORT NOTES:


Ans. Q.1. Curve of Spee.
Roth’s keys of functional occlusion are as follows: Ans.
Key 1.
Coincidence of intercuspal position (ICP) and retruded l Curve of Spee refers to anteroposterior curvature of the
contact position (RCP). occlusal surfaces beginning at the tip of the lower cus-
Key 2. pid and following the cusp tips of the bicuspids and
Maximum and stable cusp to fossa contacts throughout molars continuing as an arc through the condyle.
l If the curve is extended, it would form a circle of about
the buccal segments.
Key 3. 4 inch diameter.
l The curvature results from variations in axial alignment
Disclusion of the posterior teeth in mandibular protru-
sion by even contacts of incisors. of lower teeth.
l The long axis of each lower tooth is aligned nearly parallel
Key 4.
Lateral movements of mandible are guided by the work- to its individual arc of closure around the condylar axis.
l This requires a gradual progressive increased mesial
ing side canines, with disocclusion of all other teeth on
both working and non-working sides. tilting of teeth towards molars, which creates the curve
of Spee.
Q.4. Occlusal plane.
Q.2. Overjet and overbite?
Ans.
Ans.
Imaginary occlusal planes and curves
Curve of Spee: i. Overjet:
l Overjet is seen more initially in primary dentition.

The average overjet in primary dentition is 1–2 mm.


l With the movement of whole dental arch anteriorly,

overjet decreases.
ii. Overbite:
l In normal overbite, the upper incisors slightly overlap

the lower incisors. It is usually expressed in millimetres.


l Normal value of overbite: 1–3 mm.

l The overbite is described as the percentage of man-

dibular incisor crown length overlapped by maxillary


central incisors.
l 5%–20% of overlap of mandibular incisors is consid-

ered normal.
Q.3. Normal occlusion concept.
Ans.
l A-P curvature of occlusal surfaces beginning at the tip of
lower cuspid and following the cusp tips of tricuspids, [Ref LE Q.1]
and molars continuing as an arch through the condyle. Q.4. Centric relation and centric occlusion.
l If the curve is extended, it would follow a circle of about

4 inch diameter. Ans.

Curve of Wilson: Centric relation


l Centric relation is also called ‘ligamentous position or
l This is a curve that contacts the buccal and lingual cusp

tips of the mandibular buccal teeth. The curve of Wilson terminal hinge position’.
l Centric relation is the relation of mandible to maxilla
is mediolateral on each side of the arch.
l It results from inward inclination of the lower posterior
when the mandibular condyles are in the most superior
teeth. and retruded position in their glenoid fossa with the ar-
l It helps in two ways:
ticular disc properly interposed.
Teeth are aligned parallel to direction of medial ptery- Centric occlusion
goid for opposing resistance to mastication and elevated l Centric occlusion is also called ‘inter-cuspal position or
buccal cusps prevent the food from going palatally. convenience occlusion’.
48 Quick Review Series for BDS 4th Year, vol 1

l Centric occlusion is that position of the mandibular Q.7. Features of normal occlusion.
condyle where the teeth are in maximum intercuspation.
Ans.
Q.5. Enlist Andrew’s six keys to normal occlusion.
[Same as SN Q.3]
Ans.
Q.8. Describe six keys to normal occlusion.
[Ref LE Q.1]
Ans.
Q.6. Overjet.
[Same as SN Q.5]
Ans.
[Same as SN Q.2]

Topic 7
Classification of Malocclusion
COMMONLY ASKED QUESTIONS
LONG ESSAYS:
1. Classify malocclusion and list its advantages? Discuss different methods of classifications on malocclusion.
2. Define classification of malocclusion and explain in detail Angle’s classification and validity of Angle’s classification.
3. State how Ackerman–Proffit system is an improvement over Angle’s classification. Discuss in brief the modifi-
cations to Angle’s classification of malocclusion.
4. Describe Angle’s class II division 1 malocclusion. Discuss how it differs from Angle’s class II division 2.
5. Enumerate differences between true class III and pseudo-class III.
6. Name different methods of classification of malocclusion. Describe Simon’s classification of malocclusion.
[Same as LE Q.1]
7. Describe various classifications of malocclusion. Enumerate merits and demerits of Angle’s classification.
[Same as LE Q.2]
8. Describe Angle’s classification of malocclusion and mention the limitations of this classification. [Same as LE Q.2]
9. Discuss in brief Angle’s classification of malocclusion along with its merits, demerits and modifications.
[Same as LE Q.2]
1 0. Describe Deway–Anderson’s modification of Angle’s classification in detail. [Same as LE Q.4]
1 1. Describe the characteristics of class II division 1 and class II division 2 malocclusion. [Same as LE Q.5]

SHORT ESSAYS:
1. Ackerman–Proffit classification of malocclusion. [Ref LE Q.3]
2. Angle’s class II division 1 malocclusion – Clinical features.
3. Drawbacks of Angle’s classification.
4. Angle’s classification of malocclusion. [Ref LE Q.4]
5. Characteristics of Angle’s class II division 2 malocclusion. [Ref LE Q.2]
6. Simon’s classification of malocclusion. [Ref LE Q.1]
7. Dewey’s classification of malocclusion. [Ref LE Q.4]
8. Pseudo-class III malocclusion.
9. Limitations of Angle’s classification. [Same as SE Q.3]
1 0. Clinical features of class II division 2 malocclusion. [Same as SE Q.5]

SHORT NOTES:
1 . Bennett’s classification.
2 . Simon’s classification of malocclusion. [Ref LE Q.1]
Section | I  Topic-Wise Solved Questions of Previous Years 49

3. Validity of Angle’s classification. [Ref LE Q.2]


4. FH plane.
5. Pseudo-class III malocclusion.
6. Clinical features of class III malocclusion.
7. Clinical features of class II division 2.
8. Define malocclusion. Name few classifications of malocclusion.
9. Ackerman–Proffit classification.
10. Features of class II division 1 malocclusion.
11. Distocclusion.
12. Lischer’s modifications of Angle’s classification. [Ref LE Q.4]
13. Skeletal classification of malocclusions.
14. Mention three planes used in Simon’s classification.
15. Dewey’s modification of Angle’s classification of malocclusion.
16. Key ridge.
17. Drawbacks of Angle’s classification. [Same as SN Q.3]
18. Mention two merits and demerits of Angle’s classification. [Same as SN Q.3]
19. True class III. [Same as SN Q.6]
20. Name different classifications of malocclusion. [Same as SN Q.8]
21. Enlist various systems of classification of malocclusion. [Same as SN Q.8]

SOLVED ANSWERS
LONG ESSAYS:
Q.1. Classify malocclusion and list its advantages? Dis- Intra-arch malocclusion:
cuss different methods of classifications on malocclusion. Commonly seen individual teeth malpositions are as
follows:
Ans.
i. Buccal inclination or tipping – proclination (when the
Grouping of various malocclusions into a simpler or smaller tooth is outside the line of occlusion)
divisions or groups is known as classification. ii. Lingual inclination or tipping – retroclination (when
Strang (1938) defined classification as ‘A process of the tooth is inside the line of occlusion)
analysing cases of malocclusion for the purpose of segre- iii. Mesial inclination or tipping – crown tilted mesially
gating them into a smaller number of groups, which are or farther forward than normal
characterized by certain specific and fundamental varia- iv. Distal inclination or tipping – crown tilted distally or
tions from normal occlusion of the teeth; which variations backwards than normal
become influential and deciding factors in providing the v. Buccal displacement – tooth that is bodily moved in
fundamental data for the preparation of a systematic and labial/buccal direction
correlated plan of treatment’. vi. Lingual displacement – tooth that is bodily moved in
Advantages of classifying malocclusion: lingual direction
It helps in: vii. Mesial displacement – tooth that is bodily moved in
i. Diagnosis and treatment planning. mesial direction
ii. Visualizing and understanding the problem associated viii. Distal displacement – tooth that is bodily moved in
with malocclusion. distal direction
iii. Ease of communication among professionals. ix. Infraversion or infra-occlusion – this is a tooth which
iv. Comparison of various malocclusions. has not erupted enough as compared to other teeth in
the arch
Intra-arch malocclusion (variations in
x. Supraversion or supraocclusion – tooth which has
individual tooth position and malocclusion
affecting a group of teeth within an arch) over erupted as compared to other teeth in the arch
xi. Rotations – tooth movements around its long axis
xii. Distolingual or mesiobuccal rotation – tooth which
Types of Interarch malocclusion (malrelation of
has moved around its long axis so that distal aspect is
malocclusion dental arches to one another upon normal
skeletal bases) more lingually placed
xiii. Mesiolingual or distobuccal rotation – tooth which has
Skeletal malocclusion (malocclusion mesial aspect is more lingually placed
involving underlying bony bases)
50 Quick Review Series for BDS 4th Year, vol 1

xiv. Transposition – where the teeth have exchanged/ In Simon’s system, dental arches are related
interchanged their positions to three anthropometric planes:
Interarch malocclusion: occurs in three planes. i. Frankfort horizontal plane
ii. Orbital plane
iii. Midsagittal plane
Occurs in three planes
ATTRACTION
Dental arch or part of it is
Sagittal Vertical Transverse FH plane closer than normal to FH plane.

ABSTRACTION
Pre-normal Post-normal Includes deep bite Example: various Dental arch or part of it is
occlusion occlusion and open bite types of farther away from the FH plane.
Lower arch Lower arch (where abnormal crossbites
is more is more vertical relation (the term
forwardly distally exists between crossbite ORBITAL PLANE PROTRACTION
placed when placed when teeth of U/L arch) refers to (Simon’s law of Dental arch or part of it is farther
patient bites patient bites abnormal away from orbital plane.
canines) Sagittal
in centric in centric transverse
occlusion. occlusion. or AP direction
relationship RETRACTION
between U/L Dental arch or part of it is closer
Deep bite Open bite arches) or more posterior plane in
(overbite (no vertical
there is relation to orbital plane.
overlap
excessive between U/L
overlap teeth either DISTRACTION
MIDSAGITTAL
between U/L in anterior/ Dental arch or part of it is away
anteriors) PLANE
posteri or from midsagittal plane.
region) (transverse
direction) CONTRACTION
Dental arch or part of it is
closer to midsagittal plane.
Skeletal malocclusion: due to abnormalities in the maxilla/
mandible; defects can be in size, position or relationship Q.2. Define classification of malocclusion and explain in
between the jaws. Occurs in one or both the jaws and in detail Angle’s classification and validity of Angle’s clas-
various combinations. sification.
Ans.
Occurs in three planes l Grouping of various malocclusions into a simpler or
smaller divisions or groups is known as classification.
l Angle’s system of classification has taken into consideration
Sagittall Vertical Transverse
the anteroposterior relationship of teeth with each other.
l Angle introduced this classification with the concept of
Forward More backward Abnormalities Result of key and line of occlusion.
placement placement in vertical narrowing or l Maxillary first permanent molar is considered to be the
of jaw – of jaw – measurements widening of
prognathism. retrognathism. can affect lower key of occlusion, and he stated that the position of the
jaws, usually
facial height. maxillary first permanent molar is relatively constant.
referred to as
crossbites. Angle’s classification:
Angle’s three classes of malocclusion:

(SN Q.2 and SE Q.6)


Class I Class II Class III
{(In Simon’s classification of malocclusion, the dental
arches are related to three planes:
Division 1
i. Anteroposterior plane
and
ii. Transverse plane True Pseudo Skeletal
Division 2
iii. Vertical plane class III class III class III
Angle’s classification of malocclusion.
Section | I  Topic-Wise Solved Questions of Previous Years 51

The three classes of Angle’s malocclusion are based on the Class II molar relation:
permanent first molar relationship. Lower dental arch is distally positioned in relation to
Angle’s class I malocclusion (neutrocclusion) upper arch. The distobuccal cusp of the upper first
Molar relation in Angle’s class I: molar occludes with the mesiobuccal groove of the
The mesiobuccal cusp of the upper first molar oc- lower first molar.
cludes with the mesiobuccal groove of the lower first Class II canine relation:
molar. The distal incline of the upper canine occludes with
Canine relation: the mesial incline of the lower canine.
The mesial incline of the upper canine occludes with Line of occlusion: Altered.
the distal incline of the lower canine, whereas the Other clinical features:
distal incline of the upper canine occludes with me- i. Molars in distocclusion
sial incline of lower first premolar. ii. Retroclined central incisors and rarely other
Line of occlusion: anteriors as well
Line of occlusion will be altered in maxillary and iii. Deep bite
mandibular arches: iv. Broad square face with pleasing straight profile
Individual tooth irregularities like crowding, spacing, v. Square-shaped arch and exaggerated curve of
rotations and absence of tooth will be seen. Spee
Interarch problems like deep bite, open bite, procli- vi. Backward path of closure
nation or increased overjet and crossbite will be vii. Deep mentolabial sulcus
present. viii. Absence of abnormal muscle activity
Class I. Bimaxillary protrusion They have perfectly acceptable function as well
Class I bimaxillary malocclusion is a condition where as facial appearance. In severe cases, the bite is
both key of occlusion and line of occlusion are not often very deep and poses the risk of periodon-
altered but the upper and lower anteriors are pro- tal trauma in maxillary palatal and mandibular
clined and exist usually in an edge–edge relationship. labial aspects.}
Angle’s class II malocclusion (distocclusion) Class II division 2 subdivision:
Class II malocclusion has got two divisions, namely Condition when the class II molar relation exists on
division 1 and division 2. only one side with normal molar relation on the other
Angle’s class II division 1 malocclusion side is known as class II division 2 subdivision.
Molar relation: Angle’s class III malocclusion (mesio-occlusion)
Lower dental arch is distally positioned in relation to Class III malocclusion is a condition in which the lower
upper arch. The distobuccal cusp of the upper first molar is positioned mesial to the upper molar.
permanent molar occludes with the mesiobuccal Class III subdivision: Condition in which class III mo-
groove of the lower first permanent molar. lar relation is present only on one side with normal
Class II canine relation: molar relation on the other side is known as class III
The distal incline of upper canine occludes with the subdivision.
mesial incline of lower canine. True class III:
Other features are as follows: l This is a skeletal malocclusion; it could be due to
l V-shaped or narrow constricted maxilla retrognathic maxilla, prognathic mandible or
l Proclined maxillary incisors combination of both.
l Lip trap l In this, class III molar relation exists in both cen-
l Deep bite and exaggerated curve of Spee tric occlusion and rest position.
Class II division 1 subdivision Pseudo-class III/habitual class III:
Condition where class II molar relationship is unilat- l This is not a true class III malocclusion.
eral, i.e. present only on one side with normal class I l When the mandible moves from rest position to
molar occlusion on the other side. occlusion due to occlusal prematurities, it slides
forward into a pseudo-class III position.
[SE Q.5]
l These patients show normal molar relationship in
{Angle’s class II division 2 malocclusion: the rest position, while class III relation in centric
Class II division 2 malocclusion is characterized by occlusion.
class II molar relationship with retroclined upper Clinical features of class III malocclusion are as follows:
centrals that are overlapped by the lateral incisors. l Molar relation:
Includes variations like retroclined centrals 1 lateral Mesiobuccal cusp of the upper first permanent
incisors and very rarely includes retroclined canines molar occludes with the interdental space between
as well. the lower first and second permanent molars.
52 Quick Review Series for BDS 4th Year, vol 1

l Canine relation:
Upper canine occludes with the interdental space
between lower first and second premolars.
Group 3
l Line of occlusion: may or may not be altered. Transverse Group 4
Deviation Group 6 Sagittal Deviation
l Reverse overjet or anterior crossbite. Buccal (lateral) Trans-sagittal (A-P)
Palatal Class I ANT
l Posterior crossbite. Unilateral Alignment Type Calss II Displacement
Bilateral Profile Class Class III Division 1
Dental Division 2
Dental
True class III and pseudo-class III malocclusions can be Skeletal Group 9 Skeletal
Trans-sagittal-vertical
differentiated by taking a cephalogram in both rest position Alignment
Group 8
Alignment
Alignment
Profile Profile
and occlusion. Type Vertico
-transverse
Type Class Group 7 Profile
Class
Bite depth Sagitto-vertical
Alignment Alignment
Profile Profile
Group 2 Type Bite depth
Profile Class
Bite depth Convex
{SN Q.3} Alignment Group 5 Straight
Profile Vertical Deviation Concave
Open bite Anterior, posterior
Merits and demerits of Angle’s classification/advantages Deep bite Anterior Anterior
Collapsed Posterior Divergent
and disadvantages of Angle’s c1assification/validity of Dental Skeletal Posterior
Alignment
Angle’s classification Group 1 Bite depth
Profile Divergent

Ideal
Merits: Intra-arch
alignment–symmetry Crowding
Spacing
i. It is the most popular system of classification of (occlusal view)
Alignment

malocclusion.
ii. It is the most traditional and oldest system of clas- Fig. 7.1  Ackerman–Proffit system – Venn symbolic diagram.
sification still in use.
iii. Easy to communicate. l This is an all-inclusive method of categorizing maloc-
iv. Most practical and easy to comprehend. clusion. In this classification, each malocclusion can be
v. Widely used for academic purposes. described by five major characteristics.
Demerits: Procedure
i. Angle considered only anteroposterior plane in his Group 1 (Intra-arch alignment and symmetry):
l Involves assessment of alignment and symmetry of den-
system of classification. He did not consider trans-
verse and vertical planes. tal arches. Individual tooth irregularities are described.
l As the degree of alignment and symmetry is common to
ii. The position of the maxillary first permanent molar
is not stable as considered by Angle. all dentition, it is represented as Group 1.
l It is classified in this step as ideal, crowded or spaced.
iii. Classification is not applicable when first perma-
l Crowding, rotations, spacing, and mutilated conditions
nent molars are missing.
iv. Not applicable in deciduous dentition. are possible malocclusions in this step. If no abnormal-
v. Consideration is not given to skeletal problems. ity is present, it is called ideal.
vi. Angle’s classification considers only static occlu- Group 2 (Profile):
l Proceeding inside the Venn diagram, it involves consid-
sion.
vii. Aetiology of malocclusion is not highlighted. eration of the profile of the patient.
l The profile is described as straight, convex and concave.
viii. This classification does not differentiate between
l The facial divergence is considered as anterior or poste-
dentoalveolar and skeletal malocclusions.
ix. Angle did not consider individual tooth malposi- rior divergence.
tions. Group 3 (Type):
The term type is used to describe the kinds of crossbites.
There are two modifications to Angle’s classification: Lateral or transverse arch characteristics are evaluated.
i. Lischer’s modification Crossbites are classified as:
ii. Dewey’s modification i. Buccal and palatal
ii. Unilateral and bilateral
Q.3. State how Ackerman–Proffit system is an improve- iii. Skeletal and dental
ment over Angle’s classification. Group 4 (Class):
i. It involves assessment of sagittal relationship.
Ans. ii. It is classified as Angle’s class I/class II/class III
malocclusion.
[SE Q.1]
iii. Differentiation is made between skeletal and
l {Ackerman and Proffit introduced a new method of clas- dental malocclusions.
sification to overcome the defects of Angle’s classification. Group 5 (Bite depth):
l This system included Angle’s classification and five l Malocclusion in vertical plane is considered.
major characteristics of malocclusion within a Venn l Patient’s skeletal and dental relationships are ana-
symbolic diagram (Fig. 7.1). lysed for problems in the vertical plane.
Section | I  Topic-Wise Solved Questions of Previous Years 53

Vertical deviations:
l
l Supraversion – crossing the line of occlusion
i. Open bite n Anterior open bite, posterior open
l Axiversion – wrong axial inclination
bite, skeletal open bite and dental open bite.
l Torsiversion – rotated on its long axis
ii. Deep bite n Dental or skeletal and posterior
l Transiversion – transposition, wrong position in
collapsed bite.
the arch
The overlapping groups are seen in the centre
of the Venn diagram.
Group 9 will have the most severe form of malocclusion Dewey’s modification
comprising problems in all the three dimensions.} [SE Q.7]
Advantages and disadvantages of Ackerman and Proffit {Martin Dewey has further divided Angle’s class I and
method of classification are as follows: III into various types:
Advantages: l Class I is divided into five types
i. The complexities of malocclusion are explained. l Class III is divided into three types
ii. In addition to anteroposterior malrelations, trans- l Class II has no types
verse as well as vertical discrepancies are taken into Class I
consideration, i.e. all three planes are considered. Type 1: Crowded maxillary anterior teeth. Canines
iii. Profile of the patient is given due consideration. may be abnormally positioned. Other individual
iv. Differentiation between skeletal and dental prob- teeth irregularities present.
lems is made. Type 2: Proclined or labioversion of maxillary
v. This classification helps in complete diagnosis and central and lateral incisors.
differential treatment planning. Type 3: Class I with anterior crossbite present.
vi. Readily adaptable to computer processing. Type 4: Class I with posterior crossbite present.
Disadvantages: Molars and premolars are in buccoversion.
i. Classification did not consider the aetiological Type 5: Mesioversion of molars.
aspects. Class III
ii. It is based only on static occlusion, whereas func- Type1: Well-aligned teeth and dental arches.
tional occlusion is not considered. Edge–edge relationship exists.
Q.4. Discuss in brief the modifications to Angle’s classi- Type 2: Crowded mandibular incisors. Normally
fication of malocclusion. placed lower incisors behind the upper incisors.
Type3: Crowded maxillary incisors. Underdevel-
Ans. oped maxilla anterior crossbite present.}
There are two modifications to Angle’s classification of Q.5. Describe Angle’s class II division 1 malocclusion.
malocclusion: Discuss how it differs from Angle’s class II division 2.
i. Lischer’s modification
ii. Dewey’s modification Ans.
Lischer’s modification Differences between class II division 1 and class II division
2 malocclusion are as follows:
{SN Q.12}
i. The following names are given by Lischer to Angle’s Feature Class II division 1 Class II division 2
classification: i. Profile Convex Straight to mild
l Neutrocclusion – class I
convexity
l Distocclusion – class II ii. Lips Incompetent short Competent normal
l Mesio-occlusion – class III upper lip and upper and lower
ii. To describe malpositions of individual teeth, the fol- everted lower lip lips
lowing nomenclature was given by Lischer which indi- iii. Mentolabial Deep Normal, or may be
cates the direction of deviation from normal position. sulcus deep sometimes
l Mesioversion – mesial to the normal position iv. Mentalis Hyperactive Normal
l Distoversion – distal to the normal position muscle
l Linguoversion – lingual to the normal position v. Molar process Not prominent Prominent
l Labioversion/buccoversion – towards the lip or
vi. Lower facial Normal or may Decreased
cheek height be increased or
l Infraversion – away from the line of occlusion decreased
54 Quick Review Series for BDS 4th Year, vol 1

vii. Arch form ‘V’-shaped ‘U’-shaped or Q.7. Name different methods of classification of maloc-
square shape clusion. Describe Simon’s classification of malocclusion.
viii. Palate Deep Normal Ans.
ix. Incisors Proclined Central incisors are
(maxillary) retroclined, lateral
[Same as LE Q.1]
incisors are pro- Q.8. Describe various classifications of malocclusion.
clined
Enumerate merits and demerits of Angle’s classification.
x. Overjet Increased Decreased
Ans.
xi. Overbite Deep overbite Closed bite
xii. Crown root Normal Axis of crown and
[Same as LE Q.2]
angulation root is bent and re-
Q.9. Describe Angle’s classification of malocclusion and
ferred to as collum
angle mention the limitations of this classification.
xiii. Path of Normal Backward path of Ans.
closure closure
[Same as LE Q.2]
Q.10. Discuss in brief Angle’s classification of malocclu-
Q.6. Enumerate differences between true class III and
sion along with its merits, demerits and modifications.
pseudo-class III.
Ans.
Ans.
[Same as LE Q.2]
Differences between true class III and pseudo-class III are
as follows: Q.11. Describe Deway–Anderson’s modification of
Angle’s classification in detail.
Feature True class III Pseudo-class III
Ans.
i. Profile Concave Straight or concave
[Same as LE Q.4]
ii. Aetiology Heredity Habitual or
developmental Q.12. Describe the characteristics of class II division 1
iii. Premature Absent Present and class II division 2 malocclusion.
contacts
Ans.
iv. Path of closure Forward Deviated
[Same as LE Q.5]
v. Gonial angle h or g Normal
vi. Retrusion of Not possible Possible
mandible SHORT ESSAYS:
vii. Treatment Orthopaedic Elimination of prematu-
or surgical rities and replacement Q.1. Ackerman–Proffit classification of malocclusion.
correction. of last posterior teeth by
Ans.
No further functional space main-
changes tainers. [Ref LE Q.3]
occur if left If left untreated, it
untreated. becomes established Q.2. Angle’s class II division 1 malocclusion – clinical
into true class III maloc- features.
clusion.
Ans.
Angle’s class II malocclusion has got two divisions, namely
Surgical procedures that can be carried out for skeletal class
division 1 and division 2.
III malocclusion are as follows:
Angle’s class II division 1 malocclusion:
i. Le Fort I osteotomy – for maxillary deficiency
Class II division 1 malocclusion is characterized by class
ii. Osteotomy of the mandible
II molar relation with proclined maxillary anterior teeth.
iii. Sliding ostecotomy in ramus or body of the mandible
Clinical features:
iv. Sagittal split osteotomy – to correct mandibular prog-
i. Class II molar relation: Lower dental arch is distally
nathism
positioned in relation to upper arch. The distobuccal
v. Genioplasty – to correct chin prominence
cusp of the upper first permanent molar occludes
These skeletal procedures have to be modified according to with the mesiobuccal groove of the lower first per-
the vertical malrelation, i.e. associated open bite or deep bite. manent molar.
Section | I  Topic-Wise Solved Questions of Previous Years 55

ii. Class II canine relation: The distal incline of upper Class III malocclusion is a condition in which the lower
canine occludes with mesial incline of lower canine. molar is positioned mesial to the upper molar.
iii. Proclined maxillary anteriors with V-shaped or nar- l Pseudo-class III/habitual class III is not a true class III

row constricted maxilla. malocclusion.


iv. Convex profile. l When the mandible moves from rest position to occlu-

v. Increased overjet, deep bite and exaggerated curve sion due to occlusal prematurities, it slides forward into
of Spee. a pseudo-class III position.
vi. Short hypotonic upper lip, lip trap and lack of ante- l These patients show normal molar relationship in rest

rior lip seal. position, while class III relation in centric occlusion.
vi. Patient exhibits abnormal muscle activity. Aetiology:
vii. Proclined lower anteriors, a natural compensation Habitual or developmental.
to decrease overjet. The clinical features of pseudo-class III malocclusion are
Class II division 1 subdivision: as follows:
Condition where the class II molar relationship is uni- i. Profile is either straight or concave.
lateral, i.e. present only on one side with normal class I ii. Premature contacts and deviated path of closure are
molar occlusion on the other side. present.
iii. Gonial angle is normal.
Q.3. Drawbacks of Angle’s classification.
iv. Retrusion of mandible is possible.
Ans. Treatment of pseudo-class III consists of the following:
Limitations or demerits or drawbacks of Angle’s classification: Elimination of prematurities and replacement of last
i. Angle considered only anteroposterior plane in his posterior teeth by functional space maintainers. If left
system of classification. He did not consider trans- untreated, it becomes established into true class III
verse and vertical planes. malocclusion.
ii. The position of the maxillary first permanent molar is Q9. Limitations of Angle’s classification.
not stable as considered by Angle.
iii. Classification is not applicable when first permanent Ans.
molars are missing. [Same as SE Q.3]
iv. Not applicable in deciduous dentition.
v. Consideration is not given to skeletal problems. Q10. Clinical features of class II division 2 malocclusion.
vi. Angle’s classification considers only static occlusion. Ans.
vii. Aetiology of malocclusion is not highlighted.
viii. This classification does not differentiate between [Same as SE Q.5]
dentoalveolar and skeletal malocclusions.
ix. Angle has not considered individual tooth malpositions. SHORT NOTES:
Q.4. Angle’s classification of malocclusion. Q.1. Bennett’s classification.
Ans. Ans.
[Ref LE Q.4 ]
Norman Bennet classified malocclusion based on its aetiology
Q.5. Characteristics of Angle’s class II division 2 maloc- Class I – Malocclusion or abnormal position of one or more
clusion. teeth due to local causes
Ans. Class II – Malocclusion due to developmental defects of
bone in either arches
[Ref LE Q.2] Class III – Malocclusion due to abnormal relationship between
Q.6. Simon’s classification of malocclusion. upper and lower arches and between either arch or facial con-
tour, and correlated abnormal formation of either arch
Ans.
Q.2. Simon’s classification of malocclusion.
[Ref LE Q.1]
Ans.
Q.7. Dewey’s classification of malocclusion.
Ans. [Ref LE Q.1]

[Ref LE Q.4] Q.3. Validity of Angle’s classification.

Q.8. Pseudo-class III malocclusion. Ans.


Ans. [Ref LE Q.2]
56 Quick Review Series for BDS 4th Year, vol 1

Q.4. FH plane. Clinical features:


i. Molars in distocclusion
Ans.
ii. Retroclined central incisors l
i. Frankfort horizontal plane or eye-ear-plane (E-E-P) is ob- iii. Deep bite
tained by drawing a line through the margin of inferior or- iv. Broad square face with pleasing straight profile
bit below the eyeball and upper margin of auditory meatus. v. Backward path of closure
ii. This plane helps to detect deviations in the vertical plane. vi. Deep mentolabial sulcus
iii. Height of the dental arches and teeth is related to the vii. Absence of abnormal muscle activity
cranium.
These have perfectly acceptable function as well as facial
iv. Dental arch closer to this plane is called attraction, and
appearance.
farther away from this plane is called abstraction.
Q.5. Pseudo-class III malocclusion. Q.8. Define malocclusion. Name few classifications of
malocclusion.
Ans.
Ans.
l Pseudo-class III/habitual class III is not a true class III
malocclusion. Malocclusion is defined as a condition where there is depar-
l When the mandible moves from rest position to occlu- ture from normal relation of teeth in the same arch and to
sion due to occlusal prematurities, it slides forward into teeth in the opposing arch.
a pseudo-class III position. Quantitative and qualitative methods of classification
l These patients show normal molar relationship in rest are as follows:
position, while class III relation in centric occlusion. Qualitative methods of classification:
l It is habitual or developmental in origin. i. Angle’s classification
l Profile is either straight or concave with premature ii. Simon’s classification
contacts and deviated path of closure. iii. Bjork’s classification
l Retrusion of mandible is possible. iv. Ackerman–Proffit classification
l Treatment of pseudo-class III consists of elimination of v. Bennett’s classification, etc.
prematurities and replacement of last posterior teeth by Quantitative methods of classification:
functional space maintainers. i. Massler and Frankel
ii. Summer’s occlusal index
Q.6. Clinical features of class III malocclusion. iii. Occlusal feature index by Poulton
Ans. iv. Index for orthodontic treatment need by Shaw, etc.

l Angle’s class III malocclusion (mesio-occlusion) is a Q.9. Ackerman–Proffit classification.


condition in which the lower molar is positioned mesial
Ans.
to the upper molar.
l True class III is a skeletal malocclusion, it could be due l Ackerman and Proffit introduced a new method of clas-
to retrognathic maxilla, prognathic mandible or combi- sification system, which included Angle’s classification
nation of both. Here class III molar relation exists in and five major characteristics of malocclusion within a
both centric occlusion and rest position. Venn symbolic diagram.
l The Venn diagram is analysed in the following steps:
Clinical features of class III malocclusion are as follows:
Step 1 (Assessment of intra-arch alignment and symmetry):
l Molar relation: Mesiobuccal cusp of the upper first
l It is classified as ideal, crowded or spaced.
permanent molar occludes with the interdental space
Step 2 (Profile):
between the lower first and second permanent
l The profile is described as straight, convex and concave.
molars.
Step 3 (Type):
l Canine relation: Upper canine occludes with the inter-
The term type is used to describe the various kinds
dental space between lower first and second premolars.
of crossbites like buccal and palatal, unilateral and
l Line of occlusion: may or may not be altered.
bilateral or skeletal and dental.
l Reverse overjet or anterior crossbite and posterior crossbite.
Step 4 (Class; assessment of sagittal relationship)
l Concave profile.
It is classified as Angle’s class I/class II/class III
Q.7. Clinical features of class II division 2. malocclusion.
Step 5 (Bite depth; vertical plane is considered)
Ans.
l Open bite n anterior, posterior, skeletal or dental
l Angle’s class II division 2 malocclusion is characterized l Deep bite n dental or skeletal and posterior collapsed bite
by class II molar relationship with retroclined upper The overlapping groups are seen in the centre of a Venn
centrals that are overlapped by the lateral incisors. diagram. It has the most severe form of malocclusion
comprising problems in all the three dimensions.
Section | I  Topic-Wise Solved Questions of Previous Years 57

Q.10. Features of class II division 1 malocclusion. Skeletal class III:


Increased growth of mandible with prognathic profile.
Ans.
Q.14. Mention two differences between true and pseudo-
Angle’s class II division 1 malocclusion:
class III.
Class II division 1 malocclusion is characterized by
class II molar relation, i.e. the distobuccal cusp of the Ans.
upper first permanent molar occludes with the mesio-
Differences between true and pseudo-class III are as fol-
buccal groove of the lower first permanent molar with
lows:
proclined maxillary anterior teeth.
Clinical features: Feature True class III Pseudo-class III
i. Convex profile
i. Profile Concave Straight or concave
ii. Increased overjet, deep bite and exaggerated curve
of Spee ii. Premature contacts Absent Present
iii. Short hypotonic upper lip, lip trap and lack of ante- iii. Path of closure Forward Deviated
rior lip seal
iv. Gonial angle h or g Normal
iv. Patient exhibits abnormal muscle activity
v. Proclined lower anteriors, a natural compensation to v. Retrusion of mandible Not possible Possible
decrease overjet.
Q.15. Mention three planes used in Simon’s classification.
Q.11. Distocclusion.
Ans.
Ans.
l In Simon’s system, dental arches are related to three
l Angle’s class II malocclusion is known as distocclusion. anthropometric planes.
l In class II molar relation, the distobuccal cusp of the
l Simon had put forward a craniometric classification of
upper first molar occludes with the mesiobuccal groove malocclusion using three anthropometric planes, i.e.
of the lower first molar. i. The Frankfort horizontal plane
l Class II malocclusion has got two divisions, namely
ii. The orbital plane
division 1 and division 2. iii. The midsagittal plane
Angle’s class II division 1 malocclusion:
Class II division 1 malocclusion is characterized by class Q.16. Dewey’s modification of Angle’s classification of
II molar relation with proclined maxillary anterior teeth. malocclusion.
Angle’s class II division 2 malocclusion: Ans.
Class II division 2 malocclusion is characterized by
class II molar relationship with retroclined upper cen- Dewey’s modification of Angle’s classification:
trals that are overlapped by the lateral incisors. Martin Dewey has further divided Angle’s class I and III
Class II division 1 or 2 subdivision: into various types:
Condition when class II molar relation exists on only l Class I: Divided into five types.

one side with normal molar relation on the other side is Type 1: Crowded maxillary anterior teeth.
known as class II division 1 or 2 subdivision. Type 2: Proclined or labioversion of maxillary cen-
tral and lateral incisors.
Q.12. Lischer’s modifications of Angle’s classification. Type 3: Class I with anterior crossbite present.
Ans. Type 4: Class I with posterior crossbite present.
Type 5: Mesioversion of molars.
[Ref LE Q.4 ] l Class II: No types.

l Class III: Divided into three types.


Q.13. Skeletal classification of malocclusions.
Type 1: Well-aligned teeth and dental arches. Edge–
Ans. edge relationship exists.
Type 2: Crowded mandibular incisors.
Skeletal classification is based on the facial skeletal pattern
Type3: Crowded maxillary incisors.
and also relationship of teeth.
Skeletal class I: Q.17. Key ridge.
The bones of the face, maxilla and mandible are in nor-
Ans.
mal relation to each other.
Skeletal class II: i. Key ridge is the inferior point of anterior border of bony
l Mandibular development is retarded when compared buttress of zygoma.
with maxilla. ii. Angle said maxillary first molar is the most constant in
l Distal relationship of mandible to maxilla. position, and related it to key ridge position.
58 Quick Review Series for BDS 4th Year, vol 1

iii. Angles conviction was supported by Atkinson, who Q.20. True class III.
suggested a relative constancy of maxillary first molar
Ans.
and the bony buttress of the zygoma, which he called
the key ridge. [Same as SN Q.6]
Q.18. Drawbacks of Angle’s classification. Q.21. Name different classifications of malocclusion.
Ans. Ans.
[Same as SN Q.3] [Same as SN Q.8]
Q.19. Mention two merits and demerits of Angle’s clas- Q.22. Enlist various systems of classification of maloc-
sification. clusion.
Ans. Ans.
[Same as SN Q.3] [Same as SN Q.8]

Topic 8
Aetiology of Malocclusion
COMMONLY ASKED QUESTIONS
LONG ESSAYS:
1 . Define malocclusions. Discuss aetiology of malocclusion.
2. Discuss the environmental or local causes of malocclusion.
3. Enumerate various postnatal causes of malocclusion. Elaborate endocrinal factors.
4. Discuss the genetic and hereditary factors contributing towards the formation of malocclusion.
5. Explain in detail aetiology of malocclusion. [Same as LE Q.1]
6. Classify the aetiology of malocclusion. Discuss general factors in detail. [Same as LE Q.1]
7. Role of genetics in malocclusion. [Same as LE Q.4]

SHORT ESSAYS:
1 . Supernumerary tooth. [Ref LE Q.2]
2. Importance of genetics in malocclusion.
3. Mention local factors in the aetiology of malocclusion.
4. Graber’s classification of aetiological factors in malocclusion. [Ref LE Q.2]
5. Enumerate the prenatal causes of malocclusion.
6. Butler’s field theory.
7. Teratogens.
8. Genetic malocclusions. [Same as SE Q.2]

SHORT NOTES:
1 . Ankylosis.
2. General factors causing malocclusion.
3. Prenatal causes for malocclusion.
4. Local factors causing malocclusion. [Ref LE Q.2]
5. Teratogens.
6. Supernumerary tooth. [Ref LE Q.2]
7. Dilacerated tooth.
Section | I  Topic-Wise Solved Questions of Previous Years 59

8. Submerged tooth.
9. Supplemental teeth. [Ref LE Q.2]
10. Genetic malocclusions.
11. Enlist causes of midline diastema.
12. Acromegaly.
13. How dental caries cause malocclusion. [Ref LE Q.2]
14. Aetiology of crowding.
15. Prolonged retention of deciduous teeth. [Ref LE Q.2]
16. Blanch test.
17. Mention local factors in aetiology of malocclusion. [Same as SN Q.4]

SOLVED ANSWERS
LONG ESSAYS:
Q.1. Define malocclusions. Discuss aetiology of maloc- Moyer’s classification:–
clusion.
v. Habits: a. Thumb sucking and finger sucking
Ans. b. Tongue thrusting
c. Lip sucking and lip biting
According to Gardiner, White and Leighton, malocclusion d. Posture
is defined as a condition in which there is a departure from e. Nail biting
f. Other habits
the normal occlusion of the teeth to other teeth within the
same arch and to the teeth in the opposing arch. vi. Diseases: a. Systemic diseases
Aetiology of malocclusion: b. Endocrine disorders
c. Local diseases
(i) Nasopharyngeal diseases and
Genetic factors
disturbed respiratory function
(ii) Gingival and periodontal disease
Broad aetiological (iii) Tumours
factors (iv) Caries
Environmental vii. Malnutrition
factors

General White and Gardiner’s classification:

Graber has classified


aetiological factors as
Dental base Pre-eruption Posteruption
Local abnormalities abnormalities Abnormalities

Classification of aetiology of malocclusion i. Anteroposterior i. Abnormalities in i. Muscular


malrelationship position of a. Active muscle
Moyer’s classification:– ii. Vertical developing tooth force
malrelationship germ b. Rest position of
i. Heredity: a. Neuromuscular system
iii. Lateral ii. Missing teeth musculature
b. Bone
c. Teeth
malrelationship iii. Supernumerary c. Sucking habits
d. Soft parts iv. Disproportion teeth and teeth d. Abnormalities in
of size between abnormal in form path of closure
ii. Developmental defects of unknown origin teeth and basal iv. Prolonged ii. Premature loss of
iii. Trauma: a. Prenatal trauma and birth injuries bone retention of deciduous teeth
b. Postnatal trauma v. Congenital deciduous teeth iii. Extraction of
anomalies v. Large labial permanent teeth
iv. Physical agents: a. Premature extraction of primary teeth
frenum
b. Nature of food
vi. Traumatic injury
60 Quick Review Series for BDS 4th Year, vol 1

[SE Q.4]
{Graber’s classification of malocclusion:

General Factors Local Factors


1. Heredity Supernumerary teeth
1. Anomalies
of no. Missing teeth
(congenital absence of loss
2. Congenital due to accidents, caries etc.)
Prenatal
(trauma, maternal diet German measles) 2. Anomalies of tooth size
3. Anomalies of tooth shape
3. Environmental 4. Abnormal labial frenum: mucosal barriers
5. Premature loss of deciduous teeth
Postnatal 6. Prolonged retention of teeth
(birth injury, cerebral-palsy, TMJ injury) 7. Delayed eruption path

4. Predisposing metabolic and climatic diseases 8. Abnormal eruptive path


a. Endocrine imbalance
b. Metabolic disturbances
c. Infectious diseases
5. Dietary problems (nutritional deficiency) 9. Ankylosis
6. Abnormal pressure habits and functional aberrations 10. Dental caries
a. Abnormal sucking 11. Improper dental restoration
b. Thumb/finger sucking
c. Tongue thrust/tongue sucking
d. Lip and nail biting
e. Abnormal swallowing habits (improper deglutition)
f. Speech defects
g. Respiratory abnormalities (mouth breathing)
h. Tonsils and adenoids
i. Psychogenic ties and bruxism
7. Posture
8. Trauma and accidents

Q.2. Discuss the environmental or local causes of maloc-


C. Mucosal barriers
clusion.
i. Abnormal labial frenal attachment
Ans. ii. Soft tissue impaction
D. Dental caries
{SN Q.4} E. Improper dental restoration

Various environmental or local factors that cause maloc-


clusion are as follows: A. Disturbances of dental development
A. Disturbances of dental development i. Anomalies of number of teeth
l Supernumerary teeth
i. Anomalies of number
l Supplemental teeth
l Supernumerary teeth
l Missing teeth
l Missing teeth

ii. Anomalies of tooth size (SN Q.6 and SE Q.1)


iii. Anomalies of tooth shape
iv. Premature loss of deciduous teeth
{(Supernumerary teeth
l Teeth that are extra to the normal complement
v. Prolonged retention of deciduous teeth
are known as supernumerary teeth. They do
vi. Delayed eruption of permanent teeth
not resemble normal teeth and are usually
vii. Abnormal eruptive path
conical in shape.
B. Trauma to the teeth
l They can occur singly or in pair.
Section | I  Topic-Wise Solved Questions of Previous Years 61

Example: Mesiodense and paramolars l Abnormally large cingulum on a maxillary inci-


Mesiodense is most frequently seen supernumer- sor tooth, prevents establishment of normal over-
ary tooth.)} bite and overjet and involved tooth is in labiover-
sion due to forces of occlusion.
{SN Q.9} l Additional lingual cusp on increases MD
5 5
dimensions of tooth.
Supplemental teeth: l Congenital syphilis causes peg laterals and mul-
l Extra teeth that resemble normal teeth are called
berry molars.
supplemental teeth. l Developmental defects like amelogenesis imper-
Example: most often seen in 245 region fecta, hypoplasia of teeth, fusion and gemination
245
l {Effects of supernumerary and supplemental teeth
causes anomalies of shape.
l Dilaceration is characterized by abnormal angula-
are as follows:
a. They cause noneruption to adjacent teeth. tion between crown and root of a tooth or angula-
b. They deflect erupting adjacent teeth into abnor- tion within the root. Dilacerated teeth fail to erupt
mal location. to normal level and can cause malocclusion.
c. They can result in crowding and rotation of adja- iv. Premature loss of deciduous teeth:
l It refers to loss of a tooth before its permanent suc-
cent teeth.
d. Unerupted mesiodense is one of the causes of cessor is sufficiently advanced in development and
midline spacing. eruption to occupy its place.
l The severity of malocclusion caused due to early loss of
e. Unerupted supernumerary teeth are potential risk
factors for cystic transformation.} a deciduous tooth depends on following factors:
a. Premature loss of deciduous molars predispose to
malocclusion due to shifting of adjacent teeth into
the space.
Missing teeth: b. The earlier the deciduous teeth are extracted be-
l Congenitally missing teeth are far more common
fore successional teeth are ready to erupt, the
than supernumerary teeth. greater is the possibility of malocclusion.
l Some of the commonly missing teeth in decreas-
c. In cases of arch length deficiency or crowding, the
ing order of frequency are as follows: third mo- early loss of deciduous teeth worsens the existing
malocclusion.
lars, 2 2 , , , 5 5 v. Prolonged retention of deciduous teeth:
5 5 1 1
l Can be unilateral or bilateral It is a condition where there is undue retention of de-
l Spacing in dental arches ciduous teeth beyond the usual eruption age of their
l Migration of adjacent teeth – abnormal location permanent successors.
and axial inclination of teeth
l Over-retained deciduous teeth – because of
{SN Q.15}
absence of permanent teeth.
ii. Anomalies of tooth size: Prolonged retention of deciduous teeth
l Increase in size of teeth results in crowding, e.g.

fusion between two adjacent teeth and between


normal tooth and supernumerary tooth.
Anteriors Buccal teeth
l Smaller size teeth result in spacing, e.g. commonly
Result in Result in
seen are smaller size 2 2 .
Lingual or palatal eruption of Eruption of permanent teeth
Size of teeth is to a large extent genetically deter- their permanent successors either buccally or lingual or
mined. may remain impacted
Most of these conditions show positive family
history.
iii. Anomalies of tooth shape:
Anomalies of tooth size and shape are interrelated; Aetiology of prolonged retention of deciduous teeth:
frequently seen tooth shape anomalies are a. Absence of underlying permanent teeth
2 2 are accompanied by spacing and b. Endocrinal disturbances, e.g. hypothyroidism
l Peg shape
c. Ankylosed deciduous teeth that fail to resorb
migration of teeth. d. Nonvital deciduous teeth that do not resorb
62 Quick Review Series for BDS 4th Year, vol 1

vi. Delayed eruption of permanent teeth:


{SN Q.13}
Delayed eruption of permanent teeth may be caused
due to the following reasons: D. Dental caries
l Congenital absence of permanent tooth

l Supernumerary tooth blocking eruption of Dental caries


permanent tooth
l Presence of a heavy mucosal barrier

l Premature loss of deciduous tooth (because of Premature loss of deciduous/permanent teeth


formation of bone over erupting permanent Causes
tooth)
l Endocrinal disorders – Hypothyroidism delays Migration of contiguous teeth
eruption of teeth
l Presence of deciduous root fragments – These

are not resorbed and can block erupting perma- Abnormal axial inclination and supra-eruption
nent teeth of opposing teeth)
vii. Abnormal eruptive path:
The abnormal path of eruption may be because of
the following reasons:
l Arch length deficiency
E. Improper dental restoration
l Presence of supernumerary teeth or retained
Improper occlusal restoration:
root fragments i. Overcontoured restoration leads to premature
l Formation of bony barrier
contacts and functional shift of mandibular during
jaw closure.
Example: 3 3 Most often found erupting in abnor- ii. Undercontoured restoration permits supraerup-
mal position as they develop almost near the floor of tion of opposing dentition.
the orbit and travel down to their final position in Proximal restoration:
oral cavity. Undercontoured restoration leads to loss of arch
B. Trauma to the teeth: length and food lodgement.
l Trauma to primary teeth may cause displacement Overcontoured restorations consume more space
of permanent teeth, defective enamel formation or leading to irregularity of dentition.
dilacerated roots. Q.3. Enumerate various postnatal causes of malocclu-
l Trauma to permanent teeth may result in ankylosis,
sion. Elaborate endocrinal factors.
nonvitality and displacement of tooth.
l In ankylosis, a part or whole of the root surface is Ans.
directly fused to bone in absence of intervening peri- Various postnatal causes of malocclusion are catogerized
odontal membrane. into:
l Ankylosed teeth fail to erupt to normal level (sub-
A. Developmental disturbances:
merged teeth within jaws). Cause migration of adja- i. Endocrine disturbances
cent teeth into the space. ii. Nutritional deficiencies
C. Mucosal barriers: iii. Allergy
i. Abnormal labial frenal attachment iv. Muscular activity
l Abnormalities of maxillary labial frenum are
v. TMJ problems
quite often associated with maxillary midline B. Functional disturbances:
spacing. i. Head and tongue postures
l Rarely a heavy fibrous frenum is found attached
ii. Various habits like mouth breathing, thumb sucking,
to the interdental papilla region. This can prevent tongue thrusting and abnormal swallowing.
the two maxillary central incisors 1 1 from iii. Functional shifts
approximating each other. C. Environmental interferences:
l Diagnosed by a positive blanch test. i. Disturbances of dental development
l A midline IOPA or occlusal radiograph exhibits l Missing teeth

notching of the interdental alveolar crest. l Malformed teeth

ii. Soft tissue impaction: l Supernumerary and supplemental teeth

Thick mucosal tissue covering erupting teeth acts as l Delayed eruption

a barrier to path of eruption and results in soft tissue l Ectopic eruption

impaction. l Early loss of primary teeth


Section | I  Topic-Wise Solved Questions of Previous Years 63

ii. Trauma to teeth l The patient may have osteoporosis, which con-
iii. Dental caries traindicates orthodontic treatment.
iv. Mucosal barrier, e.g. persistent labial frenum l Rare in children.

Endocrine disturbances or problems and their mani- l Hypertension; wide eyes staring; anxious

festations: looks; very poor dental patients.


i. Hypopituitarism: Hypoparathyroidism:
l In hypopituitary dwarfism, the eruption rate l It is associated with changes in calcium

and shedding time of teeth are delayed, as is metabolism.


the growth of the body in general. l It can cause delay in tooth eruption, altered

l The dental arch is smaller than normal and tooth morphology, delayed eruption of de-
cannot accommodate all the teeth; hence the ciduous and permanent teeth and hypoplas-
malocclusion develops. Development of max- tic teeth. Morphology of teeth is affected.
illa is not as retarded as mandible, hence re- Hyperparathyroidism:
sults in class II. l It produces increase in blood calcium. There

l Thickening of dentinal walls at the expense of is demineralization of bone and disruption


pulp chambers. Amelogenesis directly de- of trabecular pattern.
pends on pituitary hormones but dentinogene- l In growing children, tooth development is

sis and cementogenesis can proceed at de- interrupted.


creased rate without pituitary hormones. l The teeth may become mobile due to loss of
ii. Hyperpituitarism: lamina dura, cortical bone and resorption of
l It results in gigantism and acromegaly. the alveolar process.
l Gigantism is due to hypersecretion of GH in Q.4. Discuss the genetic and hereditary factors contrib-
childhood before fusion of epiphysis of bone uting towards the formation of malocclusion.
with shaft. Acromegaly is due to hypersecre-
Ans.
tion of GH in adults.
l Enlargements of bones (especially mandible), l Conditions that are caused due to disturbances in germ
kyphosis (bowing of spine) and bulldog scalp plasma or chromosomes or genes are known as genetic
are important features. disorders.
l Gigantism is characterized by a general sym- l Genetic disorders can be classified into:
metric overgrowth of the body. i. Hereditary disorders:
Skeletal changes include enlarged supraorbital The conditions which are transmitted from one gen-
ridges and prognathic mandible, enlargement of eration to another are known as hereditary disorders.
the tongue and whole face. Patients may develop Neel’s criteria for considering a problem as heredi-
class III malocclusion with interdental spacing. tary disorder are as follows:
l Mandibular prognathism, labial or buccaly l Occurrence of disease in definite numerical
tilted teeth and enlarged tongue are the fea- proportions among individuals related by
tures of acromegaly. descent.
iii. Hypothyroidism (cretinism): l Failure of disease to spread to nonrelated indi-
l Congenital hypothyroidism or cretinism leads viduals.
to stunted height, enlargement of tongue, l No known precipitation factor.
spaced dentition and delayed dental age. l Greater concordance of disorder in identical
l Constant protrusion of enlarged tongue leads twins.
to malocclusion. ii. Mutational disorders:
l The eruption rate of teeth is delayed and de- l In a previously unaffected individual, the muta-
ciduous teeth are retained beyond normal tional disorders arise de novo because of damage
shedding time. to the germ plasma.
iv. Hyperthyroidism: l If mutational disorders are transmitted to the
l This condition is characterized by increase in future generation, it becomes hereditary.
the rate of maturation and metabolic rate.
Types of transmission of malocclusion:
l The patient exhibits premature eruption of de-
Malocclusions are transmitted by three ways:
ciduous teeth, disturbed root resorption of
i. Repetitive
deciduous teeth and accelerated eruption of
ii. Discontinuous
permanent teeth.
iii. Variable
64 Quick Review Series for BDS 4th Year, vol 1

l Repetitive: Recurrence of a single dentofacial deformity According to Lundstrom, the human traits influenced by
within the immediate family. the genes include:
l Discontinuous: Recurrence of tendency for a malocclu-

sal trait to reappear after few generations. Some genera-


tions will be skipped.
l Variable: Expression of different but related types of mal-
Microdontia
occlusion within the several generations of the same family. I. Tooth size
Genetic influence: Macrodontia
Malocclusions could be produced by heredity in two major
Arch length
possible ways:
i. Inherited disproportion between size of teeth and size II. Arch dimensions
of jaws. Arch width
ii. Inherited disproportion between size and shape of up- III. Crowding and spacing Uncoordinated inheritance
per and lower jaws, which leads to occlusal malrela- of arch length and tooth
tionships. material
l Genetic disorders seen at the time of birth are
IV. Abnormalities of tooth shape – e.g. peg laterals
called congenital defects.
(high genetic predisposition)
l Heredity plays a role in the following conditions:

a. Congenital deformities Anodontia


b. Facial asymmetry V. Abnormalities of tooth number
c. Cleft lip and palate Oligodontia
d. Mandibular prognathism and retrognathism
e. Micrognathia and macrognathia VI. Overjet – believed to be influenced genetically.
f. Variations of tooth shape etc. VII. Interarch variations: Discrepancies in
l Various modes of inheritance are:

a. Autosomal dominance and recessive


b. X-linked
Transverse Sagittal Vertical planes
c. Polygenic
can be inherited
d. Chromosomal
l Contemporary views on aetiology of malocclusion
VIII. Frenum: size, position and shape: genetically influenced.
attribute some of the malocclusions to hereditary or Example: midline diastema
genetic causes.
i. Dental problem:
a. Crowding – hereditary and environmental reasons According to Harris and Johnson: A number of cranio-
b. Individual tooth malalignments and crossbites – facial parameters showed significant genetic influence.
pressure environment Example: Sella – gnathion
ii. Skeletal problem: Sella – point A
Mostly attributed to inherited or genetic cause. Sella – gonion
Example: Nasion – anterior nasal spine
a. Retrognathic mandible and maxilla Articulare – pogonion
b. Prognathic mandible Bizygomatic width
c. Skeletal deep bite Anterior facial height
Heredity: As so many traits show a strong genetic pattern, a
It has long been attributed as one of the causes of real number of malocclusions can be partly or solely at-
malocclusion. tributed to genetic factors. These genetic traits can be
i. The child inherits conflicting traits from both par- further influenced by existing prenatal and postnatal
ents who have dissimilar genetic material, resulting environmental factors.
in abnormalities of dentofacial region.
ii. Uncoordinated inheritance of teeth and jaws is a Q.5. Explain in detail aetiology of malocclusion.
result of racial, ethnic and regional intermixture. Ans.
This is another reason attributed to genetically de-
termined malocclusions. [Same as LE Q.1]
Section | I  Topic-Wise Solved Questions of Previous Years 65

Q.6. Classify the aetiology of malocclusion. Discuss According to Harris and Johnson: A number of cranio-
general factors in detail. facial parameters showed significant genetic influence.
Example: Sella – gnathion, Nasion – anterior nasal
Ans.
spine, articulare – pogonion, bizygomatic width, ante-
[Same as LE Q.1] rior facial height.
A number of malocclusions can be partly or solely at-
Q.7. Role of genetics in malocclusion.
tributed to genetic factors. These genetic traits can be
Ans. further influenced by existing prenatal and postnatal
environmental factors.
[Same as LE Q.4]
Q.3. Mention local factors in the aetiology of malocclusion.
SHORT ESSAYS: Ans.
Q.1. Supernumerary tooth. Various environmental or local factors that cause malocclu-
sion are as follows:
Ans. A. Disturbances of dental development:
[Ref LE Q.2] i. Anomalies of number
ii. Anomalies of tooth size
Q.2. Importance of genetics in malocclusion. iii. Anomalies of tooth shape
Ans. iv. Premature loss of deciduous teeth
v. Prolonged retention of deciduous teeth
l Conditions that are caused due to disturbances in germ vi. Delayed eruption of permanent teeth
plasma or chromosomes or genes are known as genetic vii. Abnormal eruptive path
disorders. B. Trauma to the teeth
l Genetic disorders can be classified into: C. Mucosal barriers
i. Hereditary D. Dental caries
ii. Mutational E. Improper dental restoration
l The conditions which are transmitted from one genera-

tion to another are known as hereditary disorders. A. Disturbances of dental development


l According to Lundstrom, the human traits influenced by
i. Anomalies in number of teeth
l Teeth that are extra to the normal complement
the genes include:
are known as supernumerary teeth. They do
not resemble normal teeth and are usually
Microdontia conical in shape.
I. Tooth size Example: mesiodense and paramolars.
l Extra teeth that resemble normal teeth are
Macrodontia called supplemental teeth.
Arch length 245
For example, most often seen in region.
II. Arch dimensions 245
l Congenitally missing teeth are far more com-
Arch width mon than supernumerary teeth.
III. Crowding and spacing Uncoordinated inheritance Example: third molars and maxillary lateral
of archlength and tooth incisors.
material ii. Anomalies of tooth size:
IV. Abnormalities of tooth shape, e.g. peg laterals l Increase in size of teeth macrodontia results

in crowding, e.g. fusion between two adjacent


Anodontia teeth and between normal tooth and supernu-
merary tooth.
V. Abnormalities of tooth number l Smaller size teeth, i.e. microdontia results in
Oligodontia spacing 2 2 .
VI. Overjet – believed to be influenced genetically. iii. Anomalies of tooth shape:
VII. Interarch variations: Discrepancies in l Anomalies of tooth size and shape are inter-

related. Frequently seen tooth shape anoma-


lies are peg shape 2 2 .
Transverse Sagittal Vertical planes l Developmental defects like amelogenesis im-
can be inherited perfecta, hypoplasia of teeth, fusion and gem-
VIII. Frenum: Size, position and shape: influenced genetically. ination cause anomalies of shape.
66 Quick Review Series for BDS 4th Year, vol 1

iv. Premature loss of deciduous teeth: jaw closure, while undercontoured restoration
l It refers to loss of a tooth before its permanent permits supraeruption of opposing dentition.
successor is sufficiently advanced in develop- l Proximal restoration:

ment and eruption to occupy its place. Undercontoured restoration leads to loss of arch
v. Prolonged retention of deciduous teeth: length and food lodgement, while overcontoured
restorations consume more space, leading to ir-
Prolonged retention of deciduous teeth regularity of dentition.
Q.4. Graber’s classification of aetiological factors in
malocclusion.
Anteriors Buccal teeth
Result in Result in Ans.

Lingual or palatal eruption of Eruption of permanent teeth


[Ref LE Q.2]
their permanent successors either buccally or lingually or Q.5. Enumerate the prenatal causes of malocclusion.
may remain impacted
Ans.
vi. Delayed eruption of permanent teeth:
Delayed eruption of permanent teeth may be The various prenatal causes of malocclusion are as follows:
l The foetus is well protected against injuries and nutri-
caused due to congenital absence of permanent
tooth, supernumerary tooth blocking eruption of tional deficiencies during pregnancy, but certain factors
permanent tooth, endocrinal disorders like hypo- can result in abnormal growth of the orofacial region,
thyroidism, etc. thereby predisposing to malocclusion.
l Abnormal fetal posture during gestation is said to inter-
vii. Abnormal eruptive path:
The abnormal path of eruption may be because fere with symmetric development of the face. Most of
of arch length deficiency, presence of supernu- these deformities are temporary and usually disappear
merary teeth or retained root fragments etc. as age advances.
l The other prenatal influences include maternal fibroids,
B. Trauma to the teeth
l Trauma to primary teeth may cause displacement
amniotic lesions, maternal diet and metabolism.
l Maternal infection such as German measles and use of
of permanent teeth, defective enamel formation or
dilacerated roots. certain drugs during pregnancy such as thalidomide can
l Trauma to permanent teeth may result in ankylo-
cause gross congenital deformities, including clefts.
sis, nonvitality and displacement of tooth. Q.6. Butler’s field theory.
C. Mucosal barriers
l Abnormal labial frenal attachment is quite often Ans.
associated with maxillary midline spacing. l The human dentition is divided into four fields: (i) inci-
l Thick mucosal tissue covering erupting teeth acts
sor (ii) canine, (iii) premolar and (iv) molar regions.
as a barrier to path of eruption, and results in soft l The most distal tooth in each field is the most suscepti-
tissue impaction. ble to changes or variations like absence of tooth, varia-
D. Dental caries tion in size, shape and structure.
l Accordingly, lateral incisors, second premolars and
Dental caries
third molars are the most variable tooth in their group,
this is called ‘Butler’s field theory’.
l Canine is the least variable tooth in the arch.
Premature loss of deciduous/permanent teeth
Butler’s field theory does not apply in lower anterior
Causes region, where mandibular central incisor is more com-
monly missing than lateral incisor.
Migration of contiguous teeth
Q.7. Teratogens.
Ans.
Abnormal axial inclination and supraeruption
of opposing teeth l Chemical or other agents which cross the placental barrier
and produce embryologic defects are called teratogens.
E. Improper dental restoration l The various teratogens and their effects are as follows:
l Occlusal restoration: Teratogens effect
Overcontoured restoration leads to premature i. Aspirin, cigarette smoke – cleft lip and palate di-
contacts and functional shift of mandible during lantin and valium
Section | I  Topic-Wise Solved Questions of Previous Years 67

ii. 6-Mercaptopurine – cleft palate Q.4. Local factors causing malocclusion.


iii. Aminopterin – anencephaly
Ans.
iv. Cytomegalovirus – microcephaly, hydrocephaly
v. Ethyl alcohol – central midface deficiency [Ref LE Q.2]
vi. 13-cis-retinoic acid – retinoic acid syndrome
Q.5. Teratogens.
vii. Rubella virus – microphthalmia, deafness
viii. Thalidomide – hemifacial microsomia-like Ans.
features
l Chemical or other agents which cross the placental
ix. Toxoplasma – microcephaly, hydrocephaly
barrier and produce embryologic defects are called
x. Radiation – microcephaly
teratogens.
xi. Vitamin D excess – premature suture closure
l Examples of various teratogens and their effects are as

Q.8. Genetic malocclusions. follows:


Teratogens effect
Ans.
i. Aspirin, cigarette smoke, dilantin and valium –
[Same as SE Q.2] cleft lip and palate
ii. 6-mercaptopurine – cleft palate
iii. Cytomegalovirus – microcephaly, hydrocephaly
SHORT NOTES: iv. Ethyl alcohol – central midface deficiency
Q.1. Ankylosis. v. Radiation – microcephaly
vi. Vitamin D excess – premature suture closure
Ans.
Q.6. Supernumerary tooth.
i. Ankylosis is a condition wherein a part or whole of the
root surface is directly fused to the bone with the ab- Ans.
sence of the intervening periodontal membrane. [Ref LE Q.2]
ii. It occurs most often as a result of trauma to the tooth
which perforates the periodontal membrane. Q.7. Dilacerated tooth.
iii. It can also be associated with certain infections, endo- Ans.
crinal disorders and congenital disorders like cleidocra-
nial dysostosis. l Dilaceration is described as a condition characterized
iv. Clinically, these teeth fail to erupt to the normal level by an abnormal angulation between the crown and root
and are, therefore, called submerged teeth. of a tooth or angulation within the root.
l It usually occurs due to trauma to a deciduous tooth and
Q.2. General factors causing malocclusion. is transmitted to the underlying permanent tooth bud.
l Dilacerated teeth fail to erupt to normal level and can
Ans.
thus cause malocclusion.
General factors causing malocclusion are as follows:
i. Heredity Q.8. Submerged tooth.
ii. Congenital Ans.
iii. Predisposing metabolic and climatic diseases
iv. Dietary problems (nutritional deficiency) i. Clinically, ankylosed teeth fail to erupt to the normal
v. Abnormal pressure habits and functional aberrations, level and are therefore called submerged teeth.
e.g. thumb/finger sucking, tongue thrust and nail biting ii. At times these teeth are totally submerged within the
vi. Posture jaw and therefore cause migration of adjacent teeth into
vii. Trauma and accidents the space.
Q.9. Supplemental teeth.
Q.3. Prenatal causes for malocclusion.
Ans.
Ans.
[Ref LE Q.2]
The various prenatal causes of malocclusion are as follows:
l Abnormal fetal posture during gestation interferes with Q.10. Genetic malocclusions.
symmetric development of the face.
Ans.
l Prenatal influences include maternal fibroids, amniotic

lesions, maternal diet and metabolism. l Conditions that are caused due to disturbances in germ
l Maternal infections, e.g. German measles. plasma or chromosomes or genes are known as genetic
l Use of certain drugs during pregnancy such as thalido- disorders.
mide can cause gross congenital deformities, including l Genetic disorders can be classified as hereditary and

clefts. mutational.
68 Quick Review Series for BDS 4th Year, vol 1

l According to Lundstrom, the human traits influenced by l Mandibular prognathism, labial or buccaly tilted teeth
the genes include, e.g. and enlarged tongue are the features of acromegaly.
Tooth size like microdontia and macrodontia.
Q.13. How dental caries cause malocclusion.
Arch dimensions, i.e. arch length and width.
Abnormalities of tooth shape – e.g. peg laterals. Ans.
Crowding and spacing because of uncoordinated inheri- [Ref LE Q.2]
tance of arch length and tooth material.
Q.14. Aetiology of crowding.
Q.11. Enlist causes of midline diastema.
Ans.
Ans.
l Crowding and spacing of teeth are believed to be of
l Midline diastema is a form of localized spacing, where genetic origin.
spacing is present between two central incisors. l Most of these conditions are believed to be a result of un-
l It occurs due to a number of causes: coordinated inheritance of arch length and tooth material.
A. Normal developmental causes: l Arch length–tooth material discrepancy leads to crowding.
i. Physiological median daistema
ii. Ethnic and familial Q.15. Prolonged retention of deciduous teeth.
B. Tooth material deficiency: Ans.
i. Microdontia
ii. Missing lateral and peg lateral [Ref LE Q.2]
C. Physical impediment: Q.16. Blanch test.
i. Retained deciduous teeth
Ans.
ii. Mesiodens
D. Habits: l High frenal attachment condition is diagnosed by a
Thumb sucking, tongue thrusting etc. positive blanch test.
l A heavy fibrous frenum is found attached to the inter-
Q.12. Acromegaly.
dental papilla region. This type of frenal attachment can
Ans. prevent the two maxillary central incisors from ap-
proximating each other, leading to midline diastema.
l Hyperpituitarism results in gigantism and acromegaly.
l The procedure of blanch test
l Acromegaly is due to hypersecretion of GH in adults.
When the upper lip is stretched for a period, a notice-
Gigantism is due to hypersecretion of GH in childhood
able blanching occurs over the interdental papilla. A
before fusion of epiphysis of bone with shaft.
midline intraoral periapical radiograph usually exhibits
l Gigantism is characterized by a general symmetric
notching of interdental alveolar crest.
overgrowth of the body.
l Skeletal changes include enlarged supraorbital ridges Q.17. Mention local factors in aetiology of malocclusion.
and prognathic mandible, enlargement of the tongue
Ans.
and whole face. Patients may develop class III maloc-
clusion with interdental spacing. [Same as SN Q.4]

Topic 9
Oral Habits
COMMONLY ASKED QUESTIONS
LONG ESSAYS:
1. Define habits. Classify and discuss in detail the features of tongue thrusting habit and its treatment modalities.
2 . Enumerate the aetiological factors causing mouth breathing in children and discuss the line of treatment.
3. Define and classify habits. Discuss clinical pictures and management of thumb-sucking habit.
4. Describe how tongue thrust, mouth breathing and thumb-sucking habits cause malocclusion.
5. Classify tongue thrust habit. Discuss its aetiology, clinical features and management. [Same as LE Q.1]
Section | I  Topic-Wise Solved Questions of Previous Years 69

6. Define abnormal pressure habits. Write in detail about the classification, clinical findings and treatment for
mouth breathers. [Same as LE Q.2]
7. Mouth breathing habit and its effects on the development of occlusion. [Same as LE Q.2]
8. Describe various oral habits directly responsible for causing malocclusion. Discuss management of one such
habit in detail. [Same as LE Q.3]
9. Define a habit. How do you classify habits; discuss the aetiology, effects and management of thumb sucking.
[Same as LE Q.3]
10. Define preventive and interceptive orthodontics. Discuss the management of thumb-sucking habit. [Same as LE Q.3]

SHORT ESSAYS:
1. Thumb-sucking habit.
2 . Mouth breathing habit. [Ref LE Q.2]
3. Classify tongue thrust habit. Add a note on its management. [Ref LE Q.1]
4. Bruxism.
5. Mouth breathing habit and its effects on the development of occlusion. [Same as SE Q.2]
6. Write briefly about tongue thrust habit. [Same as SE Q.3]

SHORT NOTES:
1. Mouth breathing.
2. Thumb-sucking habit.
3. Lip sucking and lip biting.
4. Define tongue thrusting habit.
5. Adenoid facies.
6. Classify tongue thrust habit. [Ref LE Q.1]
7. Write a note on management of tongue thrust habit.
8. Habit breaking appliances. [Ref LE Q.3]
9. Lip biting.
10. Bruxism.
11. Management of thumb sucking habit.
12. Pernicious oral habits.
13. Lip bumper.
14. Nail biting.
15. Beta (b) hypothesis.
16. Role of mouth breathing in the aetiology of malocclusion. [Same as SN Q.1]
17. Aetiology of tongue thrust. [Same as SN Q.4]

SOLVED ANSWERS
LONG ESSAYS:
Q.1. Define habits. Classify and discuss in detail the l This deleterious habit clinically presents with open bite
features of tongue thrusting habit and its treatment and anterior proclination.
modalities. Aetiology
The factors considered as a cause of tongue thrusting
Ans.
habit according to Fletcher are as follows:
i. Genetic factors
[SE Q.3] ii. Learned behaviour (habit)
l {Habit can be defined as the tendency towards an act
iii. Maturational factors
that has become a repeated performance, relatively iv. Mechanical restrictions
fixed, consistent and easy to perform by an individual. v. Neurological disturbance
l Tongue thrusting is defined as a condition in which the vi. Psychogenic factors
tongue makes contact with any teeth anterior to the mo- i. Genetic factors: Specific anatomic or neuromus-
lars during swallowing. cular variations in orofacial region can precipitate
70 Quick Review Series for BDS 4th Year, vol 1

tongue thrust, e.g. hypertonic orbicularis oris Clinical features:


activity. The clinical features seen in tongue thrusting condition
ii. Learned behaviour (habit): Tongue thrusting can be are dependent on type of tongue thrusting.
acquired as a habit due to following predisposing Some common clinical features of tongue thrust habit
factors: are as follows:
a. Improper bottle feeding l Proclination of anterior teeth

b. Prolonged thumb sucking l Bimaxillary protrusion

c. Prolonged tonsillar or upper respiratory tract l Anterior open bite

infections l In case of lateral tongue thrust, posterior open

d. Prolonged duration of tenderness of gums or bite and posterior crossbite


teeth tends to change the swallowing pattern i. The Simple tongue thrust habit:
iii. Maturational factors: l It is also called teeth together swallow. There is

a. Macroglossia normal tooth contact during swallowing.


b. Constricted dental arches l Generalized spacing and proclination may be

c. Enlarged adenoids and tonsils seen in the upper and lower anterior teeth.
They cause tongue to be positioned anteriorly l Increased overjet, reduced overbite or presence

to prevent blocking of the oropharynx. of anterior open bite may be seen.


iv. Neurological disturbance: Orofacial region can l Tongue is thrust forward during swallowing to
cause tongue thrust habit, e.g. hyposensitive palate help in establishing anterior lip seal.
and moderate motor disability. l Exaggerated perioral musculature during the
v. Psychogenic factors: Tongue thrusting habit may swallowing action. Especially hyperactive men-
develop because of forced discontinuation of other talis muscle activity is seen.
habits like thumb sucking. ii. Complex tongue thrust habit:
l It is defined as tongue thrust with teeth apart

swallow.
{SN Q.6} l There are two important diagnostic features:

generalized open bite and poor occlusal fit of


Classification of tongue thrusting habit teeth which leads to sliding occlusion.
According to James S. Braner and Holt. l Absence of temporal muscle constriction during

swallowing, and the mandible is not stabilized by


Type I: Non-deforming tongue thrust
the elevator muscles.
Type II: Deforming anterior tongue thrust
iii. Retained infantile swallow:
i. Anterior open bite
Little is known about the exact aetiology of this severe
3 subgroups: ii. Anteriorproclination
problem.
iii. Posterior crossbite
Treatment
Type III: Deforming lateral tongue thrust
l Interception and treatment of tongue thrusting is age-
i. Posterior open bite
and severity-dependent. In children younger than
3 Subgroups ii. Posterior crossbite
iii. Deep over bite 3 years, no active intervention is instituted whereas
children older than this age can be trained for tongue
Type IV: Deforming anterior and lateral tongue thrust
i. Anterior and posterior open bite
swallowing exercises.
l The various modalities of treatment of tongue thrust-
3 Subgroups ii. Proclination of anterior teeth
iii. Posterior crossbite ing are as follows:
I. Reminder therapy/interception of habit
II. Corrective therapy}
I. Reminder therapy/interception of habit:
Moyers classified tongue thrusting into three types: l Use of habit breakers both fixed and remov-

i. Simple tongue thrusting: characterized by teeth to- able cribs and rakes. Some of the commonly
gether swallow used removable appliances include upper
ii. Complex thrusting: characterized by teeth apart Hawley’s plate with tongue cribs and roller
swallow balls for tongue exercise.
iii. Retained infantile swallow l Teaching correct method of swallowing to child.
Section | I  Topic-Wise Solved Questions of Previous Years 71

II. Corrective therapy: e. Treatment of malocclusion:


a. Removal of obstruction Malocclusion is treated with either removable or
b. Tongue exercises fixed orthodontic appliances.
c. Lip exercises
d. Habit breaking appliance
e. Treatment of malocclusion Q.2. Enumerate the aetiological factors causing
mouth breathing in children and discuss the line of
a. Removal of obstruction: treatment.
l Surgery for adenoids and macroglossia

l Closure of anterior and posterior open bite, Ans.


anterior spaces with either fixed or removable
[SE Q.2]
orthodontic appliance.
b. Tongue exercises: {i. Mouth breathing is an altered way of breathing through
Various muscle exercises of tongue to adapt it to mouth and is an adaptation to obstruction in nasal pas-
new swallowing pattern with removable or fixed sages. The obstruction may be temporary or recurrent.
orthodontic appliances are advised after habit is While more often it is partial than complete. The air
intercepted, they are as follows: way resistance may be enough to force the subject to
i. Elastic band swallow exercise: Small orthodon- breathe through the mouth.
tic elastic band is held up the tongue tip against ii. Mouth breathing is usually attributed as an aetiological
the palate during swallowing and asked to factor for malocclusion.
practise. If the swallow is correct, patient will iii. Mouth breathing results in altered jaw and tongue pos-
be able to hold the elastic, otherwise it falls or ture, which, in turn, alters orofacial equilibrium leading
will be swallowed. to malocclusion.
ii. Water swallow exercise: Patient is advised to Classification of mouth breathers:
keep water in mouth and a mirror in the hand
and swallowing is practised daily.
iii. Candy swallow exercise: A flat, sugarless Mouth breathers
candy is placed between tongue and palate Three types
and swallowing is practised.
This exercise reinforces the learning of new
swallowing pattern to be transferred to the Obstructive Habitual Anatomic
subconscious level. Complete or partial Due to deep-rooted Patients with short
iv. Speech exercise: Patient practises syllables like obstruction of nasal habit that is upper lip that does
c, g, h, k while keeping an elastic band between passage results in unconsciously not permit complete
the tongue and the palate. mouth breathing performed mouth closure
c. Lip exercises:
Patient practises stretching of lips to achieve an-
terior lip seal. Aetiology
d. Habit breaking appliance: A. Obstructive causes:
i. The tongue thrusting appliance has fixed l Nasal polyps
tongue spikes fabricated with 0.040˝ stain- l Obstructive adenoids
less steel alloy. It is V-shaped with three or l Congenital enlargement of nasal turbinates
four projections which extend up to cingu- l Chronic inflammation of nasal mucosa
lum of lower incisors soldered to molar l Benign tumours
bands or crowns. It acts as picket fence, l Deviated nasal septum
preventing or limiting the tongue. B. Anatomic causes:
ii. 5–10 years is the optimum age to use this l Short upper lip
appliance. l Underdeveloped nasal cavity
iii. A modified tongue crib is used in patients C. Obstructive sleep apnoea}
with lateral tongue thrusting habit. Pathophysiology of mouth breathing
72 Quick Review Series for BDS 4th Year, vol 1

Oral respiration Q.3. Define and classify habits. Discuss clinical pictures
and management of thumb-sucking habit.
Three changes in posture
Ans.

Lowering of Position of tongue Tipping back of Habit can be defined as the tendency towards an act that has
mandible (downward and forward) head become a repeated performance, relatively fixed, consistent
and easy to perform by an individual.
Classification:
Upset orofacial equilibrium According to different authors, habits have been classified
Unrestricted buccinators activity as follows:
Influences position of teeth
and growth of jaws Author Classification
Useful habits
[SE Q 2]
James (1923)
{Clinical features: Harmful habits
i. The type of malocclusion associated with mouth
breathing is called ‘Long Face Syndrome’ or clas- Morris and Pressure habits
sic adenoid facies or vertical maxillary excess. Bohanna Nonpressure habits
ii. Long and narrow face with short and flaccid upper (1969) Biting habits
lip.
Empty habits
iii. Expressionless, blank face.
Klein (1971)
iv. Anterior open bite.
v. Contraction of upper arch, narrow V-shaped upper Meaningful habits
jaw with a high narrow palate, posterior crossbite.
Compulsive habits
vi. h Overjet due to flaring of maxillary anteriors.
Finn (1987)
vii. Anterior marginal gingivitis and h caries incidence.
viii. Narrow nose and nasal passage, widely flared Noncompulsive habits
external nares.
ix. Excessive appearance of maxillary anterior teeth Classification of Habits
with a ‘GUMMY SMILE’.
Diagnosis is based on: Classification of Habits
i. History of the patient I. According to James
ii. Clinical examination
iii. Some simple tests:
l Mirror condensation test
Useful habits Harmful habits
l Water holding test

l Cotton wisp test The habits that are essential for The habits that have deleterious
iv. Cephalometrics: It helps to assess the normal function effect on the teeth and their
amount of nasopharyngeal space, size of supporting structures
adenoids and diagnosing long face.
Example: Example:
v. Rhinomanometry:
• Proper positioning of tongue • Thumb sucking
l Study of nasal airflow characteristics us-
• Respiration • Tongue thrusting etc.
ing devices like flow meter and pressure • Normal deglutition
gauges.
l Estimation of airflow through nasal pas- II. According to Morris and Bohanna
sage and amount of nasal resistance.
Treatment:
l Removal of nasal or pharyngeal obstruction by
Pressure habits Nonpressure habits Biting habits
ENT surgeon.
l Interception of the habit – vestibular screen adhe- Include sucking habits Habits which do not Include various
sive tapes for lip seal such as apply direct force on biting habits
l Rapid maxillary expansion – causes widening • Thumb sucking teeth or its supporting such as
of arch, resulting in h nasal flow and g nasal air • Lip sucking structures, such as • Nail biting
• Finger sucking • Pencil biting
resistance} • Mouth breathing
• Tongue thrusting • Lip biting
Section | I  Topic-Wise Solved Questions of Previous Years 73

III. According to Klein


The child has tendency to place his fingers
or any object into the oral cavity.
Empty habits Meaningful habits
Habits not associated with any Habits that have a psychological
Prevention of such habit is believed to result in
deep-rooted psychological bearing, e.g. nail biting, lip biting,
emotional insecurity and poses risk of child
problems, e.g. abnormal digit sucking
pillowing, chin propping diversifying into other habits.

IV. According to Finn


ii. Benjamin’s theory – suggests that thumb sucking
arises from rooting or placing reflex seen in all mam-
malian infants.
Compulsive habits Noncompulsive habits iii. Psychological aspects – feeling of insecurity in chil-
i. These are deep-rooted habits i. Include habits that are easily dren deprived of parental love and affection is believed
that have acquired fixation in learned, and dropped as the to resort to this habit.
the child to the extent that child matures. iv. Oral drive theory (1950) – proposed by Sears and
the child retreats to the habit Wise (1982), it stated that prolonged sucking can lead
whenever his security is to thumb sucking.
threatened by events that
occur around him.
ii. The child tends to suffer
Phases of development of thumb sucking
increased anxiety when an
attempt is made to correct Phase I Phase II Phase III
the habits. Normal and Clinical Intractable sucking
subclinically significant
Thumb sucking significant sucking
Digit sucking or thumb sucking is defined as placement Seen during first Seen during Persistent beyond the
of the thumb or one or more fingers in varying depths 3 years of life 3–6½ years fourth or fifth year of
into the mouth. and is considered of life life
quite normal
Aetiology
Causative factors include: No treatment Treatment: Treatment: Consulta-
(i) Parents’ occupation: Low socioeconomic groups required, usually Dental problems tion with a psycholo-
terminates at the should be solved gist is required in this
are more prone to thumb sucking. end of phase I in this phase phase
(ii) Working mother: Children with insecure feeling
cultivate the habit.
(iii) Number of siblings: More number of children leads
to neglection of child and development of this habit. Clinical effects of thumb sucking:
(iv) Order of birth of child: Later the sibling rank of a l Dentofacial changes associated with thumb sucking
child, greater the chances of oral habit. can affect – maxilla, mandible, interarch relationship,
(v) Social adjustment and stress. lip placement and function, etc.
(vi) Feeding practices. l The severity of malocclusion caused by thumb sucking
(vii) Age of the child. depends on the following factors:
a. Duration – amount of time spent in indulging in
Some of the more commonly accepted theories to explain
the habit
the thumb-sucking habit are as follows:
b. Frequency – number of times habit is activated/day
i. Freudian theory
c. Intensity – vigour with which the habit is
ii. Benjamin’s theory
performed
i ii. Psychological aspects
l The effects of thumb sucking on dental arch and its
iv. Oral drive theory
supporting structures are as follows:
v. Learned pattern theory
i. Proclination or labial tipping of maxillary ante-
i. Freudian theory (1905) – proposed by Sigmund Freud rior teeth.
l He suggested that a child passes through various ii. Increasing overjet – due to proclination of maxil-
phases of psychological development – of which oral lary anterior 1 lingual tipping of mandibular
and anal phases are seen in first 3 years of life. incisors.
74 Quick Review Series for BDS 4th Year, vol 1

iii. Anterior open bite (g overbite) – restriction of Two types of habit breakers
incisor eruption 1 supra-eruption of buccal teeth.
iv. Posterior crossbite – narrow maxillary arch Removable Fixed
predisposing to crossbite due to contraction of i. These are passive i. This appliance consists of
cheek muscles during thumb sucking. and removable molar bands/crowns on first
v. Tongue thrust – develops because of open bite. appliances permanent molars with palatal
vi. Effects on lip – hypotonic upper lip, hyperactive consisting of a assembly and soldered spurs
lower lip. h Lip incompetence, hyperactive men- crib placed pala- made of either nickel-chrome
tal to 21 12 and or stainless steel.
talis activity. anchored to oral Example:
vii. Other effects are psychological health, risk cavity by means l Quadhelix
of malposition of jaws, speech defects, digit of clasps on l Hay rakes

defects. posterior teeth. l Maxillary lingual arch

Diagnosis: Example: Tongue with palatal crib


spikes, tongue ii. The best appliance is heavy
i. History – frequency and duration of habit guard spurs/rake gauge stainless steel wire
ii. Child’s emotional status – assessed by enquiring designed to form a frame that
l Feeding habits is soldered to molar bands
l Parental care of child

l Working parents

iii. Examination of child’s fingers – presence of clean


nails and callus on finger
iv. Intraoral clinical examination – proclination, open
bite, etc.
Treatment:
l The child 1 parent 1 dentist form a team to assist

child in stopping the habit


l Treatment is mainly of the following three categories:

A. Psychological therapy
B. Reminder therapy or mechanotherapy – remov-
able habits breakers, and fixed habit breakers
C. Chemical approach
A. Psychological treatment:
a. Screening patients for underlying psychologi-
cal disturbances and referring to professionals
for counselling. Fig. 9.1  Tongue spike.
b. Children between 4 and 8 years age need
only reassurance, positive reinforcement and
friendly reminders to divert child’s attention to
other things like play and toys.
c. Dunlop’s beta hypothesis
l Forced purposeful repetition of a habit

eventually associated with unpleasant reac-


tions and habit is abandoned.
l Dunlop’s beta hypothesis is especially prac-

tised in older children, i.e. aged 8 years and


above.
B. Reminder therapy or mechanical aids:

{SN Q.8}
{The habit breaking appliances are basically reminding
appliances that assist to quit the habit.
Fig. 9.2  Tongue guard.
Section | I  Topic-Wise Solved Questions of Previous Years 75

C. Sucking habits:
i. Development of normal orofacial function is greatly
hindered by continuation of nonnutritive sucking
habits beyond 4–5 years of age.
ii. During thumb or finger sucking, mouth remains
open and the tongue is positioned forward and low
in the mouth, because of this an abnormal pressure
is generated by contraction of cheek muscles result-
ing in imbalance in intraoral force system.
iii. Exaggerated buccinator activity during sucking and
swallowing results in constricted maxilla, buccal
crossbite, lowered and backward mandibular pos-
ture resulting in class II division 1 malocclusion.
Q.5. Classify tongue thrust habit. Discuss its aetiology,
clinical features and management.
Fig. 9.3  Fixed habit breaker.
Ans.
[Same as LE Q.1]
Other mechanical aids used to intercept the
habit include: Q.6. Define abnormal pressure habits. Write in detail
l Bandaging of the thumb about the classification, clinical findings and treatment
l Bandaging of the elbow for mouth breathers.
C. Chemical approach: Ans.
l Use of bitter tasting or foul smelling prepara-

tions placed on thumb that is sucked can make [Same as LE Q.2]


the habit distasteful. Q.7. Mouth breathing habit and its effects on the devel-
l Commonly used medicaments are as follows: opment of occlusion.
a. Pepper dissolved in a volatile medium
b. Quinine Ans.
c. Asafoetida [Same as LE Q.2]
Q.4. Describe how tongue thrust, mouth breathing and Q.8. Describe various oral habits directly responsible
thumb-sucking habits cause malocclusion. for causing malocclusion. Discuss management of one
such habit in detail.
Ans.
Ans.
l Habit can be defined as the tendency towards an act that
has become a repeated performance, relatively fixed, [Same as LE Q.3]
consistent and easy to perform by an individual. Q.9. Define a habit. How do you classify habits; discuss
l Due to their repetitive nature and longer duration, the
the aetiology, effects and management of thumb-sucking.
orofacial habits influence the form of orofacial structures.
Ans.
The pathophysiology of various habits causing malocclu-
sion is as follows: [Same as LE Q.3]
A. Tongue thrusting habit: Q.10. Define preventive and interceptive orthodontics.
The repeated anterior positioning of the tongue leads to Discuss the management of thumb-sucking habit.
anterior open bite, protruded and spaced anterior teeth
and an incompetent anterior lip seal, all of these effects Ans.
lead to tongue thrusting-like situation. [Same as LE Q.3]
B. Mouth breathing:
i. A child suffering from nasorespiratory blockage due
SHORT ESSAYS:
to any reason like enlarged tonsils, recurrent throat
infections etc. tends to keep the tongue low and for- Q.1. Thumb-sucking habit.
ward and is unable to maintain anterior lip seal.
Ans.
ii. Such patients develop a mouth breathing habit with
consequent mouth open posture. These children de- l Thumb sucking is defined as placement of thumb or one
velop a long face known as adenoid facies. or more fingers in varying depths into the mouth.
76 Quick Review Series for BDS 4th Year, vol 1

l Theories involved in explaining thumb-sucking habit are Q.3. Classify tongue thrust habit. Add a note on its man-
i. Freudian theory agement.
ii. Oral drive theory
Ans.
iii. Benjamin’s theory
iv. Psychological aspects [Ref LE Q.1]
v. Learned pattern theory
Q.4. Bruxism.
l Phases of development of thumb sucking:

Phase I – normal – seen during the first 3 years of life Ans.


Phase II – clinically significant – 3–6½ years
Grinding of teeth for nonfunctional purposes is known as
Phase III – intractable sucking – beyond fourth or fifth
bruxism.
year of life
l Clinical features: Nocturnal grinding Bruxism
i. Proclination of maxillary anteriors
ii. h Overjet and g overbite
iii. Anterior open bite and posterior crossbite Two types
iv. Tongue thrust, hypotonic upper lip and hyperactive
mentalis muscle Day grinding Bruxomania
l Diagnosis:

It is based on: Psychological and emotional stresses


i. History of habit
ii. Child’s emotional status Occlusal interference or discrepancy
iii. Examination of child’s fingers – clean nails and Aetiology between centric relation (CR) and
callus on finger centric occlusion (CO)
iv. Intraoral clinical examination
l Treatment of thumb sucking:
Pericoronitis and periodontal
i. Psychological therapy pain triggers bruxism
ii. Mechanical aids: Clinical features:
a. Removable i. Occlusal wear facets on teeth
b. Fixed habit breakers ii. Mobility of teeth
c. Chemical approach iii. Fractured teeth and restorations
i. Psychological therapy: iv. Muscle pain – on waking up in the morning
l Counselling of parents to provide adequate love v. Temporomandibular joint (TMJ) pain and discom-
and affection to the child. fort
l Advice parents to divert the child’s attention to vi. Tenderness and hypertrophy of masticatory muscles
other things like play and toys. Diagnosis:
ii. Mechanical aids: i. History and clinical examination are helpful in diag-
l Habit breaking appliances: nosis of bruxism.
a. Removable, e.g. ii. Check for occlusal prematurities using articulating
i. Tongue spikes paper.
ii. Tongue guard iii. Check for hyperactivity of muscles of mastication,
b. Fixed, e.g. i.e. electromyography examination.
i. Quad helix Treatment:
ii. Hay rakes i. Psychological counselling
l Other mechanical aids used to intercept the habit ii. Relieving muscle tension by
are bandaging of thumb and elbow l Hypnosis
iii. Chemical approach: It is by using bitter tasting l Relaxing exercises
and foul smelling chemicals placed on thumb, e.g. l Massage
pepper dissolved in a volatile medium, quinine, iii. Occlusal adjustments
asafoetida. iv. Night guards or occlusal splints
Q.2. Mouth breathing habit. Q.5. Mouth breathing habit and its effects on the devel-
opment of occlusion.
Ans.
Ans.
[Ref LE Q.2]
[Same as SE Q.2]
Section | I  Topic-Wise Solved Questions of Previous Years 77

Q.6. Write briefly about tongue thrust habit. iv. Hypertrophied lower lips and associated hyperactive
mentalis activity are seen.
Ans.
v. Lip bumpers are used to treat upper and lower lip prob-
[Same as SE Q.3] lems. This appliance prevents abnormal force acting on
the incisors and hyperactivity of mentalis muscle.
SHORT NOTES: Q.4. Define tongue thrusting habit.
Q.1. Mouth breathing. Ans.
Ans. l Tongue thrusting is defined as a condition in which the
tongue makes contact with any teeth anterior to the mo-
i. Mouth breathing is an altered way of breathing through
lars during swallowing.
mouth and is an adaptation to obstruction in nasal
Aetiology:
passages.
The factors considered as a cause of tongue thrusting
ii. Mouth breathing results in altered jaw and tongue pos-
habit according to Fletcher are as follows:
ture, which in turn alters orofacial equilibrium leading
i. Genetic factors, e.g. hypertonic orbicularis oris
to malocclusion.
activity
iii. The type of malocclusion associated with mouth breath-
ii. Learned behaviour (habit) due to improper bot-
ing is called ‘Long Face Syndrome’ or classic adenoid
tle feeding, prolonged thumb sucking, prolonged
facies or vertical maxillary excess.
tonsillar or upper respiratory tract infections, etc.
iv. Anterior open bite and posterior crossbite, narrow
iii. Maturational factors, e.g. macroglossia, con-
V-shaped upper jaw with a high narrow palate, short
stricted dental arches and enlarged adenoids and
and flaccid upper lip.
tonsils, etc.
v. Treatment of mouth breathing includes removal of
iv. Mechanical restrictions
nasal or pharyngeal obstruction by ENT surgeon, inter-
v. Neurological disturbance, e.g. hyposensitive
ception of the habit using vestibular screen adhesive
palate and moderate motor disability.
tapes for lip seal and rapid maxillary expansion.
vi. Psychogenic factors
Q.2. Thumb-sucking habit.
Q.5. Adenoid facies.
Ans.
Ans.
i. Thumb sucking is defined as placement of thumb or
i. The type of malocclusion associated with mouth breath-
one or more fingers in varying depths into the mouth.
ing is called long face syndrome, adenoid facies.
ii. Clinical features: Proclination of maxillary anteriors,
ii. The term ‘adenoid facies’ was coined by Tomes in 1872
h overjet and g overbite, anterior open bite and posterior
to describe dentofacial changes associated with chronic
crossbite, tongue thrust, hypotonic upper lip and hyper-
nasal airway obstruction.
active mentalis muscle.
iii. These patients have increased lower anterior face height
iii. It is diagnosed based on history, child emotional status,
associated with unfavourable ‘clockwise’ rotation of
examination of child’s fingers, i.e. clean nails and callus
the mandible in a more vertical and posterior direction,
on finger and intraoral clinical examination.
open bite, crossbite and retrognathia.
iv. Treatment of thumb sucking includes psychological
iv. In growing patients, following adenoidectomy and
therapy, use of mechanical aids like habit breaking ap-
orthodontic treatment, changes would reverse back to
pliances, e.g. tongue spikes, tongue guard, hay rakes,
normal.
etc., and chemical approach using bitter tasting and foul
smelling chemicals placed on thumb. Q.6. Classify tongue thrust habit.
Q.3. Lip sucking and lip biting. Ans.
Ans. [Ref LE Q.1]
i. Lip sucking and lip biting are usually seen in cases of Q.7. Write a note on management of tongue thrust
excessive overjet. habit.
ii. They involve cushioning the lower lip against palatal sur-
Ans.
faces of maxillary incisors, causing them flare forward.
iii. Due to indirect pressure delivered to labial surface of Management of tongue thrusting habit:
mandibular incisors, they move lingually, increasing l Interception and treatment of tongue thrusting is age-

the overjet. and severity-dependent.


78 Quick Review Series for BDS 4th Year, vol 1

l The various modalities of treatment of tongue thrusting Management of thumb-sucking habit is as follows:
are as follows: l The child 1 parent 1 dentist form a team to assist child

I. Reminder therapy/interception of habit: Use of habit in stopping the habit.


breakers both fixed and removable cribs and rakes. l Treatment is mainly of three categories: Psychological

II. Corrective therapy: therapy, reminder therapy or mechanotherapy and


a. Removal of obstruction: Surgery for adenoids and chemical approach.
macroglossia. l Psychological therapy: Forced purposeful repetition of

b. Tongue exercises: a habit eventually associated with unpleasant reactions


i. Elastic band swallow exercise and habit is abandoned.
ii. Water swallow exercise l Reminder therapy or mechanical aids: They are habit

iii. Candy swallow exercise breaking appliances that assist to quit the habit and are
iv. Speech exercise of two types: removable and fixed.
c. Lip exercises: Patient practises stretching lips to Example: removable appliances: tongue spikes, tongue
achieve anterior lip seal. guard and spurs/rake
d. Habit breaking appliance, e.g. tongue crib. Fixed appliances: Quadhelix, hay rakes and maxillary
e. Treatment of malocclusion: Either removable or lingual arch with palatal crib.
fixed orthodontic appliances. l Chemical approach: Use of bitter tasting or foul smell-

ing preparations placed on the thumb that is sucked can


Q.8. Habit breaking appliances.
make the habit distasteful.
Ans.
Q.12. Pernicious oral habits.
[Ref LE Q.3]
Ans.
Q.9. Lip biting.
i. Pernicious oral habits are one of the factors influencing
Ans. the occlusion.
ii. Abnormal oral habits, like sucking habits and tongue
i. Lip biting habit is usually seen in cases with excessive over-jet thrusting, alter the equilibrium of buccinators mechanism.
iii. Various pernicious oral habits, like digit sucking,
Proclined upper anteriors
tongue thrusting and mouth breathing, can cause class
II, division 1 features.
ii. Clinical features: Hypertrophic and redundant iv. Median diastema can also be caused due to these habits.
lower lip v. Discontinuation of these habits is required for correc-
Cracking of lips tion of acquired malocclusions.
iii. Treatment : Lip bumpers Q.13. Lip bumper
Ans.

Q.10. Bruxism. i. The lip bumper is a ‘combined removable fixed appliance’.


ii. The appliance can be used in both the maxilla
Ans. (Denholtz appliance) and the mandible to shield the lips
i. Grinding of teeth for nonfunctional purposes is known away from the teeth.
as bruxism. iii. Typically, it is a vestibular arch carrying an acrylic pad
ii. It is mainly due to psychological and emotional stresses engaged to lower molar bands with 0.93-mm diameter
or occlusal interference. Pericoronitis and periodontal wire and it stands 2–3 mm away from the teeth and
pain triggers bruxism. gingiva and lies about 4 mm below the cervical margins
ii. Occlusal wear facets are seen on teeth, mobility of of the lower incisors. The lower lip is thus held forward.
l It prevents the hyperactivity of the mentalis muscle
teeth, fractured teeth and restorations.
iii. Muscle pain on waking up in the morning, TMJ pain or and the abnormal force from acting on the incisors.
l The other effect of lip bumper is that it causes pro-
discomfort and tenderness and hypertrophy of mastica-
tory muscles. clination of the incisors and distalization of molars.
l It is used in the correction or elimination of lip trap.
iv. Treatment consists of psychological counselling, re-
lieving muscle tension by occlusal adjustments and use Q.14. Nail biting
of night guards or occlusal splints.
Ans.
Q.11. Management of thumb-sucking habit.
i. Nail biting is one of the deleterious oral habits.
Ans. ii. It does not produce gross malocclusion.
Section | I  Topic-Wise Solved Questions of Previous Years 79

iii. It results in minor tooth irregularities similar to nut sucking thumb will hamper the pleasure derived from
notch because of cracking hard nuts. that act and the child will slowly try to avoid the habit
iv. It is treated with lip bumpers. by himself.
Q.15. Beta (b) hypothesis. Q.16. Role of mouth breathing in the aetiology of maloc-
clusion.
Ans.
i. ‘Beta hypothesis’ or concept of ‘negative practice’ was Ans.
discovered by Dr. Knight Dunlap (1929). [Same as SN Q.1]
ii. When the concept of b hypothesis or negative practice
is applied to oral habits, it helps as a self-correcting Q.17. Aetiology of tongue thrust.
mechanism. Ans.
iii. A child is encouraged to watch himself in front of a
large mirror while sucking the digit, the sight of oneself [Same as SN Q.4]

Topic 10
Orthodontic Diagnosis
COMMONLY ASKED QUESTIONS
LONG ESSAYS:
1. Classify diagnostic aids and enumerate all the essential diagnostic aids.
2. Discuss the importance of intraoral X-rays in orthodontic diagnosis and treatment planning.
3. Enumerate various diagnostic aids used in orthodontics. Add a note on study models.
4. What do you understand by diagnostic aids? Classify them. Describe the role of cephalometrics in orthodontics.
5. Classify diagnostic aids and write in detail about case history and clinical examination. [Same as LE Q.1]
6. What are diagnostic aids used in orthodontics? Describe any one of them in detail. [Same as LE Q.1]
7. Discuss in brief the various diagnostic aids used in orthodontic case analysis. [Same as LE Q.1]
8. Describe your procedure for clinical examination of face and intraoral tissue. [Same as LE Q.1]
9. What are the diagnostic aids used in orthodontics? Describe anyone of them. [Same as LE Q.2]
1 0. Describe the role of X-ray in orthodontic diagnosis and OPG in detail. [Same as LE Q.2]
1 1. Enumerate essential diagnostic aids. Describe study models in detail. [Same as LE Q.3]
1 2. Classify the diagnostic aids in orthodontics. Write the importance of study models and enumerate various types
of models used in orthodontics. [Same as LE Q.3]
1 3. What are the diagnostics aids? Describe in detail cephalometrics and its uses in orthodontics. [Same as LE Q.4]
1 4. Classify diagnostic aids in orthodontics. Describe the uses of roentgenographic cephalometry in orthodontics.
[Same as LE Q.4]

SHORT ESSAYS:
1. Electromyography.
2. Diagnostic aids in orthodontics. [Ref LE Q.1]
3. What are the study models and uses of the same in orthodontics. [Ref LE Q.3]
4. Discuss supplementary diagnostic aids used in orthodontics.
5. Uses of radiographs in orthodontics. [Ref LE Q.2]
6. Intraoral X-rays in orthodontics. [Ref LE Q.2]
7. Hand-wrist radiographs.
8. Trimming of study model.
9. Advanced diagnostic aids.
1 0. What are orthodontic diagnostic aids? Enumerate essential diagnostic aids. [Same as SE Q.2]
1 1. Essential diagnostic aids. [Same as SE Q.2]
80 Quick Review Series for BDS 4th Year, vol 1

1 2. Study models. [Same as SE Q.3]


13. Role of X-rays in orthodontics. [Same as SE Q.5]

SHORT NOTES:
1. Diagnostic aids. [Ref LE Q.1]
2. Study models.
3. Occlusal X-ray. [Ref LE Q.2]
4. CT or CAT.
5. Orthopantomogram (OPG). [Ref LE Q.2]
6. Classify X-rays in orthodontics. [Ref LE Q.2]
7. Head types/facial types.
8. Path of closure. [Ref LE Q.1]
9. Electromyograms. [Ref SE Q.1]
10. Intraoral periapical radiograph uses. [Ref LE Q.2]
11. Hand-wrist X-rays.
12. Cephalometric radiography. [Ref LE Q.2]
13. Facial forms. [Ref LE Q.1]
14. Bite-wing radiographs. [Ref LE Q.2]
15. Kesling set-up.
16. Carpal bones.
17. Gnathostatic models.
18. Incompetent lips.
19. Facial profile.
20. Evaluation of smile.
21. Facial divergence.
22. Lateral cephalogram.
23. Sheldon’s body type.
24. Uses of study models. [Same as SN Q.2]
25. Occlusal radiograph. [Same as SN Q.3]
26. OPG. [Same as SN Q.5]
27. Panoramic radiograph. [Same as SN Q.5]
28. Electromyogram as diagnostic aid. [Same as SN Q.9]
29. Hand-wrist radiographs. [Same as SN Q.11]

SOLVED ANSWERS
LONG ESSAYS:
Q.1. Classify diagnostic aids and enumerate all the Orthodontic Diagnostic Aids
essential diagnostic aids.
Ans. Essential diagnostic aids Nonessential diagnostic aids
(supplemental diagnostic aids)
(SN Q.1 and SE Q.2)
i.
Case history i. Supplemental radiographs
{(Diagnosis involves development of a comprehensive ii.
Clinical examination
iii.
Study models
a. Occlusal films (intraoral)
b. Lateral jaw views
database of pertinent information. The data are derived
iv.Certain radiographs: c. Coneshift technique
from both essential and nonessential diagnostic aids. • IOPA radiograph d. Cephalomteric radiographs
Orthodontic diagnostic aids are of two types, namely • Bite-wing ii. EMG (electromyographic
A. Essential diagnostic aids: They are considered very • Orthopantomogram examination of muscle activity)
important for all the cases. They are simple and do (OPG) iii. Hand-wrist radiographs
not require expensive equipment. v. Facial photographs iv. Endocrine tests
B. Nonessential or supplemental diagnostic aids: v. Estimation of BMR
They are not essential in all cases and require spe- vi. Diagnostic set-up
cialized equipment.) vii. Occlusograms }
Section | I  Topic-Wise Solved Questions of Previous Years 81

i. Case history: Includes the information collected from E. Prenatal and postnatal history:
the patient and parent or guardian to aid in the overall l Prenatal history includes the information on the

diagnosis of the case. Case history includes certain per- condition of the mother during pregnancy and
sonal details, the chief complaint, past and present dental type of delivery.
as well as medical history and associated family history. l The use of certain teratogenic drugs like thalido-

A. Personal details: mide and some infections like German measles


(a) Name during pregnancy result in congenital deformities
l The patient’s name should be recorded for of the child.
the purpose of not only communication and l Forceps delivery predisposes to TMJ injuries and

identification but also addressing a patient by associated mandibular growth retardation.


name which has a positive beneficial psycho- F. Postnatal history:
logical effect on the patient. l Type of feeding

(b) Age l Presence of habits, e.g. digit/thumb sucking

l The chronological age of the patient helps in l Milestones of normal development are included

diagnosis as well as treatment planning. in the postnatal history


l Certain treatment protocols are dictated by G. Family history:
the age of the patient, like growth modifica- l Records the details of malocclusion existing in

tion procedures using functional and ortho- other members of the family, which gives a hint
paedic appliances are best carried out during of inherited conditions, e.g. skeletal class II and
growth period, whereas surgical respective class III malocclusions and congenital conditions
procedures are better carried out after the like cleft lip and palate.
cessation of growth. ii. Clinical examination
(c) Sex Clinical examination of a patient includes the following:
l It is important in treatment planning as the
A. General examination
timing of growth events differs in males and B. Extraoral examination
females. C. Intraoral examination
(d) Address and occupation D. Functional examination
l These are important for communication and

evaluation of socioeconomic status of the A. General examination:


patient. Comprises general assessment of the patient, and
l It helps in selection of appropriate appliance. usually begins as soon as the patient enters the
B. Chief complaint: clinic, and includes:
l It should be recorded in patients’ own words. l Gait

l This helps the clinician in identifying the priori- l Posture

ties and desires of the patients, which helps in l Body built

setting treatment objectives that can satisfy the Gait: It is the way the person walks. Any ab-
patient as well as their family in general. normality in the gait suggests neuromuscular
C. Medical history: disorders, which have a dental correction.
l A very few medical conditions contraindicate the Posture: It is the way the person stands. Any
use of orthodontic appliances. abnormality in posture can predispose to mal-
l Most of the medical conditions require certain pre- occlusion due to alteration in maxillomandib-
cautionary measures to be taken prior to or during ular relationship.
orthodontic therapy, e.g. antibiotic coverage may be Body built: Sheldon has classified the general
required in patients with rheumatic fever or cardiac body built into three types:
anomalies even for molar band placement and re- l Ectomorphic – tall and thin physique

moval. l Mesomorphic – average physique

D. Dental history: l Endomorphic – short and obese


l Past dental history helps in assessing the pa- physique
tient’s and parent’s attitude towards dental health B. Extraoral examination:
and dental treatment. It includes the following:
l Dental history should include information on the (a) Type of the head: Mesocephalic, i.e. aver-
age of eruption of deciduous and permanent age shape head.
teeth, history of extraction, decay restoration and Dolicocephalic, i.e. long and narrow head.
trauma to the dentition. Brachycephalic, i.e. broad and round head.
82 Quick Review Series for BDS 4th Year, vol 1

(j) Clinical FMA:


{SN Q.13}
l The inclination of mandibular plane angle
(b) Facial form: Mesoproscopic (average facial to Frankfort horizontal plane should be
form) noted clinically by placing measuring
Euryproscopic (broad and short facial form) scales on patient’s face corresponding to
Leptoproscopic (long and narrow facial form) respective planes.
l In average FMA cases, the angle meets

behind the occiput.


l In a high angle case, the posterior ends of

(c) Facial profile: Straight, convex and concave the angle meet behind the auricle or within
(d) Facial divergence: It is defined as inclination the occiput.
of lower face relative to forehead. l Steep mandibular plane angle is seen in

Anterior divergent: class III cases patients with long face and open bites,
Posterior divergent: class II cases while flat mandibular plane angle is seen
Straight or orthognathic: class I cases in short faces and skeletal deep bite cases.
(e) Assessment of facial symmetry and propor- l In a low angle case, the two lines are paral-

tions: lel and meet very far away.


l For assessment of symmetry, the face is The examination of profile, divergence,
examined in the frontal and lateral views. vertical facial proportions, lip posture, inci-
l In the frontal plane, intercanthal distance sor protrusion and clinical FMA constitute
equals width of the nose, and interpupil- the facial profile analysis. It is also called
lary distance equals width of the mouth. ‘poor man’s cephalometric analysis’.
l Vertical height of the midface should (k) Chin:
equal the height of lower face. The height l Chin is examined for height, width and

of the forehead is measured from hairline contour.


to glabella, it measures one-third of the l Mentolabialsulcus: It is shallow in bimax-

total facial height. illary protrusion, while deep in class II,


l In normally balanced face, upper facial division 1 malocclusion.
height, middle face and lower facial height l Hyperactive mentalis activity is also seen

should be equal. along with lip habits like lip sucking and
(f) Examination of lips: thrusting.
l Competent lips: Lips which are in slight (l) Chin and soft tissues:
contact when the musculature is relaxed. Throat form
l Incompetent lips: Anatomically short lips Throat length
which do not contact each other when the Chin–throat angle
musculature is relaxed. C. Intraoral examination:
l Everted lips: They are hypertrophied lips a. Mouth opening
with redundant tissue exhibiting weak l Intraoral examination starts with measuring of

muscular tonicity. mouth opening.


(g) Examination of nose: Includes examination l Normal mouth opening is 45–55 mm for

of size, contour and nostrils. adults and less than 45 mm for children.
(h) Examination of chin: Mentolabialsulcus, men- b. Tongue
talis activity, chin position as well as promi- l The size, colour and configuration of tongue

nence should be examined. should be assessed.


(i) Nasolabial angle: c. Palate
l It is the angle formed by tangent to base of l Palate is assessed for contour.

the nose and a tangent to upper lip. l Palatal mucosa is examined for ulcerations,

l Normal angulation is 110°. In cases of indentations, clefts or pathologic swellings.


proclination of upper incisors, NLA is d. Gingiva
acute or decreased and NLA is obtuse or l The gingiva is examined for signs of inflam-

increased in retroclination of incisors. mation, hypertrophy or recession.


Section | I  Topic-Wise Solved Questions of Previous Years 83

e. Frenal attachments bite opening by molar extrusion can be at-


l Midline diastemas may arise due to thick tempted.
maxillary labial frenum. l Pseudo-deep bite with normal freeway space

l High attachment of mandibular labial frenum has normal eruption of posteriors. Bite open-
leads to gingival recession. ing by intrusion of incisors is recommended.
f. Adenoid and tonsils ii. Path of closure:
They are examined for enlargement and inflam-
mation.
g. Dentition
{SN Q.8}
l Number of deciduous and permanent teeth.

l Size, shape and form of teeth. The path of closure of mandible from the postural rest
l Presence of supernumerary or missing tooth, position to maximum intercuspation is evaluated in sag-
caries, attrition, erosion, fractures, etc. ittal, vertical and transverse planes.
h. Intra-arch examination Patient is examined for the presence of functional shifts
l Assessment of shape, symmetry and align- in anterior, posterior or lateral directions.
ment of arch. Examples: Upward and forward – normal.
l Location of midlines, rotations of teeth, Upward and backward – class II, division 2
crowding, spacing and contact areas are Upward and forward – pseudo-class III and anterior
checked. crossbites.
i. Interarch examination
l Midline shift between the maxilla and man-

dible recorded. iii. Examination of temporomandibular joint:


l Sagittal relations: Molar relationship, canine l Palpation: The TMJ is palpated for tenderness

relationship, overjet and anterior crossbite are and synchrony of action.


recorded. l Auscultation: The joint is checked for click-

l Vertical relation: Deep bite, open bite. ing or crepitus using a stethoscope.
l Transverse relation: Posterior crossbites. l Functional analysis of TMJ: The opening and

D. Functional examination: closing movements of the mandible as well as


l The dynamic nature of the stomatognathic sys- its protrusive, retrusive and lateral excursions
tem for optimal function is studied in functional are examined clinically.

examination. It helps in identifying the aetiology iv. Examination of orofacial dysfunctions:
of malocclusion and therefore helps in planning Examination of orofacial dysfunctions includes
the type of orthodontic treatment to be initiated. analysis of the functions like swallowing,
Detailed functional examination includes the fol- speech, respiration and actions of tongue and
lowing: lips.
i. Examination of postural rest position and a. Examination of swallowing pattern:
maximum intercuspation. l The normal swallowing pattern exhibits
ii. Examination of path of closure. contraction of mandibular elevators, the
iii. Examination of temporomandibular joint. tongue is enclosed in the oral cavity, teeth
iv. Examination of orofacial dysfunctions. occlude momentarily, dorsum of the
tongue approaches the palate.
i. Postural rest position: l Infants swallow in a different manner.
l The position of mandible where the syner- Retained infantile swallow leads to mal-
gistic and antagonistic muscular compo- occlusion.
nents are in dynamic equilibrium with their l Signs of infantile swallow: Jaws are
balance being maintained by basic muscle apart, while in swallowing, tongue is
tonus is known as postural rest position. placed between the teeth, mandible is
Clinical significance: stabilized by contraction of lips and
l In true deep bite cases, increased freeway tongue, muscles of facial expression are
space is seen where there is infra occlu- involved, anterior mandibular thrust,
sion of posteriors. In such conditions, caving in of cheeks.
84 Quick Review Series for BDS 4th Year, vol 1

b. Examination of tongue: The size, shape, pos- Example: Patients having tongue thrust habit tend
ture and function of the tongue are assessed. to lisp while cleft palate patients may have a nasal
Tongue size and shape: Abnormal tongue size tone.
like microglossia, i.e. small tongue and col- Q.2. Discuss the importance of intraoral X-ray in orth-
lapsed arch, macroglossia exhibits spaced odontic diagnosis and treatment planning.
dentition and crenations in lateral border of
Ans.
tongue.
Tongue position/posture: Diagnosis involves development of a comprehensive data-
l Position of tongue is a very important base of pertinent information. The data are derived from
factor in the development of malocclu- both essential and nonessential diagnostic aids.
sion. Orthodontic diagnostic aids are of two types, namely:
l Normal resting position of tongue is re- A. Essential diagnostic aids: They are considered very
tracted, tip just behind the lower incisors important for all the cases and are simple and do not
and lateral border resting on the linguo- require expensive equipment.
occlusal surfaces of lower posterior B. Nonessential or supplemental diagnostic aids: They are not
teeth. essential in all cases and require specialized equipment.
l In cases of class II malocclusion, tongue

tip is more retruded in rest position, [SE Q.5]


while it lies far forward in class III {Role of radiographs in orthodontic diagnosis:
cases. The uses of radiographs in orthodontics are as follows:
Tongue function (thrust): l To assess general development of the dentition,
The various types of tongue thrust are pres- presence, absence and state of eruption of the teeth.
ent like anterior, lateral, complex, endoge- l To determine the extent of root resorption of
nous, habitual and adaptive tongue thrust. deciduous teeth and the extent of root forma-
Anterior tongue thrust is associated with tion of permanent teeth.
anterior open bite. Lateral open bite is seen l To confirm the diagnosis of any supernumerary
in lateral tongue thrust. Complex tongue teeth, teeth that are morphologically abnormal,
thrust patient occludes teeth only in the extent of pathological and traumatic conditions.
molar region. l To study the character of alveolar bone.
c. Examination of lips: l They are a valuable aid in craniodentofacial
l Examination of lips consists of assessment analysis.}
of configuration, functioning and presence Various radiographs used in orthodontics are clas-
of any dysfunctions. sified as follows:
l The common lip dysfunctions include lip

sucking, lip thrust and lip insufficiency. {SN Q.6}


d. Examination of respiration:
l The mode of respiration is examined
I. Based on the location of radiographic film
a. Intraoral radiographs:
to establish any impediment in nasal
i. Intraoral periapical (IOPA) radiographs
breathing.
ii. Bite-wing radiographs
l Mouth breathing results due to prolonged
iii. Occlusal radiographs
difficulty in nasal breathing. This type of
b. Extraoral radiographs:
breathing results in disturbed orofacial
i. Panoramic radiographs
musculature, which leads to long face syn-
ii. TMJ radiographs
drome or adenoid facies.
iii. Cephalograms
e. Examination of speech:
l In structural defects involving the palate,
  II.  Based on the area of interest
tongue, lips and dentition, etc. the speech is i. Intraoral periapical (IOPA) radiographs
affected and the area of abnormality is ii. Bite-wing radiographs
identified by observing pronunciations of iii. Occlusal radiographs
different consonants. iv. Panoramic radiographs
Section | I  Topic-Wise Solved Questions of Previous Years 85

v. Radiographs to assess facial skeleton l To view the size, location and angulation of impacted
a. Lateral cephalometric radiographs teeth.
b. Frontal cephalometric radiographs l To study quality of alveolar bone and periodontal

c. Oblique cephalometric radiographs ligament space.


III.  Based on importance in diagnosis l To confirm the congenital absence of teeth or pres-

Essential diagnostic radiographs: ence of supernumerary tooth.}


i. Intraoral periapical (IOPA) radiographs
ii. Bite-wing radiographs
iii. Panoramic radiographs
Supplementary diagnostic radiographs:
Advantages of IOPA are as follows:
i. Occlusal radiographs
l The area of interest can be visualized in high
ii. Cephalometric radiographs
detail.
iii. Hand-wrist radiographs
l Minimal radiation exposure to patient.
iv. TMJ radiographs
l Easy to store and transport.

Disadvantages of IOPA are as follows:


a. Role of intraoral radiographs in orthodontics: l While placing the film, pain and gagging may

occur.
[SE Q.6] l Many radiographs are required for full mouth

l {Intraoral radiographs are called so as they are taken survey.


with the film placed inside the patient’s mouth. l Exposure to radiation increases when used for full

l They provide vital information about the teeth and their mouth examination.
supporting structures. There are two methods of obtaining IOPA radio-
l Commonly, there are three types of intraoral radio- graphs:
graphs used in orthodontics: a. Paralleling technique
i. Intraoral periapical radiographs b. Bisecting angle technique
ii. Bite-wing radiographs Paralleling technique
iii. Occlusal radiographs l In this technique, the X-ray film is placed parallel

to the long axis of the teeth and the central ray of


i. Intraoral periapical (IOPA) radiographs the collimated X-ray beam is passed perpendicular
Most commonly used radiographs in dentistry to to long axis of the tooth and the film.
visualize the teeth and the supporting alveolar bone. l In this technique there is reduced geometric

distortion.
l Morphological limitations imposed by oral

{SN Q.10} cavity in the correct placement of the film.


Bisecting angle technique
Uses of IOPA radiographs are as follows:
l This technique uses Cieszynski’s rule of isom-
l To examine the amount of demineralization in cari-
etry, which is a geometric theorem.
ous tooth.
l According to the rule of isometry, two trian-
l To assess the height of alveolar bone crest around the
gles are equal when they share a complete side
tooth.
and two equal angles.
l To study the extent of root resorption and root forma-
l In this technique, the central ray is directed at
tion in the deciduous and permanent teeth respec-
right angles to a plane bisecting the angle be-
tively.
tween the long axis of the teeth and the film.
l To visualize the root for any fractures or external
l This technique is convenient to the operator
resorption.
and the film is placed close to lingual surfaces
l To examine any calcific changes or internal resorp-
of the tooth.
tion in the dental pulp.
l The disadvantage of this technique is that
l To visualize the apical area of teeth to rule out
faulty X-ray beam angulation results in fore
pathology.
shortening or elongation of the image.
86 Quick Review Series for BDS 4th Year, vol 1

(SN Q.14 & SE Q.6) iii. To study clearly buccal or bucco-lingual


{(ii. Bite-wing radiographs expansions of cortical plate due to patho-
logical lesions of the jaw.
l Bite-wing radiographs give information about
iv. To diagnose the extent of fractures.
the tooth and the supporting alveolar bone up to
v. They are useful in orthodontics to analyse
half the length of the root.
treatment effects of arch expansion pro-
Uses of bite-wing radiographs are as follows:
cedures.
Bite-wing radiographs are used for detecting
vi. They are helpful in patients with limited
i. Interproximal caries at an early stage and
mouth opening.
secondary caries under restorations.
vii. Only minimal error is possible.
ii. Bone loss at the alveolar crest.
viii. Used to observe abnormal eruption pat-
iii. Calculus deposits and overhangs in the
tern of the canines and also to differentiate
restoration at the interproximal areas.
buccal or lingual positioning of tooth.)}
iv. Occlusal pattern.)
Advantages of bite-wing radiographs: b. Extraoral radiographs
l No geometric distortions and magnifications
Commonly used extraoral radiographs are as follows:
l Convenient to the operator and comfortable
i. Panoramic radiographs
for the patient ii. Cephalograms
l When compared to periapical films, it is more
iii. TMJ radiographs
easier to use in children.
i. Panoramic radiography
iii. Occlusal radiographs: l The Panoramic radiography was introduced

(SN Q.3 & SE Q.6) by Dickson and Copola.


l It is a radiographic procedure used to record a
l {(When there is a need to visualize a large seg- single image which covers major part of facial
ment of the dental arch with reasonable extent region. The maxillary and mandibular arches
of adjacent structures, occlusal radiograph is in- and their supporting structures are all visual-
dicated. ized in a single film.
The radiographic film 3 3 2 14 inches in size is Procedure:
placed between the occlusal surfaces of the teeth in l Position the patient’s head in the panorex
the plane of occlusion.) machine by making the patient stand or sit
biting the bite-fork with the incisors.
Classification of occlusal radiographs based on the l An X-ray source rotates around the pa-
projection of the X-ray is as follows: tient’s head. The rate at which the X-ray
i. Maxillary occlusal radiographs source moves is kept the same as the rate at
l Topographical maxillary occlusal projec-
which the film rotates.
tion l The image seen on the film is composed of
l Cross-sectional maxillary occlusal projec-
anatomic structures lying along predeter-
tion mined curved image zone called focal trough.
ii. Mandibular occlusal radiographs Uses:
l Topographical mandibular occlusal projec-

tion
l Cross-sectional mandibular occlusal pro-
{SN Q.5}
jection
l Mental spine view l Used for evaluation of dental development in the
mixed dentition by assessing the extent of root re-
(SN Q.3 & SE Q.6) sorption in the primary teeth and the amount of root
formation in the permanent teeth.
{(Uses of occlusal radiographs are as follows: l To locate supernumerary tooth or congenitally miss-
i. To locate impacted or unerupted or supernu-
ing tooth or impacted tooth.
merary teeth.
l It is used to assess the development and position of
ii. To locate foreign bodies in the jaws and
third molars.
calculi in salivary ducts.
Section | I  Topic-Wise Solved Questions of Previous Years 87

[SE Q.3]
l To detect presence of pathologic lesions in the jaw
bones and to ascertain its extent. Also useful in diag- l {Study models are accurate reproduction of teeth and
nosis of jaw bone fractures. their surrounding soft tissues that provide a reasonable
l It is used to locate carious lesions, bone loss due to ‘facsimile’ of the occlusion of the patient (Graber).
periodontal diseases, retained deciduous tooth, etc. l They are essential diagnostic aids, and have a prominent

Advantages: role in diagnosis and treatment planning. Before skull ra-


l The amount of radiation exposure to the patient is diography became popular, they were in widespread use.
low when compared with full mouth intraoral l Study casts reproduce the teeth and surrounding struc-

periapical radiographs. tures with a fair degree of accuracy. Even in the absence
l Highly valuable as an initial survey radiograph as of the patient, the clinician can study the occlusion from
broad anatomic area can be visualized. all directions using the study casts.
l It is useful in patients with extreme gag reflex and
Parts of study model are as follows:
those unable to open the mouth, i.e. trismus.
i. Anatomic portion
Disadvantages:
ii. Artistic portion
l The teeth and the supporting periodontal struc-

tures are not as clear as in periapical films. Hence, i. Anatomic portion: It is the actual replica of the patient’s
OPG is not suitable for diagnosis of lesions re- teeth and supporting structures obtained by impression.
quiring high resolution, e.g. early alveolar bone It is usually made of stone plaster.
loss. ii. Artistic portion: Artistic portion is nothing but plaster
l Image magnification, minimization or blurring of base around the anatomic portion. The artistic portion
incisor teeth may take place. supports the anatomic portion and orients it in a manner
l Radiopaque shadows are formed by structures that is pleasing to eye.}
like earrings, ramus of mandible and spinal cord. In a well-trimmed study casts, the ratio between ana-
l Overlapping of the adjacent structures occurs. tomic portion and artistic portion should be 2:1. The
l It requires very expensive equipment. tooth portion, soft tissue portion and the artistic portion
l Anterior teeth inclination cannot be visualized.) are related in 1:1:1 manner. The completed model should
be 13 mm in height in the anterior and posterior regions.
The steps involved in fabrication of study casts are as
follows:
{SN Q.12} l Impression making

l Disinfection of the impression


ii. Cephalometric radiographs:
l Casting the impression
l ‘Cephalo’ means head and ‘metric’ means mea-
l Basing and trimming of the cast
surement. The measurement of the head from the
l Finishing and polishing
shadows of the bony and soft tissue landmark on
the radiographic image is known as roentgeno- Impression making
l Obtaining a good impression is an essential fac-
graphic cephalometry.
l They are specialized radiographs of skull, in
tor in the proper fabrication of orthodontic casts.
l Care must be taken to obtain as nearly a perfect
which the head is positioned in a specially de-
signed head holder called cephalostat. Thus, it is reproduction of teeth and investing tissues as
a standardized technique wherein the head is held possible.
l Use of a mouth wash prior to impression re-
in a predetermined position.
l Cephalograms are also used for longitudinal stud-
moves debris and reduces surface tension on
ies, i.e. comparison of serial radiographs. tooth surface, cutting down bubble formation
l Cephalometric radiographs are of two types:
during impression making.
l Alginate impressions are ideally suited for this.
a. Lateral cephalogram
l A quick setting type with a time span not exceed-
b. Posteroanterior cephalogram
ing 90 s from mixing to completion of the mix.
l The orthodontic study models should repro-

duce as much of the supporting structures as


Q.3. Enumerate various diagnostic aids used in ortho-
possible; hence it is recommended to use high
dontics. Add a note on study models.
flange orthodontic trays that extend deep into
Ans. buccal and lingual sulci.
88 Quick Review Series for BDS 4th Year, vol 1

l The selected trays should include the last ii. The sides of the model are cut symmetri-
erupted molars and should have clearance of cally about the middle line.
around 3 mm between teeth and the tray. iii. The upper model is used as a guide in trim-
l A good maxillary and mandibular impression ming the lower model.
will show a peripheral roll and record the iv. By using the set square, the back corners of
muscle attachments. The retromolar pads in the upper and lower models are trimmed
the lower jaw and the tuberosity in the upper simultaneously. The front of the lower
jaw should be included. model is trimmed to a smooth curve.
Disinfecting the impression: iv. The distal corners are cut symmetrically to
l The impression is rinsed thoroughly in water, the middle line conveniently with the mod-
then disinfected to remove microorganisms, els in occlusion. The sides of the model are
plaque, mucin and other debris that might re- cut symmetrically about the middle line.
duce the quality of surface reproduction. v. The occlusal plane should be parallel to the
l Disinfectant solution, such as Biocide, may be top and bottom of the study casts.
used for this purpose. vi. After trimming, the study casts should be
l After disinfection, once again the impressions symmetrical. Upper study cast should have
are rinsed in water to clear any residual disin- seven sides and lower study cast should
fectant. have six sides when viewed from occlusal
Casting the impression: plane.
l The impression is rinsed and the excess water Finishing and polishing of the cast:
is shaken out. l Final finishing of artistic portion of dental

l A good grade of white stone model plaster is casts can be done with fine-grained waterproof
usually satisfactory to pour the impression. sand paper.
l It is best to use a mechanical spatulator or l Bubbles that appear at the gingival margin

vacuum mixer. If this is not available, it is ben- should be removed with a small universal
eficial to use some form of vibrator. sealer and those in the mucobuccal fold area
l The mechanical vibrator not only eliminates in- can be removed with a Kingsley type scraper.
corporation of air bubbles but also permits the l Final polishing is done by placing the casts in

use of heavier mix, which is much easier to han- soap solution for 1 h and later removed and
dle in pouring up and producing a strong cast. rinsed under warm water.
Basing and trimming of the cast: l The casts are then dried and buffed so that they

l The rubber base formers are readily available acquire smooth and shiny surface.
to pour the art portion or base. They serve to l Model storage boxes are used to store the fin-

confine the plaster and are fabricated to shape ished study models for future reference.
the base in artistically pleasing contours.
[SE Q.3]
l Orientation of the tray is done in such a way
that the anatomic portion is in the centre of the {Uses of study casts are as follows:
rubber mould with the occlusal plane parallel i. Study casts are considered as one of the es-
with the cast base of the base former. sential diagnostic aids in orthodontic diagno-
l Various types of base formers are used, e.g. sis and treatment planning.
Broussard cast former and the Columbia ante- ii. As they provide a three-dimensional precise
rior segment single unit study cast former. record of the teeth and their supporting struc-
Guidelines for trimming of the casts are as tures, they enable the study of the occlusion
follows: from all aspects.
Some orthodontists prefer individualizing casts iii. They help in assessing the nature and severity
because of additional information that can be of malocclusion.
gained and more accurate orientation with the iv. They are valuable aids in patient education
occlusal plane parallel with cast base. and motivation by enabling to explain the
These modifications require a cast trimmer or treatment plan as well as progress to the
a set of a plaster files. patient and parents.
The procedure of trimming is as follows: v. It makes it possible to simulate surgical treat-
i. The upper model is cut with the back edge ment procedures on the cast such as mock
at right angles to the middle line of the pal- surgery.
ate and the front surfaces are cut so that the vi. Study models help in communication between
point of intersection of the front surfaces is orthodontists to transfer records in case the
in line with the middle line of the palate. patient is to be treated by another clinician.
Section | I  Topic-Wise Solved Questions of Previous Years 89

vii. They help in assessing treatment progress by Cephalometrics


the dentist as well as the patient. l In 1931, Holly Broadbent of the USA and Herbert

viii. They are used in mixed dentition analyses Hofrath of Germany introduced the roentgeno-
along with charts and radiographs.} graphic cephalometric technique to orthodontics.
Advantages: l ‘Cephalo’ means head and ‘metric’ means mea-

l Compared with clinical examination or photo- surement. The measurement of the head from the
graphs, the study casts permit a more objective shadows of the bony and soft tissue landmarks on
assessment of malocclusion. the radiographic image is known as roentgeno-
l They make it possible to view the patient’s oc- graphic cephalometry.
clusion from all aspects, which is impossible in l Cephalometric radiographs have become an inte-

patient’s mouth. gral part of orthodontic practice and enable the


l They are a permanent record of the patient’s clinicians to quantify facial and dental relation-
malocclusion and can be duplicated easily ships.
when needed. l They provide information about the spatial rela-

l They are economical. tionship of superficial and deep structures.


Disadvantages: Various types of cephalograms are as follows:
l Considerable storage space is required to pre- i. Lateral cephalogram
serve them. ii. Frontal or anteroposterior cephalogram
l Mass storage retrieval takes time and there is iii. Oblique cephalogram
possibility of breakage. Technique of cephalometric radiography:
l No detailed information is provided about soft l An apparatus that consists of an X-ray

tissues in the oral cavity. source and a head-holding device called


l No relationship can be elicited between the cephalostatis is used to take a cephalometric
teeth and the facial profile. radiograph.
l The cephalostat prevents the movement of
Q.4. What do you understand by diagnostic aids? Classify
the head in the horizontal plane. Vertical
them. Describe the role of cephalometrics in orthodontics.
stabilization of the head is brought about by
Ans. an orbital pointer that contacts the lower
border of the left orbit. The upper part of the
Diagnosis involves development of a comprehensive data-
face is supported by the forehead clamp
base of pertinent information. The data are derived from
positioned above the region of the nasal
both essential and nonessential diagnostic aids.
bridge.
Orthodontic diagnostic aids are of two types, namely:
l The equipment helps in standardizing the
a. Essential diagnostic aids: They are considered very
radiographs by use of constant head posi-
important for all the cases. They are simple and do not
tion and a fixed source film distance (5 feet)
require expensive equipment.
so that serial radiographs can be compared.
b
. Nonessential or supplemental diagnostic aids:
Uses of cephalograms are as follows:
They are not essential in all cases and require special-
i. Cephalometrics is one of the valuable tools
ized equipment.
in treatment planning and follow-up of
patients undergoing orthodontic treatment.
Orthodontic Diagnostic Aids
ii. Cephalogram is used to elucidate the skel-
etal, dental and soft tissue relationships of
the craniofacial region and helps in identi-
Essential diagnostic aids Nonessential diagnostic aids
(supplemental diagnostic aids)
fication and classification of skeletal and
dental anomalies.
i.
Case history i. Supplemental radiographs
iii. They are useful in estimating the facial
ii.
Clinical examination a. Occlusal films (intraoral)
iii.
Study models b. Lateral jaw views
type.
iv.Certain radiographs: c. Coneshift technique iv. They help in treatment planning and evalu-
IOPA radiograph d. Cephalomteric radiographs ation of the treatment results for an indi-
Bite-wing ii. EMG (electromyographic vidual.
(OPG) examination of muscle activity) v. Serial cephalograms help in assessment of
v. Facial photographs iii. Hand-wrist radiographs growth of facial skeleton and are also used
iv. Endocrine tests in growth prediction.
v. Estimation of BMR vi. They help in predicting the growth-related
vi. Diagnostic set-up
changes and changes associated with or-
vii. Occlusograms
thognathic surgical treatment and also help
90 Quick Review Series for BDS 4th Year, vol 1

to distinguish changes produced by natural Q.6. What are diagnostic aids used in orthodontics?
growth and orthodontic treatment. Describe any one of them in detail.
vii. Cephalograms also help to carry out func-
Ans.
tional analysis.
viii. Cephalograms are relatively nondestruc- [Same as LE Q.1]
tive and noninvasive, producing a high
Q.7. Discuss in brief the various diagnostic aids used in
yield of information at relatively economi-
orthodontic case analysis.
cal cost.
ix. Cephalograms are relatively permanent re- Ans.
cords and are easy to store, transport and
reproduce. [Same as LE Q.1]
x. Cephalometrics is a valuable aid in re- Q.8. Describe your procedure for clinical examination
search work involving the cranio-dento- of face and intraoral tissue.
facial region.
Limitations/drawbacks of cephalogram: Ans.
i. It should be used only when it is diagnostically [Same as LE Q.1]
and therapeutically desirable, as patient is ex-
posed to ionizing radiation, which is harmful. Q.9. What are the diagnostic aids used in orthodontics?
ii. Due to lack of anatomical references which re- Describe anyone of them.
main constant with time, it is a serious disad- Ans.
vantage when clinicians wish to compare ceph-
alograms taken at different times. [Same as LE Q.2]
iii. Some reference landmarks and planes do not Q.10. Describe the role of X-ray in orthodontic diagno-
agree with the anatomical landmarks. sis and OPG in detail.
iv. The processes of image acquisition as well as mea-
surement procedures are not well standardized. Ans.
v. It is difficult to locate landmarks and surfaces [Same as LE Q.2]
on the X-ray image as the image lacks hard
edges and well-defined outlines. Q.11. Enumerate essential diagnostic aids. Describe
vi. The radiographic image is a two-dimensional study models in detail.
picture of three-dimensional structures being Ans.
imaged.
vii. Anatomical structures lying at different planes [Same as LE Q.3]
within the head undergo projective displacement. Q.12. Classify the diagnostic aids in orthodontics. Write
viii. There could be a mandibular shift from centric the importance of study models and enumerate various
relation as the patient is made to bite in maximum types of models used in orthodontics.
intercuspation while taking the cephalogram.
ix. A cephalometric analysis makes use of means Ans.
obtained from different population samples. [Same as LE Q.3]
They have only limited relevance when applied
to individual patient. Q.13. What are the diagnostics aids? Describe in detail
x. The composite of lines and angles used in the cephalometrics and its uses in orthodontics.
cephalometric analysis yields limited informa-
Ans.
tion about the patient’s dento-skeletal patterns.
xi. An orthodontic diagnosis cannot be made solely [Same as LE Q.4]
on the basis of cephalometric analysis.
Q.14. Classify diagnostic aids in orthodontics. De-
Q.5. Classify diagnostic aids and write in detail about scribe the uses of roentgenographic cephalometry in
case history and clinical examination. orthodontics.
Ans. Ans.
[Same as LE Q.1] [Same as LE Q.4]
Section | I  Topic-Wise Solved Questions of Previous Years 91

SHORT ESSAYS: l Following orthodontic therapy, EMG can be carried out


to see whether muscle balance is achieved.
Q.1. Electromyography. Disadvantages of EMG
Ans. l Unknown levels of muscular fatigue or pain may

compromise EMG readings.


l Muscle activity varies at different periods even dur-
{SN Q.9} ing the day time.
l The procedure that is used for recording the electri- l Measures of EMG activity during clenching, swal-

cal activity of the muscles when they are excited is lowing, etc. vary considerably between subjects.
known as electromyography (EMG). The instrument Q.2. Diagnostic aids in orthodontics.
used is called electromyograph and the output is
called electromyogram. Ans.
l The electromyograph is a machine which is used to [Ref LE Q.1]
receive, amplify and record the action potential dur-
Q.3. What are the study models and uses of the same in
ing muscle activity. The record obtained by such a
orthodontics.
procedure is known as electromyogram.
Ans.
l The surface membrane of the muscle cells is positively [Ref LE Q.3]
charged on the external surface and negatively charged Q.4. Discuss supplementary diagnostic aids used in
on the internal surface. The action potential reverses the orthodontics.
charge on the muscle membrane. A series of changes
that occur will bring about muscle contraction. Ans.
l Recording of the electrical charges from the muscle are Supplemental diagnostic aids are certain aids which are not
accomplished by means of surface electrodes or needle essential in all cases. They may require specialized equip-
electrodes. ment that an average dentist may not possess.
a. Surface electrodes: These electrodes are used when The supplemental diagnostic aids are as follows:
the muscle is superficially placed just below the skin. i. Specialized radiographs
b. Needle electrodes: They are used when the muscle is Examples:
placed deep inside, e.g. pterygoid muscles. a. Cephalometric radiographs
l The action potential is picked up by surface or needle b. Occlusal intraoral films
electrodes and recorded either with the help of a moving c. Selected lateral jaw views
pen in the form of a graph or recorded in the form of d. Cone shift technique
sound with the help of a magnetic tape recorder. ii. Electromyographic examination of muscle activity
l Permanent paper record is obtained with a pen writing iii. Hand-wrist radiographs to assess bone age or matura-
device. Electromyogram can be displayed on an oscil- tion age
loscope. iv. Endocrine tests
v. Estimation of basal metabolic rate
{SN Q.9} vi. Diagnostic set-up
vii. Occlusograms
Uses of EMG
l The role of musculature in craniofacial growth Q.5. Uses of radiographs in orthodontics.
can be studied with the help of EMG. Ans.
l In mouth breathers, EMG is used to study the

activity of mandibular elevators and depressors. [Ref LE Q.2]


l EMG is used to assess the aberrant muscular Q.6. Intraoral X-rays in orthodontics.
activity associated with various habits.
l Abnormal muscle activity associated with certain
Ans.
forms of malocclusion can be detected with the [Ref LE Q.2]
help of EMG.
Q.7. Hand-wrist radiographs.
Examples: Abnormal buccinator activity in class II,
division 1. Ans.
The hypo-functional upper lip in severe class II, divi-
i. Among the various skeletal maturity indicators, hand-
sion 1 malocclusion.
wrist radiographs are commonly used in orthodontics.
92 Quick Review Series for BDS 4th Year, vol 1

ii. The numerous small bones in the hand-wrist region l To assess the skeletal age in a patient whose
show a predictable and scheduled pattern of appear- growth is affected by infections or neoplasms
ance, ossification and union from birth to maturity. or traumatic conditions.
Thus, by comparing patients hand-wrist radiograph with l To evaluate growth status prior to orthognathic
standard radiographs that represent different skeletal surgery in young adults so that the chances of
ages, the skeletal maturation status of the individual can relapse linked to postsurgical growth can be
be determined. minimized.
iii. Among various methods described to assess skeletal l Serial assessment of skeletal age is used for
maturity using hand-wrist radiographs, the most com- studying growth of an individual.
monly used ones are as follows: l It is a valuable aid in research aimed at study-
a. Atlas method by Greulich and Pyle ing the effect of heredity, environment, nutri-
b. Bjork, Grave and Brown method tion, etc. on the skeletal maturation pattern.
c. Fishman’s skeletal maturity indicators
Q.8. Trimming of study model.
d. Hagg and Taranger method
Ans.
a. Atlas method by Greulich and Pyle
l Greulich and Pyle published an atlas which con- Basing and trimming of the cast
tains pictures of the hand-wrist for different l The rubber base formers are readily available to pour
chronological ages for both the sexes. the art portion or base. They serve to confine the
l The patients radiographs are matched with one of plaster and are fabricated to shape the base in artisti-
the photographs in the atlas which is representa- cally pleasing contours.
tive of a particular skeletal age. l The various types of base formers are used, e.g.

b. Bjork, Grave and Brown method Broussard case former, the Columbia anterior seg-
l According to Bjork, the skeletal development in ment single unit study cast former.
the hand-wrist area is divided into eight stages, l The trimming of the orthodontic model is carried out

each of them represents a particular level of skel- on an electric plaster trimming machine having a
etal maturity. medium-grit carborundum wheel.
c. Fishman’s skeletal maturity indicators Guidelines for trimming of orthodontic casts are as
l Leonard S. Fishman proposed a system for evalu- follows:
ation of skeletal maturation by making use of ana- Step 1: Orientation of the tray is done in such a way that
tomical sites located on the thumb, third finger, the anatomic portion is in the centre of the rubber mould
fifth finger and radius. with the occlusal plane parallel with the cast base of the
l Covering entire period of adolescent develop- base former.
ment, 11 descrete skeletal maturity indicators The lower model is inverted over a ‘T’-shaped piece
have been described. of rubber, and a marking is circumscribed all around
d. Hagg and Taranger method the base of the model using a marker mounted on a
l Hagg and Taranger noted that skeletal develop- vertical stand. Once the marking is made, the base of
ment in hand and wrist can be analysed from as- the cast is trimmed up to the marking.
sessment of ossification of ulnar sesamoid of Step 2: The back of the mandibular model is trimmed
metacrpo-phalangeal joint of the first finger (S) perpendicular to the midline leaving 5 mm of the plaster
and certain specified stages of three epiphyseal base distal to the most posterior teeth.
bones: the middle and distal phalanges of the third l The back of the model should be at 90° to the base

finger (MP3 and DP3) and the distal epiphysis of of the model.
the radius (R) by taking annual radiographs be- Step 3: Occlude both upper and lower models together
tween 6 and 18 years of age. and trim the maxillary back surface so that it is in flush
Indications of hand-wrist radiographs are as follows: with the mandibular back.
l It is indicated when there is a major discrep- Step 4: The upper and lower models are occluded to-
ancy between the dental age and the chrono- gether and placed on model trimmer with their backs.
logical age of the patient. l The base of the maxillary cast is trimmed so that

l For determination of skeletal maturity status to it is parallel to the base of the lower model.
assess the potential for growth prior to treating l At the end of this step, the backs of both upper

the patient with skeletal class II or class III and lower casts are at right angles to the bases.
malocclusion. l The bases of the maxillary and the mandibular casts

l To predict the pubertal growth spurt. are parallel to each other and to the occlusal plane.
Section | I  Topic-Wise Solved Questions of Previous Years 93

Step 5: The buccal cuts are made on the mandibular l On exposure to X-rays, selective discharge occurs
cast 5–6 mm away from the buccal surface of the poste- from the areas of photoreceptors proportional to
rior teeth and at an angulation of 60° to the back of the the density of the rays.
model. l The latent image is formed together from the

Step 6: The anterior portion of the lower arch is areas with discharge and without discharge and is
trimmed into a curve that follows the curvature of the converted to visible image by exposing it to
lower anterior teeth and is 5–6 mm away from the labial charged powder particles in toner.
surface of the anterior teeth. l The toner image is then transferred from the pho-

Step 7: The posterior cuts of the mandibular model toreceptor and fixed to a white plastic substrate
measuring 13–15 mm are trimmed at an angle of ap- for viewing.
proximately 115° to the back of the model. l The photoreceptor is available for reuse after

Step 8: The buccal cuts are made at an angle of 65° to cleaning.


the back of the maxillary cast at a distance of 5 mm Uses:
away from the buccal surface of the most posterior l Xerox radiography is a valuable alternative

teeth. to conventional radiography for detecting all


Step 9: On the maxillary cast, the anterior cuts are made pathologies.
at an angle of 30° to the back of the cast. The cuts on ei- l It is also useful in interpreting periapical struc-

ther side should be of equal length and should be 5–6 mm tures.


away from the labial surface of the anterior teeth. Advantages:
The anterior cuts on either side should meet at the l Wide exposure latitude, high edge enhancement
midline of the cast and should extend till the midline and good detail.
of the canine. l Choice of positive and negative displays.

Step 10: The posterior cuts of the maxillary cast are l Compared with conventional radiographs, it re-

made in such a way that they are in flush with the pos- quires only about one-third of the radiation dose.
terior cuts of the mandibular cast. ii. Magnetic resonance imaging (MRI) in orthodontics
This is done by occluding the models and trimming l Magnetic resonance imaging is a nonradiographic

the maxillary posterior cuts till they are in line with method used to visualize the craniofacial structures.
the mandibular posterior cuts. l The major constituent of the body is water which has

After trimming, the study casts should be symmetri- two hydrogen atoms, each one has a single proton.
cal. Upper study cast should have seven sides and l The hydrogen protons behave like small magnets

lower study cast should have six sides when viewed when they are placed in magnetic field, and they will
from occlusal plane. move around the magnetic field inducing a minute
current which is amplified and displayed on an oscil-
Q.9. Advanced diagnostic aids.
loscope.
Ans. Uses of MRI:
l Useful in studying internal derangements of the
The advanced diagnostic aids or the newer techniques used
articular disc of TMJ.
in orthodontic diagnosis are as follows:
l Used to evaluate the position of the articular disc
i. Xeroradiography
before and after treatment with functional and
ii. MRI
orthopaedic appliances.
iii. Computed tomography (CT) scan
l Examination of tongue movements during deglu-
iv. Computerized cephalometric systems
tition.
v. Photocephalometry
Advantages of MRI:
vi. Cinefluororadiography
l No radiation exposure.
vii. Laser holography
l Compared with CT scanning, greater tissue char-

i. Xeroradiography: acterization and a better spatial resolution.


l Xeroradiography was invented by Chester Carlson l Any choice of sectional view can be created in

for copying purposes. any desired direction in the human body.


l Xeroradiography is a radiographic method which Disadvantages:
works in a manner similar to xerox machines. No l Not useful to visualize bony lesions.

radiographic film is employed, rather a printout is l Contraindicated in patients with cardiac pacemakers.

taken on paper. l Compared with CT scanning, it is more time-

Method: consuming and expensive.


l A light-tight cassette with a photoreceptor is iii. CT in orthodontics
placed intraorally and X-ray exposed like film. l CT is the radiographic examination of section or

slice of body structures in the cross-sectional form.


94 Quick Review Series for BDS 4th Year, vol 1

l Digital data are produced by using scanners that l Parts of study model:
measure the extent of X-ray transmission through the i. Anatomic portion
object. ii. Artistic portion
Uses of CT: l Uses of study casts are as follows:
l Useful to examine TMJ. i. They are one of the essential diagnostic aids in
l Evaluation of amount of cortical bone for orth- orthodontic diagnosis and treatment planning.
odontic implants. ii. They enable the study of the occlusion from
l Diagnosis and treatment planning in maxillary all aspects and help in assessing the nature
canine impactions. and severity of malocclusion.
l To study the effects of rapid maxillary expansion iii. They are valuable aids in patient education,
and distraction osteogenesis devices. motivation and assessment of treatment
Advantages: progress.
l Large amount of useful information secured in iv. They help in communication between ortho-
very short period with high geometric precision. dontists.
l Very well discriminates between objects with mi-
Q.3. Occlusal X-ray.
nor difference in density.
l Images can be manipulated by highlighting or Ans.
accentuating areas of interest.
[Ref LE Q.2]
Disadvantages:
l Radiation exposure. Q.4. CT or CAT.
l Procedure is very expensive.
Ans.
Q.10. What are orthodontic diagnostic aids? Enumerate
l Computed axial tomography (CAT) or CT is the radio-
essential diagnostic aids.
graphic examination of section or slice of body struc-
Ans. tures in the cross-sectional form.
l Digital data are produced by using scanners that mea-
[Same as SE Q.1]
sure the extent of X-ray transmission through the object.
Q.11. Essential diagnostic aids. Uses of CT:
l Useful to examine TMJ.
Ans.
l Evaluation of amount of cortical bone for orthodon-
[Same as SE Q.1] tic implants.
l Diagnosis and treatment planning in maxillary canine
Q.12. Study models.
impactions.
Ans. l To study the effects of rapid maxillary expansion and

distraction osteogenesis devices.


[Same as SE Q.3]
Q.5. Orthopantomogram (OPG).
Q.13. Role of X-rays in orthodontics.
Ans.
Ans.
[Ref LE Q.2]
[Same as SE Q.5]
Q.6. Classify X-rays in orthodontics.
SHORT NOTES: Ans.
Q.1. Diagnostic aids. [Ref LE Q.2]
Ans. Q.7. Head types/facial types.
[Ref LE Q.1] Ans.
Q.2. Study models. Head type is determined based on the anthropometric determi-
nation of maximum skull width and maximum skull length.
Ans.
Head types are classified as follows:
l Study models are accurate reproduction of teeth and i. Mesocephalic: average shape head
their surrounding soft tissues that provide a reasonable ii. Brachycephalic: broad and round head
‘facsimile’ of the occlusion of the patient (Graber). iii. Dolichocephalic: long and narrow head
Section | I  Topic-Wise Solved Questions of Previous Years 95

Q.8. Path of closure. gingival margin. Vertical cuts are made to separate indi-
vidual teeth.
Ans.
l The individual teeth and their associated alveolar pro-

[Ref LE Q.1] cesses are sectioned off and replaced on the model base
in the desired positions using red wax.
Q.9. Electromyograms.
Uses of diagnostic set-up:
Ans. l It helps in simulating various tooth movements that

are planned for patients.


[Ref SE Q.1]
l The patients can be motivated by simulating various

Q.10. Intraoral periapical radiograph – uses. corrective procedures on the cast.


l Tooth size–arch length discrepancies can be visual-
Ans.
ized by means of a set-up.
[Ref LE Q.2] Q.16. Carpal bones.
Q.11. Hand-wrist X-rays. Ans.
Ans. l Carpal bones were first named by Lyser in 1683.
i. The hand-wrist radiographs are commonly used among l Each hand-wrist area has 8 carpals, 5 metacarpals and
the various skeletal maturity indicators in orthodontics. 14 phalanges.
ii. The numerous small bones in the hand-wrist region l The carpal bones are arranged in two rows.

show a predictable and scheduled pattern of appear- a. Distal row: trapezium, trapezoid, capitate, hamate
ance, ossification and union from birth to maturity. b. Proximal row: scaphoid, lunate, triquetral, pisiform
Thus, by comparing patients hand-wrist radiograph l These small irregular bones lie in-between the long

with standard radiographs that represent different skel- bones of forearm and the metacarpals.
etal ages, the skeletal maturation status of an individual Q.17. Gnathostatic models.
can be determined.
i ii. They are indicated when there is a major discrepancy Ans.
between the dental age and the chronological age of the Gnathostatic models are orthodontic study models where
patient for determination of skeletal maturity status. the base of the maxillary cast is trimmed to correspond to
iv. Serial assessment of skeletal age is used in studying the Frankfort horizontal plane.
growth of an individual.
Q.18. Incompetent lips.
v. It is a valuable aid in research aimed at studying the
effect of heredity, environment, nutrition, etc. on the Ans.
skeletal maturation pattern.
l Competency of lips can be defined as the ability to ap-
Q.12. Cephalometric radiolography. proximate the lips without any strain.
l Lips which are in slight contact when the musculature is
Ans. relaxed and 2 mm of incisal edges of upper incisors
[Ref LE Q.2] showing at rest is considered normal competent lips.
l Anatomically, lips which do not contact each other
Q.13. Facial forms. when the musculature is relaxed are known as incompe-
Ans. tent lips.
l They are morphologically short lips which do not form a
[Ref LE Q.1] lip seal in a relaxed state. The lip seal can only be achieved
Q.14. Bite-wing radiographs. by active contraction of the perioral and mentalis muscles.
l Potentially incompetent lips are the normal lips that fail
Ans. to form a lip seal due to proclined upper incisors.
[Ref LE Q.2] Q.19. Facial profile.
Q.15. Kesling set-up. Ans.
Ans. l The facial profile is examined by viewing the patient
l The diagnostic set-up was first proposed by H.D. Kesling. from the side.
The Kesling diagnostic set-up is made from an extra set of l The facial profile is assessed by joining the two refer-

trimmed and polished study models. ence lines:


l The cast is cut using a fretsaw blade to separate indi- i. A line joining the forehead and the soft tissue point A
vidual teeth. A horizontal cut is made 3 mm apical to the ii. A line joining point A and the soft tissue pogonion
96 Quick Review Series for BDS 4th Year, vol 1

l Three types of profiles exist based on the relationship Facial divergence can be of following three types:
between these two lines: i. Anterior divergent: A line drawn between the forehead
a. Straight profile: The lines form a straight line. and chin is inclined anteriorly towards the chin, e.g.
b. Convex profile: The lines form an angle, which is class III cases.
pointed away from the face, e.g. class II skeletal pattern. ii. Posterior divergent: A line drawn between the forehead
c. Concave profile: The lines form an angle which is and chin slants posteriorly towards chin, e.g. class II cases.
pointed towards the face, e.g. class III skeletal pattern. iii. Straight or orthognathic: The line between the fore-
l The facial profile helps in diagnosing gross deviations head and chin is straight or perpendicular to the floor,
in the maxillomandibular relationship. e.g. class I cases.
Q.20. Evaluation of smile. Q.22. Lateral cephalogram.
Ans. Ans.
l One of the important aspects of orthodontic treatment is l The measurement of the head from the shadows of the
creation of pleasing smile. bony and soft tissue landmarks on the radiographic im-
Evaluation of smile: age is known as roentgenographic cephalometry.
Smile is evaluated by its vertical, transverse and oblique l Cephalometric radiographs are of two types:
characteristics. a. Lateral cephalogram
Vertical characteristics: b. Posteroanterior cephalogram
l Incisor and gingival displays are two main features l Lateral cephalogram: This provides a lateral view of
of vertical characteristics of smile. the skull. It is taken with the head in a standardized
l Inadequate incisor display can be due to vertical reproducible position at a specified distance from the
maxillary deficiency, restricted lip mobility and short source of the X-ray.
clinical crown.
Q.23. Sheldon’s body type.
l Gummy smile will be associated with vertical maxil-

lary excess. Ans.


Transverse characteristics:
Sheldon’s classification of body type or build-up is as follows:
There are three important features: buccal corridor
i. Ectomorphic: Tall and thin body tissue is made of pri-
width, arch form and transverse cant.
marily skin and neural elements.
Buccal corridor width:
ii. Mesomorphic: Average body tissue is made of primar-
l Buccal corridor is represented by a ratio of intercom-
ily mesodermal tissue: muscular and robust individuals.
missure width divided by the distance from one max-
iii. Endomorphic: Short and obese body tissue is made of
illary first premolar to opposite side first premolar.
primarily fat tissues.
l Excessively wide buccal corridor is referred to as

‘negative space’. Q.24. Uses of study models.


Arch form: Ans.
l Arch form plays an important role in the form of
[Same as SN Q.2]
smile.
l In patients with collapsed arch or narrow maxilla, Q.25. Occlusal radiograph.
smile also is narrow. Ans.
Transverse cant:
[Same as SN Q.3]
l Asymmetric vertical growth of the arches or dif-

ferential eruption of teeth can cause appearance of Q.26. OPG.


transverse cant or tilt of the smile line. Ans.
l Ideally, there should not be any transverse cant.
[Same as SN Q.5]
Oblique characteristics:
l Maxillary occlusal plane from premolar to premolar Q.27. Panoramic radiograph.
should be in consonant with the curvature of the Ans.
lower lip on smile.
[Same as SN Q.5]
l Downward tilt of the posterior maxilla or upward tilt

of anterior maxilla can result in deviation. Q.28. Electromyogram as diagnostic aid.


Q.21. Facial divergence. Ans.
[Same as SN Q.9]
Ans.
Q.29. Hand-wrist radiographs.
Facial divergence is defined as an inclination of lower face
relative to forehead. It is influenced to a large extent by the Ans.
patient’s ethnic and racial background. [Same as SN Q.11]
Section | I  Topic-Wise Solved Questions of Previous Years 97

Topic 11
Cephalometrics
COMMONLY ASKED QUESTIONS
LONG ESSAYS:
1 . Discuss cephalometrics as a diagnostic aid.
2. Write in detail about any one cephalometric analysis of your choice used in orthodontics.
3. Describe the role of cephalometric radiography in orthodontics. Give various planes and angles used in cepha-
lometric analysis.
4. Classify diagnostic aids. Discuss the uses of cephalometrics. [Same as LE Q.1]
5. Classify the diagnostic aids. Define caphalometric landmarks and planes. [Same as LE Q.1]
6. What is standardization in cephalometrics? Discuss Steiner’s cephalometric analysis. [Same as LE Q.2]
7. Enumerate cephalometric analysis and elaborate on Steiner’s analysis. [Same as LE Q.2]
8. Discuss in detail the clinical implications of growth and development enumerating the use of cephalometrics to
study the same. [Same as LE Q.3]

SHORT ESSAYS:
1 . Uses of cephalometrics. [Ref LE Q.1]
2. Tweed’s triangle.
3. Down’s analysis.
4. Enumerate five horizontal planes used in cephalometrics and give their uses and significance. [Ref LE Q.1]
5. What is conventional cephalometrics? Give its drawbacks.
6. Visual treatment objective (VTO).
7. Steiner’s skeletal analysis. [Ref LE Q.2]
8. Write the skeletal measurements of Down’s analysis. [Same as SE Q.3]

SHORT NOTES:
1. y-axis.
2. Key ridge.
3. Limitations of cephalogram.
4. Enumerate horizontal planes in cephalometry.
5. Interincisal angle. [Ref LE Q.2]
6. S-N plane.
7. ANB angle. [Ref LE Q.2]
8. FMA angle.
9. SNA angle. [Ref LE Q.2]
10. Two uses of cephalometrics in orthodontia.
11. Tweed’s diagnostic triangle.
12. Occlusal plane angle.
13. Steiner’s soft tissue analyses.
14. Registration point.
15. Facial divergence.
16. Angle SNB. [Ref LE Q.2]
17. Mandibular plane angle.
18. Cephalostat.
19. The Wits appraisal.
20. Frankfort horizontal plane.
21. Define cephalometric points.
22. ‘E’ plane or aesthetic plane.
98 Quick Review Series for BDS 4th Year, vol 1

2 3. Computerized cephalometric system.


24. Nasolabial angle.
25. Advantages of computerized cephalometric system.
26. Significance of ANB angle. [Same as SN Q.7]
27. Uses of cephalometrics. [Same as SN Q.10]
28. Tweed’s triangle. [Same as SN Q.11]
29. Occlusal plane. [Same as SN Q.12]
30. Mandibular plane. [Same as SN Q.17]
31. Name some data management programmes in computerized cephalometric system. [Same as SN Q.23]

SOLVED ANSWERS
LONG ESSAYS:
Q.1. Discuss cephalometrics as a diagnostic aid. Cephalograms are of two types
Ans.
l Diagnostic aids are means by which a comprehensive
database of pertinent information is derived. (i) Lateral cephalogram (ii) Frontal cephalogram
(Lateral view of skull) (AP view of skull)
The orthodontic diagnostic aids are of two types
[SE Q.1]
{Uses of cephalometrics:
I. Essential diagnostic aids II. Nonessential diagnostic aids Cephalometrics is a valuable tool in treatment planning
Or Supplemental diagnostic aids. and follow-up of orthodontic patients.
For example For example Cephalometrics helps in:
i. Case history i. Supplemental radiographs i. Orthodontic diagnosis (by studying skeletal 1
ii. Clinical examination a. Occlusal films
dental and supporting structures)
iii. Study models b. Lateral jaw views
iv. Certain radiographs: c. Cephalomteric radiographs
ii. Classification of facial type (skeletal 1 dental)
a. IOPA ii. EMG iii. Treatment planning
b. Bite-wing iii. Hand-wrist radiographs iv. Evaluation of treatment results
c. OPG iv. Endocrine tests v. Prediction of growth-related changes and changes
v. Facial photographs v. Estimation of BMR associated with surgical treatment
vi. Diagnostic set-up vi. Research works involving craniodentofacial
vii. Occlusograms region.}
Technical aspects of cephalometrics Fig 11.1:
l In 1931, Holly Broad Bent (USA) and Herbert Ho Frath The standard apparatus used to take cephalograms con-
(Germany) simultaneously presented a standardized sists of the following:
cephalometric technique. i. An X-ray source
l Cephalometrics is used to describe the analysis and ii. Acephalostat (head-holding device)
measurements made on the cephalometric radiographs. iii. A cassette holder

Cephalostat
ds
r ro
Ea

X-ray film cassette

Film plane
X-ray source
Midsagittal
plane

5 Feet
Fig. 11.1  A standard cephalometric arrangement.
Section | I  Topic-Wise Solved Questions of Previous Years 99

Cephalostat stabilizes the head of the patient


l

with the help of ear rods, orbital pointer and


forehead clamp.
l The distance between the X-ray source and

the midsagittal plane of the patient is fixed at N


5 feet.
Cephalometric landmarks: P
l Certain landmarks or points on the skull are used by
O
cephalometrics for quantitative analysis and mea- Ar
surements. Bo
PTM PNS
Ba ANS
l The landmarks used in cephalometrics should be eas-
A
ily visible on radiographs, uniform in outline and
easily reproducible, permitting valid quantitative Go
measurements of lines and angles projected from
them. B
l Cephalomteric landmarks are of the following types:
Pog
Gn
Anatomic landmarks – Represent actual Me

I. anatomic structures of skull


Fig. 11.2  Lateral cephalometric landmarks.
Derived landmarks – They are obtained
secondarily from anatomic structures iv. Posterior Nasal Spine (PNS): It is the tip of the pos-
Hard tissue landmarks terior spine of the palatine bone in the hard palate.
II. v. Point A (sub-spinale): It is the deepest point in the
Soft tissue landmarks midline between the anterior nasal spine and (alveo-
lar crest between two central incisors) prosthion.
vi. Point B (supramentale): It is the deepest point in
Some of the important points and landmarks described
the midline between the alveolar crest of the man-
in lateral cephalometric projection are as follows.
dible and the mental process.
Unilateral landmarks Bilateral landmarks include
vii. Basion: The lower most point on the anterior mar-
gin of the foramen magnum in the midsagittal
i. Sella i. Orbital plane.
ii. Nasion ii. Porion
iii. Anterior nasal spine (ANS) iii. Bolton point
viii. Gnathion: The most anteroinferior point in the
iv. Posterior nasal spine (PNS) iv. Gonion contour of the chin.
v. Point A v. Pogonion ix. Menton: The lower most point on the mandibular
vi. Point B vi. Articulare symphysis.
vii. Basion vii. Condylion x. Prosthion (supra-dentale): The lowest and the
viii. Gnathion viii. The key ridge
ix. Menton ix. Broadbent registration
most anterior point on the alveolar bone in the
x. Prosthion point midline between the upper central incisors.
xi. Infradentale x. Glabella xi. Infradentale: The highest and the most anterior
xii. Ptm point xi. Chelion point on the alveolar process, in the median plane
xiii. Sub-nasal between mandibular central incisors.
xiv. Glabella
xii. Ptm point: It is the intersection of the inferior bor-
der of the foramen rotundum with the posterior
The definitions of each of the above points or landmarks wall of the pterygomaxillary fissure.
used in cephalometric are as follows: xiii. Subnasale: The point where the lowest border of
Unilateral landmarks the nose meets the outer contour of the upper lip.
i. Sella: The point representing the midpoint of the xiv. Glabella: It is the most prominent point of the
pituitary fossa or sella turcica. forehead in the midsagittal plane.
ii. Nasion: The intersection of internasal suture with Bilateral cephalometric landmarks
the nasofrontal suture in the midsagittal plane. i. Orbitale: The lowest point on the inferior bony mar-
iii. Anterior Nasal Spine (ANS): It is the tip of the an- gin of the orbit.
terior nasal spine seen on the X-ray film from nor- ii. Porion: The highest bony point on the upper margin
mal ateralis. of external auditory meatus.
100 Quick Review Series for BDS 4th Year, vol 1

iii. Bolton point: The highest point in the upward cur- (ii) Frankfort horizontal plane
vature of the retrocondylar fossa (Broadbent). This plane connects the orbitale and porion.
iv. Gonion: Is a constructed point at the junction of
ramal and mandibular planes. It is the most inferi-
orly, posteriorly and outwardly directed.
v. Pogonion: It is the most anterior point in the con-
tour of the chin.
vi. Articulare: It is a point at the junction of the poste-
rior border of ramus and the inferior border of the P
basilar part of the occipital bar. O
vii. Condylion: The most superior point on the head of
the condyle.
viii. Thekey ridge: The lower most point on the contour
of the anterior wall of the infratemporal fossa.
ix. Broadbent registration point: It is the midpoint of
the perpendicular from the centre of sellatursica to
the Bolton plane. Fig. 11.4  Frankfort horizontal plane.
x. Glabella: Is the most prominent point of the fore-
head in the midsagittal plane. (iii) Occlusal plane
xi. Chelion: Is the lateral terminus of the oral slit on It is a denture plane bisecting the posterior occlu-
the outer corner of the mouth. sion of permanent molars and premolars and ex-
Lines and planes in cephalometrics tends anteriorly.
These lines and planes are obtained by connecting two
landmarks. Based on their orientation, they are classi-
fied as horizontal and vertical planes.
[SE Q.4]
{Horizontal planes:
i. S-N plane
ii. Frankfort horizontal plane
iii. Occlusal plane
iv. Mandibular plane
v. Basion–nasion plane
vi. Bolton’s plane
(i)  S-N plane
It represents anterior cranial base. It is the cranial
line between the sella and nasion. Fig. 11.5  Occlusal Plane.

(iv) Mandibular plane:


According to Down, mandibular plane is a line
connecting gonion and menton.

N
S

Go

M
Fig. 11.3  S-N Plane (Sella-Nasion plane).
Section | I  Topic-Wise Solved Questions of Previous Years 101

(v) Basion–nasion plane: (ii) Facial plane


It is a line connecting the Basion and Nasion and It is a line from the nasion to pogonion.
represents cranial base.

N
N

Ba

Pog

Fig. 11.7  Facial Plane.

(vi) Bolton’s plane: It is a plane that connects Bolton’s (iii) Facial axis
point posterior to the occipital condyles and na- It is a line from ptm point to cephalometricgna-
sion.} thion.
Vertical planes:
Various commonly used vertical planes in cephalomet-
rics are as follows:
i. A-pog line
ii. Facial plane
iii. Facial axis
iv. Aesthetic plane
(i) A-pog line Ptm
It is a line extending from point A on the maxilla
to pogonion on the mandible.

Gn

Fig. 11.8  Facial axis.

(iv) ‘E’ plane or aesthetic plane


Pog It is a line between the most anterior point of
the soft tissue nose and soft tissue chin.
With all the above-mentioned standard points
and measurable planes and angles, cephalo-
Fig. 11.6  A-Pogonion plane. metric aids in skeletal, dental and soft tissue
102 Quick Review Series for BDS 4th Year, vol 1

The mean values of parameters considered in Steiner’s


skeletal analysis are as follows:

SNA SNB ANB Mandibular Occlusal


angle angle angle plane angle plane angle
82° 80° 2° 32° 14.5°}

(SN Q.9 and SE Q.7)

(a)  {(SNA angle:


l The angle formed between S-N plane and

line joining nasion to point A is SNA angle.


l It indicates anteroposterior positioning of

maxilla in relation to cranial base.


l The mean value is 82°.

SNA . 82° n maxillary prognathism


Fig. 11.9  Aesthetic plane. SNA , 80° n retrognathism of maxilla)

(SE Q.7 and SN Q.16)


{((b)  SNB angle:
analysis and classification of various maloc-
The angle formed between S-N plane and
clusions.
line joining nasion and point B.
Q.2. Write in detail about any one cephalometric analysis l SNB angle indicates anteroposterior
of your choice used in orthodontics. positioning of mandible in relation to
Ans. cranial base.
l The mean value of SNB angle is 80°.
Various cephalometric analyses are as follows: SNB . 80° n mandibular retrogna-
A. Methodological classification thism (class III).
1. Angular analyses – SNA, SNB, ANB, Tweeds SNB , 80° n mandibular retrogna-
analysis thism (class II).)}
2. Linear analyses – MacNamara analysis, COGS
B. According to area of analysis
1. Skeletal analysis – SNA, SNB, ANB {SN Q.7}
2. Dentoalveolar analysis – upper central incisor to NA
3. Soft tissue analysis – ‘E’ plane (c) ANB angle:
l Angle formed by intersection of line joining na-
[SE Q.7] sion to point A and nasion to point B is known as
ANB angle.
{Steiner’s analysis l ANB angle indicates relative position of maxilla
Cecil C. Steiner in 1930 developed this analysis. The
and mandible to each other.
idea is to provide maximum clinical information with
l The mean value of ANB angle is 2°.
the least number of measurements.
ANB . 2° n class II tendency.
The Steiner’s analysis is divided into three parts.
ANB , 2° n class III relationship.)
I. Skeletal analysis
II. Dental analysis
III. Soft tissue analysis [SE Q.7]
I. Skeletal analysis {(d) Mandibular plane angle:
The Steiner’s skeletal analysis considers the fol- l Mandibular plane angle is the angle between
lowing parameters: S-N plane and mandibular plane (Go-Gn).
a. SNA angle l It indicates the growth pattern of an indi-
b. SNB angle vidual:
c. ANB angle Average value is 32°.
d. Mandibular plane angle ,32° suggests horizontally growing face.
e. Occlusal plane angle .32° suggests vertically growing face.
Section | I  Topic-Wise Solved Questions of Previous Years 103

S N
N

Lon
ga
xis
of
NA line

A
Go

Gn

Fig. 11.10  Mandibular plane angle.


Fig. 11.12  Upper incisor to NA (angle).

(e)  Occlusal plane angle:


l It is the angle formed between occlusal plane
(a) Upper incisor to NA angle:
l Angle formed by intersection of the long axis of
and S-N plane.
l It indicates the relation of occlusal plane to
upper central incisors and the line joining nasion
cranium and face the growth pattern of an to point A.
l The mean value is 22°. It indicates relative incli-
individual.
It has a mean value of 14.5°. nation of the upper incisors.
h occlusal plane angle – clockwise rotation of h upper incisor to NA angle – upper incisor pro-
occlusal plane. clination, e.g. class II, division I.
g occlusal plane angle – counterclockwise g upper incisor to NA angle – upper incisor ret-
rotation of occlusal plane.} roclination.
(b) Upper incisor to NA linear: It is the lineal measure-
ment between labial surface of central incisor and
the line joining nasion to point A.
l It is the linear measurement between labial sur-

S N face of central incisor and line joining nasion to


point A.
l This helps to determine upper incisor position:

Mean value is 4 mm.


l h value seen in upper incisor proclination.

Fig. 11.11  Occlusal plane angle.

Steiner’s dental analysis


The mean values of parameters considered in Steiner’s
dental analysis are as follows:

Parameter Upper Upper Lower Lower Interincisal


incisor incisor incisor incisor angle
to NA to NA to NB to NB
(angle) (linear) (angle) (linear)
Mean 22° 4 mm 25° 4 mm 131°
value Fig. 11.13  Upper incisor to NA linear.
104 Quick Review Series for BDS 4th Year, vol 1

(c) Lower incisor to NB (angle):


{SN Q.5}
l It is the angle between long axis of lower incisor

to the NB line. The mean value is 25°. (e) Interincisal angle:


l h Value seen in lower incisor proclination. Angle formed between long axis of upper and lower
central incisors is known as interincisal angle. Mean
value is 130–131°.
l g Interincisal angle – class II, division 1 or class I

bimaxillary protrusion.
l h Interincisal angle – class II, division 2 cases.

(d) Lower incisor to NB (linear): The linear distance


between labial surface of lower central incisor and
the line joining nasion to point B.
l It helps to access lower incisor inclination. Aver-

age value is 4 mm.


l h Value seen in proclined lower incisors.
Fig. 11.15  Interincisor angle.

Steiner’s soft tissue analysis


According to Steiner, in a well-balanced face, lips
should touch the line extending from soft tissue contour

S-Line

Fig. 11.14  Lower incisor to NB linear.


Fig. 11.16  S-line.
Section | I  Topic-Wise Solved Questions of Previous Years 105

of chin to midline of an ‘S’ formed by the lower border (f) Growth changes associated with treatment: Rickett’s
of nose. four-step analysis is used to study growth versus treat-
Lips located beyond this line – protrusive (convex profile) ment changes.
Lips located behind this line – retrusive (concave profit) (g) Growth following conclusion of treatment: Growth
changes which take place after active treatment can be
Q.3. Describe the role of cephalometric radiography in
predicted using cephalograms.
orthodontics. Give various planes and angles used in
Various planes used in cephalometrics are as follows:
cephalometric analysis.
These planes are obtained by connecting two land-
Ans. marks. Based on their orientation, they are classified
as horizontal and vertical planes.
The various ways of assessment of growth using cephalo-
Horizontal planes:
metric radiography are as follows:
i. S-N plane
a. Assessment of growth expected
ii. Frankfort horizontal plane
b. Determination of direction of growth
iii. Occlusal plane
c. Growth time table
iv. Mandibular plane
d. Assessment of growth rates
v. Basion–nasion plane
e. Growth prediction
vi. Bolton’s plane
f. Growth changes associated with treatment
Vertical planes:
g. Growth following treatment conclusion
i. A-pog line
(a) Assessment of growth expected: In orthodontic treat- ii. Facial plane
ment, planning quantitative assessment of growth is an iii. Facial axis
important part. iv. Aesthetic plane
Biological age is determined chiefly by hand-wrist
Q.4. Classify diagnostic aids. Discuss the uses of cepha-
radiographs.
lometrics.
(b) Determination of direction of growth: Cephalometric
radiography differentiates between horizontal and ver- Ans.
tical growth patterns. [Same as LE Q.1]
Q.5. Classify the diagnostic aids. Define caphalometric
• Broad mandibular base
landmarks and planes.
Horizontal growth
Ramus with a thick sympysis Ans.
[Same as LE Q.1]
• Narrow mandible with thin symphysis – vertical growth
Q.6. What is standardization in cephalometrics? Discuss
Frank Fort mandibular angle Less than normal in Steiner’s cephalometric analysis.
horizontal growth Ans.
Go-Gn to SN angle More than N in [Same as LE Q.2]
vertical growth
Q.7. Enumerate cephalometric analysis and elaborate
on Steiner’s analysis.

(c) Growth timetable: Timing of growth rates can be assed Ans.


from developmental stages, and occurrence of growth [Same as LE Q.2]
spurts can be estimated.
Q.8. Discuss in detail the clinical implications of growth
(d) Assessment of growth rates: Increase in size correlating
and development enumerating the use of cephalomet-
with growth rates in certain regions like N-mg, S-as,
rics to study the same.
Ar, Gn can be assessed by cephalometrics.
(e) Growth prediction: Ans.
l Various methods of growth prediction are Johnston
[Same as LE Q.3]
grid method, Holdaway’s prediction, Rickett’s predic-
tion (visual treatment objective).
l Visual treatment objective (VTO) acts as a blue
SHORT ESSAYS:
print to establish objectives of treatment for a par- Q.1. Uses of cephalometrics.
ticular patient, it is a visual plan to predict the
Ans.
normal growth of patient and anticipates the effects
of treatment. [Ref LE Q.1]
106 Quick Review Series for BDS 4th Year, vol 1

Q.2. Tweed’s triangle. dental pattern of the patients to define the underlying
facial type and establish the relation of dentition to the
Ans.
underlying bony structures.
l Down’s analysis consists of following skeletal and

dental parameters, five each.

Skeletal parameters Dental parameters


FH plane a. Facial angle a. Cant of occlusal plane
FMA b. Angle of convexity b. Interincisal angle
FMA c. A-B plane angle c. Incisor mandibular plane angle
d. Mandibular plane d. Incisor mandibular plane angle
Ma angle e. Upper incisor to A-pog
nd e. y-axis (growth axis)
ibu
lar
pla
ne
Skeletal Parameters:
PA
IM

(a) Facial angle:


l Facial angle is the inside inferior angle formed
Long axis of lower by the intersection of FH plane and nasion–
central incisor
pogonion plane.
Fig. 11.17  Tweed analysis. l This angle indicates anteroposterior positioning

of the mandible in relation to the upper face.


l The average value is n 87.8° with a range of 82–95°.
The objectives of Tweed’s analysis are as follows: h Facial angle – skeletal class III with prominent chin.
l Determination of lower incisor position g Facial angle – skeletal class II.
l Evaluation of prognosis of a case

I. The planes used in Tweed’s analysis that form a


diagnostic triangle are as follows:
l Frankfort horizontal plane

l Mandibular plane

l Long axis of lower incisor


N
ne

II. The significance of angles formed from these three


pla

planes is as follows:
FH

P
(a) Frankfort mandibular plane angle (FMA): The
angle formed by the Frankfort horizontal plane
with the mandibular plane.
The average value is 25°
If FMA is 16–28° – prognosis is good. Nasion-pogonion
plane
If FMA is 28–35° – prognosis is fair.
If FMA is .35° – prognosis is bad. Extractions
frequently complicate the problems.
(b) Incisor mandibular plane angle (IMPA): Angle Pog
formed between long axis of lower incisor with
mandibular plane. Mean value is 90°.
If IMPA . 110° – proclined lower incisors Fig. 11.18  Facial angle.
IMPA , 85° – retroclined lower incisors
(c) Frankfort mandibular incisor angle (FMIA): An-
gle formed between long axis of lower incisor and (b) Angle of convexity:
l Angle of convexity is formed by intersection of a
Frankfort horizontal plane. Mean value is 65°.
Tweed’s diagnostic triangle is used in diagnosis, line from nasion to point A and a line from point
classification, treatment planning as well as for A to pogonion.
l This angle reveals the convexity or concavity of
prognosis of orthodontic cases.
skeletal profile.
Q.3. Down’s analysis. l Average value is n 0° with a range of –8.5 to 10°.

l Positive or h angle of convexity – prominent


Ans.
maxillary denture base relative to mandible.
l Down’s analysis is one of the most frequently used Negative or g angle of convexity – prognathic
cephalometric analysis which measures the skeletal and profile.
Section | I  Topic-Wise Solved Questions of Previous Years 107

(N-A Line) nasion P FH plane


to point line O

Point A to pogonion
line (A-Pog line)
Ma
Pog ndi
bul
ar p
lan
e

Fig. 11.19  Angle of convexity.


Fig. 11.21  Mandibular plane Angle.

(c) A-B plane angle:


l A-B plane angle is formed between a line con- (e) y-axis (growth axis)
necting point A to point B and N-pog line. l y-axis or growth axis is an acute angle formed by

l This angle is indicative of maxillomandibular the intersection of Frankfort horizontal plane


relationship in relation to facial plane. with a line from sella turcica to gnathion.
Mean value is –4.6° with range of –9 to 0°. Usu- l y-axis indicates the growth pattern of the individ-

ally A-B plane angle is negative. ual; mean value is 59.4° with a range of 53–66°.
In class III cases A-B plane angle is positive. y-axis . normal – indicates greater vertical
growth of mandible.
y-axis , normal – indicates greater horizontal
growth of mandible.
l This angle is greater in class II than in class III

facial patterns.
N

B
Pog

Fig. 11.20  A-B plane angle.

(d) Mandibular plane angle:


l Mandibular plane angle is formed between FH

plane and mandibular plane.


l Mean value is 21.9° with range of 17–28°.

l h Mandibular plane angle suggests vertical

growth with hyper divergent facial pattern. Fig. 11.22  y-axis (growth axis).
108 Quick Review Series for BDS 4th Year, vol 1

Dental parameters: (c) Incisor occlusal plane angle:


(a) Cant of occlusal plane: l It is an inside inferior angle formed by the inter-

l This angle is formed between the occlusal plane section of the long axis of lower central incisor
and the Frankfort horizontal plane. and the occlusal plane.
l This angle gives a measure of the slope of l The average value is 14.5° with a range of 3.5–20°.

occlusal plane relative to the FH plane. l It is read as a plus or minus deviation from a

l Mean value is 9.3° with a range of 1.5–14°. right angle.


h Cant of occlusal plane seen in class II cases l h Incisor occlusal plane angle suggests lower

(short ramus). incisal proclination.


g Cant of occlusal plane seen in long mandibular
ramus.

Occlu
sal p
P lane
HP plane O

Occlus
al plan Long axis of lower
e
central incisor
Fig. 11.25  Incisor occlusal plane Angle.

(d) Incisor mandibular plane angle:


l This angle is formed by intersection of long axis
Fig. 11.23  Cant of occlusal plane.
of lower incisor and mandibular plane.
l Mean value is 1.4° with a range of –8.5 to 7°.

(b) Interincisal angle: l h Incisor mandibular plane angle – lower incisor

l The angle formed between the long axes of the proclination.


upper and lower incisors is known as interincisal
angle.
l The average is 135.4° with a range of 130–150.5°. Long axis of incisor
l g Interincisal angle – class I bimaxillary protru-

sion and class II, division 1 cases.


h Interincisal angle – class II, division 2 cases.

Ma
ndi
bul
ar p
lan
e

Fig. 11.26  Incisor mandibular plane angle.

(e) Upper incisor to A-pog line:


l Upper incisor to A-pog line or protrusion of

maxillary incisors is a linear measurement be-


tween the incisal edge of maxillary central inci-
Fig. 11.24  Interincisal angle. sor and line joining point A to pogonion.
Section | I  Topic-Wise Solved Questions of Previous Years 109

l Overage value is 2.7 mm with range of 1–5 mm. x. Solely based on cephalometric analysis, orthodon-
l The measurement is more in patients with upper tic diagnosis cannot be made.
incisor proclination.
Q.6. Visual treatment objective (VTO).
Ans.
l Visual treatment objective, i.e. VTO is of two types:
A. Clinical VTO
B. Cephalometric VTO
A. Clinical VTO
Upper i. Creekmore advocated clinical VTO as an aid to
incisor decide the type of appliance in skeletal class II
malocclusion.
ii. Procedure consists of asking the patient to bring the
mandible to an edge-to-edge bite relation, and note
Pog
the change in appearance of patient at two levels,
one at edge to edge and another at a position mid-
way between existing occlusion and edge-to-edge
Fig. 11.27  Upper incisor to A-Pog line. position.
iii. If the profile worsens in edge-to-edge position
n it means fault lies in maxilla, and to correct
Q.4. Enumerate five horizontal planes used in cephalo-
maxillary prognathism, appliances like maxil-
metrics and give their uses and significance.
lary intrusion splint or head gears are advised.
Ans. iv. If the profile improves at edge-to-edge position
n it indicates fault in mandible, then functional
[Ref LE Q.1]
appliances, which stimulate the growth of man-
Q.5. What is conventional cephalometrics? Give its dible, are indicated.
drawbacks. v. If the profile improves at midway position n then it
is a combination case of fault in both maxilla and
Ans.
mandible, and hence appliances like activator, head-
l Cephalogram is a cephalometric radiography intro- gear and twin block with headgears are indicated.
duced to orthodontics by Holly Broadbent and Herbert B. Cephalometric VTO
Hofrath in 1931. l It is like a blue print or a visual plan to predict the

l These are used in orthodontics to elucidate the skeletal, normal growth of the patient and the anticipated
dental and soft tissues relationships. effects of the treatment in order to establish the
l Limitations of cephalogram are as follows: objectives of treatment for individual patient.
i. Exposure of patient to harmful ionizing radiation. l VTO permits development of alternative treatment

ii. A serious disadvantage is the absence of anatomi- plans and to set the goals in advance for the treatment.
cal references which remain constant with time. Q.7. Steiner’s skeletal analysis.
iii. The process of image acquisition and measure-
ment procedures are not well standardized. Ans.
iv. The structures being imaged are three-dimensional [Ref LE Q.2]
whereas the radiographic image is two-dimensional.
v. Anatomical structures lying at different planes with Q.8. Write the skeletal measurements of Down’s analysis.
in the head undergoes projective displacement. Ans.
vi. Some reference landmarks and planes do not agree
[Same as SE Q.3]
with anatomical landmarks.
vii. There could be a mandibular shift from centric
relation as the patient is made to bite in maximum SHORT NOTES:
intercuspation. Q.1. y-axis.
viii. The cephalometrics makes use of means obtained
Ans.
from different population samples; hence they
have only limited relevance when applied to indi- y-axis (growth axis) is considered as one of the skeletal
vidual patient. parameters in Down’s analysis.
ix. The composite of lines and angles used in cepha- l y-axis or growth axis is an acute angle formed by the

lometric analysis yields limited information about intersection of Frankfort horizontal plane with a line
patient’s dento-skeletal patterns. from sella turcica to gnathion.
110 Quick Review Series for BDS 4th Year, vol 1

iii. The process of image acquisition and measurement


procedures are not well standardized.
iv. The structures being imaged are three-dimensional
S
whereas the radiographic image is two-dimensional.
N
So, anatomical structures lying at different planes
within the head undergo projective displacement.
Y-axis
FH
v. The cephalometrics make use of means obtained from
plane different population samples; hence they have only
limited relevance when applied to individual patient.
Q.4. Enumerate horizontal planes in cephalometry.
Ans.
In cephalometrics, various lines and planes are obtained by
connecting two landmarks. Based on their orientation, they
Gn are classified as horizontal and vertical planes.
Various horizontal planes are as follows:
Fig. 11.28  y-axis (growth axis). i. S-N plane
ii. Frankfort horizontal plane
iii. Occlusal plane
l y-axis indicates the growth pattern of the individual.
iv. Mandibular plane
l Mean value is 59.4° with a range between 53 and 66°.
v. Basion–nasion plane
y-axis . normal – indicates greater vertical growth of
mandible. Q.5. Interincisal angle.
y-axis , normal – indicates greater horizontal growth of
Ans.
mandible.
l This angle is greater in class II than in class III facial [Ref LE Q.2]
patterns.
Q.6. S-N plane.
l During orthodontic treatment:

Increase in y-axis suggests – vertical growth pattern or Ans.


open bite. i. S-N plane represents the anterior cranial base.
Decrease in y-axis suggests – horizontal growth pattern ii. It is a horizontal cephalometric plane between the cen-
or deepening of bite. tre of sellatursica (S) and the most anterior point of the
frontonasal suture (N), i.e. nasion.
Q.2. Key ridge.
Ans.
l Key ridge is the lower most point on the contour of the
anterior wall of the infratemporal fossa. Or
l It is the inferior most point of the anterior border of

zygoma as seen in lateral cephalogram.


S N
l In his concept of normal occlusion, Angle related maxil-

lary first molar to key ridge position, normally the me-


siobuccal root of maxillary permanent first molar is in
line with the key ridge.
Q.3. Limitations of cephalogram.
Ans.
l Cephalogram is a cephalometric radiography intro-
duced to orthodontics by Holly Broadbent and Herbert
Hofrath in 1931.
Fig. 11.29  Sella-Nasion plane.
l These are used in orthodontics to elucidate the skeletal,

dental and soft tissue relationships.


l Limitations of cephalogram are as follows: Q.7. ANB angle.
i. Exposure of patient to harmful ionizing radiation.
Ans.
ii. A serious disadvantage is the absence of anatomical
references which remain constant with time. [Ref LE Q.2]
Section | I  Topic-Wise Solved Questions of Previous Years 111

Q.8. FMA angle.


Ans.
l FMA is an angle formed at the intersection of the
Frankfort horizontal plane with the mandibular plane.
FH plane
l Mean value of FMA is 25°.
FMA
l The Tweed’s analysis makes use of FMA angle in a FMA
diagnostic triangle to determine position of lower inci-
sor and evaluation of prognosis of a case. Ma
nd
ibu
lar
pla
ne
PA
IM

FMA Long axis of


FMA lower central
incisor
Fig. 11.31  Tweed analysis.

plane and long axis of lower incisor that form three


sides of the diagnostic triangle is as follows:
a. Frankfort mandibular plane angle (FMA): The
angle formed by the Frankfort horizontal plane
IMPA
with the mandibular plane.
Fig. 11.30  Tweed’s diagnostic triangle showing FMA angle. The average value is 25°. If FMA is 16–28° –
prognosis is good; FMA is 28–35° – prognosis is
Q.9. SNA angle. fair; FMA . 35° – prognosis is bad. Extractions
frequently complicate the problems.
Ans.
b. Incisor mandibular plane angle (IMPA): Angle
[Ref LE Q.2] formed between long axis of lower incisor with
mandibular plane.
Q.10. Two uses of cephalometrics in orthodontia.
Mean value is 90°. If IMPA .110° – proclined
Ans. lower incisors, IMPA , 85° – retroclined lower
incisors.
i. Cephalometrics is a nonessential or supplemental diag-
c. Frankfort mandibular incisor angle (FMIA): An-
nostic aid in orthodontic diagnosis.
gle formed between long axis of lower incisor and
ii. In 1931, a standardized cephalometric technique was
Frankfort horizontal plane. Mean value is 65°.
simultaneously presented by Holly Broadbent (USA)
and Herbert Ho Frath (Germany). Q.12. Occlusal plane angle.
iii. Cephalometrics helps in:
Ans.
l Orthodontic diagnosis

l Classification Occlusal plane


l Treatment planning It is a denture plane bisecting the posterior occlusion of
l Evaluation of treatment results permanent molars and premolars and extends anteriorly.
Significance:
Hence, it forms a valuable tool in treatment planning and
l The angle formed between occlusal plane and
follow-up of orthodontic patients.
S-N plane is known as occlusal plane angle.
Q.11. Tweed’s diagnostic triangle. l It indicates the relation of occlusal plane to

cranium and face, i.e. the growth pattern of an


Ans.
individual.
i. Tweed’s diagnostic triangle is used in diagnosis, classi- l It has a mean value of 14.5°.
fication, treatment planning as well as for prognosis of h Occlusal plane angle – clockwise rotation of
orthodontic cases. occlusal plane
l The significance of angles formed from the three g Occlusal plane angle – counterclockwise rota-
planes, i.e. Frankfort horizontal plane, mandibular tion of occlusal plane
112 Quick Review Series for BDS 4th Year, vol 1

Q.14. Registration point.


Ans.
The Broadbent registration point (R point) is the midpoint
of the perpendicular from the centre of sellatursica to the
Bolton–nasion plane.

S N

Fig. 11.32  Occlusal plane.


Bo
Q.13. Steiner’s soft tissue analyses. Ba

Ans.
i. Cecil C. Steiner in 1930 developed a cephalometric
analysis. The idea is to provide maximum clinical in-
formation with least number of measurements.
ii. The Steiner’s analysis consisted of three parts, namely
skeletal analysis, dental analysis and soft tissue analysis.
Broad bent
iii. Steiner’s soft tissue analysis: According to Steiner, in a
Fig. 11.34  Registration point.
well-balanced face, lips should touch the line extending
from soft tissue contour of chin to midline of an ‘S’
formed by the lower border of nose.
l Lips located beyond this line indicates n protrusive Q.15. Facial divergence.
(convex profile). Ans.
l Lips located behind this line n retrusive (concave profit).
i. Facial divergence was described by Milo Hellmann as
the inclination of lower face relative to forehead.
ii. It determines the position of lower part of the face rela-
tive to the forehead using soft tissue landmarks, i.e. soft
tissue nasion and pogonion.
iii. Based on a line drawn between the forehead and the
chin in the natural head position, the facial divergence
is of the following types:
a. Posterior divergent face: When the line is inclined
posteriorly in the chin region, it is known as poste-
rior divergent face, seen in class II cases.
b. Straight or orthognathic face: When the line is per-
pendicular to the floor, it is known as straight or
S-Line orthognathic face, seen in class I cases.
c. Anterior divergent face: When the line is inclined
anteriorly in the chin region, it is known as anterior
divergent face, seen in class III cases.
Q.16. Angle SNB.
Ans.
Fig. 11.33  S-line. [Ref LE Q.2]
Section | I  Topic-Wise Solved Questions of Previous Years 113

Q.17. Mandibular plane angle.


Ans.
Mandibular plane angle is one of the parameters used in
Steiner’s skeletal analysis.
l Mandibular plane angle is formed between S-N plane

and mandibular plane. Bo


l This angle gives an indication of the growth pattern of
Ao
an individual.
l The average value is 32°.

A lower angle indicates n horizontal growing face.


An increased angle indicates n vertical growing face.

Fig. 11.36  Wits appraisal.


SN plane
S N
ii. The distance between AO and BO gives the anteropos-
Mandibular
plane angle
terior relation between the maxilla and the mandible.
iii. Usually in males, point BO is ahead of AO by 1 mm.
In females, points AO and BO coincide.
In skeletal class II tendency, BO is behind AO (positive
reading)
In skeletal class III pattern, point BO is located ahead
Go of AO (negative reading)
The Wits appraisal is mainly used in cases where the ANB
angle is considered not reliable due to abnormal position of
Mandibular plane nasion and rotation of jaws.
Ga Q.20. Frankfort horizontal plane.

Fig. 11.35  Mandibular plane angle. Ans.


Frankfort horizontal plane
l This is a horizontal plane connecting the orbitale and

porion.
Q.18. Cephalostat.
l It is one of the cephalometric planes used in the
Ans. Down’s analysis.
l It is used as a reference plane in measuring both
i. Cephalostat is a head holder device, one of the compo-
skeletal and dental patterns of patient.
nents of standard apparatus used to take cephalograms.
ii. A cephalostat consists of ear rods, orbital pointer and
forehead clamp to stabilize the head of the patient and
position it in three dimensions to receive X-ray beam.
iii. It positions the patient’s head so that distance between
the X-ray source and midsagittal plane of patient is at a
fixed distance of 5 feet.
P
Q.19. The Wits appraisal.
O
Ans.
The Wits appraisal is a measure of the maxillomandibular
relation in the anteroposterior or sagittal plane.
i. A functional occlusal plane is drawn and perpendiculars
are dropped from points A and B on it. The points of
contact of these perpendiculars on the occlusal plane are
termed as AO and BO. Fig. 11.37  Frankfort horizontal plane.
114 Quick Review Series for BDS 4th Year, vol 1

Q.21. Define cephalometric points. i. It is one of the soft tissue measurements considered in
McNamara analysis.
Ans.
ii. Nasolabial angle is formed by drawing a line tangent to
i. Point S: The point representing the midpoint of the the upper lip.
pituitary fossa or sella turcica is known as point S. iii. Average value is 1028o.
i i. Point Me: Point Me is the inferior most point in the iv. An acute nasolabial angle indicates dentoalveolar pro-
contour of the chin. trusion. It may also be due to upturned orientation of
the base of the nose.
Q.22. ‘E’ plane or aesthetic plane.
Q.25. Advantages of computerized cephalometric
Ans.
system.
l ‘E’ plane is a line between the most anterior point of the
soft tissue nose and soft tissue chin. Ans.
l With all the above-mentioned standard points and mea- l In orthodontics, computerized cephalometric systems
surable planes and angles, cephalometric aids in skele- are employed for the purpose of diagnosis, prognosis
tal, dental and soft tissue analysis and classification of and treatment evaluation.
various malocclusions. l The advantages of computerized cephalometric system

are as follows:
i. Less time-consuming
ii. Easy to store
iii. Easy to retrieve old records
iv. Combined evaluation of patient’s photographs, casts
and cephalogram is possible
v. Efficient in research application
Q.26. Significance of ANB angle.
Ans.
[Same as SN Q.7]
Q.27. Uses of cephalometrics.

Fig. 11.38  Aesthetic plane.


Ans.
[Same as SN Q.10]
Q.23. Computerized cephalometric system.
Q.28. Tweed’s triangle.
Ans.
Ans.
There are two basic components of computerized cephalo-
metric system: [Same as SN Q.11]
a. Data acquisition Q.29. Occlusal plane.
b. Data management
l Data acquisition: It is done by either regular radio- Ans.
graphs or digital radiographs.
[Same as SN Q.12]
l Data management: Various commercially available data

management programmes are as follows: Q.30. Mandibular plane.


i. Rocky mountain orthodontics (RMOs): Jiffy orth-
odontic evaluation Ans.
ii. Pordios [Same as SN Q.17]
iii. Dentofacial planner
iv. Quickceph image Q.31. Name some data management programmes in
v. Digi graph computerized cephalometric system.

Q.24. Nasolabial angle. Ans.


Ans. [Same as SN Q.23]
Section | I  Topic-Wise Solved Questions of Previous Years 115

Topic 12
Skeletal Maturity Indicators
COMMONLY ASKED QUESTIONS
LONG ESSAYS:
1. Enumerate various methods available to assess the skeletal maturity of an individual and its implications in
orthodontic diagnosis and treatment planning. Explain in detail about hand-wrist x-rays.

SHORT ESSAYS:
1 . Hand-wrist radiography. [Ref LE Q.1]
2. Compare skeletal age and dental age.
3. Cervical vertebrae as skeletal maturity indicators.
4. Implant radiography in orthodontics.
5. How hand-wrist x-rays are useful in orthodontic diagnosis and treatment planning? [Same as SE Q.1]
6. Dental versus skeletal age. [Same as SE Q.2]

SHORT NOTES:
1 . Carpals.
2. Hand-wrist X-ray. [Ref LE Q.1]
3. Skeletal age.
4. Vital staining.
5. Dental age.
6. Carpal index. [Same as SN Q.1]
7. Hand-wrist radiography. [Same as SN Q.2]

SOLVED ANSWERS
LONG ESSAYS:
Q.1. Enumerate various methods available to assess the implications on the treatment planning and response
skeletal maturity of an individual and its implications in to treatment.
orthodontic diagnosis and treatment planning. Explain ii. While planning orthopaedic therapy, functional appli-
in detail about hand-wrist x-rays. ance therapy and orthognathic surgery, the knowledge
of position of patient in the facial growth curve is
Ans. important.
Various skeletal maturity indicators commonly used in During the periods of accelerated growth, the orthopae-
orthodontics are as follows: dic or functional appliance treatment can contribute
i. Hand-wrist radiographs significantly to correction of dentofacial deviations,
ii. Evaluation of cervical vertebrae using cephalogram leading to an improvement in facial appearance.
iii. Clinical and radiographical examination of different stages iii. The skeletal maturity or skeletal age is accurate in as-
of tooth development, especially canine calcification. sessing the physical maturity. The other parameters,
like peak height velocity, secondary sexual changes and
Clinical implications of skeletal maturity assessment sta- dental age, are inferior to skeletal age in estimating
tus on diagnosis and treatment planning in orthodontics physical maturity.
are as follows: The bones mature at different rates and follow a reason-
i. The skeletal maturity status of an individual helps in able sequence; hence, the developmental status of a
knowing whether the patient will grow appreciably child can be estimated by determination of degree of
during the treatment period. This has important completion of facial skeleton.
116 Quick Review Series for BDS 4th Year, vol 1

(SE Q.1 and SN Q.2) l Patients’ radiographs are matched with one of the
photographs in the atlas which is a representative of
{(Hand-wrist radiographs: particular skeletal age.
l Numerous small bones in the hand-wrist region
ii. Bjork, Grave and Brown method
show a predictable and scheduled pattern of appear-
l According to Bjork, skeletal development in the
ance, ossification and union from birth to maturity.
hand-wrist area is divided into eight stages, each
Thus, by comparing patient’s hand-wrist radiograph
of them represents a particular level of skeletal
with standard radiographs that represent different
maturity.}
skeletal ages, the skeletal maturation status of an
individual can be determined.
According to Bjork, stages of skeletal development in
l The left hand-wrist is used by convention and a PA
hand-wrist region
view is taken to register the hand-wrist region.
l Among various methods described to assess skeletal
S. No. Stage Object Growth Phase
maturity using hand-wrist radiographs, the most
1. PP2 Proximal Width of Slow rate of
commonly used ones are as follows:
phalanx of epiphysis 5 growth
i. Atlas method by Greulich and Pyle index finger width of
ii. Bjork, Grave and Brown method diaphysis
iii. Fishman’s skeletal maturity indicators 2. MP3 Middle phalanx Width of Maximum
iv. Hagg and Taranger method)} of middle finger epiphysis 5 long growth
width of di- imminent
Anatomy of hand-wrist region: aphysis
l Each hand-wrist area has 8 carpals, 5 metacarpals
3. S Ulnar sesamoid Sign of ossi- As stage 2
and 14 phalanges, which make a total of 27 bones.
on metacarpo- fication
l Distal ends of radius and ulna also appear in the hand-
phalangeal
wrist radiograph. Radius and ulna are the long bones joint of thumb
of the forearm. When the palm is facing front, ulna lies 4. MP3 Middle phalanx Encapsula- Maximum
in the medial aspect and radius in the distal aspect. cap of middle finger tion of di- long growth
l Carpal bones were first named by Lyser, these are eight aphysis
irregularly shaped small bones arranged in two rows: 5. DP3u Distal phalanx Epiphysis Maximum
a. Proximal row – scaphoid, lunate, triquetral and of middle finger united long growth
pisiform over
b. Distal row – trapezium, trapezoid, capitates and 6. PP3u Proximal Epiphysis Maximum
hamate phalanx of united long growth
These small irregular bones lie in-between the long middle finger over
bones of forearm and the metacarpals. 7. MP3u Middle phalanx Epiphysis Past
l Metacarpals are small long bones and are numbered of middle finger united maximum
1–5 starting from thumb to little finger. Each of the growth
five metacarpals has a base, shaft and head. These lie 8. Rc Distal epiphysis United Growth
between the carpals and phalanges, forming the skel- of radius and complete
etal framework of the palm. ulna
l Phalanges are small bones forming the fingers. Each

finger has three phalanges – proximal, middle and [SE Q.1]


distal phalanx. Middle phalanx is absent in the thumb. {iii. Fishman’s skeletal maturity indicators
l The phalanges have a pattern of ossification, which
l Leonard S. Fishman proposed a system for the
occurs in three stages: evaluation of skeletal maturation by making use of
Stage 1: The epiphysis and diaphysis are equal. anatomical sites located on the thumb, third finger,
Stage 2: The epiphysis caps the diaphysis by sur- fifth finger and radius.
rounding it. l Covering the entire period of adolescent develop-
Stage 3: The epiphysis and diaphysis are fused. ment, 11 descrete skeletal maturity indicators have
l Sesamoid bone is a small nodular bone, mostly em-
been described, which are as follows:
bedded in the tendons of thumb region.
i. Width of epiphysis and diaphysisare equal in
[SE Q.1] proximal phalanx of third finger
{i.  Atlas method by Greulich and Pyle ii. Width of epiphysis equal to that of diaphysis
l Greulich and Pyle published an atlas containing pic- in the middle phalanx of third finger
tures of hand-wrist for different chronological ages iii. Width of epiphysis equal to that of diaphysis
for both the sexes. in the middle phalanx of fifth finger
Section | I  Topic-Wise Solved Questions of Previous Years 117

iv. Appearance of adductor sesmoid of the thumb v. Stage I


v. Capping of epiphysis seen in the distal phalanx of Fusion of epiphysis and metaphysis is
third finger completed. All individuals except a few
vi. Capping of epiphysis seen in the middle phalanx of girls have ended the pubertal growth spurt.
third finger C. Distal phalanx of third finger:
vii. Capping of epiphysis seen in the middle DP3-I: Fusion of the epiphysis and metaphysis is
phalanx of fifth finger completed.
viii. Fusion of epiphysis and diaphysis seen in the This stage signifies the fusion of the epiphysis
distal phalanx of third finger and metaphysis and is attained during the decel-
ix. Fusion of epiphysis and diaphysis seen in the eration period of the pubertal growth spurt (i.e.
proximal phalanx of third finger end of PHV) by all subjects.
x. Fusion of epiphysis and diaphysis seen in the D. Radius:
middle phalanx of third finger R-I:
xi. Fusion of epiphysis and diaphysis seen in the Beginning of fusion of the epiphysis and me-
radius taphysis.
iv. Hagg and Taranger method In about 80% of the girls and 90% of the
l Hagg and Taranger noted that skeletal development boys, this stage is attained 1 year before or at
in hand and wrist can be analysed from the assess- the end of growth spurt.
ment of ossification of ulnar sesamoid of metacrpo- R-IJ:
phalangeal joint of the first finger (S) and certain Fusion is almost completed but there is still a
specified stages of three epiphyseal bones: the mid- small gap at one or both margins.
dle and distal phalanges of the third finger (MP3 and R-J:
DP3) and the distal epiphysis of the radius (R) by Fusion of the epiphysis and metaphysis oc-
taking annual radiographs between 6 and 18 years curred.
of age.} None of the subjects had attained these stages
A. Sesamoid: before the end of PHV.
It is usually attained during the acceleration pe-
riod of the pubertal growth spurt, i.e. onset of
peak height velocity (PHV).
SHORT ESSAYS:
B. Stages of ossification of middle phalanx of Q.1. Hand-wrist radiography.
third finger (MP3):
Ans.
They follow pubertal growth spurt. The stages of
ossification are outlined from stage F to stage I [Ref LE Q.1]
as follows:
Q.2. Compare skeletal age and dental age.
i. Stage F
The epiphysis is as wide as the metaphy- Ans.
sis. About 40% of individuals are before
PHV. Very few are at PHV. Skeletal age Dental age
ii. Stage FG It is based on the The formation of teeth or erup-
The epiphysis is as wide as the metaphysis, ossification of tion of the teeth is the basis for
and there is a distinct medial or lateral (or endochondral bone. calculating the dental age.
both) border of the epiphysis forming a line It is assessed based on the Assessed based on the number
of demarcation at right angles to the border. skeletal maturity indicators of teeth at each chronological
like hand-wrist radiographs age or on stages of formation of
About 90% of individuals are one year be-
and cervical vertebrae. crowns and roots of the teeth.
fore or at PHV.
iii. Stage G It helps in assessing physical It has no role in physical
maturity of an individual. maturity assessment.
The sides of the epiphysis are thickened,
and there is capping of the metaphysis, It is helpful in treatment It can reflect an assessment of
forming a sharp edge distally at one or planning and tracking physiologic age comparable to
response to treatment. age based on the skeletal de-
both sides. About 90% of individuals are at velopment, weight or height.
or one year after PHV.
iv. Stage H There is a correlation When the last tooth has
between the dental age completed its development,
Fusion of the epiphysis and metaphysis and skeletal age. it is an indication that the
has begun. About 90% of girls and all boys skeleton is approaching
are after PHV but before the end of the complete maturation.
pubertal growth spurt.
118 Quick Review Series for BDS 4th Year, vol 1

Q.3. Cervical vertebrae as skeletal maturity indicators. vi. Stage 6: Completion


l This stage corresponds to completion of growth.
Ans.
Little or no adolescent growth is expected.
l A system of skeletal maturation determination using cer- l More accentuated concavities are seen in the
vical vertebrae was introduced by Hassel and Farman. lower borders of C2, C3 and C4.
l The shapes of the cervical vertebrae were seen to differ l The body shapes of C3 and C4 were square or
at each level of skeletal development which provided a were greater in vertical dimension than in hori-
means to determine the skeletal maturity of a person and zontal dimension.
thereby estimate whether the possibility of potential
Q.4. Implant radiography in orthodontics.
growth existed.
l The six stages put forward by Hassel and Farman in as- Ans.
sessing skeletal growth are named as follows:
i. Implant radiography is an experimental method for
Stage 1: initiation
studying physical growth.
Stage 2: acceleration
ii. Professor Bjork introduced human implant radiograph
Stage 3: transition
for growth measurement.
Stage 4: deceleration
iii. Procedure:
Stage 5: maturation
l Inert metal pinseg: Tantalum pins, 1.5 mm long and
Stage 6: completion
0.5 mm in diameter, were placed in the mandible.
Changes observed in various stages are as follows:
l These metal pins get osseo-integrated and serve as
i. Stage 1: Initiation
reference points.
l Marks the beginning of adolescent growth with
l Serial cephalometric radiographs are taken repeat-
80%–95% of adolescent growth expected.
edly over a period of time, and compared.
l Inferior borders of C2, C3 and C4 were flat, and
l Only implant radiography can estimate rotation of
superior borders were tapered from posterior to
jaw bones.
anterior and vertebrae were wedge-shaped.
iv. Information obtained from implant radiography:
ii. Stage 2: Acceleration
It gives very accurate information about the site of
l Acceleration of growth begins at this stage with
growth, amount of growth and direction of growth, and
65%–85% of adolescent growth expected.
a relatively accurate information about the rate of
l Concavities are seen in the lower borders of C2
growth.
and C3 and the lower border of C4 will be flat.
v. Drawbacks:
l The bodies of C3 and C4 were nearly rectangular
l It is a two-dimensional study of three-dimensional
in shape.
process.
iii. Stage 3: Transition
l Radiation hazard.
l Corresponds to acceleration of growth to peak

height velocity with 25%–65% of adolescent Q.5. How hand-wrist x-rays are useful in orthodontic
growth expected. diagnosis and treatment planning?
l Marked concavities are seen in the lower borders
Ans.
of C2 and C3 and a concavity was beginning to
develop in the lower border of C4. [Same as SE Q.1]
l The bodies of C3 and C4 were rectangular in
Q.6. Dental versus skeletal age.
shape.
iv. Stage 4: Deceleration Ans.
l Deceleration in adolescent growth spurt with
[Same as SE Q.2]
10%–25% of adolescent growth expected.
l Marked concavities are seen in C2, C3 and C4 in

their lower borders.


SHORT NOTES:
l Vertebral bodies of C3 and C4 are square in Q.1. Carpals
shape.
Ans.
v. Stage 5: Maturation
l Final maturation of vertebrae took place during i. Carpals are the bones of hand-wrist region.
this stage with 5%–10% of adolescent growth ii. They were first named by Lyser.
expected. iii. They are eight irregularly shaped small bones arranged
l More accentuated concavities are seen in the in two rows:
lower borders of C2, C3 and C4. (a) Proximal row-scaphoid, lunate, triquetral and pisi-
l The bodies of C3 and C4 are square in shape. form.
Section | I  Topic-Wise Solved Questions of Previous Years 119

(b) Distal row-trapezium, trapezoid, capitate and hamate. Example: Commonly used dyes for vital staining are
These small irregular bones lie in-between the long alizarin S, radioactive tracers, fluorochrome, tetracy-
bones of forearm and the metacarpals. cline and trypan blue.
iv. Each carpal bone ossifies from one primary centre, iii. Animals are sacrificed and tissues are studied histo-
which appears in a predictable pattern. logically.
iv. This cross-sectional study elicits information regarding
Q.2. Hand-wrist X-ray?
detailed analysis of site and the amount of growth and
Ans. as well as the rate of growth.
v. Disadvantage: It is not a longitudinal study, i.e. re-
[Ref LE Q.1]
peated data of the same individual cannot be obtained.
Q.3. Skeletal age.
Q.5. Dental age.
Ans.
Ans.
i. The skeletal maturity or skeletal age is more superior
i. Dental age usually correlates with chronological age
and accurate in assessing the physical maturity than the
and is assessed based on the following:
other parameters like peak height, velocity, secondary
(a) Amount of root resorption of primary teeth
sexual changes and dental age.
(b) Teeth which have erupted
ii. Skeletal age is based on the ossification of endochon-
(c) Amount of permanent teeth development
dral bone.
ii. Dental age is assessed on the basis of the number of
iii. It can be assessed based on the various skeletal maturity
teeth at each chronological age or on the stages of for-
indicators like hand-wrist radiographs, evaluation of
mation of crowns and roots of the teeth.
cervical vertebrae and clinical and radiographic exami-
iii. Dental age can reflect an assessment of physiologic age
nation of different stages of tooth development.
comparable to age based on the skeletal development,
iv. Assessing the skeletal maturity is helpful in diagnosis,
weight or height.
treatment planning and response to treatment in ortho-
iv. Complete formation of the last tooth is an indication
dontics.
that the skeleton is approaching complete maturation.
v. The dental and skeletal ages correlate with each other.
Q.6. Carpal index.
Q.4. Vital staining.
Ans.
Ans.
[Same as SN Q.1]
i. Vital staining is one of the experimental methods of
measuring growth introduced in the eighteenth century Q.7. Hand-wrist radiography.
by John Hunter.
Ans.
ii. It consists of injecting dyes that stain and get deposited
in the mineralizing tissues like bones and teeth. [Same as SN Q.2]

Topic 13
Model Analysis
COMMONLY ASKED QUESTIONS
LONG ESSAYS:
1. Enumerate the various diagnostic aids used in orthodontics and add a note on study models.

SHORT ESSAYS:
1 . Pont’s analysis.
2. Korkhau’s analysis.
3. Carey’s analysis.
4. Ashley Howe’s index.
5. Mixed dentition analysis. Explain one in detail.
6. Radiographic method of mixed dentition analysis.
120 Quick Review Series for BDS 4th Year, vol 1

7. Bolton’s tooth size ratio.


8. Kesling tooth positioning.
9. Pont’s index. [Same as SE Q.1]
10. Carey’s analysis or arch perimeter analysis. [Same as SE Q.3]
11. Ashley Howe’s index. [Same as SE Q.4]
12. Assessment of tooth mass discrepancy. [Same as SE Q.4]
13. Moyer’s mixed dentition analysis. [Same as SE Q.5]
14. Bolton’s analysis. [Same as SE Q.7]
15. Kesling’s diagnostic set-up. [Same as SE Q.8]

SHORT NOTES:
1. Pont’s analysis. [Ref SE Q.1]
2. Model analysis.
3. Carey’s analysis.
4. Peck and Peck ratio.
5. Name few model analysis for mixed dentition. [Ref SE Q.5]
6. Linderhearth’s ratio.
7. Radiographic method of mixed dentition analysis.
8. Study models – uses. [Ref LE Q.1]
9. Bolton’s analysis. [Ref SE Q.7]
10. Tanaka–Johnston analysis.
11. Korkhau’s analysis.
12. Gnathostatic models.
13. Kesling’s diagnostic set-up. [Ref SE Q.8]
14. Ashley Howe’s index. [Ref SE Q.4]
15. Pont’s index. [Same as SN Q.1]
16. Arch perimeter analysis. [Same as SN Q.3]
17. Bolton’s tooth size ratio. [Same as SN Q.9]
18. Bolton index. [Same as SN Q.9]

SOLVED ANSWERS
LONG ESSAYS:
Q.1. Enumerate the various diagnostic aids used in
{SN Q.8}
orthodontics and add a note on study models.
l (Study models are one of the essential orthodontic
Ans.
diagnostic aids)
The various diagnostic aids used in orthodontics are as follows:
that make it possible to study the arrangement of teeth and
occlusion from all three planes, i.e. sagittal, vertical and
transverse planes of space.
Orthodontic Diagnostic Aids
{SN Q.8}
Uses of study models are as follows:
Essential diagnostic aids Nonessential or supplemental i. They allow study of occlusion from all aspects.
diagnostic aids
(Very important for all cases (Are not essential in all cases ii. They enable accurate measurements to be made in den-
and are simple, and do not and require special equipment)
tal arch such as arch length, arch width and tooth size.
require expensive equipment)
iii. Assessment of treatment progress by both patients and
Examples: Examples: dentist.
a. Case history and clinical a. Specialized radiographs
examination b. EMG {SN Q.8}
b. Study models c. Hand wrist X-ray
c. Certain basic radiographs d. Endocrine tests iv. They help in assessing the nature and severity of
d. Facial photographs e. Estimation of BML malocclusion as well as motivation of the patient to
f. Diagnostic set-up
orthodontic therapy.
g. Occlusograms
Section | I  Topic-Wise Solved Questions of Previous Years 121

iii. Casting the impression


v. They are useful to explain treatment plan as well as
iv. Basing and trimming of the cast
progress of treatment to the patient and parents.
v. Finishing and polishing

vi. They make it possible to simulate treatment procedures i. Impression making:


on cast called mock surgery. It is an important step in making orthodontic study
models, as they should accurately reproduce as
much of supporting structures as possible. The
{SN Q.8} ideal trays should be selected such that they in-
vii. In case the patient is to be treated by another dentist, clude last erupted molars and should have a clear-
they make it easy to transfer records of the patient. ance of around 3 mm between the teeth and the
tray. The irreversible hydrocolloids or alginates are
widely used materials for making impressions.
Ideal requirements of a study model: ii. Disinfecting the impression:
l The study models should accurately reproduce the
This can be done by soaking the impression in a
teeth and surrounding soft tissues without any disinfectant solution such as biocide, 2% glutar-
distortion. aldehyde, for a specified duration of time.
l They should not only depict the teeth but also repro-
iii. Casting the impression:
duce as much of alveolar process as possible, with a Impressions obtained are usually casted in orth-
clean, smooth and nodule-free surface. odontic stone or model stone.
l They should be trimmed in such a way that they are
iv. Basing and trimming of the cast:
symmetrical and pleasing to the eye. When placed on Rubber base formers are available to help in mak-
their backs, they should accurately reproduce the oc- ing the base (i.e. the artistic portion of the study
clusion and enable instant identification of asymme- cast) over the anatomic portion. The trimming of
tries in the arch form. orthodontic model is a meticulous process which
Parts of a study model: is done on electric plaster trimming machine.
l The orthodontic study model consists of the follow-
v. Finishing and polishing
ing parts (Fig. 13.1): The final polishing of casts is done by placing
(a) Anatomic portion them in soap solution for 1 hour and then remov-
(b) Artistic portion ing and rinsing under cold water. The casts are
l The anatomic portion of study model is the actual
then allowed to dry and buffed to acquire smooth
impression of the dental arch and its surrounding and shining appearance. They can be polished
structures. using fine-grained sandpaper.
l Usually this portion is made of stone plaster. The
The finished study models can be stored in boxes
artistic portion of study model consists of a plaster for the future reference.
base that supports the anatomic portion.
In a well-fabricated set of study models, the ratio of
anatomic portion to artistic portion should be 3:1. SHORT ESSAYS:
Q.1. Pont’s analysis.
Ans.
Pont’s analysis was presented by Pont in 1909.
Anatomic
portion
{SN Q.1}

Artistic
l Pont’s index or Pont’s analysis is a method of prede-
portion termining the ideal arch width based on mesiodistal
width of crowns of maxillary incisors.
Fig. 13.1  Orthodontic study model. l By this analysis, the width of arch in premolar and

molar regions can be established by measuring the


Construction of study models: greatest width of maxillary incisors.
l The parameters considered in this analysis are as follows:
The steps involved in the construction of study models
are as follows: (i) Determination of sum of incisors (SI) (Fig. 13.2)
i. Impression making The summed up values of mesiodistal width of
ii. Disinfection of the impression four maxillary incisors is known as the sum of
incisors (SI).
122 Quick Review Series for BDS 4th Year, vol 1

(iv) D
 etermination of calculated premolar value (CPV)
The expected arch width in the premolar region
or the calculated premolar value is determined
by the following formula: SI 3 100/80.
(v) Determination of calculated molar value (CMV)
l The expected arch width of the molar region
2 1 1 2
or the calculated molar value is determined
by the following formula: SI 3 100/64.
l Inference:

If the measured value is less than the cal-


culated value, it indicates the need for ex-
pansion.

Fig. 13.2  Sum of incisors (SI). Whether dental arch is


narrow/normal.
(ii) Determination of measured premolar value
(MPV) (Fig. 13.3) Uses of Pont’s Index
Analysis: It helps Need for lateral expansion of arch.
The MPV is the arch width of premolar region
you to determine
from the distal pit of one upper first premolar
to the distal pit of the opposite first premolar. How much expansion is possible at
premolar and molar regions?

Q.2. Korkhau’s analysis.


Ans.
Korkhau in 1938 proposed a study model analysis which
1st 1st reveals anteroposterior malpositioning of incisors in maxil-
PM PM lary and mandibular arches (Fig. 13.5).

Fig. 13.3  Measured premolar value (MPV).

(iii) D
 etermination of measured molar value
(MMV) (Fig. 13.4)
The MMV is the arch width of the molar region
from the mesial pit of one upper first molar to
the mesial pit of the opposite first molar.
Fig. 13.5  Korkhau’s analysis.

Method:
i. Sum of upper incisors (SIu) 5 Added measurement
of mesiodistal width of maxillary incisors.
ii. Available anterior arch length (AAAL) 5 Measure-
ment made from the midpoint of the interpremolar
line to the point between two maxillary incisors.
iii. The ideal anterior arch length (IAAL) is determined
by using the Korkhau’s formula:
SIu
IAAL 100
160
1st Molar 1st Molar
iv. If the AAAL . IAAL, it indicates that maxillary
Fig. 13.4  Measured molar value (MMV). central incisors are malpositioned anteriorly.
Section | I  Topic-Wise Solved Questions of Previous Years 123

Example: labioversion of anteriors, bimaxillary


{SN Q.14}
protrusion
v. If the AAAL , IAAL, it indicates that maxillary l Ashley Howe’s analysis is a model analyses to study
central incisors are malpositioned posteriorly. the relationship of tooth size to the size of supporting
Example: linguoversion of anteriors, class II structures.
division 2 malocclusion l Ashley Howe considered that tooth crowding is due
vi. The AAAL is measured in the mandibular arch in a to deficiency in arch width rather than arch length.
similar manner, whereas the arch width at the pre- l He found that a relationship exists between the total
molar region is taken from contact areas of first and width of 12 teeth anterior to the second molars and
second premolars. the width of dental arch in the 1st premolar region.
vii. According to Korkhau’s formula, for a given width l Parameters considered are as follows:
of upper incisors, a specific value of distance should i. Determination of total tooth material (TTM)
exist between the midpoint of interpremolar line to ii. Determination of premolar diameter (PMD)
the point between two maxillary incisors. iii. Determination of premolar basal arch width
An increase in this measurement denotes proclined (PMBA)
upper anterior teeth, while a decrease in this value
denotes retroclined upper anterior teeth.
(i) Total tooth material (TTM)
Q.3. Carey’s analysis. The mesiodistal width of all teeth mesial to second
permanent molars is measured and summed up. This
Ans.
value is called the total tooth material (Fig. 13.6).
The main cause of most of malocclusions is the arch length and
tooth material discrepancy. This discrepancy can be calculated 1 1
2 2
with the help of Carey’s analysis. Carey’s analysis is usually
done in the lower arch (cast). The same analysis if performed 3 3
on the upper arch (cast), it is known as arch perimeter analysis.
Method: 4 L1
C1 C2
L2 4
i. Determination of arch length C C
ii. Determination of tooth material 5 1st PM 1st PM 5
iii. Determination of discrepancy
2nd PM 2nd PM
i. Determination of arch length 6 6
It is carried with a soft brass wire. Arch length is
measured from mesial surface of the first permanent 1st M 1st M
molar of one side to the first permanent molar of the
Fig. 13.6  Total tooth material.
opposite side. If the anterior teeth are well aligned,
the brass wire passes over the incisal edges of ante-
riors; if they are retroclined, the brass wire in ante- (ii) Determination of premolar diameter (PMD)
rior segment passes labial to the teeth; in the case of The premolar diameter is the arch width measured from
proclined anteriors, the wire is passed along the the tip of buccal cusp of the first premolar to the oppo-
cingulum of anterior teeth. site first premolar (Fig. 13.7).
ii. Determination of tooth material
The tooth material is measured by summing up the
mesiodistal width of individual teeth anterior to the
first molars, i.e. second premolar to second premolar.
iii. Determination of discrepancy
The discrepancy refers to the difference between the
arch length and the tooth material.
Inference:
If the discrepancy is
0–2.5 mm – It indicates minimum tooth material excess,
and suggests proximal stripping to reduce tooth material.
2.5–5 mm – It indicates the need to extract second pre-
molar.
.5 mm – It indicates the need to extract first premolars.
Q.4. Ashley Howe’s Index.
Fig. 13.7  Premolar diameter.
Ans.
124 Quick Review Series for BDS 4th Year, vol 1

(iii) Determination of premolar basal arch width 5 after incisor alignment is determined by measur-
(PMBA) ing distance between distal surface of
The measurement of width from canine fossa of one 2 and
side to other gives the width of dental arch at the apical mesial surface of 6.
base. If canine fossa is not clear, then measurement is (iv) Based on mesiodistal width of 21 21 , the ex-
made from a point about 8 mm below the crest of the
pected width of 3, 4 and 5 is predicted by referring
interdental papilla distal to canine.
to the probability chart; 75% level of probability is
considered reliable.
{SN Q.14} (v) To determine discrepancy, the predicted tooth size
of 3, 4, 5 is compared with the arch length avail-
Inference:
able. If predicted value is greater than the arch
PMBAW and PMD are compared.
length, then crowding is expected.
If PMBAW . PMD – It is an indication that arch
expansion is possible. Q.6. Radiographic method of mixed dentition analysis.
If PMAW , PMD – The arch expansion is not possible.
Ans.
According to Ashley Howe, the ratio between the
apical base width at the premolar region and the total (i) The radiographic method of mixed dental analysis
tooth material is called the premolar basal arch width makes use of both radiograph and study cast to deter-
percentage. mine the width of unerupted tooth to compensate for
the enlargement of radiographic image.
PMBAW 100
PMBAW% (ii) A simple proportionality relationship can be set up to
determine the measurement of unerupted teeth by
Inference is as follows: studying the teeth that have already erupted in a radio-
l If PMBAW% is 37% or less – It indicates need for graph and on cast by the following formula:
extraction.
True width o f tooth that has erupted and
l If PMBAW% is 44% or more – Case can be
measured on caast width of unerupted
treated without extracting any teeth. Apparent width
tooth on radiographh
l If PMBAW% is 37%–44% – The case is a border- of unerupted
line case. tooth Width of tooth that has erupted and
measu red on the radiograph

Q.5. Mixed dentition analysis. Explain one in detail. (iii) Undistorted radiographic image is usually achieved
with individual periapical radiographs. Accuracy of
Ans. this method is fair to good, depending on the quality of
radiographs.
{SN Q.5} (iv) This technique can be used in maxillary and mandibu-
lar arches for all ethnic groups.
Various mixed dentition analyses to study the relation-
ships of tooth size and available space during mixed Q.7. Bolton’s tooth size ratio.
dentition period are as follows: Ans.
(a) Moyer’s mixed dentition analysis
(b) Tanaka–Johnston analysis {SN Q.9}
(c) Staley and Kerber analysis
(d) Radiographic method l Bolton’s analysis evaluates maxillary and mandibu-
lar teeth for tooth size discrepancies.
l According to Bolton, a ratio exists between mesio-
Moyer’s mixed dentition analysis distal widths of maxillary and mandibular teeth.
(i) The purpose of Moyer’s analysis is to evaluate the Abnormalities in tooth size are responsible for the
amount of space available in the arch for succeed- occurrence of many malocclusions.
ing permanent canines and premolars.
(ii) Moyer’s mixed dentition analysis predicts the com-
bined mesiodistal width of 3, 4 and 5 based on the l Bolton’s analysis helps in determining disproportion in
sum of widths of four lower permanent incisors, size between maxillary and mandibular teeth.
l The parameters considered are as follows:
i.e. 21 21 .
(i) Sum of mandibular 12 teeth – measured and summed
(iii) The mesio distal width of 21 21 is measured and up value of mesiodistal widths of all teeth mesial to
added. The amount of space available for 3, 4 and 7 7.
Section | I  Topic-Wise Solved Questions of Previous Years 125

(ii) Sum of maxillary 12 teeth – measured and summed Q.8. Kesling tooth positioning.
up value of mesiodistal widths of all teeth mesial to
Ans.
7 7.
(iii) Sum of mandibular 6 teeth – measured and summed
up value of mesiodistal widths of all teeth mesial to {SN Q.13}
4 4.
l H.D. Kesling (1956) proposed a diagnostic set-up
(iv) Sum of maxillary 6 teeth – measured and summed
which helps clinician to estimate arch length discrep-
up value of mesiodistal widths of all teeth mesial to
ancy.
4 4.
l Procedure:

i. Patients maxillary and mandibular study casts


revealing supporting structures to the depth of
{SN Q.9} sulcus are prepared with their bases parallel to
Determination of overall ratio: occlusal plane.
ii. Using a fretsaw blade, horizontal cuts are made
Sum of mandibular 12 100
Overall ratio in mandibular cast 3 mm below the gingival mar-
Suu m of maxillary 12 gin, and vertical cuts are made between individ-
According to Bolton, sum of mesiodistal width of man- ual teeth so that we are able to remove all teeth
dibular teeth anterior to 7 7 should be 91.3% of me- except second and third molars.
siodistal width of maxillary teeth mesial to 7 7. iii. Mesial and distal ends of the roots of teeth are
trimmed to facilitate seating in a new position.
If overall ratio is
The mandibular incisors are arranged at 65o
angle to the Frankfort horizontal plane, while
canines and premolars are placed in correct con-
tact relationship.
<91.3% >91.3% indicates
Maxillary tooth material excess Mandibular teeth material excess
iv. If the remaining space is inadequate to receive the
1st molars, the extractions are indicated so first
premolars are eliminated from the set-up and 2nd
Calculated by formula Calculated by formula premolars are placed in contact with canine.
Mandibular 12 × 100 Maxillary 12 × 91.3 v. Maxillary teeth are cut and repositioned, articu-
Maxillary 12 – Mandibular 12 –
91.3 100 lating with mandibular set-up.

Determination of anterior ratio:


Uses:
Sum of mandibular 6 1000 l The tooth size–arch length discrepancies can be
Anterior ratio
Sum of maxillary 6 directly visualized on the diagnostic set-up.
According to Bolton, sum of mesiodistal width of man- l Serves as a guide in including extractions and visual-

dibular anteriors should be 77.2% of mesiodistal width izing complex orthodontic tooth movements on the
of maxillary anteriors. study casts.
l It also helps in estimating whether only uprighting of
If anteriors ratio is 2nd molars could solve the problem of arch length
discrepancy.
l Patients can be motivated by simulating tooth move-

<77.2% >77.2% ments on the study casts.


Indicates maxillary Indicates mandibular
anterior excess anterior excess Q.9. Pont’s Index.
Ans.
Calculated as Calculated as [Same as SE Q.1]
Sum of mandibular Sum of maxillary Q.10. Carey’s analysis or arch perimeter analysis
Sum of 6 × 100 Sum of 6 × 77.2
maxillary 6 mandibular 6
77.2 100 Ans.
[Same as SE Q.3]
126 Quick Review Series for BDS 4th Year, vol 1

Q.11. Ashley Howe’s index. l The same analysis if performed on the upper arch (cast),
it is known as arch perimeter analysis.
Ans.
l It involves determination of arch length, tooth material

[Same as SE Q.4] and discrepancy.


l Arch length is measured from mesial surface of the first
Q.12. Assessment of tooth mass discrepancy. permanent molar of one side to the first permanent
Ans. molar of the opposite side.
l The tooth material is measured by summing up the me-
[Same as SE Q.4] siodistal width of individual teeth anterior to the first
Q.13. Moyer’s mixed dentition analysis. molars, i.e. second premolar to second premolar.

Ans. The discrepancy refers to the difference between arch


length and tooth material.
[Same as SE Q.5] Inference:
Q.14. Bolton’s analysis. If the discrepancy is
0–2.5 mm – It indicates minimum tooth material excess
Ans. and suggests proximal stripping to reduce tooth material.
2.5–5 mm – It indicates need to extract the second premolar.
[Same as SE Q.7]
.5 mm – It indicates need to extract first premolars.
Q.15. Kesling’s diagnostic set-up.
Q.4. Peck and Peck ratio.
Ans.
Ans.
[Same as SE Q.8] i. Chief rationale of Peck concept is stability of rotational
corrections of lower incisors rather than tooth size
SHORT NOTES: considerations.
ii. Calculated as:
Q.1. Pont’s analysis.
Mesiodistal width
Ans. Peck and peck ratio 100
Facioolingual diameter
[Ref SE Q.1]
iii. Normal ratio for central incisors is 88%–92%, and for
Q.2. Model analysis. lateral incisors the ratio is 90%–95%.
iv. Peck and Peck ratio is used to determine whether lower
Ans.
incisor teeth are excessively wider mesiodistally, if so,
l Evaluation of maxillary and mandibular teeth and their
then proximal slicing is recommended.
supporting structures using study casts is known as
model analysis. Q.5. Name few model analysis for mixed dentition?
l Model analyses can be classified as follows:
Ans.
i. Analyses to study the size relationships of groups of
teeth [Ref SE Q.5]
Examples: Bolton’s analysis, Peck and Peck ratio
Q.6. Linderhearth’s ratio.
ii. Analyses to study the relationship of teeth size to
the size of supporting structures Ans.
Examples: Ashley Howe’s analysis, Pont’s analysis
l Linderhearth’s ratio is a method of predetermining the
iii. Analyses to study mixed dentition
ideal arch width based on mesiodistal width of crowns
Examples: Moyer’s analysis, Tanaka–Johnston
of maxillary incisors similar to Pont’s Index.
analysis
l According to Linderhearth, the ratio of combined inci-
iv. Analyses to study the relationship of tooth size and
sor to transverse arch width as measured from the centre
available space in permanent dentition
of the occlusal surface of teeth is ideally 0.85 in the first
Examples: Carey’s analysis, Arch perimeter analysis
premolar area and 0.65 in the first molar area.
Q.3. Carey’s analysis. l The sum of mesiodistal widths of maxillary incisors is

measured and added (SI).


Ans.
l The calculated premolar value is determined by the
l The arch length and tooth material discrepancy can be formula: SI/85 3 100.
calculated with the help of Carey’s analysis. Carey’s l The calculated molar value is determined by the for-
analysis is usually done in the lower arch (cast). mula: SI/64 3 100.
Section | I  Topic-Wise Solved Questions of Previous Years 127

l Inference is that if the measured value is less than the v. Advantages: It is simple and practical, requires
calculated value, it indicates the need for expansion. neither radiographs nor reference tables, and
shows reasonably good accuracy.
Q.7. Radiographic method of mixed dentition analysis.
Ans. Q.11. Korkhau’s analysis?

(i) The radiographic method of mixed dental analysis Ans.


makes use of both radiograph and study cast. i. Korkhau in 1938 proposed a study model analysis
(ii) A simple proportionality relationship can be set up to which reveals anteroposterior malpositioning of inci-
determine the measurement of unerupted teeth by sors in maxillary and mandibular arches.
studying the teeth that have already erupted in a radio- ii. A measurement is made from the midpoint of interpre-
graph and on cast by the following formula: molar line to a point between the two maxillary incisors.
iii. According to Korkhau, for a given width of upper inci-
True width o f tooth that has erupted and
sors, a specific value of distance between the midpoint
measured on caast width of unerupted
Apparent width of interpremolar line and the point between two maxil-
tooth on radiographh
of unerupted lary incisors should exist.
tooth Width of tooth that has erupted and iv. An increase in this measurement denotes proclined up-
measu red on the radiograph per anterior teeth, while a decrease in this value denotes
(iii) Accuracy of this method is fair to good, depending on retroclined upper anterior teeth.
the quality of radiographs. Q.12. Gnathostatic models?
(iv) This technique can be used in maxillary and mandibu-
lar arches for all ethnic groups. Ans.

Q.8. Study models – uses. i. Gnathostatic models or gnathostatic casts reproduce


inclination of the occlusal plane with reference to the
Ans. Frankfort plane.
[Ref LE Q.1] ii. Paul Simon developed an instrument called gnathome-
ter (1928–1934).
Q.9. Bolton’s analysis. Gnathostatics is a diagnostic medium relating teeth and
Ans. their base to each other and to the craniofacial structures.
iii. Simon tried to orient and relate dentition and jaws with
[Ref SE Q.7] the help of dental study models to cranium. His effort
Q.10. Tanaka–Johnston analysis. was to give orthodontist a real insight into the orienta-
tion of dentition to facial skeleton in three planes of
Ans.
space, thereby helping to modulate treatment plan in
l Tanaka–Johnston analysis is a mixed dentition analysis. the direction of restoration of facial balance.
l It predicts the widths of unerupted canines and premo-
Q.13. Kesling’s diagnostic set-up.
lars based on the sum of widths of lower incisors.
Method: Ans.
i. Measure the total arch length.
[Ref SE Q.8]
ii. Measure the mesiodistal widths of lower four
incisors and sum it up. Q.14. Ashley Howe’s index.
iii. Divide the value obtained by 2 and
Ans.
l add 10.5 mm to obtain the sum of width of

mandibular canines and premolars in one [Ref SE Q.4]


quadrant.
Q.15. Pont’s index.
l add 11 mm to obtain the sum of widths of max-

illary canines and premolars in one quadrant. Ans.


iv. Formula to calculate the space available in the
[Same as SN Q.1]
arch after the eruption of canines and premolars
is as follows: Q.16. Arch perimeter analysis.
Space available 5 Total arch length – (Sum of
Ans.
the lower incisors 1 2 3 calculated width of
canine and premolar). [Same as SN Q.3]
128 Quick Review Series for BDS 4th Year, vol 1

Q.17. Bolton’s tooth size ratio? Q.18. Bolton index.


Ans. Ans.
[Same as SN Q.9] [Same as SN Q.9]

Topic 14
Biology and Mechanics of Tooth Movement
COMMONLY ASKED QUESTIONS
LONG ESSAYS:
1. Define optimal orthodontic force. Discuss tissue changes subsequent to light and heavy forces.
2. What are the theories of tooth movement? What factors affect the tooth movement?
3. What are the different types of tooth movement?
4. Discuss the biochemical principles involved in orthodontic tooth movement and add a note on undermining resorption.
5. Describe the tissue changes subsequent to orthodontic force application. [Same as LE Q.1]
6. Discuss the histological changes during orthodontic tooth movement. [Same as LE Q.1]
7. Describe the various histological tissue changes during active orthodontic treatment. [Same as LE Q.1]
8. What are different theories of tooth movement? Discuss the pressure–tension theory in detail. [Same as LE Q.2]
9. Describe the blood flow theory of tooth movement. [Same as LE Q.2]
10. What are the various theories that are involved in the biology of orthodontic tooth movement? Discuss in detail.
[Same as LE Q.2]

SHORT ESSAYS:
1 . Explain frontal resorption.
2. Response of bone and periodontium to orthodontic force at tension zone.
3. Enumerate the various types of tooth movements.
4. Undermining resorption.
5. Enumerate various phases of tooth movements.
6. What is bodily tooth movement? [Same as SE Q.3]

SHORT NOTES:
1. Ideal orthodontic force.
2. Name theories of tooth movement.
3. Types of orthodontic force.
4. Piezoelectric theory.
5. Undermining resorption. [Same as SE Q.4]
6. Explain frontal resorption. [Ref SE Q.1]
7. Interrupted force.
8. Physiologic tooth movement.
9. Centre of resistance.
10. Root resorption.
11. Blood flow theory to explain tooth movement.
12. Intrusion.
13. Bodily movement.
14. Define force.
Section | I  Topic-Wise Solved Questions of Previous Years 129

1 5. Name various types of tooth movements. [Ref SE Q.3]


16. Force and couple.
17. Write three advantages of optimum orthodontic force.
18. Optimum orthodontic force. [Same as SN Q.1]

SOLVED ANSWERS
LONG ESSAYS:
Q.1. Define optimal orthodontic force. Discuss tissue l Marked h in vascularity of periodontal
changes subsequent to light and heavy forces. ligament due to h capillary blood supply –
mobilization of cells fibroblasts and osteoclasts.
Ans.
l Osteoclasts lie in shallow depressions of bone
l Optimum orthodontic force is the one which moves called Howship’s lacunae – They start resorb-
teeth most rapidly in the desired direction with least ing bone.
possible damage to tissue and with minimum patient l When the forces applied are within physiologi-
discomfort. cal limits, the resorption is seen in the alveolar
l Optimum orthodontic force 5 capillary pulse pressure, plate immediately adjacent to the ligament. This
i.e. 20–26 gm/cm2 of root surface area. kind of resorption is called frontal resorption.
l Change in orientation of bony trabeculae is

Characteristics of optimum orthodontic force from seen several weeks after continued orthodontic
force application.
l The trabeculae are usually parallel to long

axis of teeth and they become horizontally


a. Clinical point of view b. Histological point of view
oriented, i.e. parallel to direction of orth-
i. Produces rapid tooth i. Vitality of tooth and
odontic force.
movement supporting PL is
l The trabecular pattern reverts to normal during
ii. Minimum patient maintained
discomfort ii. Initiates maximum retention phase of treatment.
iii. Minimum lag phase cellular response Changes on tension side (area of tooth opposite to
of tooth movement iii. Produced direct frontal direction of force):
iv. No marked mobility resorption l Periodontal ligament gets stretched – Distance
of teeth being moved between alveolar bone and tooth is widened.
l h Vascularity is seen (just as on pressure side) –

Mobilization of cells, fibroblasts, osteoblasts.


Tissue changes subsequent to orthodontic force application:
l In response to this traction, osteoid is laid
When force is applied on a tooth to bring about orth-
down by osteoblasts in periodontal ligament
odontic movement, it results in formation of:
immediately adjacent to the lamina dura. This
i. Areas of pressure (in the direction of tooth movement) –
lightly calcified bone in due course matures to
Bone subjected to pressure reacts by bone resorption.
form woven bone.
ii. Areas of tension (in the opposite direction) – Bone
Secondary remodelling changes:
subjected to tension exhibits bone deposition.
l When the force is applied to move teeth, the
iii. When tooth is moved due to application of orthodontic
bone immediately adjacent shows osteoclastic
force, there is bone resorption on pressure side and new
and osteoblastic activities on the pressure and
bone formation on tension side.
tension sides, respectively.
iv. Histological changes during tooth movements are stud-
l In addition, bony changes also take place else
ied under two headings:
where to maintain the width or thickness of the
Changes following application of: alveolar bone.
i. Mild force These changes are called secondary remodel-
ii. Extreme force ling changes, e.g. if a tooth is being moved in
i. Changes following application of mild force are as labial direction, there is compensatory depo-
follows: sition of new bone on the outer side of labial
Changes on pressure side (tooth movement side): alveolar bony plate and a compensatory re-
l Periodontal ligament compressed to one-third sorption on the lingual side of lingual alveolar
of its original thickness. bone.
130 Quick Review Series for BDS 4th Year, vol 1

During almost all the forms of orthodontic tooth


+ –
– movement, hyalinization of periodontal ligament
+
– + on the pressure side occurs in some areas. These
+ – +
– –
– areas are wider when force is applied in extreme.
+
+
– – Changes observed during formation of hyalinized
+ –
+ – zones are as follows:
Tooth moved in
+

– i. There is a gradual shrinkage of periodontal
labial direction ligament fibres.
ii. Cellular structures become indistinct; some
nuclei become smaller (pycnotic), while some
l Secondary remodelling changes seen following nuclei disappear.
application of bodily force in labial direction. iii. Compressed collagenous fitness gradually
l These compensatory structural alterations unites into a more or less cell-free mass.
maintain the thickness of supporting alveolar iv. Certain changes occur in ground substance.
process even though tooth may be moved over v. There is a breakdown of blood vessel walls
a distance several times greater than thickness leading to spilling of their contents.
of alveolar bony plates. vi. Osteoclasts formed in marrow spaces and
ii. Changes following application of extreme forces: adjacent areas of inner bone surface after a
l When extreme forces are applied to teeth – Crushing period of 20–30 h.
or total compression of periodontal ligament occurs. Presence of hyalinized zone:
l On pressure side – Root closely approximated l Periodontal ligament nonfunctioning and
lamina dura, compresses periodontal ligament bone resorption cannot occur.
and leads to occlusion of blood vessels. l Tooth is not capable of further movement
l Ligament is deprived of its nutritional supply, until local damaged tissue is removed and
leading to regressive changes called hyalinization. adjacent alveolar bone wall resorbs.
l In this case, bone cannot resorb in the frontal por- Elimination of hyalinized zone:
tion adjacent to the teeth, rather bone resorption l The two mechanisms involved are:
occurs in adjacent marrow spaces and in the al- i. Resorption of bone by osteoclasts dif-
veolar plate below, behind and above the hyalin- ferentiating in peripheral intact peri-
ized zones. This kind of resorption is called as odontal ligament and in adjacent mar-
undermining resorption or rearward resorption. row spaces.
l On tension side – Periodontal ligament is over- ii. Invasion of cells and blood vessels
stretched, leading to tearing of Blood vessels (BVs) from the periphery of compressed zone
and ischaemia. by which necrotic tissue is removed by
l When extreme force is applied during orthodontic enzymatic action and phagocytosis.
tooth movement, there is a net h in osteoclastic l Greater the force – wider the areas of hya-
activity as compared to bone formation, with the linization – large areas of periodontal liga-
result that the tooth becomes loosened in its socket. ment become functionless, thereby showing
l Pain and hyperemia of the gingiva may occur. large areas of rearward resorption.
l Hyalinization: It is a form of tissue degeneration l Light forces – hyalinized zones are smaller –
characterized by formation of a clear, eosino- large areas of functional ligament available
philic, homogenous substance. and frontal resorption predominates in case of
Conventional process of hyalinization is an irre- lighter forces.
versible process whereas hyalinization of peri- Nature of tooth movement and location of hyalinized
odontal ligament is a reversible process. tissue:
Section | I  Topic-Wise Solved Questions of Previous Years 131

Tipping tooth movement – hyalinization close to decreased O2 level in compressed area as compared
alveolar crest to tension side.
Bodily tooth movement – hyalinization close to l The formation of these aneurysms and vascular ste-

mid-portion of root nosis causes blood gases to escape into the interstitial
Excessive forces applied during tipping – two fluid, thereby creating a favourable local environ-
areas of hyalinization: one in apical and another ment for resorption.
in marginal area Bone bending and piezoelectric theories of tooth move-
Areas of bony prominences and spicules usually ment: Farrar (1876)
result in areas of hyalinization. l Farrar (1876) first noted deformation or bending of

interseptal alveolar walls.


Q.2. What are the theories of tooth movement? What l He was first to suggest that bone bending may be a pos-
factors affect the tooth movement? sible mechanism for bringing about tooth movement.
l Peizoelectricity is a phenomenon observed in many
Ans.
crystalline materials in which a deformation of crys-
Certain theories have been put forward to explain mecha- tal structure produces a flow of electric current be-
nism of movement of a tooth by an orthodontic force. cause of displacement of electrons from one part of
Accepted theories are as follows: the crystal lattice to the other. A small electric current
i. Pressure tension theory – Schwarz (1932) is generated when bone is mechanically deformed.
ii. Fluid dynamic theory/blood flow theory – Bein l The possible sources of electric current are:
iii. Bone bending piezoelectric theory – Farrar (1876) a. Collagen
Pressure tension theory: b. Hydroxyapatite
l According to Schwarz – Whenever tooth is subjected c. Collagen hydroxyapatite interface (the junction
to orthodontic force, it results in areas of pressure between the collagen and hydroxypatite crystals
and tension. when bent can be a source of piezoelectricity).
l The area of periodontium in direction of tooth move- d. Mucopolysaccharide fraction of ground substance
ment n is under pressure n shows bone resorption. is not crystalline but it may also possess the abil-
l The area of periodontium opposite to tooth move- ity to generate electric current when deformed.
ment n is under tension n shows bone deposition. l When crystal structure is deformed, the electrons

Fluid dynamic theory: migrate from one location to another n resulting in


l This theory is also called blood flow theory as pro- electric charge.
posed by Bein. l When force is released, the crystals return to their

l According to this theory n tooth movement occurs original shape and a reverse flow of electrons is
as a result of alterations in fluid dynamics in peri- observed.
odontal ligament. l Two unusual characteristics:

l Periodontal ligament – occupies periodontal space i. Quick decay rate – This piezoelectric signal
between tooth and alveolar socket. quickly dies away to zero even though the force is
l Periodontal space n consists of fluid system n made maintained.
of interstitial fluid, cellular elements and blood ves- ii. When the force is released, electron flow in the
sels viscous ground substance, in addition to peri- opposite direction is seen.
odontal fibres. l On application of force on a tooth, the adjacent

l When force of greater magnitude and direction is alveolar bone bears:


applied during orthodontic tooth movement n the Areas of concavity in bone associated with nega-
interstitial fluid in periodontal ligament squeezes out tive charge n evoke bone deposition.
and moves towards apex and cervical margins and Areas of concavity associated with positive charge n
results in decreased tooth movement n called evoke bone resorption.
Squeeze Film Effect by Bien. On application of force n alveolar and medullary
l When orthodontic force is applied, it results in n cortical plates of bone move together closely n
compression of periodontal ligament. bone becomes less concave n electrical signal
l Blood vessels of periodontal ligament are trapped be- associated with resorption is established
tween principal fibres n results in their ‘stenosis’ n l The bone which is deformed by stress becomes

The vessel above stenosis then balloons, resulting in electrically charged. Concave surfaces attain nega-
formation of ‘aneurysm’ (which are minute walled tive polarity and convex surfaces a positive polarity.
sacs of fluid). l As a result of these electrical signals, a remodel-

l Bien suggested that there is alteration in the chemical ling response is evoked; bone is added to concave
environment at the site of vascular stenosis due to surfaces and resorbed from convex surfaces.
132 Quick Review Series for BDS 4th Year, vol 1

Q.3. What are the different types of tooth movement? a. Bodily movement
l This is the most desirable type of tooth move-
Ans.
ment.
Different types of orthodontic tooth movements are as l In bodily movement, crown and root move to
follows: the same distance in the same direction, either
i. Tipping lingually or labially.
ii. Pure translation l The force applied is 70–120 g and the centre of
iii. Root movement rotation is at infinity.
iv. Rotation l Uniform stress pattern is seen in the periodon-

tal ligament.
i. Tipping
b. Intrusion
l Tipping is the simplest type of tooth movement that
l It is defined as the axial movement of the tooth
can easily be carried out with application of a single
along the long axis towards the apex of the
force to the crown.
root.
l It is of two types:
l This tooth movement requires minimum force,
a. Uncontrolled tipping
and the centre of rotation passes through the
b. Controlled tipping
centre of resistance.
a. Uncontrolled tipping l 10–20 g of force is required and the periodon-
l Uncontrolled tipping is produced when a sin- tal ligament at the apex is compressed over a
gle force is applied to the crown of a tooth, small area and no areas of tension exist.
where the crown moves in one direction and c. Extrusion
the root moves in opposite direction. l Extrusion is defined as the axial movement of
l In uncontrolled tipping, the centre of rotation the tooth along the long axis towards the coro-
is in-between the centre of resistance and apex nal part.
of the root. l 35–60 g of the force is required.
l Force required: 35–60 g. l No areas of compression in periodontal liga-
l The distribution of load is such that the peri- ment, only stretched areas are seen.
odontal ligament is stressed near the apex on iii. Root movement
the same side as the applied force and at the l This is the opposite of crown tipping, and crown of a
crest of the alveolar bone on the opposite side. tooth is kept stationary, while the root moves labio-
l It is useful when incisors have to be proclined. lingually or mesiodistally.
b. Controlled tipping l Root movement is mainly used to torque the incisor
l This is a desirable tooth movement as com- and upright the tipped teeth.
pared to uncontrolled tipping. l There are two types of root movements:
l Centre of rotation is at the root apex and crown a. Torque
moves in one direction and there is minimal or b. Uprighting
no movement of the root in opposite direction. a. Torque
l Force required is the same as that of uncon- l Labiolingual root movement is known as
trolled tipping in the range of 35–60 g. torque.
l Minimum stress of the periodontal ligament at b. Uprighting:
root apex. This prevents root movement. l This is nothing but mesiodistal root movement
l It is useful in retraction of excessively with centre of rotation at incisal edge.
proclined incisors when roots are normally l 50–100 g of force is required.
positioned. l The stress is greatest at the apex and decreases
ii. Pure translation gradually to the cervical level.
l In translation, crown and root move in the same di- iv. Rotation:
rection to the same distance. l Spinning of the tooth around its long axis.
l When two forces are applied simultaneously to the Or
crown of the tooth, the applied force passes through the A displacement of the body, produced by a couple,
centre of resistance, and translation of a tooth occurs. characterized by the centre of rotation coinciding
l Pure translation is of three types: with the centre of resistance is known as rotation.
a. Bodily movement l Pure rotations can be divided into two types:
b. Intrusion a. Transverse rotation: The tooth displacement dur-
c. Extrusion ing which the long-axis orientation changes is
Section | I  Topic-Wise Solved Questions of Previous Years 133

known as transverse rotation, e.g. tipping and l The formation of second messengers inside the cells
torquing. is believed to initiate formation of bone cells, namely
b. Long-axis rotation: In this type of tooth dis- osteoclasts and osteoblasts, which are responsible for
placement, the angulation of the long axis is not bone remodelling.
altered, e.g. rotation of a tooth around its long l It takes nearly 4 h of sustained pressure to produce

axis. second messengers; hence any appliance has to be


Generalized rotation: worn for a minimum period of 4–6 h to produce ef-
l Any movement that is not pure translation or rota- fects.
tion can be described as a combination of both Third messengers:
translation and rotation and can be termed as gen- l Within the cells, the cAMP and Ca
11
act on the pro-
eralized rotation. This type of movement can be tein kinase enzymes, which are the third messengers.
seen during routine clinical practice. l Protein kinase causes phosphorylation of the cells.

l Phosphorylation results in differentiation and activa-


Q.4. Discuss the biochemical principles involved in orth-
tion of osteoclasts and osteoblast, which ultimately
odontic tooth movement, and add a note on undermin-
produce bone remodelling.
ing resorption.
As the remodelling of bony socket starts, the tooth
Ans. movement begins.
l When orthodontic force is applied onto a tooth, it results Q.5. Describe the tissue changes subsequent to orth-
in a number of biophysical events such as compression odontic force application.
of periodontal ligament, bone deformation and tissue
Ans.
injury.
l Decreased vascularity and overstretching of periodontal [Same as LE Q.1]
ligament induces chemical changes and inflammatory
Q.6. Discuss the histological changes during orthodontic
type of response is elicited.
tooth movement.
l The biophysical events in turn lead to certain biochemi-

cal reactions at a cellular level which brings about the Ans.


release of some extracellular signalling molecules called
[Same as LE Q.1]
first messengers.
l They include hormones such as parathormone (PTH), Q.7. Describe the various histological tissue changes
local chemical mediators, such as prostaglandins, and during active orthodontic treatment.
neurotransmitters such as substance P and vasoactive
Ans.
intestinal polypeptide (VIP).
First messengers: [Same as LE Q.1]
l Prostaglandin becomes the first messengers. Prosta-
Q.8. What are different theories of tooth movement?
glandin E plays a major role in the cellular differen-
Discuss the pressure–tension theory in detail.
tiation.
l Other first messengers are PTH, substance P, vasoac- Ans.
tive peptides.
[Same as LE Q.2]
l They bind to the cell surface receptors and activate

the extracellular signals. Q.9. Describe the blood flow theory of tooth movement.
Second messengers:
Ans.
l Conversion of extracellular signal into an intracellu-

lar signal is the next step in cellular differentiation. [Same as LE Q.2]


l The first messengers bind to receptors present on the
Q.10. What are the various theories that are involved in the
cell surface of target cells and initiate a process of
biology of orthodontic tooth movement? Discuss in detail.
intracellular signalling.
l The conversion of extracellular into intracellular Ans.
signal takes place by two pathways.
[Same as LE Q.2]
i. Conversion of ATP into cyclic AMP.
ii. Opening of calcium ion channel and activate
Ca11. SHORT ESSAYS:
l The intracellular signalling results in formation of
Q.1. Explain frontal resorption.
second messengers, which include cAMP, cyclic
GMP and calcium. Ans.
134 Quick Review Series for BDS 4th Year, vol 1

l After some time, osteoid is laid on the whole of the


{SN Q.6}
alveolar wall on the tension side.
l Frontal resorption is a type of tissue change at pres- l Osteoblasts synthesize the osteoid. Subsequently, min-

sure zone in orthodontic tooth movement following eralization of osteoid takes place.
application of light force. l Rate of bone deposition is about 30 microns/day.

l Frontal resorption is also called periosteal resorption


Q.3. Enumerate the various types of tooth movements.
or direct resorption or forward resorption.
Ans.

Changes on pressure side (tooth movement side) are as


follows: {SN Q.15}
l Periodontal ligament compressed to one-third of its Various types of orthodontic tooth movements are as
original thickness. follows:
l Marked h in vascularity of periodontal ligament due i. Tipping:
to h capillary blood supply – mobilization of cells a. Controlled tipping
fibroblasts and osteoclasts. b. Uncontrolled tipping
l Osteoclasts lie in shallow depressions of bone called ii. Pure translation:
Howship’s lacunae – they start resorbing bone. a. Bodily movement
b. Intrusion
{SN Q.6} c. Extrusion
iii. Root movement
l When the forces applied are within physiological a. Torque
limits, the resorption is seen in the alveolar plate b. Uprighting
immediately adjacent to the ligament. This kind of iv. Rotation
resorption is called frontal resorption.

Bodily movement:
l In frontal resorption, the resorption process is initi-
l Bodily movement is a type of pure translation.
ated from the periodontal ligament side of the alveo-
l In translation, crown and root move in the same
lar bone.
direction to the same distance.
l When two forces are applied, simultaneously to the

{SN Q.6} crown of the tooth, the applied force passes through
the centre of resistance and translation of a tooth
l Frontal resorption usually takes place after two days occurs.
following orthodontic force application. l This is the most desirable type of tooth movement.

l In bodily movement, crown and root move to the

Q.2. Response of bone and periodontium to orthodontic same distance in the same direction, either lingually
force at tension zone. or labially.
l The force applied is 70–120 g and the centre of
Ans. rotation is at infinity.
The response of bone and periodontium at tension zone on l Uniform stress pattern is seen in the periodontal

application of orthodontic force is as follows: ligament.


l As compared to pressure zone, the cellular activity is
Q.4. Undermining resorption.
delayed in areas of tension.
It takes around 30 h for increased cellular activity to be Ans.
seen in tension zone.
l The stretched periodontal fibres are reconstructed by

changes of the original fibrils. {SN Q.5}


l In the areas of tension, macrophages are found in great
l Undermining resorption is also known as indirect
numbers and there is inflammatory type change like
resorption.
breakdown and rebuilding of fibrous elements.
l The term ‘undermining resorption’ was coined by
l Around the parts of the fibres that are close to the alveo-
Sandstedt.
lar wall, new unmineralized matrix is laid down.
Section | I  Topic-Wise Solved Questions of Previous Years 135

l When light forces are applied – areas of hyalin-


l This is a type of tissue change at pressure zone in
ization are small and frontal resorption occurs –
orthodontic tooth movement following application of
lesser will be the duration of lag phase.
heavy force.
l Heavy forces – areas of hyalinization are large –
l When extreme forces are applied to teeth – crushing
rearward resorption occurs – longer will be
or total compression of periodontal ligament occurs.
the duration of lag phase to eliminate hyalinized
l On pressure side, root closely approximates lamina
tissue.
dura, compresses periodontal ligament and leads to
c. Postlag phase:
occlusion of blood vessels.
l Tooth movement progresses rapidly as the hyalin-
l Ligament is deprived of its nutritional supply, lead-
ized zone is removed and bone resorption occurs
ing to regressive changes called hyalinization.
directly facing periodontal ligament.
l Once hyalinization occurs in periodontal ligament,
l Periodontal ligament is widened.
frontal resorption is not possible.
l In this case, bone cannot resorb in the frontal portion Q.6. What is bodily tooth movement?
adjacent to the teeth; rather bone resorption occurs in
Ans.
adjacent marrow spaces and in the alveolar plate be-
low, behind and above the hyalinized zones. This [Same as SE Q.3]
kind of resorption is called undermining resorption
or rearward resorption.
l When extreme force is applied during orthodontic
SHORT NOTES:
tooth movement, there is a net h in osteoclastic activ- Q.1. Ideal orthodontic force.
ity as compared to bone formation, with the result
Ans.
that the tooth becomes loosened in its socket.
l Ideal or optimum orthodontic force is the one which
l This method of resorption is called undermining
moves teeth most rapidly in the desired direction with
resorption because the attack is from the underside of
least possible damage to tissue and with minimum
lamina dura.
patient discomfort.
l Optimum orthodontic force 5 capillary pulse pressure,
Q.5. Enumerate various phases of tooth movements. i.e. 20–26 g/cm2 of root surface area.
Ans. l From clinical point of view, it produces rapid tooth

movement with minimum patient discomfort and mini-


l Burstone categorized the stages or phases of tooth mum lag phase and no marked mobility of teeth being
movement as follows: moved.
l Three phases of tooth movement are
l From histological point of view, vitality of tooth and
a. Initial phase supporting periodontal ligament is maintained and it
b. Lag phase produces direct frontal resorption.
c. Postlag phase
Q.2. Name theories of tooth movement.
a. Initial phase:
l Very rapid tooth movement occurs over a short Ans.
distance and then stops.
l Certain theories have been put forward to explain
l This movement represents displacement of tooth
mechanism of movement of a tooth by an orthodontic
in periodontal ligament space and probably bend-
force.
ing of alveolar bone to certain extent.
l Accepted theories are as follows:
l Both light and heavy forces displace tooth to the
i. Pressure tension theory – Schwarz (1932)
same extent. In this phase, tooth movement is
ii. Fluid dynamic theory/blood flow theory – Bein
between 0.4 and 0.9 mm in a week’s time.
iii. Bone bending piezoelectric theory – Farrar (1876)
b. Lag phase:
l This phase represents the period of hyalinization, Q.3. Types of orthodontic force.
characterized by formation of hyalinized tissue in
Ans.
periodontal ligament.
l During this phase, little or no tooth movement l Based on the duration and decay rate, orthodontic force
occurs. is classified by Proffit as follows:
l Duration of lag phase depends upon the amount a. Continuous force
of force used to move the tooth, usually extends b. Interrupted force
up to 14 days. c. Intermittent force
136 Quick Review Series for BDS 4th Year, vol 1

a. Continuous force: Q.8. Physiologic tooth movement.


l In this type, between the two successive visits of
Ans.
the patient, force is maintained at some apprecia-
ble fraction of the original force. l Physiologic tooth movement designates primarily the
l The force level does not decline to zero. slight tipping of the functioning tooth in its socket, and
l For the continuous force to be effective, it has to secondarily, the changes in tooth position that occur in
be a light continuous force. young persons during and after tooth eruption.
b. Interrupted force: l It is of the following types:

l In this type of force, the force level reduces to i. Movement during mastication
zero between the two successive visits of the ii. Eruption of tooth
patient. iii. Tooth migration
l Both light and heavy interrupted forces are clini-
i. Movement during mastication
cally acceptable, e.g. fixed appliance. During chewing, the teeth tip slightly around the
c. Intermittent force: neutral axis as fulcrum and are also displaced be-
In this type of force, there is a sudden drop of force to cause of bending of the alveolar process.
zero level when the orthodontic appliance is removed Movement during mastication is transient.
by the patient. ii. Eruption of tooth
Intermittent force acts as an impulse or a shock for Different teeth move in different directions during
short periods with a series of interruptions. eruption.
Example: functional appliances iii. Migration of teeth
Q.4. Piezoelectric theory. Migration of teeth is a slow tooth movement, usu-
ally in mesial and occlusal directions.
Ans. These movements take place to compensate for
l Bone bending and piezoelectric theories of tooth interproximal attrition and occlusal wear.
movement – Farrar (1876) Q.9. Centre of resistance.
l Farrar was the first to suggest that bone bending may

be a possible mechanism for bringing about tooth Ans.


movement. l Centre of resistance is defined as the point in the object
l Peizoelecticity is a phenomenon observed in many at which the resistance to movement is at the maximum.
crystalline materials in which a deformation of the l The centre of resistance of tooth is variable, it
crystal structure produces a flow of electric current as depends on:
a result of displacement of electrons from one part of i. Root morphology
the crystal lattice to the other. A small electric current ii. Number of roots
is generated when bone is mechanically deformed. iii. Level of alveolar bone support
l Both bone and collagen have piezoelectric property. iv. Root length
l Application of force to the centre of resistance of the
Q.5. Undermining resorption.
tooth produces true bodily movement.
Ans.
Q.10. Root resorption.
[Same as SE Q.4]
Ans.
Q.6. Explain frontal resorption.
l Root resorption is more evident when heavy orthodontic
Ans. forces are applied.
l Excessive force and prolonged duration of treatment
[Ref SE Q.1]
increase the chances of resorption.
Q.7. Interrupted force. l Types of resorption:

Generalized resorption
Ans.
Localized resorption
l Interrupted force is a type of orthodontic force. l Most of the teeth exhibit some loss of root after orth-
l In this type of force, the force level reduces to zero odontic treatment. There is generalized shortening of
between the two successive visits of the patient. root seen in majority of orthodontic patients.
l Both light and heavy interrupted forces are clinically l Orthodontic treatment causes severe localized resorp-
acceptable, e.g. fixed appliance. tion. Maxillary incisors are more prone to resorption.
Section | I  Topic-Wise Solved Questions of Previous Years 137

l One of the most important causes of root resorption in l The force applied is 70–120 g and the centre of rotation
anteriors and molars is pressing of the roots against the is at infinity.
cortical plate. l Uniform stress pattern is seen in the periodontal liga-

l Root resorption index: ment.


Grade I: irregular root contour
Q.14. Define force.
Grade II: root resorption , 2 mm at the apex
Grade III: root resorption 2 mm to one-third of root Ans.
length
l Force is a load or external influence applied to a body
Grade IV: root resorption . one-third of root length
that changes or tends to change the position of that
Q.11. Blood flow theory to explain tooth movement. body.
l It is measured in grams or ounces.
Ans.
l Types of force are as follows:

l The fluid dynamic theory is also called the blood flow i. Compression
theory as proposed by Bein. ii. Tension
l According to this theory, tooth movement occurs iii. Shear force
because of alterations in fluid dynamics in periodontal
Q.15. Name various types of tooth movements.
ligament.
l When orthodontic force is applied, it results in the com- Ans.
pression of periodontal ligament.
[Ref SE Q.3]
l Blood vessels of periodontal ligament are trapped be-

tween principal fibres, resulting in their ‘stenosis’. The Q.16. Force and couple.
vessel above stenosis then balloons, resulting in forma-
Ans.
tion of ‘aneurysm’.
l Bien suggested that there is alteration in the chemical l Force is a load or external influence applied to a body
environment at the site of vascular stenosis due to de- that changes or tends to change the position of that
creased O2 level in compressed area as compared to body.
tension side. l Being a vector, force has a definite magnitude, a specific

l The formation of these aneurysms and vascular stenosis direction and a point of application.
causes blood gases to escape into the interstitial fluid, l Couple is a pair of concentrated forces having equal

thereby creating a favourable local environment for magnitude and opposite direction with parallel but non-
resorption. collinear line of action.
l A couple when acting upon a body brings about pure
Q.12. Intrusion.
rotation.
Ans.
Q.17. Write three advantages of optimum orthodontic
l Intrusion is defined as the axial movement of the tooth force.
along the long axis towards the apex of the root.
Ans.
l This tooth movement requires minimum force and cen-

tre of rotation passes through the centre of resistance. The advantages of optimum orthodontic force are as
l 10–20 g of force is required and the periodontal liga- follows:
ment at the apex is compressed over a small area and no l Efficient tooth movement is possible.

areas of tension exist. l Resorption is mainly of the frontal type.

l Elimination of lag phase and hyalinized zone.


Q.13. Bodily movement.
l Less amount of pain and no damage to the supporting

Ans. structures.
l Chances for root resorption are minimal.
l Bodily movement is a pure translation movement.
l This is the most desirable type of tooth movement. Q.18. Optimum orthodontic force.
l In bodily movement, crown and root move to the
Ans.
same distance in the same direction, either lingually or
labially. [Same as SN Q.1]
138 Quick Review Series for BDS 4th Year, vol 1

Topic 15
Anchorage
COMMONLY ASKED QUESTIONS
LONG ESSAYS:
1 . Define anchorage. Explain in detail different types of anchorage with examples.
2. Define reinforced anchorage. Discuss method of reinforcing anchorage.
3. Define and discuss the various anchorage situations in removable and fixed appliances.
4. Define anchorage. Classify and explain orthodontic anchorage with examples. [Same as LE Q.1]
5. Define anchorage. Discuss classification of anchorage. Explain intermaxillary anchorage. [Same as LE Q.1]
6. Define orthodontic anchorage. Classify them. Discuss the uses of extraoral anchorage in orthodontics. [Same as LE Q.1]
7. Classify anchorage. Explain when, why and how would you like to reinforce it. [Same as LE Q.2]

SHORT ESSAYS:
1 . Define anchorage. Write Nanda’s classification of anchorage.
2. Extraoral anchorage.
3. Reinforced anchorage.
4. Intermaxillary anchorage.
5. Reciprocal anchorage.

SHORT NOTES:
1. Define anchorage.
2. Extraoral anchorage. [Ref SE Q.2]
3. Reciprocal anchorage. [Ref SE Q.5]
4. Intermaxillary anchorage.
5. Stationary anchorage. [Ref LE Q.1]
6. Simple anchorage. [Ref LE Q.1]
7. Define anchorage. Classify it with respect to number of teeth used. [Ref LE Q.1]
8. Define reciprocal anchorage. Give examples of reciprocal anchorage.
9. Define anchorage. Enumerate methods of reinforcing anchorage.
10. Cortical anchorage. [Ref LE Q.1]
11. Factors affecting anchorage.
12. Anchorage loss.
13. Anchorage in orthodontics. [Same as SN Q.1]
14. Define anchorage in orthodontics. Give White and Gardiner’s classification. [Same as SN Q.1]
15. Extraoral sources of anchorage. [Same as SN Q.2]
16. Baker’s anchorage. [Same as SN Q.4]
17. What is reinforced anchorage? Give two examples. [Same as SN Q.9]

SOLVED ANSWERS
LONG ESSAYS:
Q.1. Define anchorage. Explain in detail different types
{SN Q.7}
of anchorage with examples.
l Graber defined anchorage as ‘the nature and degree of
Ans.
resistance to displacement offered by an anatomic unit
when used for the purpose of effecting tooth movement’.
l Proffit defined anchorage as resistance to unwanted

tooth movement.
Section | I  Topic-Wise Solved Questions of Previous Years 139

Classification of anchorage:
A. Moyer has classified anchorage in the following ways:

I. According to manner of force application

a. Simple anchorage b. Stationary anchorage c. Reciprocal anchorage

II. According to jaws involved

a. Intramaxillary b. Intermaxillary
(Anchorage from the same jaw) (Anchorage from both jaws)

III. According to site of anchorage

(a) Intraoral (b) Extraoral (c) Muscular

(i) Intramaxillary (ii) Intermaxillary ↓

• Simple i. Cervical
• Stationary ii. Cranial
• Reciprocal iii. Occipital
iv. Facial

VI. According to the number of anchorage units

a. Single (or) primary anchorage b. Compound anchorage c. Reinforced anchorage


(Anchorage involving one tooth)

Various types of anchorage are discussed in detail below:


{SN Q.5}
I. According to manner of force application
a. Simple anchorage b. Stationary anchorage
b. Stationary anchorage l Stationary anchorage is defined as ‘the dental an-

c. Reciprocal anchorage chorage in which the manner and application of


force tend to displace the anchorage unit bodily’.
l Simply, the resistance to bodily movement is
{SN Q.6}
called the stationary anchorage.
a. Simple anchorage l An anchor tooth or source which does not move
l It is the ‘Dental anchorage in which manner and against the forces of teeth to be pulled is the sta-
application of force is such that it tends to change tionary anchorage.
the axial inclination of tooth or teeth that form the
anchorage unit’.
l The simple anchorage is nothing but resistance of Examples:
the anchorage unit to tipping. i. Retraction of maxillary incisors using molars
l The combined root surface area of the teeth form- as anchor teeth.
ing the anchorage unit must be doubled that of the ii. In real sense, only extraoral source of anchor-
teeth to be moved. age derived from headgears would be the best
l Simple anchorage has a low resistance value. example of stationary anchorage.
140 Quick Review Series for BDS 4th Year, vol 1

c. Reciprocal anchorage l Various sources of intraoral anchorage are


l The reciprocal anchorage is said to exist when as follows:
two teeth or two sets of teeth move to an equal i. Teeth
extent in an opposite direction. ii. Palate
l In reciprocal anchorage, the force applied for iii. Lingual alveolar bone of the mandible
tooth movement is dissipated to both active and l When all the anchorage units and the teeth to

reactive components and the desired tooth move- be moved are situated within the same jaw, it
ment occurs by the movement of both units. is described as intramaxillary anchorage.
l Teeth may need to be pulled against each other l When the anchorage units situated in one

to close the spaces. jaw are used to bring about tooth movement
Examples: in the opposing jaw, the anchorage is called
i. Closure of midline diastema intermaxillary anchorage.
ii. Use of crossbite elastics to correct single- Example: correction of class II and III malocclusions
tooth crossbite and class II malocclusion using intermaxillary elastics
with intermaxillary elastics b. Extraoral anchorage
iii. Dental arch expansion l Extraoral anchorage is an anchorage situation

II. According to jaws involved wherein the anchorage units are situated outside
a. Intramaxillary (anchorage from the same jaw) the oral cavity.
b. Intermaxillary (anchorage from both jaws) l Various types of extra anchorage are as follows:

i. Cervical
a. Intramaxillary anchorage
ii. Cranial
l When all the anchorage units as well as the teeth to
iii. Occipital
be moved are situated within the same jaw, the an-
iv. Facial
chorage is described as intramaxillary anchorage.
l It is also used as a form of reinforced
l Here, the appliances are placed in only one jaw,
anchorage.
either maxilla or mandible.
l Extraoral anchorage is usually used to cor-
l It may be simple, stationary or reciprocal type of
rect skeletal problems.
resistance.
l Various extraoral sources of anchorage are
Example: elastic chains used to retract the ante-
headgear, face mask, chin cup, etc.
rior segment using posterior teeth as anchorage
l Various sites of extraoral anchorage with
units
examples:
b. Intermaxillary anchorage
Occipital region: chin cap and high pull
l It is also known as ‘Baker’s anchorage’.
headgear
l When the anchorage units situated in one jaw are
Parietal: combination of headgear
used to bring about tooth movement in the oppos-
Forehead: reverse pull headgear
ing jaw, the anchorage is called intermaxillary
Back of neck: cervical headgear
anchorage.
Chin: reverse pull headgear and chin cap
Examples:
c. Muscular anchorage
l Class II elastics worn from mandibular molars
l Muscular forces can be used for anchorage pur-
to maxillary anteriors are used to retract maxil-
pose.
lary anteriors.
l Muscular forces when redirected to a favourable
l Class III elastics worn from maxillary molars
action on the teeth serve as a source of anchorage.
to mandibular anteriors are used to retract
Example: vestibular shield and lip bumper
mandibular anteriors.
A lip bumper transmits the force of hyperactive
III. According to the site of anchorage
lower lip to molars, aiding in its uprighting.
a. Intraoral
b. Extraoral
c. Muscular
{SN Q.7}
a. Intraoral anchorage
Intraoral anchorage is classified as follows: I V. According to number of anchorage units
i. Intramaxillary a. Single (or) primary anchorage (anchorage in-
ii. Intermaxillary volving one tooth)
l Intraoral anchorage is an anchorage in
b. Compound anchorage (anchorage involving mul-
which all the anchorage units are situated tiple teeth)
inside the oral cavity. c. Reinforced anchorage
Section | I  Topic-Wise Solved Questions of Previous Years 141

a. Single or primary anchorage l Onplant is a disc-like structure which can be


l In primary anchorage, the resistance provided by placed in hard palate on the posterior aspect under
a single tooth with greater alveolar support is used local anaesthesia. Orthosystem implant is a screw-
to move, the tooth with less alveolar support. type endosteal implant of about 4–6 mm in length.
l Example: Moving a tooth with smaller root sur- Aarhus implant is very small in size and early
face area against a tooth with large root surface loading is possible, hence are used in multiple
area, and is called an anchor. sites between roots. Mini implants are very small,
b. Compound anchorage 1.2 3 6 mm in dimension.
l In this, the resistance provided by more than one l Uses of implants:
tooth with greater support is used to move teeth i. Implants serve as a source of absolute an-
with lesser support. chorage
c. Reinforced anchorage Example: retromolar implant anchorage for
l It is also known as multiple anchorage. closing of edentulous space at first molar ex-
l The anchorage where more than one resistance traction sites.
unit is utilized is called reinforced anchorage. ii. They are used for anchorage and as abut-
l When more units are added, resistance units be- ments for restorations.
come more effective because reactionary force is iii. Implant is also used in distraction osteo-
distributed over a large area. genesis.
l By distribution of force over a large area and
Q.2. Define reinforced anchorage. Discuss method of
keeping the force light, trauma and pain during
reinforcing anchorage.
treatment are minimized.
Examples: Ans.
i. Use of transpalatal arch, translingual arch and
l Reinforced anchorage is also known as multiple anchorage.
Nance space holding buttons, reinforces the
l The anchorage where more than one resistance unit is
anchorage unit.
utilized is called reinforced anchorage.
ii. Usage of headgears.
l Reinforced anchorage refers to the augmentation of
iii. In cases with upper anterior bite plane, use of
anchorage by various means like extraoral appliances,
labial bow to prevent flaring of upper incisors
upper anterior inclined plane or a transpalatal arch.
is another example of reinforced anchorage.
l Extraoral forces, i.e. forces generated from extraoral
iv. Sved-type bite plates.
areas, such as cranium, back of the neck and face, can
V. Other types of anchorage
be used to reinforce anchorage.
l When more units are added, resistance units become

{SN Q.10} more effective because reactionary force is distributed


over a large area.
a. Cortical bone or cortical anchorage l By distribution of force over a large area and keeping
l Cortical bone offers more resistant to resorption the force light, trauma and pain during treatment are
than medullary bone. minimized.
l The response of cortical bone when compared to l Multiple anchorage is the augmentation of anchorage
medullary bone is different. If the roots are by various methods like the following:
torqued lingually or buccally, the resistance to Fixed appliances, rigid labial bow, Sved bite planes, in-
movement is increased, this principle is being termaxillary anchorage and use of extraoral anchorage
used by Ricketts and is called cortical anchorage. Examples:
Example: Space closure in old extraction site is dif- (i) Use of transpalatal arch, translingual arch and
ficult as the roots encounter cortical bone along the Nance space holding buttons, reinforces the an-
residual ridge. chorage unit.
a. Transpalatal arch: This is a wire that spans the
palate in transverse direction, connecting the
b. Implants as anchorage permanent upper first molars on either side
l Recently, implants are being used as anchorage with an omega loop in the midline. It is used in
units. They have been designed exclusively for fixed mechano-therapy to augment anchorage.
orthodontic purpose. It is effective as an anchorage maintenance
l Various orthodontic implants are onplant, or- device and active orthodontic appliance.
thosystem implant, Aarhus implant and mini- b. Translingual arch: It is used in mandible, and
implants. functions as anchorage device. It is usually
142 Quick Review Series for BDS 4th Year, vol 1

made up of 0.036 stainless steel wire ex- A. Anchorage sources for removable appliance
tending along lingual contour of mandibular Removable appliances derive their anchorage from oral
dentition from first molar on one side to the tissues and teeth in the following manner:
opposite side of jaw. i. Tissue born anchorage:
c. Nance palatal arch: l The acrylic base plate, which is the major compo-

i. It is used in upper arch as an anchorage nent of removable appliances, derives anchorage


device during levelling and alignment, in from tissue contact against palate and lingual sur-
molar distalization cases and as a space face of mandible.
maintainer. l The acrylic, which extends into the interdental

ii. Tissue and tooth-borne anchorage such spaces and embrasures, also secures the plate
as palatal removable appliances with in situ.
clasps of molar bands. The base plate transmits the force all over the denti-
iii. Upper anterior inclined plane: tion as well as to the underlying hard and soft tissues.
A removable appliance incorporating an l Modifications to base plate, such as bite planes,

upper anterior inclined plane results in also reinforce anchorage by transmitting muscular
forward glide of mandible during the clo- forces to the jaws.
sure of jaw. This results in the stretching l The removable appliance also derives anchorage

of retractor muscles of mandible, which from clasps, pinheads which securely fit on the
subsequently contracts and forces the teeth. These provide retention and help in the dis-
mandible against the upper inclined plane. tribution of force.
Thus, a distal force is applied on the B. Anchorage sources for fixed appliances
maxillary teeth, thereby reinforcing l The major sources of anchorage in the fixed appli-
maxillary anchorage. ances are the teeth themselves.
(ii) Usage of headgears to augment the resistance l Fixed appliances entail heavy burden on anchor
unit. teeth. Anchor units need to be supported either with
(iii) In cases with upper anterior bite plane, use of incorporation of more teeth or support from cranium
rigid labial bow to prevent flaring of upper inci- using headgears or modification of biomechanics,
sors is another example of reinforced anchorage. so-called anchorage savers.
(iv) Similarly, instead of a labial bow, the acrylic
plate is constructed in such a way that it covers Q.4. Define anchorage. Classify and explain orthodontic
the labial incisal aspect of maxillary incisors anchorage with examples.
which also prevents the labial flaring of maxil- Ans.
lary incisors. This type of reinforced anchorage
is called Sved-type bite plates. [Same as LE Q.1]
(v) The anchorage may be reinforced in the case of Q.5. Define anchorage. Discuss classification of anchor-
fixed appliances by designing the appliance so age. Explain intermaxillary anchorage.
that only bodily movement of the anchorage
teeth can occur. Ans.
Example: Passing a bow wire through the hori-
[Same as LE Q.1]
zontal tube on bands attached to adjacent teeth.
Q.6. Define orthodontic anchorage. Classify them. Dis-
Q.3. Define and discuss the various anchorage situations
cuss the uses of extraoral anchorage in orthodontics.
in removable and fixed appliances.
Ans. [Same as LE Q.1]

l Graber defined anchorage as ‘the nature and degree of Q.7. Classify anchorage. Explain when, why and how
resistance to displacement offered by an anatomic unit would you like to reinforce it.
when used for the purpose of effecting tooth movement’. Ans.
l Proffit defined anchorage as resistance to unwanted

tooth movement. [Same as LE Q.2]


Section | I  Topic-Wise Solved Questions of Previous Years 143

SHORT ESSAYS: Q.3. Reinforced anchorage.


Q.1. Define anchorage. Write Nanda’s classification of Ans.
anchorage. l Reinforced anchorage is also called multiple anchorage.
Ans. l This is an anchorage situation where more than one
l Graber defined anchorage as ‘the nature and degree of resistance unit is employed.
resistance to displacement offered by an anatomic unit l Resistance units become more effective when more

when used for the purpose of effecting tooth move- units are added because the reactionary force is distrib-
ment’. uted over a larger area.
l Proffit defined anchorage as resistance to unwanted l Distribution of force over a large area means keeping

tooth movement. the force light which minimizes trauma and pain during
Nanda’s classification of anchorage treatment.
Depending on how much anchorage unit contributes to Examples:
extraction space closure, Nanda classified the anchorage i. Use of transpalatal arch, Nance space holding but-
into three categories: Group A, B and C. tons, lingual arch reinforces the anchorage unit.
i. Group A space closure: ii. Usage of headgears to augment the resistance unit.
l 100%–75% space closure from anterior retrac- iii. Use of labial bow to prevent flaring of upper inci-
tion and 25% closure from posterior anchorage sors in cases where upper anterior bite plane is used.
movement. It is another example of reinforced anchorage.
l There is critical posterior anchorage. iv. Sved-type bite plates: Instead of a labial bow, the
ii. Group B space closure: acrylic plate is constructed in such a way that it cov-
l It includes more equal amount of anterior and ers the labial incisal aspect of maxillary incisors.
posterior tooth movement for space closure. This again prevents the labial flaring of maxillary
l It is an easy condition to deal with. incisors. This type of reinforced anchorage is called
iii. Group C space closure: Sved-type bite plates.
l Includes 75%–100% posterior protraction. There
Q.4. Intermaxillary anchorage.
is a noncritical anterior anchorage and critical
posterior anchorage. Ans.
l Anterior retraction is only 25%.
l It is also known as ‘Baker’s anchorage’.
l This is a noncritical type of anchorage.
l When the anchorage units situated in one jaw are used
Q.2. Extraoral anchorage. to bring about tooth movement in the opposing jaw, the
anchorage is called intermaxillary anchorage.
Ans. Examples:
l Baker’s anchorage is a form of intermaxillary an-

{SN Q.2} chorage to adjust the jaw relationship and teeth by


using elastics from maxilla to mandible.
l Extraoral anchorage is an anchorage situation
l Class II elastics worn from mandibular molars to max-
wherein the anchorage units are situated outside the
illary anteriors are used to retract maxillary anteriors.
oral cavity.
l Class III elastics worn from maxillary molars to
l It is also used as a form of reinforced anchorage.
mandibular anteriors are used to retract mandibular
l Extraoral anchorage is usually used to correct skele-
anteriors.
tal problems.
l Extraoral anchorage is of following types: Q.5. Reciprocal anchorage.
i. Cervical
Ans.
ii. Cranial
iii. Occipital
iv. Facial {SN Q.3}
l Various extraoral sources of anchorage are headgear,

facemask, chin cup, etc. l The reciprocal anchorage is said to exist when two
l Various sites of extraoral anchorage with examples:
teeth or two sets of teeth move to an equal extent in
Occipital region: chin cap and high pull headgear opposite direction.
l In reciprocal anchorage, the force applied for tooth
Parietal: combination headgear
Forehead: reverse pull headgear movement is dissipated to both active and reactive
Back of neck: cervical headgear components and the desired tooth movement occurs
Chin: reverse pull headgear and chin cap by the movement of both units.
144 Quick Review Series for BDS 4th Year, vol 1

Q.5. Stationary anchorage.


l Teeth may need to be pulled against each other to
close spaces. Ans.
Examples:
[Ref LE Q.1]
i. Closure of midline diastema: The midline dia-
stema in the upper arch can be closed by tying Q.6. Simple anchorage.
them with tight elastic thread.
Ans.
ii. Use of bite elastics to correct single-tooth cross-
bite. [Ref LE Q.1]
iii. Correction of class II malocclusion with inter-
Q.7. Define anchorage. Classify it with respect to num-
maxillary elastics.
ber of teeth used.
iv. Dental arch expansion.
Ans.
SHORT NOTES: [Ref LE Q.1]
Q.1. Define anchorage. Q.8. Define reciprocal anchorage. Give examples of
reciprocal anchorage.
Ans.
Ans.
l Graber defined anchorage as ‘the nature and degree of
resistance to displacement offered by an anatomic i. In reciprocal anchorage, the force applied for tooth
unit when used for the purpose of effecting tooth movement is dissipated to both active and reactive com-
movement’. ponents.
l Proffit defined anchorage as resistance to unwanted ii. The dissipation of equal and opposite force tends to
tooth movement. move both the units towards each other.
l White and Gardener’s classification of anchorage iii. The desired tooth movement occurs by the movement of
i. Simple both the units.
ii. Stationary Examples:
iii. Reciprocal l Closure of median diastema by moving both central
iv. Reinforced incisors towards each other.
v. Intermaxillary l Corrections of class II malocclusion with inter-
vi. Extraoral maxillary elastics.
l Corrections of single-tooth crossbite through bite
Q.2. Extraoral anchorage.
elastic.
Ans.
Q.9. Define anchorage. Enumerate methods of reinforc-
[Ref SE Q.2] ing anchorage.
Q.3. Reciprocal anchorage. Ans.
Ans. l The anchorage where more than one resistance unit is
utilized is called reinforced anchorage.
[Ref SE Q.5]
l Reinforced anchorage refers to the augmentation of
Q.4. Intermaxillary anchorage. anchorage by various means like extraoral appliances,
upper anterior inclined plane or a transpalatal arch.
Ans.
Examples:
i. Intermaxillary anchorage is an anchorage in which an- i. Use of transpalatal arch, translingual arch and
chorage unit situated in one jaw is used to bring about Nance space holding buttons reinforces the
tooth movement in the opposite jaw. anchorage unit.
ii. Baker’s anchorage is a form of intermaxillary anchor- ii. Usage of headgears.
age to adjust jaw relationship and teeth by using elastics iii. In cases with upper anterior bite plane, use of
from maxilla to mandible. labial bow to prevent flaring of the upper incisors
Example: correction of class II and III malocclusion by is another example of reinforced anchorage.
using intermaxillary elastics iv. Sved-type bite planes.
Section | I  Topic-Wise Solved Questions of Previous Years 145

Q.10. Cortical anchorage. l Improper treatment planning and anchorage prep-


aration
Ans.
l Resistance between arch wires and brackets

[Ref LE Q.1] iii. Methods to prevent anchorage loss are as follows:


l Use of anchorage savers like transpalatal arches,
Q.11. Factors affecting anchorage.
lingual arches etc.
Ans. l Use of optimum and differential force

l Utilizing muscular forces


Factors affecting anchorage can be considered under
l Reinforcement of anchorage
the following two headings:
A. Biological factors Q.13. Anchorage in orthodontics.
B. Mechanical factors
Ans.
. Biological factors include:
A
[Same as SN Q.1]
i. Teeth
ii. Size of the anchoring unit Q.14. Define anchorage in orthodontics. Give White and
iii. Axial inclination of teeth Gardiner’s classification.
iv. Use of optimum force
Ans.
v. Differential force system
vi. Persistant habits [Same as SN Q.1]
vii. Anchorage savers
Q.15. Extraoral sources of anchorage.
B. Mechanical factors include:
i. Friction Ans.
ii. Type of tooth movement planned
[Same as SN Q.2]
iii. Technique employed in retraction mechanics
Q.16. Baker’s anchorage.
Q.12. Anchorage loss.
Ans.
Ans.
[Same as SN Q.4]
i. The undesirable movement of anchor tooth in excess to
that of planned treatment is known as anchorage loss. Q.17. What is reinforced anchorage? Give two examples.
ii. Reasons for anchorage loss are as follows:
Ans.
l Excessive force

[Same as SN Q.9]

Topic 16
Age Factors in Orthodontics
COMMONLY ASKED QUESTIONS
SHORT ESSAYS:
1 . Age factors in orthodontics.
2. Difference between adult and adolescent patients. [Same as SE Q.1]

SHORT NOTES:
1 . List Age factors in orthodontics.
2. Ideal age for various orthodontic treatments.
3. Adult orthodontics.
146 Quick Review Series for BDS 4th Year, vol 1

SOLVED ANSWERS
SHORT ESSAYS:
Q.1. Age factors in orthodontics.
Ans.

Differences in orthodontic treatment for young and adults patients are as follows:

Factors Young patients Adult patients


1. Growth to Orthodontist has growth to work with, i.e. using Due to lack of growth, orthodontist merely relies on
work with growth potential of the patient, most orthodontic and tooth movement or surgery
orthopaedic treatments can be efficiently carried out
2. Diagnosis Routine diagnostic aids can be used Routine diagnostic aids can be used and some dormant
pathologies like impactions, periodontal problems,
decay and loss of teeth can hamper the success of
orthodontic treatment
3. Appliance These patients can benefit from orthopaedic and In these patients the options are restricted to orthodon-
selection myofunctional appliances that help to modulate tic tooth movement and surgery
growth in case of growth abnormalities
4. Periodontal Less common More common
problems
5. Patient motivation Not well motivated and do not cooperate well Well motivated and cooperate well
and cooperation
6. Tissue vitality Tissue vitality and responsiveness to force is much Due to decreased cellularity and vascularity, the tissue
greater in child vitality and responsive to force is not so much in
adults
7. Treatment Achieved well The orthodontist has to strike the best possible balance
objectives between various treatment objectives like function,
stability and aesthetics

8. Treatment Less in young patients More in adult patients


appreciation

Q.2. Difference between adult and adolescent patients. Q.2. Ideal age for various orthodontic treatments.
Ans. Ans.
[Same as SE Q.1] i. The age of the patient influences orthodontic therapeu-
tic interventions and prognosis.
ii. The effectiveness of functional appliances, like twin
SHORT NOTES: block, bionator, Frankel appliance given for class II
Q.1. List age factors in orthodontics? skeletal correction and orthopaedic appliances, like
headgears, to correct maxillary prognathism, are effec-
Ans.
tive during growing stage of the patient.
The list of various age factors considered in orthodontics iii. Maxillary expansion procedures are carried out before
are as follows: the fusion of palatal sutures during early adolescence.
i. Utilization of growth potential to work with iv. Orthognathic surgeries are best undertaken in adult
ii. Various diagnostic methods used patients after growth cessation.
iii. Selection of appliance
Q.3. Adult orthodontics.
iv. Periodontal problems
v. Patient motivation and cooperation Ans.
vi. Tissue vitality
i. Orthodontic treatment of adults is known as adult ortho-
vii. Treatment objectives
dontics.
viii. Treatment appreciation
Section | I  Topic-Wise Solved Questions of Previous Years 147

ii. Orthodontic treatment for adults is broadly classified forced eruption, crossbite correction and diastema
into two types: closure.
a. Adjunctive orthodontic treatment iv. Comprehensive orthodontic treatment is an essential
b. Comprehensive orthodontic treatment treatment procedure carried out in children for correc-
iii. Adjunctive orthodontic treatment procedures are car- tion of malocclusion. Response to orthodontic force is
ried out to facilitate other dental procedures to control slightly slower in adults as compared to children.
disease and restore function, e.g. uprighting of molars,

Topic 17
Preventive Orthodontics
COMMONLY ASKED QUESTIONS
LONG ESSAYS:
1 . Define preventive orthodontics. Discuss the various treatment plans given under the preventive orthodontics.
2 . Define preventive orthodontics and describe in brief space maintainers and their classification with
examples.
3. What are the various preventive orthodontic procedures? Explain in detail the various space maintainers.
[Same as LE Q.2]
4 . Describe in brief space maintainers and their classification with examples. [Same as LE Q.2]
5 . What are space maintainers? Describe the various types used in orthodontics and their indications.
[Same as LE Q.2]

SHORT ESSAYS:
1 . Procedures under preventive orthodontia. [Ref LE Q.1]
2 . Define space maintainer. Mention its indications and contraindications.
3 . Features of an ideal space maintainer.
4 . Preventive orthodontics.
5 . Describe the rationale or the principle of preventive orthodontic practice. [Same as SE Q.4]

SHORT NOTES:
1. Define preventive orthodontics. [Ref LE Q.1]
2. Define space maintainer. [Ref LE Q.1]
3. Fixed space maintainer.
4. Oral hygiene measures during orthodontic treatment.
5. Distal shoe space maintainer.
6. What are the requirements of space maintainers? [Same as SE Q.3]
7. Nance appliance.
8. Enumerate preventive orthodontic procedures. [Same as SN Q.1]
9. Procedures under preventive orthodontics. [Same as SN Q.1]
1 0. Classification of space maintainers. [Same as SN Q.2]
1 1. Advantages of fixed space maintainer. Give an example. [Same as SN Q.3]
1 2. Willet’s appliance. [Same as SN Q.5]
148 Quick Review Series for BDS 4th Year, vol 1

SOLVED ANSWERS
LONG ESSAYS:
Q.1. Define preventive orthodontics. Discuss the various II. Oral hygiene
treatment plans given under the preventive orthodontics. Parents and children should be taught appropri-
ate oral hygiene measures as applicable to that
Ans. age group as given below:
l Infants (younger than 1 year): Brushing
(SE Q.1 and SN Q.1) should start with the eruption of first pri-
mary teeth. Parents should do the gentle
l {(Graber has defined preventive orthodontics as the massage of gums and cleaning of the teeth.
action taken to preserve the integrity of what appears l Toddlers (children aged 1–3 years):
to be a normal occlusion at a specific time. Toothbrush should be introduced along
l Proffit and Ackerman have defined it as prevention of with some nonfluoridated pastes. Parents
potential interference with occlusal development. should do the brushing for children.
l Procedures undertaken in preventive orthodontics are l Preschoolers (children aged 3–6 years):
as follows: Children should start brushing under pa-
A. Preventive procedures without use of appliances rental supervision and fluoride toothpaste
I. Predental procedures and parental education can be introduced.
II. Maintenance of oral hygiene l School aged (children aged 6–12 years):
III. Caries control and restoration of decayed teeth Child should be taught proper brushing
IV. Care of deciduous dentition technique and regular brushing habits.
V. Management of tooth ankylosis/locked III. Caries control and restoration of decayed teeth
permanent first molar. l Unrestored caries/undercontoured restoration
VI. Extraction of supernumerary teeth/retained of proximal surface of deciduous teeth lead to
deciduous tooth loss of arch length, so to prevent reduction of
VII. Maintenance of quadrant-wise tooth shed- arch length, proper restoration of affected
ding timetable and space maintenance teeth should be undertaken immediately.
VIII. Management of abnormal frenal attachments l All possible caries prevention methods are to
and check up for deleterious oral habits be followed like proper tooth brushing and
IX. Prevention of damage to occlusion/occlu- fluoride prophylaxis at regular intervals.
sal equilibration)} IV. Care of deciduous dentition
l To prevent premature loss of deciduous teeth,

[SE Q.1] simple preventive measures like application


of topical fluoride and pit and fissure sealants
{B. Preventive procedures with use of appliances should be undertaken.
I. Mouth protectors V. Management of tooth ankylosis/locked perma-
II. Space maintenance nent first molar
A. Preventive procedures without use of appliances are l Ankylosed deciduous teeth prevent eruption
described in detail below: of permanent teeth or deflect them to erupt in
I. Predental procedures/parental education abnormal positions; hence, they should be
(a) All these preventive procedures ideally diagnosed and removed surgically at an ap-
should begin before the birth of the child and propriate time.
are instituted before the eruption of teeth. l Sometimes, permanent first molars are
(b) The expecting mothers should be educated on deeply locked by a prominent distal bulge on
the following matters: second deciduous molar which prevents their
l Nutrition
eruption. It should be assessed and the slic-
l Proper nursing and care of the child
ing of distal surface of E E should be
l Use of physiologic nipples 6 6
l Detrimental effect of prolonged use of
undertaken to guide the eruption of
pacifiers on dentition VI. Extraction of supernumerary teeth/retained
l Correct method of brushing child’s teeth deciduous tooth
Overall, the parents should be educated on need l Supernumerary teeth should be identified

for maintaining good oral hygiene of the child. and extracted before they cause displacement
Section | I  Topic-Wise Solved Questions of Previous Years 149

of other teeth and interfere with normal erup- bruxism should be detected, and selective
tion pattern and normal occlusion. grinding should be carried out to attain oc-
VII. Maintenance of quadrant-wise tooth shedding clusal equilibration.
timetable and space maintenance (c) Pseudo-class III and crossbites caused due
(a) Maintenance of tooth shedding timetable is to functional shifts should be checked and
important as premature loss of deciduous eliminated.
teeth may cause drifting of the adjacent (d) Occlusal equilibration is performed as pre-
teeth into the space, which can result in ventive, interceptive and corrective orth-
abnormal axial inclination of teeth, spacing odontic procedure.}
between teeth and shift in the midline.
(b) There should not be more than 3 months’ Q.2. Define preventive orthodontics and describe in
gap between shedding of deciduous teeth brief space maintainer’s and their classification with
and eruption of permanent teeth as com- examples.
pared with one quadrant to other quadrants.
Ans.
(c) Space maintenance is a procedure to pre-
vent loss of arch development due to pre- Graber has defined preventive orthodontics as the action
mature loss of deciduous teeth. taken to preserve the integrity of what appears to be a nor-
(d) Space maintainer is an appliance or a device mal occlusion at a specific time.
that prevents loss of arch length and guides Proffit and Ackerman have defined it as prevention of
the permanent teeth into correct position in potential interference with occlusal development. The best
dental arch. time to initiate preventive orthodontics is ideally during
VIII. Management of abnormal frenal attachments prenatal counselling.
and check up for deleterious oral habits Procedures undertaken in preventive orthodontics are
(a) The presence of abnormally thick maxillary as follows:
labial frenum produces midline diastema. A A. Preventive procedures without use of appliances:
blanch test and notching of interdental bone I. Predental procedures and parental education
in a periapical radiograph confirms the thick II. Maintenance of oral hygiene
frenal attachment, which should be diag- III. Caries control and restoration of decayed teeth
nosed and treated at an early age. IV. Care of deciduous dentition
(b) Presence of anklyloglossia or tongue tie V. Management of tooth ankylosis/locked perma-
prevents normal functional development and nent first molar
results in abnormal speech and swallowing VI. Extraction of supernumerary teeth/retained de-
patterns. This should be treated surgically to ciduous tooth
prevent full-fledged malocclusions. VII. Maintenance of quadrant-wise tooth shedding
(c) Oral habits like finger and thumb sucking, time table and space maintenance
tongue thrusting, lip biting and nail biting VIII. Management of abnormal frenal attachments and
should be identified and stopped to en- check up for deleterious oral habits
hance normal functional and deglutitional IX. Prevention of damage to occlusion/occlusal
activity. equilibration
(d) Early correction of habits is easier and helps B. Preventive procedures with use of appliances:
in elimination of the unfavourable sequelae I. Mouth protectors
of habits which lead to malocclusion. II. Space maintenance
IX. Prevention of damage to occlusion/occlusal Space maintainer: A device used to maintain the space
equilibration created by the loss of a deciduous tooth is known as
(a) Damage to occlusion as well as retardation space maintainer.
of mandibular growth and possible deformi- The ideal requirements of a space maintainer are as
ties are caused by orthopaedic appliances follows:
used for correction of scoliosis. Example: l Should maintain the desired mesiodistal width/

Milwaukee brace should be prevented by space created by premature loss of tooth


using functional appliances and positioners l Must restore function and should be strong

made of soft materials. enough to withstand functional forces


(b) All functional prematurities leading to de- l Should not interfere with eruption of other

viation of mandibular path of closure and permanent teeth


150 Quick Review Series for BDS 4th Year, vol 1

l Should not exert excessive stress on adjoining Full or complete dentures – in case of extraction
l

teeth and should prevent supraeruption of of all primary teeth in a preschooler due to ram-
opposing teeth pant caries.
l Should be simple in fabrication l Removable distal shoe space maintainer – in case

l Should be easily cleansable and permit mainte- of loss of deciduous second molar, shortly before
nance of good oral hygiene eruption of first permanent molars, an immediate
acrylic distal shoe extension has been success-
fully used to guide the permanent first molar into
(SN Q.2)
the position.
Classification of space maintainers Some commonly used fixed space maintainers
I. According to Hitchcock: Fixed space maintainers are those which are fixed or
i. Removable or fixed or semifixed fitted on to the teeth and require minimum or no tooth
ii. With bands or without bands preparation.
iii. Functional or nonfunctional Example: band and loop space maintainer (Fig 17.1)
iv. Active or passive
v. Certain combinations of the above
II. According to Raymond C. Thurow:
i. Removable
ii. Complete arch
(a) Lingual arch
(b) Extra-oral anchorage
iii. Individual tooth
III. According to Hinrichsen: Loop

E E
Fixed space Removable space
maintainers maintainer
Example: Acrylic
partial dentures 6 6 Band

Class I Class II
Example: Cantilever Fig. 17.1  Band and loop space maintainer.
type (distal shoe,
Two types
band and loop) (i) Band and loop space maintainer
l It is a fixed, semi-rigid, nonfunctional type of pas-

sive appliance.
Nonfunctional Functional l Indicated for space maintenance in the posterior
i. Bar type i. Pontic type
segments when a single tooth is lost, e.g. prema-
ii. Loop type ii. Lingual arch type
ture loss of deciduous canines, first molars or
second molars unilaterally.
Some commonly used removable space maintainers l Advantage: ease of fabrication.
i. Acrylic partial dentures l Disadvantage: supraeruption of opposing tooth
ii. Full or complete dentures and slipping of the loop towards gingiva.
iii. Removable distal shoe space maintainers l Modifications:
Removable space maintainers are mainly indicated: a. Mayne’s modification where loop extends only
l When there are multiple losses of deciduous teeth on buccal side.
which may require functional replacement and b. Band and loop with vertical projection, where
restoration of aesthetics occlusal rest prevents tipping and sliding of the
l In case of partially erupted permanent teeth, loop gingivally.
where abutment teeth cannot support a fixed ap- c. Crown and loop: Band is replaced by a stain-
pliance less steel crown.
Examples: (ii) Crown and loop space maintainer
l Acrylic partial dentures – in cases of multiple l Crown and loop space maintainers are similar
extractions. to band and loop space maintainers in every
Section | I  Topic-Wise Solved Questions of Previous Years 151

Loop Crown

Fig. 17.2  Crown and loop appliance.

6 6
respect except that a stainless steel crown is
used for abutment tooth in the place of band.
l It is useful where the tooth used as abutment
Fig. 17.4  Nance holding arch.
is highly carious or pulpotomized or markedly
hypoplastic.
(iii) Lingual arch space maintainer
l The lower lingual holding arch is a nonfunctional,

passive fixed appliance.

Stainless steel
lingual arch wire
D

Prematurely
E lost primary
E
teeth Fig. 17.5  Transpalatal arch.
6 6 Band
l Nance holding arch is a maxillary palatal arch
Fig. 17.3  Lingual arch space maintainer. space maintainer mainly used in cases of bilateral
loss of deciduous molars in maxilla. It is the ap-
pliance of choice.
l It is the most effective space maintainer in cases
l Transpalatal arch is best indicated when one side
of multiple losses of primary molars bilaterally in
of the arch is intact and several primary teeth on
mandibular arch.
other side are missing prematurely.
l It maintains arch perimeter by preventing both
(v) Distal shoe space maintainer (intra-­alveolar appli-
mesial drifting of the molars and lingual collapse
ance)
of anterior teeth.
l Distal shoe appliance is an intra-alveolar appli-
l Example: used in cases of premature loss of de-
ance introduced by Willets (1932) and later modi-
ciduous first or second molars bilaterally
fied by Roche (1942).
(iv) Palatal arch appliances:
l It is primarily used in cases of premature loss of

deciduous second molar prior to eruption of per-


manent first molars.
Palatal arch space maintainers
l The distal shoe appliance provides a greater con-

trol on path of eruption of unerupted permanent


first molar tooth and at the same time prevents its
Nance palatal holding arch Transpalatal arch undesirable mesial migration.
152 Quick Review Series for BDS 4th Year, vol 1

Example: used in the cases of premature loss of


D deciduous first and second molars
Q.3. What are the various preventive orthodontic proce-
dures? Explain in detail the various space maintainers.
6
5
Ans.
[Same as LE Q.2]
Q.4. Describe in brief space maintainers and their
classification with examples.
Ans.
Fig. 17.6  Distal shoe space maintainer
[Same as LE Q.2]
(vi) Aesthetic anterior space maintainer Q.5. What are space maintainers? Describe the various
l Introduced and described by Steffen, Miller and types used in orthodontics and their indications.
Johnson in 1971.
l Mainly used for space maintenance in premature
Ans.
loss of deciduous incisors. It consists of plastic [Same as LE Q.2]
teeth fixed onto a lingual arch, which in turn is
attached to molar bands.
(vii) Band and bar types or crown and bar space main- SHORT ESSAYS:
tainer Q.1. Procedures under preventive orthodontia.
These are fixed space maintainers where abutment
teeth on either side of extraction space are either Ans.
banded or given stainless steel crowns and connected [Ref LE Q.1]
to each other by a bar.
Q.2. Define space maintainer. Mention its indications
3
and contraindications.
Ans.
E A device used to maintain the space created by the loss of
4 a deciduous tooth is known as space maintainer.
Indications of space maintainers:
5
Space maintainers are indicated in the following condi-
tions:
l Early loss of primary anterior teeth.
l When a second primary molar is lost before the

eruption of permanent first molars and before the


second premolars are ready to take its place.
l Early loss of the primary first molar.

Fig. 17.7  Band and bar space maintainer. l In cases of congenitally missing second premolars

where a prosthesis is planned later.


l Loss of permanent first molar after eruption of

3 second permanent molar.


l Active space maintainers or space regainers are

E 6 used when there is minor amount of loss of space


4 that has to be gained.
Contraindications to space maintainers:
5 l When there is only soft tissue covering or very mini-

mal amount of bone overlying the crown of erupting


permanent tooth.
l When the space available is in excess of the mesio-

distal dimension of the erupting succedaneous


Fig. 17.8  Crown and bar space maintainer. tooth.
Section | I  Topic-Wise Solved Questions of Previous Years 153

l When the minor arch length discrepancy exists, l Patient should be educated about periodical
which can be corrected with the amount of space checkups in identifying the problems at early
available. stage and advantages of its prevention by ap-
l Congenitally missing permanent successor. propriate measures.
l Space maintainer may not be necessary when a small ii. Need for diagnostic records.
period of gap exists between shedding of deciduous l For a 2-year-old child: Clinical examination, in-

tooth and eruption of succedaneous tooth. traoral radiographs and panoramic radiographs.
l For a 5-year-old child: Longitudinal records
Q.3. Features of an ideal space maintainer. are required.
Ans. l If there are any signs of developing malocclu-

sion, periapical radiographs should be taken


once a year.
{SN Q.6} iii. Study casts.
l Between 6 and 12 years of age, study casts
Space maintainer is a device used to maintain the space
created by the loss of a deciduous tooth. make up invaluable records.
l In required cases, study casts should be pre-
The ideal requirements of a space maintainer are as
follows: pared every year to compare and evaluate
l Should maintain the desired mesiodistal width/space
potential problems.
created by premature loss of tooth Identification of future orthodontic problems
l The critical step in preventive orthodontics is the
l Must restore function and should be strong enough to

withstand functional forces recognition of future problem.


l The possible future problems can be detected by
l Should not get deformed, distorted or break

l Should not interfere with eruption of other perma-


two ways: clinical and radiographic indicators.
nent teeth i. Clinical indicators
l A thorough visual examination will reveal
l Should not exert excessive stress on adjoining

teeth and should prevent supraeruption of opposing potential problems.


l Differentiation of potential problems from
teeth
l Should not impede the vertical eruption of adjacent
self-correcting malocclusions is essential.
tooth Example: identification of proximal caries, plan-
l Should maintain individual functional movements of
ning for space maintenance
the teeth ii. Radiographic indicators
l Most important radiographic indicators are
l Should not interfere with normal development of

occlusion resorption and eruption patterns of primary


l Should be able to provide mesiodistal space opening
and permanent dentitions respectively.
if required Benefits of preventive orthodontics are as follows:
l Psychological benefits due to prevention of mal-
l Should be simple in fabrication

l Should be easily cleansable and permit good oral


occlusion.
l Preventive measures eliminate aetiologic factors
hygiene maintenance
and make it possible to restore normal growth and
possibility of achieving better results.
Q.4. Preventive orthodontics. l Early treatment of deleterious habits eliminates

problems of malocclusion.
Ans.
l Makes the treatment economical.
l Graber has defined preventive orthodontics as the action
Q.5. Describe the rationale or the principle of preven-
taken to preserve the integrity of what appears to be a
tive orthodontic practice.
normal occlusion at a specific time.
l Proffit and Ackerman have defined it as prevention of Ans.
potential interference with occlusal development. [Same as SE Q.4]
l Preventive orthodontics is a dynamic and constant vigi-

lance to prevent malocclusion by both dentist and


patient/parent.
SHORT NOTES:
The rationale of preventive orthodontics is as follows: Q.1. Define preventive orthodontics.
Requirements
Ans.
i. Establishment of a good rapport between patient
and dental surgeon. [Ref LE Q.1]
154 Quick Review Series for BDS 4th Year, vol 1

Q.2. Define space maintainer.


Ans. D

[Ref LE Q.1]
6
Q.3. Fixed space maintainer. 5

Ans.
Fixed space maintainers are those which are fixed onto the
teeth either with bands or crowns.
Advantages of fixed space maintainers are as follows:
i. Can be used in uncooperative patients.
Fig. 17.9  Distal shoe space maintainer
ii. Minimum or no tooth preparation is required to fix
bands and crowns.
iii. Jaw growth is not affected.
iv. They permit eruption of succedaneous permanent Q.6. What are the requirements of space maintainers?
teeth and passive eruption of abutment teeth without Ans.
any interference.
v. If pontics are placed, masticatory function is [Same as SE Q.3]
restored. Q.7. Nance appliance.
Q.4. Oral hygiene measures during orthodontic treatment. Ans.
Ans. It is a type of fixed palatal arch space maintainer.
Proper oral hygiene measures throughout the orthodontic (i) Nance appliance is an appliance of choice in cases of
treatment are essential. bilateral loss of deciduous molars in maxillary arch.
Various devices and methods to maintain good oral (ii) It incorporates an acrylic button in the anterior region
hygiene during orthodontic treatment are as follows: that contacts palatal tissue in the anterior palate with-
i. Special orthodontic brushes wherein the middle row is out contacting the anterior maxillary teeth.
shortened are used for home care. (iii) Advantages:
ii. Electronic tooth brushes – for patients lacking good a. Economical
motor control. b. Allows transverse growth in intercanine and perma-
iii. Interdental stimulation with special uni-tufted brushes nent intermolar areas
helps to prevent soft tissue proliferation. (iv) Disadvantages:
iv. Digital gum massage for about 5 min a day controls a. Requires good clinical skills.
gingival proliferation. b. Inflammation of soft tissues in anterior palatal
v. Waterpik is very effective in removing the debris. region.

Q.5. Distal shoe space maintainer.


Ans.
Distal shoe space maintainer is also known as eruption guid-
ance appliance, Willet’s appliance or Roche’s appliance.
l Distal shoe appliance is an intra-alveolar appliance in-

troduced by Willets (1932) and later modified by Roche


(1942).
l It is primarily used in cases of premature loss of decidu-

ous second molar prior to eruption of permanent first


molars.
l It is of the following types:

i. Fixed: (a) functional, and (b) nonfunctional.


ii. Removable
l The distal shoe appliance provides a greater control on

path of eruption of unerupted permanent first molar


tooth and, at the same time, prevents its undesirable
Fig. 17.10  Nance appliance.
mesial migration.
Section | I  Topic-Wise Solved Questions of Previous Years 155

Q.8. Enumerate preventive orthodontic procedures. Q.11. Advantages of a fixed space maintainer. Give an
example.
Ans.
Ans.
[Same as SN Q.1]
[Same as SN Q.3]
Q.9. Procedures under preventive orthodontics.
Q.12. Willet’s appliance.
Ans.
Ans.
[Same as SN Q.1]
[Same as SN Q.5]
Q.10. Classification of space maintainers.
Ans.
[Same as SN Q.2]

Topic 18
Interceptive Orthodontics
COMMONLY ASKED QUESTIONS
LONG ESSAYS:
1. Define interceptive orthodontics. Enumerate the various interceptive orthodontic procedures and describe serial
extraction procedures in detail.
2. Define interceptive orthodontics, and discuss in detail the various procedures involved.
3. Define interceptive orthodontics. Discuss serial extraction procedure. [Same as LE Q.1]
4. Describe the indications, contraindications and technique of serial extraction. [Same as LE Q.1]
5. Define serial extraction. Discuss in detail the indications and procedures of serial extraction. [Same as LE Q.1]
6. Define serial extraction. Discuss indications and contraindications, and advantages and disadvantages of serial
extraction. [Same as LE Q.1]
7. Define interceptive orthodontics and describe various methods of interceptive orthodontics. [Same as LE Q.2]

SHORT ESSAYS:
1. Muscle exercises. [Ref LE Q.2]
2. Interceptive orthodontics. [Ref LE Q.2]
3. Serial extractions. [Ref LE Q.1]
4. Classify space regainers and write briefly on any one.
5. Developing anterior crossbite correction.
6. Indications and contraindications for serial extraction procedures. [Same as SE Q.3]
7. Indications for serial extraction. [Same as SE Q.3]

SHORT NOTES:
1. Define serial extraction. Add a note on it.
2. Muscle exercises. [Ref LE Q.2]
3. Interceptive orthodontics.
4. Advantages of serial extraction. [Ref LE Q.1]
5. Disadvantages of serial extraction. [Ref LE Q.1]
6. Define serial extraction. Give its contraindications. [Ref LE Q.1]
7. Enumerate various serial extraction procedures. [Ref LE Q.2]
8. Space regainer. [Ref LE Q.2]
156 Quick Review Series for BDS 4th Year, vol 1

9. Developing anterior crossbite correction. [Same as SE Q.5]


10. Define serial extraction. [Same as SN Q.1]
11. Write few indications of serial extractions. [Same as SN Q.1]
12. Define serial extraction and discuss any one method of serial extraction. [Same as SN Q.7]
13. Classification of anterior crossbites. [Same as SN Q.9]

SOLVED ANSWERS
LONG ESSAYS:
Q.1. Define interceptive orthodontics. Enumerate the 2. Physiologic tooth movement is being utilized
various interceptive orthodontic procedures and de- in serial extraction for self-correction.
scribe serial extraction procedures in detail. 3. Result of serial extraction is influenced by nor-
mal growth of dental, skeletal and soft tissues.
Ans.
l Interceptive orthodontics has been defined as that phase [SE Q.3]
of the science and art of orthodontics employed to rec- {Indications:
ognize and eliminate potential irregularities and malpo- i. Class I malocclusion with an arch length–tooth size
sitions of the developing dentofacial complex. deficiency of 10 mm or more per quadrant showing
l Interceptive orthodontics basically refers to measures harmony between skeletal and muscular systems.
undertaken to prevent a potential malocclusion from ii. As compared with the tooth material, the arch
progressing into a more severe one. length deficiency is the most important indica-
The procedures undertaken in interceptive orthodontics tion for serial extraction.
include the following: iii. In patients where growth is not enough to over-
l Serial extractions
come the discrepancy between tooth material and
l Correction of developing crossbite: anterior and posterior
basal bone.
l Control of abnormal habits
iv. Patients with straight profile and pleasing ap-
l Space regaining
pearance.
l Muscle exercises
v. The arch length deficiency either unilateral or bilat-
l Interception of skeletal malrelation
eral due to nonpathological causes like premature loss
l Removal of soft tissue or bony barrier to enable eruption
of canines with midline shift, malpositioned or im-
of teeth pacted lateral incisors erupting out of the arch, bimax-
l Extraction of supernumerary and ankylosed teeth
illary protrusion, ectopic eruption of teeth and local-
ized gingival recession in the lower anterior region.
[SE Q.3] vi. The arch length deficiency due to pathologic causes
like extensive proximal caries and subsequent mesial
{Serial extraction migration of buccal segment, ankylosis of tooth,
l Serial extraction is defined by Tweed as the planned and premature loss of deciduous teeth, deleterious oral
sequential removal of the primary and permanent teeth habits and improper proximal restorations.}
to intercept and reduce dental crowding problems.}
l This procedure is usually initiated in the early mixed
(SE Q.3 and SN Q.6)
dentition period. It includes the planned extraction of {(Contraindications:
certain deciduous teeth and later specific permanent i. Mild class I malocclusions with minimum space
teeth in an orderly sequence and predetermined pat- deficiency and skeletal class II or III malocclusion.
tern to guide the erupting permanent teeth into a ii. Congenital absence of teeth – anodontia/oligo-
more favourable position. dontia.
History: iii. Open bite and deep bite.
Kjellgren (1929) – used the term serial extraction. iv. Spaced dentition and midline diastema.
Nance (1940) – termed it as planned and progressive vi. Unerrupted malformed teeth, e.g. dilaceration.
extraction and popularized the technique in the vii. Extensive caries or heavily filled first permanent
USA. molars.)}
Hotz (1970) – termed it as active supervision of teeth
by extraction. Diagnosis:
Rationale: l Study model analysis – Carey’s analysis for lower

Serial extraction is based on mainly two principles: arch


1. Arch length – Tooth material discrepancy is Arch perimeter analysis for upper arch
corrected by reducing the tooth material. l OPG – for evaluation of eruption status of dentition
Section | I  Topic-Wise Solved Questions of Previous Years 157

l Cephalometrics – assessment of skeletal tissues to


iii. Reduces duration of multibanded fixed treatment
study underlying skeletal relationship
as well as retention period.
l Clinical examination 1 cephalograms – for soft
iv. Reduced risk of caries due to better oral hygiene.
tissue assessment
v. More stable results – as tooth material and arch
Procedure:
length are in harmony.
Dewel’s method

Three popular methods Tweed’s Method


{SN Q.5}
Nance’s Method Disadvantages:
i. Prolong treatment time and follow-up.
i. Dewel’s method: ii. Regular patient visits – cooperation of patient
Dewel proposed a three-step serial extraction pro- needed.
cedure as follows: iii. Tendency to develop tongue thrust – due to cre-
The sequence of proposed extractions: CD4 ated extraction spaces.
Step 1: Extraction of ‘C’ – between 8 and 9 years iv. Serial extraction requires – good clinical judge-
to create space for alignment of incisors. ment.
Step 2: Extraction of ‘D’ – one year later, i.e. at v. Extraction of buccal teeth results in – deepening
10 years of age to accelerate eruption of first of bite.
premolars. vi. This is not a definitive treatment, the axial incli-
Step 3: Extraction of ‘4’ (first premolar) – to permit nation of teeth at the end of serial extraction
the eruption of permanent canines in their place. procedure requires – short-term fixed appliance
Modified Dewel’s technique: therapy.
Wherein first premolars are enucleated at the vii. Poorly executed serial extraction programme can
time of extraction of the first deciduous molar, be worse than none at all.
especially in mandibular arch where canines
erupt before first premolars.
ii. Tweed’s method: Q.2. Define interceptive orthodontics, and discuss in
The sequence of proposed extractions: DC4 detail the various procedures involved.
Step 1: Extraction of ‘D’ (deciduous first molar) –
at 8 years of age. Ans.
Step 2: Deciduous canines are maintained till
[SE Q.2]
premolars are in advanced eruptive stage.
Then both ‘C’ along with first premolars ‘4’ are l {Interceptive orthodontics has been defined as that
extracted simultaneously. phase of the science and art of orthodontics employed to
iii. Nance’s method: recognize and eliminate potential irregularities and mal-
The sequence of proposed extractions: D4C. positions of the developing dentofacial complex.
This method is basically modified Tweed’s method. l Interceptive orthodontics basically refers to measures
Step 1: Extraction of ‘D’ (deciduous first molars) – undertaken to prevent a potential malocclusion from
at 8 years of age. progressing into a more severe one.
Step 2: Extraction of ‘4’ (first premolars) and ‘C’ l The procedures undertaken in interceptive orthodontics

(deciduous canines) simultaneously. include the following:


Postserial extraction therapy: i. Serial extractions
Most cases of serial extraction need fixed orth- ii. Correction of developing crossbite: anterior and
odontic appliance therapy for correction of posterior
axial inclination and detailing of occlusion. iii. Control of abnormal habits
iv. Space regaining
v. Muscle exercises
{SN Q.4}
vi. Interception of skeletal malrelation
Advantages of serial extraction: vii. Removal of soft tissue or bony barrier to enable
i. Treatment is more physiologic. eruption of teeth
ii. As the treatment is carried out, an early age psy- viii. Extraction of supernumerary and ankylosed
chological trauma can be avoided. teeth}
158 Quick Review Series for BDS 4th Year, vol 1

l Screening patients for underlying psychological


{SN Q.7}
disturbances or any anatomical obstructions and
i. Serial extractions: referring to concerned professionals for appropri-
i. Serial extractions is an interceptive orthodontic ate treatment.
procedure that includes the planned extraction of l Teaching child the correct method of swallowing,

certain deciduous teeth and later specific perma- removal of obstruction, tongue exercises, lip exer-
nent teeth in an orderly sequence and predeter- cises, etc.
mined pattern to guide the erupting permanent l Use of habit breaking appliances, both fixed and

teeth into a more favourable position. removable, is basically reminding appliances that
ii. Three popular methods in serial extraction pro- assist to quit the habit.
cedure are l Some of the commonly used removable appli-

a. Dewel’s method ances include upper Hawley’s plate with tongue


b. Tweed’s method cribs, roller balls for tongue exercise, etc.
c. Nance’s method l Correction of malocclusion.

iii. Dewel’s method of serial extraction is the most


popular method, where the sequence of proposed
extractions is CD4. {SN Q.8}
Extraction of deciduous canines creates the space for iv. Space regaining:
alignment of the incisors, whereas eruption of first pre- l Space regaining is one of the interceptive orth-
molars is accelerated by extraction of deciduous first odontic procedures.
molars. Finally, permanent first premolars are extracted l This procedure is preferably undertaken at an
to permit permanent canines to erupt into their place and early age prior to eruption of second molar.
achieve harmonious occlusion. l Two types of space regainers are:

A. Removable space regainers:


l Removable appliance with finger spring

ii. Correction of developing crossbite l Removable lingual arch

l The developing crossbite should be corrected l Expansion screws

before it becomes established. l Split saddle regainer

l The tongue blade therapy can be used to correct B. Fixed space regainers:
developing anterior crossbite in the cooperative l Gerber space regainer

children with adequate space for tooth in cross- l Lip bumpers

bite to be moved. l Lingual arch

iii. Control of abnormal habits l Commonly used space regainers are Gerber space

l Habit can be defined as the tendency towards an regainer, jackscrews and cantilever spring.
act that has become a repeated performance, l In the case of Gerber space regainer, the forces

relatively fixed, consistent and easy to perform generated by compressed coil springs bring about
by an individual, e.g. thumb sucking, tongue a distalization of first molar.
thrusting and mouth breathing.
l Due to their repetitive nature and longer dura-
l In the case of cantilever springs, distalization of
tion, the deleterious orofacial habits influence the
molars can be achieved by using removable appli-
form of orofacial structures.
ance incorporating simple finger springs.
l The various modalities of treatment to control
v. Muscle exercises:
these abnormal oral habits are as follows:
a. Elimination of cause [SE Q.1]
b. Reminder therapy/interception of habit
c. Corrective therapy l {The presence of normal orofacial muscle func-
l Interception and treatment of abnormal habits is
tion is essential for development of normal occlu-
age and severity-dependent. sion.}
l In children younger than 3 years, no active inter- (SE Q.1 and SN Q.2)
vention is instituted whereas children aged be-
tween 4 and 8 years age need only reassurance, l {(The aberrant muscle functions can be im-
positive reinforcement and friendly reminders to proved by certain muscle exercises as follows:
divert child’s attention to other things like play a. Exercises for masseter muscle strengthening:
and toys. Patient is asked to clench the teeth and count
Section | I  Topic-Wise Solved Questions of Previous Years 159

to 10 and then relax and repeat this over some SHORT ESSAYS:
duration of time. Q.1. Muscle exercises.
b. Exercise for the lips and cheeks (circum oral
muscles): The patient is asked to hold a piece Ans.
of paper between lips which maintains lip seal [Ref LE Q.2]
by stretching the upper lip, holding and swish-
ing the water behind the lips and Button pull Q.2. Interceptive orthodontics.
exercise, etc. Ans.
c. Exercises for the tongue: One elastic and two
elastic swallow, tongue hold exercise etc. [Ref LE Q.2]
Limitations of muscle exercises are that they Q.3. Serial extractions.
are not a substitute for corrective orthodontic
treatment, and if not done correctly, they can Ans.
be counterproductive also.)} [Ref LE Q.1]
vi. Interception of skeletal malrelation: Q.4. Classify space regainers and write briefly on any
Skeletal malrelations can be treated taking advan- one.
tage of growth potential of an individual and using
myofuntional appliances and orthopaedic appli- Ans.
ances like headgears and chin cups. (i) Space regaining is one of the interceptive orthodontic
vii. Removal of soft tissue or bony barrier: procedures.
Soft tissue or any boney barrier should be (ii) Space regaining procedure is preferably undertaken at
removed to enable proper eruption of teeth. an early age prior to eruption of second molar. In such
viii. Extraction of supernumerary and ankylosed teeth: patients space lost by mesial movement of the molars
Supernumerary teeth or any teeth which are anky- can be regained by distal movement of the first molar.
losed should be extracted so that the path of erup- (iii) Commonly used space regainers are
tion of permanent teeth is not obstructed by them. Gerber space regainer
Q.3. Define interceptive orthodontics. Discuss serial ex- Jackscrews
traction procedure. Cantilever spring
(a) Gerber space regainer (Fig 18.1)
Ans. (i) An orthodontic band or a crown is selected for
[Same as LE Q.1] tooth to be distalized.
Q.4. Describe the indications, contraindications and
technique of serial extraction.
Ans.
[Same as LE Q.1]
Q.5. Define serial extraction. Discuss in detail the indi-
cations and procedure of serial extraction.
Ans.
[Same as LE Q.1]
1st premolar
Q.6. Define serial extraction. Discuss indications and
contraindications, and advantages and disadvantages of
E
serial extraction. Gerber space
regainer
Ans.
[Same as LE Q.1]
Q.7. Define interceptive orthodontics and describe vari-
ous methods of interceptive orthodontics. 1st molar

Ans.
[Same as LE Q.2] Fig. 18.1  Gerber space regainer.
160 Quick Review Series for BDS 4th Year, vol 1

(ii) This space regainer consists of ‘U’-shaped hol- Q.5. Developing anterior crossbite correction.
low tubing soldered or welded to mesial aspect
Ans.
of first molar, which is to be moved distally.
(iii) ‘U’-shaped rods with open coil springs of ade-
quate length are fitted into the above tubing so {SN Q.9}
that they contact mesial aspect of first molar to (i) The correction of developing anterior crossbite is
be moved distally. an interceptive orthodontic procedure.
The forces generated by compressed coil springs (ii) Classification of anterior crossbites.
bring about a distal movement of first molar.
(b) Space regainer using jackscrews (Fig 18.2) Functional
This appliance consists of split acrylic plate with Dentoalveolar Skeletal (pseudo-class III)
jackscrew in relation to edentulous space, and is
Due to one or Due to skeletal Due to occlusal
retained using Adam’s clasp. more maxillary discrepancies prematurities
teeth positioned
in lingual relation
to mandibular
anterior teeth

Treated by tongue Best treated by Treated by elimina-


blades catalans growth modifica- tion of occlusal
appliance double tion procedures prematurities
cantilever springs using myofunc-
tional or orthopae-
dic appliances

(iii) Anterior crossbite is a condition characterized by


Adams Jack screws
clasp reverse overjet. ‘The best time to treat crossbites is
the first time they are seen’, because they are self-
perpetuating and if not treated early, they develop
into skeletal malocclusions, which require compli-
cated orthodontic as well as surgical procedures
later for their correction.
Split acrylic plate
Fig. 18.2  Space regainer using jackscrew.
Q.6. Indications and contraindications for serial extrac-
tion procedures.
(c) Space regainer using cantilever spring (Fig 18.3)
Ans.
Distalization of molar can be achieved by using re-
movable appliance incorporating simple finger [Same as SE Q.3]
springs.
Q.7. Indications for serial extraction.
Adams clasp Ans.
[Same as SE Q.3]

SHORT NOTES:
Q.1. Define serial extraction. Add a note on it.
Ans.
Serial extraction is defined by Tweed as the planned and
Finger
springs sequential removal of the primary and permanent teeth to
intercept and reduce dental crowding problems.
Indications:
i. Class I malocclusion with an arch length–tooth size
discrepancy (10 mm).
ii. Patients with straight profile and pleasing appear-
Fig. 18.3  Space regainer using cantilever spring ance.
Section | I  Topic-Wise Solved Questions of Previous Years 161

iii. The arch length deficiency either unilateral or bilat- Q.6. Define serial extraction. Give its contraindications.
eral due to nonpathological causes.
Ans.
Example: premature loss of canines with midline shift,
bimaxillary protrusion and ectopic eruption of teeth [Ref LE Q.1]
iv. The arch length deficiency due to pathologic causes.
Q.7. Enumerate various serial extraction procedures.
Example: extensive proximal caries, ankylosis of
tooth and deleterious oral habits Ans.
Q.2. Muscle exercises. [Ref LE Q.2]
Ans. Q.8. Space regainer.
[Ref LE Q.2] Ans.
Q.3. Interceptive orthodontics. [Ref LE Q.2]
Ans. Q.9. Developing anterior crossbite correction.
i. Interceptive orthodontics has been defined as that phase Ans.
of the science and art of orthodontics employed to rec-
ognize and eliminate potential irregularities and malpo- [Same as SE Q.5]
sitions of the developing dentofacial complex. Q.10. Define serial extraction.
ii. Interceptive orthodontics basically refers to measures
undertaken to prevent a potential malocclusion from Ans.
progressing into a more severe one. [Same as SN Q.1]
iii. A few procedures undertaken in interceptive orthodontics
include: serial extractions, correction of developing cross- Q.11. Write few indications of serial extractions.
bites, control of abnormal habits, muscle exercises, etc. Ans.
iv. Many of the interceptive orthodontic procedures are
nothing but extension of preventive orthodontic proce- [Same as SN Q.1]
dures, only the difference is timing of treatment. Q.12. Define serial extraction and discuss any one
Q.4. Advantages of serial extraction. method of serial extraction.

Ans. Ans.
[Ref LE Q.1] [Same as SN Q.7]
Q.5. Disadvantages of serial extraction. Q.13. Classification of anterior crossbites.
Ans. Ans.
[Ref LE Q.1] [Same as SN Q.9]

Topic 19
Methods of Space Gaining
COMMONLY ASKED QUESTIONS
LONG ESSAYS:
1 . Enumerate the various methods of gaining space and discuss extractions in orthodontics.
2. Describe various methods to gain space in orthodontics. Give their indications and contraindications. [Same as LE Q.1]
3. What is arch length discrepancy? How will you assess the total discrepancy in a given adult patient? What
methods can be used to correct arch length discrepancy? [Same as LE Q.1]
162 Quick Review Series for BDS 4th Year, vol 1

SHORT ESSAYS:
1 . What are the methods of gaining space in orthodontics?
2. Distal driving of molars.
3. Proximal stripping.
4. Methods of space gaining in dental arch. [Same as SE Q.1]
5. Slenderization. [Same as SE Q.3]

SHORT NOTES:
1 . Molar distilization – indications. [Ref SE Q.2]
2. Proximal stripping – mention few advantages. [Ref SE Q.3]
3. What are the methods of gaining space in orthodontics? [Ref LE Q.1]
4. Uprighting of molars.
5. Reproximation of teeth [Same as SN Q.2]
6. Slenderization. [Same as SN Q.2]
7. Enlist methods of gaining space. [Same as SN Q.3]

SOLVED ANSWERS
LONG ESSAYS:
Q.1. Enumerate the various methods of gaining space Aids/investigations:
and discuss extractions in orthodontics. a. Carey’s/arch perimeter analysis (tooth material
excess of 0–2.5 mm over arch length in diagnosis)
Ans.
b. Bolton’s analysis (reveals excess tooth mate-
rial in either of the arches)
{SN Q.3} c. Diagnostic set-up (helps to localize the problem,
and discloses amount of enamel reduction)
Planning space is an important aspect of the treatment d. IOPAs (it gives an idea of enamel thickness and
planning in orthodontics. Some of the methods of gain- extent of pulp horns, thereby helps in estimating
ing space include: amount of enamel that can be removed)
i. Proximal stripping Procedure of proximal stripping:
ii. Expansion Armamentarium:
iii. Extraction l Use of metallic abrasive strips
iv. Distalization l Safe-sided carborundum discs
v. Uprighting of molars l Safe-sided diamond discs
vi. Derotation of posterior teeth l Very long and thin tapered fissure burs
vii. Proclination of anteriors Procedure:
Proximal stripping is of two types: (i) localized
I. Proximal stripping and (ii) generalized.
l Localized reduction is usually carried out in
(Reproximation, slenderization, disking and proximal
slicing) mandibular or maxillary anterior regions.
l In the moderate space discrepancy cases, a gen-
l It is a method by which the proximal surfaces of

the teeth are sliced in order to g M-D width of the eralized interproximal reduction is carried out.
l Contact points are converted into contact
teeth.
Indications: areas taking care to establish proper contact
l When space required is minimum, i.e. 0–2.5 mm.
between the teeth.
l If the Bolton’s analysis shows mild tooth mate-
Amount of proximal stripping:
l Not more than 50% of enamel thickness should
rial excess in either of the arches.
l It can be undertaken as an aid to retention in
be reduced by proximal stripping and equally
the lower anterior region. distributed over all teeth.
Contraindications: Advantages:
l Extractions are avoided in the borderline cases
l Young patients with large pulp chamber h risk

of pulp exposure. where space requirement is minimal.


l In the patients with Bolton’s discrepancy, a normal
l Patients susceptible to caries/those with high

caries index. interarch relationship (favourable over bite and


Section | I  Topic-Wise Solved Questions of Previous Years 163

overjet relation) can be established by eliminating III. Extractions as a method of gaining space
tooth material excess in either of the arches. l In clinical orthodontics, extractions form a main

l More stable results can be established by part among all space gaining procedures.
broadening the contact area, which prevents l Extractions are indicated for correction of crowd-

slipping of contact. ing, anteroposterior dental arch relations, vertical


Disadvantages/drawbacks: problems, skeletal jaw deformities and presence of
l Causes roughened proximal surface that attracts supernumerary teeth.
plaque and calculus, resulting in gingivitis. l Therapeutic extractions are extractions that are un-

l h Caries susceptibility (proximal caries). dertaken as a part of orthodontic treatment.


l Hypersensitivity of teeth may develop. l Choice of teeth for extraction depends on various

l Alteration of teeth morphology (altered aesthetics factors like condition of teeth, position of teeth and
due to improper procedure/inexperienced hands). position of crowding. Premolars (most frequently
l Food impaction (because of loss of normal extracted teeth) utilized for correction of anterior 1
contact between adjacent teeth). posterior segments.
Comprehensive fluoride programme should fol- l Molars or lower incisors are also preferred.

low this proximal stripping procedure. IV. Distalization


I I. Expansion as a method of gaining space l Moving the molars in a distal direction so as to gain

l It is a noninvasive method. space is known as distalization.


Types of expansions: l It has become a popular technique of recent times.

l Ideal timing for distalization – during mixed denti-


Orthodontic/dentoalveolar – 7 7
produces dental expansion tion period prior to eruption of .
7 7
with no skeletal.
Two methods

Results from intrinsic


Three types of
forces exerted by the tongue.
passive-expansions
Maxillary Mandibular
molar distalization molar distalization
Skeletal/orthopaedic – Example: Lip bumper
results from splitting of
mid-palatal suture. Extraoral Intraoral (removable and fixed)
Examples: Headgears Examples:
Indications of arch expansion: Disadvantages of i. Sagittal appliance – split
l Crossbite (unilateral/bilateral) extraoral: acrylic plate joined by
l Constricted arches a. Patient cooperation is jackscrew. It can be used to
Types of expansion appliances: essential for timely wear distalize one tooth at a time.
l They are broadly of two types: (i) maxillary and
of appliance. ii. Intraoral magnets – consist
b. Appliance not worn cont- of repelling magnet placed
(ii) mandibular
inuously. Intermittent in on molar and a tooth anterior
action and prolonged to it.
Expansion appliances
treatment time. iii. Use of open coil springs –
open coil NiTi spring
compressed between molar
Maxillary Mandibular and anterior segments.
Example: Lower iv. Pendulum appliance –
Schwarz plate Incorporates a modified
Nance’s button – anchorage
and an SS wire with a helix
Slow Rapid which is inserted into a sleeve
on palatal aspect of molar to
be distalized.
Removable Fixed Banded RME Bonded RME
Examples: Examples: Examples: Example: V. Uprighting of molars
i. Coffin spring i. W-arch i. Haas i. Cast metal l Uprighting springs or coil springs are used.
ii. Active plate ii. Quad helix ii. Isaacson /acrylic
l By uprighting of mesially or distally tipped molar,
with screws iii. Fixed appli- iii. Derichs- splints
or Z-springs ance with weiler certain amount of space can be recovered.
expansion VI. Derotation of posterior teeth
screws l A little amount of space can be gained by correcting

rotated teeth.
164 Quick Review Series for BDS 4th Year, vol 1

l This is best achieved by fixed appliances incorpo- l Arch expansion is indicated in cases with crossbite
rating springs or elastics using a force couple. (unilateral/bilateral) and constricted arches.
VII.  Proclination anterior teeth l Types of expansion appliances:

l Slight proclination of anterior teeth results in gain- a. Maxillary, e.g. coffin spring, W-arch, quad helix
ing of arch length. and Isaacson.
Indications: b. Mandibular, e.g. lower Schwarz plate.
a. Retroclined anteriors iii. Extractions as a method of gaining space
b. Cases where protracting anteriors will not affect l In clinical orthodontics, extractions form a main

soft tissue profile part among all space gaining procedures.


c. In patients with obtuse nasolabial angle l Extractions are indicated for correction of crowding,

anteroposterior dental arch relations, vertical prob-


Q.2. Describe various methods to gain space in ortho-
lems, skeletal jaw deformities and presence of su-
dontics. Give their indications and contraindications.
pernumerary teeth.
Ans. l Choice of teeth for extraction depends on various

factors like condition of teeth, position of teeth.


[Same as LE Q.1]
Example: premolars (most frequently extracted teeth)
Q.3. What is arch length discrepancy? How will you as- utilized for correction of anterior 1 posterior segments
sess the total discrepancy in a given adult patient? What iv. Distalization
methods can be used to correct arch length discrepancy? l Moving the molars in a distal direction so as to gain

space is known as distalization.


Ans.
l It has become a popular technique of recent times.

[Same as LE Q.1] l Ideal timing for distalization – during mixed denti-


7 7
tion period prior to eruption of 7 7 .
SHORT ESSAYS: v. Uprighting of molars
l Uprighting springs or coil springs are used.

Q.1. What are the methods of gaining space in orthodontics. l By uprighting of mesially or distally tipped molar,

certain amount of space can be recovered.


Ans.
vi. Derotation of posterior teeth
Various space gaining methods are implemented in orthodon- l A little amount of space can be gained by correcting

tic treatment. Some of the methods of gaining space include: rotated teeth.
i. Proximal stripping l This is best achieved by fixed appliances incorporat-

ii. Expansion ing springs or elastics using a force couple.


iii. Extraction vii. Proclination anterior teeth
iv. Distalization l Slight proclination of anterior teeth results in gain-

v. Uprighting of molars ing of arch length.


vi. Derotation of posterior teeth l Indicated in the cases of retroclined anteriors or in
vii. Proclination of anteriors cases where protracting anteriors will not affect soft
tissue profile.
i. Proximal stripping
(Reproximation, slenderization, disking and proximal Q.2. Distal driving of molars.
slicing)
Ans.
l It is a method by which the proximal surfaces of the

teeth are sliced in order to g M-D width of the teeth.


l Indicated when space required is minimum, i.e.
{SN Q.1}
0–2.5 mm. l Moving the molars in a distal direction so as to gain space
l Contraindicated in young patients with large pulp is known as distal driving or distalization of molars.
chambers and patients with high caries index. l It has become a popular technique of recent times.
l Not more than 50% of enamel thickness should be l Ideal timing for distalization – during mixed denti-
reduced by proximal stripping and equally distrib- 7 7
uted overall teeth. tion period prior to eruption of 7 7 .
ii. Expansion as a method of gaining space Indications:
l Class II cases due to maxillary prognathism
l It is a noninvasive method.
l Mild-to-moderate protrusion/crowding in maxil-
l Three types of expansions:

a. Orthodontic/dentoalveolar lary arch with normal mandible


l Mild arch discrepancy in mandibular arch
b. Passive expansion
l In cases of anchorage, loss during orthodontic
c. Skeletal/orthopaedic
treatment
Section | I  Topic-Wise Solved Questions of Previous Years 165

Contraindications: l It can be undertaken as an aid to retention in the


l Dental class I or class III molar relation lower anterior region.
l Bimax protrusion Contraindications:
l Both skeletal and dental open bite cases l Young patients with large pulp chamber h risk of

l Cases with concave soft tissue profile pulp exposure.


Methods of distalization: l Patients susceptible to caries/those with high caries

Two methods
index.
Aids/investigations:
a. Carey’s/arch perimeter analysis (tooth material
excess of 0–2.5 mm over arch length in diagnosis).
Maxillary molar Mandibular molar
b. Bolton’s analysis (reveals excess tooth material in
distalization distalization
either of arches)
Example: Lip bumper
c. Diagnostic set-up (helps to localize the problem and
discloses amount of enamel reduction)
Extraoral Intraoral d. IOPAs (gives an idea of enamel thickness and extent
Example: Headgears of pulp horns, thereby helps in estimating amount of
enamel that can be removed)
Removable Fixed
Procedure of proximal stripping:
Armamentarium
Example: Finger Example: Open coil springs,
l Use of metallic abrasive strips
springs, Expansion Pendulum appliance,
l Safe-sided carborundum discs
plate Jones jig
l Safe-sided diamond discs

l Very long and thin tapered fissure burs


Disadvantages of extraoral method:
l Patient cooperation is essential for timely wear of
Procedure:
appliance. Proximal stripping is of two types: (i) localized and (ii)
l Appliance not worn continuously, intermittent in
generalized.
l Localized reduction is usually carried out in man-
action and prolonged treatment time.
Intraoral method (removable and fixed appliances): dibular or maxillary anterior regions.
l In the moderate space discrepancy cases, a gener-
l Sagittal appliance: A split acrylic plate joined

by jackscrew can be used to distalize one tooth at alized interproximal reduction is carried out.
l Contact points are converted into contact areas
a time.
l Intraoral magnets: Consist of repelling magnet
taking care to establish proper contact between
placed on molar and a tooth anterior to it. the teeth.
l Use of open coil springs: Open coil NiTi spring
Amount of proximal stripping:
l Not more than 50% of enamel thickness should be
compressed between molar and anterior segment.
l Pendulum appliance: Incorporates a modified
reduced by proximal stripping and equally distrib-
Nance’s button – anchorage and an SS wire with uted overall teeth.
a helix which is inserted into a sleeve on palatal
aspect of molar to be distalized. {SN Q.2}
Q.3. Proximal stripping. Advantages:
l Extractions are avoided in the borderline cases
Ans.
where space requirement is minimal.
l In the patients with Bolton’s discrepancy, a normal
{SN Q.2}
interarch relationship (favourable overbite and
l Proximal stripping is also known as reproximation, overjet relation) can be established by eliminating
slenderization, disking and proximal slicing. tooth material excess in either of the arches.
l It is a method by which the proximal surfaces of the l More stable results can be established by broad-

teeth are sliced in order to g M-D width of the teeth. ening the contact area, which prevents slipping of
contact.
Indications:
l When space required is minimum, i.e. 0–2.5 mm. Disadvantages/drawbacks:
l If the Bolton’s analysis shows mild tooth material l Causes roughened proximal surface that attracts

excess in either of the arches. plaque and calculus resulting in gingivitis


166 Quick Review Series for BDS 4th Year, vol 1

l h Caries susceptibility (proximal caries) Q.3. What are the methods of gaining space in ortho-
l Hypersensitivity of teeth may develop dontics?
l Alteration of teeth morphology (altered aesthet-
Ans.
ics due to improper procedure/inexperienced
hands) [Ref LE Q.1]
l Food impaction (because of loss of normal contact
Q.4. Uprighting of molars.
between adjacent teeth)
Comprehensive fluoride programme should follow this Ans.
proximal stripping procedure.
l Uprighting of molars is a method gaining space in
Q.4. Methods of space gaining in dental arch. orthodontics without reducing any tooth material.
l Uprighting of molars is required when they are tipped either
Ans.
mesially or distally occupying more space in the arch.
[Same as SE Q.1] l Uprighting springs or coil springs are commonly used

for uprighting the molars.


Q.5. Slenderization.
l By uprighting of mesially or distally tipped molar, cer-

Ans. tain amount of space can be recovered.


[Same as SE Q.3] Q.5. Reproximation of teeth.
Ans.
SHORT NOTES: [Same as SN Q.2]
Q.1. Molar distilization – indications. Q.6. Slenderization.
Ans. Ans.
[Ref SE Q.2] [Same as SN Q.2]
Q.2. Proximal stripping – mention few advantages Q.7. Enlist methods of gaining space.
Ans. Ans.
[Ref SE Q.3] [Same as SN Q.3

Topic 20
Arch Expansion
COMMONLY ASKED QUESTIONS
LONG ESSAYS:
1 . Enumerate various methods to gain space in orthodontics and write in detail about rapid maxillary expansion.
2. Classify expansion in orthodontics. Discuss your line of treatment for a case of 12 years with bilateral buccal
crossbite. [Same as LE Q.1]
3. Enumerate various methods to gain space in orthodontics. Discuss rapid palatine expansion. [Same as LE Q.1]
4. Explain rapid maxillary expansion in detail. [Same as LE Q.1]

SHORT ESSAYS:
1 . Arch expansion. [Ref LE Q.1]
2. Expansion screws.
3. Indications of rapid maxillary expansion. [Ref LE Q.1]
4. Compare rapid and slow palatal expansions.
5. Rapid palatine expansion. [Same as SE Q.3]
6. Dental versus skeletal expansion. [Same as SE Q.4]
Section | I  Topic-Wise Solved Questions of Previous Years 167

SHORT NOTES:
1. Slow expansion appliance.
2. Rapid maxillary expansion.
3. Coffins spring.
4. Expansion screws. [Ref SE Q.2]
5. Role of expansion as a method of gaining space.
6. Give indications for rapid palatine expansion. [Ref LE Q.1]
7. Derichsweiler appliance.
8. Hyrax screw.
9. Quad helix appliance.
10. RME. [Same as SN Q.2]
11. Expansion devices. [Same as SN Q.4]

SOLVED ANSWERS
LONG ESSAYS:
Q.1. Enumerate various methods to gain space in or- Indications of arch expansion:
thodontics and write in detail about rapid maxillary l Crossbite (unilateral/bilateral)
expansion. l Constricted arches

Types of expansion appliances:


Ans.
l They are broadly of two types: (i) maxillary and
Various methods to gain space in orthodontics include: (ii) mandibular.
i. Proximal stripping
ii. Expansion Expansion appliances
iii. Extraction
iv. Distalization
v. Uprighting of molars Maxillary Mandibular
vi. Derotation of posterior teeth Example: Lower
vii. Proclination of anteriors Schwarz plate
Expansion as a method of gaining space
Expansion is a noninvasive method of gaining space.
Slow Rapid
[SE Q.1]
{Types of expansions:
Removable Fixed Banded RME Bonded RME
Orthodontic/dentoalveolar – Examples: Examples: Examples: Example:
i. Coffin spring i. W-arch i. Haas i. Cast metal
Produces dental expansion
ii. Active plate ii. Quad helix ii. Isaacson /acrylic
with no skeletal change. iii. Derichs- splints
with screws iii. Fixed appli-
or Z-springs ance with weiler
expansion
Three types of Results from intrinsic forces screws
passive expansions exerted by the tongue.
Rapid maxillary expansion (RME)
Skeletal/orthopaedic results Emerson C Angell (1860) is the pioneer of rapid maxil-
from splitting of mid- lary expansions. Nowadays it is an important form of
palatal suture. orthopaedic therapy.
168 Quick Review Series for BDS 4th Year, vol 1

[SE Q.3] a. Banded RME appliances:


l Usually here the first premolars or deciduous mo-
Indications of RME:
lars and first permanent molars are banded. They
(SE Q.3 and SN Q.6) are joined labially and palatally by soldering with
heavier gauge wire.
{(Indications of rapid maxillary expansion can be l The basic RME appliance is the screw, which is
considered under the following two headings: placed in the midline.
l Different types of banded RMEs: The difference
Orthodontic indications Medical indications
in appliance design of various banded RME appli-
ances is based on various types of screws and
i. Unilateral or bilateral posterior i. Poor nasal airway mode of attachments. They are as follows:
skeletal crossbite i. HAAS type:
ii. Narrow maxilla in certain ii. Recurrent ear, nasal or l In this type of banded RME, a heavy stain-
class II cases sinus infections less steel wire (0.045 inch/1.15 mm) is
iii. Class III malocclusion iii. Allergic rhinitis and asthma welded and soldered along the palatal as-
iv. Collapsed maxillary arch due iv. Before correction of septal pects of the band.
to cleft palate deformities
l The free ends are turned back to be em-
v. Treatment along with reverse
pull headgear to loosen the bedded in acrylic.
sutures l The screw used in this type of RME is

vi. To gain space in anterior similar to Derichsweiler type.


crossbite ii. Isaacson type:
vii. High angle cases (bonded l It is a tooth-borne appliance without any
type of RME) palatal acrylic covering.
l The drawback of expansion screw is the
[SE Q.3] build-up of pressure, which is hazardous to
{Principle of RME tissue.
l A special spring-loaded screw, called
l Force applied to widen the maxilla causes opening of
the mid-palatal suture and induces new bone formation. a MINNE expander, is used in this de-
Space created in the midline is initially filled with tissue sign soldered directly to the bands to
fluids and blood; later after 3–4 months, new bone fills overcome the pressure built up and to
in the space. make the force application smooth and
Classification of rapid expansion appliances: constant.
A. Removable iii. Hyrax or Biedermann type:
l Biedermann-type RME uses Hyrax (hy-
B. Fixed
a. Bonded or banded type gienic rapid expander) screw.
l Hyrax screws have heavy gauge wire ex-
b. Tooth borne
Or tensions, which are adapted to the palatal
Tooth- and tissue-borne type} contour, welded and soldered to the pala-
A. Removable appliances tal aspect of the bands on premolars and
l They are not effective for rapid maxillary expan-
molars.
sion as they are not rigid enough to produce iv. Derichsweiler type:
l In this type of appliance, the screw is
skeletal expansion.
l A removable appliance consists of split acrylic
connected to the bands by means of wire
plate with a midline screw. tags that are welded and soldered to the
l Appreciable skeletal effects are produced with
palatal aspect of band on one side and
these appliances when treatment is performed dur- embedded in acrylic on the palatal as-
ing deciduous or early mixed dentition period. pects of all nonbanded teeth except the
B. Fixed rapid maxillary expansion appliances incisors. Acrylic adapts to the palate and
They can be bonded or banded or tooth borne, tooth is in two halves to permit activation of
and tissue borne. screw.
Section | I  Topic-Wise Solved Questions of Previous Years 169

b. Bonded RME appliances: Bone changes:


In bonded RME, instead of bands, metallic cap l Maxilla moves laterally due to expansion and also

splints or acrylic covering is used. rotates with the fulcrum at frontonasal suture.
i. Cast metal cap splints: l Downward and backward rotation of mandible

Cast cap splints to which screws are soldered with increase in mandibular angle.
are prepared for all the teeth and the entire l Increase in nasal airway, reduction in airway

assembly is cemented or bonded. resistance.


ii. Acrylic splints: Sutural changes:
l Thick gauge stainless steel wire is closely l Space created by sutural opening is filled with tis-

adapted buccally and palatally around the sue fluid and haemorrhage and later the area is
posterior teeth from premolars to molars invaded by osteoblasts.
and a screw is soldered to the wire. l New bone is deposited at the edges of palatal pro-

l Acrylic is covered over the occlusal, buccal cess and the space is gradually filled with the bone.
and palatal-occlusal third of all the poste- Dental changes:
rior teeth, and the assembly is cemented/ l Initially, teeth move labially by translation and

bonded. later there is an increased buccal inclination of the


Advantages of bonded RME: posterior teeth with slight extrusion.
l Bonded appliances are useful in high angle l Appearance of median diastema, which later

cases. closes due to pull of trans-septal fibres.


l The occlusal acrylic covering acts as a Tissue reaction after expansion:
splint and prevents increase in mandibular l At the end of active expansion, 80% skeletal and

angle. 20% dental expansion occurs.


Appliance management in children younger than l After 4 months, 50% skeletal and 50% dental

15 years: changes are observed.


l Activated twice in a day. l Relapse is highest during the first 6 weeks after

l 90° activation each time with total 180° expansion, and there is more skeletal relapse
activation every day. while dental correction is retained.
l 0.5 mm/day. Retention schedule after RME:
l Review: after 1 week. l The objective of retention is to hold the expan-

Pain is felt in patients who are in late ado- sion, while the forces generated have decayed.
lescences and adults due to build up of l The same fixed RME appliance is used as

force. retainer for first 3 months with the hole of the


Slight discomfort may be felt during expan- screw filled with self-cure acrylic.
sion. Persistent pain is noticed in patients l From fourth month onwards removable retainers

wherein suture is fused. In such cases, acti- are given and are worn for full time for about
vation should be stopped. 9 months after expansion, and later half-time wear
l Surgically assisted rapid palatal expansion (SARPE) is advised.
In adults, palatal osteotomies lateral to mid-palatal su-
ture is done to assist rapid expansion. Q.2. Classify expansion in orthodontics. Discuss your
Clinical implications of expansion line of treatment for a case of 12 years with bilateral
l Mid-palatal suture does not open evenly but buccal crossbite.
opens in a ‘V’ fashion, with the broad end of
Ans.
V in anterior region and the apex in the posterior
region. [Same as LE Q.1]
l Occlusal and frontal cephalometric radiographs
Q.3. Enumerate various methods to gain space in ortho-
will reveal suture opening.
dontics. Discuss rapid palatine expansion.
l Force recorded during rapid expansion is in the

range of 10–20 pounds. Ans.


l Usual treatment period is 2 weeks and the relapse
[Same as LE Q.1]
is higher after RME; hence overcorrection is
advised. Q.4. Explain rapid maxillary expansion in detail.
Tissue changes observed with RME
Ans.
Tissue changes can be observed in bone, sutures and
dental structures. [Same as LE Q.1]
170 Quick Review Series for BDS 4th Year, vol 1

SHORT ESSAYS: The comparable features of rapid and slow palatal


expansions are as follows:
Q.1. Arch expansion.
Features Rapid expansion Slow expansion
Ans.
i. Nature of Mostly skeletal and Mainly dental
[Ref LE Q.1] expansion even dental

Q.2. Expansion screws. ii. Age Before fusion of Any age


mid-palatal suture
Ans. (young growing
individuals)
iii. Rate of Rapid Slow
{SN Q.4} expansion

l A typical expansion screw consists of an oblong iv. Indication Skeletal crossbite Cases of minor
body divided into two halves, with each half consist- cases in class II and space discrepancy
class III
ing of threaded inner side that receives one end of a
double-ended screw. v. Force level Greater forces around Milder forces be-
l The screw has a central basing with four holes,
used 10–20 pounds tween 2–4 pounds
which receive a key, which is used to turn the screw. vi. Type of tissue Traumatic Physiological
l The turning of screw to 90° brings about linear reaction
movement of 0.18. vii. Type of Mostly fixed Either fixed or
l Various types of expansion screws used in removable appliance appliance removable
and fixed appliances are jackscrews, coffin springs, used
quad helix, Isaacson, Hyrax and Derichsweiler, for viii. Frequency More frequent Less frequent
example. of activation 0.5–1 mm/day 1 mm/month
l Activation schedule: To achieve desired results, dif- ix. Duration of Short duration of Long duration of
ferent activation schedules have been advocated by treatment about 2–3 weeks around 2–3 months
different authors; the most popular ones are: x. Retention 9 months 3 months
a. Schedule by Timms
b. Schedule by Zimring and Isaacson
Q.5. Rapid palatine expansion.
Ans.
l According to Timms:
In patients aged up to 15 years: 90° rotation in the morn- [Same as SE Q.3]
ing and evening. Q.6. Dental versus skeletal expansion.
In patients aged over 15 years: 45° activation for four
times a day. Ans.
l According to Zimring and Isaacson:
[Same as SE Q.4]
In growing individuals: Two turns each day for 4–5 days,
and later one turn per day till the desired expansion is
achieved SHORT NOTES:
In non-growing adults: Two turns each day for first Q.1. Slow expansion appliance.
2 days and later one turn per day for next 5–7 days and
one turn every alternate day till the desired expansion is Ans.
achieved i. Slow expansion is traditionally known as dentoalveolar
Q.3. Indications of rapid maxillary expansion. expansion, although some minute skeletal changes can
also be seen.
Ans. ii. Here, expansion is done slowly at the rate of 0.5–1 mm/
[Ref LE Q.1] week. The forces generated by slow expansion are
much less of around 2–4 pounds.
Q.4. Compare rapid and slow palatal expansions. iii. Compared with rapid expansion, slow expansion pro-
Ans. duces more stable results and less relapse.
Section | I  Topic-Wise Solved Questions of Previous Years 171

Q.2. Rapid maxillary expansion. i. Expansion has a unique place among various methods
to gain the space in orthodontics like proximal strip-
Ans.
ping, extractions, distalization and uprighting of molars
i. The RME is also called ‘rapid palatal expansion’ and is and proclination of anteriors.
a skeletal type of expansion. ii. Types of expansions:
ii. Emerson C. Angell (1860) is the pioneer of RME. Three types of expansions are as follows:
Nowadays, it is an important form of orthopaedic a. Orthodontic/dentoalveolar expansion: pro-
therapy. duces dental expansion with no skeletal change
iii. It is indicated in the cases of unilateral or bilateral pos- b. Passive expansion: results from intrinsic forces
terior skeletal crossbites, narrow maxilla in certain exerted by the tongue
class II and class III cases of malocclusion and col- c. Skeletal/orthopaedic expansion: results from
lapsed maxillary arch due to cleft palate. splitting of mid-palatal suture
iv. It is also indicated in certain medical conditions like iii. Arch expansion is indicated in crossbite (unilateral/
poor nasal airway, allergic rhinitis and asthma and re- bilateral) cases and constricted arches.
current ear, nasal or sinus infections. iv. There are broadly of two types of expansion appliances,
v. Both removable and fixed types of appliances are used i.e. (i) maxillary and (ii) mandibular.
for rapid maxillary expansion, e.g. Isaacson type, Hyrax
Q.6. Give indications for rapid palatine expansion.
type, Derichsweiler type and Hass type.
Ans.
Q.3. Coffins spring.
[Ref LE Q.1]
Ans.
Q.7. Derichsweiler appliance.
i. Coffins spring is a slow expansion appliance introduced Ans.
by Walter Coffin.
ii. Design: i. Derichsweiler type of appliance is a type of banded
l The spring is made up of a 1.25-mm heavy stainless RME appliance.
steel wire. It consists of U- or omega-shaped loop ii. In this type of appliance, the screw is connected to the
positioned in the mid-palatal region. The distal ends bands by means of wire tags that are welded and sol-
of the U-loop are limited to the distal of the first dered to the palatal aspect of bands on first premolars and
permanent molar. first molars on one side and embedded in acrylic on the
l It is a continuous type of spring where both ends are palatal aspects of all nonbanded teeth except the incisors.
fixed to the base plate. iii. Acrylic adapts to the palate and is in two halves to per-
l Appliance is activated by expanding the appliance mit activation of screw.
manually by pulling the sides apart, first in the ante- Q.8. Hyrax screw.
rior region and then in the posterior region.
l An expansion of 2–3 mm is made during activation.
Ans.
iii. Indications: i. Hyrax screw is a type of screw used in fixed rapid max-
l Expansion of constricted maxillary arch and correc- illary banded RME expansion appliances.
tion of crossbite. ii. Hyrax or Biedermann type of RME uses Hyrax (hy-
l Conditions requiring differential expansions. gienic rapid expander) screw.
iv. Advantages: iii. Hyrax screws have heavy gauge wire extensions, which are
l Economical adapted to the palatal contour, welded and soldered to the
l Differential expansion of arch is possible palatal aspect of the bands on premolars and molars.
l Less bulky
Q.9. Quad helix appliance.
v. Disadvantage
l It is unstable if it is not made precisely. Ans.
Q.4. Expansion screws. i. Quad helix is one of the appliances used to expand a
narrow maxilla.
Ans.
ii. The quad helix incorporates four helices that increase
[Ref SE Q.2] the wire length, therefore the flexibility and range of
action of this appliance is more.
Q.5. Role of expansion as a method of gaining space.
iii. The appliance is constructed using 0.038-inch wire and
Ans. is soldered to bands on the first molars.
172 Quick Review Series for BDS 4th Year, vol 1

iv. The quad helix can be used to expand a narrow arch as Q.10. RME.
well as to bring about rotation of molars and brings
Ans.
about a slow dentoalveolar expansion.
v. It can be pre-activated by stretching the two molar [Same as SN Q.2]
bands apart prior to cementation or by using three
Q.11. Expansion devices.
prong pliers after cementation.
vi. When used in children during deciduous and early Ans.
mixed dentition periods, a skeletal mid-palatal splitting
[Same as SN Q.4]
can be achieved.

Topic 21
Extractions
COMMONLY ASKED QUESTIONS
LONG ESSAYS:
1 . Describe in detail extractions in orthodontics.
2. What are the reasons for extractions in orthodontics? Discuss the choice of teeth for extractions. [Same as LE Q.1]
3 . How will you plan extractions in orthodontic treatment? [Same as LE Q.1]
4 . Classify extractions in orthodontics. Write about therapeutic extractions in detail. [Same as LE Q.1]

SHORT ESSAYS:
1 . Describe the factors that justify extraction of teeth for treating malocclusion. [Ref LE Q.1]
2 . Wilkinson’s extractions. [Ref LE Q.1]
3 . Serial extractions.

SHORT NOTES:
1 . Extraction in orthodontics.
2 . Therapeutic extraction in orthodontics.
3 . Wilkinson’s extraction. [Ref LE Q.1]
4 . Planning extractions.
5 . Impacted tooth and its orthodontic correction.
6 . Impacted canines. [Same as SN Q.5]

SOLVED ANSWERS
LONG ESSAYS:
Q.1. Describe in detail extractions in orthodontics. [SE Q.1]
Ans. {Reasons for extraction
l Extractions are indicated for correction of crowding,
l In clinical orthodontics extractions form a main part
anteroposterior dental arch relations, vertical prob-
among all space gaining procedures.
lems, skeletal jaw deformities and presence of super-
l Calvin S. Case was pioneer of extraction philosophy in
numerary teeth as described in detail below.
orthodontics, which was later supported by Charles
i. Correction of crowding
Tweed.
l Crowding usually results from arch length

and tooth material discrepancy, hence prior to


Section | I  Topic-Wise Solved Questions of Previous Years 173

extraction a careful analysis of tooth size– Teeth Indications for extraction


arch length discrepancy should be performed.
II. Mandibu- i. Totally locked buccaly/lingually.
l In many cases the tooth material–arch length
lar incisors ii. Severe trauma, gingival recession or
disproportion cannot be treated by h the arch bone loss.
length. Hence, g of tooth material is the only iii. Severe arch length deficiency with fan-
alternative. ning of lower anteriors.
l In case of severe tooth material, arch length
iv. For correction of lower incisor crowding
in mild class II cases.
discrepancy extraction of one or more teeth is
undertaken to correct the malocclusion. III. Canines i. Ectopically erupted or unfavourably
ii. Correction of sagittal interarch relationship (seldom impacted.
extracted) ii. Totally blocked bucally or lingually.
l The cases of abnormal sagittal malrelation-
iii. Deciduous canine extracted as part of
ships like class II or class III malocclusion serial extraction.
may require extraction of teeth to achieve iv. Premature shedding of a deciduous ca-
normal sagittal interarch relation. nine usually indicates the extraction of
l In Angle’s class I cases: It is preferable to
its fellow on the opposite side of the arch
to restore symmetry.
carry out extractions in both arches. v. In class II cases if the lower deciduous
l In Angle’s class II: In most class II cases, it is canines are shed early, the upper decidu-
possible to reduce the abnormal maxillary ous canines should also be removed to
proclination by extracting only first premolars avoid worsening of the post normalcy
in the upper arch. (class II tendency).
vi. In class III cases if the upper deciduous
l In Angle’s class III: It is beneficial to avoid
canines are shed early, it may necessitate
extraction in upper arch. They are preferably the extraction of lower deciduous ca-
treated by extraction only in the lower arch or nines to avoid worsening of prenormalcy
by extraction in both arches. (class III tendency).
iii. Abnormal size and form of teeth IV. First i. To relieve moderate-to-severe crowding
Deformed teeth which interfere with normal oc- premolars U/L arch and proclination, in class II,
clusion necessitate their extraction in order to (teeth of division I or class I bidental protrusion.
choice for Reasons for the extraction of first premolars
achieve satisfactory occlusion, e.g. macrodontia,
extraction) are as follows:
severe hypoplastic teeth, dilacerations and abnor- l Their location in the arch is such that the
mal crown morphology. space gained by their extraction can be
iv. Skeletal jaw malrelations utilized for correction in both anterior
Respective surgical procedures along with ex- and posterior regions.
l The contact that results between canine
tractions may be required in correction of severe
and second premolar is satisfactory.
skeletal malrelationship of the jaws that may not
be satisfactorily treated using orthodontic appli- V. Second i. To relieve mild discrepancy of crowding
premolars and proclination.
ances alone.
ii. Unfavourably impacted.
v. Preservation of symmetry: iii. In open bite cases.
Extractions may be undertaken to correct any iv. If grossly decayed or have a questionable
asymmetry in dental arches.} prognosis.
The choice of teeth for extraction or indications for VI. First i. To correct mild crowding/proclination.
extraction molars ii. Grossly decayed with poor prognosis.
l Choice of teeth for extraction depends on various iii. In open bite cases to encourage deepen-
factors like condition of teeth, position of teeth and ing of bite.
position of crowding. VII. Second i. To prevent third molar impaction.
molars ii. To enable distalization of first molar.

VIII. Third No extraction for orthodontic purposes.


Teeth Indications for extraction molars
I. Maxillary i. Grossly carious and unrestorable incisors
incisors ii. Unfavourably impacted or totally Various extraction techniques are as follows:
(rarely blocked buccally/lingually
A. Wilkinson’s extraction
extracted) iii. In cases where one lateral is congenitally
missing, the other may be extracted to B. Balancing extraction
maintain arch symmetry C. Compensating extraction
iv. Malformations of incisor crowns/teeth D. Serial extraction
with dilacerated root E. Extractions in camouflage treatment
v. Trauma/irreparable damage to incisors by
F. Drift odontics
fracture
174 Quick Review Series for BDS 4th Year, vol 1

(SN Q.3 and SE Q.2) l These may be instances when the extraction of a
permanent tooth is not followed by orthodontic
{(A. Wilkinson’s extraction technique
6 6 treatment for varied reasons such as nonavailabil-
l Wilkinson advocated extraction of all
6 6 ity of specialist or point unsuitable for fixed ap-
between the age of 8½ and 9½ years
pliance therapy. These types of extractions are
l Basis for this extraction is the fact that the first
preferable.
permanent molars in children are highly suscep-
tible to caries. Q.2. What are the reasons for extractions in orthodon-
l Advantages: tics? Discuss the choice of teeth for extractions.
i. Impaction of third molars can be avoided by
Ans.
making the space available for their eruption.
ii. In general, crowding of arch is minimized, [Same as LE Q.1]
thereby lowering the risk of caries.
Q.3. How will you plan extractions in orthodontic treat-
iii. Makes it possible to maintain oral hygiene
ment?
effectively.
l Drawbacks: Ans.
i. The extraction of first molars offers limited
[Same as LE Q.1]
space for alleviation of crowding.
ii. The second biscuspids and second molars Q.4. Classify extractions in orthodontics. Write about
rotate and may tip into the extraction space. therapeutic extractions in detail.
iii. Deprivation of adequate anchorage for any
Ans.
orthodontic tooth movement.
iv. Improper contacts lead to accumulation of [Same as LE Q.1]
plaque and calculus, resulting in periodontal
problems.)}
SHORT ESSAYS:
B. Balancing extraction Q.1. Describe the factors that justify extraction of teeth
l Removal of teeth symmetrically on either side of for treating malocclusion.
the arch is known as balanced extraction.
Ans.
l Removal of tooth on one side of the arch results in

asymmetry; to prevent this, extractions have to be [Ref LE Q.1]


balanced to allow equal movement of remaining
Q.2. Wilkinson’s extractions.
teeth towards extraction site on both sides of the arch.
C. Compensating extraction Ans.
l Extraction of teeth in opposing jaws or arches is
[Ref LE Q.1]
known as compensating extraction.
l This type of extraction preserves interarch rela- Q.3. Serial extractions.
tionship and maintains lateral symmetry.
Ans.
D. Serial extraction
l Robert Bunon introduced the concept of serial l Robert Bunon introduced concept of serial extraction.
extraction. The term ‘serial extraction’ was coined The term ‘serial extraction’ was coined by Kjellgren.
by Kjellgren. l Extraction of certain deciduous and permanent teeth

l Extraction of certain deciduous and permanent in a sequence to alleviate crowding is known as serial
teeth in a sequence to alleviate crowding is known extraction.
as serial extraction. l Three popular methods of serial extraction are as

l Usual sequence of extraction is first primary ca- follows:


nines followed by primary first molars, then per- I. Dewel’s method
manent first premolars (C, D, 4). II. Tweed’s method
E. Drift odontics or extractions of permanent teeth III. Nance’s method
without appliance therapy
I. Dewel’s method: Dewel proposed a three-step serial
l Extraction of the lower first premolars is
4 4 extraction procedure as follows:
often associated with spontaneous decrowding of The sequence of the proposed extractions is C, D, 4.
lower anteriors. It is referred to as drift odontics Step 1: Extraction of ‘C’ – between 8 and 9 years to
and is less frequent in the upper arch. create space for alignment of incisors.
Section | I  Topic-Wise Solved Questions of Previous Years 175

Step 2: Extraction of ‘D’ – one year later, i.e. at problems, skeletal jaw deformities and presence of super-
10 years of age to accelerate eruption of first premolars. numerary teeth as per the need of the situation.
Step 3: Extraction of ‘4’ (first premolar) – to permit
Q.3. Wilkinson’s extraction.
the eruption of permanent canines in their place.
Modified Dewel’s technique: Where first premo- Ans.
lars are enucleated at the time of extraction of first
[Ref LE Q.1]
deciduous molar, especially in mandibular arch
where canines erupt before first premolars. Q.4. Planning extractions.
II. Tweed’s method: The sequence of the proposed
extraction: D, C, 4. Ans.
Step 1: Extraction of ‘D’ (deciduous first molar) – l In clinical orthodontics, extractions form a main part
at 8 years of age. among all space gaining procedures.
Step 2: Deciduous canines are maintained till pre- l Calvin S Case was the pioneer of extraction philosophy in
molars are in advanced eruptive stage. Then both orthodontics, which was later supported by Charles Tweed.
‘C’ (deciduous canines) and first premolars ‘4’ are l In planning extractions ‘think organized’, i.e. malocclu-
extracted simultaneously. sion should be analysed first in the anteroposterior plane,
III. Nance’s method: The sequence of the proposed then the vertical plane and finally in the transverse plane.
extraction: D, 4, C. l Extractions are indicated for correction of crowding,
This method is basically modified Tweed’s method. anteroposterior dental arch relations, vertical problems,
Step 1: Extraction of ‘D’ (deciduous first molars) – skeletal jaw deformities and presence of supernumerary
at 8 years of age. teeth as the situation exists.
Step 2: Extraction of ‘4’ (first premolars) and ‘C’ l Extraction should not be done if it affects soft tissue
(deciduous canines) simultaneously. balance.
Postserial extraction therapy: Most cases of serial
extraction need fixed orthodontic appliance therapy Q.5. Impacted tooth and its orthodontic correction.
for correction of axial inclination and detailing of Ans.
occlusion.
l Teeth are impacted due to variety of reasons like arch
length discrepancy, abnormal developmental position
SHORT NOTES: and deflection in the path of eruption.
Q.1. Extraction in orthodontics. l The possible methods of treatment for an impacted

tooth are as follows:


Ans.
a. Leave as it is
l In clinical orthodontics, extractions form a main part b. Extraction
among all space gaining procedures. c. Surgical exposure only
l Extractions are indicated for correction of crowding, d. Surgical exposure and orthodontic alignment
anteroposterior dental arch relations, vertical problems, l If the impacted tooth/canine is asymptomatic and well
skeletal jaw deformities and presence of supernumerary aligned, it can be left as it is with periodic follow-up.
teeth. l If the teeth are unfavourably positioned or shows signs
l Therapeutic extractions are extractions that are under- of pathology or causes resorption of adjacent teeth, then
taken as a part of orthodontic treatment. it should be extracted.
l Choice of teeth for extraction depends on various l If the impacted tooth is favourably positioned with un-
factors like condition of teeth, position of teeth and obstructed path of eruption and is well within the erup-
position of crowding. tive period, then only surgical exposure is enough.
l Premolars (most frequently extracted teeth) utilized for l If impacted tooth is malpositioned, having insufficient
correction of anterior 1 posterior segments. space available in arch on eruption or associated with
l Molars or lower incisors are also preferred. orthodontic problems, then it should be exposed surgi-
cally and mechanical approaches, e.g. Nitinol wires,
Q.2. Therapeutic extraction in orthodontics.
auxiliary arrangement of springs like PG springs and
Ans. magnets. Should be utilized to align the tooth in the arch.
l When teeth are extracted for orthodontic correction, it is Q.6. Impacted canines.
called therapeutic extraction.
Ans.
l In orthodontics, extractions are indicated for correction of

crowding, anteroposterior dental arch relations, vertical [Same as SN Q.5]


176 Quick Review Series for BDS 4th Year, vol 1

Topic 22
Orthodontic Appliances – General Principles
COMMONLY ASKED QUESTIONS
LONG ESSAYS:
1. What are the advantages and disadvantages of removable and fixed appliances?
2. What are the basic requirements of orthodontic appliances?
3. What are the indications and contraindications of removable and fixed orthodontic appliances? [Same as LE Q.1]
4. Define orthodontic appliances. Classify them. State the advantages of fixed and removable appliances.
[Same as LE Q.1]
5. Compare the merits and demerits of removable and fixed orthodontic appliances. [Same as LE Q.1]

SHORT ESSAYS:
1. Compare fixed and removable appliances.

SHORT NOTES:
1. Mention three ideal requisites of orthodontic appliance.

SOLVED ANSWERS
LONG ESSAYS:
Q.1. What are the advantages and disadvantages of Myofunctional appliances
removable and fixed appliances? They are loose fitting or passive appliances that harness
the natural forces of orofacial musculature which are
Ans.
transmitted to the teeth and alveolar bone through the
Orthodontic appliances are the devices by means of which medium of the appliance. They transmit, eliminate or
mild pressure may be applied to a tooth or a group of teeth guide the natural perioral muscle forces onto the denti-
and their supporting structures so as to bring about neces- tion. They do not contain active components and are
sary changes within the bone which will allow the tooth used for growth modification procedures.
movement. Removable appliances
Orthodontic appliances are broadly classified into two
groups: Advantages Disadvantages
i. Ability to maintain oral i. Patient cooperation is
Removable hygiene. needed.
ii. Usually used for simple ii. Inability to perform com-
A. Mechanical appliances
tooth movements that plex tooth movements.
Fixed can be brought about
by tipping.

Removable iii. Less chair side time of iii. The treatment duration
orthodontist to fabricate is prolonged in case of
B. Myofunctional appliances them, so they can handle severe malocclusion, as
more number of patients. whenever multiple tooth
Fixed movements are to be
carried out, it should be
done one at a time.
Mechanical appliances
They exert mild pressure on a tooth or a group of teeth iv. Simple movements like iv. Complex tooth movements
and their supporting structures in a predetermined direc- tipping and overbite like multiple rotations are
reduction can be difficult to treat using re-
tion with the help of active components like springs, undertaken. movable appliances.
elastics, screws etc. which are part of the appliance itself.
Section | I  Topic-Wise Solved Questions of Previous Years 177

Advantages Disadvantages a. Biological requirements


l The appliance should bring about the desired tooth
v. Lesser forces are used. So v. It is very difficult to close
the strain on anchor teeth residual space by forward
movements.
is lesser than fixed. movement of posterior l The appliance should not interfere with normal

teeth in cases of extraction. growth and function.


l The appliance should not produce pathologic changes
vi. Can be used by general vi. There is a great chance of
dental practitioners patient misplacing or such as root resorption, periodontal damage or non-
who have received basic damaging the removable vitality of the teeth.
training. appliances. l The appliance should not bring about sudden and

vii. Relatively economical vii. They cannot be used in unwanted tooth movements.
compared with fixed severe cases of class II l The material used in fabrication should be biocom-
appliances. and III malocclusions patible and should not produce toxic effects.
with unfavourable growth
b. Mechanical requirements
pattern.
l The appliance should be
viii. Damaged appliances that i. Simple to fabricate.
apply undesirable forces
ii. Strong enough to withstand masticatory forces.
can be removed by the
patient. iii. Able to deliver controlled force of desired inten-
sity, duration and direction.
iv. Universally applicable, i.e. must be able to han-
Fixed appliances dle various malocclusions.
l The appliance should not be bulky and should be
Advantages Disadvantages comfortable for the patient using it.
i. Fixed appliances offer bet- i. Oral hygiene mainte- c. Hygienic requirement: Appliance should be self-cleans-
ter control and to a large nance is more difficult. ing or easy to clean or should not interfere with oral
extent lessen the need for hygiene maintenance.
patient cooperation.
d. Aesthetic requirements: Appliance should be aestheti-
ii. It is easy to bring about ii. They take up more chair cally acceptable and should be inconspicuous as possible.
various types of tooth side time, as they are
movements, e.g. tipping, time-consuming to fix Q.3. What are the indications and contraindications of
bodily movement, rotation, and adjust. removable and fixed orthodontic appliances?
intrusion and extrusion.
Ans.
iii. As multiple tooth move- iii. Aesthetically unpleasing
ments are possible simulta- unless modern tooth-co- [Same as LE Q.1]
neously, the treatment dura- loured appliances/lingual
tion is considerably reduced. orthodontics are used. Q.4. Define orthodontic appliances. Classify them. State
iv. More precise tooth move- iv. Requires special training the advantages of fixed and removable appliances.
ments and correction of of the operator and is in-
occlusion is possible using variably handled by spe-
Ans.
fixed appliances. cialized orthodontists. [Same as LE Q.1]
v. Fixed appliances are used v. Damaged appliances
to treat most malocclu- that apply misdirected
Q.5. Compare the merits and demerits of removable
sions, including very forces cannot be re- and fixed orthodontic appliances.
complicated ones. moved by the patient.
Ans.
vi. These appliances offer bet- vi. More expensive.
ter control over anchorage. [Same as LE Q.1]

Q.2. What are the basic requirements of orthodontic SHORT ESSAYS:


appliances?
Q.1. Compare fixed and removable appliances.
Ans.
Ans.
Ideal requirements of orthodontic appliances are catego-
rized under the following headings: Fixed appliances Removable appliances
a. Biological requirements i. Fixed appliances offer bet- i. Patient cooperation is
b. Mechanical requirements ter control, and to a large needed to a large extent.
c. Aesthetic requirements extent lessen the need for
patient cooperation.
d. Hygienic requirements
ii. Oral hygiene maintenance ii. Oral hygiene maintenance
is more difficult. is easy.
178 Quick Review Series for BDS 4th Year, vol 1

Fixed appliances Removable appliances SHORT NOTES:


iii. As multiple tooth move- iii. The treatment duration Q.1. Mention three ideal requisites of orthodontic appli-
ments are possible simul- is prolonged in case of ance.
taneously, the treatment severe malocclusion as
duration is considerably whenever multiple tooth Ans.
reduced. movements are to be car-
ried out it should be done Ideal requirements of an orthodontic appliance are as
one at a time. follows:
iv. They take up more chair iv. Less chair side time of l The appliance should bring about only desired tooth

side time as they are orthodontist to fabricate movements.


time-consuming to fix them, so they can handle l It should not interfere with normal growth and function.
and adjust them. more number of patients. l It should be simple to fabricate but strong enough to

v. More expensive. v. Relatively economical withstand masticatory forces.


compared to fixed l It should not be bulky and should be comfortable for the
appliances. patient in using it.
vi. Requires special training vi. Requires no special train- l It should be self-cleansing or easy to clean.
of the operator and are ing, can be used by gen- l It should be aesthetically acceptable.
invariably handled by eral dental practitioners
specialized orthodontists. who have received basic
training.

Topic 23
Removable Orthodontic Appliances
COMMONLY ASKED QUESTIONS
LONG ESSAYS:
1 . Discuss the advantages, disadvantages and indications for using removable orthodontic appliance.
2 . Define and classify orthodontic appliances. Discuss how the various types of tooth movements are achieved using
removable appliances.
3 . Enumerate ideal properties of an orthodontic appliance and describe various types of canine retractors.
4 . Classify orthodontic appliances and name the basic components of a removable appliance.
5 . What are the various active components of removable orthodontic appliance? Write about springs.
6 . Discuss the base plate used in orthodontics.
7 . Define orthodontic appliances. Explain various mechanical factors to be considered in designing a removable
orthodontic appliance.
8 . Discuss the treatment of different types of malocclusion with removable appliances. [Same as LE Q.2]
9 . Classify orthodontic appliances and explain the retentive components of removable orthodontic appliances and
Adams clasp in detail. [Same as LE Q.4]

SHORT ESSAYS:
1 . Give classification of canine retractors. [Ref LE Q.3]
2 . Orthodontic appliance-labial bows.
3 . Hawley’s appliance.
4 . Describe components/parts of removable appliances and write about ideal requirements of springs. [Ref LE Q.4]
5 . Enumerate various types of clasps used in an orthodontics appliances and the ideal requirements of the clasp.
6 . Advantages of removable orthodontic appliances.
7 . Adams clasp and its advantages.
8 . Springs used for distalizing the canines. [Same as SE Q.1]
9 . Hawley’s retainer. [Same as SE Q.3]
Section | I  Topic-Wise Solved Questions of Previous Years 179

SHORT NOTES:
1. Z-spring. [Ref LE Q.5]
2. Crozat clasp.
3. Adams clasp.
4. Labial bow. [Ref SE Q.2]
5. Classify canine retractors.
6. Roberts retractor.
7. Orthodontic springs – mention one classification.
8. Define removable orthodontic appliance and name its components.
9. Mention three ideal requirements of orthodontic appliance. [Ref LE Q.3]
10. Name few self-supporting springs. [Ref LE Q.5]
11. Finger spring. [Ref LE Q.5]
12. Coffin spring. [Ref LE Q.5]
13. High labial bows.
14. Arrowhead clasp.
15. Double cantilever spring.
16. Types of labial bow and their uses.
17. Mention few active plates.
18. Buccal canine retractors.
19. Adams clasp – modification.
20. Disadvantages of removable orthodontic appliances.
21. Orthodontic clasp – ideal requirements.
22. Cantilever spring.

SOLVED ANSWERS
LONG ESSAYS:
Q.1. Discuss the advantages, disadvantages and indica- Advantages Disadvantages
tions for using removable orthodontic appliance.
vi. General dental practi- vi. The chances of patient mis-
Ans. tioners who have re- placing or damaging the re-
ceived adequate train- movable appliances is
The advantages and disadvantages of removable ing can use them. more.
orthodontic appliances are as follows: vii. Less inventory is re- vii. Patients should have
quired for fabrication. enough skill to remove and
Advantages Disadvantages replace the appliance with-
i. Ability to maintain oral i. The need for patient coop- out distorting them.
hygiene. eration. viii. Relatively more eco- viii. Severe cases of class II and
ii. Usually used for simple ii. Inability to perform com- nomical than fixed ap- III malocclusions with unfa-
tooth movements that plex tooth movements. pliances. vourable growth pattern
can be brought about cannot be treated using
by tipping. removable appliances.

iii. Less chair side time of iii. In case of severe malocclu- ix. As they take less chair ix. They require more regular
orthodontist to fabri- sions, the treatment dura- side time and are more monitoring.
cate them, so they can tion is prolonged, as when- economical, they can
handle more number ever multiple tooth be used in community-
of patients. movements are to be car- based programmes,
ried out it should be done wherein large number
one at a time. of patients are treated.

iv. Various movements iv. It is difficult to treat multiple x. They are less conspicu-
like tipping, overbite rotations using removable ous and aesthetically
reduction can be un- appliances. more pleasing when
dertaken. compared with fixed
appliance.
v. Lesser forces are used. v. In cases requiring extraction,
So the strain on anchor it is very difficult to close xi. The patient can remove
teeth is lesser than residual space by forward damaged appliances
fixed. movement of posterior that apply undesirable
teeth. forces by himself.
180 Quick Review Series for BDS 4th Year, vol 1

The various types of malocclusions that can be treated by iv. Generalized anterior spacing:
removable mechanical appliances are as follows: l High labial bow with apron spring.

i. Mild and moderate proclination l Hawley appliance with the base plate

ii. Anterior and posterior crossbite trimmed on the palatal aspect.


iii. Median diastema v. Anterior crossbite:
iv. Generalized anterior spacing l Hawley appliance with posterior bite plane

v. Buccally placed canines and premolars and Z-spring to correct the tooth in crossbite.
vi. Palatally placed canines l Upper anterior expansion (Schwarz appli-

vii. Deep overbite ance) with posterior bite plane.


viii. Anterior open bite l Inclined plane if the bite is deep.

vi. Posterior crossbite:


Q.2. Define and classify orthodontic appliances. Discuss
l Schwarz-type lateral expansion with poste-
how the various types of tooth movements are achieved
rior bite plane for occlusal clearance.
using removable appliances.
l Hawley appliance with posterior bite plane

Ans. and T-springs or Z-springs to correct the


tooth in crossbite.
Orthodontic appliances are defined as devices by means of
vii. Buccally placed canine:
which mild pressure may be applied to a tooth or a group of
l Buccal canine retractor when palatal and
teeth and their supporting structures so as to bring about
distal movement is required.
necessary changes within the bone which will allow the
l Helical loop canine retractor when the sulcus
tooth movement.
depth is shallow.
Orthodontic appliances are broadly classified into two
viii. Buccally placed premolar:
groups:
l Spring for palatal movement of premolars.

ix. Palatally placed canine:


Removable l Hawley appliance with Z-spring and poste-

A. Mechanical appliances rior bite plane when only buccal movement


is required.
Fixed l Palatal canine retractor with posterior bite

Removable plane when both distal and buccal move-


ments of canine are required.
B. Myofunctional appliances
x. Deep overbite:
Fixed l Upper Hawley’s appliance with flat anterior

bite plane.
xi. Anterior open bite:
Mechanical appliances: l In cases of open bite caused due to habits,
They exert mild pressure on a tooth or a group of teeth upper Hawley’s appliance with tongue spikes
and their supporting structures in a predetermined direc- is advised and posterior bite planes to intrude
tion with the help of active components which are part molar.
of the appliance itself.
Treatment of different types of malocclusions by re- Q.3. Enumerate ideal properties of an orthodontic ap-
movable mechanical appliances is as follows: pliance and describe various types of canine retractors.
i. Mild proclination: Ans.
l Hawley’s appliance

l Hawley appliance with long labial bow if

there is space distal to canine {SN Q.9}


ii. Moderate proclination:
l Roberts retractor Ideal requirements of orthodontic appliances are catego-
iii. Median diastema: rized as follows:
l Upper Hawley appliance with two finger a. Biological requirements
springs to move both the central incisors me- l The appliance should bring about only desired

sially towards each other tooth movements without any sudden and un-
l Upper plate with split-type labial bow wanted tooth movements.
l Simple Hawley appliance if the median dia- l The appliance should not interfere with normal

stema is due to proclination growth and function.


Section | I  Topic-Wise Solved Questions of Previous Years 181

l They are used where a buccally placed


l The appliance should not produce pathologic
canine has to be moved both palatally
changes such as root resorption, periodontal dam-
and distally.
age or nonvitality of the teeth.
Spring design:
l The material used in fabrication should be bio-
l It is made up of 0.7-mm or 21-gauge
compatible and should not produce toxic effects.
stainless steel (SS) wire.
b. Mechanical requirements
l It consists of a coil of 3 mm diameter, an
l The appliance should be simple to fabricate and
active arm and a retentive arm.
strong enough to withstand masticatory forces.
l Coil is placed as high as possible in be-
l It should be able to deliver controlled force of
tween the present and future positions of
desired intensity, duration and direction.
canine, i.e. distal to long axis of canine.
l The appliance should not be bulky and should be
l The end of the spring is bent at right
comfortable to the patient.
angle to the canine to be moved and then
c. Hygienic requirement
it is shaped to the tooth.
l Appliance should be self-cleansing or easy to
l Tag should cross over the mesial side of
clean or should not interfere with oral hygiene
second premolar.
maintenance.
Modifications:
d. Aesthetic requirements
l There are two modifications of buccal
l Appliance should be aesthetically acceptable.
canine retractor:
l With more number of adults seeking orthodontic
a. Supported or sleeved
treatment, the appliance should be acceptable
b. Self-supported or stabilized
aesthetically.
l The supported canine retractors are made
l Should be less visible.
of thinner gauge wire (0.5 mm). Thus,
e. Cost factor
they are more flexible and therefore me-
l Appliance should be economical for the patient.
chanically efficient. As they lack stabil-
l It should not be expensive.
ity, they are enclosed in SS tubing.
l The self-supported canine retractors are
[SE Q.1] made of thicker gauge wire (0.7 mm) so
that the spring can support itself.
{Canine retractors Activation of buccal canine retractor:
l Canine retractors are springs that are used for distal
l It is one of the few springs where the coil
movement of canines.
is closed for activation.
l The canine retractors can be classified by a number
l The self-supported canine retractors are
of ways as follows:
activated by closing the helix 1 mm at a
i. Based on location:
time, while the supported canine retrac-
(a) Buccal – buccally placed
tors can be activated up to 2 mm at a time.
(b) Palatal – palatally placed
l Activation is done by using ‘hollow
ii. Based on presence of helix or loop:
chop’ pliers like Mathews or Andresen’s.
(a) Canine retractor with helix
b. Reverse loop canine retractor or helical loop
(b) Canine retractor with loop
canine retractor
iii. Based on the mode of action:
Indications:
(a) Push type
It is indicated in patients with shallow sul-
(b) Pull type
cus, and especially in the mandibular arch.
l Types of canine retractors based on placement posi-
Spring design:
tion are as follows:
l It is made up of 0.7-mm/21-guage SS
i. Buccally placed:
wire.
a. Buccal canine retractor
l It consists of a helix or coil of 3 mm
b. Helical loop canine retractor
diameter, an active arm and a retentive
c. U-loop canine retractor
arm.
ii. Palatally placed:
l The mesial arm or retentive arm is
a. Palatal canine retractor}
adapted between the premolars.
a. Buccal canine retractor l The distal arm is active and is bent at
Indications: right angles to engage the canine below
l It is indicated in cases of buccally placed the height of contour.
canines and canines placed high in the l The coil is placed 3–4 mm below the
vestibule. gingival margin. The height of the coil
182 Quick Review Series for BDS 4th Year, vol 1

can be adjusted based on the vestibular height. l Tension can be given to the spring by
Drawbacks: squeezing the coil with the tip of the
It is stiff in the horizontal plane and unstable ver- pliers.
tically.
Q.4. Classify orthodontic appliances and name the basic
Activation:
components of a removable appliance.
There are two methods of activation.
It is activated either by opening the helix by 1 mm Ans.
or by cutting 1 mm of the free end and readapting
Classification of Orthodontic Appliances
it around the canine.
c. U-Loop canine retractor
Broadly two groups
l It is made up of 0.7-mm/21-guage SS wire.

Indications:
l Mechanically, it is least effective and is indi-
1. Mechanical appliances 2. Myofunctional appliances
cated when minimal distal retraction of 1–2
mm of canine is required.
Spring design: i. Removable
l It consists of a U-loop, an active arm and a ii. Fixed
retentive arm which is distal. The base of the
U-loop should be 2–3 mm below the cervical
margin. The mesial arm of the U-loop is bent Mechanical appliances:
at right angles and adapted around the canine Exert mild pressure on a tooth or a group of teeth and
below its mesial contact point. their supporting structures in a predetermined direc-
Disadvantage: tion with the help of active components which are
This is the least efficient of all the canine retrac- part of the appliance itself. The active components
tors. may include springs, elastics, screws, etc.
Advantages: Myofunctional appliances:
l Ease in fabrication, simple in design and less They are loose fitting or passive appliances that har-
bulky. ness the natural forces of the orofacial musculature
Activation: which are transmitted to the teeth and alveolar bone
l It is activated by closing the loops by 1–2 mm through the medium of the appliance. They transmit,
or cutting the free end of the active arm by 2 eliminate or guide the natural perioral muscle forces
mm and readapting it. onto the dentition. They do not contain active compo-
a. Palatal canine retractors: nents and are mainly used for growth modification
l This is similar to finger springs and is made up of procedures.
0.6-mm SS wire.
[SE Q.4]
Indication:
It is indicated in retraction or distalization of pala- {Components of removable orthodontic appliance
tally placed canines. There are three basic components in a removable orth-
Spring design: odontic appliance:
l It consists of an active arm, a guide a. Retentive components
arm or retentive tag and a coil of 3-mm diam- b. Active components
eter. c. Base plate
l The active arm is placed mesial to canine. The a. Fixation or retention components
helix/coilis is placed along the long axis of the i. Clasps
canine and as far away as possible to have re- ii. Bows
tractor a good range of action. Retentive tag b. Active components
gets embedded in the acrylic plate. i. Springs
Activation: ii. Labial bows
It is done by either of the following: iii. Screws
l Opening the coil/helix by 2–3 mm at a time. iv. Elastics
l Pulling the free arm of the spring slightly c. Anchorage
away from the point of emergence from i. Clasps
coil. ii. Contact of base plate with nonmoving part
Section | I  Topic-Wise Solved Questions of Previous Years 183

iii. Headgears Requirements of an ideal clasp


iv. Intermaxillary elastics
d. Base plate: Offer adequate retention
Forms the framework}
Retentive components: Offer adequate retention even in the presence
l The retentive components help in keeping the ap- of shallow undercuts
pliance in place and resist displacement of the It should
appliance. Permit usage in both fully-erupted and
l The success of a removable appliance is to a large
partial erupted teeth
extent dependent upon good retention of the
appliance. Be easy to fabricate
l The wire components that aid in retention of a

removable appliance are called clasps. By itself apply any active force that would
l Mode of action of clasps: They act by engaging bring about undesirable tooth movements
the undercuts of the teeth. of an old age tooth
The various clasps can be classified under two broad It should not Impinge on soft tissues
headings:
A. Free-ended clasps in which one end of the
clasp is embedded in acrylic
Examples: Interfere with normal occlusion
l C-clasp

l Triangular clasp
CLASPS
l Ball end clasp
1. Circumferential i. It is also known as three-quarter
l Duyzing’s clasp
clasp (Fig 23.1) clasp or ‘C’ clasp.
l Crozat clasp

l Arrow pin clasp

l Wrought Roach clasp

l Visick’s clasp

l Lingual extension clasps

B. Continuous clasp in which both ends of the Fig. 23.1  C-clasp.


clasp are embedded in acrylic
Examples: ii. They are simple clasps that are de-
l Jacksons clasp
signed to engage the bucco-cervical
undercut.
l Arrowhead clasp
iii. Advantage: Simplicity of design and
l Eyelet clasp fabrication.
l Adams clasp iv. Disadvantage: It cannot be used in
l Delta clasp partially erupted teeth, wherein the
l Southend clasp
cervical undercut is not available for
clasp fabrication.
Types of clasps based on the undercuts used
I. Clasps using mesial/distal undercuts: 2. Jacksons clasp i. It is also called full clasp or ‘U’-clasp.
(Fig 23.2)
l Adams clasp

l Triangular clasp

l Ball end clasps


l Arrowhead/Schwarz clasp

l Crozat clasp

II. Using buccal/lingual undercuts: Fig. 23.2  Jackson’s clasp.


l Jacksons clasp

l Southend clasp ii. It engages all undercuts, i.e. buccal,


cervical, mesial and distal.
l Duyzing’s clasp
iii. Wire is adapted along the bucco-
III. Using both the proximal and buccal lin- cervical margin and both the proximal
gual undercuts: undercuts, and then carried over the
l ‘C’ clasp occlusal embrasures to end as reten-
tive arms on both sides of the molar.
iv. Advantage: It is simple to construct,
and offers adequate retention.
v. Disadvantage: It offers inadequate
retention in partially erupted teeth.
184 Quick Review Series for BDS 4th Year, vol 1

3. Southend This clasp is used when retention in ante- Adams clasp (Fig 23.8)
clasp (Fig 23.3) rior region is required. l It was described by Professor Philip Adams.

l It is also known as Liverpool clasp, universal clasp or

modified arrowhead clasp.


l Constructed using 0.7-mm round SS wire.

l Parts of Adams clasp are as follows:

a. Two arrowheads – engage mesial and distal


Fig. 23.3  Southend clasp. undercuts.
b. Bridge – connects to arrowheads and is at 45° to
4. Triangular i. It is used between two adjacent
long axis of tooth.
clasp (Fig 23.4) posterior teeth.
ii. It indicated when additional reten- c. Two retentive arms.
tion is needed. Advantages:
l It is rigid and offers excellent retention.

l Can be fabricated on both deciduous and perma-

nent teeth.
l Can be used on partially or fully erupted teeth.

l Can be used on molars, premolars and incisors.

l No specialized instrument is needed to fabricate

the clasp.
Fig. 23.4  Triangular clasp. l It is small and occupies minimum space.

l The clasp can be modified in a number of ways.


5. Ball end clasp i. Preformed wires with ball at one end
(Fig 23.5) are available. Modifications to Adams clasp are as follows:
Or l Adams with single arrowhead.
Ball can be made at the end of the l Adams with J-hook.
wire with silver solder.
l Adams with incorporated helix.
ii. Indicated when additional retention
l Adams with additional buccal tube.
is required.
l Adams with soldered buccal tube.

l Adams with distal extension.

l Adams on incisors and premolars.

i. Adams with single arrowhead:


l Usually indicated in partially erupted tooth.
Fig. 23.5  Ball end clasp. l In last erupted molar, single arrowhead is made

6. Schwarz clasp Predecessor of Adams clasp. A number of to engage the mesioproximal undercut of tooth.
(Fig 23.6) arrowheads engage interproximal under- l Bridge is modified to encircle the tooth distally
cuts between molars and premolars. and ends on palatal aspect as a retentive arm.
Drawbacks:
i. Special pliers are required.
ii. Difficult and time-consuming to
fabricate.
iii. Large amount of space in buccal
vestibule.
iv. Injures interdental soft tissue.

Fig. 23.8  Adams with single arrow head.

Fig. 23.6  Schwarz clasp. ii. Adams with J-hook (Fig 23.9):
J-hook can be soldered onto bridge of the Adams
7. Crozat clasp i. It resembles full clasp with additional
clasp which is useful in engaging elastics.
(Fig 23.7) piece of wire soldered which en-
gages mesial and distal undercuts.

Fig. 23.7  Crozat clasp.

8. Adams clasp It is also known as Liverpool clasp, uni-


Fig. 23.9  Adams with J-hook.
versal clasp or modified arrowhead clasp.
Section | I  Topic-Wise Solved Questions of Previous Years 185

iii. Adams with incorporated helix (Fig 23.10): vii. Adams on incisors and premolars (Fig 23.14):
A helix can be incorporated in the bridge of the Adams clasp can be fabricated in the incisors and
Adams clasp to help in engaging elastics. PMs when retention in those areas is required.
They may span over single tooth or two teeth.

Fig. 23.10  Adams with incorporated helix.

Fig 23.14  Adams on incisors and premolars.


iv. Adams with additional arrowhead (Fig 23.11):
This additional arrowhead engages the proximal Q.5. What are the various active components of re-
undercut of adjacent tooth and is soldered onto movable orthodontic appliance? Write about springs.
the bridge of the Adams. This type of clasp of- Ans.
fers additional retention.
The active components of removable orthodontic appli-
ances are as follows:
i. Bows
ii. Screws
iii. Elastics
iv. Springs
i. Bows
Fig. 23.11  Adams with additional arrow head. l Bows are active components of removable orthodontic

appliances which are mostly used for incisor retraction.


v. Adams with soldered buccal tube (Fig 23.12):
l There are various types of bows used routinely in
Buccal tube is soldered to the bridge of the Ad-
orthodontics. They are as follows:
ams clasp. It permits the use of extraoral an-
Short labial bow, long labial bow, split labial bow, re-
chorage using face bow head area assembly.
verse labial bow, Roberts retractor, Mills retractor, etc.
ii. Screws
l Screws are active components that can be incorpo-

rated in a removable appliance.


l The removable appliances that make use of screws

can bring about three types of tooth movements:


a. Arch expansion
b. Movement of a group of teeth in a buccal or labial
direction
c. Movement of one or more teeth in a distal or me-
Fig. 23.12  Adams with soldered buccal tube. sial direction
iii. elastics
l Elastics as active components are rarely used along
vi. Adams with distal extension (Fig 23.13):
Distal arrowhead of Adams clasp has a small with removable appliances.
l Removable appliances using elastics for anterior re-
extension incorporated distally which helps in
engaging elastics. traction generally make use of a labial bow with
hooks placed distal to the canines.
iv. Springs
l Springs are active components of removable appli-

ances.
l There are various methods of classification of

springs.
I. Classification of springs based on their ability to
withstand forces of distortion:
a. Self-supported springs
i. Buccal canine retractor
Fig. 23.13  Adams with distal extension. ii. U-loop canine retractor
186 Quick Review Series for BDS 4th Year, vol 1

iii. Helical loop canine retractor l Activation is done by using ‘hollow chop’
iv. Coffin springs pliers like Mathews or Andresen’s.
b. Guided springs b. Helical loop canine retractor or reverse loop
i. Cantilever springs canine retractor
ii. Finger springs l It is indicated in patients with shallow sulcus,

iii. Palatal canine retractor and especially in the mandibular arch.


iv. T-springs l It is made up of 0.7-mm/21-guage SS wire.

c. Auxiliary springs l It consists of a helix or coil of 3-mm diameter,

i. Apron springs an active arm and a retentive arm.


II. Based on their point of attachment, springs are l The coil is placed 3–4 mm below the gingival

classified as follows: margin. The height of the coil can be adjusted


a. Free-ended springs based on the vestibular height.
i. Cantilever springs l It is stiff in the horizontal plane and unstable

ii. Finger springs vertically.


iii. Canine retractors l It is activated either by opening the helix by

iv. T-springs 1 mm or by cutting 1 mm of the free end and


b. Springs attached at both ends readapting it around the canine.
i. Labial bows c. U-loop canine retractor
ii. Coffin springs l It is made up of 0.7-mm/21-guage SS wire.
c. Accessory springs attached to arches l It consists of a U-loop, an active arm and a
i. Apron springs retentive arm which is distal. The base of the
III. Based on the presence of loops or helix, springs are U-loop should be 2–3 mm below the cervical
classified as follows: margin.
a. Helical springs l Mechanically, it is least effective and is indi-
b. Looped springs cated when minimal distal retraction of 1–2 mm
The detail description of some commonly used springs is of canine is required.
as follows: l Ease in fabrication, simple design and less bulky.

l It is activated by closing the loops by 1–2 mm

or cutting the free end of the active arm by


{SN Q.10}
2 mm and readapting it.
Self-supported springs
l The springs which can resist on their own the {SN Q.12}
distortion forces are known as self-supported
springs. ii. Coffin springs
l It is a removable type of arch expansion spring
They are made of 0.7-mm or 0.9-mm hard SS wire.
They usually have a stability ratio of 1. introduced by Walter Coffin.
l It is used to bring about slow dentoalveolar
l The various springs that fall under the category of

self-supported springs are arch expansion in patients with constricted up-


i. Canine retractors per archor unilateral crossbite.
l It is made of 1.2-mm hard round SS wire. It
a. Buccal canine retractor
b. Helical loop canine retractor consists of a U- or omega-shaped wire placed
c. U-loop canine retractor in the mid-palatal region.
l It can be activated 1–2 mm at a time manually
ii. Coffin springs
by holding both the ends at the region of the
clasps and pulling the sides gently apart.
i. Canine retractors
a. Buccal canine retractor Guided springs
l It is indicated in cases of buccally placed ca- Guided springs are those that cannot resist distortion
nines and canines placed high in the vestibule. on their own. They are usually made of 0.5-mm
l It is made up of 0.7-mm or 21-gauge SS wire. wires.
l It consists of a coil of 3-mm diameter, an active i. Cantilever springs
arm and a retentive arm. a. Single cantilever spring
l It is one of the few springs where the coil is l The spring is constructed with 0.5-mm hard
closed for activation. SS wire.
Section | I  Topic-Wise Solved Questions of Previous Years 187

l A single cantilever spring consists of three l Spring design consists of an active arm, a
parts: guide arm or retentive tag and a 3-mm diame-
l Retention tag is embedded in the acrylic ter coil.
resin of the base plate. l Activation is done by either opening the coil/

l Coil is the active part of the spring. helix by 2–3 mm at a time or pulling the free
l Active arm is in contact with the tooth to arm of the spring slightly away from the point
be moved. of emergence from coil.
l The spring is activated by opening the coil. iv. T-springs
l First visit: Activated by 1–2 mm. l T-spring is used to bring about the buccal move-

l Subsequent visits: Activated by 2–3 mm. ment of premolars and sometimes canines.
b. Double cantilever spring or Z-spring l It is made of 0.5-mm hard round SS wire.

l The spring consists of a T-shaped arm whose

ends are embedded in acrylic. Loops can be


{SN Q.1}
incorporated in both arms of the T so that as
The Z-spring is made of 0.5-mm/23-gauge hard round the tooth moves buccally, the head of the T can
SS wire. be made to remain in contact with the crown
l The spring consists of two coils of very small inter- by slightly opening the loops.
nal diameter. The spring should be perpendicular to l The spring is activated by pulling the free end
the palatal surface of the tooth. of the T towards the intended direction of tooth
l Indications: movement.
i. When both labial and lateral movements of the Auxiliary springs
incisors are required. l They are also known as Apron springs.
ii. Minor rotation correction. l Made up of 0.35-mm to 0.40-mm SS wire.
iii. When two or more teeth have to be moved labially. They are used in correction of extreme proclination
l The Z-spring is activated by opening both the helices of incisors to move them lingually and also to correct
by about 2–3 mm at a time. In the case of minor rota- single-tooth proclination.
tion correction, one of the helices is opened.
Q.6. Discuss the base plate used in orthodontics.
Ans.
{SN Q.11} l Base plate forms the framework of removable orthodon-
ii. Finger springs tic appliance and serves to hold all the components of
l The finger spring is also called single cantilever the appliance together.
spring as one end is fixed in acrylic and the other l Usually base plates are made either from cold-cure or

end is free. heat-cure acrylic.


l It is made of 0.5-mm or 0.6-mm hard round SS l Cold-cure acrylic is commonly used because it is

wire. simple to process and chances of thermal distortion


l It is used for mesiodistal movement of teeth only are less. But the heat-cure acrylic is stronger.
when they are located correctly within the line of l Clear acrylic resin is preferred because any pressure

the arch. spots can be visualized by the presence of blanching


l The finger spring consists of an active arm, coil or with the appliance.
helix and a retention tag. The uses of base plate in removable appliances are as
l The finger spring is activated by moving the ac- follows:
tive arm towards the teeth intended to be moved. a. As a base of operation:
This is done as close to the coil as possible. Acti- i. Helps to unite all the components of the appli-
vation of up to 3 mm is considered ideal when ance into a single unit.
0.5-mm wire is used for its fabrication. Whenever ii. Provides support for the wire/screw components.
0.6-mm wire has been used the activation should ii. Transmits forces from the active components
be half of that. and distributes over a large area.
iii. Protects the palatal springs.
iv. Facilitates movement, e.g. posterior bite-
iii. Palatal canine retractor blocks.
l This is similar to finger springs and is made b. As anchorage:
up of 0.6-mm SS wire. i. Prevents unwanted movement of teeth.
l It is indicated in retraction or distalization of ii. Contacts with teeth and palate and helps in
palatally placed canines. anchoring the appliance.
188 Quick Review Series for BDS 4th Year, vol 1

c. As an active component: teeth and their supporting structures to bring about nec-
i. Split plate. essary changes within the bone which will allow the
ii. Bite planes can be incorporated into plate to tooth movement.
treat certain orthodontic problems. l Orthodontic arch wires or springs can be considered as

iii. Upper anterior inclined plane. beams supported either on one side or both sides.
Dimensions of a base plate: l Wires or appliances supported on one side act as canti-

l The base plate should be of minimum thickness to lever beams, e.g. springs projecting from the removable
help in patient acceptance. appliance. Appliances supported on both sides include
l Base plates should not be made unduly thick. labial bows and arch wire.
Single thickness of wax shall be used and the base l When a force is applied to a beam, its response can be

plate is thickened over the wire tags only. analysed as follows:


l Base plates of 1.5–2-mm thickness offer adequate Force:
strength and, at the same time, are well tolerated l It is an act or load applied to an object which

by patients. tends to change the position of object.


l Maxillary and mandibular plates extend up to l Force delivered for a given deflection depends on the

distal of first permanent molar. wire length (L), radius (r) and elastic modulus (E)
l The mandibular base plate is usually shallow to
avoid irritation to the lingual sulcus. For this rea- Er 4
F
son it should be made thicker to increase the L3
strength.
If the size or diameter of wire is doubled, it increases
l The base plate should fit snugly around the necks
the stiffness by 16 times.
of teeth that are not being moved. This helps in
Increasing the length by two times reduces the stiff-
avoiding food accumulation under the base plate.
ness by eight times.
l Bite planes can also be incorporated into the base
Stress: It is defined as force per unit area in a body
plates. These bite planes help in disengaging the
which resists an external force.
occlusion.
Strain: It is defined as the internal distortion produced
Anterior bite planes:
by load or stress.
l Anterior bite planes are used to treat the deep bite
Elastic limit: It is defined as the greatest stress to which
cases.
a material can be subjected to so that it will return to its
l They are also used for relieving occlusal interfer-
original dimension when the forces are released.
ence.
Proportional limit: It is the point at which permanent
l They are fabricated by thickening the base plate
deformation is first observed.
behind the maxillary anteriors.
Yield strength: It is the point at which 0.1 % of deforma-
l By selective eruption of the posterior teeth rela-
tion is observed.
tive to the anteriors, opening of bite occurs, bite
Ultimate tensile strength: It is the maximum load a wire can
plates are trimmed and then labial bow is acti-
sustain. This determines the maximum force a spring can
vated for lingual movement of teeth.
deliver.
Posterior bite planes/molar capping:
Springiness: It depends on the elastic or proportional limit.
l Posterior bite planes are formed by extending the
Range: It is defined as the distance the wire will bend
base plate to cover the occlusal surface of the
elastically before permanent deformation occurs.
teeth.
Resilience of the wire: It is the area under stress–strain
l The thickness of bite plane should be just enough
curve up to proportional limit. It represents the me-
to clear the occlusion.
chanical energy stored in the wire. It is a combination of
l Posterior bite planes are generally used in the
strength and springiness.
treatment of crossbites as they help in removing
Formability: It is the amount of permanent deformation
the interference of opposing teeth.
a wire can withstand before it breaks.
l It is used to get occlusal clearance to tooth move-
Fatigue: The fracture of the wire due to repeated stress
ment during correction of anterior crossbite.
is called fatigue.
Q.7. Define orthodontic appliances. Explain various Incorporating a coil: Introduction of a coil into a cantilever
mechanical factors to be considered in designing a increases the length of the spring and thereby its flexibility.
removable orthodontic appliance. Burstone enumerated three important features of an orth-
odontic appliance:
Ans.
i. Moment to force ratio
l Orthodontic appliances are devices by means of which ii. Load deflection rate
mild pressure may be applied to a tooth or a group of iii. Maximal elastic moment
Section | I  Topic-Wise Solved Questions of Previous Years 189

These three properties are called spring characteristics Short labial bow: used for minor overjet reduction and an-
and all three properties put together are found within the terior space closure.
elastic range of an orthodontic wire. Long labial bow:
Moment to force ratio: It determines the centre of rota- Uses:
tion of tooth. Varying the moment to force ratio pro- l Closure of minor anterior space and space distal
duces different types of tooth movements. to canine
Load deflection rate: It gives the force produced per unit l Minor overjet reduction
activation. Active members of the appliance should have l Guidance of canine during retraction of canine
low load deflection rate which implies light continuous using palatal retractor
force. l Retaining device at the end of the treatment
Maximum elastic moment: It is the greatest force or mo- Split labial bow: used for anterior retraction and closure of
ment that can be applied to the appliance without pro- midline diastema
ducing permanent deformation. This will prevent distor- Reverse labial bow: used for minor overjet reduction and
tion of the appliance during activation or accidental anterior space closure
overloading during a chewing. Roberts retractor: used to correct severe anterior procli-
nation
Q.8. Discuss the treatment of different types of maloc-
Mills retractor: correction of large overjet
clusion with removable appliances.
High labial bow with apron springs: used for retraction of
Ans. one or more teeth
Fitted labial bow: bring out active tooth movement
[Same as LE Q.2]
Q.3. Hawley’s appliance.
Q.9. Classify orthodontic appliances and explain the
retentive components of removable orthodontic appli- Ans.
ances and Adams clasp in detail.
l Hawley’s appliance is the most frequently used retainer
Ans. designed by Charles Hawley in 1920.
l It is a passive appliance that can be removed by the
[Same as LE Q.4]
patient and reinserted at will.
l The classic Hawley’s retainer consists of clasps on
SHORT ESSAYS: molars, short labial bow
3
to
3
having adjustment
Q.1. Give classification of canine retractors. loops.
Several modifications of Hawley’s appliance to suit specific
Ans.
requirements are as follows:
[Ref LE Q.3] I. The labial bow can be made to extend from one first
PM to opposite first PM. This design helps in closing
Q.2. Orthodontic appliance – labial bows.
spaces distal to canine.
Ans.

{SN Q.4}
l Labial bows are active components of removable
orthodontic appliances which are mostly used for
incisor retraction.
l There are various types of bows routinely used in

orthodontics, they are as follows:


i. Short labial bow
ii. Long labial bow
iii. Split labial bow
iv. Reverse labial bow
v. Roberts retractor
vi. Mills retractor
vii. High labial bow with apron springs
viii. Fitted labial bow
Fig. 23.14  Hawley’s retainer.
190 Quick Review Series for BDS 4th Year, vol 1

Q.5. Enumerate various types of clasps used in an ortho-


dontics appliances and the ideal requirements of the clasp.
Ans.
Various types of clasps used in an orthodontic appliance are
as follows:
l Circumferential clasp

l Jacksons clasp

l Adams clasps

l Southend clasp

l Triangular clasp

l Ball end clasp

l Schwarz clasp

l Crozat clasp

Ideal requirements of orthodontic clasp are as follows:


l It should be passive and should not produce any un-

desirable tooth movements.


l It should be easy to fabricate and adjust.
Fig. 23.15  HR with labial bow soldered to Adams clasp.
l It should not get distorted easily on regular usage.

l It should not interfere with occlusion and irritate the


II. To solder the bow to the bridge of the Adams clasp. soft tissues.
This design avoids the risk of space opening up l It should be versatile.
between the canine and the premolar due to the l It should provide adequate resistance against dis-
crossover wires. Fitted labial bow can also be used placement and good retention.
to offer excellent retention. l It should function as anchorage part.

Q.6. Advantages of removable orthodontic appliances.


Ans.
The removable appliances are the devices that can be in-
serted into and removed from the oral cavity by the patient
at his will.
Advantages of removable orthodontic appliances are as
follows:
i. Ability to maintain good oral hygiene.
ii. Usually used for simple tooth movements that can
be brought about by tipping.
iii. Less chair side time of orthodontist to fabricate
them so that they can handle more number of
patients.
iv. Various movements like tipping, overbite reduc-
tion can be undertaken.
v. Lesser forces are used. So the strain on anchor
teeth is lesser than fixed appliances.
vi. General dental practitioners who have received
III. Anterior bite plans can be incorporated to retain or
adequate training can use them.
correct deep bite cases.
vii. Less inventory is required for fabrication.
Advantages of Hawley’s appliance:
viii. Relatively more economical.
i. Ease of fabrication due to simple design.
ix. As they take less chair side time and are more
ii. Minimal patient discomfort due to reduced bulk.
economical, they can be used in community-based
iii. It is acceptable to most patients as it is relatively
programmes where a large number of patients are
inconspicuous.
treated.
Q.4. Describe components/parts of removable appli- x. They are less conspicuous and aesthetically more
ances and write about ideal requirements of springs. pleasing.
xi. The patient can himself remove damaged appli-
Ans.
ances that apply undesirable forces.
[Ref LE Q.4] xii. Used as retention appliances.
Section | I  Topic-Wise Solved Questions of Previous Years 191

Q.7. Adams clasp and its advantages. ii. It resembles full clasp with additional piece of wire
soldered which engages mesial and distal undercuts.
Ans.
iii. Its advantages are that it offers better retention than the
l Adams clasp was described by professor Philip Adams. full clasp and can be used for active appliances.
l It is also known as Liverpool clasp, universal clasp or iv. Its disadvantages are chances of causing tissue irrita-
modified arrowhead clasp. tion and separation of the tooth.
l Constructed using 0.7-mm round SS wire.
Q.3. Adams clasp.
l Parts of Adams clasp are as follows:

a. Two arrowheads – engage mesial and distal under- Ans.


cuts.
l The Adams clasp was first described in 1948 by Professor
b. Bridge – connects to arrowheads and is at 45° to long
Phillip Adams.
axis of tooth.
l It is also known as Liverpool clasp, universal clasp or
c. Two retentive arms.
modified arrowhead clasp.
Advantages of Adams clasp are as follows:
l The clasp is constructed using 0.7-mm hard round SS
l It is rigid and offers excellent retention.
wire.
l It can be fabricated on both deciduous and perma-
l It consists of two arrowheads, a bridge and two retentive
nent teeth.
arms.
l It can be used on partially or fully erupted teeth.
l The two arrowheads engage the mesial and the distal
l It can be used on molars, premolars and incisors.
proximal undercuts and are connected to each other by
l No specialized instrument is needed to fabricate
a bridge, which is at 45° to the long axis of the tooth.
the clasp.
l Its advantages are that it is simple, strong and can be
l It is small and occupies minimum space.
easily constructed on deciduous and permanent teeth,
l It can be modified in a number of ways.
and requires no specialized instrument to fabricate.
Q.8. Springs used for distalizing the canines.
Q.4. Labial bow.
Ans.
Ans.
[Same as SE Q.1]
[Ref SE Q.2]
Q.9. Hawley’s retainer.
Q.5. Classify canine retractors.
Ans.
Ans.
[Same as SE Q.3]
l Canine retractors are springs that are used for distal
movement of canines.
SHORT NOTES: l Types of canine retractors based on placement position

are as follows:
Q.1. Z-spring. i. Buccally placed:
Ans. a. Buccal canine retractor
b. Helical loop canine retractor
[Ref LE Q.5] c. U-loop canine retractor
Q.2. Crozat clasp. ii. Palatally placed:
a. Palatal canine retractor
Ans.
Q.6. Roberts retractor.
i. It is a free-ended clasp made of 1-mm or 18-gauge hard
SS wire. Ans.
l Roberts retractor was designed by G.H. Roberts.
l It is made up of 0.5-mm or 23-gauge hard SS wire.
l It is indicated for retraction of four incisors, as it is

highly flexible and is an excellent retraction bow.


l Roberts’ retractor consists of two sleeved canine retrac-

tors joined to form an apron spring.


l The arch should lie way up the crowns of the teeth and

extends only up to two-thirds of the width of the lateral


incisors, and coils are placed at the point of emergence
Fig. 23.16  Crozat clasp.
of the wire from the sleeves.
192 Quick Review Series for BDS 4th Year, vol 1

l Activation is done by adjusting the vertical limb below Q.12. Coffin spring.
the coil by bending it palatally.
Ans.
l Disadvantages: Breakages and damage are common

and is difficult to repair. [Ref LE Q.5]


Q.7. Orthodontic springs – mention one classification. Q.13. High labial bows.
Ans. Ans.
l Springs are active components of removable appliances. l High labial bow with apron springs is used for retraction
l There are various methods of classification of springs. of one or more teeth.
I. Classification of springs based on their ability to with- l The two components of high labial bow with apron

stand forces of distortion: springs are


a. Self-supported springs: (i) Heavy base arch wire
i. Buccal canine retractor (ii) Apron spring
ii. U-loop canine retractor l Heavy base arch wire used is a 0.9-mm or 19-gauge

iii. Helical loop canine retractor hard SS wire.


iv. Coffin springs l High labial arch should neither contact the mucosa nor

b. Guided springs: extend deep into the full depth of the sulcus.
i. Cantilever springs l Apron springs constitute the active components used
ii. Finger springs with high labial bow.
iii. Palatal canine retractor l Apron springs are attached to the base arch by winding
iv. T-springs a few turns in vertical arm and horizontal arm.
c. Auxiliary springs:
Q.14. Arrowhead clasp.
i. Apron springs
Ans.
Q.8. Define removable orthodontic appliance and name
its components. l Arrowhead clasp is also known as Schwarz clasp.
l It is made with 0.7-mm/21-gauge hard SS wire.
Ans.
l A number of arrowheads engage interproximal under-

i. Orthodontic appliances are defined as the devices by cuts between molars and premolars.
means of which mild pressure may be applied to a tooth l Advantages: More elastic, facilitates eruption of buccal

or a group of teeth and their supporting structures so as teeth, can be used in combination with posterior bite
to bring about necessary changes within the bone which blocks.
will allow tooth movement. l Drawbacks: Fabrication is difficult and time-consuming

ii. The removable orthodontic appliances are the devices and requires a special pliers like Tischler’s pliers or
that can be inserted into and removed from the oral cav- optical pliers, and injures interdental soft tissue.
ity by the patient at his will.
Q.15. Double cantilever spring.
iii. There are three basic components in a removable orth-
odontic appliance: Ans.
a. Retentive components, e.g. clasps, bows
l Double cantilever spring is also known as Z-spring.
b. Active components, e.g. springs, labial bows, screws
l It is made of 0.5-mm/23-gauge hard round SS wire.
c. Base plate – forms the framework
l The spring consists of two coils of very small internal
Q.9. Mention three ideal requirements of an orthodontic diameter. The spring should be perpendicular to the
appliance. palatal surface of the tooth.
l Indications:
Ans.
i. When both labial and lateral movements of the inci-
[Ref LE Q.3] sors are required.
Q.10. Name few self-supporting springs. ii. Minor rotation correction.
iii. When two or more teeth have to be moved labially.
Ans. l The Z-spring is activated by opening both the helices by

[Ref LE Q.5] about 2–3 mm at a time. In case of minor rotation cor-


rection, one of the helices is opened.
Q.11. Finger spring.
Ans. Q.16. Types of labial bow and their uses.

[Ref LE Q.5] Ans.


Section | I  Topic-Wise Solved Questions of Previous Years 193

l Labial bows are active components of removable orth- l It is also known as Liverpool clasp, universal clasp or
odontic appliances which are mostly used for incisor modified arrowhead clasp.
retraction. l The clasp is constructed using 0.7-mm hard round SS

l The various types of bows routinely used in orthodon- wire.


tics and their uses are as follows: Modifications of Adams clasp are as follows:
Short labial bow: Adams clasp with:
Used for minor overjet reduction and anterior space l Single arrowhead

closure l Additional arrowhead

Long labial bow: l Distal extension

Closure of minor anterior space and space distal to l J-hook

canine, minor overjet reduction l Helix

Split labial bow: l Soldered buccal tube

Used for anterior retraction and closure of midline l An incisor and premolar

diastema
Q.20. Disadvantages of removable orthodontic appliances.
Reverse labial bow:
Used for minor overjet reduction and anterior space Ans.
closure
The disadvantages of removable orthodontic appliances are
Roberts’ retractor:
as follows:
Used to correct severe anterior proclination
i. The need of patient’s cooperation is highly important.
Mills retractor:
ii. Inability to perform multiple and complex tooth move-
Correction of large overjet
ments.
High labial bow with apron springs:
iii. The chances of patient misplacing or damaging the
Used for retraction of one or more teeth
removable appliances is more.
Q.17. Mention few active plates. iv. Patients should have enough skill to remove and replace
the appliance without distorting it.
Ans.
Q.21. Orthodontic clasp – ideal requirements.
l Base plate forms the framework of removable orthodon-
tic appliance and serves to hold all the components of Ans.
the appliance together.
l Clasps are the wire components that aid in the retention
l Usually, base plates are made of either cold-cure or
of a removable appliances, e.g. C-clasp, Crozat clasp,
heat-cure acrylic.
Jacksons clasp and Adams clasp.
l It functions as an active component. A few appliances
l Mode of action of clasps: They act by engaging the
which are considered as active plates are as follows:
undercuts of the teeth.
i. Split plate.
l Requirements of an ideal clasp:
ii. Bite planes can be incorporated into plate to treat
certain orthodontic problems
Offer adequate retention
iii. Upper anterior inclined plate.
Q.18. Buccal canine retractors.
Permit usage in both fully erupted and
Ans. It should
partial erupted teeth
l Buccal canine retractors are indicated in cases of buc-
cally placed canines and canines placed high in the
vestibule. Be easy to fabricate
l They are used where a buccally placed canine has to be

moved both palatally and distally. Q.22. Cantilever spring.


l It made up of 0.7-mm or 21-gauge SS wire.
Ans.
l It consists of a coil of 3-mm diameter, an active arm and

a retentive arm. Cantilever springs are of two types:


l It is one of the few springs where the coil is closed for a. Single cantilever spring
activation. l It is constructed with 0.5-mm hard SS wire.

l A single cantilever spring consists of three parts:


Q.19. Adams clasp – modification.
retention tag, coil and an active arm.
Ans. l The spring is activated by opening the coil.
194 Quick Review Series for BDS 4th Year, vol 1

b. Double cantilever spring or Z-spring: ii. Minor rotation correction.


l The Z-spring is made of 0.5-mm/23-gauge hard iii. When two or more teeth have to be moved
round SS wire. labially.
l The spring consists of two coils of very small inter- l The Z-spring is activated by opening both the helices

nal diameter. by about 2–3 mm at a time.


l Indications:

i. When both labial and lateral movements of the


incisors are required.

Topic 24
Fixed Orthodontic Appliances
COMMONLY ASKED QUESTIONS
LONG ESSAYS:
1 . Discuss the advantages and disadvantages of fixed appliances over removable appliances.
2 . Explain various components of fixed orthodontic appliances.

SHORT ESSAYS:
1. Begg’s appliance technique.
2. Fixed appliances – advantages.
3. Active parts of fixed appliances. [Ref LE Q.2]
4. Passive components of fixed appliances. [Ref LE Q.2]
5. Name a few fixed appliance techniques. Write about anyone technique.
6. Name three fixed appliance techniques. Differentiate between Begg’s technique and edge-wise technique.
7. Pre-adjusted edge-wise appliances.
8. Indirect bonding.
9. Direct bonding.
1 0. Orthodontic bands.
1 1. Lingual orthodontics.
1 2. Write about any one fixed appliance technique. [Same as SE Q.1]
1 3. Active components of fixed appliance. [Same as SE Q.3]
1 4. Differentiate between Begg’s technique and edge-wise technique. [Same as SE Q.6]

SHORT NOTES:
1. Elastics.
2. Fixed appliance.
3. Components of fixed appliance. [Ref LE Q.2]
4. Fixed appliances versus removable appliances.
5. NiTi wires.
6. Edge-wise appliance technique.
7. Stages of Begg’s appliance treatment.
8. Advantages of fixed appliances.
9. Disadvantages of fixed appliances.
1 0. Classify brackets in fixed orthodontic appliances.
1 1. Class II elastics.
1 2. Pre-adjusted edge-wise appliance.
1 3. Ideal properties of orthodontic wires.
Section | I  Topic-Wise Solved Questions of Previous Years 195

1 4. What are the advantages of bonding over banding?


15. Enumerate the objectives of stage I of Begg’s technique.
16. Molar tubes.
17. Metallic brackets.
18. Orthodontic bands. [Ref SE Q.10]
19. Ceramic brackets.
20. Types of elastics. [Same as SN Q.1]
21. Parts of fixed orthodontic appliances. [Same as SN Q.3]

SOLVED ANSWERS
LONG ESSAYS:
Q.1. Discuss the advantages and disadvantages of fixed [SE Q.3]
appliances over removable appliances.
l {Fixed orthodontic appliances are those orthodontic
Ans. appliances that are rigidly attached to the teeth by
means of bands or rings of metal, which are closely
The advantages and disadvantages of fixed appliances adapted and cemented to the teeth. Patients cannot re-
over removable appliances are as follows: move them by themselves.
l The components/parts of fixed appliances are broadly
Advantages Disadvantages classified as follows:
i. Fixed appliances offer i. Oral hygiene mainte- i. Active components
better control and to a nance becomes more ii. Passive components}
large extent remove the difficult.
need for patient compli- (SE Q.3 and SN Q.3)
ance or cooperation.
ii. It is possible to bring about ii. They take up more chair {(1.  Active components
various types of tooth side time as they are i. Separators
movements, e.g. tipping, time-consuming to fix ii. Arch wires
bodily movement, rotation, and adjust.
intrusion and extrusion.
iii. Springs
iv. Elastics
iii. The treatment duration is iii. More conspicuous than v. Expansion screws
considerably reduced, as removable appliances.
multiple tooth movements Aesthetically unpleasing
2. Passive components
are possible simultaneously. unless modern tooth- i. Bands/band material
coloured appliance are ii. Brackets
used. iii. Buccal tubes
iv. More precise tooth move- iv. Fixed orthodontic appli- iv. Lingual attachments
ments and detailing of ances require special v. Lock pins
occlusion is possible using training of the operator vi. Ligature wire
fixed appliances. and are invariably han-
vii. Bypass clamps)}
dled by specialized or-
thodontists.
[SE Q.3]
v. Fixed appliances can be v. Damaged appliances that
used to treat most maloc- apply misdirected forces {1. Active components of fixed appliances are described
clusions, including very cannot be removed by below.
complicated ones. the patient. i. Separators
vi. Fixed appliances offer bet- vi. Patient has to visit ortho- l Separators are the active components of fixed ap-
ter control over anchorage. dontist at regular inter- pliances used to bring about separation of teeth
vals. and create space in-between two adjacent teeth
vii. More expensive. generally for the purpose of banding.
l They are used in cases of tight interdental contact

to break those contacts.


Q.2. Explain various components of fixed orthodontic
Principle of separators:
appliances.
l It is a device to wedge the teeth in a place
Ans. between the adjacent teeth.
196 Quick Review Series for BDS 4th Year, vol 1

l It causes tooth movement and separation of teeth Ideal requirements of arch wire are as follows:
for easy placement of bands. The arch wire should have:
Types of separators l Low stiffness and high spring back
One classification based on material used. l High amount of stored energy
i. Metal separators: l High resiliency and formability
a. 0.020˝ brass wire l Good biocompatibility and environmental
b. Kesling separating spring stability
ii. Elastic separators: l Capability to be welded and soldered
a. Elastic thread
b. Maxian elastic separator [SE Q.3]
c. Elastomeric rings or doughnut {Classification of arch wires:
Based on duration of action, they may be classified a. Based on cross section:
as follows: l Round
I. Slow separators: l Square
i. Brass wire separator l Rectangular
ii. Kesling’s spring separator l Multistranded
II. Rapid separators: b. Based on material used:
i. Ring separators or elastic rings or l Gold and gold alloys
doughnut l Stainless steel alloys
ii. Elastic separators or dumbbell separa- l Beta titanium alloys
tors} l Co–Cr–Ni alloys
i. Brass wire separator: l Nickel–titanium alloys
l Soft brass wire of 0.5–0.6-mm diame- l Optiflex wires
ter is rotated through the embrasure c. Based on the modifications incorporated in arch
between teeth and cut short and tucked wires:
between the teeth and left for 5–7 days. l Plain arch wires
ii. Kesling’s spring separator: l Arch wire with loops
l Separating springs like Kesling’s iii. Springs
spring exerts a scissors-like action l Springs are the active components used to bring
when kept for 7 days. about various tooth movements.
iii. Ring separators or doughnut: Various types of springs:
Ring separators are small elastic rings i. Coil springs:
passed through the contact using a special l These are two types of coil springs: open coil
plier/applicator. As the stretched elastic spring and closed coil spring.
ring encircling the interdental contact area These are used to close or open the spaces be-
contracts, the teeth are separated. tween the teeth.
iv. Elastic separators or dumbbell separa- a. Open coil springs: Springs which are com-
tors: pressed between two teeth to open the
l Elastic separator/a dumbbell piece of space between them are called open coil
elastic stretched and passed through springs.
interdental contact area brings about b. Closed coil springs: The springs which are
separation of teeth by trying to regain stretched between the teeth to close the
its original length. It is effective after 2 space are called closed coil springs.
or 3 days of placement. ii. Uprighting springs: These are used for root move-
ment in mesial ordistal direction.
[SE Q.3]
iii. Rotation springs: Used to correct rotated teeth.
{ii.  Arch wires iv. Torquing spring: Used to move the root in a
l Arch wires are one of the active components that lingual/palatal direction.

exert force to the teeth and are used for achieving iv. Elastics
all types of tooth movements.} l Elastics are made up of latex rubber material and are
l They bring about various tooth movements through available in various diameters. The force applied by
medium of brackets and buccal tubes. the elastics depends upon the diameter.
Section | I  Topic-Wise Solved Questions of Previous Years 197

l Elastics are used for a number of purposes, such ii. Orthodontic brackets
as to move the teeth, to fix arch wire to the teeth l Orthodontic brackets can be compared with door

and for separation of teeth. handles, they transmit force from the active compo-
l They are available in the form of bands, threads, nents to the teeth.
modules and rotational wedges. Classification of orthodontic brackets:
l Rotation wedges are used to correct a rotated They can be classified in number of ways as follows:
tooth. I. Based on the technique
l Elastics are available in different colours such as i. Edge-wise type of brackets
yellow, pink, green, blue, white and red. ii. Pre-adjusted edge-wise brackets

l Uses: To close the spaces, correct crossbite, open iii. Begg’s brackets
bite and interarch relationship. iv. Lingual orthodontic brackets
v. Expansion screws II. Based on the type of material used
l Expansion screws are used in cases of maxillary i. Metallic brackets
arch constriction to achieve expansion.} ii. Plastic brackets
iii. Ceramic brackets
[SE Q.4] III. Based on the method of fixing
{2. Passive components/parts of fixed appliances are i. Bondable
described below. ii. Weldable}
l Bands

l Brackets Edge-wise type of brackets


l Buccal tubes l These brackets are used in the edge-wise

l Lingual attachments and the straight wire techniques. They have


l Lock pins a horizontal or rectangular slot facing
l Ligature wires labially.
i. Bands l These brackets accept wires of rectangular

l Bands are thin strips of stainless steel which are cross section with large dimension.
adapted to the contours of the tooth to which other l They provide greater control over tooth

orthodontic attachments are welded or soldered, movement and do not permit tipping of
for example: teeth.
Buccal tubes: It holds the arch wires and the in- Ribbon arch brackets
ner bow of the face bow attachment. l These brackets are used in the Begg’s fixed

Lingual sheaths: Receive and attach lingual arch appliance and posses a vertical slot facing
wires. occlusal or gingival direction.
Molar hooks, lingual buttons and cleats: These l These types of brackets are used with round

are used for engaging elastic bands and modules. wires to bring about tipping of teeth in la-
l Bands are mainly used for the posterior teeth. biolingual or mesiodistal direction.
Nowadays with the advent of direct bonding of Metallic brackets
brackets, bands are rarely used for anterior teeth. l These are commonly used brackets.

Various classifications of bands are as follows: l Advantages:

a. Based on tooth used: They are not expensive.


They can be sterilized and recycled.
Teeth Size (inches) They resist deformation and fracture.
Molar band 0.005 3 0.20 or 0.005 3 0.18 They exhibit the least friction at the wire–
Premolars band 0.004 3 0.150 bracket interface.
Incisor band 0.003 3 0.125 l Disadvantages:
They are not aesthetic and patient tends to
b. Based on fabrication have a metallic smile.
i. Preformed – They are available ready- They can corrode and cause staining of
made in assorted sizes. teeth.
ii. Custom-made – Specially made as per the Plastic brackets
patient requirement. These are contoured l They are made up of polycarbonate or a
by the orthodontists using special pliers. modified form of polycarbonate.
198 Quick Review Series for BDS 4th Year, vol 1

l Advantage: the concept of differential force technique where the


They improve aesthetic value of appliance and bodily movements of anchor molars were pitted
are available in tooth-­coloured or transparent against the tipping movement of anterior teeth.
form. Hence, light forces are used in this technique.
l Disadvantage: l The type of tooth movement achieved in this tech-

They tend to discolour and have poor dimen- nique is tipping. Anchorage preparation is not very
sional stability, and slots tend to distort. critical.
Friction between plastic brackets and metal l There are three different stages in Begg’s treatment:

arch wire is very high. Stage 1.


Ceramic brackets This stage is concerned with:
l They are introduced in the 1980s and are made Alignment
up of aluminium oxide or zirconium oxide. Correction of spacing
l Advantages: Correction of crowding
They are durable and resist staining. Correction of rotation
They are dimensionally stable and do not dis- Overjet and overbite correction
tort in oral cavity. Achieving an edge-to-edge anterior bite
l Disadvantages: Stage 2.
They are brittle and bulky in size. Maintaining correction of achieved treatment in
They exhibit greater friction at wire–bracket stage one and space closure.
interface compared with metallic brackets. Stage 3.
[SE Q.4] Involves achieving normal axial inclination of teeth
by uprighting and torquing.
{iii. Lingual attachments
l Attachments which can be fixed on the lingual as-
Q.2. Fixed appliances – advantages.
pect are called Lingual attachments.
l They are useful for engaging elastics. Ans.
l Various lingual attachments include lingual but-

tons/lingual cleats/lingual eyelets/ball end hooks. l Fixed orthodontic appliances are those orthodontic ap-
iv. Accessories pliances that are rigidly attached to the teeth by means
i. Lock pins of bands or rings of metal, which are closely adapted
ii. Ligature wire and cemented to the teeth. Patients cannot remove them
iii. modules by themselves.
i. Fixed appliances offer better control and to a large
i. Lock pins:
extent remove the need for patient compliance or
l Lock pins are made from brass and are used to
cooperation.
secure or engage the arch wire into the vertical
ii. It is possible to bring about various types of tooth
slot of the Begg’s brackets.
movements, e.g. tipping, bodily movement, rota-
l Various types of lock pins are stage I, stage II,
tion, intrusion and extrusion.
stage III and T pins.
iii. The treatment duration is considerably reduced, as
ii. Ligature wire: They are made from soft fully an-
multiple tooth movements are possible simultane-
nealed stainless steel wires and are used to tie the
ously, e.g. correction of deep bite, de-crowding
arch wire to the brackets.
and reduction of overjet.
iii. Modules: Modules are used to fix the arch wire to the
iv. More precise tooth movements and detailing of
bracket slot. They are elastomeric rings used in pre-
occlusion is possible using fixed appliances.
adjusted edge-wise technique.}
v. Fixed appliances can be used to treat most maloc-
clusions, including very complicated ones.
SHORT ESSAYS: vi. Fixed appliances offer better control over an-
Q.1. Begg’s appliance technique. chorage.
vii. In cases where posterior segments have to be
Ans.
moved forward, it can be done with fixed appli-
Begg’s technique or modified ribbon arch technique: ances only.
l Raymond Begg, in the 1950s, introduced Begg’s viii. Teeth can be aligned better when compared with
light wire differential force technique. He introduced removable appliances.
Section | I  Topic-Wise Solved Questions of Previous Years 199

Q.3. Active parts of fixed appliances. ii. Need for extraoral anchorage and difficulty in
opening deep bites.
Ans.
A number of modifications have been proposed
[Ref LE Q.2] in this technique over a period of many years.
Q.4. Passive components of fixed appliances. Q.6. Name three fixed appliance techniques. Differenti-
ate between Begg’s technique and edge-wise technique.
Ans.
Ans.
[Ref LE Q.2]
Fixed appliances are rigidly attached to the teeth by means
Q.5. Name a few fixed appliance techniques. Write
of bands or rings of metal, which are closely adapted and
about any one technique?
cemented to the teeth. Patients cannot remove it by them-
Ans. selves.
Fixed appliance techniques:
Fixed appliance techniques are as follows:
i. E-arch or expansion arch technique – Edward H.
i. E-arch or expansion arch technique – Edward H. Angle
Angle
(1800s)
ii. Pin and tube appliance technique – Edward H. Angle
ii. Begg’s appliance technique – Raymond Begg (1900s)
iii. Edge-wise appliance technique – Edward H. Angle
iii. Pin and tube appliance technique – Edward H. Angle
iv. Begg’s appliance technique – Raymond Begg
(1912)
v. Straight wire appliance technique – L. F. Andrews
iv. Edge-wise appliance technique – Edward H. Angle
vi. Lingual orthodontic technique – Craven Kurz
(1928)
v. Straight wire appliance technique – L.F. Andrews
(1970s) Differences between Begg’s and edge-wise techniques
vi. Lingual orthodontic technique – Craven Kurz (1976) are as follows:
Edge-wise appliance (Angle, 1928):
Components Begg’s Edge-wise
l In this technique, arch wire is inserted into the

bracket with narrow dimension placed occluso- i. Bracket Single point Rectangular in
contact cross section
gingivally. This mode of insertion of wire is called
edge-wise, and hence the technique is called ii. Buccal tube Round Rectangular
edge-wise technique. iii. Arch wire Round Rectangular
l This technique having unique feature of rectangu-
iv. Engagement of Using lock pins Using ligatures
lar arch wire in rectangular slot allowed excellent wire
control of tooth movement in all the three planes
v. Forces Light Heavy
of space.
l Certain bends incorporated in ideal arch wire are vi. Duration of Relatively early Relatively slow
used to accomplish desired tooth movements; treatment
they are vii. Head gear Not used Used
i. First-order bends – in and out or labiolingual viii. Anchorage Not done Done
corrections. preparation
ii. Second-order bends – tip back bends placed
ix. Tooth Bodily movement Tipping movement
in posterior segments, they are meant to movement
achieve correct mesiodistal axial inclinations
x. Stages in Three stages Three orders
of teeth.
treatment
iii. Third-order bends – They are placed by twist-
ing the arch wire, they are placed to get correct xi. Final alignment Moderate Good
buccolingual position by moving the roots. xii. Incidence of Relatively less Relatively more
Advantages of edge-wise technique are as follows: root resorption
i. Ability to move the teeth in all the three
planes of space. Q.7. Pre-adjusted edge-wise appliances.
ii. Good control over tooth movement.
iii. Bodily tooth movement and precise finishing Ans.
are possible. Pre-adjusted edge-wise appliances or straight wire appli-
Disadvantages of edge-wise technique are as follows: ances
i. Need for complex wire bending and applica- l Straight wire technique was introduced in the 1970s
tion of heavy force. by Lawrence F. Andrews.
200 Quick Review Series for BDS 4th Year, vol 1

l This technique has eliminated the complex wire ii. Paint a thin layer of unfilled resin over the etched
bending procedures by modifying the brackets; hence enamel and over the cured composite in the tray.
it is known as pre-adjusted edge-wise appliance. iii. Place the memosil tray in the mouth, and light-
l Bodily movement type of tooth movement is cure each tooth for 30 s.
achieved. Hence, anchorage preparation is vital in iv. Peel away the transfer tray from the teeth or
pre-adjusted appliance technique. cut the Memosil with a scalpel if necessary to
l The angulations and torque values built into the pre- remove it easily.
adjusted bracket are called as appliance prescription. Advantages of indirect bonding
Stages of pre-adjusted edge-wise treatment: l More precise location of brackets is possible.

Stage 1 l Indirect bonding reduces the chair side bonding

i. Initial aligning and levelling of arches done. process.


ii. Crowding correction and establishing normal l Patient comfort and hygiene are improved.

overjet. l It is very helpful in lingual bonding technique

Stage 2 where visualization is difficult.


i. Correction of molar relationship and space Disadvantages of indirect bonding
closure. l Indirect bonding is technique-sensitive.

ii. Establishing class I molar relation and normal l Additional set of impressions are needed.

overjet. l Increased laboratory time.


Stage 3 l Achieving consistent and predictable adhesion is
i. Finishing and detailing difficult.
ii. Root movement and torque correction l Failure rates are slightly higher (Zachrisson and

Brobakken).
This technique has reduced wire bending substantially and
l Closer fitting of bracket base is better achieved in
enabled good finishing of the cases.
direct bonding than in indirect bonding.
Q.8. Indirect bonding.
Q.9. Direct bonding.
Ans.
Ans.
Indirect bonding procedure consists of the attachment of the
l Direct bonding refers to the direct attachment of the
brackets to the working cast using water soluble resins ini-
bracket to the etched enamel using self- or light-cure
tially, and then transferring it to the mouth using a custom tray.
agent. Widely used as it is simple and reliable.
Technique:
l The steps involved in direct bonding on facial or lingual
l The indirect working casts are prepared by taking
surface are as follows:
alginate impressions 1 or 2 weeks prior to bonding
i. Cleaning
procedure.
ii. Conditioning of enamel
l When the casts are dry, fill in any voids and remove
iii. Application of sealants
bubbles. Mark the position of the brackets with pencil.
iv. Bonding
l Apply two thin coats of liquid separating medium to

the facial surfaces of the teeth on the cast, and allow i. Cleaning
it to dry. l The first step in bonding is removal of plaque and

l Position the brackets using light-/self-/heat-cure un- organic pellicle that normally cover all teeth.
filled resin on the models. l Thorough cleaning of teeth with water slurry of

l Fabrication of transfer trays: pumice or prophylaxis paste is essential to achieve


Inject silicone-based, addition-cured elastomer of this.
medium viscosity, e.g. memosil over the brackets ii. Conditioning enamel
with a syringe so that it covers all the buccal, occlu- l This step involves two procedures, namely:

sal and lingual surfaces of the teeth to be bonded. a. Moisture control


Allow the tray to set for 10 min. b. Enamel pretreatment
l Soak the cast and memosil tray in cold water for

20 min and then separate both trays from the cast. a. Moisture control: Complete dry working field
The brackets will easily release from the stone and is absolutely essential for effective bonding.
remain seated in the tray. Trim the trays. This can be achieved by using the:
l Chair side bonding procedure: l Lip and cheek retractors

i. Etch the teeth to be bonded. l Saliva ejectors, salivary duct obstructors


Section | I  Topic-Wise Solved Questions of Previous Years 201

l Tongue guards with bite blocks l Proper horizontal and vertical positions of
l Cotton or gauze rolls bracket should be ensured. A placement
l Antisialogogue, e.g. atropine sulphate or pro- scaler is used to position the bracket cor-
banthine bromide rectly.
b. Enamel pretreatment or acid etching: l Once the bracket is in correct position, it is
l After drying the tooth, an etchant, usually 37% pushed firmly towards the tooth surface. The
phosphoric acid solution or gel, is applied tight fit will result in good bond strength.
lightly over enamel surface with a pellet or l The last step in direct bonding is removal of
brush for 15–60 s. excess adhesive using a scaler.
l Etching is also done by 10% polyacrylic acid
Q.10. Orthodontic bands.
or 10% maleic acid.
l Etchant is rinsed off with abundant water spray Ans.
for about 15 s. If salivary contamination occurs
after etching, re-etch for another 30 s. {SN Q.18}
l Dry the tooth thoroughly to obtain dull frosty l Orthodontic bands are one of the passive compo-
white appearance. nents of fixed orthodontic appliances.
The rationale for acid etching is as follows: re- Bands are thin strips of stainless steel which are
moves about 3–10 microns of enamel surface. adapted to the contours of the tooth to which other
l It increases the wettability and surface area of orthodontic attachments are welded or soldered, e.g.
enamel substrate. buccal tubes, lingual sheaths, molar hooks, lingual
l Primary attachment mechanism of resin is buttons and cleats.
‘resin tags’. Resin tags penetrate up to the l Bands are mainly used for the posterior teeth. Nowa-
depth of 80 microns or more and provide a days with the advent of direct bonding of brackets,
micromechanical bond. bands are rarely used for anterior teeth.
iii. Application of sealants Various classifications of bands are as follows:
l Sealants are unfilled resins with low viscosity. After a. Based on tooth used:
etching, a thin layer of sealant may be painted over
entire enamel surface. It is best applied with a small Teeth Size (inches)
foam pellet or brush. It should be thin and even. Molar band 0.005 3 0.20 or 0.005 3 0.18
l Sealants might be necessary to achieve proper bond Premolar band 0.004 3 0.150
strength and to improve resistance to micro leakage. Incisor band 0.003 3 0.125
l Moisture control may not be extremely important

after sealant application. b. Based on fabrication:


l Sealant might permit easier bracket removal and
i. Preformed – They are available ready-made in
protects against enamel tear-outs during de-bonding. assorted sizes.
l Sealants are either self-cured or light-cured.
ii. Custom-made – Specially made as per the
a. Self-polymerizing sealants: polymerize poorly, patient requirement. These are contoured by
exhibit drift, have low resistance to abrasion. the orthodontists using special pliers.
b. Light-polymerizing sealants: protect enamel ad-
jacent to brackets from dissolutions and subsur-
face lesions. Q.11. Lingual orthodontics.
iv. Bonding
Ans.
The recommended bracket bonding procedure consists
of the following steps: l The technique of lingual orthodontics, also called invis-
i. Transferring the brackets ible orthodontics, was introduced in 1976 by Craven
ii. Positioning of bracket Curz.
iii. Fitting l Craven Curz developed plastic brackets on lingual sur-

iv. Removal of excess adhesive face for easy reshaping and better fit. In 1976, Ormco
Procedure: Company along with Craven Curz, Craig Andreiko and
l Use a reverse action tweezers to grip the brackets. Frank Miller developed first-generation Curz bracket.
l Apply adhesive to the bracket base and place l The technique of lingual orthodontics involves place-

the bracket immediately on the tooth close to its ment of brackets and other attachments on the lingual
correct position. surface of the teeth.
202 Quick Review Series for BDS 4th Year, vol 1

The indications for lingual orthodontic technique are as Class III (intermaxillary) – placed between maxillary mo-
follows: lars and mandibular anteriors for correction of class III.
l Cases with good gingival and periodontal health hav- Diagonal elastics – worn for midline corrections.
ing mild incisor crowding and those with anterior Crossbite elastics (intermaxillary) – used to correct
deep bite. crossbites in buccal segment.
l Long and uniform lingual tooth surfaces without any Box elastics – used to correct anterior open bites.
restorations.
Q.2. Fixed appliance.
l Keen and compliant patient.

l In individuals with mesocephalic or mild/moderate Ans.


brachycephalic skull with skeletal class I pattern.
l Fixed orthodontic appliances are those orthodontic
l Patients with adequate mouth opening.
appliances that are rigidly attached to the teeth by
Disadvantages of lingual orthodontic technique:
means of bands or rings of metal, which are closely
l Access is more difficult for orthodontist to place
adapted and cemented to the teeth. Patients cannot re-
brackets on the lingual surface.
move them by themselves.
l Tooth control is not very effective.
l They offer better control and to a large extent remove
l There is limited scope for complex problem.
the need for patient compliance or cooperation.
l Treatment is highly expensive.
l It is possible to bring about various types of tooth move-
l Indirect bonding is mandatory in lingual orthodontics.
ments, e.g. tipping, bodily movement, rotation, intru-
l Difficulty in speech and maintaining oral hygiene.
sion and extrusion.
Q.12. Write about any one fixed appliance techniques. l More precise tooth movements and detailing of occlu-

sion are possible using fixed appliances.


Ans.
Q.3. Components of fixed appliance.
[Same as SE Q.1]
Ans.
Q.13. Active components of fixed appliance.
[Ref LE Q.2]
Ans.
Q.4. Fixed appliances versus removable appliances.
[Same as SE Q.3]
Ans.
Q.14. Differentiate between Begg’s technique and edge-
wise technique. Fixed appliances Removable appliances
Ans. i. Lessen the need for i. Patient cooperation is
patient cooperation. needed to a large extent.
[Same as SE Q.6]
ii. Oral hygiene mainte- ii. Oral hygiene mainte-
nance is more difficult. nance is easy.
SHORT NOTES:
iii. The treatment duration is iii. The treatment duration is
Q.1. Elastics. very less. prolonged in case of
severe malocclusion.
Ans.
iv. More chair side time is iv. Less chair side time of
l Elastics are made up of latex rubber material. consumed to fix them. orthodontist to fabricate
l Elastics are available in different colours such as yellow, them.
pink, green, blue, white, red etc. v. More expensive. v. Relatively economical.
l Uses:
vi. Requires special training vi. Requires no special
Elastics are used for a number of purposes like to move
of the operator. training.
the teeth, fix arch wire to the teeth, separation of teeth,
close the spaces, correct crossbite, open bite and inter-
arch relationship.
l Types of elastics: Q.5. NiTi wires.
Class I (intramaxillary) – placed between molars and
Ans.
anteriors in the same arch. Used for space closure.
Class II (intermaxillary) – placed between mandibular l NiTi is commercially available as Chinese NiTi or Japanese
molars and maxillary anteriors. Used for retraction of NiTi.
maxillary anterior teeth and mesial movement of the l Nitinol was introduced into orthodontics by G. Andreasen
mandibular molars. and William F. Buehler.
Section | I  Topic-Wise Solved Questions of Previous Years 203

l Clinical uses of NiTi alloy in orthodontics are as Disadvantages of fixed appliances are as follows:
follows: i. Oral hygiene maintenance is more difficult.
i. Initial alignment and levelling arch wires ii. More chair side time is consumed to fix them.
ii. Retraction coil springs iii. More expensive.
iii. Palatal expanders iv. Requires special training of the operator and are invari-
iv. Devices for distalization of molars ably handled by specialized orthodontists.
Q.6. Edge-wise appliance technique. Q.10. Classify brackets in fixed orthodontic appliances.
Ans. Ans.
l In edge-wise technique, arch wire is inserted into the Orthodontic brackets can be compared to door handles. They
bracket with narrow dimension placed occluso-gingivally. transmit the force from the active components to the teeth.
This mode of insertion of wire is called edge-wise, and Classification of orthodontic brackets:
hence the technique is called edge-wise technique. They can be classified in a number of ways as follows:
l This technique, having unique feature of rectangular I. Based on the technique
arch wire in rectangular slot, allowed excellent control i. Edge-wise type of brackets
of tooth movement in all the three planes of space. ii. Pre-adjusted edge-wise brackets
l Certain bends incorporated in ideal arch wire are used iii. Begg’s brackets
to accomplish desired tooth movements; they are first-, iv. Lingual orthodontic brackets
second- and third-order bends. II. Based on the type of material used
l Bodily tooth movement and precise finishing are possible. i. Metallic brackets
l Need for complex wire bending and application of ii. Plastic brackets
heavy force are some of the disadvantages of this iii. Ceramic brackets
appliance. III. Based on the method of fixing
i. Bondable
Q.7. Stages of Begg’s appliance treatment.
ii. Weldable
Ans.
Q.11. Class II elastics.
l There are three different stages in Begg’s treatment:
Ans.
Stage 1:
This stage is concerned with: l Elastics are made up of latex rubber material and are
Alignment colour-coded.
Correction of spacing, crowding and overbite correc- l Elastics are used for a number of purposes like to move

tion the teeth, fix arch wire to the teeth, for superstation of
Achieving an edge-to-edge anterior bite teeth, close the spaces, correct crossbite, open bite and
Stage 2: interarch relationship.
Maintaining correction of achieved treatment in l Class II (intermaxillary) elastics
stage one and space closure i. Used to correct class II malocclusion.
Stage 3: ii. They are placed between mandibular molars and
Involves achieving normal axial inclination of teeth maxillary anteriors, for retraction of maxillary ante-
by uprighting and torquing rior teeth and mesial movement of the mandibular
molar teeth.
Q.8. Advantages of fixed appliances.
Q.12. Pre-adjusted edge-wise appliance.
Ans.
Ans.
Advantages of fixed appliances are as follows:
i. They offer better control and to a large extent remove Pre-adjusted edge-wise appliance or straight wire appliance:
the need for patient cooperation. l Straight wire technique was introduced by Lawrence. F.

ii. Various types of tooth movements are possible, e.g. Andrews in the 1970s.
tipping, bodily movement, rotation, intrusion and l This technique has eliminated the complex wire bend-

extrusion. ing procedures by modifying the brackets, hence it is


iii. The treatment duration is considerably reduced. known as pre-adjusted edge-wise appliance.
iv. More precise tooth movements and detailing of occlu- l Bodily movement type of tooth movement is achieved.

sion are possible using fixed appliances. Hence, anchorage preparation is vital in pre-adjusted
appliance technique.
Q.9. Disadvantages of fixed appliances.
l The angulations and torque values built into the pre-

Ans. adjusted bracket are called appliance prescription.


204 Quick Review Series for BDS 4th Year, vol 1

l This technique has reduced wire bending substantially C. Based on number of tubes
and enabled good finishing of the cases. i. Single
Q.13. Ideal properties of orthodontic wires. ii. Double
iii. Triple
Ans. D. Based on technique
Ideal properties of orthodontic wire are as follows: i. Begg’s tube
i. The wire should deliver low constant force. ii. Edge-wise tube
ii. It should have high strength and range. iii. Pre-adjusted edge-wise
iii. It should have low stiffness or good spring back. Q.17. Metallic brackets.
iv. It should offer less frictional resistance between wire
and bracket base. Ans.
v. It should be easy to manipulate and biocompatible.
Brackets are passive orthodontic components. Metallic
vi. It should be stable in the oral environment.
brackets are the commonly used brackets.
vii. It should be economical.
l Advantages:
Q.14. What are the advantages of bonding over banding? They are not expensive.
Ans. They can be sterilized and recycled.
They resist deformation and fracture.
l The bonding is a method of fixing attachments directly They exhibit least friction at the wire–bracket interface.
over the enamel surface of tooth using adhesive resins. l Disadvantages:
l The advantages of bonding over banding are as follows:
They are not aesthetic and patient tends to have a metal-
i. It is easier and faster to bond than to pinch bands lic smile.
around teeth. They can corrode and cause staining of teeth.
ii. Better oral hygiene maintenance.
iii. Superior aesthetics. Q.18. Orthodontic bands.
iv. Risk of caries under loose bands is eliminated.
Ans.
v. It is the best method in case of partially erupted and
fractured teeth and teeth with abnormal shapes. [Ref SE Q.10]
Q.15. Enumerate the objectives of stage I of Begg’s tech- Q.19. Ceramic brackets.
nique.
Ans.
Ans.
Brackets are passive orthodontic components. Ceramic
There are three different stages in Begg’s treatment:
brackets are the latest brackets.
The objectives of stage I Begg’s technique are as follows:
l Ceramic brackets were introduced in the 1980s and are
Stage I is concerned with:
made of aluminium oxide or zirconium oxide.
l Alignment
l Advantages:
l Correction of spacing, crowding and rotation of teeth
They are durable and resist staining.
l Overjet and overbite reduction
They are dimensionally stable and do not distort in oral
l Achieving an edge-to-edge anterior bite
cavity.
Q.16. Molar tubes. l Disadvantages:

Ans. They are brittle and bulky in size.


They exhibit greater friction at wire–bracket interface
l The orthodontic attachments that are generally used for compared with metallic brackets.
molars are called ‘buccal tube or molar tubes’.
l They are generally used on molars and help to provide Q.20. Types of elastics.
better three-dimensional control of these anchor teeth.
Ans.
Classification of molar/buccal tubes:
A. Based on mode of attachment [Same as SN Q.1]
i. Weldable
ii. Bondable Q.21. Parts of fixed orthodontic appliances.
B. Based on lumen shape
Ans.
i. Round
ii. Oval [Same as SN Q.3]
iii. Rectangular
Section | I  Topic-Wise Solved Questions of Previous Years 205

Topic 25
Myofunctional and Orthopaedic Appliances
COMMONLY ASKED QUESTIONS
LONG ESSAYS:
1. Classify myofunctional appliances. Discuss in detail the mechanism of action, fabrication and trimming of
activator.
2. What are myofunctional appliances? Describe in detail about preparation and uses of oral screen?
3. Classify functional appliances. Describe functional appliances how do they work and modify growth.
4. Define and classify functional appliances. Write in detail about Frankel appliance.
5. What is myofunctional appliance? Discuss about bionator.
6. Enumerate various types of functional regulators and describe the functional regulator II.
7. What are the functional appliances? Give examples. Discuss any one appliance in detail? [Same as LE Q.1]
8. What are myofunctional appliances? Classify them. Explain activator in detail? [Same as LE Q.1]
9. What are functional appliances? Give examples. Describe the trimming and mechanism of action of Anderson
appliance? [Same as LE Q.1]
10. Enumerate the uses of oral screen. How will you fabricate an oral screen? [Same as LE Q.2]
11. Define a functional appliance. What are the indications, mode of action of FR II appliances? [Same as LE Q.4]
12. Discuss about various types of Frankel functional regulators. [Same as LE Q.6]
13. Name the components of FR2 (Frankel 2) appliances and describe their action? [Same as LE Q.6]

SHORT ESSAYS:
1. Catalan’s appliances.
2. Jasper jumper.
3. Write briefly on upper anterior bite plane.
4. Activator.
5. Oral screen and their indications.
6. Twin block appliance.
7. Functional appliances – Classification.
8. Philosophy of Frankel appliance.
9. Difference between activator and Frankel appliance.
10. Case selection for functional appliance.
11. Catalan’s appliance. Mention disadvantages of this appliance. [Same as SE Q.1]
12. Anderson activator. [Same as SE Q.4]

SHORT NOTES:
1. Oral screen – Uses. [Ref LE Q.2]
2. Lip bumper.
3. Components of FR II appliances.
4. Types of Frankel appliance. [Ref LE Q.6]
5. Bionator appliance.
6. Activator.
7. Catalan’s appliance.
8. Define and classify myofunctional appliances.
9. Upper anterior bite plane – Mechanism of action.
10. Sved bite plane.
11. Advantages of Jasper jumper.
12. Fixed functional appliances – Classification and advantages.
206 Quick Review Series for BDS 4th Year, vol 1

1 3. Action of functional appliances.


14. Mode of action of activator.
15. Indications of twin block.
16. Contra indications of activator. [Ref LE Q.1]
17. Denholtz appliance.
18. Orthopaedic force in orthodontics.
19. Chin cap.
20. Extraoral traction.
21. Describe various types of headgears available to control the growth of maxilla.
22. Orthopaedic appliance – Components.
23. Effects of face mask.
24. Name the headgears used in orthodontics.
25. High pull headgear.
26. Enumerate various types of facemasks.
27. Mention various functional regulators? [Same as SN Q.4]
28. Give indications for activator therapy. [Same as SN Q.6]
29. Face mask therapy – Advantages. [Same as SN Q.24]
30. Indications of face mask therapy. [Same as SN Q.23]
31. Headgear – Uses. [Same as SN Q.24]

SOLVED ANSWERS
LONG ESSAYS:
Q.1. Classify myofunctional appliances. Discuss in de- Indications:
tail the mechanism of action, fabrication and trimming i. Cases of class I malocclusion with open bite or
of activator. deep bite.
ii. Class II division 1 malocclusion
Ans.
iii. Class II division 2 malocclusion
Classification of myofunctional appliances: iv. Class III malocclusion
Myofunctional appliances can be classified in a number v. Phase I treatment before fixed appliance treatment
of ways: in children with lack of vertical development in
I. Tooth-borne active appliances lower facial height
Tooth-borne passive appliances vi. As a retention appliance
Tissue-borne passive appliances vii. As a habit breaking appliance
II. Myotonic appliances viii. Used in obstructive sleep apnoea
Myodynamic appliances
III. Removable functional appliances
Fixed functional appliances
IV. Group I appliances {SN Q.16}
Group II appliances
Group III appliances Contraindications:
The appliance cannot be used in:
l Class I with crowded teeth because of dishar-

{SN Q.16} mony between tooth size and jaw size.


l Children with excessive lower facial height
Activator: and extreme vertical mandibular growth.
l Activator is also known as Norwegian appliance,
l Children with severely procumbent lower
Monobloc, Andresen and Haupl appliance or incisors.
Andresen’s appliance. l Children with nasal stenosis caused by struc-

tural problems within nose or chronic un-


treated allergy.
l It is developed by Andresen and Haupl.
l In non-growing individuals, it has limited
l Activator is called so because the wearing of this loose
application.
fitting appliance activates the muscles.
Section | I  Topic-Wise Solved Questions of Previous Years 207

Advantages: is for vertical activators where potential en-


l It uses existing growth of the jaws. ergy is utilized in extreme opening of mandi-
l Minimal oral hygiene problems during treatment. ble (.10 mm).
l Long intervals between appointments. c. Combination of kinetic and potential energy:
l Need minimal adjustments hence require short Vertical opening is 4–6 mm.
appointment. d. Differential eruption of teeth: Selective grind-
l More economical. ing leads to differential eruption of teeth.
Disadvantages: Effects of activator treatment on:
l Good patient cooperation is required during treat- A. Maxilla:
ment. i. Slight intermolar and intercanine expansion
l Post-treatment fixed orthodontic therapy may be ii. Upper incisors are retracted
needed for detailing of the occlusion, as activator iii. Increase in the vertical height in posterior teeth
cannot produce a precise detailing and finishing of iv. Restraining effect on the maxillary arch as a unit
the occlusion. B. Mandible:
l Activators are not used in cases of excessive lower i. Lower incisors proclination and reduction in deep bite
face height because they may produce moderate ii. Increase in the vertical height of posterior teeth,
mandibular rotation anteriorly and downwards. downward and forward translation of the mandi-
Mode of action of activator is as follows: ble and the teeth as unit
Activator is a loose fitting appliance which was de- C. Soft tissue:
signed by Andresen and Haupl to correct retrognathic Potentially competent lips become sufficiently com-
mandible. petent and causes changes in lip posture.
There are different views and mechanisms by which Construction of activator:
activator works. Components:
a. First view: l Labial bow

l According to Andersen and Haupl, the activa- l Jack screw

tor induces musculoskeletal adaptation intro- l Acrylic portion

ducing a new pattern of mandibular closure. The steps in the fabrication of activator are
l The appliance loosely fits into the mouth and i. Preparation of models: Working and study models
induces musculoskeletal adaptation introduc- ii. Registration of construction bite: Horizontal or
ing a new pattern of mandibular closure. vertical bites
l The patient has to move the mandible forward iii. Articulation of models
to engage the appliance, this results in stretch- iv. Wax-up and wire bending
ing of the elevator muscles of mastication v. Processing of appliance
which starts contraction, thereby setting up a vi. Trimming of activator
myotactic reflex. Registration of construction bite:
l This generates kinetic energy which causes,
prevention of further forward growth of the Horizontal bite: l Mandible advanced by 6–7 mm
maxillary dentoalveolar process. Movement of (H activator) l Vertical opening by 2–3 mm
the maxillary dentoalveolar process distally
Vertical bite (V-activator) l Mandible advanced by 2–3 mm
causes a reciprocal forward force on the man-
dible. (High angle cases) l Vertical opening by 7–8 mm
l In addition to this myotactic reflex, a condylar Only vertical opening l In deep bite
adaptation by backward and upward growth
Retrusive bite l Class III cases
occurs.
l A third factor is the force generated while

swallowing and during sleeping. Guidelines for bite registration:


b. Second view: i. Early mixed dentition:
l According to Harvold, Woodside and Herren, l The mandible should be moved forward until on

passive tension caused by stretching muscles, an average, about 4–5 mm. Then the upper pri-
soft tissue, tendinous tissue, etc., are responsi- mary canine occludes with interproximal area
ble for the action, which they called the visco- between the lower primary canine and the first
elastic property. primary molars.
l Activator causes change in viscoelastic prop- ii. Late mixed dentition:
erties of muscles and stretches soft tissues and l The mandible should be moved forward on an

induces skeletal adaptation. This mechanism average, around 6–8 mm until the upper canine
208 Quick Review Series for BDS 4th Year, vol 1

relates directly above the interproximal between Trimming for anteroposterior or sagittal movements:
the lower cuspid and first bicuspid. The following movements can be achieved in the antero-
iii. Anterior midline: posterior plane.
l When the bite registration is taken, the upper and i. Protrusion of incisors:
lower midlines should coincide. l It can be produced by loading the entire lingual
l If there is skeletal midline deviation, bite registra- surface of the incisors with acrylic.
tion is done with midlines coinciding. Or
l No attempt should be made to correct the mid- Only the incisal portion of the lingual surface
lines if there is dental midline shift. is loaded.
iv. Articulation: l Protrusion can also be achieved with accessory
l Reverse articulation provides good access during elements like protrusion springs, wooden pegs
acrylization of the appliance. and gutta-percha.
Wire bending: ii. Retrusion of incisors:
l A passive labial bow is made with 0. 9 mm l Retrusion is achieved by trimming away the
wire. The labial bow should contact the middle acrylic from lingual surface of the incisors and
third of the labial surface of the upper anterior using ‘active labial bows’.
teeth and the ends of the bow cross between Distal movement of molars:
canine and first premolar or deciduous first l For distalizing movements, the guide planes load the
molar through the centre of interocclusal wax. molars on the mesiolingual surfaces and extends to
l It acts as a passive medium for the transmis- the area of greatest convexity.
sion of muscular forces to the maxillary teeth l Active springs can also be used to achieve distal
and arch. movement of molars.
Processing of appliance: l Distal movement of upper molars is indicated in
l This is done using either heat cure or cold cure. class II, while lower molars is indicated in class III
l Appliance consists of maxillary part, interoc- malocclusions.
clusal part and mandibular part. Mesial movement of molars:
Trimming of activator: l Mesial movement is achieved by the guide planes
After processing of the appliance, an interocclusal block contacting the teeth on the distolingual surfaces and
of acrylic is present between the upper and lower poste- extending to the greatest lingual circumference in the
rior teeth. Guiding grooves are placed in the interocclu- mesiodistal plane.
sal block to facilitate tooth movement. Guiding grooves l In class III, malocclusion mesial movement of poste-
are created using appropriate flame-shaped burs. rior teeth in upper arch is indicated.
Trimming for vertical movement: Transverse movements with activator:
The intrusion and extrusion are two movements that oc- l Activator may also be trimmed to stimulate expan-
cur in vertical plane with activator treatment. sion of buccal segment. This is done by allowing
i. Intrusion: contact of acrylic on the lingual surfaces of the teeth
l In deep bite cases intrusion of the incisor teeth can to be moved transversely.
be achieved by loading the incisal edges of teeth. l More better expansion can be achieved by incorpo-
l Molar intrusion is indicated in open bite cases rating jackscrew in the activator.
where it can be achieved by loading the cusps Guidelines for clinical control:
alone of the molars and grinding away the acrylic l It is important to ensure during treatment that the
from fossae and fissures. grooves maintain their contact.
ii. Extrusion: l Reshaping of grooves and padding with fast setting
l Extrusion of the incisor teeth indicated in open self-cure acrylic in contact areas should be carried out.
bite cases can be achieved by loading the lingual l Wearing time of the appliance should be monitored.
surfaces above the area of greatest convexity and Appliance is worn for 2–3 h during the first 2 weeks.
it can be enhanced also by placing the labial bow Then increased to full night time wear.
above the area of convexity. l Any trauma or sore spots should be corrected.
l Extrusion of molars indicated in deep bite cases is Retention period:
achieved by loading the lingual surfaces above the l Retention period starts as the bicuspid exchange has
area of greatest convexity in maxilla and below in been completed, and an adult class I occlusion is
mandible. established.
l During supraeruption of molars, selective trim- l Following active treatment average retention period is
ming is done. In this either upper or lower molars 6–8 months following which wearing of the appliance
are allowed to erupt individually or both together. is gradually tapered off over a period of 2 to 3 months.
Section | I  Topic-Wise Solved Questions of Previous Years 209

Modifications of activator:
{SN Q.1}
l Eschler’s modification

l Herren’s modification Indications/uses:


l Wunderer’s modification l Used mostly to intercept mouth breathing habit

l Bow activator of A.M. Schwarz and can also be used to intercept other habits like
l Karwetzky appliance thumb sucking, tongue thrusting, lip biting and
l Cut out or palate free activator cheek biting.
l The propulsor l Used for correction of mild distocclusions and

l Elastic open activator mild anterior proclination.


l Kinetor by Stockfish l Used to perform muscle exercises that help in cor-

rection of hypotonic lip and check muscles.


Q.2. What are myofunctional appliances? Describe in
l Correction of flaccid hypotonic orofacial muscu-
detail about preparation and uses of oral screen?
lature.
Ans. l Used as both active and passive appliances.

l Counteract deficiencies in lip posture and

function.
{SN Q.1}
l Vestibular screen or oral screen was first introduced
by ‘Newel’ in 1912 (Fig. 25.1). Steps in the fabrication of oral screen are as follows:
l Vestibular screens are also called lip moulders.
i. Appliances are preferably made in clear acrylic.
l Oral screen or vestibular screen is simple functional
ii. Upper and lower impressions are made and work-
appliance that takes the form of a curved shield of ing models are poured, reproducing depths of
acrylic placed in the labial vestibule. vestibular sulcus.
l It is a functional appliance because it has no active
iii. Upper and lower casts are occluded in normal in-
elements designed to produce force. It produces its tercuspation and sealed together using plaster.
effect by redirecting the pressure of the muscles and iv. A construction bite plane should be taken to ad-
soft tissues like lips and cheek. vance the mandible in case the appliance is being
used for correction of distocclusion.
v. Vestibular screen should extend into the sulcus to
the point of mucosal tissue reflects outwards and
should not impinge on frenum and muscle attach-
ments. Posteriorly it should extend up to the distal
margin of the last erupted molar.
vi. Models are covered with 2–3 mm of wax over the
labial surface of the teeth and alveolar process. In
case of proclined teeth which need to be retracted
the wax relief is removed to expose the incisal one-
third of the teeth.
vii. Appliance is fabricated in either self-cure/
heat-cure acrylic resin.
viii. It is trimmed and polished.
Management of appliance:
l Ask the patient to wear the appliance at night and

2–3 h during day time and patient is instructed to


maintain lip seal.
l The areas of appliance causing irritation to sulcus
Fig. 25.1  Vestibular screen.
and frenum are carefully trimmed.
l Padding with quick setting self-cure acrylic is done

Principle: in areas where tooth contact is present. Padding is


l Vestibular screen works on the principle of both done with pink acrylic.
force application as well as force elimination. l Breathing holes should be gradually reduced in size.

l The vestibular screen can be used either to apply the Modifications of the vestibular screen:
forces of the circumoral musculature to certain teeth Number of modifications of vestibular screen are as
or to relieve those forces from the teeth thereby follows:
allowing them to move due to forces exerted by the i. Hotz modification
tongue. ii. Double oral screen
210 Quick Review Series for BDS 4th Year, vol 1

iii. Screen with breathing holes l Functional appliances are defined as loose fitting or pas-
iv. Oral screen used in open bite cases sive appliances which harness natural forces of the
v. Rehak’s modification orofacial musculature that are transmitted to the teeth
and alveolar bone through the medium of the appliance.
i. Hotz modification:
l These appliances either transmit, eliminate or guide the
The oral screen can be fabricated with a metal
natural forces of the musculature and are used for
ring projecting between upper and lower lips.
growth modification procedures that are aimed at inter-
This ring can be used to carry out various muscle
cepting and treating jaw discrepancies.
exercises.
They can bring about the following changes:
ii. Double oral screen by Krauss:
l An increase or decrease in jaw size
l Useful in patients with abnormal tongue pos-
l A change in spatial relationship of the jaws
ture and tongue thrust.
l Change in direction of growth of the jaws
l In patients with tongue thrust habit an addi-
l Acceleration of desirable growth
tional screen is placed on the lingual aspect of
the teeth. This is attached to vestibular screen Treatment principles:
by means of a thick wire that runs through the Functional appliances work on two major principles:
bite in the lateral incisor region. i. Force application
iii. Oral screen with breathing holes: ii. Force elimination
l In case of mouth breathers, the vestibular i. Force application:
screen should be fabricated with a number of l Most of the fixed and removable function ap-

holes that are gradually closed in a phased pliances work on this principle.
manner as nasal breathing takes over. l Compressive stress and strain act on the sur-

l Place breathing holes in the labial aspect of rounding structures involved and primarily re-
the oral screen. sult in an alteration in form with a secondary
l A button with a string attached is placed on adaptation in function.
the lingual aspect. ii. Force elimination:
l Patient is instructed to perform exercises by l This principle allows optimal development of denti-

pulling the string through the breathing hole. tion by elimination of abnormal and restrictive envi-
iv. Oral screen used in open bite cases: ronmental influences on the dentition.
l The tongue is kept away from the dentition by l Thus function is rehabilitated with a secondary

an acrylic projection. change in form.


v. Modification of Rehak: Mode of action of functional appliances:
l In this a nipple is combined with the screen Functional appliances are capable of producing the fol-
which projects out. The nipple has to be re- lowing changes:
tained by the lips. Therefore to increase the i. Orthopaedic changes
effects of oral screen, the natural sucking ii. Dentoalveolar changes
movements are used. iii. Muscular changes
Advantages: i. Orthopaedic changes:
l Simple and versatile appliance for early intercep-

tive treatment.
l The appliance establishes a better muscle balance
Capable of accelerating the growth in
between the tongue and buccinators mechanism. the condylar region
l Corrects the abnormal relationships of upper and
Able to bring about remodelling of the
lower lips to each other and makes it possible to
glenoid fossa
achieve near normal lip seal. FA can be
l Develops effective mechanism for reducing or
Designed to have restrictive influence
eliminating hyperactive mentalis muscle. on the growth of the jaws
l Contributes to the development of a proper func-

tioning occlusion. Change the direction of growth of


Disadvantages: the jaws
l It is not a complete mechanotherapy.
ii. Dentoalveolar changes:
l It forms only an initial or phase 1 correction of
Functional appliances can bring about dentoalve-
orthodontic problems. olar changes in the:
Q.3. Classify functional appliances. Describe functional a. Sagittal direction
appliances how do they work and modify growth b. Transverse direction
c. Vertical direction
Ans.
Section | I  Topic-Wise Solved Questions of Previous Years 211

Sagittal Transverse Vertical IV. (a)  Group I appliances:


Most FA allow the FA can bring FA can be designed For example: oral screen and inclined planes
upper anteriors to about expansion to allow selective They consist of appliances that transmit the
tip palatally and of dental arches eruption of teeth. muscle force directly to the teeth for the pur-
lower anteriors to by incorporating pose of correction of the malocclusion.
tip labially. screws in them (b) Group II appliances:
or by shielding the For example: activator and bionator.
buccal muscles
These appliances reposition the mandible
away from dental
arch.
and the resultant force is transmitted to the
teeth and other structures.
(c) Group III appliances:
iii. Muscular changes – Functional appliances can
For example: Frankel appliance and vestibu-
improve the tonicity of the orofacial muscula-
lar screen
ture.
These appliances also reposition the man-
Classification of myofunctional appliances:
dible but their area of operation is vestibu-
Myofunctional appliances can be classified in a number
lar, outside the dental arch.
of ways as follows:
Visual treatment objective:
I. Tooth-borne active appliances
Is an important diagnostic test undertaken before making
Tooth-borne passive appliances
a decision to use a functional appliance. It is performed
Tissue-borne passive appliances
by asking the patient to bring the mandible forward. An
II. Myotonic appliances
improvement in profile is considered as a 1ve indication
Myodynamic appliances
for the use of functional appliance.
III. Removable functional appliances
In case the profile worsens, then other treatment mo-
Fixed functional appliances
dalities have to be considered.
IV. Group I appliances
Group II appliances
Group III appliances Q.4. Define and classify functional appliances. Write in
detail about Frankel appliance.
I. (a)  Tooth-borne active appliances:
They include modifications of activator and Ans.
bionator that include expansion screws or l Functional appliances are defined as ‘loose fitting or pas-
other active components like springs to pro- sive appliances which harness the natural forces of the
vide intrinsic force for transverse or antero- orofacial musculature that are transmitted to the teeth
posterior changes. and alveolar bone through the medium of appliance’.
(b) Tooth-borne passive appliances: Classification of functional appliances:
For example: activator, bionator and Herbst i. Functional appliances:
appliance Tooth-borne passive appliances (Profitt, 1993)
They have no intrinsic force generating compo- l They are tooth-borne appliances that have no

nents such as springs or screws. They depend on intrinsic force generating components such as
the soft tissue stretch and muscular activity to springs or screws.
produce desired treatment results. l They depend on the soft tissue stretch and

(c) Tissue-borne passive appliances: muscular activity to produce the desired treat-
For example: functional regulator of Frankel ment results.
They are mostly located in the vestibule and For example: Andresen/Haupl activator, Woodside
have little or no contact with the dentition. activator, Balter’s bionator and Herbst appliance
II (a) Myotonic appliances: Tooth-borne active appliances:
They are dependent on muscle mass for their l They include modifications of activator and

action. bionator.
(b) Myodynamic appliances: l These include expansion screws or other active

They depend on the muscle activity for their components like springs to provide intrinsic
function. force for transverse or anteroposterior changes.
III (a)  Removable functional appliances: For example: elastic open activator (EOP), modi-
They can be removed and inserted into the fied bionator, stock fish appliance
mouth by the patient. Tissue-borne passive appliances:
For example: activator, bionator l Tissue-borne appliances are usually located in

(b) Fixed functional appliances: the vestibule and have little or no contact with
They are fitted on the teeth by the operator and the dentition.
cannot be removed by the patient at will. For example: oral screen and lip bumpers
212 Quick Review Series for BDS 4th Year, vol 1

Tissue-borne active appliances: Mode of action of Frankel appliance:


For example: Frankel appliance The concepts pertaining to the mechanism of action of
Other classification: functional regulators can be dealt under the following
ii. Myotonic appliances: headings:
l They are functional appliances that depend on i. Vestibular area of operation:
the muscle mass for their action. l According to Frankel, malocclusion is a result of

For example: Andresen appliance, Woodside faulty muscle posture and muscle imbalance.
activator l It prevents the aberrant muscular force from act-

Myodynamic appliances: ing on the teeth with the help of buccal and lips
l They are functional appliances that depend on shields. By removal of the restraining influence it
the muscle activity for their function. EOP, enables outward development of the arches.
modified bionator and Bimler. l The major part of Frankel appliance is confined

iii. Removable functional appliances: to the oral vestibule. The buccal shields and lip
They are myofunctional appliances that can be re- pads hold the labial and buccal musculature
moved and inserted into the mouth by the patient. away from the teeth and prevents buccinator
For example: activator, bionator and Frankel’ ap- mechanism from acting on the dentition.
pliance l The primary aim of functional regulator is re-

l Semifixed functional appliances: establishment of adequate space condition in the


For example: Denholtz, Bass appliance lower part of the oral functioning space.
l Fixed functional appliances: l Functional regulator helps in the correction of

They are functional appliances that are fitted faulty muscle posture by acting as oral gymnas-
on the teeth by the operator and cannot be re- tics device.
moved by the patient at will. ii. Exercise device:
For example: Herbst, Jasper jumper, Saif l Frankel appliance also acts as exercise device

spring apart from restricting the faulty muscle posture.


According to Graber: l It stimulates normal function while eliminating

l Group I appliances: aberrant muscle activity. Hence full time wear of


They consist of appliances that transmit the muscle the appliance is recommended.
force directly to the teeth for the purpose of correc- iii. Tongue function:
tion of the malocclusion. Though Frankel appliance gives more importance to
For example: oral screen and inclined planes buccinator mechanism, tongue also plays significant
l Group II appliances: role in moulding the arches. Tongue force causes
These appliances reposition the mandible and the passive expansion of the arches.
resultant force is transmitted to the teeth and other iv. Anteroposterior correction:
structures.
For example: activator and bionator Stimulation of lateral pterygoid muscle
l Group III appliances:

These appliances also reposition the mandible but


Increased activity of the retrodiscal pad
their area of operation is the vestibule, outside the
dental arch.
For example: Frankel appliance and vestibular screen Increased growth of condylar cartilage
Frankel appliance:
l Frankel appliances or functional regulators (FR)

are functional appliances introduced by Professor Postero-superior deposition of bone in condyle


Dr Rolf Frankel of Germany.
l Functional regulator is also called functional correc-

tor or vestibular appliance or Frankel appliance or Growth of mandible anteroposteriorly


oral gymnastic appliance or orofacial orthopaedic
appliance. v. Maxillary restraining effect:
l Functional regulators are tissue-borne type of func- Frankel appliances have a restraining effect on the
tional appliance. As the name implies, treatment maxillary teeth and arch.
with this appliance is directed towards the func- vi. Decrowding during eruption:
tional disorders responsible for dental or skeletal l Decrowding during eruption is a feature of all the

malformations. Frankel appliances.


Section | I  Topic-Wise Solved Questions of Previous Years 213

l The tension created in the soft tissues by ves- l Acrylic extends up to the distal of the first per-
tibular screen causes outward bending of the manent molars. The maxillary plate covers
thin buccal plate, thereby facilitating outward only the molars and the premolars with ante-
drift of the teeth. rior region remaining uncovered. Acrylic
vii. Differential eruption: extends 2 mm below the gingival margin.
l Maxillary molars are prevented from downward

and forward movement by Frankel appliances. Palatal arch


l Establishment of correct sagittal relationship by (1. 2 mm diameter)
1–2 mm is possible due to differential eruption Wire components are
of lower molars.
viii. Periosteal matrix stimulation: Vestibular wire
The tension created in vestibule by buccal shields (0. 9 mm SS wire)
and lip pads elicits periosteal pull and causes bone
deposition. l Palatal arch emerges opposite middle of the
Periosteal matrix stimulation causes maxillary first premolar and follows contour of palate
arch expansion and mandibular anterior extension. following a curve that reaches distal surface of
first permanent molars. It is kept 1 mm away
Q.5. What is myofunctional appliance? Discuss bionator.
from the mucosa.
Ans. l Vestibular wire (0. 9 mm SS wire) emerges

from acrylic below the contact point between


Bionator was developed by Balters in 1950s.
canine and first premolar. It rises vertically and
Three types:
is bent at right angles to go distally along the
l Standard appliance
middle of the upper premolar crowns. Mesial
l Class III appliance
to the molar, a round bend is made so that the
l The open bite appliance
wire runs at the level of lower papilla up to
Indications:
mandibular canine where it is bent to reach the
i. In class II division 1 with following features:
upper canines. It forms mirror image on the
l Well-aligned dental arches
opposite side.
l Retruded mandibule
l The vestibular wire is kept away from the sur-
l Not very severe skeletal discrepancy
face of incisors by the thickness of a sheet of
l Labial tipping of upper incisors
paper. The lateral portions of wire are suffi-
ii. Class III malocclusion where reverse bionator
ciently away from the teeth to allow expansion
can be used.
of the arch.
iii. Open bite cases where open bite bionator can be
Class III appliance:
used.
l Used in mandibular prognathism.
Standard appliance:
l Acrylic parts are similar to standard appliances.
Palatal arch is placed in opposite direction, so that
rounded arch is placed anteriorly. Vestibular wire
runs over the lower incisors instead of terminating
at lower canines.
The open bite appliance:
l Used in open bite cases

l The palatal arch and vestibular wires – same as

standard appliance
Maxillary acrylic portion is modified – Even anterior
area is covered.
l Its purpose is to prevent tongue from thrusting

between the teeth as the tongue is responsible in


most cases for the open bite.
It consists of:
Q.6. Enumerate various types of functional regulators
l Slender acrylic body fitted to the lingual as-
and describe the functional regulator II.
pects of mandibular arch and part of the max-
illary arch. Ans.
214 Quick Review Series for BDS 4th Year, vol 1

The wire components include:


{SN Q.4}
a. Labial bow
l Frankel appliances or functional regulators (FR) b. Palatal bow
are functional appliances introduced by Professor c. Canine loops/extensions
Dr Rolf Frankel of Germany. d. Upper lingual wire
l Functional regulator is also called functional correc- e. Lingual crossover wire
tor or vestibular appliance or Frankel appliance or f. Support wire for lip pads
oral gymnastic appliance or orofacial orthopaedic g. Lower lingual springs
appliance. The acrylic parts are described below:
l Functional regulators are tissue-borne type of func- a. Buccal shields:
tional appliance. As the name implies, treatment with l The buccal shields are also called the vestibular
this appliance is directed towards the functional dis- shields.
orders responsible for dento-skeletal malformations. l It extends as deep into the vestibule as possible within
Types of functional regulators: the confines tissue attachment and patient comfort.
A. Functional regulator I (FR 1): l These shields stand away from the teeth and basal
This is used for treatment of class I and class II, alveolar bone and helps in unrestricted dentoal-
division 1 malocclusion. veolar development. In addition they also cause
The FR 1 is divided into the following three types: periosteal bone deposition.
FR 1 a – It is used for class I malocclusion where l Functions of buccal shield are physiotherapy,
there is minor crowding. training of cheek muscles to adopt to functional
It is also used for class I deep bite cases and de- performance and correction of spatial disorders.
layed development of basal bone and dental struc- b. Lip pads/labial pads:
tures. l The lower lip pads are called pelots.
FRl b – It is used for class II division 1 malocclu- l The lip pads are rhomboid shaped and fit on labial
sion with deep bite and where overjet does not surface of mandibular alveolar process.
exceed 5–7 mm. l The lip pads help in elimination of abnormal peri-
FR 1 c – It is used for severe class II, division oral muscle activity, i.e. hyperactive mentalis
1 malocclusion in which the overjet is more than muscle activity.
7 mm. l Lower lip pads help in eliminating lower lip trap
B. Functional regulator II (FR 2): which causes or accentuates the proclination of
This is used for correction of class II division 1 upper incisors.
and division 2. l Function: physiotherapy as well as forced training
C. Functional regulator III (FR 3): to prevent hyperactive mentalis from raising the
This is used for treatment of class III malocclu- lower lip.
sion due to maxillary deficiency. c. Lingual shield/pad:
D. Functional regulator IV (FR 4): l Lingual shied is situated or placed below the gin-
This is used for treatment of open bite and bimax- gival margin of the mandibular teeth and extends
illary protrusion. up to the distal surface of the second premolar.
E. Functional regulator V (FR 5): l It is positioned in place by the two connecting
This is a functional regulator that incorporates wires to the buccal shield.
headgear. It is indicated in high angle cases and l Functions: Forced training in mandibular retru-
vertical maxillary excess. sion cases to keep mandible in advanced position
by supporting action of lingual and labial shields.
It also stimulates protractor muscles of mandible
Functional regulator II (FR 2): by activating proprioceptors.
Among all of the above Frankel II (FR2) is the most The wire components are described as follows:
commonly used appliance and is discussed in detail a. Labial bow:
below: l The maxillary labial bow originates from the vestibu-
l The FR 2 consists of acrylic parts and wire com- lar shields and runs in the middle 3rd of the labial
ponents. surface of the maxillary incisors. It runs gingivally at
The acrylic parts include: right angles between lateral incisor and canine.
a. Buccal shields l It forms a gentle curve distally at the height of
b. Lip pads middle of canine root and re-embedded in buccal
c. Lower lingual pad shield.
Section | I  Topic-Wise Solved Questions of Previous Years 215

l The labial bow should be bent in an ideal contour g. Lower lingual springs:
and it should be passive in nature. l These recurved springs are embodied in the lin-

b. Palatal bow: gual pad. They are two in number rest against the
l Has a slight curve in a distal direction and stands lingual surface of the lower anteriors.
clear of the palatal tissues l The main uses of the lingual springs are as follows:

l Crosses the palate and runs interdentally between i. Prevent supraeruption of the lower incisors
the maxillary first molar and second premolar or ii. Screen the tongue pressure from lower
deciduous second molar and enters the acrylic incisor.
buccal shield iii. For proclining the lower incisors actively
l Makes a loop into the buccal shield and emerges iv. Cause bite opening by relative intrusion
to form an occlusal rest in molar between the me-
Q.7. What are the functional appliances? Give exam-
siobuccal and distobuccal cusps
ples. Discuss any one appliance in detail?
l Provides maxillary anchorage and stabilizing action

c. Canine loops: Ans.


l The canine loops are also called canine guards.
[Same as LE Q.1]
l They start with its tags in buccal shield and runs

palatally to the lingual surface of the canine for Q.8. What are myofunctional appliances? Classify them.
a distance of about 1 mm, then crosses the in- Explain activator in detail?
terproximal contact between canine and lateral
Ans.
incisor.
l They are kept 2–3 mm way from the buccal sur- [Same as LE Q.1]
face of the canines.
Q.9. What are functional appliances? Give examples.
l They help in keeping the perioral activity away
Describe the trimming and mechanism of action of
from canine and thereby help in passive expan-
Anderson appliance?
sion in canine area.
d. Upper lingual wire or lingual stabilizing bow: Ans.
l This wire is also called upper lingual wire or pro-
[Same as LE Q.1]
trusion bow.
l An upper palatal protrusion bow is present behind Q.10. Enumerate the uses of oral screen. How will you
upper incisors. This wire prevents the lingual tip- fabricate an oral screen?
ping of the incisors during treatment.
Ans.
l It originates from the vestibular shields and passes

between the upper canines and first deciduous [Same as LE Q.2]


molars and curves along the lingual surface of the
Q.11. Define a functional appliance. What are the indi-
upper incisors at the level of the cingulum.
cations, mode of action of FR II appliances?
e. Lingual crossover wire:
l It is made of 1. 25 mm stainless steel wire that Ans.
connects the lingual shield with buccal shields.
[Same as LE Q.4]
l It runs 3–4 mm below the lingual gingival margin

and follows the contour of the lingual mucosa. Q.12. Discuss about various types of Frankel functional
l It is placed 1–2 mm away from the mucosa. regulators.
l It runs between the mandibular first and second
Ans.
premolars.
f. Support wire for lip pads: [Same as LE Q.6]
l Labial support wire is made of 0. 9 mm wire. It
Q.13. Name the components of FR-2 (Frankle 2) appli-
serves as the skeleton and offers support for the
ances and describe their action?
lip pad.
l This wire should be placed at least 7 mm below Ans.
the gingival margin. The central wire is inverted
[Same as LE Q.6]
‘V’ shaped to accommodate the lower labial fre-
num. Another wire emerges from the lip pad and
gets embedded in the buccal shields. SHORT ESSAYS:
l Lower labial wires or vestibular wires are the con-
Q.1. Catalan’s appliances.
necting wires between the labial pad and the buc-
cal shield. Ans.
216 Quick Review Series for BDS 4th Year, vol 1

l Catalan’s appliance is also called ‘lower anterior in- tube of the maxillary first molar. Anteriorly the mod-
clined plane’. ule is anchored to the lower arch wire distal to the
l It is used to treat maxillary teeth in crossbite and is mandibular canine by a small bayonet bend and lexan
constructed on the lower anterior teeth. bead.
l It can be made of acrylic or cast metal and can be de- iv. The Jasper modules are available in seven sizes
signed to treat single tooth or a segment of the upper ranging from 26 mm to 38 mm in length.
arch in crossbite. Indications:
l The inclined plane is designed to have a 45° angulation l Basically indicated in class II malocclusion with

which forces the maxillary teeth in crossbite to a more maxillary excess and mandibular deficiency
labial position. Mechanism of action:
Indications: l Selection of force module is by measuring the

l Crossbite cases with adequate space in the arch for distance between mesial aspect of upper facebow
the alignment of the maxillary teeth tube and distal aspect of the lexan ball distal to
l Cases where the crossbite is due to a palatally placed mandibular canine. Add 12 mm to this length to
maxillary incisors get the required length of the force module.
Disadvantages: l When the teeth come into occlusion, the force

l Speech problems during therapy. module being longer tends to curve, thereby pro-
l Patient has to follow certain dietary restrictions. ducing a mesial force on the mandibular arch and
l Use of the appliance for more than 6 weeks can re- a distal force on the maxillary arch.
sult in anterior open bite due to supra-eruption of the Effects of Jasper jumper:
posteriors. It brings about both skeletal and dentoalveolar
l The appliance may need frequent recementation. changes in the ratio of 40:60.
a. Skeletal effects:
Q.2. Jasper jumper.
l Holds and displaces the maxilla distally

Ans. with a small shift of point A distally


l Clockwise rotation of mandible
Jasper jumper is a flexible fixed tooth-borne functional
l Forward movement of condyles
appliance introduced by J.J. Jasper in 1980.
Dental changes:
l Posterior tipping and intrusion of upper molar and

palatal tipping of maxillary incisors


l Anterior translation and tipping of mandibular

teeth and intrusion of mandibular incisors


Advantages:
l Produces continuous forces.

l Compared to Herbst appliance, it allows greater

degree of mandibular freedom.


l It is easier to maintain better oral hygiene.

Q.3. Write briefly on upper anterior bite plane.


Ans.
l Bite plane is the simplest form of functional appliance.
It is nothing but extension of base plate which serves
Appliance design: various functions apart from forming the framework of
i. The appliance uses a modular system commonly the appliance.
known as Jasper jumper, which can be attached to l Upper anterior bite plane consists of acrylic platform

fixed appliances that are placed on the U/L arches. parallel to the occlusal plane which is present behind the
ii. Jasper jumper is constructed of stainless steel coil upper incisor teeth on which the lower incisors bite.
that is attached at both ends to SS end caps. The Mechanism of action
module is given an opaque polyurethane covering l Anterior bite plane causes differential eruption of

for purpose of hygiene and comfort. The end caps posterior teeth and they also cause relative intrusion.
are attached to fixed appliance at the maxillary pos- l When appliance is worn the posterior teeth are freed

terior and mandibular anterior region. from mastication and occlusion and they supraerupt
iii. The force module is attached posteriorly to maxillary causing reduction of deep overbite, otherwise known
arch by a ball pin that passes through the facebow as ‘opening the bite’.
Section | I  Topic-Wise Solved Questions of Previous Years 217

l The bite opening should not interfere with normal l Vestibular screen or oral screen was first introduced by
freeway space and posterior teeth should be main- ‘Newel’ in 1912.
tained at 2–3 mm separation. l Oral screen or vestibular screen is a simple functional

l Anterior bite planes are more successful in patients appliance that takes the form of a curved shield of
with large interocclusal clearance. acrylic placed in the labial vestibule.
Bite plane with labial bow: l It is a functional appliance because it has no active ele-

l Labial proclination of upper incisors is the important ments designed to produce force. It produces its effect
side effect of anterior bite plane which can be mini- by redirecting the pressure of the muscles and soft tis-
mized by placing a labial bow. sues like lips and cheek.
l The labial bow should not be activated for retraction Principle:
with bite planes. l Vestibular screen works on the principle of both

Sved bite plane: force application and force elimination.


l Modification of bite plane by extending the acrylic l The vestibular screen can be used either to apply the

plate over to cover incisal edges of upper anterior forces of the circumoral musculature to certain teeth
teeth is known as Sved bite plane. or to relieve those forces from the teeth thereby
l This eliminates forwarded component of force which allowing them to move due to forces exerted by the
causes proclination. tongue.
l Using Sved bite plane is another method to prevent Indications/uses:
labial proclination of upper incisors. l Used mostly to intercept mouth breathing habit and

can also be used to intercept other habits like thumb


Q.4. Activator.
sucking, tongue thrusting, lip biting and cheek biting.
Ans. l Used for correction of mild distocclusions and mild

anterior proclination.
l Activator is also known as Norwegian appliance,
l Used to perform muscle exercises that help in correc-
Monobloc, Andresen and Haupl appliance or Andre-
tion of hypotonic lip and check muscles.
sen’s appliance.
l Correction of flaccid hypotonic orofacial muscu-
l It is developed by Andresen and Haupl.
lature.
l Activator is called so because the wearing of this loose
l Used as both active and passive appliances.
fitting appliance activates the muscles.
l Counteract deficiencies in lip posture and function.
Advantages:
l It uses existing growth of the jaws. Q.6. Twin block appliance.
l Minimal oral hygiene problems during treatment.
Ans.
l Long intervals between appointments.

l Need minimal adjustments hence require short l Twin block appliance was introduced by William Clark.
appointment. l It is a highly successful and most popular appliance
l Tissues are not injured. which effectively combines inclined planes with inter-
l Appliance is worn at night time only and helps to maxillary and extraoral traction.
eliminate abnormal habits. l Appliance design:

l More economical. It has an acrylic part and wire components.


Disadvantages: a. The acrylic part consists of (i) lower block and
l Good patient cooperation is required during (ii) upper block.
treatment. b. Wire components include (i) clasps, (ii) labial
l Post-treatment fixed orthodontic therapy may be bow and (iii) construction bite.
needed for detailing of the occlusion, as activator l The appliance consists of upper and lower plates having

cannot produce a precise detailing and finishing of simple bite blocks that modify the occlusal inclined
the occlusion. plane efficiently.
l Activators are not used in cases of excessive lower l Twin block has two separate pieces of appliance and it

face height because they may produce moderate permits all functional movements and as well eating and
mandibular rotation anteriorly and downwards. speaking are possible with the appliance.
l It is bulky and uncomfortable. l Twin block should be worn full time (8–10 h/day).

l Little value in cases with crowding and very little or l They correct the maxillo–mandibular relationship

no response in older patients. through the functional displacement.


l Twin block produces rapid functional correction of mal-
Q.5. Oral screen and their indications. occlusion by guiding the mandible forward into correct
occlusion where the forces of occlusion are used to cor-
Ans. rect the malocclusion.
218 Quick Review Series for BDS 4th Year, vol 1

l In severe skeletal discrepancy cases extraoral traction is Myodynamic appliances:


used. A concorde facebow is used along with twin block l They are functional appliances that depend on

that combines extraoral traction with intermaxillary the muscle activity for their function. EOP,
traction. modified bionator and Bimler.
l A prescribed extraoral traction of 200 g each side for iii. Removable functional appliances:
8–10 h per day and intermaxillary force of 150 g from They are myofunctional appliances that can be re-
lower appliance to facebow can be applied with twin moved and inserted into the mouth by the patient.
block. For example: activator, bionator and Frankel appliance
l Indications: l Semifixed functional appliances:

i. Class I with open bite For example: Denholtz and Bass appliance
ii. Class I with closed bite l Fixed functional appliances:

iii. Class II division I and division II They are functional appliances that are fitted on
iv. Class III the teeth by the operator and cannot be removed
v. Lateral arch constriction and TMJ problems by the patient at will.
For example: Herbst, Jasper jumper and Saif spring
Q.7. Functional appliances – Classification.
According to Graber:
Ans. l Group I appliances:

They consist of appliances that transmit the


l Functional appliances are defined as ‘loose fitting or
muscle force directly to the teeth for the pur-
passive appliances which harness the natural forces
pose of correction of the malocclusion.
of the orofacial musculature that are transmitted to
For example: oral screen and inclined planes
the teeth and alveolar bone through the medium of
l Group II appliances:
appliance’.
These appliances reposition the mandible and
Classification of functional appliances:
the resultant force is transmitted to the teeth
i. Functional appliances:
and other structures.
Tooth-borne passive appliances (Profitt, 1993)
For example: activator and bionator
l They are tooth-borne appliances that have no
l Group III appliances:
intrinsic force generating components such as
These appliances also reposition the mandible
springs or screws.
but their area of operation is the vestibule, out-
l They depend on the soft tissue stretch and
side the dental arch.
muscular activity to produce the desired treat-
For example: Frankel appliance and vestibular
ment results.
screen
For example: Andresen/Haupl activator, Woodside
activator, Balter’s bionator and Herbst appliance Q.8. Philosophy of Frankel appliance.
Tooth-borne active appliances:
Ans.
l They include modifications of activator and

bionator. l Frankel appliances or functional regulators (FR) are


l These include expansion screws or other active functional appliances introduced by Professor Dr Rolf
components like springs to provide intrinsic Frankel of Germany.
force for transverse oranteroposterior changes. Philosophy or mode of action of Frankel appliance:
For example: EOP, modified bionator and stock The concepts pertaining to the philosophy of Frankel
fish appliance appliance are as follows:
Tissue-borne passive appliances: i. Vestibular area of operation:
l Tissue-borne appliances are usually located in According to Frankel, this appliance is designed to
the vestibule and have little or no contact with prevent the aberrant muscular force from acting on
the dentition. the teeth with the help of buccal and lip shields.
For example: oral screen and lip bumpers l By removal of the restraining influence, it

Tissue-borne active appliances: enables outward development of the arches.


For example: Frankel appliance l The primary aim of functional regulator is

Other classification: re-establishment of adequate space condition


ii. Myotonic appliances: in the lower part of the oral functioning space.
l They are functional appliances that depend on the l Functional regulator helps in the correction of

muscle mass for their action. faulty muscle posture by acting as oral gym-
For example: Andresen appliance and Woodside nastics device or exercise device.
activator.
Section | I  Topic-Wise Solved Questions of Previous Years 219

ii. Tongue function: Activator Functional regulator


Though Frankel appliance gives more importance
Worn only during night Worn throughout the day and
to buccinator mechanism, tongue also plays sig- time. night.
nificant role in moulding the arches. Tongue force
causes passive expansion of the arches. It does not act as a gymnas- It acts as oral gymnastics device
tic/exercise device. or exercise device.
iv. Anteroposterior correction:
Stimulation of lateral pterygoid muscle leads to Mandibular advancement is Only minimum advancement of
possible by 6–7 mm. mandible by 2.5–3 mm is possible.
increased activity of the retrodiscal pad resulting
in increased growth of condylar cartilage and It is bulky and uncomfort- Speech not impaired.
postero-superior deposition of bone in condyle able. Speech is not possible
with the appliance in
resulting in growth of mandible anteroposteriorly.
mouth.
v. Maxillary restraining effect:
Frankel appliances have a restraining effect on the
maxillary teeth and arch. Q.10. Case selection for functional appliance.
vi. Decrowding during eruption:
Ans.
l Decrowding during eruption is a feature of all

the Frankel appliances. A wider range of cases are being treated in recent years us-
l The tension created in the soft tissues by ves- ing functional appliances.
tibular screen causes outward bending of the The factors to be considered in selecting a case for func-
thin buccal plate, thereby facilitating outward tional appliance are as follows:
drift of the teeth. i. Age:
vii. Differential eruption: l Only in growing patients the growth modification

l Maxillary molars are prevented from downward therapy using functional appliances is possible.
and forward movement by Frankel appliances. l According to most authors, the age between 10 years

l Establishment of correct sagittal relationship by and pubertal growth phase is the optimum time for
1–2 mm is possible due to differential eruption myofunctional therapy.
of lower molars. ii. Social consideration:
viii. Periosteal matrix stimulation: l Unfortunately all cases cannot be treated with func-

The tension created in vestibule by buccal shields tional appliance alone. Patients who live far away
and lip pads elicits periosteal pull and causes bone from the clinic or those attending boarding school
deposition. may benefit from this appliance provided they fulfil
Periosteal matrix stimulation causes maxillary all other criteria for case selection.
arch expansion and mandibular anterior extension. iii. Dental consideration:
l A case that is devoid of gross local irregularities of
Q.9. Difference between activator and Frankel appli-
teeth like rotation and crowding is considered as an
ance.
ideal case for functional appliance therapy.
Ans. l A malocclusion can be treated satisfactorily by func-

tional appliance alone only in uncrowded cases.


Differences between activator and functional regulator
iv. Skeletal consideration:
are as follows:
l A case with moderate-to-severe class II malocclusion

is ideally suited for functional appliance treatment.


Activator Functional regulator
l Mild class III occlusion which presents a reverse
It is a tooth-borne loose It is a tissue-borne appliance overjet and an average overbite can be regarded as
fitting appliance. having firm maxillary anchorage.
potentially treatable with functional appliances.
It activates the muscles This appliance is designed to l Low angle cases, i.e. horizontal growers respond
hence called the activator. prevent the aberrant muscular well and the high angle cases with deep overbite are
force from acting on the teeth
and dental arches.
successfully treated using functional appliance while
the open bite type of cases pose a special problem.
Bulk of the appliance is Bulk of the appliance is placed
placed within the dentition. outside the dental arches in oral Q.11. Catalan’s appliance. Mention disadvantages of
vestibule. this appliance.
Activator has a single Functional regulator has three
Ans.
acrylic and only one wire acrylic parts joined by multiple
component. wire components. [Same as SE Q.1]
220 Quick Review Series for BDS 4th Year, vol 1

Q.12. Anderson activator. . Upper lingual wire


d
e. Lingual crossover wire
Ans.
f. Support wire for lip pads
[Same as SE Q.4] g. Lower lingual springs
Q.4. Types of Frankel appliance.
SHORT NOTES: Ans.
Q.1. Oral screen – uses.
[Ref LE Q.6]
Ans.
Q.5. Bionator appliance.
[Ref LE Q.2]
Ans.
Q.2. Lip bumper.
l Bionator appliance was developed by Balters in 1950s.
Ans. l Three types of bionator:
i. Standard appliance
l The lip bumper or lip plumper is a functional component
ii. Class III appliance
that is used along with lower and upper fixed appliance.
iii. The open bite appliance
l Types of lip bumpers:
Indications:
a. Based on their ability to be removed:
i. In class II division 1 malocclusion with well-
i. Combined fixed removable
aligned dental arches, retruded mandible with not
ii. Component of fixed appliance
very severe skeletal discrepancy
b. Based on arch used:
ii. Class III malocclusion where reverse bionator
i. Maxillary lip bumper or Denholtz appliance
can be used
ii. Mandibular lip bumper
iii. Open bite cases where open bite bionator can be
l Mechanism of action:
used
The lip bumper prevents hyperactivity of mentalis mus-
cles and abnormal force acting on the incisors. Q.6. Activator.
l Uses:
Ans.
i. Correction of lip trap
ii. Uprighting molars and as anchorage savers l Activator is also known as Norwegian appliance, Monobloc,
iii. Distalization of molars and reduction overjet by Andresen and Haupl appliance or Andresen’s appliance.
proclination of mandibular incisors l Indications:

i. Cases of class I malocclusion with open bite or deep bite


Q.3. Components of FR II appliances.
ii. Class II division 1 and division 2 malocclusion
Ans. iii. Class III malocclusion
iv. Phase I treatment before fixed appliance treatment
l Frankel appliances or functional regulators (FR) are
in children with lack of vertical development in
functional appliances introduced by Professor Dr Rolf
lower facial height
Frankel of Germany.
v. As a retention appliance and as well as habit break-
l Functional regulator is also called functional corrector
ing appliance
or vestibular appliance or Frankel appliance or oral
vi. Used in obstructive sleep apnoea
gymnastic appliance or orofacial orthopaedic appliance.
Functional regulator II (FR 2): Q.7. Catalan’s appliance.
Among all of the above Frankel II (FR2) is the most
Ans.
commonly used appliance.
l The FR 2 consists of acrylic parts and wire compo- l Catalan’s appliance is also called lower anterior inclined
nents as follows: plane.
The acrylic parts include: l It is constructed on the lower anterior teeth can be used

a. Buccal shields to treat maxillary teeth in crossbite.


b. Lip pads l The inclined plane can be made of acrylic or cast metal

c. Lower lingual pad and can be designed to treat single tooth in crossbite or
The wire components include: a segment of the upper arch in crossbite.
a. Labial bow l The inclined plane is designed to have a 45° angulation

b. Palatal bow which forces the maxillary teeth in crossbite to a more


c. Canine loops/extensions labial position.
Section | I  Topic-Wise Solved Questions of Previous Years 221

Q.8. Define and classify myofunctional appliances. l Basically indicated in class II malocclusion with maxil-
lary excess and mandibular deficiency.
Ans.
It brings about both skeletal and dentoalveolar changes
Myofunctional appliances are defined as loose fitting or in the ratio of 40:60.
passive appliances which harness natural forces of the oro- l Advantages:

facial musculature that are transmitted to the teeth and i. It produces continuous forces.
alveolar bone through the medium of the appliance. ii. Compared to Herbst appliance, it allows greater
Classification of myofunctional appliances: degree of mandibular freedom.
Myofunctional appliances can be classified in a number iii. It is easier to maintain better oral hygiene.
of ways as follows:
Q.12. Fixed functional appliances – classification and
I. Tooth-borne active appliances
advantages.
Tooth-borne passive appliances
Tissue-borne passive appliances Ans.
II. Myotonic appliances
Emil Herbst introduced the concept of fixed functional
Myodynamic appliances
appliances.
III. Removable functional appliances
l They are classified as follows:
Fixed functional appliances
A. Flexible fixed functional appliances
IV. Group I appliances
For example: Jasper jumper, Amoric torsion coils,
Group II appliances
Bite fixer
Group III appliances
B. Rigid fixed functional appliances
Q.9. Upper anterior bite plane – Mechanism of action. For example: Herbst appliance, FORSUS (Fatigue-
resistant device) and Ritto appliance
Ans.
l Advantages:

l Bite plane is the simplest form of functional appliance. i. It is designed to wear 24 h continuously and
l Upper anterior bite plane consists of acrylic platform thereby reducing need for patient cooperation.
parallel to the occlusal plane which is present behind the ii. Over all treatment time and efforts are re-
upper incisor teeth on which the lower incisors bite. duced.
Mechanism of action iii. Smaller in size and is better adapted to func-
l Anterior bite plane causes differential eruption of tions like mastication, swallowing and speech
posterior teeth and they also cause relative intrusion. iv. As appliance cannot be removed by the pa-
l When appliance is worn the posterior teeth are freed tient, it allows greater control by orthodontist.
from mastication and occlusion and they supraerupt
Q.13. Action of functional appliances.
causing reduction of deep overbite, otherwise known
as ‘opening the bite’. Ans.
l The bite opening should not interfere with normal
l Functional appliances are defined as loose fitting or pas-
freeway space and posterior teeth should be main-
sive appliances which harness natural forces of the
tained at 2–3 mm separation.
orofacial musculature that are transmitted to the teeth
l Anterior bite planes are more successful in patients
and alveolar bone through the medium of the appliance.
with large interocclusal clearance.
l These appliances either transmit, eliminate or guide the

Q.10. Sved bite plane. natural forces of the musculature and are used for
growth modification procedures that are aimed at inter-
Ans.
cepting and treating jaw discrepancies.
l Modification of upper anterior bite plane by extending Mode of action of functional appliances:
the acrylic plate over to cover incisal edges of upper Functional appliances are capable of producing the fol-
anterior teeth is known as Sved bite plane. lowing changes:
l This eliminates forwarded component of force which i. Orthopaedic changes
causes proclination. ii. Dentoalveolar changes
l Using Sved bite plane is another method to prevent iii. Muscular changes
labial proclination of upper incisors.
Q.14. Mode of action of activator.
Q.11. Advantages of Jasper jumper.
Ans.
Ans.
Mode of action of activator is as follows:
l Jasper jumper is a flexible fixed tooth-borne functional There are different views and mechanisms by which
appliance introduced by J.J. Jasper in 1980. activator works.
222 Quick Review Series for BDS 4th Year, vol 1

a. First view: Q.18. Orthopaedic force in orthodontics.


l According to Andersen and Haupl, the activa-
Ans.
tor induces musculoskeletal adaptation intro-
ducing a new pattern of mandibular closure, l The forces employed in orthodontic practice are basi-
which generates kinetic energy resulting in cally of two types:
prevention of further forward growth of the i. Orthodontic force
maxillary dentoalveolar process and a recipro- ii. Orthopaedic force
cal forward force on the mandible. l Orthopaedic force is that which affects the deeper cra-

l In addition to this myotactic reflex, a condylar niofacial structures.


adaptation by backward and upward growth l The orthopaedic forces are heavy forces of over 400 g

occurs. that bring about a change in the skeletal tissue.


l A third factor is the force generated while l Thus the orthopaedic appliances utilize the teeth as

swallowing and during sleeping. handles to transmit the forces to the adjacent skeletal
b. Second view: structures.
l According to Harvold, Woodside and Herren, l Amount of orthopaedic force:

passive tension caused by stretching muscles, Heavy forces of over 400 g totally compress the peri-
soft tissue, tendinous tissue, etc., is responsible odontal ligament on the pressure side and cause hyalin-
for the action, which is known as the viscoelas- ization which prevents tooth movement. These heavy
tic property. forces are conducted to the skeletal structures to pro-
c. Combination of kinetic and potential energy. duce an orthopaedic effect.
d. Differential eruption of teeth. l Duration of force:

Intermittent forces ranging from 12 to14 h a day are


Q.15. Indications of twin block.
believed to bring about minimum tooth movement but
Ans. maximum skeletal change. Thus most extraoral ortho-
paedic appliances are worn 12–14 h a day.
l Twin block appliance was introduced by William Clark.
l The commonly used orthopaedic appliances are head-
l It is a highly successful and most popular appliance
gear, face mask and chin cup.
which effectively combines inclined planes with inter-
maxillary and extraoral traction. Q.19. Chin cap.
l Indications:
Ans.
i. Class I with open bite
ii. Class I with closed bite l Chin cap or chin cup is an extraoral orthopaedic device
iii. Class II division 1 and division 2 which exerts upward and backward force on mandible
iv. Class III by applying pressure to chin and thereby preventing its
v. Lateral arch constriction and TMJ problems forward growth.
Indications:
Q.16. Contraindications of activator.
l Skeletal class III cases due to mandibular prognathism.

Ans. l Increased anterior facial height.

l Anterior open bite.


[Ref LE Q.1]
l It is used to restrict forward and downward growth of

Q.17. Denholtz appliance. the mandible.


Types:
Ans.
l They are of two types: (i) occipital pull chin cup and

l Denholtz appliance is another name for maxillary lip (ii) vertical pull chin cup.
bumper. i. Occipital pull chin cup:
l Mechanism of action: l Most commonly used type and it derives anchor-

The lip bumper prevents hyperactivity of mentalis mus- age from occipital region of head.
cles and abnormal force acting on the incisors. l Used in cases of class III malocclusion with mild-

l Uses: to-moderate prognathism.


i. Correction of lip trap ii. Vertical pull chin cup:
ii. Uprighting molars and as anchorage savers l It is used to correct anterior open bite cases.

iii. Distalization of molars and reduction overjet by Force magnitude and duration of wear (biomechanics):
proclination of mandibular incisors Force at the start of treatment: 150–300 g/side.
Section | I  Topic-Wise Solved Questions of Previous Years 223

After 2 months, force is h to: 450–700 g/side. iii. As a retention device after surgical correction of
Duration to wear appliance to achieve desired results: skeletal class III malocclusion.
14 h a day with a range of 10–16 h. Effects of facemask:
i. Forward movement of maxilla and proclination of
Q.20. Extraoral traction.
maxillary teeth
Ans. ii. Correction of crossbite, both posterior and anterior
iii. Rotation of the mandible downwards and back-
l Extraoral traction is a form of reinforced anchorage
wards
where in the anchorage units are situated outside the
Advantages:
mouth.
i. It is the only extraoral traction device for correction
l It is used to correct skeletal problems.
of maxillary deficiency with rapid improvement.
l Various sites used for extraoral traction are as follows:
ii. Rapid treatment progress and good patient compliance.
For example:
Occipital region – chin pad and high pull headgear Q.24. Name the headgears used in orthodontics.
Back of the neck – cervical headgear Ans.
Forehead – reverse pull headgear
Chin – chin cap Headgear is an extraoral orthopaedic appliance.
Types of headgears:
Q.21. Describe various types of headgears available to Based on the site of anchorage, headgears can be three
control the growth of maxilla. types:
Ans. i. Cervical headgear
ii. Occipital headgear
l Headgear is an extraoral orthopaedic appliance used to iii. Combination headgear
restrain the downward and forward growth of maxilla. Uses of headgear are as follows:
l Headgears are classified into three types: i. Orthopaedic effect:
i. High pull or occipital headgear: It exerts superior In the correction of skeletal class II due to prog-
and distal force. nathic maxilla in young individuals
ii. Combi pull/straight headgear/medium pull-distal ii. Anchorage:
and slight upward force is exerted. It can be used for reinforcement of anchorage
iii. Cervical or low pull headgear-distal/extrusive force during fixed orthodontic therapy.
is exerted on first molars. iii. Distalization of maxillary first molars.
l Cervical headgear is also known as Kloehn headgear iv. Uprighting of molars.
and it tends to move upper jaw distally. iv. Retention.
Q.22. Orthopaedic appliance – Components. v. Space maintenance and regaining.
vi. Overjet reduction.
Ans. vii. Intrusion of molars and incisors.
l Orthopaedic appliance is defined as any manipulation Q.25. High pull headgear.
that alters the skeletal system and associated motor
Ans.
organs.
l Following are the components of orthopaedic appliance: l According to site from which anchorage is gained head-
a. Facebow – outer bow and inner bow gears are of three types:
b. Anchorage source – head strap a. High pull
c. Force element – high pull or occipital headgear b. Medium pull
Combi pull/straight headgear or medium pull c. Low pull
Cervical low pull headgear l High pull headgears derive anchorage from back of

the head in occipital region or junction of parietal


Q.23. Effects of face mask.
and occipital regions.
Ans. l They produces a distally and superiorly directed

force on maxillary teeth and as well as maxilla.


l Face mask or the reverse pull headgear is an extraoral
l When forces exerted on molars it results in distaliza-
traction appliance used for correction of skeletal class III
tion and intrusion of molars.
malocclusion. It was popularized by Delaire in 1960s.
l It is used in treating high mandibular angle cases.
Indications:
i. Correction of class III skeletal malocclusion in young Q.26. Enumerate various types of facemasks.
children due to maxillary retrognathism.
Ans.
ii. Correction of centric relation and centric occlusion
discrepancy in pseudo-class III malocclusion. l The facemask is an extraoral traction appliance.
224 Quick Review Series for BDS 4th Year, vol 1

l The various types of face masks available are as follows: Q.29. Face mask therapy – advantages.
i. Hickham – reverse pull headgrear
ii. Delaire’s facemask Ans.
iii. Petit’s facemask [Same as SN Q.23]
iv. Tubinger facemask
Q.30. Indications of face mask therapy.
Q.27. Mention various functional regulators.
Ans.
Ans.
[Same as SN Q.4] [Same as SN Q.23]

Q.28. Give indications for activator therapy. Q.31. Headgear – uses.

Ans. Ans.
[Same as SN Q.6] [Same as SN Q.24]

Topic 26
Management of Common Malocclusions
COMMONLY ASKED QUESTIONS
LONG ESSAYS:
1 . Describe the causes of midline diastema and explain how will you correct the same?
2. Discuss features and management of class I malocclusion.
3. Discuss the causes and treatment of anterior crowding of teeth.
4. Describe the causes of median diastema and the measures to correct it? [Same as LE Q.1]
5. Enumerate the aetiologic factors causing crowding of the teeth. Mention their treatment. [Same as LE Q.3]
6. A child in the mixed dentition reports to you with crowding in the lower anterior teeth. Discuss your line of
treatment. [Same as LE Q.3]

SHORT ESSAYS:
1 . Management of midline diastema.
2. Clinical features of crowding of anterior teeth. [Ref LE Q.3]
3. Aetiology of spacing.
4. Aetiology and treatment of midline diastema. [Same as SE Q.1]
5. Midline diastema. [Same as SE Q.1]

SHORT NOTES:
1 . Maxillary midline diastema – aetiology. [Ref LE Q.1]
2. Lower anterior crowding. [Ref LE Q.3]
3. Spacing.
4. Rotation.
5. Imbrications.
6. Abnormal labial frenum.
7. Derotation of teeth.
8. Blanch test.
9. Midline diastema diagnosis and its causes. [Same as SN Q.1]
Section | I  Topic-Wise Solved Questions of Previous Years 225

1 0. Aetiology of diastema. [Same as SN Q.1]


11. Causes of crowding. [Same as SN Q.2]
12. Localized spacing. [Same as SN Q.3]
13. Aetiology and treatment of imbrications. [Same as SN Q.5]

SOLVED ANSWERS
LONG ESSAYS:
Q.1. Describe the causes of midline diastema and b. Tooth material–arch length discrepancy:
explain how will you correct the same? l A disparity where the arch length exceeds the tooth

Ans. material can result in midline diastema in conditions


such as missing teeth, microdontia, macrognathia and
extractions with resultant drifting of adjacent teeth.
{SN Q.1} c. Physical impediment:
l Abnormal frenal attachment: The presence of a thick
l Median or midline diastema is a form of localized
spacing between the two maxillary central incisors. and fleshy labial frenum gives rise to a midline dia-
l It is one of the most frequently seen malocclusions
stema. It prevents the two central incisors from ap-
which is easy to treat but often difficult to retain. proximating each other due to the fibrous connective
Aetiology: tissue interposed between them.
The midline diastema can be a result of a number d. Habits:
l Abnormal pressure habits like thumb sucking and
of causes such as:
a. Normal/developmental: tongue thrusting also predispose to midline diastema.
l These patients generally present with proclination
i. Physiologic median diastema
ii. Ethnic and familial and generalized anterior spacing.
l Spacing in the midline can be caused by midline soft
iii. Imperfect fusion at midline of premaxilla
b. Tooth material deficiency: tissue and hard tissue pathologies, e.g. cysts, tumours
i. Microdontia, peg laterals and missing and odontomes.
l Presence of an unerupted mesiodens between the
laterals
ii. Macrognathia roots of the two central incisors also predispose to
iii. Extracted tooth midline diastema.
c. Physical impediment e. Other causes:
l Midline diastemas can occur when certain therapeu-
i. Retained deciduous teeth
ii. Mesiodens, midline pathology, etc. tic procedures are undertaken.
iii. Enlarged labial frenum For example: The appearance of a midline spacing is
iv. Deep bite an important prognostic sign during rapid maxillary
d. Habits expansion and it indicates the opening of the inter-
i. Thumb sucking, tongue thrusting, etc. maxillary suture.
l Racial predisposition: The presence of midline spacing
ii. Frenum thrusting
e. Other causes also has a racial and familial background. The Negroid
i. RME race shows the greatest incidence of midline diastema.
ii. Milwaukee braces Investigations:
i. A proper history and clinical examination
ii. Blanch test
iii. Any pernicious oral habits
a . Normal/developmental: iv. Periapical radiograph
l Transient malocclusion: Midline diastema is very v. Model analysis
often seen as an incipient malocclusion that is self- l A proper history and clinical examination is
correcting. necessary to confirm whether it is localized or
l A midline spacing can occur during the mixed denti- part of generalized spacing.
tion period associated with the eruption of the per- l A blanch test is performed by pulling the up-
manent canines, i.e. the ugly duckling stage. This per lip outwards, the blanching of the tissue
condition usually corrects by itself when canines in the incisive papilla region palatal to the two
erupt and the pressure is transferred from the roots to central incisors confirms the presence of a
the coronal area of the incisors. thick and fleshy frenum.
226 Quick Review Series for BDS 4th Year, vol 1

l Look for any pernicious oral habits. l Midline diastema is often considered easy
l An intraoral periapical radiograph reveals to treat but difficult to retain.
V-shaped notching present between the central l The key to its successful management is the

incisors and as well they are a valuable aid in elimination of the aetiologic factors in-
diagnosing midline pathology that causes volved and long-term retention using suit-
spacing. able retainers. Since prolonged retention is
l Tooth material–arch length discrepancies can indicated, it is advisable to use lingual
be determined using model analysis. bonded retainers. The other retainers that can
Treatment of midline diastema: be used include banded retainers, Hawley’s
The treatment of midline diastema consists of three retainer, etc.
phases: Other methods of treatment:
a. Identifying and removal of cause i. Role of cosmetic restorations:
b. Active treatment Aesthetic composite resins are generally used to
c. Retention close very small midline diastema especially in
adult patients.
a. The first phase (identifying and removal of
ii. Prosthetic management:
cause)
If the diastema is big, closure by light-cure com-
l It involves removal of the aetiology.
posite will be unaesthetic. In these cases, if the
l Habits should be eliminated using fixed or
arch is well-aligned closure by giving an implant
removable habit breaking appliances.
or bridge is suggested.
l Unerupted mesiodens should be extracted.
iii. Surgical management:
l Frenectomy should be performed.
Surgery is done in some cases where there is a
l Any midline pathology should be treated as
median diastema in otherwise normal occlusion
indicated.
iv. Prosthesis/crown:
b. The second phase (active treatment)
Presence of peg shaped laterals or teeth with
It consists of active treatment using removable
other anomalies of shape and size require pros-
or fixed appliances.
thetic rehabilitation. Missing teeth should be
i. Removable appliances:
replaced with fixed or removable prosthesis.
l Simple removable appliances incor-
v. Physiologic median diastema that occurs during
porating finger springs or a split la-
ugly duckling stage of eruption is a self-correcting
bial bow can be used to close a mid-
condition that requires no treatment.
line spacing.
l Finger springs can be given distal to Q.2. Discuss features and management of class I maloc-
the two central incisors. An alterna- clusion.
tive would be to use a split labial
Ans.
bow. The labial bows are made to
extend up to the distal aspect of the Class I malocclusion is characterized by the mesiobuccal
opposite central incisor. cusp of maxillary first permanent molar occluding with
ii. Fixed appliances: mesiobuccal groove of mandibular first permanent molar
l Fixed appliances incorporating elas- and presence of normal interarch molar relation.
tics or springs bring about the most Angle’s class I malocclusion (neutro-occlusion):
rapid correction of midline diastema. Molar relation:
l Elastics can be stretched between the The mesiobuccal cusp of the upper first molar oc-
two central incisors in order to close cludes with the mesiobuccal groove of the lower first
the space. Elastic thread or elastic chain molar.
can be used between the two central Canine relation:
incisors for the same purpose. An alter- The mesial incline of the upper canine occludes with
native is to stretch a closed coil spring the distal incline of the lower canine whereas the
between the two central incisors. ‘M’ distal incline of the upper canine occludes with me-
shaped springs incorporating three he- sial incline of lower first premolar.
lices can be inserted into the two cen- Class I bimaxillary protrusion:
tral incisor brackets. This spring is acti- i. Class I bimaxillary malocclusion is a condition
vated by closing the helices. where both the key of occlusion and line of occlu-
c. The third phase (retention) sion are not altered.
l This phase of treatment involves retaining ii. The upper and lower anteriors are proclined and
the treated malocclusion. exist usually in an edge–edge relationship.
Section | I  Topic-Wise Solved Questions of Previous Years 227

Features of class I malocclusion are as follows: c. Peg laterals or small teeth results in spacing
Extraoral: between the rest of the teeth due to drifting.
i. Straight profile In such cases, the space for the lateral inci-
ii. Competent/incompetent lips sor can be regained by using a removable
iii. Normal/shallow/deep mentolabial sulcus appliance incorporating finger spring or
Intraoral: fixed appliances incorporating an open coil
Spacing and crowding spring.
i. Spacing of teeth iii. Prosthetic treatment:
ii. Crowding of teeth a. The space regained in cases of peg laterals
iii. Rotation of teeth can be used for a prosthetic crown on the
Proclination and retroclination: lateral incisor.
i. Proclination of teeth b. In cases of absence of maxillary lateral inci-
ii. Retroclination of teeth sors, they can be replaced by a fixed or re-
Protrusion and retrusion movable partial prosthesis.
i. Bimaxillary protrusion
ii. Bimaxillary retrusion Management of class I malocclusion with crowding:
Deep bite, open bite and crossbite:
Crowding is a common manifestation of class I malocclu-
sion which usually occurs due to tooth material and arch
i. Deep bite length discrepancy.
Anterior Various methods of treating crowding are as follows:
a. Gaining space:
i. Normal alignment of crowded teeth require space.
ii. Open bite Posterior On an average for every 1 mm of crowding, an equal
amount of space is required for correction.
Lateral ii. The various methods of gaining space include:
l Proximal stripping
Anterior l Expansion

l Extraction

l Molar distalization
iii. Crossbite l Derotation

l Proclination of anterior teeth

Posterior b. Orthodontic appliances:


i. Removable appliances:
Once the provision for space is made, teeth can be
Management of class I malocclusion with spacing: moved to normal position using removable appli-
l It can be managed by: ances incorporating coil springs, canine retractors,
i. Removal of the aetiology labial bows, etc.
ii. Orthodontic treatment ii. Fixed appliances:
iii. Combined orthodontic and prosthodontic Fixed appliances with multilooped arch wires or
treatment resilient nickel–titanium wires are most effective in
i. Removal of the aetiology: correction of crowding.
The aetiologic cause for the spacing should be The cases of mild crowding:
l Teeth can be aligned with removable orthodontic
diagnosed and eliminated.
For example: appliances incorporating labial bow and springs
l Spacing resulting from abnormal pres-
such as Z-spring, T-spring and flapper springs
sure habits can be treated by using habit after gaining space using expansion/proximal
breaking appliances. stripping.
l Surgical removal of cystic lesions is in-
In case of moderate crowding
l Teeth can be aligned using removable or fixed
dicted in cases of bony pathology.
ii. Orthodontic appliances: appliances after gaining the space.
l Fixed appliances make use of multilooped arch
a. Active removable appliances incorporating
labial bows can be used to close the space that wires or resilient nickel–titanium wires which
occurs in conjugation with proclination. are very effective in correction of crowding.
b. Fixed appliances along with elastic chains or In case of severe crowding:
l In cases with sever crowding, the extraction of
elastic threads are most effective in closure of
generalized spacing. premolars are indicated to gain the space.
228 Quick Review Series for BDS 4th Year, vol 1

l Following space gaining, the stages of treat- iii. Third-degree crowding is severe malalignment of
ment are as follows: all four incisors.
Stage I – retraction of canines using canine Aetiology of crowding:
retractor
Stage II – alignment of anterior teeth using {SN Q.2}
suitable labial bow
Stage III – retraction of the teeth using a re- Some of the common causes of crowding are as follows:
tention appliances like Hawley’s retainer i. Tooth material–arch length discrepancy is the
common reason for hereditary crowding. In-
Management of class I malocclusion with rotations: creased tooth material and decreased arch
l Rotated anterior teeth occupy less space hence re-
length usually leads to crowding.
quires additional space for their derotation. ii. Presence of supernumerary or extra teeth can
l Mild rotation of teeth can be corrected by using re-
result in crowded arrangement of teeth.
movable appliances by creating couple forces with iii. Prolonged retention of deciduous teeth.
the help of double cantilever spring (Z-spring) and a iv. Discrepancy in individual tooth size and shape
labial bow. like abnormally large teeth can predispose to
l When multiple rotations are present, fixed appliances
crowding.
are required to treat the case. v. Abnormal eruption path and altered eruption
l Derotation can be brought about by the use of derota-
sequence.
tion springs/elastics. vi. Rotation and transposition of tooth.
Q.3. Discuss the causes and treatment of anterior crowd- vii. Ankylosed primary tooth.
ing of teeth. viii. Premature loss of deciduous tooth results in
drifting of adjacent teeth into extraction space
Ans. predisposing to crowding.
Crowding is a common condition in class I malocclusion, ix. Prolonged retention of primary tooth.
where there is malalignment of teeth caused by inadequate
space. [SE Q.2]
Classification of crowding:
There are different methods of classification of crowding.
{Clinical features of class I crowding
The signs of crowding with class I molar relation are as
follows:
l Crowded mandibular incisor teeth

l Premature exfoliation of deciduous canines on the

crowded side due to displacement of erupting tooth


A B C D
l Reduced Leeway space

l Splaying out of maxillary permanent lateral inci-

sors and gingival recession on the labial surface of


Heredi- Environ- Prim- Second- Tertiary Simple Complex 1° 2° 3° prominent mandibular incisors
tary mental ary ary
l Bulging of canines in the unerupted position
l If no treatment is given, impaction of second per-
l Primary crowding is determined genetically and is
manent molar
caused by disproportionately sized teeth and jaws.
l Vertical palisading of the permanent maxillary
l Secondary crowding/acquired crowding is caused by
first, second and third molars}
loss of arch length due to environmental cause.
Diagnosis:
l Tertiary crowding also called late incisor crowding is
l Clinical examination:
due to late mandibular growth.
Carried out to determine extent and location of
l Simple crowding is due to disharmony between the
crowding.
size of the teeth and the space available for them
l Model analysis:
without skeletal, muscular or functional occlusal
Mixed dentition model analysis like Moyer’s analysis
problems.
is carried out to find out the arch length discrepancy.
l Complex crowding is caused and associated with skel-
Management of crowding:
etal, muscular and functional occlusal problems.
A. Management of crowding in mixed dentition is
l Crowding in mixed dentition is of three degrees:
as follows:
i. First-degree crowding is due to slight malalignment of
Age – after eruption of 12 12
the anterior teeth. No abnormality in supporting zone.
i. Slight crowding:
ii. Second-degree crowding is pronounced malalign-
In case of slight changes in the position of
ment of anterior teeth. No abnormality in supporting
anterior teeth, just wait and watch, no treat-
zone.
ment is required.
Section | I  Topic-Wise Solved Questions of Previous Years 229

ii. Moderate crowding: SHORT ESSAYS:


Lack of space by width of one lateral inci-
sor, can wait till the eruption of premolars Q.1. Management of midline diastema.
and at a later date expansion and guidance Ans.
of eruption is the treatment required.
Minimal or moderate crowding can be cor- The treatment of midline diastema consists of three phases:
rected by passive expansion achieved with a. Phase I – identifying and removal of cause
functional regulator and vestibular appliance. b. Phase II – active treatment
iii. Pronounced crowding: c. Phase III – retention
l Immediate treatment is required. a. The first phase (identifying and removal of cause)
l It can be treated with expansion, guidance l It involves removal of the aetiologic cause of midline
of eruption, serial extraction and extrac- diastema.
tion followed by orthodontic treatment. For example: Habits should be eliminated, extraction
Management of crowding in young adults: of unerupted mesiodens, frenectomy, surgical treat-
Investigations: ment of midline pathology if any.
l Arch length analysis for permanent dentition, b. The second phase (active treatment)
e.g. Carey’s analysis should be carried out. It consists of active treatment using removable or fixed
l Complete Kesling’s diagnostic set-up should appliances.
be carried out. i. Removable appliances:
l Treatment can be either by nonextraction or l Simple removable appliances incorporating
extraction. finger springs or a split labial bow can be used
Treatment: to close a midline spacing.
a. Nonextraction method of treatment: ii. Fixed appliances:
l Indicated in cases with mild discrepancy. l Fixed appliances incorporating elastics or
l Proximal reduction and treatment with springs bring about the most rapid correction
either removable or fixed appliances. of midline diastema.
l Lip bumpers are useful in increasing the For example: Elastics can be stretched between
arch length. the two central incisors in order to close the
l In cases of minor crowding arch expansion space ‘M’ shaped springs incorporating three
procedures and molar distalization are other helices can be inserted into the two central in-
methods to gain space. cisor brackets. This spring is activated by clos-
b. Extraction method of treatment: ing the helices.
l Treatment planning includes the choice of c. The third phase(retention)
extraction. Following extraction, treatment l This phase of treatment involves retaining the treated
is done with preferably fixed appliance malocclusion.
mechanotherapy. The key to its successful management is the elimina-
l If there is any unerupted tooth, it has to be tion of the aetiologic factors and long-term retention
brought into occlusion. using suitable retainers. For example: lingual bonded
Q.4. Describe the causes of median diastema and the retainers, banded retainers, Hawley’s retainer.
measures to correct it? Other methods of treatment:
i. Cosmetic restorations: Generally used to close
Ans. very small midline diastema.
[Same as LE Q.1] ii. Prosthetic management: If the diastema is big
and the arch is well aligned. Presence of peg-
Q.5. Enumerate the aetiologic factors causing crowding
shaped laterals or teeth with other anomalies of
of the teeth. Mention their treatment.
shape and size require prosthetic rehabilitation.
Ans. iii. Surgical management.
[Same as LE Q.3] iv. Physiologic median diastema that occurs during
ugly duckling stage of eruption is a self-correcting
Q.6. A child in the mixed dentition reports to you with condition that requires no treatment.
crowding in the lower anterior teeth. Discuss your line
of treatment. Q.2. Clinical features of crowding of anterior teeth.

Ans. Ans.
[Same as LE Q.3] [Ref LE Q.3]
230 Quick Review Series for BDS 4th Year, vol 1

Q.3. Aetiology of spacing. Spacing.


Ans. Ans.
Spacing is of two types: l Spacing is defined as imperfections in the teeth align-
A. Localized spacing ment and distance, wherein there is gap between two
B. generalized spacing teeth or many teeth.
Localized spacing: l Presence of spacing between the teeth is the commonest

Localized spacing is condition where spacing is present manifestation of the Class I malocclusion. Spacing may
in localized regions or areas. be generalized or localized.
Various causes of localized spacing are l Localized spacing is condition where spacing is present

l Missing teeth: in localized regions or areas.


Congenitally missing teeth causes localized spac- Causes of localized spacing are
ing but the problem may not be restricted to one l Missing teeth:

particular spot. Congenitally missing teeth causes localized spacing


l Unerupted teeth: Impacted or unerupted tooth but the problem may not be restricted to one particu-
causes localized spacing. lar spot.
l Premature loss of primary teeth: In this situation, l Unerupted teeth: Impacted or unerupted tooth causes

decision has to be made whether to close the localized spacing.


space or maintain the space and replace with an l Premature loss of primary teeth.

implant or bridge. l Prolonged retention of primary teeth results in ecto-

l Prolonged retention of primary teeth: This results pic eruption of permanent successor, and when the
in ectopic eruption of permanent successor, and primary tooth is exfoliated after ectopic eruption of
when the primary tooth is exfoliated after ectopic permanent successor space results.
eruption of permanent successor space results. Q.4. Rotation.
Generalized spacing:
The causes of generalized spacing are Ans.
l Microdontia: The presence of smaller teeth in the l Rotation can be defined as the spinning of the tooth
normal jaws will result in generalized spacing. around its long axis.
l Macrognathia: The bigger size of arch with nor- l Types of rotation:
mal size of teeth results in generalized spacing. i. Centric rotation – only rotation around the long axis
l Macroglossia: An unduly large tongue causes ii. Eccentric rotation – rotation with tipping of the tooth
generalized spacing. also
l Abnormal tongue posture also causes generalized l Derotation can be achieved by applying a couple. The
spacing. force required for rotation correction is 35–60 g.
l Certain sucking habits may also cause generalized l Rotation can be achieved by two ways: By using a
spacing. couple force, by using a single force and a stop.
l There is greater tendency for the rotation to relapse after
Q.4. Aetiology and treatment of midline diastema.
correction.
Ans.
Q.5. Imbrications.
[Same as SE Q.1]
Ans.
Q.5. Midline diastema.
l Imbrication denotes especially lower incisors arranged
Ans. in an irregular manner within the arch due to lack of
[Same as SE Q.1] space.
l Some of the common causes of imbrications are as

follows:
SHORT NOTES: i. Tooth material–arch length discrepancy
Q.1. Maxillary midline diastema – aetiology. ii. Presence of supernumerary teeth
iii. Discrepancy in individual tooth size and shape
Ans. iv. Abnormal eruption path
[Ref LE Q.1] v. Rotation and transposition of tooth
vi. Premature loss of deciduous or prolonged retention
Q.2. Lower anterior crowding. of primary tooth
Ans. Q.6. Abnormal labial frenum?
[Ref LE Q.3] Ans.
Section | I  Topic-Wise Solved Questions of Previous Years 231

l Abnormal frenal attachment is a physical impediment l The condition of abnormal labial frenum is diagnosed
causing midline diastema. by a 1 ve blanch test.
l The presence of a thick and fleshy labial frenum gives l A blanch test is performed by retracting or pulling the

rise to a midline diastema. upper lip outwards, the blanching of the tissue in the
l It prevents the two central incisors from approximating incisive papilla region palatal to the two central incisors
each other due to the fibrous connective tissue inter- confirms the presence of a thick and fleshy frenum.
posed between them. l If the blanching is present then the frenum is responsi-

l It can be surgically excised to correct midline diastema. ble for the midline diastema.
Q.7. Derotation of teeth. Q.9. Midline diastema diagnosis and its causes.
Ans. Ans.
l Derotation of teeth is a method of space gaining in [Same as SN Q.1]
orthodontics.
Q.10. Aetiology of diastema.
l Derotation of posterior teeth occupies more spaces. By

correcting a rotated tooth little amount of space can be Ans.


gained.
[Same as SN Q.1]
l Derotation is best achieved with fixed appliances incor-

porating springs or elastics using a force couple. Q.11. Causes of crowding.


l Few removable appliances can correct rotation of teeth
Ans.
by creating couple forces with the help of flapper spring
or double cantilever spring (Z-spring) and a labial bow. [Same as SN Q.2]
l Fixed appliances are best when multiple rotations are
Q.12. Localized spacing.
present. Derotation can be brought about by the use of
derotation springs, elastics, etc. Ans.
l Semifixed appliances can also be used for the correction
[Same as SN Q.3]
of rotations.
For example: whip spring and high labial low with sol- Q.13. Aetiology and treatment of imbrications.
dered T-spring
Ans.
Q.8. Blanch test.
[Same as SN Q.5]
Ans.

Topic 27
Management of Class II Malocclusion
COMMONLY ASKED QUESTIONS
LONG ESSAYS:
1 . Discuss the treatment plan for Angle’s class II malocclusion patients in mixed dentition period?
2 . Discuss in brief the aetiology, clinical picture and treatment of Angle’s class II malocclusion?
3 . Discuss your treatment of choice of appliance for a patient aged 8 years, presenting class II division 1 malocclu-
sion with positive VTO? [Same as LE Q.1]
4 . A child in a mixed dentition having a backwardly placed chin with protrusion and spaces in upper anterior
teeth. Describe the line of treatment and appliance used. [Same as LE Q.1]
5 . A 12-year-old boy with receding chin, proclined anteriors and deep bite reports to you. Discuss your diagnosis
and justify your diagnosis. [Same as LE Q.1]
6 . How will you set out a treatment of a class II division 1 malocclusion case in the mixed dentition with moderated
crowding in the anteriors? Justify your modality of treatment. [Same as LE Q.1]
232 Quick Review Series for BDS 4th Year, vol 1

SHORT ESSAYS:
1 . Treatment of mandibular retrusion.
2. Treatment planning for class II division I malocclusion in adults.

SHORT NOTES:
1 . Clinical features of class II division 2. [Ref LE Q.2]
2. Clinical features of class II division 1 malocclusion. [Ref LE Q.2]
3. Define growth modulation and state its methods.
4. Camouflage.

SOLVED ANSWERS
LONG ESSAYS:
Q.1.Discuss the treatment plan for Angle’s class II mal- b. Retrognathic mandible with higher FMA
occlusion patients in mixed dentition period? angle:
l Activator along with high pull headgear
Ans.
should be used.
Angle’s class II Division 1 malocclusion is characterized by: ii. Maxillary prognathism:
Molar relation: l To restrict the forward growth of the max-
The distobuccal cusp of the upper first permanent molar illa extra-oral orthopaedic force in the form
occludes with the mesiobuccal groove of the lower first of headgear should be used.
permanent molar. Lower dental arch is distally posi- l Patients should wear the appliance for 12–14
tioned in relation to upper arch. days and the force applied is 350–450 g/side.
The objectives of treatment planning in a growing child l For horizontal growing patients cervical
for correcting a class II division 1 malocclusion are as pull headgear is used.
follows: l For vertically growing patients high pull or
l Correction of class II molar and canine relationship occipital pull headgear is used.
l Establishing stable class I incisor relationship and l In cases of vertical maxillary excess, max-
normal overbite illary intrusion splints are used.
l Correction of deep bite and deep curve of Spee iii. Combination of mandibular retrognathism
l To relieve crowding and irregularities of teeth and and maxillary prognathism:
proper alignment of crowded anterior teeth l Activator with headgear is used to restrict
l Correction or improvement of skeletal discrepancy the maxillary growth and promote man-
l Improvement of facial aesthetics dibular growth.
l Correction of any other problems in an individual B. Correction of dentoalveolar class II malocclusion
case with class I skeletal base:
Treatment in mixed dentition period: l In dentoalveolar class II with normal skeletal
A. Correction of skeletal class II malocclusion: base, the defect lies in the dentoalveolar portion.
Interception of abnormal skeletal patterns: l As the normal incisor and molar relationship is
l During the mixed dentition period abnormal established, the other teeth usually settle in nor-
skeletal patterns can be intercepted by means mal position and function.
of functional or orthopaedic appliances. l Space is gained either by distalization of molars
i. Mandibular retrognathism: or extraction for correction of crowding.
a. Retrognathic mandible with average FMA l In low angle case, deep bite correction is achieved
angle and lower facial height: by using anterior bite planes.
i. Activator or FR-I is commonly used. l In high angle cases, utility arches are used to
While bionator, biomodular, cybernator, achieve incisor intrusion and labial bows or fixed
propulsor, etc. are other functional ap- appliance mechanotherapy is used for retraction
pliances, which are less commonly used. of incisors.
ii. Functional appliance acts by placing the l Posterior crossbites are corrected using crossbite
mandible in anterior position and also by elastics.
eliminating functional retrusion. l Any habit should be corrected simultaneously.
Section | I  Topic-Wise Solved Questions of Previous Years 233

Retention after class II correction is achieved by


l
v. Short hypotonic upper lip, lip trap (patient
using Tweed’s type B retention plan.
places lower lip against palatal surface of
A brief diagramatic representation of treatment plan for
upper incisors).
class II malocclusion in a growing child is as follows:
vi. Lack of anterior lip seal due to short upper
Class II in growing child lip. Normal lip seal is essential to main-
tain teeth in their corrected position.
vii. Patient exhibits abnormal muscle activity:

Dentolveolar Skeletal • Abnormal buccinator Leading to constricted narrow


class II class II activity upper arch with posterior crossbites.

• A hyperactive mentalis
Correction of Growth modulation procedures muscle.
dentoalveolar viii. Proclined lower anteriors, a natural com-
structures
Prognathic Retrognathic Combination pensation to decrease overjet.
• Correction of molar
maxilla mandible
relation, deep bite
and crossbite
Headgear
• Reduction of overjet Skeletal features:
Headgear Functional with FA
appliance Abnormal skeletal features most often found are
i. Maxillary protrusion
Q.2. Discuss in brief the aetiology, clinical picture and ii. Mandibular retrusion
treatment of Angle’s class II malocclusion? iii. Combination of both of the above
Aetiology:
Ans.
Class II malocclusion is characterized by a class II molar i. Heredity
6 ii. Teratogenesis
relation i.e. DB cusp of occludes in buccal groove Prenatal
iii. Irradiation
6 .
of iv. Intrauterine fetal posture
Natal i. Improper forceps application
Aetiologic factors during delivery – Trauma to
Class II, Trauma
condylar region – Leading to
division 1 ankylosed or fibrosed TMJ
with under developed
Postnatal mandible
Two main forms:

I. Traumatic injury to mandible and TMJ.


Class II,
II. Long-term irradiation treatment of
division 2 skeletal craniofacial region.
III. Infectious conditions like rheumatoid
{SN Q.2} arthritis influences mandibular growth.
IV. Abnormal function – e.g. oral
Class II division 1 malocclusion: respiration, abnormal swallowing
Class II molar relation with proclined maxillary an- habits like thumb sucking.
terior teeth.
Clinical features: Treatment objectives:
Patient exhibits: I. Reduction of overjet and overbite.
i. Class II molar relation (may vary from end II. Correction of crowding and local irregulari-
on molar to one that is full-fledged class II). ties, unstable molar relationship and posterior
ii. Proclined maxillary anteriors with hoverjet. crossbites if any.
iii. Convex profile. III. Normalizing the musculature.
iv. Because of proclined upper anteriors –
Lower anteriors fail to make contact with Treatment of Angle’s class II malocclusion in brief is as
palatal surface of upper anterior leading to follows:
h overbite and excessive curve of Spee.
234 Quick Review Series for BDS 4th Year, vol 1

Class II malocclusion

Growing patient Non-growing patient

Skeletal Cl. II Dental Cl. II. Dental Cl. II. Skeletal Cl. II

Maxillary Maxillary Mandibular Orthodontic treatment Mild-to- Skeletal


prognathism prognathism retrognathism as needed moderate class II
+ mandibular class II
retrognathism

Orthodontic
Maxillary Mandibular
camouflage by
Facebow with Activator with Myofunctional prognathism retrognathism
extraction of
head gear headgear to therapy with
some teeth
to restrict the restrict maxillary activator or
maxillary growth and functional Surgical Surgical
growth promote regulator maxillary mandibular
mandibular Or setback advancement
growth If patient is at the
end of growth period,
fixed functional appliances
like Herbert appliance or
Jasper jumper are indicated

Correction of deep bite and crossbite: Class II maloc-


vi. Deep mentolabial sulcus
clusion can be associated with anterior deep bite and
vii. Absence of abnormal muscle activity
posterior crossbite.
Deep bites: can be treated by using:
They have perfectly acceptable function as well as facial
i. Removable anterior bite planes
appearance. In severe cases the bite is often very deep
ii. Fixed appliances to intrude upper/lower
and poses the risk of periodontal trauma in maxillary
anteriors.
palatal and mandibular labial aspects.
Crossbites: Can be corrected with appliances in-
Treatment objectives:
corporating screws/springs that expand maxillary
a. Correction of incisor relationship and buccal seg-
arch.
ment relationship
Class II division 2 malocclusion:
b. Relief of gingival trauma
Characterized by class II molar relationship with retro-
c. Relief of crowding and local irregularities
clined upper centrals that are overlapped by the lateral
Deep anterior overbite:
incisors.
i. Treated by g in incisal overbite and alteration
Includes variations like – retroclined centrals 1 lateral
of incisor inclination.
incisors and very rarely include retroclined canines
ii. Deep bite can be reduced by use of anterior
as well.
bite plane or fixed appliances incorporating
anchor bends or reverse curve of Spee.
{SN Q.1} iii. Incisor inclination treated by use of torqueing
springs to move the upper incisor roots lin-
Clinical features: gually and crowns buccally.
i. Molars in distocclusion
ii. Retroclined central incisors and rarely other an- Q.3. Discuss your treatment of choice of appliance for a
teriors as well patient aged 8 years, presenting class II division 1 mal-
iii. Deep bite occlusion with positive VTO?
iv. Broad square face with pleasing straight profile Ans.
v. Backward path of closure
[Same as LE Q.1]
Section | I  Topic-Wise Solved Questions of Previous Years 235

Q.4. A child in a mixed dentition having a backwardly b. Dentoalveolar class II malocclusion: Orthodontic cor-
placed chin with protrusion and spaces in upper anterior rection
teeth. Describe the line of treatment and appliance used. i. Orthodontic camouflage:
l Orthodontic camouflage refers to repositioning
Ans.
the teeth without correcting the skeletal problem.
[Same as LE Q.1] l The objective of orthodontic camouflage is to

correct the malocclusion which makes the un-


Q.5. A 12-year-old boy with receding chin, proclined
derlying skeletal problem less apparent.
anteriors and deep bite reports to you. Discuss your di-
l Camouflage is best performed in adolescents but
agnosis and justify your diagnosis.
it is also done in adults.
Ans. l Extractions for camouflage are done in three

possible ways: extraction of upper first premo-


[Same as LE Q.1]
lars, extraction of upper as well as lower first
Q.6. How will you set out a treatment of a class II divi- premolars and extraction of upper first and lower
sion 1 malocclusion case in the mixed dentition with second premolars.
moderated crowding in the anteriors? Justify your mo- l When only retraction of proclined incisors is

dality of treatment. required, extraction of upper first premolars


alone is done.
Ans.
l If crowding correction or proclination correction

[Same as LE Q.1] is required in the lower arch then extraction of


both upper and lower first premolars is done.
l For molar correction, extraction of lower second
SHORT ESSAYS: premolar is done.
l Orthodontic camouflage is achieved through
Q.1. Treatment of mandibular retrusion.
fixed appliance.
Ans. ii. Surgery:
l Surgery is the suitable form of treatment in se-
l Retrognathic mandible or mandibular retrusion results
vere class II skeletal malocclusion.
in class II malocclusion.
l The various surgical procedures carried out are
l Growth stimulation of the mandible is induced using
For correction of prognathic maxilla:
functional appliances.
Le Fort I osteotomy and anterior maxillary oste-
l Functional appliances act by placing the mandible in
otomy.
anterior position and also by eliminating functional
For correction of retrognathic mandible:
retrusion.
Advancement procedures like sagittal split and
l Commonly used functional appliances for correction
oblique osteotomy are carried out.
of class II cases are activator, Frankel, twin block and
In combination cases – bijaw surgery with ge-
bionator.
nioplasty if required.
l During late mixed dentition in children with residual

postpubertal growth, fixed functional appliances like iii. Orthodontic correction:


In dentoalveolar class II cases with normal skeletal
Herbst and Jasper jumpers are used.
base, the defect lies in the dentoalveolar portion.
Q.2. Treatment planning for class II division I malocclu- l As the normal incisor and molar relationship
sion in adults. is established, the other teeth usually settle in
Ans. normal position and function.
l Space is gained either by distalization of mo-
Class II division 1 malocclusion is characterized by class II lars or extraction for correction of crowding.
molar relation with proclined maxillary anterior teeth. l In low angle case deep bite correction is
Treatment objectives: achieved by using anterior bite planes and in
I. Reduction of overjet and overbite high angle cases, incisor intrusion is achieved
II. Correction of crowding and local irregularities, un- using utility arches.
stable molar relationship, posterior crossbites if any l Crossbite elastics are used to correct poste-
III. Normalizing the musculature rior crossbites.
Treatment procedures: l Retraction of incisors is achieved by using
a. Skeletal class II malocclusion: Orthodontic camou- labial bows or with fixed appliance mechano-
flage or surgery is carried out. therapy.
236 Quick Review Series for BDS 4th Year, vol 1

SHORT NOTES: iii. In case of combination of mandibular retrogna-


thism and maxillary prognathism:
Q.1. Clinical features of class II division 2. Activator with headgear is used to restrict the max-
Ans. illary growth and promote mandibular growth.
[Ref LE Q.2] Q.4. Camouflage.
Q.2. Clinical features of class II division 1 malocclusion. Ans.
Ans. l Orthodontic camouflage refers to repositioning the teeth
[Ref LE Q.2] without correcting the skeletal problem.
l The objective of orthodontic camouflage is to correct
Q.3. Define growth modulation and state its methods. the malocclusion which makes the underlying skeletal
Ans. problem less apparent.
l Camouflage is best performed in adolescents but it is
l The treatment procedures carried out during mixed or also done in adults.
early permanent dentition period prior to cessation of Indications for camouflage treatment are
growth to intercept abnormal skeletal patterns leading to a. Patients too old for growth modulation
class II division 1 malocclusion by means of functional b. Mild or moderate skeletal class II
or orthopaedic appliances are known as growth modifi- c. Good alignment of teeth and good vertical pro-
cation methods. portions
For example: l Extractions for camouflage are done in three
i. Correction of mandibular deficiency or retrogna- possible ways based on situation:
thism during mixed dentition period by use of i. Extraction of upper first premolars
myofunctional appliances like activator or func- ii. Extraction of upper, lower first premolar
tional regulator. iii. Extraction of upper first and lower second
ii. Correction of maxillary prognathism by use of premolars
face bow with headgear to restrict further maxil- l Orthodontic camouflage is achieved through
lary growth. fixed appliance.

Topic 28
Management of Class III Malocclusion
COMMONLY ASKED QUESTIONS
LONG ESSAYS:
1 . Discuss the clinical picture of Angle’s skeletal class III malocclusion and its clinical management?
2. Enumerate the differences between true and pseudo-class III.
3. Enumerate various methods in treating a case of class III malocclusion at an early age. [Same as LE Q.1]
4. Discuss in brief the aetiology, clinical picture and treatment of Angle’s class III malocclusion. [Same as LE Q.1]

SHORT ESSAYS:
1 . Aetiology and treatment of pseudo-class III malocclusion.
2. Aetiology of class III malocclusion. [Ref LE Q.1]
3. Treatment of Angle’s class III malocclusion in adults. [Ref LE Q.1]
4. Discuss your line of treatment for class III malocclusion in mixed dentition. [Ref LE Q.1]
5. A 9-year-old boy reports to the department of orthodontics with a chief complaint of prognathic mandible.
Discuss the growth modulation procedures as a line of treatment. [Same as SE Q.4]
Section | I  Topic-Wise Solved Questions of Previous Years 237

SHORT NOTES:
1 . Management of mandibular prognathism in adults.
2. Objectives of treatment of class III malocclusion.

SOLVED ANSWERS
LONG ESSAYS:
Q.1. Discuss the clinical picture of Angle’s skeletal iii. Incisor relation:
class III malocclusion and its clinical management? l The incisors may be in an edge-to-edge rela-

tionship or reverse overjet or it may exhibit


Ans. a normal incisal relationship.
iv. Line of occlusion:
[SE Q.2]
Line of occlusion may or may not be altered in
l {Class III malocclusion is found in about 3% of the the maxillary and mandibular arches.
population. It is also known as mesiocclusion or prenor- v. The upper arch is frequently narrow, while the
mal occlusion. lower arch is broad. Thus posterior crossbites
l Class III malocclusion is a condition in which the lower are a common feature of class III malocclusion.
molar is positioned mesial to the upper molar. vi. It is common for the upper teeth to be crowded
Class III subdivision: Condition in which class III molar as the arch is narrow and short in some cases.
relation is present only on one side with normal molar vii. The patients have a concave profile due to the
relation on the other side is known as class III subdivision. presence of a prominent chin.
True class III: viii. Vertical growers exhibiting an increased inter-
l This is a skeletal malocclusion, it could be due maxillary height may have an anterior open bite.
to retrognathic maxilla, prognathic mandible In some patients a deep overbite may be seen.
or combination of both. B. Skeletal features of class III malocclusion are as
l In this class III molar relation exists both in follows:
centric occlusion and in rest position. l Class III malocclusions are quite often associated
Pseudo-class III/habitual class III: with underlying skeletal malrelationships:
l This is not a true class III malocclusion. a. A short or retrognathic maxilla
l When the mandible moves from rest position b. A long or prognanthic mandible
to occlusion due to occlusal prematurities, it c. A combination of the above two
slides forward into a pseudo-class III position. C. Soft tissue features:
l These patients show normal molar relationship l Concave profile
in rest position while class III relation in cen- l Anterior divergence
tric occlusion. l Frequently incompetent lips
Aetiology: l Short upper lip
i. Heredity plays a major role in true or skeletal l Tongue more anteriorly placed
class III malocclusion. D. Functional features:
ii. Class III malocclusions are said to have a very l Forward displacement of the mandible.
strong genetic basis. l When there is unilateral crossbite, lateral man-
iii. It is seen commonly in certain races, e.g. dibular displacement is found.
Hapsburg jaw in royal families of Germany. l In pseudo-class III, patients will have class I skel-
iv. The habitual forward positioning of the man- etal pattern, the abnormality is due to tilting of
dible due to occlusal premturities or enlarged the teeth and the forward path of closure.
adenoids are the other causes of prenormalcy.} E. Growth:
Clinical features of class III malocclusion are as follows: l Unfavourable facial growth is seen in most of the
A. Occlusal features: class III cases.
i. Class III molar relation: l Tendency to open bite increases with vertical
The lower dental arch is in anterior relation to the facial growth.
maxillary arch. Mesiobuccal cusp of the upper first l Excessive horizontal growth worsens the reverse
permanent molar occludes with the interdental space overjet.
between the lower first and second permanent molars. Diagnosis:
ii. Class III canine relation: l The clinical examination: should include ob-
Upper canine occludes with the interdental space servation of path of closure
between lower first and second premolars. l Study models
238 Quick Review Series for BDS 4th Year, vol 1

Radiographs:
l [SE Q.3]
A lateral cephalogram offers valuable informa-
tion on the skeletal nature of the malocclusion.
{Treatment of severe class III after growth:
l Camouflage can be used in mild skeletal class III cases.
Treatment:
l Severe class III malocclusion after growth comple-
Occlusion should be recognized and treated early
tion is treated by surgical and corrective procedures.
due to the following reasons:
l Class III due to maxillary deficiency is treated by maxil-
a. The severity of the developing malocclu-
lary advancement procedures such as Le Fort I osteotomy.
sion can be reduced by recognizing the
l Class III malocclusions that are a result of mandibu-
malocclusion at an early age and it is
lar prognathism are treated by mandibular set back
possible to intercept the abnormal skeletal
procedures.
pattern.
Treatment of pseudo-class III:
b. The class III malocclusion characterized by
l On removal of aetiology, the pseudo-class III maloc-
anterior crossbites often results in retarded
clusion that occurs as a result of occlusal prematurity
maxillary growth due to locking of the max-
improves.
illa within the mandible.
l In the early stages, patients can be treated by equili-
c. The occlusal forces on the mandibular inci-
bration of occlusion alone.
sors exerted by the maxillary incisors in
l Later treatment consists of correction of anterior
crossbite encourage the continued forward
crossbite.
growth of mandible further worsening the
Correction of dentoalveolar structures:
prenormalcy.
l Removable or fixed appliances are effective in cor-
d. Skeletal class III malocclusion requires
rection of dentoalveolar structures.
early treatment to intercept the developing
l Inclined planes act as extension of the lower incisal
skeletal malrelation.
edges by contacting the palatal surfaces of maxillary
[SE Q.4] incisors. On closing, the mandible is forced to be
retruded. Maxillary teeth are tipped labially.
{The following are some of the growth modulation proce- l If the bite is shallow, then posterior bite blocks with
dures that can be carried out in growing children: Z-spring are used to move the palatally placed maxil-
a. During growth period to intercept a skeletal class III lary incisors.}
case due to maxillary retrusion, a Frankel III, a myo- A brief summary of treatment of class III malocclusion:
functional appliance can be used or other myofunc-
tional appliances like reverse activator, reverse bion- Face mask in
ator and twin block may also be used. maxillary
retrognathism
b. Chin cup with high pull headgear are used to inter-
Skeletal
cept class III malocclusion due to mandibular prog-
nathism. Chin cap in
mandibular
d. Severe class III malocclusions that are a result of prognathism
Growing
maxillary retrusion can be treated by reverse pull
patient
headgear (face mask) to protract the maxilla.
Treatment of anterior crossbite:
The lower anterior inclined planes or removable ap- Removable
Dental
pliances incorporating screws designed for anterior Class III /fixed
malocclusion orthodontic
expansion can be used to treat the mild anterior Dental treatment
crossbites.
Treatment of posterior crossbite: Mild-to-
Posterior crossbites are common with class III mal- Adult moderate
occlusions. They can be treated by rapid maxillary patient Cl III:
Orthodontic
expansion. camouflage
Role of extractions:
l Class III malocclusion characterized by lower Skeletal
Severe
arch length deficiency and anterior crossbite can class III:
be treated by extracting the lower first premolars Surgical
followed by fixed appliances. maxillary
l The first premolars should be extracted in both the advancement
upper and lower arches, in case of arch length or mandibular
set back based
deficiency involving both the arches.}
on case
Section | I  Topic-Wise Solved Questions of Previous Years 239

Q.2. Enumerate the differences between true and Q.4. Discuss in brief the aetiology, clinical picture and
pseudo-class III. treatment of Angle’s class III malocclusion.
Ans. Ans.
Class III malocclusion is found in about 3% of the population. [Same as LE Q.1]
It is also known as mesiocclusion or prenormal occlusion, in
which the lower molar is positioned mesial to the upper molar.
True class III: SHORT ESSAYS:
l This is a skeletal malocclusion; it could be due to Q.1. Aetiology and treatment of pseudo-class III maloc-
retrognathic maxilla, prognathic mandible or combi- clusion.
nation of both.
l In this class III, molar relation exists both in centric
Ans.
occlusion and in rest position. Pseudo-class III/habitual class III:
Pseudo-class III/habitual class III: l This is not a true class III malocclusion.
l This is not a true class III malocclusion. l When the mandible moves from rest position to oc-
l When the mandible moves from rest position to clusion due to occlusal prematurities, it slides for-
occlusion due to occlusal prematurities, it slides ward into a pseudo-class III position.
forward into a pseudo-class III position. l These patients show normal molar relationship in
l These patients show normal molar relationship in rest rest position while class III relation in centric
position while class III relation in centric occlusion. occlusion.
Aetiology:
Differences between true and pseudo-class III are The causes of prenormalcy are as follows:
as follows: l The habitual forward positioning of the mandible

due to occlusal prematurities


Features True class III Pseudo-class III l Enlarged adenoids
i. Profile Concave Straight or concave Treatment of pseudo-class III:
ii. Aetiology Heredity Habitual or l On removal of aetiology, the pseudo-class III maloc-

developmental clusion that occurs as a result of occlusal prematurity


iii. Skeletal relation Has a class Has a normal class I
improves.
III skeletal skeletal base l In the early stages, patients can be treated by equili-

base bration of occlusion alone.


l Later treatment consists of correction of anterior
iv. Molar relation Class III Class I
in rest position crossbite.
v. Molar relation in Class III Shift from class I to Q.2. Aetiology of class III malocclusion.
centric occlusion class III
Ans.
vi. Premature Absent Present
contacts [Ref LE Q.1]
vii. Path of closure Forward Deviated Q.3.Treatment of Angle’s class III malocclusion in
viii. Gonial angle h or g Normal adults.
ix. Retrusion of Not possible Possible Ans.
mandible further
[Ref LE Q.1]
x. Treatment Orthopaedic Elimination of prema-
or surgical turities and replace- Q.4. Discuss your line of treatment for class III maloc-
correction ment of last posterior clusion in mixed dentition.
No further teeth by functional
changers space maintainers Ans.
occur if left If left untreated,
untreated becomes established [Ref LE Q.1]
into true class III
malocclusion Q.5. A 9-year-old boy reports to the department of or-
thodontics with a chief complaint of prognathic mandi-
Q.3. Enumerate various methods in treating a case of ble. Discuss the growth modulation procedures as a line
class III malocclusion at an early age. of treatment.
Ans. Ans.
[Same as LE Q.1] [Same as SE Q.4]
240 Quick Review Series for BDS 4th Year, vol 1

SHORT NOTES: Q.2. Objectives of treatment of class III malocclusion.


Q.1. Management of mandibular prognathism in adults. Ans.

Ans. The objectives of treatment of class III malocclusion are as


follows:
i. Chin cup with high pull headgear is used to intercept i. Correction of open bite
class III malocclusion due to mandibular prognathism in ii. Correction of anterior deep bite
growing children. iii. Correction of anterior and posterior crossbite
ii. Class III malocclusions that are a result of mandibular iv. Correction of buccal segment relationship
prognathism are treated by mandibular set back proce- v. Correction of any other associated problems in an indi-
dures in adults who have completed their growth. vidual case

Topic 29
Management of Open Bite, Crossbite and Deep Bite
COMMONLY ASKED QUESTIONS
LONG ESSAYS:
1 . Describe the probable aetiologic factors responsible for anterior and posterior open bite and also their
treatment.
2. Discuss the aetiology, line of treatment and the design of appliance in correction of anterior crossbite.
3 . Define preventive, interceptive and corrective orthodontics. Enumerate about the various modes of posterior
crossbite correction and discuss in detail any one mode of treatment.
4 . Define and classify crossbite. Give aetiology, clinical features and diagnosis of posterior crossbite.
5 . Aetiology and treatment of open bite. [Same as LE Q.1]
6 . Describe the construction and use of a removable appliance in the treatment of anterior crossbite of one or two
teeth. [Same as LE Q.2]
7 . An 8-year-old male child is having one of the upper central incisors in anterior crossbite. What can be the prob-
able aetiology? Design an appliance for the correction of the above case. What will be your advice to the patient?
[Same as LE Q.2]

SHORT ESSAYS:
1. Anterior open bite. [Ref LE Q.1]
2. Clinical features of anterior open bite. [Ref LE Q.1]
3. Catalans appliances.
4. Anterior crossbite.
5. Posterior crossbite – aetiology and clinical features.
6. Management of crossbite.
7. Aetiology and treatment of deep bite.
8. Aetiology of open bite. [Same as SE Q.1]
9. Catalan appliances – Mention the disadvantage of this appliance. [Same as SE Q.3]
1 0. Lower anterior bite plane. [Same as SE Q.3]
1 1. Treatment of crossbite. [Same as SE Q.6]

SHORT NOTES:
1 . Open bite.
2 . Anterior open bite. [Ref LE Q.1]
3 . Treatment methods of anterior crossbite. [Ref LE Q.2]
Section | I  Topic-Wise Solved Questions of Previous Years 241

4. Catalan’s appliance. [Ref LE Q.2]


5. Management of posterior crossbite.
6. Clinical features of skeletal deep bite.
7. Tongue blade therapy. [Ref LE Q.2]
8. Aetiology of deep bite.
9. Apertognathia. [Same as SN Q.1]
10. Clinical features of anterior open bite. [Same as SN Q.2]
11. Deep overbite. [Same as SN Q.8]

SOLVED ANSWERS
LONG ESSAYS:
Q.1. Describe the probable aetiologic factors responsi- A. Epigenetic factors:
ble for anterior and posterior open bite and also their i. Disharmony of skeletal growth pattern
treatment. between maxilla and mandible
ii. Alterations in morphology of the tongue
Ans.
iii. Tongue posture
Open bite is a malocclusion that occurs in the vertical plane iv. Inherited factors – e.g. h tongue size,
characterized by lack of vertical overlap between the maxil- abnormal skeletal growth pattern
lary and mandibular dentitions. B. Environmental factors:
Open bite can be in the anterior or posterior region. i. Prolonged thumb-sucking habit
Nature and severity of open bite are affected by:
Skeletal a. Posture of thumb positioning
b. The intensity of sucking
Anterior Anterior c. The frequency of sucking
region open bite ii. Tongue thrusting
iii. Nasopharyngeal airway obstruction and as-
Dental sociated mouth breathing}
Open bite
Clinical features:
(SN Q.2 and SE Q.2)
Posterior Posterior
region open bite {(Skeletal anterior open bite exhibits following features:
i. h Lower anterior facial height.
Anterior open bite: ii. g Upper anterior facial height.
It is a condition where there is no vertical overlap iii. h Anterior 1 g posterior facial height.
between the upper and lower anteriors. iv. Patient exhibits vertical maxillary increase and
a long and narrow face, small mandibular body
[SE Q.1] ramus.
v. Steep anterior cranial base.
Skeletal vi. Cephalometric evaluation reveals downward
and forward rotation of the mandible with steep
mandibular plane angle.
Anterior open bite classified as vii. In some cases, upward tipping of maxillary
skeletal base can be observed.
viii. Divergent cephalometric planes.
Dental Dental anterior open bite exhibits following features:
i. Proclination of upper anterior teeth.
Aetiology: ii. Patient may have narrow maxillary arch due to
According to Fletcher (1975), aetiology of open bite lowered tongue posture due to any habits.
is classified under following headings: iii. Upper and lower anteriors fail to overlap each
A. Epigenetic factors other resulting in a space between incisal edges
B. Environmental factors of maxillary and mandibular anteriors.)}
242 Quick Review Series for BDS 4th Year, vol 1

[SE Q.1] A. Dental factors:


A dental anterior crossbite is because of abnormal
{Treatment of anterior open bite: axial inclination of the maxillary incisors, which
i. Removal of the cause or aetiology:
may result from one of the following causes:
Either a removable or fixed habit breaking appli-
l Trauma to primary teeth or to the permanent
ances, e.g. palatal crib, can be used to intercept
tooth bud
the habit.
l Persistence of deciduous tooth
ii. Myofunctional therapy:
l Labially positioned supernumerary tooth
Skeletal anterior open bite is treated with func-
l Arch length – tooth material discrepancy which
tional appliances, e.g. Frankel – IV or modified
causes lingual eruption of permanent tooth
activator, which incorporates bite blocks inter-
l Abnormal habits, e.g. Lip biting, thumb suck-
posed between the posterior teeth which have an
ing and mouth breathing causes lowered
intrusive action on upper and lower posterior teeth.
tongue position which no longer balances
iii. Orthodontic therapy:
forces exerted on teeth by buccal group of
Mild-to-moderate open bites successfully man-
muscles. This disharmony between external
aged with fixed orthodontic therapy in conjunc-
and internal muscle forces results in narrowing
tion with box elastics, which brings about extru-
of upper arch resulting posterior crossbite.
sion of the upper and lower anteriors. In severe
l Surgically repaired cleft lip and palate.
skeletal open bites this therapy is not advisable.
B. Skeletal factors:
iv. Surgical correction:
l Skeletal crossbite results due to excessive man-
Skeletal open bites in adults are best treated by sur-
dibular growth.
gical procedures involving maxilla and mandible.}
l It is genetic or inherited malocclusion.
Posterior open bite:
l Collapse of maxillary arch in children with cleft
Is characterized by lack of contact between the posteri-
palates where there is retrognathic maxilla.
ors when the teeth are in centric occlusion.
C. Functional factors:
Aetiology:
Functional interference of the mandible during clo-
i. Mechanical interference with eruption:
sure results in dental crossbite due to premature tooth
l Ankylosis of tooth due to trauma.
contact. This results in pseudo-class III malocclusion.
l Obstacles in path of eruption like supernumer-
Treatment of anterior crossbite:
ary teeth, nonresorbed deciduous tooth roots
Factors to be considered in treating anterior crossbite
and pressure from soft tissues interposed be-
are as follows:
tween the teeth.
l Availability of mesiodistal space
ii. Failure of the eruption mechanism of the tooth.
l Sufficient overbite
Treatment:
l Position of the tooth
Removal of aetiological factors is the primary aim of
l Type of occlusion either class I or class III
treatment.
l Extents of root formation
For example:
i. Lateral tongue spikes – to control tongue
thrust habit. {SN Q.3}
ii. Forceful extrusion of posterior teeth.
Various methods of correction of anterior crossbite are
iii. In cases of ankylosed teeth which are in infra
as follows:
occlusion, crowns can be given to posteriors
l Tongue blade therapy
to restore the normalocclusal level.
l Inclined planes

Q.2. Discuss the aetiology, line of treatment and the de- l Expansion appliances with either screws or canti-

sign of appliance in correction of anterior crossbite. lever springs


l Fixed appliances
Ans.
l Anterior crossbite is a condition in which one or more
primary or permanent maxillary incisors is lingual to {SN Q.7}
the mandibular incisor. Tongue blade therapy:
l It is also known as reverse overjet, reverse bite and
l Tongue blade can be used to treat developing
under bite. single-tooth anterior crossbites successfully, pro-
Aetiology of anterior crossbite can be studied under vided there is sufficient space for the tooth to be
following categories: brought out.
A. Dental factors l The tongue blade is a flat wooden stick resem-
B. Skeletal factors bling an ice cream stick. It is placed inside the
C. Functional factors
Section | I  Topic-Wise Solved Questions of Previous Years 243

Fixed appliances:
mouth contacting the palatal aspect of the tooth in
l Fixed appliances are used to pull the in-locked tooth
crossbite. The blade is made to rest on the man-
or teeth into correct labial position.
dibular tooth in crossbite which acts as a fulcrum
l Dental anterior crossbite involving one or two teeth
and the patient is asked to move the oral part of
can be treated with fixed appliances using multi-
the blade upwards and forward.
looped archwires.
l This is continued for 1–2 h for about 2 weeks.
Treatment of skeletal anterior crossbite during growth
l This form of therapy can be successfully used to
period:
treat most of the developing crossbites that are
l Before termination of growth, a protraction face
recognized by the dentist at an early stage.
mask or reverse head gear is used to treat skeletal
anterior crossbite that occurs as a result of a retro-
positioned maxilla.
{SN Q.4} l These face masks help in protraction of the maxilla

Catlan’s appliance or lower anterior inclined plane: thereby normalizing the skeletal crossbite.
l A chin cap should be used to intercept the exces-
l This appliance is indicated when adequate space

exists in the arch for the alignment of the maxil- sive mandibular growth leading to skeletal anterior
lary teeth in crossbite. crossbites.
l They are used only in those cases where the cross- Q.3. Define preventive, interceptive and corrective
bite is due to a palatally displaced maxillary incisor. orthodontics. Enumerate about the various modes of
l Inclined planes constructed on the lower anterior posterior crossbite correction and discuss in detail any
teeth can be used to treat maxillary teeth in crossbite. one mode of treatment.
l It can be made of acrylic or cast metal and can be
Ans.
designed to treat a single tooth in crossbite or a
segment of the upper arch in crossbite. i. Preventive orthodontics:
l The inclined plane is designed to have a 45° angu- Preventive orthodontics is the action taken to preserve
lation which forces the maxillary teeth in cross- the integrity of what appears to be a normal occlusion
bite to a more labial position. at a specific time.
ii. Interceptive orthodontics:
Interceptive orthodontics is defined as that phase of the
The disadvantages of lower anterior inclined plane art and science of orthodontics employed to recognize
include: and eliminate potential irregularities and malpositions
i. The patient encounters problems in speech and of the developing dentofacial complex.
dietary restrictions. iii. Corrective orthodontics:
ii. If the appliance is used for more than 6 weeks, it Corrective orthodontics recognizes the existence of a
can result in anterior open bite due to supraerup- malocclusion and the need for employing certain tech-
tion of the posteriors. nical procedures to reduce or eliminate the problem and
iii. The appliance may need frequent recementation. the attendant squelae.
Expansion appliances: Posterior crossbite:
i. Schwarz-type expansion plate with posterior bite l A posterior crossbite is an abnormal buccolingual rela-
plane. tionship of a tooth or teeth between maxilla and man-
ii. Upper Hawley’s appliance with cantilever spring to dible when they are brought into centric occlusion.
move the in-standing tooth. Posterior bite plane is Factors to be considered in treating posterior crossbite
added to the Hawley’s appliance. are
iii. Removable appliances with Z-spring or double can- l Availability of mesiodistal space to correct the
tilever spring. crossbites
Use of double cantilever spring (Z-spring): l Position of the apical portion of tooth after treatment
l Anterior crossbites involving one or two maxil- l Types of tooth movement required
lary teeth can be treated using a double cantilever Treatment of posterior crossbite:
spring. A. Single-tooth dental crossbites:
l The Z-spring is indicated only when there is ade- i. Crossbite elastics
quate space for labialization of the teeth in cross- B. Dentoalveolar contraction and crossbite:
bite. a. Treatment of unilateral contraction of maxil-
l In case of a deep overbite the spring should be lary arch:
given along with a posterior bite plane to help in i. Removable plates
jumping the bite. ii. Quad helix
244 Quick Review Series for BDS 4th Year, vol 1

iii. W-arch Quad helix:


iv. Coffin spring l The quad helix is a spring that consists of four

b. Treatment of bilateral contraction of maxillary helices.


arch: l It is capable of causing dentoalveolar expan-

i. Quad helix sion of the molar as well as premolar region.


ii. W-arch l It can bring about skeletal expansion when

iii. Rapid maxillary expansion (RME) used in younger individuals.


c. Mandibular dentoalveolar contraction: Removable plates:
i. Quad helix spring l Removable appliances incorporating jack
C. Skeletal crossbite screws are used to treat the unilateral cross-
a. Narrow maxilla: bites.
i. Mild cases – quad helix or W-arch l This appliance consists of a split acrylic plate,

ii. Severe cases – RME or Minnesota a jack screw and Adams clasps on the posterior
expander teeth to retain the plate. A labial bow can also
b. Narrow mandible – usually associated with be incorporated into the appliance for minor
retrognathic mandible space closure and retraction.
i. Functional appliances l The desired effect is achieved by sectioning the
Very severe cases are treated by surgery. plate in such a way that a small segment and
A. Single-tooth dental crossbite: larger segment are formed. The two segments
l Usually in single-tooth crossbite, where both the are connected by one or more jack screws. The
antagonist teeth are tipped out of position, simple smaller segment of the plate adjoins the area in
crossbite elastics are effective. crossbite whereas the larger segment is used
l Single-tooth crossbite involving the molars can be for anchorage.
treated using elastics that are stretched between Fixed appliances
the maxillary palatal surface and mandibular buc- l Unilateral crossbites can also be treated by us-

cal surface. ing fixed appliances.


l These elastics are to be worn day and night. The Removable W-arch appliance:
treatment should not be continued for more than l Due to its reciprocal action, this appliance

6 weeks as the elastics can extrude the teeth. should be limited to only bilateral dental
l Disadvantages: It requires banding of the teeth crossbite conditions.
and good patient cooperation. l Caution should be exercised since a precise

B. Dentoalveolar contraction and crossbite: control of the force being applied to the
First any functional interference present is elimi- teeth is difficult.
nated by occlusal equilibration. Rapid maxillary expansion:
Occlusal equilibration: l Rapid maxillary expansion is a skeletal type of

A dental bilateral lingual crossbite in the primary expansion that involves the separation of the
dentition or mixed dentition may be simply cor- mid-palatal suture and movement of the maxil-
rected by removing occlusal interferences, usu- lary shelves away from, each other.
ally in the cuspid areas. This may sometimes need l Bilateral skeletal crossbite can be treated by

to be accompanied by some appliances. RME where in the mid-palatal suture is split.


Appliances given after occlusal equilibration are as This is done by using appliances that incorpo-
follows: rate screws that are to be activated at regular
Coffin spring: intervals.
l Walter Coffin designed the Coffin spring. l Expansion of the arches is the one of the

l It is a removable appliance that consists of an method of gaining the space in orthodontics.


omega-shaped wire (1.25 mm diameter) l Orthodontic indications of RME are

placed in the mid-palatal region. The free i. In anterior crossbite to gain space
ends of the omega are embedded in an acrylic ii. Collapsed maxillary arch due to cleft
plate that covers the slopes of the palate. palate
l The spring brings about dentoalveolar ex- iii. Unilateral or bilateral posterior skeletal
pansion. crossbite
l When used in young patients, it is capable l Principle of RME: Application of force to

of causing skeletal changes. widen the maxilla causes opening of the mid-
Section | I  Topic-Wise Solved Questions of Previous Years 245

palatal suture, new bone formation is induced . Skeletal factors


b
and the space created in the midline is filled c. Functional factors
with tissue fluids and blood. After 3–4 months,
a. Dental factors:
new bone fills in the space.
l Irregular eruption pattern, where the tooth

Q.4. Define and classify crossbite. Give aetiology, clini- erupts out of position
cal features and diagnosis of posterior crossbite. l Insufficient arch length leading to lingual or

buccal deflection of teeth during eruption


Ans.
l Over-retained primary tooth and ectopic

l ‘Crossbite’ is defined as a condition where one or more eruption of permanent teeth


teeth may be abnormally malposed buccally or lingually l Prolonged thumb or finger-sucking habit

or labially with reference to the opposing tooth or teeth. causes narrowing of the arches and lingual
A. Classification of crossbites: tipping of the posterior teeth
a. Anterior crossbite: b. Skeletal factors:
i. Single tooth i. Asymmetric growth of maxilla or mandible
ii. Segmental (multiple teeth) due to inherited growth pattern, trauma or
b. Posterior crossbite: long-standing functional problem
i. Single-tooth crossbite ii. Difference in basal width of the maxilla and
ii. Unilateral mandible due to constricted maxilla and
iii. Bilateral cleft palate
B. Another classification of crossbite: c. Functional or muscular crossbite:
a. Dental: (i) anterior and (ii) posterior l Functional adjustments to tooth interferences

b. Skeletal: (i) anterior and (ii) posterior Diagnosis:


c. Functional crossbite i. Study models
l Anterior crossbite: ii. Grids
Condition in which one or more primary or iii. Symmetroscope
permanent maxillary incisors are lingual to iv. Boley gauge or divider
the mandibular incisors. v. Radiographs: PA view or frontal cephalograms.
l Buccal crossbite: l Study models using wax bite in centric rela-

Condition in which the maxillary posterior tion is a useful diagnostic aid.


teeth is buccal to the mandibular antagonist. Study models will show which tooth is at fault
l Lingual crossbite: in the dental crossbite whether maxillary tooth
Condition in which the maxillary posterior or mandibular tooth.
teeth is lingual to mandibular antagonist. l A dental crossbite will exhibit an abnormal

l Scissors bite or telescopic bite: buccal or lingual axial inclination.


Mandibular teeth are entirely lingual to the l A skeletal crossbite may not exhibit abnormal

maxillary arch. axial inclination of teeth.


Posterior crossbite: l Symmetry of the dental arches can be as-

l A posterior crossbite is an abnormal buccolingual rela- sessed using grids, symmetroscope, Boley
tionship of a tooth or teeth between maxilla and man- gauge or divider.
dible when they are brought into centric occlusion. This helps in diagnosing the arch at fault in
Clinical features: skeletal crossbite.
l Posterior crossbite either unilateral or bilateral l Assessment of midlines by (posteroanterior)

presents as anyone or combination of the follow- PA view radiographs or frontal cephalograms


ing types: should be done.
i. Lingual crossbite l Midline should be assessed in both rest and

ii. Buccal crossbite centric position.


iii. Complete lingual crossbite l Differential diagnosis of midline shift:

Aetiology: i. If midline shift is present only in centric


Aetiology of posterior crossbite can be studied under position, then it is functional crossbite.
following headings: ii. If midline shift is present in both centric and
a. Dental factors rest positions, then it is true skeletal crossbite.
246 Quick Review Series for BDS 4th Year, vol 1

Q.5. Aetiology and treatment of open bite. l The inclined plane is designed to have a 45° angu-
lation which forces the maxillary teeth in cross-
Ans.
bite to a more labial position.
[Same as LE Q.1] l It takes about 10–14 days for the correction of

crossbite. It should be used for a maximum period


Q.6. Describe the construction and use of a removable
of 2–3 weeks.
appliance in the treatment of anterior crossbite of one
Mechanism of action:
or two teeth.
l When the appliance is either cemented or attached

Ans. with Adams clasp there is contact only in the an-


terior region, where there is crossbite.
[Same as LE Q.2]
l During the functional movements like swallowing,

Q.7. An 8-year-old male child is having one of the upper due to lack of contact of the posterior teeth all the
central incisors in anterior crossbite. What can be the forces are transmitted to the region of contact,
probable aetiology? Design an appliance for the correc- which guides the teeth to erupt into normal position.
tion of the above case. What will be your advice to the Disadvantages:
patient? l Patient encounters problems in speech during therapy.

l Patient has to put up with dietary restrictions.


Ans.
l If the appliance is used more than 6 weeks it can
[Same as LE Q.2] results in anterior open bite due to supra-eruption
of the posteriors.
l The appliance may need frequent recementation.

SHORT ESSAYS: Q.4. Anterior crossbite.


Q.1. Anterior open bite. Ans.
Ans. l Anterior crossbite is a condition in which one or more
primary or permanent maxillary incisors are lingual to
[Ref LE Q.1]
the mandibular incisor.
Q.2. Clinical features of anterior open bite. l It is also known as reverse overjet, reverse bite and under bite.

Aetiology of anterior crossbite can be studied under


Ans.
following categories:
[Ref LE Q.1] A. Dental factors
B. Skeletal factors
Q.3. Catalan’s appliances.
C. Functional factors
Ans.
A. Dental factors:
l Catalan’s appliance is also called as lower anterior in- l A dental anterior crossbite is because of abnor-

clined plane or incisal capping. mal axial inclination of the maxillary incisors.
l This appliance was introduced by Catalan 150 years ago. B. Skeletal factors:
l It is the simplest of all the functional appliances used for l Excessive mandibular growth.

correction of a developing crossbite. It is used on the l It is genetic or inherited malocclusion.

lower anterior teeth. l Collapse of maxillary arch in children with cleft

Indications: palates where there is retrognathic maxilla.


l When there is adequate space existing in the arch C. Functional factors:
for the alignment of the maxillary teeth in crossbite Functional interference of the mandible during clo-
l In cases where the crossbite is due to a palatally sure results in dental crossbite due to premature tooth
displaced maxillary incisor contact. This results in pseudo-class III malocclusion.
Contraindications: Treatment of anterior crossbite:
l Cases where the degree of overbite is less Various methods of correction of anterior crossbite
l Crossbite, which is due to true mandibular prog- are as follows:
nathism l Tongue blade therapy

Design: l Inclined planes

l It can be made of acrylic or cast metal and can be l Expansion appliances with either screws or

designed to treat a single tooth in crossbite or a cantilever springs


segment of the upper arch in crossbite. l Fixed appliances
Section | I  Topic-Wise Solved Questions of Previous Years 247

Tongue blade therapy: l Aetiology of posterior crossbite can be studied under


l Tongue blade can be used to treat develop- following headings:
ing single-tooth anterior crossbites success- a. Dental factors
fully, provided there is sufficient space for b. Skeletal factors
the tooth to be brought out. c. Functional factors
l This form of therapy can be successfully
a. Dental factors:
used to treat most of the developing cross-
l Irregular eruption pattern, where the tooth erupts
bites that are recognized at an early stage.
out of position.
Catlan’s appliance or lower anterior inclined
l Insufficient arch length leading to lingual or buc-
plane:
cal deflection of teeth during eruption.
l This appliance is indicated when adequate
l Over-retained primary tooth and ectopic eruption
space exists in the arch for the alignment of
of permanent teeth.
the maxillary teeth in crossbite.
l Prolonged thumb or finger-sucking habit causes
l It can be made of acrylic or cast metal and
narrowing of the arches and lingual tipping of the
can be designed to treat a single tooth in
posterior teeth.
crossbite or a segment of the upper arch in
b. Skeletal factors:
crossbite.
i. Asymmetric growth of maxilla or mandible due to
l The inclined plane is designed to have a 45°
inherited growth pattern, trauma or long-standing
angulation which forces the maxillary teeth
functional problem.
in crossbite to a more labial position.
ii. Difference in basal width of the maxilla and man-
Expansion appliances:
dible due to constricted maxilla and cleft palate.
i. Schwarz-type expansion plate with poste-
c. Functional or muscular crossbite:
rior bite plane.
l Functional adjustments to tooth interferences.
ii. Upper Hawley’s appliance with cantilever
Clinical features:
spring to move the in-standing tooth.
l Posterior crossbite either unilateral or bilateral
iii. Removable appliances with Z-spring or
presents as anyone or combination of the fol-
double cantilever spring.
lowing types:
Fixed appliances:
i. Lingual crossbite
l Fixed appliances are used to pull the in-
ii. Buccal crossbite
locked tooth or teeth into correct labial
iii. Complete lingual crossbite
position.
l Dental anterior crossbite involving one or Q.6. Management of crossbite.
two teeth can be treated with fixed appli-
Ans.
ances using multilooped archwires.
Treatment of skeletal anterior crossbite during According to Graber, crossbite is defined as a condition
growth period: where one or more teeth may be abnormally malposed buc-
l Before termination of growth, a protraction cally or lingually or labially with reference to the opposing
face mask or reverse head gear is used to treat tooth or teeth.
skeletal anterior crossbite that occurs as a re- Treatment of anterior crossbite:
sult of a retro-positioned maxilla. A. Fixed appliances:
l A chin cap should be used to intercept the ex- l Dental anterior crossbite involving one or two

cessive mandibular growth leading to skeletal teeth can be treated with fixed appliances using
anterior crossbites. multilooped arch wires.
B. Removable appliances:
Q.5. Posterior crossbite – aetiology and clinical features. i. Tongue blade:
l Single-tooth anterior crossbites can be suc-
Ans.
cessfully treated by using a tongue blade.
A posterior crossbite is an abnormal buccolingual relation- l The blade is made to rest on the mandibular

ship of a tooth or teeth between maxilla and mandible when tooth in crossbite which acts as a fulcrum and
they are brought into centric occlusion. the patient is asked to rotate the oral part of
In this condition, instead of the mandibular buccal cusps the blade upwards and forward. This is contin-
occluding in the central fossae of the maxillary posterior ued for 1–2 h for about 2 weeks.
teeth, they occlude buccal to the maxillary buccal cusps. ii. Z-spring:
l A posterior crossbite is an abnormal buccolingual rela- l Anterior crossbites involving one/two maxil-

tionship of a tooth or teeth between maxilla and man- lary teeth can be treated by using a Z-spring/
dible when they are brought into centric occlusion. double cantilever spring.
248 Quick Review Series for BDS 4th Year, vol 1

l It is indicated only in those cases where there ii. Quad helix:


is an adequate space for labialization of the l The quad helix appliance is capable of pro-
teeth in crossbite. ducing dentoalveolar expansion of the molar
C. Functional appliances: as well as premolar region.
l Lower anterior inclined plane/Catalan’s appliance l It produces skeletal changes when used in
is used to treat a single tooth in anterior crossbite young patients.
or a segment of the upper arch in crossbite. iii. RME:
l Catalan’s appliance constructed on the lower an- l Bilateral skeletal crossbite with a deep palate
terior teeth can be used to treat maxillary teeth in and narrow maxilla can be treated by RME.
crossbite. l It is achieved by using appliance that incorpo-
D. Orthopaedic appliances: rates screws that are to be activated at regular
i. Headgear: intervals.
l Skeletal anterior crossbites that occur as a

result of a retro-positioned maxilla should be Q.7. Aetiology and treatment of deep bite.
treated before termination of growth by using Ans.
a protraction face mask (reverse head gear).
l These face masks help in protraction of max- l According to Graber, deep bite is a condition of exces-
illa, thereby normalizing the skeletal crossbite. sive overbite, where the vertical measurement between
ii. Chin cap: maxillary and mandibular incisal margins is excessive
l Excessive mandibular growth leading to skel- when the mandible is brought into the habitual or cen-
etal anterior crossbites should be intercepted tric occlusion.
by use of a chin cap. Aetiology:
Treatment of posterior crossbite: Skeletal deep bite:
A. Removable appliances l It is caused by convergent rotation of skeletal jaw
i. Crossbite elastics: bases. The upward and forward rotation of the
l Single-tooth crossbite involving the molars mandible and as well downward and forward in-
can be treated by crossbite elastics. clination of the maxilla leads to deep bite.
l These elastics are stretched between the max- Dental deep bite:
illary palatal surface and mandibular buccal l Supraeruption of anterior teeth
surfaces. l Premature loss of permanent teeth resulting in
l These elastics are to be worn day and night. lingual collapse of anterior teeth
The treatment should not be continued for l Infra-occlusion of molars due to tongue thrusting
more than 6 weeks as the elastics can results l Anterior tipping of posterior teeth
in extrusion of teeth. l Lateral spreading low tongue posture
ii. Removable plates: l Large size teeth
l Unilateral crossbites can be treated by remov- Treatment:
able appliances incorporating Jackscrews. Deep bites can be treated by using removable, fixed or
l The appliance consists of an acrylic plate, a myofunctional appliances.
jackscrew and an Adams clasps on the poste- Treatment mechanics/appliances used in correction of
rior teeth to retain the plate. A labial bow is deep bite:
also incorporated for minor space closure and I. Intrusion mechanics:
retraction. l Utility arches
B. Fixed appliances: l Burstone intrusion arch
l Unilateral crossbites can also be treated with l Arch wires with reverse curve of Spee
fixed appliances. Asymmetrically expanded arch l Arch wires with anchor bends/tip back bends
wires can bring about correction of crossbite. II. Relative Intrusion mechanics:
C. Expansion appliances: l Activator
i. Coffin spring: l Bionator
l It is a removable appliance that consists of an III. Extrusion mechanics:
omega-shaped wire of 1.25 mm diameter l Anterior bite plane
placed in the mid-palatal region. i. Removable appliances (anterior bite plane):
l It brings about dentoalveolar expansion, but it l Anterior bite plane is the most commonly
is capable of producing skeletal changes when used removable appliance for treatment of
used in young patients. the deep bite.
Section | I  Topic-Wise Solved Questions of Previous Years 249

l Anterior bite plane is a modified Hawley’s Q.11. Treatment of crossbite.


appliance with a flat ledge of acrylic be-
Ans.
hind the maxillary anteriors.
l When the patient bites, the mandibular in- [Same as SE Q.6]
cisors contact the bite plane thus disclos-
ing the posteriors, which are free to erupt.
l The height of the anterior bite plane should
SHORT NOTES:
be just enough to separate the posteriors by Q.1. Open bite.
1.5–2.0 mm as the posterior teeth erupt the
Ans.
height of the bite plane is gradually in-
creased. l Open bite is a malocclusion that occurs in the vertical
ii. Fixed appliances: plane characterized by lack of vertical overlap between
l Fixed orthodontic appliances can be used the maxillary and mandibular dentition
to intrude the anteriors. l It is of two types: Anterior open bite and posterior open

Anchor bends: bite.


l Anchorage bends are given in the arch l Aetiology:

wire mesial to the molar tubes so that i. Disharmony of skeletal growth pattern between
the anterior part of the arch wire lies maxilla and mandible.
gingival to the bracket slot. ii. Alterations in morphology and posture of the
l When these arch wires are pulled oc- tongue
clusally and engaged into the brackets, a iii. Prolonged thumb-sucking habit and tongue
gingivally directed intrusive force is ex- thrusting
erted on the incisors, which reduces the iv. Nasopharyngeal airway obstruction.
deep bite. l Treatment of the open bite:

Arch wires with reverse curve of Spee: i. Removal of the aetiological cause.
l They are used to intrude the anteriors. ii. Orthodontic therapy
l When these arch wires are inserted into iii. Myofunctional therapy
the molar tubes, the anterior segment iv. Surgical correction
curves gingivally. v. In cases of posterior open bite due to infra-
l This anterior segment is forced occlus- occlusion, placement of crowns on posterior
ally into the bracket slot resulting in an teeth to restore normal occlusal level is the best
intrusive force on the incisors. treatment.
iii. Myofunctional appliances:
Q.2. Anterior open bite.
l Deep bite with infra-occlusion of molars

can be treated by using activator and Ans.


bionator.
[Ref LE Q.1]
l The design of functional appliance is mod-

ified to allow the extrusion of the posterior Q.3. Treatment methods of anterior crossbite.
teeth.
Ans.
Q.8. Aetiology of open bite.
[Ref LE Q.2]
Ans. Q.4. Catalan’s appliance.
[Same as SE Q.1] Ans.
Q.9. Catalan appliances – Mention the disadvantage of [Ref LE Q.2]
this appliance.
Q.5. Management of posterior crossbite.
Ans.
Ans.
[Same as SE Q.3]
Various appliances used in treatment of posterior crossbite
Q.10. Lower anterior bite plane. are
A. Removable appliances
Ans.
i. Crossbite elastics
[Same as SE Q.3] ii. Removable plates
250 Quick Review Series for BDS 4th Year, vol 1

. Fixed appliances
B According to Graber deep bite is a condition of excessive
C. Expansion appliances: overbite.
i. Coffin spring Aetiological causes of deep bite are as follows:
ii. Quad helix l Convergent rotation of skeletal jaw bases.

iii. RME l The upward and forward rotation of the mandible.

l Downward and forward inclination of the maxilla


Q.6. Clinical features of skeletal deep bite.
leads to deep bite.
Ans. l Supraeruption of anterior teeth.

l Premature loss of permanent teeth resulting in lin-


l According to Graber, deep bite is a condition of exces-
gual collapse of anterior teeth.
sive overbite, where the vertical measurement between
l Infra-occlusion of molars due to tongue thrusting.
maxillary and mandibular incisal margins is excessive
l Anterior tipping of posterior teeth.
when the mandible is brought into the habitual or cen-
l Large size teeth.
tric occlusion.
l Clinical features of skeletal deep bite are as follows: Q.9. Apertognathia.
i. Deep curve of Spee is seen in mandibular dentition. Ans.
ii. Reverse curve of Spee is seen in maxillary dentition.
[Same as SN Q.1]
iii. Gummy smile.
iv. Decreased lower facial height. Q.10. Clinical features of anterior open bite.

Q.7. Tongue blade therapy. Ans.


[Same as SN Q.2]
Ans.
Q.11. Deep overbite.
[Ref LE Q.2]
Ans.
Q.8. Aetiology of deep bite.
[Same as SN Q.8]
Ans.

Topic 30
Cleft Lip and Palate
COMMONLY ASKED QUESTIONS
LONG ESSAYS:
1 . Describe the aetiology, classification and clinical picture of cleft lip and palate patients.
2. Describe in brief about the malformations of dental arches in cleft lip and palate cases. Write about aetiology
and orthodontic management of cleft lip and palate cases.
3. What are the causes of cleft lip and palate? How will you manage the problems associated with them? [Same as LE Q.1]
4. Briefly discuss the treatment protocol from birth to adulthood of a child born with cleft lip and palate. [Same as LE Q.2]

SHORT ESSAYS:
1 . Orthodontic management of cleft palate.
2. Enumerate various classifications of cleft lip and palate and describe Veau’s classification of cleft lip and palate.
[Ref LE Q.1]
3. Surgical closure of cleft lip? Describe briefly any one technique.
Section | I  Topic-Wise Solved Questions of Previous Years 251

SHORT NOTES:
1 . Aetiology of cleft lip and palate. [Ref LE Q.2]
2. Name various techniques of cleft lip closure. [Ref SE Q.3]
3. Name surgical management procedures of cleft palate.
4. NAM.
5. What is ‘team approach’ in rehabilitation of cleft lip/palate patients? [Ref LE Q.2]

SOLVED ANSWERS
LONG ESSAYS:
Q.1. Describe the aetiology, classification and clinical [SE Q.2]
picture of cleft lip and palate patients.

l {Various classifications of cleft lip and palate are
Ans. (a) Davis and Ritchie classification (1922)
(b) Veau’s classification (1931)
Cleft lip and palate are congenital abnormalities that affect (c) Classification by Fogh Andersen (1942)
the upper lip and the hard and soft palate. This abnormality (d) Schuchardt and Pfeifer’s symbolic classification
may range from a small notch in the lip to a complete fis- (e) Kernahan’s stripped ‘Y’ classification
sure extending up to the roof of mouth and nose. (f) LAHSHAL classification}
l Cleft lip is more common in males while cleft palate is

more common in females. (a) Davis and Ritchie classification:


l Incidence: 1 in every 600–1000 births. This is a morphological classification based of
l Negroid race has least incidence (1 in every 2000 births). location of cleft relative to the alveolar process.
l Mongoloids have the highest incidence. Classified into three groups:

Unilateral cleft n 80% (clefts involving left side account


Unilateral
for 70% of cases)
Bilateral cleft n 20%
Aetiology: • Group I Bilateral
Genetic and environmental factors causing cleft lip and (Pre-alveolar clefts)
palate are: Involves only the lip Median
i. Heredity: Is an important aetiologic factor. Clefts
of the lip and palate may be transmitted as a l Group II (postalveolar clefts): Includes
dominant or a recessive trait. different degrees of hard and soft palate
ii. Environment: Another possible environmental clefts that extend up to alveolar ridge.
factor is teratogenesis. l Group III (alveolar clefts): Complete clefts
Some of known toratogens are rubella virus, involving palate 1 alveolar ridge 1lip
cortisone, mercaptopurine, methotrexate, valium
and dilantin.
iii. Multifactorial aetiology: Recent studies have
shown that aetiology of cleft lip and palate can-
Unilateral Bilateral Median
not be attributed solely to either genetic or envi-
ronmental factors. It seems to involve more than
one factor. [SE Q.2]
l Predisposing factors: Factors that believe to increase {b. Veau’s classification (1931)
risk of cleft lip and palate incidence are Four groups:
i. Increased maternal age Group 1: Cleft involving soft palate only
ii. Racial (mongoloids have greatest percentage of Group 2: Cleft involving hard and soft palate
incidence) extending up to incisive foramen
iii. Blood supply (any factor that decreases blood Group 3: Complete unilateral clefts involving-
supply to the nasomaxillary area during embryo- soft palate 1 hard palate 1 lip and alveolar
logical development predisposes to clefts) ridge
252 Quick Review Series for BDS 4th Year, vol 1

Group 4: Complete bilateral clefts affecting- The boxes are shaded in areas where the cleft has
soft palate 1 hard palate 1 lip and alveolar occurred
ridge}
c. Classification by Foghandersen (1942) 1 4
2
Single (unilateral 5
3 6
or median clefts)
Group 1: Clefts of lip
Subdivided 7
Double (bilateral
clefts) 8
Unilateral
Group 2: Clefts of lip + plate 9
Single double
Bilateral

Group 3: Clefts of the palate extending up to incisive f. LAHSHAL classification:


foramen. Simple classification by Okriens (1987).
d. Schuchardt and Pfeifer’s symbolic classification: LAHSHAL is a paraphrase of the anatomic areas
l Classification makes use of a chart made of affected by the cleft.
three pairs of rectangles which represent lip, L – Lip
alveolus and hard palate as we go down. A – Alveolus
l One inverted triangle at bottom represents soft H – Hard palate
palate. S – Soft palate
l Areas affected by clefts are shaded in the chart. H – Hard palate
Advantage: Simplicity A – Alveolus
Disadvantage: Difficulty in writing typing and L – Lip
communication. Areas involved in cleft are denoted by specifically
Left Right
indicating the alphabet standing for it.
For example: (1) L … S … (cleft if right lip and
Lip soft palate).
(2) L A … S … L (stand for right lip, alveolus and
soft palate together with left cleft lip).
Alveolus
This classification is based on the fact that:

Total cleft Hard palate can be → Bilateral


The clefts of lip, alveolus and hard palate 
Partial cleft While clefts involving soft palate are usually
→ Unilateral
Clinical picture and problems associated with clefts:
Soft palate A cleft lip and palate patient can be afflicted by a num-
ber of problems which can be broadly classified as:
A. Dental
B. Aesthetic
e. Kernahan’s stripped ‘Y’ classification:
C. Speech and hearing
Symbolic Classification – uses a stripped ‘Y’ hav-
D. Psychologic
ing numbered blocks, which represents specific
areas of the oral cavity. A. Dental problems: Presence of the cleft is associ-
Block 1 and 4 n lip ated with division, displacement and deficiency
Block 2 and 5 n alveolus of oral tissue. Cleft lip and palate patients can
Block 3 and 6 n hard palate anterior to inci- have one or mode of following features:
sive foramen i. Congenitally missing teeth (most common 2 2 )
Block 7 and 8 n hard palate posterior to inci- ii. Presence of natal and neonatal teeth
sive foramen iii. Presence of supernumerary teeth
Block 9 n soft palate iv. Ectopically erupting teeth
Section | I  Topic-Wise Solved Questions of Previous Years 253

v. Anomalies of tooth morphology, i.e. micro-


{SN Q.1}
dontia, macrodontia, fused teeth, aberra-
tions in crown shape and enamel hypoplasia Aetiology of cleft lip and palate:
vi. Mobile and early shedding of the teeth due Genetic and environmental factors causing cleft lip
to poor periodontal support and palate are:
vii. Posterior and anterior crossbite i. Heredity: Is an important aetiologic factor.
viii. Protruding premaxilla Clefts of the lip and palate may be transmitted
ix. Deep bite as a dominant or a recessive trait.
x. Spacing/crowding ii. Environment: Another possible environmen-
B. Aesthetic problems: tal factor is teratogenesis.
l Clefts involving lip can result in facial disfig- Some of known toratogens are rubella virus,
urement (mild to severe). cortisone, mercaptopurine, methotrexate, va-
l Orofacial structures may be malformed or lium and dilantin.
congenitally missing. iii. Multifactorial aetiology: Recent studies have
l Deformities of nose can also occur. shown that aetiology of cleft lip and palate
Thus aesthetics is greatly affected. cannot be attributed solely to either genetic or
C. Hearing and speech: Cleft lip and palate are environmental factors. It seems to involve
sometimes associated with middle ear disorders more than one factor.
that affect hearing.

Presence of hearing Difficulties in language Management of cleft lip and palate patient:
cause
 → The treatment protocol from birth to adulthood of a
problems uptake and speech
child born with cleft lip and palate is as follows:
D. Psychological problems:
These patients are under a lot of psychological {SN Q.5}
stares. Due to their abnormal facial appearance
l The complexity of the problem requires that a number
they have to put up with staring, curiosity, pity,
of health care practitioners cooperate to ensure compre-
etc. They face problems in getting jobs and mak-
hensive care and successful rehabilitation of the patient.
ing friends. They fare badly in academics as a
l This led to the concept of multidisciplinary cleft pal-
result of hearing impairment speech problems
ate team comprising:
and frequent absence from school.
Paediatrician
Q.2. Describe in brief about the malformations of dental Paedodontist
arches in cleft lip and palate cases. Write about aetiology Orthodontist
and orthodontic management of cleft lip and palate cases. Oral and maxillofacial surgeon
Prosthodontist
Ans.
Social worker
l The malformations of dental arches associated with Genetic scientist
cleft lip and palate are as follows: ENT surgeon
i. Dental aberrations Plastic surgeon
ii. Aesthetic and growth problems Psychiatrist
A speech pathologist
i. Dental aberrations include:
l The cleft palate team has been described as close,
a. Malalignment of alveolar arches
cooperative, democratic, multiprofessional union de-
b. Posterior and anterior crossbites
voted to single cause, i.e. patient well-being.
c. Spacing, crowding and various anomalies of tooth
morphology etc.
ii. Aesthetic and growth problems: l The management of cleft lip and palate can be
The severity of cleft lip or palate affects degree of divided into following stages:
deformity of dentofacial structures.
a. Midface deficiency with characteristic concave
A. Stage 1 n Birth to 18 months of age

profile. B. Stage 2 n 18 months to 5th year of life (primary


b. Hypoplastic maxilla with shortened columella dentition stage)
on the cleft side. C. Stage 3 n 6–11 year of age (mixed dentition stage)
c. Grossly deficient premaxilla in bilateral clefts.
d. Hypoplastic muscles of soft palate. D. Stage 4 n 12–18 year of age (permanent dentition
stage)
e. Nasal tip is widened and flattened.
254 Quick Review Series for BDS 4th Year, vol 1

A. Stage 1 treatment: (c) Surgical lip closure:


Treatment modalities carried out during first stage
include: Surgery should be performed
(a) Fabrication of a passive obturator within 45 days of birth. The
(b) Presurgical orthopaedics early surgery improves the
(c) Surgical management of cleft lip Early facial appearance and
(d) Surgical management of cleft palate school therefore improves child
suggests acceptance and ↓ parent
(a) Fabrication of passive maxillary obtu- apprehension.
rator:
l It is an intraoral prosthetic device fabri-
Two
cated using clod cure acrylic. schools The surgery should be
l Clasps can aid in retention. In case of postponed till the
insufficient retention, wings made of completion of dentition.
thick wire can be embedded in acrylic Reason being that the tissue
and can be stabilized against cheeks us- The late would be able to grow and
ing micropore adhesive tape. school mature, thereby giving the
l It fills palatal cleft and provides a false surgeon more muscle mass
roofing against which the child can to work on.
suckle.
l It reduces incidence of feeding difficul- Millard suggested rule of 10:
ties like insufficient suction, excessive Surgery should be performed in a child
air intake and choking. at the age not less than 10 weeks of age
l It provides maxillary cross arch sta- and body weight not less there 10 pounds
bility, thus preventing arch from col- and Hb% not less than 10 g%.
lapsing. (d) Surgical palate closure:
(b) Presurgical orthopaedics: l Palate repair should be attempted be-

l The aim of presurgical orthopaedics is tween 12 and 24 months of age; this


to achieve upper arch from that con- facilitates normal speech, hearing and
forms to the lower arch. improves swallowing.
l The orthodontist should try to correct l Repair is done using bone transplants

the displacement such as outward dis- from rib, iliac bone, mandibular sym-
placement of premaxilla (in bilateral physis, tibial bone or outer table of pa-
cleft) and displacement of greatest seg- rietal bone.
ment (in unilateral clefts) by extra-oral B. Stage two treatment:
strapping across the premaxilla at- This stage of treatment is carried out during pri-
tached directly to the face or to some mary dentition period.
form of head cap. A micropore adhe- The procedures include:
sive tape can also be strapped across the (a) Adjustments in the intraoral obturator to
premaxilla. accommodate the erupting deciduous teeth
l In case of a narrow, collapsed maxillary (b) To maintain a check on eruption pattern
arch – expansion can be achieved by a and timing
suitable appliance incorporating screws (c) Oral hygiene instructions
or springs. (d) Restoration of decayed teeth
l Advantages of presurgical orthopaedic No orthodontic treatment is usually initiated dur-
phase are ing this phase.
i. g size of clefts thereby aiding in C. Stage three treatment:
surgery It includes treatments carried out during mixed
ii. Partial obturation of clefts assists dentition phase.
feeding Orthodontic procedures usually carried out are
iii. Improved speech – As size of (a) Correction of anterior crossbites with re-
defect g movable or fixed appliances.
iv. Reassures the parents at a crucial For example: removable appliance with
time Z-spring to treat anterior crossbite
Section | I  Topic-Wise Solved Questions of Previous Years 255

(b) Buccal segment crossbites are treated us- appliance pinned to the segments in severe cases and
ing quad helix or expansion screws. pressure from repaired cleft lip.
D. Stage four treatment: l Construction of feeding plates to assist the child in

l Consists of treatment during permanent den- feeding.


tition – with fixed orthodontic appliance. ii. Late primary and mixed dentition period:
l All local irregularities – crowding, spacing, l Orthodontic treatment is not required at this stage.

crossbites and overjet lover bite problems are iii. Early permanent dentition period:
corrected. l Fixed appliance orthodontic treatment is started at this

l Patients with hypoplastic maxilla – are given age, as there is tendency to develop posterior crossbites.
face mask to advance the maxilla. l Space closure is carried out after successful bone

l In case of missing teeth – prosthesis can be grafting.


given after completion of orthodontic treat- iv. After growth completion:
ment. l Comprehensive orthodontic treatment using pre-adjusted

l Following completion of orthodontic treat- edge-wise appliances for precise positioning of teeth.
ment long retention phase is required in these l Chin cup can be used to redirect the mandibular

patients. growth.
Because of inadequate bone support, absence l Maxillary protraction by using orthognathic surgery.

of some teeth presence of stretched scar tissue. l Maxillar distraction osteogenesis that provides simul-
These patients should be treated with sympa- taneous maxillary advancement as well as expansion
thy and concern in addition to flexibility and of soft tissues.
multidisciplinary approach.
Q.2. Enumerare various classifications of cleft lip and pal-
ate and describe Veau’s classification of cleft lip and palate.
Q.3. What are the causes of cleft lip and palate? How
will you manage the problems associated with them? Ans.
Ans. [Ref LE Q.1]
[Same as LE Q.1] Q.3. Surgical closure of cleft lip? Describe briefly any
one technique.
Q.4. Briefly discuss the treatment protocol from birth to
adulthood of a child born with cleft lip and palate. Ans.
Ans. l Surgical closure of cleft lip is known as cheiloplasty.
l There are three principles in closure of a cleft lip:
[Same as LE Q.2]
i. The positioning of structures in their anatomically
most appropriate position (primary muscle continuity)
SHORT ESSAYS: ii. Reconstruction of a muscle sphincter
iii. The avoidance of a straight line lip scar
Q.1. Orthodontic management of cleft palate.
Principles of symmetry:
Ans. l Primary muscle continuity

l Proper philtral shape and size


Orthodontic treatment of cleft lip and palate can be studied
l Formation of median tubercle from lateral
under four stages:
labial elements
i. Infancy stage
ii. Late primary and mixed dentition period
{SN Q.2}
iii. Early permanent dentition period
iv. After growth completion Various techniques of cleft lip repair are:
i. Tension – Randall repair (triangular flap repair)
i. Infancy stage:
ii. Lemesurier (1949) technique (quadrangular flap
l Infantorthopaedics is usually carried out at 3–6
repair)
weeks of age so as to facilitate lip closure done at
iii. Millard (1976) lip repair (rotation and advance-
about 10 weeks of age.
ment tech)
l Different orthopaedic solutions to reposition the max-
iv. Delaires technique
illary segments and retract premaxilla are: light elas-
v. Modified Millard’s technique
tic strap across premaxillary segment, orthodontic
256 Quick Review Series for BDS 4th Year, vol 1

Tennison–Randall repair: ii. The quadric lateral scar has a horizontal


l This technique deploys triangular flap. element which does not approximate to
l Points in technique: Mathematical precision in mea- any natural structure of the lip and will
surement is necessary (Randall 1971). always look unnatural.
Advantages: iii. Thus procedures accidentally reorientate
i. Skin incision can be combined with a muscle muscle bundles within quadrilateral flaps
dissection and repositioning. in a nonfunctional way. Thus muscle tissue
ii. Straight line scar is avoided. may even be denervated.
iii. The cupids bow in preserved by advancing a iv. Later revision of the scar can be extremely
triangular flap into the free edge of the medial lip difficult if the lip is too long or too short.
to bring the vermilion down, thereby equalizing Millard lip repair:
the columella length on cleft and noncleft side. l Most popular technique in current times internationally.

l The technique is described as skin rotation advance-

ment.
l The entire cupids bow and dimple component is low-

ered (rotation) by incising in the line of the hypoplas-


tic or missing philtral column. This opens a gap into
which the lateral lip element and flared alar base can
be placed (advancement).
l Points of technique:

The vertical component of the scar seems nearly


Marked incisions Closure always to be short and this should perhaps be de-
signed a little longer than is necessary.
Disadvantages:
i. Philtrum is broken up by triangular flap.
ii. Secondary correction can be difficult.
Le mesurier technique:
(Le mesurier right-angled flap lip repair) 1
2

1 2

Incision marked Closure after freely of rotation flap


(1) and advancement flap (2)

Advantages:
Incision marked
i. The flaps can be modified after initial cutting
(cut as you go) to bring down the cleft side to the
This technique uses quadrilateral flap to achieve a level of the noncleft side.
nonstraight line closure. ii. This technique sacrifices some little amount of
Advantages: tissue from the margin of the cleft.
l As a quadrilateral flap was raised compris- iii. Dissection of the muscle as a separate layer is
ing full thickness skin, muscle and mucosa, relatively straight forward and a three layer clo-
it provides much tissue for closing a wide sure can be achieved.
cleft by bringing musculocutaneous flap iv. Scar is excellent for later revision.
from upper part of the lateral lip down to fill v. It stimulates normal philtral column and tech-
the deficiency in the lower part of the lip nique provides best possible nasal philtrum.
defect. Disadvantages of Millard’s lip repair:
Disadvantages: i. The scar is almost always little short.
i. The workings are made preoperatively on ii. Even when the static length of new philtral col-
the basis of arbitrary measurements and no umn is satisfactory, dynamic motion will not be
modification can be made during operation. natural.
Section | I  Topic-Wise Solved Questions of Previous Years 257

iii. In a wide cleft the large closure can be difficult v. Oslo technique
to achieve and it can seem as a very radical pro- vi. Delaires technique
cedure when the cleft is very minimal.
Q.4. NAM.
In spite of some disadvantages the Millard’s
technique remains the most versatile technique Ans.
and probably the best available.
i. Nasoalveolar moulding (NAM) is a nonsurgical tech-
nique developed by Dr Court Cutting and Dr Barry
SHORT NOTES: Grayson at New York University.
ii. They combined the moulding techniques used in orth-
Q.1. Aetiology of cleft lip and palate.
odontic treatment and latest cosmetic surgical tech-
Ans. niques to develop nasoalveolar moulding.
iii. NAM is only effective in infants below the age of
[Ref LE Q.2]
6 months as their cartilage is malleable.
Q.2. Name various techniques of cleft lip closure. iv. Advantages of NAM:
l NAM device reduces the number of surgeries re-
Ans.
quired during patient’s life time thereby reducing
[Ref SE Q.3] the facial scarring, trauma, inconvenience and cost
involved in additional surgeries.
Q.3.Name surgical management procedures of cleft
l It also helps with feeding.
palate.
l Nose has been lifted and narrowed by the time of

Ans. surgery, smaller gap in cleft region reduces tension


when surgeon closes the cleft.
Various surgical procedures in management of cleft palate
are as follows: Q.5. What is ‘team approach’ in rehabilitation of cleft
i. Bardach’s two flap technique lip/palate patients?
ii. Cutting’s technique
Ans.
iii. Salyer’s modified two flap technique
iv. Oxford technique [Ref LE Q.2]

Topic 31
Surgical Orthodontics
COMMONLY ASKED QUESTIONS
LONG ESSAYS:
1 . Classify surgical orthodontics. Discuss about minor surgical orthodontic procedures.
2 . What is surgical orthodontics? Enumerate the minor and major surgical orthodontic procedures. Discuss any
three minor surgical orthodontic procedures. [Same as LE Q.1]
3 . Define surgical orthodontics. Give objectives, classification of surgical orthodontics and discuss in detail about
minor surgical orthodontic procedures. [Same as LE Q.1]

SHORT ESSAYS:
1 . Genioplasty.
2 . Enumerate the surgical modalities in correction of mandibular prognathism. Describe any one in brief.
3 . Enumerate minor surgical procedures in orthodontics. Write briefly about transplantation of teeth.
4 . Sterilization in orthodontics.
5 . Cosmetic surgeries. [Same as SE Q.1]
258 Quick Review Series for BDS 4th Year, vol 1

SHORT NOTES:
1. Pericision. [Ref LE Q.1]
2. Surgical orthodontics. [Ref LE Q.1]
3. Genioplasty. [Ref SE Q.1]
4. Impacted upper permanent canine.
5. Frenectomy.
6. Orthognathic surgery in the maxilla.
7. Resection.
8. Supracrestal fibrotomy. [Same as SN Q.1]
9. Surgical aids in orthodontics. [Same as SN Q.2]
10. Minor surgical procedures. [Same as SN Q.2]
11. Cosmetic surgeries. [Same as SN Q.3]
12. Osteotomies. [Same as SN Q.7]

SOLVED ANSWERS
LONG ESSAYS:
Q.1. Classify surgical orthodontics. Discuss about minor Various minor surgical procedures are discussed in
l
surgical orthodontic procedures. detail below:
i. Extractions:
Ans.
These are the most commonly undertaken minor
surgical procedures in conjunction with orthodontic
{SN Q.2} therapy.
a. Therapeutic extractions:
l Surgical orthodontics is a term that refers to surgical
l Undertaken mainly to gain the space.
procedures carried out as an adjunct to or in conjunc-
l Preoperative radiographs are valuable aid in
tion with orthodontic treatment.
planning and execution of extraction.
l Surgical orthodontic procedures are broadly classi-
b. Serial extraction:
fied as minor surgical procedures and major surgical
Involves removal of some deciduous teeth fol-
procedures.
lowed by specific permanent teeth in an orderly
A. Minor surgical procedures:
sequence to guide the rest of the permanent teeth
i. Extractions:
into a more favourable position during mixed
a. Therapeutic extractions
dentition period.
b. Serial extractions
c. Extraction of supernumerary, impacted and anky-
c. Extraction of carious teeth
losed teeth:
d. Extraction of malformed teeth
The presence of supernumerary, impacted and
e. Extraction of supernumerary teeth
ankylosed teeth are important local causes of mal-
f. Extraction of impacted teeth
occlusion prior to removal of these teeth their
ii. Surgical uncovering/exposure of teeth
exact location and their relationship with adjacent
iii. Frenectomy
structures should be ascertained by radiographs.
iv. Pericision
ii. Surgical uncovering of impacted teeth:
v. Transplantation of teeth
Impacted teeth due to presence of mucosal and bony
vi. Corticotomy
barriers that prevent their eruption should be man-
vii. Removal of soft tissue barrier
aged in the following steps:
viii. Removal of cysts and odontomes
l Location of the tooth – Clark’s technique or
ix. Orthodontic implants
right angle technique
B. Major surgical procedures:
l Evaluation of favourability – consider favour-
i. Orthodontic surgeries
able whenever apex of canine is close to its
ii. Cosmetic surgeries
normal position
iii. Surgical corrections in cleft lip and palate
l Evaluation of space adequacy.
patients
l Surgical excision and bone removal – overall
iv. Surgical assisted rapid maxillary expansion
tooth
v. Distraction osteogenesis
l Fixing orthodontic attachments
Section | I  Topic-Wise Solved Questions of Previous Years 259

iii. Frenectomy:
l Pericision is a minor surgical procedure that is
Maxillary midline frenum, mandibular labial and
undertaken under local anaesthesia to counter the
lingual frenum might contribute to orthodontic
relapse tendency of the stretched gingival fibres.
problems.
The transeptal and alveolar crest group of gingival
Frenectomy is a surgical procedure performed to
fibres remains stretched and do not readily re-
excise the frenum and remove the deeply embedded
adapt to the new tooth position following correc-
fibrous tissue.
tion of rotation, hence causing relapse.
l Pericision involves surgical sectioning of these
Timing of frenectomy
fibres by passing a sharp narrow scalpel through
the gingival sulcus around the tooth to a depth of
2-mm apical to the alveolar crest.
According to one school According to another school l Pericision is generally undertaken as an adjunc-

of thought tive retention procedure after the correction of


rotations.
It should be performed prior It should be performed after
to orthodontic closure of orthodontic space closure Q.2. What is surgical orthodontics? Enumerate the mi-
midline diastema. as it reduces the risk of nor and major surgical orthodontic procedures. Discuss
scar tissue formation any three minor surgical orthodontic procedures.
that can prevent closure
Ans.
of the midline diastema.
[Same as LE Q.1]
The following points should be remembered during
Q.3. Define surgical orthodontics. Give objectives, clas-
frenectomy regardless of timing when the procedure
sification of surgical orthodontics and discuss in detail
in performed.
about minor surgical orthodontic procedures.
a. The frenum should not merely be clipped. It
should be totally excised to bone level. Ans.
b. Any palatally attached fibrous tissue should
[Same as LE Q.1]
be removed.
c. Fibrous tissue attached to the intermaxillary
suture area should be removed. SHORT ESSAYS:
d. The mucosa of the lip is undermined to pre-
Q.1. Genioplasty
vent reattachment of the fibrous tissue.
iv. Corticotomy: Ans.
l Usually undertaken in patients having dental pro-

clination with spacing and in median diastema


without any other features of malocclusion. {SN Q.3}
l It involves sectioning of the dentoalveolar region l Cosmetic surgeries are surgical procedures carried
into multiple small units to speed up orthodontic out to improve the aesthetic appearance of the patient.
tooth movement.
l Surgical steps in corticotomy are as follows:
Nose Rhinoplasty
a. Raising labial flaps.
b. Interdental bony cuts are made parallel to
• Cosmetic surgery of the
long axis of teeth.
c. Joining of the cuts together by a horizontal
bony cut above the apices of the roots. Chin Genioplasty
d. Following surgery orthodontic tooth move-
ments is reiterated using fixed appliances. Genioplasty:
l Surgical correction of chin is known as genio-

plasty.
{SN Q.1}
l It is of following types:

v. Pericision: Augmentation and reduction of genioplasties.


l Pericision is also known as supracrestal fibrot- i. Single section horizontal sliding genio-
omy, circumferential supracrestal fibrotomy, sul- plasty for augmentation of chin. Tran-
cus slice procedure and Edwards procedure. sosseous wires are placed to decrease the
260 Quick Review Series for BDS 4th Year, vol 1

It is a versatile intraoral surgical procedure per-


l
possibility of relapse. Slight overcorrec-
formed under general anaesthesia.
tion is employed.
l Asymmetry and crossbite corrections are possible

with this procedure.


Surgical steps:
l On the medial aspect of the ramus, a horizontal

cut is done only in the cortical bone above the


mandibular foramen.
ii. Double section horizontal sliding geni- l The cut is stopped just behind the mandibular fo-

oplasty. The additional section allows ramen. The cut is then taken just medial to the
for greater augmentation of chin promi- lateral oblique ridge to the horizontal ramus about
nence 2.5 mm below the bony cervical margin. This is
brought up to the level of second molar or a little
anterior.
l The vertical cut of the bone is done from the

anterior end of the cut downward to the inferior


border.
l Bone splitting is started at the anterior aspect of

iii. Genioplasty for length reduction of chin the medial cut on the vertical ramus using a 4 mm
prominence. Osteotomy bony incisions osteotome. Spreader (Smith bone spreader) is en-
are made in more vertical plane and a gaged in the split and the split is spread out to
section equal to the desired reduction is completion. Care should be taken not to cut the
removed. inferior alveolar nerve.
l All the above-mentioned steps are repeated on the

opposite side.
l Mandibular setback is achieved and the prefabri-

cated splint is placed in proper occlusion and


intermaxillary fixation is done.
l BSSO can be used to correct mandibular progna-
iv. Genioplasty for height reduction of an- thism, mandibular retrognathism and facial
terior portion of mandible. asymmetry.
l Depending on the purpose of the surgery, man-

dibular setback is done for mandibular progna-


thism, mandibular advancement is done for re-
truded mandible and mandibular repositioning is
done for facial asymmetry.
Q.3. Enumerate minor surgical procedures in orthodon-
tics. Write briefly about transplantation of teeth.
Q.2. Enumerate the surgical modalities in correction of
Ans.
mandibular prognathism. Describe any one in brief.
Surgical orthodontic procedures are broadly classified as
Ans.
minor surgical procedures and major surgical procedures.
The mandibular prognathism can be corrected by various A. Minor surgical procedures include:
surgical procedures like: i. Extractions:
i. Mandibular body osteotomy a. Therapeutic extractions
ii. Mandibular ramus osteotomy b. Serial extractions
iii. Sagittal split technique c. Extraction of carious teeth
iv. C and L osteotomies d. Extraction of malformed teeth
v. Vertical subsigmoid osteotomy e. Extraction of supernumerary teeth
Sagittal split technique: f. Extraction of impacted teeth
l One of the most widely used procedure for man- ii. Surgical uncovering/exposure of teeth
dibular reduction is bilateral sagittal split osteot- iii. Frenectomy
omy (BSSO). iv. Pericision
Section | I  Topic-Wise Solved Questions of Previous Years 261

v. Transplantation of teeth Q.3. Genioplasty.


vi. Corticotomy
Ans.
vii. Removal of soft tissue barrier
viii. Removal of cysts and odontomes [Ref SE Q.1]
ix. Orthodontic implants
Q.4. Impacted upper permanent canine.
Transplantation or transpositioning of teeth:
l It is a technique of reimplanting tooth after being Ans.
removed into a newly created socket.
l Impacted teeth due to presence of mucosal and bony
For example: Teeth most commonly transplanted
barriers that prevent their eruption should be managed
are 3rd molars into first molar region.
in the following steps:
l Criteria for selection of case: Sufficient space
i. Location of the tooth and their relation with adjacent
should be available and minimum trauma to the
structures should be confirmed by radiographs –
prepared socket and tooth to be reimplanted
Clark’s technique or right angle technique.
should have a wide open apex, should be ex-
l Evaluation of favourability.
tracted atraumatically without damaging peri-
l Evaluation of space adequacy.
odontal ligament and cementum.
l Depending on location, tooth should be surgically
l Procedure:
approached by either a buccal or palatal flap and
a. After the preparation of recipient site, tooth to
adequate amount of bone should be removed us-
be transplanted is uncovered extracted a trau-
ing rotary cutting instruments.
matically.
l Fixing orthodontic attachments, the tooth can be
b. Extracted tooth is placed in the prepared bony
aligned in occlusion if possible, otherwise it is
socket.
extracted and the wound is closed with sutures.
c. Wound is closed and sutured with 3-0 black silk.
Q.5. Frenectomy.
Q.4. Sterilization in orthodontics.
Ans.
Ans.
l Frenectomy is a surgical procedure performed to excise the
Various methods of sterilization commonly used are
frenum and remove the deeply embedded fibrous tissue.
i. Hot air oven
l Timing of frenectomy: According to one school, it
ii. Boiling water
should be performed prior to orthodontic closure of mid-
iii. Autoclave
line diastema and according to another school of thought,
iv. Cold sterilization
it should be performed after orthodontic space closure.
The best method of sterilization used is autoclave:
l Regardless of timing when the procedure is performed,
l It provides moist heat in the form of saturated
the following points should be remembered during fre-
steam under pressure.
nectomy:
l Holding temperature is 121°C for 20 min under
a. The frenum should be totally excised to bone level.
15 lb pressure or 134°C for 3 min.
b. Fibrous tissue attached palatally or to the intermaxil-
l It is most effective and time-efficient method.
lary suture area should be removed.
l It kills microorganisms by coagulation and dena-
c. The mucosa of the lip is undermined to prevent reat-
turation of proteins.
tachment of the fibrous tissue.
Q.5. Cosmetic surgeries.
Q.6. Orthognathic surgery in the maxilla.
Ans.
Ans.
[Same as SE Q.1]
l Orthognathic surgery is an art and science of diagnosis,
treatment planning and execution of treatment to correct
SHORT NOTES: musculoskeletal, dento-osseous and soft tissue deformi-
ties of orofacial region.
Q.1. Pericision.
l Orthognathic surgeries are major surgical procedures

Ans. carried out along with orthodontic therapy to correct


dentofacial or orofacial deformities.
[Ref LE Q.1]
l Maxilla is corrected using Le Fort I osteotomy as per

Q.2. Surgical orthodontics. requirement by individual case as following:


i. Maxillary retrusion or hypoplasia: Le Fort I osteot-
Ans.
omy with maxillary advancement.
[Ref LE Q.1] ii. Maxillary protrusion: Maxillary segmental setback.
262 Quick Review Series for BDS 4th Year, vol 1

iii. Maxillary deficiency: Le Fort I osteotomy to ad- Q.8. Supracrestal fibrotomy.


vance and impact maxilla.
Ans.
iv. Maxillary vertical excess: Le Fort I osteotomy with
maxillary impaction. [Same as SN Q.1]

Q.7. Resectionor osteotomies. Q.9. Surgical aids in orthodontics.

Ans. Ans.
[Same as SN Q.2]
l The fundamental or basic biology behind all orthogna-
thic surgical procedures is that the bones of maxilla and Q.10. Minor surgical procedures.
mandible are intentionally sectioned and repositioned to
Ans.
desired sites to correct dentofacial deformities.
l Osteotomies refers to simple bone cuts, whereas ostec- [Same as SN Q.2]
tomy or resection means removal of a portion of bone.
Q.11. Cosmetic surgeries.
For example:
i. Le Fort I osteotomy to correct maxillary retru- Ans.
sion or hypoplasia
[Same as SN Q.3]
ii. Mandibular ramus osteotomy
iii. C and L osteotomies for repositioning of body Q.12. Osteotomies.
and dentoalveolar segment of mandible
Ans.
iv. BSSO (bilateral sagittal split osteotomy) for
mandibular reduction [Same as SN Q.7]

Topic 32
Retention and Relapse
COMMONLY ASKED QUESTIONS
LONG ESSAYS:
1 . Define retention and relapse. Write briefly about causes of relapse.
2. Enumerate theories of retention.
3. What is retention? Describe in brief various retention appliances commonly used in orthodontic practice.
4. Define retention and relapse in orthodontics. What are the causes of relapse and how would you prevent them?
[Same as LE Q.1]
5. Discuss the role of periodontium and growth in causing relapse. Write in brief four other causes of relapse?
[Same as LE Q.1]
6 . Define retention and discuss various theories of retention. [Same as LE Q.2]
7. Write the theories of retention and types of retention. [Same as LE Q.2]
8. Classify retention appliances and describe in detail the Hawley retainer. [Same as LE Q.3]
9. Explain in detail retention after treatment of malocclusion. [Same as LE Q.3]

SHORT ESSAYS:
1 . Retention in orthodontics.
2. What is relapse? What are the causes of relapse? [Ref LE Q.1]
3. Fixed retainer.
4. Theories of retention. [Ref LE Q.2]
5. Define retention. Explain the schools of retention.
6. Permanent retention. [Same as SE Q.1]
7. Relapse in orthodontics. [Same as SE Q.2]
8. Define relapse. Add a note on the role of third molars in causing relapse. [Same as SE Q.2]
Section | I  Topic-Wise Solved Questions of Previous Years 263

SHORT NOTES:
1. Relapse in orthodontics. [Ref LE Q.1]
2. Hawley’s retainer. [Ref LE Q.3]
3. Permanent retention. [Ref SE Q.1]
4. Retention appliance. [Ref LE Q.3]
5. ‘Relapse’ and ‘retention’. [Ref SE Q.1]
6. Name different schools of thought for retention.
7. What is the role of third molar in causing the relapse?
8. Adjunctive periodontal surgeries to minimize relapse.
9. Riedel’s theorems of retention and relapse. [Ref LE Q.2]
10. Define retention period.
11. Give an example of natural or self-retention. [Ref SE Q.1]
12. What is relapse? How to prevent relapse?
13. Define and classify retention giving examples. [Ref SE Q.1]
14. Upper Hawley’s appliance. [Same as SN Q.2]
15. Retainers. [Same as SN Q.4]
16. Define retention. [Same as SN Q.5]
17. Types of retention. [Same as SN Q.13]

SOLVED ANSWERS
LONG ESSAYS:
Q.1. Define retention and relapse. Write briefly about l The principle fibres of periodontal ligament
causes of relapse. rearrange themselves quite rapidly to the
new position within about 4 weeks time. The
Ans.
supra-alveolar gingival fibres take around
l Retention has been defined by Moyer’s as ‘maintaining 40 weeks to rearrange around new position
newly moved teeth in position, long enough to aid in and thus predispose to relapse provided
stabilizing their correction’. proper retention period is not continued.
II. Relapse due to growth-related changes:
(SN Q.1 and SE Q.2)
l Patients with skeletal Class II, Class III, open
l {(Relapse can be defined as ‘the loss of any correction bite or deep bite malocclusion may exhibit
achieved by orthodontic treatment’. relapse due to continuation of abnormal
Or in simple terms it is the tendency of teeth to return growth pattern after orthodontic treatment.
back to original position post-treatment. l Hence prolonged retention is indicated until
l Causes of relapse: active growth is completed.
Numerous causes are attributed to relapse. There is III. Bone adaptation:
no single factor that can be said to be the sole cause l Teeth moved recently are surrounded by
of relapse. In most cases, relapse occurs due to a lightly calcified osteoid bone. Thus the
combination of causes. teeth are not adequately stabilized and
l The main aetiologic causes of relapse are as follows: have a tendency to move to their original
i. Periodontal ligament traction position.
ii. Relapse due to growth-related changes l Normally bony trabecular are arranged per-
iii. Bone adaptation pendicular to long axis of teeth, whereas
iv. Muscular forces during orthodontic treatment, they get
v. Failure to eliminate the original cause aligned parallel to direction of force. During
vi. Role of 3rd molars retention phase they revert back to their
vii. Role of occlusion)} normal alignment.
IV. Muscular forces:
[SE Q.2] Muscle imbalance at the end of orthodontic treat-
{i. Periodontal ligament traction: ment result in reappearance of malocclusion.
l Orthodontic treatment causes n Stretching of V. Failure to elucidate original cause:
periodontal ligament and gingival fibres; there Failure to remove the aetiology of malocclusion
fibres can contract and cause relapse. can result in relapse.
264 Quick Review Series for BDS 4th Year, vol 1

VI. Role of 3rd molars:


iii. Theorem 3:
l Most patients would have completed their
l ‘Malocclusion should be over-corrected as a safety
orthodontic treatment by the time 3rd molars
factor’. So as to give a Leeway for a certain
start erupting, i.e. around 18–21 years of age.
amount of relapse.
l The pressure exerted by erupting 3rd molars
l This is being practised by many orthodontists, to
is believed to cause late anterior crowding,
allow some amount of relapse so that after relapse
predisposing to relapse.
the occlusion will be normal.
VII. Role of occlusion:
For example, overcorrection is done in treating
l The CR and CO should coincide or the slide
class II and III malocclusions and rotations.
from centric should not be .1.5–2 mm in
iv. Theorem 4:
order to have greater stability of orthodontic
l ‘Proper occlusion is a potent factor in holding
treatment results.
teeth in their corrected positions’.
l Presence of certain occlusal mannerisms
l Post-treatment stability is increased by good occlusion.
such as clenching, grinding, nail biting and
l Obtaining correct intercuspation and proper func-
lip biting are important causes of relapse.}
tional occlusion are essential factors in occlusal
Q.2. Enumerate theories of retention. stabilization.
v. Theorem 5:
Ans.
l ‘Bone and adjacent tissues must be allowed timely

[SE Q.4] to reorganize around newly positioned teeth’.


l When teeth are moved orthodontically, numerous
{Retention has been defined by Moyer’s as ‘maintaining changes occur in the bone and surrounding tissues.
newly moved teeth in position, long enough to aid in stabi- It takes considerable time for the reorganization to
lizing their correction’. be completed.
Stabilizing the treatment results by retention procedures vi. Theorem 6:
is an integral part of orthodontic therapy and therefore pro- l ‘If the lower incisors are placed upright over basal bone
vision should be made in the treatment plan for adequate they are more likely to remain in good alignment’.
retention keeping in mind the destabilizing factors. l Most stable results are obtained by placing the

(SE Q.4 and SN Q.9) mandibular incisors upright over the basal bone.
vii. Theorem 7:
{(There are about 10 theorems on retention. The first l ‘Corrections carried out during periods of growth
nine theorems are put forward by Riedel, while tenth theo- are less likely to relapse’.
rem was included by Moyer. l Treatment modalities carried out during the active
Theories of retention are as follows: growth period allows the tissue systems to adapt
i. Theorem 1: well and therefore reduce the relapse potential.
l ‘Teeth that have been moved tend to return to their
l Especially in skeletal malocclusion, early treat-
former position’. ment planning offers greater advantage in long-
l The causes for this relapse are many and a single
term stability.
aetiology cannot be highlighted. viii. Theorem 8:
l This theorem mainly applies to correction of rota-
l ‘The farther the teeth have been moved, the lesser is
tions. The apical base, trans-septal fibres and mus- the risk of relapse’, i.e. farther the teeth are moved,
culature may be responsible for teeth to go back to lesser the risk of it returning to its original position.
their original position. l There is only a little evidence to support this concept.
ii. Theorem 2: ix. Theorem 9:
l ‘Elimination of the cause of malocclusion will pre-
l ‘Arch form particularly in the mandibular arch, can-
vent relapse’. not be permanently altered by appliance therapy’.
l The cause for the malocclusion should be identified
l Alteration of existing arch form results in h risk of
at the time of diagnosis and adequate steps should relapse.)}
be taken during the treatment plan to eliminate it.
l Failure to remove the cause h relapse potential. This
[SE Q.4]
theorem can be applied in cases of malocclusion
where the cause is obvious, e.g. thumb sucking, x {Theorem 10:
tongue thrusting and not in any malocclusion where ‘Many treated malocclusions require permanent re-
l

the cause is elusive. taining devices’. This theorem was subsequently


added by Moyer.
Section | I  Topic-Wise Solved Questions of Previous Years 265

l This is true in cases that have not been treated to


l Several modifications to suit specific require-
achieve occlusal goals that stand for stability.
ments are as follows:
l No permanent retention is required in correction of
i. A long labial bow instead of a short labial
certain malocclusions with specific occlusal goals
bow used in cases of first premolar extraction,
and regard to growth and functional aspect.}
helps to prevent wedging effect in extracted
Q.3. What is retention? Describe in brief various reten- site and closes spaces distal to canine.
tion appliances commonly used in orthodontic practice. ii. Soldering labial bow to the bridge of Adam’s
clasp, is another alternative in extraction cases
Ans.
to offer excellent retention and avoid risk of
l Retention has been defined by Moyer’s as ‘maintaining space opening between canine and premolar.
newly moved teeth in position, long enough to aid in iii. Incorporation of anterior bite planes helps to
stabilizing their correction’. control overbite or correct deep bite cases.
l Retainers are passive orthodontic appliances that help in l Advantages of this appliance are:

maintaining and stabilizing the position of teeth long Ease of fabrication, simple design, minimal pa-
enough to permit reorganization of the supporting struc- tient discomfort and highly acceptable by the pa-
tures after the active phase of orthodontic therapy. tients and is relatively inconspicuous.
l Ideal requirements of a good retainer are as follows:

i. It should restrain all teeth that have been moved into


b. Begg retainer:
desired positions.
l Begg retainer is made up of a single wrap
ii. It should allow the normal functional forces to act
round labial bow extending from the distal of
freely on the dentition.
second molar to the opposite second molar.
iii. It should be self-cleansable and should permit oral
l Advantages:
hygiene maintenance.
i. The wrap around wire eliminates the poten-
iv. It should be as inconspicuous as possible.
tial occlusal interferences and allows verti-
v. It should be strong enough to achieve the objectives
cal settling of occlusion
of retention.
ii. In extraction cases it maintains canine and
second premolar in tight contact, thereby
{SN Q.4} eliminating the risk of space opening up.
l Retainers can be classified into: c. Clip-on retainer/spring aligner:
l This is widely used in lower anterior region in

Removable retainers Fixed retainers correcting minor rotations of anterior teeth.


(These are passive appli- (These retainers are fixed or l This appliance is made up of a wire frame

ances that can be removed fitted onto teeth and can- work which runs labially as well lingually over
by the patient and rein- not be removal and rein- incisors passing between canine and premolar.
serted at will.) serted by the patient.) Both labial as well as lingual wire segments
a. Hawley’s appliance a. Banded canine-to-canine are embedded in a strip of clear acrylic.
b. Begg retainer retainer l Advantage:
c. Clip-on retainer/spring b. Bonded canine-to-canine
It is well tolerated by the patients and can be
aligner lingual retainers
d. Wrap around retainer c. Band and spur retainer used to realign lower incisors.
e. Kesling tooth positioner d. Antirotation band d. Wrap around retainer:
f. Essix retainer/invisible re- l This is an extended version of spring aligner
tainers that covers all the teeth.
g. Functional appliances
l It consists of a wire reinforced plastic bar made

with clear acrylic along the labial and lingual


Removable retainers: surfaces of the teeth.
l This is not routinely used in orthodontic

{SN Q.2} practice.


l This retainer splints the teeth together firmly
a. Hawley’s appliance: and is usually used in stabilizing the periodon-
l It was designed by Charles Hawley in 1920 and is
tally weak dentition.
most commonly used retentive appliance. e. Kesling tooth positioner:
l Classic Hawley’s appliance consists of clasps on
l A tooth positioner devised by Kesling is usu-
molar teeth and a short labial bow which spans ally used as a finishing appliance, sometimes
from canine to canine with adjustment loops. itself can be used as a retaining appliance.
266 Quick Review Series for BDS 4th Year, vol 1

l Advantage: It maintains intra arch tooth posi- Q.4. Define retention and relapse in orthodontics. What are
tion and also occlusal relationships. the causes of relapse and how would you prevent them?
l Disadvantage: Bulky, has different pattern of
Ans.
wear compared to retention appliance, has ten-
dency to deepen the bite, speech difficulties [Same as LE Q.1]
and risk of TMJ problems.
Q.5. Discuss the role of periodontium and growth in
f. Invisible retainers:
causing relapse. Write in brief four other causes of
l The invisible retainers are standard Essix
relapse?
canine-to-canine retainers made of ultrathin
clear thermoplastic sheets. Ans.
l Advantage:
[Same as LE Q.1]
They are aesthetically acceptable and in ex-
traction cases they are extended to cover the Q.6. Define retention and discuss various theories of
extraction site. retention.
g. Functional appliances:
Ans.
l They are used in the subjects who have still

growth potential. For example, activators and [Same as LE Q.2]


oral screen.
Q.7. Write the theories of retention and types of
Fixed retainers:
retention.
They are used in conditions where long-term reten-
tion is required and intra-arch instability is antici- Ans.
pated.
[Same as LE Q.2]
Various fixed retainers are as follows:
a. Banded canine-to-canine retainer: Q.8. Classify retention appliances and describe in detail
l Commonly used in lower anterior region, the Hawley retainer.
for maintenance of lower incisor position
Ans.
during growth.
l The canines/premolars are banded and a [Same as LE Q.3]
thick wire is contoured over the lingual as-
Q.9. Explain in detail retention after treatment of mal-
pect and soldered to the bands.
occlusion.
l Disadvantages:

Predisposition to poor oral hygiene and are Ans.


unaesthetic
[Same as LE Q.3]
b. Bonded canine-to-canine lingual retainers:
l Various prefabricated lingual retainers are

available that can be bonded directly on SHORT ESSAYS:


lingual aspect of canines.
l They are made from heavier wire to resist. Q.1. Retention in orthodontics.
c. Band and spur retainer: Ans.
l It is used in cases of orthodontic correction

of single-tooth rotation or labiolingual dis- {SN Q.3, Q.5 and Q.13}


placement.
l Retention has been defined by Moyer’s as ‘maintaining
l It holds the tooth in its corrected position
newly moved teeth in position, long enough to aid in
and prevents it from returning to its original
stabilizing their correction’.
position.
l Retention can be of three types
d. Antirotation band:
i. Natural or no retention
l It is used to maintain corrected single-tooth
ii. Limited or short-term retention
rotation.
iii. Prolonged or permanent retention
l The band on the rotated tooth has two spurs

welded one each on labial and lingual sides, i. Natural or no retention:


so that they rest on adjacent teeth and pre- Some conditions that do not require any retention
vents relapse. include
Section | I  Topic-Wise Solved Questions of Previous Years 267

i ii. Band and spur retainer


{SN Q.11}
iv. Antirotation band
a. Anterior crossbite
i. Banded canine-to-canine retainer
b. Serial extraction procedures
l Commonly used in lower anterior region for
c. Blocked out/highly placed canines in class I extrac-
maintenance of lower incisor position during
tion cases
growth.
Posterior crossbite in patients having steep cusps
l The canines/premolars are banded and a thick

wire is contoured over the lingual aspect and


ii. Limited or short-term retention: soldered to the bands.
Most cases treated routinely in the orthodontic ii. Bonded canine-to-canine lingual retainers: Various
clinic fall into this category. Retention is recom- prefabricated lingual retainers are available that can
mended to allow the bone and other periodontal be bonded directly on lingual aspect of canines.
tissues to readapt to their new location. iii. Band and spur retainer: It is used in cases of orth-
For example: class I nonextraction with dental odontic correction of single-tooth rotation or labio-
arches lingual displacement.
a. Showing proclination and spacing iv. Antirotation band
b. Deep bites l It is used to maintain corrected single-tooth

c. Class I, class II division 1 and division 2 rotation.


cases treated by extraction. l The band on the rotated tooth has two spurs

iii. Prolonged or permanent retention: welded one on each on labial and lingual sides,
so that they rest on adjacent teeth and prevent
relapse.
{SN Q.3}
Q.4. Theories of retention.
Cases that require prolonged or indefinite retention
include Ans.
For example: [Ref LE Q.2]
a. Midline diastema
b. Severe rotations Q.5. Define retention. Explain the schools of retention.
c. Arch expansion achieved without ensuring Ans.
good occlusion
c. Certain class II division 2 deep bite cases Retention has been defined by Moyer as ‘maintaining
d. Patients exhibiting abnormal musculature newly moved teeth in position, long enough to aid in stabi-
or tongue habits lizing their correction’.
e. Expanded arches in cleft palate patients There are four schools of thought related to retention in
orthodontics:
i. Kingsley’s occlusion school of thought: According to
Q.2. What is relapse? What are the causes of relapse? this, proper occlusion of teeth, i.e. a good cusp to fossa
relationship between maxillary and mandibular teeth is
Ans.
a potent factor in maintaining stability of achieved orth-
[Ref LE Q.1] odontic results.
ii. Axel Lundstrom apical base school of thought: Apical
Q.3. Fixed retainer.
base is one of the most important factors in orthodontic
Ans. correction as well as its post-treatment maintenance.
iii. Mandibular incisor school of thought by Grieve and
l Retainers are passive orthodontic appliances that help in
Tweed: According to this, mandibular incisors should
maintaining and stabilizing the position of teeth long
be placed upright and over the basal bone.
enough to permit reorganization of the supporting struc-
iv. Roger’s musculature school of thought: According to
tures after the active phase of orthodontic therapy.
this, to achieve stable occlusion and prevent relapse,
l Retainers can be classified as: (i) removable and (ii)
it is essential to establish proper functional muscle
fixed retainers.
balance.
l Fixed retainers are fixed or fitted onto teeth, and cannot

be removal and reinserted by the patient. Q.6. Permanent retention.


l Various fixed retainers are as follows:
Ans.
i. Banded canine-to-canine retainer
ii. Bonded canine-to-canine lingual retainers [Same as SE Q.1]
268 Quick Review Series for BDS 4th Year, vol 1

Q.7. Relapse in orthodontics. l Role of third molars


i. Most patients would have completed their orthodon-
Ans.
tic treatment by the time third molars start erupting,
[Same as SE Q.2] i.e. around 18–21 years of age.
ii. The pressure exerted by erupting third molars is be-
Q.8. Define relapse. Add a note on the role of third mo-
lieved to cause late anterior crowding, predisposing
lars in causing relapse.
to relapse.
Ans.
Q.8. Adjunctive periodontal surgeries to minimize
[Same as SE Q.2] relapse.
Ans.
SHORT NOTES:
Certain minor surgical procedures performed as adjunctive
Q.1. Relapse in orthodontics. periodontal surgeries to minimize relapse following orth-
Ans. odontic treatment are
i. Frenectomy: It is a surgical procedure to remove entire
[Ref LE Q.1] frenum, sometimes even along with fibrous tissue pres-
Q.2. Hawley’s retainer. ent between the roots of central incisors.
ii. Pericision: It is also known as ‘supracrestal fibrotomy’,
Ans. involves severing the fibres connecting tooth to gingival
[Ref LE Q.3] soft tissues by passing a no.15 BP blade around circum-
ference of the tooth.
Q.3. Permanent retention. iii. Papilla-dividing procedure: This procedure is usually
Ans. undertaken a few weeks before removal of active orth-
odontic appliance.
[Ref SE Q.1]
It is nothing but making a vertical incision in the centre
Q.4. Retention appliance. of each gingival papilla 1–2 mm away from the gingival
Ans. margin.
[Ref LE Q.3] Q.9. Riedel’s theorems of retention and relapse.
Q.5. ‘Relapse’ and ‘retention’. Ans.
Ans. [Ref LE Q.2]
[Ref SE Q.1] Q.10. Define retention period.
Q.6. Name different schools of thought for retention. Ans.
Ans. Retention period can be defined as time required after orth-
odontic therapy for holding teeth in optimal aesthetic and
l Retention has been defined by Moyer as ‘maintaining
functional position for long enough to aid in their stabiliza-
newly moved teeth in position, long enough to aid in
tion with the help of retention appliances.
stabilizing their correction’.
l There are four schools of thought related to retention in Q.11. Give an example of natural or self-retention.
orthodontics:
Ans.
i. Occlusion school of thought by Kingsley
ii. Apical base school of thought by Axel Lundstrom [Ref SE Q.1]
iii. Mandibular incisor school of thought by Grieve and
Q.12. What is relapse? How to prevent relapse?
Tweed
iv. Musculature school of thought by Roger Ans.
Q.7. What is the role of third molar in causing the relapse? l Relapse can be defined as ‘the loss of any correction
achieved by orthodontic treatment’. In simple terms, it
Ans.
is the tendency of teeth to return back to original posi-
l Relapse can be defined as ‘the loss of any correction tion post-treatment.
achieved by orthodontic treatment’. In simple terms, it l Relapse can be prevented by:

is the tendency of teeth to return back to original posi- i. Achieving proper occlusal stability, e.g. corrected
tion post-treatment. crossbite cases
Section | I  Topic-Wise Solved Questions of Previous Years 269

ii. Using retention appliances – either fixed or Q.15. Retainers.


removable
Ans.
iii. Adjunctive procedures like frenectomy, pericision
or papilla-dividing procedures. [Same as SN Q.4]
Q.13. Define and classify retention giving examples. Q.16. Define retention.
Ans. Ans.
[Ref SE Q.1] [Same as SN Q.5]
Q.14. Upper Hawley’s appliance. Q.17. Types of retention.
Ans. Ans.
[Same as SN Q.2] [Same as SN Q.13]

Topic 33
Genetics in Orthodontics
COMMONLY ASKED QUESTIONS
SHORT ESSAYS:
1 . Importance of genetics in orthodontics.
2. Methods of genetic studies.
3. Twin studies. [Same as SE Q.2]

SHORT NOTES:
1 . Genetics in orthodontics. [Ref SE Q.1]
2. Teratogens.
3. Dentofacial disturbances of genetic origin. [Ref SE Q.1]
4. Mutation.
5. Chromosomes.
6. Pedigree studies. [Ref SE Q.2]
7. Role of genetics in aetiology of malocclusion. [Same as SN Q.1]
8. Importance of genetics in malocclusion. [Same as SN Q.1]
9. Genetic disorders. [Same as SN Q.3]

SOLVED ANSWERS
SHORT ESSAYS:
Q.1. Importance of genetics in orthodontics.
{SN Q.3}
Ans.
l Genetic disorders seen at the time of birth are called
congenital defects. Heredity plays a role in the fol-
{SN Q.1} lowing conditions:
l Genetics plays a major role in aetiology of malocclusion. i. Congenital deformities
l Genetic disorders are caused due to disturbance in ii. Facial asymmetry
germ plasm or chromosomes or genes. They are clas- iii. Mandibular prognathism and retrognathism
sified as: iv. Macrognathia and micrognathia
i. Hereditary v. Deep bite
ii. Mutational vi. Macrodontia and microdontia
270 Quick Review Series for BDS 4th Year, vol 1

vi. Disadvantages of twin study are


vii. Anodontia, oligodontia and hypodontia
l Difficulty in identifying the identical twins
viii. Cleft lip and palate
l Difficult to establish same environment for both
ix. Variations of tooth shape
the twins
x. Abnormalfrenal attachments (resulting in diastema)

{SN Q.6}
l Hereditary disorders are transmitted from one genera-
tion to another B. Family study or pedigree study:
l The modes of inheritance of genetic disorders are as i. Pedigree study is a type of genetic study. It is
follows: also known as family study where occlusal fea-
a. Autosomal dominant tures and differences between mother–child, fa-
b. Autosomal recessive ther–child and siblings are analysed.
c. X-linked ii. It helps to differentiate between dominant and
d. Chromosomal recessive traits.
e. Polygenic iii. Dominant traits will be expressed in all the sub-
l Mutational disorders occur de novo in a previously unaf- sequent generations, while recessive traits will
fected individual as result of damage to germ plasm. If they be expressed in children born of consanguineous
get transmitted to next generation it becomes hereditary. marriage.
l The two major possible ways in which malocclusion

could be produced by heredity are as follows: Q.3. Twin studies.


Inherited disproportion between:
i. Size of teeth and jaws Ans.
ii. Size or shape of upper and lower jaws [Same as SE Q.2]
l Examples of some of the malocclusions caused due to

heredity or genetic cause are as follows:


SHORT NOTES:
Q.1. Genetics in orthodontics.
{SN Q.1}
Ans.
i. Dental problems:
a. Crowding – Hereditary and environmental reasons [Ref SE Q.1]
b. Individual tooth malalignments and crossbites –
Q.2. Teratogens.
Pressure environment
ii. Skeletal problems: Ans.
a. Retrognathic mandible and retrognathic maxilla
l The various chemicals or agents which crosses the pla-
(achondroplasia)
cental barrier and produce embryologic defects are
b. Prognathic mandible (Hapsburg jaw)
called teratogens.
c. Skeletal deep bite
l The various teratogens and their effects are as follows:

For example:
Q.2. Methods of genetic studies.
Name of the teratogens Effect seen
Ans.
i. Aspirin, valium, Cleft lip and palate and cigarette
Genetic studies are basically of two types: dilantin smoke
A. Twin study ii. Aminopterin Anencephaly
B. Family study or pedigree study
iii. X-ray radiation Microcephaly
A. Twin study: iv. Toxoplasma and Microcephaly and hydrocephaly
i. Twins are compared in this study. cytomegalovirus
ii. The heritability of malocclusion can be determined v. Thalidomide Hemifacial microsomia-like features
by comparing the monozygotic twins, dizygotic
twins and normal siblings. vi. Ethyl alcohol Central midface deficiency
iii. The best way to determine the genetic effect on
malocclusion is to compare monozygotic (identical) Q.3. Dentofacial disturbances of genetic origin.
twins with dizygotic (fraternal) twins. Ans.
iv. In monozygotic twins as both have same DNA any
change in occlusion or features could be attributed [Ref SE Q.1]
to environmental factors. Q.4. Mutation.
v. In dizygotic (fraternal) twins, interplay of genetic
and environmental factors is studied. Ans.
Section | I  Topic-Wise Solved Questions of Previous Years 271

l The term mutation refers to permanent change in the ii. Each chromosome has a very large number of struc-
DNA. The genetic disorders arising from chromosomal tures called genes on it, which guide the performance of
aberrations includes disorders that are consequence of particular cellular functions, and in turn lead to the de-
numeric or structural abnormalities in the chromosomes. velopment of particular features of a species or an indi-
l Some of the general features of chromosomal disorders vidual.
and some specific examples of diseases involving iii. Chromosomes control the development and functioning
changes in the karyotype are as follows: of cells, by determining what types of proteins will be
i. Chromosomal disorders resulting from mutations synthesized in them.
may be associated with absence, i.e. deletion or iv. Chromosomal aberrations result in various syn-
monosomy, excess, i.e. trisomy or abnormal rear- dromes which exhibit different types of malocclu-
rangements, i.e. translocation of chromosomes. sion as follows:
ii. In general the loss of chromosomal material pro- For example:
duces more severe defects than does the gain of l Class II malocclusion is seen in Pierre Robin

chromosomal material. syndrome, mandibulofacial dysostosis, Golden-


iii. The excess chromosomal material may result from har syndrome, etc.
a complete chromosome as in trisomy or from part l Class III malocclusion is observed in Down

of a chromosome as in Robertsonian translocation. syndrome, Marfan syndrome, Gorlin syn-


iv. Most of the times chromosomal disorders result drome, etc.
from de novo changes. An uncommon but important
Q.6. Pedigree studies.
exception to this principle is exhibited by the trans-
location form of Down syndrome. Ans.
Q.5. Chromosomes. [Ref SE Q.2]
Ans. Q.7. Role of genetics in aetiology of malocclusion.
i. A typical chromosome is made up of two rod-shaped Ans.
structures or chromatids placed more or less parallel to
[Same as SN Q.1]
each other and are united by a centromere or kinetochore.
Q.8. Importance of genetics in malocclusion.
Satellite Ans.
Secondary
Short arm of constriction [Same as SN Q.1]
chromatid
Q.9. Genetic disorders.
Ans.
Centromere [Same as SN Q.3]

Long arm of
chromatid

Typical chromosome
272 Quick Review Series for BDS 4th Year, vol 1

Topic 34
Lab Procedures
COMMONLY ASKED QUESTIONS
SHORT ESSAYS:
1. Soldering and welding.
2. Solder and flux.
3. Soldering and spot welding. [Same as SE Q.1]

SHORT NOTES:
1. Soldering. [Ref SE Q.1]
2. Spot welding. [Ref SE Q.1]
3. Flux and antiflux.
4. Dental spot welder.
5. Heat treatment of orthodontic wire.
6. Sensitization and stabilization.
7. Soldering and welding. [Same as SN Q.1]
8. Welding in orthodontics. [Same as SN Q.1]
9. Soldering welding and brazing. [Same as SN Q.1]
10. Antiflux. [Same as SN Q.3]

SOLVED ANSWERS
SHORT ESSAYS:
Q.1. Soldering and welding. a. Investment Embedding the metallic
soldering parts in an invest leaving
Ans.
a gap of 0.13 mm between
Two metal ends.
{SN Q.1} methods

Soldering is defined as a process of joining metals by the b. Free-hand Used when the area of
use of a filler metal which has substantially lower fusion soldering contact between metallic
temperature than that of the metals being joined. parts to be joined
is large.
If fusion temperature of filler metal is
Most of the orthodontic
,450°C Procedure is known as Soldering soldering procedures fall
.450°C Procedure is known as Brazing in this category.

Solders are the alloys that are used as intermediary or a Involves soldering of two
filler metal to join two or more metallic parts. They are metallic parts together after
composed of gold, silver, copper, zinc, tin and nickel. adequate stabilization without
use of investment to precisely
hold the parts together.
{SN Q.1}
Applications of soldering in orthodontics: Practical considerations in soldering:
Used to joins parts of orthodontic appliances i. Use the reducing flame of the soldering torch
Used to fasten attachments to bands ii. Use wet cotton and asbestos to limit the spread of
Types of soldering: heat
Section | I  Topic-Wise Solved Questions of Previous Years 273

iii. Soldered joint should not be polished as it weakens


employ the electrode technique and are used instead of
the joint.
soldering in cases where the heating cycle must be very
iv. Antiflux should be used to prevent excessive spread
short, in order to prevent changes in the physical proper-
of soldering.
ties of the components being joined.
Steps in soldering:
Application of welding in orthodontics:
i. Cleaning the surfaces to be joined
Joining of metal strips during bending.
ii. Assembling the parts to be joined
Fixing attachments such as brackets and molar tubes
iii. Selecting the right solder and flux
onto bands.
iv. Selection of proper joint
Principle of spot welding:
v. Application of flux
i. Heat and pressure are the two basic principles in-
vi. Heating and introduction of solder
volved in spot welding.
vii. Quenching
ii. Electric current (AC) is made to pass through a
i. Cleaning the surfaces to be joined: In order to re- stepdown transformer to obtain a low voltage and
move dirt and other surface contaminants which high amperage current that is conducted through
result in poor solder flow and therefore failure of two copper electrodes on either side of the metals
the procedure. being joined.
ii. Assembling the parts to be joined: The part to be
joined is stabilization in desired fashion using plaster
iii. The resistance offered by stainless steel to current
or orthophosphate cement. A gap of about 0.5 mm is
of high amperage generates very high temperature
considered adequate between parts to be joined.
at the electrodes. Thus the area of metal under the
iii. Selection of right solders and flux: Based on metal-
electrodes becomes plastic.
lic parts to be joined solder should be compatible
iv. The copper electrodes simultaneously apply pres-
with metals in aspects of strength and colour and
sure on the metals and therefore squeeze the metals
they should exhibit lower fusion temperature. A
into each other.
good flux is also selected.
v. It is very important that the passage of current at the
iv. Selection of proper joint: Between metals being
weld spot be of very short duration, i.e. not more
joined greatly enhances the strength of the joint.
than one-tenth of a second.
Point of contact – Do not offer adequate strength
In case the current is passed for a long duration of
when two wires are being joined together. It would
time, it results in weld decay due to precipitation of
be beneficial to wrap one of the wires around the
carbides from the metal. Thus most of the modern
other.
welders have an electronic timer that helps in dis-
v. Application of flux: The flux applied in the gap
charging current of very short duration.
between the parts also covers a portion of the parts
vi. Welding of stainless steel depends on proper use of
being joined. The flow of the solder can be limited
following three variables:
by using antiflux.
a. The current flowing through the circuit.
vi. Heating and introduction of solder: The area to be
b. Time during which the current is allowed of
joined is heated using a soldering torch. As soon as
flow.
the flux begins to fuse, the solder is introduced. The
c. The mechanical press is applied at the weld
solder melts and encases the joint. The flame
heads.
should be maintained until the filler metal has
Procedure of spot welding:
flowed completely into the joint.
i. Select proper electrode in the thickness or shape
vii. Quenching: The assembly is immediately quenched
of the material to be welded.
in water to limit the spread of heat.
A broad electrode should be used for thin material.
Welding
A narrow electrode should be used for thick material.
ii. The electrodes of welder are cleaned so as to
{SN Q.2} remove any carbide precipitates.
iii. The surface of each electrode must be smooth,
Welding involves the joining to two or more metal flat and perpendicular to its long axis.
pieces directly under pressure without the introduction iv. When the electrodes are together, they should be
of an intermediary or filter material. in total contact.
a. Cold welding n Done by hammering or pressure v. The welder has a timer that is set to the required
b. Hot welding n Uses heat of sufficient intensity to reading. The metals to be joined are placed be-
melt the metals being joined tween the two electrodes and the switch is turned
The type of welding used to join orthodontic compo- on. The electrode pressure can be maintained for
nents is called spot welding. Orthodontic spot welders a few seconds to help obtain a good joint.
274 Quick Review Series for BDS 4th Year, vol 1

Q.2. Solder and flux. Q.3. Flux and antiflux.


Ans. Ans.
Solder Flux:
l Solders are the filler metals used in process of solder- l Flux is a Latin word meaning ‘flow’.
ing; they have substantially lower fusion temperature l Flux is defined as compound applied to metal sur-
than the metal parts being joined. faces that dissolves or prevents the formation of
l Orthodontic silver solders are alloys of silver, copper oxides and other undesirable substances that may
and zinc to which tin and indium are added to lower reduce the quality or strength of a soldered or
the fusion temperature. brazed area.
l Properties of a solder are as follows: l Functions of a flux:
i. It should have ability to wet the substrate metal. i. To remove any oxide coating on the parent metal
ii. It should have sufficient fluidity at the flow tem- and increase the flow of molten solder.
peratures. ii. To protect the metal surface from oxidation dur-
iii. Solder alloy should have a small liquidus–solidus ing soldering procedures.
range, which means that it should harden in- iii. Significantly decreases melting point of the dental
stantly. solder.
iv. It should have adequate strength and hardness l Composition of flux used in dentistry:
and as well good resistance to tarnish and cor- Borax glass – 55%
rosion. Boric acid – 35%
v. Should also have colour compatibility. Silica – 10%
Flux Fluoride fluxes containing boric acid and potassium
l ‘Flux’ is a Latin word which means ‘flow’. fluoride in a 1:1 ratio also produce excellent soldered
l Flux is defined as compound applied to metal surfaces joints.
that dissolves or prevents the formation of oxides and Antiflux:
other undesirable substances that may reduce the qual- l Antiflux is a material that is used to confine the
ity or strength of a soldered or brazed area. flow of the molten solder over the metals being
l Composition: joined.
Flux is made up of borax, boric acid and potassium l The commonly used antifluxes are
fluoride. a. Lead pencil markings
l Types: b. Graphite lines
i. Type 1 – Surface protection: c. Iron rouge
Coats the metal surface and prevents entry of
oxygen Q.4. Dental spot welder.
ii. Type 2 – Reducing agent: Ans.
Reduces any oxide present and exposes clean
metal surface l Dental spot welder is used to perform spot welding of
iii. Type 3 – Solvent: bands, attachments to bands and fine springs to heavy
Dissolves oxides present on the surface and car- wire in orthodontic appliances.
ries them away l It welds by a process of fusing two or more metal parts

Used for orthodontic purpose through the application of heat, pressure or both without
using a filler metal.
Q.3. Soldering and spot welding. l The parts of a welder are electric transformer, copper

Ans. electrodes, pressure mechanism and timer switch.


l The electric transformer reduces voltage of the main
[Same as SE Q.1] supply to a low value, which is safe to handle and the
copper electrodes convey the current to the work pieces.
SHORT NOTES: l Main heating takes place between the work pieces

which soften and get welded together by the pressure of


Q.1. Soldering. the electrodes.
Ans. Q.5. Heat treatment of orthodontic wire.
[Ref SE Q.1]
Ans.
Q.2. Spot welding.
l Heat treatment or annealing is a controlled heating and
Ans. cooling process designed to produce desired properties
[Ref SE Q.1] in a metal.
Section | I  Topic-Wise Solved Questions of Previous Years 275

l The unwanted effects of cold working a cast metal like Stabilization:


strain hardening, reduced ductility and distorted grains l Introduction of some other elements like titanium

can be eliminated by annealing. The annealing process which prevents chromium carbide formation at the
includes simply heating the metal to a temperature that grain boundaries of stainless steel is known as stabi-
is approximately half the melting point of the metal or lization.
fusion temperature of the alloy. l Addition of titanium usually six times that of carbon

l Annealing takes place in three successive stages: Re- is the most successful method employed to eliminate
covery, recrystallization and grain growth. chromium carbide precipitation. Stainless steel that
l Clinical applications of annealing: Altering the proper- is modified in this manner is said to be ‘stabilized’.
ties of wires; softening a metal (e.g. steel); and increas-
Q.7. Soldering and welding.
ing their plastic deformation potential to stabilize shape
and to increase machinability. Ans.
Q.6. Sensitization and stabilization. [Same as SN Q.1]

Ans. Q.8. Welding in orthodontics.


Ans.
Sensitization:
l Heating of stainless steel between 400°C and 900°C [Same as SN Q.1]
leads to reaction between chromium and carbon
Q.9. Soldering welding and brazing.
resulting in the formation of chromium carbide at the
grain boundaries. Ans.
l Depletion of chromium content near grain boundar-
[Same as SN Q.1]
ies to less than 12% causes stainless steel to become
susceptible to corrosion. Q.10. Antiflux.
l This effect of losing resistance to corrosion by form-
Ans.
ing chromium carbide at the grain boundaries is
called ‘sensitization’. [Same as SN Q.3]

Topic 35
Materials Used in Orthodontics
COMMONLY ASKED QUESTIONS
SHORT ESSAYS:
1 . Properties of an ideal orthodontic wire.
2. 18-8 stainless steel.
3. Direct bonding.
4. Elgiloy wires.

SHORT NOTES:
1 . Irreversible hydrocolloids.
2. Properties of ideal orthodontic wires. [Same as SE Q.1]
3. Stainless steel in orthodontic practice. [Same as SE Q.2]
4. Name various light sources used in curing composite.
5. Direct bonding. [Ref SE Q.3]
6. Shape memory alloys.
7. Glass ionomer cements.
8. Uses of NiTi alloys in orthodontics.
9. Braided and twisted wires.
276 Quick Review Series for BDS 4th Year, vol 1

1 0. Super elasticity.
11. Wires used in orthodontics.
12. Elgiloy wires. [Ref SE Q.4]
13. Molar tubes.
14. Tensile strength.
15. Cold-cure acrylic resins.
16. Composition of wrought cobalt–chromium–nickel alloys.
17. Nitinol wires/NiTi wires.
18. Stainless steel. [Same as SN Q.3]
19. Nickel–titanium wires. [Same as SN Q.17]
20. Nickel–titanium alloy. [Same as SN Q.17]

SOLVED ANSWERS
SHORT ESSAYS:
Q.1. Properties of an ideal orthodontic wire.
carbon, while type 304 has similar content but car-
Ans. bon content is 0.08%.

{SN Q.2} Q.3. Direct bonding.

Desirable properties or ideal requirements of orthodon- Ans.


tic wire are as follows:
i. The wire should deliver low constant force. {SN Q.5}
ii. It should have high strength and range.
iii. It should have low stiffness or good spring back. i. The direct bonding is a technique, sensitive proce-
iv. It should be easy to manipulate. dure, where brackets can be directly attached on to
v. It should exhibit ease of joining, i.e. solderable and the teeth. The development of resins by Newman in
weldable. 1960s replaced banding with bonding.
vi. It should offer less frictional resistance between ii. The introduction of acid-etching technique and com-
wire and bracket base. posites has revolutionized direct bonding of brackets.
vii. It should be biocompatible.
viii. It should be stable in the oral environment, i.e. Two types of bonding materials like self-cure and light-
resistant to tarnish and corrosion. cure bonding materials are available.
ix. It should be economical. iii. The most important requirements of orthodontic bond-
ing agents are that it should be biologically safe in oral
cavity and should generate minimum bond strength of
Q.2. 18-8 stainless steel. 7–15 MPa.
Currently available bonding agents offer bond strength
Ans. of 12–20 MPa which exceeds the minimum required
bond strength.
{SN Q.3}
{SN Q.5}
l Steel is an alloy of iron containing less than 1.2%
carbon. The alloy is referred stainless steel when the iv. Successful direct bonding includes:
chromium content of the steel exceeds 11%. l Optimum etching

l Based on crystal structure arrangement, stainless steel is l Maintenance of moisture-free environment of

of three types, namely ferritic, martensitic and austenitic. bonding site


l Austenitic type of stainless steel is most corrosion l A thin and uniform layer of primer

resistant and is widely used in dentistry. l Optimum quantity of bonding agent on brackets

l Type 302 and 304 austenitic types of stainless steel l Correct placement of brackets,

are called 18-8 stainless steel. l A slight but firm, vibration-free pressure on

Type 302 austenite is the basic alloy containing bracket to ensure good flow of bond material into
17%–20% chromium, 8%–12% nickel and 0.15% enamel micropores and bracket base mesh
Section | I  Topic-Wise Solved Questions of Previous Years 277

Q.4. Elgiloy wires. l Composition of alginate is as follows:


i. Sodium or potassium or triethanolamine alginate
Ans.
– 15%
ii. Calcium sulphate (reactor) – 16%
{SN Q.12) iii. Zinc oxide – 4%
iv. Diatomaceous earth – 60%
l Elgiloy, i.e. cobalt–chromium–nickel alloy was de-
v. Potassium titanium fluoride – 3%
veloped during 1950s by Eligin Watch Company
vi. Sodium phosphate (retarder) – 2%
(USA). This belongs to group of alloys called satel-
vii. Colouring and flavouring agents – traces
lite alloys.
l When alginate powder is mixed with water a sol is

{SN Q.16} formed which later sets to a gel by a chemical reaction.


l The calcium sulphate reacts with sodium alginate to form
l The typical composition of Co–Cr alloy is insoluble calcium alginate which forms a gel with water.
Cobalt – 40% l Applications of alginate hydrocolloid are as follows:
Chromium – 20% i. It is used for impression making when there are un-
Nickel – 15% dercuts and also in mouths with excessive salivation
Iron – 15.8% ii. For making preliminary impressions for complete
Molybdenum – 7% dentures.
Manganese – 2% iii. For impressions to make study models and working
Carbon – 0.16% casts.
Beryllium – 0.04% iv. For duplicating models.
Q.2. Properties of ideal orthodontic wires.
l Co–Cr alloys are available commercially as Elgiloy (Rocky
Mountain Orthodontics), Azura (Ormco Corporation) and Ans.
Multiphase (American Orthodontics Corporation). [Ref SE Q.1]
Q.3. Stainless steel in orthodontic practice.
{SN Q.12}
Ans.
l Elgiloy wires by Rocky Mountain Orthodontics are
supplied in four tempers (level of resilience) which [Ref SE Q.2]
are colour coded. Q.4. Name various light sources used in curing composite.
a. Blue (soft)
Ans.
b. Yellow (ductile)
c. Green (semiresilient) Various light sources used in curing composites are as follows:
d. Red (resilient) i. Conventional and fast halogen lights
l Co–Cr–Ni alloys have: ii. Argon lasers
a. Best formability among all the wires. iii. Plasma arc lights (xenon)
b. Greater resistance to fatigue and distortion than SS. iv. Light-emitting diodes
c. High yield strength on heat treatment.
Q.5. Direct bonding.
d. Good biocompatibility and high corrosion resis-
tance. Ans.
e. Good join ability and can be easily soldered or
[Ref SE Q.5]
welded.
f. Low coefficient of friction like SS. Q.6. Shape memory alloys.
g. Stiffness is a bit higher than SS after heat
Ans.
treatment.
h. Cost is slightly higher than SS. i. Shape memory was defined by Andreasen as ‘Ability of
wire to return to a previously manufactured shape when it
is heated through a transition temperature range (TTR)’.
SHORT NOTES: ii. Shape memory is also known as ‘thermoelasticity’ and
Q.1. Irreversible hydrocolloids. is one of the distinctive properties of Nitinol, where in
the material has ability to remember its ‘original shape’.
Ans. iii. It is because of temperature induced crystallo-
l Irreversible hydrocolloid or alginate is available as a graphic transformation. It is associated with revers-
powder. ible martensitic–austenitic transformation.
278 Quick Review Series for BDS 4th Year, vol 1

Q.7. Glass ionomer cements. v. Many researchers have found that the properties of
these wires are comparable with nickel–titanium
Ans.
wires.
l Glass ionomer cements (GIC) are adhesive tooth-
Q.10. Super elasticity.
coloured anticariogenic restorative materials and is one
of the cements widely used in orthodontics for cementa- Ans.
tion of orthodontic bands.
l Super elasticity is one of the distinct properties of
l Composition:
Nitinol.
Powder: an acid-soluble calcium fluoroaluminosilicate glass.
l It is the ability of wire to sustain or deliver a near con-
Liquid: in most current cements, the liquid contains
stant force over a wide range of activation. Instead of
polyacrylic acid, tartaric acid and water.
temperature, stress is used to bring about changes in
l It is available in four forms:
crystalline structure, i.e. from austenite to martensite
a. Conventional luting glass ionomer
and back to austenite.
b. Light-cured resin modified GIC
l The elasticity of the wire increases during activation;
c. Chemically cured resin modified GIC
this is called super elasticity, which is associate with
d. Glass ionomer for orthodontics
very large reversible strain and nonelastic force deflec-
l Glass ionomer–resin combination materials are relatively
tion curves.
new materials having various names like compomer,
For example: Chinese NiTi and Japanese NiTi
resin–ionomers, RMGI, light-cured GIC, dual-cure GIC
and tri-cure GIC. Q.11. Wires used in orthodontics.
They are preferable as adhesives for orthodontic brackets. Ans.
Q.8. Uses of NiTi alloys in orthodontics. Various wires used in orthodontics are classified based on
composition, number of filaments, cross section and diam-
Ans.
eter as follows:
l Nitinol was introduced into orthodontics by G. Andreasen Based on:
and William F. Buehler.
l NiTi is commercially available as Chinese NiTi or Japanese i. Composition
NiTi.
l Clinical uses of NiTi alloy in orthodontics are as

follows: Metallic Nonmetallic


They are used as For example: For example:
i. Initial alignment and levelling archwires Stainless steel, Elgiloy, Composites and
ii. Retraction coil springs β–Ti and nickel–titanium optiflex
iii. Palatal expanders
iv. Devices for distalization of molars ii. Number of filaments

Q.9. Braided and twisted wires.


Ans. Monofilament Polyfilament Twisted or
braided
i. The wires used in orthodontics may be:
iii. Cross Section
l Single stranded

l Multi-stranded
l Twisted or braided

ii. Very thin small diameter stainless steel wires can be Round Square Rectangular
twisted or braided together to form wires in clinical iv. Diameter/dimension of wire available in inches
orthodontics which may be round or rectangular in
shape.
iii. They apply low forces for a given deflection when com-
Round wires Square wires Rectangular wires
pared to similar size solid wire and are able to sustain
(0.010, 0.012, (0.016 × 0.016) (0.016 × 0.022;
large elastic deflections in bending.
0.014, 0.016, 0.017 × 0.025;
iv. Coaxial/braided wires with greater working range offer
0.018, 0.020, 0.022) 0.018 × 0.025;
a good choice of wire for initial alignment and levelling
0.019 × 0.025;
of untreated malocclusion.
0.0215 × 0.0275)
Section | I  Topic-Wise Solved Questions of Previous Years 279

Q.12. Elgiloy wires. iii. Instead of heat as in heat-cure resins, the polymeriza-
tion is achieved at room temperature.
Ans.
Q.16. Composition of wrought cobalt–chromium–nickel
[Ref SE Q.4]
alloys.
Q.13. Molar tubes. Ans.
Ans. l Cobalt–chromium–nickel alloy belongs to a group of
l Molar tubes are both bondable and weldable. Weldable alloys called satellite alloys.
tubes are more common in use. l Elgiloy wires by Rocky Mountain Orthodontics are sup-

l Buccal tubes are housed on first and second molars; plied in four tempers based on level of resilience which
they are usually 0.75 long with internal dimensions of are colour-coded.
0.022 3 0.028. [Refer SE Q.4]
l Maxillary molar tubes are often a combination of rect-

angular edge-wise and round tube called double tube. Q.17. Nitinol wires/NiTi wires.
The rectangular tube is the main tube and round tube is Ans.
for headgear. The tube has built in offset to compensate
for molar rotation. They are designed to enhance an- l Nitinol was introduced into orthodontics by G. Andreasen
chorage and produce sufficient torque for normal buc- and William F. Buehler.
colingual inclination of molars. l NiTi is commercially available as Chinese NiTi or

l The triple buccal tubes or auxiliary edge-wise tubes are Japanese NiTi.
used for two active archwires simultaneously. l Clinical uses of NiTi alloy in orthodontics are as follows:

They are used as


Q.14. Tensile strength. i. Initial alignment and levelling archwires
Ans. ii. Retraction coil springs
iii. Palatal expanders
l Tensile strength or ultimate tensile strength is tensile iv. Devices for distalization of molars.
stress at the point of fracture.
l It is the maximum load a wire can sustain.
Q.18. Stainless steel.
l This determines the maximum force a spring can Ans.
deliver.
[Same as SN Q.3]
Q.15. Cold-cure acrylic resins.
Q.19. Nickel–titanium wires.
Ans.
Ans.
i. The chemically activated acrylic resins polymerize at
[Same as SN Q.17]
room temperature. They are also known as ‘self-curing’
or cold-cure or autopolymerizing resins. Q.20. Nickel–titanium alloy.
ii. In cold-cured acrylic resins, the chemical initiator ben-
Ans.
zoyl peroxide is activated by another chemical di-
methyl-para toluidine which is present in the monomer. [Same as SN Q.17]
This page intentionally left blank
Section I

Topic-Wise Solved Questions


of Previous Years

PART II PAEDODONTICS
Topic 1 Introduction to Paedodontics 283
Topic 2 Examination, Diagnosis and Radiographic Techniques 288
Topic 3 Theories of Child Development 300
Topic 4 Parent Counselling and Child Behaviour 308
Topic 5 Behavioural Science and Psychologic Management
of Children’s Behaviour 318
Topic 6 Therapeutic Management 338
Topic 7 Management of Handicapped Children 347
Topic 8 Management of Children with Systemic Diseases
and HIV Infection 353
Topic 9 Management of Children with Cleft Lip and Palate 362
Topic 10 Growth and Development of the Face and Dental
Arches 364
Topic 11 Development and Morphology of Primary Teeth
and Occlusion 373
Topic 12 Acquired and Developmental Disturbances of the
Teeth and Associated Oral Structures 391
Topic 13 Developing Malocclusion and Its Management
and Preventive Measures 403
Topic 14 Oral Habits 427
Topic 15 Gingival and Periodontal Diseases in Children 438
Topic 16 Home Oral Hygiene for Children and Adolescents 447
Topic 17 Dental Caries in Children and Adolescents 458
Topic 18 Pit and Fissure Sealants 483
Topic 19 Atraumatic Restorative Treatment 489
Topic 20 Fluorides and Oral Habits 492
Topic 21 Paediatric Restorative Materials and Rubber
Dam Application 507
Topic 22 Restoration of Primary Carious Teeth 523
Topic 23 Paediatric Endodontics 533
Topic 24 Traumatic Injuries of Anterior Teeth and Management 547
Topic 25 Local Anaesthesia and Oral Surgery for the Child Patient 562
Topic 26 NSAIDs, Antimicrobial Drugs and Miscellaneous 568
Section I

Topic-Wise Solved Questions


of Previous Years

Part II
Paedodontics

Topic 1
Introduction to Paedodontics
COMMONLY ASKED QUESTIONS

LONG ESSAYS:
1 . Define paediatric dentistry. Explain paediatric practice management.
2. Define paediatric dentistry. What are the aims and objectives of paediatric dentistry? Mention responsibilities
of a paedodontist.

SHORT ESSAYS:
1. Scope of paedodontics.

SHORT NOTES:
1 . Define paedodontics.
2. Scope of paedodontics. [Ref SE Q.1]
3. Define paediatric dentistry. [Same as SN Q.1]

283
284 Quick Review Series for BDS 4th Year, Vol 1

SOLVED ANSWERS
LONG ESSAYS:
Q.1. Define paediatric dentistry. Explain paediatric l The assistant keeps the working area and all
practice management. the necessary equipment ready for the work.
l The appointment time can be reduced; hence,
Ans.
more number of patients can be attended.
Paediatric dentistry is defined as an age-related specialty, l Paedodontist’s income may increase.
meant for preventive and curative dental and oral health l More complicated treatments can be undertaken.
care of infants and children, up to the age of 15 years. For smooth and efficient working:
Paediatric practice management: The proper positioning of patient, assistant’s
l The main aim of paediatric practice management is de-
position and paedodontist’s position are very
veloping management skills to achieve a good practice, important.
harmonious staff, satisfied patient and good income. Position of child patient:
l Success of the dental practice depends on the office
l Child’s body should be parallel to the floor
staff, patients and the entire operational system. and legs slightly elevated.
l Some important things to be considered for a good pae-
l Paedodontist’s hand should be slightly
dodontic practice are as follows: above the patient’s chest.
i. Situation and office decoration l The instruments should be out of vision of
ii. Ancillary personnel the child to lessen apprehension to the child
iii. Health education room patient.
iv. Introductory information to patient Assistant’s position:
v. Patient history l Assistant should be seated opposite to the
vi. Preliminary examination and consultation paedodontist.
vii. Recalls l His/her level should be slightly above the

i. Situation and office decoration paedodontist’s level from the floor.


l Paedodontic clinic should be situated in an area, l Everything should be within the reach of

where economic and social condition or status the assistant without leaving the chair.
of the people living there is good. Paedodontist’s position:
l Access to the clinic from the schools and resi- l The paedodontist should be comfortably

dential area should be good. seated on an operating stool.


l The office should be decorated according to the l His/her back should be straight, he/she

fantasies of children because a child patient is should not lean on the patient. His/her feet
often fearful when he/she comes to a dental should rest on the floor and thighs should be
clinic for the first time. parallel to the floor.
l Office walls should have posters of circus, nurs- Four-handed dentistry:
ery rhymes, etc. If space permits, an aquarium l In four-handed dentistry, two hands of the

should be installed at such a place that it should paedodontist and two hands of the assistant
be visible from the reception and also from the are consistently used. All four hands must
dental chair. By such a type of decoration child’s be utilized.
fear can be reduced to a great extent. l The exchange of instruments should be very

l For parents or accompanying persons who smooth. The assistant exchanges the instru-
bring the child to a dental clinic, good reading ments as and when required, using one hand
material should be available, like different types to give and take instruments simultaneously.
of magazines and newspapers. iii. Health education room:
l Good, soft and mild music always has a sooth- l In the paediatric dental office, there should be a

ing effect on children and parents, so clinic health education room, where instructions
should have a good music system. about preventive procedures should be given
ii. Ancillary personnel and demonstrated to the parents.
l To save precious time of paedodontist, they l Proper toothbrushing technique should be demon-

should have a hygienist and an assistant or sec- strated to the patient. Education to the parents is
retary. These ancillary personnel should work necessary so that they can guide their child at home.
under the direction of paedodontist. l Children and parents can be educated with the help

The importance of these ancillary personnel is as of slides, posters, models, films, pamphlets, etc.
follows: l Children should be educated on how to brush

l Paedodontist can work more rapidly and effi- and floss in front of a mirror so that they can
ciently. learn it easily.
Section | I  Topic-Wise Solved Questions of Previous Years 285

iv. Introductory information Q.2. Define paediatric dentistry. What are the aims and
l First visit of a child patient to a dental clinic objectives of paediatric dentistry? Mention responsibili-
should be with his/her parents because a child ties of a paedodontist.
cannot give all the necessary information and
Ans.
cannot fully understand what the paedodontist
or his/her staff tell him/her. l Paediatric dentistry is defined as an age-related spe-
l On subsequent visits for minor work, a child cialty, meant for preventive and curative dental and
above 12 years of age can come alone. oral health care of infants and children up to the age of
l Paedodontist should also record the name of the 15 years.
person who had referred the patient to him/her and The aims and objectives of paediatric dentistry:
should send an appreciation card to that person. l One of the important aims of a paedodontist is preven-

v. Patient history: tion of diseases, because it can be very effectively im-


l Chief complaint and past medical and dental plemented in younger age groups and prevention is al-
history should be recorded. ways better than cure.
l If the chief complaint needs emergency. A thor- l The general and dental health of a child should be visual-

ough and revealing past history should be taken to ized as a whole and dental health of the child should al-
avoid any untoward incident during the treatment. ways be improved in accordance with their general health.
l A child avoids eating if his/her teeth hurt during l The developing dentition of the child should be

treatment; therefore palliative or corrective treat- observed and controlled as necessary.


ment should be done. Proper restoration and cor- l The parents and patients should be convinced about the

rect occlusion should be given on the first visit. dental treatment and its importance with respect to pri-
l All the records should be signed by the mary dentition and young permanent teeth to avoid
person who has given the history. further dental diseases.
vi. Preliminary examination and consultation: l The aesthetics should be maintained or achieved as the

l After preliminary examination, the patient and case demands.


parent should be taken into the treatment room. Responsibilities of a paedodontist
A thorough examination of the oral cavity l The three main groups of responsibilities or objec-

should be done. tives of a paedodontist are as follows:


l If required, full mouth X-ray is done and im- i. Responsibility towards patient.
pressions are taken, topical fluoride applied and ii. Responsibility towards community.
instructions about proper brushing technique iii. Responsibility toward himself/herself and their
are given. family.
After all these information and examination, a
i. Responsibility towards patient:
diagnosis and treatment plan can be made.
l The main objective of a paedodontic practice
l On next appointment, problems can be pre-
is correct diagnosis and proper treatment plan-
sented in front of parents more accurately
ning. The paedodontists should provide the
and alternate treatment plans may be
best possible service according to their knowl-
suggested.
edge and experience.
l A good first appointment experience provides
l The paedodontist should know good office and
the foundation for enjoyable long-term relation-
patient management techniques. They should
ships with the patients and parents.
know how to handle an infant and a child patient
Recalls:
to perform high quality paedodontic service.
l Recall check-up appointments should be given
l Paedodontist should stress on prevention and
for better results.
this will require knowledge of preventive tech-
l Hygienist or assistant should explain the im-
niques and functions.
portance of periodic examination to keep teeth
l A paedodontist should assure the parents of the
and gums in good health.
child patient that the money which they are
Operational systems:
spending is not a wastage but a wise investment
l For smooth functioning of the dental clinic it is
for future dental health of the child.
very important to schedule and document all
l Good paedodontic practice always begins at
the necessary activities required.
home in the form of proper brushing, diet coun-
l Efficient appointment scheduling makes the
selling, patient education and motivation, etc.
office functioning pleasant.
l A paedodontist should have a good relation-
l Patients who miss their appointments should
ship with the paediatricians and physicians
be separately listed and seen that they are not
practising in the same area, so that they can
lost. They should be contacted periodically and
refer the child patients to the paediatrician in
invited to reschedule the appointment.
case of any medical problems.
286 Quick Review Series for BDS 4th Year, Vol 1

l If the physician and paediatrician find a child with b. Dietary factors:


any dental problems, they will refer them to the pae- l The children should be taught proper

dodontist and that will be helpful for the child also. food habits and about fixed time for
l Whenever a paedodontist opens a clinic, he/she meals, and avoiding in-between meals
should introduce himself/herself to the paediatri- and taking carbohydrates frequently.
cians, physicians and general practitioners practising l If caries susceptibility is very high then

in the same area. This will build up mutual respect, patient can use artificial sweeteners and
understanding and practice. they must brush teeth after every meal.
ii. Responsibility towards community: c. Fluoride prophylaxis:
l Paedodontist should think not only about the chil- l Paedodontist should teach the parents about

dren coming to their clinic but also about the other the importance of fluoride in preventive
children who are unable to afford the treatment. dentistry.
They should try for the betterment of oral health of l If a child is living in a fluoride deficient area,

all children of the community. he/she should be advised fluoride toothpaste


l There should be an earnest desire for better dental and tablets under supervision as home care.
health of children. The dentist should educate the l Periodic topical application of fluoride

children and parents about dental health the public should be done on caries susceptible teeth.
dental health programmes. Water fluoridation and milk fluoridation,
l These programmes should be repeated periodically wherever required, should be carried out.
to remind the people about importance of dental d. Pit and fissure sealants:
health. The pit and fissure sealants must be applied in
l Children Dental Health Day (or Week) should be cel- appropriate time to prevent the decay of
ebrated once or twice a year. This can help to motivate young permanent teeth.
the people. By this programme, parents can bring e. Prevention of malocclusion:
their child to a paedodontist for dental check-up. l Premature loss of deciduous teeth causes

l Community dental health programmes have two development of malocclusion.


components: l The community should be taught about the

i. Preventive programmes causes and consequences of malocclusion


ii. Curative programmes through more attractive, educational pro-
i. Preventive programmes: grammes like the motion pictures, puppet
l Preventive programmes deal with the preven- shows, newspaper articles, radio and TV pro-
tion of initiation of oral and dental diseases in grammes, posters on vehicles, school proj-
children and interception of their progress. ects and talk and slide shows with projectors.
l Space maintenance is a widely accepted
l Most dental diseases cause irreparable damage

to the tissues. By proper planning and imple- and practised preventive procedure.
l The preservation of deciduous dentition
mentation of preventive measures, most of the
dental diseases can be prevented. till their physiological exfoliation not only
l A paedodontist should teach about the follow- prevents malocclusion from developing
ing, to the parents and children: but also makes it less severe.
a. Oral hygiene and prevention of dental dis- ii. Corrective and curative programmes:
l Corrective programmes include the treatment of
eases
b. Dietary factors for proper health dental caries, habit-breaking appliances, myo-
c. Fluoride prophylaxis functional appliances, etc.
l Some dental care programmes should be started
d. Pit and fissure sealants
e. Prevention of malocclusion in play, nursery and primary schools, religious
f. Soft tissue treatment places and public health buildings.
l If a programme is running on a small scale and can-
g. Periodic recall check-up
a. Oral hygiene and prevention of dental not treat all the problems of children, then stress
diseases: should be given on treating and preserving the per-
This includes teaching the techniques of manent first molars because they are the most vul-
proper toothbrushing, demonstration on a nerable, but most important teeth in all respects.
l If a programme is on a large scale, then it should
big model by a big toothbrush or by audio-
visual aids. include the examination and treatment of both
After teaching, they should ask the chil- preschool and school-going children. An early
dren and parents to do what was demon- school dental treatment programme up to the age
strated because they can understand more of 12 years is more economical than teenage treat-
quickly by doing than by only observing. ment programmes.
Section | I  Topic-Wise Solved Questions of Previous Years 287

iii. Responsibility towards himself/herself and their


vii. The paedodontist can now expand the scope of
family:
practice towards the goal of prevention, end-
l As the ability and experience of the paedodontist
odontic treatments and high quality restorative
increases, his/her responsibility also increases.
and cosmetic dentistry and early treatment of
Paedodontics is a continuously growing profes-
periodontal diseases.
sion in knowledge and technique.
viii. The scope of paedodontics is increasing and its
l A paedodontist should always try to improve
horizon widening. It is exciting to look ahead as
his/her knowledge by participating in profes-
the specialty evolves, redefines its boundaries
sional meetings, refresher courses, lectures,
and seeks to collaborate with other specialties in
seminars, continued education programmes and
dentistry and paediatrics.
conferences. They should also read professional
journals, new textbooks and literature to in-
crease their knowledge and ability.
l A paedodontist should try to treat the patients, ac- SHORT NOTES:
cording to the best of the knowledge and ability.
l Paedodontists should have the necessary ability Q.1. Define paedodontics.
and a good personality to earn livelihood for their Ans.
family and for himself/herself. They should es-
tablish good relationship with public, private The paedodontics is defined in number of ways as follows:
school teachers and administrators. According to Stewart, Barber, Troutman and Wei
(1982):
SHORT ESSAYS: l Paediatric dentistry is the practice and teaching of

comprehensive preventive and therapeutic oral health


Q.1. Scope of paedodontics. care of child from birth to adolescence. It is con-
Ans. structed to include care of special patients who dem-
onstrate mental, physical or emotional problems.
l Paedodontics or paediatric dentistry is an age-specific According to American Academy of Pediatric Den-
specialty that provides both primary and comprehensive tistry (1985):
preventive and therapeutic oral health care for infants l Paediatric dentistry is also known as Paedodontics
and children through adolescence including those with and as dentistry for adolescents and children, it is
special health care needs. the area of dentistry concerned with preventive
and therapeutic oral health care for children from
{SN Q.2} birth through adolescence. It also includes special
Scope of paedodontics: care for special patients beyond the age of adoles-
i. It encompasses a variety of disciplines, tech- cence who demonstrate mental, physical or emo-
niques, procedures and skills that logically share tional problems.
a common basis with other specialties. According to Boucher’s dental terminology (1993):
l Paedodontics is the branch of dentistry that in-
ii. To understand special needs of children and ado-
lescents and also of those with special health care cludes having a child accept dentistry, prevention,
needs. detection, restoration of primary and permanent
iii. Paediatric dentistry concentrates mainly on the dentition, applying preventive measures for peri-
integration of appropriate didactic and clinical odontal therapy, dental caries prevalence, intercept-
knowledge from various specialties into a frame- ing and correcting various areas of malocclusion.
work of quality oral health care for children. According to American Association of Pediatric Den-
iv. Paediatric dentistry is an age-specific specialty; tistry (1999):
l Paediatric dentistry is defined as a specialty that
hence, it encompasses all aspects of dentistry,
including some aspects of other specialties. provides both primary and specialty, comprehen-
v. They are fortunate enough of being important sive, preventive and therapeutic oral health care
team members in children’s hospital to work for infants and children through adolescence in-
with members of other disciplines and in plan- cluding those with special health care needs.
ning and execution of treatment of cleft lip and Q.2. Scope of paedodontics.
palate patients.
vi. Paediatric dentistry is the only specialty, which [Ref SE Q.1]
has an intensity of knowledge for the manage- Q.3. Define paediatric dentistry.
ment, and treatment of the oral health needs of
infants, children and adolescents especially Ans.
those with special care needs. [Same as SN Q.1]
288 Quick Review Series for BDS 4th Year, Vol 1

Topic 2
Examination, Diagnosis and Radiographic Techniques
COMMONLY ASKED QUESTIONS
LONG ESSAYS:
1 . Write in detail about examination diagnosis in children.
2. What is treatment planning in paedodontics?
3. Discuss case taking, clinical examination and diagnosis of trauma to anterior teeth.

SHORT ESSAYS:
1 . Intraoral radiographic techniques.
2. Describe briefly radiographic techniques for paedodontic patient.
3. What are the various diagnostic aids in the selection of tooth for vital pulp therapy?
4. Radiographic survey in paediatric dentistry.
5. Sterilization and disinfection techniques used in dental offices.
6. Infection control.

SHORT NOTES:
1. Bitewing radiographs in children.
2. Sterilization methods in the dental clinic.
3. Intraoral periapical radiography in children.
4. Orthopantomograph.
5. Clark’s rule.
6. Digital subtraction radiograph.
7. SLOB rule.
8. Skeletal age versus dental age.
9. Digital radiograph.
10. Miller’s technique.

SOLVED ANSWERS
LONG ESSAYS:
Q.1. Write in detail about examination diagnosis in l Temporomandibular joint
children. l Oral hygiene and periodontal health
l Developing occlusion
Ans. l Caries risk

l Traditionally a complete examination of the patient is l Additional diagnostic aids include:

performed to develop a treatment plan. l Radiographs

l It is presented to the patient parents, outlining the rec- l Study models

ommended course of treatment. l Photographs

l A thorough examination diagnosis of the paediatric l Pulp tests

dental patient includes assessment of following: l Laboratory tests

l General growth Preliminary medical and dental examination:


l Behaviour l It is important for the dentist to be familiar with the

l General health medical and dental history of the paediatric patient.


l Chief complaint, e.g. pain l Familial history may also be relevant to the patient’s

l Extraoral and intraoral soft tissues oral condition and may provide important diagnostic
l Intraoral hard tissue information in some hereditary disorders.
Section | I  Topic-Wise Solved Questions of Previous Years 289

l Prior to dentist examining the child, the dental as- office and the facts related by the patient and the
sistant can obtain sufficient information to provide parent regarding previous care in another office.
the dentist with knowledge of the child’s general Chief complaint:
health and can alert the dentist to the need for obtain- l The reason which prompted the patient to seek

ing additional information from the parent or the dental treatment


child’s physician. l Most common reasons are pain, swelling and to

Behaviour: improve aesthetics


l Information regarding the child’s social and psy- l May be referred from other practitioner

chological development is important. Clinical examination:


l Accurate information reflecting a child’s learning, l A thorough clinical and radiographic examination

behavioural or communication problems is espe- helps in obtaining most facts needed for a com-
cially important when the parents are aware of prehensive oral diagnosis in the young patient.
their child’s developmental disorder. l In addition to examining the structures in the oral

l Behaviour problems in the dental office are often cavity, the dentist may in some cases wish to note
related to the child’s inability to communicate the patient’s size, stature, gait or involuntary
with the dentist and to follow instructions. movements.
l This inability may be attributable to a learning l The first clue to malnutrition may come from ob-

disorder. An indication of learning disorders can serving a patient’s abnormal size or stature.
usually be obtained by the dental assistant while l Similarly, the severity of a child’s illness, even if

asking questions about the child’s learning process. oral in origin, may be recognized by observing a
For example, asking a young school-aged child weak, unsteady gait of lethargy and malaise as the
how he/she is doing in school is a good lead ques- patient walks into the office.
tion. The questions should be age appropriate for l The clinical examination, whether the first

the child. examination or a regular recall examination,


General health: should be all inclusive.
l When there is indication of an acute or chronic l Attention to the patient’s hair, head, face, neck

systemic disease or anomaly, the dentist should and hands should be among the first observations
consult the child’s physician to learn the status of made by the dentist after the patient is seated in
the condition, the long range prognosis and the the chair.
current drug therapy. l The patient’s hands may reveal information perti-

l In addition to consulting the child’s physician, the nent to the comprehensive diagnosis.
dentist may decide to record additional data con- l The dentist may first detect an elevated tempera-

cerning the child’s current physical condition, ture by holding the patient’s hand.
such as blood pressure, body temperature, heart Cold, clammy hands or bitten fingernails may be
sounds, height and weight, pulse and respiration. the first indication of abnormal anxiety in the
l Before treatment is initiated, certain laboratory child. A callused or unusually clean digit suggests
tests may be indicated and special precautions a persistent sucking habit.
may be necessary. l Clubbing of the fingers or a bluish colour in the

l If the dentist is aware that a child was previously nail beds suggests congenital heart disease that
hospitalized or the child fears strangers in clinic may require special precautions during dental
attire, the necessary time and procedures can be treatment.
planned to help the child overcome the fear and l Inspection and palpation of the patient’s head and

accept dental treatment. neck are also indicated. Unusual characteristics of


l The dentist’s personal involvement at this early the hair or skin should be noted.
time strengthens the confidence of the parents. l The dentist may observe signs of head lice, ring-

l It is advisable to postpone nonemergency dental worm or impetigo during the examination. Proper
care for a patient exhibiting signs or symptoms of referral is indicated immediately, because these
acute infectious disease until the patient recovers. conditions are contagious. After the child’s physi-
The pertinent facts of the medical history can be cian has supervised the treatment to control the
transferred to the oral examination record for easy condition, the child’s dental appointment may be
reference by the dentist. rescheduled.
l The patient’s dental history should also be sum- l If a contagious condition is identified but the child

marized on the examination chart. This should also has a dental emergency, the dentist and the
include a record of previous care in the dentist’s staff must take appropriate precautions to prevent
290 Quick Review Series for BDS 4th Year, Vol 1

spread of the disease to others while the emer- infection is suspected, immediate referral to
gency is alleviated. Further treatment should be the child’s physician is indicated.
postponed until the contagious condition is con- Hard tissues:
trolled. l After thoroughly examining the oral soft tis-

l Variations in size, shape, symmetry or function of sues, the dentist should inspect the occlusion
the head and neck structures should be recorded. and note any dental or skeletal irregularities.
Abnormalities of these structures may indicate This dynamic developmental process occurs in
various syndromes or conditions associated with all three planes and with periodic evaluation
oral abnormalities. the dentist can intercept and favourably influ-
Temporomandibular evaluation ence undesirable changes.
l One should evaluate Temporomandibular joint l Monitoring of the patient’s facial profile and

(TMJ) function by palpating the head of each symmetry; molar, canine and anterior segment
mandibular condyle and observing the patient relationships; dental midlines and relation of
while the mouth is closed with teeth clenched, at arch length to tooth mass should be routinely
rest and in various open positions. included in the clinical examination.
l Movements of the condyles or jaw that are not l The teeth should be inspected carefully for

smoothly flowing or deviate from the expected evidence of carious lesions and hereditary or
norm should be noted. acquired anomalies. They should also be
l Similarly, any crepitus that may be heard or iden- counted and identified individually to ensure
tified by palpation, or any other abnormal sounds, recognition of supernumerary or missing teeth.
should be noted. l Identification of carious lesions is important in

l Sore masticatory muscles may also signal TMJ patients of all age groups but is especially
dysfunction. critical in young patients because the lesions
Such deviations from normal TMJ function may may progress rapidly in early childhood caries
require further evaluation and treatment. if not controlled.
The extraoral examination: l The decision whether to place a sealant or to

l The extraoral examination continues with palpa- restore a defect depends on the patient’s his-
tion of the patient’s neck and submandibular tory of dental caries, the parents’ or patient’s
area. acceptance of a comprehensive preventive den-
l Any deviations from normal, such as unusual ten- tistry programme and the patient’s dependabil-
derness or enlargement, should be noted and follow- ity in returning for recare appointments.
up tests performed or referrals made as indicated. l In patients with severe dental caries, caries

The intraoral examination: activity tests and diet analysis may contribute
Soft tissues: to the diagnostic process by helping define
l The intraoral examination of a paediatric pa- specific aetiologic factors.
tient should be comprehensive. l Additional diagnostic aids include:
The dentist should first evaluate the condition l Radiographs

of the oral soft tissues. l Study models

l The buccal tissues, lips, floor of the mouth, l Photographs

palate and gingivae should be carefully in- l Pulp tests

spected and palpated. l Laboratory tests

l Soft tissue should be examined for 3Cs, i.e. Radiographic examination:


change in the colour, contour and consistency. l When indicated, radiographic examination for
l The use of the periodontal screening and re- children must be completed before the compre-
cording programme (PSR) is often a helpful hensive oral health care plan can be developed.
adjunct in children. It is designed to facilitate l Obtaining isolated occlusal, periapical or bite-

early detection of periodontal diseases with a wing films is sometimes indicated in very young
simplified probing technique and minimal doc- children due to trauma, toothache, suspected de-
umentation. velopmental disturbances or proximal caries.
l The tongue and oropharynx should be closely l Intraoral periapical radiograph (IOPA) gives in-

inspected. Enlarged tonsils accompanied by formation regarding the presence or absence of


purulent exudate may be the initial sign of a permanent teeth, shape and position of the teeth
streptococcal infection, which can lead to present, relative state of development of teeth,
rheumatic fever. When streptococcal throat extent of calcification of developing tooth, path of
Section | I  Topic-Wise Solved Questions of Previous Years 291

eruption of permanent teeth, morphology and in- c. Preventive treatment phase:


clination of the root of permanent tooth. l Oral prophylaxis

Study models: l Caries control

l Study models should be neat and record surround- l Orthodontic consultation

ing anatomical structures like alveolar process. l Oral surgery

l When models are in occlusion, the top surface of l Endodontic therapy

upper model and lower surface of the lower base d. Corrective treatment phase:
should be parallel. l Operative dentistry

l In total height of the cast, the anatomic portion l Prosthetic dentistry

should be three-fourth and artistic portion should l Orthodontic therapy

be one-fourth. e. Maintenance phase:


l Model analysis is done to detect arch size and l It includes periodic recall examination and main-

tooth size discrepancies. tenance treatment.


Photographs: The detailed description of ideal treatment plan is as
Ideally extra oral and intraoral maxillary and man- follows:
dibular photographs are taken. a. Medical treatment:
They are useful to: l During this phase patients with positive medi-

l Assess the symmetry of the face cal history are referred to paediatrician for
l Assess profile and facial type evaluation and consent.
l Serve as a record l It may also be required to modify the dosage or

Pulp testing: change a particular drug, etc. as per the re-


This is not a routine diagnostic aid used in children quirement of the treatment.
as their pain perception varies due to the resorbing or l When the history and examination of the child

developing roots. suggest a medical problem, the dentist should


Response to vitality testing is read as follows: consult the child’s physician to insure the
Nil – nonvital pulp or false –ve health and safety of the child during treatment.
Moderate transient – normal l Blood dyscrasias are often reflected in the oral

Painful transient – reversible pulpitis cavity by changes in colour, size, shape and
Painful lingering – irreversible pulpitis consistency of oral soft tissues. Therefore, the
dentist should evaluate tissue changes and re-
Q.2. What is treatment planning in paedodontics?
lay any pertinent information to the child’s
Ans. physician or paediatrician.
b. Systemic treatment:
l Successful dental treatment is based on an accurate
l This phase includes any medication given to
diagnosis and careful treatment planning.
modify dental treatment, such as premedication
l The history, clinical examination and laboratory diag-
for behaviour management or antibiotic prophy-
nostic aids will provide the essential facts necessary to
laxis to a child with congenital cardiac defects.
make an accurate diagnosis and appropriate treatment
l Premedication of apprehensive children, spas-
plan.
tic patients or those with cardiac problems is
l A pattern of sequence serves as a reminder of the phases
frequently necessary and should be done only
of treatment which must be considered in caring for the
after consultation with the child’s physician.
total needs of the child. The following general outline is
The exact dosages of all drugs to be used
suggested for paedodontic treatment planning.
should be entered on the treatment plan.
l Treatment planning can be made based on following
l Systemic drug therapy may cause oral tissue
five different phases:
changes which make restorative work difficult
a. Medical phase
or even impossible.
b. Systemic phase
For example, a child taking Dilantin sodium
c. Preventive phase
develops severely hypertrophic gingivae. Such
d. Corrective phase
a problem should be discussed with the physi-
e. Maintenance phase
cian so that drug substitution may alleviate the
Outline for paedodontic treatment planning consists of:
problem and dental treatment can proceed.
a. Medical treatment phase:
c. Preventive treatment phase:
l Referral to a physician
l Preventive phase is aimed at providing preven-
b. Systemic treatment phase:
tive therapy to patient and minimize dental
l Premedication
disease.
l Therapy for oral infections if any
292 Quick Review Series for BDS 4th Year, Vol 1

l Preventive phase is the first phase of treatment l All possibilities of alternate future treat-
and it includes: ment, such as restorative and endodontic
i. Oral prophylaxis and fluoride treatment procedures, replacement problems with
ii. Oral hygiene counselling removable or fixed prostheses, must be
iii. Diet counselling considered from a practical and a dental
iv. Pit and fissure sealant application health point of view.
v. Orthodontic consultation d. Corrective treatment
l Final corrective treatment can be started only

i. Oral prophylaxis and fluoride treatment: after the medical and preparatory phases have
l After establishment of the medical status and been initiated.
premedication regime of the child, his/her l Sequence is important even in this phase of the

teeth should be thoroughly cleaned. treatment plan.


l The concerned dentist can offer the parent For example, caries should be eliminated from
an interesting and challenging opportunity the teeth and the restorations polished before
to facilitate the reduction of caries in the orthodontic treatment is initiated.
child by topical fluoride application or use e. Maintenance phase:
of systemic fluorides. l It includes periodic recall examination and

ii. Oral hygiene counselling: maintenance treatment.


l The oral prophylaxis gives the dentist a l Upon completion of treatment, the wise dentist

splendid opportunity to teach the child tooth gives an explicit appointment for the next re-
brushing and other elements of home care. call visit. The interval may vary from 3 months
l Also, much can be learned about the pa- to 6 months.
tient’s temperament, apprehensiveness Alternate treatment plans
and oral health status during the oral pro- l There is an ideal treatment plan for every

phylaxis and the home care instructions. child that should be presented to the parent
iii. Diet counselling: using models, X-rays and other aids.
l Closely following the initial prophylaxis, l Unlike adult treatment, dental care for chil-

an evaluation of the caries susceptibility of dren must not be delayed or even spaced
the child should be made. If active caries is over a long period of time. There are few
evident, the child’s mother should be ques- opportunities for alternate treatment plans
tioned closely about his/her diet. in paedodontics. When an alternate plan is
l With the proper approach, the concerned suggested, the dentist must be sure that the
dentist can offer the parent an interesting results will be as beneficial as possible and
and challenging opportunity to facilitate not in any way detrimental to the future
the reduction of caries in the child by sev- dental health of the child.
eral available means, like topical and sys- Advantages of treatment planning:
temic fluorides, diet substitutions and l Repeat of diagnosis at every visit is

more regular meals without in-between avoided.


snacks. l Instruments can be prepared well in

iv. Pit and fissure sealant application: advance.


The incidence of caries can be reduced by l Serial appointments can be given on the

application of pit and fissure sealants. first visit.


Age range for sealant application l Total fee estimation can be done.

3–4 years age – for primary molar sealant


application Q.3. Discuss case taking, clinical examination and diag-
6–7 years age – for the first permanent molar nosis of trauma to anterior teeth.
11–13 years age – for the second permanent History and examination:
molars and premolars l The routine clinical evaluation sheet is helpful dur-

v. Orthodontic consultation: ing the initial examination and subsequent examina-


l When crowding or malalignment is evident, tions of an injured tooth.
an orthodontist should be consulted immedi- l It serves as a checklist of important questions that

ately. Frequently, preventive orthodontic mea- must be asked and observations that must be made by
sures can be performed concurrently with re- the dentist and the auxiliary personnel during the
storative procedures. examination of the child.
Section | I  Topic-Wise Solved Questions of Previous Years 293

History of the injury: extent of the injury and in estimating the abil-
l The time of the injury should first be established. ity of the injured pulp and supporting tissues to
l Sometimes the accident is so severe that dental treat- overcome the effects of the injury.
ment cannot be started immediately because other inju- l Trauma to the supporting tissues may cause
ries have higher priority. sufficient inflammation to initiate external root
l Davis and Vogel emphasized that a force strong enough resorption. In instances of severe injury, teeth
to fracture, intrude or avulse a tooth is equally strong can be lost as a result of pathologic root resorp-
enough to result in cervical spine or intracranial injury. tion and pulpal degeneration.
l The dentist must be particularly alert to such potential l The clinical examination should be conducted
problems, be prepared ahead of time to make a neuro- after the teeth in the area of injury have been
logic assessment and make appropriate medical referral carefully cleaned of debris.
when indicated promptly. l When a fracture of the crown has resulted due
l Davis recommends a quick cranial nerve evaluation in- to injury, the dentist should observe the
volving the following four areas: amount of tooth structure that has been lost
a. Extraocular muscles: and should look for evidence of a pulp expo-
If they are intact and functioning appropriately then, sure.
the patient can track a finger moving vertically and l Under a good light, the clinical crown should
horizontally through the visual field with the eyes be examined carefully for cracks and craze
remaining in tandem. lines, the presence of which could influence
b. Pupils: the type of permanent restoration used for the
Should be equal, round and reactive to light with tooth.
accommodation. l With light transmitted through the teeth, a se-
c. Sensory function: verely traumatized tooth often appears darker
Is measured through light contact to various areas of and reddish, although not actually discoloured,
the face. which indicates pulpal hyperaemia. This ap-
d. Symmetry of motor function: pearance suggests that at some later time the
Is assessed by having the patient frown, smile, move pulp may undergo degenerative change termi-
the tongue and perform several voluntary muscular nating in pulpal necrosis.
movements. If they are equal on both sides then it is l Historically, the Ellis and Davey classification
assumed that symmetry is present of crown fractures is useful in recording the
l The patient should be assessed for nausea, extent of damage to the crown.
vomiting, drowsiness or possible cerebral l A vitality test of the injured tooth should be
spinal fluid leakage from the nose and ears, performed, and the teeth in the immediate
which would be indicative of a skull fracture. area, as well as those in the opposing arch,
l The patient should be evaluated for lacerations should be tested. The best prediction of con-
and facial bone fractures. tinued vitality of the pulp of a damaged or
l For practical and especially economic reasons, traumatized tooth is the vital response to elec-
Andreasen and colleagues have attempted to tric pulp testing at the time of the initial ex-
classify pulpal and periodontal healing of trau- amination.
matic dental injuries based on the effect of l A negative response, however, is not reliable
treatment delay. evidence of pulp death because some teeth that
l Unfortunately, there is limited knowledge of give such a response soon after the injury may
the effect of treatment delay on wound healing recover vitality after a time. When the electric
available in the literature. pulp tester is used, the dentist should first deter-
l Taking a complete dental history can help the mine the normal reading by testing an unin-
dentist learn of previous injuries to the teeth in jured tooth on the opposite side of the mouth
the area. The dentist must rule out the possibil- and recording the lowest number at which the
ity of a degenerative pulp or adverse reaction tooth responds.
of the supporting tissues as a result of previous If the injured tooth requires more current
trauma. than does a normal tooth, the pulp may be
l The patient’s complaints and experiences after undergoing degenerative change. If less cur-
the injury are often valuable in determining the rent is needed to elicit a response from a
294 Quick Review Series for BDS 4th Year, Vol 1

traumatized tooth, pulpal inflammation is SHORT ESSAYS:


usually indicated.
l The reliability of the electric pulp test depends
Q.1. Intraoral radiographic techniques.
on eliciting valid responses from the patient. It Ans.
is frequently unreliable even on normal teeth
when apices are incompletely formed. The various intraoral projection techniques are as follows:
l The thermal test is also somewhat helpful in
a. Paralleling technique
determining the degree of pulpal damage after b. Bisecting angle technique
trauma. It is probably more reliable in testing c. Specialized intraoral radiographic technique
primary incisors in young children than the d. Supplementary intraoral radiographic techniques
l The paralleling technique and bisecting angle tech-
electric pulp test.
l Failure of a recently traumatized tooth to re-
nique are two intraoral projection techniques that are
spond to the pulp test is not uncommon. How- used for periapical radiography.
ever, the traumatized tooth may be in a state a. Paralleling technique:
of shock and as a result may fail to respond to l Dr Gordan is the pioneer of paralleling technique.
the accepted methods of determining pulp l It is also called right-angle technique/long cone tech-
vitality. nique/Fitzgerald technique.
l Laser Doppler flowmetry has been reported to l Paralleling principle of intraoral X-ray is technique
be a significant aid in determining vascular of choice, because it is more accurate and produces
vitality of traumatized teeth. less distortion than bisecting angle technique.
Radiographic examination: l The primary purpose of this is to obtain a true radio-
l The examination of traumatized teeth is not consid- graphic orientation of teeth and supporting structures.
ered to be complete without a radiograph of the l It is based on the principle that central ray should be
injured tooth, the adjacent teeth and sometimes the focused perpendicular to long axis of the film with
teeth in the opposing arch. the X-ray film being parallel to long axis of tooth.
l It may even be necessary to obtain a radiograph of l To obtain parallelism and to reduce distortion the
the soft tissue surrounding the injury site to rule out film is placed away from tooth but the use of long
presence of a fractured tooth fragment. source to object distance reduces the size of the ap-
l In young patients, the stage of apical development parent focal spot and leads to less magnification and
often indicates the type of treatment. increased definition.
l The presence of a root fracture may not influence the l Film holders are used to ensure proper position of the
course of treatment, particularly if the fracture line is film and to maintain it in position.
in the region of the apical third. Teeth with root frac- l To assure that the periapical areas are be projected
tures in this area rarely need stabilization, and a onto the film, it is necessary that the film be posi-
fibrous or calcified union usually results. tioned away from the teeth and towards the centre of
l If teeth have been discernibly dislocated, with or the mouth, where the maximum height of the palate
without root fracture, two or three radiographs of the can be utilized.
area at different angles may be needed to clearly de- l For maxillary projections, the superior border of the
fine the defect and aid the dentist in deciding on a film will generally rest at the height of the palatal
course of treatment. vault in the midline.
l The radiographs provide a record of the tooth im- l For mandibular projections, the inferior border of the
mediately after the injury. Frequent, periodic radio- film is depressed into the floor of the mouth away from
graphs reveal evidence of continued pulp vitality or the mucosa on the lingual surface on the mandible.
adverse changes that take place within the pulp or the l In case of children, as there is high muscle activity in
supporting tissues. the mandible and shallow palate, the film cannot be
l When more complex facial injuries have occurred or placed parallel to the long axis of the teeth but it has
jaw fractures are suspected, extraoral films may also been proved that even if the film is placed within 20°
be necessary to identify the extent and location of all of the parallel to the long axis, with the beam di-
injury sequelae. Oblique lateral jaw radiographs and rected to the film, the radiograph produced by paral-
panoramic films are often useful adjuncts to this di- leling technique will be far superior than bisecting
agnostic process. angle technique.
Section | I  Topic-Wise Solved Questions of Previous Years 295

Advantages: l This technique can be used both with paralleling


i. Images obtained are accurate with minimum cone or bisecting angle technique.
magnification. l This technique works well with young children,

ii. Interdental bone levels are very well represented. requires little skill as patient bites down.
iii. Periapical tissue will be accurately demonstrated l Top portion of the film is bent at right angle and this

with minimal foreshortening or elongation. serves as a bite block to hold the film in place. Pa-
Disadvantages: tient is instructed to bite the film slowly and radio-
i. Positioning of the film packet is very uncomfort- graph is taken. Care must be taken to straighten the
able for patient especially in the posterior aspect film before processing.
of teeth, often causing gagging. . Supplementary intraoral radiographic techniques:
d
ii. Anatomy of mouth sometimes makes the tech- i. Bitewing radiography:
nique difficult. l Periapical films are used to record the coronal

iii. Positioning the holders in the lower 3rd molar portions of both maxillary and mandibular teeth
region can be very difficult. in one image.
b. Bisecting angle technique: l Size 1 film is used in children and size 2 films are

l It is also called Millers right-angle technique/short used in adults.


cone technique/isometric triangulation technique. l Used mostly to detect interproximal caries and to

l This tech is based on the principle of Cieszynsky rule check the level of bone.
of isometry. ii. Occlusal radiography:
l In this technique the film is placed close to the teeth l Used to take the jaw radiographs of maxilla and man-

and central ray is directed at right angles to the line dible to detect large lesions, fractures, impactions,
bisecting the angle formed by the plane of the film supernumerary teeth and to localize foreign bodies.
and the long axis of the tooth. l The film is partially held in-between teeth and

l Angulations of tube head is different for different teeth. partially supported by patient.
Maxillary: l The vertical angulation for maxilla is 145° and

Incisor: 140°, premolar: 130°, canine: 145°, molar: for mandible is 255°.
120°.
Q.2. Describe briefly radiographic techniques for
In deciduous: Anterior: 145°, posterior: 130°
paedodontic patient.
Mandibular:
Incisor: 215°, premolar: 210°, canine: 220°, Ans.
molar: 25°.
Commonly used radiographic techniques in paedodontic
In deciduous: Anterior: 215°, posterior: 210°
patients depend primarily on the size of the oral cavity, the
Advantages:
number of teeth present and patient cooperation.
i. Positioning of film or film packet is simple,
The procedures commonly used by paedodontic practi-
quick and reasonably comfortable for patient in
tioner include the following:
all areas of mouth.
i. Bitewing
ii. If proper angulation is given there will not be
ii. Periapical
any distortion of image.
iii. Occlusal
Disadvantages:
iv. Panoramic
i. Improper vertical angulations may lead to
shortening or lengthening of image. i. Bitewing technique:
ii. Interdental bone less will be poorly demon- l A No. 0 bitewing film is usually the most suitable size

strated. for the smaller patient, while some children’s mouths


iii. Shadow of zygomatic bone frequently overlies are large enough to receive a No. 2 bitewing film.
the roots of upper molars. l The head is positioned so that the mid sagittal plane

iv. Incorrect horizontal angulation will result in is perpendicular and the ala-tragus line is parallel to
horizontal overlapping of crowns and roots. the floor.
v. Crowns of teeth are often distorted, thus pre- l The inferior edge of the bitewing film packet is

venting detection of proximal caries. placed in the floor of the mouth between the tongue
c. Specialized intraoral radiographic technique: and the lingual aspect of the mandible and the bite-
l Bent film radiographic technique is used exclusively tab or positioning device is placed on the occlusal
for children who do not tolerate the placing of a film surfaces of the mandibular teeth.
holder inside their mouths. l The anterior edge of the film packet is located as

l Size 1 or 2 should be used. far anteriorly as possible in the region of the


296 Quick Review Series for BDS 4th Year, Vol 1

canine so that the distal aspect of the canine will includes the teeth, the supporting structures, the
be recorded. maxillary region extending to the superior third of
l The dentist holds the bite-tab against the occlusal the orbit and the entire mandible including the tem-
surfaces of the patient’s mandibular teeth with an poromandibular joint region.
index finger, and the patient is instructed to ‘close l Panoramic radiology can be valuable when disabled

slowly’. The finger is rolled out of the way onto the patients are examined if the patient can sit in a chair
buccal surfaces of the teeth as the patient closes in and hold his/her head in position.
centric occlusion. l The only inherent drawback to panoramic radiogra-

l The central ray enters through the occlusal plane at phy is lack of image detail for diagnosing early cari-
a point below the pupil of the eye. The vertical angle ous lesions.
is 18° to 1 10°. v. Lateral jaw technique:
ii. Periapical technique: l A 5 3 7 inch X-ray film is used for the lateral jaw

l There are essentially two methods of taking periapi- technique. The film is marked with a right or left lead
cal radiographs: paralleling and bisecting angle identification letter placed on the film packet slightly
techniques. anterior and superior to the central portion of the film.
l Each has benefits and limitations when used with the l The patient’s head is positioned so that the occlusal

paediatric patient. plane is parallel and the sagittal plane is perpendicu-


l Regardless of which technique is used, film posi- lar the floor.
tioning for the two techniques is identical. In all l The long axis of the film, also perpendicular to the

cases the identification dot is placed towards the floor, rests on the patient’s shoulder and against the
occlusal surface. face. The patient is instructed to rotate the head to-
iii. Occlusal radiography: wards the film until the nose rests against it. Then the
l Occlusal radiographs are used to take the entire jaw chin is raised and the head tilted approximately 15°
radiographs of maxilla and mandible to detect large towards the film.
bony lesions, fractures, impactions, supernumerary l The patient secures the film with the palm of the

teeth and to localize foreign bodies. hand and with fingers extended. The cone is posi-
l The patient’s occlusal plane should be parallel to the tioned so that the central X-ray beam enters at a point
floor, and the sagittal plane should be perpendicular a half-inch behind and below the angle of the man-
to the floor. dible on the side opposite the film. The vertical angle
l A No. 2 periapical film is placed in patient’s mouth is 17°.
so that the long axis of the film is parallel to the floor l The central X-ray beam is perpendicular to the hori-

partially held in-between teeth and partially sup- zontal plane of the film.
ported by patient. l With advent of specialized panoramic views this

l The vertical angulation for maxilla is 145° and for technique is slowly becoming obsolete.
mandible is 255°.
iv. Panoramic radiography: Q.3. What are the various diagnostic aids in the selec-
l Frequently employed extra oral radiographs include tion of tooth for vital pulp therapy?
panoramic and cephalometric radiographs.
Ans.
l Panoramic radiographs aid in visualizing the various

structures in the child’s developing dentition. l Prior to the selection of primary tooth for initiation
l They are excellent for determining the position of the of pulp therapy a thorough clinical and radiographic
unerupted permanent teeth and their likely path of examination must be made.
eruption. Various pathologic conditions like cysts, The clinical examination would normally include:
tumours and so on can also be detected on these films. l A case history, using the classic format with ap-

l Numerous panoramic X-ray units are available to propriate alterations as required.


the dental profession. The use of a machine with a l History of present illness (PI), with leading ques-

digital receptor allows a very low dose and means tions like ‘Does the tooth hurt now?’ ‘Has it ever
that the operator, in many cases, may not even need hurt you?’ ‘Does it hurt when you drink cold wa-
to place an image receptor in the mouth. This would ter?’ ‘Does it hurt when you chew?’ may well
be a great advantage in some cases where the child determine whether one is dealing with a pulpitis
is not tolerant of an intraoral image acquisition. or an apical periodontitis.
l It provides an excellent coverage of the structures l Personal past history (PPH), ‘Is your child in

that are viewed during paediatric dental diagnosis. A good physical health at this time?’ ‘Has he ever
typical diagnostic film or digital panoramic image had any serious illnesses – diabetes, rheumatic
Section | I  Topic-Wise Solved Questions of Previous Years 297

fever or the like?’ ‘Is he allergic to any drugs?’ l These series of radiographic examinations include the
will give indications as to the condition of his/her following.
general health and any limitations on treatment. i. Four-film series:
l Examination of the specific area is best started l This series consists of a maxillary and mandibu-
with an examination of the soft tissue. lar anterior occlusal and two posterior bitewing
Any signs such as discolouration, a draining or radiographs.
quiescent fistula or inflammation should cause ii. Eight-film survey:
serious doubts about proceeding with pulp ther- l This survey includes a maxillary and mandibular
apy short of endodontics. anterior occlusal (or periapicals), a right and left
l Then the tooth itself should be examined for maxillary posterior occlusal (or periapicals),
clinical destruction of the crown and possible right and left primary mandibular molar periapi-
presence of a hypertrophied pulp. cals and two posterior bitewings.
l The tooth’s mobility should be determined, which, iii. Twelve-film survey:
if present, should warn one about a possibly ne- l This includes four primary molar-premolar peri-
crotic pulp. apical radiographs, four canine periapical radio-
l Percussion of the tooth should follow, for if any graphs, two incisor periapical radiographs and
sensitivity is registered by the patient, the proba- two posterior bitewing radiographs.
ble periapical involvement should make one dubi- l Several techniques are commonly used to radio-
ous of pulpal therapy success. graph a child’s dentition are bitewing, periapi-
l A vitality test may be made, but the results ob- cal, occlusal and panoramic views.
tained from primary teeth by this technique have l The technique used depends primarily on the
been mostly unreliable. size of the oral cavity, the number of teeth pres-
Radiographic examination: ent and choice of patient practitioner.
l To complete the diagnosis good radiographs are

required subsequent to the choice of treatment Q.5. Sterilization and disinfection techniques used in
and prognosis. dental offices.
l Both periapical and bitewing films are necessary. Ans.
They make it possible to acquire some idea as to
the pulp’s condition. l Most commonly used methods of sterilization in den-
l Any form of internal resorption be present in the
tistry are as follows:
coronal or apical portions, it is unlikely that the i. Steam autoclave
pulp will respond well to treatment. ii. By dry heat in a hot air oven
l The radiograph might indicate periapical or bifur-
iii. By unsaturated chemical vapour sterilizer (chemi-
cation involvement that suggests a degenerated clave)
l Other methods
pulp.
l The presence of calcified bodies, or pulp stones, has
i. Low-temperature steam and formaldehyde (LTSF)
been reported to be evidence of pulpal degenera- ii. Use of ethylene oxide
tion. Such an obvious finding as prematurely re- iii. Irradiation
l Cleaning and disinfection of dental unit and environ-
sorbed roots would be detrimental to pulp therapy.
l In summary, wherever possible, it is desirable to
mental surfaces:
evaluate as many diagnostic criteria as possible i. After treatment of each patient and at the comple-
before proceeding with pulp therapy and particu- tion of daily work activities, dental unit surfaces
larly prior to any anaesthesia. should be cleaned with disposable towels, using
l If the decision to perform pulp therapy has to be
an appropriate cleaning agent and water as
made after the tooth has been entered, then radio- necessary.
graphs and obvious clinical symptoms should be ii. Surfaces then should be disinfected with suitable
relied upon. chemical germicides such as phenols, idophors,
quaternary ammonium compounds, household
Q.4. Radiographic survey in paediatric dentistry. bleach and chlorine-contained compounds.
Ans. iii. Laboratory materials and other items that have
been used in the mouth, e.g. impressions, appli-
l Whenever a new patient is examined at the dental office ances and prosthesis, should be cleaned and
and no previous radiographs are available, it may be disinfected before being manipulated in the
necessary to obtain a baseline series of radiographs. laboratory.
298 Quick Review Series for BDS 4th Year, Vol 1

iv. These items also should be cleaned and disinfected viii. Dental instruments must be wrapped before steril-
after being manipulated in the dental laboratory ization. Unwrapped instruments have no shelf life.
and before placement in patient’s mouth. ix. Barrier protection of surfaces and equipment can
v. Single-use disposable instruments (prophylaxis prevent contamination of clinical contact sur-
tips and saliva ejectors) should be used for one faces, but is particularly effective for those that
patient only and discarded appropriately. are difficult to clean.
l Treatment and disposal technique for health Barriers include clear plastic wrap, bags, sheets,
care waste are as follows: tubing and plastic-backed paper or other materi-
Incineration, safe burying, encapsulation, chem- als impervious to moisture
ical disinfection, microwave irradiation and wet x. Hand hygiene, e.g. handwashing and hand anti-
thermal treatment. sepsis substantially reduces potential pathogens
on the hands. Evidence indicates that proper
Q.6. Infection control.
hand hygiene is the single most critical measure
Ans. for reducing the risk of transmitting organisms.
xi. For routine dental examinations and nonsurgical
l The dental team is exposed to a wide variety of micro-
procedures, handwashing and hand antisepsis is
organisms in the saliva and blood of their patients,
achieved by using a plain or antimicrobial soap
which may include hepatitis B and C, herpesviruses,
and water. If the hands are not visibly soiled, an
cytomegalovirus, measles virus, mumps virus, chicken-
alcohol-based hand rub is adequate.
pox virus, human immunodeficiency virus, Mycobacte-
xii. Biofilms form quickly and serve as continuous
rium tuberculosis, streptococci, staphylococci and other
sources of contamination for dental unit water
nonvaccine-preventable infections.
lines water. Flushing of lines temporarily reduces
l As it is impossible to identify all of those patients who
microbial emissions, but does not remove biofilm.
may harbour dangerous microorganisms, it is necessary
xiii. Use of sterile water does not reduce the level of
to use standard precautions and practice infection con-
microorganisms released. The only remedy is to
trol procedures routinely to avoid spread of disease.
remove effectively the biofilms through the ap-
l The following infection control procedures as described
plication of certain chemicals.
by Miller and Paienikle are based on those recom-
xiv. Routine use of additional chemicals helps retard
mended for dentistry by the Centres for Disease Control
biofilm development.
and Prevention (CDC) in the Public Health Service of
As exposure to microorganisms can cause infec-
the US Department of Health and Human Services:
tions, it is the responsibility of dental health care
i. Always obtain an updated thorough medical his-
practitioners to use water that has the lowest level
tory, which include enquiry about medications,
of microbial contamination.
current illnesses, hepatitis, unintentional weight
loss, lymphadenopathy, oral soft tissue lesions or
any other infections. SHORT NOTES:
ii. Clean all reusable instruments in an ultrasonic
cleaner or washer/disinfector and minimize the Q.1. Bitewing radiographs in children.
amount of hand scrubbing performed.
Ans.
iii. Wearing of personal protective equipment like
heavy rubber gloves, mask and protective clothing l Bitewing films are used to record the crowns of maxil-
and eyewear to protect against puncture injuries lary and mandibular teeth in one film.
and splashing is required when treating patients. l These are one of the best amongst the currently avail-
iv. Sterilize all reusable instruments that penetrate or able methods, to detect or rule out early interproximal
come into contact with oral tissues or that become carious lesions.
contaminated with saliva or blood. l Sizes:

v. Metal or heat-stable instruments should be steril- i. Size 0 – for children – posterior (22 3 35 mm)
ized in a steam autoclave, a dry heat oven or an ii. Size 1 – for children – anterior (24 3 40 mm)
unsaturated chemical vapour sterilizer. Uses:
vi. Heat-sensitive items may require up to 10 h expo- i. They are particularly valuable for detecting in-
sure time for sterilization in a liquid chemical ter-proximal caries in the early stages of devel-
agent/ sterilant, followed by rinsing with sterile opment before it becomes clinically apparent.
water. ii. Visualize the alveolar crest and assessment of
vii. Biological monitoring must occur weekly. periodontal disease in easier way.
Section | I  Topic-Wise Solved Questions of Previous Years 299

iii. They are especially effective, and useful for Indications:


detecting calculus deposits in inter-proximal i. Evaluation of tooth development during (mixed
areas, because of projection directly through dentition), developmental anomalies, etc.
the inter-proximal spaces. ii. It is indicated to detect fracture cases, various
iv. Useful in periodic check-up of the teeth for pathologic conditions like cysts and tumours.
detection of new caries and of early periodontal Advantages:
change. l Panoramic radiographs aid in visualizing broad

anatomic region exhibiting various structures in


Q.2. Sterilization methods in the dental clinic.
the child’s developing dentition.
Ans. l They are excellent for determining the position of

the unerupted permanent teeth and their likely


l Sterilization is the process by which an article, surface
path of eruption.
or medium is freed of all living microorganisms either
l Relatively low radiation dose, convenience, speed
in the vegetative or spore state.
and ease.
Methods of sterilization:
l Useful in patients who are unable to open mouth.
l It can be achieved by both physical and chemical
Disadvantages:
methods.
l Lack of image detail for diagnosis of early carious
A. Physical
lesion.
a. Heat
l Cost of X-ray machine.
l Dry heat – Hot air oven, glass bead

sterilizer Q.5. Clark’s rule.


l Moist heat – Autoclaving
Ans.
b. Radiation
l Gamma rays l Clark’s technique is used to localize the buccolingual
l Ultraviolet rays position of supernumerary/impacted/unerupted teeth or
l Infrared rays foreign body, there are three techniques:
c. Sonic and ultrasonic vibrations l This is also called SLOB rule (Same side Lingual –

B. Chemical Opposite Buccal), tube shift localization technique


l Ethylene oxide gas or buccal object rule.
l Gluteraldehyde l It is based on principle of parallax which states that

when an object is viewed from two different positions,


Q.3. Intraoral periapical radiography in children. the object appears to move in two different directions.
Ans. l The basic principle is that the relative position of the

radiographic images of two separate objects changes


l Intraoral periapical radiographs are indicated to visual- when the projection angle at which the projection was
ize the roots, furcation and periapical areas of erupted made is changed.
teeth and to assess the developmental status of un- l Buccal object rule states that the image of a buccally
erupted permanent teeth. oriented object appears to move in the opposite direc-
l Intraoral films are available in different sizes, i.e. #0, #1,
tion from a moving X-ray source, and the image of any
#2. The 0 size (22 3 35 mm) which is most comfortable lingually oriented object appears to move in the same
to the child patient should be used. direction as a moving X-ray source.
l Once the film is positioned the X-ray beam should be

directed perpendicular to the film in the vertical plane. Q.6. Digital subtraction radiograph.
l There are essentially two methods of taking periapical Ans.
radiographs: paralleling and bisecting angle techniques.
l Each has benefits and limitations when used with the
l Digital subtraction radiography is a technique that
paediatric patient. allows determination of quantitative changes in radio-
l Regardless of which technique is used, film positioning
graphs.
l A series of radiographic images are taken before and
for the two techniques is identical. In all cases the iden-
tification dot is placed towards the occlusal surface. after treatment. The two images are digitalized and
compared. The resultant image shows only the changes
Q.4. Orthopantomograph. that have occurred and ‘subtracts’ those components of
the image that have not changed.
Ans.
Advantage:
l Orthopantomograph is also called maxillomandibular l The magnitude of changes that have occurred due
radiograph or pantomograph or rotational tomograph. to therapeutic intervention can be measured.
300 Quick Review Series for BDS 4th Year, Vol 1

Disadvantage: There is correlation between When the last tooth has been
l It is difficult to replicate the exact projection ge- the dental age and the skele- completed, the skeleton is
ometry and receptor placement thus the changes tal age. approaching complete
in the subtracted image may demonstrate false maturation
changes.
Q.9. Digital radiograph.
Q.7. SLOB rule.
Ans.
Ans.
l Digital radiography is a promising technology, which
l SLOB rule or the buccal object rule is one method of
has revolutionized dental imaging. It has provided new
localizing embedded or unerupted teeth.
diagnostic procedures that are not available with tradi-
l SLOB rule states that the image of any buccally ori-
tional film-based imaging.
ented object appears to move in the opposite direction
l Digital radiography is of two types:
from a moving X-ray source. On the other hand, the im-
i. Direct digital radiography, e.g. radiovisiography (RVG)
age of any lingually oriented object appears to move in
ii. Indirect digital radiography, e.g. subtraction radiog-
the same direction as a moving X-ray source.
raphy
Q.8. Skeletal age versus dental age. l In radiography the electronic sensors have gradually

replaced the radiographic film.


Ans.
l Charged coupled devices (CCD), complementary metal

oxide semiconductors (CMOS) and photo-stimulable


Skeletal age Dental age
phosphorplates (PSP) are used for capturing the image
It is based on the ossification It is based on the formation of
in digital radiography.
of endochondral bone. or eruption of the teeth.
It can be assessed based It is assessed on the basis of Q.10. Miller’s technique.
on the skeletal maturity the number of teeth at each
Ans.
indicators/hand-wrist chronological age or on stages
radiographs. of formation of crowns and To localize the buccolingual position of supernumerary/
roots of the teeth.
impacted/unerupted teeth or foreign body, there are three
The physical maturity of an The physical maturity of an techniques:
individual can be assessed individual is not related to i. Clark’s technique
by skeletal age. dental age.
ii. Miller’s technique
Assessing the skeletal matu- Dental age can reflect an as- iii. Cross-sectional occlusal radiograph
rity is helpful in treatment sessment of physiologic age l Miller’s technique is also called right-angle tech-
planning and response to comparable to age based on
treatment. the skeletal development,
nique.
weight or height. l It is used to achieve the same goal as Clark’s tech-

nique, but only in case of mandible.

Topic 3
Theories of Child Development
COMMONLY ASKED QUESTIONS
LONG ESSAYS:
1 . Explain the psychological development of a child according to Sigmund Freud.
2. Classify theories of child psychology. Describe in detail the cognitive development theory.
3.
Classify theories of child psychology. Describe in detail the classical conditioning theory.
4.
Describe the psychological development of child from birth through adolescent as per Sigmund Freud. 
[Same as LE Q.1]
5. Describe in detail psychosexual theory. [Same as LE Q.1]
Section | I  Topic-Wise Solved Questions of Previous Years 301

6. Enumerate the different theories of child psychology. Explain the different stages of Jean Piaget’s cognitive
theory in detail. [Same as LE Q.2]
7. Classify theories of child psychology. Write in detail about cognitive theory. [Same as LE Q.2]

SHORT ESSAYS:
1 . What is Oedipus complex and its significance? [Ref LE Q.1]
2. Id, ego and superego.
3. Oedipus complex. [Same as SE Q.1]
4. Oedipus conflict and Electra conflict. [Ref LE Q.1]

SHORT NOTES:
1 . Stimulus–response theory.
2. Superego. [Ref LE Q.1]
3. Trust versus mistrust.
4. Enumerate basic emotions of children. What is Id and ego. [Ref LE Q.1]
5. Oedipus complex. [Ref LE Q.1]
6. Sensorimotor period.

SOLVED ANSWERS
LONG ESSAYS:
Q.1. Explain the psychological development of a child
l It is present at birth, is impulse ridden and strives
according to Sigmund Freud.
for immediate pleasure and gratification without
Ans. regard to rules, realities of life or morals of any
kind.
l Psychology is the science dealing with human nature,
Ego:
function and phenomenon of his/her soul mainly.
l Ego is governed by ‘The Reality Principle’.
l Child psychology is the science that deals with the men-
l Ego tries to satisfy the Id’s urge of pleasure but
tal power or an interaction between the conscious and
only in the realistic ways that take account on
subconscious elements in a child.
what is possible in the real world.
l Sigmund Freud gave two theories:
l It is the mediator between Id and superego. It
A. Psycoanalytical theory
channelizes Id into a socially acceptable way.
B. Psycosexual theory
A. Psychoanalytical theory:
l This theory helps in understanding intrapsychic

process and personality development.


l This theory was proposed by Sigmund Freud

(1905). He compared human mind to an iceberg. {SN Q.2}


Elements of psychic triad:
i. According to Freud, personality is composed Superego:
of three parts and is known as psychic triad – l It is governed by ‘The Moral Principle’.

Id, ego and superego. l It develops around 5 years. It establishes and

ii. Each system has its own functions but the three maintains the persons moral conscious on the ba-
are required to be intact to govern the behaviour. sis of a complex of ideas and values, internalized
by parents.
l It stems from internalization of feeling good and
{SN Q.4}
bad, love and hate, praising and forbidding and
Id (instinctual drives): rewards and punishment.
l It is governed by ‘The Pleasure Principle’. l It is linked to a social consciousness derived in

l Id is that portion of mind that contains uncon- part from the familial cultural restrictions placed
scious drives for pleasure and destruction. on the growing child.
302 Quick Review Series for BDS 4th Year, Vol 1

B.  Psychosexual theory: vi. Over-emphasis by adults on toilet training will


l Freud believed that an individual progresses result in compulsive, obstinate and the perfec-
through several developmental stages that affect tionist behaviour in later life called ‘anal per-
personality. sonality’.
l According to Freud, development stages are classi- vii. Anal personality is characterized by abnormal
fied into: behaviour like disorderliness, abstinence, stub-
a. Oral stage: 0–1 year bornness, wilfulness and frugality.
b. Anal stage: 1–3 years viii. Less controlled toilet training results in an impul-
c. Phallic/Oedipal stage: 3–7 years sive personality in later life. The transition be-
d. Latency period: 7–12 years tween the anal and phallic stages is known as
e. Genital stage: 12–18 years urethral stage. It is also characterized by competi-
tiveness. Child derives pleasure from exercising
a. Oral stage: control over the urinary sphincter.
i. First year of child life is called oral stage and
is earliest stage of development characterized [SE Q.1 & SE Q.4]
by passiveness and dependency. {c. Phallic stage:
ii. The oral cavity is the primary zone of plea- i. The sex identification, which occurs between
sure because hunger is satisfied by oral stim- 3 and 6 years of age, is an important feature of
ulation. this stage.
iii. In infants oral cavity is the site for the identi- ii. During this stage the child explores and experi-
fying needs. Children will put their thumb or ences his/her genital organs as pleasurable.
anything else they can reach, into their Hallmarks of this stage:
mouths.
iv. This is a dependent stage, since the infant is
dependent on parents for their oral needs. {SN Q.5}
v. If the child does not receive the sufficient
gratification of pleasure at this age, fixation to Phallic stage is characterized by:
l Oedipus complex
this stage occurs, as the individual grows older.
l Electra complex
For example, oral dependency in the form of
digit sucking in older individuals. Oedipus complex:
l The child begins to direct their awakened sex-
vi. If the child’s needs are not adequately met in
this age, the following traits may develop: pes- ual impulses towards the parent of the opposite
simism, demandism, frustration and jealousy. sex.
l Attachment between young boys and mother
vii. Excessive oral gratification leads to excessive
optimism or narcissism. and they consider father as their enemy.
l The name oedipal complex comes from Greek
b. Anal stage:
i. This stage, also known as ‘Terrible tows’, oc- mythology. Oedipus the King of Thebe unwill-
curs between 1 and 3 years, and is marked by ingly slew this father and married his mother.
l The little boys strive to imitate their father to
the egocentric behaviour.
ii. During this stage, the anal zone becomes the gain affection of mother. Freud also described
principle zone of pleasure. Gratification is oedipal complex as a desire to have a sexual
derived from expelling or with hoarding relation with the mother.}
feces.
iii. During this stage, neuromuscular control oc-
curs. Control over the sphincter results in in- Electra complex:
creased voluntary activity. l Young girls develop an attraction towards their
iv. This stage is characterized by development of father and they resent the mother being close to
personal autonomy and independence. Child the father.
realizes the increased voluntary control that l The child realizes the sexual qualities without
provides him/her with the sense of indepen- embarrassment. In a phobic child these uncon-
dence and autonomy. scious, unacceptable wishes and feeling asso-
v. Child realizes his/her control over his/her ciated with Oedipal situation do not enter
needs and practices with a sense of shame. consciousness.
Section | I  Topic-Wise Solved Questions of Previous Years 303

l If the characteristic features of this stage are groups: psychodynamic theories and behaviour
not resolved, the balance between male and learning theories.
female roles does not develop. I. Psychodynamic theories:
Clinical significance: i. Psychosexual theory – Freud
l Keep the opposite sex during the child man- ii. Psychosocial theory – Eric Erickson
agement in the dental office/clinic. Keep iii. Cognitive theory – Jean Piaget
mother in case of boy or father in case of II. Behaviour learning theories:
baby girl during the child management.} i. Classical conditioning theory – Ivan Pavlov
d. Latency stage: ii. Hierarchy of needs – Abraham Maslow
i. This stage begins with resolution of Oedipus iii. Social learning theory – Albert Bandura
complex around the age of 5–6 years and ends iv. Operant conditioning theory – B.F. Skinner
with onset of puberty 6–12 years and is a Cognitive theory:
period of consolidation of sex roles. l The term ‘cognitive’ refers to element of perception,

ii. Maturation of ego takes place and the superego awareness and the ability to comprehend empirical
becomes firmly internalized. This stage is quiet knowledge.
as compared to the stages before and after. l The cognitive theory was proposed by Jean Piaget

iii. There develops a greater degree of control over (world’s leading theorist in the field of cognitive de-
instinctual impulses. velopment of children) in 1952.
iv. Child gains better sense of initiative and starts l He emphasized that childhood development pro-

adapting to the adverse environment. ceeds from an egocentric position through a pre-
v. The goal of this phase is the further develop- dictable step like consistent expansion by incorpo-
ment of personality. ration of learned experiences.
vi. Lack of inner control or excessive inner con- Concepts of cognitive theory are
trol results in an immature behaviour and de- i. Schemata
creased development of skill. ii. Assimilation and adaptation
e. Genital stage: iii. Accommodation
i. It begins with puberty and extends to young iv. Equilibrium
adulthood and is characterized by reopening of l Piaget names the major mechanisms that

ego struggle to gain mastery and control over allow children to progress from one stage of
the impulses of id and superego. cognitive functioning to the next stage as as-
ii. Child has a material personality. Sense of iden- similation, accommodation and equilibration.
tity develops. Helps to separate from the de- Schemata:
pendence of parents. l Schemata describe both the mental and physi-

iii. He/she can satisfy genital potency and realizes cal actions involved in knowing and under-
his/her goals for reproduction and survival. standing.
iv. Fluctuating extremities in emotional behaviour Assimilation:
and preoccupation with philosophical and ab- l Assimilation describes the ability of the child

stract thoughts predominate due to struggle to to deal with new situations and problems
attain a firm sense of self. within his/her age-specific skills.
v. The individual becomes more matured and The process of taking in new information into
will be able to make right decision and pre- the previously existing schemas is known as
pared to face the world. assimilation.
Q.2. Classify theories of child psychology. Describe in Accommodation:
l Accommodation is an individual’s tendency to
detail the cognitive development theory.
modify action to fit into a new situation.
Ans.
It is a process which enables him/her to adapt
l Psychology is the science dealing with human nature, and change his/her way of dealing with the
function and phenomenon of his/her soul in the main. world to handle a problem.
l Child psychology is the science that deals with the men- Equilibrium:
tal power or an interaction between the conscious and l Equilibration refers to changing basic assump-

subconscious elements it child. tions following adjustments in assimilated


Theories of child psychology: knowledge so that the facts fit better.
l Many theories have been proposed to explain and As a result of new knowledge, the child is tem-
identify the processes involved in personality de- porarily in a state of equilibrium or cognitive
velopment. These theories can be divided into two harmony.
304 Quick Review Series for BDS 4th Year, Vol 1

Merits of cognitive theory: l Sensorimotor stage starts with automatic in-


l It is the most comprehensive theory of de- form reflexes which include the ability of child
velopment. to suck, cry, move his/her arms and legs, track
l It illustrates that we can learn as much a moving object and orient to a sound. Then
about children’s intellectual development coordination of these reflexes improves.
from examining their incorrect answers to l Children acquire memory or object perma-

the items as from examining their correct nence at about 7 months of age. Physical de-
answers. velopment leading to mobility helps the child
Demerits of cognitive theory: to develop new intellectual abilities.
l Vagueness about the process of change l The actions of infants are increasingly goal-

l Overestimates age differences in thinking directed. Piaget describes this behaviour as


l Underestimates children’s abilities and role genuinely adaptive and intelligent at a behav-
of social environment ioural level but not totally concrete and not
Stages of development according to cognitive theory: accompanied by cognitive awareness of the
According to Piaget, there are four distinctive periods causality or outcome of action.
of cognitive development under two broad categories: l By the end of the 1st year, children will learn

A. i.  Sensorimotor period (birth to 2 years) to coordinate their sensorimotor schemes bet-
B. Period of conceptual intelligence (2 years and ter, to attain an external goal. For example,
above) child will pack up a cover in order to retrieve a
ii. Preoperational period (2–7 years) try he/his saw his father place their earlier.
a. Preconceptual stage (2–4 years) l Towards the end of the sensorimotor period,

b. Intuitive stage (4–7 years) child learns to invent new schemes through a
iii. Period of concrete operations (7–11 years) kind of mental exploration in which they imag-
iv. Period of formal operations (11 years and ine certain event and outcomes.
above) l By the end of sensorimotor stage the child will

l Each stage is characterized by distinct have transformed himself/herself from an or-


types of thinking and in which the child ganism totally dependent on reflex and other
successfully relies more upon internal hereditary equipment to a person capable of
stimuli and symbolic thought and less symbolic thought.
upon external stimulation. Dental applications:
i. Sensorimotor period: l Child begins to interact with the environment

l It lasts from birth to 2 years of age. Object per- and can be given toys while sitting on the den-
manence, primitive beginning of symbolic tal chair.
thought and animism are the hallmarks of this ii. Preoperational stage:
stage. l Preoperational stage is divided into – precon-

l It is the stage of practical intelligence. During ceptual period (18 months to 4 years) and in-
this period infants are busy discovering rela- tuitive period (4–7 years).
tionships between the body and the environ- l The essential characteristic of this stage is the

ment (seeing, touching, sucking, etc.). manipulation of symbols or words. This mani-
l It can be further divided as follows. fests in delayed imitation and children’s imag-
inative or pretend play.
Egocentrism:
Sensorimotor l Egocentrisim refers to child’s tendency to
Substage Period of time development conceptualize no point of view other than
i l Birth to l The use of reflexes his/her own.
ii 1 month l Primary circular reactions l The child is incapable of assuming or think-
iii l 1–4 months l Secondary circular
ing the role of another person.
iv l 4–8 months reactions
l During this stage the child has difficulty in
v l 8–12 months l Coordination of secondary

vi l 12–18 months schemata and their appli- understanding life from any other perspective
l 18–24 months cation to new situations other than his/her own.
l Tertiary circular reactions
l His/her thinking is so egocentric that
l Invention of means
when he covers his eyes, he/she thinks
through mental combi-
nations that since he/she can’t see you, you can’t
see him/her.
Section | I  Topic-Wise Solved Questions of Previous Years 305

Concentrate: l The main limitation of this stage is their inability


l At the school age level a child concentrates to reason about abstraction, hypothetical proposi-
on only one aspect of a situation and ne- tions or imaginary events.
glects the importance of other aspects. Dental applications:
l During this stage the child’s thinking is self- l Concrete instructions like this is a retainer, brush

centred. The child is very ‘me, myself and I, like this can be given to the child.
oriented. l Abstract instructions, like wear the retainer

Animism: every night and keep clean, can be given to the


It means imparting life to inanimate objects. child.
His/her view of world is animistic or artificial- l Child can be allowed to hold the mirror to see

istic and he/she cannot distinguish what is real what is being done on his/her teeth.
from what is not real. iv. Formal-operational stage:
For example, if hurts himself/herself by bumping l It begins approximately around age of 11 years

against the door, he/she may hit the door as he/she and continuous through adulthood. The indi-
feels that it gets hurt or will be happy if the parents vidual is highly versatile and flexible in thought
slam at the door. and reasoning.
Dental applications: l Child acquires ability to deal with abstract

Constructivism: concepts and abstract reasoning. This stage


l The child likes to explore things and make is characterized by the ability to formulate
own observations. For example, child sur- hypothesis and systematically test them
veys the dental chair and airway syringe. to arrive at an answer to a problem. This
Cognitive equilibrium type of thinking is important in long-term
l Child is explained about the equipment planning.
or instrument and allowed to deal with it. l Thinking becomes quite adult-like, most adult

For example, airway syringe. capabilities are thought to be in place by the age
Animism: of 16 years; hence, the child should be treated as
l Child correlates things with other objects adult at this stage.
to which they are more used to or accus- l Adolescents feel that they are constantly ‘on

tomed. For example, explaining about stage’ being observed and criticized by others;
radiograph as tooth picture. this phenomenon is called ‘imaginary audi-
iii. Concrete-operational stage: ence’.
l During this stage child is about 7–11 years l The second phenomenon is called ‘Personal

age and his/her thinking is still strongly tied Fable’ in which they think they are unique. It is
to concrete situation and has limited abstract a powerful motivator that allows them to cope in
reasoning. Animism declines. a dangerous world.
l During this stage child begins to reason logically Dental applications:
and organizes thoughts coherently. l Peer influence and abstract thinking increases.

l They are able to decanter, that is, they focus This can play an important role in orthodontic
their attention on several attributes of an object appliances and braces.
or even understand the relations between di- l Acceptance from peers can be used for motiva-

mensions. For example, same quantity of milk tion for dental treatment.
in tall and thin glasses and short and broad Q.3. Classify theories of child psychology. Describe in
glasses. detail the classical conditioning theory.
l During this stage child has the ability to ar-

range objects according to some quantified Ans.


dimensions such as weight/size/shape/colour. Many theories have been proposed to explain and identify
This is called serration. For example, arrange- the processes involved in personality development. These
ment of different sizes of colour blocks ac- theories can be divided into two groups: Psychodynamic
cording to size. theories and behaviour learning theories.
l Child acquires the concepts of time, space, num- I. Psychodynamic theories:
bers and logic. He/she integrates his/her own ex- i. Psychosexual theory – Freud
periences with other experiences he/she has read ii. Psychosocial theory – Eric Erickson
or observed in others. iii. Cognitive theory – Jean Piaget
306 Quick Review Series for BDS 4th Year, Vol 1

II. Behaviour learning theories: l The acquisition of conditioned response is


i. Classical conditioning theory – Ivan Pavlov usually gradual and requires more trials but
ii. Hierarchy of needs- – Abraham Maslow once acquired is more strong.
iii. Social learning theory – Albert Bandura ii. Generalization:
iv. Operant conditioning theory – B.F. Skinner l Generalization which means stimuli similar

Classical conditioning theory: to the original conditional stimulus also


l It is also known as stimulus–response theory or evoke a conditioned response.
Pavlov conditioning theory. For example, development of phobia in
l Classical conditioning is a type of learning in children to a specific environment or action
which stimulus acquires the capacity to evoke may be due to stimulus generalization most
response that was originally evoked by another of the time.
stimulus. iii. Discrimination:
Pavlov experiment: Discrimination is the opposite of generaliza-
l Classic conditioning theory is the result of tion. Learning to discriminate or learning to
the classic experiments of Ivan Pertovich respond only to certain stimuli but not to re-
Pavlov. spond to others.
l Presentation of food to a hungry dog along For example, if the child is exposed to clinic
with some other stimulus like ringing bell is settings, which are different, those associated
done repeatedly. The stimulus evoked by with painful experiences the child learns to
sound of ringing bell becomes associated discriminate between the two clinics.
with food presentation stimulus. iv. Extinction:
l He trained the dog by sounding the bell and l The conditioned response gradually disappears

shortly afterward presented the food. After when the unconditioned stimulus is removed.
few sessions the dog would salivate at the l Extinction of the conditioned behaviour re-

sound of the bell. This is the conditioned re- sults if the association between the condi-
sponse what the dog has learned. tioned and the unconditioned response is
l The unconditioned stimulus is a stimulus that not reinforced.
evokes an unconditioned response without v. Spontaneous recovery:
previous conditioning. Unconditioned re- The reappearance of an extinguished response
sponse is an unlearned reaction to an uncon- after a period of nonexposure to the extin-
ditioned stimulus that occurs without previ- guished stimulus.
ous conditioning. Dental applications:
l The conditioned stimulus is previously neu- l Simple to understand and very applica-

tral stimulus that has through conditioning ble on a child in dental clinic.
acquired capacity to evoke a conditioned l Three-year-old child taken to paediatri-

response. cian – nurse comes with needle – child


l In Pavlov’s experiment, presentation of food given injection badly – child cries.
is unconditioned stimulus and salivation is Next visit – sight of needle – child cries (needle that was a
unconditioned response. Here ringing bell is neutral stimulus has become a Conditioned stimulus (CS).)
the neutral stimulus, over a period of time it ● First visit
becomes conditioned stimulus and salivation White coat Pain on injection
becomes conditioned response. (Neutral stimulus) (Unconditioned stimulus)
Process of classical conditioning:
The processes included under classical conditioning Pain on injection Fear and crying
are (Unconditioned stimulus) (Response)
i. Acquisition
● Second visit
ii. Generalization
iii. Descrimination Sight of white coat Pain on injection
(Conditioned stimulus) (Unconditioned stimulus)
iv. Extinction
v. Spontaneous recovery Pain on injection Fear and crying
i. Acquisition: (Unconditioned stimulus) (Response)
l Acquisition is learning a new response or

forming a new response tendency due to White coats, hospital atmosphere, etc. (neutral stimuli –
conditioning. NS) associated with pain, fear (unconditioned response)
Section | I  Topic-Wise Solved Questions of Previous Years 307

producing procedures (unconditioned stimulus) will pro- Id (pleasure Ego (reality Superego (ethics
duce fear (conditioned response) in future, even when not principle) principle) by morals)
associated with original unconditioned stimulus, here, the
2. It is governed 2. It is determined
‘neutral stimulus will become conditioned stimulus’. With by reality prin- by regulations im-
repetition, there will be reinforcement of the unconditioned ciple and is posed on child by
response (fear). concerned with parents, society
memory and and culture (eth-
Q.4. Describe the psychological development of child judgement. ics and morals).
from birth through adolescent as per Sigmund Freud.
3. It develops af- 3. It is internalized
Ans. ter birth ex- control which
pands with age produces the feel-
[Same as LE Q.1] and it delays, ing of shame and
modifies and guilt.
Q.5. Describe in detail psychosexual theory. controls Id im-
pulses on a re-
Ans. alistic level; it
[Same as LE Q.1] is mediation
between Id and
Q.6. Enumerate the different theories of child psychol- superego.
ogy. Explain the different stages of Jean Piaget’s cogni-
tive theory in detail. Q.3. Oedipus complex.
Ans. Ans.
[Same as LE Q.2] [Same as SE Q.1]
Q.7. Classify theories of child psychology. Write in detail Q.4. Oedipus conflict and Electra conflict.
about cognitive theory.
Ans.
Ans.
[Ref LE Q.1]
[Same as LE Q.2]
SHORT NOTES:
SHORT ESSAYS:
Q.1. What is Oedipus complex and its significance. Q.1. Stimulus–response theory.
Ans. Ans.
[Ref LE Q.1] l A theory based on stimulus–response reflex given by
Ivan Petrovich Pavlov is known as stimulus–response
Q.2. Id, Ego and Superego.
theory.
Ans. l It was developed through experimentation with dogs.

Stimulus–response reflex is an involuntary response to


Psychoanalytical theory was given by Sigmund Freud (1905). an external stimulus.
This theory of personality was based on the interaction l When two stimuli occur together at the same time, re-
between three systems called Id, ego and superego within each sulting in a response, this response can be also obtained
individual.
by stimulating anyone of the original stimuli.
Psychic structure l In dentistry we can use this theory for stimulating the

development of good habits, breaking old habits, to


remove fear and to develop positive attitude.
Three parts proposed by Freud in psychodynamic theory
Q.2. Superego.
Id (pleasure Ego (reality Superego (ethics
principle) principle) by morals) Ans.
1. Basic structure 1. Develops in 1. It is prohibition [Ref LE Q.1]
of personality 2nd–6th month learned from en-
present at birth of life when in- vironment (more Q.3. Trust versus mistrust?
impulse ridden fant injury to from parent and
and strives for distinguish be- authorities).
Ans.
immediate tween itself l The trust versus mistrust is dealt in Stage I of psycho-
pleasure and and outside
gratification. world.
logical theory given by Eric H. Erikson which extends
over 1st year of life.
308 Quick Review Series for BDS 4th Year, Vol 1

l Basic needs are met by the person whom he trusts. l Piaget has described four major periods of cognitive
When these are not met he develops mistrust. Child with growth, among which the sensorimotor period is the
a sense of trust may have the sense of mistrust activated first stage extending from birth to 18 months, in which
at a later stage such as parents getting divorced. an integration of sensory modalities is learned by the
l Hope and danger present. infant.
For example, child with a sense of mistrust may come to l In this period the infant responds in a relatively undif-

trust a particular person like a teacher who has taken the ferentiated reflexive pattern. He/she learns to integrate
trouble to be trust worthy developing hope. the sensory modalities and is able to look towards the
object and make a sound or reach and hold the object
Q.4. Enumerate basic emotions of children. What is Id
he/she desires.
and ego?
l It lasts from birth to approximately 2 years of age: Ob-

Ans. ject permanence, primitive beginning of symbolic


thought and animism are the hallmarks of this stage.
[Ref LE Q.1]
l During this stage, infants understand their environment

Q.5. Oedipus complex. by physically manipulating the world around them.


l Knowledge of the world is limited to sensory percep-
Ans.
tions and motor activities. Behaviour is limited to sim-
[Ref LE Q.1] ple motor responses to sensory stimuli.
Q.6. Sensorimotor period.
Ans.

Topic 4
Parent Counselling and Child Behaviour
COMMONLY ASKED QUESTIONS
LONG ESSAYS:
1 . Discuss the importance of first appointment for the child patient.
2. Enumerate the factors influencing child’s behaviour. Describe the parental influences on the behaviour of chil-
dren during dental treatment.
3. Describe in detail child abuse and dental neglect.

SHORT ESSAYS:
1. Child abuse. [Ref LE Q.3]
2. Parent counselling.
3. First dental visit. [Ref LE Q.1]
4. Parental attitudes and behaviour of children. [Ref LE Q.2]
5. Types of fear.
6. Write briefly about examination of abused child. [Same as SE Q.1]
7. Orofacial signs of child abuse. [Same as SE Q.1]
8. Importance of child’s first dental visit. [Same as SE Q.3]
9. Write briefly about parent–child relationship. [Same as SE Q.4]
10. What is objective fear and subjective fear? [Same as SE Q.5]
11. Fear in paedodontia. [Same as SE Q.5]

SHORT NOTES:
1 . Define fear and name the types of fear. [Ref SE Q.5]
2. Communication.
Section | I  Topic-Wise Solved Questions of Previous Years 309

3 . Different types of cry.


4. Child abuse. [Ref LE Q.3]
5. Anticipatory guidance.
6. Parent counselling. [Ref SE Q.2]
7. Dental neglect.
8. Types of cry. [Same as SN Q.3]
9. Type of child abuse. [Same as SN Q.4]

SOLVED ANSWERS
LONG ESSAYS:
Q.1. Discuss the importance of first appointment for the l A packet mailed before the first examination is an
child patient. excellent introduction to the practice. The items
that a packet might contain are as follows:
Ans.
i. A brochure that provides information about
the practice, the practice web address, a list of
[SE Q.3]
services offered, general advice about how
{ It is generally recommended that a child’s first visit be
l parents can best help children prepare for the
made at no later than 3–4 years of age. first dental appointment, a map to the office
l Dental care for children has been designed primarily to and a reminder of appointment date and time.
prevent oral pain and infection, the occurrence and ii. A medical health history form stamped in red
progress of dental caries, the premature loss of primary ink in an upper corner, ‘Please complete and
teeth, the loss of arch length and the development of an bring with you’.
association between fear and dental care. iii. A note of welcome addressing the young patient.
Importance of first appointment for a child patient is l The physical layout of the reception area and
as follows: business desk must allow staff members a full
l A good first appointment experience provides the view of the reception room. On arrival, the patient
foundation for an enjoyable, long-term relation- and parents should be greeted by a team member
ship with patients and parents. whether or not a sign-in sheet is used.
l Unless the young child presents with an acute l A well-trained team member should escort new
dental problem, the first visit usually involves patient arrivals to a conference room or the opera-
only an examination, radiographic evaluation and tory. At this time, the staff person should review
if possible a prophylaxis and topical fluoride the child’s health history with parents, recording
treatment. It is readily accepted by most of the pertinent notes for the dentist.
children. l The patient and parents may be given a brief tour
l The impression given on the very first visit is long of the office, and the tour guide can reinforce in-
lasting, although walk-in emergencies occur oc- formation given in the practice brochure.
casionally, most initial contacts with a practice are l The dentist should then be introduced by the staff
by telephone. member who interviewed the new patient and par-
l A caller frequently judges the entire practice by a ents. After a short conversation addressed mainly
voice on the telephone, the business staff should to the patient, the dentist should again review the
be trained in correct telephone etiquette and pro- health history with the parents, reading notes made
cedures. by the staff member during the interview.
l During the initial telephone conversation, certain l The above routine provides two opportunities to
information should be gathered from the parent- ensure that the health history has been correctly
ing adult. completed and that the review with parents is
l Use of a form can help to standardize the proce- documented.
dure. A completed form about patient details by l If the initial conversation occurred in a conference
the receptionist and used during the first visit is an room, the dentist should leave while the team
invaluable aid in patient registration and can be member prepares to take the child to the opera-
kept permanently in the patient chart. tory. In this way, if the child is upset, the dentist
310 Quick Review Series for BDS 4th Year, Vol 1

who must gain full cooperation during the dental d. Social and adaptive skill
examination is not involved. e. Position of the child in the family
l In many offices, parents accompany their child to II. Factors involving the parents:
the operatory. Although this often adds stress and a. Family influence
causes the child’s behaviour to be worse, the pres- b. Parent–child relationship
ence of parents in the operatory is a reality in c. Maternal anxiety
many paediatric dental offices in today’s litigious d. Attitude of parents to dentistry
society. III. Factors under the control of dentist:
l The dentist has to wisely limit the number a. Environment of the dental office
of adults accompanying each patient to the b. Personality of the dentist (his/her attitude and attire)
operatory. c. Scheduling of appointment
l After the examination, the dentist should present d. Dentist’s skill and speed
the case to the parents who usually want answers e. Use of fear promoting words, use of subtle, flattery,
to the following questions: What is wrong? Can it praise and reward
be fixed? How much will it cost? The practitioner f. Presence of parents in the operatory
should keep these questions in mind and plan case I. Factors involving the child:
presentations accordingly. a. Growth and development
l Many new-to-practice dentists over talk during i. The genetic, familial, cultural, interpersonal
case presentations, confusing parents and patients and psychic factors influence the growth and
with excessive clinical terminology. development processes of a child.
l At some point during the first appointment, an ii. Most children demonstrate emotional matura-
assistant or hygienist should give home care in- tion along with physical growth.
structions. The patient and parents are then es- iii. During maturation the child’s behaviour is sys-
corted to a checkout area. temically affected by the inherent genetic makeup.
l The next appointment is usually scheduled before iv. With each new experience, a new behaviour
the fee is collected. After checkout, a staff mem- develops as directed by the child’s internal sys-
ber should express thanks to the parents for tem from his/her motivation and from the con-
choosing the office and remind them that the prac- sequences of his/her behaviour.
tice appreciates referrals. b. IQ of the child:
l The child and parents should leave the office feel- i. IQ assessment is the method of quantifying the
ing that the dentist and staff are extraordinarily mental ability in relation to chronological age.
caring, thorough, skilled and efficient. ii. It is assessed and measured by tasks, examining
l The first appointment should make parents eager memory, spatial relationship, reasoning, etc.
to recommend the practice to other family mem- iii. A positive relationship exists between IQ and
bers and friends. the acceptance of dental treatment.
l Except for the emergency treatment or noninva- c. Past dental experience:
sive procedure it is wise not to perform any other i. A fear might develop in the child towards den-
treatment during the first visit, even in a very co- tal treatment, due to any of the previous painful
operative child. experiences.
l The highest rates of uncooperative behaviour dur- ii. It is very important to alleviate this fear before
ing the first session occur when the child is sepa- one can accept a tolerable behaviour.
rated from his/her mother and during the taking of d. Social and adaptive skills:
the radiograph. This behaviour may be due to the i. The level of child’s social and adaptive skills is
fear of abandonment, which is common in chil- an important aspect of the overall functioning
dren younger than four.} of the child.
ii. It is important to consider how effective the
Q.2. Enumerate the factors influencing child’s behav-
child is in meeting the standards for personal
iour. Describe the parental influences on the behaviour
independence and social responsibility in ev-
of children during dental treatment.
eryday situations.
Ans. e. Position of the child in the family:
The various factors influencing behaviour of a child are as The child’s position in the family also influences
follows: their behaviour as follows:
I. Factors involving the child: i. 1st child:
a. Growth and development Uncertainty, mistrustful, insecurity, stinginess,
b. IQ (intelligent quotient) of the child dependence, responsibility, authoritarianism,
c. Past dental experience jealousy, sensitive, etc.
Section | I  Topic-Wise Solved Questions of Previous Years 311

ii. 2nd child: l Conversely, punitive mothers and those who ig-
Independence, aggressive, extrovert and ad- nored their children did not exhibit these positive
venturesome behavioural characteristics. Friendly, cooperative
iii. Middle child: child will probably also exhibit these traits in the
Aggressiveness, easily distracted, infertility dental office.
and prone behaviour disorders
iv. Last child: [SE Q.4]
Secure, confident, immature, envy, irresponsi- {b. Parent–child relationship
ble, spontaneous good and bad behaviour l Characteristics of parent child relationships that may
II. Factors involving the parents: affect child’s behaviour in the dental office are as
a. Family influence: follows:
l The home environment is an important factor in i. Overprotective/overanxious
the development of a child’s personality and his/
ii. Overindulgence
her behaviour patterns. iii. Underaffection and rejection
l Socioeconomic status of parents has some iv. Domination
consideration in behaviour modulation in den- v. Identification
tal office. Parents belonging to low socioeco- vi. Authoritarian
nomic status show authorization in controlling
the child than the middle- and high-income i. Overprotective/overanxious parents:
groups. Factors responsible for maternal overprotection
l Maternal influence on the children’s mental, may be:
physical and emotional development begins l History of previous miscarriage or a long delay

even before birth. in conception.


Mother’s nutritional status and the state of phys- l Family’s financial condition.

ical health can affect the neurological as well as l Death of a sibling.

somatic development of the fetus, which di- l Serious illness or handicapped condition.

rectly influences the children’s mental, physical l Parental absence by divorce or death.

and emotional development. l This overprotective attitude is characterized

l Parent–child relationship: by undue concern for the child in terms of


Parents play an important role in the child’s feeding, dressing, bathing, etc.
psychological development but more emphasis l Parents constantly involve with child’s daily

is on the mother. social activities and may not allow him/her


l Bell has termed the parent–child relation- to participate in risk involving games/
ship as ‘one tailed’, since parental charac- situations.
teristics are viewed as having a unilateral l The overprotective mother retards the normal

influences on those developing in the psychologic maturation of the child and tends
child. to infantize him/her.
l According to this theory, the child’s char- l The child is not permitted to play alone.

acteristics including the personality, be- l The overprotective mother is associated with a

haviour and reaction to stressful situation submissive, shy and anxious child. These chil-
are the direct product of various maternal dren are usually shy, timid and fearful.
characteristics. l Aggressive child, demanding and expects con-

l It was found that loving mothers tend to stant attention and services.
have calm, happy children, while hostile l They lack the ability to make decisions for

mothers tend to have children who are themselves.


excitable and unhappy. l With encouragement and assurance the child

l Most of the relevant mother–child rela- usually responds in a satisfactory manner. The
tionship falls into two broad categories: child will be a cooperative dental patient.
i. Autonomy versus control ii. Overindulgence:
ii. Hostility versus love l It may be associated with overprotective or

l Mothers who allowed autonomy and who dominant natural trait.


expressed affection had children who l Relatives such as grandparents are also over-

were friendly and cooperative. indulgent.


312 Quick Review Series for BDS 4th Year, Vol 1

l The parents give the child whatever he/she might l With kindness and consideration they
want, as far as financially possible including toys, generally develop into good dental
candy and clothes. patients.
l They usually place very little restraint upon their v. Identification:
child’s behaviour. l This type of parents try to relive their own

The behaviour of this type of child is as follows: lives in those of their children.
l Child is spoiled and is accustomed to getting l In doing so, they try to give the child every

his/her own way. advantage denied to them.


l His/her emotional development is impeded, l If the child does not respond favourably, the

and he/she is aggressive, demanding and dis- parent shows disappointment and the child
plays temper tantrums. has a feeling of guilt.
l He/she is usually incapable of amusing him- The behaviour of this type of child is as follows:
self/herself and keeps the adults around him/ l He/she cries easily and lacks confidence.

her busy devising diversion for him/her. l These children should be handled kindly

l In the dental office, when they cannot control and with consideration.
the situation the way they control the situation vi. Authoritarian:
at home they may show bursts of temper. l Some parents choose nonloving-oriented

iii. Underaffection and rejection: techniques for controlling child behaviour.


l Underaffection may vary from mild detachment l Discipline takes the form of physical pun-

to indifference to neglect. ishment or verbal ridicule.


l Mother becomes less emotionally supportive of l The mother feels that the child should

her child due to her outside interests, employment follow her set of norms and ideas.
or because the child is unwanted. The behaviour of this type of child is as follows:
l She may neglect the child, gives severe punish- l The response of the child will be submis-

ment, nagging and resistant to spend time and sion, coupled with resentment and evasion.
money on the child. l This type of child will not directly dis-

The behaviour of this type of child is as follows: obey a command, he/she has heightened
l These children are well behaved and outwardly avoidance gradient.
appear to be well adjusted. l Parents are nonsupportive and are criti-

l They however develop resentment and become cizing them. Therefore, these children
completely withdrawn to a shell. are often afraid of dentists and resist the
l Since they have not experienced love and dental treatment.}
affection at home, such a child usually lacks a c. Maternal anxiety
feeling of belonging or worthiness. l Highly anxious parents tend to affect their child’s

l He/she may show anxiety, cry easily, unable behaviour negatively.


or unwilling to cooperate and he/she will re- l Although the scientific data revealed that children
sort to any behaviour to attract attention. of all ages can be affected by their mother’s anxi-
l Such children are usually demanding and at eties, children under the age of 4 years are greatly
extreme, rejection can lead to rivalry. affected.
l They respond well to a dentist who gives them d. Attitude of parents to dentistry
emotional support and affection. l Parents with positive dental attitude will develop

iv. Domination: the same in the child, whereas fearful parent may
l Parents with dominant attitude demand from their develop fear unknowingly in a child.
children excessive responsibility, which is incom- III. Factors under the control of dentist:
patible with their chronological age. a. Environment of the dental office:
l They cannot accept the child as he/she is, but com- l The physical layout of the reception area and

pare him/her with others older to him/her. business desk must allow staff members a full
l They force the child and criticize him/her and this view of the reception room. On arrival, the pa-
results in resentment, evasion, submission and tient and parents should be greeted by a team
restlessness in the child. member whether or not a sign-in sheet is used.
The behaviour of this type of child is as follows: l The patient and parents may be given a brief tour

l They are fearful of resisting openly and will of the office, and the tour guide can reinforce
obey commands slowly. information given in the practice brochure.
Section | I  Topic-Wise Solved Questions of Previous Years 313

l Since the child may enter the dental office with l In praising a child, it is better to praise the behav-
some fear, the paedodontist or his/her auxiliary iour that of the individual.
must put the child at ease, waiting room must be l Tiny gifts such as alphabet erasers, tiny gold stars,

made comfortable by having one corner set aside toys or stickers make good gifts.
for their own use. The corner must have books, l The recognition makes the child more happy than

toys and chairs set aside for them. the material, flattery can be used as a reward after the
l Operating room can be made more appealing by treatment.
having pictures of laughing, playing and carefree
children.
Q.3. Describe in detail child abuse and dental neglect.
l The child patient should not see others in pain or

blood. Ans.
b. Personality of the dentist (his/her attitude and attire):
l The approach of the dentist should be casual,
(SE Q.1 and SN Q.4)
confident and friendly towards the child.
l The dentist should never loose his/her temper as this

will create feeling of success in the mind of the child {(l Child abuse and neglect (CAN) is defined ‘as any
and will ruin the child for all future dental visits. interaction or lack of interaction between a caregiver
l Always call the child by his/her nickname or at least
and a child resulting in nonaccidental harm to the
the first name, when approaching a new child patient. child’s physical or developmental state’.
l Child abuse and neglect include various experiences
l All conversations should be directed towards him/

her. They must include the subject of interest to of a child which are threatening or harmful. These
the child and never underestimate the intelligence are always the result of improper conduct and re-
of the child. sponsibility of a caretaker or parents.
l Emotion expressed actively or passively against the
c. Scheduling of appointment (time and length of
appointment): child is often unplanned, but nonetheless can result
l Both time and length of appointment are impor-
in significant harm or death.
l Child maltreatment is usually divided into following
tant, when dealing with children.
l Children cannot sit in one position for longer time
categories:
and their threshold of tolerance is very low; i. Physical abuse
hence, they should not be kept in the chair for ii. Sexual abuse
periods longer than 30 min. iii. Emotional or psychological abuse
l With longer appointment the children tend to be-
iv. Failure to thrive
come less cooperative. Once they loose their self- v. Intentional poisoning/drugging
composer, the cooperation is very difficult to regain. vi. Munchausen syndrome by proxy
l Children should not be given appointment during
vii. Neglect in its many forms
their naptime. viii. Health care neglect
ix. Safety neglect
d. Dentist’s skill and speed:
x. Educational abuse)}
l To avoid any loss of time, the dentist should per-

form his/her duties with dexterity, in a preplanned


manner. i. Physical abuse:
l A child can endure discomfort if he/she knows it l It forms 60% of the child abuse–related
is soon going to end. fatalities.
e. Use of fear promoting words: l It includes injuries that are inflicted upon child
l Avoid the use of fear promoting words such as that result from punishment, which are inap-
needle, injection, etc. propriate for the child’s age.
l The alternative words that can be used are called l These type of injuries may be inflicted by the
as euphemisms. parents, relatives or baby sitters.
For example: Mosquito bite – Needle prick l Groups living in poverty may have an in-
Raincoat – Rubber dam creased incidence of child abuse because of
Tooth paint – Sealant the increased number of crisis in their lives.
f. Use of subtle, flattery, praise and reward: l Over 90% of abusing parents have neither
l One of the most important rewards sought by the psychotic nor criminal personalities, tend to
child is approval of the dentist. be lonely, unhappy and angry adults under
314 Quick Review Series for BDS 4th Year, Vol 1

stress. They injure their children in anger after l Such children are usually younger than 2 years
being provoked by some misbehaviour. and are malnourished and are underweighted.
l The physical injuries sustained may be: l The mother may neglect to feed, because she is

Mild – few bruises, scratches, etc. busy with external problems, preoccupied with
Moderate – numerous bruises, minor burns or inner problems or does not like the infant.
single fractures. l Most of the mothers feel deprived and un-

Severe – large burns, CNS injury, abdominal in- loved themselves and are acutely or chroni-
jury, multiple fractures or other life-threatening cally depressed.
injuries. v. Intentional poisoning/drugging:
l Children living in violent homes are increas-
l Intentional overdrugging or poisoning consists
ingly recognized as victims of maltreatment. of the intake of harmful drugs, not intended
l Identification, treatment and intervention are
normally for use in a child.
the tasks of professionals from multidisci- l It is an uncommon lethal type of child abuse.
plinary backgrounds working together to pro- Parents who poison their children may have se-
vide care and evaluation in the best interests of vere marital problems or may be drug abusers.
the child.
l Education and prevention efforts may teach par-
vi. Munchausen syndrome by proxy:
l In Munchausen syndrome by proxy, children
ents to redirect their actions and explore more
appropriate discipline techniques and ways to younger than 6 years and too young to reveal the
manage anger or frustration. deception exhibit parentally fabricated or in-
ii. Sexual abuse: duced illness.
l The induced symptoms and signs lead to unnec-
l The National Center on Child Abuse and Ne-

glect defines sexual abuse ‘to include contacts essary medical investigations, hospital admis-
or interactions between a child and an adult’. sions and treatment.
l Any kind of intentional sexual misuse on a child
Various factitious signs and symptoms include:
by a person is termed as sexual abuse. i. Bleeding from various sites. If specimens are
iii. Emotional abuse and neglect: required, the parent adds his/her own blood to
l Emotional abuse is defined as ‘the continual
them.
scapegoating and rejection of a child by parents, ii. Recurrent sepsis from injecting contaminated
caretakers or teachers’. fluids.
l Any form of abnormal behaviours or mental
iii. Chronic diarrhoea from laxatives.
health problems that harm a child are called iv. Fever from rubbing or heating thermometers.
emotional abuse, e.g. continuous isolation ac- v. Rashes from rubbing the skin or applying
tion, degradation, terrorization, exploitation or caustic substances.
denial of affection. vii. Neglect in its many forms:
l Emotional and verbal abuse involve interac- l Neglect consists of failure to provide the neces-

tions or lack of interactions on the part of the sities of adequate food, shelter, clothing and
caretaker that inflict damage on the child’s also health care needs according to accepted or
personality, emotional well-being or devel- appropriate standards. Neglect is a chronic form
opment. of child abuse.
l Harm to the child generally occurs in various Physically neglected children tend to exhibit at
ways over a prolonged period. least several of these characteristics.
l It is often difficult to demonstrate the direct or a. Dirty clothing and skin with foul smell, lice
causal link between the emotional and verbal and unkempt appearance.
abuse and the harm to the child. Such harm is b. Under-nourished.
usually seen as abnormal behaviours or men- c. Rampant caries, abscess, periodontal lesions, etc.
tal health problems that are multifactorial in d. Uncared wounds.
origin. e. Constant sleepiness or hunger.
iv. Failure to thrive: g. Pain and emergency type situations are the
l Failure to thrive due to lack of attention to di- only reasons for the patients appearing at the
etary intake of child by parent or caretaker, the dental office.
child becomes malnourished and underweight h The reason for neglect may be parents’ illness,
and exhibits failure to thrive. poverty, ignorance, unusual stress on the family.
Section | I  Topic-Wise Solved Questions of Previous Years 315

viii. Health care neglect: Q.2. Parent counselling.


l When a parent or caretaker ignores the treatment
Ans.
recommendations of a health professional for
the management of a treatable illness that a child
has and that is becoming worse. {SN Q.6}
l Dental care neglect also comes under this.

The American Academy of Pediatric Dentistry l Parent counselling can be defined as educating the
defines dental neglect as ‘the failure by a parent parents regarding the child’s oral health status, opti-
or guardian to seek treatment for visually un- mal health care and informing them about the pre-
treated caries, oral infection and/or oral pain’. vention of potential dental diseases.
l Parent education is very important to get a satisfac-
ix. Safety neglect:
l It includes gross lack of direct or indirect super- tory rapport between the entire family and the
vision of a child that results in an injury. dentist.
x. Educational abuse: Purpose:
l Discussion of emotional problems of children,
l When a parent or a caretaker knowingly or in-

tentionally keeps the child at home or fails to particularly in relation to dental treatment.
l Knowing about the attitudes of parents towards
enrol the child in the school.
behaviour management techniques used during
dental treatment of children.
SHORT ESSAYS:
l Obtaining the cooperation of a child patient,
Q.1. Child abuse. establishing a good rapport with the child and
also using effective techniques of behaviour
Ans.
management.
[Ref LE Q.3] l Educating the parents about various dental prob-

lems and diseases and their sequelae and how they


Examination of abused children includes:
can be prevented with accurate preventive mea-
l Assessment of the child and abuser’s behaviour is
sures if recognized earlier.
done by separate interviews.
l By counselling the parent in a few simple rules to
l The abused child usually does not make eye-to-eye
follow before bringing the child to the dentist for
contact and looks afraid.
the first time, a great service can be rendered to
l Signs of overall poor care are seen such as improper
the parent and the child.
clothing, multiple skin injuries, malnourishment.
l Counselling parents about dental treatment should
l The abuser either parent/guardian shows violent tem-
begin preferably before children are old enough to
per, poor coping skills and always describes the child
be impressed adversely by outside influences.
in negative terms.
Counselling can be done on a mass scale through
l At times, abuser’s behaviour reflects guilt, question-
various media, or individually.
ing regarding the incident can cause him/her to feel
Some points to be discussed during parent counselling
embarrassed.
are
General physical examination:
l Tell the parents not to voice their own personal
l On examination of body parts and skin, for fractures/
fears in front of the child.
lacerations, burns, bruises, bite marks, head injuries,
l Tell the parents never to use dentistry as a threat
haemorrhages, etc. Hair pulling can be seen in the
or punishment.
forms of bald patches on the scalp.
l Intraoral examination may reveal trauma to denti-
The feel of punishment in the child’s mind is
tion, injury to upper lip or labial fraenum and other associated with unpleasantness and pain.
l Tell the parents to familiarize their child with
mucosal conditions like warts, ecchymosis, ery-
thema, venereal diseases. dentistry by taking the child to the dentist to be-
l Such children should be managed by first treating the
come accustomed to the dental office and the
emergency condition and if required, referral to other dentist.
l Counsel the parent about the home environment
clinician should be done. Injuries to soft tissue and
and the importance of moderate parental attitudes
dentition should be treated appropriately.
in building well-adjusted children. A well-adjusted
l If required, tetanus toxoid should be given prior to
child is generally a good dental patient.
procedure.
l Explain to the parent that an occasional display of
l Documentation records in the form of case history,
courage on his/her part in dental matters will build
photographs and radiographs should be maintained.
316 Quick Review Series for BDS 4th Year, Vol 1

courage in the child. There is a correlation be- Types of fear:


tween child and parent fears. a. Innate fear

l Stress to the parent, the value of regular dental b. Objective fear
care, not only in preserving the teeth, but also in c. Subjective fear
the formation of good dental patients. d. Suggestive fear

l Discourage parents from bribing their children, to e. Imitative fear
go to the dentist. f. Imaginative fear

l The parent should be instructed never to scold,
shame or ridicule to overcome the fear of dental
treatment. This only builds resentment towards Innate fear:
l Innate fear is the fear without stimuli or previous
the dentist and makes the dentist’s efforts more
difficult. experience and is thus also dependant on the vulner-

l The parent should be informed of the need for ability of the individual.
combating all deleterious impressions of dentistry Objective fear:
l Objective fear is based on the child’s own experiences.
from outside the home.

l The parent should not promise the child what the They are produced by direct physical stimulation.
dentist is not going to do. Lying only leads to They are the responses to stimuli that are felt, seen,
disappointment and mistrust. heard, smelt or tasted and are not linked or accepted.
l Children fear white uniforms and smell of certain

l Several days before the appointment, the parent
should be instructed to convey to the child in a drugs and chemicals in the hospital.
casual manner that they have been invited to visit Subjective fear:
l A child develops subjective fear based on somebody
the dentist.

l The parent should commit the child to the den- else’s experience without actually undergoing dental
tist’s care once the office is reached and should treatment himself/herself.
l Parents may tell the child about an unpleasant or
not enter the treatment room unless requested to
do so by the dentist. Once in the treatment room pain-producing situation undergone by them and this
he/she should act as an invited spectator only. fear may be retained in the child’s mind.

l The parent who educates his/her child to be recep- Suggestive fear:
l It may be acquired by observing fear in other and
tive to dental treatment will find that it pays divi-
dends in the child’s enjoyment of dentistry. then the child develops a fear for the same object as
Parent counselling and education is very important real and genuine.
l Child’s anxiety is closely correlated with parental
to get a satisfactory rapport between the entire
family and the dentist. anxiety. If the parent is sad the child feels sad, and if
the parent display fear the child is fearful.
Imitative fear:
Q.3. First dental visit. l A mother who fears going to the dentist may transmit

this unconsciously to her child who is observing her.


Ans.
l Imitative fears may be transmitted subtly and may be
[Ref LE Q.1] displayed by the parent and acquired by the child
without either being aware of it.
Q.4. Parental attitudes and behaviour of children.
l They are generally recurrent fears and therefore are
Ans. more deep-seated and difficult to eradicate.
l Displayed emotion, such as anxiety observed in the
[Ref LE Q.2]
parent’s face, may create more of an impression than
Q.5. Types of fear. verbal suggestion. For example, a mother who fears
going to the dentist and goes only under great emo-
Ans.
tional stress transmits this fear unconsciously to her
child who is observing her.
{SN Q.1} Imaginative fear:
l As the imaginative capacities of the child develop,
l Fear is the physio psychological response to a realis- imaginary fears become more intense. Imaginary
tic threat or danger to one’s existence. fears therefore become greater with age and mental
l It is the primary emotion for survival against danger,
development, up to a certain age, when reason shows
which acquired soon after birth. them to be ill founded.
Section | I  Topic-Wise Solved Questions of Previous Years 317

Fears may be irrational in the sense that the child


l l Effective vocabulary is an important aspect as the dentist
may not know why he/she is frightened. Memories of must only use the words that are understandable by the
past experiences may fade entirely from his/her con- child.
sciousness, but the emotion associated with the for- l The important aspect of communication is getting the

gotten experience determines, to a large measure, child to respond to dentist’s commands. It is imperative
his/her reaction to a similar event in the future. to use positive language which is pleasing like ‘please
can you move your hand’ rather than use negative aspect
Q.6. Write briefly about examination of abused child. like ‘do not get your hand here’.
Ans. l The three most important facets of communication are

source, medium and receiver.


[Same as SE Q.1] In dentistry, dentist is the source, dental clinic is me-
Q.7. Orofacial signs of child abuse. dium and child is the receiver.
l If the dentist is good, sympathetic, confident and honest,
Ans. dental clinic is neat and attractive to children, with full
[Same as SE Q.1] of toys, then automatically the child will be communi-
cating and is well managed.
Q.8. Importance of child’s first dental visit. l The fear and natural innate curiosity of the child de-

Ans. mand that explanations be given for each and every step
of dental treatment.
[Same as SE Q.3] l Honesty of approach is also very important, if the child

Q.9. Write briefly about parent–child relationship. knows that dentist is honest with his/her words it will
bring out a cooperative behaviour in him/her.
Ans.
Q.3. Different types of cry.
[Same as SE Q.4]
Ans.
Q.10. What is objective fear and subjective fear?
l Cry is defined as a loud utterance of emotion especially
Ans. when inarticulate.
[Same as SE Q.5] Types of cry:
l According to Eisbach (1963), four types of crying
Q.11. Fear in paedodontia. are usually seen in children.
a. Obstinate cry
Ans.
b. Frightened cry
[Same as SE Q.5] c. Hurt cry
d. Compensatory cry
Obstinate cry:
SHORT NOTES: It is a loud and high-pitched emotion. It is
characterized as a siren-like wail.
Q.1. Define fear and name the types of fear. Frightened cry:
Ans. Usually accompanied by a torrent of tears,
convulsive breath-catching sobs can be seen.
[Ref SE Q.5] Hurt cry:
It may be loud and more frequently accompa-
Q.2. Communication.
nied by a small whimper.
Ans. Compensatory cry:
It is not a cry at all; usually the cry is slow in
l Establishing communication is the first objective in suc-
sound and monotone.
cessful management of the young child.
l The dentist not only learns about the patient but also Q.4. Child abuse.
may relax the youngster, by involving the child in con-
Ans.
versation.
l There are two ways of establishing communication: [Ref LE Q.3]
i. Verbal – spoken language to gain confidence
Q.5. Anticipatory guidance.
ii. Nonverbal – expression without words like welcome
hand shake, patting and eye contact Ans.
318 Quick Review Series for BDS 4th Year, Vol 1

Anticipatory guidance includes the following things: l Denial of attention to the basic needs of a child, such as
l Clean infant’s gums after each feeding using a clean food, clothing, shelter, medical care education and su-
damp cloth or an infant toothbrush with a small head pervision is known as neglect.
using plain water. l Neglect is a chronic form of child abuse.

l Plan an appointment for the infant’s first dental visit l The reason for neglect may be parents’ illness, poverty,

within 6 months of eruption of the first tooth and before ignorance and unusual stress on the family.
12 months of age. l Physically neglected children tend to exhibit following

l After initial dental visit make future appointments based characteristics.


on the schedule suggested by the dentist, based on the a. Dirty clothing and skin with foul smell, lice and
infant’s individual needs. unkempt appearance
l As soon as the first tooth erupts (i.e. 6–10 months of b. Undernourished
age) start brushing twice a day using a soft bristled l Health care neglect:

toothbrush designed for infants. In this, parents of the child do not seek treatment for an
l In case infant has sore gums due to eruption of teeth, illness of the child.
give infant a clean teething ring, cool spoon or cold wet l Dental neglect:

wash cloth or even rub his/her gums with a clean finger. This is the failure to seek treatment for dental caries,
l Following brushing at night, after the meal do not give pain and oral infection.
any eatables except water. l Safety neglect:

l For infants at increased risk of tooth decay consult a This means the insufficient supervision by parents or
dentist regarding use of fluoridated toothpaste. caretakers.
l Give recommended doses of fluoride supplements to the l Emotional neglect:

infants of 6 months or older based on water fluoride level. Inadequate mothering or affection due to an infant.
l Become familiar with the normal appearance of the in-
Q.8. Types of cry.
fant’s teeth and gums, so problems can be identified if
they occur. Ans.

Q.6. Parent counselling. [Same as SN Q.3]

Ans. Q.9. Type of child abuse.

[Ref SE Q.2] Ans.

Q.7. Dental neglect. [Same as SN Q.4]

Ans.

Topic 5
Behavioural Science and Psychologic
Management of Children’s Behaviour
COMMONLY ASKED QUESTIONS
LONG ESSAYS:
1 . Define behaviour. Classify behaviour and discuss in detail factors affecting child behaviour.
2. Discuss the importance of first appointment for the child patient.
3. How will you manage a rebellious child in dental clinic?
4. Describe the parental influences on the behaviour of children during dental treatment.
5.
Define behaviour management. Enumerate the fundamentals of behaviour management techniques and explain
aversive conditioning in detail.
6 . Discuss the various behaviour modification techniques for child management in dental office.
7. Describe the different types of behaviour of children. [Same as LE Q.1]
8. How does dental office atmosphere affect behaviour of the children? [Same as LE Q.1]
Section | I  Topic-Wise Solved Questions of Previous Years 319

9. Discuss the management of the first time dental patient. [Same as LE Q.2]
10. Discuss the child’s first visit to the dental clinic. [Same as LE Q.2]
11. How does parental influence affect the behaviour of a child in paedodontic practice. Add a note on parent
counselling. [Same as LE Q.4]
12. Briefly mention about HOME care for the child patient. [Same as LE Q.5]
13. Describe modelling therapy in behaviour management in paedodontic practice. [Same as LE Q.6]
14. What are the different methods you would adopt to manage a difficult child in the dental clinic? [Same as LE Q.6]
15. Discuss various behavioural managements of a 4-year-old child throwing temper tantrums. [Same as LE Q.6]
16. Classify behaviour management techniques. Explain how you would manage a 5½-year-old boy exhibiting
temper tantrums. [Same as LE Q.6]
17. Define behaviour management and behaviour shaping. Enumerate various on pharmacological and pharma-
cological techniques used for behaviour management. Explain in detail tell-show-do. [Same as LE Q.6]
18. Define behaviour management. Explain in detail TSD technique. [Same as LE Q.6]

SHORT ESSAYS:
1. Paediatric treatment triangle.
2. Aversive conditioning. [Ref LE Q.5]
3. Wright’s classification of behaviour of children in dental office. [Ref LE Q.1]
4. TSD. [Ref LE Q.6]
5. Describe modelling technique.
6. Operant conditioning.
7. Factors affecting child behaviour in the dental office. [Ref LE Q.1]
8. Types of cry.
9. Frankle’s behaviour rating scale. [Ref LE Q.1]
10. Voice control. [Ref LE Q.6 and Q.3]
11. What is behaviour management? How will you manage a fearful child of 2 years using different behaviour
modification techniques?
1 2. Psychic triad. [Same as SE Q.1]
13. What do you understand by paediatric triangle? Give its significance. [Same as SE Q.1]
14. HOME. [Same as SE Q.2]
15. Hand over mouth exercise (HOME). [Same as SE Q.2]
16. Modelling. [Same as SE Q.5]

SHORT NOTES:
1. Psychic triad. [Ref SE Q.1]
2. Stoic behaviour.
3. Classical conditioning.
4. Give the Frankel behaviour rating scale for recording children’s behaviour in dental office. [Ref LE Q.1]
5. Desensitization. [Ref LE Q.6]
6. Define TSD. [Ref LE Q.6]
7. Define HOME technique. [Ref LE Q.5]
8. Name different types of cry. [Ref SE Q.8]
9. Define behaviour management and behaviour shaping.
10. Reinforcement. [Ref LE Q.6]
11. Define modelling. [Ref LE Q.6]
12. Name few physical restraints. [Ref LE Q.5]
13. Communicative management technique.
14. Euphemisms.
15. Contingency management. [Ref LE Q.6]
16. Rewarding.
17. Paedodontic triangle. [Same as SN Q.1]
320 Quick Review Series for BDS 4th Year, Vol 1

1 8. Keye’s triad. [Same as SN Q.1]


19. Frankel rating. [Same as SN Q.2]
20. Frankel’s behaviour rating scale. [Same as SN Q.2]
21. Tell-show-do technique. [Same as SN Q.6]

SOLVED ANSWERS
LONG ESSAYS:
Q.1. Define behaviour. Classify behaviour and discuss in (SE Q.9 and SN Q.4)
detail factors affecting child behaviour.
Ans.
{(Frankl’s behaviour rating scale (1962):
Behaviour paedodontics is defined as a discipline which Rating No. 1 Rating No. 4
focuses upon the psychological, social and learning prob- (definitely Rating No. 2 Rating No. (definitely
lems of children and adolescents as they relate to the dental negative) (negative) 3 (positive) positive)
situations. Refuses treat- Reluctant to Accepts Unique
Behaviour is defined as any change in the functioning of ment accept treat- treatment behaviour
ment
an organism.
Immature Immature Tense, co- Good rapport
[SE Q.3] operative

{Various classifications of behaviour are as follows: Defiant be- Timid and Timid and Understand-
I. Frankl’s classification (1962) haviour and whining whining ing and
II. Lampshires classification
crying force- interested) }
fully
III. Wrights classification
IV. Kopels classification
II. Lampshires classification:
V. Modified Wrights classification (addition of symbolic
i. Cooperative:
modifications to the Frankl’s rating scale)}
l Physically and emotionally relaxed

[SE Q.9] ii. Tense cooperative:


l Tensed and cooperative
{I. Frankl’s Behaviour Rating Scale (1962): iii. Outwardly apprehensive:
Rating No.1 – Definitely negative l Avoids treatment initially
l Refuses treatment
l Hides behind mother and avoids looking or
l Immature, uncontrollable
talking to dentist but eventually accepts the
l Defiant behaviour
treatment
l Crying forcefully
iv. Fearful
Rating No.2 – Negative l Requires considerable support so as to over-
l Reluctance to accept treatment
come the fears of dental treatment.
l Immature, timid and whining.
v. Stubborn or defiant:
l Some evidence of negative attitude but not
l Passively resists treatment by using tech-
pronounced. niques.
Rating No.3 – Positive vi. Hypermotive:
l Accepts treatment
l Child is acutely agitated resorts to screaming
l Tense cooperative
and kicking.
l Willingness to comply with dentist
vii. Handicapped:
Rating No.4 – Definitely positive l Physical or mental.
l Good rapport with the dentist
viii. Emotionally immature:
l Understanding and interested in the dental pro-
l Emotionally handicapped.
cedures
l Laughs and enjoys the situation}
Section | I  Topic-Wise Solved Questions of Previous Years 321

[SE Q.3] IV. Kopels classification:


l Very young patient
{III. Wrights classification of cooperativeness of chil-
l Emotionally disturbed patient
dren in dental office:
l Child from a broken or poor family

l Pampered or spoiled child


Children’s behaviour
l Neurotic child

l Excessively fearful child


Cooperative Lacking Potentially l Hyperactive child
behaviour cooperative cooperative l Physically handicapped child
behaviour behaviour l Child with precious untouched medical or dental

experience
i. Uncontrolled
[SE Q.9]
behaviour
ii. Defiant behaviour {V. Wrights modification of Frankl’s behaviour
iii. Timid behaviour rating:
iv. Tense cooperative Rating No.1 – Definitely negative (– –)
behaviour Rating No.2 – Negative (–)
v. Whining behaviour Rating No.3 – Positive (1)
I. Cooperative behaviour: Rating No.4 – Definitely positive (1 1)}
l Child is cooperative
[SE Q.7]
l Reasonably relaxed

l Develop good rapport with the dentist {Factors influencing child’s behaviour:
l Laugh and enjoy the situation Factors influencing child’s behaviour are categorized
II. Lacking cooperative behaviour under following headings:
l This behaviour is contrast to cooperative child.

l It includes young children (0–3 years), disabled Factors involving


child, physically and mentally handicap. the child/out of Factors involving Factors involving
l They can have major behavioural problems. control of dentists the parents the dentist
III. Potentially cooperative behaviour Growth and Family influence Appearance of the
l Child is cooperative. development dental office
l Physically and mentally fit. IQ of the child Parent–child rela- Time of appoint-
l Childs behaviour can be modified. tionship ment
l Group of children’s require behavioural modifi-
Past dental Maternal anxiety Length of appoint-
cation procedures. experience ment

Potentially cooperative behaviour is subclassified into Social and adoptive Attitude of par- Dentists skill
following: skills ents to dentistry

Uncon- Defiant Tense Whining Position of the child Use of good


in the family words and
trolled behav- Timid cooper- behav- rewarding
behaviour iour behaviour ative iour
Age group Found Seen in Border Whining I. Factors involving the child/out of control of dentists:
3–6 years in all overprotec- line be- through- i. Growth and development:
ages tive child haviour out the l Growth is defined as an increase in size, whereas
procedure
the development is progression towards maturity.
Also called Also re- Milder but Child is Cry is l Both the processes proceed in a relatively predict-
incorrigible ferred to highly anx- tensed in controlled able logical step like sequential order.
behaviour as stub- ious mind constant
l These processes are influenced by genetic, famil-
born or and not
spoilt loud ial, cultural, interpersonal and psychic factors.
The basic developmental parameters that influ-
Tears, loud They do Is shy but Seldom
ence behaviour are
crying not like coopera- are the
l Biologic – motor maturation as well as
physical to go to tive tears} bodily development
lashing out dental
and flailing clinic l Cognitive development – intellectual devel-
of the opment such as thinking and reasoning
hands
l Emotion
322 Quick Review Series for BDS 4th Year, Vol 1

l Perceptual development – integration of Mother’s behaviour Child’s behaviour


senses such as hearing and sight Overprotective Submissive, shy, anxious
l Personality and social development –

habitual way of behaving with others Overindulgent Aggressive, spoiled, demanding, display
of temper
l Growth of language skills

ii. IQ of the child: Under affectionate Well behaved, unable to cooperate,


shy, may cry easily
l Intelligent quotient (IQ) is the method of quan-

tifying the mental ability in relation to chrono- Rejection Aggressive, overreactive, disobedient
logical age formulated by Alfred Binet. Dominant Aggressive, demanding, display of
IQ 5 mental age/chronological age 3 100. temper
iii. Past dental experience: Identification Feel of guilt, cries easily, loss of confi-
l Attachment attained by the child during the past dence
dental visit plays a very important role than the
Authoritarian Submissive, resentment, evasive.
number of visits.
l Any past unpleasant dental experience results in

uncooperative behaviour.
iv. Social and adaptive skills: iii. Maternal anxiety:
l Highly anxious parents affect child’s behav-
l If a child gets whatever he/she wants there are

high chances of child getting spoilt. iour, and it is more at the age of 4 years.
l Independence of the child also plays a major
iv. Attitude of parents to dentistry:
l If parents behave positively even the child
role.
v. Position of the child in the family and child’s will behave in the same way.
behaviour: III. Factors involving the dentist:
i. First child: uncertainty, mistrustfulness, inse- i. Appearance of dental office:
l Appearance of dental office should make the
curity, dependence, responsibility and jealousy
ii. Second child: independence, aggressive, extro- child comfortable.
l Lamps and shades should be small.
vert, fun-loving and adventurous
l Toys should be kept near the dental chair.
iii. Middle child: aggressive nature, feel of inferi-
l Protect child from seeing adult’s pain or proce-
ority
iv. Last child: secure, confident, immature, envy, dures like extractions.
l Dental office staffs should show enthusiasm.
irresponsible, good and bad behaviour
II. Factors involving the parents: ii. Dentist behaviour:
l Permitting the child to express.
i. Family influence:
l Communicating with the child in the sweet way.
l Home is the first school and it’s the place where
l Making the child comfortable.
the child learns to behave.
l Listening to children’s comments.
l Mother’s behaviour plays a major role in the be-
l Providing them the comfortable environment.
haviour of the child compared to the other family
members. iii. Time and length of appointment:
l Child should not be kept for more than half an
l One tailed – is the description for the mother–

child relationship. hour on a dental chair.


l Mother’s nutritional status also places a role in
iv. Dentists skill and speed:
l The dentist should perform his/her duties with
development of the child.
ii. Parent–child relationship: dexterity, and should avoid loss of time.
l Two broad categories:
v. Use of good word’s, subtle, flattery, praise and
i. Autonomy versus control reward:
l Fear-promoting words should be avoided like
ii. Hostility versus love
Characteristics of parent–child relationship Mosquito bite n needle prick
that may influence child’s behaviour in dental Rain coat n rubber dam
clinic: Coat rack n rubber dam frame
a. Domination Cavity fighter n fluoride
l One of the most important rewards sought by the
b. Identification
c. Overindulgence child is the approval of the dentist.
l In praising a child, it is better to praise the be-
d. Overprotective nature
e. Authoritarian haviour than the individual.
Section | I  Topic-Wise Solved Questions of Previous Years 323

Tiny gifts make fine reward. Tiny gold stars, toys


l common in children younger than 4 years or it
or stickers make good gifts. It is the recognition may be because of fear of the unknown.
more than the material that makes the child l Subjective fear and maternal anxiety have been
happy. associated with the reactions children exhibit at
l Flattery can also be used as a reward after the their first dental visit.
treatment.} l Several other factors have been associated with
Q.2. Discuss the importance of first appointment for the the reactions children exhibit at their first dental
child patient. visit. The prominent factor that has been related is
Ans. the maternal anxiety.
l Both a negative attitude towards the physician and
l It is generally recommended that a child’s first visit be a previous history of pain associated with medical
made at no later than 3–4 years of age. appointments has also been identified as factors.
Goals of a child’s first dental visit are as follows: l Few studies have found that children of lower
l To familiarize children with the dental setting and socioeconomic status exhibit more negative be-
various types of dental treatment. haviour at their first visit.
l To provide children of all age groups a pleasant, l Except for the emergency treatment or noninva-
fun-filled, enjoyable nonthreatening introduction sive procedure it is wise not to perform any other
to dentistry. treatment during the first visit, even in a very co-
l To introduce your philosophy of holistic, child- operative behaviour.
centred and preventive practice. l If the initial visit happens uneventfully, the child
l To determine the relative risk and susceptibility of believes the dentist and starts trusting the stranger
the child to dental disease he/she met.
l To detect the signs of early dental disease, e.g. l Once children are aware as to what would hap-
white spot lesions. pen, most of them are very prepared and they
l To assist and provide support to parents and help tend to cooperate for the procedure. One may
them to reduce caries incidence in children. not have any difficulty in convincing them. Ev-
Common guidelines to be followed during child’s first ery effort should be made to keep the first visit
dental visit are as follows: uneventful.
l During the first visit, an easy, comfortable and The following age-wise guidelines may prove useful
real fun way of easing the child into the dental for the paediatric dentist:
setting may be done by doing something that the i. 0–3 years (lacking cooperative ability):
child really enjoys. l They should be examined either in knee-to-
l Children may have varied interests, but one thing knee position, on an infant examination table
that never fails to arouse their attention is colours. in parent’s lap, on the dental chair selectively.
Have a variety of crayons, colour pencils, sketch Suggestions during first visit are
pens, etc. that the child might find attractive. Brief examination
l Generally it has been found that the degree of l Not to separate from the parents if
cooperation exhibited by preschool children at possible.
their first appointment is high since the first visit l Avoid using airotor, if necessary us-
usually involves only an examination, radio- ing micromotor to be considered.
graphic evaluation and if possible a prophylaxis l Intermittent preparation of the tooth
and topical fluorides treatment unless the child ii. 4–6 years children:
presents with an acute dental problem. Most chil- l They should be examined on the dental
dren readily accept this. chair, sometimes in parent’s lap on the den-
l However there are certain procedures during the tal chair.
first appointment that are frequently associated Suggestions during first visit treatment of these
with un-cooperative behaviour. children are
l The highest rates of uncooperative behaviour dur- Effective TSD for the following:
ing the first session occur when the child is sepa- l Chair movements and chair light
rated from his/her mother. l Suction apparatus
l Taking radiographs can cause some un-coopera- l Three-way syringe
tive behaviour during the first session, since child l Airotor
may experience a fear of abandonment which is l Micromotor on the nails
324 Quick Review Series for BDS 4th Year, Vol 1

iii. 7–12 years children: [SE Q.10]


Suggestions during first visit treatment of these
children are
{Method:
l Voice control is usually carried out on children of
a. Effective TSD for the following:
3–6 years.
l Chair movements and chair light
l Sudden and firm commands can be used to get the
l Suction apparatus
child’s attention or stop the child from whatever is
l Three-way syringe
being done.
l Airotor
l There is an abrupt and emphatic change in the
l Micromotor on the nails
dentist’s tone of voice to emphasize his/her dis-
b. Identifying and eliminating or desensitizing
pleasure with the child’s inattention.
any existing specific fears.
l Most often children seeing the dentist’s unhappi-
iv. Above 12 years:
ness become quiet and start listening to dentist’s
Suggestions during first visit treatment of these
instructions.
children are
l If used appropriately, this is one of the effec-
l Information is conveyed in such a way that
tive behaviour management techniques to con-
their dental health would improve their self-
trol child exhibiting temper tantrums or defiant
image and peer influence.
behaviour.
l Personal perception of appearance needs to
l As soon as the child complies we should thank
be evaluated before the need for any teeth
him/her and compliment him/her for the resultant
alignment.
excellent behaviour.}
Every effort should be made to keep the first
iii. Pharmacological methods:
visit uneventful. The time spent during first
This method includes:
dental visit is a valuable investment for the
a. Oral sedation
future dental visits.
b. Intravenous sedation
a. Oral sedation:
Q.3. How will you manage a rebellious child in dental l Most accepted route of drug administration

clinic? l Absorption is not consistent

l Recovery can be prolonged if drug is slowly


Ans.
metabolized
Managing a rebellious child in the dental clinic or a hospital b. Intravenous sedation:
is a common task. l It can be used if the child refuses to take the drugs.
Managing can be done by following below mentioned l Action cannot be reversed.
three steps: l Site of injection is upper part of gluteal
i. Parental counselling region.
ii. Voice control
iii. Pharmacological methods Common oral agents for conscious sedation:

i. Parental counselling: Agents Supplied as Recommended dose*


l Instructing the parents to make sure that the child is Narcotics
comfortable when the child comes for next appoint- Mepiridine Tablets (50 mg) 1–1.5 mg/kg with
ment. By commanding the child and making him/ (Demerol) Syrup (50 mg/ agents from other
her comfortable about the next appointment. 5 mL) group or 2 mg/kg with-
ii. Voice control: out other agents
l Voice control is a controlled alteration of voice, Sedative hypnotics
volume, tone or pace to influence and direct the Chloral hydrate Capsule 250 and 50–70 mg/kg
patient’s behaviour. (Noctec) 500 mg
Objectives: Syrup (50 mg/mL)
a. To gain the patients attention and compliance. Diazepam Tablets (2,5 and 0.2–0.5 mg/kg alone
b. To avert negative or avoidance behaviour. (Valium) 10 mg)
c. To establish authority. Phenobarbital Capsules (30, 50 2–3 mg/kg alone
Indications: (Nembutal) and 100 mg) Elixir
l Voice control is indicated for the uncooperative, (30 mg/5 mL)
inattentive and communicative child. Anithistaminics
Contraindications:
Diphenhydr- Capsules (25 and 1.0–1.5 mg/kg with
l In children who due to age disability, medication
amine 50 mg) narcotic
or emotional immaturity are unable to understand (Benadryl) Elixir (12.5 mg/
and cooperate. mL)
Section | I  Topic-Wise Solved Questions of Previous Years 325

Agents Supplied as Recommended dose* Q.4. Describe the parental influences on the behaviour
of children during dental treatment.
Hydroxyzine Syrup (10mg/5mL) P.o.: 1–2 mg /kg
I.m.: 1.1 mg/kg with Ans.
narcotic l In the treatment of children, a child cannot be treated as

Promethazine Tablets (10, 25, 50 0.5–1 mg/kg with nar- a single separate entity since his/her behaviour will de-
Phenergan and 100 mg) cotic pend a great deal on his/her parents, siblings or the fam-
Syrup (2, 5, 6.25 ily as a whole.
mg/mL) l The paedodontic treatment triangle depicts this. All

parts of the triangle are interrelated and the arrows show


they are dynamic or ever changing.

Common parenteral agents for conscious sedation:


Recommended Child
Agents Supplied as dose
Narcotics:
Fentanyl (subli- Ampule (2 and Submucosal (SM)
maze) 5 mL) (0.0005 n 0.002 mg/kg
mg/mL) I.m. n 0.002 mg/ Society
kg Dentists
I.v. n not recom- /surgeon
mended Parent

Alphaprodine Ampule (40 mg/mL, SM n 0.6 mg/kg


1 mL) I.m. n Not rec-
Ampule (60 mg/ ommended l Here the child is the most significant part and is depicted
mL, 1 mL) I.v. n 0.2 mg/kg at the apex of the triangle. Mother plays the most signifi-
Vial (60 mg/mL,
cant part in the family’s influence in child behaviour since
10 mL)
she is the one the child will be usually most attached to,
Mepiridine Ampule (50 mg/ SM n 2 mg/kg though other members also play a significant role.
(Demerol) mL; 0.5, 1, 1.5 and I.m. n 2 mg/kg
l The parent’s attitudinal structure, which moulds, shapes
2 mL) I.v. n 0.3 mg/kg
Vial (50 mg/mL, and directs child behaviour in the early period of the
30 mL) offspring development, is affected by socioeconomic
Neuroleptics:
position, cultural development and ethnic background.
l From such sources emerge gradations of parenting rang-
Diphenhydramine Ampule (10,50 mg/ SM n Not recom-
ing from the authoritative progenitor to the shy one. The
(Benadryl) mL, 10 mL) mended
Ampule (50 mg/mL, I.m. n 1 mg/kg parent who is personally distraught or depressed has
1 mL) I.v. n 0.5 mg/kg difficulty parenting in an effective manner.
Maternal influence on personality development:
Hydroxyzine Vial (50 mg/mL, SM n not recom- l There is a mother–child interdependency that initiates
(Vistaril) 2 mL) Vial (25, 50 mended at infancy, and builds well into the preschool period.
mg/mL, 10 mL) I.m. n 1 mg /kg l Should this interdependency extend beyond its in-
I.v. n Not recom-
tended period, dual ambivalences may emerge be-
mended
tween the mother and her child with resultant mater-
Promethazine Ampule (25,50 mg/ I.m. n 1 mg/kg nal anxieties and development of aberrant behaviour
(Phenergan) mL, 1 mL) I.v. n 0.5 mg/kg
pattern on the part of the offspring.
Reversal agents: l Bayley and Shaefar indicate that most of the relevant

Naloxone (Opoid Ampule (0.4 mg/mL, I.v./i.m./s.c. n 0.1 mother–child relationships fall into following two
reversal agent) 1 mL) mg/kg/dose to a broad categories.
maximum of A. Autonomy versus control
2 mg/dose B. Hostility versus love
Flumazenil (Benzo- I.v. n 0.02 mg/kg/ l Mothers who allowed enough autonomy and who
diazepine reversal dose, may be re- expressed affection had children who were friendly
agent) peated every and cooperative and those who ignored their children
1 min to a maxi-
mum 1 mg
did not have children who exhibited these positive
behavioural features.
326 Quick Review Series for BDS 4th Year, Vol 1

‘Bell’ termed the parent–child relationship as ‘one-


l l Parental attitudes: Counsel the parent about the home
tailed’; since parental characteristics are viewed as environment and the importance of moderate paren-
having a unilateral influence on those developing in tal attitudes in building well-adjusted children.
the child. l Valuing dental care: Stress to parent, the value of

l According to the ‘one-tailed’ theory, many of the regular dental care, not only in preserving the
child characteristics including his/her personality, teeth, but also in the formation of good dental
behaviour and reaction to stressful situations are the patients.
direct product of various parental – especially mater- l No bribing the children

nal – characteristics. l Never scold the child

Parental attitudes: l Encourage the child

l Some of the parental attitudes can adversely affect the


Q.5. Define behaviour management. Enumerate the
child’s developing personality; these specific types of fundamentals of behaviour management techniques
parental behaviours leading to characteristic mal- and explain aversive conditioning in detail.
adaptive behaviour in their children are as follows:
a. Overprotective attitude Ans.
b. Overindulgent attitude Behaviour management is a means by which the dental
c. Overauthoritative attitude health team effectively performs treatment for a child and
d. Under-affection/rejecting attitude at the same time, installs a positive dental attitude.
Mother–child behaviour interactions: Fundamentals of behaviour management are as follows:
l To establish effective communication with the child and
Mother’s behaviour Child’s behaviour the parent.
Overprotective Shy, anxious, lacking confidence, l To gain the confidence of both the child and the parent
submissive and make them accept dental treatment.
Overindulgent Aggressive, spoilt, demanding, dis- l To teach the child and the parent the positive aspects of
plays temper preventive dental care.
Under-affection Usually well behaved, shy, unable to l To provide a relaxation and comfortable environment
cooperate for the dental team to work in while treating the child.
Rejecting Overactive, disobedient, aggressive
[SE Q.2]
Authoritarian Delayed response, evasive
{Aversive conditioning:
Effects of maternal anxiety on child behaviour Aversive conditioning includes the following two tech-
l Highly anxious parents tend to affect their child’s niques:
behaviour negatively. i. Hand over mouth exercise (HOME)
l Although the scientific data revealed that children of ii. Physical restraints}
all ages can be affected by their mother’s anxiety.
(SE Q.2 and SN Q.7)
l The effect is greatest with those under 4 years of age.

l These might be anticipated because of the close

child–parent symbiosis that begins in infancy and {(Hand over mouth exercise technique (HOME):
gradually diminishes. l This technique was first described in the 1920s
Parent counselling: by Dr Evangeline Jordan who wrote ‘If a normal
l Educating the patient is very important to get a satis- child will not listen but continue to cry and
factory rapport between the entire family and the struggle hold a folded napkin over the child’s
dentist. mouth and gently but firmly hold his mouth shut.
l Personal fears: Tell the parents not to voice their own His screams increase his condition of hysteria,
personal fears in front of the child. but if the mouth is held closed, there is little
l Dentistry not a threat: Tell the parents never to use sound and he soon begins to reason’.
dentistry as a threat or punishment. l Levitas described hand over mouth technique

l Familiarize dentistry: Tell the parents to familiarize (HOMT) as HOME. A hand is placed over the
their child with dentistry by taking the child to the child’s mouth and behavioural expectations are
dentist to become accustomed to the dental office calmly explained. The child is told that the hand
and the dentist. will be removed as soon as appropriate behaviour
l Courage display: Explain to the parent that an occa- begins. When the child responds, the hand is re-
sional display of courage on his/her part in dental moved and the child’s appropriate behaviour is re-
matters will build courage in the child. inforced. The method may require reapplication.)}
Section | I  Topic-Wise Solved Questions of Previous Years 327

[SE Q.2] l Once communication is achieved with HOME,


other management techniques can be em-
{ It was called emotional surprise therapy by Lampshire
l ployed.
and as aversive conditioning by Kramer. There are several variations to HOME, they are as
Objectives of HOME therapy are as follows: follows:
i. To gain the child’s attention enabling communi- l Hand over mouth – airway unrestricted
cation with the dentist so that appropriate behav- l Hand over both mouth and nose – airway
ioural expectations can be explained. restricted
ii. To eliminate inappropriate avoidance responses l Towel held over mouth only
to dental treatment and to establish appropriate l Dry towel held over mouth and nose
learned responses. l Wet towel held over mouth and nose
iii. To enhance the child’s self-confidence in coping But it is always recommended that the hand be
with the anxiety of dental treatment. placed only over the mouth so that the child’s airway
iv. To ensure the child’s safety in the delivery of should never be restricted. It is very important to
quality dental treatment. ensure that the child has no nasal blockage and can
Technique of HOME: satisfactorily breathe through nose when the mouth
l Dentist gently but firmly places his/her is closed.
hand over the child’s mouth. Physical restraints:
l In a loud voice the child is told that if he/ l Protective stabilization or restraining is defined as
she cooperates, the hand will be removed the restriction of patient’s freedom of movement,
from the mouth. with or without the patient’s permission, to de-
l When the patient indicates his/her willing- crease risk of injury while allowing safe comple-
ness to cooperate, usually by a nod of the tion of treatment.
head or the scream, the hand is removed l Partial or complete immobilization of the patient is
and patient is revaluated. necessary sometimes to protect the patient and/or
l If the disruptive behaviour continues, the dental staff from injury while providing dental care.
dentist again places his/her hand over l Physical restraints in the dental office can range
the child’s mouth and tells him/her to from gently holding a child’s hands during injec-
cooperate. tion procedure to full body restraint with a pa-
l Once the child cooperates he/she must be poose board.
complimented. l Restraints can be performed by the dentist, staff
l The need to diagnose and treat as well as or parent with or without the aid of a
the safety of the patient and practitioner restraining device.
must justify the use of HOME. l Parental or guardian consent must be obtained
l Parents or guardians consent should be prior to use of restraints, and the following must
obtained prior to the use of HOME. be included in the patient record:
l Informed consent and indication for the i. Informed consent
use of HOME should be included in the ii. Type of restraint used
patient record. iii. Indication for the stabilization
Indications: iv. The time and the duration of application of
l In case of a healthy child who is able to restraint
understand and cooperate, but who exhibits Objectives of using restraints are
defiant, obstreperous or hysterical avoid- i. To reduce or eliminate untoward movement.
ance of behaviour to dental treatment. ii. To protect the patient and dental staff from
l For normal children who are hysterical injury.
and belligerent. iii. To facilitate delivery of quality dental treat-
l Used for children with sufficient maturity ment.
to understand simple verbal commands. Indications:
Contraindications: i. A patient who requires immediate diagnosis
l In children, who due to age, disability, or treatment and cannot cooperate due to lack
medication or emotional immaturity are of maturity, mental or physical disability.
unable to understand and cooperate. ii. A patient who requires diagnosis and/or treat-
l When it will prevent the child from ment and does not cooperate after other be-
breathing. haviour management techniques.
328 Quick Review Series for BDS 4th Year, Vol 1

iii. When the safety of the patient and/or practi- d. Rubber bite blocks
tioner would be at risk without the protective l Available in various sizes to fit on the occlusal
use of restraints. surfaces of the teeth and stabilize the mouth in
Contraindications: an open position.
i. A cooperative patient II. Restraints for body:
ii. A patient who cannot be restrained safely due a. Papoose Board:
to underlying medical or systemic conditions.} l It is a commercial wrapping for the body.

l Secures child against a rigid base with three


(SE Q.2 and SN Q.12)
pairs of canvas straps or single strap for very
young child. Velcro system is easy to close
{(The following are some commonly used physical and adjusting by pressing or peel apart
restraints: fasteners.
I. Oral: l Restrains even the most uncooperative patients.
Mouth props, padded wrapped tongue blades, a b. Pedi-wrap:
finger guard or an intraocclusal thimble and rubber/ l Reinforced nylon meshsheet with velcro clo-
plastic bite blocks sures available in small, medium and large sizes
II. Body: which is placed on prepositioned chair.
Papoose board, triangular sheet, Pedi-wrap, bean l Child is placed on the wrap and velcro fastners
bag dental chair insert, safety belt, extra-assistant. are put on chest followed by arms and legs to
III. Extremities: avoid movements.
Posey strap, Velcro straps, towel/tape, extra-assistant. c. Bean bag dental chair insert:
IV. Head: l It was developed to help comfortably accom-
Forearm support, head positioner, plastic bowl, extra- modate the hypotonic and severely spastic
assistant)} patients who need more support and less
restraining.
I. Oral restraints: III. Restraints for extremities:
Mechanical aids to maintain the mouth in an open position: a. Posey straps:
a. Padded and wrapped tongue blades: l To restrain the child’s arms and legs.

l Easy to use l Posey straps fasten to the arms of the dental

l Disposable chair and allow limited movement.


l Inexpensive b. Towel and tape
b. Molt mouth prop: l A towel wrapped around patients fore arms and

l Very helpful for management of a difficult fastened with adhesive tapes without impeding
patient for a prolonged period. circulation.
l Made in both adult and children sizes. c. Velco straps and an extra-assistant:
l Allows accessibility to the opposite side of the l Velcro straps are available in small, medium and

mouth. large sizes.


l Disadvantages include the possibility of lip and IV. Restraints for head:
palatal lacerations and luxation of teeth if not a. Forearm body support:
used correctly. l Head position can usually be maintained by the

l Caution must be exercised to prevent injury to the use of forearm body pressure by the dental
patient, and the prop should not be allowed to rest surgeon.
on anterior teeth and patient’s mouth should not b. Head positioner:
be forced beyond its natural limits. l The papoose board comes with a head posi-

c. A finger guard or an intraocclusal thimble: tioned to stabilize the head.


The advantages are c. Extra-assistant:
l It prevents mouth closure. l An extra-assistant is required to stabilize the

l Inexpensive. head.
l Fits dentist’s finger. l Physical restraints should never be used as pun-

l Main disadvantage is the limited mobility of ishment. An explanation of their benefits should
the dentists hand once the splint is in place and be presented if communication is possible with
functioning. the child as well as parents.
Section | I  Topic-Wise Solved Questions of Previous Years 329

Q.6. Discuss the various behaviour modification tech- . Antianxiety drugs


b
niques for child management in dental office. c. Antihistaminics
II. Conscious sedation
Ans.
III. General anaesthesia
l Behaviour management is defined as the means by A. Nonpharmacological methods:
which the dental health team effectively and efficiently The nonpharmacological methods of behaviour man-
performs treatment for a child and at the same installs a agement are explained in detail below:
positive dental attitude. a. Preappointment behaviour modification:
l Behaviour shaping is that procedure which very slowly l Preappointment behaviour modification in-

develops behaviour by reinforcing successive approxi- cludes everything that is to be done to posi-
mations of the desired behaviour until the desired be- tively influence the child’s behaviour, be-
haviour comes to be. fore the child enters the dental clinic.
It is sometime called as ‘stimulus–response theory’. l Various methods used for this purpose are

According to AAPD Guidelines, 2002–2003: as follows:


I. Basic behaviour management i. Films or videotapes showing a model of a
1. Communicative management cooperative child patient.
a. Voice control ii. It can also be performed on live models,
b. Nonverbal communication such as other children or parents.
c. Tell-show-do (TSD) b. Communication:
d. Positive reinforcement l Effective communication, i.e. imparting or

e. Distraction interchange of thoughts, opinions or infor-


2. Parental presence or absence mation is important factor in dealing with
3. Nitrous oxide/oxygen inhalation sedation children.
II. Advanced behaviour management l The first objective of successful manage-

1. HOME ment of a young dental patient is to estab-


2. Medical immobilization lish communication.
3. Sedation l Based on the age of the child there are many

4. General anaesthesia ways of initiating a verbal communication


In general behaviour management methods are clas- but, in dental setups, it is affected primarily
sified as follows: through dialogue, tone of voice, facial ex-
A. Nonpharmacological methods pression and body language.
B. Pharmacological methods l Involving the child in conversation will re-

A. Nonpharmacological methods: lax the child and enables the dentist to learn
a. Preappointment behaviour modifications about the patient.
b. Communication l Verbal communication with young children is

c. Behavioural shaping techniques: best initiated with complementary comments


i. TSD about their dress or interests followed by some
ii. Desensitization questions that elicit an answer other than yes
iii. Modelling or no.
iv. Contingency l For the successful communication to take

d. Behavioural management techniques place all the four elements of communica-


i. Audioanalgesia tion, i.e. sender, message, context and re-
ii. Biofeedback ceiver must be present and consistent.
iii. Voice control
iv. Hypnodontics [SE Q.4]
v. Coping {c. Behavioural shaping techniques:
vi. Relaxation l Behaviour shaping is the procedure by
vii. Aversive conditioning which the desired behaviour is instilled
viii. Implosion therapy and inculcated in the child.
ix. Retraining l Behaviour shaping is based on a planned
B. Pharmacological methods of behaviour man- introduction of treatment procedures, so
agement that the child is gradually trained to ac-
I. Premedication: cept treatment in a relaxed and coopera-
a. Sedatives and hypnotics tive manner.
330 Quick Review Series for BDS 4th Year, Vol 1

l Various behaviour shaping techniques are as l TSD contains certain elements of systematic desensiti-
follows: zation.
i. TSD l Ingersoll (1982), however, considers TSD to be an in-

ii. Desensitization formation exposure method of behaviour shaping be-


iii. Modelling cause it excludes the preparatory format contained in
iv. Contingency} the original studies on systematic desensitization.
Indications:
(SE Q.4 and SN Q.6) l All patients who can communicate regardless of

the level or the method of communication.}


{(i. Tell-show-do technique:
l Almost five decades ago (1959), Addelston for-

malized a technique that encompasses several {SN Q.5}


concepts from the social learning theory. It was ii. Desensitization:
called TSD technique. l Desensitization technique involves three stages:
l The TSD is a behaviour shaping technique. It
a. Training the patients to relax
gives good results and is recommended to be b. Constructing a hierarchy of fear producing
followed in routine practice. stimuli
l The TSD method of introducing dental tech-
c. Introducing each stimulus in hierarchy to re-
niques is extremely effective for shaping the laxed patient starting with the stimulus which
child’s behaviour and conditioning him/her to causes least fear
accept treatment.
Objectives of TSD technique are For example, if the child is afraid of dental clinic, the
l Teach the patient important aspects of the general desensitization should include gradual introduc-
dental visit. tion of the child to following:
l Familiarization of dental setting. l Reception, waiting room and receptionist

l Shape the patient’s response to procedures l Paedodontist, hygienist and nurse

through desensitization and well-described l Dental surgery

expectations. l Dental chair

TSD is a series of successive approximations, l Oral examination

the steps followed in TSD are l Prophylaxis

A. Tell the child about the treatment to be At each stage the child’s fears are allayed by the
carried-out. kind, friendly and reassuring manner of recep-
B. Show him/her part of it, how it will be tionist, nurse, hygienist and paedodontist and
done. positive approach of the child is reinforced.
C. Then do it.)}
iii. Modelling (imitation):
[SE Q.4] l Children are capable of acquiring almost any behav-

iour that they observe closely and that is not too


{ TSD technique includes verbal explanations at the patient’s
l
complex for them to perform at their level of physi-
understanding level (tell) appropriately using second lan- cal development.
guage or word substitutes wherever necessary.
l Demonstration of the visual, auditory, olfactory and
tactile aspects of the procedure in a nonthreatening fash- {SN Q.11}
ion (show).
l Modelling is learning by imitation based on observa-
l Completion of the procedure without deviating from
tional learning theory by Albert Bandura, 1969.
what was explained and demonstrated (do).
l Goal of modelling is to have the patient reproduce
l While working intraorally, the child should be shown
the behaviour exhibited by the models.
as much of the procedure as possible. One should be
truthful with the child and yet should not frighten
him/her. Stages and requirements of modelling:
l Any deviation from that of originally explained or dem- l Two stages of modelling are acquisition and perfor-
onstrated can affect the relation between the child and mance.
the dentist. So honesty is required, to achieve positive l The four requirements are attention, retention, motoric
attitude of children in future. reproduction, reinforcement and motivation which
Section | I  Topic-Wise Solved Questions of Previous Years 331

were discussed under observational learning theory 2. Omission or time out:


and child psychology. l It is the means of increasing the (probability (fre-

The advantages of modelling procedures are the following: quency) of a desired behaviour by withdrawal of
l Stimulation of good behaviour. or threatening to withdraw a pleasant stimulus.
l Facilitation of behaviour in more appropriate For example, warning or threatening the child that
manner. you will send the mother outside the operatory, if
l Extinction of fears and apprehensions. the child is not cooperating for the procedure.
Modelling is effective particularly in the following condi- 3. Punishment:
tions: l It is the means of increasing the frequency of a

l When the patient is in state of arousal. desired behaviour by the presentation of an aver-
l When model’s behaviour has positive consequences. sive stimulus.
l When modelling is performed on models having For example, the use of voice control, protective
higher status and prestige. stabilization or hand over mouth.
d. Behavioural management techniques:
{SN Q.15} i. Audioanalgesia
ii. Biofeedback
iv. Contingency management:
iii. Voice control
l This technique is based on the operant condition-
iv. Coping
ing theory of B.F. Skinner.
v. Relaxation
l It is a method of modifying behaviour by presen-
vi. Aversive conditioning
tation or withdrawal of the reinforcers. These re-
vii. Implosion therapy
inforcers are the pleasant or unpleasant stimuli
viii. Retraining
mentioned in the operant conditioning theory in
child psychology. i. Audioanalgesia:
l Contingency management includes: l It is also called as ‘white noise’. This con-

1. Reinforcement: sists of providing a sound stimulus intensity


Either positive reinforcement or negative rein- that the patient finds it difficult to attend to
forcement anything else. The effect is due to distrac-
2. Omission/time out tion, displacement of attention and a posi-
3. Punishment tive feeling on the part of the dentist that it
can help.
ii. Biofeed back:
{SN Q.10}
l In this method certain physiologic reaction

1. Reinforcement: of the body are detected that may indicate


l It is a method of increasing the probability (fre- fear. For example, checking heart rate that
quency) of a desired behaviour by presentation of may increase when the patient is under
a pleasant stimulus or withdrawal of an aversive stress.
or unpleasant stimulus. Here the stimulus is
termed as reinforcer. [SE Q.10]
l Positive reinforcers are presented:
{iii. Voice control:
For example, a pat on the back or shoulder,
l Voice control is a controlled alteration of
shaking hand, verbal praise in the presence of
volume, pace or tone of voice to control
parent for which the child will be happy. In the
child’s disruptive behaviour.
form of gifts like toothbrush kits, drawing kits,
l To establish authority, gain the child’s atten-
favourite cartoon stickers or toys appropriate
tion and compliance and avert negative be-
for their age.
haviour, voice control is done.
Or negative reinforcers are withdrawn:
l Voice control is most effective when used in
For example, withdrawal of handpiece if the
conjunction with other communication. A
child is afraid of the noise of that and using
sudden command ‘to stop crying and pay
hand instruments so that the child will accept
attention’ may be a necessary preliminary
dental treatment in the next appointment.
measure for future communication.
l Presence (positive) or absence (negative) of these
l Sudden and firm commands can be used to
reinforcers increases the frequency of desired be-
get the child’s attention or stop the child
haviour.
from whatever is being done.
332 Quick Review Series for BDS 4th Year, Vol 1

l This is an abrupt and emphatic change in the l Telling oneself to relax is another mechanism
dentist’s tone of voice to emphasize his/her for personal coping. This technique apparently
displeasure with the child’s inattention. works by reducing tension, well-known poten-
l As soon as the child complies we should thank tiator of pain.
him/her and compliment him/her for the resul- vi. Aversive conditioning:
tant excellent behaviour. Aversive conditioning includes the following two
l Used properly in correct situations, voice con- techniques:
trol is an effective management tool. a. HOME
Objectives: b. Physical restraints
l To gain the patients attention and com-

pliance. a. Hand over mouth exercise technique


l To avert negative or avoidance behav- (HOME):
iour. l This technique was first described in the

l To establish authority. 1920s by Dr Evangeline Jordan.


Indications: l Levitas described hand over mouth

l Voice control is indicated for the unco- technique (HOMT) as HOME. A hand
operative, inattentive and communica- is placed over the child’s mouth and
tive child. behavioural expectations are calmly
Contraindications: explained. The child is told that the
l In children who due to age disability, hand will be removed as soon as ap-
medication or emotional immaturity are propriate behaviour begins. When the
unable to understand and cooperate.} child responds the hand is removed and
iv. Coping: the child’s appropriate behaviour is
l Patients differ not only in their perception and reinforced. The method may require
response to pain but also in their ways of deal- reapplication.
ing or coping with the stress associated with l It was called emotional surprise therapy

painful experiences. by Lampshire and as aversive condition-


l Same can be used to modify child’s behaviour ing by Kramer.
in the dental clinic. Objectives of HOME therapy are as follows:
Different coping mechanisms are i. To gain the child’s attention enabling
l Distraction or displacement of atten- communication with the dentist so that
tion away from the threat. Frequently appropriate behavioural expectations can
the patient spontaneously endeavours be explained.
to utilize this coping behaviour by ii. To ensure the child’s safety in the deliv-
thinking of something pleasant or di- ery of quality dental treatment.
verting or this may be done deliber- Indications of HOME:
ately by the dentist by talking to the l For normal children who are momen-

child and asking him/her interesting tarily hysterical, belligerent or defiant.


questions. l Used for children with sufficient matu-

l Verbalizing fears to others is another rity to understand simple verbal com-


well-known way of coping. Expressive mands.
communication serves to release Contraindication of HOME:
tension. l Immature, frightened or the child with a

l Another best way is facing the threat of serious physical, mental or emotional
stress or the pain enjoying affiliative handicap.
behaviour when people feel threatened, b. Physical restraints:
they prefer to be with others. l Protective stabilization or restraining is

v. Relaxation: defined as the restriction of patient’s


l Jacobson and others used specific relaxation freedom of movement, with or without
technique. Relaxation usually involves a se- the patient’s permission, to decrease risk
ries of basic exercise that may take several of injury while allowing safe completion
months to learn and which require the pa- of treatment.
tient to practice at home for at least 15 min l Parental or guardian consent must be

each day. obtained prior to use of restraints.


Section | I  Topic-Wise Solved Questions of Previous Years 333

Objectives of using restraints are Q.7. Describe the different types of behaviour of chil-
i. To reduce or eliminate untoward movement. dren.
ii. To protect the patient and dental staff from Ans.
injury.
iii. To facilitate delivery of quality dental treat- [Same as LE Q.1]
ment. Q.8. How does dental office atmosphere affect behav-
Indications: iour of the children?
l A patient who requires immediate diagno-

sis or treatment and cannot cooperate due Ans.


to lack of maturity, mental or physical dis- [Same as LE Q.1]
ability.
The following are some commonly used physical Q.9. Discuss the management of the first time dental
restraints: patient.
l Oral: Ans.
Mouth props, padded wrapped tongue
blades, a finger guard or an intraocclusal [Same as LE Q.2]
thimble and rubber/plastic bite blocks Q.10. Discuss the child’s first visit to the dental clinic.
l Body:
Ans.
Papoose board, triangular sheet, Pedi-wrap,
bean bag dental chair insert, safety belt and [Same as LE Q.2]
extra-assistant.
Q.11. How does parental influence affect the behaviour
l Extremities:
of a child in paedodontic practice? Add a note on parent
Posey strap, velcro straps, towel/tape and
counselling.
extra-assistant.
l Head: Ans.
Forearm support, head positioner, plastic [Same as LE Q.4]
bowl and extra-assistant.
l Physical restraints should never be used Q.12. Briefly mention about HOME care for the child
as punishment. An explanation of their patient.
benefits should be presented if commu- Ans.
nication is possible with the child as well
as parents. [Same as LE Q.5]
vii. Implosion therapy: Q.13. Describe modelling therapy in behaviour manage-
l In this technique the patient is flooded with ment in paedodontic practice.
many stimuli which have affected him/her
Ans.
adversely.
l The child has no other choice but to face it [Same as LE Q.6]
until the negative behaviour disappear. Q.14. What are the different methods you would adopt
l It comprises of HOME technique, voice
to manage a difficult child in the dental clinic.
control and physical restraints together.
viii. Retraining: Ans.
l It is required in children displaying consid- [Same as LE Q.6]
erable apprehension or negative or unco-
Q.15. Discuss various behavioural managements of a
operative behaviour, which may be due to
4-year-old child throwing temper tantrums.
a previous dental visit.
l The objective of retraining is to build a new Ans.
series of images and associations in child’s
[Same as LE Q.6]
mind.
l If the child’s expectancy of getting hurt Q.16. Classify behaviour management techniques. Ex-
does not come true, a new series of expec- plain how you would manage a 5½-year-old boy exhibit-
tancies are learned that the paedodontist and ing temper tantrums.
his/her auxiliaries can be trusted. The child
Ans.
develops a new perception of dental clinic
and a new relationship to dentistry. [Same as LE Q.6]
334 Quick Review Series for BDS 4th Year, Vol 1

Q.17. Define behaviour management and behaviour


{SN Q.1}
shaping. Enumerate various on pharmacological and
pharmacological techniques used for behaviour man- l The child is the most significant part and is depicted
agement. Explain in detail tell-show-do. at the apex of the triangle.
l Mother plays the most significant part in the family’s
Ans.
influence in child behaviour since she is the one the
[Same as LE Q.6] child will be usually most attached to, though other
members also play a significant role.
Q.18. Define behaviour management. Explain in detail
l The parent’s attitudinal structure, which moulds,
TSD technique.
shapes and directs child behaviour in the early period
Ans. of the offspring development, is affected by socio-
economic position, cultural development and ethnic
[Same as LE Q.6]
background

SHORT ESSAYS: l The socioeconomic status of the family unit directly af-
fects its attitude towards the values of the dental health
Q.1. Paediatric treatment triangle. process.
l The low-income group or parents with below average
Ans.
education have a tendency to attend dental needs when
the symptoms dictate. Certain of these families harbour
anxieties and fear of dental treatment, and their children
{SN Q.1} take on these fears and tend to be less cooperative.
l Cultural standards and ethnic orthodoxies have some
Paedodontic triangle was given by Wright. bearing on the degree of acceptance of dental health
l The child occupies the apex of the triangle. measures. Their closed attitude foster anxieties among
l Focus is on the dentist and the parent. the children producing disparate behaviour forms.
l All the three are interrelated.

Recently ‘Society’ also has been added. Q.2. Aversive conditioning.


Ans.

CHILD [Ref LE Q.5]


Q.3. Wright’s classification of behaviour of children in
dental office.
Ans.
SOCIETY
[Ref LE Q.1]
Q.4. TSD.

PARENT DENTISTS Ans.


[Ref LE Q.6]
Q.5. Describe modelling technique.
l In the treatment of children, a child cannot be treated as Ans.
a single separate entity since his/her behaviour will de-
pend a great deal on his/her parents, siblings or the fam- l Bandura (1969) developed from social learning princi-
ily as a whole. ples a behavioural modification technique called model-
l The paedodontic treatment triangle depicts this. All ling or imitation.
parts of the triangle are interrelated and the arrows show l The basic modelling procedure involves allowing a

they are dynamic or ever changing. patient to observe one or more individuals (models)
Section | I  Topic-Wise Solved Questions of Previous Years 335

who demonstrate appropriate behaviours in a particular l The response to a stimulus which produces a satisfac-
situation. tory outcome will be repeated whereas those which re-
l The patient will frequently imitate the models behaviour sult in disagreeable results will tend to diminish.
when placed in a similar situation. The model can be l Some of the terms identified with operant conditioning

live, e.g. siblings, other children or parents or filmed. and commonly used methods of influencing an individ-
For example Mickey mouse undergoing dental treat- ual response are
ment in picture or video format, with equally successful A. Reinforcement:
results. It can be positive or negative.
Objectives of modelling according to Rimm and Masters: B. Systemic desensitization/counterconditioning:
l Stimulation of acquisition of new behaviours. Method to eliminate learned maladaptive responses
l Facilitation of behaviours already in the patient’s by substituting more appropriate ones.
repertoire in a more appropriate manner or time C. Reward:
l Disinhibition of behaviour avoided because of fear The result of adding positive outcomes or removing
l Extinction of fears negative ones.
Steps in modelling: D. Punishment:
i. First the patient’s attention is obtained. The result of adding negative outcomes and or re-
ii. The desired behaviour is modelled. moving positive ones, thus weakening the behaviour
iii. Physical guidance of the desired behaviour may be or responses.
necessary when the patient is initially expected to Q.7. Factors affecting child behaviour in the dental
mimic the modelled behaviour. office.
iv. Reinforcement of the required behaviour:
l In private practices modelling technique yields Ans.
significant benefit with minimum effort. [Ref LE Q.1]
Rather than making the child patient wait in
the waiting room where they may be adversely Q.8. Types of cry.
influenced by maternal anxiety associated Ans.
with the dental situation, children may be
brought into an operatory immediately upon
arrival in the office if a suitable model is being
treated. {SN Q.8}
l It is observed that multiple model simultane-

ously undergoing dental procedures seems to Elsbach (1963) described four types of children’s cries
have remarkable calming effects on the anxious as follows:
child. i. Obstinate cry
l According to Rimm and Masters, the effective- ii. Frightened cry
ness of multiple model is that ‘while single iii. Hurt cry
model might be presumed by the child to have iv. Compensatory cry
some special talents that allow them to be fear-
less, this is less likely to be the case among a
i. Obstinate cry:
group of divergent models’.
l It is exhibited by a child who throws a temper tan-
l Furthermore, multiple models are likely to
trum and is loud, high pitched and has been charac-
vary slightly in the ways in which they demon-
terized as a siren-like wail.
strate fearless behaviour, thus providing
l This form of belligerence represents the child’s exter-
greater latitudes of behaviour possibilities for
nal response to his/her anxiety in the dental situation.
the child.
ii. The frightened cry:
Q.6. Operant conditioning. l It is usually accompanied by a torrent of tears and

convulsive breath catching sobs.


Ans.
l The child emitting this type of cry has been over-

l An individual learns to produce voluntary responses whelmed by the situation.


which plays a major role where the outcome is instrumen- l It is the dentist’s responsibility to instil confidence

tal in bringing about the reoccurrence of the stimulus. in the frightened child by providing a series of care-
l This theory explains development or continuation of fully structured dental experiences that will allow
behaviour as a result of reinforcement. the child to cope.
336 Quick Review Series for BDS 4th Year, Vol 1

iii. The hurt cry: l His/her vocabulary may vary greatly (12–1000 words)
l It may be loud and is frequently accompanied by a and thus his/her comprehension may be more depen-
small whimper. dent on facial expression and tone of voice than words
l The first indication that the child is in discomfort alone.
may be a single tear falling from the corner of the l Also he/she will need to grasp and feel objects to

eye and running down the child’s cheek. totally understand their meaning.
l The hurt cry is easily identified because the child l The 2-year-old fears falling, sudden unexpected

will state either voluntarily or when asked that he/ movements, loud noises and strangers. The dental
she is being hurt. situation with its unusual sounds, smells, bright
l Some children may be in pain but control their lights and tilting chairs can produce fear in the child.
physical activity so that the dentist is unaware of a l It is advisable to have the parent accompany him/her

problem. into the operatory to provide him/her with security


l When it is recognized that the child is in pain, den- and reassurance.
tal procedure should be stopped and satisfactory l Various behaviour management techniques like par-

pain control obtained. ent counselling, voice control and so on can be used
iv. Compensatory cry: in this age group.
l According to Elsbach, it is not a cry at all. l Different behaviour modification techniques used for

l It is a droning monotone, the child makes to drown children of this age group are
out the noise of the dentist’s drill. i. TSD technique
l While it may be annoying to the dentist, it is the ii. Desensitization
child’s way of coping with what he/she considers iii. Modelling
unpleasant auditory stimuli. iv. Contingency
l It is a successful coping strategy the child has de-
Q.12. Psychic triad.
veloped to cope with the anxiety he/she is experi-
encing and therefore the dentist should make no Ans.
attempt to stop it. [Same as SE Q.1]
Q.9. Frankle’s behaviour rating scale. Q.13. What do you understand by paediatric triangle?
Give its significance.
Ans.
Ans.
[Ref LE Q.1]
[Same as SE Q.1]
Q.10. Voice control.
Q.14. HOME.
Ans.
Ans.
[Ref LE Q.6 and Q.3]
Q.11. What is behaviour management? How will you [Same as SE Q.2]
manage a fearful child of 2 years using different behav- Q.15. Hand over mouth exercise (HOME).
iour modification techniques?
Ans.
Ans.
[Same as SE Q.2]
Behaviour management is a means by which the dental
health team effectively performs treatment for a child and Q.16. Modelling.
at the same time installs a positive dental attitude. Ans.
Management of child of 2 years age:
l This is a period of tremendous physical, intellectual [Same as SE Q.5]
and emotional growth of the child.
l His/her mastery of toilet training in this year gives
SHORT NOTES:
him/her a sense of achievement; self-control and in-
dependence from others. Q.1. Psychic triad.
l Through his/her increased language capabilities he/ Ans.
she learns to express how he/she feels and to make
his/her needs known. [Ref SE Q.1]
Section | I  Topic-Wise Solved Questions of Previous Years 337

Q.2. Stoic behaviour. Q.9. Define behaviour management and behaviour


shaping.
Ans.
Ans.
l The stoic behaviour is seen in physically abused
children. Behaviour management:
The characteristics of this behaviour are as follows: It is a means by which the dental health team effectively
l Accepts treatment and efficiently performs treatment for a child and at the
l Tense cooperative same time, installs a positive dental attitude.
l Whining and timid Behaviour shaping:
l Milder but little anxious It is that procedure which very slowly develops behaviour
l May shield behind the parent by reinforcing successive approximations of the desired
l May whimper but does not cry hysterically, seldom behaviour until the desired behaviour comes to be.
are the tears seen.
Q.10. Reinforcement.
Q.3. Classical conditioning.
Ans.
Ans.
[Ref LE Q.6]
l Classical conditioning theory was given by Ivan Petriv-
Q.11. Define modelling.
ich based on stimulus–response reflex.
l It was developed through experimentation with dogs on Ans.
stimulus–response to an external stimulus.
[Ref LE Q.6]
l When two stimuli occur together at the same time, re-

sulting in a response, this response can be obtained by Q.12. Name few physical restraints.
stimulating any the one of original stimuli.
Ans.
l In dentistry we can use this theory for stimulating the

development of good habits, breaking old habits, to re- l Protective stabilization or restraining is defined as the
move fear and to develop positive attitude. restriction of patient’s freedom of movement, with or
without the patient’s permission, to decrease risk of in-
Q.4. Give the Frankel behaviour rating scale for record- jury while allowing safe completion of treatment.
ing children’s behaviour in dental office.
[Ref LE Q.5 for rest of the answer]
Ans.
Q.13. Communicative management technique.
[Ref LE Q.1]
Ans.
Q.5. Desensitisation. l Basic ways of communication are verbal and nonverbal.
Ans. Verbal communication:
l Verbal communication is through conversation. By
[Ref LE Q.6] involving the child in a conversation the paedodontist
not only learns about the patient, but also may relax
Q.6. Define TSD.
the youngster.
Ans. l Generally verbal communication is best initiated for

younger children with complimentary comments,


[Ref LE Q.6]
followed by questions that elicit an answer other than
Q.7. Define HOME technique. yes or no.
l It is important that communication occur from a
Ans. single source. The message must be understood in
[Ref LE Q.5] the same way by both the sender and the receiver.
Nonverbal communication (multisensory communica-
Q.8. Name different types of cry. tion):
l Nonverbal message also can be sent to patients or
Ans.
received from them. Body contact can be a form of
[Ref SE Q.8] nonverbal communication.
338 Quick Review Series for BDS 4th Year, Vol 1

For example, the clinician’s simple act of placing a Q.16. Rewarding.


hand on a child’s shoulder while sitting on a chair
Ans.
side stool conveys a feeling of warmth and friendship.
l Eye contact is also important. A child who avoids it l Reward is one of the factors under the control of dentist
is often not fully prepared to cooperate. Sitting and which influences the child behaviour.
speaking at eye level allows for friendlier and less l One of the most important rewards sought by the child

authoritative communication. is the approval of the paedodontist.


l Active listening by the paedodontist is very impor- l Tiny gifts make fine reward. Tiny gold stars, toys or

tant thus encouraging the kind of genuine communi- stickers make good gifts.
cation in which the patient is stimulated to express l Small token gifts like baby toothbrushes after good be-

feeling and the paedodontist does the same as a nec- haviour also make fine rewards. It is the recognition
essary process in communication. more than the material that makes the child happy.
l Flattery can also be used as a reward after the treatment.
Q.14. Euphemisms.
Q.17. Paedodontic triangle.
Ans.
Ans.
l To improve the clarity of message to young patients
‘euphemisms’ or word substitutes are used to explain [Same as SN Q.1]
things and procedures better.
Q.18. Keye’s triad.
l Various euphemisms used in case of paediatric patients are
Ans.
Dental terminology Word substitutes
[Same as SN Q.1]
i. Rubber dam Rubber rain coat
Q.19. Frankel rating.
ii. Sealant Tooth paint
iii. Topical fluoride gel Cavity fighter Ans.
iv. Air syringe Wind gun [Same as SN Q.4]
v. Water syringe Water gun Q.20. Frankel’s behaviour rating scale.
vi. Suction Vacuum cleaner
Ans.
vii. Needle and anaesthesia Sleepy water
[Same as SN Q.4]
viii. Radiographic equipment Camera, etc.
Q.21. Tell-show-do technique.
Q.15. Contingency management. Ans.
Ans. [Same as SN Q.6]
[Ref LE Q.6]

Topic 6
Therapeutic Management
COMMONLY ASKED QUESTIONS
LONG ESSAY:
1 . Classify pharmacological behaviour management. Describe the premedication in detail.
2. Define and discuss briefly conscious sedation. Give indications and contraindications of N2O–O2 analgesia.
Describe the equipment, clinical features at various concentration and complications of N2O–O2 analgesia.
Section | I  Topic-Wise Solved Questions of Previous Years 339

SHORT ESSAYS:
1 . Enumerate differences between conscious sedation and general anaesthesia.
2. Conscious sedation in paediatric dentistry. [Ref LE Q.2]
3. Midazolam.
4. Advantages of nitrous oxide and oxygen conscious sedation. [Ref LE Q.2]
5. Anaesthetic preparation of child.
6. Indications and contraindications for general anaesthesia.
7. Conscious sedation. [Same as SE Q.2]
8. Nitrous oxide sedation. Advantages and disadvantages. [Same as SE Q.4]

SHORT NOTES:
1 . Nitrous oxide analgesia. [Ref LE Q.2]
2. Ketamine.
3. Diazepam.
4. Promethazine.
5. Define conscious sedation and enumerate various agents used for the same. [Ref LE Q.2]
6. Pulse oximeter and its applications in paediatric dentistry.
7. Indications for conscious sedation.
8. What do you understand by nitrous oxide–oxygen analgesia? [Same as SN Q.1]
9. Conscious sedation. [Same as SN Q.2]

SOLVED ANSWERS
LONG ESSAY:
Q.1. Classify pharmacological behaviour management. General anaesthesia:
Describe the premedication in detail. A controlled state of unconsciousness, accompanied
by partial or complete loss of protective reflexes, in-
Ans.
cluding inability to maintain an airway indepen-
l To provide the best quality dental service for the paedi- dently and respond purposefully to physical stimula-
atric patient, one may need to utilize pharmacological tion or verbal command.
means to obtain a quiescent, cooperative patient. Premedication:
Indications for pharmacological behaviour manage- Guidelines for the use of premedication are
ment techniques are as follows: i. Detailed medical history:
l Children who are either extremely young It helps to prevent undesired drug interac-
l Have reduced mental capacity tions.
l Intensely fearful or have severe medical problems ii. Selecting a premedication agent:
which affect their ability to be cooperative The type and dosage used should never
Different types of pharmacological behaviour manage- impair the vital reflexes of the child.
ment techniques are as follows: iii. Consent and preoperative instructions:
i. Premedication Should be given before any procedure.
ii. Conscious sedation – parental/inhalation iv. The method or route of administration
iii. General anaesthesia should be clearly explained.
Conscious sedation: v. Specific instruction regarding eating and
A minimally depressed level of consciousness drinking prior to administration should be
that retains the patient’s ability to maintain an given.
airway independently and respond appropriately vi. Information should be provided about side
to physical stimulation and verbal command. effects like drowsiness, vertigo, exhilara-
Deep sedation: tion or agitation.
A controlled state of depressed consciousness, viii. High levels of personnel training is a
accompanied by a partial loss of protective re- must
flexes including inability to respond purposefully ix. Call for documentation of events during
to a verbal command. the treatment (vital signs, etc.)
340 Quick Review Series for BDS 4th Year, Vol 1

x. Postoperative care includes – discharge only l The child who displays greater physical
when vital signs are stable, patient is alert, can activity will usually require higher dosage
walk with minimal assistance. than will a child who is more passive.
xi. While most of the oral premedications act d. Route of administration:
best when taken on an empty stomach, they l Drugs given by i.v. will act more rapidly

may also be administered with a liquid or with and are given in lower dose, whereas a drug
food in order to disguise the unpleasant taste given orally act more slowly and dosage
or to prevent nausea. requirement is more.
Administration of premedication: l Intramuscular administration of drugs re-

l It is better to administer premedication in the sults in intermediate onset of action and


dental office as the dentist can use routes other dosage requirements.
than oral and also accurate timing of the ad- e. Environment:
ministration can be done. l Generally lower doses are required when a

l Another advantage of office administration is drug is taken in a nonstress full environment


that treatment can be begun at the time of opti- and higher doses are required under stress-
mum effect on the child who responds quickly ful environment of the dental office, where
to the drug, whereas if the drug is administered auditory, tactile and visual stimulation can
at home, the child may be in transit during be intense.
peak drug activity. f. Time of the day:
Care during premedication: Dosage may be reduced if given during the nap
l Child should never be left unattended. time of the child, conversely dosages may have
l The child’s environment should be kept as to be elevated when the drug is administered
quiet as possible to enhance drug efficacy. during active play time of the child.
l The child who is aroused before the medication General causes of premedication failure:
has reached peak activity may remain excited. l Prescription of an insufficient dose of drug or

l Child aroused by painful stimuli may display intentional reduction of dosage by the parents.
considerable agitation and confusion. l Failure of the child to swallow the drug or ex-

l Once the desired level of sedation is obtained it pectoration or vomiting of a portion of the
is still essential to administer local anaesthesia. drug.
Postoperative instructions: l Children with medical condition such as brain

l After the completion of the treatment the child damage and other problems are often inade-
whether is asleep or awake, will be in a sedated quately premedicated and may require in-
condition for many hours, depending on the creased doses or different drugs.
drug and the dosage used.
l Upon awakening, the child may complain of

hunger or thirst if the sleep has been pro- Q.2. Define and discuss briefly conscious sedation. Give
longed. It is better to start with little water and indications and contraindications of N2O–O2 analgesia.
then to proceed with solid foods. Describe the equipment, clinical features at various con-
l Recovery period should be under supervision. centration and complications of N2O–O2 analgesia.
Factors influencing dosage: Ans.
a. Age:
Young’s rule has not been found to be an effec-
[SE Q.2]
tive method of determining premedication
dosage. {Conscious sedation:
b. Weight: A minimally depressed level of consciousness that re-
Clarke’s rule has also found to be ineffective in tains the patient’s ability to maintain an airway indepen-
determining premedication dosages. dently and respond appropriately to physical stimula-
Clinical experience has proved to be better tion and verbal command.}
than Clarke’s rule for premedication.
c. Emotional state and activity: (SE Q.2 and SN Q.5)
l Extremely anxious or defiant child will re- {(Sedation: Routes and agents:
quire more premedication than will the I. Inhalation:
mildly apprehensive child. l Nitrous oxide
Section | I  Topic-Wise Solved Questions of Previous Years 341

II. Oral route (several drugs): Objectives:


3
l Hydroxyzine (vistaril) 25 mg/5 cm . Benett (1978) stated the objectives as follows:
3
l Promethazine (phenergan) 12.5 mg/5 cm ., l Patients mood should be altered.
2.5 mg/5 cm3 l Patients should be conscious, respond to
3
l Chloralhydrate (Noctec) 500 mg/5 cm (not verbal stimuli and be cooperative.
recount for children younger than 6 years) l All protective reflexes intact, vital signs
3
l Meperidine (Demerol) 50 mg/5 cm , contra- stable and normal.
indicated (C/I) in children with chronic ob- l Childs pain threshold should be increased.
structive pulmonary disorder (COPD), hypo- l Amnesia should occur.
thyroid or liver dysfunction There is only one inhalation agent that meets the
3
l Diazepam (valium) 5 mg/5 cm elixir, 2, 5, requirement of conscious sedation and that is ni-
10, 15 mg tabs (indicated in children ,6 trous oxide and is described below:
years age)
l Triazolam (Halcion) 0.125, 0.25 mg tab. {SN Q.1}
l Chlorpromazine (Thorazine) 10 mg/mL
syrup, 10, 25, 100, 200 mg tab (useful in sever Nitrous oxide (N2O):
l It is the most frequently used sedation agent by
behavioural problems)
III. Intramuscular: 85% of paediatric dentists.
l Nitrous oxide is slightly sweet smelling, colour-
l Ketamine (Ketalar) 10, 50 mg/mL parenteral

or oral use. less, noninflammable and inert gas heavier than air.
l It is a weak analgesic, although this effect can be
l Midazolam (versed) 1 and 5 mg/mL. Rapid

onset of action and used mainly for short influenced by the psychological preparation of the
procedures patient.
l It is compressed in cylinders as a liquid that
IV. Intravenous:
l Midazolam is commonly used agent and is
vaporizes on release.
l It has a blood gas coefficient of 0.47 and has rapid
best for invasive procedures of short duration.
l Mechanism in conscious sedation or relative
onset and recovery time due to low solubility in
analgesia is that the patient’s threshold to blood.
l Should be offered to children with mild-to-
pain, cold, warmth and light touch is in-
creased. moderate anxiety to enable them to accept den-
l Although the special senses may be partly
tal treatment better and to facilitate coping
obtunded and sensation of numbers is de- across sequential visits.
l Can be used to facilitate dental extractions in children.
scribed, superficial and deep reflexes remain
active and the sensorium remains clear.)} l It is absorbed quickly from the alveoli of the

lungs and is physically dissolved in the blood


[SE Q.2] with no chemical combination anywhere in the
body and excreted through lungs without any
{Indications: biotransformation.
l Patients who cannot cooperate or understand defini-
Actions (pharmacodynamics) of nitrous oxide:
tive treatment.
l Without impairing motor function, it creates an
l Patients lacking cooperation of lack of psychological
altered state of awareness and is a CNS depressant.
or emotional maturity.
l Increases the respiratory rate and decreases the
l Patients who are fearful and anxious with dental care
tidal volume.
requirements.
l Cardiac output is decreased and peripheral vascu-
Contraindications:
lar resistance is increased.
l COPD pregnancy, myasthenia, epilepsy, obe-
Absorption, fate and excretion:
sity and bleeding disorders
l Enters blood by crossing pulmonary epithelium.
l Unwilling or unaccompanied patients
l During early phases of administration – brain,
l Dental difficulties: prolonged surgery and in-
heart, liver and kidney absorbs the major portion
adequate personnel
of nitrous oxide from blood.
l 1st trimester of pregnancy
l Exhaled through lungs.
l Hypersensitivity to the agent}
to patient for 2–3 min

342
Slowly introduce nitrous oxide and encourage
Quick Review Series for BDS 4th Year, Vol 1
the patient to breathe through nose

Nasal in haler
Fresh gas Explain the sensation to be felt as – floating,
giddy and tingling of digits

ly ng
mb bi
Flow meter Exhaled gas

se l tu
Bag tee

as rnia
Co
22 mL Adjust the concentration to 30% nitrous oxide
flow
adapter and 70% oxygen
3-Lbag Slide
adjuster
Continuous monitoring is required throughout
Fresh gas Vacuum hose
‘Y’connector ‘Y’connector the procedure
Fresh gas
tubing
Vacuum
hose
Vacuum
Outer mask One way valve After completion of procedure give 100% oxygen
control bock for 5 min
Inner mask

Two techniques have been described:


Gas leakage a. Slow induction technique
Systems nasal hood
b. Rapid induction or ‘surge’ technique
To vacuum
Slow induction technique:
l Described by Langa (1968).
Requirements of the equipment used for the induction of
l First with 100% oxygen total litre flow rate/min of
nitrous oxide:
gases should be established. For adults 5–7 L/min
i. The equipment should have a continuous flow de-
and children of 3–4 years – 3 L/min.
sign with flow meters capable of accurate regulation.
l Tell-show-do approach should be used to introduce
ii. Automatic shut down if oxygen level falls ,20 %.
the child to the operatory.
iii. Flush level for easy and immediate flushing of the
l After stabilization of the nose piece, 100% oxygen is
system with 100% oxygen.
delivered for 3–5 min.
iv. They can be either mobile units or operating from
l Then the nitrous oxide level is increased to 30%–
a central supply.
35% for 3–5 min (induction period).
v. Good and efficient scavenger system.
l During this induction period, the dentist continu-
vi. Adequate size nasal hood should be used for the
ously communicates with the child to promote relax-
adults and children.
ation and reinforce cooperative behaviour.
Techniques:
l If the child is older, he/she can be asked for the
l Critical to the nitrous oxide procedure is the graceful
physical changes like tingling sensation in the finger
acceptance of the nosepiece by the child, since this
and toe and the eyes will take a distant gaze with
treatment is not advised for the resistant paedodontic
sagging of eyelids.
patient.
l Most dentists will prefer to increase the level of ni-
l This requires explanation at the youngster’s level of
trous oxide to 50% for 3–5 min to provide the maxi-
comprehension, a slow approach and behaviour
mum effect for the administration of LA.
shaping with positive reinforcement throughout.
l In dental practice, concentration of N2O . 50% is
The common procedure of nitrous oxide induction in gen- contraindicated. After local anaesthesia the concen-
eral is as follows: tration can be brought down to 30%–35%.
l After the treatment-inhalation of 100% oxygen for
Thorough inspection of equipment not less than 5 min should be continued.
l This allows diffusion of nitrogen from the venous

blood into the alveolus, which is then exhaled as ni-


The mask is placed over nose trous oxide through respiratory tract and also allows
the patient to return to pretreatment activities without
any incident. Inadequate oxygenation may produce
Bag is filled with 100% oxygen and delivered nausea, light headedness or dizziness.
to patient for 2–3 min l After the procedure, the child should be kept in

supine position or on his/her side to maintain air-


way patency. Upon arriving home the child should
Slowly introduce nitrous oxide and encourage be placed on his/her side and observed carefully for
the patient to breathe through nose the first hour. If he/she wishes to sleep, he/she can
be allowed to do so.

Explain the sensation to be felt as – floating,


giddy and tingling of digits
Section | I  Topic-Wise Solved Questions of Previous Years 343

Rapid induction technique: more appropriate to administer sedation in hospital


l Described by Sorenson and Roth (1973) and Simon environment supported by a consultant anaesthetist.
and Vogelsberg (1975). Disadvantages:
l Initiation phase is started by administering equal l The common acute adverse effects associated with

parts of nitrous oxide and oxygen for 10–15 min. this type of sedation are nausea, whereas chronic
l This is followed by maintenance phase where the effects may be impotence, liver toxicity and recre-
nitrous oxide is reduced by half for 40 min. ational abuse.
l Withdrawal is by administering oxygen only. l Exposure to nitrous oxide can result in depression of

l Oxygen is used to prevent anoxia, which is produced vitamin B12 activity resulting in impaired synthesis
if nitrous oxide is used alone. of RNA.}
Potential problems and solutions:
i. Sleep-frequent arousal or communication is advised.
ii. Airway obstruction – frequent repositioning of the
SHORT ESSAYS:
head is needed to hyper extend the mandible so that
Q.1. Enumerate differences between conscious sedation
the tongue is brought forward
and general anaesthesia.
iii. Vomiting can be prevented by
l Using minimum effective concentration. Ans.
l Avoiding prolonged procedure.

l Empty stomach inhalation. Conscious sedation General anaesthesia


l Slow return to upright position. i. At several visits the treat- i. Generally single sitting,
l Aspiration is unlikely – so just ask the patient ment procedures may be once in a lifetime
to vomit in a chair side emesis basin if there is performed. procedure.
ii. Patient is cooperative but ii. Patient is uncooperative.
vomiting.
anxious and fearful.
iv. Diffusion hypoxia – Since nitrous oxide has a iii. No extensive investiga- iii. At least basic investiga-
lower blood solubility, it rapidly diffuses into al- tions and no premedica- tions and also premedica-
veoli and dilutes the alveoli air causing a fall in the tion is required; no NPO tion and Nil per oral
partial pressure of oxygen in alveoli. 100% oxygen required. (NPO) is strictly required.
iv. Airway is maintained as iv. Ventilation is required.
for 10 min.
patient is conscious.
v. High concentration of nitrous oxide should be v. No mortality. v. 99% success rate
avoided as the pressure will be created in the air reported.
filled body cavities especially in the middle ear. vi. Reoperation period is vi. Time-consuming
Contraindications: 1–2 min. procedure.
l Very young children
vii. Patient feels he/she is in vii. Patient cannot control the
control of the situation. situation.
l Children having common cold, tonsillitis and nasal

blockage Q.2. Conscious sedation in paediatric dentistry.


l Bleomycin chemotherapy
l Pre-cooperative children Ans.
l First trimester of pregnancy
[Ref LE Q.2]
[SE Q.4] Q.3. Midazolam.
{Advantages: Ans.
l It is a viable and cost-effective alternative to general
anaesthesia. l Midazolam is a benzodiazepine similar to diazepam but
l Nitrous oxide sedation has minimal effect on cardio- with twice the potency.
vascular and respiratory function and the laryngeal l The drug is highly lipophilic, providing for rapid ab-
reflex. sorption from the gastrointestinal tract as well as rapid
l Using nitrous oxide inhalation sedation in conjunc- entry into brain tissue.
tion with other sedatives may rapidly produce a state l Elimination is also rapid, giving a shorter duration of
of deep sedation or general anaesthesia. activity.
l Nitrous oxide should be used with caution on ASA 3 l The elimination half-life is 10 times less than that of
and ASA 4 status patients, for whom it would be diazepam.
344 Quick Review Series for BDS 4th Year, Vol 1

l After intravenous administration, sedation occurs in ii. Preparation indications


3–5 min. Recovery occurs in 2 h but is variable and may iii. Informed consent
require up to 6 h for complete return to baseline values. iv. Instructions to parents
There is no rebound phenomenon from metabolites. v. Documentation
Available forms:
i. Patient selection and choice of technique used:
l IV Midazolam use is widely reported in adults.
l The practitioner should have a rationale for making
l Midazolam can also be effectively given intra-
the choice as to which patients will most likely ben-
muscularly.
efit from the use of sedation as it embodies a group
l Recently the oral form has become available and
of techniques designed to alter patient behaviour.
holds great promise for paediatric conscious seda-
l The indiscriminate application of these techniques
tion.
to all patients must be avoided.
l Intranasal midazolam produces sedative effect
l Several behavioural or anxiety assessment profiles
within 5 min of its administration.
have been developed that can be of great help to the
Advantages:
practitioner as the various techniques are intro-
l High water solubility.
duced into a practice.
l The possibility of thrombophlebitis is reduced to
l As one gains experience, this decision becomes one of
a minimum.
clinical judgment as to which approach produces the
Dosage:
most successful results for specific types of patients
l Oral – 0.25–1 mg/kg to a maximum single dose of
for that individual practitioner.
20 mg.
l No one technique or agent, or combination of agents,
l I.m. – 0.1–0. 15 mg/kg to a maximum dose of
should be expected to be successful every time.
10 mg.
l One should choose the agent and technique that
l Midazolam may produce respiratory depression
best fits the patient type as well as the nature of
with higher doses.
what needs to be accomplished.
Q.4. Advantages of nitrous oxide and oxygen conscious ii. Preparation indications:
sedation. l A thorough medical history is required to deter-

mine whether a patient is suitable for sedative


Ans.
procedures.
l Nitrous oxide (N2O) is the most frequently used seda- l This, along with a recent physical examination,

tion agent by 85% of paediatric dentists. constitutes a risk assessment or physiologic status
l It is slightly sweet smelling, colourless, noninflamma- evaluation.
ble and inert gas heavier than air. l This health evaluation should be used to place the

l It is a weak analgesic, although this effect can be influ- patient in one of the categories set forth by the
enced by the psychological preparation of the patient. American Society of Anesthesiologists.
l It is compressed in cylinders as a liquid that vaporizes l Patients who are in ASA I are frequently considered

on release. appropriate candidates for minimal, moderate or


l It has a blood gas coefficient of 0.47 and has rapid onset deep sedation.
and recovery time due to low solubility in blood. l Some children assigned to ASA class II or III may

l It should be offered to children with mild-to-moderate actually benefit from this approach, but this must be
anxiety to enable them to accept dental treatment better determined in consultation with the child’s physician.
and to facilitate coping across sequential visits. l Generally, patients categorized into classes III and

l It can be used to facilitate dental extractions in children. IV, children with special needs and those with ana-
l It is absorbed quickly from the alveoli of the lungs and tomic airway abnormalities or extreme tonsillar
is physically dissolved in the blood with no chemical hypertrophy are better managed in a hospital set-
combination anywhere in the body and excreted through ting, according to AAPD guidelines.
lungs without any biotransformation. The physical evaluation should include the following:
[Ref LE Q.2 for rest of the answer] i. Vital signs, including heart, respiratory rates,
blood pressure and temperature.
Q.5. Anaesthetic preparation of child. ii. Evaluation of airway patency to include tonsil-
Ans. lar size and any anatomic abnormalities like
mandibular hypoplasia that may increase the
The aesthetic preparation of child includes the following: risk of airway obstruction.
i. Patient selection and choice of technique iii. ASA classification.
Section | I  Topic-Wise Solved Questions of Previous Years 345

iv. Name, address and telephone number of the l Once solids are tolerated, there are no dietary restric-
child’s medical home. tions other than those imposed as a result of the
iii. Informed consent: dental procedure performed.
l The parent or legal guardian must be agreeable to l Knowledge on the part of the parent of what to ex-

the use of sedation for the child. pect is the most reliable way to ensure a calm, com-
l These individuals are provided complete informa- fortable and uncomplicated postsedation period.
tion regarding the reasonably foreseeable risks and l These instructions and recommendations should

the benefits associated with the particular technique be in written form and should be reviewed again
and agents being used in clear, concise terms that with the person responsible for the patient and
are familiar to them. given to this person at the time of discharge from
l The consent form can be on or part of a sedation the office.
record with space provided for the signatures of all v. Documentation:
parties. l Meticulous and accurate documentation of the seda-

iv. Instructions to parents: tion incidence is imperative.


l Information in written form should be reviewed l Procedural records should document:

with the person caring for the child and given to i. Proper once to food and liquid intake restric-
this person along with the notice of the scheduled tions
appointment. ii. The preoperative health evaluation, including
l This information should include a 24-hour contact the patient’s history and a complete physical
number for the practitioner. assessment along the patient’s current weight,
Dietary instructions should be as follows (AAPD age and baseline vital signs.
guidelines): iii. Name and address of the physician who usually
i. Clear liquids: water, fruit juices without pulp, cares for the child.
carbonated beverages, clear tea and black cof- iv. A note as to why the particular method of man-
fee up to 2 h before the procedure. agement was chosen.
ii. Breast milk up to 4 h before the procedure. v. The presence of informed consent.
iii. Infant formula up to 6 h before the procedure. vi. The delivery of instructions to the caregiver.
iv. Nonhuman milk up to 6 h before the procedure. vii. Before the sedation, a ‘time out’ should be
v. A light meal up to 6 h before the procedure, performed to confirm the patient’s name, the
e.g. toast and clear liquids. procedure to be performed and the site of
vi. It is permissible for routine necessary medica- the procedure should be documented in the re-
tions to be taken with a sip of water on the day cord (AAPD #43).
of the procedure. viii. Intraoperatively the appropriate vital signs
l The parent or guardian should also be advised that should be recorded as they are assessed. Timed
he/she will be expected to remain in the area of the notations regarding the patient’s appearance
office during the sedation appointment. should be included.
l With regard to transportation, the instructions The type of drug, the dose given and the route,
should request that a second person accompany the site and time of administration should be clearly
parent so that the person caring for the child may indicated.
be free to attend to the child’s needs during the trip If a prescription is used, either a copy of the
home. prescription or a note as to what was prescribed
l The caregiver should be advised that, on arriving should also be a part of the permanent record.
home, the child may sleep for several hours and ix. After completion of treatment, the patient
may be drowsy and irritable for up to 24 h after the should be continuously observed in an appro-
sedation. priately equipped recovery area. The patient
l It is important to stress the need for frequent obser- should remain under direct observation until
vation if the child is sleeping, to ensure an open respiratory and cardiovascular stability have
airway. been ensured.
l Activity should be restricted to quieter pursuits and x. The patient should not be discharged until the
be closely supervised for the remainder of the day. presedation level of consciousness or a level as
l Following treatment, the child should first be of- close as possible for that child has been
fered clear liquids and may advance to solid foods achieved. At the time of discharge, the condi-
as tolerated. tion of the patient should be noted.
346 Quick Review Series for BDS 4th Year, Vol 1

Q.6. Indications and contraindications for general Q.3. Diazepam.


anaesthesia.
Ans.
Ans.
l Diazepam is a benzodiazepine that is lipid soluble and
General anaesthesia produces reversible loss of all sensa- water insoluble, reaching peak levels at 2 h.
tion and consciousness. l It is rapidly absorbed from the gastrointestinal tract.

The indications and contraindications for general an- l Diazepam has strong anticonvulsant activity and pro-

aesthesia are as follows: vides some prophylaxis against this adverse reaction of
Indications: other drugs during the operative procedure.
i. Patients who cannot cooperate due to a lack of l Dosage: Oral or rectal – 0.2–0.5 mg/kg to a maximum

psychological or emotional maturity and those single dose of 10 mg, i.v. 0.25 mg/kg
with mental, physical or medical disability Supplied as: Tablets – 2, 5 and 10 mg and suspension
ii. Patients for whom local anaesthesia is ineffec- 5 mg/mL.
tive because of acute infection, anatomic varia- l Biotransformation of the drug occurs quite slowly, with

tions or allergy a half-life of 20 to 50 h. The drug has three active me-


iii. Highly uncooperative, fearful, anxious or un- tabolites, and these are more anxiolytic than sedative.
communicative child or adolescent l Ataxia and prolonged CNS effects are the only common

iv. Patients requiring prolonged surgical proce- adverse reactions that can be anticipated when diaze-
dures pam is used for conscious sedation.
v. Patients requiring immediate, comprehensive
Q.4. Promethazine.
oral–dental care
Contraindications Ans.
i. A cooperative and healthy patient with mini-
l Promethazine or Phenergan is a phenothiazine with
mal dental needs
sedative and antihistaminic properties.
ii. Medically compromised conditions which
l It is well absorbed after oral ingestion.
would make general anaesthesia inadvisable
Onset is within 15–60 min, with a peak at 1–2 h and a
Q.7. Conscious sedation. duration of 4–6 h.
l Metabolized by the liver.
Ans.
l Should be used with caution in children with a history

[Same as SE Q.2] of asthma, sleep apnoea or a family history of sudden


infant death syndrome (SIDS).
Q.8. Nitrous oxide sedation. Advantages and disadvan-
Phenothiazines lower the seizure threshold and should
tages.
be avoided in seizure-prone patients.
Ans. l Interactions: Potentiates other CNS depressants.

l Adverse reactions: Dry mouth, blurred vision, thicken-


[Same as SE Q.4]
ing of bronchial secretions, mild hypotension and extra-
pyramidal effects.
SHORT NOTES: Dosage: Oral/intramuscular – 0.5–1.1 mg/kg.
Subcutaneous – not recommended
Q.1. Nitrous oxide analgesia. Maximum recommended single dose – 25 mg
Ans. Supplied: Tablets – 12.5, 25 and 50 mg
Syrup – 6.25 and 25 mg/mL
[Ref LE Q.2] Injectable – 25 and 50 mg/mL ampules
Q.2. Ketamine. Q.5. Define conscious sedation and enumerate various
Ans. agents used for the same.
l Ketamine is a powerful analgesic, which, in small dos- Ans.
ages, can produce a state of dissociation while maintain-
ing the protective reflexes. [Ref LE Q.2]
l Side effects include hypertension, vivid hallucinations
Q.6. Pulse oximeter and its applications in paediatric
and physical movement although these are less preva- dentistry.
lent in children.
l Known to increase secretions, including salivation. Ans.
Section | I  Topic-Wise Solved Questions of Previous Years 347

l Pulse monitors are available that attach to the finger or Q.7. Indications for conscious sedation.
ear lobe and produce both visual and audible signals.
Ans.
l The pulse oximeter is one of the most valuable pieces of

electronic monitoring equipment. Conscious sedation is defined as a controlled, pharmaco-


l This device continuously assesses arterial haemoglobin logically induced, minimally depressed state or level of
oxygen saturation and pulse rate, with values updated consciousness in which the patient retains the ability to
with every heartbeat. maintain a patent airway independently and continuously
l An oxygen sensor is attached noninvasively to a digit on and to respond appropriately to physical stimulation or
the hand or foot or to the earlobe and consists of a light- verbal command.
emitting diode and light-detecting diode. Indications:
l The light-emitting diode emits both red and infrared Patient’s requiring dental treatment but cannot cooper-
wavelengths of light, and the light-detecting diode de- ate due to:
tects light transmitted through the tissue. l Lack of psychological or emotional maturity

l Red wavelengths are absorbed primarily by oxygenated l Medical, physical and cognitive disability

haemoglobin, whereas infrared wavelengths are ab- l Fearful and highly anxious behaviour

sorbed primarily by deoxygenated haemoglobin.


Q.8. What do you understand by nitrous oxide–oxygen
l The device’s processor then calculates the percent of
analgesia.
oxygenation of haemoglobin and the results are con-
veyed both audibly and visually. Ans.
l Sensor displacement is the most common cause for false

readings in children and can be minimized by using a [Same as SN Q.1]


sensor with adhesive tabs rather than a clip-on sensor. Q.9. Conscious sedation.
l The new generation of pulse oximeters is less suscepti-

ble to motion artefacts and may be more useful than Ans.


older oximeters that do not contain the updated software
(AAPD guidelines). [Same as SN Q.2]

Topic 7
Management of Handicapped Children
COMMONLY ASKED QUESTIONS
LONG ESSAYS:
1 . What are intelligence quotient (IQ) and mental retardation? Describe the features of a Down syndrome child.
2. What special attention is to be taken in the dental treatment and management of handicapped children?
3. Define and classify handicapped child. Explain how you will manage mentally handicapped children in your
dental clinic. [Same as LE Q.2]

SHORT ESSAYS:
1 . Cerebral palsy.
2. Trisomy 21. [Ref LE Q.1]
3. Learning disorders.
4. Autism.
5. Discuss various measures in the treatment and management of the mentally retarded children. [Ref LE Q.2]
6. Down syndrome. [Same as SE Q.2]

SHORT NOTES:
1 . Definition of handicapped child. [Ref LE Q.2]
2. Name the signs of learning disorders. [Ref SE Q.3]
348 Quick Review Series for BDS 4th Year, Vol 1

3 . Munchausen syndrome by proxy.


4. Autism. [Ref SE Q.4]
5. Down syndrome.
6. Handicapped children. [Same as SN Q.1]
7. Trisomy. [Same as SN Q.5]
8. Down syndrome. [Same as SN Q.5]

SOLVED ANSWERS
LONG ESSAYS:
Q.1. What are intelligence quotient (IQ) and mental re- l It is reasonable to assume that most children
tardation? Describe the features of a Down syndrome with mild retardation can cope with simple,
child. preventive and short procedures.
ii. Moderate retardation:
Ans.
l Children in this category have vocabulary and
Intelligent quotient (IQ) is the method of quantifying the language skills such that the child can commu-
mental ability in relation to chronological age. nicate at a basic level with others.
Formulated by Alfred Binet (1900s) as: l Their IQ score is in the range of 40–55.
It is measured by tasks, examining memory, spatial rela- l The children in this group are generally trained
tionship, reasoning, etc. to master certain self-help skills like dressing,
Several tests used to determine the IQ are grooming, feeding and cleaning.
i. The Cattell infant intelligence scale (used in children iii. Profound or severe retardation:
with developmental age less than 2 years) l Children in this category have little or no com-
ii. The Stanford–Binet intelligence scale (used in children munication skills.
with developmental age at least 2 years) l Their IQ may be in the range of less than 35–39.
iii. WIPPSI (Welschsler preschool and primary scale of l They invariably need pharmacological behav-
intelligence used in children with chronological ages of iour management methods to provide dental
6–17 years) care.
iv. WISC-R (Welschsler intelligence scale for children-
[SE Q.2]
revised used in individuals 16 and above)
A positive relation exists between IQ and acceptance of {Down syndrome:
dental treatment. l Most common chromosomal aberration.
Mental retardation: l Incidence is 1 in every 600 newborns.
Mental retardation has been defined by the American l It may occur due to trisomy of chromosome 21 in
Association of Mental Deficiency (AAMD) as ‘Subaverage 95%, translocation (3%) or due to mosaicism.
general intellectual functioning which originates during the Predisposing factors:
developmental period and is associated with impairment in l Advanced maternal age
adaptive behaviour’. l Uterine and placental abnormalities
l Initially, mental deficiency was assessed using intelli- l Chromosomal aberrations
gence scores and they were treated inferiorly. Now a di- Clinical features:
agnosis of mental deficiency is made when there is inad- Head:
equate adaptive functioning and intellectual deficiency. l Microcephaly with prominent forehead
l A child with mental retardation is classified as: l Flattening of the occiput
i. Mild l Brachycephalic skull
ii. Moderate Face:
iii. Severe or profound l Flat nasal bridge
i. Mild retardation: l Epicanthal fold
l Children are categorized under this category l Upward slanting palpebral fissures
when they are able to speak well enough for Eyes:
most of their communication needs. l Hypoplasia of iris
l Their IQ score is usually in the range of 55–70. l Brushfield spots
l Most of the children in this category are edu- l Chronic infections of conjunctiva
cable and trainable and function eventually as Mouth:
acceptable adults. l Underdeveloped maxilla
Section | I  Topic-Wise Solved Questions of Previous Years 349

l Both maxilla and mandible positioned anteri- Q.2. What special attention is to be taken in the dental
orly under the cranial base treatment and management of handicapped children.
l Protruding tongue, hypertrophy of vallate Ans.
papillae
l Narrow and flat palate {SN Q.1}
l Delayed eruption of teeth

l Congenitally missing, abnormal shaped small


‘WHO’ defines a handicapped individual as ‘one who,
and hypoplastic teeth over an appreciable time, is prevented by physical or
Limbs: mental condition from full participation in the normal
l Broad hands, feet and digits
activities of his/her age group including those of a so-
l Wide space between the first and second toes
cial, recreational, educational and vocational nature’.
CNS: Classification:
l Mental retardation is another characteristic
A. Nowak (1976) has classified handicapping condi-
finding. Level of intelligentsia may range from tion into nine categories as follows:
mild to severe retardation.
l IQ often severely retarded with on IQ of 25–50. i. Physical handicapped, e.g. poliomyelitis, scoliosis
l They are very docile. ii. Mentally handicapped, e.g. mental retardation
l Generally movements are slow, clumpsy and iii. Congenital defects, e.g. cleft palate congenital
poorly coordinated. heart disease
Cardiac problems: iv. Convulsive disorder, e.g. epilepsy
l Septal defects are common ASD, VSD, etc. v. Communication disorder, e.g. deafness and blindness
Leukaemia: vi. Systemic disorder, e.g. hypothyroidism and hae-
l Children with Down syndrome have a 10–20 mophilia
fold greater incidence of leukaemia compared vii. Metabolic disorders, e.g. juvenile diabetes
to general population. viii. Osseous disorders, e.g. rickets and osteoporosis
l Acute lymphoblastic leukaemia – 20 times ix. Malignant disorders, e.g. leukaemia
more common in these children. B. Agerholm (1975) classified handicapping conditions
Dental treatment: into:
l The greatest problem in management is due to i. Intrinsic – ‘One from which the person cannot be
the presence of mental retardation (10%), separated’
otherwise they are very friendly and willing For example, all the medical and physical disabilities
to cooperate. ii. Extrinsic – ‘One from which the person can be
l Children are generally affectionate and coop- removed’
erative and present no special problems during For example, social deprivation
management. C. Frank and winter (1974) have classified handicapping
l Dentist should introduce treatment in a non- as:
threatening and friendly manner. i. Blind or partially sighted
l h incidence of leukaemia and acute and chronic ii. Deaf or partially deaf
infections of URT (upper respiratory tract) can iii. Educationally subnormal
also effect treatment. iv. Epileptic
l Incidence of cardiac disease in Down syn- v. Maladjusted
drome is 40% and will require adequate pro- vi. Physically handicapped
phylaxis. vii. Defective of speech
l Preventive procedures along with chlorhex- viii. Senile
ieline mouth wash may be beneficial. D. Considering the variations in the types of treatment mo-
l N2O analgesia or TSD in mildly apprehensive dalities for handicapped children, for the convenience of
patients can be used and GA in those patients management, they can be categorized into two:
who are severely resistant to dental treatment. i. Developmentally disabled child
l Pulp treatment of deciduous teeth is contrain- ii. Medically compromised patients
dicated in cardiac patients; therefore risk of Management of handicapped child:
bacteraemia in permanent teeth can be consid- l Systematic treatment planning is required in provid-

ered if adequate apical seal can be replaced.} ing appropriate care to children with special needs.
350 Quick Review Series for BDS 4th Year, Vol 1

l One of the biggest challenges is that the patients with patients with mental retardation undergoing den-
special needs may not be able to participate in the tal care.
discussions on a particular treatment plan and are l Providing dental treatment for a person with men-
unable to make their opinions known. tal retardation requires adjusting to social, intel-
The accompanying relatives and caretakers have a lectual and emotional delays.
crucial role to play in deciding the eventual care re- The following procedures have proved beneficial in
ceived by a mentally challenged patient. establishing dentist patient rapport and reducing the
l Most of the traditional behaviour management meth- patient’s anxiety about dental care:
ods may not work and the paediatric dentist may be l Give the family a brief tour of the office before
left with pharmacological behaviour management attempting treatment and introduce the office staff
methods to provide high quality dental care. to patient and their family in order to reduce the
Protective stabilization: patient’s fear of the unknown.
l Effective physical restraints or immobilization l Allow the patient to bring a favourite item, e.g.
may be needed in infants and patients with neuro- stuffed animal or toy to hold for the visit.
muscular disorders, to diagnose and provide den- l Keep the parents inside the operatory.
tal care. l Be repetitive, speak slowly and in simple terms.
l The main purpose of restraints is to limit or stop l Give only one instruction at a time.
the movements of the patient’s head, extremities l Actively and carefully listen to the patient.
and torso. l Reward the patient with compliments after the
l Immobilization is also useful in controlling resis- successful completion of each procedure.
tant patients. l Invite the parent into the operatory for assistance
l Immobilization can be provided by extra assis- and to aid in communication with the patient.
tants or any device made for that purpose and in- l Ask the parents not to communicate when dentist
formed consent to be taken before use of any type is communicating.
of physical restraints. l Keep appointment short.
Immobilization is indicated in the following situations: l Gradually progress to more difficult and lengthy
l Patient who cannot cooperate because of lack of procedures.
emotional maturity or physically or mentally l Schedule the patient’s visit early in the day.
challenging conditions. l Generally, patients with mild retardation can be
l When no other behaviour management techniques treated as regular patients. If the extent of work is
work. more, N2O sedation or general anaesthesia may be
l When there is a risk for the patient or the practi- needed.
tioner, if physical restraints are not used. l However, sedation or general anaesthesia may be
Contraindications for protective immobilization are invariably needed to provide dental treatment for
l A cooperative patient. children with moderate and severe retardation.}
l Patients with underlying medical conditions

which contraindicate the use of physical restraints. Q.3. Define and classify handicapped child. Explain how
l Treatment immobilization should never be used you will manage mentally handicapped children in your
as a threat or punishment for children. dental clinic.
Various physical restraints used on different parts of
Ans.
body are as follows:
l Entire body: triangular sheet, papoose board,

Pedi-wrap, safety belt and an extra-assistant [Same as LE Q.2]


l Extremities: Posey and Velcro straps, towel and

tape or an extra-assistant SHORT ESSAYS:


l Head: forearm body support, head positioner and

an extra-assistant Q.1. Cerebral palsy.


l Intraoral: mouthprops, McKesson bite blocks and
Ans.
wrapped tongue blades
Cerebral palsy is one of the most severely handicapping
[SE Q.5]
conditions affecting childhood.
{Dental treatment of a person with mental retardation: ‘Nelson’ described cerebral palsy as a group of nonpro-
A short attention span, restlessness, hyperactivity
l gressive disorders resulting from malfunction of the motor
and erratic emotional behaviour may characterize centres and pathways of the brain.
Section | I  Topic-Wise Solved Questions of Previous Years 351

It is characterized by paralysis, weakness, incoordination x. Local anaesthesia can be used with care and stabi-
or other aberrations of motor function and has its origin lization against any sudden movement by the child.
either prenatally or before the CNS has reached maturity. xi. Rubber dam can be used to protect working
Incidence: 0.6–5.9 per 1000 births. area from hyperactive tongue movement.
Classification: xii. Guaze shields used during extraction to avoid
tooth aspiration.
There are five types of cerebral palsy:
xiii. Premedication can be used to reduce hyperto-
Occurs in Lesion in nicity, involuntary movements and anxiety.
(a) Spasticity (.40% cases) Cerebral cortex xiv. General anaesthesia should be used as last re-
sort if the case is not manageable.
(b) Athetosis (25%) Basal ganglion
xv. Do permanent restorations.
(c) Ataxia (10%) Cerebellum
Q.2. Trisomy 21.
(d) Rigidity (5%) Basal ganglion

(e) Tremors (5%) Basal ganglion


Ans.
[Ref LE Q.1]
l Characteristic of cerebral palsy is the persistent neona-
tal reflexes, i.e. asymmetric tonic neck reflex, tonic Q.3. Learning disorders.
labyrinthine reflux and startle reflex.
Ans.
l Along with these, mental retardation, seizure disor-

ders, sensory deficits like visual defects and deafness l The children affected with minimal brain dysfunction
and speech disorders like dysarthria are seen. were known to be effected by ‘learning disability’.
Dental problems encountered are as follows:
l Dental caries because of poor oral hygiene {SN Q.2}
l Periodontal disease because of poor oral hygiene
l The signs of learning disability are as follows:
and phenytoin treatment i. Attention for short spans
l Malocclusions seen in spastics Class II division 2
ii. Distractibility
and in athetoids Class II division I iii. Hyperactivity
l Bruxism seen especially in athetoid cerebral palsy
iv. Awkwardness
l Decreased Vertical dimension (VD) and TMJ dis-
v. Mild speech impairment
orders nTrauma especially in maxillary anteriors
Treatment: l The term learning disability is applied to children who
i. Communication forms an important aspect of exhibit a disorder in one or more of the basic physio-
management of patients with cerebral palsy. logic processes involved in understanding or using
ii. Routine procedures can be accomplished, gen- spoken or written language.
erally they will understand. Cooperate when l It may be manifested in disorders of listening, thinking,
the dentist explains before starting a proce- reading, talking, writing or spelling.
dure, unless severely mentally retarded. l Learning disability includes dyslexia, developmental
iii. Maintain a calm, friendly and professional at- aphasia, brain injury and minimal brain dysfunction.
mosphere, be empathetic about the child’s l Boys are more commonly affected than girls.
problems. l One form of learning disability which causes manage-
iv. Thorough medical and dental history should ment problems is hyperactivity.
be taken along with consultation with the l Their nervousness makes cooperation through long pro-
child’s physician. cedures difficult.
v. Many patients can and prefer to be treated in Paediatric significance:
wheel chairs, which may be tipped back into l Most of the children cooperate for the dental pro-
the dentists lap. cedures.
vi. Patients head should be stabilized throughout l If the child resists dental treatment occasionally,
the procedure and back should be elevated to one has to use sedation or general anaesthesia.
reduce swallowing problems.
vii. Use physical restraints judiciously for control Q.4. Autism.
of flailing extremities. Ans.
viii. The variety of mouth props and finger splints
can be used for control of involuntary jaw {SN Q.4}
movements.
ix. Avoid abrupt movements, lights and noises to l Childhood autism is also known as Kanner syn-
minimize startle reflex reactions and introduce drome, early infantile autism and infantile psychosis
intra oral stimuli slowly to avoid gag reflex. or childhood schizophrenia.
352 Quick Review Series for BDS 4th Year, Vol 1

Q.6. Down syndrome.


l Nowadays the term autism is used to describe an in-
capacitating disturbance of mental and emotional Ans.
development that causes problem in learning, com-
[Same as SE Q.2]
municating and relating with others.
l The term ‘autism’ is derived from the Greek word

‘autos’ meaning ‘self’, which appropriately describes SHORT NOTES:


the characteristic feature of this disorder namely a Q.1. Definition of handicapped child.
profound withdrawal from people and from social
Ans.
reactions with people, even parents.
Aetiology: [Ref LE Q.2]
l Personalities attitudes and behaviour of par- Q.2. Name the signs of learning disorders.
ents contributes to psychodynamics of autism.
Ans.

[Ref SE Q.3]
Clinical features:
Q.3. Munchausen syndrome by proxy.
l It usually manifests in the first 3 years of life.
l Males more commonly affected than females. Ans.
l Kopel in 1977 has described 12 behavioural charac-
l Munchausen syndrome is defined as significantly sub-
teristics of this disorder they are as follows: average general intellectual functioning existing con-
i. Extreme aloneness currently with deficits in adaptive behaviour and mani-
ii. Language disturbances fested during the developmental period.
iii. Mutism
iv. Parrot-like repetitious speech Q.4. Autism.
v. Difficulty with the concept of ‘yes’ Ans.
vi. Confusion in the use of personal pronouns
[Ref SE Q.4]
vii. Obsessive desire for the maintenance of
sameness Q.5. Down syndrome.
viii. Eating disturbances such as holding food in the Ans.
mouth and preference for a soft diet
ix. Intrigue with spinning objects l Down syndrome is also called as trisomy 21 syndrome
x. Self-stimulatory behaviour or mongolism.
l Cardiovascular defects include ventricular septal defect,
xi. Hyperactivity, nystagmus and mental retardation
xii. Seizure disorder ALV communication, patent ductus arteriosus and mi-
Treatment and paediatric significance: tral valve prolapse.
l Haematological: Impaired immunodeficiency, risk of
l Dentist should have lot of patience and use a gentle

and slow approach to the oral cavity. neutropenia, eosinophilia and leukaemia.
l Musculoskeletal: Atlantoaxial instability, midface is
l Maintain consistency in the environment.

l Behavioural management techniques like tell-show-


underdeveloped with relative prognathism and open
do (TSD), positive reinforcement and rewards may bite.
l Nervous: Delayed motor function and dementia.
be helpful in some children.
l Oral: V-shaped, high vault palate, soft palate insuffi-
l Immobilization devices like Papoose board and Pedi-

wrap may produce calming effect in some children. ciency, open mouth and macroglossia.
l In case of very uncooperative children, when treat- Q.6. Handicapped children.
ment is not possible at the chair side, use of sedation Ans.
and general anaesthesia is necessary.
l The use of positive reinforcement to promote desir-
[Same as SN Q.1]
able behaviour is the key to all behaviour modifica- Q.7. Trisomy.
tion programs. Ans.
Q.5. Discuss various measures in the treatment and [Same as SN Q.5]
management of the mentally retarded children. Q.8. Down syndrome.
Ans. Ans.
[Ref LE Q.2] [Same as SN Q.5]
Section | I  Topic-Wise Solved Questions of Previous Years 353

Topic 8
Management of Children with Systemic
Diseases and HIV Infection
COMMONLY ASKED QUESTIONS
LONG ESSAYS:
1. Classify handicapping conditions and discuss the antibiotic prophylaxis in management of patient with con-
genital cardiac disease.
2. Define handicapped children. What precautions you take while carrying out dental treatment for a patient suf-
fering from haemophilia?
3. Give oral manifestations of systemic disease in children. What are the AHA guidelines for the prevention of
bacterial endocarditis?
4. Give oral manifestations of leukaemia. What precaution would you take to treat such a child?
5. Define handicapped child and discuss the management of a haemophilic child in the dental clinic. [Same as LE Q.2]
6. Define handicapped child in paedodontic patients and discuss the management of haemophilic child for an
extraction of teeth? [Same as LE Q.2]

SHORT ESSAYS:
1 . Describe briefly the management of a child suffering with mumps.
2. Oral manifestations of AIDS.
3. Dental management of von Willebrand disease.
4. AIDS in children.
5. Management of haemophilic child in dental office. [Ref LE Q.2]
6. Describe the dental management of epileptic patient.
7. Recent prophylactic regime against bacterial endocarditis.

SHORT NOTES:
1 . Erythroblastosis fetalis.
2. Enumerate congenital anomalies.
3. Give oral manifestations of leukaemia. [Ref LE Q.4]
4. Management of a purpuric patient in dental clinic.
5. Describe the dental management of epileptic patient. [Ref SE Q.6]
6. Clinical importance of platelet count in a leukaemic patient.
7. Clinical importance of WBC counts in a leukaemic patient.

SOLVED ANSWERS
LONG ESSAYS:
Q.1. Classify handicapping conditions and discuss the their age groups including those of a social, recreational,
antibiotic prophylaxis in management of patient with educational and vocational nature’.
congenital cardiac disease. Classification of handicapping conditions:
The following are the various classifications of handi-
Ans.
capping conditions:
WHO has defined a handicapped person as ‘one who over I. Nowak (1976) has classified handicapping condition
an appreciable period is prevented by physical or mental into nine categories as follows:
conditions from full participation in the normal activities of i. Physically handicapped, e.g. poliomyelitis and scoliosis
354 Quick Review Series for BDS 4th Year, Vol 1

ii. Mentally handicapped, e.g. mental retardation l This group mainly includes ventricular and
iii. Congenital defects, e.g. cleft palate and congeni- atrial septal defects and defects that cause
tal heart disease obstruction.
iv. Convulsive disorders, e.g. epilepsy l Clinical manifestations include congestive

v. Communication disorders, e.g. deafness and heart failure, pulmonary congestion, heart
blindness murmur, laboured breathing and so on.
vi. Systemic disorders, e.g. hypothyroidism and hae- Acquired heart disease:
mophilia Types of acquired heart disease are as follows:
vii. Metabolic disorders, e.g. juvenile diabetes (I) Rheumatic heart fever:
viii. Osseous disorders, e.g. rickets and osteoporosis l It is a very serious inflammatory disease that

ix. Malignant disorders, e.g. leukaemia occurs as a delayed sequela to pharyngeal


II. Agerholm (1975) classified handicapping conditions: infections with group A beta haemolytic
i. Intrinsic – ‘One from which the person cannot be streptococci.
separated’ l The infection can involve the heart, joints,

For example, all the medical and physical disabili- skin and central nervous system.
ties l It occurs most commonly under 40 years of

ii. Extrinsic – ‘One from which the person can be age specially between 6 and 15 years of age but
removed’ it can occur at any age.
For example, social deprivation l It is more common in poor children living

III. Frank and Winter (1974) have classified handicap as: in temperate climate at high altitude.
i. Blind or partially sighted l Cardiac involvement is the most significant

ii. Deaf or partially deaf pathologic sequela of rheumatic fever which


iii. Educationally subnormal can be fatal during the acute phase or can
iv. Epileptic lead to chronic rheumatic heart disease as a
v. Maladjusted result of scarring and deformity of heart
vi. Physically handicapped valves.
vii. Defective of speech (II) Infective bacterial endocarditis:
viii. Senile l It is characterized by microbial infection of

IV. Considering the variations in the types of treatment the heart valves or endocardium in proxim-
modalities for handicapped children, for the conve- ity to congenital or acquired cardiac defects.
nience of management, they can be categorized into: l It has been divided into acute and subacute

A. Developmentally disabled child forms.


B. Medically compromised patients l The acute form is a fulminating disease that
Management of patient with cardiac disease: usually occurs as a result of microorganisms
Cardiac disease can be divided into two general of high pathogenicity attacking a normal
types: heart.
(a)   Congenital l In this, erosive destruction of the valves
(b)   Acquired takes place. Subacute bacterial endocardi-
a. Congenital heart diseases (CHD): tis (SABE) usually develops in persons
The CHD can be classified into the following with pre-existing congenital cardiac dis-
types: ease or rheumatic valvular lesions. Embo-
l Cyanotic lization is usually the characteristic fea-
l Acyanotic ture of infective endocarditis. Vegetation
Cyanotic heart disease: composed of microorganisms and fibrous
l It is characterized by right to left shunt- exudate develops on damaged valves or
ing of blood within heart. endocardium.
l Cyanosis is observed even on minor l They might separate and pass into systemic
exertion. or pulmonary circulation.
For example, Fallot tetralogy’s clinical l Clinical symptoms include low, irregular fe-
manifestations include cyanosis, hy- ver, more in afternoon and evenings with
poxic spells, poor physical development sweating, malaise, anorexia, weight loss and
and clubbing. arthralgia. Inflammation of endocardium in-
Acyanotic heart disease: creases cardiac destruction and murmurs de-
l Here, due to a cardiac defect blood is velop. Painful fingers and toes and skin le-
shunted from left to right. sions develop.
Section | I  Topic-Wise Solved Questions of Previous Years 355

l Laboratory findings can include leucocytosis, Use of antifibrinolytics:


neutrophils and anaemia with rapid ESR. l Antifibrinolytic agents are an adjunctive ther-

Management: apy for dental management of patients with


l Careful consultation with cardiologist is es- bleeding disorders and are important for pre-
sential before any major dental procedure. vention or treatment of oral bleeding.
l Information concerning dental needs, anaes- l These agents include: Epsilon-aminocaproic

thesia, sedation, drug therapy should be dis- acid (Amicar, Xanodyne Pharmaceuticals, Flor-
cussed beforehand. ence, KY) and tranexamic acid (Cyklokapron,
Antibiotic prophylaxis: Pfizer, New York).
l Bacterial endocarditis could occur following l Haemophilic patients form loose, friable clots

any dental procedure capable of producing that may be readily dislodged or quickly dis-
transient bacteraemia. solved, especially in the oral cavity where local
l Therefore all patients with cerebrovascular fibrinolysis is increased.
system (CVS) disorders should be adminis- l Antifibrinolytics prevent clot lysis within the

tered antibiotic prophylaxis. oral cavity. They are often used as an adjunct
Antibiotic prophylaxis is recommended in following to factor concentrate replacement. For some
dental procedures: dental procedures in which minimal bleeding
l Dental procedures likely to induce any is anticipated, they may be used alone.
bleeding Dosages:
l Surgical operations involving respiratory mu- l In children, Epsilon-aminocaproic acid is

cosa of maxillary sinus given immediately before dental treatment in


l Incision and drainage of infected tissue an initial loading dose of 100–200 mg/kg by
l Intraligamentary operations mouth up to a maximum total dose of 10 g.
Dental procedures not requiring antibiotic coverage Subsequently, 50–100 mg/kg per dose up to a
are as follows: total maximum dose of 5 g is administered
l Simple adjustments of orthodontic appliances orally every 6 h for 5–7 days.
l Fillings above gingiva l Alternatively, for patients of approximately

l Intra oral Injection of local anaesthetics (ex- adult size or heavier than 30 kg, a regimen of
cept intraligamentary) 3 g by mouth four times daily without a load-
l Exfoliation of deciduous teeth ing dose may be used. The advantage of Epsi-
l New denture or orthodontic appliance lon-aminocaproic acid for children is that it is
insertion available in both tablet and liquid form.
l The adult and paediatric dosage of tranexamic
Q.2. Define handicapped children. What precautions acid is 25 mg/kg given immediately before
you take while carrying out dental treatment for a dental treatment. The same dose is continued
patient suffering from haemophilia? every 8 h for 5–7 days.
l The oral preparation of tranexamic acid is not
Ans.
available in the United States but the intravenous
WHO has defined a handicapped person as ‘one who over formulation is available. The intravenous formu-
an appreciable period is prevented by physical or mental lation may be administered orally if required.
conditions from full participation in the normal activities of Side effects:
their age groups including those of a social, recreational, l The common side effects associated with
educational and vocational nature’. the use of antifibrinolytics include head-
The management of haemophilic child for an extraction ache, nausea and dry mouth.
of teeth: l These side effects are usually tolerable and,
l The dentist should confer with the patient’s physician unless severe, do not require discontinua-
and haematologist to formulate an appropriate treat- tion of the medication. Other less common
ment plan. side effects have also been reported.
l The following factors help in establishing the appropri- l To avoid thrombosis, antifibrinolytics
ate treatment plan: should not be used when renal or urinary
i. The invasiveness of the dental procedure tract bleeding is present or when there is
ii. The amount of bleeding anticipated any evidence of disseminated intravascular
iii. The time involved in oral wound healing coagulation.
356 Quick Review Series for BDS 4th Year, Vol 1

Pain control in haemophilic patients: highly vascular and accidental lacerations may
l Intramuscular injections of hypnotic, tran- present a difficult management problem.
quilizing or analgesic agents are contraindi- l Thin rubber dam is used to decrease the torque

cated due to the risk of hematoma forma- and retainer should be placed carefully.
tion. Analgesics containing aspirin or l High-speed vacuum ejectors must be used with

anti-inflammatory agents (e.g. ibuprofen) caution so that sublingual haematomas do not


may affect platelet function and should be occur.
avoided. l After tooth preparation periphery wax is used

l Acute pain of moderate intensity can fre- on the impression tray to prevent possible in-
quently be managed using acetaminophen traoral laceration during tray placement.
Propoxyphene hydrochloride (Darvon). l Undue trauma is avoided in cementing or fin-

l For severe pain, narcotic analgesics may be ishing a crown.


required and are not contraindicated in the Pulpal therapy:
haemophilic patient. l A pulpotomy or a pulpectomy is preferable to

Local anaesthesia: extraction


l A minimum of a 40% factor correction is man- l If the pulp of the vital tooth is exposed an intra

datory with block anaesthesia. pulpal injection may be used safely to control
l All patients should be observed for develop- the pain.
ment of a haematoma and immediately re-
ferred for treatment in case haematoma [SE Q.5]
forms after the administration of local an- {Oral surgery:
aesthesia. l For simple extractions of erupted permanent
l In the absence of factor replacement, periodon-
teeth and multirooted primary teeth, a 30%–
tal ligament (PDL) injections may be used. 40% factor correction is administered within 1 h
l The aesthetic is administered along the four
before dental treatment.
axial surfaces of the tooth by placement of the l Antifibrinolytic therapy should be started im-
needle into the gingival sulcus and the peri- mediately before or after the procedure and
odontal ligament space. should be continued for 5–10 days.
l The patient should be placed on a clear liquid
[SE Q.5]
diet for the first 72 h.
{Dental management: For the next week, a soft cold diet is recom-
Appointments should be arranged so that max-
l mended. During this time, the patient should not
imum treatment is accomplished per visit to use straws, metal utensils, pacifiers or bottles.
minimize the need for unscheduled factor infu- After 10 days, the patient may begin to con-
sions and hence cost. sume a more normal diet.
l Rubber cup prophylaxis and supragingival l Specific postoperative instructions should be
scaling may be safely performed without prior provided to the patient and parent.
factor replacement therapy. l All extractions should be completed in one ap-
l Minor bleeding can be readily controlled with pointment if possible.
local measures, such as direct pressure with a l After extractions are completed, the direct topi-
moistened gauze square. cal application of haemostatic agents, such as
l If bleeding persists for several minutes, the thrombin or microfibrillar collagen haemostat
topical application of bovine thrombin, micro- (Avitene), may assist with local haemostasis.
fibrillar collagen and local fibrin glue may be l The socket should be packed with an absorbable
of value. gelatine sponge (e.g. Gelfoam). Microfibrillar
Periodontal therapy: collagen or topical thrombin or fibrin glue may
l Replacement therapy is required for proce- then be placed in the wound. Direct pressure
dures like subgingival scaling, frenotomy and with gauze should then be applied to the area for
periodontal surgeries.} additional protection from the oral environment.
Restorative procedures: l In general, the use of sutures should be avoided
l A rubber dam should be used to isolate the unless suturing is expected to markedly en-
operating field and to retract and protect the hance healing, in which case resorbable su-
cheeks, lips and tongue. These soft tissues are tures are recommended.
Section | I  Topic-Wise Solved Questions of Previous Years 357

l For surgical extractions of impacted, partially l Deposition of blood pigments in developing teeth
erupted or unerupted teeth, a higher factor ac- results in staining
tivity level may be targeted before surgery. l Hypoplasia with a classic appearance of regular

Surgical complications: incremental defects


l Despite all precautions, bleeding may occur ii. Cyanotic congenital heart disease:
3–4 days postoperatively when the clot begins l Cyanotic gingivitis and stomatitis

to break down. l Glossitis

l Sufficient replacement factor should be admin- l Delayed eruption of teeth

istered to control recurrent bleeding. l Increased caries activity

l The typical clot in this situation is character- l Intrinsic dyschromia of dentition resulting from

ized as a ‘liver clot’ and is dark red, usually medications or blood by-product deposition
protruding from the surgical site and often cov- iii. Diabetes mellitus:
ers the surfaces of several teeth. l Xerostomia

l Following adequate replacement with factor l Increased caries rate

concentrate, usually to a 30%–40% activity l Oral candidiasis

level, the abnormal clot should be removed and l Oral ulcerations and increased severity of peri-

the area cleansed to help isolate the source of odontitis and bone loss
bleeding. iv. Hypo pituitarism:
l The socket should then be repacked and use of l Tooth eruption is incomplete and delayed.

antifibrinolytic agents considered.} l The formation of the root and closure of the apical

Antibiotic prophylaxis: foramen are also delayed and incomplete.


l The antibiotic prophylaxis is followed for hae- l Vertical height of the mandible is reduced resulting

mophilic patients with prior joint replacement in open bite, immature facial patterns, reduced in-
surgeries. termaxillary space and crowding of the teeth.
l If the patient is immunocompromised because v. Hypo thyroidism:
of HIV infection, intravenous antibiotic pro- l Delayed eruption

phylaxis may be considered. l Malocclusion

l Increased susceptibility to periodontal diseases,


Q.3. Give oral manifestations of systemic disease in chil-
caries and oral ulcerations
dren. What are the AHA guidelines for the prevention of
l Developmental retardation and formation of teeth
bacterial endocarditis ?
vi. Hyper thyroidism:
Ans. l Susceptibility to periodontal disease and caries

Oral manifestations of systemic diseases in children are as l Periodontal/periapical destruction

follows: l Premature loss of deciduous teeth and early erup-

i. Chronic renal failure: tion of permanent teeth


Oral manifestations of CRF depend upon the time of vii. Hypo parathyroidism:
onset, duration, severity and nature of the underlying l Oral candidiasis

disease. l Hypoplasia of enamel

Manifestations in soft tissues: l Hypodontia

l Generalized pallor of the oral mucosa l Root dysmorphogenesis

l Intraoral haematoma (a tendency to bruise) l Delayed tooth eruption

l Uremic gingivostomatitis l Thickened lamina dura

l Bad mouth odour l Chvostek sign positive:

l Metastatic calcifications in maxillary sinus and That is, a sharp tap in front of the ear over the facial
sometimes in other areas of oral cavity. nerve causes twitching of facial muscles around the
Manifestations in hard tissues: mouth.
l Malocclusion due to growth retardation viii. Hyper parathyroidism:
l Loss of laminadura l Tooth drifting, mobility of tooth

l Loss of trabeculation l Disappearance of lamina dura

l Ground glass appearance of the jaws l Radiographically ‘ground glass’ appearance of the jaws

l Large bony lesions like giant cell tumours of ix. Iron deficiency anaemia:
hyperpara thyroidism l Cracking or splitting of nails
358 Quick Review Series for BDS 4th Year, Vol 1

l Painful tongue Based on situation Drugs Adults Children


l Decreased healing capacity to oral and periodon-
Allergic to penicillin Cefazolin or 1 g i.m. 50 mg/kg
tal surgery
or ampicillin and Ceftriaxone or i.v.; i.m. or i.v.
l Mucosal pallor
unable to take oral 600 mg 20 mg/kg
x. Pernicious anaemia: medication or i.m. or i.m. or i.v.
l Glossitis – Painful and burning sensations. Clindamycin i.v.
Tongue is generally inflamed and beefy red in
colour over the dorsum and lateral borders. Q.4. Give oral manifestations of leukaemia. What pre-
l Hunter glossitis – Atrophy of the papillae of the caution would you take to treat such a child?
tongue resulting in a smooth or bald tongue.
xi. Aplastic anaemia: Ans.
l Pallor l Leukaemias are haematopoietic malignancies where
l Purpura there is uncontrolled neoplastic proliferation of abnor-
l Spontaneous bleeding mal leukocytes in the bone marrow and dissemination
xii. Polycythemia vera: of these cells into blood.
l The gingiva and tongue appear deep purplish red. l ALL (acute lymphoid leukaemia) accounting for 75%
l Cyanosis due to reduced haemoglobin. of all child hood leukaemias is the most common malig-
l The gingiva bleeds on slightest provocation. nancy of all childhood malignancies.
l Pale mucosal petechiae, ecchymoses and haema-

tomas.
{SN Q.3}
xiii. Leukaemia:
l Gingival hyperplasia The following are the oral manifestations of leukaemia:
l Ulceration of mucosa l Commonly observed oral manifestations are pete-
l Petechiae chiae, ecchymoses, gingival bleeding, pallor and
xiv. Leukopenia: nonspecific ulcerations.
l Inability of the tissue to react to infection or l Gangrenous stomatitis.
trauma in usual manner. l Direct invasion of tissue by an infiltrate of leukaemic
xvi. Cyclic neutropenia: cells can produce gingival hypertrophy.
l Gingivitis l Infiltration of leukaemic cells along vascular chan-
l Stomatitis, sometimes with ulceration nels can result in strangulation of pulpal tissue and
xvii. Haemophilia: spontaneous abscess formation as a result of infec-
l Gingival bleeding tion or focal areas of liquefaction necrosis in the
xviii. Thrombocytopenia: dental pulp of sound teeth.
l Severe and profuse gingival bleeding l Skeletal lesions caused by leukaemic infiltration of
l Petechiae on oral mucosa bone are common in childhood leukaemia.
l Regimen for dental procedure as given by the l The most common finding is a generalized osteopo-
American Heart Association in 2007 is as follows: rosis caused by enlargement of the Haversian canals
and Volkmann’s canals.

Based on situation Drugs Adults Children l Manifestations in the jaws include generalized loss of
Standard general Amoxicillin 2g 50 mg/kg trabeculation, destruction of the crypts of developing
oral prophylaxis teeth, loss of lamina dura, widening of the periodontal
ligament space and displacement of teeth and tooth buds.
Management of a leukaemic patient in dental clinic:
Unable to take oral Ampicillin 2 g IM 50 mg/kg
l Before any dental treatment is administered to a
medication or or IV i.m. or i.v.
Cefazolin 2 g IM 50 mg/kg child with leukaemia, the child’s haematologist or
or or IV i.m. or i.v. oncologist or primary care physician should be
Cefitriaxone consulted.
Allergic to penicillin Cephalexin 2g 50 mg/kg l The following information is ascertained:

or ampicillin – oral or i. Primary medical diagnosis


Clindamycin 600 mg 20 mg/kg ii. Anticipated clinical course
or iii. Present and future therapeutic modalities
Azithromycin 500 mg 15 mg/kg
or
iv. Present general state of health
Clarithromycin v. Present haematological status
Section | I  Topic-Wise Solved Questions of Previous Years 359

l Pulp therapy on primary teeth is contraindicated l It is typically benign and resolves within a
in any patient with a history of leukaemia. week.
l Routine preventive and restorative treatment, in- l The two complications are of significance:
cluding injections, may be considered when there i. Orchitis with painful swelling of the testi-
are at least 50,000 platelets/mm3. cles in postpubertal males, which can result
3
l If there are less than 20,000 platelets/mm , no in sterility and
dental treatment should be performed at such a ii. Deafness in children.
time without a preceding prophylactic platelet Management:
transfusion. l Oral hygiene is important when the mouth is
l Prophylactic platelet transfusions are given for dry from lack of saliva.
platelet levels below 10,000 cells/mm3. l Orchitis can be relieved by prednisolone 40 mg
l The use of a soft nylon tooth brush for the re- orally daily for 4 days.
moval of plaque is recommended. Prevention:
l For patients who are thrombocytic or at risk for l It is prevented by immunization with live attenu-
intermittent episodes of thrombocytopenia be- ated virus as part of measles–mumps–rubella
cause of chemotherapy or active disease, the den- (MMM) vaccine.
tist should avoid prescribing drugs that may alter
platelet function, such as salicylates like aspirin Q.2. Oral manifestations of AIDS.
and other nonsteroidal anti-inflammatory drugs.
Ans.
Q.5. Define handicapped child and discuss the manage-
Oral manifestation of AIDS patients are as follows:
ment of a haemophilic child in the dental clinic.
l Fungal infection like candidiasis
Ans. l Bacterial infections either generalized, localized or

pyogenic
[Same as LE Q.2]
l Viral infections like herpes zoster, herpes simplex and
Q.6. Define handicapped child in paedodontic patients hairy leukoplakia
and discuss the management of haemophilic child for an l Linear gingival erythema
extraction of teeth. l Gingival and periodontal lesions like ANUG and necro-

tizing ulcerative periodontitis


Ans.
l Pulmonary lymphoid hyperplasia
[Same as LE Q.2] l Pyogenic bacterial infections like otitis media

l Salivary gland enlargement

l Developmental craniofacial features


SHORT ESSAYS:
l Progressive encephalopathy

Q.1. Describe briefly the management of a child suffer- Q.3. Dental management of von Willebrand disease.
ing with mumps.
Ans.
Ans.
l von Willebrand disease is a hereditary bleeding disorder
l Mumps is caused by paramyxo virus which is transmit- resulting from impairment of von Willebrand factor
ted via respiratory droplets. (vWF).
l It occurs worldwide with peak incidence in the winter. l The main function of vWF is primary platelet plug for-
l Paramixo virus causes an acute contagious nonsuppura- mation.
tive parotitis. l As a general rule treatment planning should be modified
l Incubation period: whenever possible to provide nonsurgical treatment.
It is about 18 days. l Patients with von Willebrand disease should undergo
Clinical features: subtyping to determine optimal therapy.
l The most noticeable symptom of mumps is the l DDAVP (desmopressin acetate) may be used to
painful swelling of the parotid glands, either achieve haemostasis in most patients with type I von
unilateral or bilateral. Willebrand disease, where type I represents a quanti-
l Malaise, fever, trismus and pain near the angle tative vWF deficiency with intact multimers.
of the jaw are soon followed by tender swell- l When DDAVP is used, a test dose should be ad-
ing of one or both parotid glands. ministered to document an adequate haemostatic
l Less frequent clinical sequelae include orchitis response.
occurring in 20%–35% of postpubertal males l DDAVP is not used in the patients:
and aseptic meningitis. With less common subtypes of VWD
360 Quick Review Series for BDS 4th Year, Vol 1

Who do not respond to DDAVP


{SN Q.5}
With history of bleeding events and
For whom replacement with exogenous vWF through Dental management of epileptic patients should be
l

the use of a concentrate is recommended. aimed at both prevention and control of the epileptic
attacks.
Q.4. AIDS in children.
Prevention of seizures in dental office can be best
Ans. managed by:
i. Complete medical history, i.e. type and fre-
l Acquired immunodeficiency syndrome (AIDS) is a
quency of seizure episodes and time and situa-
clinically defined condition caused by infection with
tion of the last attack.
HIV type I or much less commonly type II.
ii. Reduce stress on the patients with behavioural
l The incubation period from the time of infection to
preparations, sedation, etc.
the appearance of symptoms of AIDS is approximately
iii. Avoid use of dental chair light.
11 years in adults.
iv. Avoid seizure promoting drugs, e.g. phenothi-
l Therefore HIV infected individuals can unknowingly
azines, local anaesthetics i.v.
spread the virus to the sexual or needle sharing partners,
v. Appropriate drug therapy for seizures – Dilantin
in case of infected mothers, to their children.
sodium and recent drugs like Vigabatrin, La-
l Infants and children with AIDS have clinical findings
motrigine, Gabapentine and Topiramate.
similar to those in adults.
vi. Typical fibrous gingival hyperplasia may occur
l Early manifestations of HIV infection include pneumo-
this require surgical removal.
cystis, pneumonia, interstitial pneumonitis, weight loss
and failure to thrive, hepatomegaly or splenomegaly,
generalized lymphadenopathy and chronic diarrhoea.
l Recurrent and severe bacterial infections are common in Dental treatment:
paediatric patients with HIV infection. l Appointments should be kept short.

l Oral manifestations of AIDS patients include fungal, l Importance of tooth brushing procedure and regular

bacterial and viral infections, linear gingival erythema, dental review must be stressed.
pyogenic bacterial infection such as otitis media, epato- l Boxed type of appliances are indicated for tooth

splenomegaly, chronic pneumonitis and progressive movement and tooth replacement.


encephalopathy. Office management of seizure attack:
Preventive measures to be followed are: The following procedures are done if the seizure
l Barrier techniques and proper sterilization. occurs in dental chair:
l HIV is sensitive to autoclaving at 121°C for 15 min l Lower the chair to supine position.

at 1 atmospheric pressure. l Prevent the child from injuring himself/herself

l Dry heat sterilization of instruments up to 170°C. For example, to prevent tongue biting – Mouth
l Virus can be inactivated by heating lyophilized factor prop either rubber/plastic is used.
at 68°C for 72 h. Patient shifted to place where they cannot
l Disinfectants for innate objects: harm themselves.
l Calcium hypochlorite 0.2%, sodium hypochlorite l Maintain patent airway: Suction is useful to

l 6% hydrogen peroxide for more than 30 min avoid aspiration of secretions, if it is not avail-
l 2% glutaraldehyde and 6% hydrogen peroxide able head should be turned to a side.
l Sodium dichloro-isocyanate treatment for 10 min at l If convulsions do not stop within few minutes:

room temperature with 10% household bleach, 50% give diazepam 1 mg/kg i.v. and slowly up to
ethanol and 3% hydrogen peroxide. 10 mg and O2.
l Gloves may be disinfected by immersing them in
Q.7. Recent prophylactic regime against bacterial endo-
boiling water for 20 min and alternatively overnight carditis.
soaking in 1% sodium hypochlorite.
Ans.
Q.5. Management of haemophilic child in dental office.
l Transient bacteraemia is an important initiating factor in
Ans. infective endocarditis.
[Ref LE Q.2] l Procedures known to precipitate transient bacteraemias

in dentistry are all those that involve manipulation of


Q.6. Describe the dental management of epileptic patient. gingival tissue or the periapical region of teeth or perfo-
Ans. ration of the vital mucosa.
Section | I  Topic-Wise Solved Questions of Previous Years 361

l Regimen for dental procedure as given by the American Q.3. Give oral manifestations of leukaemia.
Heart Association in 2007 is as follows:
Ans.
[Ref LE Q.4]
Based on situation Drugs Adults Children
Standard general Amoxicillin 2g 50 mg/kg Q.4. Management of a purpuric patient in dental clinic.
oral prophylaxis
Ans.
l The information regarding primary medical diagnosis,
Unable to take oral Ampicillin 2 g i.m. 50 mg/kg present general state of health and present haematologi-
medication or or i.v. i.m. or i.v.
cal status is ascertained.
Cefazolin 2 g i.m. 50 mg/kg 3
l When there are at least 50,000 platelets/mm , routine
or or i.v. i.m. or i.v.
Cefitriaxone preventive and restorative treatment, including injec-
tions, may be considered.
Allergic to penicil- Cephalexin 2g 50 mg/kg 3
l If there are less than 20,000 platelets/mm , no dental
lin or ampicillin – or
oral Clindamycin 600 mg 20 mg/kg treatment should be performed at such a time without a
or preceding prophylactic platelet transfusion.
Azithromycin 500 mg 15 mg/kg l Prophylactic platelet transfusions are given for platelet
or
levels below 10,000 cells/mm3.
Clarithromycin
l The use of a soft nylon toothbrush for the removal of
Allergic to penicil- Cefazolin or 1 g i.m. 50 mg/kg plaque is recommended.
lin or ampicillin Ceftriaxone or i.v.; i.m. or i.v.
l For patients who are thrombocytic or at risk for inter-
and unable to take or 600 mg 20 mg/kg
oral medication Clindamycin i.m. or i.m. or i.v. mittent episodes of thrombocytopenia because of che-
i.v. motherapy or active disease, the dentist should avoid
prescribing drugs that may affect platelet function, such
as salicylates (aspirin) and nonsteroidal anti-inflamma-
tory drugs.
SHORT NOTES: Q.5. Describe the dental management of epileptic
patient.
Q.1. Erythroblastosis fetalis.
Ans.
Ans.
[Ref SE Q.6]
Erythroblastosis fetalis:
l Anaemia due to Rh-positive red blood cells in the Q.6. Clinical importance of platelet count in a leukae-
fetus being attacked by antibodies from Rh-negative mic patient.
mother.
Ans.
Oral manifestations:
l Deposition of the blood pigment in the enamel and l A platelet level of 100,000/mm3 is adequate for most
dentin of the developing teeth. dental procedures
l The pigment colour ranges from green, brown to l Clinical importance of platelet count is as follows:

blue hue.
Platelet count (cells/mm3) Significance

Q.2. Enumerate congenital anomalies. 150,000–4,00,000 Normal


50,000–150,000 Bleeding time prolonged, but
Ans. patient would tolerate most rou-
The various congenital anomalies are as follows: tine procedures
l Cleft lip and palate 20,000–50,000 At moderate risk for bleeding
l Down syndrome hence defer elective surgical
l Fallots tetralogy
procedures
l Septal defects and ,20,000 At significant risk for bleeding;
l Congenital cardiac disorders defer elective dental procedures
362 Quick Review Series for BDS 4th Year, Vol 1

l Routine preventive and restorative treatment, including l Clinical importance of the WBC count is as follows:
nonblock injections, may be considered when the plate-
Absolute neutrophil
let count is at least 50,000/mm3.
count(cells/mm3) Significance
Q.7. Clinical importance of WBC counts in a leukaemic .1500 Normal
patient.
500–1000 Patient at some risk for infection; defer
Ans. elective procedures that could induce
significant transient bacteraemia.
l The absolute neutrophil count is an indicator of the
200–500 Patient must be admitted to the hospi-
host’s ability to suppress or eliminate infection. tal if febrile and given broad-spectrum
l It is calculated a follows: antibiotics; at moderate risk for sepsis;
ANC 5 (% of neutrophils 1 % of bands) total white defer all elective procedures.
cell count/100 ,200 At significant risk for sepsis.

Topic 9
Management of Children with Cleft Lip and Palate
COMMONLY ASKED QUESTIONS
SHORT ESSAYS:
1 . Define cleft lip and cleft palate.
2. Obturator in paediatric prosthodontics.
3. Treatment schedule of cleft palate. [Same as SE Q.1]

SHORT NOTES:
1 . Veau’s classification of cleft lip and cleft palate.
2. Kernahan and Starks classification of cleft palate.

SOLVED ANSWERS
SHORT ESSAYS: may range from a small notch in the lip to a complete fis-
sure extending up to the roof of mouth and nose.
Q.1. Define cleft lip and cleft palate. Treatment schedule for patient with cleft lip and palate:
A child with cleft lip and palate needs continuous care
Ans. from the team members until the late teenage.
Cleft lip and palate are congenital abnormalities that affect
the upper lip and the hard and soft palate. This abnormality
Section | I  Topic-Wise Solved Questions of Previous Years 363

The various treatments to be carried out at different ages Types of obturator:


are summarized as follows: i. Feeding obturator: Used to cover maxillary defects
Age Treatment in newborns to aid in feeding and suckling.
ii. Surgical obturator: Given after surgery to aid in
At birth l Construction of feeding plate
l Refer to a centre where a multidisci-
wound healing, hold dressings, maintain pressure
plinary cleft palate team exists on split thickness skin grafts
l Primary care advice about weight gain iii. Functional obturator: To help in deglutition
for fitness to surgery iv. Speech obturator:
l Infant oral care measures
l It is also known as speech aid prosthesis, naso-
3 months l Surgical repair of the lip pharyngeal obturator, speech appliance, pros-
l Monitoring speech and hearing thetic speech aid and speech bulb.
6 months l Preventive oral care measures l It is a temporary or interim prosthesis used to
l Discussion of anticipatory guidance close a defect in the hard and/or soft palate to
protocol with paediatric dentist replace tissue lost due to developmental or sur-
l Reinforcement of infant oral care mea-
gical alterations which is necessary for the pro-
sures and importance of preventive
dentistry duction of intelligible speech.
Indications:
12–18 months l Surgical repair of cleft palate
l To serve as a temporary prosthesis during the period
l Preliminary speech assessment
l Oral hygiene instructions
of surgical correction.
l To restore a patient’s cosmetic appearance rapidly
2–3 years l Initial assessment by orthodontist, ENT
surgeon, speech therapist
for social contacts.
l Assessment of surgical result by plastic l To act as a framework over which tissues may be

surgeon or oral and maxillofacial surgeon shaped by the surgeon.


l Quarterly dental check-ups l When the patient’s age or the local avascular condi-
l Parental education of effective oral
tion of the tissues contraindicates surgery.
hygiene measures
l When the patient is susceptible to recurrence of the
4–7 years l ENT assessment lesion which produced the deformity.
Beginning of speech therapy
l
Uses:
l Early correction of crossbite
l It may help to reconstruct the palatal contour and soft
l Orthodontist’s assessment on early
orthodontic treatment palate.
l It may be used for feeding purposes.
9 years l Aesthetic surgery – lip and nose
l It improves speech or, in some instances makes
revision
l Growth modification treatment speech possible.
l Early orthodontic treatment l It may be used to keep the wound or defective area

12 years l Correction of malalignment begins clean and may enhance the healing of postsurgical
l Creation of space for replacement of defects.
missing teeth l It can benefit the morale of patients with maxillary
l Alveolar bone graft
defects.
l Access school psychological
l It reduces the flow of exudates into the mouth.
adjustment
l The obturator may be used as a stent to hold
16 years l Aesthetic surgery to improve dressings or packs postsurgically in maxillary
appearance
resections.
l Any major orthognathic surgeries
l It reduces the possibility of postoperative haemor-
l Fixed and permanent replacement of

missing teeth rhage, and maintains pressure either directly or indi-


l Regular restorative care rectly on split thickness skin grafts, thus causing
close adaptation of the graft to the wound which
Q.2. Obturator in paediatric prosthodontics. prevents the formation of a hematoma and ultimate
Ans. failure of the graft.
An obturator is a disc or plate, natural or artificial, which Q.3. Treatment schedule of cleft palate.
closes an opening or defect of the maxilla as a result of a Ans.
cleft palate or partial or total removal of maxilla for a
tumour mass (Chalian 1971). [Same as SE Q.1]
364 Quick Review Series for BDS 4th Year, Vol 1

SHORT NOTES: 1 4
2
Q.1. Veau’s classification of cleft lip and cleft palate. 3 6
5

Ans.
Veau’s classification of cleft lip and cleft palate (1931): 7
l Group 1: Cleft of the soft palate only

l Group 2: Cleft of the hard and soft palate to the incisive 8


foramen
l Group 3: Complete unilateral cleft of the soft, hard pal-
9
ate and lip and alveolar ridge on one side
l Group 4: Complete bilateral cleft of the soft hard palate

and lip and alveolar ridge on both sides Fig. 9.1  Kernahan striped ‘Y’.

Q.2. Kernahan and Starks classification of cleft palate.


Ans.
Kernahan striped ‘Y’ classification (Fig. 9.1): Blocks 3 and 6 n Hard palate anterior to incisive foramen
Symbolic classification – uses a striped ‘Y’ having num- Blocks 7 and 8 n Hard palate posterior to incisive foramen
bered blocks, which represents specific areas of the oral Blocks 9 n Soft palate
cavity.
Blocks 1 and 4 n Lip The boxes are shaded in areas where the cleft has
Blocks 2 and 5 n Alveolus occurred.

Topic 10
Growth and Development of the Face
and Dental Arches
COMMONLY ASKED QUESTIONS
LONG ESSAYS:
1 . Discuss in detail growth of mandible.
2. Discuss the growth and development of dental arches from birth to adolescence.
3. Define growth and development. Discuss growth and their clinical implications.

SHORT ESSAYS:
1 . Growth spurts and growth trends.
2. Scammon’s growth curve.

SHORT NOTES:
1 . Skeletal age versus dental age.
2. Name few hormones influencing growth.
3. What are growth spurts? In which age are they seen?
4. Gum pads.
5. Broadbent phenomenon.
6. Age changes in mandible.
7. Significance of growth spurts. [Same as SN Q.3]
8. Growth spurts. [Same as SN Q.3]
continues on its own to form the sphenomandibular
ligament and the spinous process of the sphenoid bone
which are remnants of it.
Section | I  Topic-Wise Solved Questions of Previous Years 365

Between 8th and 12th weeks of IU life.


There is marked acceleration of mandibular growth as a
SOLVED ANSWERS
result mandibular length increases, the external auditory
meatus appears to move posteriorly.
LONG ESSAYS:
Q.1. Discuss in detail growth of mandible. Between 10th and 14th weeks of IU life.
Ans. Secondary accessory cartilages appear to form the head of
the condyle, part of the coronoid process and mental pro-
Prenatal growth of mandible: tuberance. Soon the growing intramembranous ossification
fuses the coronoid process to the ramus.
First structure to develop in primordium of lower jaw.
Is
Mandibular division of V nerve (induces osteogenesis by The ossification of the ramus proceeds and the condyle is
production of neurotrophic factors). soon fused to the mandible at about 16 weeks.

Followed by
Mesenchymal condensation forming the first arch Meckel’s cartilage does persists until as long as 24th week of
(mandibular arch). IU life before it disappears.

Postnatal growth of mandible:


Mandible is derived from ossification of an osteogenic l Among all the facial bones, the mandible undergoes
membrane formed from ecto-mesenchymal the largest amount of growth postnatally and also
condensation at around 36–38 days. exhibits the largest variability in morphology.
l Mandibular growth in the postnatal life shows the

integration of the periosteal and capsular matrices of


Resulting intramembranous bone lies lateral to Meckel’s functional matrix theory by Moss.
cartilage of 1st arch (mandibular arch). l Capsular matrix involves the oropharyngeal func-

tional spaces and the mandible grows according to


the functional needs of the particular functional sys-
At 6th week of IU life tem. The process of surface remodelling usually in-
In the region of bifurcation of inferior alveolar nerve into volves the activity of the periosteal matrix, i.e.
mental and incisive branches, a single ossification centre muscle fibres.
for each half of the mandible arises. Mandible at birth:
l Mandible at birth is much smaller in size and var-

ies in shape from the adult form. The infant man-


During 7th week of IU life. dible has a short more or less horizontal ramus
The bone begins to develop lateral to Meckel’s cartilage and with obtuse gonial angle.
continues until the posterior aspect is covered by the bone. l The condyles are low and at the position of

Ossification stops at the point which will later become the occlusal plane. The symphyseal suture has not
the mandibular lingula, from where Meckel’s cartilage ossified.
continues into middle ear and develops into auditory Growth in the first year:
ossicles, i.e. malleus and incus. l It involves growth at the symphyseal suture and

The remaining part of the Meckel’s cartilage lateral expansion in the anterior region to accom-
continues on its own to form the sphenomandibular modate the erupting anterior teeth.
ligament and the spinous process of the sphenoid bone l The mental foramen is directed at right angle to

which are remnants of it. the surface of the corpus.


l There is increased bone deposition in the poste-

rior surface of the ramus of the mandible.


Between 8th and 12th weeks of IU life. l The infant mandible is suited for the suckling ac-
There is marked acceleration of mandibular growth as a tivity since the condyle and the glenoid fossa is
result mandibular length increases, the external auditory flat, which helps in the anteroposterior movement
meatus appears to move posteriorly. of the mandible.

Between 10th and 14th weeks of IU life.


Secondary accessory cartilages appear to form the head of
the condyle, part of the coronoid process and mental pro-
tuberance. Soon the growing intramembranous ossification
fuses the coronoid process to the ramus.
366 Quick Review Series for BDS 4th Year, Vol 1

Mandible in the adult/concept of V principle: Rotation of mandible:


The adult mandible differs from the mandible of an l Bjork used implants to study the growth pattern of

infant in that mandible and found that mandible undergoes growth


l The ramus is longer and the gonial angle is less rotation. It was found that though mandible under-
obtuse. goes rotation, the effects seen are minimal due to
l The bone is larger on the whole and the con- external compensation.
dyle is well developed. It was concluded that the growth of mandible is largely
l All these changes take place in the growth of influenced by the functional matrices, and condylar
the mandible in the form of an expanding V. cartilage has little influence in its overall growth.
l Because of its horseshoe shape, it is easier to Summary of mandibular growth:
visualize mandible as the V-shaped bone than Length increases by:
the maxilla. i. Surface apposition at posterior border of ramus
V-principle of growth: and resorption at anterior border
According to this principle growth of mandible in ii. Deposition at bony chin
length, width and height is as follows: iii. Growth at condylar cartilage
Length: Height increases by:
l The growth of the mandible in length antero-posteri- i. Surface apposition at alveolar border
orly is by the deposition of bone at the posterior ii. Apposition at the lower border of mandible
border of the ramus and resorption at the anterior iii. Growth at the condylar cartilage
surface, which helps to lengthen mandible so that the Width increases by:
anterior part of the ramus is occupied by the poste- i. Sutural growth up to first year postnatally
rior part of the body in the future and accommodates ii. Later surface apposition at outer surface
the developing permanent molars. Growth sites in mandible are
l As the articulation of the condyle to the glenoid fossa i. Mandibular condyle
is constant, the anterior displacement causes displace- ii. Posterior border of ramus
ment of the mandible anteriorly as it grows posteriorly. iii. Alveolar process
l There is corresponding surface remodelling at the iv. Lower border of mandible
anterior border with deposition in the posterior sur- v. Suture
face of the symphysis and resorption in the superior
Q.2. Discuss the growth and development of dental
part of the anterior surface and deposition in the in-
arches from birth to adolescence.
ferior aspect.
Width: Ans.
l There is deposition in the lateral surface of the ramus
The growth and development of dental arches or occlusal
and resorption on the lingual surface of mandible
development from birth to adolescence can be divided into
below the mylohyoid ridge. In contrast, the coronoid
the following periods or stages:
process undergoes apposition at the medial surface
Predental period:
and resorption at the lateral surface. This expands the
l This period extends from birth to 6 months of age
mandible like a V.
after birth, i.e. 0–6 months.
l The condyle undergoes reduction of bone on the lat-
l The neonate is without teeth for about 6 months of
eral aspect of neck and deposition corresponding to
life. The alveolar arches of an infant at this time
the V principle makes the condyle longer at the neck.
period are known as gum pads.
l Following the V principle, the inter-ramal distance is
The features of gum pads are as follows (Fig. 10.1):
efficiently increased by the growth of mandible,
l They are pink in colour and firm in consistency cov-
which helps the mandible to keep pace with the
ered by dense layer of fibrous periosteum.
growth of the cranial base.
l They are horseshoe shaped and develop in two parts:
Height:
(a)   Labiobuccal portion
l Alveolar process height increases well with eruption
(b) Lingual portion
of teeth.
Dental groove:
l Bone deposition taking place in the lower border of
These two portions are separated by a groove called the
mandible also contribute to increase in height of the
dental groove.
mandible.
Section | I  Topic-Wise Solved Questions of Previous Years 367

ii. Primate/anthropoid/simian spaces:


l These physiologic spaces are present invariably on

mesial side of maxillary canines and distal side of


Lateral sulcus
mandibular canines.
l As these spaces are commonly seen in primates.

Dental groove They are known as primate spaces, simian spaces


or anthropoid spaces. These spaces help in place-
ment of the canine cusps of the opposing arch.
Gingival groove iii. Shallow overjet and overbite.
iv. Ovoid arch form.
Fig. 10.1  Maxillary gum pad. v. Almost vertical inclination of anterior teeth.
vi. Flush terminal plane:
The mesiodistal relation between the distal surfaces of
Transverse grooves: the upper and lower second deciduous molars (E) is
l The gum pads are divided into 10 segments by trans- called the terminal plane.
verse grooves. Each segment consists of one devel- A normal feature of deciduous dentition is a flush
oping deciduous tooth sac. terminal plane, where the distal surfaces of the upper
Lateral sulcus: and lower second deciduous molars are in the same
l The transverse groove between the canine and first plane.
deciduous molar segment is called the lateral sulcus. vii. Deep bite:
l The lateral sulcus of mandibular arch is normally The deep bite occurs in the initial stages of develop-
more distal to that of maxillary arch. ment and is accentuated by the more upright decidu-
The lateral sulci are useful in judging the interarch ous incisors compared to their successors.
relationship of maxilla and mandible at very early This deep bite is reduced later due to:
stage. (a)   Eruption of deciduous molars
l Upper and lower gum pads are almost similar to each (b)   Attrition of incisors
other. (c)  Forward movement of the mandible due to
Relationship of gum pads: growth
l When upper and lower gum pads are approximated The mixed dentition period:
there in a complete overjet all around, as the upper This period ranges from 6 years to 12 years of age.
gum pad is wider as well as longer than lower gum This period can be divided into three phases:
pad. (a)   First transitional period
l Class II pattern is exhibited as maxillary gum pad (b) Intertransitional period
being more prominent. (c)   Second transitional period
l Anterior open bite: This infantile open bite is consid- (A) First transitional period:
ered normal. Contact occurs between the upper and It is characterized by:

lower gum pads in first molar region and a space ex- (i) Emergence of the first permanent molars.

ists between them anteriorly known as infantile open (ii) Exchange of the deciduous incisors with
bite which helps in sucking. permanent incisors.
The deciduous dentition period:
(i) Emergence of the first permanent molars:
l The deciduous dentition period extends from
l Mandibular first molar is the first permanent
6 months to 6 years of postnatal life.
tooth to erupt at around 6 years of age.
l It starts with eruption of deciduous mandibular
l The distal surface of the second deciduous
central incisors and completes with second de-
molar, i.e. (E), guides the first permanent
ciduous molars coming into occlusion.
molars into the dental arch.
l The eruption of all primary teeth is completed by
l The location and relationship of the first
2½–3½ years of age.
permanent molars depend much on the dis-
The normal features of the ideal occlusion in the primary
tal surface relationship between upper and
dentition are as follows:
lower second deciduous molars E .
i. Spacing of anterior teeth: E
l Spaces existing between the deciduous teeth l The distal surface relationship between the

called physiological or developmental spaces are upper and lower second deciduous molars
important for normal development of permanent can be of three types:
dentition. a. Flush terminal plane (76%)
368 Quick Review Series for BDS 4th Year, Vol 1

. Mesial step terminal plane (14%)


b In this type of relationship:
c. Distal step terminal plane (10%) l Distal surface of mandibular second decidu-

) )
ous molar E E is more mesial than that of

6 E D
maxillary second deciduous molar E E .
) )
l The permanent molars erupt directly into
the Angle’s class I occlusion.

If forward growth
6 E D
of mandible

Fig. 10.2  Flush terminal plane. Persists Minimal

a. Flush terminal plane (Fig. 10.2): Leads to Establishes


l The distal surface of the upper and lower sec-

ond deciduous molars are in one vertical plane. Angle’s class III molar Angle’s class I molar
This type of relationship is called flush or ver- relationship relationship
tical terminal plane relationship, which is a
normal feature of deciduous dentition. C. Distal step terminal plane (Fig. 10.4):
l The erupting first permanent molars may also

be in a flush or end on relationship which shifts


to class relation by either:
a. Early shift 6 E D
b. Late shift
l Early shift:

Occurs during early mixed dentition pe-


riod, where eruptive force of the first 6 E D
permanent molar is sufficient to push the
deciduous first and second molars for-
ward to close the primate spaces and es- Fig. 10.4  Distal step terminal plane.
tablish class I molar relationship.
l Late shift:

Occurs in the late mixed dentition pe- l In this type of relationship, the distal
riod. In children lacking primate spaces, surface of E E is more distal to the of
the erupting permanent first molars drift E E
mesially utilizing the leeway space,
when deciduous 2nd molars exfoliate. l Erupting permanent molars assume
b. Mesial step terminal plane (Fig. 10.3): Angle’s class II occlusion here.
(ii) The exchange of incisors:
l The deciduous incisors are replaced by

the permanent incisors during first tran-


sition period.
l The mesiodistal width of permanent
6 E D
incisors is larger than deciduous teeth
they replace.
l Incisal liability is the difference be-

6 E D tween the amount of space needed for


accommodation of the incisors and the
amount of space available for them to
Fig. 10.3  Mesial step terminal plane. occupy. It was described by Warren–
Mayne in 1969.
Section | I  Topic-Wise Solved Questions of Previous Years 369

l Incisal liability is Q.3. Define growth and development. Discuss growth


a. 7 mm in maxillary arch and their clinical implications.
b. 5 mm in mandibular arch Ans.
l Incisal liability can be overcome by:

i. Utilization of interdental spaces seen in Growth has been described variedly by various paedodon-
primary dentition tists as below:
ii. Increase in intercanine width Definition of growth according to:
l Stewart, 1982:
iii. Change in incisor inclination
(B) Inter transitional period: Growth may be defined as developmental increase in
l It is relatively stable and no changes occur dur- mass. In other words, it is a process that leads to an
ing this phase to mixed dentition. increase in the physical size of the cells, tissue or-
l The maxillary and mandibular arches consist gans or organisms as a whole.
l Proffit, 1986:
of sets of deciduous and permanent teeth dur-
ing this period. Growth refers to an increase in size or numbers.
l Stedman, 1990:
(C) Second transitional period:
i. The replacement of deciduous molars and ca- Growth is an increase in size of a living being or any
nines by the premolars and permanent cus- of its parts, occurring in the process of development.
pids, respectively, is characteristic of this l Pinkham, 1994:

phase. Growth signifies an increase, expansion of any given


ii. The leeway space of Nance is the excess space tissue.
available after the exchange of the deciduous Definition of development according to:
l Todd (1931):
molars and canines with permanent teeth. It is
utilized for mesial drift of mandibular molars to Development is an increase in complexity.
l Moyers (1988):
establish class I molar relation.
iii. Ugly duckling stage (7–11 years of age): Development refers to all the naturally occurring
l It is also known as Broadbent’s phenomena; unidirectional changes in the life of an individual
it is a transient or self-correcting malocclu- from its existence as a single cell to its elaboration as
sion seen in maxillary incisor region par- a multifunctional unit terminating in death.
l Stedman (1990):
ticularly during eruption of permanent
canines. Development refers to the act or process of natural
l During eruption of permanent canines progression from previous, lower or embryonic stage
they impinge on roots of lateral incisors to a later, more complex adult stage.
l Pinkham (1994):
displacing them mesially, which in turn re-
sults in transmission of force on to the roots Development addresses the progressive evolution of
of central incisors which also get displaced a tissue.
mesially. Importance of study of growth and development in paedo-
l A resultant distal divergence of crowns of dontics is as follows:
two central incisors causes a midline dia- l Knowledge of normal human growth is essential for

stema. the recognition of abnormal or pathologic growth.


This situation has been described by Broad- l Estimation of growth potential is necessary to achieve

bent as ugly duckling stage as children tend a stable functional and an aesthetic result in cases of
to look ugly during this phase. orthodontic, surgical orthognathic and orthopaedic
(D) The permanent dentition period: corrections of dentofacial disharmony.
This period extends from shedding of last primary l The study of growth in children is done to assess

tooth and eruption of all permanent teeth. the health and nutrition of children living in a
The frequently seen eruption sequence of the per- nation.
manent dentition is as follows: l The study of growth in children is done to compare

l Maxillary arch 6-1-2-4-3-5-7 the growth of an individual child with the growth of
Or 6-1-2-3-4-5-7 a large sample of other children. This is important for
l Mandibular arch 6-1-2-3-4-5-7 health education professionals and parents who care
Or 6-1-2-4-3-5-7 for growing children.
370 Quick Review Series for BDS 4th Year, Vol 1

l Growth rate may be the best indicator of the physical indicates the magnitude of skeletal join discrep-
and psychologic wellbeing of children. ancies. The normal value of ANB angle is 2°.
l To use the myofunctional appliances appropriately to l If it is less than 2° then it is indicative of class

know the growth spurt period. II and if it is more than 2° then indicative of
class III malocclusion.
SHORT ESSAYS: Type A:
l The maxilla and mandible grow together and
Q.1. Growth spurts and growth trends. thus ANB angle remains the same. Should this
Ans. be accompanied by class I relationship and ANB
does not exceed 4.5˝, no treatment is indicated.
Growth spurts and differential growth:
Type A subdivision
l Growth does not continue uniformly at all times.
l Maxilla is protruding with an ANB angle of
There seems to be periods when a sudden accelera-
more than 4.5°. The treatment is to restrict the
tion of growth occurs. This sudden increase in growth
growth of maxilla while allowing mandible to
is termed as ‘growth spurt’.
catch up. The prognosis is good, but may
l Such accentuated growth is believed to be the
sometimes require extraction of premolars.
caused due to the physiological alteration in hor-
Type B:
monal secretion.
l Mandible and maxilla grow forward and down-
l The timing of growth spurt differs in boys and girls.
wards with the growth of maxilla exceeding
l Growth spurts are mainly due to following causes:
that of mandible.
i. Prebirth – cell division
l Poor prognosis and indicates that point B will
ii. Postbirth – hormonal influence
not catch up with point A.
Timings of growth spurts:
l Growth of middle and lower face is predomi-
i. Just before birth.
nantly in vertical direction.
ii. One year after birth.
Type B subdivision:
iii. Mixed dentition growth spurt:
l The ANB angle is large and continues to
Boys: 8–11 years and girls: 7–9 years
grow indicating unfavourable growth trend.
iv. Prepubertal growth spurt/adolescent growth spurt:
Type C:
Boys: 14–16 years and girls: 11–13 years.
l The maxilla and mandible grow forward and
v. Prepubertal growth spurt/adolescent growth spurt
downwards with mandible growing forward
has been divided into three phases:
more rapidly.
a. Prepubertal take off stage – moderate increment
l The ANB angle is seen to be decreasing with
in the height velocity.
the mandible catching up with maxilla. This
b. Pubescent phase – very rapid growth phase.
indicates favourable trend.
c. Postpubescent phase – decelerating height ve-
l No treatment is required until eruption of
locity finally, linear growth comes to a stop
canine.
with fusion of the epiphyses.
Type C subdivision:
Clinical application:
l The mandible is found to be growing more
l Growth modification treatments by means of func-
forward when compared to maxilla. With
tional and orthopaedic appliances elicit better re-
this the mandible incisors touch the lingual
sponse during growth spurts.
surface of maxillary incisors.
l Surgical corrections involving maxilla and mandible
l Therefore lingual tipping of mandibular in-
should be carried out only after cessation of the
cisors and labial tipping of maxillary inci-
growth spurts.
sors are obvious.
Growth trends:
l By overlapping consequent cephalograms, Tweed Q.2. Scammon’s growth curve.
discerned a pattern of growth and termed it as
growth trends. Ans.
ANB angle: l Throughout life human body does not grow at the same
l According to Sterner it is the angle between rate, but different organs grow at different rates to a dif-
point A on maxilla and point B on mandible. It ferent amount and at different times; this is known as
is the difference between SNA and SNB and differential growth.
Section | I  Topic-Wise Solved Questions of Previous Years 371

l The concepts of differential growth are more clearly Effect of Scammon’s growth in facial region:
understood by two important aspects of growth: l Mandible follows somatic growth pattern.
a. Cephalo caudal gradient of growth Long-time growth is seen until about 18–20
b. Scammon’s curve of growth years in males.
l Maxilla follows neural growth pattern and

a. Cephalocaudal gradient of growth: growth ceases earlier hence skeletal problems


l An axis of increased growth gradient extending from of the maxilla should be treated earlier to
head towards the feet is called ‘cephalocaudal mandible.
growth’. l Scammon’s growth curve indicates that growth

l In fetal life, head constitutes 50% of total body rate of different tissues are different at different
length while limbs are primitive (30%). ages.
At the time of birth, head constitutes 25%–30% and For example:
there is increased growth of body and limbs. l The various tissues for which Scammon’s

In an adult the head constitutes only 12%, while limbs growth curve is plotted are lymphoid tis-
accounts to 50%. These changes in the pattern of sue, neural tissue, general or visceral tissue
growth are because of cephalocaudal gradient. and genital tissue.
Cephalocaudal growth in face: l Lymphoid tissue proliferates rapidly in late

l At the time of birth, jaws and face are less childhood to almost 200% of adult size.
developed compared to skull. Maxilla being By 18 years, it undergoes involution to
closer to head grows faster and growth is reach adult size.
completed before mandibular growth. Man- l Neural tissue grows very rapidly and al-

dible being away from the brain grows more most reaches adult size by 6–7 years of age
and growth completes later than maxilla. after that a very little growth occurs in neu-
b. Scammon’s curve of growth: ral tissue.
Major tissues of the human body are divided into four l General/visceral tissue exhibits ‘S’-shaped

types: curve which indicates rapid growth up to


i. Lymphoid tissue 2–3 years of age followed by slow phase
ii. Neural tissue between 3 and 10 years of age and followed
iii. General tissue again by rapid phase of growth occurring
iv. Genital tissue after 10th year terminating by 18–20 years.
These different tissues grow at different time and l Genital tissue shows negligible growth un-

at different rates til puberty. They grow rapidly at puberty


i. Lymphoid tissue: reaching adult size after which growth
Proliferates rapidly in late childhood to ceases.
almost 200% of adult size; (adaptation to
protect child from infection) by 18 years it SHORT NOTES:
undergoes involution to reach adult size.
ii. Neural tissue: Q.1. Skeletal age versus dental age.
Grows very rapidly and almost reaches
adult size by 6–7 years of age after that a Ans.
very little growth occurs in neural tissue.
iii. General/visceral 1 (muscle, bone and Skeletal age Dental age
other organs): It is based on the ossification The formation of teeth or
They exhibit ‘S’-shaped curve; rapid of endochondral bone. eruption of the teeth is basis
for calculating dental age.
growth up to 2–3 years of age followed by
slow phase between 3 and 10 years of age Assessed based on the skeletal Assessed based on the number
and followed again by rapid phase of maturity indicators like hand- of teeth at each chronological
wrist radiographs, cervical age or on stages of formation
growth occurring after 10th year terminat-
vertebrae. of crowns and roots of the
ing by 18–20 years. teeth.
iv. Genital tissue (reproductive organs):
Negligible growth until puberty. They
Q.2. Name few hormones influencing growth.
grow rapidly at puberty reaching adult size
after which growth ceases. Ans.
372 Quick Review Series for BDS 4th Year, Vol 1

There are three types of hormones responsible for growth: v. Lower gum pads are V-shaped and similar to maxillary
Group I: Hormones influencing skeletal bone growth gum pads but the segments are less defined when com-
l Growth hormone pared to maxillary gum pad.
l Insulin vi. Gum pads’ relationship is arbitrary; they do not have
l Thyrotropic hormone definite relationship.
Group II: Hormones responsible for ossification of long
Q.5. Broadbent phenomenon.
bones
l Parathormone Ans.
Group III: Hormones responsible for pubertal growth spurt
l Ugly duckling stage (7–11 years of age) is also known
l Androgens
as Broadbent’s phenomena.
l Progestrone and oestrogen
l It is a transient or self-correcting malocclusion seen in

Q.3. What are growth spurts? In which age are they maxillary incisor region particularly during eruption of
seen? permanent canines.
l During eruption of permanent canines they impinge on
Ans.
roots of lateral incisors displacing them mesially, which
l During process of growth, there seems to be periods in turn results in transmission of force on to the roots of
when a sudden acceleration of growth occurs. This sud- central incisors which also get displaced mesially.
den increase in growth is termed growth spurt. l A resultant distal divergence of crowns of two central

l The timing of the growth spurts differ in boys and girls incisors causes a midline diastema. This situation has
as follows: been described by Broadbent as ugly duckling stage as
a. Just before birth children tend to look ugly during this phase.
b. One year after birth
Q.6. Age changes in mandible.
c. Mixed dentition growth spurt (boys: 8–11 years and
girls: 7–9 years) Ans.
d. Prepubertal growth spurt (boys: 14–16 years and
Infant mandible:
girls: 11–13 years)
l The infant mandible has a short, more or less hori-
l Clinical importance:
zontal ramus with obtuse gonial angle.
a. Knowledge of growth spurts is essential for success-
l The condyles are low and at the position of the oc-
ful treatment planning in orthodontics.
clusal plane.
b. Growth modulation by means of functional and orth-
The adult mandible differs from the mandible of an infant
odontic appliances elicit better response during
in that:
growth spurts.
l The ramus is longer and the gonial angle is less obtuse.

Q.4. Gum pads. l The bone is larger on the whole and the condyle is

well developed.
Ans.
l All these changes take place in the growth of the
i. The alveolar arches at the time of birth are called gum mandible in the form of an expanding V.
pads and are firm and pink.
Q.7. Significance of growth spurts.
ii. Maxillary gum pads develop in two parts, namely
labiobuccal and lingual; they are demarcated by the Ans.
dental groove. Labiobuccal part grows fast and is di-
[Same as SN Q.3]
vided into 10 segments by transverse grooves which
correspond to the deciduous tooth sac. Q.8. Growth spurts.
iii. The groove between the canine and deciduous first
Ans.
molar is called lateral sulcus.
iv. Gingival groove demarcates the palate from gum pads. [Same as SN Q.3]
Section | I  Topic-Wise Solved Questions of Previous Years 373

Topic 11
Development and Morphology of Primary
Teeth and Occlusion
COMMONLY ASKED QUESTIONS
LONG ESSAYS:
1. What is importance of deciduous teeth? Describe anatomic and histologic differences between the primary and
permanent dentition.
2. Describe the stages of development of dentition.
3. Define normal occlusion. Describe in brief the development of occlusion from 6 to 12 years.
4. Write in detail about the eruption sequence in deciduous dentition which deciduous and permanent teeth are
present in child aged 10 years.
5. What are the causes, sequelae and management of early loss of primary teeth?
6. What are the morphologic differences between maxillary and mandibular primary teeth?
7. Discuss the various treatment modalities in case of a premature loss of first permanent molar in a developing
occlusion.
8. Discuss the morphological and histological differences of primary and permanent teeth and its significance.
[Same as LE Q.1]
9. What are the morphologic differences between primary and permanent teeth? [Same as LE Q.1]
10. Enumerate the different stages of tooth development. Discuss various developmental abnormalities and distur-
bances of teeth and other oral structure during these developmental stages. [Same as LE Q.2]
11. Describe the development of occlusion from deciduous to permanent stages. [Same as LE Q.3]
12. Discuss the development of normal occlusion from gum pad relationship till the eruption of 2nd permanent
molar. [Same as LE Q.3]
13. What is transitional period? Describe the changes seen in occlusion in this stage. [Same as LE Q.3]
14. Discuss in brief the classification of occlusal relationship in primary dentition, its variation and disharmony.
[Same as LE Q.3]
15. Define growth and development. Discuss the development of normal occlusion from gum pad relationship till
the eruption of 2nd permanent molar. [Same as LE Q.3]
16. Discuss the development of dentition from 6 to 10 years. [Same as LE Q.4]

SHORT ESSAYS:
1. Explain local and systemic factors for delayed eruption of teeth.
2. Predentate period. [Ref LE Q.3]
3. What is teething disorder? Explain the management.
4. Chronology of human primary teeth. [Ref LE Q.4]
5. Transient malocclusion.
6. Young permanent first molar tooth. [Ref LE Q.7]
7. Explain morphological differences between primary and permanent teeth. [Ref LE Q.1]
8. Importance of deciduous dentition. [Ref LE Q.1]
9. Development of occlusion from 6 to 12 years. [Ref LE Q.3]
10. Characteristics of primary dentition. [Ref LE Q.3]
11. Local and systemic causes of delayed eruption. [Same as SE Q.1]
12. Sequence and time of eruption of primary teeth. [Same as SE Q.4]
13. Self-correcting anomalies. [Same as SE Q.5]
14. Importance of young permanent tooth. [Same as SE Q.6]
15. Importance of first permanent molar. [Same as SE Q.6]
16. What is transitional period? Describe the changes seen in occlusion at the stage. [Same as SE Q.9]
374 Quick Review Series for BDS 4th Year, Vol 1

SHORT NOTES:
1. Teething. [Ref SE Q.3]
2. Primate spaces. [Ref LE Q.3]
3. Gum pads.
4. Eruption cyst.
5. Eruption sequestrum.
6. Chronology of deciduous dentition.
7. Mention different stages of tooth development. [Ref LE Q.2]
8. Mamelons.
9. Causes of precocious eruption.
10. Incisal liability.
11. Mulberry molars.
12. Leeway space.
13. Ugly duckling stage.
14. Primary molar relationship. [Ref LE Q.3]
15. Early and late mesial shift. [Ref LE Q.3]
16. Young permanent first molar.
17. FDI tooth numbering system.
18. Skeletal age versus dental age.
19. Natal and neonatal teeth.
20. Pulpal differences in primary and permanent teeth.
21. Define eruption.
22. Explain dental age and chronological age.
23. Neonatal line.
24. Features of primary dentition.
25. Teeth present in the jaws at birth.
26. Ectopic eruption.
27. Eruption sequence.
28. Teething disorder. [Same as SN Q.1]
29. Chronology of eruption of primary teeth. [Same as SN Q.6]
30. Leeway space of Nance. [Same as SN Q.12]
31. Terminal plane relationship. [Same as SN Q.14]
32. Flush terminal plane. [Same as SN Q.14]
33. Late mesial shift. [Same as SN Q.15]
34. Give the importance of 1st permanent molar. [Same as SN Q.16]

SOLVED ANSWERS
LONG ESSAYS:
Q.1. What is importance of deciduous teeth? Describe Importance of primary teeth:
anatomic and histologic differences between the pri-
mary and permanent dentition. [SE Q.8]
Ans. {l   The emergence of the primary dentition through
l Emergence of the primary dentition takes place between the alveolar mucosa is an important time for the
the 6th and 13th months of postnatal life. development of oral motor behaviour and the ac-
l It takes from 2 to 3 years for the primary dentition to be quisition of masticatory skills.
completed beginning with the initial calcification of the l At this time of development, the presence of

primary central incisor to the completion of the roots of ‘teething’ problems suggests how the primary
the primary second molar. dentition can affect the development of future
Section | I  Topic-Wise Solved Questions of Previous Years 375

neurobehavioural mechanisms, including jaw Primary teeth Permanent teeth


movements and mastication.
Molars have narrow occlusal There is less convergence of
l Learning of mastication may be highly dependent table in a bucco-lingual plane; buccal and lingual surfaces of
on the stage and development of the dentition, e.g. occlusal plane is relatively molars towards the occlusal
type and number of teeth present and occlusal flat. surface.
relations, the maturation of the neuromuscular Molars are more bulbous and Have more curved contours;
system and such factors as diet. are sharply constricted cervi- they have less constriction at
l Even though, for the individual child, consider- cally (bell shaped). the neck.
able variation in the times of emergence of the The enamel is thinner and has Enamel is thicker and has a
primary dentition may occur, the primary denti- a more consistent depth of thickness of about 2–3 mm.
tion is completely formed by about age of 3 years about 1 mm thickness
and functions for a relatively short period of time throughout the entire crown.
before it is lost completely about age of 11 years. Contact areas between molars Contact point between
l The loss of the deciduous teeth tends to mirror the are broader, flatter and situ- permanent molars is situated
eruption sequence: incisors, first molars, canines ated gingivally. occlusally.
and second molars, with the mandibular pairs The enamel rods at the cervix The enamel rods at the region
preceding the maxillary teeth. The increase in slopes occlusally from the DEJ. are oriented gingivally.
prevalence of dental caries among tooth types Supplemental grooves are Supplemental grooves are less.
reverses their order of eruption. more.
l The role of the primary teeth in mastication and Mamelons are absent. Mamelons present on incisal
their function in maintaining the space for erup- edges of newly erupted molars.
tion of the permanent teeth.
First molar is smaller in di- First molar is larger in dimen-
l A lack of space associated with premature loss of mension than 2nd molar. sion than 2nd molar.
deciduous teeth is a significant factor in the devel-
Roots are larger and more Roots are shorter and more
opment of malocclusion.
slender. bulbous.
l The development of adequate spacing is an im-
portant factor in the development of normal oc- Pulpal outline follows the DEJ Pulp outline follows the DEJ
more closely; the pulp horns less closely.
clusal relations in the permanent dentition. There- are closer to the outer surface.
fore it is important to prevent and treat dental
decay by providing the child with a comfortable Root canals are more ribbon Root canals are well defined
functional occlusion of the deciduous teeth.}
like; the radicular pulp follows with less branching.
a thin, tortuous and branching
path.
[SE Q.7]
Accessory canals in the floor Floor of the pulp chamber
{Major differences between primary and permanent teeth of pulp chamber leads directly
into inter-radicular furcation.
does not have any accessory
canals.
are as follows:
Enamel and dentin are less They are more mineralized.
Primary teeth Permanent teeth mineralized.
Teeth are lighter in colour; Darker in colour; greyish or Secondary cementum is Secondary cementum is
bluish white. yellowish white. absent. present.
Number of teeth is 20; premo- Number of teeth is 32. Neonatal lines are present. Neonatal line is seen only in
lars and molars are absent. first molar.
Crowns are wider mesiodis- Crowns are larger in cervico- High potential for repair. Comparatively less potential
tally in relation to the cervico-
occlusal height, which gives a
occlusal dimension than the
mesio-distal dimension
}
for repair .

cup-shaped appearance to an- Cuspids are less conical.


terior teeth and square-shaped
appearance to molars; cuspids Q.2. Describe the stages of development of dentition.
are more conical.
Ans.
Cervical ridges are more pro- Cervical ridges are flatter.
nounced especially on the Tooth development is a dynamic process and goes through
buccal aspect of first primary various stages of formation. It begins from the differentia-
molar.
tion of the oral ectoderm to dental lamina formation.
376 Quick Review Series for BDS 4th Year, Vol 1

l This extends along the entire margin of the jaws and


{SN Q.7}
is known as the primordium of the ectodermal por-
Different stages of development of teeth: tion of the teeth which results in dental lamina.
The tooth development progresses in the following l Ten round or ovoid swellings occur in each jaw dur-

stages according to American Academy of Paediatric ing the same time, in the positions to be occupied by
Dentistry: the primary dentition.
A. Morphological developmental stages: l The entire primary dentition and permanent molars

l Dental lamina arise from the dental lamina. The permanent incisors,
l Bud stage canines and premolars develop from the buds of their
l Cap stage predecessors.
l Bell stage Proliferation (cap) stage:
l Advanced bell stage l The method of proliferation of cells continues in the

l Hertwig epithelial root sheath cap stage.


l Formation of enamel and dentin matrices l A cap is formed as a result of uneven growth in dif-

B. Histo-physiological development stages: ferent parts of the bud.


l Initiation l A shallow invagination appears on the deep surface

l Proliferation of the bud, with the peripheral cells of the cap later
l Histodifferentiation forming the outer and inner enamel epithelium.
l Morphodifferentiation Histodifferentiation and morphodifferentiation (bell)
l Apposition stages:
l Calcification (mineralization) and maturation l The epithelium continues to invaginate and gets

deeper until the enamel organ assumes the shape of a


bell.
Dental lamina: l There is differentiation of the cells in the dental
l Two types of cells are involved in development of
papilla into odontoblasts and the cells of the inner
mammalian teeth, they are enamel epithelium into ameloblasts.
i. Stomodeal ectoderm, which forms ameloblasts. l The histodifferentiation marks the end of prolifera-
ii. Cranial neural crest-derived (ecto) mesenchyme tion as the cells lose the capacity to multiply. This
cells, which form odontoblasts and cementoblasts. stage is the predecessor of apposition stage.
l These two cell types juxtaposed in the developing
l In the morphodifferentiation stage, the formative
oral cavity, interact to control the entire process of cells are arranged to outline the shape and size of
tooth development like initiation, morphogenesis and the tooth, the process that occurs before matrix
cytodifferentiation. deposition.
l Dental lamina begins at 6 weeks of intrauterine life
l The morphology of the tooth gets established when
and tooth buds arise from the lamina. Initiation of the inner enamel epithelium is arranged such that the
primary teeth is seen from second month in utero, margin between it and the odontoblasts delineates
successor teeth from 5 months in utero and initiation the future dentino-enamel junction.
of accessory teeth from 4 months in utero for first l Mammalian tooth morphogenesis must be controlled
permanent molars and 1 year after birth for second by ectodermally derived cells and/or ecto-mesenchy-
permanent molars. mally derived cells, since these are the only cell
l The enamel of teeth is derived from the ectoderm,
types that form teeth.
and the mesoderm provides the anlage for the dentin, l Current evidence suggests that the information for
pulp and periodontal tissues. Neural crest cells are generation of an incisor or a molar tooth is inherent
responsible for tissues like bone, cartilage, dentin in the ectomesenchyme, whereas the establishment
and dermis but not the enamel which is derived from and actual mechanics of using this information are
the stomodeal ectoderm. carried out by ectodermally derived cells of the
Initiation (bud stage): enamel knot.
l The first sign of tooth development can be observed l Spatial domains of homeobox genes are established
as early as the 6th week of intrauterine life. in the ectomesenchyme and provide the positional
l The tooth bud consists of enamel organ, dental information for the specification of tooth shape.
papilla and dental sac. Apposition stage:
l An epithelial thickening in the region of the future l A layer-like deposition of a nonvital extracellular
dental arch is formed by proliferation of cells from secretion in the form of a tissue matrix results in
the basal layer of oral epithelium. apposition.
Section | I  Topic-Wise Solved Questions of Previous Years 377

l This tissue matrix is deposited by the formative cells, In cementum: Anomalies of cementum as in hypo-
ameloblasts and the odontoblasts that line up along phosphatasia, epidermolysis bullosa and cleidocra-
the future dentinoenamel and dentinocemental junc- nial dysplasia.
tions at morphodifferentiation stage. Calcification stage:
l The enamel and the dentin matrix are deposited by Anomalies of structure also occur due to disturbances in
these cells according to a definite pattern and rate. calcification stage of teeth.
l The formative cells begin their act at specific sites In enamel: Hypocalcification occurs as in amelogen-
referred to as growth centres as soon as the dentino­ esis imperfecta type III – hypocalcified and enamel
enamel junction is formed. fluorosis.
Calcification and maturation: In dentin: Sclerotic dentin results.
l Matrix deposition is followed by calcification or
Q.3. Define normal occlusion. Describe in brief the
mineralization which involves the precipitation of
development of occlusion from 6 to 12 years.
inorganic calcium salts within the deposited matrix.
l The process begins with a nidus around which fur- Ans.
ther precipitation occurs. The nidus increases in size
Growth:
by addition of concentric laminations.
l According to Stewart, 1982: Growth may be defined
l These individual calcospherites approximate and
as developmental increase in mass. In other words, it
fuse with each other to form a homogenously miner-
is a process that leads to an increase in the physical
alized layer of tissue matrix.
size of the cells, tissue organs or organisms as a whole.
Various developmental abnormalities and disturbances of
or
teeth and other oral structure during these developmental
l According to Proffit, 1986: Growth refers to an
stages:
increase in size or numbers.
l Due to the disturbances that occur at different stages
Development:
of development of dentition, various anomalies of
According to Todd (1931): Development is an increase
development appear which are as follows:
in complexity.
i. Initiation or bud stage:
or
l If there is any disturbance in the initiation or
According to Moyers (1988): Development refers to all
bud stage, anomalies of number occur.
the naturally occurring unidirectional changes in the life
l For example, hyperdontia, hypodontia (oligo-
of an individual from its existence as a single cell to its
dontia) or anodontia
elaboration as a multifunctional unit terminating in
ii. Proliferation or cap stage:
death.
l Anomalies of size like microdontia and mac-
Occlusion:
rodontia occur due to disturbances in prolif-
The term occlusion is derived from the Latin word, ‘oc-
eration or cap stage
cluso’ defined as the relationship between all the com-
For example, twinning/conjoined teeth in-
ponents of the masticatory system in normal function,
cluding gemination, fusion, conation and con-
dysfunction and parafunction.
crescence
The various stages of occlusal development are
iii. Morphodifferentiation stage:
i. Predentate jaw relationship
l Anomalies of size and shape occur due to
ii. The deciduous dentition period
disturbances in morphodifferentiation stage.
iii. The mixed (transitional) dentition period
For example, dens in dente, dens evaginatus,
iv. The permanent dentition period
talon cusp, taurodontism and dilacerations
Histodifferentiation or bell stage: [SE Q.2]
l Anomalies of structure will occur in this stage.

For example, this includes amelogenesis imperfecta {Predentate period


AI type 1: hypoplastic and dentinogenesis imper- l This is the period soon after birth. During this the
fecta: shields type I, II and III neonate has no teeth but the relation of the gum
Apposition stage: pads is of equal importance. The alveolar process
l Anomalies of structure in enamel, dentin and cemen-
at the time of birth is called the gum pads.
l The gum pads are horseshoe shaped that are pink,
tum occur due to disturbances in apposition stage.
In enamel: AI types II and IV, hypo-maturation type, firm and covered with a layer of dense perios-
acquired enamel hypoplasia due to systemic and teum. They are divided into two parts labio-buccal
local causes. and lingual by dental groove.
l The gum pad is further divided into 10 segments
In dentin: Dentin dysplasia – two types (shields),
regional odontodysplasia – ‘ghost teeth’ and other by transverse groove; each segment has one de-
conditions with dentin abnormalities. veloping tooth sac.
378 Quick Review Series for BDS 4th Year, Vol 1

l A very important landmark in gum pads is lateral


a. Primate spaces:
sulcus, which is the transverse groove between
l Exist between the upper lateral incisors and the
canine and 1st molar. This is helpful in predicting
canines (present mesial to maxillary deciduous
interarch relation at a very early stage.
canines) and lower canines and first deciduous mo-
l The maxillary gum pad is wider and longer than
lars (present distal to mandibular deciduous canines).
the mandibular thus when they are approximated,
These spaces are also called anthropoid or simian spaces.
there is a complete overjet all around.
l The only contact that occurs is around the molar

region while space exists in anterior region. This b. Physiologic spaces:


is called infantile open bite, which is considered l They are present in between the primary

normal and helpful during suckling. teeth and play an important role in normal
l At birth the gum pads are not sufficiently wide to development of the permanent dentition.
accommodate teeth and there is relative crowding l The total space present may vary from 0 to

of developing tooth crypts.} 8 mm with the average 4 mm in the maxil-


lary arch and 1 to 7 mm with the average of
[SE Q.10] 3 mm in the mandibular arch.
{Primary or deciduous dentition period: Nonspaced dentition:
l Lack of space between primary teeth either
l The initiation of primary teeth occurs during first
six weeks of intrauterine life and the first primary due to small jaw or larger teeth. This type of
tooth erupts at the age of 6 months. dentition usually indicates to crowding in de-
l It takes around 2½ to 3½ years for all the primary
veloping permanent dentition.
teeth to establish their occlusion. Deep bite:
l This occurs during initial stages of develop-
l Some of the characteristic clinical features of

deciduous dentition according to Baume are as ment and is accentuated because the deciduous
follows: incisors are more upright than their successors.
i. Both the dental arches are half round or
ovoid in shape. {SN Q.14}
ii. Almost flat or no curve of Spee is present.
Terminal planes:
iii. Shallow cuspal interdigitation.
The mesio-distal relation between the distal surfaces
iv. Slight overjet.
of maxillary and mandibular 2nd deciduous molars is
v. Deep bite.
called terminal plane.
vi. Vertical inclination of the incisors (90°).
This is of three types:
vii. Spaced dentition.
a. Flush terminal plane (74%):
viii. Different maxillo-mandibular relations like
l The distal surfaces of the upper and
flush, mesial and distal terminal planes.}
lower teeth are in a straight plane (flush)
Spacing:
and therefore situated on the same verti-
l Delabarre in 1918 was the first to describe in-
cal plane.
terdental spacing in primary dentition
l It is usually most favourable relationship
l Baume in 1950 divided the primary dentition
to guide the permanent molars into class I.
into two types, i.e.
b. Mesial step terminal plane (14%):
I. Spaced dentition
l The distal surface of the lower molar is
II. Nonspaced dentition
more mesial to that of the upper molar.
l He/she also concluded that primary spacing oc-
Invariably, this guides the permanent
curs around 70% in maxilla and 63% in mandible.
molars into a class I relationship.
l Spaced dentition is supposed to be good, as
l However, a few can proceed into half
spaces in between the teeth can be utilized for
cusp class III during molar transition and
adjustment of permanent successors, which are
further into full class III relationship
always larger in size compared to the decidu-
with continued mandibular growth.
ous teeth.
c. Distal step terminal plane (10%):
l The distal surface of the lower molar is
{SN Q.2} more distal to that of the upper molar.
l The spaces present are of two types: l This relationship is unfavourable as it

a. Primate spaces guides the permanent molars into distal


b. Physiologic spaces occlusion.
Section | I  Topic-Wise Solved Questions of Previous Years 379

Anterior teeth relationship: Mixed dentition period:


a. Overbite: l The period during which both the primary and per-

l It is the distance, which the incisal edge of the manent teeth are present in the mouth together is
maxillary incisors overlaps vertically past the in- known as mixed dentition.
cisal edge of the mandibular incisors.
l The primary incisors erupt in a deep overbite
[SE Q.9]
which is corrected by eruption of posterior teeth
around 5 years of age. l { The permanent teeth erupting in place of previ-

l The average overbite in the primary dentition is ous deciduous teeth are the successional teeth,
2 mm. whereas those erupting posteriorly to the primary
b. Edge-to-edge bite: teeth are called the accessional teeth. This phase
l When the incisal edges of the two incisors are in begins at around 6 years with the eruption of
the same plane. This is also called a zero overbite. 1st permanent molars and lasts till about 12 years
l This is most common due to attrition, lengthening of age.
of ramus and downward-forward growth of l It can be divided as:

mandible. I. First transitional period:


c. Canine relationship: a. Emergence of the first permanent molars
l The relationship of the maxillary and mandibular b. Incisors transition
deciduous canines is one of the most stable in II. Intertransitional period:
primary dentition. III. Second transitional period:
Class I: l Emergence of cuspids, bicuspids and the

l The mandibular canine interdigitates in embra- second permanent molars


sure between the maxillary lateral incisor and l Establishment of occlusion

canine. I. First transitional period:


Class II: l This is characterized by emergence of 1st

l The mandibular canine interdigitates distal to permanent molars and exchange of decidu-
embrasure between the maxillary lateral inci- ous incisors with permanent incisors.
sor and canine. a. Emergence of 1st permanent molars:
Class III: l The antero-posterior relation between the

l The mandibular canine interdigitates in any two opposing first molars after eruption
other relation. depends on:
Arch dimensions: l Their positions previously occupied

These were first measured by Zsigmondy in 1890. within the jaws.


Frank and Baume later described the changes which can l The sagittal relation between the max-

take place in arch dimensions by loss of primary teeth illa and mandible.
and during the development of occlusion. l The occlusal relationship is estab-

Arch size: lished by the cone and funnel mecha-


l Size of the primary dental arch is the arch width nism with the upper palatal cusp
between primary canine and second molars. (cone) sliding into the lower occlusal
Arch length: fossa (funnel).
l Measured from the most labial surface of primary l The mandibular molars are the first to

central incisor to canine and to second primary erupt at around 6 years of age. Their
molars. position and relation is dependent on
Arch circumference: the relation of second deciduous mo-
l It is determined by measuring the length of curved lars as they are guided into dental
line passing over the incisal edges and buccal arch by the distal surfaces of these
cusps of teeth from the distal surfaces of primary teeth.}
second molar around the arch to the distal surface i. If the 2nd deciduous molar is in flush termi-
of second primary molar on the other side. nal plane:
Arch width: Then the erupting permanent molar will
l Bicanine or bimolar width is called the arch also be in the same relation. For this to
width. change into class I relation the molar has to
380 Quick Review Series for BDS 4th Year, Vol 1

move 2–3 mm in a forward direction, this is l Increase in intercanine arch length – This is
accomplished by: due to growth of jaws.
l Change in interincisal angulations – The angle

{SN Q.15} between the maxillary and mandibular incisors


is about 150° in primary dentition, whereas it
l Early mesial shift:
is about 123° in permanent dentition, thus al-
This occurs during early mixed dentition period. The
lowing more proclination and gaining space
eruptive forces of 1st permanent molars are strong
for incisor alignment. This is called incisor
enough to push the deciduous molars forward in the
liability.
arch thereby utilizing the primate spaces and thus
II. Intertransitional period:
establishing class I relationship.
l In this period the maxillary and mandibular arches
l Late mesial shift:
consist of permanent incisors and permanent mo-
Many children lack primate spaces and have a nons-
lars that sandwich the deciduous canines and
paced dentition and thus erupting permanent molars are
molars.
not able to establish class I relation even as they erupt.
l This phase lasts for 1½ years and is relatively
In these cases, the molars establish class I relation by
stable. Only a few changes in the morphology of
drifting mesially and utilizing the Leeway space after
deciduous teeth are seen because they undergo
exfoliation of deciduous molars and this is called late
attrition.
mesial shift.
III. Second transitional period:
l This phase is characterized by replacement of

deciduous molars and canines by premolars and


ii. If the 2nd deciduous molar is in mesial-step termi- permanent cuspids.
nal plane: l This takes place around 9–10 years of age and is
Then the erupting permanent molar will directly very critical for the alignment of the erupting per-
erupt in class I relation. But if further growth occurs manent teeth.
or if there is more utilization of spaces, the relation l The most common sequence of eruption of per-
can even change to class III. manent teeth in the maxilla is 4-3-5 (1st premolar-
iii. If the 2nd deciduous molar is in distal-step terminal canine-2nd premolar) and in the mandible 3-4-5
plane: (canine-1st premolar-2nd premolar).
Then the erupting permanent molar will erupt into l The combined mesio-distal width of permanent
class II relation. If further growth occurs or there is canine and premolars is less than that of decidu-
more utilization of spaces then it can lead into end ous canine and molars. This is called leeway
on molar relation. space of Nance or E space. It is 1.8 mm (0.9 mm
[SE Q.9] on each side) in maxillary arch and 3.4 mm (1.7
mm on each side) in mandibular arch.
{b. Exchange of incisors: l This excess space is utilized by mandibular mo-
l The deciduous incisors are replaced by permanent lars to establish class I relationship through late
incisors during this phase. mesial shift.}
l This period of transition is from 6½ to 8½ years. Broadbent phenomenon:
l The permanent incisors are larger as compared to their l Another common occurrence during this transitional
primary counterparts and thus require more space for period is the ugly Duckling Stage or Broadbent phe-
their alignment. nomenon. This self-correcting malocclusion is seen
l This difference between space available and space around 9–11 years of age or during eruption of canines
required is called the incisor liability. This is 7 mm and was first described by Broadbent in 1937.
for maxillary arch and 5 mm for mandibular arch. l As the permanent canines erupt they displace the roots
Some of the factors that help in alignment of incisors by of lateral incisors mesially. This force is transmitted to
gaining space are: the central incisors and their roots are also displaced
l Utilization of interdental spacing of primary inci- mesially. Thus the resultant force causes the distal di-
sors – Averages 4 mm in the maxillary arch and vergence of the crown in an opposite direction, leading
3 mm in the mandibular arch. to midline spacing.
l Increase in intercanine arch width – This occurs l The term ugly duckling stage indicates the unaesthetic
as the child grows. In males it is 6 mm for maxilla appearance of child during this stage.
and 4 mm for mandible whereas in females it is 4; l This condition corrects itself after the canines have
5 mm in maxilla and 4 mm in mandible. erupted. The canines after eruption apply pressure on
Section | I  Topic-Wise Solved Questions of Previous Years 381

the crowns of incisors thereby causing them to development and the one in which malocclusion is
shift back to original positions. most likely to develop.
l No orthodontic treatment should be attempted at l A long and valuable period of 3–7 years of quies-

this stage as there is danger of deflecting the cence follows before eruption of the lower 3rd mo-
canine from its normal path of eruption. lars to complete the dentition. The third molars do
Permanent dentition: not begin calcification until 9th year of age
l The entire permanent dentition is formed within and their eruption from the 16th year onwards her-
the jaws after birth except for the cusps of 1st alds the completion of dento-facial growth and
molar, which are formed before birth. development.
l The most frequent sequence of eruption for max-
Chronology primary dentition
illary arch is 6-1-2-4-5-3-7-8 (1st molar-central
incisor-lateral incisor-1st premolar-2nd premolar- Tooth Hard tissue Crown Eruption Root com-
canine-2nd molar-3rd molar) and in mandibular formation completed pleted
begins
arch is 6-1-2-3-4-5-7-8 (1st molar-central incisor-
lateral incisor-canine-1st premolar-2nd premolar-
2nd molar-3rd molar). Maxilla
Some changes that can be seen in permanent denti- Central in- 4 months 4 months 7½ months 1½ years
tion are cisor in utero
l Horizontal overbite decreases.
Lateral in- 4½ months 5 months 9 months 2 years
l Dental arches become shorter. cisor in utero
l Vertical overbite decreases up to the age of
Canine 5 months 9 months 18 months 3¼ years
18 years by 0.5 mm. in utero
l Overjet decreases by 0.7 mm between 12 and
1st molar 5 months 6 months 14 months 2½ years
20 years of age. in utero
Q.4. Write in detail about the eruption sequence in de- 2nd molar 6 months 11 months 24 months 3 years
ciduous dentition which deciduous and permanent teeth in utero
are present in child aged 10 years.
Ans. Mandible
Central in- 4½ months 4½ months 6 months 1½ years
[SE Q.4] cisor in utero

{Chronology of human dentition: Lateral in-


cisor
4½ months
in utero
4 months 7 months 1½ years

l The regular sequence of eruption suggests that it is


under genetic control, while the same is an event Canine 5 months 9 months 16 months 3 years
in utero
highly subject to nutritional, hormonal and disease
states. 1st molar 5 months 5½ months 12 months 2¼ years
l At birth jaws contain the partly calcified crowns of
in utero
20 deciduous teeth and beginning of calcification of 2nd molar 6 months 10 months 20 months }
3 years
the 1st permanent molars. in utero
l Eruption of deciduous dentition begins at an average

of 7½ months of age and terminates at about Chronology permanent dentition


29 months. Dental eruption is then quiescent for
Tooth Hard tissue Crown Eruption Root com-
nearly 4 years.
formation completed pleted
l At the age of 6 years, the jaws contain more teeth
begins
than at any other time; 48 teeth are filling the body of
mandible. After this extreme activity there is a 2½
Maxilla
years of quite period until 10½ years of age. Then
during the next 18 months the remaining 12 decidu- Central in- 3–4 4–5 years 7–8 years 10 years
cisor months
ous teeth are lost and 16 permanent teeth erupt.
l The 6 years of period of the mixed dentition from 6 Lateral in- 10–12 4–5 years 8–9 years 11 years
to 12 years in the most complicated period of dental cisor months
382 Quick Review Series for BDS 4th Year, Vol 1

Canine 4–5 6–7 years 11–12 13–15 measures such as passive space maintenance, active
months years years tooth guidance with space regaining or a combination
1st premo- 1½–1¾ 5–6 years 10–11 12–13
of both may be needed.
lar years years years l A study has shown that children who had premature
loss of one or more primary canines or molars were
2nd pre- 2–2 ¼ 6–7 years 10–12 12–14
molar years years years more likely to receive orthodontic treatment in the
permanent dentition with the need more than three
1st molar Birth 2½–3 6–7 years 9–10 years
times greater in children who had lost one or more
years
primary teeth through 9 years of age than the control
2nd molar 2½–3 7–8 years 12–15 14–16 group.
years years years
l Premature loss of primary molars was especially
3rd molar 7–9 years 12–16 17–24 18–25 associated with major malalignment of permanent
years years years teeth.
l No differences were observed in effects between loss
Mandible of first and second primary molars.
Central 3–4 4–5 years 6–7 years 9 years l Crowding of anterior teeth was directly affected by
incisor months the premature loss of primary canines.
Lateral 3–4 4–5 years 7–8 years 10 years
incisor months Q.6. What are the morphologic differences between
maxillary and mandibular primary teeth?
Canine 4–5 6–7 years 9–10 12–14
months years years Ans.
1st 1¾–2 5–6 years 10–11 12–13
The morphological differences between maxillary and
premolar years years years
mandibular primary teeth are as follows:
2nd 2¼–2 ½ 6–7 years 11–12 13–14 i. Maxillary central incisor:
premolar years years years
l The first notable difference between the maxillary
1st molar Birth 2½–3 years 6–7 years 9–10 years central incisor and its permanent successor is the fact
2nd molar 2½–3 7–8 years 11–13 14–15 that it has a mesio-distal measurement greater than
years years years the inciso-cervical measurement.
l The labial surface is slightly convex and relatively
3rd molar 8–10 years 12–16years 17–21 18–25
years years smooth, with little evidence of developmental lines
or grooves.
l The incisal edge joins the mesial surface at an
Q.5. What are the causes, sequelae and management of
early loss of primary teeth? acute angle and the distal surface at a more obtuse
angle.
Ans. l The lingual surface shows a well-developed cingu-

Early loss of teeth and space maintenance: lum and marginal ridges but developmental ana-
l If arch integrity is disrupted by early loss of primary tomic features such as pits and grooves are usually
teeth, problems may arise that affect the alignment of missing.
the permanent dentition. l The root of the maxillary central is conical and tapered

l Opposing teeth can supraerupt, more distal teeth can towards the apex.
drift and tip mesially and more forward teeth can ii. Mandibular central incisor:
drift and tip distally. l The mandibular central incisor is smaller in all di-

l Altered tooth positions may include a ‘symptomatic’ mensions than the maxillary central incisor.
space deficiency with loss of arch length and circum- l When viewed from the labial aspect, the tooth is

ference, blocked or deflected eruption of permanent symmetric with both the mesio- and distoincisal an-
teeth, unattractive appearance, food impaction areas, gles joining the incisal edge at almost right angles.
increased caries and periodontal disease and other l The incisal edge is usually perfectly straight in the

negative aspects of malocclusion. horizontal plane.


l The altered occlusal relationships may evidence trau- l The labial surface is less convex than that of the

matic interferences and untoward jaw relationships. maxillary central incisor, but it is also smooth with-
Management of early loss of primary teeth: out evidence of developmental anatomic landmarks.
l When early primary tooth loss occurs, to optimize the The lingual surface is usually smooth with a poorly
normal process of occlusion development corrective defined fossa and marginal ridges.
Section | I  Topic-Wise Solved Questions of Previous Years 383

l The root of the mandibular central incisor is long, l The proximal surfaces converge towards the lingual,
evenly tapered towards the apex and at times slightly creating a crown that is wider mesio-distally at the
compressed on its mesial and distal surfaces. buccal surface. The mesio-lingual cusp is the largest,
iii. Maxillary lateral incisor: followed by the mesio-buccal and the disto-buccal.
l The maxillary lateral incisor is essentially smaller l The mesio-buccal shows a greater mesio-distal devel-

in most dimensions than the central incisor. The opment than the disto-buccal cusp, occupying two-
disto-incisal angle is more rounded than the corre- thirds of the buccal surface. The mesio-buccal cusp is
sponding angle on the central incisor and the lingual also developed to a greater degree in an inciso-cervical
anatomy is usually less prominent. direction, creating an increased curvature in the cervi-
l The morphology of the root is essentially the same cal line in the mesial half of the crown.
as that of the central incisor, except that it is longer l A view of the crown from the mesial aspect shows

in proportion to the crown. the prominent bucco-cervical ridge which is charac-


iv. Mandibular lateral incisor: teristic of primary molars and, in particular, first
l The morphology of the mandibular lateral incisor is primary molars.
similar to that of the central incisor, except that the l The maxillary first molar has three long and slender

incisal edge slopes downward distally forming a roots. The lingual root is the longest, followed by
more obtuse disto-incisal angle. the mesio-buccal and the disto-buccal.
l The crown is also slightly larger inciso-cervically l All three roots extend from extremely short root

and mesio-distally than that of the central incisor. base in a divergent manner which is characteristic of
l The root is conical, longer than that of the central the primary molars.
incisors, and shows a definite distal inclination at its viii. Mandibular first molar:
apex. l The general outline of the crown of the mandibular

l The distal surface of the root will often show a lon- first primary molar when viewed from the occlusal
gitudinal depression or groove, separating the root aspect is rhomboid. There are usually two buccal
into labial and lingual moieties. and two lingual cusps.
v. Maxillary canine: l When viewed from the buccal, the greater mesio-

l It is larger than maxillary incisors in all dimensions. distal and inciso-cervical development of the mesio-
All surfaces of the crown are convex, creating a buccal cusp is immediately noticed.
more pronounced constriction at the cervix than is l A marked apical curvature of the cervical line and a

seen in the maxillary incisors. well-developed bucco-cervical ridge occur in tooth as


l It has a prominent cusp dividing the incisal aspect a characteristic of the mandibular first primary molar.
into a mesio-incisal and a disto-incisal edge, the l A distinguishing characteristic of this molar when

mesio-incisal edge being the longer of the two. viewed from the occlusal is the heavy transverse
l The lingual surface presents a prominent lingual ridge connecting the mesio-buccal and mesio-lingual
ridge, lingual fossae and marginal ridges. cusps.
l The root of the maxillary canine is long and tapered l There are generally three pits found on the occlusal
towards the apex, but shows a characteristic in- surface-central, mesial and distal, with the first the
crease in diameter just apical to the cervical line. most prominent of the three.
vi. Mandibular canine l The two roots mesial and distal show the typical flar-

l The mandibular canine appears more slender than ing characteristic of primary molars both, however,
the maxillary canine because of the smaller mesio- end in a sharp edge which may be slightly bifid.
distal diameter in relation to crown height. ix. Maxillary second molar:
l The relative lengths of the incisal edges are reversed l The morphology of the maxillary second molar is
in the mandibular canine, making the disto-incisal similar to that of the maxillary first permanent mo-
edge the longer of the two. lar, with a similar crown form, pit, groove and cus-
l The marginal ridges and cingulum are much less pal arrangement.
prominent, making the labio-lingual diameter l There are four major cusps. The largest is the me-

smaller than that of the maxillary canine. sio-lingual. The disto-lingual is the smallest, while
l The root is smoothly tapered from the cervical line the mesio-buccal and disto-buccal cusps are nearly
to the apex. equal in size.
vii. Maxillary first molar: l The occlusal surface shows three pits – distal, cen-

l The geometric form of the maxillary first molar tral and mesial which mark the intersection of the
when viewed from the occlusal aspect is triangular. developmental grooves.
384 Quick Review Series for BDS 4th Year, Vol 1

l The root morphology is similar to that of the maxil- l When the maxillary first permanent molar loses its op-
lary first permanent molar, except that the roots of ponent, it erupts at a faster rate than the adjacent teeth.
the second primary molar are thinner and diverge l The alveolar process is also carried along with the mo-

more from the root base. lars and causes problems when prosthetic replacements
x . Mandibular second molar: are needed.
l The mandibular second primary molar is a smaller l The treatment of patients with the loss of first perma-

replica of the mandibular first permanent molar. nent molars must be approached on an individual basis.
l There are three buccal cusps: the disto-buccal is the A superimposed existing malocclusion, abnormal mus-
largest, followed by the mesio-buccal and the distal. culature or the presence of deleterious oral habits can
There are two lingual cusps which are similar in size. affect the result, as in the case of the premature loss of
l There are three pits on the occlusal surface, the cen- primary molars.}
tral pit being the deepest and the distal and mesial l Loss of a first permanent molar before the eruption of

pits less prominent. the second permanent molar presents problems in both
l The crown morphology shows the typical cervical anteroposterior space control and vertical eruption con-
constriction and bucco-cervical ridge seen on the trol of opposing molars.
other primary molars. l Although it is possible to prevent overeruption of a

l As in the mandibular first primary molar, the two maxillary first permanent molar by placing a lower par-
roots of the mandibular second molar are narrow tial denture, there is no completely effective way to in-
mesio-distally. fluence the path of eruption of the developing second
permanent molar other than the use of an acrylic distal
Q.7. Discuss the various treatment modalities in case of shoe extension on a partial denture.
a premature loss of first permanent molar in a develop- l The second molar drifts mesially before eruption when
ing occlusion. the first permanent molar has been extracted. Reposi-
Ans. tioning this tooth orthodontically is possible after its
eruption.
[SE Q.6] l However, the child must then be considered for pro-

l {The first permanent molar is unquestionably the most longed space maintenance until the time when a more
important unit of mastication and is essential in the de- permanent tooth replacement can be inserted. The re-
velopment of functionally desirable occlusion. moval of the opposing first permanent molar, even when
l A carious lesion may develop rapidly in the first perma- the tooth appears to be sound and caries free, is some-
nent molar and occasionally progress from an incipient times recommended in preference to allowing it to ex-
lesion to a pulp exposure in a 6-month period. trude or to subjecting the child to prolonged space
l The loss of a first permanent molar in a child can lead maintenance and eventual fixed replacement.
l If the first permanent molars are removed several years
to changes in the dental arches that can be traced
throughout the life of that person. before eruption of the second permanent molars, there
l Unless appropriate corrective measures are instituted, is an excellent chance that the second molars will erupt
these changes include diminished local function, drift- in an acceptable position. However, the axial inclination
ing of teeth and continued eruption of opposing teeth. of the second molars, particularly in the lower arch, may
l The second molars, even if unerupted, start to drift me- be greater than normal.
l The decision whether to allow the second molar to drift
sially after the loss of the first permanent molar. A
greater degree of forward bodily movement will occur mesially or to guide it forward in an upright position
with loss of the first permanent molar in children in the may be influenced by the presence of a third molar of
8–12 year age group. normal size.
l If there is a question regarding the favourable develop-
l In older children, if the loss occurs after eruption of the
second permanent molar, more exaggerated mesial tip- ment of a third molar on the affected side, repositioning
ping of the second molar can be the expected outcome. the drifted second molar and holding space for a re-
Although the premolars undergo the greatest amount of placement prosthesis is usually the treatment of choice.
l When the first permanent molar is lost after the eruption
distal drifting, all the teeth anterior to the space, includ-
ing the central and lateral incisors on the side where the of the second permanent molar, orthodontic evaluation is
loss occurred, may show evidence of movement. indicated, and the following points should be considered:
l Is there any need of corrective treatment other than
l Contacts open and the premolars, in particular, rotate as
they fall distally. There is a tendency for the maxillary in the first permanent molar area?
l Should the space be maintained for a replacement
premolars to move distally in unison, whereas those in
the lower arch may move separately. prosthesis?
Section | I  Topic-Wise Solved Questions of Previous Years 385

l Should the second molar be moved forward into the Q.14. Discuss in brief the classification of occlusal relation-
area formerly occupied by the first molar? ship in primary dentition, its variation and disharmony.
l The latter choice is often the more satisfactory, even
Ans.
though there will be a difference in the number of mo-
lars in the opposing arch. A third molar can often be [Same as LE Q.3]
removed to compensate for the difference. Without
Q.15. Define growth and development. Discuss the de-
treatment the second molar will tip forward within a
velopment of normal occlusion from gum pad relation-
matter of weeks.
ship till the eruption of 2nd permanent molar.
l Another option to consider is autotransplantation of a

third molar into the first molar position. According to Ans.


Bauss and colleagues, autotransplantation has become a
[Same as LE Q.3]
well-established treatment modality in cases of early
tooth loss or aplasia. Q.16. Discuss the development of dentition from 6 to
10 years.
Q.8. Discuss the morphological and histological differ-
Ans.
ences of primary and permanent teeth and its signifi-
cance. [Same as LE Q.4]
Ans.
[Same as LE Q.1]
SHORT ESSAYS:
Q.1. Explain local and systemic factors for delayed
Q.9. What are the morphologic differences between pri-
eruption of teeth.
mary and permanent teeth?
Ans.
Ans.
The various local and systemic factors that influence the
[Same as LE Q.1]
eruption of teeth are as follows:
Local factors:
Q.10. Enumerate the different stages of tooth develop-
l Lack of space in the arch
ment. Discuss various developmental abnormalities and
l Early loss of primary tooth
disturbances of teeth and other oral structure during
l Ankylosed primary teeth, retained roots
these developmental stages.
l Supernumerary teeth
Ans. l Cysts/tumours of jaws

l Abnormal musculature
[Same as LE Q.2]
Systemic factors:
Acceleration of eruption
Q.11. Describe the development of occlusion from de-
l Hyperthyroidism
ciduous to permanent stages.
l Hyperpituitarism
Ans. l Turner syndrome

Delayed eruption
[Same as LE Q.3]
l Hypopituitarism

l Hypothyroidism
Q.12. Discuss the development of normal occlusion from
l Down syndrome
gum pad relationship till the eruption of 2nd permanent
l Cleidocranial dysostosis
molar.
l Hypovitaminosis A and D

Ans.
Q.2. Predentate period.
[Same as LE Q.3]
Ans.
Q.13. What is transitional period. Describe the changes [Ref LE Q.3]
seen in occlusion in this stage?
Q.3. What is teething disorder? Explain the manage-
Ans. ment.
[Same as LE Q.3] Ans.
386 Quick Review Series for BDS 4th Year, Vol 1

l Alternative nonpharmacological holistic therapies


{SN Q.1}
(acupressure, aromatherapy, massage and homeopa-
The term ‘teething’ literally means, ‘eruption of primary thy) have been suggested as giving relief from the
teeth’. symptoms of teething.
The various clinical features of teething disorder are as l Nonirritating topical anaesthetics, some systemic

follows: analgesics and antipyretics can be advised to give


l Increased salivation, putting fingers in the mouth, relief from pain and fever.
diarrhoea, fever, convulsions, acute herpetic gin- l The child may be referred to paediatrician or family

givo-stomatitis, photophobia, blinking eyes, vom- physician in case of any systemic disturbances and
iting, neuralgia, severe headache, cold, weight fever with temperature more than 101°F.
loss, toxemia, tonsillitis, paralysis, cholera, men-
Q.4. Chronology of human primary teeth.
ingitis, tetanus and even death have been attrib-
uted to eruption. Ans.
l In 19th century, infant mortality has been attrib-
[Ref LE Q.4]
uted to teething. It was proved that no evidence
exists of teething causing fever, convulsions, Q.5. Transient malocclusion.
bronchitis or diarrhoea and all other conditions
Ans.
mentioned.
l Teething neither increases the incidence of infec- l Anomalies, which arise in the child’s developing denti-
tion nor the erythrocyte sedimentation rate (ESR) tion during the period of transition from predentate pe-
or white blood cell (WBC) counts. It does not riod to permanent dentition period and get corrected on
cause fever. Any fever that shows more than their own without any dental, treatment are known as
101°F temperature is not attributed to teething. self-correcting anomalies.
l It does cause day-time restlessness, increase in Period of development – self-correcting anomaly
amount of finger sucking, drooling or rubbing of During predentate period:
gums and possibly loss of appetite. One-third of l Retrognathic mandible

children only show pronounced change in mucosa l Anterior open bite

with small haemorrhage. l Infantile swallow

l Eruption of teeth is a normal physiological pro- During deciduous dentition:


cess, the fever or other infections are considered l Deep bite

coincidental. l Decrease overjet

l Flush terminal plane

l Primate and physiologic spacing


l Inflammation of tissues over erupting teeth results in During mixed dentition period:
pain temporarily. l Anterior deep bite
Management: l Mandibular anterior crowding
l Removal of tissue covering over the erupting teeth is
l End-on molar relation
not recommended. l Ugly duckling stage
l Nontoxic unbreakable teething toys or rings are

advised. Q.6. Young permanent first molar tooth.


l Solid silicone-based teething rings are superior to
Ans.
their liquid-filled counterparts, as the potentially ir-
ritant contents may leak, if they are damaged and [Ref LE Q.7]
they cannot be sterilized. Q.7. Explain morphological differences between pri-
l Temporary pain relief is provided by the pressure
mary and permanent teeth.
produced by chewing the teething ring, preferably
chilled one. Ans.
l Pacifiers can also be used for this purpose. Teething
[Ref LE Q.1]
rings should be attached to the infants clothing and
not tied around the neck, as strangulation could result. Q.8. Importance of deciduous dentition.
l Hard, nonsweetened rusks, made from flour and
Ans.
wheat germ with no sugar or sweetener can also be
attached on to the infant’s clothing. [Ref LE Q.1]
Section | I  Topic-Wise Solved Questions of Previous Years 387

Q.9. Development of occlusion from 6 to 12 years. l The gum pads are horseshoe shaped and are pink, firm
and covered with a layer of dense periosteum.
Ans. l They are divided into two parts labio-buccal and lingual

[Ref LE Q.3] by dental groove.


l The gum pad is further divided into 10 segments by
Q.10. Characteristics of primary dentition. transverse groove; each segment has one developing
Ans. tooth sac.
l A very important landmark in gum pads is lateral sul-
[Ref LE Q.3] cus, which is the transverse groove between canine and
Q.11. Local and systemic causes of delayed eruption. 1st molar. This is helpful in predicting interarch relation
at a very early stage.
Ans. l The maxillary gum pad is wider and longer than the

[Same as SE Q.1] mandibular; thus when they are approximated, there is a


complete overjet all around.
Q.12. Sequence and time of eruption of primary teeth. l The only contact that occurs is around the molar region

Ans. while space exists in anterior region. This is called in-


fantile open bite, which is considered normal and help-
[Same as SE Q.4] ful during suckling.
Q.13. Self-correcting anomalies. Q.4. Eruption cyst.
Ans. Ans.
[Same as SE Q.5] l Eruption cyst is a bluish purple, elevated area of tissue,
Q.14. Importance of young permanent tooth. commonly called eruption hematoma, occasionally de-
velops few weeks before the eruption of primary or
Ans. permanent tooth.
[Same as SE Q.6] l The blood filled cyst is most frequently seen in the pri-

mary second molar or the first permanent molar regions.


Q.15. Importance of first permanent molar. l This fact substantiates the belief that the condition de-

Ans. velops as a result of trauma to the soft tissue during


function and usually within a few days the tooth breaks
[Same as SE Q.6]
through the tissue, and the hematoma subsides.
Q.16. What is transitional period? Describe the changes l Because the condition is almost always self-limited,

seen in occlusion at the stage. treatment may occasionally be justified.


Ans. Q.5. Eruption sequestrum.
[Same as SE Q.9] Ans.
l The eruption sequestrum is seen occasionally in chil-
SHORT NOTES: dren at the time of the eruption of the first permanent
molar.
Q.1. Teething. l An eruption sequestrum is composed of cementum like

Ans. material formed within the dental follicle.


l Regardless of its origin, the hard tissue fragment is gen-

[Ref SE Q.3] erally overlying the central fossa of the associated tooth
Q.2. Primate spaces. embedded and contoured within the soft tissue.
l As the tooth erupts and the cusps emerge the fragment
Ans. sequestrates.
l Eruption sequestra are usually of little or no clinical
[Ref LE Q.3]
significance as it may spontaneously resolve without
Q.3. Gum pads. noticeable symptoms. In cases where eruption seques-
Ans. trum is causing local irritation and has surfaced through
the mucosa it may easily be removed.
l During predentate period, i.e. the period soon after
birth, the neonate has no teeth. The alveolar processes at Q.6. Chronology of deciduous dentition.
the time of birth are called the gum pads. Ans.
388 Quick Review Series for BDS 4th Year, Vol 1

The chronology of primary dentition is as follows: Q.10. Incisal liability.


Maxilla Eruption Ans.
Central incisor 7½ months
It refers to the difference between the amount of space
Lateral incisor 9 months needed for the permanent incisors and the amount of space
Canine 18 months available for them. This result in crowding of permanent
incisors in the mandibular arch (sometimes in maxillary
1st molar 14 months
arch too) at age 8–9. There are different mechanisms by
2nd molar 24 months which this incisal liability gets compensated. They are
l Utilization of interdental spacing or the spaces between

Mandible Eruption the primary incisors and canines in spaced dentition.


l Increase in the arch width or intercanine width changes
Central incisor 6 months
leading to more outward positioning of canines after
Lateral incisor 7 months eruption. This accounts for a space of about 2 mm.
Canine 16 months l Arch length changes: labial inclination and positioning

1st molar 12 months


of permanent incisors in a larger arc compared to up-
right primary incisors. This contributes to additional
2nd molar 20 months 1–2 mm space.
l Repositioning of the canines in the mandibular arch
Q.7. Mention different stages of tooth development. slightly back into the primate spaces along with the wid-
Ans. ening of arch accounting for another extra-millimetre of
space.
[Ref LE Q.2]
Q.11. Mulberry molars.
Q.8. Mamelons.
Ans.
Ans.
l Mulberry molars occur due to enamel hypoplasia of
l Mamelons are developmental grooves present on the congenital syphilis.
incisal edges of a newly erupted incisor. l It is one of the symptoms of Hutchinson triad for the
l These are seen in permanent teeth because the enamel
diagnosis of congenital syphilis.
formation in permanent teeth occurs in lobes. l These molars show many small globular malformations
l These lobes fuse together to form the labial and lingual
on the occlusal surface rather than the normal cusps and
surfaces of teeth. The lines of fusion are seen as grooves groove patterns.
on the incisal edge of newly erupted incisor which are
called mamelons. Q.12. Leeway space.
l These grooves wear off as the age advances; hence,
Ans.
presence of mamelons is an indication that they are
young permanent incisors. l The combined mesio-distal width of permanent canine
l The mamelons are not seen in primary incisors as the and premolars is less than that of deciduous canine
enamel formation takes place from a single lobe. and molars. This is called Leeway space of Nance or
E space.
Q.9. Causes of precocious eruption.
l It is 1.8 mm (0.9 mm on each side) in maxillary arch

Ans. and 3.4 mm (1.7 mm on each side) in mandibular arch.


l This excess space is utilized by mandibular molars to
Aetiology of precocious eruption is unknown, but superfi-
establish class I relationship through late mesial shift.
cially positioned tooth bud may be a reason and most of
these teeth are poorly formed. Q.13. Ugly duckling stage.
The natal and neonatal teeth are also termed as prema-
ture teeth or predeciduous dentition. These teeth erupt prior Ans.
to 3 months of age. Natal teeth are present at birth and neo- l Another common occurrence during the second transi-
natal teeth erupt within the first 30 days of life. tional period is the ugly duckling stage or Broadbent
Natal and neonatal teeth may be associated with Riga– phenomenon.
Fede disease in which trauma to tongue during feeding is l This self-correcting malocclusion is seen around 9–11
observed. These may also be observed in chondro-ectodermal years of age or during eruption of canines and was first
dysplasia. described by Broadbent in 1937.
Section | I  Topic-Wise Solved Questions of Previous Years 389

l As the permanent canines erupt they displace the roots teeth, respectively. The teeth are numbered in each
of lateral incisors mesially. This force is transmitted to quadrant from 1 to 8 for permanent teeth and 1 to 5 for
the central incisors and their roots are also displaced primary teeth starting from the central incisor towards
mesially. Thus the resultant force causes the distal di- the distal.
vergence of the crown in an opposite direction, leading For example:
to midline spacing. In the number 14, 1 denotes maxillary right quadrant,
l The term ugly duckling stage indicates the unaesthetic and 4 denotes the fourth tooth in the arch from midline,
appearance of child during this stage. i.e. first premolar and it is pronounced as one-four.
l This condition corrects itself after the canines have In the expression 65 in primary teeth: 6 denotes max-
erupted. No orthodontic treatment should be attempted illary left quadrant and 5 denotes the fifth (last) tooth
at this stage. in the quadrant from midline, i.e. second primary
molar and it is pronounced as six-five.
Q.14. Primary molar relationship.
The permanent teeth can be identified and charted as:
Ans.
18 17 16 15 14 13 1211 21 22 23 24 25 26 27 28
[Ref LE Q.3] 48 47 46 45 44 43 4241 31 32 33 34 35 36 37 38
Q.15. Early and late mesial shift.
The primary teeth can be identified and charted as:
Ans.
55 54 53 52 51 61 62 63 64 65
[Ref LE Q.3]
85 84 83 82 81 71 72 73 74 75
Q.16. Young permanent first molar.
Ans. Q.18. Skeletal age versus dental age.
l The first permanent molar is unquestionably the most Ans.
important unit of mastication and is essential in the de-
Skeletal age:
velopment of functionally desirable occlusion.
l Skeletal age assessment is often made with the help
l The loss of a first permanent molar in a child can lead
of hand-wrist radiograph which can be considered
to changes in the dental arches that can be traced
the ‘biological clock’.
throughout the life of that person.
l The ossification events are localized in the area of the
l Unless appropriate corrective measures are instituted,
phalanges, carpal bones and radius.
these changes include diminished local function, drift-
l Leonard S. Fishman (1982) outlined four stages of
ing of teeth and continued eruption of opposing teeth.
bone maturation found at six anatomical sites located
l In older children, if the loss occurs after eruption of the
on the thumb, 3rd finger, 5th finger and radius.
second permanent molar, more exaggerated mesial tip-
Eleven skeletal maturity indicators are found in these
ping of the second molar can be the expected outcome.
six anatomic sites.
l All the teeth anterior to the space, including the central
Dental age:
and lateral incisors on the side where the loss occurred,
l Dental age has been based on two different methods
may show evidence of movement.
of assessment:
l Contacts open and the premolars, in particular, rotate as
i. Tooth eruption age
they fall distally.
ii. Tooth mineralization stage
l When the maxillary first permanent molar loses its op-

ponent, it erupts at a faster rate than the adjacent teeth. Q.19. Natal and neonatal teeth.
l The treatment of patients with the loss of first perma-

nent molars must be approached on an individual basis. Ans.


Q.17. FDI tooth numbering system. l The natal and neonatal teeth are also termed as prema-
ture teeth or predeciduous dentition. These teeth erupt
Ans.
prior to 3 months of age.
l FDI tooth numbering system was proposed by Federa- l Natal teeth are present at birth and neonatal teeth erupt

tion Dentaire Internationale (FDI). within the first 30 days of life.


l This is a two digit numbering system. l These teeth commonly occur in pairs, mostly in the

The first digit represents the quadrant the second digit lower anterior region.
represents the individual tooth. l The natal to neonatal teeth ratio is 3:1 with the inci-

l The quadrants are denoted with the numbers 1, 2, 3 and dence as 1 in 2000–3500. Ninety per cent of these teeth
4 for permanent teeth and 5, 6, 7 and 8 for primary are true primary teeth, others are supernumerary.
390 Quick Review Series for BDS 4th Year, Vol 1

Q.20. Pulpal differences in primary and permanent Chronological age:


teeth l It is the most commonly and easily determined de-

velopmental age parameter which is simply figured


Ans.
out from the child’s date of birth.
l It is neither an accurate indicator of stage of de-
Primary teeth Permanent teeth
velopment nor it is a good predictor of growth
First molar is smaller in dimension First molar is larger in
potential.
than 2nd molar. dimension than 2nd
l Chronological age is often not sufficient for assess-
Roots are larger and more slender. molar.
Roots are shorter and ing the developmental stage and somatic maturity of
more bulbous. the patient.
Pulpal outline follows the DEJ more Pulp outline follows Q.23. Neonatal line.
closely; the pulp horns are closer to the DEJ less closely.
the outer surface; the pulp horns are Ans.
high, and the pulp chambers are large.
l In the deciduous teeth and in the first permanent mo-
High potential for repair. Comparatively less
potential for repair.
lars, where dentin is formed partly before and partly
after birth, the prenatal and postnatal dentine are
Root canals are more ribbon-like; the Root canals are well separated by an accentuated contour line. This is
radicular pulp follows a thin, tortuous defined with less
and branching path. branching.
termed the neonatal line and is seen in enamel as well
as dentin.
Enamel and dentin are less mineral- They are more mineral-
l This line reflects the abrupt change in environment
ized; the dentin thickness between ized.
the pulp chambers and the enamel is that occurs at birth. The dentin matrix formed prior to
limited, particularly in some areas birth is usually of better quality than that formed after
(lower second primary molar). birth, and the neonatal line may be a zone of hypocal-
Neonatal lines are present; secondary Neonatal line is seen cification.
cementum is absent. only in first molar; sec-
Q.24. Features of primary dentition.
ondary cementum is
present. Ans.
Accessory canals in the floor of pulp Floor of the pulp
Some of the characteristic clinical features of deciduous
chamber leads directly into inter- chamber does not have
radicular furcation. any accessory canals. dentition are
i. Both the dental arches are half round in shape or ovoid.
ii. Almost no curve of Spee is present.
Q.21. Define eruption.
iii. Shallow cuspal interdigitation.
Ans
iv. Slight overjet.
l Maury Massier and Schour (1941) defined eruption as a
v. Deep bite.
process whereby the forming tooth migrates from its
vi. Vertical inclination of the incisors.
intraosseous location in the jaw to its functional position
vii. Spaced dentition.
within the oral cavity.
viii. Different maxillo-mandibular relations like flush,
or
mesial and distal terminal planes.
l James K. Avery defined eruption as the movement of the

teeth through the bone of the jaws and the overlying Q.25. Teeth present in the jaws at birth.
mucosa to appear and function in the oral cavity.
Ans.
or
l The term eruption has been used to denote the tooth l At birth jaws contain the partly calcified crowns of 20
emerging through the gingiva, but then it became more deciduous teeth and beginning of calcification of the 1st
completely defined to mean, continuous tooth move- permanent molars.
ment from the dental bud to occlusal contact.
Q.26. Ectopic eruption.
Q.22. Explain dental age and chronological age.
Ans.
Ans.
l Arch length inadequacy or a variety of local factors may
Dental age: influence a tooth to erupt in a position other than nor-
Dental age has been based on two different methods of mal, it is known as ectopic eruption.
assessment:
Q.27. Eruption sequence.
i. Tooth eruption age
ii. Tooth mineralization stage Ans.
Section | I  Topic-Wise Solved Questions of Previous Years 391

l The sequence of eruption of teeth can vary with indi- Q.30. Leeway space of Nance.
viduals.
Ans.
l The favourable sequence observed for permanent teeth

in the maxilla is as follows: [Same as SN Q.12]


6, 1, 2, 4, 5, 3, 7, 8 (first molar, central incisor, lateral
Q.31. Terminal plane relationship.
incisor, first premolar, second premolar, canine, second
molar and third molar, respectively). Ans.
l The favourable sequence observed for permanent teeth
[Same as SN Q.14]
in the mandible is as follows:
6, 1, 2, 3, 4, 5, 7, 8 (first molar, central incisor, lateral Q.32. Flush terminal plane.
incisor, canine, first premolar, second premolar, second
Ans.
molar and third molar, respectively).
l In primary teeth, the normal eruption sequence ob- [Same as SN Q.14]
served in both maxilla and mandible is A, B, D, C, E
Q.33. Late mesial shift.
(central incisor, lateral incisor, first primary molar, ca-
nine and second primary molar, respectively). Ans.
Q.28. Teething disorder. [Same as SN Q.15]
Ans. Q.34. Give the importance of 1st permanent molar.
[Same as SN Q.1] Ans.
Q.29. Chronology of eruption of primary teeth. [Same as SN Q.16]
Ans.
[Same as SN Q.6]

Topic 12
Acquired and Developmental Disturbances
of the Teeth and Associated Oral Structures
COMMONLY ASKED QUESTIONS
LONG ESSAYS:
1 . Write in detail about the developmental anomalies of shape of teeth in children.
2. Describe the various causes of enamel hypoplasia.

SHORT ESSAYS:
1. Submerged teeth.
2. Enamel hypoplasia. [Ref LE Q.2]
3. Turner (hypoplasia) tooth.
4. Germination and fusion. [Ref LE Q.1]
5. Amelogenesis imperfecta.
6. Anomalies of the number of teeth.
7. Pink tooth.
8. Supernumerary teeth.
9. Tooth discolouration (intrinsic).
10. Intrinsic discolouration of teeth. [Same as SE Q.9]
392 Quick Review Series for BDS 4th Year, Vol 1

SHORT NOTES:
1. Mesiodens.
2. Twinning.
3. Anodontia.
4. Dilaceration. [Ref LE Q.1]
5. Talon cusps. [Ref LE Q.1]
6. Taurodontism.
7. Ankyloglossia.
8. Dens in dente.
9. Ankylosed teeth.
10. Partial anodontia. [Ref SE Q.6]
11. Supernumerary teeth. [Ref SE Q.8]
12. Bohn nodules.
13. Turner hypoplasia.
14. Hyspoplasia due to nutritional deficiency.
15. Internal resorption. [Ref SE Q.7]
16. Eruption cyst.
17. Epstein pearls.
18. Riga–Fede disease.
19. Acrodynia.
20. Mucocoele.
21. Dental effects of bobby pin opening.
22. Germination and fusion.
23. Dentinogenesis imperfecta.
24. Natal and neonatal teeth.
25. Tongue tie. [Same as SN Q.7]
26. Turner tooth. [Same as SN Q.13]
27. Pink tooth. [Same as SN Q.15]

SOLVED ANSWERS
LONG ESSAYS:
Q.1. Write in detail about the developmental anomalies [SE Q.4]
of shape of teeth in children.
{i. a.  Germination:
Ans. l Germination is a developmental anomaly which re-
fers to division of single tooth germ into incomplete
The developmental disturbances affecting shape of teeth
or complete formation of two teeth.
are as follows:
l A geminated tooth represents an attempted division
i. Germination, fusion and concrescence
of a single tooth germ by invagination occurring dur-
ii. Accessory cusps
ing the proliferation stage of the growth cycle of the
l Cusp of Carabelli
tooth.
l Talon cusp
Aetiology:
l Dens invaginatus
Division of single tooth bud.
l Dens evaginatus
Clinical features:
iii. Ectopic enamel
l Bifid crown on a single root.
a. Enamel pearls
l Crowns may be partially or totally separated from
b. Cervical enamel extensions
each other.
iv. Taurodontism
l Roots are fused and single root canal is present
v. Dilaceration
within the root.
vi. Supernumerary roots
Section | I  Topic-Wise Solved Questions of Previous Years 393

l The structure is usually one with two completely the case, and a post and core and a crown restora-
or incompletely separated crowns that have a tion were anticipated for the future.}
single root and a root canal. i. c.  Concrescence:
l The condition is seen in both deciduous and per- l Concrescence is defined as union of two adjacent

manent dentition, with a higher frequency in the teeth by cementum only without confluence of the
anterior and maxillary region. underlying dentin.
The treatment: l It is the type of fusion which is limited only to the

l The treatment of a permanent anterior geminated roots of teeth and it occurs after the root formation of
tooth may involve reduction of the mesiodistal involved teeth is completed.
width of the tooth to allow normal development of l Concrescence may be developmental or postinflam-

the occlusion. matory or due to traumatic injury.


l Devitalization of the tooth and root canal therapy l Commonly seen between maxillary 2nd molar and

followed by the construction of a postcrown may unerupted 3rd molar.


be needed when the geminated tooth is large and ii. Accessory cusps:
malformed. a. Cusp of Carabelli
i. b.  Fusion: b. Talon cusp
l Fusion represents the union of two independently c. Dens invaginatus (dens in dente)
developing primary or permanent teeth. d. Dens evaginatus
l The condition is almost always limited to the anterior a. Cusp of Carabelli:
teeth and, like germination, may show a familial l Present on mesiopalatal cusp of maxillary 1st

tendency. molars.
Aetiology: l An analogous accessory cusp in seen occasionally

l Incomplete attempt of two tooth buds to fuse into on the mesiobuccal cusp of a mandibular perma-
one. nent or deciduous molar known as protostylid.
l Physical tear and premature union of two tooth

buds or two developing teeth.


Clinical features: {SN Q.5}
l One of the most important criteria for fusion is the b. Talon cusp:
fused tooth must exhibit confluent dentin. l Talon cusp is an anomalous projection resembling
l Two separate roots 1 root canals – (complete or eagle’s talon, projects lingually from cingulum
incomplete). area of permanent incisors.
l Both permanent and deciduous dentition are af- l A developmental groove is present at the site,
fected in case of fusion, although it is more com- where this projection meets with the lingual sur-
mon in deciduous teeth. face of tooth.
l Fusion can be complete or incomplete and its l This groove is prone to caries, so it should be re-
extent will depend on stage of odontogenesis at moved. If pulp exposure is present then endodon-
which fusion takes place. tic therapy is done.
l The incisor teeth are more frequently affected in l Found in association with ‘Rubinstein Taybi
both the dentitions. syndrome’.
l A frequent finding in fusion of primary teeth is

the congenital absence of one of the correspond-


ing permanent teeth.
Radiographic examination: c. Dens in dente or dens invaginatus:
l The radiograph may show that the fusion is lim- l Dens in dente is a developmental variation which

ited to the crowns and roots. Fused teeth will have arises as a result of enamel epithelial invagination
separate pulp chambers and separate pulp canals. of the crown surface before calcification.
Treatment: l Several causes of this condition: It is because of

l Surgical division and selective shaping of crowns. focal growth proliferation and focal growth retar-
l The multidisciplinary approach may be indicated dation that takes place in certain areas of tooth
in the clinical management of certain problems bud, increased localized external pressure.
associated with fused teeth. l After calcification it appears as accentuation of

l The disciplines of paediatric dentistry, endodon- lingual pit.


tics, surgery, restorative dentistry and orthodon- l Teeth most frequently involved are maxillary lat-

tics were represented in the initial management of eral and maxillary central incisors.
394 Quick Review Series for BDS 4th Year, Vol 1

Radiographic features: l Cervical enamel extension, also occurs along the


l Appearance of tooth within tooth due to deep surface of dental roots.
pear-shaped invagination from lingual pits, l Maxillary and mandibular molars are most com-

approximating to pulp. monly affected.


Treatment: l Predisposes to development of buccal bifurcation

l Application of sealant or a restoration in the cysts.


opening of the invagination is the recom- iv. Taurodontism (bull-like teeth):
mended treatment to prevent pulpal involve- l Taurodontism is a dental anomaly in which the body

ment. of the tooth is enlarged at the expense of roots.


l If the condition is detected before complete Aetiology:
eruption of the tooth, the removal of gingival l A specialized or retrograde character or a mutation

tissue to facilitate cavity preparation and resto- l A primitive pattern and an atavistic feature

ration may be indicated. l Mendalian recessive trait

l This anomaly makes teeth prone to caries so l Associated with Klinefelter syndrome

endodontic therapy should be done. l Due to failure of Hertwig epithelial root sheath to

l The advisability of performing endodontic invaginate at proper horizontal level


procedures on such a tooth with pulpal degen- Clinical findings:
eration depends on its pulp morphology and l It may affect both deciduous and permanent den-

the restorability of the crown. tition, but more common in permanent dentition.
d. Dens evaginatus (occlusal tuberculated premolar, l Molars are commonly affected.

Leong premolar, evaginated odontome) l Tooth morphology is normal.

l Dens evaginatus is a developmental condition Radiographic features:


which appears as an accessory cusp or globule l Enlarged and rectangular pulp chamber is present.

of enamel on occlusal surface between buccal l No constriction of pulp at cervical area.

and lingual cusps of premolars unilaterally or l Roots are very short.

bilaterally. l Furcation is present just above root apex.

l This is opposite of invagination. That means there Treatment:


occurs extrusion of the dental papilla outwards No treatment is required.
into the enamel organ.
Clinical findings: {SN Q.4}
l This condition is more common in people of
v. Dilaceration:
Chinese race.
l Dilaceration refers to angulation or curve in root
l More common in maxillary 1st premolars
or crown of tooth.
but also occurs rarely on molars, cuspids and
l Angulation is caused due to trauma to the tooth
incisors.
during formative stage of tooth.
l Presents a tubercle of enamel with a core of
l Curve is present at apical, middle or at cervical
dentine with a narrow pulp chamber.
portion depending on the portion which is form-
l When the tooth erupts, this bit of enamel is
ing at the time of trauma.
higher than the cusps, and covers the underly-
l Occlusal trauma in deciduous tooth may also
ing mass of dentine.
cause dilaceration of permanent tooth.
l If present in deciduous teeth, it causes diffi-
l More common in the maxillary anterior region.
culty in feeding.
l Significance is that the tooth with bent root is dif-
l When the thin surface enamel of the tubercle
ficult to extract.
breaks down, infection of the tooth takes
place resulting in death of the pulp and ab-
scess formation. vi. Supernumerary roots:
Treatment: l One or more extra roots may be present in tooth.

l It consists of extraction of the tooth. l Usually single-rooted teeth such as mandibular cus-

iii. Ectopic enamel pids and bicuspids are involved.


l Enamel pearls l Third molars of both jaws also present one or more

l Cervical enamel extensions extra roots.


l Ectopic enamel or enamel pearls or enameloma
Q.2. Describe the various causes of enamel hypoplasia.
or enamel drop usually occurs in furcation area
below the crest of gingiva. Ans.
Section | I  Topic-Wise Solved Questions of Previous Years 395

[SE Q.2] l Birth injuries


Ingestion of fluoride
l {Enamel hypoplasia is defined as an incomplete or de- l

l Local infection
fective formation of the organic enamel matrix of teeth.
l Idiopathic
l Amelogenesis occurs in two stages. In the first stage, the
Nutritional deficiencies and exanthematous diseases:
enamel matrix forms, and in the second stage, the ma-
l Vitamins A, C and D, calcium, and phosphorus
trix undergoes calcification.
deficiencies and exanthematous diseases
l Local or systemic factors that interfere with normal
(e.g. chickenpox, measles) and scarlet fever cause
matrix formation cause enamel surface defects and ir-
enamel hypoplasia of pitting type
regularities called enamel hypoplasia.
if deficiency occurs during teeth formation.
l Enamel hypoplasia is of two types:
Enamel-hypoplasia due to congenital syphilis:
(a)   Hereditary (also called amelogenesis imperfecta)
l In congenital syphilis, crowns of maxillary central
(b)   Environmental}
incisors become screw driver shaped.
(a)  Hereditary enamel hypoplasia (amelogenesis im-
l Crown of first molar at occlusal surface arranged
perfecta):
into agglomerated mass of globule and termed as
l It is a group of hereditary defects of enamel as-
mulberry molars. The teeth affected are called
sociated with other generalized defects, dentin is
Hutchinson teeth.
usually normal.
Enamel hypoplasia due to hypocalcaemia:
l It may be of three types:
l Calcium level ,6–8 mg/dL may cause enamel
(a)  Hypoplastic type: It is the defect of enamel
hypoplasia of pitting type.
organic matrix formation.
Enamel hypoplasia due to birth injuries:
(b)  Hypocalcification type: It is the defect of
l Premature children affected with Rh haemo-
mineralization of enamel.
lytic diseases at birth may suffer from enamel
(c)   Hypomaturation type: It is the defect of
hypoplasia.
enamel crystal maturation.
Enamel hypoplasia due to ingestion of fluoride:
Clinical features:
l If drinking water contains fluoride content more
i. In hypoplastic type, enamel thickness is
than 1 ppm at the time of amelogenesis, it can
not complete.
cause death of ameloblasts so it leads to defective
ii. In hypocalcification type, enamel is soft
matrix formation.
and can be removed by prophylactic in-
l It may also result in disturbances in calcification.
strument.
Enamel affected is termed as mottled enamel.
iii. In hypomaturative type:
l In mild cases white flecks appear in enamel. Mod-
l Enamel can be pierced with explorer
erate degree of fluoride toxicity may lead to white
point and chipped off.
opaque areas in enamel. In more severe case,
l Teeth are brownish in colour.
brown discolouration of enamel occurs.
l Vertical lines or grooves may be present
l Still more severe toxicity of fluoride causes cor-
on surface.
roded crown surface.
l Enamel is chalky and it can be chipped
Treatment:
off with exposure of underlying dentin.
l Defluoridation of drinking water is done
l Contact points are abraded.
if drinking water contains excess fluoride
Treatment:
content.
l There is no treatment except for improve-
l The staining of enamel surface can be removed
ment of cosmetic appearance by veneering
by bleaching with hydrogen peroxide (30%
or capping of teeth.
solution), grinding or capping.
(b)   Environmental enamel hypoplasia:
Enamel hypoplasia due to local infection or trauma:
l In this type of enamel hypoplasia both enamel
l Turner first described this localized type of hy-
and dentin are affected.
poplasia. He/she noted defects in the enamel of
[SE Q.2] two premolars and traced the defects to apical
{Causes: infection of the nearest primary molar. Enamel
l Nutritional deficiencies hypoplasia resulting from local infection is
l Exanthematous diseases called Turner tooth.
l Only single tooth, generally maxillary incisor or
l Congenital syphilis
l Hypocalcaemia premolar, is affected.
396 Quick Review Series for BDS 4th Year, Vol 1

l The apically infected deciduous tooth may affect Hypoplasia associated with cleft lip and palate:
the ameloblastic layer of permanent tooth. l Mink studied the incidence of enamel hypoplasia

l Ameloblastic layer may also get disturbed due to of the maxillary anterior teeth in 98 patients with
occlusal trauma to deciduous tooth. repaired bilateral and unilateral complete cleft lip
l The permanent tooth beneath infected or trauma- and palate.
tized deciduous tooth is discoloured or pitted. l Among patients in the repaired unilateral and bi-

l The teeth are called Turner teeth and this type of lateral complete cleft lip and palate group, 66% of
enamel hypoplasia is termed as Turner hypopla- those with maxillary anterior primary teeth had
sia.} one or more primary teeth affected with enamel
Enamel hypoplasia related to brain injury and neuro- hypoplasia and 92% of those with erupted maxil-
logic defects: lary anterior permanent teeth had one or more
l Herman and McDonald observed enamel hypo- permanent teeth affected with enamel hypoplasia.
plasia in 36% of the group with cerebral palsy and l Mink concluded that the permanent teeth are in

in 6% of the group without the disorder. earlier stages of development at the time of the
l A definite relationship between the time of occur- surgical procedure and are more subject to dam-
rence of the possible factors that could have age.
caused brain damage and the apparent time of Hypoplasia caused by X-radiation and chemotherapy:
origination of the enamel defect was established l Numerous dental abnormalities may result in sur-
for 70% of the affected teeth of children with ce- viving children who receive high-dose radiother-
rebral palsy. apy and chemotherapy during the time their teeth
l Cohen and Diner observed that enamel defects are forming.
occurred with greatest frequency in children with l Children who receive high-dose X-radiation in the

low intelligence quotients and a high incidence of treatment of a malignancy are at risk for develop-
neurologic defects. ing rampant caries in the irradiated area.
Hypoplasia associated with nephrotic syndrome: l Ameloblasts are somewhat resistant to X-radia-

l Oliver and Owings observed enamel hypoplasia in tion. However, a line of hypoplastic enamel that
permanent teeth in a high percentage of children corresponds to the stage of development at the
with nephrotic syndrome and found a correlation time of therapy may be seen.
between the time of severe renal disease and the es-
timated time at which the defective enamel forma-
tion occurred.
SHORT ESSAYS:
l Koch and colleagues found a high incidence of Q.1. Submerged teeth.
enamel defects in the primary teeth of children
Ans.
who were diagnosed with chronic renal failure
early in infancy. l Ankylosed teeth are submerged teeth, most commonly
Hypoplasia associated with allergies: deciduous mandibular second molars that have under-
l Rattner and Myers discovered a correlation be- gone a variable degree of root resorption and have be-
tween enamel defects of the primary dentition and come ankylosed to the bone.
the presence of severe allergic reactions. l They may exist in three conditions:
l Enamel defects were present in 26 of 45 children i. Minimal infraocclusion: Marginal ridge of sub-
with congenital allergies. The enamel lesions merged tooth is occlusal to adjacent areas
were localized in the occlusal third of the primary ii. Moderate infraocclusion: Marginal ridge of sub-
canines and first molars. merged tooth is just cervical to adjacent contact areas
Hypoplasa associated with lead poisoning (plumbism): iii. Severe infraocclusion: Marginal ridge of sub-
l Lawson and Stout observed that in areas of merged tooth is at gingival level
Charleston, South Carolina, where there were Causes:
very old frame buildings, the incidence of pitting l Trauma, infection, disturbed local metabolism
hypoplasia was approximately 100% greater than and genetic influence
their control group of children. Treatment:
l Pearl and Roland have pointed out that the fetus l This condition is usually treated by surgical
of a lead-poisoned mother can be affected because removal of ankylosed teeth to prevent the de-
lead readily crosses the placenta during preg- velopment of malocclusion, local periodontal
nancy. disturbance or dental caries
Section | I  Topic-Wise Solved Questions of Previous Years 397

l In case of moderate infra-occlusion there are two l Two clinically distinct forms of autosomal domi-
treatment alternatives: nant amelogenesis imperfect a smooth hypoplastic
a. Retain the submerged tooth: amelogenesis imperfecta and local hypoplastic am-
Maintain the adjacent tooth contacts and op- elogenesis imperfecta are associated with mutations
posite tooth contacts by fitting a stainless steel in the enamelin (ENAM) gene located at 4q21.
crown or by building up the occlusal surface l In addition, autosomal dominant amelogenesis im-

b. Extract the submerged tooth – if it interferes perfecta can be associated with mutation in the
with normal eruption of premolars kallikrein-4 (KLK4) gene, and autosomal recessive
l In case of severe infra-occlusion, extract the sub- pigmented hypomaturation amelogenesis imper-
merged tooth fecta with an enamelysin (MMP-20) gene muta-
tion, illustrating the heterogeneity of the condition.
Q.2. Enamel hypoplasia. Clinical features:
l Amelogenesis imperfecta has a wide range of
Ans. clinical appearances with three broad catego-
[Ref LE Q.2] ries the hypocalcified type, the hypomaturation
type and the hypoplastic type.
Q.3. Turner (hypoplasia) tooth. l Hypoplastic teeth lack normal enamel thick-

Ans. ness due to inadequate deposition of matrix.


Enamel is pitted with horizontal and vertical
l Turner first described this localized type of hypoplasia. ridges.
l Individual permanent teeth have hypoplastic or hypo l In hypomaturation, type there is normal depo-
calcified areas on the crown that result from infection or sition of enamel and defective maturation of
trauma. crystal structure.
l Periapical inflammatory processes of primary teeth extend l Affected teeth are motteled, opaque and white
towards the buds of the pertinent permanent teeth and af- brown yellowish discolouration.
fect them during their prefunctional stage of eruption. l Enamel is soft and chips off from dentin.
l The infection fails to stimulate the development of a l Hypomaturation hypoplastic with taurodontism:
fibrous wall that would localize the lesion. Enamel is mottled yellow brown with areas of
l Instead the infection spreads diffusely through the bone hypomaturation. Molar teeth have taurodont
around the buds of the successors and thereby affects shape and other teeth have enlarged pulp change.
the important protective layer of the young enamel, the Treatment:
reduced enamel epithelium. l There is no treatment except for improvement
l The permanent tooth beneath infected or traumatized of cosmetic appearance by veneering or cap-
deciduous tooth is discoloured or pitted. The teeth are ping of teeth.
called Turner teeth and this type of enamel hypoplasia
is termed as Turner hypoplasia. Poor aesthetics Primary teeth restored with glass ion-
Q.4. Germination and fusion. omer/composite veneer
Dentinal sensitivity Full coverage with stainless steel
Ans. crowns
[Ref LE Q.1] Dental caries Dietary advice, fluoride therapy;
glass ionomer/composite restorations
Q.5. Amelogenesis imperfecta. and stainless steel crowns
Ans. Gingival inflammation Improve preventive oral health care
practices
l Amelogenesis imperfecta is a developmental defect of
the enamel with a heterogeneous aetiology that affects
the enamel of both the primary and permanent dentition. Q.6. Anomalies of the number of teeth.
l Amelogenesis imperfecta is alteration in both quality

and quantity of enamel. Ans.


Aetiology: Anomalies of the number of teeth are as follows:
l Gene mutations in enamel matrix results in hypo- A. Anodontia: total lack of tooth development
plasia, hypocalcification and hypomaturation. B. Hypodontia: lack of development of one or more teeth
398 Quick Review Series for BDS 4th Year, Vol 1

C. Oligodontia: lack of development of six or more teeth


{SN Q.10}
(a subdivision of hypodontia)
D. Hyperdontia – development of increased number of Partial anodontia:
teeth l It is a common phenomenon and is characterized

A. Anodontia: by congenital absence of one or few teeth.


l Anodontia is defined as the condition in which there l In partial anodontia any tooth can be congenitally

is congenital absence of teeth in oral cavity. missing.


l Anodontia is rare and most cases occur in the pres- For example: The third molars are most fre-
ence of ectodermal dysplasia. quently observed congenitally missing teeth.
l Aetiology: The mandibular first molars and the mandibular
lateral incisors are least likely to be missing.
The causes of anodontia are
i. Hereditary factor
ii. Environmental factor
iii. Familial factor B. Hypodontia and oligodontia:
iv. Syndrome associated l Oligodontia refers to lack of development of six or
v. Radiation injury to the developing tooth germ more teeth.
Types: l Damage to dental lamina before tooth formation can
l Anodontia can also be divided into following result in hypodontia.
types: Aetiology:
a. True anodontia: It occurs due to failure of l May be caused by genetic factors, trauma, endo-
development or formation of tooth in jaw crine disturbances, infection, radiation and che-
bone. motherapeutic medications.
b. Pseudo-anodontia: It refers to the condition l It may also occur in hereditary syndromes such as
in which teeth are present within the jaw Crouzon syndrome, Down syndrome, ectodermal
bone but are not clinically visible in the dysplasia, Hurler syndrome and Turner syndrome.
mouth, as they have not erupted, e.g. im- Clinical features:
pacted teeth. l It usually affects permanent third molars, second
c. Induced or false anodontia: It is the condi- premolars and lateral incisors in that order.
tion in which teeth are missing in the oral l Oligodontia and hypodontia may cause abnormal
cavity because of their previous extractions. spacing of teeth, delayed tooth formation, delayed
deciduous tooth exfoliation and late permanent
{SN Q.10} tooth eruption.
Treatment:
l True anodontia is of two types: l Orthodontic closure of space or prosthetic re-
a. Complete anodontia: There is congenital absence placement of teeth may be needed.
of all the teeth. D. Hyperdontia:
b. Partial anodontia: Congenital absence of one or Increase in number of teeth (excess teeth) is known as
few teeth. hyperdontia.
Aetiology:
Complete anodontia: l Multiple supernumerary teeth can occur in asso-
l It is the condition in which there is ciation with the syndromes like Gardener syn-
neither any deciduous tooth nor any drome and cleidocranial dysplasia.
permanent tooth present in the oral l Continue activity of dental lamina.
cavity. l Complete division of eruptive teeth.
l A complete anodontia is a common fea- Clinical features:
ture of hereditary ectodermal dysplasia; l Rudimentary teeth
however, in many cases cuspids are pres- l Erupted/impacted teeth
ent in this disease. l Supplemental teeth like mesiodens, para molars
l Complete anodontia occurs among chil- and disto molars
dren those who have received high doses l Displacement of adjacent teeth
of radiation to the jaws as infants for Treatment:
therapeutic reasons. Surgical removal or orthodontic alignment of teeth
Section | I  Topic-Wise Solved Questions of Previous Years 399

Q.7. Pink tooth. Q.9. Tooth discolouration (intrinsic).


Ans. Ans.
Intrinsic discolouration of teeth (pigmentation of teeth):
{SN Q.15} l The primary teeth occasionally have unusual pig-

l Internal resorption is also known as chronic perforat- mentation.


l Certain conditions arising from the pulp can cause
ing hyperplasia of pulp, odontoclastoma or pink
tooth of Mummery. the whole tooth to appear discoloured.
l It is an unusual form of resorption that begins cen- Intrinsic discolouration of teeth is seen in:
trally within the pulp, apparently initiated by a pecu- i. Erythroblastosis fetalis
liar inflammatory hyperplasia of the pulp. ii. Porphyria
Aetiology: iii. Cystic fibrosis
l Idiopathic iv. Tetracycline therapy
Clinical features Intrinsic discolouration of teeth in various conditions is
l No early clinical signs and symptoms. described below:
l Tooth may show pink spot (pink tooth) when Discolouration in hyperbilirubinaemia:
l If teeth are developing during periods of hy-
more of dentine is resorbed from one area of
the crown, leaving a covering of translucent perbilirubinaemia they may become intrinsi-
enamel. cally stained.
l Excess levels of bilirubin are released into the
l It appears as a pink area due to vascular pulp

visible through the translucent enamel. circulating blood in a number of conditions.


Radiographic appearance: The two most common disorders that cause
Pink spot appears as round or ovoid area of radiolu- this intrinsic staining are erythroblastosis feta-
cency in the central portion of the tooth. lis and biliary atresia.
l Erythroblastosis fetalis results from the trans-
Treatment:
l If condition is discovered before perforation of placental passage of maternal antibody active
crown, root canal therapy may be carried out. against red blood cell antigens of the infant,
l Once perforation has occurred, extraction of tooth which leads to an increased rate of red blood
is the treatment. cell destruction.
l If an infant has had severe, persistent jaundice

during the neonatal period, the primary teeth


Q.8. Supernumerary teeth.
may have a characteristic blue-green colour,
Ans. although in a few instances brown teeth have
been observed.
{SN Q.11} Discolouration in porphyria:
l The porphyrias are inherited and acquired dis-
l Supernumerary teeth are a developmental distur- orders in which the activities of the enzymes of
bance in the number of teeth. the haem biosynthetic pathway are partially or
l A supernumerary tooth is an additional entity to the almost completely deficient.
normal series and is seen in all quadrants of the jaw. l As a result, abnormally elevated levels of por-
l Morphological types of supernumerary teeth: phyrins and/or their precursors are produced,
Conical accumulate in tissues, and are excreted.
Tuberculate l The primary teeth of children with congenital
Supplemental erythropoietic porphyria are purplish brown as a
Odontome result of the deposition of porphyrin in the devel-
l Supplemental supernumerary teeth are teeth that re- oping structures. The permanent teeth also show
semble the typical anatomy of posterior and anterior evidence of intrinsic staining but to a lesser degree.
teeth. Discolouration in cystic fibrosis:
l Rudimentary supernumerary teeth are conical in l Cystic fibrosis is an inherited, chronic, multi-
shape. Usually they are found in syndromes like clei- system, life shortening disorder characterized
docranial dysplasia and orofacial digital syndrome. primarily by obstruction and infection of the
l Multiple supernumerary teeth can occur in associa- airways and poor digestion.
tion with the conditions like Gardener syndrome and l Tetracycline is the drug of choice in these
cleidocranial dysplasia. patients.
400 Quick Review Series for BDS 4th Year, Vol 1

l The unsightly yellowish gray to dark brown Types:


discolourations of the teeth in these patients is l Anodontia can be divided into following types:
due to tetracycline therapy during a period a. True anodontia:
when their tooth crowns were forming. l It occurs due to failure of development or
Discolouration in tetracycline therapy: formation of tooth in jaw bone.
l Dentists and physicians have observed that l True anodontia is of two types:
children who have received tetracycline ther- i. Complete anodontia: There is congenital
apy during the period of calcification of the absence of all the teeth.
primary or permanent teeth show a degree of ii. Partial anodontia: Congenital absence of
pigmentation of the clinical crowns of the one or few teeth.
teeth. b. Pseudo-anodontia:
l The crowns of affected teeth are discoloured, It refers to the condition in which teeth are
ranging from yellow to brown and from gray to present within the jaw bone but are not clini-
black. cally visible in the mouth, as they have not
l Tetracycline is deposited in the dentin and to erupted, e.g. impacted teeth.
a lesser extent in the enamel of teeth that are c. Induced or false anodontia:
calcifying during the time the drug is admin- It is the condition in which teeth are missing in the
istered. oral cavity because of their previous extractions.
l The tetracyclines, which are yellow, fluoresce

under ultraviolet light. Q.4. Dilaceration.

Q.10. Intrinsic discolouration of teeth. Ans.

Ans. [Ref LE Q.1]

[Same as SE Q.9] Q.5. Talon cusps.


Ans.
SHORT NOTES:
[Ref LE Q.1]
Q.1. Mesiodens.
Q.6. Taurodontism.
Ans.
Ans.
l Mesiodens is a maxillary supernumerary tooth between
two central incisors and is the most common supernu- l Taurodontism is a dental anomaly in which the body of
merary teeth. the tooth is enlarged at the expense of the roots.
l The term taurodontism refers to ‘bull-like teeth’
l Autosomal dominant type of inheritance.

l 90% occur in maxilla.


Aetiology:
l Failure of Hertwig epithelial sheath to invaginate
l Develops from third tooth bud or splitting of permanent

tooth bud. at the proper horizontal level.


l More common in males compared to females.
Clinical features:
l Most commonly involves permanent dentition
l Occurrence is very less in deciduous teeth.
followed by deciduous dentition, molars are com-
Q.2. Twinning. monly involved.
l Condition may be unilateral or bilateral.
Ans.
l Involved teeth are rectangular in shape with a long
l The division of a single tooth resulting in one normal body and short roots.
and one supernumerary tooth. l Jaspers and Witkop noted that taurodontism is
l The number of teeth will be more than normal. found in about 2.5% of adult whites as an isolated
Q.3. Anodontia. trait, as well as in individuals with syndromes
such as trichodento-osseous syndrome, otodental
Ans. dysplasia, and X-chromosome aneuploidies.
l Anodontia is defined as the condition in which there is Q.7. Ankyloglossia.
congenital absence of teeth in oral cavity.
Aetiology: Ans.
l Hereditary, environmental or familial factors or l In ankyloglossia a short lingual frenum extending from
syndrome associated. the tip of the tongue to the floor of the mouth and onto
Section | I  Topic-Wise Solved Questions of Previous Years 401

the lingual gingival tissue limits movements of the


tongue and causes speech difficulties.
l In the older child a reduction of the frenum should be
recommended only if local conditions or speech prob-
lems warrant the treatment.
l Stripping of the lingual tissues may occur if the tongue-
tie is not corrected.
l Surgical reduction of the abnormal lingual frenum by
lingual frenectomy, frenotomy or frenuloplasty is indi-
cated if it interferes with the infant’s nursing.
l Tongue mobility and speech patterns improved sig- Radiograph:
nificantly after the frenum attachment was released l Radiograph exhibits break in periodontal liga-

surgically. ment continuity.


Treatment:
Q.8. Dens in dente.
l Extraction (surgical removal), if permanent suc-

Ans. cessor is present.


l Functional occlusion with stainless steel crowns
l Dens in dente is a developmental variation which arises
should be planned, if permanent teeth are missing.
as a result of enamel epithelial invagination of the
crown surface before calcification. Q.10. Partial anodontia.
Aetiology:
Ans.
l It is because of focal growth proliferation and

focal growth retardation that takes place in certain [Ref SE Q.6]


areas of tooth bud.
Q.11. Supernumerary teeth.
l Increased localized external pressure.

Clinical features: Ans.


l Occurs in both primary and permanent teeth.
[Ref SE Q.8]
l Teeth most frequently involved are permanent

maxillary lateral and central incisors. Q.12. Bohn nodules.


l Anterior teeth with dens in dente are usually of
Ans.
normal shape and size.
l After calcification it appears as accentuation of l Bohn nodules are small cystic lesions found on the crest
lingual pit. of the maxillary and mandibular ridges.
Radiographic features: l Bohn nodules are formed along the buccal and lingual

l Appearance of tooth within tooth due to deep aspects of the dental ridges and on the palate away from
pear-shaped invagination from lingual pits, ap- the raphe.
proximating to pulp. l They are remnants of the dental lamina and may also be

Treatment: called ectopic or pseudodental lamina cysts.


l Application of sealant or a restoration in the open- l No treatment is required for dental lamina cysts as they

ing of the invagination is the recommended treat- disappear within a month after birth.
ment to prevent pulpal involvement.
Q.13. Turner hypoplasia.
l This anomaly makes teeth prone to caries so, end-

odontic therapy should be done. Ans.


Q.9. Ankylosed teeth. l Turner hypoplasia is enamel hypoplasia due to local
infection or trauma.
Ans.
l Only single tooth, generally maxillary incisor or premo-

l In ankylosis there is lack of periodontal ligament conti- lar is affected.


nuity and tooth is directly attached to the bone. l The apically infected deciduous tooth may affect the

l Ankylosed or submerged teeth is most commonly seen ameloblastic layer of permanent tooth.
in mandibular primary molars, especially E. l Ameloblastic layer may also get disturbed due to oc-

l No contact with opposing tooth. clusal trauma to deciduous tooth.


l No mobility in spite of advanced root resorption. l The permanent tooth may get discoloured or pitted. The

l Solid sound on percussion (normal teeth produce cush- teeth are called Turner teeth and this type of enamel
ion effect sound). hypoplasia is termed as Turner hypoplasia.
402 Quick Review Series for BDS 4th Year, Vol 1

Q.14. Hyspoplasia due to nutritional deficiency. l In 1881 and 1890, Riga and Fede described this lesion
histologically; hence, it is known as Riga–Fede disease.
Ans.
l It is one of the complications of the natal teeth.

l Vitamins A, C and D, calcium and phosphorus deficien- l In this condition laceration and traumatic ulceration of

cies cause enamel hypoplasia of pitting type if defi- the ventral surface of the tongue, frenulum of the lip due
ciency occurs during teeth formation. to the sharp incisal edge of the natal teeth are seen.
l Deficiencies of vitamins A, C, and D, calcium, and l The more appropriate and descriptive term is ‘neo natal

phosphorus were the most common causes of defective sublingual traumatic ulceration’.
enamel formation. l Tooth may have to be removed in these cases.

l Enamel hypoplasia and neonatal tetany can be manifes- l Inflamed tissue around teeth should be controlled by

tations of vitamin D deficiency during pregnancy and applying chlorhexidine gluconate gel three times a day.
are most likely the result of secondary hyperparathy-
Q.19. Acrodynia.
roidism in the mother.
l Apparently in some children a mild vitamin D defi- Ans.
ciency state or systemic condition without clinical
l The exposure of young children to minute amounts of
symptoms can interfere with ameloblastic activity and
mercury is responsible for a condition referred to as
can produce a permanent defect in the developing
acrodynia or pink disease.
enamel.
l Ointments and medications are the usual sources of the
Q.15. Internal resorption. mercury. Dental amalgam restorations do not cause
acrodynia.
Ans.
l The clinical features of the disease include fever, an-
[Ref SE Q.7] orexia, desquamation of the soles and palms causing
them to be pink, sweating, tachycardia, gastrointestinal
Q.16. Eruption cyst.
disturbance and hypotonia.
Ans. l The oral findings include inflammation and ulceration

of the mucous membrane, excessive salivation, loss of


l Eruption haematoma or eruption cyst is a bluish purple,
alveolar bone and premature exfoliation of teeth.
elevated area of tissue, commonly called eruption hae-
matoma. Q.20. Mucocoele.
l Occasionally develops few weeks before the eruption of
Ans.
primary or permanent tooth.
l The blood filled cyst is most frequently seen in the pri- l The mucocoele, or mucus retention phenomenon, as it
mary second molar or the first permanent molar regions. is often called, is a salivary gland lesion of traumatic
l This fact substantiates the belief that the condition de- origin that forms when the main duct of a minor salivary
velops as a result of trauma to the soft tissue during gland is torn with subsequent extravasation of mucus
function and usually within a few days the tooth breaks into the fibrous connective tissue so that a cystlike cav-
through the tissue, and the haematoma subsides. ity is produced.
l The condition is almost always self-limited treatment l The wall of this cavity is formed by compressed bundles of

may occasionally be justified. collagen fibrils, and its lumen contains inspissated mucin.
l Mucocoeles occur most commonly on the lower lip,
Q.17. Epstein pearls.
with the floor of the mouth and buccal mucosa being the
Ans. next most frequent sites of involvement.
l They tend to be noted most frequently in the second and
l Epstein pearls are small, white or greyish white lesions
third decades of life. No obvious sex predilection is
on the alveolar mucosa of the newborn and on rare oc-
noted.
casions may be incorrectly diagnosed as natal teeth.
l Treatment is by surgical excision, with removal of the
l The lesions are usually multiple but do not increase in
involved accessory salivary gland.
size. No treatment is indicated, since the lesions are
spontaneously shed a few weeks after birth. Q.21. Dental effects of bobby pin opening.
l They are considered remnants of epithelial tissue
Ans.
trapped along the raphe as the fetus grew.
l Bobby pin opening is usually seen in teenage girls
Q.18. Riga–Fede disease.
wherein opening bobby pin with anterior incisors is
Ans. done.
Section | I  Topic-Wise Solved Questions of Previous Years 403

Dental effects: l Affected teeth are grey to yellowish brown and have
l Clinically there is notching of incisors and par- broad crowns with a constriction area of the cervical
tially denuded labial enamel. area resulting in a ‘tulip’ shape.
l Gingival laceration can also be noted. l Types:

l In long-term cases, there may be diastema that i. Dentinogenesis imperfect type I (opalescent dentin)
can be seen. ii. Dentionogensis imperfect type II (brandywine type)
Prevention
Q.24. Natal and neonatal teeth.
l Educating the teenagers about the harmful effects

of bobby pin opening is the best of its prevention. Ans.


Q.22. Germination and fusion. Natal teeth:
l Teeth present at birth are known as natal teeth. In
Ans.
these teeth there is almost no root present.
l Germination is a developmental anomaly which refers Neonatal teeth:
to division of single tooth germ into incomplete or com- l Teeth that erupt within 30 days after birth are called

plete formation of two teeth. neonatal teeth.


l Crowns may be partially or totally separated from each l Preferably natal or neonatal tooth should not be ex-

other while roots are fused and single root canal is pres- tracted for normal growth and uncomplicated erup-
ent within the root. tion of the adjacent teeth. But in case it is hypermo-
l Fusion is defined as single enlarged tooth or joined bile and there is a danger of its avulsion and
tooth in which the tooth count reveals a missing tooth swallowing by the child, it should be extracted.
when the anamolous tooth is counted as one. l The mother may have some problem in breastfeeding

l Fused teeth arise through union of two normally sepa- the child with natal or neonatal teeth. If the mother can-
rated tooth germs. not bear this discomfort, she can use the breast pump.
l Both permanent and deciduous dentition are affected in Q.25. Tongue tie.
case of fusion, although it is more common in decidu- Ans.
ous teeth.
l The incisor teeth are more frequently affected in both [Same as SN Q.7]
the dentitions. Q.26. Turner tooth.
Q.23. Dentinogenesis imperfecta. Ans.
Ans. [Same as SN Q.13]
l Dentinogenesis imperfect is a developmental distur- Q.27. Pink tooth.
bance in the structure of the teeth. Ans.
l This is an autosomal condition affecting both deciduous

and permanent teeth. [Same as SN Q.15]

Topic 13
Developing Malocclusion and Its Management
and Preventive Measures
COMMONLY ASKED QUESTIONS
LONG ESSAYS:
1 . Define and classify space maintainers. Describe Willet’s guiding shoe appliance.
2. Define and classify space maintainers. What factors would you consider before planning space maintainer to the
child patient?
404 Quick Review Series for BDS 4th Year, Vol 1

3. Discuss recent concepts on preventive and interceptive orthodontics for children.


4. Define serial extraction. Discuss the indication, contraindications, advantages, disadvantages and the proce-
dure of serial extraction.
5. Define space management and describe removable space maintainers in detail their indications, contraindica-
tions, advantages and disadvantages.
6. Define space maintainers. Describe about fixed space maintainers and enumerate fixed space maintainers used
for bilateral loss of deciduous molars.
7. Define space maintainers and objectives of space management. Classify space regainers. Write in detail about
space regainers.
8. Define and classify space maintainers. Write in detail about distal shoe space maintainers. [Same as LE Q.1]
9. Define space maintainers and space management. Classify space maintainers. Write in detail about eruption
guidance appliance. [Same as LE Q.1]
10. Discuss the indications, contraindications and classification of space maintainers. [Same as LE Q.1]
11. What are the indications for space maintainers? Describe briefly the various types of the same. [Same as LE Q.1]
12. A 8-year-old child reports to the clinic with bilateral loss of primary molars in the arch. What is the line of
approach? [Same as LE Q.6]

SHORT ESSAYS:
1. Distal shoe space maintainer.
2. Nance’s appliance.
3. Describe how labial frenum affects the teeth in the arch.
4. Classify anterior crossbite and how do you correct them.
5. Explain band and loop space maintainer construction procedure. [Ref LE Q.6]
6. Mixed dentition analysis. Tanaka–Johnson analysis.
7. Tongue blade therapy.
8. Catalan’s appliances.
9. Fixed space regainers. [Ref LE Q.7]
10. Removable space maintainers for multiple loss of teeth.
11. Indications of serial extraction. [Ref LE Q.4]
12. Lingual arch space maintainer. [Ref LE Q.6]
13. Willet’s appliance. [Same as SE Q.1]
14. Anterior diastema. [Same as SE Q.3]
15. Describe causes of midline diastema and its management. [Same as SE Q.3]
16. Median diastema. Treatment of mid-line diastema. [Same as SE Q.3]
17. Define crossbite. Write briefly the treatment of anterior crossbite at the age of 8 years. [Same as SE Q.4]
18. Explain any one of the fixed space maintainer. [Same as SE Q.5]
19. Tanaka–Johnson analysis. [Same as SE Q.6]
20. Inclined plane. [Same as SE Q.8]
21. Lower anterior inclined plane. [Same as SE Q.8]
22. Lingual arch holding appliance. [Same as SE Q.12]

SHORT NOTES:
1. Name fixed space regainers. [Ref LE Q.7]
2. Define serial extraction. [Ref LE Q.4]
3. Space maintainers. [Ref LE Q.2]
4. Tongue blade therapy.
5. Active space maintainers.
6. Eruption guidance appliance.
7. Blue-grass appliance.
8. Genetic aspect of malocclusion.
9. Anterior crossbite. [Ref SE Q.4]
10. Name the mixed dentition analysis. [Ref SE Q.6]
11. Inclined plane.
Section | I  Topic-Wise Solved Questions of Previous Years 405

1 2. Indications for removable space maintainers. [Ref LE Q.5]


13. Lip bumper. [Ref LE Q.7]
14. Trans-palatal arch appliance. [Ref LE Q.6]
15. Lingual arch space maintainer – Indications.
16. Common procedure done in interceptive orthodontics. [Ref LE Q.2]
17. Define preventive orthodontics. [Ref LE Q.2]
18. Moyer’s mixed dentition analysis.
19. Mention four indications of serial extraction. [Same as SN Q.2]
20. Classification of space maintainers according to the anchorage and support available. [Same as SN Q.3]
21. Correction of a developing single tooth crossbite. [Same as SN Q.4]
22. Aetiology of anterior crossbite. [Same as SN Q.4]
23. Catalan’s appliance. [Same as SN Q.11]

SOLVED ANSWERS
LONG ESSAYS:
Q.1. Define and classify space maintainers. Describe According to the anchorage and support available,
Willet’s guiding shoe appliance. space maintainers can be classified as:
Ans. i. Semifixed type space maintainer
Space maintainers are devices used to maintain the space l Crown-distal-shoe space maintainer

following the loss of deciduous tooth/teeth, so as to guide l Crown-loop space maintainer

the unerupted tooth into a proper position in the arch. l Band-loop space maintainer

or ii. Fixed type space maintainer


According to Boucher, it is a fixed or removable appli- l Lingual-holding arch space maintainer

ance designed to preserve the space created by the prema- l Nance’s holding arch

ture loss of a primary tooth or a group of teeth. iii. Acrylic partial denture
or l Acrylic partial denture

According to J.C. Brauer, space maintainer refers to an l Complete denture

appliance designed to retain a given area or space, generally Indications of space maintainers are as follows:
in the primary and mixed dentitions. i. In cases of possible space inadequacy indicated
Classification of space maintainers: by space analysis for succedaneous teeth.
According to Hitchcock: ii. To prevent further complication of existing mal-
l Removable or fixed or semifixed occlusion by loss of space.
l With bands or without bands iii. After extraction of deciduous teeth, within first
l Functional or nonfunctional 6 months.
l Active or passive iv. Delayed or altered eruption of permanent tooth.
l Certain combinations of the above v. Congenital absence of permanent tooth.
According to Raymond C. Thurow: vi. In case of premature signs of loss of deciduous
l Removable teeth.
l Complete arch – lingual arch and extraoral vii. To avoid supraeruption of opposing tooth.
anchorage viii. In case of erupting premolars (as they require
l Individual tooth 4–5 months to erupt or move 1 mm in bone).
According to Hinrichsen: ix. To improve physiology of child’s masticatory
l Fixed space maintainers system and to restore dental health.
i. Class I Contra indications of space maintainers are as follows:
a. Nonfunctional types – bar type, loop type i. When there is sufficient space for eruption and
b. Functional types – pontic type, lingual arch alveolar bone covering over the erupting crown of
type tooth is absent.
ii. Class II ii. When space left is greater than mesiodistal dimen-
a. Cantilever type (distal shoe, band and loop) sion of erupting tooth and space loss is not expected.
l Removable space maintainers iii. When the radiograph of extracted region shows one
Acrylic partial dentures third of root of succedaneous tooth.
406 Quick Review Series for BDS 4th Year, Vol 1

iv. Where permanent succeeding tooth is congenitally the loop are soldered to the band or directly to the
absent and space closure is desired. crown in some cases.
v. If the space shows no signs of closing. Position and width of the distal extension:
Distal shoe space maintainer: l The primary function of distal shoe appliance is to

l Distal shoe appliance is otherwise known as the intra provide a guide plane for the eruption path of
alveolar appliance. unerupted first permanent molar.
l One of the early designs of distal shoe space main- l The mandibular and maxillary first permanent

tainers was cast gold or Willet’s distal shoe. molars differ markedly in their paths of eruption.
l This appliance is rarely used because of the increased The normal path of eruption of the mandibular
cost of the materials, difficulties in tooth preparation first permanent molar is in a mesial and lingual
and more complicated fabrication procedures. direction, erupting against the distal surface of the
l The appliance in practice now is Roche’s distal shoe second primary molar, using it as a buttress to
or modifications of it using crown and band appli- guide itself into position.
ances with a distal intragingival extension. l Whereas, the maxillary first permanent molar

l V-shaped end offers a broader surface and helps pre- erupts in a distal and buccal direction until it
vent rotations. Broader surface also holds greater meets muscular resistance. It then erupts in a me-
chance of success, if the unerupted tooth is posi- sial direction until contact is made with the distal
tioned bucally or lingually in the dental arch. surface of the second primary molar.
l Distal surface of the second primary molar provides l The design and placement of the distal extension

a guide for unerupted first permanent molar. of the appliance will differ for upper and lower
l When the second primary molar is removed prior to arches, as the eruption patterns of the mandibular
the eruption of first permanent molar, the intra-alveolar and maxillary first permanent molars differ.
appliance provides greater control of the path of erup- l In the lower arch, the contact area of distal exten-

tion of the unerupted tooth and prevents undesirable sion should have slight lingual position over the
mesial migration. crest of the alveolar ridge in order to engage the
Indications: mesial contact area of the first permanent molar as
l It is indicated when the second deciduous molar is it begins its mesial and lingual movements. The
extracted or lost before the eruption of first perma- contact area of the distal extension of the maxil-
nent molar. lary appliance should be slightly buccal to the
Contraindications: crest of the alveolar ridge.
l Inadequate abutments due to multiple loss of teeth. l These considerations are important in preventing

l Poor oral hygiene or lack of parent and patient coop- the erupting permanent molar from slipping contact
eration. with the appliance. The width should closely ap-
l Medically compromised patients. proximate the normal contact area of the distal sur-
For example: Patients with congenital diseases, kid- face of the second primary molar being replaced.
ney problems, juvenile diabetics, history of rheu- Length of the distal extension (horizontal bar):
matic fever and haemophilia l The determination of the proper length of the dis-

l Congenitally missing first permanent molar which is tal extension of the appliance is essential.
a rare situation. l If the second primary molar is still present to

Method of fabrication: serve as a guide on the working model then the


l A stainless steel band is adopted using the first pri- problem is simplified. In such cases it should be
mary molar as an abutment. maintained, if possible until the appliance is ready
l If the morphology of the tooth does not permit easy to be sealed.
placement and adaptation of band then stainless steel l If the second primary molar is missing, then it is

crown provides a desirable contour for the placement recommended to measure radiographically the
of stainless steel band. The band is placed over the distance between the distal surface of the first
stainless steel crown or abutment tooth. primary molar and mesial surface of the un-
l A compound impression is made, the band is re- erupted first permanent molar.
moved and placed in the impression and a stone If the opposite second primary molar is present,
model is prepared then record the mesiodistal width of the tooth and
Construction of loop: compare that with the radiographic measurement.
l Using a 0.040-inch wire, the tissue bearing loop is l It is not necessary in all cases, therefore, to extend

then contoured extending distally and into the the appliance to the mesial surface of the first
prepared opening on the model. The free ends of permanent molar.
Section | I  Topic-Wise Solved Questions of Previous Years 407

Depth of the gingival extension (vertical bar): root surface of the second deciduous molar and
l Another determination to be made in constructing does not use the root for eruption guidance at
this appliance is the intra-alveolar depth of the gingi- all.
val extension. c. Instead, the lower first permanent molar normally
l If the extension is left too long, possible harm to the erupts in occlusal direction to contact first the
developing second premolar may result. If the exten- distal crown surface of the deciduous molar and
sion is too short, the first permanent molar could uses that to buttress for uprighting and establish-
erupt underneath the appliance. ing a mesial position.
l The gingival extension of the appliance should be

constructed to extend about 1 mm below the


mesial marginal ridge of the first permanent molar or Q.2. Define and classify space maintainers. What factors
just sufficient to ‘capture’ its mesial surface as the would you consider before planning space maintainer to
tooth erupts and moves forward. the child patient?
l For indirect construction techniques, a good preop-

erative radiograph that is slightly under exposed to Ans.


show the thickness of the overlying soft tissues will
aid in determining the depth of the groove to be cut {SN Q.3}
in the working model for constructing the gingival l Space maintainers are devices used to maintain the
extension. space following the loss of deciduous tooth/teeth, so
Appliance placement: as to guide the unerupted tooth into a proper position
l The appliance is removed from the model and the V
in the arch.
of the tissue extension is filled in and soldered with or
pieces of 0.040 inch wire. l According to Boucher, it is a fixed or removable ap-
l A knife edge is formed at the apex of the V, if the
pliance designed to preserve the space created by the
second primary molar has previously been extracted premature loss of a primary tooth or a group of teeth.
and the extraction site has healed.
l The sharpened distal shoe may be passed through a

sterilized and anesthetized area of the ridge. If the or


appliance is delivered at the time of extraction, the l According to J.C. Brauer, space maintainer refers to an
intragingival extension is just polished and not sharp- appliance designed to retain a given area or space, gen-
ened. erally in the primary and mixed dentitions.
l Before final placement of the maintainer in the Classification of space maintainers:
mouth, a radiograph is taken to determine whether According to Hitchcock:
the tissue extension of the appliance is in proper re- l Removable or fixed or semifixed
lationship with the unerupted first permanent molar. l With bands or without bands
Final adjustments in length and contour of the distal l Functional or nonfunctional
shoe can be made at this time. l Active or passive
l For the cases, where the distal shoe is contra- l Certain combinations of the above
indicated, two possibilities for treatment exist: According to Raymond C. Thurow:
i. To allow the tooth to erupt and regain space later. l Removable
ii. Use a removable or fixed appliance that does not Complete arch – lingual arch and extraoral
penetrate the tissue but places pressure on the anchorage
ridge mesial to the unerupted permanent molar. l Individual tooth
l Cases have been reported in the literature where a According to Hinrichsen:
pressure appliance, removable or fixed, was used to l Fixed space maintainers
guide the permanent molar as it erupted. i. Class I
l According to Barber, the appliance has become con- a. Nonfunctional types – bar type, loop
troversial in recent years due to following reasons: type
a. First, there have been reports of trauma and dam- b. Functional types – pontic type, lingual
age to the unerupted permanent teeth by appliance arch type
or procedure. ii. Class II
b. Second, it is felt that the normal eruption of the a. Cantilever type (distal shoe, band and
lower first permanent molar rarely contacts the loop)
408 Quick Review Series for BDS 4th Year, Vol 1

l Removable space maintainers Mandibular 1st molar is 0.9 mm


Acrylic partial dentures Maxillary 2nd molar is 2.2 mm
Mandibular 2nd molar is 1.7 mm
{SN Q.3} l According to Clinch and Healy:

Space loss before eruption of permanent molar


According to the anchorage and support available,
is 6.1 mm.
space maintainers can be classified as:
Space loss after eruption of permanent molar is
i. Semi-fixed type space maintainer
3.7 mm.
l Crown-distal-shoe space maintainer
iii. Rate of space closure:
l Crown-loop space maintainer
l Maxillary spaces close faster as compared to
l Band-loop space maintainer
mandibular spaces.
ii. Fixed type space maintainer
l Younger the patient, more is the space loss.
l Lingual-holding arch space maintainer
l Maximum space is lost during first 6 months of
l Nance’s holding arch
extraction and most immediate loss is within
iii. Acrylic partial denture
first 76 h.
l Acrylic partial denture
iv. Direction of space closure:
l Complete denture
l F.S. Stewart observed that in maxilla all except

1 of 12 extraction spaces closed by mesial mi-


Factors affecting planning for space maintainers: gration of teeth distal to the extraction space.
i. Time elapsed since tooth loss l In mandible all space losses greater than
ii. Amount of space loss 2 mm were brought about mainly by a distal
iii. Rate of space closure movement of the teeth mesial to the space.
iv. Direction of space closure l J.S. Rose (1966) states that space closure can
v. Eruption status of the adjacent teeth occur in two ways:
vi. Eruption status of the succedaneous tooth i. Through forward migration
vii. Congenital absence of teeth ii. Rotation of teeth distal to the site of extrac-
viii. Sequence of eruption tion
ix. Delayed eruption of permanent teeth v. Eruption status of the adjacent teeth:
x. Amount of bone coverage over the tooth l It helps us ascertain mesial shift for molars and
xi. Available space and arch length adequacy distal tipping for canines.
xii. Curve of Spee vi. Eruption status of the succedaneous tooth:
xiii. Abnormal oral habits l It is estimated by the amount of root comple-
xiv. Miscellaneous factors tion.
xv. Congenital absence vii. Congenital absence of teeth:
l Congenital absence of teeth may alter the erup-
i. Time elapsed since tooth loss: tion path of other teeth.
l It was stated by McDonald and Avery that if viii. Sequence of eruption:
space closure is going to occur, it will usually l Knowledge of usual eruption sequence is im-
take place within six months after the loss of portant.
tooth. For example: If the mandibular primary sec-
l In order to prevent space closure, the appliance ond molar is prematurely lost and mandibu-
must be placed as soon as possible, following lar second permanent molar is erupting be-
the extraction of tooth. fore the second premolar, arch length loss
ii. Amount of space loss: secondary to mesial forces generated on first
l Olsen (1959) stated that greater loss occurs in permanent molar as the second permanent
mandible owing to a mesial axial orientation of molar erupts can occur with subsequent
1st molar. space loss.
l Richardson (1965), Cohen (1941) and Seipel ix. Delayed eruption of permanent teeth:
(1949) stated that loss of 2nd deciduous l Over-retained or ankylosed primary teeth, or im-
molar will cause greater space loss. pacted permanent teeth, can result in a delay of
l According to Breakspear the space loss follow- the eruption process.
ing loss of individual tooth is as follows: x. Amount of bone coverage over the tooth:
Space loss following the loss of: l According to McDonald 1 mm of bone resorbs in
Maxillary 1st molar is 0.8 mm 4–5 months and so if the bone is present over the
Section | I  Topic-Wise Solved Questions of Previous Years 409

succedaneous tooth it is an indication for space oral health care including diagnosis, prevention, restora-
maintainer. tion and correction of malocclusion are increasingly the
xi. Available space: responsibility of the paediatric dentist.
l An evaluation of the available space should be Preventive orthodontics:
performed to determine whether the deficiency
is developmental or a result of the pre-existing
condition. {SN Q.17}
l A space analysis conducted in the mixed denti- Graber (1966) has defined preventive orthodontics as the
tion will aid the practitioner in a prediction of action taken to preserve the integrity of what appears to
the amount of available space for the be normal occlusion at a specific time.
unerupted permanent teeth. A decision may be or
made about the type of space maintenance that is Profft and Ackerman (1980) have defined it as preven-
appropriate. tion of potential interference with occlusal development.
xii. Arch length adequacy:
This will be estimated by position of incisors, Lee-
way space and incisor liability. Procedures undertaken in preventive orthodontics
xiii. Curve of Spee: are as follows:
According to Andrews, ideal occlusion will have a l Parent education

near flat curve of Spee; thus additional space can l Caries control

be gained (l mm of space is gained per 1 mm of l Care of primary dentition

depth of curve of Spee). l Management of ankylosed tooth

xiv. Abnormal oral habits: l Maintenance of quadrant wise tooth shedding

They will exert abnormal pressure on dental arches time table


and may influence the type and planning of space l Check-up for oral habits and habit breaking

maintainer. appliances, if necessary


xv. Miscellaneous factors: l Occlusal equilibration, if there are any occlusal

These factors influence planning because they prematurities


may be associated with either space gain or space l Prevention of damage to occlusion

loss. Some of these factors are growth of jaws, l Extraction of supernumerary teeth

proximal caries, wear and attrition. l Space maintenance

l Management of deeply locked first permanent


Q.3. Discuss recent concepts on preventive and intercep-
molars
tive orthodontics for children.
l Management of abnormal frenal attachments

Ans.
l The purpose of early orthodontic treatment is to guide
the dentitions towards the development of a functionally {SN Q.16}
and morphologically harmonious occlusion.
Interceptive orthodontics:
l To intercept a developing malocclusion, a number of
American Association of Orthodontics (1969)
procedures can be undertaken. Unlike preventive orth-
defined interceptive orthodontics as that phase
odontic procedures that are aimed at elimination of fac-
of science and art of orthodontics employed to
tors that may lead to malocclusion.
recognize and eliminate the potential irregularities
l The terms preventive and interceptive orthodontics are
and malpositions in the developing dento-facial
sometimes used synonymously. But preventive orth-
complex.
odontic procedures are undertaken when the dentition
Procedures undertaken in interceptive orthodontics:
and occlusion are perfectly normal, while the intercep-
l Serial extraction
tive procedures are carried out when the signs and
l Correction of developing crossbite
symptoms of a malocclusion have appeared.
l Control of abnormal habits
l Some of the procedures carried out in preventive ortho-
l Space regaining
dontics can also be performed in interceptive orthodon-
l Muscle exercises
tics but the timings are different.
l Interception of skeletal malrelation
l Paediatric dentistry has increasingly shifted from a
l Removal of soft tissue or bony barrier to en-
conservative – restorative approach towards a concept
able eruption of teeth
of total paediatric patient care. Thus, all the aspects of
410 Quick Review Series for BDS 4th Year, Vol 1

Q.4. Define serial extraction. Discus the indication, con- ii. Congenital absence of teeth – anodontia/oligo-
traindications, advantages, disadvantages and the pro- dontia
cedure of serial extraction. iii. Open bite and deep bite
iv. Spaced dentition and midline diastema
Ans.
vi. Unerupted malformed teeth, e.g. dilaceration
vii. Extensive caries or heavily filled 1st permanent
(SE Q.11 and SN Q.2) molars
viii. Cleft lip and cleft palate cases
{l (The term serial extraction was first introduced by ix. Patients with reverse overjet, deep bite, crossbite,
Kjellgren in the year 1929. Nance (1940) actually rotation, gross malposition, etc.
popularized the same on presenting his/her tech- Procedure of serial extraction:
nique of progressive extraction and he/she is con-
sidered to be the father of serial extraction. Hotz in Dewel’s method
1970 preferred to term it as ‘Guidance of Eruption’.
l The term serial extraction was defined by Dewel in
Three popular methods Tweed’s method
1969 as an orthodontic treatment procedure that
involves the orderly removal of selected primary
Nance’s method
and permanent teeth in a predetermined sequence.
l Serial extraction is defined by Tweed as the planned and i. Dewel’s method:
sequential removal of the primary and permanent teeth Dewel proposed a three-step serial extraction procedure
to intercept and reduce dental crowding problems. as follows:
l This procedure is usually initiated in the early mixed The sequence of proposed extractions: CD4
dentition period. It includes the planned extraction Step 1: Extraction of primary canines ‘C’ –
of certain deciduous teeth and later specific perma- between 8 and 9 years to create space for align-
nent teeth in an orderly sequence and predetermined ment of incisors.
pattern to guide the erupting permanent teeth into a Step 2: Extraction of first primary molars ‘D’ –
more favourable position. 1 year later, i.e. at 10 years of age to accelerate
eruption of first premolars.
Indications for serial extraction: Step 3: Extraction of first premolars, i. e. ‘4’ – to
i. Class I malocclusion showing harmony between permit the eruption of permanent canines in their
skeletal and muscular system place.
ii. Premature loss of primary teeth Modified Dewel’s technique:
iii. Arch length deficiency and tooth size discrepancies Wherein first premolars are enucleated at the
iv. In patients where growth is not enough to over- time of extraction of first deciduous molar es-
come the discrepancy between tooth material pecially in mandibular arch where canines
and basal bone)} erupt before first premolars.
ii. Tweed’s method:
[SE Q.11]
The sequence of proposed extractions: DC4
{v. Lingual eruption of permanent lateral incisors Step 1: Extraction of ‘D’ (deciduous first molar) – at
vi. Unilateral primary canine loss and shift to the 8 year of age.
same side Step 2: Deciduous canines are maintained till pre-
vii. Mesial eruption of canines over lateral incisor molars are in advanced eruptive stage.
and mesial drift of buccal segments Then both ‘C’ along with first premolars ‘4’ are
viii. Abnormal eruption direction and eruption sequence extracted simultaneously.
ix. Flaring of incisors, ectopic eruption of mandibu- When the permanent canines erupt, they migrate pos-
lar first primary molar and ankylosis teriorly into a good position. Any irregularities in
x. Abnormal resorption of second primary molar mandibular incisors if not too severe, get corrected
xi. Labial stripping or gingival recession, usually of themselves and they are also tipped lingually due to
lower incisors normal muscular forces.
xii. Deleterious oral habits iii. Nance’s method:
xiii. Crowded maxillary and mandibular incisors with This method is similar to Tweed’s method.
extreme labial proclination The sequence of proposed extractions: D4C
Contraindications: This method is basically modified Tweed’s method.
i. Mild class I malocclusions with minimum space Step 1: Extraction of ‘D’ (deciduous first molars) – at
deficiency and skeletal class II or III malocclusion 8 years of age.
Section | I  Topic-Wise Solved Questions of Previous Years 411

Step 2: Extraction of ‘4’ (first premolars) and ‘C’ Removable space maintainers:
(deciduous canines) simultaneously.
Postserial extraction therapy:
Most cases of serial extraction need fixed orth- {SN Q.12}
odontic appliance therapy for correction of axial
l Removable space maintainers are the appliances
inclination and detailing of occlusion.
designed for easy removal for cleansing and/or
Advantages of serial extraction:
adjustment.
i. Treatment is more physiologic, as it involves
l They are space maintainers that can be removed and
guidance of teeth into normal positions mak-
reinserted into the oral cavity by the patients.
ing use of physiologic forces.
ii. As the treatment is carried out, an early age
psychological trauma can be avoided.
ii. Reduces duration of multibanded fixed treat- Classification of removable space maintainers
ment as well as retention period. A. Removable appliances can be classified as:
iii. Reduced risk of caries due to better oral i. Functional
hygiene. ii. Nonfunctional
iv. More stable results – as tooth material and iii. With clasps
arch length are in harmony. iv. Without clasps
v. It lessens the cost of treatment. B. Brauer classified removable dentures for children
Disadvantages of serial extraction: as follows:
i. Prolong treatment time and follow up, as the Class 1: Unilateral maxillary posterior
treatment is carried out in stages spread over Class 2: Unilateral mandibular posterior
2–3 years. Class 3: Bilateral maxillary posterior
ii. Regular patient visits – cooperation of Class 4: Bilateral mandibular posterior
patient is needed. Class 5: Bilateral maxillary anterior posterior
iii. Tendency to develop tongue thrust due to cre- Class 6: Bilateral mandibular anterior posterior
ated extraction spaces. Class 7: One or more primary of permanent anterior
iv. Serial extraction requires good clinical judge- Class 8: Complete primary
ment.
v. Extraction of buccal teeth results in deepening
of bite. {SN Q.12}
vi. This is not a definitive treatment, the axial (Indications:
inclination of teeth at the end of serial extrac- l They are indicated when aesthetics is of impor-
tion procedure requires short-term fixed ap- tance.
pliance therapy. l The abutment teeth cannot support a fixed appli-
vii. There is no single approach that can be uni- ance) due to
versally applied to all patients. Each patient
has to be assessed and a suitable extraction
time table planned. a. Expected early loss because of normal root
viii. Poorly executed serial extraction programme resorption.
can be worse than none at all. b. Previous injury or extended caries which has
ix. Serial extraction is not very useful in class II involved the pulp.
and class III malocclusion cases.
Q.5. Define space management and describe removable
space maintainers in detail their indications, contrain- {SN Q.12}
dications, advantages and disadvantages. l In a cleft palate patient.
Ans. l The child has reached a mental age of 2.5 years and
all the primary teeth have erupted.
l The term space maintenance was coined by J.C. Brauer l The permanent teeth are not fully erupted for the
in 1941. It is defined as the process of maintaining a adaptation of bands.
space in a given arch previously occupied by a tooth or l Multiple loss of primary teeth.
a group of teeth.
412 Quick Review Series for BDS 4th Year, Vol 1

Contraindications: l Restore vertical dimension


l Lack of patient-parent cooperation l Help in mastication
l If the child has not attained a mental age of l Stimulate eruption of underlying tooth
2.5 years l Elaborate skills and instrumentation are not required
l If the patients are allergic to acrylic materials l Alterations made easily without changing the
l Epileptic patients appliance
l Children with possible caries activity Disadvantages of removable space maintainers:
Components of removable appliances: l May be lost or broken by the patient.

l Acrylic plate with extensions on to the edentu- l Not suitable in uncooperative patients; they may

lous space not wear them regularly.


l Clasps for retention l Lateral jaw growth may be hampered.

l Occlusal rests if first permanent molars are to l May be allergic to underlying tissues.

be clasped
Q.6. Define space maintainers. Describe about fixed
l Acrylic teeth, if the appliance is functional
space maintainers and enumerate fixed space maintain-
Removable appliances for multiple loss of teeth
ers used for bilateral loss of deciduous molars.
Acrylic partial denture:
When the number of missing teeth prevents the use Ans.
of fixed partial denture, a removable partial denture Space maintainers are devices used to maintain the space
becomes a restoration of necessity. following the loss of deciduous tooth/teeth, so as to guide
Indications: the unerupted tooth into a proper position in the arch.
l Excessive span length or
l Inability to achieve adequate retention for a fixed According to Boucher, it is a fixed or removable appli-
prosthesis ance designed to preserve the space created by the prema-
l Congenital malformations that result in only a ture loss of a primary tooth or a group of teeth.
few widely spaced permanent teeth
l Injuries that have caused multiple teeth and often
[SE Q.5]
alveolar bone to be lost {Various fixed space maintainers are as follows:
Three major objectives concerned in an adoles- I. Band and loop
cent patient who needs removable partial denture II. Crown and loop
are III. Lingual arch
l The restoration of functions of mastication and IV. Palatal arches
speech A. Nance’s palatal holding arch
l The restoration of dental and facial aesthetics B. Trans-palatal arch
l The preservation of remaining teeth and their sup- V. Distal shoe appliance
portive tissues VI. Fixed appliances for anterior space maintenance
Removable distal shoe maintainer: VII. Bonded space maintainers
l The acrylic removable appliance can be consid- Classification of fixed space maintainers according to
ered if one or both second primary molars are lost Hinrichsen:
at a short time before the eruption of the first per- l Fixed space maintainers
manent molars. i. Class I
l An ‘immediate’ acrylic partial denture with an a. Nonfunctional types – bar type and loop type.
acrylic distal shoe extension successfully guides b. Functional types – pontic type, lingual arch
first permanent molar into position. type and Nance’s holding arch.
l The tooth to be extracted is cut away from the ii. Class II
stone mode and a depression is cut into the stone a. Cantilever type (distal shoe, band and loop)
model to allow the fabrication of the acrylic The detail description of various fixed space maintain-
extension. ers is as follows:
l The acrylic will extend into the alveolus after the I. Band and loop space maintainer:
removal of the primary tooth. The extension may It is a unilateral, nonfunctional, passive, fixed ap-
be removed after the eruption of the permanent pliance indicated for space maintenance in the
tooth. posterior segments when single tooth is lost.
Advantages of removable space maintainers: Indications:
l Easy to clean and permit maintenance of proper i. Usually for preserving the space created by
oral hygiene premature loss of single primary molar
Section | I  Topic-Wise Solved Questions of Previous Years 413

ii. Bilateral loss of single primary molar before iii. Impression making and cast preparation:
eruption of permanent incisors l Make compound index to lubricate tooth and
iii. It is also indicated when second primary molar is do not overheat the compound. It should
lost after the eruption of first permanent molar completely cover the occlusal surface of
Advantages: band, extending 2 mm past the band.
l Effective space maintainer for unilateral loss of l If index is too large, it will be dislodged when
single tooth in buccal segments the impression tray with alginate is placed.
l Adjusts easily to accommodate the changing den- l Take an alginate impression. Compound in-
tition dex should come off the tooth and be embed-
l Less chair side time, if preformed bands are used ded in the alginate impression.
l Economical l Check whether the occlusal margin of the
Disadvantages: band is clearly reproduced. Remove the band
l Decalcification under bands and orient in compound index. Stabilize band
l Does not prevent the continuing eruption of with sticky wax.
opposing teeth l Pour the cast using white plaster, if you are
l Does not restore chewing function going to abrade cast or else use stone. Pre-
l Limited to maintenance of single tooth loss pare a flat base so that cast does not rock
Procedure for placement of band and loop space main- during the fabrication of appliance.
tainer: iv. Loop fabrication:
i. Design of wire loop l The mesial end of the loop contacts the distal
ii. Selection and adaptation of band surface of the tooth mesial to space, at a point
iii. Impression making and cast preparation just below the height of contour. The wire
iv. Loop fabrication should be above the gingiva at the point of
v. Soldering contact with the abutment tooth.
vi. Polishing l Central portion of the loop is shaped wide
vii. Cementation enough to allow the full eruption of the per-
i. Design of the wire loop: manent tooth.
l Mesial end of the loop should start distally l The buccolingual width of a maxillary pre-
from lingual to buccal to allow lateral shift of molar is 9 mm. The loop should be contoured
canine. to follow the edentulous ridge, but l mm off
l Loop should be wide enough to allow erup- the tissue. The anterior curve of the loop is
tion of premolar. shaped to approximate the shape of the distal
l Distal ends of the loop should overlap the surface of the abutment tooth and to match its
mesial one-third of the buccal and lingual width.
surfaces of the band. v. Soldering:
ii. Band selection and adaptation: l Stabilize the wire in position on the cast with
l A range of preformed bands from 1 to 32 wax. Remove all particles of wax, plaster and
sizes are available commercially depending debris from soldering site.
upon the mesiodistal width of the tooth for l The solder encircles the wire and extends to
maxillary and mandibular arches. the full length of the contact area between the
l Select the smallest band that will fit over the wire and the band.
height of contour of the tooth. l The occlusal and gingival margins of the
l The selected band is seated with digital pressure. band are free of solder.
l Utilizing a band seater ensure that the band is l The surface of the solder joints are free of
fully seated. Adapt margins to the tooth mor- pitting and voids. The wire mesial to the sol-
phology by utilizing the band adapter. der joints has not been annealed.
l Occlusal margin of band is apical to the vi. Polishing:
proximal ridges. Gingival margin of the band l Use a heatless green stone to reduce the distal
is in the gingival sulcus. Band is snugly ends of wire to form a smooth curve continu-
adapted to the tooth’s surface. ous with the band.
l Utilize the band removing pliers to remove l Appliance is finally polished with gold rouge
the band. or rag wheel later it is cleaned and dried.
414 Quick Review Series for BDS 4th Year, Vol 1

vii. Cementation: l Serves as a space maintainer for more than one


l The band and loop space maintainer is fi- succedaneous tooth in the arch
nally cemented on the tooth.} Disadvantages:
II. Crown and loop space maintainer: l Decalcification of the teeth.

Fabrication: l Arch wire may become embedded into the soft

l Abutment tooth is prepared for stainless steel tissue.


crown. l Wire may be distorted by masticatory forces and

l Properly contoured crown is seated on the move teeth into undesirable positions.}
prepared tooth. IV. Palatal arch appliances:
l A compound impression is taken and the A.  Nance’s palatal holding arch:
stainless steel crown is placed in the impres- Indications:
sion and a working model is prepared. l Maintaining the maxillary first permanent

l Then a loop is made as explained above for molars in place when there is bilateral pre-
the band and loop space maintainer and sol- mature loss of primary teeth.
dered. l Combined with habit breaking appliance by

l The appliance is removed from the working incorporating spurs in the acrylic button.
model, finished, polished and cemented to the Nance’s arch fabrication:
tooth. l Bands are fitted on molars. Impression is

taken in compound/alginate.
[SE Q.12] l A working model is prepared and an arch

{III.   Lingual arch space maintainer: wire is adapted such that it will traverse the
l It is a bilateral, nonfunctional, passive or palatal vault.
active and mandibular fixed appliance. l Adapted arch wire is soldered to bands.

l Most effective for space maintenance and minor l Acrylic button is added to embed the wire; it

tooth movements in lower arch. provides anchorage and prevents mesial


Indications: drifting of anteriors.
l Bilateral single or multiple tooth loss in mandible l Completed arch is now ready for try-in and

l Not recommended when primary incisors still cementation.


present Advantages:
Lingual arch fabrication: l Very effective.

l Arch wire should contact the erupted permanent Disadvantages:


incisors at the cingulum about 1–1.5 mm above l Soft tissue irritation.

gingival margin.
l The solder joint should be in mid-third and paral-

lel to band. {SN Q.14}


l Arch wire should be below the plane of occlusion
B. Transpalatal arch appliance:
in the posterior segment. l Transpalatal arch (TPA) was introduced by
l Fit molar bands on the compound/alginate im-
Robert A. Goshgarian of Waukegan, Illinois.
pression. Check for passivity on the model and in Indications:
the mouth before cementation. l When one side of the arch is intact and sev-
Modifications: eral primary teeth on the other side are
With spurs, i.e. projections of the wire used as stop- missing.
pers, and U bends as in Hotz lingual arch. l When correction of molar rotation, molar
Advantages: stabilization and anchorage and molar dis-
l Excellent source of anchorage, because it incor-
talization as well as other molar movements
porates resistance of several teeth. are needed.
l Allows free individual movement of teeth while
Contraindications:
maintaining space l Certain class II malocclusions in which up-
l Causes little or no inconvenience to the patient
per first premolars are removed.
l Less bulky than acrylic space maintainer
l In class III nonsurgical cases.
l Less conspicuous than other space maintainers
Section | I  Topic-Wise Solved Questions of Previous Years 415

Fabrication: l Construction of loop:


l Molar bands are fitted to the maxillary teeth. Using a 0.040-inch wire, the tissue bearing loop is
l Alginate impressions are made. then contoured extending distally and into the
l Bands removed from the teeth and placed in prepared opening on the model. The free ends of
the impression and secured and work model the loop are soldered to the band or directly to the
poured. crown in some cases.
l TPA is made with SS wire maintaining 1–1.5 mm l Position and width of the distal extension:

clearance in the palatal area. The design and placement of the distal extension
l Right-angled bands are placed in the arch, so of the appliance will differ for upper and lower
that the wire follows the lingual contour of the arches, as the eruption patterns of the mandibular
molar bands. and maxillary first permanent molars differ.
l The wire should initially contact the molar l In the lower arch, the contact area of distal exten-

band at the mesiolingual line angles to facili- sion should have slight lingual position over the
tate the production of rotational movements. crest of the alveolar ridge in order to engage the
l After TPA has been formed, it is secured in mesial contact area of the first permanent molar as
palatal area using modelling clay. TPA at- it begins its mesial and lingual movements. The
tached to the molar band by low-fusing solder. contact area of the distal extension of the maxil-
l Bands removed from model and the entire ap- lary appliance should be slightly buccal to the
pliance is smoothened, polished and disin- crest of the alveolar ridge.
fected. l These considerations are important in preventing

l Activation of this arch is done with Weingart the erupting permanent molar from slipping con-
plier. tact with the appliance.
Clinical problems: l The determination of the proper length of the dis-

l Soft tissue irritation, most common tal extension of the appliance is essential.
l Slight grooving and ulceration of tongue l The gingival extension of the appliance should

V. Distal shoe space maintainer: be constructed to extend about 1 mm below


l Distal shoe appliance is otherwise known as the intra the mesial marginal ridge of the first perma-
alveolar appliance. nent molar or just sufficient to ‘capture’ its
l The appliance in practice now is Roche’s distal shoe mesial surface as the tooth erupts and moves
or modifications of it using crown and band appli- forward.
ances with a distal intragingival extension. l Before final placement of the maintainer in the

Indications: mouth, a radiograph is taken to determine whether


l It is indicated when the second deciduous molar is the tissue extension of the appliance is in proper
extracted or lost before the eruption of first per- relationship with the unerupted first permanent
manent molar. molar. Final adjustments in length and contour of
Contraindications: the distal shoe can be made at this time.
l Inadequate abutments due to multiple loss of l According to Barber, the appliance has become

teeth. controversial in recent years due to reports of


l Poor oral hygiene or lack of parent and patient trauma and damage to the unerupted permanent
cooperation. teeth by appliance or procedure and also the nor-
l Medically compromised patients. mal eruption of the lower first permanent molar
Method of fabrication: rarely contacts the root surface of the second de-
l A stainless steel band is adopted using the first ciduous molar and does not use the root for erup-
primary molar as an abutment. tion guidance at all.
l If the morphology of the tooth does not permit VI. Fixed appliances for space maintenance in primary
easy placement and adaptation of band then stain- incisor region:
less steel crown provides a desirable contour for A. Fixed bridges and cast overlays:
the placement of stainless steel band. The band is l In the past, fixed bridges and the cast gold over-

placed over the stainless steel crown or abutment lays and loop were sometimes used for space
tooth. maintenance in the primary incisor area.
l A compound impression is made, the band is re- l Now economic considerations of both dentist

moved and placed in the impression and a stone and patient usually preclude the use of gold
model is prepared. appliances.
416 Quick Review Series for BDS 4th Year, Vol 1

B. Aesthetic anterior space maintainers: anchored in place at the canine area and then sol-
l It was described by Steffen, Miller and Johnson dered to the crown or band.
in 1971. It is a simple method of constructing a iv. Functional maintenance of arch length:
space maintainer and also provides an aesthetic Norman P. Martinez and Henry G. Elsbach (1984)
component. introduced a technique to maintain arch length utiliz-
l The space maintainer also consists of a plastic ing fixed functional space maintainer that can be con-
tooth processed on to a lingual arch which, in structed by direct or indirect method.
turn, is attached to bands for the molars. Indications:
VII. Bonded space maintainer: l When the space for a permanent tooth should be

l The loop that is similar to band and loop space maintained for 2 years or longer.
maintainer design is bonded with resin to buccal l To avoid supraeruption of a tooth from the op-

and lingual surfaces of both the abutment teeth. posing arch.


The wire passes from one abutment to the other l To improve the physiology of a child’s masticatory

crossing the alveolar ridge. system and restore dental health optimally.
Advantages: Contraindications:
l Made more easily Same as that for all other space maintainers
l Better gingival health v. Glass fibre-reinforced composite resin space main-
l Economical tainer:
Disadvantages: l Glass fibre-reinforced composite resin space main-

l Difficult to retain due to shearing forces of tainer is a translucent-coloured, semimanufactured


occlusion. product made of glass fibres, thermoplastic poly-
l Flexure in function will debond the space mer, light-curing resin matrix for reinforcing dental
maintainer. polymer.
l Not easy to adjust. l It is made of unidirectional fibres which increase

Other fixed appliances: the strength and stiffness of the final product per-
i. Bar type space maintainer pendicular to the direction of the fibres.
l A bar is attached to the mesial aspect of a crown l It is also used in frames of bridges and crowns,

or band on the second deciduous molar or first resin-bonded bridges, permanent splinting in oc-
permanent molar and on the distal aspect of the clusal and palatal sides, removable dentures and
first deciduous molar. devices and for reinforcing thick areas in remov-
l It may take the following forms: able devices.
a. A bar touching the next tooth in the arch Indications:
thus restoring the former contact point. l Unilateral loss of 1 or 2 teeth in maxillary or

b. A bar bent into an ‘S’ shape before touching mandibular arch


the next tooth. l In case of metal allergy

c. A bar attached to the next tooth through a Advantages:


ring soldered to a band or crown. l Time-saving, need for lab procedure elimi-

d. A bar which passes into a tube soldered to a nated


band or crown on the next tooth or vice l Single sitting procedure

versa. l Easy to apply and reliable adhesive bonding

e. A bar incorporating a screw to reopen space. l Easy to clean

f. The coil spring-ligature space maintainer. l Aesthetic

ii. Broken stress type functional space maintainer: Disadvantages:


l It was described by Graber. It may prevent intol- l Cost

erable loads from being thrust on the supporting l Invasive technique for tooth preparation

teeth. Procedure:
l The stress breaker should be designed to allow l Grooves drilled in a mesiodistal direction on

vertical movement of the supporting teeth con- abutment tooth for reinforcing the retention.
sistent with normal functional demands and, to l Space length is measured and a preimpregnated

a lesser degree, adjustive labial or lingual glass fibre is cut in proper length to the size after
movement. intraoral measurement.
iii. Modified loop appliance: l The prepared dental surfaces were cleaned with

l Stainless steel crown or band is adapted to the pumice, etched by using 35% phosphoric acid
tooth. An ‘L’ loop is adapted and contoured and and then viewed.
Section | I  Topic-Wise Solved Questions of Previous Years 417

l A thin layer of flowable composite is applied on c . Jack screw


to the tooth surfaces or cavity without light. Fi- d. Sling shot
bres are inserted and positioned simultaneously i. Fixed space regainers:
on to the intended area. a. Jaffe’s appliance
l After preliminary curing had been done in both l An appliance for certain minor tooth movement

teeth, the restorative composite is further cured by was described by Paul E. Jaffe in 1963.
40 s exposure at several points. l It is useful in the presence of ankylosed tooth,

l Excess material is removed and the occlusion is early loss of a deciduous molar or an extraction,
carefully checked. Appropriate polishing should filling of adjacent segments into proximal dental
be done. area.
l Movement is obtained by the use of light spring
Q.7. Define space maintainers and objectives of space pressure against a sliding section or arch.
management. Classify space regainers. Write in detail b. Gerber space regainers
about space regainers. l Gerber’s appliance may be fabricated directly in

Ans. the mouth or in a relatively short appointment


period and requires no lab work.
[SE Q.9]
{According to J.C. Brauer, the term space maintainer re-
fers to an appliance designed to retain a given area or space,
generally in the primary and mixed dentitions.
or
According to Boucher: Space maintainer is a fixed or
removable appliance designed to preserve the space created
by the premature loss of a tooth. 1st premolar
Objectives of space maintenance are as follows:
l Preservation of primate space. E
l Preservation of the integrity of the dental arches and Gerber space
regainer
normal occlusal planes.
l Aid in aesthetics and phonetics.

l Space maintenance is necessary in early loss of pos-

terior primary teeth because early loss contributes to


the development of occlusal disharmonies. 1st molar
l When space is progressively lost, the therapy should

be considered to regain it so that additional dishar-


monies do not develop.
l A seamless orthodontic band or crown is selected
(SE Q.9 and SN Q.1) for the tooth to be distalized.
l This space regainer consists of ‘U’ shaped hollow
Space regainers: tubing soldered or welded to mesial aspect of first
Some of the appliances that can be used to regain molar which is to be moved distally.
space are classified as follows:} l ‘U’ shaped rods with open coil springs of ade-
{(i. Fixed: quate length are fitted into the above tubing so
a. Open coil that they contact mesial aspect of first molar to be
b. Gerber moved distally.
c. Hotz l The forces generated by compressed coil springs
d. Sectional bring about a distal movement of first molar.
e. Lip bumper)} Advantages:
l Can be fabricated directly in the mouth
[SE Q.9]
l Relatively short appointments

{ii. Removable: l Requires no lab work

a . Free-end loop c. Hotz lingual arch:


b. Split saddle l It is another method for distalization of molars.
418 Quick Review Series for BDS 4th Year, Vol 1

l It utilizes the looped Hotz lingual arch. This is l After the desired movement of the permanent
appropriate in a situation where the lower first molar has been attained, the appliance may be
permanent molar has drifted mesially, but the used as a space maintainer by soldering the acti-
premolar or cuspid has not drifted distally. vator portion of the spring to the guide wire in its
l But there must be X-ray evidence that there is suf- passive position or by filling in the edentulous
ficient space between first molar and developing region with additional resin.}
second molar. ii. Removable space regainers
d. King’s appliance a. Free-end loop spring space regainer
l King in 1977 described an appliance for regaining l The appliance utilizes a labial arch wire for stabil-

of space in both maxillary and mandibular arch. ity and retention with a back action loop spring
l The anchorage unit for the mandibular arch is constructed of No. 0.025 wire. The base of the
basically a fixed lingual arch with bands fitted appliance is made of acrylic resin.
on the first deciduous molar of the treatment l Movement of the permanent molar is achieved by

side and the first permanent molar on the op- activating the free end of the wire loop at certain
posite side. Then a wide siamese edgewise intervals of time.
bracket is spot-welded to the buccal surface of l The lower arch has a shorter wire loop resulting in

the primary molar band and the completed an- less distortion when the child inserts the appli-
chorage unit is cemented in place. The anchor- ance.
age unit must be modified for treatment in the b. Split-block or split saddle space regainer
maxillary arch. l The appliance differs from the free-end spring

e. Anterior space regainer: type, in that the functional part of the appliance
l Bayardo in 1986 described an anterior space re- consists of an acrylic block that is split buccolin-
gainer utilizing direct bond technique. gually and joined by No. 0.025 wire in the form
l The enamel of the labial surface of left central and of a buccal and lingual loop.
right lateral incisors was etched with 35% phos- l The appliance is activated by periodic spreading

phoric acid and labial tube was individually of the loops. The activator block is split with a
bonded to each abutment tooth and a 0.014 inch disk after the appliance has been processed.
round wire is inserted in an open coil spring and c. Space regainer utilizing jack-screw
activated, thus causing space regaining.} l This appliance incorporates an expansion screw in

the edentulous space.


(SE Q.9 and SN Q.13) l Space is opened by expanding the plates antero-

posteriorly.
{(f. Lip bumper: d. Sling shot space regainer
l Most easily used for space regaining procedure
l This appliance consists of a wire elastic holder
in which bilateral movement is desired.
with hooks, instead of a wire spring that transmits
l It consists of a heavy labial arch wire over which
a force against the molar to be distalized.
an acrylic flange is prepared in the anterior region
l Since the distalizing force is produced by the
such that it does not contact the lower anterior
elastic stretched on the middle of the lingual sur-
teeth. It is used to relieve the lip pressure.
face of the molar to be moved the other is ar-
l This pressure can be used to distalize the molars
ranged in the same position on the buccal surface
by:
of the molar. The elastic can be changed once
i. Incorporating loops in the arch wire just before
each day.
it enters the buccal tube.
Other removable appliances are
ii. Utilizing a coil spring.
l Hawley’s appliance with dumbbell spring
l It can also be used unilaterally.)}
l Hawley’s appliance with split acrylic

l Hawley’s appliance with elastics


[SE Q.9]
{g. Fixed, loop-spring space regainer: Q.8. Define and classify space maintainers. Write in
l This appliance differs from other types mainly in detail about distal shoe space maintainers.
the design of spring activation.
l This appliance resists breakage and provides a Ans.
satisfactory method of moving the molar distally. [Same as LE Q.1]
Section | I  Topic-Wise Solved Questions of Previous Years 419

Q.9. Define space maintainers and space management. crown provides a desirable contour for the placement
Classify space maintainers. Write in detail about erup- of stainless steel band.
tion guidance appliance. l A compound impression is made, the band is re-

moved and placed in the impression and a stone


Ans. model is prepared.
[Same as LE Q.1] Construction of loop:
l Using a 0.040-inch wire, the tissue bearing loop is
Q.10. Discuss the indications, contraindications and then contoured extending distally and into the
classification of space maintainers. prepared opening on the model. The free ends of
the loop are soldered to the band or directly to the
Ans. crown in some cases.
[Same as LE Q.1] Position and width of the distal extension:
l The design and placement of the distal extension
Q.11. What are the indications for space maintainers. of the appliance will differ for upper and lower
Describe briefly the various types of the same. arches, as the eruption patterns of the mandibular
and maxillary first permanent molars differ.
Ans.
l In the lower arch, the contact area of distal exten-

[Same as LE Q.1] sion should have slight lingual position over the
crest of the alveolar ridge in order to engage the
Q.12. A 8-year-old child reports to the clinic with bilat-
mesial contact area of the first permanent molar as
eral loss of primary molars in the arch. What is the line
it begins its mesial and lingual movements. The
of approach.
contact area of the distal extension of the maxil-
Ans. lary appliance should be slightly buccal to the
crest of the alveolar ridge.
[Same as LE Q.6] l These considerations are important in prevent-

ing the erupting permanent molar from slipping


contact with the appliance. The width should
SHORT ESSAYS: closely approximate the normal contact area of
Q.1. Distal shoe space maintainer. the distal surface of the second primary molar
being replaced.
Ans.
Length of the distal extension (horizontal bar):
l Distal shoe space maintainer is otherwise known as the l The determination of the proper length of the dis-

intra-alveolar appliance. tal extension of the appliance is essential.


l The appliance in practice now is Roche’s distal shoe or l If the second primary molar is still present to

modifications of it using crown and band appliances serve as a guide on the working model then the
with a distal intragingival extension. problem is simplified. In such cases it should be
Indications: maintained, if possible until the appliance is ready
l It is indicated when the second deciduous molar is to be sealed.
extracted or lost before the eruption of first perma- l If the second primary molar is missing, then it is

nent molar. recommended to measure radiographically the


Contraindications: distance between the distal surface of the first
l Inadequate abutments due to multiple loss of teeth. primary molar and mesial surface of the un-
l Poor oral hygiene or lack of parent and patient coop- erupted first permanent molar.
eration. If the opposite second primary molar is present,
l Medically compromised patients. For example, pa- then record the mesiodistal width of the tooth and
tients with congenital diseases and kidney problems. compare that with the radiographic measurement.
l Congenitally missing first permanent molar which is Depth of the gingival extension (vertical bar):
a rare situation. l The intra-alveolar depth of the gingival extension

Method of fabrication: should be accurate. If the extension is left too


l A stainless steel band is adopted using the first long, possible harm to the developing second pre-
primary molar as an abutment. molar may result. If the extension is too short, the
l If the morphology of the tooth does not permit easy first permanent molar could erupt underneath the
placement and adaptation of band then stainless steel appliance.
420 Quick Review Series for BDS 4th Year, Vol 1

The gingival extension of the appliance should be


l Nance’s arch fabrication:
constructed to extend about 1 mm below the me- l Bands are fitted on molars. Impression is taken in
sial marginal ridge of the first permanent molar or compound/alginate.
just sufficient to ‘capture’ its mesial surface as the l A working model is prepared and an arch wire is
tooth erupts and moves forward. adapted such that it will traverse the palatal vault.
Appliance placement: l Adapted arch wire is soldered to bands.
l The appliance is removed from the model and the l Acrylic button is added to embed the wire; it pro-
V of the tissue extension is filled in and soldered vides anchorage and prevents mesial drifting of
with pieces of 0.040 inch wire. anteriors.
l A knife edge is formed at the apex of the V, if the l Completed arch is now ready for try-in and cemen-
second primary molar has previously been ex- tation.
tracted and the extraction site has healed. Advantages:
l The sharpened distal shoe may be passed through l Very effective
a sterilized and anesthetized area of the ridge. If l Arch stabilization
the appliance is delivered at the time of extraction, Disadvantages:
the intragingival extension is just polished and not l Soft tissue irritation
sharpened. l Pressure effects
l Before final placement of the maintainer in the

mouth, a radiograph is taken to determine whether Q.3. Describe how labial frenum affects the teeth in the
the tissue extension of the appliance is in proper arch.
relationship with the unerupted first permanent
Ans.
molar. Final adjustments in length and contour of
the distal shoe can be made at this time. l Midline diastema or median diastema refers to anterior
l According to Barber, the appliance has become mid-line spacing between the two maxillary central
controversial in recent years due to reports of incisors.
trauma and damage to the unerupted permanent l The prevalence of midline diastema is said to be almost
teeth by appliance or procedure. 48% in 9–11 years. It reduces to 7% in the 12–18 years
age group.
Q.2. Nance’s appliance.
Aetiology:
Ans.
i. Normal developmental phase (ugly duckling
l Nance’s palatal holding arch is a bilateral, nonfunc- stage)
tional, passive, and maxillary fixed appliance that does ii. Deep overbites
not contact the anterior teeth, but approximates the an- iii. High labial frenal attachments
terior palate via an acrylic button that contacts the pala- iv. Mesiodens (supernumerary teeth)
tal tissue, which provides resistance to the anterior v. Missing lateral incisors or peg laterals
movement of posterior teeth in a horizontal direction. vi. Macroglossia
Indications: vii. Pathology due to cysts
l Maintaining the maxillary first permanent molars in viii. Habits such as thumb sucking or finger sucking
place when there is bilateral premature loss of pri- ix. Genetic
mary teeth with no loss of space in arch and a favour- Diagnosis
able mixed dentition analysis. l Proper history and clinical examination is necessary
l Combined with habit breaking appliance by incorpo- to make a diagnosis
rating spurs in the acrylic button. l Blanch test is performed to diagnose high frenal
Design of the wire loop: attachments
l The arch wire extends anteriorly without touching l Intraoral periapical radiographs to be taken in case of
against the surface of the primary molars. supernumerary tooth
l At the rugae area, a small V-shaped bend should be in- Treatment
corporated in the wire, which is approximately 1–2 mm Treatment of midline diastema is done in three phases:
away from the soft tissue. The bend will enhance the i. Removal of cause:
retention of acrylic to the wire. l Habits should be eliminated using fixed or
l The acrylic button, 0.5 inch in diameter, is placed removable habit breakers.
usually on the descending portion of the palatal vault l Unerupted mesiodens should be extracted.
1–2 mm below the incisive papilla. l Frenectomy.
Section | I  Topic-Wise Solved Questions of Previous Years 421

ii. Active treatment:


{SN Q.9}
l Removable appliances such as finger springs,

or split labial bow with a Hawley’s appliance Aetiology of anterior crossbite:


can be used to correct midline diastema. i. Traumatic injuries to primary dentition and lin-
l Fixed appliances can also be used to correct gual displacement of permanent tooth bud
midline diastema. ii. Supernumerary teeth
iii. Retention: iii. Inadequate arch length
After the active treatment, retention is mandatory iv. Habit of biting upper lip
to prevent relapse. v. Repaired cleft lip
This is achieved by:
l Hawley’s retainer

l Banded retainers vi. Functional crossbite due to premature tooth


l Lingual bonded retainers contact
iv. Cosmetic restorations: vii. Skeletal – increased mandibular growth (true
Closure of diastema using composites class III)
viii. Delayed eruption of the primary dentition
Q.4. Classify anterior crossbite and how do you correct ix. An over-retained, necrotic, pulpless primary
them. tooth or root
Ans. x. A sclerosed bony or fibrous tissue barrier
caused by premature loss of a primary tooth
Crossbite is defined by Graber as a condition where one or Diagnosis:
more teeth may be abnormally malposed buccally or lingually l Dental crossbites are diagnosed by clinical
or labially with reference to the opposing tooth or teeth. examination.
or l If a skeletal discrepancy is suspected, study
Definition of American Association of Orthodontists: An models and a lateral cephalometric radiograph
abnormal relationship of a tooth or teeth to the opposing will help in diagnosis and treatment planning.
tooth or teeth, in which normal buccolingual or labiolingual Treatment:
relationships are reversed. l Occlusal equilibration
Classification of crossbite l Tongue blade therapy
I. Based on location: l Lower inclined plane (Catalan’s appliance 45°)
l Anterior crossbite
l Reverse stainless steel crown
a. Single tooth l Composite inclines
b. Segmental l Removable Hawley’s appliance with ‘Z’-
l Posterior crossbite
spring
a. Unilateral l Fixed appliance (lingual/palatal arch) with
b. Bilateral auxiliary springs
II. Based on nature of crossbite: Tongue blade therapy:
a. Skeletal crossbite l This method is used to correct a developing ante-
b. Dental crossbite rior crossbite involving a single tooth.
c. Functional crossbite l The treatment of anterior crossbite at the age of
Anterior crossbite: 8 years indicates that it is correction of developing
l Malocclusion resulting from the lingual position
crossbite.
of the maxillary anterior teeth in relationship with l Tooth in crossbite is in the initial stage of eruption
the mandibular anterior teeth. with a minimal degree of in-locking and can often
Fundamentals: be repositioned by this method.
l Anterior crossbite involving all the anterior
l The child is instructed to place the tongue blade,
teeth may indicate a skeletal growth problem which can be made by uniting 2–3 ice cream
and developing class III malocclusion. sticks behind the inlocked tooth, and to exert pres-
l Anterior crossbite of one or more of the perma-
sure on the tooth using the chin as fulcrum. The
nent incisors may be an evidence of localized pressure is directed towards the labial side. It
discrepancy. should be practiced at least 5 min per hour and as
l Generally, it involves one or more incisors or
often as possible during the day.
canines. It has to be corrected as soon as it is l The major limitation of this method is its total
diagnosed. There is little possibility of self- dependence on patient cooperation for frequency
correction. of performance and accuracy of placement.
422 Quick Review Series for BDS 4th Year, Vol 1

Inclined plane – Catalan’s appliance: l Minimum discomfort due to reduced bulk.


l An acrylic inclined plane cemented to the lower l The forces applied are light.
anterior teeth is another way to reposition one or l This method is the first choice for correction of
more inlocked anterior teeth. anterior crossbite.
l With this appliance, crossbite is corrected in 7–10
Q.5. Explain band and loop space maintainer construc-
days. If crossbite is not corrected within 7–10 tion procedure.
days, a different appliance should be considered.
Limitations: Ans.
l The physical activities of children wearing in-
[Ref LE Q.6]
clined plane should be sharply restricted to
minimize the possibility of injury to the teeth. Q.6. Mixed dentition analysis. Tanaka–Johnson analysis.
Teeth that occlude on an inclined plane are Ans.
especially vulnerable to avulsion or luxation
from a blow to the chin.
l Patient encounters problems in speech.
{SN Q.10}
l Inconvenience for eating as the posterior teeth

are not in contact. l Mixed dentition analyses are used to predict the size
l Worn for more than 4 weeks and failure to re- of unerupted tooth by using two dimensions of tooth
view in 7–10 days can lead to anterior open present in the mouth.
bite because of supraeruption of posterior l Hence, an assessment about the space needed or de-

teeth. ficiency in the arch can be calculated.


Reversed stainless steel crown: l Most of these mixed dentition analyses utilize the

l Anterior stainless steel crowns cemented back- dimensions of mandibular permanent central and
wards on the maxillary teeth can correct anterior lateral incisors as standards.
crossbite. l Mandibular incisors are generally used as standards

l Stainless steel crown needs to open the bite 2–3 because:


mm and establish at least a 25% overbite for suc- i. They erupt into the mouth earlier than maxillary
cessful treatment. When the child bites down on incisors and offer the earliest opportunity of mea-
lower teeth, the upper teeth are forced to move out surement.
towards the lips into their proper place. ii. These teeth are least susceptible for morphologic
l It is cemented on the child’s central incisor and it variations, hence less variable and more reliable
can correct the crossbite in 2–4 weeks. than maxillary incisors.
l If they worsen or fail to correct the crossbite, one The most commonly used mixed dentition analyses
of the other alternatives can be considered. are
l The limitation of this procedure is that, it is an a. Huckaba analysis
unaesthetic procedure. b. Moyer’s mixed dentition analysis
Removable Hawley’s appliance with Z-springs: c. Tanaka–Johnson analysis
l The most frequently used appliance is with acrylic

palatal coverage and wire clasps on the molars


and a short labial bow extending from canine to a. Huckaba analysis (Huckaba 1964):
canine having adjustment loops. l This analysis compensates for radiographic enlarge-
l The acrylic is usually extended to create posterior ment of tooth image in intraoral periapical radiograph.
bite plates. Method:
l If the child is motivated to wear continuously, i. Width of primary tooth on IOPA-Y1
crossbite is usually corrected in 7–10 days and ii. Width of its underlying successor on IOPA-XI
maximum it takes 6–8 weeks. iii. Width of primary tooth on the cast – Y
l Use of inclined plane is recommended only iv. Width of the unerupted permanent tooth – X
when the posterior teeth are absent for retaining The formula is X 5 YXl/Y1
a Hawley’s appliance with Z-springs. b. Moyer’s mixed dentition analysis:
l Patient compliance is necessary for successful l This mixed dentition analysis is simple to use.
correction of crossbite. It has the following advantages:
Advantages: l Minimal systematic error
l Ease of fabrication. l Even beginners can carry out this analysis with
l Simple design. equal reliability
Section | I  Topic-Wise Solved Questions of Previous Years 423

l Less time-consuming. l In the maxillary arch, half the sum of the mandibu-
l No special equipment or radiographs are lar incisors is still used, but 11 mm is substituted
required to perform this analysis. for 10.5 mm because the unerupted permanent
l This analysis can be completed in the mouth maxillary teeth are slightly larger.
and on study casts also.
Q.7. Tongue blade therapy.
l It can be carried out for both arches.

Technique: Ans.
l The combined width of the mandibular perma-

nent central and lateral incisors is measured.


This value is used in the probability chart (75%
of the value) and a value obtained. This value
gives the predicted width of unerupted canine
and premolars.
l Space available is measured with a brass wire Teeth
extending from the mesial side of first perma-
nent molar on one side, passing through the
buccal cusps and incisal edges of teeth to the
mesial side of opposite first permanent molar.
l The brass wire is straightened and measured
Clean
with Boley gauge. This gives the space avail-
able. The difference between the space avail-
able and space needed gives the discrepancy.
c. Tanaka–Johnson analysis:
This analysis is very useful because it requires no l Tongue blade therapy is used to correct a developing
additional radiographs or tables to predict tooth size. anterior crossbite involving a single tooth rather than a
The first step: developed crossbite.
l Determination of the available arch length. l Tooth in crossbite is in the initial stage of eruption with
l The distance from the mesial of the permanent a minimal degree of in-locking and can often be reposi-
first molar to the mesial of the contralateral per- tioned by this method.
manent first molar is measured by dividing the l The child is instructed to place the tongue blade (which
arch into several segments. can be made by uniting 2–3 ice cream sticks) behind the
l Each segment is measured over the contact points in-locked tooth, and to exert pressure on the tooth using
and incisal edges of the teeth. The segments are the chin as fulcrum.
added together to provide an approximation of the l The pressure is directed towards the labial side. It
total arch length. should be practiced at least 5 min per hour and as often
The second step: as possible during the day.
l Measurement of the width of four mandibular l The major limitation of this method is its total depen-
incisors. dence on patient cooperation for frequency of perfor-
l The width of four incisors are added together to mance and accuracy of placement.
determine the amount of room necessary for ideal
Q.8. Catalan’s appliances.
alignment.
l The mesiodistal width of the unerupted mandibu- Ans.
lar canine and the premolars in one quadrant is
l Catalan’s appliance or an acrylic inclined plane ce-
predicted by adding 10.5 mm to half the width of
mented to the lower anterior teeth is another way to re-
the four lower incisors.
position one or more in-locked anterior teeth.
The final step:
l This acrylic inclined plane is constructed on a stone
l Subtract the width of the lower incisors and the
model. The length of the inclined plane is preferably 1/4
two times the calculated premolar and canine
inch extending lingually at a 45° angle to the long axis
width (both sides) from the total area length ap-
of the lower incisor teeth.
proximation.
l Steeper the angle of inclined plane greater the forces
l If the result is positive, there is more space avail-
applied. Only the in-locked tooth should be in contact
able in the arch than is needed for the unerupted
with it and no other tooth should contact the inclined
teeth.
plane.
l If the result is negative, the unerupted teeth will
l The plane should not touch the palatal tissue and the
need more space than is available to erupt in ideal
posterior teeth should be out of occlusion by 2–3 mm.
alignment.
424 Quick Review Series for BDS 4th Year, Vol 1

l With this appliance, crossbite is corrected in 7–10 days. l Congenital malformations that result in
If crossbite is not corrected within 7–10 days, a different only a few widely spaced permanent teeth.
appliance should be considered. l Injuries that have caused multiple teeth and

Limitations: often alveolar bone to be lost.


l The physical activities of children wearing in- II. Removable distal shoe maintainer:
clined plane should be sharply restricted to mini- l If one or both second primary molars are lost

mize the possibility of injury to the teeth. at a short time before the eruption of the first
l Teeth that occlude on an inclined plane are espe- permanent molars, the acrylic removable ap-
cially vulnerable to avulsion or luxation from a pliance can be considered.
blow to the chin. l An ‘immediate’ acrylic partial denture with

l Patient encounters problems in speech. an acrylic distal shoe extension successfully


l Inconvenience for eating as the posterior teeth are guides first permanent molar into position. The
not in contact. tooth to be extracted is cut away from the stone
l Worn for more than 4 weeks and failure to review model and a depression is cut into the stone
in 7–10 days can lead to anterior open bite be- model to allow the fabrication of the acrylic
cause of supraeruption of posterior teeth. extension.
l The acrylic will extend into the alveolus after
Q.9. Fixed space regainers.
the removal of the primary tooth.
Ans. l The extension may be removed after the erup-

tion of the permanent tooth.


[Ref LE Q.7]
Q.11. Indications of serial extraction.
Q.10. Removable space maintainers for multiple loss of
teeth. Ans.
Ans. [Ref LE Q.4]
Removable space maintainers are the appliances designed Q.12. Lingual arch space maintainer.
for easy removal for cleansing and/or adjustment. Ans.
Indications:
l Removable dentures are indicated when aesthet- [Ref LE Q.6]
ics is of importance. Q.13. Willet’s appliance.
l The abutment teeth cannot support a fixed appli-

ances either because of expected early loss, due to Ans.


normal root resorption or previous injury or ex- [Same as SE Q.1]
tended caries which has involved the pulp.
l A cleft palate is to be closed with a denture. Q.14. Anterior diastema.
l The permanent teeth are not fully erupted for the
Ans.
adaptation of bands.
l Multiple loss of primary teeth. [Same as SE Q.3]
Classification: Q.15. Describe causes of midline diastema and its man-
Removable appliances can be classified as: agement.
a. Functional
b. Nonfunctional Ans.
c. With clasps [Same as SE Q.3]
d. Without clasps
Removable appliances for multiple loss of teeth are as Q.16. Median diastema. Treatment of mid-line dia-
follows: stema.
I. Acrylic partial denture: Ans.
When the number of missing teeth prevents use of
[Same as SE Q.3]
a fixed partial denture, a removable partial denture
becomes a restoration of necessity. Q.17. Define crossbite. Write briefly the treatment of
Indications: anterior crossbite at the age of 8 years.
l Excessive span length.
Ans.
l Inability to achieve adequate retention for a

fixed prosthesis. [Same as SE Q.4]


Section | I  Topic-Wise Solved Questions of Previous Years 425

Q.18. Explain any one of the fixed space maintainer. l The major limitation of this method is its total depen-
dence on patient cooperation for frequency of perfor-
Ans.
mance and accuracy of placement.
[Same as SE Q.5]
Q.5. Active space maintainers.
Q.19. Tanaka–Johnson analysis.
Ans.
Ans.
l Active space maintainers are categorized into two
[Same as SE Q.6] types:
i. Active fixed maintainers
Q.20. Inclined plane.
ii. Active removable maintainers
Ans. Active fixed maintainers:
l A case in which there is not enough space for a
[Same as SE Q.8]
lower second premolar, but there is a space be-
Q.21. Lower anterior inclined plane. tween the distally slanting first premolar and the
cuspid, and the first molar is slanting slightly
Ans.
mesially.
[Same as SE Q.8] l A band is made on the first permanent molar.

Q.22. Lingual arch holding appliance. Precious-metal tubes are soldered on the band
buccally and lingually.
Ans. l In this case a wire is bent to a U shape which will

[Same as SE Q.12] fit passively in both the buccal and lingual tubes.
l The anterior curved part of the ‘U’ should have a

reverse bend where it contacts the distal outline of


SHORT NOTES: the first premolar.
Active removable maintainers:
Q.1. Name fixed space regainers. l A removable wire and plastic space maintainer is

Ans. sometimes used for the active backward reposi-


tioning movement of a molar to allow a second
[Ref LE Q.1] premolar to erupt.
Q.2. Define serial extraction.
Q.6. Eruption guidance appliance.
Ans.
Ans.
[Ref LE Q.4]
l Eruption guidance appliance is one of the early designs
Q.3. Space maintainers. of distal shoe space maintainers made in cast gold or
Willet’s distal shoe.
Ans.
l This appliance is rarely used because of the increased

[Ref LE Q.2] cost of the materials, difficulties in tooth preparation


and more complicated fabrication procedures.
Q.4. Tongue blade therapy.
l The appliance in practice now is Roche’s distal shoe or

Ans. modifications of it using crown and band appliances


with a distal intragingival extension.
l Tongue blade therapy is used to correct a developing
anterior crossbite involving a single tooth rather than a Q.7. Blue-grass appliance.
developed crossbite.
Ans.
l Tooth in crossbite is in the initial stage of eruption with

a minimal degree of in-locking and can often be reposi- l This appliance has special wire that is connected to the
tioned by this method. upper right and left first molars that extend forward to-
l The child is instructed to place the tongue blade behind wards the roof of the mouth behind front teeth.
the in-locked tooth, and to exert pressure on the tooth l There is an acrylic or plastic wheel that rests against the

using the chin as fulcrum. roof of the mouth.


l The pressure is directed towards the labial side. It l This appliance is used for breaking the habit of tongue

should be practiced at least 5 min per hour and as often thrusting and thumb sucking, by spinning the wheel in-
as possible during the day. stead of performing the habit.
426 Quick Review Series for BDS 4th Year, Vol 1

Q.8. Genetic aspect of malocclusion. Q.12. Indications for removable space maintainers.
Ans. Ans.
l The aetiology of malocclusion is complex, multifacto- [Ref LE Q.5]
rial and polygenic. This is discussed in two ways:
Q.13. Lip bumper.
i. Inheritance of disproportion between size of the teeth
and jaws leading to crowding or spacing. Ans.
ii. Inheritance of disproportion in the size and shape of
[Ref LE Q.7]
the mandible and maxilla.
l The role of genetics in malocclusion is more obvious in Q.14. Trans-palatal arch appliance.
persons who have severe class II and class III maloc-
Ans.
clusion.
l Observations indicated that both the size and shape of [Ref LE Q.6]
the teeth are greatly affected by genetic factors in
Q.15. Lingual arch space maintainer – Indications.
twins. So, examination of old sibling can provide a
clue to the need for interception and early treatment of Ans.
malocclusion.
l Lingual arch holding appliance is a bilateral, nonfunc-
l The craniofacial complex is under the rigid control of
tional, passive/active and mandibular fixed appliance.
hereditary factors.
Indication of lingual arch holding appliance is as follows:
For example: Skeletal structures were more frequently
i. To preserve the space created by multiple loss of
transmitted from mother to son and facial height from
primary molars when there is no loss of space in
mother to offspring. Genetic correlation exits between
the arch.
father and children in mandibular dimensions.
ii. It helps in maintaining the arch perimeter.
l It is clear that genetics plays an important role in maloc-
iii. Bilateral loss of primary molars after eruption of
clusion.
lower lateral incisors.
Q.9. Aetiology of anterior crossbite. iv. Unilateral loss of primary molars after eruption
of lower lateral incisors.
Ans.
v. Minor space regaining.
[Ref SE Q.4] Q.16. Common procedure done in interceptive ortho-
Q.10. Name mixed dentition analysis. dontics.

Ans. Ans.
[Ref LE Q.2]
[Ref SE Q.6]
Q.17. Define preventive orthodontics.
Q.11. Inclined plane.
Ans.
Ans.
[Ref LE Q.2]
l Inclined plane or Catalan’s appliance is an acrylic in-
clined plane cemented to the lower anterior teeth is one Q.18. Moyer’s mixed dentition analysis.
way to treat localized anterior crossbite. Ans.
l This appliance corrects the crossbite in 7–10 days. If
crossbite is not corrected within 7–10 days, a different l This mixed dentition analysis is simple to use.
appliance should be considered. l Even beginners can carry out this analysis with equal
Limitations: reliability.
l The physical activities of children wearing in- l Less time-consuming.

clined plane should be sharply restricted to mini- l No special equipment or radiographs are required to

mize the possibility of injury to the teeth. perform this analysis.


l Patient encounters problems in speech and incon- l This analysis can be completed also in the mouth and

venience in eating. casts.


l Worn for more than 4 weeks and failure to review Technique:
in 7–10 days can lead to anterior open bite be- l The combined width of the mandibular perma-

cause of supraeruption of posterior teeth. nent central and lateral incisors is measured.
Section | I  Topic-Wise Solved Questions of Previous Years 427

This value is used in the probability chart (75% Q.20. Classification of space maintainers according to
of the value) and a value obtained. This value the anchorage and support available.
gives the predicted width of unerupted canine Ans.
and premolars.
l Space available is measured with a brass wire
[Same as SN Q.3]
extending from the mesial side of first permanent Q.21. Correction of a developing single tooth crossbite.
molar on one side, passing through the buccal
Ans.
cusps and incisal edges of teeth to the mesial side
of opposite first permanent molar. [Same as SN Q.4]
l The brass wire is straightened and measured with
Q.22. Aetiology of anterior crossbite.
Boley gauge. This gives the space available. The
difference between the space available and space Ans.
needed gives the discrepancy.
[Same as SN Q.4]
Q.19. Mention four indications of serial extraction.
Q.23. Catalan’s appliance.
Ans. Ans.
[Same as SN Q.2] [Same as SN Q.11]

Topic 14
Oral Habits
COMMONLY ASKED QUESTIONS
LONG ESSAYS:
1 . Write briefly on tongue-thrusting habit.
2. Define and classify oral habits and discuss aetiology and treatment of thumb-sucking habit.
3. What is mouth-breathing habit? How does it develop? Describe treatment plan of mouth-breathing habit in
children.
4. Classification, aetiology and deleterious effects of tongue-thrusting habit on teeth and associated structures and
their treatment. [Same as LE Q.1]
5. Define and classify oral habits. Write in detail about the aetiology, clinical features and management of thumb-
sucking habit. [Same as LE Q.2]
6. Define and classify oral habits. Discuss in detail thumb-sucking habit. [Same as LE Q.2]
7. Write the effects of oral habits on the growing jaws and dentition. How will you manage a case of mouth breath-
ing? [Same as LE Q.3]
8. Define and classify habits. Discuss the aetiology, clinical features and management of mouth breathing. 
[Same as LE Q.3]
9. Define and classify mouth-breathing habits. Explain in detail mouth-breathing habit and its treatment.
[Same as LE Q.3]

SHORT ESSAYS:
1 . Define and classify oral habits. [Ref LE Q.2]
2. Classify tongue-thrusting habit and describe the management of tongue thrusting. [Ref LE Q.1]
3. Clinical features of mouth-breathing habit. [Ref LE Q.3]
4. Masochistic habits.
5. Define thumb-sucking habit and its management. [Ref LE Q.2]
428 Quick Review Series for BDS 4th Year, Vol 1

6. Pacifier.
7. Bruxism.
8. Management of tongue-thrusting habit. [Same as SE Q.2]
9. Clinical manifestations of mouth-breathing habit. [Same as SE Q.3]
10. Aetiology of thumb sucking. [Same as SE Q.5]
11. Management of thumb sucking. [Same as SE Q.5]

SHORT NOTES:
1. Define oral habits. [Ref LE Q.2]
2. Define thumb sucking. [Ref LE Q.2]
3. Tongue thrusting. [Ref LE Q.1]
4. Mouth breathing. [Ref LE Q.3]
5. Define masochistic habits and mention two aetiological factors.
6. Vestibular screen indications.
7. Non-nutritive sucking habit.
8. Mention two advantages of oral screen.
9. Lip bumper.
10. Tests for mouth breathing. [Ref LE Q.3]
11. Oro-facial habits. [Same as SN Q.1]
12. Causative factors of thumb-sucking habit. [Same as SN Q.2]
13. Salient features of tongue-thrusting habit. [Same as SN Q.3]
14. Causes of mouth breathing. [Same as SN Q.4]

SOLVED ANSWERS
LONG ESSAYS:
Q.1. Write briefly on tongue-thrusting habit. Type III: Deforming lateral tongue thrust
Ans. i. Posterior open bite

[SE Q.2] Three subgroups: ii. Posterior crossbite

{Tulley (1969) defined tongue thrust as the forward move- iii. Deep overbite
ment of tongue tip between the teeth to meet the lower lip
during deglutition and in the sounds of speech, so that
tongue lies interdentition. Type IV: Deforming anterior and lateral tongue
According to Schneider (1982) – Tongue thrust is de- thrust
fined as a forward placement of the tongue between the i. Anterior and posterior open bite
anterior teeth and against the lower lip during swallowing.
Classification of tongue-thrusting habit: Three subgroups: ii. Proclination of anterior teeth
According to James S. Brauer and Holt:
Type I: Nondeforming tongue thrust. iii. Posterior crossbite
Type II: Deforming anterior tongue thrust.
i. Anterior open bite Moyers classified tongue thrusting into three types:
i. Simple tongue thrusting: Characterized by teeth
together swallow
Three subgroups: ii. Anterior proclination ii. Complex thrusting: Characterized by teeth apart
swallow
iii. Posterior crossbite iii. Retained infantile swallow
Section | I  Topic-Wise Solved Questions of Previous Years 429

Another classification: i. The simple tongue thrust habit:


i. Physiologic: l It is also called teeth together swallow. There is nor-
Normal tongue thrust swallow of infancy. mal tooth contact during swallowing.
ii. Habitual: l Generalized spacing and proclination may be seen in
After correction of malocclusion tongue thrust is the upper and lower anterior teeth.
present as a habit. l Increased overjet, reduced overbite or presence of
iii. Functional: anterior open bite may be seen.
To achieve oral seal the tongue thrust may develop l Tongue is thrust forward during swallowing to help
as an adaptive behaviour. in establishing anterior lip seal.
iv. Anatomic: l Exaggerated perioral musculature during the swallow-
Anterior tongue posture develops due to enlarged ing action. Especially hyperactive mentalis muscle
tongue.} activity is seen.
Aetiology: ii. Complex tongue thrust habit:
The factors considered as a cause of tongue-thrusting l It is defined as tongue thrust with teeth apart swal-
habit according to Fletcher are as follows: low.
i. Genetic factors l There are two important diagnostic features: Gener-
ii. Learned behaviour (habit) alized open bite and poor occlusal fit of teeth which
iii. Maturational factors leads to sliding occlusion.
iv. Mechanical restrictions l Absence of temporal muscle constriction during
v. Neurological disturbance swallowing and the mandible is not stabilized by the
vi. Psychogenic factors elevator muscles.
i. Genetic factors: iii. Retained infantile swallow:
Specific anatomic or neuromuscular variations Little is known about the exact aetiology of this
in orofacial region can precipitate tongue thrust, severe problem.
e.g. hypertonic orbicularis oris activity.
ii. Learned behaviour (habit): [SE Q.2]
Tongue thrusting can be acquired as a habit due
to improper bottle feeding, prolonged thumb
{Treatment of tongue thrusting:
I. Training of correct swallow and posture of the
sucking, prolonged tonsillar or upper respiratory
tongue.
tract infections etc.
A. Myofunctional exercises:
iii. Maturational factors:
B. Using appliances as a guide in the correct posi-
a. Macroglossia
tioning of tongue
b. Constricted dental arches
A. Myofunctional exercises:
c. Enlarged adenoids and tonsils
i. Place the tip of tongue in rugae area for 5 min
They cause tongue to be positioned anteriorly
and ask the patient to swallow.
to prevent blocking of the oropharynx.
ii. Orthodontic elastic or a sugarless fruit drop
iv. Neurological disturbance:
exercise, where it is held against the palate by
These disturbances of orofacial region can cause
tongue.
tongue thrust habit, e.g. hyposensitive palate and
iii. 45, 25 exercise – includes identify spot, sali-
moderate motor disability.
vating squeezing the spot and swallowing.
v. Psychogenic factors:
iv. Other exercises – whistling, reciting from 61 to
Tongue-thrusting habit may develop as a result
69, gargling yawning, etc. to tone the respec-
of forced discontinuation of other habits like
tive muscles.
thumb sucking.
B. Using appliances as a guide in the correct posi-
tioning of tongue:
{SN Q.3} The correct method of swallowing should be prac-
Clinical features: ticed once/twice a day; once patient is familiar
The clinical features seen in tongue thrusting condi- with the new position an appliance is given for
tion are dependent on the type of tongue thrusting. training the correct positioning of the tongue.
Some common clinical features of tongue thrust For example: preorthodontic trainer for myofunc-
habit are as follows: tional training and Nance palatal arch appliance
l Proclination of anterior teeth II. Speech therapy:
l Bimaxillary protrusion Train correct positioning of tongue not indicated
l Anterior open bite before age of 8 years; child patient is asked to re-
l In case of lateral tongue thrust, posterior open peat simple multiplication tables of sixes and to
bite and posterior crossbite pronounce words beginning with ‘S’ sounds.
430 Quick Review Series for BDS 4th Year, Vol 1

III. Mechano therapy: Based on causative factors


i. Tongue thrust appliance: The mature swallow- Nonobsessive (easily learned
ing act is stimulated by this type of appliance as Obsessive (deep rooted) and dropped)
the tongue adapts to new position and function.
Intentional Masochistic Uintentional Functional
ii. Removable appliance therapy: A variety of new or mean- or self- or empty habits
modifications of Hawley’s appliance are avail- ingful inflicting For example: For example:
able to treat tongue thrust, a crib is incorporated For exam- injurious abnormal pil- mouth breath-
which serves as reminder. ple: nail habit lowing chin ing, tongue
iii. Fixed habit breaking appliance: The base wire biting, digit For exam- propping thrusting,
sucking, lip ple: gingival bruxism
is adapted to follow the contour of the palate biting stripping
and carried posterior to contact metal crown on
1st permanent molar. 3–4 ‘V’-shaped projec-
Classification according to various authors
tions extending downward to point just behind
cingulum of mandibular incisors are made as of Author Classification
crib soldered to the base. James (1923) i. Useful habits
iv. Oral screen: Vestibular or oral screen or combi- ii. Harmful habits
nation is another effective means of controlling Klein (1971) i. Empty habits
abnormal muscle habits like tongue thrusting. ii. Meaningful habits
v. Correction of malocclusion.
Morris and Bohanna i. Pressure habits
IV. Surgical: (1969) ii. Nonpressure habits
Orthognathic surgical procedures to correct skele- iii. Biting habits
tal malocclusions.} iv. Postural habit
v. Miscellaneous
Q.2. Define and classify oral habits and discuss aetiology
Finn (1987) i. Compulsive habits, noncompulsive
and treatment of thumb-sucking habit.
ii. Primary habits, secondary habits
Ans. Kingsley (1958) i. Functional oral habits
ii. Muscular habits
[SE Q.1 and SN Q.2] iii. Combined ones

Classification of habits
{(According to Dorland (1957) habit can be defined as
a fired or constant practice established by frequent
I. According to James
repetition.
Butter Sworth (1961) defined a habit as a frequent or
constant practice or acquired tendency, which has been
fixed by frequent repetition. Useful habits Harmful habits
Mathewson (1982): Oral habits are learned patterns of The habits that are essential The habits that have
muscular contractions.)} for normal function deleterious effect on
For example: the teeth and their
[SE Q.1] • Proper positioning of supporting structures
tongue For example:
{Classification: • Respiration • Thumb sucking
• Normal deglutition • Tongue thrusting

II. According to Morris and Bohanna

Pressure habits Nonpressure habits Postural habit Harmful habits


Include: Habits which do not For example: For example:
Sucking habits: apply direct force on Chin rest Bruxism
• Thumb sucking teeth or its supporting
• Lip sucking structures
• Finger sucking For example:
• Tongue thrusting • Mouth breathing
Biting habits:
• Nail biting
• Pencil biting
• Lip biting
Section | I  Topic-Wise Solved Questions of Previous Years 431

III. According to Klein that influence the severity of the habit into
six levels.
According to Subtelny (1973):
Group I:
Empty habits Meaningful habits
Thumb is inserted beyond the first joint,
Habits not associated Habits that have a
pressing against the palatal mucosa and al-
with any deep-rooted psychological bearing
psychological problems For example:
veolar tissue: lower incisors press against the
For example: Nail biting thumb.
Abnormal pillowing Lip biting Group II:
Chin propping Digit sucking The thumb extends up to the first joint or just
anterior to it; no palatal contact is present with
only anterior teeth.
IV. According to Finn Group III:
Thumb is placed fully into the mouth in con-
tact with palate as in group I, but the lower
Compulsive habits Noncompulsive habits incisors do not contact the thumb.
i. These are deep-rooted i. Include habits that are easily Group IV:
habits that have acqui- learned and dropped as the The thumb does not progress appreciably into
red a fixation in the child matures. the mouth. The lower incisors contacted the
child to the extent that thumb at the nails.
the child retreats to the Theories associated with thumb sucking:
habit whenever his/her i. Classical Freudian theory (1919)
security is threateneda ii. Sucking reflex (Ergel – 1962)
by events that occur iii. Rooting Reflex – Benjamin (1962)
around him/her. ii. Learning theory – Davidson (1967)
ii. The child tends to suffer iii. Oral drive theory – Sears and Wise (1982)
increased anxiety when iv. Johnson and Lasson (1993) – Combination of
an attempt is made to psychoanalytic and learning theories.
correct the habits.}
[SE Q.5]

Thumb sucking: {Aetiology of sucking habits:


Thumb sucking can be defined as placement of the l Sucking urge – Unrestricted breastfeeding,
thumb into the various depths of the mouth. bottle or cup feeding
Classification: l Surplus sucking urge – Non-nutritive sucking
A. Normal thumb sucking: For example: thumb or dummy sucking satis-
During 1st and 2nd year of life thumb sucking is faction}
considered normal, it usually disappears as child (SE Q.5 and SN Q.2)
matures and do not generate any malocclusion.
B. Abnormal thumb sucking: {(Various causative factors are as follows:
Habit persists beyond preschool period and has i. Parents occupation:
deleterious effects on dento-facial structures. Low socioeconomic group are more prone to
C. Psychological: thumb sucking as the mother is unable to pro-
Habit may have a deep-rooted emotional factor vide sufficient breast milk to infants; hence
involved such as insecurity, neglect or loneliness in the process the infant suckles intensively
of child. for a long time thereby exhausting the sucking
i. Habitual: No psychological bearing just as urge. Whereas in high socioeconomic status
habit. the mother is in a better position to feed the
Sucking habits can also be classified as: child and in a short time the baby’s hunger is
ii. Nutritive sucking habits: satisfied.
For example: breast/bottle feeding ii. Working mother:
iii. Non-nutritive sucking habits: Child due to insecure feeling cultivates thumb-
For example: thumb/finger sucking, paci- sucking habit.
fier sucking iii. No of siblings:
Johanson (1993) classified NNS (non- More number of children, so neglection of child
nutritive sucking) habits based on factor leads to development of this habit.
432 Quick Review Series for BDS 4th Year, Vol 1

iv. Order of birth of child: l Treatment is mainly of three categories:


In case of order of birth, the later the rank of sibling A. Psychological therapy
there will be greater chances of developing oral habit B. Reminder therapy or mechanotherapy – remov-
v. Social adjustment and stress: able habit breakers and fixed habit breakers
Digit sucking develops as an emotionally based C. Chemical approach
behaviour.)} A. Psychological treatment:
a. Screening patients for underlying psychologi-
[SE Q.5] cal disturbances and referring to professionals
{vi. Age of the child: for counselling.
The age at which the digit sucking habit appears b. Children between 4 and 8 years of age need
has a significance.} only reassurance, positive reinforcement and
friendly reminders to divert child’s attention to
Clinical effects of thumb sucking: other things like plays and toys.
l Dentofacial changes associated with thumb sucking
c. Dunlop’s beta-hypothesis:
can affect maxilla, mandible, interarch relationship, l Forced purposeful repetition of a habit
lip placement and function, etc. eventually associated with unpleasant reac-
l The severity of malocclusion caused by thumb suck-
tions and habit is abandoned.
ing depends on the following factors: l Dunlop’s beta-hypothesis is especially prac-
a. Duration – Amount of time spent in indulging in ticed in older children, i.e. 8 years and above.
the habit B. Reminder therapy or mechanical aids:
b. Frequency – number of times habit activated/day They are basically reminding appliances that as-
c. Intensity – vigour with which the habit is performed sist to quit the habit.
l The effects of thumb sucking on dental arch and its
Two types of habit breakers:
supporting structures are as follows:
i. Proclination or labial tipping of maxillary ante-
rior teeth. Removable Fixed
ii. Increasing overjet – Due to proclination of i. They are passive and re- i. This appliance consists of
maxillalry anterior 1 lingual tipping of man- movable appliances con- molar bands/crowns on 1st
sisting of a crib-placed permanent molars with pal-
dibular incisors. palatal to 21 12 and an- atal assembly and soldered
iii. Anterior open bite (g overbite) – Restriction of chored to oral cavity by spurs made of either nickel-
incisor eruption 1 supra eruption of buccal teeth means of clasps on poste- chrome or stainless steel.
iv. Posterior crossbite – Narrow maxillary arch rior teeth. For example:
predisposing to crossbite due to contraction of For example: l Quadhelix

l Tongue spikes l Hay rakes


cheek muscles during thumb sucking. l Tongue guard l Maxillary lingual arch with
v. Tongue thrust – Develops as a result of open bite l Spurs/rake palatal crib
vi. Effects on lip – Hypotonic upper lip, hyperac- ii. The best appliance is heavy-
tive lower lip. h lip incompetence, hyperactive guage stainless-steel wire
mentalis activity. designed to form a frame
that is soldered to molar
vii. Other effects are psychological health, risk of bands.
malposition of jaws, speech defects, digit defects.
Diagnosis:
i. History – frequency and duration of habit Fig.14.1  Tongue spike.
ii. Child emotional status – assessed by enquiring
l Feeding habits

l Parental care of child

l Working parents

iii. Examination of child’s fingers – presence of clean


Fig.14.3  Fixed habit breaker.
nails and callus on finger
iv. Intraoral clinical examination – proclination, open
Fig.14.2  Tongue guard.
bite, etc.

[SE Q.5] Other mechanical aids used to intercept the


habit include:
{Treatment: l Bandaging of the thumb
The child 1 parent 1 dentist form a team to assist
l
l Bandaging of the elbow
child in stopping the habit.
Section | I  Topic-Wise Solved Questions of Previous Years 433

Fixed intraoral anti-thumb-sucking appliance: Classification of mouth breathers:


A lingual arch forms base of the appliance to According to Sim and Finn, mouth breathing can be
which are added inter-lacking wires, the anterior categorized into three types:
position in area of anterior part of hard palate
Mouth breathers
disturbs contact of thumb to palate. ↓Three types
Blue grass appliance (1991) by Haskell
It consists of modified six-sided roller ma-
chined from Teflon to prevent thumb place- Obstructive Habitual Anatomic
ment. This is slipped over 0. 045 SS wire sol- Complete or partial Due to deep- Patients with short
dered to molar orthodontic bands duration 3–6 obstruction of nasal rooted habit upper lip that does
mouths. Instruct the patient to turn the roller passage results in that is uncon- not permit complete
instead of sucking digit. Digit sucking is often mouth breathing sciously mouth closure
seen to stop immediately. performed
Quadhelix:
Aetiology:
Prevents thumb from being inserted and also
corrects malocclusion by expanding the
arch. {SN Q.4}
C. Chemical approach: A. obstructive causes:
l Use of bitter tasting or foul smelling preparations
l Nasal polyps
placed on thumb that is sucked can make the habit l Obstructive adenoids
distasteful. l Congenital enlargement of nasal turbinates
l Commonly used medicaments are:
l Chronic inflammation of nasal mucosa
a. Pepper dissolved in a volatile medium l Benign tumours
b. Quinine l Deviated nasal septum
c. Asafoetida B. Anatomic causes like:
Current strategies in treating thumb-sucking habit are l Short upper lip
l Increasing the arm length of night suit l Under-developed nasal cavity
l Thumb home concept

l Use of hand puppets


C. Obstructive sleep apnoea
l Thumb-sucking books
Pathophysiology of mouth breathing:
l My special shirt}
Oral respiration

Q.3. What is mouth-breathing habit? How does it de- Three changes in posture
velop? Describe treatment plan of mouth-breathing
habit in children.
Lowering of Position of tongue Tipping back of
Ans. mandible (downward and head
forward)
[SE Q.3]
{Sassouni (1971) defined mouth breathing as habitual res- Upset orofacial equilibrium
piration through the mouth instead of nose. Unrestricted buccinators activity
l Mouth breathing is an altered way of breathing Influences position of teeth
through mouth and is an adaptation to obstruction in and growth of jaws
nasal passages.
l The obstruction may be temporary or recurrent.
[SE Q.3]
While more often it is partial than complete. The {Clinical features:
airway resistance may be enough to force the subject i. The type of malocclusion associated with mouth-
to breathe through the mouth. breathing is called ‘long face syndrome’ or classic
l Mouth breathing results in altered jaw and tongue adenoid facies or vertical maxillary excess.
posture which in turn alters orofacial equilibrium ii. Dolichofacial skeletal pattern. Long and narrow
leading to malocclusion.} face with short and flaccid upper lip.
434 Quick Review Series for BDS 4th Year, Vol 1

iii. Expression less, blank face. l Removal of nasal or pharyngeal obstruction


iv. Anterior open bite. by ENT surgeon.
v. Contraction of upper arch, narrow V-shaped upper l Interception of the habit – vestibular screen
jaw with a high narrow palate, posterior crossbite. adhesive tapes for lip seal.
vi. h overjet due to labial flaring of maxillary anteriors. l Rapid maxillary expansion – causes widen-
vii. Anterior marginal gingivitis and gingival hypertrophy ing of arch, resulting in h nasal flow and g
is seen in mouth breathers and h caries incidence. nasal air resistance.
viii. Narrow nose and nasal passage, widely flared ex- l Symptomatic treatment like deep breathing
ternal nares. exercises and lip exercises.
ix. Excessive appearance of maxillary anterior teeth
with a ‘gummy smile’. Q.4. Classification, aetiology and deleterious effects of
x. Nasal tone in voice.} tongue-thrusting habit on teeth and associated struc-
Examination of a child for mouth-breathing habit: tures and their treatment.
Diagnosis:
Ans.
i. Observe the patient:
l Mouth breathers – lips will be apart [Same as LE Q.1]
l Nasal breathers – lips will be touching
Q.5. Define and classify oral habits. Write in detail
ii. Ask the patient to take a deep breath through
about the aetiology, clinical features and management
nose:
of thumb-sucking habit.
l Mouth breather – no change in shape or

size of external nares Ans.


l Nasal breather – demonstrates good control of
[Same as LE Q.2]
alar muscles
Q.6. Define and classify oral habits. Discuss in detail
{SN Q.10} thumb-sucking habit.
iii. Mirror test (fog test): Two-surfaced mirror is placed Ans.
on the patient’s upper lip. If air condenses on upper
side of mirror the patient is nasal breather and if it [Same as LE Q.2]
does sooner on the opposite side then he/she is a Q.7. Write the effects of oral habits on the growing jaws
mouth breather. and dentition. How will you manage a case of mouth
iv. Massier’s water holding test: Patient is asked to breathing?
hold mouthfull of water for few minutes without
swallowing. Mouth breathers cannot retain water Ans.
for more than 2–3 min. [Same as LE Q.3]
v. Zwemer’s butterfly test: Take a few fibres of cotton
and place them just below the nasal opening. On Q.8. Define and classify habits. Discuss the aetiology,
exhalation, if the fibres of the cotton flutter down- clinical features and management of mouth breathing.
wards patient is nasal breather and if fibres flutter Ans.
upward he/she is a mouth breather. The child should
close his/her eyes before the cotton is held to his/her [Same as LE Q.3]
nostrils and mouth, so that breathing will be entirely Q.9. Define and classify mouth-breathing habits. Ex-
natural and not forced as when the child is deliber- plain in detail mouth-breathing habit and its treatment.
ately instructed to breathe through his/her nose.
vi. Rhinometry (inductive plethysmography): The total Ans.
air flow through the nose and mouth can be quanti- [Same as LE Q.3]
fied using inductive plethysmography.
vii. Cephalometrics: It can be used to calculate amount
of nasopharyngeal space. SHORT ESSAYS:
Treatment: Q.1. Define and classify oral habits.
l The major aspect of treating mouth breathing

patients is to treat and eliminate underlying Ans.


pathology that has created the habit. [Ref LE Q.2]
Section | I  Topic-Wise Solved Questions of Previous Years 435

Q.2. Classify tongue-thrusting habit and describe the l Psychotherapy.


management of tongue thrusting. l Palliative therapy followed by mechanotherapy
using protective padding and mouth guards.
Ans.
Q.5. Define thumb-sucking habit and its management.
[Ref LE Q.1]
Ans.
Q.3. Mouth-breathing habit – clinical features.
[Ref LE Q.2]
Ans.
Q.6. Pacifier.
[Ref LE Q.3]
Ans.
Q.4. Masochistic habits.
l Pacifiers help the child in transitioning to sleep and
Ans.
provide comfort while teething.
l Self-injurious behaviours or masochistic habits or self- l They are available in various sizes and shapes according

mutilating habits are an extremely rare entity in normal to the age of the child.
children. The incidence of such habits is around 10%– l Pacifier use can lead to malocclusion depending upon

20% in special children. the intensity, frequency and duration of its use.
Aetiology: l Christensen in 1988 reported that anterior open bites and

i. Organic: maxillary constrictions are consistently seen with paci-


l Self-mutilation is associated with Lesch– fier use. Pacifier use can also lead to posterior crossbites.
Nyhan and De Lange syndrome. l Prolonged use of pacifiers with sweetened liquids has a

ii. Functional: positive relation with caries.


l Given by Stewart and Kernohan in 1972. l Recommendations or important information to parents

They are of following types: before using pacifier are


Type A: Injuries super imposed on pre-existing a. Pacifier should be a sturdy one piece construction.
lesion. b. It should have an easily grasped handle.
Type B: Injuries secondary to another estab- c. It should be kept clean.
lished habit. d. Parents should be suggested that pacifier use be cur-
Type C: Injuries of unknown or complex aeti- tailed at beginning of 2 year of age.
ology.
Q.7. Bruxism.
Clinical features:
l The most common form of its manifestation is Ans.
labial loss of gingival tissue and even alveolar Grinding of teeth for nonfunctional purposes is known as
bone. bruxism.
l Finger or thumb habit, whereby the nail of the

finger or thumb is used to strip the tissue from Nocturnal grinding → bruxism
the tooth. Two types
l Another frequent manifestation is biting of the
Day grinding → bruxomania
lips, tongue and oral mucosa.
l A child presents with a localized lesion that
Psychological and emotional stresses
can only be ascribed to repeated trauma to the
affected area, a masochistic behaviour should Occlusal interference or discrepancy
be suspected. Aetiology between CR (centric relation) and
l Insertion of sharp objects like pencils, ball CO (centric occlusion)
point pens and other sharp small objects in- Pericoronitis and periodontal pain
flicts the damage. triggers bruxism
l Any child who wilfully inflicts pain or damage

on himself/herself should be considered psy- Clinical features:


chologically abnormal and should be referred i. Occlusal wear facets on teeth
for psychological evaluation and treatment. ii. Mobility of teeth
Treatment: iii. Fractured teeth and restorations
l Usually, the parents are unaware of the habit, iv. Muscle pain – on waking up in the morning
and bringing it to their attention, along with v. TMJ pain and discomfort
explaining the danger of such practice of the vi. Tenderness and hypertrophy of masticatory muscles
child, is of paramount importance in managing Forces of bruxism are transmitted to the structures of mas-
such habits. ticatory apparatus and depending on the resistance of the
436 Quick Review Series for BDS 4th Year, Vol 1

individual, certain amount of the forces are absorbed and SHORT NOTES:
the rest are passed to other structures.
Diagnosis: Q.1. Define oral habits.
i. History and clinical examination are helpful in
diagnosis of bruxism. Ans.
ii. Check for occlusal prematurities using articulating [Ref LE Q.2]
paper.
iii. Check for hyperactivity of muscles of mastication, Q.2. Define thumb sucking.
i.e. electromyography examination. Ans.
Treatment:
i. Psychological counselling [Ref LE Q.2]
ii. Relieving muscle tension by hypnosis, relaxing Q.3. Tongue thrusting.
exercises and massage.
Other lesser used methods of management are Ans.
physiotherapy, drugs – local anaesthetic injections, [Ref LE Q.1]
tranquilizers and muscle relaxants; electric method
– electro-galvanic stimulation for muscle relax- Q.4. Mouth breathing.
ation and acupuncture.
Ans.
iii. Occlusal adjustments:
Occlusal interferences, if any, should be eliminated. [Ref LE Q.3]
Any possible systemic conditions should be treated.
Q.5. Define masochistic habits and mention two aetio-
iv. Night guards or occlusal splints.
logical factors.
To prevent damage to the teeth structures, a mouth
guard-like appliance or a soft splint can be given to Ans.
the patient to wear at night time.
v. Stainless steel crowns: l Self-injurious behaviours or masochistic habits or self-
l Stainless steel crowns are necessary sometimes
mutilating habits are an extremely rare entity in normal
to prevent any sensitivity and pulpal exposures children. The incidence of such habits is around 10%–
in challenged children. 20% in special children.
l This restores the damaged coronal tissue and the
For example: finger or thumb habit, whereby the nail of
lost vertical dimension. the finger or thumb is used to strip the tissue from the
l When these crowns are properly placed, they
tooth and biting of the lips, tongue and oral mucosa
eliminate occlusal interferences and decrease Aetiology:
the lateral working resistance during grinding. i. Organic:
l Self-mutilation is associated with Lesch–Nyhan
l The parents report that bruxism is eliminated, in

most cases where this treatment option has been and De Lange syndrome.
used. ii. Functional:
l Given by Stewart and Kernohan in 1972.
Q.8. Management of tongue-thrusting habit. They are of following types:
Ans. Type A: Injuries superimposed on pre-existing
lesion.
[Same as SE Q.2] Type B: Injuries secondary to another estab-
Q.9. Clinical manifestations of mouth-breathing habit. lished habit.
Type C: Injuries of unknown or complex aeti-
Ans. ology.
[Same as SE Q.3] Q.6. Vestibular screen indications.
Q.10. Aetiology of thumb sucking.
Ans.
Ans.
Vestibular screen or oral screen was first introduced by
[Same as SE Q.5] ‘Newel’ in 1912.
l Vestibular screens are also called lip moulders.
Q.11. Management of thumb sucking.
l Oral screen or vestibular screen is simple functional
Ans. appliance that takes the form of a curved shield of
[Same as SE Q.5] acrylic placed in the labial vestibule.
Section | I  Topic-Wise Solved Questions of Previous Years 437

Indications/uses: Q.9. Lip bumper.


l Used mostly to intercept mouth-breathing habit
Ans.
and also other habits like thumb sucking, tongue
thrusting, lip biting and cheek biting l Lip bumper is a habit-breaking appliance that can be
l Used for correction of mild distoclusions and used to make the practice of drawing the lower lip be-
mild anterior proclination tween the anterior teeth more difficult and thus remind-
l Used to perform muscle exercises that help in cor- ing the child of the habit.
rection of hypotonic lip and check muscles l Types of lip bumpers:

l Correction of flaccid hypotonic orofacial muscu- a. Based on their ability to be removed


lature i. Combined fixed removable
l Used as both active and passive appliances ii. Component of fixed appliance
b. Based on arch used:
Q.7. Non-nutritive sucking habit.
i. Maxillary lip bumper or Denholtz appliance
Ans. ii. Mandibular lip bumper
l Mechanism of action:
Non-nutritive sucking habits:
The lip bumper prevents hyperactivity of mentalis
For example: thumb/finger sucking, pacifier sucking
muscles and abnormal force acting on the incisors.
Johanson (1993):
l Uses:
Classified NNS (non-nutritive sucking) habits based on
i. Correction of lip trap
factor that influence the severity of the habit into six
ii. Uprighting molars and as anchorage savers
levels.
iii. Distalization of molars and reduction overjet by
Q.8. Mention two advantages of oral screen. proclination of mandibular incisors
Ans. Q.10. Tests for mouth breathing.
Oral screen is a functional appliance introduced by Newell Ans.
in 1912. It produces its effects by redirecting the pressure
of the muscular and soft tissue curtain of the cheeks and [Ref LE Q.3]
lips.
Q.11. Oro-facial habits.
It will block the passage of air through the mouth and
force the inspiration and expiration of air through the nares. Ans.
Advantages:
[Same as SN Q.1]
l Simple and versatile appliance for early interceptive

treatment. Q.12. Causative factors of thumb-sucking habit.


l It works on the principle of both force application

and force elimination. Ans.


For example: Posterior crossbite can be corrected [Same as SN Q.2]
utilizing the principle of force elimination by provid-
ing a space between the teeth and the screen. Q.13. Salient features of tongue-thrusting habit.
l Anterior teeth proclination can be corrected by utiliz-
Ans.
ing the principle of force application, the forces from
the lips are transmitted directly to the proclined teeth [Same as SN Q.3]
through the screen.
l Lip exercises are possible with oral screen, which Q.14. Causes of mouth breathing.
improves the tonicity of the lips. Ans.
l Develops effective mechanism for reducing or elimi-

nating hyperactive mentalis muscle. [Same as SN Q.4]


438 Quick Review Series for BDS 4th Year, Vol 1

Topic 15
Gingival and Periodontal Diseases in Children
COMMONLY ASKED QUESTIONS
LONG ESSAYS:
1 . Describe normal gingiva of a child. Discuss in detail causes of gingival disorders in children.
2. Classify periodontal diseases in children and adolescents. Discuss localized aggressive periodontitis.
3. Enumerate various gingival and periodontal diseases seen in children and write in detail acute herpetic gingivo-
stomatitis.
4. Which are the common gingival disorders affecting children? How do you diagnose and manage ANUG in
children? [Same as LE Q.1]

SHORT ESSAYS:
1. Scorbutic gingivitis.
2. Juvenile periodontitis.
3. Normal gingiva in a child. [Ref LE Q.1]
4. Causes of gingival recession and treatment.
5. ANUG. [Ref LE Q.1]
6. Herpetic gingivitis. [Ref LE Q.3]
7. Gingival enlargements. [Ref LE Q.1]
8. Enumerate common gingival and periodontal diseases in children. [Ref LE Q.1 and Q.2]
9. Localized aggressive periodontitis.
10. Periodontium in children. [Same as SE Q.3]
11. Acute necrotizing ulcerative gingivitis. [Same as SE Q.5]
12. Vincent infection. [Same as SE Q.5]
13. Acute herpetic gingivo-stomatitis. [Same as SE Q.6]

SHORT NOTES:
1. Dilantin gingivitis.
2. Vincent infection. [Ref LE Q.1]
3. Juvenile periodontitis. [Ref SE Q.2]
4. Fenestration.
5. Eruption gingivitis. [Ref LE Q.1]
6. Give the clinical features and management of acute herpetic gingivo-stomatitis. [Ref LE Q.1]
7. Gingival recession predisposing factors. [Ref SE Q.4]
8. What is PMA index.
9. Phenytoin-induced gingival overgrowth. [Ref LE Q.1]
10. Define dental plaque.
11. General principles of treatment of periodontal conditions in children.
12. ANUG. [Same as SN Q.2]
13. Acute herpetic gingivo-stomatitis. [Same as SN Q.6]
Section | I  Topic-Wise Solved Questions of Previous Years 439

SOLVED ANSWERS
LONG ESSAYS:
Q.1. Describe normal gingiva of a child. Discuss in detail iii. Allergy and gingival inflammation
causes of gingival disorders in children. iv. Acute gingival disease
v. Chronic nonspecific gingivitis
Ans.
vi. Gingival diseases modified by systemic factors}
Each one of the above gingival diseases is
[SE Q.3] described below:
{Normal gingiva
l The gingival tissues are normally light pink, whereas
in the young child it may be more reddish due to in-
creased vascularity and thinner epithelium. {SN Q.5}
l The surface of the gingiva of a child appears less Eruption gingivitis:
stippled or smoother than that of an adult. In the l Eruption gingivitis is a transitory type of gingivitis,
healthy adult, the marginal gingiva has a sharp, often observed in young children when the primary
knife-like edge. teeth are erupting.
l Normal probing depths around primary teeth is ap- l It is often localized and associated with difficult
proximately 2 mm, with the facial and lingual probe eruption, subsides after the teeth emerge into the oral
sites shallower than the proximal sites. cavity.
l The width of the attached gingiva is narrower in the l The greatest incidence of this gingivitis in children is
mandible than in the maxilla. often seen in the 6- to 7-year age group when the
l The alveolar bone surrounding the primary dentition permanent teeth begin to erupt.
demonstrates fewer trabeculae, less calcification and l It apparently occurs because the gingival margin re-
larger marrow spaces. ceives no protection from the coronal contour of the
l Marginal gingivitis is the most common form of gin- tooth during the early stage of active eruption, and
gival disease and starts in early childhood.} the continual impingement of food on the gingivae
Causes of gingival disorders causes the inflammatory process.
The major aetiologic factors associated with gingivi- l Food debris, materia alba and bacterial plaque often
tis are collect around and beneath the free tissue, partially
i. Bacterial plaque cover the crown of the erupting tooth and cause the
ii. Dental calculus development of an inflammatory process.
l Bacterial plaque both uncalcified and calci- l This inflammation is most commonly associated
fied. with the eruption of the first and second permanent
l Bacterial plaque is composed of soft bacte- molars, and the condition can be painful and can
rial deposits and adheres firmly to the teeth. develop into a pericoronitis or a pericoronal abscess.
It is a complex, metabolically intercon- l Mild eruption gingivitis requires no treatment other
nected, highly organized bacterial system than improved oral hygiene.
consisting of dense masses of microorgan- l To prevent this an oral hygiene program should be
isms embedded in an intermicrobial matrix initiated by parents when the child is very young.
(biofilm). It can cause dental caries and
periodontal disease also.
l Dental calculus, both supragingival and l Painful pericoronitis may be helped when the
subgingival, which is considered to be cal- area is irrigated with a counterirritant, such as
cified dental plaque, is an important factor Peroxyl (Colgate-Palmolive Co., New York,
in the development of gingival and peri- NY).
odontal disease. l Pericoronitis accompanied by swelling and

lymph node involvement should be treated


[SE Q.8]
with antibiotic therapy.
{The common gingival disorders affecting children are Dental plaque–induced gingivitis
as follows: l Plaque-induced gingivitis associated with poor

i. Eruption gingivitis oral hygiene is usually classified as early (slight),


ii. Dental plaque–induced gingivitis moderate or advanced.
440 Quick Review Series for BDS 4th Year, Vol 1

l Gingivitis is generally less severe in children than agents. Currently, a topical corticosteroid (e.g.
in adults with similar plaque levels. 0.5% fluocinonide, 0.025% triamcinolone, 0.5%
l Favourable occlusion and the chewing of coarse, clobetasol) is applied to the area with a mucosal
detergent-type foods, such as raw carrots, celery adherent (e.g. isobutyl cyanoacrylate, Orabase)
and apples, have a beneficial effect on oral clean- l The application of triamcinolone acetonide
liness. (Kenalog in Orabase) to the surface of the le-
l Early gingivitis is quickly reversible and can be sions before meals and before sleeping may also
treated with a good oral prophylactic treatment be helpful.
and instruction in good tooth brushing and floss-
ing techniques to keep the teeth free of bacterial (SN Q.2 and SE Q.5)
plaque.
Allergy and gingival inflammation {(Acute necrotizing ulcerative gingivitis
l The significance of gingival reaction during short l It is an infectious disease which occurs occasion-
allergic seasons is difficult to assess. ally in children of 6–12 years old, and is common
l It is speculated that patients with complex aller- in young adults.
gies who have symptoms for longer periods may l ANUG can be diagnosed by the involvement of
be at higher risk for adverse periodontal changes. the interproximal papillae and the presence of a
Acute gingival disease pseudomembranous necrotic covering of the mar-
ginal tissue.
l Two microorganisms, Borrelia vincentii and fusi-
{SN Q.6} form bacilli, referred to as spirochetal organisms,
Herpes simplex virus infection are considered to be responsible for the disease.
l Herpes virus causes one of the most widespread l The clinical manifestations of the disease include
viral infections. inflamed, painful, bleeding gingival tissue, poor
l The primary infection usually occurs in a child appetite, temperature as high as 40°C (104°F),
younger than 6 years of age who has had no con- general malaise and a fetid odour.
tact with the type 1 herpes simplex virus (HSV-1) l The disease responds dramatically within 24–48 h
and who therefore has no neutralizing antibodies. to subgingival curettage, debridement and the use
l Treatment of acute herpetic gingivostomatitis in of mild oxidizing solutions.
children, which runs a course of 10–14 days, l Improved oral hygiene, the use of mild oxidizing
should include specific antiviral medication as mouth rinses after each meal and twice daily rins-
well as provision for the relief of the acute symp- ing with chlorhexidine will aid in overcoming the
toms so that fluid and nutritional intake can be infection.)}
maintained.
Acute candidiasis (thrush, candidosis, moniliasis)
l Candida (Monilia) albicans is a common inhabitant

Recurrent aphthous ulcer (canker sore) of the oral cavity but may multiply rapidly and cause
l The recurrent aphthous ulcer (RAU) – also re- a pathogenic state when host resistance is lowered.
ferred to as recurrent aphthous stomatitis (RAS) – l Thrush may develop after prolonged local anti-

is a painful ulceration on the unattached mucous biotic therapy, which allows the fungus to pro-
membrane that occurs in school-aged children liferate.
and adults. l The lesion of the oral disease appears as raised,

l It is the most common mucosal disorder in people furry, white patches, which can be removed easily
of all ages and races in the world, but the peak age to produce a bleeding underlying surface.
of occurrence is between 10 and 19 years of age. l Neonatal candidiasis, contracted during passage

l The exact cause of RAU is unknown. through the vagina and erupting clinically during
l A variety of treatments have been recommended the first 2 weeks of life, is a common occurrence.
for RAU, but a completely successful therapy has This infection is also common in immunosup-
not been found. Topical anti-inflammatory and pressed patients.
analgesic medicines and/or systemic immuno- l Antifungal antibiotics control thrush.

modulating and immunosuppression agents have l Improved oral hygiene is important in treating the

been used for RAU. infection


l The primary line of treatment uses topical gels, l Nystatin and chlorhexidine mouth rinses are also

creams and ointments as anti-inflammatory appropriate.


Section | I  Topic-Wise Solved Questions of Previous Years 441

Chronic nonspecific gingivitis (SN Q 9 and SE Q.7)


l This type of gingivitis is commonly seen during

the preteenage and teenage years. {(Phenytoin-induced gingival overgrowth


l The chronic gingival inflammation may be local-
l Phenytoin (Dilantin or diphenylhydantoin), a ma-
ized to the anterior region, or it may be more jor anticonvulsant agent used in the treatment of
generalized. epilepsy.
l Although the condition is rarely painful, it
l There is neither excessive collagen accumulation
may persist for long periods without much per unit of tissue nor the fibroblasts appear abnor-
improvement. mal in number or size, so it is not a true hyperplasia.
l Inadequate oral hygiene, which allows food im-
l Phenytoin induced gingival over growth (PIGO),
paction and the accumulation of material alba when it develops, begins to appear as early 2–3
and bacterial plaque, is undoubtedly the major weeks after initiation of phenytoin therapy and
cause of this chronic type of gingivitis. peaks at 18–24 months.
l An improved dietary intake of vitamins and the
l The initial clinical appearance is painless enlarge-
use of multiple-vitamin supplements improve the ment of the interproximal gingiva.
gingival condition in many children. l The buccal and anterior segments are most often
Gingival diseases modified by systemic factors affected than the lingual and posterior segments;
Gingival diseases associated with the endocrine in some cases, the entire occlusal surface of the
system: teeth becomes covered. These lesions may remain
l Puberty gingivitis is a distinctive type of gingi-
purely fibrotic in nature or may be combined with
vitis that occasionally develops in children in a noticeable inflammatory component.
the prepubertal and pubertal period. l No cure exists and treatment is often symptomatic
l The enlargement of the gingival tissues in pu-
in nature.
berty gingivitis is confined to the anterior seg- l Other drugs that can induce gingival overgrowth in
ment and may be present in only one arch. The some patients include cyclosporin, calcium chan-
lingual gingival tissue generally remains unaf- nel blockers, valproic acid and phenobarbital.
fected. l Maintaining excellent oral hygiene is the primary
l Treatment of puberty gingivitis should be di-
key to successful therapy.)}
rected towards improved oral hygiene, removal
Ascorbic acid–deficiency gingivitis (scorbutic gingi-
of all local irritants, restoration of carious teeth
vitis)
and dietary changes necessary to ensure an
l Scorbutic gingivitis is associated with vitamin
adequate nutritional status.
C deficiency and differs from the type of gin-
Gingival lesions of genetic origin
givitis related to poor oral hygiene.
l Hereditary gingival fibromatosis (HGF) is charac-
l The involvement is usually limited to the marginal
terized by a slow, progressive, benign enlarge-
tissues and papillae.
ment of the gingivae.
l The child with scorbutic gingivitis may complain
l The most common genetic form, HGF, usually
of severe pain, and spontaneous haemorrhage is
has an autosomal dominant mode of inheritance.
evident.
This rare type of gingivitis has been referred to as
l The gingivitis responds dramatically to the daily
elephantiasis gingivae or hereditary hyperplasia
administration of 250–500 mg of ascorbic acid.
of the gums.
l Older children and adults may require 1 g of vita-
l The gingival tissues appear normal at birth but
min C for 2 weeks to speed recovery.
begin to enlarge with the eruption of the primary
teeth. Q.2. Classify periodontal diseases in children and ado-
l Although mild cases are observed, the gingival
lescents. Discuss localized aggressive periodontitis.
tissues usually continue to enlarge with eruption
of the permanent teeth until the tissues essentially Periodontitis, an inflammatory disease of the gingival and
cover the clinical crowns of the teeth. deeper tissues of periodontium, is characterized by pocket
l The dense fibrous tissue often causes displace-
formation and destruction of supporting alveolar bone.
ment of the teeth and maloclusion.
l Surgical removal of the hyperplastic tissue [SE Q.8]
achieves a more favourable oral and facial {Periodontal diseases in children
appearance. 1. Early onset periodontitis (aggressive periodontitis)
442 Quick Review Series for BDS 4th Year, Vol 1

2 . Localized aggressive periodontitis (LAP) B. Associated with genetic disorders


3. Generalized aggressive periodontitis 1. Familial and cyclic neutropenia
4. Periodontitis as a manifestation of systemic 2. Down syndrome
disease} 3. Leukocyte adhesion–deficiency syndromes
4. Papillon–Lefevre syndrome
1. Early onset periodontitis (aggressive periodontitis): 5. Chediak–Higashi syndrome
Aggressive periodontitis is a heterogeneous group of 6. Histiocytosis syndromes
periodontal disease occurring in younger individuals 7. Glycogen storage disease
who are otherwise healthy. Aggressive periodontitis 8. Infantile genetic agranulocytosis
can be 9. Cohen syndrome
(1)   A localized form (LAP) 10. Ehlers–Danlos syndrome (types IV and VIII)
(2)  A generalized form (generalized aggressive 11. Hypophosphatasia
periodontitis [GAP]) Leukaemia
l LAP is localized attachment loss and al- l Leukemic cells are capable of infiltrating the

veolar bone loss only in the primary denti- gingiva and the deeper periodontal tissues,
tion in an otherwise healthy child. leading to gingival hyperplasia and pocket for-
l It appears around or before 4 years of age, mation.
when the bone loss is usually seen on ra- l The pockets can be colonized by bacteria and
diographs around the primary molars and/ become inflamed.
or incisors. l Gingival bleeding is a common finding.

l Abnormal probing depths with minor gin- Cyclic neutropenia


gival inflammation, rapid bone loss and l Oral manifestations include

minimal to varying amounts of plaque is i. Oral mucosal ulceration.


seen at the affected sites. ii. Severe gingivitis.
l Abnormalities in host defences (e.g. leu- iii. The periodontal destruction can lead to
kocyte chemotaxis), extensive proximal premature exfoliation of teeth.
caries facilitating plaque retention and Hypophosphatasia
bone loss, and a family history of peri- l Hypophosphatasia is characterized by incom-

odontitis have been associated with LAP plete bone mineralization.


in children. l Its clinical features are rickets, osteomalacia,

l The child’s periodontium shows signs of poor cranial-bone formation and craniosteno-
gingival inflammation with gingival clefts sis.
and localized ulceration of the gingival l Premature loss of primary teeth appear to have

margin. a ‘shell’ appearance due to the widened pulp


l The onset of GAP is during or soon after chambers and thin root canal walls.
the eruption of the primary teeth and re- LANGERHANS CELL HISTIOCYTOSIS
sults in severe gingival inflammation and l Clinical features are loosened teeth, prema-

generalized attachment loss, tooth mobil- turely erupted primary teeth, oral soft tissue
ity and rapid alveolar bone loss with pre- invasion of Langerhans cells leading to gingi-
mature exfoliation of the teeth. val inflammation, ulceration, hypertrophy, bad
Microorganisms predominating in the gin- taste and halitosis are some of the oral mani-
gival pocket include Aggregatibacter acti- festations.
nomycetemcomitans (Aa), Porphyromonas Because of destruction of alveolar boil the
(Bacteroides) gingivalis, Bacteroides mel- teeth in radiographs give a ‘floating teeth’
aninogenicus, Prevotella intermedia, appearance.
Capnocytophaga sputigena and Fusobac- l Acrodynia

terium nucleatum. It is a hypersensitivity reaction to mercury.


PERIODONTITIS AS A MANIFESTATION OF SYS- l Increased salivation, sore mouth, gingival hy-

TEMIC DISEASE perplasia, alveolar bone destruction and mobil-


Several of the periodontal conditions affected by ity of deciduous teeth are some of the common
systemic diseases are identified in the paediatric oral manifestation.
population: DOWN SYNDROME
A. Associated with haematological disorders l Affected children have growth retardation,

1. Acquired neutropenia mental deficiency, joint hyperfexibility and


2. Leukaemias cardiac problems.
Section | I  Topic-Wise Solved Questions of Previous Years 443

Children with Down syndrome are affected


l l In a few days the vesicles rupture and form pain-
with periodontal disease involving both pri- ful ulcers, 1–3 mm in diameter, which are covered
mary and permanent dentition. with a whitish grey membrane and have a circum-
DIABETES MELLITUS scribed area of inflammation.
l Periodontal disease may be more frequent and l The ulcers may be observed on any area of the mu-

severe on diabetic individuals with more ad- cous membrane, including buccal mucosa, tongue,
vanced systemic complications. lips, hard and soft palate and the tonsillar areas.
l Affected patient lacks adequate metabolic con- l The lesion culture also shows positive results for

trol, periodontal complications usually start HSV-1.


with the onset of gingivitis in the circumpuber- l Treatment of acute herpetic gingivostomatitis in

tal period. children, which runs a course of 10–14 days,


l Early onset of this disease is associated with should include specific antiviral medication as
severe periodontal destruction, leading to se- well as fluid and nutritional intake should be
vere mobility and early exfoliation of teeth. maintained.
l The application of a mild topical anaesthetic, such
Q.3. Enumerate various gingival and periodontal dis- as dyclonine hydrochloride (0.5%) (Dyclone) be-
eases seen in children and write in detail acute herpetic fore mealtime temporarily relieves the pain and
gingivo-stomatitis. allows the child to take in soft food.
Ans. l Another topical anaesthetic, lidocaine (xylocaine

viscous), can be prescribed.


The common gingival disorders affecting children are as l Since fruit juices are usually irritating to the ul-
follows: cerated area, ingestion of a vitamin supplement
i. Eruption gingivitis during the course of the disease is indicated.
ii. Dental plaque–induced gingivitis l Bed rest and isolation from other children in the
iii. Allergy and gingival inflammation family are also recommended.
iv. Acute gingival disease l After the initial primary attack during early child-
v. Chronic nonspecific gingivitis hood, the herpes simplex virus becomes inactive
vi. Gingival diseases modified by systemic factors and resides in sensory nerve ganglia.
Periodontal diseases in children l The virus often reappears later as the familiar cold
a. Early onset periodontitis (aggressive periodon- sore or fever blister, usually on the outside of the
titis) lips. Thus the disease has been commonly re-
b. LAP ferred to as recurrent herpes labialis (RHL).}
c. Generalized aggressive periodontitis
d. Periodontitis as a manifestation of systemic Q.4. Which are the common gingival disorders affecting
disease children? How do you diagnose and manage ANUG in
children?
[SE Q.6]
Ans.
{Acute herpetic gingivostomatitis
l Herpes virus causes one of the most widespread [Same as LE Q.1]
viral infections
l The primary infection usually occurs in a child SHORT ESSAYS:
younger than 6 years of age who has had no
contact with the type 1 herpes simplex virus Q.1. Scorbutic gingivitis.
(HSV-1) and who therefore has no neutralizing
antibodies. Ans.
l In some children the primary infection may be Scorbutic gingivitis.
characterized by only one or two mild sores on the l Scorbutic gingivitis is associated with vitamin C defi-
oral mucous membranes ciency and differs from the type of gingivitis related to
l The symptoms of the disease develop suddenly poor oral hygiene.
and include, in addition to the fiery red gingival l It is usually limited to the marginal tissues and papillae.
tissues, malaise, irritability, headache and pain l The child with scorbutic gingivitis may complain of
associated with the intake of food and liquids of severe pain, and spontaneous haemorrhage is evident.
acid content. l Severe clinical scorbutic gingivitis is rare in children.
l A characteristic oral finding in the acute primary However, it may occur in children allergic to fruit juices
disease is the presence of yellow or white liquid- when provision of an adequate dietary supplement of
filled vesicles. vitamin C is neglected.
444 Quick Review Series for BDS 4th Year, Vol 1

l Scorbutic gingivitis responds dramatically to the daily Q.4. Causes of gingival recession and treatment.
administration of 250–500 mg of ascorbic acid. Older
Ans.
children and adults may require 1 g of vitamin C for
2 weeks to speed recovery.
l Inflammation and enlargement of the marginal gingival {SN Q.7}
tissue and papillae in the absence of local predisposing l Gingival recession is often observed in children.
factors are possible evidence of scorbutic gingivitis. l Factors predisposing to gingival recession include
l Questioning the child and parents regarding eating hab- 1. Presence of a narrow band of attached or kera-
its and using the 7-day diet survey frequently reveals tinized gingiva
that the child is receiving inadequate amounts of foods 2. Alveolar bony dehiscence
containing vitamin C. 3. Toothbrush trauma
l Complete dental care, improved oral hygiene and sup- 4. Tooth prominence
plementation with vitamin C and other water-soluble 5. Impinging frenum attachment
vitamins fill greatly improve the gingival condition. 6. Soft tissue impingement by opposing occlusion
Q.2. Juvenile periodontitis 7. Orthodontic tooth movement
8. Use of impression techniques including subgin-
Ans. gival tissue retraction
9. Oral habits
{SN Q.3} 10. Periodontitis
Juvenile periodontitis also known as LAP is a heteroge- 11. Pseudorecession (extrusion of teeth)
neous group of periodontal disease occurring in younger
individuals who are otherwise healthy. Treatment
l Juvenile periodontitis (LAP) is localized attachment
l Recession is dealt with conservatively by elimination
loss and alveolar bone loss only in the primary denti- of the stimulus if possible, while excellent oral hy-
tion in an otherwise healthy child. giene is maintained in the affected areas.
l It appears around or before 4 years of age, with se-
l If the recession of the affected area remains un-
vere bone loss usually seen on radiographs around changed (nonprogressive) or improves (less recession
more than one permanent tooth usually the first mo- observed) over time, continued periodic monitoring is
lars and incisors. recommended.
l Abnormal probing depths with minor gingival in-
l If the recession has progressed after a 4- to 8-week
flammation, rapid bone loss and minimal to varying period of observation, other periodontal procedures
amounts of plaque is seen at the affected sites. may be required based on the identified predisposing
l Progression of bone loss is 3–4 times faster than in
factor.
chronic periodontitis. l The treatment for gingival recession is aimed at con-
l The probable causative microbial species are Aa or
trolling the predisposing factors and maintenance of
Aa in combination with porphyromonas-like species. excellent oral hygiene.
l Abnormalities in peripheral neutrophils, chemotaxis,
l Even after controlling the predisposing factors, if the
phagocytosis, leukotriene generation and bacterial recession is not improved, periodontal surgical pro-
activity are seen. cedures (flap procedures) need to be carried out to
l Treatment of juvenile periodontitis depends on
correct the recession.
(a) Early diagnosis
(b) Dental curettage Q.5. ANUG.
(c) Root planing Ans.
(d) Prophylaxis
(e) Removal of primary teeth that have lost bony [Ref LE Q.1]
support Q.6. Herpetic gingivitis.
(f) Restoration of decayed teeth
(g) Oral hygiene instruction Ans.
(h) Broad spectrum antimicrobials [Ref LE Q.3]
(i) Chlorhexidine rinses
Q.7. Gingival enlargements.
Q.3. Normal gingiva in a child. Ans.
Ans. Gingival enlargement refers to the overgrowth of marginal
[Ref LE Q.1] gingival caused due to continuous irritation of a stimulant.
Section | I  Topic-Wise Solved Questions of Previous Years 445

This mostly occurs due to long-term use of certain drugs. periodontal disease occurring in younger individuals
Of which the most common being phenytoin. who are otherwise healthy.
Phenytoin-induced gingival overgrowth: l LAP is localized attachment loss and alveolar bone loss
l Phenytoin (Dilantin or diphenylhydantoin), a major only in the primary dentition in an otherwise healthy
anticonvulsant agent used in the treatment of child.
epilepsy. l The probable causative microbial species are Actinomy-
l There is neither excessive collagen accumulation per ces actinomycetemcomitans with porphyromonas like
unit of tissue nor the fibroblasts appear abnormal in species.
number or size, so it is not a true hyperplasia. l It appears around or before 4 years of age, when the
l PIGO, when it develops, begins to appear as early bone loss is usually seen on radiographs around the
2–3 weeks after initiation of phenytoin therapy and primary molars and/or incisors.
peaks at 18–24 months. l Abnormal probing depths with minor gingival inflam-
l The initial clinical appearance is painless enlarge- mation, rapid bone loss and minimal to varying amounts
ment of the interproximal gingiva. of plaque are seen at the affected sites
l The buccal and anterior segments are most often l Abnormalities in host defences (e.g. leukocyte chemo-
affected than the lingual and posterior segments; in taxis), extensive proximal caries facilitating plaque re-
some cases, the entire occlusal surface of the teeth tention and bone loss and a family history of periodon-
becomes covered. These lesions may remain purely titis have been associated with LAP in children.
fibrotic in nature or may be combined with a notice- l The child’s periodontium shows signs of gingival in-
able inflammatory component. flammation with gingival clefts and localized ulceration
l No cure exists and treatment is often symptomatic in of the gingival margin.
nature.
Q.10. Periodontium in children.
l Antihistamines, topical corticosteroids, ascorbic acid

(vitamin C) supplements, topical antibiotics and al- Ans.


kaline mouthwashes have been used with limited
[Same as SE Q.3]
success and are considered to be ineffective.
l Other drugs that can induce gingival overgrowth in l The gingival tissues are normally light pink, and the
some patients include cyclosporin, calcium channel colour depends on the complexion of the person, the
blockers, valproic acid and phenobarbital. thickness of the tissue and the degree of keratinization.
l Maintaining excellent oral hygiene is the primary l The gingival colour of the young child may be more red-

key to successful therapy. dish due to increased vascularity and thinner epithelium.
l The surface of the gingiva of a child appears less stippled
Q.8. Enumerate common gingival and periodontal dis-
or smoother than that of an adult. In the healthy adult,
eases in children.
the marginal gingiva has a sharp, knifelike edge.
Ans. l Normal probing depths around primary teeth is ap-

proximately 2 mm, with the facial and lingual probe


The common gingival disorders affecting children are as
sites shallower than the proximal sites.
follows:
l The width of the attached gingiva is narrower in the
i. Eruption gingivitis
mandible than in the maxilla.
ii. Dental plaque–induced gingivitis
l The periodontal ligament has more width in children
iii. Allergy and gingival inflammation
than in adults.
iv. Acute gingival disease
l The alveolar bone surrounding the primary dentition
v. Chronic nonspecific gingivitis
demonstrates fewer trabeculae, less calcification and
vi. Gingival diseases modified by systemic factors
larger marrow spaces.
Periodontal diseases in children
i. Early onset periodontitis (aggressive periodontitis) SHORT NOTES:
ii. LAP
iii. Generalized aggressive periodontitis Q.1. Dilantin gingivitis.
iv. Periodontitis as a manifestation of systemic Ans.
disease
l Dilantin, diphenylhydantoin or phenytoin is a major
Q.9. Localized aggressive periodontitis. anticonvulsant agent used in the treatment of epilepsy,
Ans. which causes varying degrees of gingival overgrowth.
l Dilantin gingivitis, when it develops, begins to appear
l Localized aggressive periodontitis is also known as as early as 2–3 weeks after initiation of phenytoin ther-
juvenile periodontitis. It is a heterogeneous group of apy and peaks at 18–24 months.
446 Quick Review Series for BDS 4th Year, Vol 1

l The initial clinical appearance is painless enlargement l The P, M and A numerical values for all the teeth are
of the interproximal gingiva. added separately and then added together to express
l The buccal and anterior segments are most often af- the PMA index score per person.
fected than the lingual and posterior segments; in some Calculation of PMA score:
cases, the entire occlusal surface of the teeth becomes PMA score 5 P 1 M 1 A
covered. l The developers of this index eventually added a se-

l No cure exists and treatment is often symptomatic in verity component for assessing gingivitis, the papil-
nature. lary units (P) were scored on a scale of 0–5 and the
l Maintaining excellent oral hygiene is the primary key to marginal (M) and attached gingiva were scored on a
successful therapy. scale of 0–3.

Q.2. Vincent infection. Q.9. Phenytoin-induced gingival overgrowth.

Ans. Ans.

[Ref LE Q.1] [Ref LE Q.1]

Q.3. Juvenile periodontitis. Q.10. Define dental plaque.

Ans. Ans.
Dental plaque
[Ref SE Q.2]
Dental plaque is defined as a thin biofilm present on the
Q.4. Fenestration. surface of teeth.
It acts as a niche for accumulation of microbes and food
Ans.
particles which further results in varying degrees of gingi-
Fenestration is a term given for isolated areas in which the val disorders usually starting with marginal gingivitis.
root is denuded of bone and the root surface is covered only l Gingivitis associated with poor oral hygiene is usually
by periosteum and overlying gingival. In these areas the classified as early (slight), moderate or advanced.
marginal bone remains intact. l Gingivitis is generally less severe in children than in

adults with similar plaque levels.


Q.5. Eruption gingivitis.
l Favourable occlusion and the chewing of coarse, deter-
Ans. gent-type foods, such as raw carrots, celery and apples,
have a beneficial effect on oral cleanliness as it helps in
[Ref LE Q.1]
removal of plaque.
Q.6. Give the clinical features and management of acute l Early gingivitis is quickly reversible and can be treated
herpetic gingivo-stomatitis. with a good oral prophylactic treatment and instruction
in good tooth brushing and flossing techniques to keep
Ans.
the teeth free of bacterial plaque.
[Ref LE Q.1] Q.11. General principles of treatment of periodontal
Q.7. Gingival recession predisposing factors. conditions in children.
Ans. Ans.

[Ref SE Q.4] Periodontal conditions develop in children depending upon


the age and cleanliness. So the basic step involved in the
Q.8. What is PMA index? treatment of such condition is maintaining the oral health of
Ans. child at a very young age. Other than this, symptomatic
treatment of inflammatory diseases is necessary. Antibiot-
Papillary marginal attachment (PMA) index by Schour ics, antiseptic gels and mouthwashes need to be prescribed
and Massler (1944): in cases of infection.
l In this index, the number of gingival units affected
Q.12. ANUG.
were counted rather than the severity of the inflam-
mation. Ans.
l A gingival unit is divided into three component parts:
[Same as SN Q.2]
i. Papillary gingiva (P)
ii. Marginal gingiva (M) Q.13. Acute herpetic gingivo-stomatitis.
iii. Attached gingiva (A) Ans.
l The presence or absence of inflammation on each

gingival unit is recorded as 1 or 0, respectively. [Same as SN Q.6]


Section | I  Topic-Wise Solved Questions of Previous Years 447

Topic 16
Home Oral Hygiene for Children and Adolescents
COMMONLY ASKED QUESTIONS
LONG ESSAYS:
1 . Write briefly about chemical control of dental plaque.
2. Describe microbial aspects of oral hygiene and plaque formation.
3. Define dentifrice and give composition of the same. Describe briefly about fluoride dentifrices.
4. Give detail account of maintaining optimum dental health in children.
5. Describe the various biological factors responsible for the maintenance of oral health. [Same as LE Q.2]

SHORT ESSAYS:
1. Discuss plaque control methods and agents used in children.
2. Disclosing solutions.
3. Role of toothbrushes.
4. Chemical control of dental plaque.
5. Composition of ideal dentifrice (toothpaste).
6. Flossing for children.
7. Brushing methods for children.
8. Chemicals used in control of plaque in preventive dentistry. [Same as SE Q.4]
9. Chemical plaque control agents. [Same as SE Q.4]
10. Toothbrushing techniques in children. [Same as SE Q.7]

SHORT NOTES:
1. Define dental plaque.
2. Commonly used dentifrices in children.
3. Disclosing solution.
4. Types of dental floss. [Ref SE Q.6]
5. Plaque control methods. [Same as SE Q.1]
6. Horizontal scrub technique.
7. Acquired pellicle.
8. Electrical toothbrushes.
9. Oral hygiene measures for handicapped children.
10. Fone’s method of toothbrushing.
11. Dentifrices for children. [Same as SN Q.2]
12. Modification of toothbrush for handicapped children. [Same as SN Q.9]

SOLVED ANSWERS
LONG ESSAYS:
Q.1. Write briefly about chemical control of dental l In addition, certain patients with dental diseases or
plaque. medical diseases require additional assistance be-
yond mechanotherapy to maintain a normal state of
Ans.
oral health.
Chemical plaque control: l In some patients who are unable, unwilling or un-
l Interest in chemical methods of plaque control has trained to practice, effective mechanotherapy has re-
dramatically increased due to advancement in sci- sulted in development of chemotherapeutic agents as
ence and research methodologies. adjuncts to plaque control.
448 Quick Review Series for BDS 4th Year, Vol 1

Ideal properties of an antiplaque agent are as follows: F. Fluorides and inorganic ions:
l Safe at concentration and dosage recommended. l Strontium fluoride, stannous fluoride, chlorine diox-
l Affects only the target tissue and bacteria known to ide, sodium bicarbonate, sodium chloride, etc.
cause gingivitis or periodontitis or both. G. Oxygenating agents:
l Meaningful reduction in gingivitis or periodontitis or l Hydrogen peroxide
both. H. Organic compounds/phenolic compounds
l It acts only on the tooth or root surface and not oral l Sanguinarine, thymol, menthol, eucalyptol, etc.
mucosa. I. Other antiseptics:
l Remains at the site of action and has a substantive effect. l Iodine and povidone iodine
l Desirable anamnestic characteristics to enhance l Sodium hypochlorite
compliance. l Hexetidine
l Economical. l Triclosan
Depending on antimicrobial efficacy and substantivity, The most commonly used ones are described below:
the chemical antiplaque agents are classified as follows: Chlorhexidine:
i. First generation agents: l It is classified into first or second generation anti-
l Reduces plaque scores by 20%–50%.
plaque agents.
l Efficacy is limited by their poor retention in
l The usual concentration of chlorhexidine used in
the oral cavity. mouth rinses is between 0.1% and 0.2%.
l Used 4–6 times daily due to poor substantivity.
l It has a broad spectrum of antimicrobial activity
For example: Antibiotics, quaternary ammo- against Gram-positive bacteria, Gram-negative
nium compounds, phenols and sanguinarine. bacteria and yeast and Streptococciis mutans.
ii. Second generation agents: Mechanism of action:
l These are retained longer in the oral cavity or
l It acts by blocking acidic groups on salivary
tissues. glycoproteins which in turn prevents pellicle
l Slow release property provides overall reduc-
formation.
tion in plaque score by 70%–90%. l It also retards the adsorption of plaque on to
l Used 1–2 times daily due to higher substan-
tooth surface by binding to the bacterial sur-
tivity. face in sublethal amounts and prevents matura-
For example: Bisbiguanides. tion of plaque and calculus formation by dis-
iii. Third generation agents: placing calcium from plaque matrix.
l It should be effective against specific peri-
l Chlorhexidine spray has been used for dis-
odontopathic organisms. abled persons with limited abilities for me-
l Yet to be developed clinically.
chanical plaque control.
The most commonly used antiplaque agents are classified Chlorhexidine with other agents:
into different groups: l It is an effective anticaries agent when com-
A. Antibiotics bined with topical fluoride gel and acts as an
l Penicillin
antimicrobial varnish when used along with
l Vancomycin
tincture of benzoin (chlorzion).
l Kanamycin
Adverse effects:
l Erythromycin
l Bitter taste.
l Spiramycin
l Staining of teeth, restorations and tongue.
l Metronidazole, etc.
l Desquamation and soreness of oral mucosa
B. Enzymes l Defective perception of sweet and salty taste.
l Dextranase glucose-amyloglucosidase
l As mentioned above, there are many other agents
l Amylase
used for plaque control in adults and their use in
l Lactoperoxidase, etc.
children needs further research.
C. Antiseptics (quaternary ammonium compounds): Triclosan
l Cetylpyridinium chloride
l It is a broad G spectrum antimicrobial effective
l Benzethonium chloride
against Gram-negative and Gram-positive
l Benzalkonium chloride, etc.
bacteria.
D. Bisbiguanides l It acts on the microbial cytoplasmic membrane
l Chlorhexidine
causing leakage of the cell contents or bacteri-
l Alexidine
olysis.
l Octenidine/bispyridines
l Its use has reduced plaque formation and de-
E. Metallic salts creased gingival bleeding.
l Copper, tin, zinc, etc.
Section | I  Topic-Wise Solved Questions of Previous Years 449

Q.2. Describe microbial aspects of oral hygiene and l Efforts of plaque control should be directed towards fol-
plaque formation. lowing goals:
i. Limiting the numbers of mutans streptococci in den-
Ans. tal plaques for prevention of caries by mechanical
The various biological factors responsible for the mainte- elimination of supragingival plaque and limitation of
nance of oral health are as follows: dietary sucrose
l Although Miller proposed in the late nineteenth century ii. Maintaining the predominantly Gram-positive flora
that microorganisms play a role in dental disease, de- associated with gingival health by mechanical removal
finitive evidence of the microbial aetiology of dental of plaque from the subgingival area on a regular basis
caries and periodontal diseases did not appear until l The use of chemotherapeutic agents, particularly chlorhexi-

three-fourths of a century later with the work of Keyes dine, can also play a role in maintenance of gingival health.
and of Loe and colleagues. l The incorporation of these methods into the daily rou-

l The major focus of dental research has been directed to tines of patients is perhaps the greatest challenge facing
define the specific microorganisms in dental plaque that the dentist.
mediate these diseases. Q.3. Define dentifrice and give composition of the same.
l Although pathogens are identified, mechanical removal of Describe briefly about fluoride dentifrices.
plaque and promotion of the remineralization of the tooth
Ans.
surface remain as primary tools in preventing dental diseases.
l The development of anaerobic culturing techniques and, l A dentifrice is a substance used with a toothbrush or
more recently, genetic techniques that allow for the de- other applicator to remove the dental plaque, materia
tection of uncultivable species have identified more than alba, debris, stains from the teeth, tongue and gingiva
700 bacterial species and numerous distinct bacterial for cosmetic, therapeutic or preventive purposes.
habitats in the mouth. l Dentifrices serve multiple functions in oral hygiene

l Only a limited number of species are found in high through the inclusion of a variety of agents.
numbers in dental plaque. These species are uniquely l Children’s dentifrice should contain fluoride, rank low

suited to this habitat. in abrasiveness and carry the ADA seal of acceptance.
l The formation of plaque on the tooth surface is charac- l Dentifrices are available in powder, paste and gel form:

terized by progression from a limited number of pioneer The powder form contains:
species mainly streptococci and other Gram-positive i. Abrasives, e.g. calcium carbonate and calcium
organisms to the complex flora of mature dental plaque. pyrophosphate
l This maturation involves initial adherence of bacteria to ii. Detergents, e.g. sodium lauryl sulphate
the salivary pellicle and subsequent formation of a com- iii. Flavouring agents, e.g. peppermint oil and menthol
plex multispecies biofilm. iv. Sweeteners, e.g. sorbitol and glycerine
l Most oral bacteria have evolved specific adherence l Paste and gel forms contain all the above-mentioned

mechanisms that enable them to colonize the tooth sur- components as well as following components:
face. In addition, bacteria undergo a number of pheno- i. Binders, e.g. organic hydrophilic colloids and algi-
typic changes as they initiate the formation of a biofilm. nates colloidal silica
The molecular mechanisms that underlie these pro- ii. Humectants, e.g. glycerine and sorbitol
cesses have been intensively studied. iii. Preservative, e.g. formaldehyde and dichlorinated
l Although the possibility of new methods of plaque con- phenols
trol exists, mechanical plaque removal with supplemen- iv. Water
tation by chemotherapeutic agents currently offers the v. Colouring agent
most practical method of controlling plaque. l A therapeutic dentifrice may have a chemical added for

l As plaque matures on the tooth surface, not only do a specific preventive or treatment outcome.
microbial changes occur, but mature dental plaques as- l These dentifrices for kids are not sold over the counter.

sociated with oral diseases appear to differ from those They are issued against a prescription by the dentist.
associated with oral health. l They have tartar control properties due to addition of

l In dental caries, the pathogenicity of plaque is related to pyrophosphates and anticaries properties through action
the numbers of Streptococcus mutans and related spe- of fluoride and other agents.
cies present. Fluoride dentifrices:
l The plaque associated with gingival inflammation is l Fluoride dentifrices play a significant role in caries

characterized by a predominance of Gram-negative bac- prevention. Stannous fluoride dentifrice was the first
teria rather than the predominantly Gram-positive flora to be recognized by the FDA in the year 1955, as an
found in oral health. effective tooth decay preventive product.
This transition seems to coincide with inflammatory l ADA accepted the fluoride dentifrice in the year

changes that occur at the gingival margin. 1964. The effect of these dentifrices on tooth
450 Quick Review Series for BDS 4th Year, Vol 1

surfaces can be of physio-mechanical and chemical The dental health for infants:
functions. l Generally it is recommended that parents start clear-

l The physio-mechanical effect of the toothbrush and ing the infant’s mouth by the time first tooth erupts.
dentifrice may reduce the cariogenicity of the remain- l The parents should wrap a damp washcloth or a
ing plaque by rinsing or flushing action which re- piece of gauze around the index finger and clean the
moves fermentable food debris which may form acid, teeth and gum pads once a day. As more teeth erupt,
thereby preventing formation of cariogenic plaque. the parents can begin using a small soft toothbrush.

l The chemical functions are based on the anticario- l The toothpaste is not necessary at this age.
genic property of fluoride which is mediated through l Several methods of positioning the infants for daily
the deposition of fluoride into enamel and plaque, or hygiene procedures have been suggested.
thus reducing the chances of enamel dissolution. One effective method is to have the parent cuddle the in-

l The composition of fluoride dentifrice is a mixture of fant in his/her arm with one of the child arms gently
an abrasive or polishing agent, a detergent, a binder, slipped around the parents back. In this way the parent can
favouring agents and substances and a fluoride agent. stabilize the child with one hand and work with the other.

l Exact formulations of dentifrices vary from manu- The dental health for toddler:
facturers’ specifications. There are different fluoride l The parent should be totally responsible for oral hy-
compounds present in dentifrices like sodium fluo- giene maintenance of the toddler. Establishing a
ride (0.188% to 0.254% approved by FDA), stannous specific routine is generally most convenient for par-
fluoride (1% of stannous pyrophosphate added to ents and encourages the young child to develop good
maintain soluble stannous ions), sodium monofluo- dental habits.
rophosphate (0.564% to 0.884%) and amine fluoride. l As more teeth begin to erupt, parents should ap-

l These fluoridated dentifrices will yield 800 to 1000 proach brushing in a systematic way by beginning in
ppm of fluoride. one area of the mouth and progressing in an orderly

l Presently best practice includes recommending twice fashion. This should be accomplished by the use of a
daily use of a fluoridated toothpaste for dentate chil- dampened cloth or soft bristled toothbrush.
dren in optimally fluoridated and fluoride-deficient l Although parents have the responsibility of perform-
communities. Twice daily use has benefits greater ing a thorough, daily plaque removal for their tod-
than once-daily brushing. dlers, children at this age begin to demonstrate an

l A ‘smear’ of fluoridated toothpaste for children less interest in the procedure and a desire to take part.
than 2 years of age may decrease risk of fluorosis. A l Parents should encourage the behaviour and allow
‘pea-size’ amount of toothpaste is appropriate for the child to attempt brushing procedures. Parents
children aged 2 to 5 years. should, however, be advised that the child efforts will

l For preschool-aged children, parents should dispense be inadequate in thoroughly removing plaque; hence
the toothpaste on to a soft, age-appropriate sized they must perform a thorough plaque removal for the
toothbrush and perform or assist with toothbrushing. child at least once a day.

l To maximize the beneficial effect of fluoride in the l Many of the techniques employed with the infants
toothpaste, rinsing after brushing should be kept to a for positioning and stabilizing may also be applied to
minimum or eliminated altogether. the toddler. One of the most effective positions is to
Q.4. Give detail account of maintaining optimum dental have the parents face each other while the child is
health in children. supine on the parent’s knees.
l In this position, one parent assumes the role of
Ans.
brusher while the other parent stabilizes the child.
Guidelines for maintaining optimum dental health in chil- l The preschool child is usually unable to expectorate
dren are as follows: effectively, and any dentifrice that is placed on the
Prenatal counselling: toothbrush is generally ingested. Repeated ingestion
l The goal of prenatal dental counselling is one coun- of large amount of dentifrice may increase the sys-
selling of dental health education even before the temic fluoride intake to undesirable levels.
baby is born. l Until the child can expectorate effectively, the parent
l Parents should be counselled on how to provide an should be responsible for dispensing the toothpaste
environment that will nurture good oral health habits and should place only a small pea-sized portion of
that contribute to life-long dental health for their child. dentifrice on the brush for the child.
l During this period the parents are more open to The early school age child:
health information for their child than during any l As they begin to develop the necessary skill, early
other time; hence, prenatal counselling can be quite school-aged children should be encouraged to rou-
effective. tinely attempt brushing and flossing.
Section | I  Topic-Wise Solved Questions of Previous Years 451

l The parents must still continue to maintain the major Category Oral hygiene measure
responsibility by providing a thorough plaque re-
Preschoolers l Children can manipulate toothbrush and
moval for the child each evening before bed. (3–6 years old) can be allowed to practice brushing.
l When one is teaching brushing and flossing tech- l Fluoride dentifrice can be introduced, use
niques, disclosing agents may be useful in this age a pea-sized toothpaste.
group. l Flossing can be done by the parent or

l The key to success of any oral hygiene programme


caretaker.
l Use of fluoride rinses can be started in
for the preadolescent child is to encourage parents to high caries risk children.
reinforce the instructions given in the dental office. l Frequency of cleaning – twice daily.
l After the child attempts plaque removal procedures, l Should be supervised by the parents for a

the parent can promote learning by staining the teeth complete brushing.
l Fone’s or circular technique of brushing
with disclosing solution and showing where the im-
should be followed.
provement is needed.
l The child should also be praised for his/her efforts School-aged l Children start brushing their teeth on their
when plaque has been successfully removed. Chil- children own under supervision of parents.
(6–12 years) l Flossing is taught to them.
dren in this age group generally demonstrate the l Use of disclosing agents can be initiated
ability to expectorate and should use a fluoridated for self-evaluation.
dentifrice each time they brush. l Use of fluoride rinses and pastes are rec-

The preadolescent: ommended.


l Cleaning should be performed twice daily.
l The children in this age group require instruction on
l Horizontal scrub or Bass method of brush-
proper brushing and flossing techniques. ing technique should be followed.
l During this period, the child will gradually assume

more responsibility for his/her own hygiene. Adolescents l May neglect oral care – persistent
(12–19 years) reinforcement is necessary.
l By 10 or 11 years of age, the child has often achieved
l Increasing their knowledge in good oral
the coordination necessary for effective brushing and health can help them to perform oral
flossing. hygiene measures adequately.
The adolescent: l Provider – self.

l Cleaning should be performed twice daily.


l In this stage generally the manual dexterity needed to
l Brushing technique – modified Stillman or
brush properly, and floss without direct help from an Charter’s method.
adult is attained.
l Although children in this age group probably have
Q.5. Describe the various biological factors responsible
the ability to adequately perform thorough oral hy-
for the maintenance of oral health.
giene procedure, they may lack the motivation to do
so on a routine basis. Ans.
Age-wise oral hygiene measure recommended for children
[Same as LE Q.2]
are as follows:

Category Oral hygiene measure SHORT ESSAYS:


Infants l Cleaning the gum pads with wet gauze
(0–1 year) and the erupted teeth with a finger brush Q.1. Discuss plaque control methods and agents used in
or small toothbrush. children.
l No need to use a dentifrice.

l Stabilizing the child by cradling with one Ans.


arm and cleaning with the other arm is
recommended.
l Parents should be providers.
{SN Q.5}
l Cleaning should be performed after every

meal or feed.
l Plaque control is the key to prevention and success-
ful treatment of periodontal diseases.
Toddlers l Toothbrush should be introduced along l The removal of plaque and the prevention of its ac-
(1–3 years) with pea-sized nonfluoridated dentifrice.
l Knee-to-knee position helps in carrying
cumulation on the teeth and adjacent gingival sur-
out oral hygiene measures. faces are known as plaque control.
l Parents should be providers.

l Cleaning should be performed twice daily.


452 Quick Review Series for BDS 4th Year, Vol 1

Indications:
l It is accomplished by professional plaque removal
i. Patient education:
and by patient performed oral hygiene.
l These agents are particularly helpful in teach-
l Removal of microbial plaque leads to resolution of
ing children toothbrushing techniques and ed-
gingival inflammation in its early stages, and cessa-
ucating them on the rationale for oral hygiene.
tion of plaque control measures leads to its recur-
ii. Assessment by the clinician and self-assessment
rence.
by the patient:
Some common methods and agents that are used in
l Use a topical application or provide diluted
plaque control are
concentrate for a rinse or request the patient
A. Mechanical plaque control:
to chew a tablet, swish for approximately 1
Some common agents that are used in mechanical
min and rinse.
plaque control are
l Stained plaque is pointed out and explained
l Dentifrice
how the bacteria must be removed to control
l Toothbrush
inflammation.
l Dental floss
iii. Evaluation of effectiveness of plaque control
l Oral irrigation
measures:
l Interdental cleaning aids
l The plaque score is recorded and explained to
B. Chemical plaque control:
the patient and kept for future reference.
Some common chemical agents that are com-
l For minimal amounts of plaque, the disclos-
monly used in plaque control are
ing solutions were found to be the most sensi-
Mouth rinses:
tive assessment techniques. For moderate and
i. Chlorhexidine
abundant plaque deposits, however, the probe
ii. Triclosan
techniques were more sensitive.
Medicated dentifrices:
i. Fluoridated toothpaste Q.3. Role of toothbrushes.
ii. Tetra sodium phosphate
Ans.
Antibiotics:
i. Penicillin l In 1938 first brushes made with nylon were introduced.
ii. Vancomycin Since then numerous designs of toothbrushes have
iii. Kanamycin come into the market.
iv. Erythromycin Some common designs of toothbrushes used in children:
Antiseptics: Size of toothbrush head:
i. Chloramines As recommended by American Dental Association is
ii. Chlorhexidine as follows:
iii. Alexdine l Length: 1–1.25 inch (25.4–31.8 mm).

l Width: 5/16–3/8 inch (7.9–9.5 mm).

l The brush head should have 2–4 rows of bristles


Q.2. Disclosing solutions. and 5–12 tufts per row.
Tufts (bristles) design:
Ans.
l Bristles are grouped in tufts that are arranged in
l Disclosing agents are solutions used to educate and three or four rows on the brush head.
motivate patients for performing better oral hygiene l Rounded bristle ends are preferred than flat-cut or
practices. coarse cut bristles as the former is more gentle on
l Several agents have been developed to allow for patient the gingiva.
visualization of plaque. l Diameters of commonly used bristles range from
Various substances used as disclosing agents are 0.007 inch for soft brushes to 0.012 inch for me-
i. Basic fuchsin dium bristles. Kid’s toothbrushes should have
ii. Erythrocin thinner 0.005 inches bristles.
iii. Fast green l The length of the bristles for an adult toothbrush
iv. Iodine should be 0.406 inches, whereas a kid’s tooth-
v. Gentian violet brush bristle should be 0.344 inches.
vi. Fluorescein l Generally softer bristles are more flexible and
vii. A two-tone disclosing agent use of hard bristles is associated with gingival
viii. Vegetable and food colouring dyes recession.
Section | I  Topic-Wise Solved Questions of Previous Years 453

Two types of bristle material used in dentistry are E. Sugar substitutes:


  i. Natural bristles from hogs l Xylitol and mannitol

ii. Artificial filaments of nylon F. Plaque attachment interference agents:


Majority of the toothbrushes in the market have l Sodium polyvinylphosphonic acid and perfluoro-

nylon bristles. alkyl


Indications for various types of brushes based on bristle
Q.5. Composition of ideal dentifrice (toothpaste).
texture:
Extra soft: tender gingival tissue, prone to bleeding Ans.
Soft: young children
Dentifrices:
Medium: most preferred
l According to the American Dental Association
Hard brush: brushing forces are heavy, in heavily
Council on Dental Therapeutics ‘A dentifrice is a
keratinized gingival tissues
substance used with a toothbrush for the purpose of
Extra-hard bristles: mostly not recommended
cleaning accessible surfaces of the teeth’.
Usage:
l Webster described the term dentifrice as derived
l The toothbrushes should be replaced every 3 months
from dens (tooth) and fricare (to rub).
or when it appears well worn.
l Dorland described it as a preparation for cleaning
Powered toothbrushes:
and polishing the tooth surfaces.
l Powered toothbrushes were introduced in 1939.
Composition of dentifrices:
These brushes had back and forth motions.
l Currently the electronic toothbrushes have oscil-

lating and rotating motions. Commonly used


l They are generally used in paediatric dentistry for
Ingredients chemicals % Functions
children who lack adequate manual dexterity to Polishing/ l Calcium car- 15%–45% i. Mechani-
manipulate a manual toothbrush. abrasive bonate cally clean
agents l Dicalcium the teeth
Q.4. Chemical control of dental plaque. phosphate de- ii. Removes
hydrate stained pelli-
Ans. l Alumina cle from the
l Silica tooth sur-
Chemotherapeutic plaque control agents: face, restores
A. Antiseptic agents: natural lustre
i. Positively charged organic molecules: iii. Mild abra-
l Quaternary ammonium compounds – cetyl-
sive action
that aids in
pyridinium chloride eliminating
l Pyrimidines – hexedine plaque from
l Bis-biguanides – chlorhexidine and alexidine tooth surface
ii. Noncharged phenolic agents: Binding/ Water-soluble Up to 2% i. Bind the sol-
Listerine (thymol, eucalyptol, menthol and meth- thickening agents: ids to form
ylsalicylate), triclosan, phenol and thymol. agents l Alginates homoge-
iii. Oxygenating agents: l Sodium car- neous paste
boxyl methyl and eases
Peroxides and perborate
cellulose dispersion of
iv. Bis-pyridines: Water-insoluble the paste in
Octenidine agents: the mouth
v. Halogens: l Magnesium al- ii. Control sta-
Iodine, iodophors and fluorides uminium sili- bility and
cate consistency
vi. Heavy metal salts:
l Colloidal silica of a tooth-
Silver, mercury, zinc, copper and tin l Sodium mag- paste
B. Antibiotics: nesium silicate
l Penicillin, kanamycin sulphate, tetracycline
Detergents Sodium lauryl 1%–5% i. Produce the
hydrochloride and vancomycin hydrochloride or sulphate foam, which
C. Enzymes: surfactants Sodium dodecyl aids in the
l Mucinases, pancreatin, fungal enzymes and sulphate removal of
protease food debris.
ii. Antimicro-
D. Plaque-modifying agents:
bial property
l Urea peroxide
454 Quick Review Series for BDS 4th Year, Vol 1

Commonly used Several types of dental floss are available they are
Ingredients chemicals % Functions i. Flavoured and unflavoured
Humectants Sorbitol 25%–40% i. Maintain the ii. Waxed and unwaxed
Glycerine consistency iii. Thin and thick
Polyethylene of the paste iv. Banded and nonbanded
glycol ii. Aids in re-
ducing the
v. Twisted and nontwisted
loss of mois- vi. Nylon and teflon
ture from the vii. Microfilament and multifilament
toothpaste Types of floss According to ADA specification are
Flavouring Peppermint oil 1% i. Render the i. Type I – Unbonded dental floss composed of
agents Spearmint oil product yarn having no additives
Oil of Winter- pleasant to ii. Type II – Bonded dental floss composed of yarn
green use having no additives other than binding agent or
ii. Leave a fresh
taste in the
agent cosmetic performance
mouth after iii. Type III – Bonded or unbonded having drug for
use therapeutic usage
l Based on the work by Bass, it is said that the nylon
Sweeteners Saccharin Up to 2% Impart sweetness
and and makes it unwaxed floss is generally considered the floss of
colouring more pleasant to choice because of the ease of use.
agents look at l Use of dental floss in children should be attempted

Water Double distilled 20%–30% Acts as vehicle cautiously under the supervision of parents.
water and solvent l Floss holders can be used.
medium
Preserva- Benzoic acid Up to Prevent microbial
tives 0.5% growth Techniques:
Therapeutic Tetrasodium Up to 2% Provide specific i. String floss method:
agents Pyrophosphatase therapeutic ac- l Use 18 inches length of floss.
Zinc chloride tion to the paste` l Wrap 2–3 inches of floss around middle finger of

left hand as well as to the right hand.


Toothpastes, with their cleaning potential by way of
l
ii. Circle of floss method:
detergents and toothpaste abrasives, have been rec-
l Take floss and tie a double knot to secure it. The
ognized for years, but now they are also considered
size of the circle is like an orange. Position the
to be an appropriate vehicle for the incorporation of
knot to the left side of working area and place
chemicals that may have a preventive and/or thera-
middle, little and ring fingers of both hand on the
peutic role in oral disease.
inside of circle to keep it taut. Rotate counter-
Q.6. Flossing for children. clockwise for fresh segments.
Application:
Ans.
Maxillary teeth:
l Direct the floss by holding the floss over two

thumbs or a thumb and an index finger. Rest a side


{SN Q.4} of a finger on teeth of opposite side of the arch to
provide balance and a fulcrum.
l In 1882, the first commercial dental floss made by
Mandibular teeth:
Codman and Shurtuff of Massachusetts was intro-
l Direct the floss down by holding the two index
duced.
fingers on top of the strand. One index finger
l Dental floss was first patented in 1876 and the Johnson
holds the floss on the lingual aspect and the other
and Johnson Co. manufactured silk dental floss as
on the facial aspect.
early as 1898.
l The side of the finger on the lingual side is held
l The effective use of dental floss helps to improve the
on the teeth of the opposite side of the mouth to
gingival health by effectively removing dental plaque
serve as a fulcrum or rest.
from the interdental areas.
l Hold floss firmly in a diagonal or oblique position.
l Size of dental floss can vary from 300 to 1500 de-
l Guide the floss past catch contact area with a
nier (D).
gentle saving motion.
Section | I  Topic-Wise Solved Questions of Previous Years 455

l Snapping the floss through the contact area should Method of Bristle Movement Advantages/
be avoided. brushing placement of bristles disadvantage
l Control floss to prevent snapping through the con-
Scrub tech- l Bristles l Scrub mo- Advantages:
tact area onto the gingival tissue. nique are placed tion in an- l Easy to learn and
l Clean adjacent teeth separately. horizon- terior-pos- master the tech-
l Slide the floss to a new, unused portion for suc- tally on terior nique.
ceeding proximal tooth surfaces. gingival direction l Best suited for

l Floss may be doubled to provide a wide rubbing


margin. keeping children.
brush hori- Disadvantages:
surface. zontal. l Inefficient plaque
l When a dentifrice is used with the floss, use den- removal.
tal tape. l It results in tooth

abrasion and gin-


Q.7. Brushing methods for children. gival recession.
l It is detrimental to
Ans. the general oral
Various types of brushing techniques used in children are as health.
follows: Charters tech- l Brush l Small cir- Advantages:
i. Sulcular technique – Bass method nique head is cular mo- l Increases the

ii. Roll technique – Modified Stillman placed tions are cleansing effec-
coronally, given with tiveness and gingi-
iii. Vibratory technique – Charter’s and Bass method 45° to apical val stimulation in
iv. Circular technique – Fone’s method long axis movement the interproximal
v. Horizontal technique – Horizontal Scrub method of the towards areas.
vi. Physiological technique – Smith method tooth. gingival Disadvantages:
l Ends of margin. l Hard to learn and
The detail description of each method of brushing is as
bristles position brush.
follows: are placed
Method of Bristle Movement Advantages/ half on
teeth and
brushing placement of bristles disadvantage
half on
Bass tech- Apically, to- l Short back Advantages: gingiva.
nique (intra- wards gingi- and forth l Easy to learn.

sulcular val sulcus at vibratory l Good gingival


Fone’s The brush is With teeth in Advantages:
method). 45°, to long motion is stimulation. method (cir- firmly placed occlusion, l It is very easy to

axis of tooth. given, l Removes plaque


cular scrub against the move brush in learn.
while bris- from interdental, method) teeth and rotary motion l Good gingival
tles remain cervical and sul- gingiva with over both stimulation.
in sulcus. cular areas. the bristles at arches and l Because of its sim-

l The occlu- Disadvantage: right angles gingival plicity, it is mainly


sal surfaces l It is time-
to buccal margin. indicated for
are brushed consuming. surfaces and school children/
with ante- l Overzealous
the handle young children,
rior poste- brushing may parallel with physically or emo-
rior short cause injury to the occlusal tionally handi-
strokes. gingival margin. plane. capped individuals
and patients who
Modified Bass l The bris- l A vibratory Advantages: lack dexterity.
technique tles are motion is l The technique is Disadvantages:
placed given in the easy to master. l Possible trauma to
sweeping gingival l It dislodges and gingival tissues.
down- sulci. removes the l Interproximal ar-
wards plaque from the eas are not prop-
over the cervical and inter- erly cleaned.
tooth sur- proximal areas of l Detrimental for
face. the teeth. adults especially
Disadvantages: who use the brush
l It is time- vigorously.
consuming.
456 Quick Review Series for BDS 4th Year, Vol 1

Method of Bristle Movement Advantages/ Q.10. Toothbrushing techniques in children.


brushing placement of bristles disadvantage Ans.
Roll tech- Apically, par- On buccal Advantages:
nique (ADA allel to tooth and lingual l It is more appro-
[Same as SE Q.7]
method/ and then inward pres- priate when the
sweep over tooth sure, then patient is in nor-
method/ surface. rolling of head mal health.
SHORT NOTES:
rolling stroke to sweep bris- l Easy to learn.

method) tle over gin- l Good gingival Q.1. Define dental plaque.
giva and stimulation.
tooth. Disadvantages: Ans.
l Difficult to learn

and requires suffi-


l Dental plaque is defined clinically as a structured resil-
cient dexterity. ient, yellowish grey substance consisting of bacterial
l Does not remove aggregations that adheres tenaciously to teeth and other
plaque from the intraoral hard surfaces such as restorations.
sulcus area. l Plaque is broadly differentiated into two categories,
l Possible laceration

of the alveolar
based on its relationship to the gingival margin:
mucosa. i. Supragingival plaque
ii. Subgingival plaque
Stillman’s Bristles are Keeping the l Excellent gingival
method placed on bristle ends stimulation. Q.2. Commonly used dentifrices in children.
buccal and stationary, l Moderate clean-

lingual sides, both on buc- ing of interproxi- Ans.


apically at cal and lin- mal areas.
an oblique gual sides Commonly available kid’s toothpastes in India are
angle to long slight rotary i. Kidodent:
axis of tooth; motions are It contains sodium monofluorophosphate 0.38% w/w,
ends rest on given. xylitol, fluoride content 500 ppm.
gingiva and
cervical part.
ii. Cheerio:
It contains sodium monofluorophosphate 0.35% w/w,
Modified Bristles are Apply pres- Advantages: fluoride content 458 ppm.
Stillman’s placed point- sure as in l Good gingival
method ing apically Stillman’s stimulation.
iii. Bubble:
at an angle method; the l Cleaning of inter- It contains sodium monofluorophosphate 0.35% w/w,
of 45° to brush is vi- proximal areas fluoride content 500 ppm.
tooth sur- brated mesio- and exposed area iv. Inissiate:
face. distally with a of tooth. It contains sodium monofluorophosphate 0.38% w/w,
gradual move- l Easy to master.
ment towards Disadvantages
xylitol, fluoride content 500 ppm.
occlusal l Improper brushing
Q.3. Disclosing solution.
plane. technique can
cause damage to Ans.
the epithelial at-
tachment. l Disclosing agents are solutions used to educate and
l Chances of miss- motivate patients for performing better oral hygiene
ing the gingival practices.
and cervical areas
of teeth, thus leav-
Various substances used as disclosing agents are
ing behind plaque. i. Basic fuchsin
ii. Erythrocin
Q.8. Chemicals used in control of plaque in preventive iii. Fast green
dentistry. iv. Gentian viole
v. Fluorescein, etc.
Ans. Indications:
[Same as SE Q.4] i. Patient education: It allows for patient visualiza-
tion of plaque.
Q.9. Chemical plaque control agents. ii. Assessment by the clinician and self-assessment by
Ans. the patient.
iii. Evaluation of effectiveness of plaque control
[Same as SE Q.4] measures.
Section | I  Topic-Wise Solved Questions of Previous Years 457

Q.4. Types of dental floss. l Braun-Oral B kids power toothbrush DIG is most effec-
tive in removing plaque in children. It has an oscillatory
Ans.
round brush head so causes no soft tissue damage.
[Ref SE Q.6] Advantages of powered toothbrushes:
l It increases patient motivation.
Q.5. Plaque control methods.
l Increased accessibility in interproximal and lingual

Ans. tooth surfaces.


[Same as SE Q.1] Q.9. Oral hygiene measures for handicapped children.
Q.6. Horizontal scrub technique. Ans.
Ans. l Home dental care for handicapped children should be-
l In horizontal scrub technique, bristles of toothbrush are gin in infancy. The dentist should instruct the parents to
placed horizontally on gingival margin with motion of gently cleanse the teeth daily with a soft cloth or finger
scrub in anterior–posterior direction keeping brush brush or an infant toothbrush.
horizontal. l For older children who are unwilling or physically un-

l Advantages: able to cooperate, the dentist should teach the parent or


i. Easy to learn guardian correct toothbrushing techniques.
ii. Best suited for children l If a child is institutionalized, the staff should be in-

structed in the proper dental care regimen for the child.


Q.7. Acquired pellicle. l A soft multitufted nylon brush should be used.

Ans. l A powered toothbrush will be helpful.

l Mouth rinses can be used.


l A thin, saliva derived layer, called the acquired pellicle, l Modification on the brush handle may be necessary.
covers the tooth surface and it consists of numerous
components, including glycoproteins, proline-rich pro- Q.10. Fone’s method of toothbrushing.
teins, phosphoproteins, histidine-rich proteins, enzymes
and other molecules that can function as adhesion sites Ans.
for bacteria. l Fone’s technique of brushing uses a circular motion and
l This involves the adsorption of positively charged sali-
it may be recommended as an easy to learn first tech-
vary, crevicular fluid and other environmental macro- nique for young children.
molecules to negatively charged hydroxyapatite sur- Technique:
faces of teeth through electrostatic, van der Waals and l With the teeth closed, a fast wide circular motion is
hydrophobic forces. used that sweeps from the maxillary gingiva to the
l Although this pellicle is protective in nature, providing
mandibular gingiva with very little pressure.
lubrication and preventing tissue desiccation, its forma- l The lingual and palatal surfaces need an in-and-out
tion on the teeth surfaces forms the substrate for coloni- stroke.
zation and subsequent proliferation of microorganisms. l This technique is very effective for young children

Q.8. Electrical toothbrushes. with minimal manual dexterity, while it is considered


detrimental to adults.
Ans.
l Powered toothbrushes are also known as automatic Q.11. Dentifrices for children.
toothbrushes or electric toothbrushes. Ans.
l Powered toothbrushes were introduced in 1939 and are

recommended for: [Same as SN Q.2]


l Individuals lacking fine motor skills

l Young children or handicapped or hospitalized pa- Q.12. Modification of toothbrush for handicapped
tients who need to have their teeth cleaned by some- children.
one else Ans.
l Patients with orthodontic appliances

l Patients who prefer them [Same as SN Q.9]


458 Quick Review Series for BDS 4th Year, Vol 1

Topic 17
Dental Caries in Children and Adolescents
COMMONLY ASKED QUESTIONS
LONG ESSAYS:
1. Define rampant caries. A 2-year-old child comes to you with a complaint of blackish discolouration of upper
incisors. Explain in detail your evaluation of this case and step-by-step management of the problem.
2. Discuss in detail the current concepts of caries predication in paediatric patient.
3. Classify dental caries. Give detailed description of how will you treat a case of nursing bottle caries.
4. Define rampant caries. How would you differentiate rampant caries from nursing bottle caries and also distin-
guish active (acute) caries and chronic/arrested caries.
5. Define early childhood caries. Write a note on window of infectivity. Explain the stages and management of
ECC.
6. Define diet and nutrition. Enumerate caries-inducing factors and caries protective factors in diet. Why sucrose
is called ‘Arch-criminal of dental caries’? Specify along with Stephan’s curve. Describe ‘Hopewood House
study’.
7. What are the different caries activity tests? State their importance. Describe Snyder’s test and discuss its
applicability in preventive dentistry.
8. Discuss about role of carbohydrates in dental caries.
9. Define rampant caries. Discuss its management. [Same as LE Q.1]
10. Define rampart caries. Write in detail about it. [Same as LE Q.1]
11. Define rampart caries. Discuss the aetiology, clinical features and step-by-step management of the same.
[Same as LE Q.1]
12. What is early childhood caries? Write in detail about the management and prevention of early childhood car-
ies. [Same as LE Q.5]
13. Discuss about cariogenic potential of diet in dental caries. [Same as LE Q.8]

SHORT ESSAYS:
1. What is plaque? Mention its role in dental caries.
2. Zones of dental caries in enamel and dentin.
3. Hopewood House study.
4. Diet and dental caries.
5. Cariogenic diet substitutes.
6. Preventive protocol for nursing caries.
7. Snyder’s test.
8. Caries activity tests.
9. Milk bottle syndrome.
10. Difference between ‘nursing caries’ and ‘rampant caries’.
11. Prevention of early childhood caries.
12. Acidogenic theory.
13. Diet factors in dental caries. [Same as SE Q.4]
14. Role of diet in the aetiology of dental caries. [Same as SE Q.4]
15. Role of sugar substitutes for the prevention of dental caries. [Same as SE Q.5]
16. Anticariogenic diet. [Same as SE Q.5]
17. Nursing caries. [Same as SE Q.6]
18. Nursing bottle caries. [Same as SE Q.6]
19. Caries activity tests/Snyder’s test. [Same as SE Q.7]
20. Nursing versus rampant caries. [Same as SE Q.10]
Section | I  Topic-Wise Solved Questions of Previous Years 459

SHORT NOTES:
1. Caries vaccine.
2. Salivary defence factors.
3. Give in short the importance of dietary record for paediatric patient.
4. Caries severity index for primary teeth.
5. Caries detection.
6. Pre-eruptive caries.
7. Rampant caries.
8. Nursing bottle caries.
9. Guidelines for prevention of baby bottle syndrome.
10. Trace elements.
11. Sugar substitutes.
12. Caries is combined process of demineralization/remineralization – justify.
13. Critical pH.
14. Diet analysis.
15. Window of infectivity.
16. Hidden caries.
17. Stephen’s curve.
18. Sucrose an arch criminal – Comment.
19. Arrested caries.
20. Fluoride varnishes.
21. Hopewood study. [Same as SE Q.3]
22. Diet counselling.
23. Define early childhood caries and severe early childhood caries.
24. Snyder’s test.
25. Define balanced diet.
26. Incipient caries.
27. Dyes used to detect caries.
28. Low calorie sweeteners.
29. White spot lesion.
30. Shoe leather survey.
31. Define dietary caries control and diet counselling.
32. Define caries vaccine. Discuss its routes of administration. [Same as SN Q.1]
33. Methods of caries detection. [Same as SN Q.5]
34. Define rampart caries. [Same as SN Q.7]
35. Cariostatic trace elements. [Same as SN Q.10]
36. Classify sugar substitutes. [Same as SN Q.11]
37. Diet charting. [Same as SN Q.14]
38. Diet analysis of sugar exposure. [Same as SN Q.14]
39. Diet diary. [Same as SN Q.14]
40. Define diet and nutrition. [Same as SN Q.22]

SOLVED ANSWERS
LONG ESSAYS:
Q.1. Define rampant caries. A 2-year-old child comes to type of caries, resulting in early involvement of the pulp
you with a complaint of blackish discolouration of up- and affecting those teeth usually regarded as immune to
per incisors. Explain in detail your evaluation of this ordinary decay’.
case and step by-step management of the problem. l Winter et al. (1966) defined rampant caries as caries of

acute onset involving many or all the teeth in areas that


Ans.
are usually not susceptible. They further defined the
l According to Massler, 1945 rampant caries is defined as condition to be associated with rapid destruction of
a ‘suddenly appearing, widespread, rapidly burrowing crowns with frequent involvement of dental pulp.
460 Quick Review Series for BDS 4th Year, Vol 1

Predisposing factors: Clinical features:


According to McDonald, some of the factors which may l Rampant caries involves surfaces of teeth that are

predispose an individual to rampant caries are as fol- ordinarily relatively caries-free.


lows: l Proximal and cervical surfaces of anterior teeth, in-

l Repressed emotions and fears. cluding the mandibular incisors which are relatively
l A traumatic school experience. caries-free, may be affected.
l Dissatisfaction with achievement. l Most often observed in the primary dentition of

l Rebellion against a home situation. young children and the permanent dentition of teen-
l Continuous general tension and anxiety. agers.
Such emotional disturbances may create an unusual l Dietary factors affecting oral substrate and oral flora

craving for sweets or the habit of snacking which in turn and physiological factors affecting saliva are often
might influence the incidence of dental caries. significant in the development of rampant caries.
l A noticeable decrease in salivary flow is observed in l Types of rampant caries are most common in infancy

individuals with emotional disturbance which can in- or sooner caries and adolescent caries.
crease their risk for caries. Management of rampant caries:
l Radiation therapy for any tumour of head and neck can Factors involved while treating a case of rampant
also lead to decreased salivary flow leading rampant caries. caries are
l Patient motivation
Aetiology:
l Extent of the decay
The two major predisposing factors in rampant
l Age
caries are
l Cooperation of the child
a. Specific microorganisms
Factors to be assessed:
b. Diet
l Initial treatment
Microorganisms:
l Provisional restorations
l Streptococcus mutans is an important pathogen in
l Diet assessment
the development of dental caries. The main source of
Treatment consists of:
S. mutans is primary infection in mother.
l Oral hygiene instructions.
l The cariogenicity of S. mutans is probably related to
l Home and professional restorations.
its unique combination of properties, which include:
l Comprehensive restorative treatment.
i. Colonization of teeth
l In patients presenting with acute and severe
ii. Production of large amount of extracellular
signs and symptoms of gross caries, pain,
polysaccharides that enable voluminous plaque
abscess, sinus or facial swelling immediate
formation
treatment is indicated.
iii. Production of large amounts of acids, even with
l Once the history is received, recommenda-
low pH
tions can be offered.
iv. Breakdown of salivary glycoprotein, which might
l Food intake and dietary habits are very dif-
be of great importance for initiation of caries
ficult to modify.
Diet:
l Successful management of rampant caries
l The classic Swedish Vipeholm study demonstrated
necessitates severe dietary modifications.
that cariogenic potential is closely related to the tex-
ture of the carbohydrate and the frequency of con- Prevention of rampant caries
sumption of sticky sugars, rather than to the amount
Dentition: 0–5 years
of sugar eaten.
l The carbohydrate component of the diet is associated Advice: Diet counselling with parents on good nursing
techniques
with the formation of dental caries.
Carbohydrate component particularly sucrose is con- Therapy: Toothpaste
sidered to be the most cariogenic sugar in human diet. Tablets if in area without water fluoridation
Professional topical fluoride application every
l It is highly soluble and acts as a substrate both for the
6 months
production of extracellular polysaccharide and for
acid production. Control: Oral hygiene instructions to parents
Toothbrushing with parental supervision list to
l It favours the establishment of S. mutans on the teeth.
dental office at around 12 months of age to
l Breast milk contains a higher amount of lactose than 6-month recall
bovine milk, so greater caries potential.
Section | I  Topic-Wise Solved Questions of Previous Years 461

Dentition: 5–12 years


Type 0–2 years 2–3 years 3–13 years .13 years
Advice: Diet counselling with parents and patients
Self- Not indi- Not indi- Self-appli- Self-appli-
Therapy: Toothpaste applied cated cated cation of cation of
Tablets up to 8 years if in area without water topical gel tray gel tray
fluoridation fluoride daily for daily for
Mouthrinse approxi- approxi-
Professional topical fluoride application every mately mately
6 months 4 weeks; 4 weeks;
thereafter thereafter
Control: Oral hygiene instructions to patient continue continue
Toothbrushing without parental supervision with a with a
Disclosing tablets daily fluo- daily fluo-
Sealants month recalls ride rinse ride rinse
(0.05% (0.05%
Permanent dentition: 12 years onward NaF) NaF)

Advice: Diet counselling with parents and patients Fluoride Brush with Brush with Brush with Brush with
dentifrice F-contain- F-contain- F-contain- F-contain-
Therapy: Toothpaste
ing denti- ing denti- ing denti- ing denti-
Mouth rinse
frice frice frice frice
Professional topical fluoride application every
6 months

Control: Oral hygiene instructions to patient


Q.2. Discuss in detail the current concepts of caries
Toothbrushing predication in paediatric patient.
Disclosing tablets
Interdental cleaning with floss or toothpicks
Ans.

Sealants month recalls. Current concept of dental caries predication are as


follows:
Restorative strategies for rampant caries: l In the epidemiological model, a disease state is due
i. Early caries with minimal loss of enamel: to interplay of three primary factors. According to
Weekly professionally applied topical fluoride therapy Keyes and Jordan, 1960:
ii. Extensive cavitation with no pulpal involvement: i. The host
a. Anterior teeth: ii. The agent or recruiting factor
l Acid-etched composite resin restorations
iii. Environmental influences
l Paedo-strip crowns
l Interaction between three primary factors is essential
l Glass ionomer cement restorations
for the initiation and progression of caries:
b. Posterior teeth: i. A susceptible host tissue, i.e. the tooth
l Posterior composite cement restorations
ii. Microflora with a cariogenic potential
l Glass ionomer cement restorations
iii. A suitable local substrate to meet the require-
l Stainless steel crowns
ments of the pathodontic flora
c. Extensive cavitation with pulpal involvement: l The tooth is the target tissue destroyed in the dental
l Pulpotomy or pulpectomy, where appropriate,
caries process.
followed by permanent restoration l The cariogenic oral flora, which is localized to specific
l Extraction followed by space maintainer or
sites on teeth, is the agent that produces and secretes
partial or complete dentures the chemical substances that causes the destruction of
the inorganic components and the subsequent break-
Fluoride treatment for children with rampant caries down of the organic moieties of enamel and dentin.
(0.3–0.7 ppm water fluoride level) l The local substrate provides the nutritional and en-

Type 0–2 years 2–3 years 3–13 years .13 years


ergy requirement for the oral microflora, thereby
permitting them to colonize, grow and metabolize on
Dietary flu- Not indi- 0.25 mg F 0.5 mg F Not indi-
selective surfaces of teeth.
oride sup- cated daily daily cated
l The third factor, the resistance of the tooth, is obvi-
plement
ously important since this determines the overall
Operator- APF topi- APF topi- APF topi- APF topi-
applied cal solu- cal solu- cal solu- cal solu-
effects of the attack.
topical flu- tion or gel, tion or gel, tion or gel tion or gel, l The hypothesis that bacteria are a prerequisite for the

oride 1.23% F, 1.23% F, 1.23% F, 1.23% F, initiation and progression of dental caries was clinched
applied applied applied applied by Orland (1954) at the University of Chicago. This
four times four times four times four times study laid to rest debates extending over a century
a year a year a years a year
about the role of bacteria in dental caries.
462 Quick Review Series for BDS 4th Year, Vol 1

Fitzgerald in 1968 concluded that: b. Dynamics, i.e. according to severity and rate of pro-
l Microorganisms are a prerequisite for caries gression of lesion
initiation. c. Chronology, i.e. according to age patterns at which
l A single type of organism is capable of inducing lesions predominate
caries. I. Based on morphology or anatomical position:
l The ability of producing acid is prerequisite for i. Occlusal or pit and fissure
caries induction but not all acid-producing organ- ii. Smooth surface:
isms are cariogenic. (a)   Proximal
l Organisms vary greatly in their capacity (viru- (b)   Cervical
lence) to induce caries. iii. Root caries (Billing’s 1986)
The current determinants of dental caries can be summa- (a) Grade I initial
rized as: (b) Grade II shallow
(c) Grade III cavitations
Bacterial Mechanism Role
(d) Grade IV pulpal
Primary Secondary metabo- of destruc- in
II. According to severity and progression:
factors factors lites tion caries
i. Incipient caries
Plaque Oral hygiene Organic Dissolve Major
l
ii. Rampant caries
l Oral Flora acids Organic
l Saliva – pH, phase iii. Arrested caries or eburnations of dentin
composition, iv. Recurrent caries or secondary caries
flow, buffer v. Radiation caries
capacity vi. Occult or hidden caries (not clinical but radio-
l Fluoride in
graphic destruction)
plaque
l Diet
III. According to chronology:
l Transmissibility i. Early childhood caries (ECCs)
ii. Nursing bottle caries
Substrate l Type of carbo- Chelators Bind cal- Minor
hydrates proteolytic cium ions; Major ii. Rampant caries
l Chemical com- enzymes breakdown in den- iii. Linear enamel caries or odontoclasia (neonatal
position of food of organic tinal line)
l Physical charac- matrix caries iv. Teenage caries:
teristics of food
(a)   Adolescent caries
l Oral clearance

l Frequency of
(b)   Adult caries
eating Clinical classification of dental caries:
l Sugar intake A. According to location:
and frequency l Pit and fissure caries

Tooth l Fluoride con- Major l Smooth surface caries


centration l Root caries
l Carbonate and
B. According to rapidity with which it progresses:
citrate level
l Acute dental caries
l Age of tooth
l Chronic dental caries
l Morphology of

tooth C. According to whether the lesion is a new one or


l Trace elements not:
l Nutrition
l Primary caries
l Saliva
l Secondary caries
l Composition of

enamel D. Based on extent of damage:


l Incipient caries

l Occult caries
Q.3. Classify dental caries. Give detailed description of
Other classifications:
how will you treat a case of nursing bottle caries.
Based on the number of surfaces involved:
Ans. i. Simple caries – one surface of the tooth
ii. Compound caries – two surfaces of tooth
l There is currently no universally accepted classification
iii. Complex caries – three or more surfaces
of dental caries.
Based on the treatment and restorative design:
l On the basis of clinical features and patterns, dental car-
i. Class 1 lesion
ies may be classified according to three basic factors:
ii. Class 2 lesion
a. Morphology, i.e. according to anatomical site of the
iii. Class 3 lesion
lesion
Section | I  Topic-Wise Solved Questions of Previous Years 463

iv. Class 3 lesion


Oral hygiene instruction.
l
v. Class 5 lesion
Home and professional restorations.
l
vi. Class 6 lesion l
Comprehensive restorative treatment.
Bitewing radiograph classification (by Grondahl l
Patient presenting with acute and severe signs and
et al., 1977): symptoms of gross caries, pain, abscess, sinus or
0 – Sound on the bitewing facial swelling immediate treatment is indicated.
1 – Radiolucency confined to enamel l Once the history is received, recommendations can
2 – Radiolucency up to Dentino enamel junction (DEJ) be offered.
3 – Radiolucency in enamel and outer half of dentin l Food intake and dietary habits are very difficult to
4 – Radiolucency in enamel and reaching to inner modify. Successful management of rampant caries
half of enamel necessitates severe dietary modifications.
Classification of proximal caries on radiograph: Prevention of nursing bottle caries:
(According to Norwegian system by Espelid and
Tveit) Dentition: 0–5 years
Grade 1 lesion on the outer half of the enamel Advice Diet counselling with parents on good nursing
Grade 2 lesion on the inner half of the enamel but techniques
not into the dentin Therapy Toothpaste
Grade 3 lesion into the outer third of the dentin Tablets if in area without water fluoridation
Grade 4 lesion into middle third of dentin Professional topical fluoride application every
6 months
Grade 5 lesion into the inner third of dentin
According to the World Health Organization (WHO) Control Oral hygiene instructions to parents
system, caries lesion can be scored on a four point Toothbrushing with parental supervision list to den-
tal office at around 12 months of age to 6-month
scale. recall
D 1 – clinically detectable enamel lesions with intact
surface Restorative strategies for nursing bottle caries:
D 2 – clinically detectable cavities limited to the Early caries with minimal loss of enamel:
enamel l Weekly professionally applied topical fluoride
D3 – clinically detectable lesions in dentin (with or therapy
without cavitation of dentin) Extensive cavitation with no pulpal involvement:
D 4 – lesions into the pulp Anterior teeth:
Classification of root caries (by Billings, 1986) l Acid-etched composite resin restorations
Grade 1 (incipient) l Paedo-strip crowns
Grade 2 (shallow) l Glass ionomer cement restorations
Grade 3 (cavitation) Posterior teeth:
Grade 4 (pulpal) l Posterior composite cement restorations
Treatment of nursing bottle caries: l Glass ionomer cement restorations
l Nursing bottle caries is a form of rampant dental car- l Stainless steel crowns
ies in the primary dentition of infants and children. It Extensive cavitation with pulpal involvement:
is characterized by rampant caries pattern initially l Pulpotomy or pulpectomy, where appropriate,
involving first deciduous anterior teeth, posterior followed by permanent restoration
teeth are then involved and mandibular anterior are l Extraction followed by space maintainer or partial
usually spared. or complete dentures
Factors involved while treating a case of nursing bottle caries
are as follows: Q.4. Define rampant caries. How would you differenti-
l Patient motivation
ate rampant caries from nursing bottle caries and also
l Extent of the decay
distinguish active (acute) caries and chronic/arrested
l Age
caries.
l Cooperation of the child Ans.
Factors to be assessed prior to treating a child with nursing
bottle caries are l Rampant caries is defined as a ‘ suddenly appearing,
l Initial treatment.
widespread, rapidly burrowing type of caries, resulting
l Provisional restorations.
in early involvement of the pulp and affecting those
l Diet assessment.
teeth usually regarded as immune to ordinary decay’.
464 Quick Review Series for BDS 4th Year, Vol 1

Differences between nursing bottle caries and rampant l The most beneficial time for vaccination against dental
caries are as follows: caries would be in infancy prior to eruption of teeth.
l This would promote the induction of adherence in-
Rampant caries Nursing bottle caries hibiting salivary IgA thus delaying colonization of
Aetiology: Aetiology: S. mutans.
Inadequate maintenance of Prolonged bottle feeding. l With the establishment of early colonizers there
oral hygiene.
would be a synergistic effect of suppressing the colo-
Involvement of proximal sur- Involvement of maxillary ante- nization of S. mutans during the time span of window
face of lower anterior teeth rior teeth, the maxillary and
of infectivity.
and development of cervical mandibular posterior teeth
l A booster dose of vaccination may be required at
type of caries is the main and mandibular canine is
characteristic. seen. time of eruption of first permanent molars.
Clinical features of early childhood caries:
Mandibular incisors are Mandibular incisors are not
l Early childhood caries can develop as soon as teeth
affected. affected due to protection
from the tongue. erupt.
l Cavities may be visible as early as 10 months of age.

l It occurs in following stages:


Differences between acute (active) caries and chronic
(arrested) caries are as follows:
i. Very mild:
Clinical appearance shows slight demineraliza-
Acute dental caries Chronic dental caries tion usually at gingival crest and no cavitation.
This type of caries runs a rapid This form of caries progresses ii. Mild:
clinical course and results in slowly and tends to involve Clinical appearance shows demineralization in
early pulp involvement by the the pulp much later than gingival third of tooth and moderate cavitation.
carious process. acute dental caries.
iii. Moderate:
It occurs most often in chil- It is seen most commonly in Clinical appearance shows frank cavitation on
dren and young adolescents adults.
multiple tooth surfaces.
because the dentinal tubules
are large and open and show iv. Severe:
no sclerosis. l Clinical appearance consists of widespread

destruction of tooth and partial to complete


The process is so rapid that The slow progress of the lesion
there is little or no time for de- often allows deposition of loss of clinical crown typically presents in
position of secondary dentin. secondary dentin. children as white lines or spots on the maxil-
lary incisors, which are among the first teeth
Early childhood caries caused The entrance to the lesion is
by prolonged bottle feeding almost invariably larger than to erupt and least protected by saliva.
and on demand breast-feeding that of acute caries. Because l Caries progresses from decalcification of up-
can be categorized into a form of this there is not only less per primary incisors to primary molars and
of acute dental caries. food retention but also greater canines, if not controlled.
cleansing by saliva.
l While the four upper incisors are the most

severely affected by ECC, lower incisors re-


Q.5. Define early childhood caries. Write a note on main intact because they are protected by the
window of infectivity. Explain the stages and manage- tongue and moistened by the saliva from sub-
ment of ECC. mandibular salivary glands.
l The distribution of ECC follows the sequence
Ans.
of eruption. If left untreated, the decay of the
l Davies in 1998 defined early childhood caries as a com- maxillary incisors eventually continues to
plex disease involving maxillary primary incisors within such an extent that the crowns are weakened
a month after eruption and spreading rapidly to other and fractured. This process may be so rapid
primary teeth. that the parents often perceive the teeth as
l American Academy of Paediatric Dentistry (AAPD) defected from the moment of eruption.
defines early childhood caries as ‘the presence of one or Management of early childhood caries:
more decayed (noncavitated or cavitated lesions), miss- l Managing ECC is a challenging and an arduous task
ing (due to caries) or filled tooth surfaces’ in any pri- for even a trained paediatric dentist.
mary tooth in a child 71 months of age or younger. l Children aged 3 years and younger are categorized
Window of infectivity: by Wright as ‘children who lack cooperative ability’.
l Caufield (1996) stated that there is a window of in- l Hence treatment for ECC in such situations may
fectivity between 19 and 33 months during which need pharmacological management like sedation and
teeth get infected with S. mutans. general anaesthesia.
Section | I  Topic-Wise Solved Questions of Previous Years 465

Very mild: Some teeth may need restorations while some may
l

l When the clinical appearance do not show any cavi- need stainless steel crowns with or without pulp
tation on the teeth surfaces, it is possible to arrest the therapy, etc.
process by active preventive measures. l Hence treatment planning for such children depends

l If appropriate preventive measures are not taken, the on the extension of decay, number of teeth involved,
possibility of developing a full blown ECC is put age of the child, risk for future caries, parental atti-
forward to parents. The consequences of severe ECC tudes towards oral health, cost of the treatment and
are described to them with the possibility of an inter- availability of the specialist and facilities.
vention in the hospital under general anaesthesia. View on benefits of treatment under general anaesthesia:
l Counselling regarding diet, oral hygiene measures, i. The anterior decayed teeth can be pulpectomised
fluoride adequacy and the need for frequent recall to and strip crowns or anterior stainless steel crowns
monitor the progress of the ECC are explained to can be placed for children going to school. Com-
them. posite restorations can be carried out comfortably
l The need for assistance in performing oral hygiene to the highest quality as the child is sleeping.
measures (mother or father brushing the child’s ii. Once the posterior teeth with deep cavities are
teeth) is also emphasized. Systemic fluorides are treated and restored with crowns, the eating pattern
given, if necessary, in the form of drops or tablets of the infant improves to a great extent.
depending upon the age and the other sources of iii. As the active caries lesions are arrested, the micro-
fluoride intake for the child. bial population of the mouth decreases gradually
l If proper care is taken by the parents and the profes- over a period of time which, in turn decreases the
sional advice is followed meticulously, this process risk of new cavities on newly erupting primary or
of very mild ECC can be arrested at that stage itself. permanent dentition.
Mild: iv. As the sensitive tooth surfaces are covered and the
l Children or infants with mild ECC need intervention carious teeth are restored, the child can allow or
of the carious process by a dentist who can handle perform better oral hygiene practices like brushing.
children and infants. v. As the entire treatment is carried out in one visit
l If the child does not cooperate for the procedure, under general anaesthesia, the child will not have
then treatment under general anaesthesia can be rec- any unpleasant dental experience.
ommended and carried out. vi. Behaviour of some children improves after a hospi-
l Controlling the active caries process at this stage will talization procedure.
prevent an extensive and invasive treatment with Prevention of early childhood caries:
multiple pulp therapies and crowns at a later stage, l Early screening for signs of caries development,

under general anaesthesia. starting from the first year of life, could identify in-
l Most parents of these patients would have met a fants and toddlers showing the risk of developing
dentist at this stage of ECC, where they were advised ECC and could also assist in providing information
against any treatment stating that these are milk teeth to parents about how to promote oral health and pre-
and they will be replaced. vent the development of tooth decay.
l Unfortunately, these children are left untreated and l High-risk children should be targeted with a profes-

not referred to the right sources also. sional preventive programme that includes fluoride var-
Moderate and severe: nish application, fluoridated dentifrices, fluoride supple-
l Children with moderate and severe ECC may need ments, sealants, diet counselling and chlorhexidine.
pharmacological intervention. l Prevention of ECC also requires addressing the so-

l Children with multiple decayed teeth and with deep cial and economic factors that are faced by many
caries lesions on the posterior teeth need multiple families where ECC is endemic.
appointments and many local anaesthetic injections l The education of mothers or care givers to promote

to carry out pulp therapy and stainless steel crowns healthy dietary habits in infants has been the main
in a regular dental setting. strategy used for the prevention of ECC.
l Children of even 4–5 years also may become unco- l There are three general approaches that have been

operative after the initial visits with injections, etc. used to prevent ECC:
l However, the benefits and the risks involved should a. First is the community-based strategy.
be weighed for each patient and a decision should be b. Second approach is based on the provision of ex-
taken accordingly. amination and preventive care in dental clinics.
l Children with ECC usually present with multiple c. The third approach involves the development of
decayed teeth at various stages of progression. appropriate dietary and self-care habits at home.
466 Quick Review Series for BDS 4th Year, Vol 1

Q.6. Define diet and nutrition. Enumerate caries induc- ‘Hopewood House study’:
ing factors and caries protective factors in diet. Why l The Hopewood House ‘is the home for orphans in Aus-

sucrose is called ‘arch-criminal of dental caries’. Specify tralia where the children were brought up from infancy’.
along with Stephan’s curve? Describe ‘Hopewood House l They were on vegetarian diet with occasional serving

study’. of egg yolk. Sugar and other refined carbohydrates


were excluded from the diet of children.
Ans.
l The caries incidence of primary dentition was almost

Diet: negligible and that of permanent teeth was 1/10th of


Diet refers to the customary allowance of food and the average Australian child.
drink taken by any person from day to day. l The oral hygiene was extremely poor and about 75%

Nutrition: had gingivitis.


Nutrition is the provision of the materials to cells and When these children left the institution and exposed
organisms necessary to support life. to routine diet, there was a sharp rise in caries rate.
Caries inducing factors: l This work shows that in institutionalized children, at

i. Carbohydrate content of diet least, dental caries can be reduced to insignificant


ii. Sugar levels by a spartan diet, and without beneficial influ-
Caries protective factors: ence of fluoride and in the presence of unfavourable
i. Phosphates oral hygiene.
a. Trimetaphosphates
b. Sodium metaphosphates Q.7. What are the different caries activity tests? State
c. Calcium sucrose phosphate their importance. Describe Snyder’s test and discuss its
d. Organic phosphates applicability in preventive dentistry.
ii. Glycyrrhizinic
Ans.
iii. Fats and protein
Sucrose – ‘Arch-criminal of dental caries’: l Caries activity tests are used in dental research for many
l Sucrose is said to have a major role in the aetiology years and some of them are employed for routine use in
of dental caries. the dental office.
l There is a well-documented information on the rela- l These tests are used as a valuable adjunct for patient

tionship between dietary sugars and development of motivation in plaque control programmes.
dental caries. l The objective of this test is to identify some parameters

l The extracellular polysaccharide produced by the associated with the caries process, which indicates the
bacteria utilizing sucrose, functions in a dual role as extent and the adequacy of the defence or repair capa-
a structural matrix of dental plaque and a reservoir of bility of the tooth.
substrate for the plaque organisms between meals. Such information is utilized to estimate the probability
l Some bacteria synthesize glucans, the polymers of for caries and thereby helps in prevention of disease.
glucose while others form levans from fructose. Uses or applications of caries susceptibility assessment
l Biosynthesis of these polysaccharides occurs by the tests for the clinician:
agency of extracellular enzymes which show a spec- l To determine the need for caries control measures.

ificity for sucrose. l To serve an indicator of patient cooperation.

l Patients on a soft protein–fat diet developed a thin l To act as an aid in timing of recall appointments.

structure less plaque after few days. l As a guide to insertion of expensive restorations.

l When sucrose was included in diet a striking differ- l To aid in the determination of prognosis.

ence in the appearance of the plaque became notice- l As a precautionary signal to the orthodontist in plac-

able as it attained a considerable size and grew to ing bands.


form voluminous and turgid mass. Ideal requirements of caries activity tests:
Stephan’s curve: Suitable caries activity test should:
l Acidogenic bacteria in dental plaque can rapidly l Have a sound theoretical basis

metabolize certain carbohydrates to acids. In the l Show maximum correlation with clinical status

mouth, the resultant change in the plaque pH over l Be accurate with respect to duplication of

time is called Stephan’s curve. results


l Within 2–4 min of rinsing with a solution of glucose l Be simple and should take little time

or sucrose, plaque pH is reduced from about 6.5 to l Should be valid, reliable and feasible

5.0 and gradually returns to original value within Different types of caries activity tests:
40 min. This when graphically plotted is the curve i. Lactobacillus colony count
called Stephan’s curve. ii. Snyder’s test
Section | I  Topic-Wise Solved Questions of Previous Years 467

iii. Salivary reductase test d. Score 4 5 100.000 and above (marked car-
iv. Saliva tongue blade method ies activity)
v. Rapid caries activity test by resazurin l When the score increases for a group of indi-

Bacterial caries activity tests: viduals, so does the caries score. There is usu-
i. Lactobacilli count test: ally an excellent correlation between a zero
l This test was put forward by Hadley in 1933. count and caries resistance.
l This was the first microbiologic caries activity test l In spite of the well-established direct relation-

that was extensively used by practitioners. ship between lactobacilli counts and DMFS
Principle involved: scores, this method of caries activity evalua-
l This test estimates the number of acidogenic tion fell into disuse, partially because of its
and aciduric bacteria in the patient’s saliva. demerits and the introduction of the Snyder,
l A selective medium of pH 5.0, which favours Alban and dip slide tests that are easier to per-
the growth of Lactobacillus, is the basis of the form.
test. Demerits of this test:
l Improved selective medium (LBS agar) with l Lactobacilli may be responsible for the pro-

an acid pH, a high content of acetate and other gression of the lesion, as they are probably not
salts and low surface tension that is highly essential for the initiation of a lesion, these
selective for growth of lactobacilli are used. levels in saliva reflect the number of existing
l The number of colonies that grow on the me- lesions and acidic conditions in the mouth.
dium upon incubation is an index of the acidu- l Test results are not available for several days.

ric flora in saliva. l Counting of the colonies is a very tedious

Procedure: process.
l A stimulated, whole saliva specimen is col- l The test is not simple as it requires relatively

lected over a 5-min period preferably before complex equipment and personnel with bacte-
breakfast, by chewing a 1 g paraffin wafer or a riological training.
sterilized rubber band. l The cost is relatively high.

l The laboratory procedure begins with lining up ii. Snyder’s test:


of seven test tubes, each with 9 mL of saline. l Marshall L. Snyder in the early 1940s proposed a

One millilitre of the saliva is placed in the first lactobacilli test that was much easier to accom-
test tube and the contents shaken. One millili- plish than the lactobacilli counts.
tre from this tube is transferred to the second Principle involved:
tube, which is also shaken before another serial l This test was based on the fact that lactobacilli

transfer is made to the third tube. The serial are acidogenic and aciduric. Since the amount
transfers and shaking are repeated until all of acid produced is directly proportional to the
tubes have been inoculated. In this way, tube number of lactobacilli, both the counting
one has 10–1 bacteria, tube two 10–2, and so on. method and the Snyder’s method measure the
l A 1 mL aliquot is taken from each of these same cariogenic potential.
tubes and placed in a series of Petri dishes. l The selective medium used for the Snyder’s test

Then approximately 10 mL of Ragosa’s lacto- has a pH of approximately 5, which is optimum


bacilli selective medium which has been al- for lactobacilli growth but extremely restrictive
lowed to cool to 45°C is added. for other organisms that are not aciduric.
l After 4 days, the number of colonies on the l In order to evaluate visually the rapidity and

plates is counted. The most accurate counting extent of acid production, bromocresol green is
can be obtained on plates containing 35–100 incorporated into the medium to indicate pH
colonies. Thus, if the plate selected had the change. The medium is blue at pH 5 and green
10–3 dilution and 50 colonies were counted, the at pH 4.6, yellowish at pH 4.2 and yellow at
total count of bacteria from the saliva would be pH 3.8. A standardized colour chart is used as
50 3 103, or 50,000. Counts can be scored an aid in determining the colour changes.
from 1 to 4, depending on whether they fall l The medium is initially prepared by adding 1 L

within the range as: of boiling water to 61 g of the powdered


a. Score 1 5 0–1000 (light or no caries activity) Snyder’s medium and adjusting the pH with
b. Score 2 5 1000–10,000 (slight caries activity) glacial acetic acid. Approximately 5 mL of
c. Score 3 5 10,000–100,000 (moderate car- medium is placed in sterile test tubes that are
ies activity) stored in the refrigerator.
468 Quick Review Series for BDS 4th Year, Vol 1

Procedure: the swab inserted beneath the surface of the


l To perform the test, a specimen of stimulated agar. The tube is then incubated for 4 days
whole saliva is secured with paraffin stimulation. and the colour change is noted every day.
l One tube from the refrigerator is heated to l The colour changes are scored from 0 to 4,

100°C to liquefy the agar. It is then cooled to with the score being based on the amount of
45°C before 0.1 mL of the saliva specimen is colour changes occurring from top to bottom
added and the tube shaken before being placed in the tube.
in the incubator for 72 h at 37°C. a. A zero score indicates no colour change.
l At the end of 24 h, and again at 48 and 72 h, b. 11 score is a colour change to yellow in
the colour of the medium is recorded as 1–4 on the top one-fourth of the tube.
the basis of whether the colour remains the c. 21 to the halfway mark.
same or changes to a light green, a light yel- d. 31 to the three-fourths mark.
low, or a definite yellow. e. 41 when the entire length of the agar
l If the colour changes to a definite yellow in column has changed to yellow.
24 h, the individual is considered as caries sus- l Alban’s test is probably most predictive when

ceptible. If no changes occur in 72 h, the indi- the scores are at the 0 or 41 level at the end of
vidual is caries resistant. In between scores are 24 and 96 h, respectively.
less informative but can be used along with clini- l Like other lactobacilli tests, the Alban’s test is
cal judgment as an aid in evaluating caries status. outstanding for indicating caries inactivity.
The interpretation of the results is given in table Alban’s test is ideal for educating the patients
below: as they can understand the role of acids in car-
Snyder’s test: ies process.
l Tests can be repeated and favourable changes

Time in hours in diet and plaque control procedures are re-


24 48 72
flected within a few weeks by corresponding
changes in Alban’s test score.
Colour Yellow Yellow Yellow iv. The swab test:
Caries activity Marked Definite Limited l This test was developed by Grainger et al. in

Colour Green Green Green 1965.


Principle involved:
Caries activity Continue test Continue test Inactive
l It is based on the same principle as the Snyder’s

Advantages of Snyder’s test: test.


l It measures the aciduric-acidogenic compo-
l Simple

l Less armamentarium
nent of the oral flora after a suitable incuba-
l Cost is moderate
tion period by employing a colour indicator
iii. Alban’s test (modified Snyder’s test): in the test medium or by directly reading the
l Arthur L. Alban, a paedodontist from California,
pH on a pH meter.
in 1970 modified the Snyder’s test to make avail- Procedure:
l The buccal surfaces of the teeth are swabbed
able an easy to accomplish caries activity test for
routine dental office use. with a cotton applicator which is incubated in
l This method uses the same medium with the
the medium.
l The change in pH following a 48-h incuba-
exception that less agar is used which allows
easier permeation of bacteria and end products tion is read on a pH meter, or the colour
through the agar column. change is read by use of a colour comparator.
Procedure: Results of the swab test with pH meter are as
l A 5 mL tube of agar is removed from the
follows:
refrigerator, but not heated.
pH Caries activity
l The patient is asked to drool or spit unstimu-

lated saliva directly into the tube until there is . 4.6 Inactive
a thin layer of salvia covering the agar. 4.6–4.5 Mildly active
l A small funnel can be used in the sample col-
4.4–4.2 Active
lection. With small children, a cotton swab
4.1 and less Very active
can be rubbed across the tooth surface and
Section | I  Topic-Wise Solved Questions of Previous Years 469

l The swab test is advantageous over the then plated using conventional mitis salivarius
Snyder’s test, as no collection of saliva is nec- agar with the addition of sucrose and bacitracin.
essary. Therefore, it is particularly valuable l This is followed by an incubation period of

in evaluating caries activity in very young 4 days, at which time the CFUs are counted. A
children. threshold value of 2.5 3 106 CFU/mL of saliva
But, the swab test is not widely used now. has been suggested to select children considered
v. Dip slide methods: to be at a high caries risk.
l A specially designed dip slide of plastic is b. Dip slide method for S. mutans:
coated with LBS agar. l This method is very similar to that of dip slide

l Undiluted, paraffin stimulated saliva is flowed method for lactobacilli.


over the agar surface. The amount of saliva in- l Undiluted paraffin-stimulated saliva is poured on

oculated on the dip slide is relatively constant in a special plastic slide that is coated with mitis
spite of the method of inoculation. salivarius agar, containing 20% sucrose.
l The plastic slide holders are positioned verti- l The agar surface is thoroughly moistened and the

cally with a slight tilt to assure both agar sur- excess saliva is allowed to drain off two discs
faces are wetted with saliva. Excessive saliva is containing 5 mcg bacitracin are placed on the
allowed to drain on to a clean absorbent paper. agar, 20 mm apart.
l The slide is then placed into a sterile tube, l The slide is then tightly screwed into a cover tube

which is tightly closed and incubated at 35°C and incubated at 37°C for 48 h in a scaled jar.
for 4 days. The scoring is done as follows:
l It is then removed and the colony density is de- i. Low:
termined by comparing it with a model chart Colonies are discrete and could be readily
that is provided. counted at 153 magnification with the total
l The lactobacilli will form transparent or white counted of CFU inside the inhibition zone less
colonies. than 200.
l Readings of more than 10,000 colonies per mL ii. Medium:
of saliva are considered high, whereas the read- The colonies are discrete and the number in the
ings of less than 1000 colony counts are consid- zone of inhibition is more than 200 at 323
ered low. magnification.
l Any result between 1000 and 10,000 is consid- iii. High:
ered medium. The colonies are tiny and almost completely or
Tests based on Streptococcus mutans: totally cover the inhibition zone, with the num-
Principle: ber of colonies uncountable, when using a 323
l The tests measure the number of S. mutans magnification.
colony-forming units per unit volume of saliva. l S. mutans tests using mitis salivarius

l Culturing plaque samples from discrete sites such medium with bacitracin are very good in
as occlusal surfaces and proximal areas is an ideal identifying children with 1 mm caries
method for the purpose of quantifying the S. mu- increments.
tans that have colonized on teeth. l Positive scores do not correlate well with

l However, this is not practical and hence salivary high caries increments.
samples may be used as a workable alternative. l The tests are economical and suitable for

l Incubation on a selective Streptococcus medium mass screening to identify low-risk pop-


known as mitis-salivarius agar (MSA) with the ulation who do not require preventive
high concentration of sucrose (20%) and 0.2% treatment.
J–L bacitracin/millilitre (MSB) suppresses the Based on both S. mutans and lactobacilli :
growth of most non-S. mutans colonies. Advanced dip slide methods of S. mutans and lactoba-
a. Streptococcus mutans count tests: cilli (Dentocult and S trip mutans):
l The number of S. mutans in human’s saliva has S. mutans in saliva:
been proposed as a reliable indicator of caries l S. mutans level in saliva is done using Dento-

activity. cult SM (Orion Diagnostica, Finland), follow-


l In the quantitative evaluation of the number of ing the instructions of the manufacturer.
S. mutans colony-forming units, a serial dilution l A disk impregnated with bacitracin is dropped

is accomplished, using 1 mL of saliva specimen. inside the tube that contains selective culture
One millilitre aliquots of these serial dilutions are media for S. mutans to sample saliva, the strip
470 Quick Review Series for BDS 4th Year, Vol 1

is rotated 10 times on the surface of the tongue and Principle involved:


put it into the tube with media and incubated at 37°C l The reductase enzyme is involved in the forma-
for 48 h. tion of products dangerous to the tooth surface.
l The results of the strip were compared with the chart l The test measures the rate at which an indicator
of the manufacturer. molecule diazoresorcinol, changes from blue to
l The data were coded as follows: red to colourless or leukoform on reduction by the
l Code 0 and 1 – ,l05 mixed salivary flora.
l Code 2 – .105, ,106 Procedure:
l Code 3 – .106 l A kit is available under the trade name Treatex.
Lactobacillus levels in saliva – Dentocult LB: l 5 mL of stimulated saliva is collected in a plastic
l Dentocult method is a simple and highly practical container with paraffin wax stimulation. The sam-
method for estimating salivary levels of Lactobacillus ple is then mixed with the dye diazoresorcinol
and other aciduric microorganisms. which colours the saliva blue. As the dye is re-
l It was introduced by Larmas in 1975. This method duced, the colour changes and the caries condu-
makes use of a self-contained kit with a shelf life of civeness reading are taken after 15 min. No incu-
at least 1 year. bation is needed. The results are interpreted as
l This simplified, prepackaged selective culture sys- given in table below:
tem is easily adapted for office use and does not re- Interpretation of salivary reductase test: results
quire special equipment.
Method: Colour change Caries conductiveness
l Undiluted paraffin-stimulated saliva is poured Blue in 15 min Nonconducive
over a plastic slide that is coated with LBS agar Orchid in 15 min Slightly conducive
on both sides. Excess saliva is allowed to drain off
Red in 15 min Moderately conducive
and the slide is placed into a sterile tube. The tube
which is tightly closed is incubated at 35°C–37°C Red immediately on mixing Highly conducive
for 4 days. Colourless in 15 min Extremely conducive
l At the end of four days, the colony density on the

slide is not counted, but is compared with a model Buffer capacity test:
chart and classified as about 1000, 10,000, l Buffer capacity can be quantitated using either a pH

100,000 or 1,000,000 aciduric organisms/millili- meter or colour indicators.


tre of saliva. l The test measures the number of millilitres of acid
3
l Code 0: 10 mfc/mL required to lower the pH of saliva through an arbi-
4
l Code 1: 10 mfc/mL trary pH interval, such as from pH 7.0 to 6.0 or the
5
l Code 2: 10 mfc/mL amount of acid or base required to bring colour indi-
6
l Code 3: 10 mfc/mL cators to their end point.
Older methods: Procedure:
Enamel solubility test: l Ten millilitres of stimulated saliva is collected

l It is same as Fosdick dissolution test. This test under oil at least 1 hour after eating. Five millili-
is not generally suited for office procedures. tres of this is taken in a beaker. The pH of the sa-
l It is based on the fact that when glucose is added liva is adjusted to 7.0 by addition of lactic acid or
to saliva containing powdered enamel, organic base. The level of lactic acid in the graduated
acids are formed. These in turn decalcify the cylinder is re-recorded.
enamel resulting in an increase in the amount of l Lactic acid is then added to the sample until a pH

soluble calcium in the saliva-glucose enamel of 6.0 is reached. The number of millilitres of
mixture. lactic acid needed to reduce pH from 7.0 to 6.0 is
l The extent of increased calcium is supposedly a measure of buffer capacity.
a direct measure of the degree of caries suscep- l This number can be converted to milliequivalents

tibility. per litre.


Salivary reductase test: l There is an inverse relationship between buffering

Rapp, in 1962, claimed that this test measures the capacity of saliva and caries activity.
activity of the reductase enzyme present in sali- l The saliva of individuals whose mouth contains a

vary bacteria. considerable number of carious lesions frequently


Section | I  Topic-Wise Solved Questions of Previous Years 471

has a lower acid – buffering capacity. This test, iv. Frequency:


however, does not correlate adequately with car- l Frequency of consumption of sugar-containing

ies activity. foods


Fosdick calcium dissolution test: l Frequency of ingestion of sucrose, even little

Principle: concentration
l This test measures the milligrams of powdered l Frequency of in between meal snacks in-

enamel dissolved in 4 h by acid formed when the creases caries prevalence


patient’s saliva is mixed with glucose and pow- v. Cooking and processing:
dered enamel. l At temperature used for cooking carbohy-

Procedure: drates interact with proteins; this decreases


l A 25 mL of stimulated saliva is collected, part of the bioavailability of sugar and amino acids.
which is analysed for calcium content. The rest is vi. Other factors:
placed in an 8-inch sterile test tube with about 0.1 g They include detergent quality, texture and effect
of powdered human enamel. of mixing foods.
l The tube is sealed and shaken for 4 h at body tem-

perature with test tube agitation equipment. After Role of carbohydrates in dental caries:
l Among all the nutrients, carbohydrates are sugars.
agitation, it is again analysed for calcium content.
l Fermentable dietary carbohydrates (e.g. glucose,
l The amount of enamel dissolution increases as the

caries activity increases. fructose and sucrose) play an important role in the
causation of caries.
Q.8. Discuss the role of carbohydrates in dental caries. l The glucose, sucrose, fructose, etc., are rapidly diffused

into the plaque due to their low molecular weight.


Ans.
l These sugars are easily and rapidly fermented by
Diet and dental caries: cariogenic bacteria in the oral cavity to produce acid,
‘Diet refers to the customary allowance of food and at or near the tooth surface that causes dissolution of
drink taken by any person from day to day’. the hydroxyapatite crystals of the enamel followed
l The role of dietary factors in the occurrence of by the dentin.
dental caries is well established. l Risk of caries incidence increases greatly if the dietary
l Carbohydrates are one of the main dietary factors sugar is sticky in nature which remains adherent to the
that are thought to be associated with increased tooth surface for long time after taking the meal.
incidence of dental caries. l Following the ingestion of these sugars, the pH of the
Cariogenic potential of food: plaque falls to 4.5 to 5 within 1– 3 min and neutral-
The absolute cariogenic potential of food is influenced ization occurs after 10–30 min.
by the following factors: l The dental caries occurs due to the interplay of fac-
i. Fermentable carbohydrate content: tors such as oral microorganisms, local carbohydrates
l The mono- and disaccharides are more harm- and tooth surface.
ful as they can be easily fermentable than l The following equation describes the role of these
polysaccharides. factors in caries:
Cariogenicity of dietary carbohydrates varies de- Bacteria 1 Sugars 1 Teeth n Organic acids n Dental
pending on: caries.
l Frequency of ingestion Role of carbohydrates in dental caries is as follows:
l Physical form l The four carbohydrates – starch, sucrose, fructose
l Chemical composition and glucose comprise the great proportion of
l Route of administration foods consumed by man.
l Presence of other food constituents l The main polysaccharide (starch) is not highly
ii. Food retention: cariogenic in man at least in some circumstances.
Duration of presence of carbohydrates in oral l Controlled studies in experimental animals and in
cavity influences period of time acid remains in humans have confirmed that excessive and fre-
contact with tooth. Retentive, sticky and sweet quent use of highly fermentable mono- and disac-
foods are potentially highly cariogenic. charides is correlated with high caries rates.
iii. Eating pattern: l Sucrose is by far the commonest dietary sugar and
Sequence of food intake. most cariogenic.
472 Quick Review Series for BDS 4th Year, Vol 1

While glucose, fructose, lactose and mannose have


l l Because foods containing sugars in solution as
been shown to be cariogenic in animal experiments well as retentive sugars are included in the diet
they are usually minor constituents of human foods analysis, 20 min may be considered as the
as they are present only in dried fruits, honey and minimal time each exposure permits acid con-
milk. centrations to be available in the bacterial
Carbohydrates are three types as follows: plaque.
i. Monosaccharides l The following can be used in explaining the
ii. Disaccharides dental caries process to a parent or child:
iii. Polysaccharides i. Fermentable carbohydrate 1 Oral bacteria
i. Monosaccharides: within plaque n Acid within plaque.
l Simple sugars that cannot be decomposed ii. Acid 1 Susceptible tooth n Tooth decay.
by hydrolysis, e.g. glucose, galactose and fruc- Hence, diet can affect the structure of
tose. teeth and their future susceptibility to den-
l They can be fermented into cariogenic acids by tal caries.
bacteria. Q.9. Define rampant caries. Discuss its management.
ii. Disaccharides:
l In this, two monosaccharide molecules are linked Ans.
together, e.g. sucrose, lactose and maltose. [Same as LE Q.1]
l Bacteria can ferment them to produce cariogenic

acids. Q.10. Define rampart caries. Write in detail about it.


iii. Polysaccharides: Ans.
l In this more than two monosaccharide molecules

are linked together. [Same as LE Q.1]


l They are least cariogenic of all the three types of
Q.11. Define rampart caries. Discuss the aetiology, clini-
carbohydrates. cal features and step-by-step management of the same.
Sucrose:
l Sucrose is most cariogenic as it can penetrate dental Ans.
plaque and produce complex organic acids by fer- [Same as LE Q.1]
mentation. It has been labelled as ‘arch criminal of
dental caries’. Q.12. What is early childhood caries. Write in detail
l Sucrose also stimulates plaque formation and its at- about the management and prevention of early child-
tachment. hood caries.
l It also implants caries producing streptococci even
Ans.
on smooth surface of teeth.
l Sucrose function is a dual role: [Same as LE Q.5]
(a)   As structural matrix of dental plaque Q.13. Discuss about cariogenic potential of diet in dental
(b) As reservoir of substrate for plaque organisms caries.
test
l Cariogenic microorganisms like S. mutans produce Ans.
large quantities of water insoluble glucan from su- [Same as LE Q.8]
crose.
Glucans:
Serves as structural components of plaque matrix to act
SHORT ESSAYS:
as ‘glue’ for certain bacteria. Q.1. What is plaque, mention its role in dental caries?
Levans:
Ans.
Degraded by oral flora and serve as transient reserves of
fermentable carbohydrates prolonging the duration of l A gelatinous mass of bacteria, adhering to the tooth
acid formation. surface is known as plaque.
l Investigations by Schachtele and Jensen as well as Role of plaque in dental caries:
by Park and colleagues have indicated that the l Teeth provide ideal conditions for bacterial coloniza-

acidity of plaque located in interproximal areas, tion and growth.


which generally have less exposure to saliva, may In contrast to mucosal surfaces, shedding of colo-
remain below the critical pH for periods in excess nized epithelial cells does not constantly renew the
of 2 h after carbohydrate ingestion. surfaces of teeth.
Section | I  Topic-Wise Solved Questions of Previous Years 473

l Microbial colonization of teeth, however, is not l Mature plaque communities rapidly metabolize
merely a question of passive retention, but requires sucrose through glycolytic pathways to organic
bacteria to adhere to the surface. acids, primarily lactic acid. Thus acidic environ-
l A firm attachment may subsequently be achieved by ment favours the tooth decay.
specific mechanisms.
Q.2. Zones of dental caries in enamel and dentin.
Pellicle formation
l Microorganisms do not attach themselves directly Ans.
to the mineralized tooth surface as the teeth are
Zones of dental caries in enamel:
always covered by an acellular proteinaceous
a. Zone 1: Translucent zone:
film, the pellicle.
l It lies at the advancing front of the enamel lesion.
l The pellicle forms on the naked tooth surface
It is not always present.
within minutes to hours.
l It has been shown that this zone is slightly more
The major constituents of the pellicle are salivary
porous than sound enamel. Pore volume is 1% as
glycoproteins, phosphoproteins, lipids and com-
compared to sound enamel, i.e. 0.1%.
ponents from gingival crevicular fluid.
b. Zone 2: Dark zone:
l Because of its selective permeability, it restricts
l It lies adjacent and superficial to zone 1. It is re-
the transport of ions in and out of the dental hard
ferred to as the positive zone, because it is usually
tissues. Thus, it plays an important modifying role
present.
in caries.
l This zone is formed as a result of demineraliza-
Microbial colonization:
tion.
l The early stages of recolonization of the cleaned
c. Zone 3: Body of the lesion:
tooth surface involve adhesion between the pellicle
l It lies between the relatively unaffected surface
and the pioneering organisms.
layer and dark zone. It is the area of greatest
l The adhesion process is very selective and requires
demineralization.
specific organism receptors capable of binding to
l Pore volume being 5% at the periphery and 25%
certain areas on the precipitated salivary proteins of
at the centre.
the pellicle.
d. Zone 4: Surface zone:
Initial microbial colonization
l Appears relatively unaffected due to greater de-
l Irrespective of the type of tooth surface the initial
gree of mineralization and greater concentration
colonizers constitute a highly selected part of the
of fluoride in the surface enamel.
oral microflora mainly Streptococcus sanguis,
l It has lower pore volume than the body of the le-
Streptococcus oralis and Streptococcus mitis.
sion. Intact surface serves as a barrier to bacterial
In addition, the initial microflora comprises minor
invasion.
proportions of Actinomyces species and Gram-
Zones of dental caries in dentin:
negative bacteria, e.g. Haemophilus species.
a. Zone – 1: Normal dentin:
l Within 1 day, the tooth surface is almost com-
l The deepest area which has tubules with odonto-
pletely covered by a blanket of microorganisms.
blastic processes that are smooth, and no crystals
However, the microbial deposits are not uniform
are in lumens.
in thickness. At this early stage of colonization,
l Zones of fatty degeneration of Tome’s process are
Gram positive and Gram-negative bacteria within
seen.
a single microcolony are not recognized accord-
b. Zone – 2: Subtransparent dentin:
ing to any particular pattern
l Zone of dentinal sclerosis characterized by depo-
l After 1 day the surface of the microbiota is mainly
sition of calcium salts in dentinal tubules. Dentin
made up of coccoid bacteria, with a few filaments.
is capable of remineralization.
Mature dental plaque:
c. Zone – 3: Transparent dentin:
l As the plaque matures, the production of cells and
l Zone softer than carious dentin and shows further
matrix slows and utilization of energy for the total
loss of mineral from the intertubular dentin.
community metabolism results in acid production.
l Also called zone of decalcification of dentin, a
l Since the mature plaque is primarily anaerobic, it
narrow zone, preceding bacterial invasion.
reduces the available nutrients to anaerobic metabo-
e. Zone – 4: Turbid dentin:
lites, that is, fermentation products including weak
l Zone of bacterial invasion of decalcified but intact
organic acids, amines and alcohol.
dentin.
474 Quick Review Series for BDS 4th Year, Vol 1

f. Zone – 5: Infected dentin: Dietary constituents and dental caries:


l Zone of decomposed dentin. i. Polysaccharides and sugars:
l The four carbohydrates – starch, sucrose, fruc-
Q.3. Hopewood House study.
tose and glucose – comprise the great propor-
Ans. tion of foods consumed by humans.
l The main polysaccharide (starch) is not highly

cariogenic in man at least in some circum-


{SN Q.21} stances.
l The various experimental studies have con-
l In 1942, an eccentric, wealthy Australian busi- firmed that excessive and frequent use of highly
nessman transformed what was formerly a spa- fermentable mono- and disaccharides are cor-
cious country mansion, Hopewood House, into a related with high caries rates.
‘motherhouse’ for young orphan children at NSW, l While glucose, fructose, lactose and mannose
Australia. have been shown to be cariogenic, sucrose is by
l His own experience of dramatic recovery in health far the commonest dietary sugar and most cario-
due to a drastic change in dietary habits has stipu- genic.
lated that the children of Hopewood House should be ii. Lipids:
raised on a natural diet that excluded refined carbo- l Fat consumed has been somewhat responsible
hydrates. for anticariogenic effect.
l The basically vegetarian diet of these children was l This mechanism can be due to protection from
adequate but spartan porridge, biscuits, wheat gram, demineralization by formation of fatty film in
fresh and dried fruit, vegetables (cooked and raw), proximal areas.
along with butter cheese, eggs, milk and fruit juices. iii. Vitamins:
Vitamin concentrates and an occasional serving of l Vitamin D and vitamin A are most important
nuts and a sweetening agent such as honey supple- with respect to development of teeth.
mented the meals. l Decrease of vitamin D will lead to calcium and
l In order to retain its natural state, the food was un- phosphate derangement and, in turn, cause
cooked as far as possible. The most striking feature hypoplasia of teeth.
of this diet was the notable absence of sugar. l Deficiency of vitamin A can lead to changes in
l The fluoride content of the water and food was insig- ameloblasts thereby causing alteration in tooth
nificant and no tea was consumed. morphology and can also have deleterious effects
All meals and between-meal eating were controlled with on salivary glands.
great regularity. Physical properties of foods and cariogenicity:
l From a dental standpoint the physical properties of
l At the end of 1-year period, the 13-year-old chil-

dren of Hopewood House had a mean DMF per food may have significance by affecting: food reten-
child of 1.6, the corresponding figure for the gen- tion, food clearance, solubility and oral hygiene.
l Obviously if a type of food is more sticky then there
eral child population of the State of NSW was
10.7. Only 0.4% of the l3-year-old state school are more chances of getting caries as compared to a
children were free from dental caries, whereas food that readily cleared from oral cavity.
l Those foods that improve the cleansing action and
53% of the Hopewood House children experi-
enced no caries. reduce the retention of food within the oral cavity
l The children’s oral hygiene was poor, dental cal-
and increase saliva flow are to be encouraged in ev-
culus was uncommon, but gingivitis was preva- eryday diets. Clinical evidence shows that consump-
lent in about 75% of the children. tion of these food items will significantly reduce
l This work shows that in institutionalized children,
caries incidence.
at least, dental caries can be reduced to insignifi- The physical texture and chemical composition of food
cant levels by a spartan diet, and without the Some important physical properties that determine food
beneficial influence of fluoride and in the pres- texture are
l Mechanical properties: hardness, cohesiveness
ence of unfavourable oral hygiene.
and viscosity.
l Geometric properties: particle size and shape.

l Others: moisture and fat content.


Q.4. Diet and dental caries.
l Physical texture is known to effect salivary flow
Ans. rates. Saliva that is rapidly flowing is more alkaline
Section | I  Topic-Wise Solved Questions of Previous Years 475

than resting saliva and more supersaturated with Commonly used substitutes are
calcium and phosphate and thus may be more car- l Xylitol
ies inhibitory. l Sorbitol
l Some dietary items are highly acidic and there- l Saccharine
fore affect, usually in a transient manner, the pH l Cyclamates
in plaque and saliva. l Aspartame
l Natural foods, such as lemons, apples, fruit juices
Q.6. Preventive protocol for nursing caries.
and carbonated beverages, are sufficiently acidic
so as to cause demineralization of enamel that is Ans.
in prolonged contact with them.
Preventive protocol for nursing caries is as follows:
l These items, under normal dietary use, are of no
i. Professional care:
consequence in the dental caries process. How-
l Educating parents regarding importance of de-
ever, excessive (habitual) use of these foods and
ciduous teeth
beverages may cause etching of enamel with
l Diet counselling
cavitations.
l Dental health education to parents regarding gum

Q.5. Cariogenic diet substitutes. pads cleaning, toothbrushing, frequent mouth rinsing
l Advocating fluoride supplementation if needed
Ans. l Advocating fluoride varnish topically

l Application of fissure sealants in first and second


Sugar substitutes:
primary molars
l Artificial sweeteners are called sugar substitutes.
l Regular recalls for routine monitoring for dental
l The ideal sweetener should provide sweetness, with
health
no unpleasant after-taste, have little or no calories,
l Reinforcing and motivating parents to continue
not be carcinogenic or mutagenic, be economical to
supervised home care
produce and should not be degraded by heat when
ii. Home care:
cooked.
l Elimination of cariogenic food items from the diet
l There are two kinds of sweeteners/sugar substitutes:
l Substitution with tooth friendly food.
a. Non-nutritive sweeteners/noncaloric sweeteners/
l Discouraging bottle feeding at night.
intense sweeteners
l Falling asleep with pacifiers should be stopped.
b. Nutritive sugars/caloric sweeteners/sugar substitutes
l Cleaning of gum pads during infancy period is
l Nutritive sweeteners provide some calories, while non-
encouraged.
nutritive sweeteners typically provide zero calories.
l Digital or baby toothbrushing should be started as
a. Non-nutritive/noncaloric sweeteners:
the teeth erupts.
l They are the substances of synthetic or natural
l Initializing mouth rinsing habit after consuming
origin that tastes much sweeter than sugar.
any solid or drinks.
l They yield little or no energy, provide no bulk
l Regular visit to dental clinic once in 6 months.
and are to be used in very small quantities in
drinks or blended with sugar substitutes in Q.7. Snyder’s test.
foods and snacks.
Ans.
l Their main commercial success is based on

weight control and diabetic products, e.g. sac- l Snyder’s test measures the ability of salivary microorgan-
charin, cyclamates and aspartame. isms to form organic acids from carbohydrate medium.
b. Sugar substitutes/caloric sweeteners: Action:
l They are usually carbohydrates or carbohy- l The Snyder’s test measures the rapidity of acid for-
drates substitutes. mation when a sample of stimulated saliva is inocu-
l They can be metabolized to yield energy and lated into glucose agar adjusted to pH 4.7 to 5 and
they add bulk to the food products. with bromcresol green as colour indicator.
l They are equally or less sweet tasting than su- Equipment:
crose and require blending with intense sweet- l The equipment includes saliva-collecting bottles,
ness, e.g. sorbitol, xylitol, fructose and glucose. paraffin, a tube of Snyder’s glucose agar containing
Ideal sweetener selection is based on: bromcresol green, pipettes and incubating facilities.
l Consumer acceptance. Procedure:
l Consumer tolerance based on metabolic capacity. l Saliva is collected before breakfast by chewing
l Product should be noncariogenic. paraffin.
476 Quick Review Series for BDS 4th Year, Vol 1

A tube of Snyder’s glucose agar is melted and then


l l The Snyder’s test measures the rapidity of acid
cooled to 50°C. formation when a sample of stimulated saliva is
l Saliva specimen is shaken vigorously for 3 min, then inoculated into glucose agar adjusted to pH 4.7 to
0.2 mL of saliva is pipetted into the tube of agar and 5 and with bromcresol green as colour indicator.
immediately mixed by rotating the tube. l Colour change of the indicator is observed after

l Agar is allowed to solidify in the tube and is incu- 24, 48 and 72 h of incubation by comparison
bated at 37°C with an inoculated tube against a white back-
l Colour change of the indicator is observed after 24, ground.
48 and 72 h of incubation by comparison with an iii. Salivary reductase test:
inoculated tube against a white background. l It measures the enzyme reductase in the saliva

l If the colour changes to a definite yellow in 24 h, the and the rate at which an indicator dye, diazores-
individual is considered as caries susceptible. If no orcinol changes its colour from blue to red to
changes occur in 72 h, the individual is caries resis- colourless on reduction by the mixed salivary
tant. In between scores are less informative but can flora.
be used along with clinical judgment as an aid in iv. Saliva tongue blade method:
evaluating caries status. l The test estimates the number of S. mutans in

Advantages: mixed paraffin stimulated saliva, when cultured


l Simple on mitis salivarius bacitracin (MSB) agar me-
l Requires only simple equipment dium.
l Only some training is needed l After patient chews paraffin wax for 1 min, they

l Cost-effective are given a sterile tongue blade which they ro-


tate in their mouth ten times, so that both the
Q.8. Caries activity tests. sides of the tongue blades are thoroughly coated.
Ans. Excess saliva is removed by withdrawing the
tongue blade through closed lips.
l Caries activity is defined as the speed with which teeth l Both sides of the tongue blade are then pressed
are destroyed by caries, which includes new carious le- on to an MSB agar medium in Petri dish. It is
sions and enlargement of existing cavities during a cer- then incubated at 37° for 48 h.
tain period. l Counts of .100 colony-forming units (CFU) by
Ideal requisites of a caries activity test are this method is proportional to greater than 10
l Should have a sound theoretical basis CFU of S. mutans per millilitre of saliva by con-
l Show maximal correlation with clinical status ventional methods.
l Be accurate with respect to duplication of results v. Rapid caries activity test by resazurin disc:
l Be simple, inexpensive and take less time to perform l This method of caries activity test has a charac-
l Should possess validity, reliability and feasibility teristic colour reaction developing within 15 min
Some of the caries activity tests are: at 32–37°C.
i. Lactobacillus colony count test l The colour of the disc changes from blue to blu-
ii. Snyder’s test ish violet, reddish violet and then to red or co-
iii. Salivary reductase test lourless with saliva of the individuals.
iv. Saliva tongue blade method l The resazurin disc is highly sensitive to Gram-
v. Rapid caries activity by resazurin disc positive microorganisms such as S. mutans,
i. Lactobacillus colony count: S. mitis, Lactobacilli and actinomyces series.
l Introduced by Hadley in 1933. It estimates the l The colour change was due to a chemical reac-
number of acidogenic bacteria and aciduric bac- tion and not a pH effect.
teria in patient’s saliva.
l Saliva is collected after having the patient chew
Q.9. Milk bottle syndrome.
paraffin before breakfast. Ans.
l Saliva is diluted to 1:100 dilution and 0.4 mL of

this is spread on the surface of an agar plate and l Milk bottle syndrome is the cause of early childhood
are incubated at 37o for 3–4 days. tooth decay.
l It is the rapid decay of baby teeth in infants and chil-
l A count of the number of colonies is the made

by using Quebec counter. dren from frequent, long exposure to liquids containing
ii. Snyder’s test: sugars.
l Most often the upper four front teeth are affected,
l It measures the ability of salivary microorganisms

to form organic acids from carbohydrate medium. usually this problem is caused by the baby or child
Section | I  Topic-Wise Solved Questions of Previous Years 477

falling asleep while drinking a bottle or while breast varnish application, fluoridated dentifrices, fluoride
feeding. supplements, sealants, diet counselling and chlorhex-
l The sugary liquid from milk to juice pools around the idine.
front teeth and reacts with the bacteria in the child’s l Prevention of ECC also requires addressing the so-

mouth causing tooth decay. cial and economic factors that face many families
l A tooth can begin the process of decaying as soon as it where ECC is endemic.
has erupted. l The education of mothers or caregivers to promote

This means that a child as young as 1 year old can start healthy dietary habits in infants has been the main
having cavities. strategy used for the prevention of ECC.
l This is a severe problem and causes debilitating tooth l There are three general approaches that have been

destruction and even pain. used to prevent ECC:


l It is the only severe dental diseases common to children i. First is the community-based strategy.
under the age of three. ii. Second approach is based on the provision of
The following guidelines from the American academy of examination and preventive care in dental clinics.
paediatric dentistry should be followed to prevent baby- iii. The third involves the development of appropri-
bottle tooth decay: ate dietary and self-care habits at home.
l Clean your child’s teeth daily.
Q.12. Acidogenic theory.
l Do not allow your child to sip on a bottle filled

with juice, milk or formula for long periods of Ans.


time.
l Acidogenic theory is also known as Miller’s chemico-
l Make sure the child gets the fluoride needed to
parasitic theory.
prevent the decay.
l W.D. Miller, an American dental scientist, made the
l Introduce children to a cup as they approach
significant observation that many organisms could pro-
1 year of age.
duce acid from the fermentation of sugar.
l Children should stop drinking from bottles soon
l This theory states that the process of dental caries is a
after their first birthdays.
chemico-parasitic process consisting of two stages:
Q.10. Difference between ‘nursing caries’ and ‘rampant a. Preliminary stage:
caries’. Production of organic acid occurs as a result of fer-
mentation of the carbohydrates by the plaque bacte-
Ans.
ria. The decalcification of enamel which results in its
total destruction and decalcification of dentin.
Rampant caries Nursing bottle caries
b. Later stage:
Cause: inadequate maintenance Cause: prolonged bottle
l Preliminary stage followed by dissolution of the
of oral hygiene. feeding.
softened residue.
Involvement of proximal surface Involvement of maxillary l Acids resulting in primary decalcification is pro-
of lower anterior teeth and de- anterior teeth, the maxillary
duced by the fermentation of dietary carbohy-
velopment of cervical type of and mandibular posterior
caries is the main characteristic. teeth and mandibular canine drates, i.e. starches and sugar from the retaining
is seen. centres of teeth.
l Essentially three factors are recognized in Miller’s
Mandibular incisors are Mandibular incisors are not
affected. affected due to protection observations.
from the tongue. i. Carbohydrate substrate over the tooth surface.
ii. Variety of microorganisms responsible for the
Q.11. Prevention of early childhood caries. caries process.
iii. Production of acids initially and protein degra-
Ans. dation subsequently.
l This theory has been accepted by the majority of
Prevention of early childhood caries includes:
l Early screening for signs of caries development,
investigators since scientific evidence does impli-
starting from the first year of life, could identify in- cate carbohydrates, oral microorganisms and acids.
fants and toddlers showing the risk of developing i. Role of carbohydrates:
l Fermentable dietary carbohydrates play an im-
ECC and could also assist in providing information
of parents about how to promote oral health and pre- portant role in the causation of caries, e.g.
vent the development of tooth decay. glucose, fructose and sucrose.
l These sugars are easily and rapidly fermented
l High-risk children should be targeted with a profes-

sional preventive programme that includes fluoride by cariogenic bacteria in the oral cavity to
478 Quick Review Series for BDS 4th Year, Vol 1

produce acid at or near the tooth surface and Q.18. Nursing bottle caries.
cause dissolution of the hydroxyapatite crys-
Ans.
tals of the enamel followed by the dentin.
l Following the ingestion of these sugars the pH
[Same as SE Q.6]
of the plaque falls to 4.5 to 5 within 1–3 min
and neutralization occurs after 10 to 30 min. Q.19. Caries activity tests/Snyder’s test.
ii. Role of microorganisms:
Ans.
l A large number of microorganisms play indi-

vidual role in dental caries production and


[Same as SE Q.7]
among them the most important one is Strep-
tococcus mutans. Q.20. Nursing versus rampant caries.
l It synthesizes dextran from sucrose, which
Ans.
helps in adhering the plaque bacteria as well
as the acid on to the tooth surface.
[Same as SE Q.10]
l The actinomycosis group, e.g. Actinomycosis

israelii, Actinomycosis viscosus, etc. are the


important organisms to cause root caries. SHORT NOTES:
l The Lactobacillus acidophilus is an important

organism for the progress of dental caries. Q.1. Caries vaccine.


iii. Role of acids:
Ans.
l During the process of caries formation, a

large variety of acids are produced in the oral l Caries vaccine is a vaccine to prevent and protect
cavity due to the bacterial fermentation of against the tooth decay
dietary carbohydrate. For example: lactic l Routes of administration:
acid, aspartic acid, acetic acid, butyric acid i. Oral
and glutamic acid. ii. Systemic
They can cause demineralization of enamel iii. Active gingivo-salivary
and dentin and causes the tooth decay. iv. Passive dental immunization
l This theory summarizes that the interplay
Q.2. Salivary defence factors.
between Bacteria 1 Sugars 1 Teeth n
Organic acids n Caries. Ans.
Q.13. Diet factors in dental caries. l The antibacterial property of the saliva is associated
with the presence of the below-mentioned defence com-
Ans.
ponents.
[Same as SE Q.4] Lysozyme:
l More in sublingual and submandibular saliva.
Q.14. Role of diet in the aetiology of dental caries.
l Increased lysozyme activity was seen in caries-
Ans. free children.
Salivary lactoperoxidase:
[Same as SE Q.4]
l Peroxidase and thiocyanate act on hydrogen per-
Q.15. Role of sugar substitutes for the prevention of oxidase to produce hypothiocyanate; this in turn
dental caries. inactivates various bacterial enzymes and tempo-
rarily inhibits growth.
Ans.
Immunoglobulins:
[Same as SE Q.5] l Secretory IgA is secreted in the saliva and has

Q.16. Anticariogenic diet. antibacterial property.


Ans. Q.3. Give in short the importance of dietary record for
paediatric patient.
[Same as SE Q.5]
Ans.
Q.17. Nursing caries.
l Dietary record is the accounts of food consumed re-
Ans.
corded over a certain period of time for 3–7 days includ-
[Same as SE Q.6] ing weekends.
Section | I  Topic-Wise Solved Questions of Previous Years 479

Importance of dietary record for paediatric patient is as l Salivary analysis


follows: l Tooth separation
l The collection of information about food consump- l Radiographic assessment
tion and dietary habits is essential when relating diet l Digital radiographic methods
to caries prevalence or incidence. l Xeroradiography
l It can be used by a group of people living in institu- l Fibre-optic transillumination
tion, families or on individual basis. l Dyes (caries detector dyes)
l The amount of sugar-containing foods can also be l Electrical resistance measurements
written specifically. l Ultra sonography
l Large sample can be obtained by it. l Light-induced fluorescence
l It is economical as there is no need for trained l Endoscopic-filtered fluorescence method
persons. l Magnetic resonance microimaging.

Q.4. Caries severity index for primary teeth. Q.6. Pre-eruptive caries.

Ans. Ans.

Dental caries severity index for primary teeth: l Occasionally, defects on the crowns of developing perma-
l This was designed by Aubrey Chosack in 1985. nent teeth are evident radiographically, even though no in-
l It comprised of clinical examination of all individual fection of the primary tooth or surrounding area is apparent.
surfaces and scoring them individually as follows: Muhler referred to this condition as pre-eruptive ‘caries’.
I. Occlusal surfaces l Such a lesion often does resemble caries when it is ob-

Score criteria served clinically, and the destructive lesion progresses if


1. Early pit and fissure caries it is not restored.
2. Cavitation of 1 mm l As soon as the lesion is reasonably accessible, the tooth

3. Cavitation with breakdown of half of tooth should be uncovered by removal of the overlying pri-
or any cusp mary tooth or by surgical exposure.
II. Buccal-lingual and palatal smooth surface: l The caries-like dentin is then excavated, and the tooth is

Score criteria restored with a durable temporary or permanent restor-


1. White lesion not extending to embrasure ative material.
2. Cavitation of 1–2 mm extending to one em- l In some cases, the lesion may be so extensive that indi-

brasure rect pulp therapy is justified.


3. Cavitation of 2 mm extending to both em- Q.7. Rampant caries.
brasures
III. Proximal surfaces of molars Ans.
Score criteria l Sudden, rapid and almost uncontrollable destruction of
1. Discontinuity of enamel teeth is known as rampant caries. It also involves sur-
2. Cavitation with breakdown of marginal faces of teeth that are ordinarily relatively caries-free.
ridge For example: Proximal and cervical surfaces of anterior
3. Breakdown of marginal ridge with cavita- teeth, including the mandibular incisors which are rela-
tion extending to proximal extensions of tively caries-free, may be affected.
occlusal surfaces l Most often observed in the primary dentition of young
IV. Proximal surfaces of incisors: children and the permanent dentition of teenagers.
Score criteria l Dietary factors affecting oral substrate and oral flora
1. Discontinuity of enamel and physiological factors affecting saliva are often sig-
2. Cavitation with breakdown undermining nificant in the development of rampant caries.
the buccal or lingual surface
3. Cavitation with breakdown of incisal edge Q.8. Nursing bottle caries.
Ans.
Q.5. Caries detection.
l Nursing bottle caries is the rapid decay of baby teeth in
Ans.
infants and children from frequent, long exposure to
Various methods of caries detection are as follows: liquids containing sugars.
l Patient history l Cause: prolonged bottle feeding.

l Clinical examination – visual and tactile l Involvement of maxillary anterior teeth, the maxillary and

l Nutritional analysis mandibular posterior teeth and mandibular canine is seen.


480 Quick Review Series for BDS 4th Year, Vol 1

l Mandibular incisors are not affected due to protection and this precipitation is hastened by the presence of
from the tongue. fluoride.
l It is the only severe dental diseases common to children
Q.13. Critical pH.
under the age of three.
Ans.
Q.9. Guidelines for prevention of baby bottle syndrome.
l Critical pH is that pH at which any particular saliva
Ans. ceases to be saturated with calcium and phosphate.
l The following guidelines from the American academy l Below this value, the organic material of the tooth may

of paediatric dentistry should be followed to prevent dissolve.


baby bottle tooth decay: Q.14. Diet analysis.
l Clean your child’s teeth daily.
Ans.
l Do not allow your child to sip on a bottle filled with

juice, milk, or formula for long period of time as a l The collection of information about food consumption
pacifier. and dietary habits is essential when relating diet to
l Make sure the child gets the fluoride needed to prevent caries prevalence or incidence.
the decay. l Various methods employed for collection are
l Introduce children to a cup as they approach 1 year of age. a. Food balance sheets
l Children should stop drinking from bottles soon after b. Food accounts and estimated food records
their first birthdays. c. Weighing methods and duplicate portion technique
d. Interview methods
Q.10. Trace elements.
l Diet recall
Ans. l Diet history

e. Questionnaires
l Trace elements are variously defined depending upon
l The choice of method is greatly affected by the size of
the field of chemical, physical or biologic sciences be-
the sample and compromise may have to be made re-
ing discussed.
garding the size sample and accuracy of the method.
l In the field of biology, elements that are present in only

minute quantities in animal tissues are called trace elements. Q.15. Window of infectivity.
l Trace elements can be grouped into
Ans.
Cariosatic – F, P
Mildly cariostatic – Mo, V, Cu, Sr l Caufield (1996) stated that there is a window of infec-
Doubtfully cariostatic – Be, Co, Mn, Sn, Zn tivity between 19 and 33 months during which teeth get
Caries inert – Ba, Al, Ni, Pd infected with S. mutans.
Caries promoting – Se, Mg, Cd, Pt l The most beneficial time for vaccination against dental

caries would be in infancy prior to eruption of teeth.


Q.11. Sugar substitutes. l This would promote the induction of adherence inhib-

Ans. iting salivary IgA, thus delaying colonization of


S. mutans.
l Various sugar substitutes are of two types: l With the establishment of early colonizers there
i. Noncaloric sweeteners: would be a synergistic effect of suppressing the colo-
Examples: saccharin, cyclamates and aspartame nization of S. mutans during the time span of window
ii. Caloric sweeteners: of infectivity.
Examples: sorbitol, xylitol, fructose and glucose l A booster dose of vaccination may be required at time

Q.12. Caries is combined process of demineralization/ of eruption of first permanent molars.


remineralization – justify. Q.16. Hidden caries.
Ans. Ans.
l Caries is actually the combined process of demineraliza- l Hidden caries is the term used to describe dentine caries
tion and remineralization. If the process of demineraliza- that is missed on a visual examination, but is large
tion exceeds at a faster rate than that of remineralization, enough and demineralized enough to be detected radio-
it results in the formation of caries. graphically.
l Demineralization is due to the acids especially the lactic l It is possible that an improved visual examination with
acid produced by the bacteria as the end product of car- cleaning and drying of the teeth, may improve occlusal
bohydrate metabolism. caries detection to a point where hidden caries no longer
l Remineralization is due to the precipitation of minerals exists.
especially calcium and phosphates present in the saliva l It can be detected using bitewing radiograph.
Section | I  Topic-Wise Solved Questions of Previous Years 481

Q.17. Stephen’s curve. l Fluoride varnishes were first developed in Europe


(1964). They increase the time the fluoride in contact
Ans.
with the tooth.
Within 2–4 min of rinsing with a solution of glucose or Indications
sucrose, plaque pH is reduced from about 6.5 to 5.0 and l Handicapped children

gradually returns to original value within 40 min. This l Incipient caries lesion

when graphically plotted is the curve called Stephen’s l After restorative treatment is completed under gen-

curve. eral anaesthesia


Commonly used fluoride varnishes are
Q.18. Sucrose an arch-criminal – Comment.
l Duraphat

Ans. l Fluor protector

l Carex
Sucrose – ‘arch-criminal of dental caries’:
l Sucrose is said to have a major role in the aetiology Q.21. Hopewood study.
of dental caries.
Ans:
l The extracellular polysaccharide produced by the

bacteria utilizing sucrose, functions in a dual role as [Same as SE Q.3]


a structural matrix of dental plaque and a reservoir of
Q.22. Diet counselling.
substrate for the plaque organisms between meals.
l Some bacteria synthesize glucans, the polymers of Ans.
glucose while others form levans from fructose.
Areas of influence for promoting dietary behaviour modifi-
l When sucrose was included in diet a striking differ-
cations are as follows:
ence in the appearance of the plaque was noticeable
l Laws and regulation
as it attained a considerable size and grew to form
l Cultural norms and values
voluminous and turgid mass.
l Education
l Mature plaque communities rapidly metabolize su-
l Food production
crose through glycolytic pathways to organic acids,
l Availability of food
primarily lactic acid. Thus, acidic environment fa-
l Improvement of socioeconomic status
vours the tooth decay.
l Media

Q.19. Arrested caries. l Dietary modifications are synonymous with re-

stricted intake of sugars. Xylitol is one of the promis-


Ans.
ing dietary approaches on current scene.
l There is clinical evidence that incipient and even more l Xylitol chewing gums have shown reduced levels of

advanced carious lesions may become arrested if there S. mutans.


is a significant shift in oral environmental conditions l A number of potentially effective strategies have

from those that predispose to those that tend to slow the been implemented such as use of natural inhibitors of
caries process. demineralization such as phosphates, components
l This kind of caries becomes stationary and does not like polyphenols, oat and pecan hulls and cheese and
progress. Superficial dentin is soft and decalcified but other bovine milk products.
gradually gets burnished and has a brown polished
Q.23. Define early childhood caries and severe early
appearance and is hard. This is called ‘eburnation’ of
childhood caries.
dentin.
l Sclerosis of dentinal tubules and secondary dentin for- Ans.
mation are seen.
Early childhood caries:
l Another type of arrested caries is seen on proximal sur-
American Academy of Pediatric Dentistry (AAPD) de-
face of teeth when adjacent tooth is extracted. It shows
fines early childhood caries as ‘The presence of one or
brown area at or below the contact point of tooth. This
more decayed (noncavitated or cavitated lesions), miss-
type of caries is early caries which gets arrested after
ing (due to caries) or filled tooth surfaces in any primary
extraction due to the formation of self-cleansing area.
tooth in a child 71 months of age or younger’.
Q.20. Fluoride varnishes. Severe early childhood caries:
Is defined as a ‘ suddenly appearing, widespread, rapidly
Ans.
burrowing type of caries, resulting in early involvement
482 Quick Review Series for BDS 4th Year, Vol 1

of the pulp and affecting those teeth usually regarded as l Propylene glycol based Caries Finder is available
immune to ordinary decay’. in red and easy to see green, 10 mL bottle or 1 g
syringe.
Q.24. Snyder’s test.
l Some caries detection products contain a red and

Ans. blue disodium disclosing solution, e.g. Cari-D-Tect,


Gresco products, Stafford, Texas. These products
l Snyder’s test measures the ability of salivary microor-
stain infected caries dark blue to bluish green.
ganisms to form organic acids from carbohydrate me-
dium. Q.28. Low calorie sweeteners.
l Saliva is collected before breakfast by chewing paraffin
Ans.
and inoculated on agar medium and immediately mixed
by rotating the tube. Agar is allowed to solidify in the l Low or noncaloric sweeteners are the substances of
tube and is incubated at 37°C. synthetic or natural origin that tastes much sweeter than
l Colour change of the indicator is observed after 24, 48 sugar.
and 72 h of incubation by comparison with an inocu- l They yield little or no energy, provide no bulk and are

lated tube against a white background. to be used in very small quantities in drinks or blended
l If the colour changes to a definite yellow in 24 h, the with sugar substitutes in food and snacks, e.g. saccha-
individual is considered as caries susceptible. If no rin, cyclamates and aspartame.
changes occur in 72 h, the individual is caries resistant.
Q.29. White spot lesion.
In-between scores are less informative but can be used
along with clinical judgment as an aid in evaluating car- Ans.
ies status.
l Small opaque white region, called white spot lesion,
Q.25. Define balanced diet. forms the initial feature of dental caries.
l This is the zone of demineralization that can be remin-
Ans.
eralized.
A balanced diet is a diet which contains the quantity and l Enamel overlying this white spot is hard and shiny with

proportions of nutrients needed to maintain good health and no morphologic changes from the sound enamel.
sustain life.
Q.30. Shoe leather survey.
Q.26. Incipient caries.
Ans.
Ans.
l In 1931, Trendley H. Dean carried out a survey in the
l Incipient caries is the first evidence of caries activity in USA, which was a continuation of McKay’s work, to
the enamel which has not extended to the DEJ and find out the extent and geographical distribution of
enamel is hard and intact. mottled enamel.
l The early carious lesion on visible smooth surfaces of l This survey was done to study the relationship between

teeth is clinically manifested as a white, opaque region, fluoride concentration in drinking water, mottled enamel
which is best demonstrated when the area is air dried. and dental caries; it was known as shoe leather survey.
l At this stage some demineralization of enamel has oc- l In 1939, Dean and McKay reported that fluoride in the

curred but there is no cavity and no major histological domestic water is the primary cause of human mottled
change of the organic matrix of enamel. enamel.
l However, at this stage it is important to recognize that
Q.31. Define dietary caries control and diet counselling.
only chemical change (loss of mineral salts) has oc-
curred in the tissue and that there is no significant pene- Ans.
tration of the bacteria associated with the dentinal lesion.
l Dietary caries control: Most important dietary modifica-
Q.27. Dyes used to detect caries. tion is to decrease frequency of eating sugars.
Diet counselling:
Ans.
i. Restrict the number of eating times to three main
Various dyes used for caries detection are as follows: meals
l Products based on acid red 52 are marketed by a num- ii. Avoid carbohydrate snacks in between meals
ber of manufacturers, e.g. Caries Detector, Kuraray, iii. Take low carbohydrate and high protein snacks
Osaka, Japan. and fruits in between meals, if required.
l Acid reds 50, 51, 54 have provided good success to iv. Increase eating of high protein foods, e.g. meat,
many clinicians. fish and milk.
Section | I  Topic-Wise Solved Questions of Previous Years 483

v. Restrict carbohydrate eating so that they provide Q.36. Classify sugar substitutes.
only 30%–50% of total calories requirement by
Ans.
the body.
vi. Eliminate eating sticky sweets like chocolates, [Same as SN Q.11]
toffees, candies, cakes and pastries if not com-
Q.37. Diet charting.
pletely, as far as possible.
vii. Eat firm detersive food like raw vegetables and Ans.
fruits liberally. This reduces dental plaque forma-
[Same as SN Q.14]
tion and increases salivary flow.
Q.38. Diet analysis of sugar exposure.
Q.32. Define caries vaccine. Discuss its routes of admin-
istration. Ans.
Ans. [Same as SN Q.14]
[Same as SN Q.1] Q.39. Diet diary.
Q.33. Methods of caries detection. Ans.
Ans. [Same as SN Q.14]
[Same as SN Q.5] Q.40. Define diet and nutrition.
Q.34. Define rampart caries. Ans.
Ans. [Same as SN Q.22]
[Same as SN Q.7]
Q.35. Cariostatic trace elements.
Ans.
[Same as SN Q.10]

Topic 18
Pit and Fissure Sealants
COMMONLY ASKED QUESTIONS
LONG ESSAYS:
1. What are fissure sealants? Enumerate in detail their importance indications, contraindications and their method
of application.
2. Classify pit and fissure sealants. Give indications, contraindications and detailed procedure of application of pit
and fissure sealants. [Same as LE Q.1]
3. What are fissure sealants? Write indications and contraindications. Describe in detail the method of application
of pit and fissure sealant. [Same as LE Q.1]

SHORT ESSAYS:
1 . Pit and fissure sealants.
2. Prophylactic odontotomy.
3. Indications and contraindications for pit and fissure sealant placement. [Ref LE Q.1]
4. Ideal properties of pit and fissure sealant material.
5. Preventive resin restorations
6. Application of pit and fissure sealants. [Ref LE Q.1]
7. What are pit and fissure sealants? Mention different materials currently used for the procedure. [Same as LE Q.1]
484 Quick Review Series for BDS 4th Year, Vol 1

SHORT NOTES:
1 . Define pit and fissure sealant.
2. Advantages of pit and fissure sealants.
3. Classify pit and fissure sealants based on curing method. [Ref LE Q.1]
4. Mention steps in the procedure of application of pit and fissure sealants. [Ref LE Q.1]
5. Indications for pit and fissure sealants placement. [Ref LE Q.1]
6. Name few sealants materials. [Ref LE Q.1]

SOLVED ANSWERS
LONG ESSAYS:
Q.1. What are fissure sealants? Enumerate in detail Classification of resin sealants:
their importance indications, contraindications and
their method of application. {SN Q.3}
Ans. A. Based on curing method:
l First-generation sealants – polymerized with UV
l Pit is defined as a small pin point depression located at
the junction of developmental grooves or at terminals of light of 350 microns
l Second-generation sealants – self-cured or chemi-
those grooves.
l Fissure is defined as deep clefts between adjoining
cally cured
l Third-generation sealants – visible light-cured
cusps.
Based on alphabetical description of shape, Nango (1960) 430–490 microns wavelength
l Fourth-generation sealants – fluoride-releasing
described four principal types of fissures:
sealants, addition of fluoride for added benefit
V-type Tend to be self-cleansing Noninvasive tech-
U-type and somewhat caries nique is recom-
resistant mended.
B. Based on presence of filler:
I-type Susceptible to caries Invasive technique is l Unfilled – better flow
K-type recommended. l Semi-filled – strong and resistant to wear

C. Based on colour:
According to Nugano (1961) there are five types of pits and
l Tinted – for easy identification
fissures:
l Clear – difficult to detect
i. V-type (34%)
l Opaque – for easy identification
ii. U-type (14%)
l Coloured – e.g. pink (Fuji VII, G. C. company) –
iii. I-type (19%)
better fluoride release
iv. IK-type (26%)
Requirements of an efficient sealant are as follows
v. Inverted Y-type (7%)
(Brauer, 1978):
i. Good flow, a viscosity allowing penetration
into deep and narrow fissures even in way teeth
{SN Q.6} ii. Adequate working time and rapid cure (short
setting time)
Classification of pit and fissure sealants:
iii. Good prolonged adhesion to enamel (good
Mitchell and Gordon (1990) stated that sealants can
bond strength)
be differentiated in the following ways:
iv. Low sorption and solubility
i. Cyanoacrylates
v. Increased hardness and abrasion resistance
ii. Polyurethanes, e.g. epoxylite
after curing
iii. Dimethacrylates, e.g. BISGMA (diluted with
vi. Sufficient strength, surface hardness, dimen-
MMA ratio 3:1)
sional stability and so on
iv. Glass ionomer
vii. Chemically inert and anticariogenic
Section | I  Topic-Wise Solved Questions of Previous Years 485

Age range for sealant application: l Rinse with water thoroughly and dry the tooth with
l 3–4 years age – for primary molar sealant a jet of oil-free air.
application l Brockleherst (1992) suggested that air abrasion with

l 6–7 years age – for the first permanent molar aluminium oxide particles is the best method of
l 11–13 years age – for the second permanent cleaning as it results in an improved surface for resin
molars and premolars wetting, more number of resin tag formation and
more depth of sealant penetration.
(SN Q.5 and SE Q.3) l Sol et al. (2000) found out that use of sodium bicar-

{(Indications for placement of pit and fissure sealant: bonate air-polishing system resulted in higher reten-
i. Pre-carious deep and narrow retentive pits and fis- tion of cement.
sures, which may cause wedging of an explorer l It can be concluded that irrespective of type of pro-

ii. Stained pits and fissures with minimum decalcifi- phylaxis medium, a prophylaxis before sealing is not
cation essential, although tooth preparation can be useful.
iii. No radiographic or clinical evidence of proximal ii. Isolation:
caries and possibility of adequate isolation l The tooth should be isolated from salivary contami-

iv. In caries free pit and fissures, if the patient desires nation by use of rubber dam or by cotton rolls and
v. Caries pattern indicative of more than one lesion suctioning.
per year l Rubber dam is ideal and should be used in fully

vi. Morphology of pit at risk of caries and other fac- erupted teeth but may not be feasible in certain cir-
tors associated with increased caries incidence cumstances, then cotton rolls can be used.
vii. Community-based sealant program)} l Isolate and dry the quadrant with cotton rolls and a

saliva ejector.
[SE Q.3] l This procedure is very technique sensitive so mois-

{Contraindications for sealant usage: ture control is essential to achieve optimum bond
i. Well-coalesced, self-cleansing pits and fissures strength.
ii. Radiographic or clinical evidence of presence of iii. Acid etching:
l Acid etching technique creates microporosities in
interproximal carious lesions
iii. Tooth not fully erupted and where isolation of the enamel surface.
l Various phosphoric acid solutions have been evalu-
tooth is not possible
iv. Life expectancy of tooth is limited ated for the etching procedure. Generally 30%–50%
v. Occlusal surfaces that are already carious and acid solutions or gels are recommended. Buonocore
require restoration initially used 80% phosphoric acid but nowadays
vi. Presence of rampant caries} 37% phosphoric acid is recommended.
l Apply the etching agent to the tooth surface using a

(SN Q.4 and SE Q.6) fine brush or a minisponge or a cotton pellet or ap-
plicator provided by the manufacturer’s and gently
{(Clinical technique for placement of pit and fissure sealants: rub the etchant applicator over tooth surface includ-
The steps involved in placement of sealants are as
ing 2–3 mm of cuspal inclines and reaching into any
follows:
pits and grooves that are present.
i. Cleaning
l Periodically add fresh etchant to the tooth surface
ii. Isolation
carefully avoiding spillage of etchant on the in-
iii. Etching
terproximal areas as it may lead to gingival irrita-
iv. Washing
tion and sealing of adjacent interproximal spaces
v. Application of sealant
together.
vi. Check of occlusal interferences
l Etchant can be either in liquid form or gel form but
vii. Recall and re-evaluation)}
gel is preferred as it is more effective and its flow
[SE Q.6] can be controlled.
l Etching time – It is very critical for this procedure.
{Step I: Cleaning: As recommended the etching time for primary teeth
l The enamel surface preparation can be performed in be double than that of permanent teeth.
different ways prior to etching and sealant application. l Tandon et al. (1989) have proposed an etching time
l Early concept was to polish occlusal surfaces with
of 15 s to be sufficient for primary teeth but the most
pumice and oil-free water using a prophy cup or accepted times and the currently applicable times
bristle brush. were given in IADR sealant symposium in 1991.
486 Quick Review Series for BDS 4th Year, Vol 1

Steps Primary tooth Permanent tooth v. Application of sealant:


Chemically cured sealant:
Acid etch 30 s 20 s
l Manufacturers, instructions should be followed.
Wash 30 s 30 s
Dry 15 s 15 s Precise mixing without any agitation helps to pre-
vent formation of air bubbles.
Acid etching on the surface enamel has shown to produce a l As working time is limited with chemically cured
degree of porosity as follows: material carry it immediately to etched and dried
l First zone, a narrow zone of enamel is removed by tooth surface.
etching. This zone is 10 microns in depth. Visible light–cured sealant:
l The second zone is qualitative porous zone, which is l Apply the material and allow it to flow into
20 microns in depth. pits and fissures. In mandibular teeth, apply
l The third zone is quantitative porous zone with small the sealant distally and allow it to flow me-
porosities and is 20 microns deep. sially with the converse being true for the
Types of etching pattern – Silverstone in 1975 identified maxillary teeth.
three basic patterns of etching: l The sealant is applied to prepared tooth sur-
Type 1: face and gently teased into pits and grooves.
There is a generalized roughening of enamel surface, l Using a fine brush or applicator carry a thin
but with a distinct hollowing of prism centres and layer up the cuspal inclines to seal second-
relatively intact peripheral regions. ary and supplemental fissures.
Type 2: l With careful application incorporation of
Prism peripheries appear to be damaged. Prism cores air bubbles is avoided. As no mixing of ma-
are left projecting towards original enamel surface. terial is required with light-cure sealants
Type 3: there is less chance of incorporation of air
Shows neither type 1 nor type 2 etching pattern but bubbles.
appears as generalized surface roughening. l According to the manufacturer’s recom-
l The surface morphology also changes. The mended time for curing, Hicks et al. (2000)
interface between enamel and resin is an inti- found that argon laser curing of sealant ma-
mate one and shows no detectable microspaces. terial may enhance caries resistance.
l Sealant materials penetrate into the micropo- vi. Check of occlusal interferences:
rosities created in the surface enamel during l Using articulating paper evaluate occlusal interferences
the etching procedure. of sealed tooth surface to determine if any excessive
l Infiltration of etching enamel results in the sealant is present and adjust occlusion if needed.
formation of resin tags, which provide me- l A small discrepancy in occlusion in case of unfilled
chanical means for sealant retention. sealant is easily tolerated as the cement abrades
iv. Washing: away but in case of filled resin sealant occlusal ad-
l Most manufacturers recommend thorough rinsing justment is a must to avoid discomfort.
and drying of the etched tooth surface without l A small round bur at slow speed will remove the
specifying the time interval. excess effectively.
l This removes the etching agent and reaction prod- vii. Recall and re-evaluation:
ucts from etched enamel surface. l Recall and check the patient at subsequent visits. It
l Dry the tooth for 15 s with uncontaminated com- is important to recognize that sealed teeth should be
pressed air. observed clinically at periodic intervals to determine
l The dried etched enamel should have a frosted white effectiveness of sealants.
appearance. If salivary contamination has occurred, l It is necessary to re-evaluate sealed tooth surface for
re-etch for 10 s and repeat the procedure. loss of material, exposure of voids and caries develop-
Application of bonding agent: ment especially in the first 6 months of placement.}
l Application of halogenated bonding agent af- Q.2. Classify pit and fissure sealants. Give indications,
ter etching displaces saliva from enamel contraindications and detailed procedure of applica-
thereby improving sealant wetting of surface tion of pit and fissure sealants.
and increases the bond strength both in saliva-
contaminated enamel and in uncontaminated Ans.
enamel. [Same as LE Q.1]
Section | I  Topic-Wise Solved Questions of Previous Years 487

Q.3. What are fissure sealants. Write indications and v. Wet-bond pit and fissure sealant:
contraindications. Describe in detail the method of ap- l This is the first pit and fissure sealant resin that
plication of pit and fissure sealant. can be applied in a moist field.
l It bonds chemically and micromechanically to
Ans.
the moist tooth, integrating with the tooth struc-
[Same as LE Q.1] ture to create a strong, margin-free bond that
virtually eliminates microleakage.
l It forms a unique resin acid-integrating network
SHORT ESSAYS:
(RAIN) that improves penetration into pits and
Q.1. Pit and fissure sealants. fissures and provides superior sealing of the
margins, e.g. Embrace Wet Bond™ (Pulp dent
Ans. Corporation).
vi. Pit and fissure sealant with ACP:
Pit and fissure sealants are defined as thin plastic coatings l It is a light-cured sealant that contains the ‘smart
placed on the occlusal surfaces of posterior teeth to form a material’ amorphous calcium phosphate (ACP)
mechanical barrier between tooth structure and the oral creating a stronger and longer lasting sealant.
environment. l It has a controlled flow ability that keeps the
Different materials currently used as pit and fissure seal- sealant on the tooth structure while completely
ants are as follows: filling occlusal surfaces and it forms a chemical
i. Fluoride-releasing sealants: and thermal barrier protecting the tooth.
l Garcia Godoy (1997) found out that all the fluo- l To make significant gain in decreasing caries in
ridated sealant had the greatest amount of fluo- children, it is necessary for dentist to educate
ride release by 24 h after mixing and the fluoride and inform the parents and physicians about
release declined sharply thereafter. cost-effectiveness and caries preventive benefits
l Cooley et al. and Hicks et al. in 1992 conducted of sealants.
various studies on a fluoride-releasing sealant
material composed of a modified urethane Bis- Q.2. Prophylactic odontotomy.
GMA resin and concluded that fluoride release
Ans.
dips considerably as the days go by.
l They help in 60% reduction in secondary caries l The technique called prophylactic odontomy was pro-
and enhanced degree of caries resistance. posed by Hyatt (1936).
ii. Clear pit and fissure sealant: l He/she advocated the placement of small amalgam res-
l This type of sealant is aesthetic but, difficult to torations in pits and fissures of newly erupted teeth be-
detect in recall visits, e.g. Helioseal changes fore the appearance of clinical signs of decay. Later
from green to white. when they are fully erupted, a small occlusal cavity
iii. Coloured pit and fissure sealant: should be prepared and filled with amalgam.
l In the beginning the sealant is clear but after l He/she recommended eliminating all susceptible fis-
polymerization it changes its colour. sures by cutting a shallow, minimal width Class I cavity
l The degree of colour change is also an indicator in enamel and then filling it with amalgam.
of its setting and adequate polymerization. l Prophylactic odontotomy is no more recommended, due
l Easy and convenient to see during placement to the encouraging results with fissure sealants.
and recall, e.g. Clinpro pink changes to pink on Advantages:
setting. l Small filling, minimum pulpal irritation and ex-
iv. Fluorescing pit and fissure sealant: tension for prevention is not required.
l With the use of a UV pen light, this sealants l Serious injury of the deep decay can be prevented.
fluoresces a blue/white colour. l The procedure is relatively painless.
l The fluorescent glow provides clinicians with l The surfaces treated do not force accumulation of
a visual verification of the sealant margins at carbohydrates and food debris and may be remin-
the time of placement and offers the easiest eralized by salivary constituents.
way to verify retention and inspect margins Disadvantages:
during patient recall appointments, e.g. Delton l Cutting instrument must be used.
Seal-N-Glo (Dentsply). l Tooth is always subjected to unnecessary reduc-
l The fluorescing sealant eliminates the guess- tion and there is a risk of sealing caries if the fis-
work involved with placing sealants. sure extends to the dentino-enamel junction.
488 Quick Review Series for BDS 4th Year, Vol 1

Q.3. Indications and contraindications for pit and fis- l Classification according to type is necessary so as to
sure sealant placement. determine the type of restorative material to be selected.
Type A: This comprises suspicious pits and fissures
Ans. where caries removal is limited to enamel only. A slow
[Ref LE Q.1] speed round bur is used to remove any decalcified
enamel and local anaesthesia is not required.
Q.4. Ideal properties of pit and fissure sealant material. Type B: This comprises incipient lesion in dentin that is
Ans. small and confined.
Type C: This is characterized by the need for greater
l Pit and fissure sealants are defined as ‘a cement or a exploratory preparation in dentin.
resin which is introduced into unprepared occlusal pits This would require local anaesthesia and placement
and fissures of caries susceptible teeth forming a me- of a liner like calcium hydroxide over the exposed
chanical and physical protective layer against the action dentin.
of acid producing bacteria and their substances’. l Initially, Simonsen advocated an unfilled resin for type
l Ideal properties of pit and fissure sealant materials are
A, a diluted composite resin (a combination of unfilled
as follows: bond agent and filled resins) for type B and a filled
i. Good flow, a viscosity a allowing penetration into composite for type C.
deep and narrow fissures even in way teeth. l The greater retention of diluted composite resin was at-
ii. Adequate working time and rapid cure (short set- tributed to the greater wear resistance of filled particles,
ting time). whereas the dilution with unfilled resin provided the
iii. Good prolonged adhesion to enamel (good bond viscosity necessary to flow.
strength). l With the advent of wear resistant (filled), light activated,
iv. Low sorption and solubility. radiopaque composite resin systems for posterior teeth,
v. Increased hardness and abrasion resistance after these new generation of materials will enhance success
curing. of preventive resin restorations.
vi. Sufficient strength, surface hardness, dimensional
Placement technique:
stability, etc.
Armamentarium:
vii. Chemically inert, anticariogenic and cariostatic in
l Local anaesthesia (optional)
action.
l Rubber dam or cotton rolls
viii. It should have the same thermal conductivity as the
l Cotton pellets
tooth.
l Burs-slow-speed, high-speed and carbide fluted
ix. It should have the reduced polymerization shrink-
finishing
age.
l Etching gel, sealant, bonding agent, calcium hy-
Mechanical properties of an ideal sealant are summarized
droxide liner and composite resin material.
as follows:
l Polymerization unit (visible light)
l High penetration and wear resistance
l Plastic (teflon) instrument or condenser
l Medium working time
l Marking paper
l Low water sorption and thermal expansion
Procedure:
Q.5. Preventive resin restorations l Clean the surface of the tooth surface.

l Isolate with cotton rolls or preferably rubber dam.


Ans.
l Remove decalcified pits and fissures with a slow

l Preventive resin restorations are also known as conser- speed 1/4 or 1/2 round bur.
vative composite restorations. l Place etchant gel all over the occlusal surface for

l The preventive resin restoration is an alternative proce- 20–60 s and Wash (20 s) and dry (10 s) the surface
dure for restoring young permanent teeth that require thoroughly.
only minimal tooth preparation for caries removal but l Apply the sealant carefully, avoiding air entrapment/

also have adjacent susceptible fissures. voids in the preparation site.


l Simonsen and Stallard described the technique of re- l Polymerize with visible light-curing light for 20 s or

moving only the carious tooth structure in small class 1 according to manufacturer’s instructions, adjust the
cavities. A resin restoration was then placed, and the occlusion, if needed, with finishing burs.
adjacent pits and fissures were sealed at the same time. l Type-B restorations are indicated when exploratory

l There are three types of preventive resin restorations as removal of caries has included dentin to a slight
determined by the exploratory preparation based the extent.
extent and depth of the caries lesion. These are A, B and l Type C restorations are larger and deeper hence add

C (Simonsen, 1978). additional polymerization time (30 s). In most cases,


Section | I  Topic-Wise Solved Questions of Previous Years 489

local anaesthesia will also be required. A base or Advantages of pit and fissure sealants are as follows:
sub-base may be needed before the placement of the i. Noninvasive procedure
final restoration. ii. Easy to apply
iii. Prevent occurrence of fissure caries
Q.6. Application of pit and fissure sealants.
iv. Minimum armemtarium required for their placement
Ans.
Q.3. Classify pit and fissure sealants based on curing
[Ref LE Q.1] method.
Q.7. What are pit and fissure sealants? Mention differ- Ans.
ent materials currently used for the procedure.
[Ref LE Q.1]
Ans.
[Same as SE Q.1] Q.4. Mention steps in the procedure of application of pit
and fissure sealants.

SHORT NOTES: Ans.

Q.1. Define pit and fissure sealant. [Ref LE Q.1]

Ans. Q.5. Indications for pit and fissure sealants placement.


Pit and fissure sealants are defined as ‘a cement or a resin Ans.
which is introduced into unprepared occlusal pits and fis-
sures of caries susceptible teeth forming a mechanical and [Ref LE Q.1]
physical protective layer against the action of acid produc-
Q.6. Name few sealants materials.
ing bacteria and their substances’.
Q.2. Advantages of pit and fissure sealants Ans.

Ans. [Ref LE Q.1]

Topic 19
Atraumatic Restorative Treatment
COMMONLY ASKED QUESTIONS
LONG ESSAYS:
1. ART (atraumatic restorative technique).

SHORT ESSAYS:
1. Define ART and write briefly its advantages. [Ref LE Q.1]

SHORT NOTES:
1 . Define ART.
2. Prophylactic odontotomy.
490 Quick Review Series for BDS 4th Year, Vol 1

SOLVED ANSWERS
LONG ESSAYS:
Q.1. ART (atraumatic restorative technique). l The assistant’s head should be at a higher level
than the operator, so that the assistant can also see
Ans.
the operating field and can pass the correct instru-
[SE Q.1] ments as needed.
Patient position:
{l The atraumatic restorative treatment (ART) is also l A patient lying on the back on a flat surface will
known as alternate restorative treatment. provide safe and secure body support and com-
l The ART is a procedure based on removing carious fortable and stable position for lengthy periods of
tooth tissues using hand instruments alone and restoring time.
the cavity with an adhesive restorative material. Arrangements in the mouth:
l The results of ART were so encouraging that the system l A very important aspect for the success of ART is
has been adopted by WHO and is being promoted control of saliva around the tooth being treated.
worldwide as a useful technique for communities that l Cotton wool rolls are quite effective at absorbing
lack regular dental facilities.} saliva and can provide short-term protection from
Principles of ART: moisture or saliva.
The two main principles of ART are Essential instruments and materials:
i. Removing carious tooth substance using hand l Following instruments and materials are used for
instruments only. ART:
ii. Restoring the cavity with a restorative material i. Cotton rolls
that adheres to tooth, e.g. glass ionomer cement. ii. Small or medium size spoon shaped excavator
Armamentarium, methods, materials and patient prepara- iii. Calcium hydroxide paste
tion for ART: iv. Glass ionomer cement
The correct positioning of both the operator and patient Procedure:
is essential to achieve good quality care. l The tooth is isolated with cotton rolls and the tooth
The operator’s work posture and positions: surface to be treated is cleaned with a wet cotton
l The work posture and position of the operator pellet.
should be comfortable and provide the best view l The entrance of the lesion is slightly widened by
of the patient’s oral cavity. hand instruments to remove gross overhanging un-
l The operator sits firmly on the stool, with straight supported enamel rods.
back, thighs parallel to the floor and both feet flat l By using either the small or medium size spoon-
on the floor. shaped excavator, the dental caries is removed and if
l The operator should be positioned behind the necessary pulpal protection is provide by calcium
head of the patient. The exact position will de- hydroxide paste.
pend on the area of the patient’s mouth to be l The cavity surface along with occlusal margins is
treated. cleaned and acid-etched.
l Good vision is essential, artificial light is more l The mixed glass ionomer is inserted into the cavity
reliable, constant and can also be focused on a and slightly overfilled. A gloved finger, which is
particular spot; hence a portable light source is smeared with petroleum jelly, is pressed on top of the
recommended, e.g. a headlamp, glasses with a entire occlusal surface and slight pressure is applied.
light source attached or a light attached to the l The bite is checked and excess material is removed
mouth mirror. with a sharp carver and all high points are removed.
Assistance: l The filling is covered with petroleum jelly once
l Oral care is best provided by a team consisting of again or the varnish may be applied and patient is
an operator and an assistant. instructed not to bite with the tooth for at least half
l Assistant is of great advantage particularly in an hour.
treating children using ART. He/she allows the The reasons for using hand instruments are
operator to concentrate on the cavity and maintain l It makes restorative care accessible to all population
effective saliva control. groups who are living in remote areas without access
l The assistant works at the left side of a right- to electric supply.
handed operator and does not change position and l The use of a biological approach, which requires
should be as close to the patient support as pos- minimal cavity preparation that conserves sound
sible, facing the patient’s mouth. tooth structure.
Section | I  Topic-Wise Solved Questions of Previous Years 491

l Compared to electrically driven dental equipment the limited to small and medium sized one surface cavi-
hand instruments are economical. ties only.
l The limitation of pain that reduces the need for local l The continuous use of hand instruments over long
anaesthesia to a minimum and reduce psychological period of time may result in hand fatigue.
trauma to patients. l A relatively unstandardized mix of glass ionomer
l Simplified infection control. Hand instruments can may be produced due to hand mixing.
be easily cleaned and sterilized after every patient.
The reasons for using glass ionomer cement:
l As the glass ionomer chemically bonds to both
SHORT ESSAYS:
enamel and dentin, it reduces the need to cut sound
tooth tissue to prepare the cavity. Q.1. Define ART and write briefly its advantages.
l Fluoride is released from the restoration to prevent Ans.
and arrest secondary caries.
l Glass ionomer is biocompatible, does not inflame
[Ref LE Q.1]
pulp or gingiva.
l It has a coefficient of thermal expansion similar to SHORT NOTES:
tooth structure.
Q.1. Define ART.
[SE Q.1]
Ans.
{Advantages of ART:
l Easily available inexpensive hand instruments are l The ART means atraumatic restorative treatment. It is
used rather than the expensive electrically driven also known as alternate restorative treatment.
l The ART is a procedure based on removing carious
dental equipment.
l As it is almost a painless procedure the need for local
tooth tissues using hand instruments alone and restoring
anaesthesia is eliminated or minimized. the cavity with an adhesive restorative material.
l The ART system has been adopted by WHO and is be-
l ART involves the removal of only decalcified tooth

tissues, which results in relatively small cavities and ing promoted worldwide as a useful technique for com-
conserves sound tooth tissue as much as possible. munities that lack regular dental facilities.
l As ART is based on modern concepts of cavity prepa-
l Sound tooth tissue need not be cut for retention of

filling material. The retention is obtained by the mi- ration where minimal intervention and invasion is
crotags produced due to etching and also because of emphasized; this approach is applicable also in the
the chemical adhesion of glass ionomer restorative industrialized countries for special groups such as the
material with cavity walls. physically and mentally handicapped and the elderly
l A practice of straight forward and simple infection
patients.
control is used without the need to use autoclaved Q.2. Prophylactic odontotomy.
hand pieces.
l The leaching of fluoride from glass ionomer proba-
Ans.
bly remineralizes sterile demineralized dentin and l The prophylactic odontomy technique was proposed by
prevents development of secondary caries. Hyatt (1936).
l The combined preventive and curative treatment can l He/she advocated the placement of small amalgam res-
be done in one appointment. torations in pits and fissures of newly erupted teeth be-
l Restorations can be easily repaired. fore the appearance of clinical signs of decay. Later
l It is economical and less time-consuming procedure when they are fully erupted, a small occlusal cavity
as in one sitting several fillings can be done. should be prepared and filled with amalgam.
l One of the greatest advantages of ART is that it l He/she recommended eliminating all susceptible fis-
enables oral health workers to reach people who sures by cutting a shallow, minimal width class I cavity
otherwise never would have received any oral health in enamel and then filling it with amalgam.
service.} l Prophylactic odontotomy is no more recommended, due
Disadvantages of ART: to the encouraging results with fissure sealants.
l ART restorations are not long lasting. The average Advantages:
life is 2 years. l Small filling and minimum pulpal irritation.
l As fundamental principles of cavity preparation are l The procedure is relatively painless.
not followed all oral health workers may not accept it. Disadvantages:
l Because of the low wear resistance and low strength l Use of cutting instruments.
of the existing glass ionomer materials, their use is l Tooth is always subjected to unnecessary reduction.
492 Quick Review Series for BDS 4th Year, Vol 1

Topic 20
Fluorides and Oral Habits
COMMONLY ASKED QUESTIONS
LONG ESSAYS:
1 . What is topical fluoride? Explain in detail about sodium fluoride and APF.
2. Describe the mechanism of action of stannous fluoride and advantages and disadvantages of stannous fluoride.
3. Give a detailed account of role of fluorides in preventive dentistry.
4. Define water fluoridation. Discuss school water fluoridation in India.
5. Discuss in detail the role of fluoride varnish used for the prevention of dental caries.
6. Describe the role of systemic fluorides in the prevention of dental caries.
7. Describe the role of topical fluorides in the prevention of dental caries. [Same as LE Q.1]
8. Explain mechanism of action of fluoride. [Same as LE Q.3]
9. Discuss in detail systemic use of ‘fluorides’ for the prevention of dental caries in children. [Same as LE Q.6]

SHORT ESSAYS:
1. Fluoride gels.
2. Defluoridation.
3. What do you know about fluoride varnishes and their importance? [Ref LE Q.5]
4. Brudevold’s solution.
5. Stannous fluoride.
6. Knutson’s technique.
7. Water fluoridation. [Ref LE Q.6]
8. Nalgonda technique.
9. Topical fluorides.
10. Shoe leather survey.
11. Mechanism of action of fluoride in preventing dental caries.
12. Endemic fluorosis.
13. Give differences between sodium fluoride and stannous fluoride.
14.
What is optimum level of fluoride in water? List the recommendations for use of fluoridated tooth paste in
children.
15. What is the difference between acute fluoride toxicity and chronic fluoride toxicity?
1 6. Thixotropic gel.

SHORT NOTES:
1. Milk fluoridation. [Ref LE Q.6]
2. Fluoride mouthwash.
3. APF gel.
4. Fluoride varnish. [Ref LE Q.5]
5. Fluoride tablets.
6. Brudevold’s solution. [Ref SE Q.4]
7. Dean’s index of fluorosis.
8. Choking phenomenon.
9. Advantages and disadvantages of sodium fluoride. [Ref LE Q.1]
10. School water fluoridation. [Ref LE Q.4]
11. Shoe leather survey. [Ref SE Q.10]
12. Fluoride dentifrice.
13. Fluoride gels. [Same as SN Q.3]
Section | I  Topic-Wise Solved Questions of Previous Years 493

SOLVED ANSWERS
LONG ESSAYS:
Q.1. What is topical fluoride? Explain in detail about SiF2 thus reducing the availability of free active
sodium fluoride and APF. fluoride for proposed anticaries action.
Method of application:
Ans.
l Cleaning and polishing of the teeth is done only
l Topical fluorides are those fluoride-containing agents during the first of the four applications.
which are applied to the tooth surfaces in regular inter- l Upper and opposing lower quadrants are isolated
vals in order to prevent the development of caries. with cotton rolls and the teeth are dried thor-
l Topical fluorides are the fluoride supplements acting lo- oughly.
cally or topically. For their local effect the fluoride must l 2% NaF is then applied with cotton applicators
be either contacting tooth surface before it is swallowed and is permitted to dry on the teeth for about
or pass through the circulation and be secreted in saliva. 4 min.
l Dean proved that individuals continuously living in a l The procedure is repeated for the remaining quad-
fluoride-rich area had less caries as compared to those rants.
living in nonfluoride areas. Thus, the idea of topical l After completion of the treatment the patient is
application of fluorides for prevention of dental caries instructed to avoid eating, drinking or rinsing for
came forth. 30 min in order to prolong the availability of fluo-
Topical fluorides can be divided into: ride ions to react with the tooth surfaces.
Professionally applied fluorides: l Second, third and fourth applications are given at
l Neutral sodium fluoride weekly intervals at ages 3, 7, 11 and 13 years.
l Stannous fluoride Mechanism of action:
l Acidulated phosphate fluoride (APF) l When NaF is applied topically, it reacts with hy-
l Amine fluoride droxyapatite crystals to form CaF2.
l Fluoride gels l Due to high concentration of fluoride there is ini-
l Fluoride varnishes tial formation of CaF2 on the tooth surface which
Self-applied fluorides: prevents further entry of fluoride into the tooth.
l Toothbrushing dentifrices This is called choking off effect as it blocks the
l Toothbrushing solutions or gels diffusion.
l Toothbrushing prophylaxis pastes l CaF2 reacts with hydroxyapatite to form fluori-
l Applying gels in trays dated hydroxyapatite which increases the concen-
l Mouth rinses tration of surface fluoride.
A. Sodium fluoride: l It makes the tooth structure more stable, less sus-

ceptible to dissolution by acids, interferes with


plaque metabolism through antienzymatic action
{SN Q.9}
and also helps in remineralization of the initial
Knutson and Feldman recommended a technique of
l decalcified areas.
four applications of 2% NaF at weekly intervals in a
year at 3, 7, 11 and 13 years.
{SN Q.9}
Properties of sodium fluoride: Advantages:
l Neutral pH l Chemically stable
l 9200 ppm of F l Acceptable taste
l Caries reduction in 1st year was 45% and in 2nd l Nonirritating to gingival tissues
year was 36% l Does not discolour the teeth
Method of preparation: l Economical/inexpensive
l 2% NaF solution can be prepared by dissolving

20 g of NaF powder in 1 L of distilled water in a


plastic bottle. Disadvantages:
l It is essential to store fluoride in plastic bottles be- l Continuous application for 4 min.

cause if stored in glass containers, the fluoride ion l Patient has to make four visits in a short time.

of solution can react with silica of glass forming l Follow-up is difficult.


494 Quick Review Series for BDS 4th Year, Vol 1

B. Acidulated phosphate fluoride: l Solution is acidic.


l Acidulated phosphate fluoride is simply an acidified l It is sour and bitter in taste.
sodium fluoride with phosphoric acid in order to gain Comparison of APF gel and APF solution:
more depth of fluoride penetration when applied on
the tooth. APF gel APF sol
l It can be used as a topical agent either solution or gel.
l Relatively costly l Relatively cheaper
l A semiannual application of 1.23 APF for 4 min is
l Readily available l Prepared easily
helpful in reducing caries by 28%. l Self-application possible l Applied by dentist or auxil-
Method of preparation: iary staff
l It is prepared by dissolving 20 g of NaF in l L of

0.1 M phosphoric acid, to this 50% hydrofluoric Q.2. Describe the mechanism of action of stannous fluoride
acid is added to adjust the pH at 3.0 and F concen- and advantages and disadvantages of stannous fluoride.
trations at 1.23%.
l For the preparation of APF gel, a gelling agent Ans.
like methylcellulose or hydroxyethyl cellulose is Stannous fluoride occupies a central role in preventive
to be added to the solution and the pH is to be dentistry.
adjusted between 4 and 5. Method of preparation:
Method of application: l Stannous fluoride solution has to be freshly prepared
l After thorough prophylaxis, the teeth are isolated
before use each time, as it has no shelf life.
with cotton rolls on both lingual and buccal sides l For preparation 0.8 g of SnF2 powder is dissolved in
and dried. 10 mL of distilled water in a plastic container and the
l Using cotton applicators APF solution is continu-
solution thus prepared is shaken briefly.
ously and repeatedly applied and the teeth are l The fresh solution is then applied immediately.
kept moist for 4 min. Method of application:
l Floss may be drawn through each interpromixal
(Muhler technique)
embrasure to ensure wetting of these surfaces. l The recommended procedure for application of
l APF in gel form is applied using trays and patient
SnF2 begins with thorough prophylaxis.
is asked to bite tightly for 4 min. The gel thins out l The teeth are then isolated with cotton rolls and
under the biting force because of its thixotropic dried with compressed air.
nature. Salivary ejectors are used during this l Either a quadrant or half of the mouth can be
application. treated at one time.
l The recommended frequency of APF topical
l A freshly prepared 8% solution of SnF2 is applied
application is semiannual. continuously to the teeth with cotton applicator
l Instruct the patient to expectorate immediately
and reapplication of the solution to a particular
and avoid drinking and eating for the next 30 min. tooth is done every 15–30 s so that the teeth are
Mechanism of action: kept wet for 4 min.
l When APF is applied on the teeth, it initially leads
l The recommended frequency of application is
to dehydration and shrinkage in the volume of once per year.
hydroxyapatite crystals, which further on hydro- Mechanism of action:
lysis form an intermediate product called dical- l SnF2 reacts with hydroxyapatite of dental enamel, in
cium phosphate dihydrate (DCPD). addition to fluoride the tin combines with it and
l This DCPD is highly reactive with fluoride lead-
forms a new crystalline product – stannous trifluoro-
ing to formation of fluorapatite. phosphate that is more resistant to decay than enamel.
l The amount and depth of fluoride deposited as
l There is rapid penetration of tin and fluoride in 30 s
fluorapatite would be dependent on the amount therefore continuous reapplication after 15–30 s is
and depth at which DCPD gets formed. needed.
Advantages: l In addition to Stannous trifluorophosphate, three more
l Has acceptable taste
additional products are formed are tin hydroxyphos-
l No staining
phate, calcium fluoride and calcium trifluorostannate.
l No gingival irritation
Advantages:
l Stable with long shelf life
l Rapid penetration of tin and fluoride within 30 s.
l Economical
l The tin–trifluorophosphate complex formed on
Disadvantages: the enamel surface is more resistant to decay than
l Teeth have to be kept wet for 4 min.
enamel.
Section | I  Topic-Wise Solved Questions of Previous Years 495

Disadvantages: Q.4. Define water fluoridation. Discuss school water


l Highly unstable, hence should be prepared freshly fluoridation in India.
l Has a metallic taste
Ans.
l Causes gingival irritation

l Produces discolouration of teeth l Water fluoridation is defined as the upward adjustment


l Causes staining on margins of restorations of the concentration of fluoride ion in public water sup-
Q.3. Give a detailed account of role of fluorides in pre- ply in such a way that the concentration of fluoride ion
ventive dentistry. in the water may be consistently maintained at one part
per million (ppm) by weight.
Ans.
l It has been concluded that 1 ppm fluoride is not only
Mechanism of action of fluoride: best for caries control, but was also well within limits of
i. Improved crystallinity: safety.
l Fluoride increases the crystal size and produces

less strain in crystal lattice. Fluoride compounds used in water fluoridation:


l This takes place through conversion of amor- i. Fluorspar
phous calcium phosphate into crystalline hy- ii. Sodium fluoride
droxyphosphate. iii. Silicofluorides
ii. Void theory: iv. Sodium silicofluoride
l Fluoride fills the voids in the hydroxyapatite v. Hydrofluosilicic acid
crystal and attains a stable form with formation vi. Ammonium silicofluoride
of more and stronger hydrogen bonds. Optimum level of fluoride:
l Greater stability will lead to lower solubility and l Optimum concentration for fluoride in the water
hence greater resistance to dissolution in acids. is in the range of 0.7 to 1.2 parts per million. This
iii. Acid solubility: range effectively reduces tooth decay, while mini-
l Fluorapatite or fluoridated hydroxyapatite is mizing the occurrence of dental fluorosis.
less soluble than hydroxyapatite, therefore has Optimal fluoride concentrations and climatic condi-
greater stability. tions:
iv. Enzyme inhibition: l The water intake of individuals varies widely and
l Fluoride forms a complex with enolase enzyme is influenced significantly by climate.
to inhibit glucose transport. It also binds with l Children living in a 1 ppm fluoridated area are
phosphatases to reduce acid production. assumed to receive an optimal intake of fluoride
v. Suppressing the flora: from water and food of 1 mg fluoride daily.
l Stannous fluoride oxidizes the thiol group present l The optimal concentration of fluoride in drinking
in bacteria thus inhibiting bacterial metabolism. water was assumed to be 1 ppm for the incorpo-
vi. Antibacterial action: ration of fluoride in dental enamel during its
l The concentration of fluoride above 2 ppm in formative stages for children living in temperate
solution progressively decreases the transport of climates.
uptake of glucose into cells of oral streptococci l An empiric formula developed by Galagan and
and also reduces ATP synthesis. Vermillion for estimating the amount of daily
vii. Lowering free surface energy: fluoride is as follows:
l Fluoride substitutes the hydroxyl ions to reduce ppm of F 5 0.34/E
the free surface energy and thus indirectly re- where E520.038 1 0.0062 3 t
duces the deposition of pellicle and subsequent ‘E’ is the estimated daily water intake of children
plaque formation. in oz/lb of body weight.
viii. Desorption of protein and bacteria: ‘t’ is the mean maximum daily air temperature in
l Hydroxyapatite crystals have both positive and degree Fahrenheit of the area.
negative receptor sites. Acidic protein group binds Advantages of water fluoridation:
to calcium site and basic to phosphate site. Fluoride l Large number of people are benefited.
inhibits the binding of acidic protein to hydroxy- l Consumption is regular.
apatite thereby displaying its beneficial effects. l Fluoridated drinking water not only acts systemi-
ix. Alteration in tooth morphology: cally during tooth formation to make dental
l Dentition in fluoridated communities showed a enamel more resistant to dental decay, but also
tendency towards rounded cusps, shallow fis- has topical effect through the release in saliva
sures due to selective inhibition of ameloblasts. after ingestion.
496 Quick Review Series for BDS 4th Year, Vol 1

Fluoridation of community water is the least ex-


l
l The two most commonly used varnishes are
pensive and most effective way to provide fluo- i. Duraphat (NaF varnish containing 2.26%F) in
ride to a large group of people. organic lacquer
Disadvantages of water fluoridation: ii. Fluorprotector (silane fluoride with 0.7% F))}
l Interferes with human rights.

l Other modes are not considered.

l Common source of water supply may not be present.


[SE Q.3]
{Duraphat:
{SN Q.10} Duraphat is a trade name of sodium fluoride var-
l

nish which is available in the market.


School water fluoridation:
l Duraphat is sodium fluoride in varnish form con-
l School water fluoridation is a suitable alternative
taining 22.6 mg F/mL (2.26%) suspended in an
where community water fluoridation is not feasible.
alcoholic solution of natural organic varnishes.
l The amount of fluoride added in school drinking
l It is available in bottles of 30 mL suspension con-
water should be greater than normal about 3–5 ppm
taining 50 mg NaF/mL.
fluoride because children would be exposed to the
l The active fluoride available is 22,600 ppm.
fluoridated water for a limited period in a day and
l It has neutral pH.
to compensate for holidays and vacations.
l The duraphat remains on tooth surface and its
l The current recommended regimen for school wa-
ability to inhibit caries is more than fluorpro-
ter fluoridation is adding 4.5 times more fluoride.
tector.
l There has been around 25% to 40% decrease in
Mechanism of action:
dental caries with this program.
l Duraphat is NaF in varnish form, with neutral
Advantages:
pH.
l Unlike other school-based preventive pro-
l When applied topically under clinically con-
grams like fluoride tablets and mouth rinses,
trolled conditions, a reservoir of fluoride ions
no effort is required by the recipients.
gets built up around the enamel of teeth.
l Good results in reducing caries.
l From this, fluoride keeps on slowly releasing
l Minimal equipment.
and continuously reacting with the hydroxy-
l Not expensive.
apatite crystals of enamel over a long period of
Disadvantages:
time leading to deeper penetration of fluoride
l Children do not receive the benefit until they
and more formation of fluorapatite.})
go to school. It allows only fewer benefits in
l 10Ca5 (PO4) 3OH 1 10F 5 6Ca5(PO4) 3F 1
the primary dentitions.
2CaF2 1 6Ca3(PO4) 2 1 10OH
l Not all children go to school in poor countries
l A part of CaF2 so formed in low concentrations
like India.
further reacts with crystals of hydroxyapatite
l Amount of water drunk cannot be regulated.
and forms fluorapetite.
l Continuous, careful monitoring is required.
l 2Ca5 (PO4) 30H 1 CaF2 5 2Ca5(PO4) 3F 1

Ca(OH)2
Q.5. Discuss in detail the role of fluoride varnish used Fluorprotector:
for the prevention of dental caries. l Fluorprotector is a colourless, polyurethane lac-

quer dissolved in chloroform and dispensed in


Ans. 1 mL ampules.
l This product contains 2% difluorosilane.
(SN Q.4 and SE Q.3) Mechanism of action:
l Silane fluoride of fluorprotector reacts with
{( Fluoride varnishes are developed in order to in-
l water to form considerable amounts of hydro-
crease the retention of topical fluoride on to the fluoric acid (HF), which penetrates enamel
enamel for a longer period. more rapidly than fluoride.
l To enhance the caries inhibitory property of topical l Fluorosilanes enhance retention and penetra-
fluorides, a new coating method was developed in tion of fluoride in enamel by utilizing enamel
which the teeth coated with a lacquer-containing network as a conduit.
fluoride called F-lacquer, which released fluoride l Compared to Duraphat, the fluoride deposited
ions to the dental enamel in high concentrations for in the enamel is more in case of fluorprotector.
several hours in the atmosphere of the mouth. R-SiF2OH 1 H2O 5 R-Si (OH)3 1 2HF.
l Consequently the use of fluoride-containing var- Technique of varnish application:
nishes in caries prevention has become the treatment l Following thorough oral prophylaxis, teeth
of choice. are dried and isolated, but cotton rolls should
Section | I  Topic-Wise Solved Questions of Previous Years 497

not be used for isolation, as the varnish sticks to iv. Sodium silicofluoride
them. v. Hydrofluosilicic acid
l A total of 0.3–0.5 mL (6.9–11.5 mg F) of varnish vi. Ammonium silicofluoride
is required to cover the full dentition. Equipments for water fluoridation:
l The application is done first on lower arch and There are three systems for water fluoridation:
then on upper arch with a small brush, starting i. Saturator system
with the proximal surfaces and covering over all ii. Dry feeder system
surfaces of teeth. iii. Solution feeder system
l After application the patient is made to sit with Optimal fluoride concentrations and climatic condi-
mouth open for 4 min before spitting. tions:
l The patients is instructed not to rinse or drink l The water intake of individuals varies widely

anything for 1 h, not to eat anything solid but with climate


take liquids and semisolids only, till next morn- l Children living in a 1 ppm fluoridated area are

ing to prolong the contact between varnish and assumed to receive an optimal intake of fluo-
enamel. ride from water and food of 1mg fluoride daily
Galagan and Vermillion developed an empiric
formula for estimating the amount of daily
{SN Q.4} fluid intake:
Safety aspect of fluoride varnish: ppm F 5 0.34/E
l If the fluoride varnish is used well within the rec- where E 520.038 1 0.0062 3 t
ommended doses then the plasma concentrations ‘E’ is the estimated daily water intake of chil-
thus attained are far below the toxic levels. dren in oz/lb of body weight.
‘t’ is the mean maximum daily air temperature
in degree Fahrenheit of the area.
Q.6. Describe the role of systemic fluorides in the pre- Advantages of communal water fluoridation:
vention of dental caries. i. Large number of people are benefited.
Ans. ii. Consumption is regular.
iii. Fluoridated drinking water makes dental
In children, systemic fluorides are used in various ways and enamel more resistant to dental decay, and also
forms for the prevention of dental caries: has topical effect through the release in saliva.
Systemic fluorides: iv. Fluoridation of community water is the least
a. Water fluoridation expensive and most effective way to provide
b. School water fluoridation fluoride to a large group of people.
c. Salt fluoridation Disadvantages of water fluoridation:
d. Milk fluoridation i. Interfere with human rights.
e. Fluoride tablets and vitamins ii. Other modes are not considered.
f. Fluoride drops iii. Common source of water supply may not be
[SE Q.7] present.}
b. School water fluoridation:
{a. Water fluoridation: l This program helps in limiting caries in school
l It is defined as the upward adjustment of the con- children.
centration of fluoride ion in public water supply in l The amount of fluoride added in school drinking
such way that the concentration of fluoride ion in water should be greater than normal because chil-
the water may be consistently maintained at one dren stay in school for a short time.
part per million by weight. l The current recommended regimen for school wa-
l Since 1 ppm fluoride is not only best for caries ter fluoridation is adding 4.5 times more fluoride.
control, but also well within limits of safety, it is Advantages:
added to communal drinking water. l Good results in reducing caries
l Optimum concentration for fluoride in the water l Minimal equipment
is 0.7–1.2 parts per million. This range effectively l Not expensive
reduces tooth decay, and minimizes the occur- Disadvantages:
rence of dental fluorosis. l Children do not receive the benefit until they
Various fluoride compounds used in water fluorida- go to school.
tion are l Not all children go to school in poor countries
i. Fluorspar like India.
ii. Sodium fluoride l Amount of water drunk cannot be regulated.
iii. Silicofluorides
498 Quick Review Series for BDS 4th Year, Vol 1

c. Salt fluoridation: Dosage depends upon the age of the child and con-
l As a dietary vehicle for ensuring adequate inges- centration of fluoride in that area.
tion of fluoride domestic salt comes next to drink- l It is recommended that a child consume no
ing water. more than 1 mg of fluoride per day from fluo-
l Concentration of fluoride is 200–350 mgF/kg. ride supplements and from the drinking water.
Advantages: l Fluoride doses are given depending upon the
i. Fluoridated salt is safe. patient’s age and level of fluoride in the drink-
ii. Theoretically fluoridated salt prevents dental ing water.
caries by both systemic and topical actions.
Birth to 24 25–36 37 months to
iii. No supervision of set up or distribution system.
Water (ppm) months months 13 years
iv. Cost-effective.
v. Depends on individual acceptance or rejection. 0.3 or less 0.25 mg 0.5 mg 1.0 mg
0.3–0.7 mg 0.0 mg 0.25 mg 0.5 mg
Disadvantages:
i. No precise control over indicated consump-
tion, since salt intake varies greatly among Q.7. Describe the role of topical fluorides in the preven-
people. tion of dental caries.
ii. International efforts to reduce sodium uptake. Ans.
iii. Fluoridated salt consumption is lowest when
the need for fluorides is greatest during the [Same as LE Q.1]
early years of life. Q.8. Explain mechanism of action of fluoride.
Ans.
{SN Q.1}
[Same as LE Q.3]
d. Milk fluoridation:
l Milk fluoridation process targets the fluoride di-
Q.9. Discuss in detail systemic use of ‘fluorides’ for the
rectly to the children and is less expensive than prevention of dental caries in children.
water fluoridation. Ans.
l The amount of fluoride to be added depends upon

the age of the child and the fluoride concentration [Same as LE Q.6]
in water.
Compounds used for milk fluoridation are SHORT ESSAYS:
l Calcium fluoride

l Sodium fluoride Q.1. Fluoride gels.


l Disodium monofluorophosphate

l Disodium silicofluoride
Ans.
Feasibility of milk fluoridation in India: l Fluoride gels are also a form for topical application of
Though milk fluoridation is advantageous, this fluorides.
method does not seem to be viable and feasible l Most commonly used compound in form of gels is
because of the following facts: acidulated phosphate fluoride as it is acidic and sour
and bitter in taste, so repeated applications are often
i. In India, majority of the children population difficult.
living in rural and urban areas cannot afford Method of preparation:
l APF gel is prepared by dissolving 20 g of NaF in l L
milk daily.
ii. Variation of intake and quantity of milk is an- of 0.1 M phosphoric acid.
l To this 50% hydrofluoric acid is added to adjust the
other factor which cannot be controlled.
e. Dietary fluoride supplements: pH at 3.0 and F concentration at 1.23%.
l A gelling agent like methylcellulose or hydroxyethyl
l Fluoride supplements are available as drops, tab-

lets and lozenges, fluoride-vitamin preparations cellulose is to be added to the solution and the pH is
and oral rinse supplements. to be adjusted between 4 and 5.
l For the application of gel, patient is positioned up-
Factors to be considered before determining proper
fluoride dosage: right and saliva ejector is kept in place.
l Enough gel is placed to fill one-third of the trough area
i. Concentration of fluoride in drinking water.
ii. Total amount of bioavailable fluoride. of tray so that it is sufficient to cover dental arches.
l Loaded tray is placed over the arch and buccal and lin-
iii. Age of the child.
iv. Dosage forms commercially available. gual surfaces are squeezed forcing gel between them.
Section | I  Topic-Wise Solved Questions of Previous Years 499

l Tray is allowed to remain in mouth for 4 min. has a good shelf life of 1–2 years and is very cost-
l Patient is instructed to expectorate immediately and effective.
avoid drinking and eating for the next 30 min. Magnesia: It removes the excess fluorides but pH
of treated water is beyond 10 and its correction by
Comparison of APF gel and APF solution
acidification or recarbonation is necessary due to
APF gel APF sol which it proves very costly.
i. Relatively costly i. Relatively cheap B. Addition of chemicals:
ii. Readily available ii. Prepared easily a. Lime
(imported in India) iii. Applied by dentist or b. Lime with magnesium salts
iii. Self-application possible auxiliary staff
c. Aluminium salts
d. Magnesia
Q.2. Defluoridation. e. Calcium phosphate
f. Benthonite
Ans. g. Fuller’s earth
l Defluoridation is the process of removing excess, h. Diatomaceous earth
naturally occurring fluorides from drinking water in Nalgonda technique:
l This technique was introduced in the town of
order to reduce the prevalence and severity of dental
fluorosis. Kadiri in Andhra Pradesh in 1980.
l This is a more economical method of defluorida-
or
l It is downward adjustment of fluoride ion in high fluo-
tion.
l In this method, the fluoridated water is defluori-
ride-containing drinking water to the optimum level
thereby providing optimal protection from dental caries dated by addition of lime and aluminium sul-
and reducing fluorosis. phate. To this aluminium chloride can also be
l Optimum limit of fluoride in drinking water for the pre-
added.
l This technique can be used both for domestic and
vention of dental caries is 0.7–1.2 ppm.
Various methods for removal of fluoride may be divided into for community water supplies.
two basic types those based upon: Advantages:
l Simplicity in design, construction and mainte-
A. Ion exchange process
B. Addition of chemicals to water nance
l Adaptable to domestic usage
C. Reverse osmosis
l Readily available chemicals are used
D. Electrolysis
l No regeneration of media
A. Ion exchange process:
l No handling of caustic acids and alkalies
i. Anion exchange resins:
l Little wastage of water
These include polystyrene anion exchange resins and
basic quarternary ammonium type resins. C. Reverse osmosis:
l Reverse osmosis (RO) is a water filtration process in
For example: Tulsion A27, deacedite, FF-IP,
Lewatit, MIH-59 and Ambertite IRA-400. which drinking water, which contains dissolved sol-
ii. Cation exchange resins: ids, is run through a membrane. The water then goes
For example: to a storage tank and 99.9% of unwanted inorganic
l Saw dust carbon
compounds, sediment and other contaminants are
l Defluoron-1 (saw dust impregnated with 2%
flushed down the drain.
l This form of water treatment is one of the most effec-
alum)
l Carbion
tive methods for producing high quality drinking
l Defluoron 2
water.
l Magnesia
D. Electrolysis:
l Electrochemically generated aluminium can remove
Defluoron-1: It is a combination of sulphonated
saw dust impregnated with 2% alum solution. The most contaminants present in water by precipitation
disadvantage is poor hydraulic properties and and adsorption.
l Through the process of electrolysis, coagulating
heavy attritional losses.
Carbion: It is a cation exchange resin of good agents such as metal hydroxides are produced.
l The aluminium species act as a coagulant by com-
durability and can be used both on sodium and
hydrogen cycles. bining with the pollutants to form large size flakes
Defluoron 2: It is a suphonated coal and works on and then can be removed by settling and flotation
the aluminium cycles. It gives excellent results, known as electrocoagulation/flotation process.
500 Quick Review Series for BDS 4th Year, Vol 1

Q.3. What do you know about fluoride varnishes and hydroxyapatite crystals, which further on hydroly-
their importance. sis form an intermediate product called DCPD.
l This DCPD is highly reactive with fluoride leading
Ans.
to formation of fluorapatite.
[Ref LE Q.5]
Q.4. Brudevold’s solution. {SN Q.6}
Ans. Advantages:
l Has greater fluoride uptake

l No staining and no gingival irritation

{SN Q.6} l Stable with long shelf life

l Economical and easy to prepare


l APF solution is also known as Brudevold’s solution.
Disadvantages:
l Brudevold and his co-workers did systematic investi-
l Teeth have to be kept wet for 4 min.
gation to find out an optimal fluoride acid solution
l Solution is acidic, and is sour and bitter in taste.
which would provide maximal fluoride deposition
while causing minimal demineralization. They con-
cluded that semiannual application of 1.23% APF for Q.5. Stannous fluoride.
4 min is helpful in reducing caries by 28%. Ans.

l One of the practical difficulties of doing the topical ap- l Stannous fluoride occupies a central role in preventive
plication is that the teeth must be kept wet with solution dentistry. The stannous fluoride 8% solution is commonly
for 4 min and, moreover, APF solution is acidic and used, the concentration of fluoride in it is 19,360 ppm.
l Stannous fluoride solution has to be freshly prepared
sour and bitter in taste, so repeated applications are
often difficult. before use each time, as it has no shelf life.
l To overcome these problems APF gels were introduced.
Method of preparation:
l 0.8 g of stannous fluoride powder is dissolved in
l Acidulated phosphate fluoride can be used as a topical

agent as it is found that as the pH of the NaF solution 10 mL of distilled water in a plastic container and the
was lowered, fluoride was absorbed into enamel more solution thus prepared is shaken briefly.
l The solution is then applied immediately.
effectively.
Method of application (Muhler technique – 1957):
l The first step in recommended procedure is thorough
{SN Q.6} prophylaxis.
l The teeth are then isolated with cotton rolls and dried
Method of preparation:
with compressed air.
l It is prepared by dissolving 20 g of NaF in l L of
l Either a quadrant or half of the mouth can be treated
0.1 M phosphoric acid.
at one time.
l To this 50% hydrofluorideacid is added to adjust
l A freshly prepared 8% solution of SnF2 is applied
the pH at 3.0 and F concentrations at 1.23%.
continuously to the teeth with cotton applicator and
l For the preparation of APF gel, a gelling agent
reapplication of the solution to a particular tooth is
like methylcellulose or hydroxyethyl cellulose is
done every 15–30 s so that the teeth are kept moist
to be added to the solution and the pH is to be
for 4 min.
adjusted between 4 and 5.
l Patient should refrain from eating or drinking for
Method of application:
next 30 min.
l After thorough prophylaxis and isolation of teeth,
l The recommended frequency of application is once
APF solution is continuously and repeatedly ap-
per year (annually).
plied with cotton applicators and the teeth kept
Mechanism of action:
moist for 4 min.
l SnF2 reacts with hydroxyapatite in addition to fluo-
l Floss may be drawn through each interproximal
ride and forms a new crystalline product – stannous
embrasures to ensure wetting of these surfaces.
trifluorophosphate that is more resistant to decay
l Instruct the patient to expectorate immediately
than enamel.
and avoid drinking and eating for the next 30 min.
l There is rapid penetration of tin and fluoride in 30 s

therefore continuous reapplication after 15–30 s is


Mechanism of action: needed.
l When APF is applied on the teeth, it initially leads l In addition to stannous trifluorophosphate, three
to dehydration and shrinkage in the volume of more additional products are formed, i.e. stannous
Section | I  Topic-Wise Solved Questions of Previous Years 501

hydroxyphosphate, calcium fluoride and calcium tri- l It makes the tooth structure more stable, less suscepti-
fluorostannate. ble to dissolution by acids, interferes with plaque me-
Advantages: tabolism through antienzymatic action and also helps
l Rapid penetration of tin and fluoride within 30 s. in remineralization of the initial decalcified areas.
l The tin–trifluorophosphate complex formed on Advantages:
the enamel surface is more resistant to decay than l Chemically stable

enamel itself. l Acceptable taste

Disadvantage: l Nonirritating to gingival tissues

l Highly unstable, should be prepared freshly l Does not discolour the teeth

l Low pH l Cheap and inexpensive

l Metallic taste Disadvantages:


l Causes gingival irritation l Continous application for 4 min.

l Produces discolouration of teeth l Patient has to make four visits in a short time.

l Causes staining on margins of restorations l Follow-up is difficult.

Q.7. Water fluoridation.


Q.6. Knutson’s technique.
Ans.
Ans.
[Ref LE Q.6]
Knutson’s technique is a method for topical application of
sodium fluoride. Q.8. Nalgonda technique.
Knutson and Feldman recommended a technique of four
Ans.
applications of 2% NaF at weekly intervals in a year at 3,
7, 11 and 13 years. l Nalgonda technique is a method for defluoridation of
Method of preparation: water.
2% NaF solution can be prepared by dissolving 20 g of l This technique was introduced in the town of Kadiri in
NaF powder in 1 L of distilled water in a plastic bottle. Andhra Pradesh in 1980.
Method of application (Knutson’s technique): l This is a more economical method of defluoridation.
l Thorough oral prophylaxis, i.e. cleaning and polish- l This technique can be used both for domestic and for
ing of the teeth is done in only the first of the four community water supplies.
applications. l Although defluoron-2 was successful in removing fluo-
l Upper and opposing lower quadrants are isolated rides, the regeneration and maintenance of the plant re-
with cotton rolls and the teeth are dried thoroughly. quired skilled operation, which may not be readily
l 2% NaF is then applied only once with cotton ap- available.
plicators and is permitted to dry on the teeth for In order to overcome this problem a method was
about 4 min. evolved by Nawalakhe in 1974, which so simple and
l The procedure is repeated for the remaining quad- adaptable that even illiterate persons can make use of it.
rants. l The method used involves the addition of three readily
l After completion of the treatment the patient is available chemicals:
instructed to avoid eating, drinking or rinsing for (a)   Sodium aluminate or lime
30 min so as to prolong the availability of fluoride (b)   Bleaching powder
ion to react with the tooth surfaces. (c)   Filter alum
l Second, third and fourth applications are given at These are added to the fluoride water in the same
weekly intervals at ages 3, 7, 11 and 13 years. sequence which leads to flocculation, sedimenta-
Mechanism of action: tion and filtration.
l When NaF is applied topically, it reacts with hy- l Sodium aluminate or lime hastens settlement
droxyapatite crystals to form CaF2. of precipitate and bleaching powder ensures
l Due to high concentration of fluoride there is initial disinfection.
formation of CaF2 on the tooth surface which pre- l This technique can be used both for domestic
vents further entry of fluoride into the tooth. This is and for community water supplies.
called choking off effect as it prevents further diffu- l For domestic treatment any container of 20–
sion of F-ions. 25 L capacity is suitable.
l CaF2 reacts with hydroxyapatite to form fluoridated l A tap 3–5 cm above the bottom of the con-
hydroxyapatite which increases the concentration of tainer is useful to withdraw treated water but
surface fluoride. is not essential.
502 Quick Review Series for BDS 4th Year, Vol 1

l Adequate amount of lime water and bleaching Acidulated phosphate fluoride:


powder are sprinkled into water first and mixed l Acidulated phosphate fluoride can be used as a

well with it. topical agent as it is found that as the pH of the


l Alum solution is then poured and the water is NaF solution was lowered, fluoride was absorbed
stirred for 10 min. into enamel more effectively.
l The contents are settled for 1 h and the clear l A semiannual application of 1.23% APF for 4 min

water is withdrawn either through the tap or is helpful in reducing caries by 28%.
decanted slowly without disturbing the sedi- Advantages:
ment. l Has acceptable taste

Advantages: l No staining

l Simplicity in design, construction and mainte- l No gingival irritation

nance l Stable with long shelf life

l Adaptable to domestic usage l Economical

l Readily available chemicals are used Disadvantages:


l No regeneration of media l Teeth have to be kept wet for 4 min.

l No handling of caustic acids and alkalies l Solution is acidic.

l Little wastage of water l It is sour and bitter in taste.

Q.9. Topical fluorides. Q.10. Shoe leather survey.


Ans. Ans.
Topical fluoride is the most effective of anticaries agents.
Topical fluorides can be divided into:
{SN Q.11}
A. Professionally applied:
l Neutral sodium fluoride (2%) l Dr H. Trendley Dean was appointed to find the extent
l Stannous fluoride (8%) and geographical distribution of mottled enamel in
l Acidulated phosphate fluoride (1.23%) the USA.
l Amine fluoride l Shoe leather survey is a study of relationship be-

l Fluoride gels tween fluoride concentration in drinking water, mot-


l Fluoride varnishes tled enamel and dental caries.
B. Self-applied fluorides: l Dr H. Trendley Dean pursued full time research on

l Toothbrushing dentifrices mottled enamel. He continued Mckay’s work to find


l Toothbrushing solutions or gels the extent and geographical distribution of mottled
l Toothbrushing prophylaxis pastes enamel in the USA.
l Applying gels in trays l His aim was to find out the minimal threshold of

l Mouth rinses fluoride – the level at which fluorine began to blem-


Commonly used agents are described below: ish the teeth.
Sodium fluoride: l He showed conclusively that the severity of mottling

Knutson and Feldman recommended a technique of increased with increasing fluoride concentrations in
four applications of 2% NaF at weekly intervals in a the drinking water.
year at 3, 7, 11 and 13 years.
Properties of sodium fluoride:
l Neutral pH. He gave the following observations according to water
l 9200 ppm of F2. concentration as below:
l Caries reduction in 1st year was 45% and in When water concentration was:
2nd year was 36%. i. 4 ppm or more – signs of discrete pitting
Advantages: ii. 3 ppm or more – mottling was widespread
l Chemically stable iii. 2–3 ppm – teeth had dull chalky appearance
l Acceptable taste iv. 1 ppm or less – no mottling of any aesthetic signifi-
l Nonirritating to gingival tissues cance
l Does not cause any discolouration of the teeth v. He also reported that the incidence of caries in these
l Economical teeth was less as compared to nonfluoridated teeth
Disadvantages: l With the excitement of the early results, water fluo-

l Continuous application for 4 min. ridation started in the USA.


l Patient has to make four visits in a short time. l It was then believed that the addition of 0.1 ppm

l Follow-up is difficult. fluoride to the drinking water would result in a


Section | I  Topic-Wise Solved Questions of Previous Years 503

maximal reduction in the permanent tooth caries Q.12. Endemic fluorosis.


experience of 6–8 year old children.
Ans.
l With similar conditions, a significant, but not a

complete reduction in the permanent tooth dental In many parts of the world where drinking water contains
caries experience had been achieved in 12–14 excessive amounts of fluorine, endemic fluorosis has been
years old. observed.
Endemic fluorosis results in:
Q.11. Mechanism of action of fluoride in preventing
(a)   Dental fluorosis:
dental caries.
l It can be defined as hypoplasia or hypomatura-

Ans. tion of tooth enamel or dentin produced by the


chronic ingestion of excessive amounts of fluo-
Mechanism of action of fluoride:
ride during the period when teeth are developing.
i. Improved crystallinity:
l There is direct inhibitory effect on enzymatic
Fluoride increases the crystal size and produces less
action of ameloblasts leading to defective matrix
strain in crystal lattice. This takes place through con-
formation and subsequent hypomineralization.
version of amorphous calcium phosphate into crystal-
l Tooth absorbs stains from oral cavity due to its
line hydroxyphosphate.
porous structure. This discolouration can be re-
ii. Void theory:
duced to certain degree by bleaching the teeth
Fluoride fills the voids in the hydroxyapatite crystal
externally.
and attains a stable form with formation of more and
l Major cause is water consumption containing
stronger hydrogen bonds. Greater stability will lead to
high levels of fluoride during the first 6 years of
lower solubility and hence greater resistance to dis-
life.
solution in acids.
l Both primary and permanent teeth will be af-
iii. Acid solubility:
fected, but greater fluorosis in permanent teeth is
Fluorapatite or fluoridated hydroxyapatite is less
seen because, much of the mineralization of
soluble than hydroxyapatite therefore has greater
primary teeth occurs before birth.
stability.
(b)  Skeletal fluorosis:
iv. Enzyme inhibition:
l There is heavy deposition of fluoride in the skel-
Fluoride forms a complex with enolase enzyme to
eton. When a concentration of 10 mg/L is ex-
inhibit glucose transport. It also binds with phospha-
ceeded, crippling fluorosis can ensue leading to
tases to reduce acid production.
permanent disability.
v. Suppressing the flora:
(c)   Genu valgum:
Stannous fluoride oxidizes the thiol group present in
l A new form of fluorosis characterized by genu
bacteria thus inhibiting bacterial metabolism.
valgum and osteoporosis of lower extremities.
vi. Antibacterial action:
The concentration of fluoride above 2 ppm in solution Q.13. Give differences between sodium fluoride and
progressively decreases the transport of uptake of stannous fluoride.
glucose into cells of oral streptococci and also reduces
Ans.
ATP synthesis.
vii. Lowering free surface energy: l Although sodium fluoride and stannous fluoride are
Fluoride substitutes the hydroxyl ions to reduce both used for topical application, there are some basic
the free surface energy and thus indirectly reduces differences in their preparation, application and other
the deposition of pellicle and subsequent plaque properties, they are as follows:
formation.
viii. Desorption of protein and bacteria: Sodium fluoride Stannous fluoride
Hydroxyapatite crystals have both positive and nega- It has a neutral pH of 7. It has an acidic pH of 2.1–2.3.
tive receptor sites. Acidic protein group binds to cal- Available fluoride in ppm is Available fluoride in ppm is
cium site and basic to phosphate site. Fluoride inhibits 9040. 1930 pmm.
the binding of acidic protein to hydroxyapatite thereby Solution is made by dissolving It is made by mixing 0.8 g
displaying its beneficial effects. 20 g (2%) NaF powder in 1 L SnF2 powder in 10 mL dis-
ix. Alteration in tooth morphology: of distilled water. tilled water (8%).
Dentition in fluoridated communities showed a ten- It can be stored and kept in It is always freshly prepared
dency towards rounded cusps, shallow fissures due to plastic bottles. before each use, as the solu-
selective inhibition of ameloblasts. tion is unstable.
504 Quick Review Series for BDS 4th Year, Vol 1

Sodium fluoride Stannous fluoride Acute fluoride toxicity:


l Ingestion of large doses of fluoride at one time.
Four applications are required Number of applications
l Probably toxic dose (PTD) is defined as the threshold
each at the ages of 3, 7, 11 required is once per year.
and 13 years. dose that could cause serious or life-threatening sys-
temic signs and symptoms.
When NaF is applied topically, SnF2 reacts with hydroxyapa-
it reacts with hydroxyapatite tite in addition to fluoride and
Safely tolerated dose: 8–16 mg/kg body wt.
crystals to form CaF2. forms a new crystalline Toxic dose: 16–32 mg/kg body wt.
product – stannous trifluoro- Lethal dose: 32–64 mg/kg body wt.
phosphate that is more resis- Factors affecting acute toxicity:
tant to decay than enamel. l Bioavailability

Advantages: Advantages: l Route of administration


l Chemically stable. l Rapid penetration of tin l Age
l Acceptable taste. and fluoride within 30 s. l Rate of absorption
l Nonirritating to gingival l The tin–trifluorophosphate
l Acid–base status
tissues. complex formed on the
l Does not discolour the enamel surface is more Signs and symptoms:
teeth. resistant to decay than l Nausea and vomiting
l Economical. enamel itself. l Abdominal pain and diarrhoea
Disadvantages: Disadvantages: l Excess salivation and mucosal discharge
l Continuous application for l Should be prepared freshly.
l Generalized weakness and carpopedal spasms
4 min. l Metallic taste.

l Patient has to make four l Causes gingival irritation. l Weak thready pulse and fall in blood pressure

visits in a short time. l Produces discolouration of l Depression of respiratory centre


l Follow-up is difficult. teeth. l Decreased plasma calcium level and increased
l Causes staining on margins
plasma potassium level
of restorations.
l Cardiac arrhythmia

l Coma and death


Q.14. What is optimum level of fluoride in water? List
the recommendations for use of fluoridated tooth paste Management:
in children. Immediate:
l Aimed at reducing fluoride absorption
Ans. l Induce vomiting

Optimum level of fluoride: l Fluid replacement

Optimum concentration for fluoride in the water in the l Monitoring levels of plasma calcium and

range of 0.7–1.2 parts per million. This range effec- potassium


tively reduces tooth decay, while minimizing the occur- Less than 5 mg/kg fluoride ingested:
rence of dental fluorosis. l Give milk

Fluoride tooth pastes: l Induce vomiting

Recommended dentifrices by ADA or FDA: More than 5 mg/kg fluoride ingested:


l In Europe: Extra strength Aim, Improved crest, l Give milk

Colgate and Aquafresh. l Induce vomiting

l In India: Stolin-R, Pepsodent, Cibaca, Colgate l 5% calcium gluconate

total and Senquel. l Hospitalization

Recommendations for use of fluoride dentifrice: More than 15 mg/kg fluoride ingested:
l Induce vomiting
Age Recommendation l Hospitalization

Younger than 4 years Not recommended l Cardiac monitoring (peaking of T-wave or pro-
4–6 years Once daily with fluoride paste and longed Q-T interval)
6–10 years twice without paste l Slow administration of 10% calcium gluconate
Older than 10 years Twice daily with fluoride paste and
l Maintain urinary output – supportive measures for
once without paste
Thrice daily with fluoride paste shock
Chronic toxicity:
l Ingestion of variant doses of fluoride over a pro-
Q.15. What is the difference between acute fluoride tox-
longed period of time
icity and chronic fluoride toxicity?
l It is of two types – dental fluorosis and skeletal

Ans. fluorosis
Section | I  Topic-Wise Solved Questions of Previous Years 505

Dental fluorosis: iii. Self-application is possible.


l It can be defined as hypoplasia or hypomaturation iv. It also provides the beneficial properties of fluo-
of tooth enamel or dentin produced by the chronic rides which include remineralization, decreased
ingestion of excessive amounts of fluoride during incidence of caries, shallow pits and fissures.
the period when teeth are developing.
l There is direct inhibitory effect on enzymatic ac-

tion of ameloblasts leading to defective matrix


SHORT NOTES:
formation and subsequent hypomineralization.
Q.1. Milk fluoridation.
l Major cause is water consumption containing

high levels of fluoride during the first 6 years of Ans.


life.
[Ref LE Q.6]
l Both primary and permanent teeth will be af-

fected, but greater fluorosis in permanent teeth is Q.2. Fluoride mouthwash.


seen because, much of the mineralization of pri-
Ans.
mary teeth occurs before birth and also because
the placenta serves as the barrier to the transfer of l It has been found that use of lesser concentration of
high concentrations of plasma fluoride from a fluoride as fluoride mouthwash is more cariostatic than
pregnant mother to her developing fetus. less frequent use of higher concentration of fluoride.
Skeletal fluorosis: l In areas, where water fluoridation is not possible or has

l It is associated with various forms of metabolic not been implemented, the fluoride mouth rinses (0.5%
bone diseases. NaF daily) have been found to be an effective tool in
prevention of dental caries.
Q.16. Thixotropic gel. l This procedure can be carried out by dissolving 200 mg

Ans. NaF tablet in 25 mL of fresh clean water, which is suf-


ficient for daily mouth rinse for a family of about four
l Fluoride gels are also a form of topical application of members.
fluorides. l These rinses are very cost-effective. So those individu-
l Most commonly used compound in form of gels is
als who cannot buy brush and tooth paste are also ben-
acidulated phosphate fluoride. efitted.
l As APF solution is acidic and sour and bitter in taste, so

repeated applications are often difficult. To overcome Q.3. APF gel.


these problems APF gels were introduced. Ans.
Method of preparation:
l APF gel prepared by dissolving 20 g of NaF in
l Fluoride gels are also a form for topical application of
l L of 0.1 M phosphoric acid. fluorides.
l Most commonly used compound in form of gels is
l To this 50% hydrofluoride acid is added to adjust

the pH at 3.0 and F2 concentration at 1.23%. acidulated phosphate fluoride.


l Acidulated phosphate fluoride is commonly used as an
l A gelling agent like methylcellulose or hydroxy-

ethyl cellulose is to be added to the solution and anticaries agent.


l The solution form of APF is bitter in taste and is acidic
the pH is to be adjusted between 4 and 5.
l For the application of gel, patient is positioned
in nature, to overcome this the concept of gels was
upright and saliva ejector is kept in place. developed.
l Enough gel is placed to fill one-third of the trough
Properties of APF gel:
area of tray so that it is sufficient to cover dental i. Relatively costly.
arches. ii. Readily available (imported in India).
l Loaded tray is placed over the arch and buccal
iii. Self-application is possible.
and lingual surfaces are squeezed forcing gel iv. APF gel also provides the beneficial properties
between them. of fluorides which include remineralization, de-
l Tray is allowed to remain in mouth for 4 min.
creased incidence of caries, shallow pits and
l Patient is instructed to expectorate immediately
fissures.
and avoid drinking and eating for the next 30 min. Q.4. Fluoride varnish.
Properties of APF gel:
Ans.
i. Relatively costly
ii. Readily available (imported in India) [Ref LE Q.5]
506 Quick Review Series for BDS 4th Year, Vol 1

Q.5. Fluoride tablets. l Water concentration was 2–3 ppm – teeth had dull
chalky appearance.
Ans.
l Water concentration was 1 ppm or less – no mot-

l Fluoride tablets provide systemic effect before mineral- tling of any aesthetic significance.
ization of primary and permanent teeth and a topical He also reported that the incidence of caries in
effect thereafter these teeth was less as compared to nonfluori-
l Caries reduction in the range of 50%–80% has been dated teeth.
reported, when fluoride administration in the form of
Q.8. Choking phenomenon.
tablets was started before 2 years of age and continued
for a minimum of 3–4 years. Ans.
Availability:
l Choking phenomenon occurs during application of
l Fluoride tablets are commercially available as NaF
sodium fluoride.
tablets of 2.2 mg, 1.1 mg and 0.55 mg yielding 1 mg,
It can be explained as follows:
0.5 mg and 0.25 mg fluoride, respectively.
l When NaF is applied topically, it reacts with
l Sodium fluoride tablets with vitamin combinations
hydroxyapatite crystals to form CaF2.
are also available.
l Due to high concentration of fluoride there is ini-
Recommended dose:
tial formation of CaF2 on the tooth surface which
l The daily recommended dose of fluoride for child
prevents further entry of fluoride into the tooth.
younger than 2 years is 0.5 mg, between 2 and 3years
This is called choking off effect as it blocks the
is 0.5–0.7 mg and older than 3 years is 1–1.5 mg.
diffusion.
Swish and swallow technique:
l For best topical effect, fluoride tablets should be first Q.9. Advantages and disadvantages of sodium
chewed and then swallowed. fluoride.
l The use of tablets is logical that if a child chew the
Ans.
tablets, then swish the saliva between the teeth for a
minute before swallowing, both topical and systemic [Ref LE Q.1]
dosages of fluoride would be achieved. This method Q.10. School water fluoridation.
is advocated whenever tablets are used.
Advantages: Ans.
l Ready for use. [Ref LE Q.4]
l Requires little time to dispense.

l Some tablets have a flavour that enhances child mo-


Q.11. Shoe leather survey.
tivation to participate in the daily ingestion of fluo- Ans.
ride tablets.
[Ref SE Q.10]
Q.6. Brudevold’s solution.
Q.12. Fluoride dentifrice.
Ans.
Ans.
[Ref SE Q.4]
l Fluoride dentifrices have been proven to be effective
Q.7. Dean’s index of fluorosis. anticaries agents.
l The most commonly used fluoride dentifrices are
Ans.
sodium fluoride and stannous fluoride, sodium mono-
l Dr H. Trendley Dean developed an index of fluorosis fluorophosphate and amine fluoride.
based on relationship between fluoride concentration in Sodium fluoride and stannous fluoride dentifrices:
drinking water, mottled enamel and dental caries. l NaF was the first fluoride compound to be used but

l Index is based on the level at which fluorine begins to had limited efficacy.
blemish the teeth. l Later SnF2 was also added. However, this leads to

The index is as follows: staining of restorations and had astringent taste.


l Water concentration was 4 ppm or more – signs of Amine fluoride:
discrete pitting. l It has antibacterial and anticariogenic properties,

l Water concentration was 3 ppm or more – mottling superior to inorganic fluorides and shows significant
was widespread. reduction in caries rate.
Section | I  Topic-Wise Solved Questions of Previous Years 507

Sodium monofluorophosphate: Recommendations for use of fluoride dentifrice:


l It is the preferred chemical form of fluoride in most
Age Recommendation
of the fluoridated tooth pastes. Dentifrices containing
Younger than Not recommended
MFP at a concentration of 0.76%, 0.1% F with so- 4 years Once daily with fluoride paste and twice
dium metaphosphate as abrasive have led to variable 4–6 years without paste
reductions in caries rates. 6–10 years Twice daily with fluoride paste and once
Fluoride tooth pastes: Older than without paste
Recommended dentifrices by ADA or FDA. 10 years Thrice daily with fluoride paste
l Europe – Extra strength Aim, Improved crest,

Colgate and Aquafresh. Q.13. Fluoride gels.


l India – Stolin-R, Pepsodent, Cibaca, Colgate total
Ans.
and Senquel.
[Same as SN Q.3]

Topic 21
Paediatric Restorative Materials
and Rubber Dam Application
COMMONLY ASKED QUESTIONS
LONG ESSAYS:
1 . Write briefly about rubber dam application and its advantages and disadvantages.
2. Describe aesthetically acceptable anterior crowns used in pedodontics.
3. Classify glass ionomer cements. Discuss their role in paediatric dentistry.

SHORT ESSAYS:
1. Indications and contraindications of stainless steel crowns.
2. High copper alloys.
3. Modifications of glass ionomer cement.
4. Bonding agents.
5. Zinc phosphate cement.
6. Glass ionomer cements classification, uses and properties.
7. Anterior aesthetic crowns.
8. Describe briefly about isolation.
9. Advantages and disadvantages of rubber dam in pedodontics.
10. Polycarboxylate cement.
11. Silicate cement.
12. EBA cement.
13. Clinical steps for glass ionomer restoration.
14. Dentine-bonding agents. [Same as SE Q.4]

SHORT NOTES:
1 . Cavity varnish.
2. Preformed crown.
3. Calcium hydroxide.
4. Indications and contraindications of stainless steel crown.
5. Polycarbonate crowns. [Ref LE Q.2]
6. Name glass ionomer cements used in pedodontics.
508 Quick Review Series for BDS 4th Year, Vol 1

7. g2 Phase of amalgam.
8. Mention four advantages of rubber dam applications. [Ref SE Q.9]
9. Indications of glass ionomer cement.
10. Mechanical properties of glass ionomer cement.
11. Advantages of GIC.
12. Compomer.

SOLVED ANSWERS
LONG ESSAYS:
Q.1. Write briefly about rubber dam application and its Slit-dam method:
advantages and disadvantages. l When a quadrant of restorations in the primary

dentition is planned and no pulp therapy is antici-


Ans.
pated, Croll recommends the ‘slit-dam method’.
Various types of isolation are as follows: l One long opening is made in the dam and the
i. Rubber dam isolation entire quadrant is isolated without interseptal dam
ii. Cotton roll isolation and cellulose wafers material between the teeth
iii. Throat shields Rubber dam application:
iv. High volume evacuators and saliva ejectors Preparation of child:
v. Retraction cord l Euphemisms used for preparation of child for rub-
vi. Mirror and evacuator tip retraction ber dam placement are
vii. Mouth props Raincoat – rubber dam sheet
In 1864, S.C. Barnum, a New York dentist, introduced Hanger – frame
rubber dam to dentistry. Clip – clamp
Advantages of using rubber dam are as follows: Armamentaria required for application of rubber dam
i. It helps in moisture control and maintains dry clean are as follows:
operating field thereby improving properties of i. Rubber dam sheets
dental materials. ii. Retainers or clamps
ii. It improves accessibility and visibility with a good iii. Rubber dam retaining forceps
operating efficiency. iv. Rubber dam punch
iii. It helps in retraction of soft tissues and protects v. Rubber dam frame
patient by reducing chances of soft tissue injuries. vi. Rubber dam napkin
iv. It saves operators time. vii. Template
v. It prevents aspiration or swallowing of small in- viii. Lubricant
struments and restorative materials. ix. Dental floss
vi. It maintains aseptic environment and helps in x. Scissors
effective infection control. i. Rubber dam sheets:
vii. It facilitates quadrant restorative procedures. l They are available in 5˝ 3 5˝ or 6˝ 3 6˝ sizes
viii. It helps in minimization of mouth breathing. l Available thickness are
ix. It helps the dentist to educate the parents when l Thin: 0.15 mm
treating children by showing what was done to l Medium: 0.20 mm
their kid. l Heavy: 0.25 mm
Disadvantages of using rubber dam are as follows: l Extra heavy: 030 mm
i. Due to psychological intolerance and latex allergy, l Special heavy: 0.35 mm
patients may show objection to use of rubber dam. l It is available in green, blue, black, pink and
ii. Build-up of saliva. burgundy colours and mint, banana and
iii. Poor retention of clamps on partially erupted tooth strawberry flavours.
as they cannot receive a retainer. l The rubber dam sheet has a darker side and a
iv. Frame can cause pressure marks on face. shiny side. The shiny side should always be
v. Time-consuming. towards the tissue so that the dam can pass
Types of rubber dam: easily over them with minimal irritation,
Quick dam or insta-dam: whereas the dull side should be towards the
l These are new types of rubber dams that have occlusal aspect so that it does not reflect the
preattached frame for the ease of application. light from it to obstruct vision of operator.
Section | I  Topic-Wise Solved Questions of Previous Years 509

ii. Retainers or clamps: iv. Placement of clamp:


l The clamp or retainer has 4 prongs and 2 jaws l Place the clamp on the tooth with the help of
that are connected by a bow. retainer forceps and check for stability.
l Its use is to anchor the most posterior tooth to v. Lubrication and placement of rubber dam sheet:
be isolated and also to retract gingival tissue. l Now lubricate the punched hole in the sheet
l They are available in various types and sizes and also apply lubricant like petroleum jelly on
for each individual tooth. the gingival tissues and lips of the patient.
iii. Rubber dam retaining forceps: l The sheet is stretched with the index finger of
l It is used for placement and removal of both the hands spreading the hole and placed
clamps. over the clamp and other teeth which need to
iv. Rubber dam punch: be exposed.
l It is a precision instrument having a rotating l Apply the Young’s frame and stretch the dam
metal table with six holes of varying sizes and on to it. A wedge is placed between the contacts
a tapered, sharp, pointed plunger. of the most anteriorly isolated tooth and cut if
l The largest hole being for molars and the there is any excess in nasal area.
smallest for mandibular incisors. Steps for removal of rubber dam:
v. Rubber dam frame: l The first step in removing the rubber dam is removal
l It is of two types – metallic (Young’s frame) of wedges.
and plastic (Nygaard Ostby frame). l With forceps, the clamp is removed with the frame as
l It holds and positions the border of rubber a unit.
dam. Complications:
vi. Rubber dam napkin: i. Trauma to lips and gingiva due to poor clamp
l It is placed between rubber dam and patient’s selection
skin. ii. Loss of springiness of clamp may lead to loss of
l It prevents allergy, acts as a cushion and pre- retention and worn out clamps can fracture dur-
vents pressure marks on patient’s cheeks and ing treatment.
also it is the convenient method for wiping the iii. Pressure marks on face.
patient’s lips on removal of dam. iv. High dam can block nasal passage.
vii. Lubricant:
Q.2. Describe aesthetically acceptable anterior crowns
l Commonly used lubricants are soap solution,
used in pedodontics.
petroleum jelly and cocoa butter.
l Facilitates passing of dam through posterior Ans.
contacts and also help the dam to pass over
l The use of anterior crowns in case of severely decayed
clamps. It is also applied over patient’s tissues
primary teeth in anterior region is more cost-effective
to prevent injury and dryness.
and a viable option.
viii. Dental floss:
l These can either be polycarbonate crowns or strip
l Used to secure the rubber dam
crowns or stainless steel crowns with composite facing
ix Rubber dam template:
or the newest material called Artglass crowns.
l To check for exact placement of rubber dam.

x. Procedure for placement of rubber dam:


i. Administration of local anaesthesia:
l Local anaesthesia is administered first.

ii. Selection of clamp: {SN Q.5}


l Appropriate clamp should be selected, un-
i. Polycarbonate crowns:
less the clamp is firmly anchored to the l Polycarbonates are aromatic linear polyesters of
tooth, the tension of the stretched rubber carbonic acid. They exhibit high impact strength
will easily dislodge it. and rigidity.
iii. Selection of rubber dam sheet: l They are termed as thermoplastic resins since they
l Usually 5 3 5 inch sheets of medium latex
are moulded as solids by heat and pressure into
is kept on a template, where the appropriate the desired form.
holes are marked and punched with a rub- Advantages:
ber dam punch. l They have good dimensional stability.
l An 18-inch long dental floss is secured on
l They are unaffected by dilute mineral acids,
the clamp by wrapping it all around the bow ether and alcohol.
and passing it from both the holes in wings.
510 Quick Review Series for BDS 4th Year, Vol 1

l The crown is then lined with acrylic or composite


Disadvantage:
material. Lining a polycarbonate crown will en-
l Poor abrasion resistance.
sure good marginal adaptation to the preparation.
Indications:
Cold-cure acrylics chemically bond with poly-
l Full coverage restoration of primary maxillary
carbonate crowns. Composite materials need
anterior teeth with extensive caries.
some retention, by mechanically roughening the
l Early childhood caries.
inside crown surface. A chemical bond to com-
l In case of teeth with structural deformities and
posite can be obtained by priming the fitting
discolouration.
surface of the polycarbonate with methyl methac-
Contraindications:
rylate liquid.
l Deep bite
l After the lining material has set, the crown is re-
l Bruxism
moved from the tooth and the margins carefully
l High functionality of teeth
trimmed and finished it is important that an ac-
curate fit is obtained at the preparation margin to
Technique: maintain gingival health.
l After checking the fit and occlusion, the polycar-
l Crown is selected according to the mesiodistal di-

mension of tooth to be restored. bonate crown should be cemented using propri-


l Using a tapering diamond bur the proximal aspects
etary temporary luting cement and the excess
are reduced until contacts are open and surface is removed.
parallel. iii. Stainless steel crowns with composite facing:
l These are indicated in maxillary canines where
l Preparation of finish line (according to Stewart a

chamfer finish line should be created, whereas Meyers strength is a major requirement as compared to aes-
advocated no finish line). thetics.
l The labial portion of anterior stainless steel crown is
l Labial and lingual reduction should not be more than

0.5 mm as this is sufficient to allow for crown form. removed and composite is placed as a facing thereby
l Incisal reduction of 1–2 mm.
providing adequate strength and acceptable aesthetics.
l The disadvantage of these crowns is that they are not
l Selected crown is adapted by selective grinding of

gingival portion and internal margin and then it is easily removed.


cemented. Technique:
l Select an appropriate crown.
ii. Modified polycarbonate crowns:
l Start with proximal reduction creating a chamfer
(3M ESPE polycarbonate prefabricated crowns)
l The crowns are made of a polycarbonate resin
finish line.
l Minimal palatal and buccal reduction has to be
incorporating micro-glass fibres which permit
crown adjustment with pliers and also give these done as these provide retentive undercuts.
l Check for fitness and reduce the crown wherever
crowns good durability and strength.
l Have good anatomic form and aesthetics.
needed by contouring and crimping with pliers.
l Finally cementation of crown is done.
l They save operators time, as they are easy to trim

with dental burs or crown scissors, and can be iv. Strip crowns:
l These are celluloid crown forms that are the most
easily adjusted with pliers.
l Provides good durability, strength as well as pro-
effective for use in paediatric patients with extensive
tection to the prepared crown margins. caries in anterior teeth.
l The advantage is that, they are transparent so appro-
l They have good anatomic form and aesthetics.

l Smooth surface finish for patient comfort and to


priate shade of composite can be selected.
l Easy to place and remove, and hence are less time-
help minimize plaque build-up.
l They are manufactured in a universal shade which
consuming.
is translucent enough to allow shade adjustment Technique:
l Isolate and anesthetize the teeth.
as per the type of lining material used.
l Size of celluloid crowns should be selected by
Technique:
l Crown is selected by measuring the mesiodistal
measuring mesiodistal dimension of the space
width at the level of the contact point of the pre- available.
l Caries is removed using a small round bur and
pared tooth, or by measuring the width of the
contralateral tooth in the same arch. tooth preparation requires only proximal slicing.
l Select the shade of composite.
l Cervical crown margin is trimmed to the required
l Celluloid crowns are trimmed using curved scis-
contour with crown scissors or by grinding with a
trimming bur or stone. sors, then length and cervical fit should be checked.
Section | I  Topic-Wise Solved Questions of Previous Years 511

l Vent holes are prepared in the crown for escape of . Low viscosity GIC
d
air and excess material. e. Base/liner
l Acid etching of the tooth should be done. f. Luting
l Composite-filled celluloid crown should be placed B. Resin-modified GIC
on to tooth and excess material is removed and a. Restorative
then cured. b. Base/liner
l Excavator or probe is used to insert beneath the c. Pit and fissure sealant
crown and crown form is stripped off. d. Luting
l Final adjustment includes smoothening and polishing. e. Orthodonic cementation material
v. Artglass crowns (Kulzer): C. Polyoid-modified resin composites/compomers
l These are the only patented preformed crowns for III. According to Sturdvent:
paediatric usage. i. Traditional or conventional
l These crowns provide the natural feel, bondability and ii. Metal-modified GIC
kindness associated with composites, along with the a. Ceremets
aesthetics and longevity associated with porcelain. b. Miracle mix
l Artglass contains multifunctional methacrylate, which c. Light-cured GIC
has the ability to form three-dimensional molecular d. Hybrid (resin-modified GIC)
networks with a highly cross-linked structure. e. Polyacid-modified resin composites or compomer
l High inorganic filler makes Artglass colour stable IV. According to Wilson and McLean (1998):
and plaque resistant. The total filler content of Art- a. Type I luting
glass is only 75% but when the matrix is cured, the b. Type II
amorphous, highly cross-linked organic glass forms, a. Aesthetic filling material
which we call Polymer glass which is one of the b. Bis-reinforced filling material (indudes ceremets)
toughest materials available to dentistry. c. Type III – lining base and fissure sealant
l Wear of Artglass is similar to enamel and kind to V. Based on chemical constituents of cement:
opposing dentition. a. Conventional
l Matched to the Vita shade system simplifies shade b. Metal reinforced
selection. i. Miracle mix
l Flexural strength over 50% higher than porcelain, ii. Ceremets
less chance of fracture. c. Resin modified
l Easily adjusted or repaired intraorally, less chair time
VI. According to intended applications:
for dentists. a. Type I – luting
Q.3. Classify glass ionomer cements. Discuss their role b. Type II – restorative
in paediatric dentistry. c. Type III – fast setting lining
d. Type IV – fissure sealants
Ans. e. Type V – orthodontic cements
l Glass ionomer cements were developed by A.D. Wilson f. Type VI – core build-up material
and B.E. Kent in the laboratory of the Government g. Type VII – high fluoride releasing command set GIC
chemist, London in 1972. h. Type VIII – gIC for atraumatic restorative treat-
l 1994 – Resin-glass ionomer hybrids officially named as ment (ART)
‘Resin-Modified Glass Ionomer Cements’ at the Inter- i. Type IX – geriatric and paediatric GIC
national Symposium on Glass Ionomer Cements. VII. According to McLean, Nicholson and Wilson (1994):
l 1995 to Present – Introduction of compomers and pack- a. Glass ionomer cement
able glass ionomers.   i. Glass polyalkeonates
ii. Glass polyphosphonates
Classification of glass ionomer cements is as follows:
b. Resin-modified GIC
I. According to Philips:
c. Polyacid-modified GIC
a. Type I – luting
Role of glass ionomer cements in paediatric dentistry:
b. Type II – restorative
Type I: Luting and lining cement
c. Type III – liner and base
Shade: Light yellow.
II. According to Davidson and Major:
Indications:
A. Conventional/traditional:
l Cementation of stainless steel crowns
a. Glass ionomer for direct restorations
l Cementation of high strength all ceramic
b. Metal reinforced GIC
crowns, jacket crowns and bridges
c. High viscosity GIC
512 Quick Review Series for BDS 4th Year, Vol 1

Contraindications: l Transfer cement to the preparation using a sy-


l Pulp capping. ringe or suitable placement instrument. Avoid
l In some rare cases, where the product may air bubbles. Form the contour, and place a ma-
cause sensitivity in some people. trix, if required during the first 2 min of setting.
Manipulation: Setting time is 5 min 30 s after start of mixing.
l After tooth preparation, the prepared tooth Once set, remove the matrix and immediately
should be cleaned with pumice and water. Ex- apply varnish.
cess moisture should be removed by blotting l For one visit treatment, after initial set, correct
with a cotton pellet or gently blowing air. the contour without water spray and apply var-
l Prepared surfaces should appear moist and nish. Wait for 15 min and finishing can be car-
glistening. ried out under water spray. Apply a final coat
l The powder and liquid should be dispensed in of varnish.
a standard ratio 1.8 g/1.0 g (1 level scoop of Type VII protection and stabilization material:
powder to 2 drops of liquid) on the mixing pad. l It is available in pink and white shades.
l Using the plastic spatula, all the powder should l Pink shade is command set and white shade is
be added to the liquid. Mix rapidly for 20 s. chemical set only.
When mixing large amounts, divide the powder Indications:
into two equal parts. Mix the first portion with l Fissure protection
all the liquid for 5 s. Incorporate the remaining l Root surface protection
portion and mix the whole thoroughly for 15 s. l Hypersensitivity prevention and control
l Then coat the internal surface of the restoration l Intermediate restorations and intermediate
with sufficient cement and seat immediately. endodontic sealing
l The working time is 2 min from start of mixing Contraindications:
at 23°C. Higher temperatures shorten the l Pulp capping.
working time. Maintain moderate pressure and l In rare cases, the product may cause sensitivity
remove the excess cement when the excess ce- in some people.
ment feels rubbery. Manipulation:
l Finishing can be started 4 min 30 s after seat- l After cleaning the tooth surface in usual man-
ing the restoration. ner, rinse thoroughly with water. Dry by blot-
Type II: Restorative cement: ting with a cotton pellet or gently blowing with
Available shades: pale yellow, yellow brown, dark an air syringe.
grey and deep brown l The standard powder to liquid ratio is 1.8 g:1.9 g
Indications: (1 level pink scoop of powder to 1 drop of
l Restoration of primary teeth liquid). To extend working time, the powder to
l Core build-up liquid ratio is 1.1 g:1.0 g (1 level yellow scoop
l Restorations of class III, V and limited class I of powder to 1 drop of liquid).
cavities l Using the plastic spatula, divide the powder
Manipulation: into two equal parts. Spread the liquid over the
l After tooth preparation, the prepared tooth pad and mix the first portion with all the liquid
should be washed and dried but do not desic- for 10 s. Incorporate the remaining portion and
cate. Using dentine conditioner, remove the mix the whole amount thoroughly for 10–15 s
smear layer. (total time within 25 s).
l The powder and liquid should be dispensed in l Working time is 1 min 40 s from the start of
a standard ratio 2.7 g/1.0 g (1 level scoop of mixing at 23°C. Take the mixed material using
powder to 1 drop of liquid) on the mixing pad. a suitable placement instrument or brush and
l Using the plastic spatula, divide the powder apply to the tooth surface. Then use a brush to
into two equal parts. Mix the first portion with spread over the pits and fissures. Then protect
all of the liquid for 10 s. Incorporate the re- the surface with varnish or cocoa butter.
maining portion and mix the whole thoroughly Type IX High strength posterior restorative cement:
for 15–20 s (total time within 30 s). Mix the It is available in various shades according to vita
required amount of cement. shade guide: A2, A3, A3.5, B2, B3, C4.
l The working time is 2 min from start of mixing Indications:
at 23°C. Higher temperatures shorten working l Class I and class II restorations in decidu-
time. ous teeth
Section | I  Topic-Wise Solved Questions of Previous Years 513

l Nonload–bearing class I and class II resto- Admixed alloy powder:


rations in permanent teeth i. In admixed system, small spherical particles of a silver–
l Intermediate restorative and base material copper alloy were added to filings of a conventional
for heavy stress situation in class I and class silver–tin alloy.
II cavities using sandwich laminate tech-
nique Composition of admixed alloy powder is
l Class V and root surface restorations
Silver: 69%
l Core build-up
Tin: 17%
Manipulation: Copper: 13%
l The clinical procedure for type IX glass
Zinc: 1%
ionomer cement is the same like type II i. The total copper content ranges from 9 to 20 weight%.
glass ionomer cement. Except for the mix- In admixed alloy powders which usually contain 30–55
ing and working times weight% is spherical high-copper powder.
ii. Setting reaction:
l Silver enters the mercury from the silver copper eu-
SHORT ESSAYS: tectic alloy particles, and both silver and tin enter the
mercury from the silver–tin alloy particles. The mer-
Q.1. Indications and contraindications of stainless steel cury dissolved in the silver–tin particles will react
crowns. like low copper alloys and will form the gl and g2
Ans. phases, leaving some silver–tin particles unreacted.
l The newly formed g2 phase (Sn8Hg) will react with
Indications of stainless crowns: silver–copper particles forming Cu6Sn5 (h or eta)
l Teeth involved with extensive and recurrent caries, phase. Some g1 phase (Ag2Hg3) will also form
rampant caries around the silver–copper particles.
l Intermediate restoration l The reaction may be shown as follows:
l After pulp therapy

l Fractures of primary and permanent incisors Ag3Sn + Ag-Cu + Hg → Ag2Hg3 + Sn8Hg + Ag3Sn unreacted + Ag-Cu unreacted
l Severe bruxism (γ) (eutectic) (γl) (γ2) (γ) (eutectic)
l Inherited or acquired enamel defects and later,
l Abutment teeth to prosthesis

l As part of a space maintainer Sn6Hg + Ag-Cu → Cu6Sn5 + Ag2Hg3


(γ2) (eutectic) (η) (γ1)
Contraindications of stainless steel crowns:
l In anterior teeth where aesthetics is of more concern In the above reaction, g2 has been eliminated and is
l

l Patients allergic to nickel replaced by h phase. To accomplish this, it is neces-


l Imminent exfoliating primary teeth sary to have a net copper content of at least 12% in
the alloy powder.
Q.2. High copper alloys. l They are preferred because of their improved me-

Ans. chanical properties, resistance to corrosion, and bet-


ter marginal integrity.
l The original dental amalgam alloys were alloys of silver Single-composition alloys:
and tin with a maximum of 6% copper. l In single-composition alloy, each particle of the alloy
Alloys that contain enough copper to eliminate the for- powder has the same composition. Therefore, they
mation of the tin–mercury phase (11%–30%) are called are called single-composition or ‘unicompositional’
high-copper amalgam alloys. alloys, e.g. silver, copper, and tin.
l Amalgams made from high-copper alloys have low
Composition:
creep. This property has been associated with the mar- Silver: 40%–60%
ginal breakdown (ditching) commonly noted with amal- Tin: 22%–30%
gam restoration. Copper : 13%–30%
l Although the ADA specification for dental amalgam
Zinc: 0%–4%
permits a maximum of 3% creep, creep of a modern Indium or palladium – small amounts.
high-copper amalgam alloy should not exceed 1%. Setting reaction:
Types of high copper alloys are l When triturated, silver and tin from Ag–Sn phases

a. Admixed alloy powder dissolve in mercury. Very little copper dissolves in


b. Single-composition alloy powder mercury. The Ag2–Hg3 (gl) crystals grow forming a
514 Quick Review Series for BDS 4th Year, Vol 1

matrix that binds together the partially dissolved ii. Cermet ionomer cements (glass sintered with
alloy particles. Later, h (Cu6–Sn5) crystals are formed silver):
at the surface of alloy particles. l This cement was developed by McLean and
l The overall reaction is as below: Gasser as a solution to the problem of improv-
AgSnCu 1 Hg n Cu6Sn5 1 Ag2Hg3 1 AgSnCu ing resistance to abrasion.
(g 1 E)      (h)       (g1) (unreacted) l Cermet ionomer cements have greatly im-
The difference between the elimination of the g2 proved resistance to abrasion when compared
phase in an admixed and unicompositional alloy is with glass ionomer cements and their flexural
that: strength is also higher.
l In the admixed type the g2 forms around the silver– l But their use is confined to low stress-bearing
tin (lathe-cut) particles and is eliminated around the cavity preparations.
silver–copper (spherical) particles. Resin-modified glass ionomer (visible light-cure glass
In unicompositional alloy, the particles at the begin- ionomers, hybrid glass ionomers):
ning of the reaction function like silver–tin particles l To overcome the problem of moisture sensitivity and
of the admixed type, and later the same particles lack of command cure, attempts have made to com-
function like the silver–copper particles of the ad- bine glass ionomers with composite resins.
mixed type, eliminating g2 phase. Composition:
Applications or uses of high-copper alloys: Powder:
i. As a permanent filling material in class I and class l Same powder as chemical-cure glass ionomer
II cavities, and in class V cavities where aesthetics l Photosensitizer
is not important Liquid
ii. To restore a crown, in combination with retentive l Polyacrylic acid copolymer.
pins l Tartaric acid.
iii. For making dies l Methacrylate groups (HEMA).
iv. In retrograde root canal fillings l Photoinitiator.
v. As a core material l Resin modification of glass ionomer cement was

designed to produce favourable physical proper-


Q.3. Modifications of glass ionomer cement. ties similar to those of resin composites while
Ans. maintaining the basic features of the conventional
glass ionomer cement.
The latest modifications of glass ionomer cement are as l In these newer materials the fundamental acid/
follows: base curing reaction is supplemented by a sec-
A. Metal-modified glass ionomer ond curing process, which is initiated by light
B. Resin-modified glass ionomer or chemical.
Metal-reinforced glass ionomer cement: l These products are considered to be dual-cure
Metal powders or fibres are added to reinforce GIC cements if only one polymerization mechanism
which improves strength. is used; if both mechanisms are used, they are
They are of two types: considered to be tri-cure cements.
i. Silver alloy admix (silver amalgam alloy par- l These new materials are called as resin-modi-
ticles mixed with glass particles) fied glass ionomer cements or hybrid iono-
ii. Cermet (glass sintered with silver) mers.
i. Silver alloy admix (silver amalgam alloy mixed l The first commercial RM GICs available were
with glass particles): liners, Vitrebond (3M Dental Pdts) being the
l Addition of metal powders or fibres to glass first introduced.
ionomer cements can improve the strength. Advantages:
Simmons suggested mixing amalgam alloy l More working time
powders with the cements and developed l Command set on application of visible light
this system known as ‘miracle mix’. l Good adaptation and adhesion
l This mix has high strength and resistance to l Acceptable fluoride release
abrasion. l Aesthetics similar to those of composites
l This alloy/glass ionomer mix is used for l Superior strength characteristics
core building and for the treatment of Disadvantages:
mouths with high caries incidence. l Setting shrinkage
l Disadvantage is that, their aesthetics are l Limited depth of cure especially with more
poor and they do not take polish. opaque lining cements
Section | I  Topic-Wise Solved Questions of Previous Years 515

Q.4. Bonding agents. ii. Second generation (early 1980s): Developed as ad-
hesive agents for composite resins which had re-
Ans.
placed acrylic restorations.
l Bonding agents are those materials that help to improve Disadvantage: Adhesion was short term, the bond
the bond between the tooth and the restoration. They are eventually hydrolysed,
the materials of low viscosity, when applied on the tooth For example: Prisma, Universal Bond, Clearfil and
surface forms a thin film after setting. Scotch Bond.
l Bonding agents are used as an adjunct to the acid-etch iii. Third generation (late 1980s): These coupling agents
technique. After acid etching of the enamel, the bonding had bond strengths comparable to that of resin to
agent was applied. etched enamel. However, their use is more complex
and requires two to three application steps.
The bonding agents may be classified according to:
For example: Tenure, Scotch bond 2, Prisma, Uni-
I. The mechanism of bonding:
versal bond and Mirage bond
a. Mechanical bonding
iv. Fourth generation: All bond-2 system consists of
For example: Composite resins, bis-GMA, TEG-
2 primers (NPG-GMA and Biphenyl dimethacrylate
DMA and cyanoacrylates
(BPDM) and an unfilled resin adhesive (40% bis-
b. Chemical bonding
GMA, 30% UDMA and 30% HEMA). This system
For example: Polycarboxylate cements (Zn Poly
bonds composite not only to dentine but to most
Carboxylate), GIC
dental related surfaces like enamel, casting alloys,
II. The application:
amalgam, porcelain and composite.
a. Enamel-bonding agents
v. Fifth generation: The most recent product is the sin-
b. Dentine-bonding agents
gle bond adhesive. Unlike the previous generations,
a. Enamel-bonding agents: this system is more simple to use as it needs only a
i. They were the earliest agents. single-step application, e.g. 3M Single Bond, One
ii. The enamel-bonding agents helped to improve the Step (BISCO), Prime and Bond (Dentsply).
bond by flowing into all the microporosities of the The advantages are single-step application, less tech-
etched enamel. nique sensitive, pleasant odour and higher bond
These materials have been largely replaced by agents strength.
that bond to dentine also. vi. Sixth and Seventh generation:
l Manufacturers formerly supplied enamel- These are single component system liquids, which re-
bonding agents which consisted of bis-GMA quire single application without washing.
resin matrix material diluted with a low-vis- For example: Organic phosphates, Polyurethanes,
cosity methacrylate monomer. HEMA1 bis-GMA 1 Maleic acid, Polyalkenoites
For example: thin composite resins, TEGDMA, cya- (trade name: VLC).
noacrylates and glass ionomers l The most recently introduced systems mix to-
b. Dentine-bonding agents: gether the acidic primer and resin adhesive
l Dentine-bonding agent is a thin layer of resin be- before they are placed on the tooth surface.
tween conditioned dentine and the resin matrix of a Use of these materials remains extremely tech-
composite. nique sensitive.
For example: several dimethacrylate systems, NPG-
Q.5. Zinc phosphate cement.
GMA, bis-GMA, Polyurethanes, 4-META and ferric
oxalate systems Ans.
l Dentine-bonding agents are either chemically acti-
l Formerly, zinc phosphate cement was the most widely
vated or light cured. The resulting bond to tooth
used luting agent.
structure was strictly mechanical.
l It is composed essentially of phosphoric acid liquid that
Dentine-bonding agents are classified as follows:
is mixed with zinc oxide powder.
i. First generation (developed before 1980s): They
l The cement has excellent handling characteristics such
used glycerophosphoric acid dimethycrylate to
as setting time, fluidity and film thickness.
provide a bifunctional molecule. The hydrophilic
l This cement has a long history of successful application
phosphate part reacted with calcium ions of the
for permanent cementation.
hydroxyapatite. The hydrophobic methacrylate
l It does not have an anticariogenic effect, does not ad-
groups bonded to the acrylic restoration resin.
here to tooth structure and does demonstrate a moderate
The main disadvantage was their low bond
degree of intraoral solubility.
strength.
516 Quick Review Series for BDS 4th Year, Vol 1

l Because of the phosphoric acid liquid, zinc phosphate iii. Adhesion:


cement is an irritant and proper pulp protection is l The glass ionomer bonds chemically to tooth
recommended. structure. The bonding is due to the reaction
Q.6. Glass ionomer cements classification, uses and between the carboxyl groups of the polyacids
properties. and the calcium in the enamel and dentine.
l It adheres well to enamel and dentine. The bond
Ans. to enamel is always higher than that to dentine,
l Glass ionomer cements are adhesive tooth coloured probably due to the greater inorganic content of
anticariogenic restorative materials. enamel and its greater homogeneity.
l It was named glass ionomer, because the powder is iv. Aesthetics:
glass and the setting reaction and adhesive bonding to l Aesthetically they are inferior to silicates and

tooth structure is due to ionic bond. composites.


l Due to its adhesive bonding to tooth structures this l They lack translucency and have a rough surface

cement requires minimal cavity preparation. texture. They may accumulate stain with time.
Classification of GIC: v. Biocompatibility:
a. Type I – for luting l They are relatively biocompatible and the pulpal

b. Type II – for restorations response is mild.


c. Type III – liners and bases vi. Anticariogenic properties:
Various other types of GIC available are l Type II glass ionomers release fluoride in

l Pit and fissure sealants. amounts comparable to silicate cements initially


l Metal-modified GIC – e.g. Miracle mix (silver and continue to do so over an extended period of
alloy admix). time.
Glass cermet cement, i.e. ketac silver. l In addition, due to its adhesive effect they have

l Resin-modified GIC, e.g. compomer. the potential for reducing infiltration of oral flu-
l Fuji VII – world’s first high-fluoride nonresin- ids at the cement-tooth interface, thereby pre-
containing autocure GIC. venting secondary caries.
l Fuji VIII and Fuji IX — new high viscosity Q.7. Anterior aesthetic crowns.
GIC’s (ART – atraumatic restorative mate-
rial). Ans.
Uses: Anterior crowns in paediatric dentistry:
i. Anterior aesthetic restorative material for class i. Stainless steel crowns with composite facing:
III cavities l These are indicated in maxillary canines where
ii. For eroded areas and class V restorations strength is a major requirement as compared to
iii. As a luting agent aesthetics.
iv. As liners and bases l The labial portion of anterior stainless steel crown
v. For core build-up is removed and composite is placed as a facing
vi. To a limited extent as pit and fissure sealants thereby providing adequate strength and acceptable
Properties of GIC are as follows: aesthetics.
i. Mechanical properties: l The disadvantage of these crowns is that they are
a. Compressive strength: (150 Mpa). It is less not easily removed.
than silicate ii. Strip crowns:
b. Tensile strength: (6.6 Mpa). Higher than l These are celluloid crown forms that are the most
silicate effective for use in paediatric patients with exten-
c. Hardness: (49 KHN). Less hard than silicates. sive caries in anterior teeth.
The wear resistance is less when compared to l They are transparent, so shade selection of compos-
composites. ite is advantageous.
d. Fracture toughness: A measure of energy re- l Easy to place and remove.
quired to produce fracture is known as fracture l Less time-consuming.
toughness. iii. Artglass crowns (Kulzer):
ii. Solubility and disintegration: l These are the only patented, preformed crowns for
l The initial solubility is high (0.4%) due to paediatric usage.
leaching of intermediate products. l These crowns provide the natural feel, bondability
l Glass ionomer cements are more resistant to and kindness associated with composites, but the
attack by organic acids. aesthetics and longevity associated with porcelain.
Section | I  Topic-Wise Solved Questions of Previous Years 517

l Artglass contains multifunctional methacrylate, v. Retraction cord


which has the ability to form three-dimensional vi. Mirror and evacuator tip retraction
molecular networks with a highly cross-linked vii. Mouth props
structure. i. Rubber dam isolation:
l Wear of Artglass is similar to enamel and kind to op-
l Rubber dam was introduced by S.C. Barnum in
posing dentition. 1864. It maintains a clean operating field during
l High inorganic filler makes Artglass colour stable
restorative procedures.
and plaque resistant. The advantages of using rubber dam are as follows:
l Matched to the Vita shade system simplifies shade
l It increases the visibility and accessibility to
selection. the dentist.
l Flexural strength over 50% higher than porcelain,
l It maintains isolation of the teeth, provides a
less chance of fracture. dry field and effectively retracts the tongue and
l Easily adjusted or repaired intraorally, less chair time
cheeks away from the field of operation.
for dentists. l It saves time.
l Provides the aesthetics and lasting qualities of porcelain.
l It reduces the chances of injury to soft tissues.
l Offers the ease and bondability of a composite.
ii. Cotton roll isolation and cellulose wafers:
Q.8. Describe briefly about isolation. l Isolation can also be provided by using cotton

rolls and cellulose wafers.


Ans. l When rubber dam application is impractical or

The goals of isolation of operating field are impossible; absorbants are isolation alternatives.
i. Moisture control l Several commercial devices for holding cotton

ii. Retraction and access rolls in position are available. An advantage of


iii. Prevention of harm to adjacent structures cotton roll holders is that they might slightly re-
i. Moisture control: tract the cheeks and tongue from the teeth, which
l The moisture control in the oral cavity is essential in enhances access and visibility.
order to execute operative dentistry properly. l Cellulose wafers may be used to retract the cheek

l Moisture control refers to excluding sulcular fluid, and provide additional absorbency.
saliva and gingival bleeding from the operating l When removing cotton rolls or cellulose wafers, it

field. It also refers to preventing the hand piece may be necessary to moisten them using the air–
spray and restorative debris from being swallowed water syringe to prevent inadvertent removal of
or aspirated by the patient. the epithelium from the cheeks, floor of the mouth
l Use of local anesthetizing agents reduces saliva- or lips.
tion, apparently because the patient is more com- iii. Throat shields:
fortable, less anxious and less sensitive to oral l They are particularly important when treating

stimuli, thus reducing salivary flow. teeth in the maxillary arch. A gauze sponge un-
ii. Retraction and access: folded and spread over the tongue and the poste-
l This provides maximal exposure of the operating rior part of the mouth, is helpful in recovering
site and usually involves maintaining an open small objects dropped accidentally during treat-
mouth and depressing or retracting the gingival tis- ment.
sue, tongue, lips and cheek. iv. High-volume evacuators and saliva ejectors:
iii. Harm prevention: l High-volume evacuators are preferred for suction-

l An axiom taught to every member of health profes- ing water and debris from the mouth because sa-
sion is ‘do no harm’, and an important consider- liva ejector removes water slowly and have little
ation of isolation is preventing the patient from capacity for picking up solids.
being harmed during the operation. l The tip of the saliva ejector should be smooth

Various methods employed for adequate isolation are and made from a nonirritating material.
as follows: l A Svedopter is a helpful device that serves both as

i. Rubber dam isolation saliva ejector and a tongue retractor.


ii. Cotton roll isolation and cellulose wafers v. Retraction cord:
iii. Throat shields l Retraction cord may help restrict excess restor-

iv. High volume evacuators and saliva ejectors ative material from entering the gingival sulcus
518 Quick Review Series for BDS 4th Year, Vol 1

and provide better access for contouring and


It helps the dentist to educate the parents when
l
finishing the restorative material.
treating children by showing what was done to
l The diameter of the cord should be selected such
their kid.
that it is gently inserted into gingival sulcus, pro-
ducing lateral displacement of the free gingiva
without blanching it.
Disadvantages of using rubber dam are as follows:
l The objective is to obtain minimal yet suffi-
l Due to psychological intolerance and latex allergy,
cient lateral displacement of free gingiva and
patients may show objection to use of rubber dam.
not to force it apically. The cord may be
l Build-up of saliva.
moistened with a noncaustic styptic before
l Poor retention of clamps on partially erupted tooth as
insertion, if bleeding of the fragile tissue is
they cannot receive a retainer.
anticipated.
l Frame can cause pressure marks on face.
vi. Mirror and evacuator tip retraction:
l Time-consuming.
l A secondary function of the mirror and evacua-

tion tip is to retract the cheek, lip and tongue. Q.10. Polycarboxylate cement.
vii. Mouth props:
Ans.
l It can be of potential aid for a lengthy

appointment on posterior teeth. l Polycarboxylate cement was the first cement system
l A mouth prop should establish and maintain suit- developed with a potential for adhesion to tooth struc-
able mouth opening, thereby relieving the pa- ture.
tient’s muscles of this task, which often produces Mode of supply:
fatigue and sometimes pain. l The polycarboxylate cement is available as powder
l Mouth props of different designs and materials and liquid in bottles.
are available. l Some manufacturers supply this cement as precapsu-
Drugs: lated powder/liquid system.
l The use of drugs to control salivation is rarely Composition:
indicated. Powder:
l Antisialagogues and local anaesthesia decrease Zinc oxide – basic ingredient
salivary secretions. Magnesium oxide – principle modifier and also aids
in sintering
Q.9. Advantages and disadvantages of rubber dam in Other oxides like bismuth and aluminium – small
pedodontics. amounts
Ans. Stannous fluoride – increases strength, modifies set-
ting time and imparts anticariogenic proprieties
Rubber dam was introduced by S.C. Barnum in 1864. It Liquid:
maintains a clean operating field during restorative proce- l Aqueous solution of polyacrylic acid or copolymer
dures. of acrylic acid with other unsaturated carboxylic
acids, i.e. iticonic, maleic or tricarboxylic acid.
{SN Q.8} Setting time:
l 7–9 min.
The advantages of using rubber dam regularly are
l The setting time can be increased by cooling the
l It increases the visibility and accessibility to the

dentist. glass slab.


l It maintains isolation of the teeth, provides a dry
Properties:
field and effectively retracts the tongue and cheeks i. Mechanical properties:
away from the field of operation. (a)  Compressive strength: Polycarboxylate cement
l It saves time.
is inferior to zinc phosphate cement (55 Mpa –
l It reduces the chances of injury to soft tissues.
8000 Psi).
l It also prevents any aspiration or ingestion of den-
(b)  Tensile strength: Its tensile strength is slightly
tal instruments or materials. higher than that of zinc phosphate cement (6.2
l It protects the patients from any bad taste of the
Mpa – 900 Psi).
l Powder/liquid ratio: Increase in P/L ratio in-
materials used.
creases strength.
Section | I  Topic-Wise Solved Questions of Previous Years 519

l Molecular weight of polyacrylic acid also af- ii. The powder and liquid should be mixed rapidly and
fects the strength. the mix should be completed within 30 s.
l A mix from a low viscosity liquid is weaker When properly prepared, the mix has a glossy ap-
than a high viscosity. pearance and can be extruded into a thin film.
ii. Solubility and disintegration: iii. It is important that minimal time elapsed between
l It tends to absorb water and is slightly more soluble completion of the mix and placement of the cement;
than zinc phosphate cement (0.06%). the mix must not have lost its glossy appearance.
l The marginal dissolution of cement is more when iv. To prolong working time, a chilled mixing slab
used as cementing medium. may be used.
l A reduction in the P/L ratio results in a signifi- v. Although polycarboxylate cement demonstrates
cantly higher solubility and disintegration in the adhesion to tooth structure, it has a relatively low
oral cavity. tensile strength, no significant fluoride release and
iii. Biocompatibility: modest intraoral solubility.
l The pulpal response of polycarboxylate cement is vi. Good practices of tooth preparation should be used
mild. to ensure retention of the restoration.
l The pH of the liquid is 1.0–1.7 and that of freshly

mixed cement is 3.0–4.0. After 24 h, pH of the ce- Q.11. Silicate cement.


ment is 5.0–6.0.
l They are less irritant to the pulp than zinc phos-
Ans.
phate cement, because l Silicate cements are available as powder and liquid.
a. The liquid is rapidly neutralized by the powder. l The powder is a finely ground ceramic that is essentially
The pH of polycarboxylate cement rises more an acid soluble glass.
rapidly than that of zinc phosphate.
The composition of silicate cement is as follows:
b. Penetration of polyacrylic acid into the dentinal
Powder:
tubules is less because of its higher molecular
weight and larger phosphate molecules.
Components Approximate wt% Functions
l Reparative dentine formation is observed with

polycarboxylate. Silica (SiO2) 40% Provides strength


and translucency
iv. Adhesion:
l An outstanding characteristic of zinc polycarboxyl- Alumina with phos- 30% Provides Al, Ca, K
ate cement is that the cement bonds chemically with phoric acid (Al2O3) ions by reacting
the tooth structure due to the ability of the carboxyl Sodium fluoride 23% Acts as a flux, g
group in the polymer molecules to chelate with (NaF) Melting point or
Cryolite (Na3AlF6) Calcium fluoride
calcium in the tooth structure.
(CaF2) firing temperature
v. Optical properties:
They are very opaque due to large quantities of unre- Calcium phosphate 7% Act as modifier
Ca(H2PO4)2 H2O (opacifiers)
acted zinc oxide.
or
vi. Thermal properties: Lime (CaO)
Polycarboxylate cements are good thermal insulators.
Liquid:
Uses:
i. It is primarily used for cementation of restoration and Phosphoric acid 52% Reactor
as thermal insulating base. Aluminium 2% Buffers
ii. It is also used as an intermediate restoration. phosphate
iii. Primarily used for luting permanent restorations. Zinc phosphate 6% Control setting
iv. As bases and liners. or magnesium time
v. Used in orthodontics for cementation of bands. phosphate
vi. Also used as root canal fillings in endodontic. Water 40% Controls pH
Manipulation:
i. The recommended powder/liquid ratio should be used. Manipulation:
The powder is dispensed on a thick, cool, dry glass l Dry field is required during manipulation. Because

slab and divided into two or three large increments. on exposure to oral fluids prior to formation of final
Liquid should be dispensed just before the mix is to reaction products results in increased solubility and a
be made. poor surface.
520 Quick Review Series for BDS 4th Year, Vol 1

l The liquid is dispensed just prior to the mixing, in Q.13. Clinical steps for glass ionomer restoration.
order to preserve the acid–water balance.
Ans.
l Mixing is done with an agate, plastic or cobalt–

chromium spatula. The steel spatulas are contraindi- The clinical steps involved in achieving long-lasting resto-
cated, as they are liable to be abraded by the silicate rations with glass ionomer cement are as follows:
powder leading to discolouration of the mix. i. Isolation
Procedure: ii. Tooth preparation includes:
l Powder/liquid ratio: Approximately around 1.6 g (a)   Cavity preparation
of powder/per 4 mL of liquid. (b)   Prophylaxis
l The powder is dispensed on a thick, cool, dry (c)   Surface conditioning
glass slab and divided into two or three large in- iii. Proper manipulation of the cement
crements. The increments are then rapidly folded iv. Careful finishing and polishing
into the liquid over a small area, in order to pre- v. Protection of the restoration surface
serve the gel structure. i. Isolation:
l Particles of the powder should be properly wetted. l Glass ionomer cements are highly sensitive to mois-

Mix for 1 min. ture contamination during placement, so care should


l The mixed material should have consistency like be taken to isolate the tooth surface properly using
putty. The surface of the mix should have a shiny rubber dam, cotton rolls, retraction cords and saliva
appearance. ejectors.
l The mixed material should be inserted into the ii. Tooth preparation:
cavity in one portion. If small increments are used l The tooth surface should be prepared properly to

complete bonding between the portions will not enhance adhesion of the glass ionomer cement.
occur and the set material will be weaker. This consists of:
l A cellulose acetate strip is held against the setting (a)   Cavity preparation:
material in the cavity. The strip is removed after l This is required while restoring class III or

the material sets. Gross excess cement is then re- class V carious lesions. Cavity preparation
moved from the margins at that time. The restora- should be dictated only by the extent of
tion is the painted with a water insoluble varnish caries. No mechanical retentive features are
to protect it from contact with oral fluids. necessary.
l The final finishing should be delayed for several l For abrasion and erosion defects there is no

days. Early finish could disturb or fracture the need for cavity preparation.
margin before maximum properties are attained. (b)   Prophylaxis:
l Silicate cements are subject to dehydration through- l Prophylaxis followed by surface condition-

out their lifetime. Therefore during subsequent op- ing is mandatory.


erative procedures, they should be protected from l This is done using pumice slurry carried in a

exposure to air by a coat of vanish or silicone grease. bristle brush.


l This will remove any plaque or salivary pel-
Q.12. EBA cement.
licle from the tooth surface.
Ans. (c)   Surface conditioning:
l This is an important step in promoting good
l EBA-alumina-modified cements were introduced in an
adhesion of glass ionomers.
effort to improve the mechanical properties of zinc ox-
l Various agents used for conditioning the tooth
ide-eugenol cement.
surface, are
Composition:
10% citric acid
Powder:
3% hydrogen peroxide
Zinc oxide: 70%
10% EDTA
Alumina: 30%
25% tannic acid
Liquid:
10% polyacrylic acid
EBA: 62.5%
l Of these agents, 10% polyacrylic acid applied
Eugenol: 37.5%
for 10–15 s is most widely accepted
l In general, their properties are better than that
iii. Proper manipulation of the cement:
of unmodified ZOE.
l Proper powder liquid ratio should be used and mix
i. Compressive strength is increased 55 Mpa
can be made either on a disposable, moisture resis-
(8000 psi).
tant paper pad or glass slab. A plastic spatula is
ii. Solubility and disintegration in water is
preferred.
decreased – 0.05%wt.
Section | I  Topic-Wise Solved Questions of Previous Years 521

l The GICs are mixed in large increments of powder Q.2. Preformed crown
incorporated into liquid rapidly; mixing should be
Ans.
completed in 40 s. Working time is not more than
3 min from the start of the mix. Various types of preformed crowns available for paediatric
iv. Careful finishing and polishing: use are as follows:
l Bard Parker blades or sharp carvers may be used. i. Pretrimmed crowns (Unitek stainless steel crowns, 3M,
l Final finishing and polishing is done after 24 h. Fin- de novo crowns):
ishing diamond points are used to contour the resto- The crowns are of normal length of the tooth, sides are
ration and Sof-Lex discs from coarse to fine are used straight but shorter when compared to untrimmed
for final finishing. crowns. Need for trimming is minimal. Contouring and
l The finishing has to be done under moist conditions crimping is required.
as dry cutting will dehydrate the cement making it ii. Precontoured crowns (Ni-chrome ion crowns and
chalky and porous, damaging its properties. Unitek):
v. Surface protection: They are similar to pretrimmed crowns in height of the
l Finally the surface has to be protected with the resin- crowns but they are more rounded in the gingival mar-
bonding agent. An alternative method of surface gin and they stimulate the normal appearance of the
protection, though not as effective as the low viscos- tooth.
ity resin sealant, would be to apply two coats of iii. Artglass crowns (Kulzer):
varnish. These are the only patented, preformed crowns for paedi-
l They can be finished using fine finishing diamond atric usage. These crowns provide the natural feel, bond-
points, 12-fluted carbide burs and flexible Sof-Lex ability and kindness associated with composites, but the
discs. Surface protection is required for resin-modified aesthetics and longevity associated with porcelain.
glass ionomer cements also. Q.3. Calcium hydroxide.
Q.14. Dentine-bonding agents. Ans.
Ans. l Calcium hydroxide is relatively weak cement but it is
commonly employed as liner in deep cavities, because
[Same as SE Q.4] of its pulpal biocompatibility, i.e. ability to stimulate
reparative dentine formation and antibacterial activity.
SHORT NOTES: l Uses of calcium hydroxide cement:

a. For direct and indirect pulp capping.


Q.1. Cavity varnish. b. As low strength bases beneath silicate and composite
restorations for pulp protection.
Ans. c. Apexification procedure in young permanent teeth
i. Cavity varnish is a material used to provide the barrier where root formation is incomplete.
against the passage of irritants from the restorative ma- Q.4. Indications and contraindications of stainless steel
terials and to reduce the penetration of oral fluids at the crown.
restoration tooth interface into the pulp.
ii. Cavity varnish is a solution of one or more resins, which Ans.
when applied on to the cavity walls, evaporates leaving Indications of stainless crowns:
a thin resin film that serves as a barrier between the l Teeth involved with extensive and recurrent caries,
restoration and the dentinal tubules. rampant caries
iii. The film thickness ranges between 2 and 400 microns l After pulp therapy
iv. Functions of varnish are l Fractures of primary and permanent incisors
l Reduces the marginal leakage
l Severe bruxism
l Pulp protection
Contraindications of stainless steel crowns:
l Reduces tooth discolouration.
l In anterior teeth where aesthetics is of more concern
Contraindications: l Patients allergic to nickel
Contraindicated when therapeutic action is expected l Imminent exfoliating primary teeth
from some materials like
l Composite resins Q.5. Polycarbonate crowns.
l Glass ionomer
Ans.
l Some cements, e.g. zinc oxide eugenol and

calcium hydroxide [Ref LE Q.2]


522 Quick Review Series for BDS 4th Year, Vol 1

Q.6. Name glass ionomer cements used in pedodontics. Q.9. Indications of glass ionomer cement.
Ans. Ans.
Types of GIC commonly used in pedodontics: Indications of glass ionomer cement are as follows:
Type I: luting and lining cement l In class I, class II, class III and class V restorations
Type II: restorative cement l In pit and fissure sealant
Type VII: protection and stabilization material l Tunnel restorations
Type IX: high strength posterior restorative cement l Root caries
i. Type I: luting and lining cement available in light l As a liner\base and luting cement
yellow shade l As a repair material and in endodontics
Indications:
l Cementation of stainless steel crowns, high
Q.10. Mechanical properties of glass ionomer cement.
strength all ceramic crowns, jacket crowns Ans.
and bridges.
ii. Type II: Restorative cements are available in pale Mechanical properties of glass ionomer cement are
yellow, yellow brown, dark grey and deep brown. a. Compressive strength: (150 Mpa). It is less than silicate.
Indications: b. Tensile strength: (6.6 Mpa). Higher than silicate.
l They are used for restoration of class III, V
c. Hardness: (49 KHN). Less hard than silicates. The wear
and limited class I cavities. resistance is less when compared to composites.
l Core build-up.
d. Fracture toughness: A measure of energy required to
iii. Type VII: Protection and stabilization material. produce fracture is known as fracture toughness.
Available in pink and white shades. Q.11. Advantages of GIC.
Pink shade is command set, while white shade is
chemical set only. Ans.
Indications: Advantages of GIC:
l Fissure protection and root surface protection. l Greater working time
l Intermediate restorations. l Command set on application of visible light
iv. Type IX High strength posterior restorative cement: l Good adaptation and adhesion
Available in shades: A2, A3, A3.5, B2, B3, C4 (ac- l Acceptable fluoride release
cording to vita shade guide). l Aesthetics similar to those of composites
Indications: l Superior strength characteristics
l Class I and class II restorations in deciduous

teeth Q.12. Compomer.


l Nonload bearing class I and class II restora- Ans.
tions in permanent teeth
l Core build-up
l Compomers are a new variety of tooth-coloured restor-
ative materials introduced in the early 1990s.
Q.7. g2 Phase of amalgam. l They are developed to combine the durability of com-

Ans. posite resins and the fluoride releasing ability of GIC.


l They are mainly composed of a resin with minimal GIC
i. The stoichiometric formula of g2 phase in amalgam characteristics. Hence the appropriate nomenclature is
alloys and set dental amalgams is Sn8Hg. ‘polyacid-modified composite resins’.
ii. The g2 phase is the weakest component. The hardness l Available as a single paste, light-curable material in a
of g2 is approximately 10% of the hardness of g1, syringe or compule.
whereas the g phase hardness is somewhat higher than Composition:
that of g1. i. Resin matrix – UDMA and TCB resins
iii. The g2 phase is also the least stable in a corrosive en- ii. Glass fillers – strontium fluorosilicate glass fillers
vironment and may suffer corrosion attack, especially iii. Photoinitiators
in ‘crevices’ of the restorations. iv. Stabilizers
iv. In general, g(Ag3Sn) and pure g1 (Ag2Hg3) phases are Indications:
stable in an oral environment. l Restorations of class III and class V cavities.

l Aesthetics and fracture toughness of composites,


Q.8. Mention four advantages of rubber dam applications.
combined with fluoride release is desired.
Ans. l Deciduous restorations in anterior and posterior

[Ref SE Q.9] regions.


Section | I  Topic-Wise Solved Questions of Previous Years 523

Topic 22
Restoration of Primary Carious Teeth
COMMONLY ASKED QUESTIONS

LONG ESSAYS:
1 . Describe briefly about willets inlay preparation.
2. Define class II cavity. Explain the modifications considered.
3. Discuss the differences in cavity preparation between primary and permanent molars.
4. Classify stainless steel crowns. Give composition and procedure of stainless steel crown in detail.
5. Write briefly about cavity preparation of class II in a primary molar tooth. [Same as LE Q.2]

SHORT ESSAYS:
1 . Explain tooth preparation for stain less steel crown.
2. Infected versus affected dentin.
3. Separators.
4. Matrices used in paediatric dentistry.
5. Willet inlay preparation.
6. Indications and contraindications of stainless steel crowns.
7. Full crown restorations for deciduous anterior teeth.
8. Wedges. [Same as SE Q.3]
9. Matrix bands.[Same as SE Q.4]

SHORT NOTES:
1. T-band matrix.
2. Classify different types of matrices used in paediatric dentistry. [Ref SE Q.4]
3. Enumerate factors one must consider before restoring the teeth in paediatric dentistry.
4. Proximal slicing of primary teeth. [Ref LE Q.4]
5. Strip crowns. [Ref SE Q.7]
6. Preformed crown.
7. Uses of stainless steel crowns.
8. Saliva ejectors.
9. Semipermanent restorations.
10. Polycarbonate crowns.
11. Intermediary restorations.
12. Tunnel cavity preparation.
13. Mention five causes of class II amalgam failures in primary molar.
14. Enumerate the factors one must consider before restoring the teeth in paediatric dentistry.

SOLVED ANSWERS
LONG ESSAYS:
Q.1. Describe briefly about willets inlay preparation. l These inlays are prepared on primary molars and are
expensive. However, these inlays can be prepared fast
Ans. with little chair time.
Preparation of tooth:
l Willett inlay preparation can be considered, when a fill- l Using a safe-sided disk at slow speed, slicing of the
ing on a primary molar will not have longevity. proximal areas is done.
524 Quick Review Series for BDS 4th Year, Vol 1

The slicing is done at right angles to the occlusal retention. Removing tooth structure to produce
plane of the tooth and carrying it just below the free a lock would only serve to weaken the tooth
margin of gingiva and laterally into self-cleansing unnecessarily.
areas. b. Proximal box:

l With a tapering fissure bur, a trench cut is made l Buccal and lingual walls of proximal box

through the enamel beginning one-third up the buc- should have an occlusal convergence and ap-
cal groove, extending over the occlusal surface and proximately follow the buccal and lingual sur-
down through the lingual surface. faces of the tooth.

l The trench cut ends in a feather edge at the gingival l This provides increased retention and also ex-

third and follows the contour of the tooth. From the tends the preparation into self-cleansing areas.
central trench, a similar cut is made to connect with l There is no need to bevel the gingival seat in

the proximal slice. primary molars as the enamel rods are inclined

l The walls must be parallel and all undercuts are occlusally. Axio-pulpal line angle should be
removed to permit the wax pattern to separate freely rounded to prevent stress concentration and to
and without distortion. add bulk of material into the restoration.

l Casting is carried out with base metals or gold. c. Isthmus:

l After the advent of stainless steel crowns, these inlays l The width of the isthmus should be approxi-

are not practiced widely in paediatric dentistry. mately one-third of inter-occlusal distance.
l Increasing the depth of the cavity will aid in
Q.2. Define class II cavity. Explain the modifications
better retention of restoration than the width.
considered.
l Increasing the width unnecessarily removes

Ans. sound tooth structure.


l Isthmus fracture seems to occur only when
l According to Black’s classification class II lesions occur
there is a premature contact from the opposing
on the proximal surfaces of posterior teeth.
tooth. Reduction of high points with articulat-
l Class II lesions occur after the primary molars contacts
ing paper is of paramount importance while
are established. If the class II cavity becomes larger than
doing class II restorations.
the minimal classical dimensions then the tooth would
Modifications of class II preparation:
be better served by a stainless steel crown.
l Modifications of class II preparations are used when
l Preparation of class II includes, class I cavity prepara-
proximal caries is not involving the occlusal surface
tion and preparation of gingival or proximal box.
and when conservative preparations are needed.
Traditional class II cavity preparation:
l They include slot cavity preparation, tunnel cavity
l The main concern is the preservation of tooth struc-
preparations and proximal approach.
ture to leave the tooth as strong as possible.
l Traditional class II cavity preparation consists of Q.3. Discuss the differences in cavity preparation be-
three components: tween primary and permanent molars.
a. Occlusal extension
Ans.
b. Proximal box
c. Isthmus l Principles involved in cavity preparation for primary
a. Occlusal extension: teeth differ from that of permanent teeth.
l Caries is removed with a round bur and ideal l Morphology of primary teeth is responsible for these

depth is 0.5 mm into dentino-enamel junction. differences and they are discussed in relation to their
l The buccal and lingual walls are prepared with clinical significance.
occlusal convergence. Various morphologic considerations in cavity preparation
l The cavity outline is extended into all the suscep- of primary teeth:
tible grooves and fissures. The buccal and lingual Shape of the crown:
walls should be at right angles to the surface of l Generally, the size of the primary teeth is smaller

the tooth and in the direction of enamel rods. but crown is more bulbous than permanent teeth.
l The pulpal floor should be flat mesiodistally. The crowns are wider mesiodistally than occluso-
l The dovetail becomes unnecessary, when the gingivally.
fissures and grooves are included in the cavity l As the occlusal table is narrow due to the occlusal

preparation. convergence of buccal and lingual surfaces, it auto-


l The idea of the dovetail lock cut at the opposite matically results in reduction of the bucco-lingual
end of the occlusal part to the proximal box is dimensions of the occlusal part of any class I or II
outdated. The purpose of dovetail is to increase cavity to prevent weakening of cusps.
Section | I  Topic-Wise Solved Questions of Previous Years 525

This is more pronounced in first primary molars The differences in the cavity preparation of primary teeth
than second primary molars. can be summarized as follows:
l In class II preparations, the isthmus is narrow i. In the primary teeth the cavity preparation is smaller,
exactly at a point where strength is needed to shallower and narrower due to smaller crown size,
withstand the forces of occlusion. thin enamel and narrow occlusal table, respectively.
l The thickness of enamel and dentin is very less in ii. The pulpal floor is made saucer shaped because of
primary teeth so the pulp is proportionately larger. higher pulp horns.
The clinical significance of this entity is caries iii. The occlusal walls are less convergent.
can progress to the pulp faster if not diagnosed iv. In proximal box preparation, the buccal and lingual
early, smaller burs needs to be used, sufficient walls are more convergent occlusally.
bulk of restorative material is to be placed without v. Buccal and lingual retentive grooves are contrain-
exposing the pulp to provide retention. dicated.
l The distance between the mesial surface of the vi. Bevel is given at the gingival seat as the enamel
mandibular first primary molar and the pulp may rods are directed occlusally.
be as little as 1.6 mm; this should be considered
Q.4. Classify stainless steel crowns. Give composition
during cavity preparation.
and procedure of stainless steel crown in detail.
l The sharp constriction at the neck of primary

teeth necessitates special care in the formation of Ans.


gingival floor during class II preparation. l Humphrey introduced stainless steel crowns in the year
Contact areas: 1950.
l Contact areas between primary molars are broad,
l These stainless steel crowns are semipermanent resto-
fatter and situated farther gingivally than those of rations used to overcome a wide range of problems
permanent teeth. encountered in paediatric dentistry on both primary and
l The clinical significance of this factor is
permanent teeth.
i. Interproximal caries need to be extensive be- Classification of stainless steel crowns.
fore they are clinically observable, so one has I. Based on composition:
to rely on bitewing radiographs for its early i. Stainless steel crown (Unitek and Rocky Moun-
diagnosis. tain crown)
ii. Buccal and lingual margins of proximal box in ii. Nickel-base crowns (Ion Ni-Chro from 3M)
class II restorations should extend towards the iii. Tin-base crowns
embrasure to make them accessible for self- iv. Aluminium-base crowns
cleansing. II. Based on morphology:
iii. As the proximal caries starts below the contact i. Untrimmed/uncontoured crowns (Unitek)
area, gingival seat must be taken below the ii. Pretrimmed crowns (Unitek stainless steel
contact area. crowns, 3M, De novo crowns)
Enamel rod inclination: iii. Precontoured crowns (Ni-Chro ion crowns and
l The inclination of enamel rods of primary molars
Unitek)
in the gingival one-third is towards the occlusal Based on composition the stainless steel crowns are
surface. described as follows:
l Hence, there is no need for bevelling the gingival
i. Stainless steel crown (Unitek and Rocky Moun-
seat in primary molars, which is done routinely to tain crown):
remove the unsupported enamel rods in perma- l These alloys are made of authentic steels with
nent teeth where enamel rods show apical or hori- the composition of 17%–19% chromium, 9%–
zontal inclination. 13% nickel and 0.08%–0.12% carbon.
Pulp morphology: l These alloys have good formability and ductil-
l The pulp horns in primary teeth are more promi-
ity necessary for the clinical adaptation of
nent than that of permanent teeth. crowns.
Especially the first primary molar has an occlusal l They have adequate hardness and wear resis-
ridge of enamel below which is a pulp horn. This tance to resist opposing occlusal forces.
horn is very fine and may extend to the dentino- ii. Nickel-base crowns (Ion Ni-Chro from 3M):
enamel junction. Hence, there is considerable risk l These are an Iconel type of alloy with the
when a cavity preparation is attempted on the first nominal composition: 76% nickel, 15.5%
primary molar. chromium, 8% iron, 0.04% carbon and 0.35%
l In class I and II restorations, the depth of the cav-
manganese.
ity should be kept, to a minimum to prevent inad- l These crowns are strain hardened during man-
vertent pulp exposures. ufacturing.
526 Quick Review Series for BDS 4th Year, Vol 1

l They have formability and ductility, necessary iii. The Abel ball-and-socket plier no. 112:
for the clinical manipulation. Can be used to produce contact points
ii. Tin-base crowns: iv. Gordon plier no. 137:
l They have a nominal composition of 96% of Used to contour the cervical margin
tin and 4% of silver. v. Crown and bridge scissors
l They are readily adaptable but are not as per- Tooth preparation for stainless steel crowns:
manent as above crowns. l Prior to starting a tooth preparation, occlusal assess-
iv. Aluminium-base crowns: ment should be done and, anaesthesia should be
l These are made of aluminium alloys contain- given to the patient.
ing 1.2% manganese, 10% magnesium, 0.7% l The three steps in tooth preparation are:
iron, 0.3% silicon and 0.25% copper. i. Occlusal preparation
l Even though they are readily adaptable, their ii. Proximal reduction
clinical durability is less. iii. Rounding of line angles
Based on morphology: i. Occlusal preparation:
i. Untrimmed/uncontoured crowns (Unitek): l According to Nash in 1981, 69 or 169 L bur is
l They were first developed crowns whose sides used to reduce the occlusal surface by 1.5–2.0
are straight and longer. mm following the cuspal outline and main-
l Nowadays they are preferred only in deep taining the original contour of the cusp keep-
interproximal lesions. ing in the view, the currently available nickel-
ii. Pretrimmed crowns (Unitek stainless steel chromium crowns.
crowns, 3M, De novo crowns): l Humphrey in 1950 recommended that the
l The crowns are of normal length of the tooth; cusp be reduced, if necessary and the four
sides are straight but shorter when compared sides of the tooth were reduced but preserving
to untrimmed crowns. as much tooth structure as possible.
l They are not contoured. Need for trimming is l Occlusal preparation is done prior to the prox-
minimal when compared to untrimmed imal reduction, as the chances of gingival
crowns. Contouring and crimping is required. bleeding due to proximal reduction can be a
iii. Precontoured crowns (Ni-Chro ion crowns and hindrance in diagnosing the bleeding from the
Unitek): pulp.
l They are similar to pretrimmed crowns in ii. Proximal reduction:
height of the crowns but they are more
rounded in the gingival margin and they sim-
{SN Q.4}
ulate the normal appearance of the tooth.
Clinical availability: l Wooden wedges are used in the interproximal em-
l They are available from size 2 to 7. brasures to reduce the risk of damage to the adjacent
l The crowns are available clinically in different tooth enamel.
sizes for both primary and permanent molar teeth l The bur is swept buccolingually across the proximal
individually. surface, beginning at the marginal ridge and at an
Procedure for placement of stainless steel crowns: angle slightly convergent to the occlusal surface.
Armamentarium: l The bur should follow a path tangential to the proxi-
A. For tooth preparation: mal surface and the depth of the slice should be suf-
Burs: ficient to break the contact with the adjacent teeth
i. 69 or 169 L bur: and produce a knife-edge finish line.
For tooth reduction l Care must be taken to extend the preparation gingi-
ii. No. 1/2 round bur: vally far enough to avoid the development of a ledge,
For marking on the stainless steel crown which would make it difficult to seat the crown prop-
B. For crown adaptation erly. The sufficient reduction is checked by passing
Pliers: an explorer through the interproximal aspect at the
i. The no. 114 (wide) and 115 (medium) John- gingival portion.
son pliers:
Used for general contouring of the stainless
steel crown iii. Roundening line angles:
ii. Crimper no. 800-417: l All line angles created by the occlusal prepara-

Used to crimp the last 1–2 mm of the gingi- tion and proximal reductions should be
val margin rounded.
Section | I  Topic-Wise Solved Questions of Previous Years 527

l Moving the bur at an angle of 45° rounds the oc- l Nowadays, the crown crimper no. 800-417 is used to
clusobuccal and lingual surfaces. crimp the last 1–2 mm of the gingival margin. This
l The other line angles are slightly rounded into the may produce a scalloped margin, which can be
proximal preparation to avoid any sharp margins. smoothened by drawing the crown while squeezing
Crown selection: the plier.
l The three main considerations in selecting the crown l The Abel ball-and-socket plier can be used to pro-
are duce contact points.
i. Adequate-mesiodistal diameter l Gordon plier no. 137 is used to contour the cervical
ii. Light resistance to seating margin.
iii. Proper occlusal height During this time:
l The correct size crown may be selected:
a. The crown must snap into the place, which cannot be
i. Prior to tooth preparation by measuring the me- removed with finger pressure.
siodistal dimension of the tooth to be restored b. It must fit snugly against the tooth.
using a Boley gauge. c. There is no blanching or cutting of gingiva.
ii. It can also be selected after the preparation of the d. The margin of the crown is 1 mm into the sulcus and
tooth. should closely engage the tooth structure on all surfaces.
iii. Trial and error method.
e. The occlusion should be normal without any inter-
l Crowns are manufactured such that the length is
ference and the contact with the adjacent tooth, if
proportional to the mesiodistal and circumferential present, should be re-established.
measurements. l In 1984, Spedding gave two principles for obtain-
l It is generally advisable to select a medium- size
ing optimal adaptation of stainless steel crowns to
crown such as a no. 4 and progress to a larger or a primary molars.
smaller crown as required.
l A crown selected should be somewhat larger than the They are
tooth to which it is being adapted. The most common i. Crown length
sizes used are the size 4 or 5. ii. Shape of crown’s margin
l The correct crown will approximate the mesiodistal l As said earlier, the crown margin should just be

width and the circumference of the tooth, having into the sulcus engaging the undercut. The outline
been placed with some resistance and yet completely of the crown margin should follow the gingival
enveloping the tooth at the cervical aspect. margin of the tooth. For the second primary mo-
Crown seating: lar, it should resemble smiles on the buccal and
l The selected crown should be placed on the lingual lingual margins.
side and rotated to the buccal side . l The buccal marginal gingiva of the first primary

l The crown should fit loosely with 2–3 mm of excess molar has a different outline due to the mesiobuc-
on the gingival side. cal bulge. It will be like the letter S that is
1
l Using either a scaler or the no. /2 slow-speed round stretched out.
bur, the marking should be done on the gingival l The contour on the lingual gingival margin of all

margin. first primary molars resembles smiles.


l From this mark, the excess length of the crown be- l The proximal contour of all primary molars re-

yond 1 mm can be cut using scissors or trimming sembles frown, as the occlusogingival height is
using heatless stone. shortest at midpoint buccolingually. While trim-
l A properly seated crown will correspond to the mar- ming the crown, the shape of the margin should be
ginal ridge height of the adjacent teeth and is not kept in mind.
rotated on the tooth. l Contouring and crimping the crown margin

Adapting the crown: help to get a tight fit around the crown. A tight
l As the nickel-chromium crowns are pretrimmed marginal fit aids in mechanical retention of the
and contoured, only few adjustments are needed. crown, protection of the cement from oral fluids
l Adaptation is very important to the gingival health and maintenance of gingival health.
of the supporting tooth. Finishing:
l The various pliers used for this are no. 114 (wide) l After adaptation, the crown margin should be

and 115 (medium) Johnson pliers to contour the trimmed using green stone in counterclockwise di-
gingival edges to tighten the fit of the crown. rection at 45° angle.
l Poorly adapted crowns will serve as a collection l This will help to achieve what is called a minicrimp.

area for bacteria and can cause recurrent caries. Then the rubber wheel is used to smoothen it after
528 Quick Review Series for BDS 4th Year, Vol 1

which the crown is polished using Tripoli/iron ii. Proximal reduction:


oxide. l Wooden wedges are used in the interproximal
Cementation: embrasures to reduce the risk of damage to the
l The cement should be selected mainly based on adjacent tooth enamel.
the status of the pulp. While cementing a crown on l The bur is swept buccolingually across the prox-
a vital tooth cavity, varnish should be routinely imal surface, beginning at the marginal ridge and
used. The most commonly used cement is the at an angle slightly convergent to the occlusal
glass ionomer cement. surface.
l Mathewson in 1975 said that the retention of l The bur should follow a path tangential to the
stainless steel crowns is due to cementing me- proximal surface and the depth of the slice
dium used rather than the mechanical adaptation. should be sufficient to break the contact with
l After cementation, the excess cement in the inter- the adjacent teeth and produce a knife-edge fin-
proximal surfaces is removed using the floss with ish line.
a knot and drawing it through. Cement should be l Care must be taken to extend the preparation
expressed around all the crown margins; this en- gingivally far enough to avoid the development
sures that all the spaces between crown and tooth of a ledge, which would make it difficult to seat
have been completely filled by the cement. the crown properly. The sufficient reduction is
checked by passing an explorer through the inter-
Q.5. Write briefly about cavity preparation of class II in proximal aspect at the gingival portion.
a primary molar tooth. iii. Roundening line angles:
l All line angles created by the occlusal preparation
Ans.
and proximal reductions should be rounded.
[Same as LE Q.2] l Moving the bur at an angle of 45° rounds the

occlusobuccal and lingual surfaces.


l The other line angles are slightly rounded into
SHORT ESSAYS:
the proximal preparation to avoid any sharp
margins.
Q.1. Explain tooth preparation for stain less steel crown.
Ans. Q.2. Infected versus affected dentin.
Tooth preparation for stainless steel crowns is as follows: Ans.
l Prior to starting a tooth preparation, occlusal assessment

should be done, and anaesthesia should be given to the The differences between infected and affected dentin are as
patient. follows:
l The three steps in tooth preparation are
Infected dentin Affected dentin
i. Occlusal preparation
More superficial layer of Deeper layer
ii. Proximal reduction l l

carious dentin l Dark brown in colour


iii. Rounding of line angles l Light brown in colour l Hard in consistency
i. Occlusal preparation: l Soft and leathery in consistency l Sensitive to touch
l According to Nash in 1981, No. 69 or 169 L bur l Not sensitive to touch. l Collagen reversibly
l Collagen is irreversibly dena- denatured
is used to reduce the occlusal surface by 1.5–2.0
tured No bacteria are found
mm following the cuspal outline and maintain- l

l Has a high concentration of in this zone


ing the original contour of the cusp keeping in bacteria l Not stained by caries
the view, the currently available nickel-chro- l Stained by caries detecting detecting dyes
mium crowns. dyes l Capable of remineral-
l Not remineralizable ization
l Humphrey in 1950 recommended that the cusp be
l Should be removed Should be retained
reduced if necessary, and the four sides of the l

tooth were reduced but preserving as much tooth


structure as possible. Q.3. Separators.
l Occlusal preparation is done prior to the proximal
Ans.
reduction, as the chances of gingival bleeding due
to proximal reduction can be a hindrance in diag- l Wedge is defined as a piece of wood, metal, etc., one
nosing the bleeding from the pulp. end of which is an acute angled edge formed by two
Section | I  Topic-Wise Solved Questions of Previous Years 529

converging planes used to tighten or exert force in vari- iii. Assures proper health of interdental col
ous ways. iv. Absorbs fluid and aids in tooth separation
l At present all types of wedges like plastic, metal, wood
Q.4. Matrices used in paediatric dentistry.
and celluloid are available depending upon side and
type of tooth. Ans.
l The newest type of wedge is the light reflecting
Matricing is a procedure where by a temporary wall is cre-
one introduced by Luci-wedge Hawe-Neos dental,
ated opposite the axial wall surrounding the areas of tooth
Switzerland.
structure lost during preparation. The appliance used for
Various types of wedges are as follows:
building these walls is called matrix.
i. Anatomical – same as shape of embrasure
Rationale for using matrix
ii. Nonanatomical – round
i. Accurate reproduction of tooth contour, to main-
iii. Wooden – can be made of either hard or soft
tain integrity of normal gingival papillae.
wood
ii. To establish tight contact areas.
iv. Plastic – available in various shapes
iii. To maintain arch dimensions in primary dentition.
v. Coloured – all types
Ideal requirements of matrix are as follows:
vi. Light reflecting – to be used with composites
i. Should be rigid to allow condensation and promote
Ideal requirements of wedges are as follows:
desired contour
i. Easy to apply and withdraw
ii. Should be easy to apply and form positive contact
ii. Should be rigid, disposable and radiopaque, and
with tooth
should adapt to shape of tooth embrasure
iii. Should be of minimal thickness
iii. Should not cause deformation of matrix band
iv. Compatible with restorative material
iv. Should be nontoxic, nonirritant and stable in oral
v. Economical
fluids
Selection of a correct wedge:
l A wedge should compress the matrix band to re-

maining healthy tooth structure through its entire


bucco-lingual length apically to gingival cavosurface {SN Q.2}
line angle. Classification of matrix:
l To select a correct wedge four variables are to be I. According to region of application:
considered: l Anterior – Celluloid matrix
i. Convergence angle of the base l Posterior – T-band, Tofflemire
ii. Gingival base width II. According to constituents:
iii. Wedge height l Metallic – Ivory no. 1, Ivory No. 8, Tofflemire
iv. Concavity of side walls l Nonmetallic – Mylar strips
i. Convergence angle of the base: III. According to presence or absence of retainer:
l The angle created by two tangential lines l With retainer – Ivory No. 1, Ivory No. 8
drawn to adjacent tooth structures at gingival l Without retainer – S-band
cavosurface line angles should match the con- IV. According to form:
vergence angle of wedge to ensure maximum l Anatomical – Celluloid crown form
rigid support. l Nonanatomical – Ivory No. 1
ii. Gingival base width: V. According to use
l It should be slightly greater than inter-dental l Universal – Ivory No. 8, Tofflemire
space width in order to achieve stability. l Unilateral – Ivory No. 1
iii. Wedge height:
It is critical to establish contact point.
iv. Concavity of side walls: Functions of matrix are as follows:
It dictates proximal contour of the restored tooth i. To replace the missing wall and allow close adapta-
surface. tion of restorative material.
Functions of a wedge are as follows: ii. Should retain restorative material during placement.
i. Assures close adaptation of matrix band to tooth iii. Allows restoration of contact point and external
and prevents gingival overhang crown contour.
ii. Helps in stabilization of band iv. Helps in isolation of cavity.
530 Quick Review Series for BDS 4th Year, Vol 1

Q.5. Willet inlay preparation. l The durability is good like steel crowns, but the
facings can fracture.
Ans. l These are indicated in maxillary canines where

Willet inlay preparation of tooth: strength is a major requirement as compared to


l Using a safe-sided disk at slow speed, slicing of the aesthetics.
proximal areas is done. l The labial portion of anterior stainless steel crown

l With a tapering fissure bur, a trench cut is made is removed and composite is placed as a facing,
through the enamel beginning one-third up the buc- thereby providing adequate strength and accept-
cal groove, extending over the occlusal surface and able aesthetics.
down through the lingual surface. l The disadvantage of this crown is that some

l The trench cut ends in a feather edge at the gingival amount of metal margins is visible and these
third and follows the contour of the tooth. From the crowns are not easily removed.
central trench, a similar cut is made to connect with ii. Resin-veneered stainless steel crowns:
the proximal slice. l Prefabricated resin-veneered crowns are available in

l The walls must be parallel and all undercuts are re- which the composite resin and thermoplastics are
moved to permit the wax pattern to separate freely bound to the metal.
and without distortion. l The advantages of these crowns are that an aestheti-

l Casting is carried out with base metals or gold. cally pleasing result can be obtained with relatively
short operating time but occasionally the veneer can
Q.6. Indications and contraindications of stainless steel fracture.
crowns. l They are expensive.

Ans. l More recently, a new resin-veneered Dura crown is

introduced which has labial gingival margin crimped


Indications of stainless steel crowns are as follows: and resin adapted to the gingival edge of the anterior
l Teeth involved with extensive and recurrent caries
aspect of the crown.
and rampant caries iii. Polycarbonate crowns:
l After pulp therapy
l Polycarbonate crowns are heat-moulded acrylic res-
l Intermediate restoration
ins used to restore anterior teeth.
l Fractures of primary and permanent incisors
l These do not resist strong abrasive forces, leading to
l Severe bruxism
occasional fracture and debonding or dislodgement.
l Inherited or acquired enamel defects
These are contraindicated:
l As abutment teeth to prosthesis
l When there is inadequate spacing between teeth
l As part of a space maintainer
l Crowding of anterior and deep overbite
Contraindications of stainless steel crowns are as follows: l Bruxism habit
l In anterior teeth where aesthetics is of more concern
l When there is evidence of abrasion in the anterior
l Patients allergic to nickel
teeth
l Imminent exfoliating primary teeth

Q.7. Full crown restorations for deciduous anterior {SN Q.5}


teeth.
iii. Strip crowns:
Ans. l These are celluloid crown forms that are the most

effective for use in paediatric patients with exten-


Full crown restorations in paediatric dentistry are as
sive caries in anterior teeth.
follows:
l They are transparent so shade selection of com-
i. Stainless steel crowns with composite facing.
or posite is advantageous.
l Easy to place and remove.
Facial cutout stainless steel crowns:
l Less time-consuming.
l These are referred to as ‘open-face steel crown’ in

which a window preparation is done on to the


cemented stainless steel crown and over which iv. Artglass crowns (Kulzer):
composite material is placed. This technique is a l These are the only patented, preformed crowns for

dramatic improvement over the plain metallic ap- paediatric usage.


pearance of stainless steel crown. l These crowns provide the natural feel, bondability

l Placement of this crown takes a longer time as it and kindness associated with composites, but the
is a two-step procedure: aesthetics and longevity associated with porcelain.
i. Crown placement l Wear of Artglass is similar to enamel and kind to op-

ii. Composite placement posing dentition.


Section | I  Topic-Wise Solved Questions of Previous Years 531

l High inorganic filler, makes Artglass colour stable Q.4. Proximal slicing of primary teeth.
and plaque resistant.
Ans.
l Matched to the Vita shade system, simplifies shade

selection. [Ref LE Q.4]


l Easily adjusted or repaired intraorally, less chair time
Q.5. Strip crowns.
for dentists.
l Provides the aesthetics and lasting qualities of Ans.
porcelain.
[Ref SE Q.7]
Q.8. Wedges. Q.6. Preformed crown.
Ans. Ans.
[Same as SE Q.3] l The stainless steel crowns are semipermanent restora-
Q.9. Matrix bands. tions used to overcome a wide range of problems en-
countered in paediatric dentistry on both primary and
Ans. permanent teeth.
Based on morphology preformed crowns are as follows:
[Same as SE Q.4]
i. Pretrimmed crowns (Unitek stainless steel crowns,
3M, De novo crowns):
SHORT NOTES: l The crowns are of normal length of the tooth,

sides are straight but shorter when compared to


Q.1. T-band matrix. untrimmed crowns.
l They are not contoured. Need for trimming is
Ans.
minimal when compared to untrimmed crowns.
l T-band is a preformed T-shaped stainless steel matrix Contouring and crimping is required.
band without a retainer. ii. Precontoured crowns (Ni-Chro ion crowns and
l Band is stabilized by wedging and supported with low- Unitek):
fusing compound. l They are similar to pretrimmed crowns in height
l It is indicated in class II cavities involving one or both of the crowns but they are more rounded in the
proximal surfaces of a posterior tooth. gingival margin and they simulate the normal
Advantages: appearance of the tooth.
l It is simple and inexpensive matrix system.
Q.7. Uses of stainless steel crowns.
l Rapid and easy to apply.

l Disadvantage is that it is flimsy in structure and not Ans.


very stable.
The uses of stainless steel crowns are as follows:
Q.2. Classify different types of matrices used in paediatric i. Protection of pulpectomized teeth
dentistry. ii. Optimum tooth structure reduction for adequate crown
retention
Ans.
iii. Maintains arch length
[Ref SE Q.4] iv. Economical and less traumatic to the tooth and sur-
rounding structures
Q.3. Enumerate factors one must consider before restoring v. Provides accurate marginal adaptation and gingival health
the teeth in paediatric dentistry. vi. Provides good functional occlusion
Ans. Q.8. Saliva ejectors.
Factors to be considered before restoring the teeth in pae- Ans.
diatric patients are as follows:
i. Age of the child l Saliva ejector removes water slowly and have little
ii. Extent of damage to the sound tooth due to caries, frac- capacity for picking up solids.
tures or physiological variations like abrasion, erosion l The tip of the saliva ejector should be smooth and made

and abfraction from a nonirritating material.


iii. Isolation and anaesthesia l Disposable, inexpensive plastic ejectors that may be

iv. Time of normal exfoliation shaped by bending with the fingers are preferable, be-
v. Space consideration in arch cause of improved infection control.
532 Quick Review Series for BDS 4th Year, Vol 1

l Svedopter is a helpful device that serves both as saliva tissue tolerance and ability to minimize initial microle-
ejector and a tongue retractor. akage.
l The strength, rigidity and resistance to abrasion of the
Q.9. Semipermanent restorations.
conventional ZOE mixture are improved by the addition
Ans. of polymers and by the surface treatment of the zinc
oxide powder.
l The stainless steel crowns are semipermanent restora-
tions used to overcome a wide range of problems en- Q.12. Tunnel cavity preparation.
countered in paediatric dentistry on both primary and
Ans.
permanent teeth.
l Stainless steel crowns are introduced by Humphrey in l The tunnel cavity preparation is a modification of
the year 1950. class II and is indicated for small cavities and for
l In the small deciduous teeth, neglected caries can de- cavities which are placed 2–2.5 mm below the mar-
stroy the tooth integrity faster than that in the larger ginal ridge.
permanent teeth and moreover the deciduous pulp is l The aim is to develop:

larger than the permanent pulp in relation to its dentin i. Access via the occlusal aspect.
and enamel envelope. In these situations, the clinician’s ii. To preserve the strength of marginal ridge.
best choice is semipermanent stainless steel crowns. iii. To prevent formation of proximal cavity.
l Procedure includes cavity preparation in the direction of
Q.10. Polycarbonate crowns.
lesion, once the lesion is spotted.
Ans. l Caries is removed by using slow speed round bur and

proximal wall is not fractured, if it is not involved.


l Polycarbonate crowns are heat-moulded acrylic resins
l Remaining caries is removed with spoon excavator and
used to restore anterior teeth.
cavity is restored using glass ionomer cement.
l These do not resist strong abrasive forces, leading to

occasional fracture and debonding or dislodgement. Q.13. Mention five causes of class II amalgam failures in
These are contraindicated: primary molar.
l When there is inadequate spacing between teeth
Ans.
l Crowding of anterior and deep overbite

l Bruxism habit Causes for class II amalgam failures in primary molars in


l When there is evidence of abrasion in the anterior teeth comparison with stainless steel crowns are as follows:
Advantages: i. Low fracture resistance.
l They are extremely stable dimensionally. ii. Child below the age of 4 years.
l Unaffected by dilute mineral, acids, ether and iii. Less cost-effective.
alcohol. iv. More chair side time.
Disadvantage: v. Success rate is less requiring retreatment.
l Poor abrasion resistance Q.14. Enumerate the factors one must consider before
Indications: restoring the teeth in paediatric dentistry.
l Full coverage restoration of primary maxillary

anterior teeth with extensive caries Ans.


l Early childhood caries The factors one must consider in restoring the teeth in pae-
l Deformities in structure of teeth diatric dentistry are as follows:
l Discoloured teeth i. In the primary teeth, the cavity preparation is
Q.11. Intermediary restorations. smaller due to smaller crown size.
ii. Shallower due to thin enamel and narrower due to
Ans. narrow occlusal table.
l The temporary or intermediate restoration should iii. The pulpal floor is made saucer shaped because of
possess: higher pulp horns.
i. Good biologic characteristics iv. The occlusal walls are less convergent.
ii. Minimal solubility v. In proximal box preparation, the buccal and lingual
iii. Should be rigid, strong and resistant to abrasion walls are more convergent occlusally.
l Zinc oxide eugenol cement is commonly used as inter- vi. Bevel is not given at the gingival seat in primary
mediate restorative material because of its excellent molars as the enamel rods are directed occlusally.
Section | I  Topic-Wise Solved Questions of Previous Years 533

Topic 23
Paediatric Endodontics
COMMONLY ASKED QUESTIONS
LONG ESSAYS:
1. Describe briefly about pulpotomy.
2. Write briefly about the apexification procedure.
3. Enumerate different root canal filling materials used in paedodontics. Describe any one in detail.
4. Describe your treatment plan when you are encountered with a small traumatic exposure while performing a
cavity in primary molar.
5. Write briefly on formocresol pulputomy.
6. Describe the procedure in treating a young permanent fractured central incisor with wide apical foramen and
necrotic pulp.
7. Define pulpectomy. What are the indications and contraindications for pulpectomy. Write the step-by-step
procedure for pulpectomy in 85.
8. What is pulpotomy. Discuss the reaction of pulp to glutaraldehyde, formocresol and calcium hydroxide.
9. Write the indications, contraindications and procedure of pulpotomy in a deciduous molar. [Same as LE Q.1]
10. Define pulpotomy. Describe the indications, contraindications and procedure of pulpotomy. [Same as LE Q.1]
11. Write briefly on formocresol pulputomy. [Same as LE Q.1]
12. Define pulpotomy. Describe the technique and pulpal tissue changes following formocresol pulpotomy in
primary molar. [Same as LE Q.1]
13. Define pulpotomy. Mention the step-by-step formocresol pulpotomy procedure in primary teeth. [Same as LE Q.1]
14. What is the difference between apexogensis and apexification? [Same as LE Q.2]
15. Direct pulp capping. [Same as LE Q.4]
16. Define pulpotomy. Describe the technique and pulpal tissue changes following formocresol pulpotomy in pri-
mary molar. [Same as LE Q.5]
17. A 5-year-old patient comes in your clinic with a badly carious nonrestorable mandibular 2nd deciduous molar.
Give your treatment plan. [Same as LE Q.7]
18. Define pulpotomy. Describe the histological picture of dental pulp after a glutaraldehyde pulpotomy. Why is
glutaraldehyde preferred over formocresol? [Same as LE Q.8]

SHORT ESSAYS:
1. Apexogenesis versus apexification. [Ref LE Q.2]
2. Indications and contraindications of pulpotomy. [Ref LE Q.1]
3. Explain vitality test, describe various methods.
4. What are the requirements of root canal obturation materials for primary teeth?
5. Glutaraldehyde.
6. Direct pulp capping. [Ref LE Q.4]
7. Define indirect pulp capping. Mention objectives indications and contraindications.
8. Describe formocresol pulpotomy. Add a note on Sweet’s formocresol pulpotomy. [Ref LE Q.1]
9. Reaction of pulp to glutaraldehyde and formocresol. [Ref LE Q.8]
10. Bleaching of teeth.
11. Pulp vitality tests. [Same as SE Q.3]
12. Root canal filling materials used in 75. [Same as SE Q.4]
13. Obturating materials for primary teeth. [Same as SE Q.4]
14. Ideal requirements of obturating material of primary teeth. [Same as SE Q.4]
15. Glutaraldehyde pulpotomy. [Same as SE Q.5]
534 Quick Review Series for BDS 4th Year, Vol 1

SHORT NOTES:
1. Define pulpotomy. [Ref LE Q.1]
2. Root canal obturating materials used for deciduous teeth. [Ref LE Q.3]
3. Pulpotomy medicaments. [Ref LE Q.1]
4. Mention various types of formocresol pulpotomy. [Ref LE Q.5]
5. Name four vitality tests. [Ref SE Q.3]
6. Formocresol.
7. Indirect pulp capping. [Ref SE Q.7 SE]
8. Root canal irrigation.
9. Define apexification. [Ref SE Q.1]
10. Indications for direct pulp capping in permanent teeth.
11. Histological changes or reaction of pulp to calcium hydroxide. [Ref LE Q.8]
12. Define direct pulp capping. [Ref SE Q.6]
13. Pulp capping agents. [Ref SE Q.6]
14. KRI paste.
15. MTA.
16. Iodoform-based obturating pastes.
17. Lasers in pulpotomy.
18. Electrosurgical pulpotomy.
19. List the obturating materials used in primary teeth. [Same as SN Q.2]

SOLVED ANSWERS
LONG ESSAYS:
Q.1. Describe briefly about pulpotomy. iv. There is no evidence of internal resorption, inter-
radicular bone loss, abscesses or fistulas.
Ans.
v. The haemorrhage from the amputation site is easy
to control.
(SN Q.1 and SE Q.2, SE Q.8) Contraindications:
l {(Pulpotomy is defined as the surgical removal of the
i. The tooth crown is nonrestorable and tender on
entire coronal portion of the pulp presumed to be par- percussion.
tially or totally inflamed, followed by placement of suit- ii. Highly viscous haemorrhage seen at the radicular
able medicament either to heal or preserve (fixation) the canal orifices.
remaining vital radicular pulp within the canals under iii. Mobility or radiolucency with marked root resorp-
aseptic conditions. tion exists.
l Pulpotomy is a procedure for teeth with healthy pulps or iv. Persistent toothaches and coronal pus.}
teeth with symptoms of reversible pulpitis and deep car- Diagnosis:
ies. Radiographically, the tooth should not show signs Irreversible pulpits limited to the coronal pulp.
of pathological resorption or radiolucency.
[SE Q.8]
l Sweet developed a multiappointment pulpotomy proce-

dure in 1930s; it has become popular and is predecessor {Technique:


to currently used single visit pulpotomy. l After achieving adequate anaesthesia and rubber
l The current technique followed is the use of diluted dam placement, all the superficial caries should be
Buckley’s formocresol technique for 5 min in a single removed before pulpal exposure.
visit.)} l The roof of the pulp chamber should be removed by

joining the pulp horns with bur and the coronal pulp
[SE Q.2] is amputated.
{Indications: l This procedure should be carried out carefully to

i. Vital teeth which is free of radicular pulpitis are prevent further damage to the pulp and perforation to
considered suitable for pulp capping. the pulpal floor.
ii. Pain, if present, is neither spontaneous nor persistent. l Following coronal pulp amputation, one or more cot-

iii. The tooth is restorable and possesses at least two- ton pellets should be placed over each amputation site
thirds of its root length. and pressure should be applied for a few minutes.
Section | I  Topic-Wise Solved Questions of Previous Years 535

l When the cotton pellets are removed, haemostasis Electrosurgery:


should be apparent. l Controlled energy in the form of electrosurgical
l Excessive bleeding that persists in spite of cotton heat application to the pulp stumps at the canal
pellet pressure and a deep purple colour of tissue orifice site has been used for pulpotomy.
may indicate inflammation has extended to the ra- Lasers
dicular pulp. l Lasers have been suggested for a number of
l Following haemostasis, a cotton pellet dipped in procedures in dentistry including pulpotomy.
Buckley’s formocresol solution (one-fifth dilution Nd:YAG, diode, CO2, argon, Er:YAG lasers all
or full strength) is placed over the pulp stumps for have been tried as an alternate to formocresol
5 min. pulpotomy.
l A base of zinc oxide eugenol is placed over the am-

putation site and lightly condensed to cover the Q.2. Write briefly about the apexification procedure.
pulpal floor. Ans.
l A second layer is then condensed to fill the access

opening completely. Apexification:


l ‘Apexification is defined as chemically induced root
l The final restoration should, preferably, be a stainless

steel crown.} formation by calcium hydroxide or CMCP in nonvi-


tal immature, blunderbuss canals of young perma-
nent teeth’.
{SN Q.3} l Calcium hydroxide has become the most widely used

material to promote apexification.


Agents commonly used for pulpotomy are as follows: l Mineral trioxide aggregate (MTA) is one of the most
i. Formocresol recently popularized materials for this purpose.
ii. Glutaraldehyde Apexogensis:
iii. Ferricsulphate ‘Apexogenesis is defined as the vital pulpotomy proce-
iv. Electrosurgery dure undertaken for young permanent teeth with
v. Lasers calcium hydroxide which lays down a dentinal bridge
thereby allowing the radicular portion of the pulp for
root elongation or lengthening’.
Formocresol: Technique:
l Buckley’s formocresol is used in this technique l As the tooth is nonvital, it may not be necessary

which contains 35% cresol, 19% formalin in a to use local anaesthesia.


vehicle of glycerine and water at a pH of approxi- l An access opening is made and then the pulp is

mately 5.1. extirpated.


l To dilute this to one-fifth strength, thoroughly mix l Canals are irrigated and dried.

three parts of glycerine with one part of distilled l Injectable calcium hydroxide is filled inside the

water and add these four parts to one part of con- root canals and entrance filling is done with GIC.
centrated commercial formocresol compound. l Postoperatively a radiograph is taken to check the

l Add these four parts to one part of concentrated intracanal calcium hydroxide filling.
commercial formocresol compound. The stopera- l Periodic recall is scheduled at 2 weeks, 3 months

tive effect of formocresol pulpotomy on the pri- and 6 months.


mary teeth, and also the succedaneous premolars, l Usually it takes around 6 months for barrier to

is mostly without concrete evidence. Formocresol form.


has been found to be cytotoxic. l Once the barrier formation is confirmed clinically

Glutaraldehyde: by passing an endodontic instrument, obturation


l 2% glutaraldehyde could be an alternative pulp- of the tooth is done with gutta-percha.
otomy fixative medicament. l A postendodontic jacket crown is done or an in-

l It has better fixative properties with true cross terim composite restoration is done depending on
linking and effective disinfecting properties. the age of the patient.
l It has lower cytotoxicity compared to formocresol. l Recently, a single visit apexification has become

Ferric sulphate: popular with the advent of MTA.


l Ferric sulphate causes early exfoliation of the l This material can be mixed and placed at the open

treated teeth and has the same radiographic signs apex with special carriers.
of failure like calcific metamorphosis and internal l It usually sets within 4 h and then the obturation

resorption. with gutta-percha can be carried out on the next day.


536 Quick Review Series for BDS 4th Year, Vol 1

Follow-up of pulp treated tooth: vii. Mineral trioxide aggregate (a mixture of tricalcium
l All pulpally treated teeth should be observed and aluminate, tricalcium silicate, silicateoxide, tricalcium
assessment should be made every 6 months. oxide and bismuth oxide)
l Any abnormalities like mobility, pain, swelling, viii. Endoflas (a mixture of barium sulphate, calcium
sinus or a fistula on the gingiva adjacent to the hydroxide, iodoform and zinc oxide eugenol)
tooth, persistent tenderness to percussion should I. Zinc oxide eugenol:
be clinically managed. l Eugenol is obtained primarily from the oil of clove.
Difference between apexogensis and apexification: l Eugenol is treated with hydrochloric acid and washed

until a purified preparation is obtained.


[SE Q.1]
l Zinc oxide is produced by exposing a zinc mineral to
l {Apexogenesis is defined as the vital pulpotomy high temperature.
procedure undertaken for young permanent teeth l The litoberated volatile zinc is oxidized to form zinc
with calcium hydroxide which lays down a den- oxide.
tinal bridge thereby allowing the radicular portion l It is insoluble in water.
of the pulp for root elongation/lengthening, l It acts as mild astringent and antiseptic.
whereas apexification is chemically induced in II. Calcium hydroxide
nonvital or immature permanent teeth. l Calcium hydroxide is one of the most commonly

used agents for root canal filling.


Apexogenesis Apexification l Pure calcium hydroxide has a pH of 11.
i. It is defined as the treat- i. It is defined as induction l Commercially, Dycal is the most commonly used in
ment of vital pulp to per- of apical closure of a non- India and it has a neutral pH.
mit growth of root and vital young permanent
Other products are Pulpdent and Hydrex.
closure of apex. tooth.
l Pulp in direct contact with calcium hydroxide (alka-
ii. Only coronal pulp tissue ii. The entire pulp is removed line pH) becomes necrotic after 24 h.
is removed radicular pulp thoroughly.
l Partially calcified fibrous tissue lined by odonto-
is prevented from damage.
blasts is seen below the calcium proteinate zone in
iii. Ca(OH)2 is placed over iii. The entire pulp chamber 14 days.
amputated root stumps. is filled with Ca(OH)2 .
l A zone of new dentin is observed within 28 days.
iv. Ca(OH)2 lays down a iv. Ca(OH)2 induces forma- l Dycal was introduced in 1962 as two paste system a
dentinal bridge. tion of an apical barrier. base and a catalyst and it has neutral pH of 7.
v. The apex is formed by v. Apex formation is due to l Histological changes vary with Dycal due to neutral
normal growth of roots. formation of osteocemen- pH.
tum/bone like tissue.} l Pulp in direct contact with material becomes necrotic

and the necrotic material is removed by macro-


Q.3. Enumerate different root canal filling materials phages. Granulation tissue is seen in the layer.
used in paedodontics. Describe any one in detail. l Odontoblasts are seen differentiating in the granula-

Ans. tion tissue and form dentin.


l The dentin formed is in contact with Dycal makes it
Some of the most commonly used materials for obturation difficult to visualize the dentin bridge from the radi-
of root canals in primary teeth are as follows: opaque material.
Q.4. Describe your treatment plan when you are en-
{SN Q.2} countered with a small traumatic exposure while per-
i. Unrenreinforced zinc oxide eugenol forming a cavity in primary molar.
ii. Calcium hydroxide (calcicure) Ans.
iii. Calcium hydroxide with iodoform (Vitapex or
Metapex) [SE Q.6]
iv. KRI paste (a mixture of para-chlorophenol, cam-
phor, menthol and iodoform) l {Direct pulp capping is the treatment of choice for a
v. Walkhoff paste (a mixture of parachlorophenol, small traumatic exposures caused while performing a
camphor andmenthol) cavity preparation in primary teeth.
vi. Maisto paste (a mixture of zinc oxide, iodoform, l ‘Direct pulp capping is defined as the treatment of an

thymol, chlorophenol, camphor and lanolin) exposed vital pulp by sealing the pulpal wound with a
dental material placed directly on a mechanical or
Section | I  Topic-Wise Solved Questions of Previous Years 537

traumatic exposure to facilitate the formation of re- l The 2nd stage treatment should be the conven-
parative dentin and maintenance of the vital pulp.’ tional root canal therapy, once the apices are
Or closed.
l ‘Direct pulp capping is defined as 1 mm2 or less than
Q.5. Write briefly on formocresol pulputomy.
1 mm2 mechanical exposure during cavity preparation
or caries or trauma left behind with a sound surrounding Ans.
dentin and dressed with a biocompatible radiopaque
l Pulpotomy is defined as the surgical removal of the en-
base in contact with the exposed pulp and should heal
tire coronal portion of the pulp presumed to be partially
the tissue and deposit the reparative dentin prior to plac-
or totally inflamed, followed by placement of suitable
ing the restoration.’
medicament either to heal or preserve (fixation) the re-
l While performing a cavity in primary molar, if a trau-
maining vital radicular pulp within the canals under
matic exposure is encountered, a series of various steps
aseptic conditions.
need to be followed.
l Pulpotomy is a procedure for teeth with healthy pulps or
l Cvek has shown that, in exposures resulting from trau-
teeth with symptoms of reversible pulpitis and deep car-
matic incidence, pulpal changes are characterized by a
ies. Radiographically, the tooth should not show signs
proliferative response, with inflammation extending
of pathological resorption or radiolucency.
only a few millimetres into the pulp.
l When the hyperplastic tissue is removed, healthy pulp is
seen. {SN Q.4}
l In cases with carious exposure, it might be necessary to
Formocresol pulpotomy:
remove tissue to a greater depth in order to reach nonin-
Formocresol was introduced by Buckley in 1904 and
flamed pulp.
since then a lot of modifications have been tried and
l Cutting of the tissue with a diamond bur using high
advocated regarding the techniques of formocresol
speed with water cooling is the best method.
pulpotomies.
l After pulp amputation, saline is used for irrigation and
l Sweet (1930): formulated multivisit technique.
the haemorrhage is controlled with cotton pellets.
A multiappointment pulpotomy procedure in-
l A dressing of calcium hydroxide is gently placed on the
troduced by Sweet has become popular and is
amputation site. Care should be taken not to push the
predecessor to currently used single visit pulp-
material into the pulp.
otomy.
l A temporary restoration is placed first and after deter-
l Doyle (1962): advocated two sitting procedure
mining the success of pulp capping through determina-
(complete devitalization).
tion of dentinal bridge, maintenance of pulp vitality,
l Spedding (1965): gave 5 min protocol (partial
lack of pain and minimal inflammatory response the fi-
devitalization).
nal restoration is carried out.
l Venham (1967): proposed 15 s procedure.
l An entrance filling is done using GIC or composite.
l Current concept uses 4 min of application
l Recently MTA (mineral trioxide aggregate) has been
used to carry out these procedures.} time.
The formocresol used in the technique may be
On follow-up examination:
obtained under the trade name Buckley’s
l Apical development is seen by comparison with
formocresol.
the preoperative radiograph (IOPA).
l Formation of calcific bridge, continued apical

development, absence of internal resorption and


periapical radiolucency are evidences of success.
l It is referred to as nature’s efforts at root filling. l Composition of formocresol: Buckley’s formula:
l Pulp remnants slowly become nonvital. cresol – 35%, formaldehyde – 19%, glycerol – 15%
l Bacteria might migrate into these root canal and water – 31%.
spaces and periapical pathology might result. l Currently we use one-fifth concentration of Buckley’s

l Once the canal calcifies to a great extent, it may formula, which is prepared by the following method:
be impossible to negotiate it with instruments, l To dilute Buckley’s formocresol to one-fifth strength,

even the use of chelating agents like EDTA (eth- thoroughly mix three parts of glycerine with one part
ylene diamine tetra-acetic acid) might not help. of distilled water. Add these four parts to one part of
l Calcium hydroxide pulpotomy can thus be con- concentrated commercial formocresol compound to
sidered as the 1st stage of treatment for vital, obtain formocresol of one-fifth strength.
cariously or traumatically exposed, permanent l The glycerine is added to prevent polymerization of

teeth with incompletely formed apices. formaldehyde to paraformaldehyde.


538 Quick Review Series for BDS 4th Year, Vol 1

l Once fixation of pulp tissue is achieved, zinc oxide


{SN Q.4}
eugenol is placed in pulp chamber over the amputa-
Mechanism of action: tion site and lightly condensed to cover the pulpal
l It prevents tissue autolysis by bonding to the pro- floor.
teins. This bonding is of peptide groups of side l A second layer is then condensed to fill the access

chain amino acids and is a reversible process ac- opening completely.


complished without changing the basic structure l After 1 week, recall the patient and restore with per-

of protein molecules. manent restorations if they are asymptomatic.


l The final restoration should, preferably, be a stainless

steel crown.}
Histological changes l Pulpotomized primary molars can be successfully
Mass and Zilbermann in 1933 and also Massler and restored with one surface amalgam, if their natural
Mansokhani in 1959, demonstrated the histologic exfoliationis expected within not more than 2 years.
changes. l For all other cases, including multisurface restora-
i. Immediately: tions stainless steel crowns are the treatment of
Pulp becomes fibrous and acidophillic. choice.
ii. In 7–14 days:
Q.6. Describe the procedure in treating a young perma-
Three zones appear:
nent fractured central incisor with wide apical foramen
l A broad eosinophilic zone of fixation
and necrotic pulp.
l A broad pale-staining zone of atrophy with

poor cellular definition Ans.


l A broad zone of inflammation extending
For treating a young permanent central incisor with wide
apically into normal pulp tissue
apical foramen and necrotic pulp a procedure known as
iii. After 1 year:
apexification needs to be followed which can be described
Progressive apical movement of these zones with
as follows:
only acidophilic zone left at the end of 1 year.
l Apexification is defined as chemically induced root
Technique:
formation by calcium hydroxide or CMCP in nonvital
[SE Q.8] immature, blunderbuss canals of young permanent
teeth.

l {After achieving adequate anaesthesia and rubber l It is a method of inducing apical closure by formation
dam placement, all the superficial caries should be of a mineralized tissue in the apical region of a nonvital
removed before pulpal exposure. permanent tooth with an incompletely formed root

l The roof of the pulp chamber should be removed by apex.
joining the pulp horns with bur and the coronal pulp l It is defined by Cohen as a method of induce develop-
is amputated. ment of the root apex of an immature pulpless tooth by

l Sharp spoon excavators are used to scoop out coronal formation of osteocementum/bone like tissue.
pulp and pulp remnants. l The main objective of apexification is to induce either

l This procedure should be carried out carefully to closure of open apical third of root canal or the forma-
prevent further damage to the pulp and perforation to tion of an apical calcific barrier against which obtura-
the pulpal floor. tion can be achieved.

l Clean the pulp chamber with saline and remove all Technique
debris. i. Use of local anaesthesia depends on patient
l Following coronal pulp amputation, one or more response as the pulp is necrosed.
cotton pellets should be placed over each amputa- ii. Apply the rubber dam.
tion site and pressure should be applied for a few iii. Make an access opening and determine the work-
minutes. ing length.

l When the cotton pellets are removed, haemostasis iv. Irrigate the canals with saline and dry them.
should be apparent. v. Injectable calcium hydroxide is injected inside the

l Following haemostasis, a cotton pellet dipped in canal and temporary restoration is done.
Buckley’s formocresol solution (one-fifth dilution or vi. Post-treatment radiograph is taken to check the in-
full strength) is placed over the pulp stumps for 4 min. tracanal calcium hydroxide.

l Place a small dry cotton pellet over it to avoid con- vii. Periodic recall is scheduled at 2 weeks, 3 months
tact of tissues with formocresol. and 6 months.
Section | I  Topic-Wise Solved Questions of Previous Years 539

viii. Usually it takes around 6 months for barrier to the ‘so-called inaccessible canals’ rather than the con-
form. ventional ‘shaping’ of the canals.
ix. Once the barrier formation is confirmed clinically Indications:
by passing an endodontic instrument, obturation of l Primary tooth with irreversibly inflamed pulp or

the tooth is done with gutta-percha. necrotic pulps and having minimum, i.e. not more
l Depending on the age of the patient, a post- than one-third of root resorption.
endodontic jacket crown is done or an interim l Primary tooth with an abscess or sinus opening.

composite restoration is done. l Primary anterior tooth with atraumatic fracture or

l Recently, a single visit apexification can also be caries involving the pulp irreversibly.
done with the advent of MTA. This material can l No pathologic resorption associated with apical re-

be mixed and placed at the open apex with spe- gion of root.
cial carriers. l Teeth exhibiting internal resorption with no visible

l It usually sets within 4 h and then the obturation perforation.


with gutta-percha can be carried out on the next l Presence of pus at the exposure site or in the pulp

day. chamber.
Follow-up of pulp treated tooth: l Pulpless primary teeth without permanent successor.

l All pulpally treated teeth should be monitored l Pulpless primary second molars before the eruption

regularly. of the permanent first molars.


l A clinical and radiographic assessment should be l Children suffering from haemophilia.

made every 6 months. l Teeth adjacent to the line of a palatal cleft.

l Any abnormalities such as mobility, or pain and l Primary molars where arch length is deficient or in

swelling, a sinus or a fistula on the gingiva adja- case they support orthodontic appliances.
cent to the tooth, persistent tenderness to percus- l Handicapped children where monitoring of space

sion should be managed. maintainers or continuous supervision is not


Frank’s criteria for apexification are as follows: possible.
i. Apex is closed, through minimum recession of l Pulpless primary anterior teeth when speech,

the canal. crowded arches or aesthetics is a factor.


ii. Apex is closed with no change in root space. Contraindications:
iii. Radiographically apparent calcific bridge at the l Primary tooth with excessive mobility

apex. l A primary tooth with a irreversibly inflamed pulp

iv. There is no radiographic evidence of apical clo- with excessive root resorption involving more than
sure but upon clinical instrumentation there is two-thirds of the root
definite stop at the apex, indicating calcific repair. l Tooth with internal resorption

l A nonrestorable tooth with insufficient tooth struc-


Q.7. Define pulpectomy. What are the indications and ture to support rubber dam
contraindications for pulpectomy? Write the step-by-step l Teeth with mechanical or carious perforations of the
procedure for pulpectomy in 85. floor of the pulp chamber
Ans. l Primary teeth associated with underlying dentiger-

ous or follicular cysts


l Mathewson (1995) defined pulpectomy as the complete l Young patients with systemic illnesses such as
removal of the necrotic pulp from the root canals of congenital or rheumatic heart disease, hepatitis or
primary teeth and filling them with an inert resorbable leukaemia
material so as to maintain the tooth in the dental arch. l Children on long-term corticosteroid therapy or
l Finn defines pulpectomy as removal of all pulpal tissue those who are immune-compromised
from the coronal and radicular portions of the tooth. Technique:
l ‘Pulpectomy involves removal of the nonvital cariously l The child with necrotic primary tooth can present
exposed roof of the pulp chamber and contents of the with varied clinical signs and symptoms ranging
chamber in order to gain access to the root canals which from being absolutely asymptomatic to an acutely or
are debrided, enlarged and disinfected followed by the chronically abscessed, mobile and painful, teeth with
obturation with the resorbable material.’ swollen periodontal tissues.
l The goal of pulpectomy procedure is to eliminate the l In cases of nondraining alveolar abscess and celluli-
bacteria and the contaminated pulp tissue from the canal. tis from odontogenic origin, antibiotic therapy using
l In primary teeth, more emphasis has been given on dis- first- or second-generation penicillin for a period of
infection and removal of necrotic pulp remnants from 5–7 days must be the first line of treatment.
540 Quick Review Series for BDS 4th Year, Vol 1

The steps involved in pulpectomy are as follows: l Avoid excessive cleaning and shaping as it may
i. Application of rubber dam and securing anaesthesia lead to lateral wall perforations or floor perfora-
ii. Access opening tions.
iii. Working length determination l The purpose of filling is to remove the pulp tis-

iv. Pulp extirpation sue from the canals to create space for the pri-
v. Biomechanical preparation mary teeth obturating material.
vi. Obturation vi. Obturation:
i. Application of rubber dam and securing anaesthe- l Obturation of primary teeth is done with a re-

sia: sorbable material which will give way for the


l After securing profound anaesthesia and place- erupting permanent tooth. Calcium hydroxide
ment of the rubber dam for isolation, all caries is with iodoform comes closest to the ideal obturat-
removed with a slow-speed round bur or a sharp ing material.
spoon excavator. l Following the pulp extirpation and biomechani-

l The pulp chamber is carefully opened to relieve cal preparation, irrigate and debride the canals
any pressure from the infected pulp, with a high- are dry them with paper points.
speed bur. l Start obturating the canals

ii. Access opening: a. First a thin mix of zinc oxide eugenol is used
l The access opening is refined to make a straight to coat the walls of the canals using a reamer.
line access with slight flaring to allow ease of b. First the reamer is rotated clockwise inside
Insertion of the files. the canals for 10–15 rotations keeping in
l When access opening is made the bur penetrates mind not to extend beyond the working
the enamel and dentin and enters the chamber by length.
penetrating the roof. c. Use thick mix of zinc oxide eugenol and fill
l Using a straight fissure bur or a round diamond the canals using lentulospirals.
bur, attempt should be made to remove the entire d. Once filling of root canals is completed, seal
roof of the pulp chamber to facilitate access to the pulp chamber with temporary restoration.
all the root canal orifices and also to access the e. Recall after a week days and if patient is to-
necrotic pulp tissue. tally asymptomatic do a final restoration and
Access openings for various teeth: give stainless steel crown.
a. Primary anterior teeth:
An access opening can be successfully made Various obturation techniques are as follows:
from the facial surface with more extension to A. Obturation using freshly mixed zinc oxide eugenol
the incisal edge. i. Using reamer
b. Primary posterior teeth: ii. Using wet cotton
The access openings are essentially same as iii. Using lentulospirals
that of permanent teeth, except that the length B. Obturation using injectable syringes:
of the crown is short and primary teeth have The materials used with this technique are
very thin dentinal walls in the root canals and i. Metapex (calcium hydroxide and iodoform
the floor. The depth necessary to penetrate into paste)
the pulp chamber is much less than that in the ii. Vitapex (calcium hydroxide and iodoform paste)
permanent teeth. iii. Calcicur (calcium hydroxide paste)
iii. Working length determination: C. Endodontic pressure syringe:
l The working length of the root canals is deter- It was designed by Greenberg and Katz. Using this
mined from a radiograph with an endodontic file syringe, a very thick mix of zinc oxide eugenol can
inside the canal. be forced through an extremely narrow gauge needle.
vi. Pulp extirpation: The flexibility of the needle allows them to be bent
l The entire accessible coronal and radicular pulp to reach the apex, of even inaccessible posterior
tissue should be removed with broaches and teeth.
Hedstroem or K files. Other methods:
v. Biomechanical preparation: D. Jiffy tube:
l File the canals, progressively increasing the file The regular mix of zinc oxide eugenol is backloaded
diameter and complete the biomechanical (BMP) into the tube. Then the tube tip is placed into the canal
preparation. orifice and the material expressed into the canal with
Section | I  Topic-Wise Solved Questions of Previous Years 541

a downward squeezing motion until the orifice Advantages of glutaraldehyde over formocresol are as
appeared visibly filled. follows:
E. Tuberculin syringe: l It is excellent antimicrobial.
Syringe utilized by Aylord and Johnson in 1987 was l Causes less necrosis of the pulpal tissue.
a standard 26 gauge, three-eighth inch needle. l ‘However, it was found that glutaraldehyde is less
Various materials used for obturation of primary teeth are toxic than formocresol’.
l Unreinforced zinc oxide eugenol l It has better fixative properties with true cross
l Calcium hydroxide (calcicure) linking, the larger molecule size than formalde-
l Calcium hydroxide with iodoform (Vitapex or Metapex) hyde will prevent it to diffuse out of the apical
l KRI paste (a mixture of para-chlorophenol, camphor, foramen, effective disinfecting properties and also
menthol and iodoform) has a low mutagenicity.
l Walkhoff paste (parachlorophenol, camphor and l Antigenecity is also less as compared to formo-
menthol) cresol.
l Maisto paste (zinc oxide, iodoform, thymol, chloro- l It has lower cytotoxicity compared to formocre-
phenol, camphor and lanolin) sol; no damage to the erupted permanent teeth.
l Causes less dystrophic calcification in pulp canals.
Q.8. What is pulpotomy? Discuss the reaction of pulp to
l Demonstrates less systemic distribution.
glutaraldehyde, formocresol and calcium hydroxide.
l It is low tissue binding, readily metabolized,
Ans. eliminated in urine and expired in gases – 90% of
the drug is gone in 3 days.
l ‘Pulpotomy is defined as the surgical removal of the
Disadvantages of glutaraldehyde:
entire coronal portion of the pulp presumed to be par-
l Success rate is low compared to formocresol
tially or totally inflamed and quite possibly infected at
pulpotomies.
the amputation site, followed by placement of suitable
l A potential drawback is the poor shelf life of the
medicament either to heal or preserve (fixation) the re-
product.}
maining vital radicular pulp within the canals, under
Histological changes or reaction of pulp to calcium hy-
aseptic conditions’.
droxide:
l Most commonly used agents for pulpotomy are formo-
l Pulp in direct contact with calcium hydroxide (alka-
cresol and glutaraldehyde.
line pH) becomes necrotic after 24 h.
[SE Q.9] l Necrotic layer is separated from healthy tissue

by a deep staining basophilic material-calcium


{Histological changes following formocresol pulpotomy: proteinate.
Mass and Zilbermann in 1933 and also Massler and l Partially calcified fibrous tissue lined by odonto-
Mansokhani in 1959 demonstrated the histologic blasts is seen below the calcium proteinate zone in
changes following formocresol pulpotomy as follows: 14 days.
i. Immediately: l A zone of new dentin is observed within 28 days.
Pulp becomes fibrous and acidophillic.
ii. In 7–14 days: Reaction of pulp to Ca(OH)2 can be summarized as
l After a 7–14 day application, the pulp devel- below:
oped three distinctive zones:
i. A broad eosinophilic zone of fixation
{SN Q.11}
ii. A broad pale-staining zone with poor cellular
definition After 1 day:
iii. A zone of inflammation diffusing apically into l A deep staining basophilic layer appears between
normal pulp tissue healthy pulp tissue and necrotic zone adjacent to
iii. After 60 days: Ca(OH)2.
l After 60 days, the remaining tissue is com- After 7 days:
pletely fixed, and appears as a strand of eosino- l Increase in cellular and fibroblastic activity.
philic fibrous tissue. After 14 days:
Histological picture of dental pulp after a glutaraldehyde l Necrotic zone disappears and a partly calcified
pulpotomy: fibrous tissue lined by odontoblastic cells is seen
In 1975s, Gravenmade and Dankert et al. proposed that below Ca(OH)2 zone.
2% glutaraldehyde could be an alternative pulpotomy After 28 days:
fixative medicament. l A layer of newly formed dentin.
542 Quick Review Series for BDS 4th Year, Vol 1

Q.9. Write the indications, contraindications and proce- Q.19. Define pulpotomy. Describe the histological pic-
dure of pulpotomy in a deciduous molar. ture of dental pulp after a glutaraldehyde pulpotomy.
Why is glutaraldehyde preferred over formocresol?
Ans.
Ans.
[Same as LE Q.1]
[Same as LE Q.8]
Q.10. Define pulpotomy. Describe the indications, con-
traindications and procedure of pulpotomy.
Ans.
SHORT ESSAYS:
[Same as LE Q.1] Q.1. Apexogenesis versus apexification.
Q.11. Write briefly on formocresol pulputomy. Ans.
Ans. [Ref LE Q.2]
[Same as LE Q.1] Q.2. Indications and contraindications of pulpotomy.
Q.12. Define pulpotomy. Describe the technique and Ans.
pulpal tissue changes following formocresol pulpotomy
[Ref LE Q.1]
in primary molar.
Q.3. Explain vitality test and describe various methods.
Ans.
Ans.
[Same as LE Q.1]
Q.13. Define pulpotomy. Mention the step-by-step
formocresol pulpotomy procedure in primary teeth. {SN Q.5}
Ans. The pulp vitality testing includes accurate measurement
of the vitality of the tooth to establish the vitality of
[Same as LE Q.1] normal teeth to that of traumatized teeth or affected by
Q.14. What is the difference between apexogensis and dental caries.
apexification? Methods used for pulp vitality testing are
i. Electric pulp testing
Ans. ii. Thermal tests
[Same as LE Q.2] iii. Pulse oximeter
iv. Laser Doppler flowmetry
Q.15. Direct pulp capping. v. Dual wavelength spectrophotometry
Ans. vi. Test cavity
[Same as LE Q.4]
Q.16. Define pulpotomy. Describe the technique and i. Electric pulp testing:
l The procedure must be explained to the patient be-
pulpal tissue changes following formocresol pulpotomy
in primary molar. fore doing electric pulp testing.
l First dry the teeth to be tested and isolate them with
Ans. cotton rolls.
l Cover the tip of the electrode with toothpaste or a
[Same as LE Q.5]
similar electrical conductor and to complete the circuit,
Q.17. Define pulpotomy. Mention the step-by-step a ground attachment may be clipped on patient’s lip.
formocresol pulpotomy procedure in primary teeth. l Then the electric current slowly is passed through the

Ans. tooth using the control in the pulp tester.


ii. Thermal tests:
[Same as LE Q.5] Two types of thermal tests are available:
Q.18. A 5-year-old patient comes in your clinic with a a. Cold
badly carious non restorable mandibular 2nd deciduous b. Hot stimuli
molar. Give your treatment plan. Cold testing:
l It can be made with an air blast, a cold drink,
Ans. an ice stick, ethyl chloride or fluorimethane
[Same as LE Q.7] spray or a carbon dioxide ‘ice stick’.
Section | I  Topic-Wise Solved Questions of Previous Years 543

Hot testing: vi. Radiopaque


l Hot testing can be made with a stick of heated vii. Should not discolour the tooth
gutta-percha or hot water. l Calcium hydroxide with iodoform comes closest to the

l When using cold stimulus, one must try to de- ideal obturating material.
termine if the effect of stimulus application l Nonresorbable materials such as gutta-percha or silver

produces a lingering effect or if the pain sub- points are contraindicated as they will not enhance the
sides immediately on removal of the stimulus primary root physiologic resorptive process.
from the tooth. Various materials used for obturation of primary teeth
l In testing, if the pain lingers, that is taken as are
evidence for irreversible pulpitis. If pain sub- l Unreinforced zinc oxide eugenol

sides immediately after stimulus removal, hy- l Calcium hydroxide (calcicure)

persensitivity or reversible pulpitis is the more l Calcium hydroxide with iodoform (Vitapex or

likely diagnosis. Metapex)


Direct measurement of the pulpal circulation: l KRI paste (a mixture of para-chlorophenol, cam-

l Direct measurement of the pulpal circula- phor, menthol and iodoform)


tion is the real measure of its vitality. l Walkhoff paste (parachlorophenol, camphor and

l Laser Doppler flowmetry, dual wavelength menthol)


spectrophotometry and pulse oximetry all l Maisto paste (zinc oxide, iodoform, thymol, chlo-

establish the same by a noninvasive subjec- rophenol, camphor and lanolin)


tive method as compared to the traditional
Q.5. Glutaraldehyde.
objective techniques like electric pulp tester
or thermal testing. Ans.
l Laser Doppler flowmeter has also been
‘Pulpotomy is defined as the surgical removal of the entire
shown to measure pulpal blood flow thereby
coronal portion of the pulp presumed to be partially or to-
assessing the vitality of the tooth.
tally inflamed, followed by placement of suitable medica-
l Pulse oximeter probes tend to assess the
ment either to heal or preserve (fixation) the remaining vital
oxygen saturation of a tooth and thereby
radicular pulp within the canals’.
indicating the status of the pulp.
Glutaraldehyde:
Test cavity:
l Two per cent glutaraldehyde could be an alternative
l This procedure is used as a last resort when
pulpotomy fixative medicament.
the other tests are inconclusive.
l It has better fixative properties with true cross-linking,
l This procedure involves slow removal of
effective disinfecting properties.
enamel and dentin through an existing resto-
l It has lower cytotoxicity compared to formocresol.
ration, using a small round bur to test vitality
Technique:
of the tooth.
l After achieving adequate anaesthesia and rubber
l Once the bur reaches the dentinoenamel
dam placement, all the superficial caries should
junction, patient perceives sensitivity or
be removed before pulpal exposure.
sharp pain indicating the vitality of the tooth.
l The roof of the pulp chamber should be removed

Q.4. What are the requirements of root canal obturation by joining the pulp horns with bur.
materials for primary teeth? l The coronal pulp is then amputated using either a

sharp spoon excavator or a slowly revolving round


Ans.
bur leaving an intact vital radicular pulp.
l Obturation of primary teeth is done with a resorbable l This procedure should be carried out carefully to

material, so that it resorbs and gives way for the erupt- prevent further damage to the pulp and perfora-
ing permanent tooth. tion to the pulpal floor.
l Ideal requirements of a root filling material for primary l Following coronal pulp amputation, one or more

teeth are as follows: cotton pellets should be placed over each amputa-
They should be tion site and pressure should be applied for a few
i. Resorbable minutes.
ii. Nonirritating to the underlying permanent tooth l When the cotton pellets are removed, haemostasis

germ should be apparent.


iii. Noninflammatory l Using cotton pellets, apply 2% glutaraldehyde to

iv. Having antiseptic properties the pulp and then remove cotton pellets and check
v. Easily inserted and removed for fixation.
544 Quick Review Series for BDS 4th Year, Vol 1

l A base of zinc oxide eugenol is placed over the


{SN Q.13}
amputation site and lightly condensed to cover the
pulpal floor. A variety of materials used for direct pulp capping are
l Glutaraldehyde solution might replace formocre- as follows:
sol in endodontics, because it appears to have l Calcium hydroxide

fixative properties with less destruction of tissue l Zinc oxide eugenol

and at the same time appears to be bactericidal. l Corticosteroids and antibiotics

l Disadvantages of glutaraldehyde are that: l Polycarboxylate cements

l Success rate is low compared to formocresol l Tricalcium phosphate cement

pulpotomies. l Cyanoacrylate

l A potential drawback is the poor shelf life of l Collagen

the product. l 4-META (4-methacryloxyethy ltrimellitate anhy-

l It appears that it is not an ideal material to dride)


replace formocresol. l MTA (mineral trioxide aggregate)

Q.6. Direct pulp capping.


Q.7. Define indirect pulp capping. Mention objectives
Ans. indications and contraindications.
Ans.
{SN Q.12}
l Direct pulp capping is the treatment of choice for a {SN Q.7}
small traumatic exposures caused while performing
a cavity preparation in primary teeth. l ‘Indirect pulp capping is defined as a procedure in
l ‘Direct pulp capping is defined as the treatment of an
which a material is placed on a thin partition of
exposed vital pulp by sealing the pulpal wound with remaining carious dentin that, if removed, might
a dental material placed directly on a mechanical or expose the pulp.’
l Indirect pulp therapy (IPT) is recommended for
traumatic exposure to facilitate the formation of re-
parative dentin and maintenance of the vital pulp.’ teeth that have deep carious lesions approximating
l Direct pulp capping is a treatment option for teeth
the pulp but no signs and symptoms of pulp degen-
with traumatic or mechanical pulp exposures inad- eration.
l It is performed on a lesion where it is anticipated that
vertently occurred during an operative procedure.
l It should be attempted only when the pulp is vital
the pulp would be exposed when the caries is exca-
and does not show any signs or have any symptoms vated completely.
of irreversible pulpitis. Objectives:
l A calcium hydroxide medicament is placed over the
The ultimate objective of this procedure is to preserve
exposure site to stimulate dentin formation and thus pulp vitality by:
l Arresting the carious process
‘heal’ the wound and maintain the vitality of the
l Promoting dentin sclerosis
pulp.
l Stimulating the formation of tertiary dentin

and
l Remineralizing the carious dentin
l MTA (mineral trioxide aggregate) can also be used as an
alternative material.
l A temporary restoration is placed first and after deter- Indications:
mining the success of pulp capping through determina- l Mild discomfort from chemical or thermal stimuli
tion of dentinal bridge, maintenance of pulp vitality, and on eating
lack of pain and minimal inflammatory response the l Negative history of spontaneous or nocturnal pain
final restoration is carried out. l A deep carious lesion radiographically close to the
l An entrance filling is done using GIC or composite. pulp in vital primary or young permanent teeth
l An important criterion for the case selection in these l Normal colour of the tooth
cases, in addition to absence of pain, is either no bleed- l Normal appearance of adjacent gingiva with radio-
ing from the exposure site or bleeding in an amount that graphic evidence of normal laminadura and peri-
would be considered normal in the absence of a hyper- odontal ligament space
aemic or inflamed pulp. l No periradicular or furcal radiolucency
Section | I  Topic-Wise Solved Questions of Previous Years 545

Contraindications: l High molecular weight compounds degrade to less


l Sharp continuous pain, persisting after the with- complex molecules that reflect less light.
drawal of the stimulus Bleaching techniques for nonvital teeth:
l Nocturnal pain i. Walking bleach:
l Tenderness on percussion In this technique, sodium perborate is placed inside
l Excessive tooth mobility the tooth over a protective filling like GIC, ZnPO4,
l Parulis in gingiva adjacent to the tooth polycarboxylate and the patient is evaluated after
l Discoloured tooth 2 weeks.
l Large carious lesion with obvious radiographic pulp ii. Thermocatalytic bleaching:
exposure Hydrogen peroxide is used followed by heat ap-
l Interrupted laminadura with widened periodontal plication.
ligament space iii. UV photo-oxidation:
l Peri-radicular or inter-radicular or furcal radiolu- H2O2 is placed in pulp chamber followed by expo-
cency sure to UV light.
l Pulpal calcifications
Q.11. Pulp vitality tests.
l Soft leathery dentin covering a very large area in the

cavity, in a nonrestorable tooth Ans.

Q.8. Describe formocresol pulpotomy. Add a note on [Same as SE Q.3]


Sweet’s formocresol pulpotomy. Q.12. Root canal filling materials used in 75.
Ans.
Ans.
[Ref LE Q.1]
[Same as SE Q.4]
Q.9. Reaction of pulp to glutaraldehyde and formocre-
Q.13. Obturating materials for primary teeth.
sol.
Ans.
Ans.
[Ref LE Q.8] [Same as SE Q.4]

Q.10. Bleaching of teeth. Q.14. Ideal requirements of obturating material of


primary teeth.
Ans.
Ans.
Bleaching is a cosmetic procedure resulting in whitening of
a tooth by using chemical agents that reduce the organic [Same as SE Q.4]
pigmentation in the tooth. Q.15. Glutaraldehyde pulpotomy.
Agents commonly used for bleaching are
i. Hydrogen peroxide Ans.
ii. Sodium perborate [Same as SE Q.5]
iii. Carbamide peroxide
i. Hydrogen peroxide:
l It is usually used in form of superoxol which is SHORT NOTES:
30% solution of H2O2 by weight and100% vol-
ume distilled water. Q.1. Define pulpotomy.
l It is colourless, odourless and unstable.
Ans.
l It is caustic and burns tissues on contact.

ii. Sodiumperborate: [Ref LE Q.1]


l It decomposes in the presence of acid, water air
Q.2. Root canal obturating materials used for deciduous
to form hydrogen peroxide. teeth.
l It is safer than H2O2.

iii. Carbamide peroxide: Ans.


l Also known as urea hydrogen peroxide.
[Ref LE Q.3]
l It is nontoxic and has no side effects.

Mechanism of bleaching: Q.3. Pulpotomy medicaments.


l The bleaching agents act on organic content of dental
Ans.
hard tissues and degrade them into chemical by prod-
ucts which are lighter in colour. [Ref LE Q.1]
546 Quick Review Series for BDS 4th Year, Vol 1

Q.4. Mention various types of formocresol pulpotomy. l An important criterion for the case selection in these
cases, in addition to absence of pain, is either no bleed-
Ans.
ing from the exposure site or bleeding in an amount that
[Ref LE Q.5] would be considered normal in the absence of a hyper-
aemic or inflamed pulp.
Q.5. Name four vitality tests.
Q.11. Histological changes or reaction of pulp to cal-
Ans.
cium hydroxide.
[Ref SE Q.3]
Ans.
Q.6. Formocresol.
[Ref LE Q.8]
Ans.
Q.12. Define direct pulp capping.
l The formocresol used for pulpotomy technique may be
Ans.
obtained under the trade name Buckley’s formocresol.
l Composition of commercially available Buckley’s for- [Ref SE Q.6]
mula contains 35% cresol, 19% formalin in a vehicle of
Q.13. Pulp capping agents.
glycerine and water at a pH of approximately 5.1.
l The glycerine is added to prevent polymerization of Ans.
formaldehyde to paraformaldehyde.
[Ref SE Q.6]
l Preparation of dilute formocresol solution: To dilute this

to one-fifth strength, thoroughly mix three parts of glyc- Q.14. KRI paste.
erine with one part of distilled water. Add these four
Ans.
parts to one part of concentrated commercial formocre-
sol compound. l It is one among some of the most commonly used ma-
terials for obturation of root canals in primary teeth.
Q.7. Indirect pulp capping.
l KRI paste is a mixture of para-chlorophenol, camphor,

Ans. menthol and iodoform.


[Ref SE Q.7] Q.15. MTA.
Q.8. Root canal irrigation. Ans.
Ans. l Mineral trioxide aggregate has been only recently rec-
ommended as a pulp capping agent in primary teeth.
Root canal irrigation is needed to:
l A number of human studies have demonstrated a better
i. Flush out root canal debris
success rates with MTA than formocresol over 6–7 months.
ii. Decrease bacterial count
l Pulp canal obliteration is a common finding but the inhibi-
iii. For lubrication during instrumentation
tory factor in the use of this material is its exorbitant cost.
iv. Removal of smear layer
l Over a period, MTA will replace the gold standard, as
v. To debride inaccessible areas like accessory/lateral
the most commonly used pulp medicament and set a
canals
new standard of its own.
Q.9. Define apexification.
Q.16. Iodoform-based obturating pastes.
Ans.
Ans.
[Ref SE Q.1]
Iodoform based obturating pastes
Q.10. Indications for direct pulp capping in permanent i. They have a pH ranging between 8 and 9.
teeth. ii. They contain calcium hydroxide, 40% iodoform and
silicone oil.
Ans.
iii. Commercially available as vitapex, Metapex and
Direct pulp cap is a treatment option for teeth with: Calform RC.
l Traumatic or mechanical pulp exposures (inadvertently iv. They have an alkalizing effect on periapical tissues and
exposed during an operative procedure). promote apical healing.
l It should be attempted only when the pulp is vital and
Q.17. Lasers in pulpotomy.
does not show any signs or have any symptoms of irre-
versible pulpitis. Ans.
Section | I  Topic-Wise Solved Questions of Previous Years 547

l Lasers have been suggested for a number of procedures canal orifice site has been proposed as an alternate to the
in dentistry including pulpotomy. more traditional pharmacotherapeutic techniques, par-
l Nd:YAG, diode, CO2, argon, Er:YAG lasers all have ticularly those using formocresol.
been tried as an alternate to formocresol pulpotomy l Numerous researchers have tried this as an alternate to

with similar or slightly lower success rates than the gold formocresol pulpotomy but none could prove that it is
standard. superior to formocresol pulpotomy.
l As lasers become more common in dentistry, dentists
Q.19. List the obturating materials used in primary
owning a laser can use this as an alternate to pulpoto-
teeth.
mies in children in their practices.
Ans.
Q.18. Electrosurgical pulpotomy.
[Same as SN Q.2]
Ans.
l In electrosurgery, controlled energy in the form of elec-
trosurgical heat application to the pulp stumps at the

Topic 24
Traumatic Injuries of Anterior Teeth and Management
COMMONLY ASKED QUESTIONS
LONG ESSAYS:
1. Discuss the management of Ellis class II fracture in incisor tooth.
2. Classify fractures of central incisor in an 8-year-old child following on automobile accident.
3. Classify injuries to anterior teeth how do you treat a case of avulsion.
4. Give WHO classification of traumatic injuries to the anterior teeth and management of complicated crown
fracture of maxillary anterior teeth.
5. Classify anterior teeth fractures according to Ellis. Write in detail about management of root fracture.
6. Describe step-by-step the procedure of a vital class II fracture of an incisor tooth with composite material using
acid etch technique. [Same as LE Q.1]
7. A 9-year-old child sustained fracture of upper right central incisor; the fracture involved enamel and dentine
but without pulp exposure. Discuss your immediate, intermediate and permanent treatment in the patient.
[Same as LE Q.1]
8. Classify the injuries to anterior teeth (any one classification) and discuss in detail the management of trauma-
tized permanent central incisor in a 7-year-old child with pulpal involvement. [Same as LE Q.2]
9. Discuss the management of a class III fracture of central incisor in a 9-year-old child. [Same as LE Q.2]
10. Give Ellis and Davey’s classification for anterior tooth trauma. A 10-year-old patient comes with Ellis class III
facture. Give your line of treatment. [Same as LE Q.2]
1 1. Describe in detail the management of avulsed permanent tooth. [Same as LE Q.4]
12. Classify injuries to anterior teeth. How do you treat a case of avulsion? What are the steps you like to take for
prevention of trauma to teeth. [Same as LE Q.4]
13. Mention predisposing factors for traumatic injuries. How will you manage the complicated crown fracture of
upper central incisors? [Same as LE Q.5]
14. Classify traumatic injuries of teeth. Write about management of root fractures. [Same as LE Q.6]

SHORT ESSAYS:
1 . Luxation injuries – their management.
2. Ellis classification for fracture of anterior teeth. [Ref LE Q.1]
3. Sequelae of trauma to the primary teeth.
548 Quick Review Series for BDS 4th Year, Vol 1

4. Soft tissue injuries related to trauma.


5. Oral guards.
6. Give WHO classification of traumatic injuries to the anterior teeth.
7. Healing of replanted teeth.
8. Predisposing factors for trauma to anterior teeth.
9. Treatment of root fractures.
10. Medias used to store avulsed tooth.
11. Treatment of avulsion of permanent teeth.
12. Classify traumatic injuries to anterior teeth (Ellis classification). [Same as SE Q.2]
13. Ellis and Davey’s classification. [Same as SE Q.1]
14. Mouth guards. [Same as SE Q.5]
15. Mouth protectors. [Same as SE Q.5]
16. Reattachment of tooth fragment. [Same as SE Q.7]
17. Management of avulsion. [Same as SE Q.11]

SHORT NOTES:
1. Splints.
2. Avulsion.
3. Management Ellis class III fracture.
4. Management of intrusive luxation.
5. Properties of an ideal splint.
6. Materials used to store avulsed tooth. [Ref LE Q.3]
7. Nonaccidental injury.
8. Clinical examination of child reporting to the clinic with anterior tooth trauma.
9. Mouth guards. [Ref SE Q.5]
10. Hanks balanced salt solution. [Ref SE Q.10]
11. Ellis class V injuries. [Same as SN Q.2]

SOLVED ANSWERS
LONG ESSAYS:
Q.1. Discuss the management of Ellis class II fracture in Class VIII – fracture of crown en mass and its displacement
incisor tooth. Class IX – traumatic injuries of primary teeth}
Ans. Ellis and Davey’s class II (enamel and dentine fractures):
The major steps involved in treating the class II frac-
[SE Q.2] tures are as follows:
{Classification of injuries to teeth by Ellis and Davey i. Indirect pulp capping
(1960) is as follows: ii. Composite restoration with strip crowns or polycar-
Class I – simple fracture of crown involving only bonate crowns
enamel with little or no dentine iii. Fragment reattachment
Class II – extensive fracture of crown involving consid- iv. Finishing and polishing of the attachment area with
erable dentine but not exposing dental pulp finishing burs and disks
l The fractured tooth should be thoroughly cleaned
Class III – extensive fracture of crown involving consid-
erable dentine and exposing dental pulp and examined for assessment of the extent of
Class IV – traumatized tooth that becomes nonvital with exposed dentine and minute pulp exposures.
l Pulp protection and aesthetics are the main con-
or without loss of crown structure
Class V – total tooth loss – avulsion cerns.
Class VI – fracture of the root with or without loss of Immediate provisional treatment:
l Hard setting calcium hydroxide (e.g. Dycal)
crown structure
Class VII – displacement of tooth without crown or root is placed over the exposed dentine as a part
fracture of indirect pulp capping.
Section | I  Topic-Wise Solved Questions of Previous Years 549

l Calcium hydroxide can be stabilized by an open- Class VIII – fracture of crown en mass and its displace-
faced stainless steel crown or orthodontic band. ment
Permanent treatment: Class IX – traumatic injuries of primary teeth
l A composite or glass ionomer dressing initially Ellis and Davey, class III (enamel–dentine fracture
followed by a definitive composite restoration us- involving pulp):
ing strip crowns or polycarbonate crowns also can The main objectives of treating pulp exposed teeth are
be used for aesthetics. l To retain the tooth.

Fragment reattachment: l To maintain vitality if possible, to allow apex

l Fragment reattachment can be done for Ellis class closure.


II and III or uncomplicated and complicated frac- l To ensure apex maturity by chemical means.

tures of crown. The factors that should be considered during the man-
The procedure includes reattachment of the frac- agement of crown fractures with pulp exposure are
tured tooth fragment which was retrieved by the l Vitality of the pulp

patient/parent or by the dentist from the soft tis- l Size of pulp exposure

sues. l Time elapsed since exposure

Procedure of fragment reattachment: l Stage of development of root apex

l Under local anaesthesia with rubber dam isola- l Restorability of the fractured crown

tion. Size of pulp exposure and time elapsed since


l Calcium hydroxide dressing is given to the den- exposure:
tine portion over the pulp. i. If the pulp exposure is small and is not been
l Preparation of the fragment and the attachment exposed for more than 4–5 min, then pulp cap-
site is done for taking the composite resin restora- ping is the treatment of choice.
tion. ii. If the pulp exposure is large and if it has been
l To facilitate handling of the tooth fragment, it can exposed for more than 5 min, then pulpotomy
be attached to a piece of sticky wax. is the ideal treatment of choice.
l It is followed by acid etching; bonding and attach- l Stage of development of root apex and vitality of

ment of the fragment with chemical cure or light the pulp:


cure composite resins. Complete development (Closed apex):
l Finally, finishing and polishing of the attachment In case of closed apex, the procedure is extirpa-
area is done with finishing burs and disks. tion of pulp and conventional endodontic
l The pulp vitality should be monitored regularly. treatment.
Early treatment improves the chance of maintain- Consideration is given for calcium hydroxide
ing pulp vitality. therapy for 6–12 months before definitive root
l Initial follow-up is after 3 months and thereafter filling.
at 6 monthly intervals. Developing tooth with open apex (immature tooth):
Treatment of open or immature apex varies based
Q.2. Classify fractures of central incisor in an 8-year- on the vitality of the tooth. In case of:
old child following on automobile accident. a. Vital tooth:
Direct pulp capping or pulpotomy with cal-
Ans.
cium hydroxide is performed based on the
Classification of injuries to teeth by Ellis and Davey amount of pulp exposure to allow apexo-
(1960) is as follows: genesis.
Class I – simple fracture of crown involving only b. Nonvital tooth:
enamel with little or no dentine In case of nonvital tooth, induction of root
Class II – extensive fracture of crown involving consid- end closure or apexification is done with
erable dentine but not exposing dental pulp calcium hydroxide and CMCP.
Class III – extensive fracture of crown involving consid- The steps involved in treatment of class III fractures
erable dentine and exposing dental pulp are as follows:
Class IV – traumatized tooth that becomes nonvital with i. Direct pulp capping
or without loss of crown structure ii. Composite restoration
ClassV – total tooth loss – avulsion iii. Fragment reattachment
Class VI – fracture of the root with or without loss of iv. Cvek pulpotomy or calcium hydroxide pulp-
crown structure otomy (apexogenesis)
Class VII – displacement of tooth without crown or root v. Apexification
fracture vi. Conventional root canal treatment
550 Quick Review Series for BDS 4th Year, Vol 1

Q.3. Classify injuries to anterior teeth how do you treat Agents used for root surface treatment to prevent sur-
a case of avulsion. face resorption:
l Citric acid
Ans.
l Doxycycline
Classification of injuries to teeth by Ellis and Davey l Tetracyclines
(1960) is as follows: l Fluorides
Class I – simple fracture of crown involving only l Enamel matrix derivative (Emdogain)
enamel with little or no dentine l Diphosphonates
Class II – extensive fracture of crown involving consid- The prognosis and long-term success of avulsion inju-
erable dentine but not exposing dental pulp ries depend on many factors like:
Class III – extensive fracture of crown involving consid- l Length of the time the tooth was outside the
erable dentine and exposing dental pulp socket
Class IV – the traumatized tooth that becomes nonvital l Condition in which the tooth is brought
with or without loss of crown structure l Replantation of the tooth is always the treatment of
Class V – total tooth loss – avulsion choice, which may have some psychological value
Class VI – fracture of the root with or without loss of too for both the child and the parent.
crown structure Replantation:
Class VII – displacement of tooth without crown or root l If the tooth is brought within 30 min of trauma in
fracture suitable medium then immediate replantation is pos-
Class VIII – fracture of crown en mass and its displacement sible.
Class IX – traumatic injuries of primary teeth l If the periodontal ligament left attached to the root
Avulsion (Ellis and Davey’s class VIII): surface does not dry out, the consequences of tooth
The complete displacement of tooth from its alveolus avulsion are usually minimal. The hydrated peri-
is known as avulsion. odontal ligament cells allow them to reattach on re-
Consequences of tooth avulsion: plantation without causing any more than minimal
l When a tooth is avulsed, damage to periodontal
destructive inflammation.
attachment and pulp necrosis occur The steps involved in replantation of teeth are as fol-
l The tearing of the periodontal ligament leaves vi-
lows:
able periodontal ligament cells on the root surface. i. Preparation of root
l A small, localized cementum damage also occurs
ii. Preparation of socket
due to the crushing of the tooth against the socket. iii. Splinting
Instructions on telephone: i. Preparation of root:
l Locate the tooth
l Preparation of the root is dependent on the
l Just rinse under running tap water with plug to
maturity of the tooth (open vs. closed apex)
sink without use soap or alcohol and on the dry time of the tooth before it was
l No scrubbing or scraping of the root should be done
placed in a storage medium.
l To be handled by the crown portion of the
l A dry time of 60 min is considered the point
tooth only where survival of root periodontal ligament
l Insert tooth back into socket and gently bite
cells is unlikely.
with gauze or l In case of extraoral dry time, less than 60 min
l Bring the tooth in a storage medium, if not
for a tooth with closed apex the root should be
manageable by you rinsed of debris with water or saline and re-
Various storage media used for carrying avulsed planted in as gentle fashion as possible.
teeth are as follows: l Revascularization is not possible with closed

apices but, because the tooth was dry for less


{SN Q.6} than 60 min and is placed in suitable medium,
l Tissue or cell culture media like Hank’s balanced salt the chance for periodontal healing exists.
solution (HBSS) l A dry time of less than 15–20 min is consid-

l Milk (ultra-heat treated milk) ered optimal where periodontal healing would
l Isotonic saline be expected.
l Contact lens solution l If the tooth has been dry for more than

l Buccal vestibule or under the tongue 20 min but less than 60 min it can be treated
l Unsalted water with Emdogain® (Biora, Sweden), the medi-
l Saliva cament presumed to be valuable in such con-
ditions.
Section | I  Topic-Wise Solved Questions of Previous Years 551

l In case of extraoral dry time, less than 60 min for a l When the tooth is in the best possible position, it
tooth with open apex the tooth is soaked in doxy- is important to adjust the bite to ensure that it has
cycline for 5 min, then debris is gently rinsed off, not been splinted in a position that will cause
and replantation is done. In an open apex tooth, traumatic occlusion.
revascularization of the pulp as well as continued l To maintain the avulsed tooth in position,

root development is possible with tooth soaked in 1 week is sufficient to create periodontal support;
doxycycline for 5 min before replantation. therefore, the splint should be removed after 7–10
l In case of extraoral dry time, more than days.
60 min for a tooth with closed apex, the periodon- l When avulsion occurs in conjunction with alveo-

tal ligament cells are not expected to survive. The lar fractures, in which case it is suggested that the
root is prepared to be as resistant to resorption as tooth should be splinted for a period of 4–8
possible. Removal of the periodontal ligament is weeks.
done by placing in acid for 5 min, then tooth is l Healing after replantation can occur through ei-

soaked in 2% stannous fluoride or cover the root ther normal periodontal ligament or with surface
with Emdogain®, followed by replantation. resorption or with ankyloses, i.e. replacement re-
l In these cases, endodontics may be performed sorption or with inflammatory resorption.
extraorally before replantation. Complications following replantation are as follows:
i. In the case of a tooth with a closed apex, no l Pulp necrosis

advantage exists to this additional step at the l Obliteration of root canal

emergency visit. l Internal resorption

ii. Completing the root canal treatment extra- l External root resorption in the form of surface re-

orally where a seal in the blunderbuss apex is placement, replacement resorption (ankylosis) or
easier to achieve, may be advantageous. soft tissue replacement
Since these teeth are in young patients The instructions to be given to the parent after traumatic
whose facial development is usually incom- injury are
plete, the prognosis may be good. l Give the child soft diet for few days after injury to

ii. Preparation of socket: avoid discomfort.


l The socket should be left undisturbed before l Maintain good oral hygiene by brushing the teeth

replantation. after every meal to avoid infection.


l Remove any obstacles within the socket to facilitate l Topical use of chlorhexidine mouthwash twice a day

the replacement of the tooth into the socket. If the for 1 week.
alveolar bone has collapsed, a factor which may l Inform the possible complications.

prevent replantation or cause it to be traumatic, a Preventive measures:


blunt instrument should be inserted carefully into l Offering anticipatory guidance to the parents and

the socket in an attempt to reposition the wall. caregivers regarding dental problems is a very im-
iii. Splinting: portant part in the prevention of dental diseases and
l A splinting technique that allows physiological conditions.
movement of the tooth during healing and that is in l Certain precautionary measures should be taken like

place for a minimal time period results in a de- unstable chairs, swings and walkers should not be
creased incidence of ankylosis. used.
l Semirigidor physiologic fixation for 7–10 days is l When in motion, the parents should always place

recommended. The splint should allow movement young children in an appropriate-sized child seat that
of the tooth and should not impinge on the gin- is in proper position and location. A toddler in a
giva and/or prevent maintenance of oral hygiene stroller should be belted and never left alone.
in the area. l Mouth guards are effective for those older children

l A new titanium trauma splint recently been shown who are involved in contact sports.
to be particularly effective and easy to use. l When the time comes for a child to ride bicycle,

l A radiograph should be taken after the splint is in tricycle or traveling in scooters or other wheeled
place, to verify the positioning of the tooth and as riding toy, an appropriately fitting helmet is
a preoperative reference for further treatment and recommended which is also available in small
follow-up. sizes.
552 Quick Review Series for BDS 4th Year, Vol 1

Q.4. Give WHO classification of traumatic injuries to Treatment:


the anterior teeth and management of complicated The type of treatment will depend upon the extent
crown fracture of maxillary anterior teeth. and time of pulp exposure.
l When the exposure is small and pulp has not
Ans.
been exposed for more than 4–5 min, then pulp
WHO classification (1993) of traumatic injuries to the an- capping is advisable.
terior teeth is as follows: l When the exposure is large and pulp has been
873.60 – enamel fracture exposed for more than 5 min, then it is ideal to
873.61 – enamel and dentine fracture without pulp do pulpotomy.
exposure l In young patients with immature, still develop-
873.62 – enamel and dentine fracture with pulp ing teeth, it is advantageous to preserve pulp
exposure vitality by pulp capping or partial pulpotomy.
873.63 – root fracture This treatment is also the choice in young patients
873.64 – crown–root fracture with completely formed teeth. Calcium hydroxide
873.66 – concussion, luxation and MTA are suitable materials for such procedures.
873.67 – intrusion, extrusion l In older patient, root canal treatment can be the
873.68 – avulsion treatment of choice, although pulp capping or
873.69 – soft tissue injuries partial pulpotomy may also be selected.
Predisposing factors for traumatic injuries are as follows: l If too much time elapses between accident and
i. Accident prone profile – Class II division 1 and treatment and the pulp becomes necrotic, root
class I type 1 canal treatment is indicated to preserve the tooth.
ii. Inadequate lip coverage – According to Jarvinen, l In extensive crown fractures the extraction is
frequency of injury with normal overjet (0–3 mm) is feasible treatment.
14.2%, while with increased overjet (3.1–6 mm) it
Q.5. Classify anterior teeth fractures according to Ellis.
is 28.4%. In cases of extreme overjet (.6 mm), it is
Write in detail about management of root fracture.
38.6%.
iii. Handicapped children – Abnormal muscle tone and Ans.
poor skeletal coordination in epileptic patients. By Ellis and Davey (1960):
iv. Dental anomalies and caries – Hypoplasia and den- Class I – simple fracture of crown involving only
tal caries weakening of crown structures. enamel with little or no dentine
v. Mechanical factors – According to Hallet severity Class II – extensive fracture of crown involving consid-
of injury depends on: erable dentine but not exposing dental pulp
a. Energy impact Class III – extensive fracture of crown involving consid-
b. Resiliency of impacting object erable dentine and exposing dental pulp
c. Shape of impacting object Class IV – traumatized tooth that becomes nonvital with
d. Angle of direction of impacting force or without loss of crown structure
Complicated crown fractures: Class V – total tooth loss, i.e. avulsion
Clinical features: Class VI – fracture of the root with or without loss of
l Complicated crown fracture involves enamel and crown structure
dentine and the pulp is exposed. Class VII – displacement of tooth with neither crown
l This usually presents as a fractured segment of nor root fracture
tooth with frank bleeding from the exposed pulp. Class VIII – fracture of crown en masse and its displace-
l Sensibility testing is usually not indicated initially ment
since vitality of the pulp can be visualized. Class IX – traumatic injuries of primary teeth
l To monitor pulpal status follow-up control visits According to Cohen: cracked tooth
includes sensibility testing. According to Mathewson: cyclic dislocation of tooth
Radiographic findings: Root fracture:
l Three angulations were described in radiographic l Root fracture is defined as fractures involving den-
examination to rule out displacement or fracture tine, cementum and pulp. They are relatively uncom-
of the root. mon ranging from 0.5% to 7% in permanent dentition
l Radiograph of lip or cheek lacerations is recom- and 2% to 4% in primary dentition.
mended to search for tooth fragments or foreign l The mechanism of root fractures is usually a frontal
material. impact, which creates compression zones labially
l The stage of root development can be determined and lingually. The resulting shearing stress zone then
from the radiographs. dictates the plane of fracture.
Section | I  Topic-Wise Solved Questions of Previous Years 553

Clinical features: iii. When fracture is near to gingival margin –


l Root fractures involving the permanent dentition orthodontic or surgical extrusion of the fragment
predominantly affect the maxillary central incisor followed by immobilization and later crown
region in age group of 11–20 years. fabrication
l Coronal fragments are displaced lingually or slightly iv. Vertical root fracture:
extruded. l It is also called cracked tooth syndrome. It

l The coronal segment may be mobile and may be runs lengthwise from crown towards the apex.
displaced. It is mostly found in posterior teeth and its
l The tooth may be tender to percussion. aetiology is mostly iatrogenic like insertion of
l Temporary loss of sensitivity. screws or after pulp therapy.
l Monitoring the status of the pulp is recommended. l It is indicated by persistent dull pain of long

Sensibility testing may give negative results initially, standing origin and pain is elicited by apply-
indicating transient or permanent pulpal damage. ing pressure.
l Transient crown discolouration (red or grey) may occur. l Radiographically the line of fracture is visible

l The fracture involves the root of the tooth and is in a as radiolucent line and thickening of Peri-
horizontal or diagonal plane. odontal ligament (PDL) is also seen.
l Fractures that are in the horizontal plane can usually l Treatment of single rooted teeth consists of

be detected in the regular 90° angle film with the extraction and in case of multirooted teeth
central beam through the tooth. This is usually the hemisection and the remaining tooth is end-
case with fractures in the cervical third of the root. odontically treated and resorted with crown.
l If the plane of fracture is more diagonal, which is
Q.6. Describe step-by-step the procedure of a vital class
common with apical third fractures, an occlusal view
II fracture of an incisor tooth with composite material
is more likely to demonstrate the fracture including
using acid etch technique.
those located in the middle third.
l Reposition, if displaced, the coronal segment of the Ans.
tooth as soon as possible. Check position radiograph-
[Same as LE Q.1]
ically. Stabilize the tooth with a flexible splint for
4 weeks. Q.7. A 9-year-old child sustained fracture of upper
l If the root fracture is near the cervical area of the right central incisor; the fracture involved enamel and
tooth, stabilization is beneficial for a longer period of dentine but without pulp exposure. Discuss your im-
time. mediate, intermediate and permanent treatment in the
l It is advisable to monitor healing for at least 1 year patient.
to determine pulpal status. If pulp necrosis develops, Ans.
root canal treatment of the coronal tooth segment to
the fracture line is indicated to preserve the tooth. [Same as LE Q.1]
Radiographic features: Q.8. Classify the injuries to anterior teeth (any one clas-
l Radiographic demonstration of root fractures is fa-
sification) and discuss in detail the management of trau-
cilitated by the fact that the fracture line is most often matized permanent central incisor in a 7-year-old child
oblique and at an optimal angle for radiographic with pulpal involvement.
disclosure.
l The root fracture would normally be visible only if
Ans.
the central beam is directed within a maximum range [Same as LE Q.2]
of 15°–20° of fracture plane.
Treatment: Q.9. Discuss the management of a class III fracture of
l The principle of treatment of permanent teeth is re-
central incisor in a 9-year-old child.
duction of displaced coronal fragments and firm im- Ans.
mobilization.
Following treatment modalities are recommended based [Same as LE Q.2]
on the fracture line: Q.10. Give Ellis and Davey’s classification for anterior
i. When fracture is present in middle third – tooth trauma. A 10-year-old patient comes with Ellis
extraction class III facture. Give your line of treatment.
ii. When fracture is in apical third – obturation till
Ans.
the possible working length and apicectomy to
remove the fragment [Same as LE Q.2]
554 Quick Review Series for BDS 4th Year, Vol 1

Q.11. Describe in detail the management of avulsed per- l Percussion may reveal a metallic sound but
manent tooth. tenderness to percussion is a rare finding.
l Crushing or compression of the alveolar bone
Ans.
is associated with intrusive injury, detected by
[Same as LE Q.4] gentle palpation of the mucosa in the trauma-
tized area. In this situation, the tooth and the
Q.12. Classify injuries to anterior teeth. How do you
cortical plate move as a single unit.
treat a case of avulsion? What are the steps you like to
Radiographic features:
take for prevention of trauma to teeth?
Periapical and occlusal radiographs are adequate
Ans. to diagnose an intruded incisor.
l The absence/obliteration of periodontal lig-
[Same as LE Q.4]
ament space.
Q.13. Mention predisposing factors for traumatic inju- l The occlusal image determines the direc-

ries. How will you manage the complicated crown frac- tion of intrusion.
ture of upper central incisors? l An extraorallateral radiograph will help in

determining the proximity of the intruded in-


Ans.
cisor root apex to the succeeding permanent
[Same as LE Q.5] tooth germ and fracture of labial cortical plate.
Management:
Q.14. Classify traumatic injuries of teeth. Write about
It mainly depends on the direction and severity of
management of root fractures.
the intrusion and the presence of alveolar bone
Ans. fracture.
Two main treatment modalities are
[Same as LE Q.6]
i. Wait and watch for spontaneous re-eruption
ii. Extraction if permanent tooth injury is sus-
SHORT ESSAYS: pected
i. Wait and watch for spontaneous re-eruption:
Q.1. Luxation injuries – their management. l If the root of the primary incisor is forced

Ans. towards the labial bone then spontaneous


re-eruption is anticipated within 1–6
Luxation (Ellis and Davey’s class IX): months, only when the intrusion is mild.
It is of three types: ii. Extraction is indicated in cases of:
i. Intrusion l Moderate or severe intrusion of the tooth
ii. Extrusion l Palatally directed root which may contact
iii. Lateral luxation with or invades the follicle of the permanent
Intrusion: tooth germ
l Intrusion is the displacement of tooth into the l Buccal cortical plate perforation
alveolar bone. l Bone fracture was present
Intrusions can be divided into three grades based on l Signs of re-eruption not evident after 4 to
visual examination: 8 weeks that may indicate ankylosis of the
a. Grade I: mild partial intrusion with more than intruded tooth
50% of the crown visible l Infection of the tooth
b. Grade II: moderate partial intrusion with less Extrusion and lateral luxation:
than 50% of the crown visible l Extrusion is the partial displacement of tooth
c. Grade III: severe or complete intrusion of the out of the socket. Extruded teeth might have
crown occlusal interference with the opposing tooth.
Clinical features: l Lateral luxation includes teeth that are displaced
l Intrusion is always associated with soft tissue in a direction other than axial direction.
injuries of which contusion of the lower lip and Clinical features:
the chin is frequent. l Tooth is mobile and tender to percussion and
l Presence of submucosal haemorrhage. masticatory forces.
l Bleeding may be noticed around the tooth. l Bleeding from gingival crevice.
Section | I  Topic-Wise Solved Questions of Previous Years 555

Radiographic features: For example: White or yellow brown discolouration of


l Widening of periodontal ligament space in case of enamel
extrusion l Infection and abscess

l Widening of periodontal ligament space on one l Odontoma like malformations

side and obliteration on other side in case of lat- l Loss of space in the dental arch

eral luxation l Ankylosis

The various treatment options are: l Injury to developing permanent tooth

i. No treatment l Abnormal exfoliation

i. Repositioning and splinting l Financial costs for maintaining dental arch space

ii. Extraction or restoration


l If the displacement is minimal, they can be left
Q.4. Soft tissue injuries related to trauma.
without any treatment but periodic monitoring
should be done. Ans.
l If they are interfering with occlusion, reposi-
l Soft tissue injuries are common in trauma to the primary
tioning can be done using digital pressure fol-
dentition causing displacement of tooth/teeth.
lowed by splinting.
Soft tissue injuries can be one of the following:
l If the palatal displacement is so severe such
i. Laceration: A shallow or deep wound in the mu-
that the injured teeth are in crossbite on centric
cosa resulting from a tear and is usually produced
occlusion, then these teeth are to be splinted
by a sharp object.
after repositioning for 2 weeks. If improve-
ii. Contusion: A bruise is usually produced by impact
ment is not seen within 2 weeks, extraction is
with a blunt object and not accompanied by a break in
indicated.
the mucosa usually causing submucosal haemorrhage.
l In case of any chance of damage to the un-
iii. Abrasion: A superficial wound produced by rub-
derlying permanent tooth germ is anticipated
bing or scraping of the mucosa leaving a raw bleed-
or if injured tooth is nearing time for exfo-
ing surface.
liation then the extraction is the treatment of
l Swelling and bruising of the lips are common
choice.
findings after oral trauma, even in the absence of
laceration.
Q.2. Ellis classification for fracture of anterior teeth.
l Lips often cushion the teeth during a fall, and

Ans. may be bruised or lacerated by the impact of


teeth against tissue.
[Ref LE Q.1]
l If any laceration is present, check for any foreign

Q.3. Sequelae of trauma to the primary teeth. body impaction, such as a tooth fragment or gravel.
l A child who falls while holding an object in his/
Ans.
her mouth-most commonly, a stick, pencil or
l Injuries to the primary dentition are estimated to affect pen or a toothbrush-often suffers an impalement
30% of preschool children. The most serious primary injury of the oropharynx.
tooth injuries in terms of damage to the permanent suc- l Most impalement injuries heal spontaneously, but

cessor are intrusion, avulsions (52%), extrusions and the wound should be explored for possible for-
subluxation (each 34%). eign bodies and a prophylactic antibiotic should
The sequelae of dental trauma to primary teeth are as be prescribed to avoid infectious complications.
follows: l If there is significant soft tissue injury, the soft

l Different injuries of teeth involving crown, root or tissues should be sutured under local anaesthesia
whole tooth for cooperative child and general anaesthesia for
For example: crown dilacerations, root dilacerations or an uncooperative child.
angulation l If the soft tissue injury is severe or site of trauma

l Failure to continue eruption: is dirty, antibiotic coverage for 5 days may be


The eruption of succeeding permanent teeth is generally necessary and patients’ antitetanus status should
delayed after premature loss of primary teeth be evaluated.
l Colour changes or discolouration-slight, moderate or l A recall after 7–10 days in order to check heal-

severe which can be transient or permanent ing and for suture removal.
556 Quick Review Series for BDS 4th Year, Vol 1

Q.5. Oral guards. They are unacceptable for most athletes and offer the
l

least protection for the prevention of sports-related


Ans.
traumatic dental injuries.
Advantages:
{SN Q.9} l Least expensive

l Ready to wear
l Mouth guards/protectors were initially introduced in
Disadvantages:
1913 as the boxers’ mouthpiece. This device gained
l Do not fit to the dentition of the athlete and must
importance particularly in boxing.
be held in place by biting the teeth together.
l The mouth guards are designed to protect the lips
l Bulky and least retentive.
and intraoral soft tissues from bruises and lacera-
l Interfere with the athlete’s ability to speak and
tions. They also protect the teeth from any kind of
breathe.
fractures or luxation injuries and the jaws from any
Mouth-formed mouth guards (type II):
dislocations and fractures.
Mouth-formed mouth guards come in two varieties:
l The American Dental Association and the Academy
 i. Shell-lined
for Sports Dentistry (ASD) recommend properly fit-
ii. Boil and bite
ted mouth guards for a variety of sports and recre-
 i. Shell-lined:
ational activities which predispose the participants to
This mouth guard is fabricated by placing freshly
oral injuries.
mixed ethyl methacrylate into a hard shell, which
For example: acrobatics, basketball, bicycling, box-
is then inserted into the athlete’s mouth until it sets.
ing, football, martial arts, rugby and wrestling
The advantages of shell-lined mouth guards:
l Relatively good adaptation to dental arch
ASTM and ASD requirements: compared to stock mouth guards
ASTM: The disadvantages of shell-lined mouth guards:
The mouth guard should cover all teeth in one arch- l Too bulky and uncomfortable to wear
the maxillary arch in players with class I or class II l Have an unpleasant odour and taste
malocclusion and the mandibular arch in players l The liner should be changed after every
with class III malocclusion. use
ASD: Due to the above-mentioned limitations, shell-
The properties of a properly fitted mouth guard as lined mouth guards lost their favour and are
defined by the ASD are as follows: not often used by the athletes.
l Adequate thickness in all areas to provide for ii. Boil and bite:
the reduction of impact force l These mouth guards are fabricated by placing
l A fit that is retentive and not dislodged on the material into boiling water to soften the
impact material and this softened material is placed
l Speech considerations equal to the demands of inside the athlete’s mouth where it is moulded
the playing status of the athlete with finger pressure.
l A material that meets US Food and Drug l Facial and intraoral muscular movements are
Administration approval used to enhance adaptation to the hard and soft
l Preferably a wearing duration of time equal to tissue structures of the mouth.
one season of play l After removal from the athlete’s mouth, it is

placed inside cold water until the shape is set


{SN Q.9} firmly. It can be re-boiled and readapted to
improve retention.
Classification of mouth guards The advantages of boil and bite mouth guards:
The ASTM classifies mouth guards as: l Easy to use and fit can be modified.
Type I: stock mouth guards l Comes in a wide variety of styles and co-
Type II: mouth-formed mouth guards lours with and without strap attachments.
Type III: custom fabricated mouth guards l The price range is from inexpensive to mod-
The type III mouth guards are the most preferred. erately expensive depending on the quality.
The disadvantages of boil and bite mouth
Stock mouth guards (type I): guards:
l Stock mouth guards are readily available in sporting l It may be too bulky at the periphery or

goods stores. They are purchased over the counter too thin at the occlusal table.
and come in a variety of colours and styles with and l If fitted by the athlete himself/herself, it

without straps. may give poor results.


Section | I  Topic-Wise Solved Questions of Previous Years 557

Custom-fabricated mouth guards (type III): periods of time in comparison to the other fab-
l Custom-fabricated mouth guards are made profes- rication techniques.
sionally over a dental cast of the patients. Maintenance of mouth guards:
l They are fabricated over the maxillary cast for pa- l Mouth guard should be washed after each use in

tients with class I or class II malocclusion and man- cold or lukewarm water to minimize the accumu-
dibular cast for patients with class III malocclusion. lation of debris, while hot water should be avoided.
l They are superior to either stock or mouth-formed l It can be cleansed with a toothbrush with tooth-

mouth guards. paste and rinsed with mouthwash to reduce bad


l They do not interfere with breathing and speech be- odour.
cause of their superior adaptation and retention. l It should be stored in a closed plastic container

Advantages of custom fabricated mouth guards: when not in use.


l Better fit and comfort. l It should be rinsed with water prior to insertion.

l Less interference with speech and breathing. l Mouth guard should be periodically inspected for

l Occlusal table thickness can be controlled during distortions and occlusal perforations. When these
laboratory procedure. abnormalities are found, a new mouth guard
l Straps may not be necessary. needs to be fabricated.
Disadvantages of custom fabricated mouth guards: The education on effectiveness of the protective
l Multiple dental visits (two or more) needed for equipment in prevention of traumatic sports-
fabrication related injuries and regulations for usage of such
l Involvement of laboratory procedures for its fab- equipments can lead to a more positive attitude
rication makes it expensive regarding regular usage.
Q.6. Give WHO classification of traumatic injuries to
the anterior teeth.
{SN Q.9}
Ans.
Other forms of mouth guards:
l Jaw joint positioner WHO classification, 1978:
l Anatomically designed mouth guard The World Health Organization has adopted a system of
classification in its application of international classifi-
cation of diseases to dentistry and stomatology in 1978.
Techniques of fabrication of mouth guards: Every injury has a code number, which are as follows:
Many methods and techniques have been published in a. Injuries to hard dental tissues and pulp:
the literature for fabrication of mouth guards. l Enamel fracture – 873.60

Two commonly used methods for fabrication are l Crown fracture without pulpal involvement –

 i. Vacuum-forming technique 873.61


ii. Heat pressure–laminating technique l Crown fracture with pulpal involvement –

 i. Vacuum-forming technique: 873.62


l In this technique, ethylene vinyl acetate (EVA) l Root fracture – 873.63

material, available as single or double lami- l Crown–root fracture without pulpal involve-

nated sheets are used. ment – 873.64


l The vacuum-forming machine uses 1 atmo- l Crown–root fracture with pulpal involvement

spheric pressure to vacuum the EVA material – 873.64


down over the dental cast. b. Injuries to periodontal tissues:
ii. Heat pressure-laminating technique: l Luxation – 873.66

l This technique uses a positive pressure of 10 l Concussion – 873.66

atmosphere to mould the EVA material over l Subluxation – 873.66

the dental cast. l Lateral luxation – 873.66

l Its primary advantage is that it offers the added l Intrusion/extrusion – 873.67

capability of fusing multiple layers of EVA l Avulsion – 873.68

material to the desired thickness to meet the l Others – 873.69

desired needs of the individual athlete. WHO classification, 1992:


l The names or logos of the teams can be added According to the International Classification diseases
between the layers of EVA sheets. (1992) by World Health Organization, the code numbers
l The proponents of this technique assert that the are changed as follows:
material is better adapted, the fit is precise and a. Injuries to hard dental tissues and pulp:
the structural integrity is retained for longer l Enamel infraction – N 502.50
558 Quick Review Series for BDS 4th Year, Vol 1

l Enamel fracture – N 502.50 Histologic examination of replanted human teeth has


l Crown fracture without pulpal involvement –
revealed four different healing modalities in PDL:
N 502.51 i. Healing with a normal periodontal ligament:
l This type of healing will only occur if innermost
l Crown fracture with pulpal involvement – N

502.52 cell layers along the root surface are vital.


l As tooth avulsion will result in at least minimal
l Root fracture – N 502.53

l Crown-root fracture without pulpal involve-


injury to innermost layer of PDL, this type of
ment – N 502.54 healing will probably never take place.
l Clinically tooth is in normal position and a nor-
l Crown-root fracture with pulpal involvement

– N 502.54 mal percussion tone can be elicited.


l Histologically it is characterized by complete
b. Injuries to periodontal tissues:
l Concussion – N 503.20
regeneration of PDL, which usually takes 2–4
l Subluxation – N 503.20
weeks to complete.
l Radiographically there is normal PDL space
l Extrusive luxation – N 503.20

l Lateral luxation – N 503.20


without signs of root resorption.
l Intrusion – N 503.21
ii. Healing with surface resorption:
l The surface resorption presumably represents
l Avulsion – N 503.22

c. Injuries to the supporting bone: localized areas of damage to PDL or cementum,


l Communition of the mandibular (N 502.60) or
which is healed by PDL, derived cells.
l Clinically the tooth is in normal position and a
maxillary (N 502.40) alveolar socket
l Fracture of the mandibular (N 502.60) or max-
normal percussion tone can be heard.
l Histologically this type of healing is characterized
illary (N 502.40) alveolar socket wall
l Fracture of the mandibular (N 502.60) or max-
by localized areas along the root surface, which
illary (N 502.40) alveolar process show superficial resorption lacunae repaired by
l Fracture of mandible (N 502.61) or maxilla (N
new cementum.
502.42) iii. Healing with ankylosis (replacement resorption):
l The aetiology of replacement resorption appears
d. Injuries to gingiva or oral mucosa:
l Laceration of gingiva or oral mucosa (S 01.50)
to be related to the absence of vital PDL cover
l Contusion of gingiva or oral mucosa (S 00.50)
on the root surface.
l Replacement resorption develops in two differ-
l Abrasion of gingiva or oral mucosa (S 00.50)

WHO has modified its classification based on ent directions depending upon the extent of
Application of the International Classification damage to the PDL surface of the root.
l Progressive replacement resorption, which
of Diseases to Dentistry and Stomatology
(1995), by substituting ‘S’ in place of N5. For gradually resorbs the entire root, is always elic-
example, The number for root fracture is ited when the entire PDL is removed before
S02.53 instead of N 502.53. reimplantation or after extensive drying of the
tooth before reimplantation.
Q.7. Healing of replanted teeth. l It is assumed that the damaged PDL is repopu-

Ans. lated from adjacent bone marrow cells, which


have osteogenic potential and will consequently
l Immediately after reimplantation, a coagulum is formed form ankylosis.
between two parts of severed periodontal ligament. l The ankylosed root becomes part of the normal bone
l Proliferation of connective tissue soon occurs and after remodelling system and is gradually replaced by
3–4 days the gap in the periodontal ligament is obliter- bone. After some time little of tooth substance re-
ated by young connective tissue. mains, at this stage the resorptive process are usually
l After 1 week the epithelium is reattached at the cemento- intensified along the surface of the root canal filling
enamel junction. a phenomenon known as tunnelling resorption.
l This is of clinical importance because it reduces risk of l Histologically ankylosis represents a fusion of
gingival infection and reduced risk of bacterial invasion the alveolar bone and the root surface and can
of root canal via the gingival pocket. be demonstrated 2 weeks after reimplantation.
l After 2 weeks the split line in the PDL is healed and iv. Healing with inflammatory resorption:
collagen fibres are seen extending from the cemental l Pathogenesis is that minor injuries to PDL and
surface to alveolar bone. cementum due to trauma or contamination with
Section | I  Topic-Wise Solved Questions of Previous Years 559

bacteria induce small resorption cavities on the l Direction of impact


root surface. l Type of impact – direct or indirect
l If these resorption cavities expose dentinal tu-
Q.9. Treatment of root fractures.
bules and root canal contains infected necrotic
tissue, toxins from these areas will penetrate Ans.
along dentinal tubules to lateral periodontal tis-
Root fractures (Ellis and Davey’s class VI):
sue and provoke an inflammatory response. This
l Root fractures are relatively uncommon in permanent
in turn will intensify the resorption process,
dentition. They usually account for ,5% of injuries,
which advances towards root canal, and within a
the maxillary central incisors are most commonly
few months entire root can be resorbed.
affected.
l Clinically the replanted tooth is loose, extruded
l The prognosis for root fractures is best when the
and sensitive to percussion with dull tone.
fracture occurs in the apical third of the root.
l Histologically inflammatory resorption is char-
The root fractures are diagnosed by:
acterized by bowl-shaped resorption cavities in
Clinical examination:
cementum and dentine associated with inflam-
l Clinical examination of the root fractures re-
matory changes in the adjacent periodontal
veal slightly extruded tooth often displaced
space.
lingually.
l Radiographically inflammatory resorption is
Radiographic examination:
characterized by radiolucent bowl-shaped cavi-
l Radiographs will only show a root fracture, if
tations along root surface with corresponding
the central beam is directed within a maximum
excavations in adjacent bone.
range of 15–20° of the fracture plane. There-
Q.8. Predisposing factors for trauma to anterior teeth. fore, it is necessary to take two views for ex-
amination of root fractures.
Ans.
The principles of treatment of root fractures:
Predisposing factors for trauma to anterior teeth are as l Reduction of displaced fragments is done fol-

follows: lowed by firm immobilization or splinting.


i. Age of the child: l The repair is by either calcified material, a combi-

l Children of 1.5 years due to falls when they nation of dentine and cementum, connective tis-
learn to stand and walk because of lack of devel- sue and bone.
opment of neuromuscular coordination. l The splint must be rigid and cemented in place;

l Children of 8–11 years, which is school age with any movement during splinting period may cause
increased physical activity and contact sports. fracture of the tiny links in the calcifying tissues
ii. Occlusion with increased overjet: as soon as they are formed.
l Occlusion with overjet . 4 mm is termed as l The pulp is more likely to survive after root frac-

accident-prone profile as seen in class II divi- ture than after luxation, but pulp canal obliteration
sion 1 and class I type 2 malocclusion. and external and internal resorption are common.
iii. Developmental defects of teeth and dental caries: Treatment of root fractures:
l Developmental defects of teeth like dentinogen- l Passively applied acid-etch splints for 2–3

esis imperfecta and dental caries will weaken weeks are required for treating root fractures.
the tooth structure, in turn affecting the resis- Apical third root fractures:
tance of tooth to fracture. l Apical third root fractures are not often

iv. Mental retardation and epilepsy: mobile enough to need splinting.


l They might result in falls of children resulting in l For apical third fractures with loss of vital-

trauma to the teeth. ity and small apical segment, apicoectomy


v. Sports injuries is done followed by an endodontic implant
For example: volley ball, baseball, cricket and after root canal therapy of the coronal por-
horse riding. tion of the root.
vi. Automobile injury: Middle third root fractures:
Various road traffic accidents. l Middle third root fractures will require

Mechanical factors affecting dental trauma: splinting in this way.


Mechanical factors affecting dental trauma are as follows: l For middle third root fractures, one has to

l Shape and mass of the impacting object observe for vitality, andendodontic treatment
l Velocity of the impacting object is done, if necessary. Root canal obturation
560 Quick Review Series for BDS 4th Year, Vol 1

of both fragments is done followed by endodontic Treatment of avulsed teeth:


screw fixation. Treatment of avulsed tooth is based on various catego-
Coronal third root fractures: ries as follows:
l Coronal third root fractures will have insufficient

root length on the crown for adequate function.


Extrusion of the root portion by orthodontic Mature apex Immature apex
means, followed by endodontic treatment, coronal Less than 15 min out of Less than 15 min out of
restoration with postcore and crown is the treat- mouth: mouth:
ment of choice. Rinse the tooth with physio- Soak the tooth in doxycycline
logic solution to remove de- solution for 5 min.
l For coronal third root factures, if splinting period
bris from the root surface. Reimplant the tooth in the
is uneventful, leave it like that. In case of pulp Flush the socket with sterile socket.
necrosis, removal of coronal fragment is done, water or saline. Splint the tooth in a functional
followed by root canal treatment. Reimplant the tooth in the position.
socket. Check the tooth for vitality
and apex closure every month.
Q.10. Media used to store avulsed tooth.
15 min to 6 h in physiologic 15 min to 6 h in physiologic
Ans. solution: solution:
Place the tooth in Hank’s bal- Soak the tooth in doxycycline
Storage media used to store avulsed tooth are as follows: anced salt solution for 30 solution for 5 min.
l Tissue or cell culture media like Hank’s balanced salt min. Flush the socket with sterile
solution (HBSS) Flush the socket with sterile water or saline.
l Milk (ultra-heat treated milk) water or saline. Reimplant the tooth in the
Reimplant the tooth in the socket.
l Isotonic saline
socket. Splint the tooth in a functional
l Contact lens solution
Splint the tooth in a functional position.
l Buccal vestibule or under the tongue position.
l Unsalted water
15 min to 1 h in nonphysio- 15 min to 1 h in nonphysio-
l Saliva
logic solution: logic solution:
Place the tooth in Hank’s Place the tooth in Hank’s bal-
balanced salt solution for anced salt Solution for 30 min.
{SN Q.10} 30 min. Soak the tooth in doxycycline
Flush the socket with sterile solution for 5 min.
HBSS (EMT Tooth Saver®) contains: water or saline. Flush the socket with sterile
l Sodium chloride Reimplant the tooth in the water or saline.
l Potassium chloride socket. Reimplant the tooth in the
Splint the tooth in a functional socket.
l Glucose
position. Splint the tooth in a functional
l Sodium bicarbonate
position.
l Sodium phosphate
Greater than 1 h dry storage: Greater than 1 h dry storage:
l Calcium chloride
Remove remnants of the peri- Remove remnants of the peri-
l Magnesium chloride
odontal ligament by soaking odontal ligament by soaking in
l Magnesium sulphate in sodium hypochlorite for sodium hypochlorite for 10–15
10–15 min. Instrument the min. Instrument the root canal
root canal with the tooth out with the tooth out of the
Agents used for root surface treatment to prevent surface of the mouth. mouth.
resorption are as follows: Soak the tooth in 2% stannous Soak the tooth in 2% stannous
l Citric acid fluoride solution for 5 min. fluoride solution for 5 min.
l Doxycycline
Obturate the root canal with Obturate the root canal with
gutta-percha. gutta-percha.
l Tetracyclines
Coat the tooth root with Em- Coat the tooth root with Em-
l Fluorides dogain and place Emdogain dogain and place Emdogain
l Enamel matrix derivative (Emdogain) (Biora, Chicago, III, and in the socket.
l Diphosphonates Malmo, Sweden) in the socket. Reimplant the tooth in the
Reimplant the tooth in the socket.
Q.11. Treatment of avulsion of permanent teeth. socket. Splint the tooth in a functional
Splint the tooth in a functional position.
Ans. position.
Section | I  Topic-Wise Solved Questions of Previous Years 561

Q.12. Classify traumatic injuries to anterior teeth (Ellis Q.2. Avulsion.


classification).
Ans.
Ans.
l Ellis Class V injuries are total tooth loss as a result of
[Same as SE Q.2] trauma, i.e. avulsion.
l The term avulsion is used to describe complete dis-
Q.13. Ellis and Davey’s classification.
placement of tooth from its alveolus. It is also called
Ans. exarticulation.
Maxillary teeth are most commonly involved.
[Same as SE Q.2]
Clinical features:
Q.14. Mouth guards. l Bleeding socket with missing tooth

Radiographic features
Ans.
l Empty socket.

[Same as SE Q.5] l Associated bone fractures.

l If the wound is recent, then lamina dura is visible


Q.15. Mouth protectors.
otherwise it is obliterated.
Ans. Treatment
l Reimplantation.
[Same as SE Q.5]
l If apical foramen is not closed, endodontic ther-
Q.16. Reattachment of tooth fragment. apy is delayed till first signs of apical closure are
seen.
Ans.
l If apical foramen is closed, endodontic therapy is
[Same as SE Q.7] done after 1–2 weeks depending on type of
reimplantation.
Q.17. Management of avulsion.
Ans. Q.3. Management Ellis class III fracture.

[Same as SE Q.11] Ans.


l Ellis Class III fractures are extensive fractures of crown
SHORT NOTES: involving considerable dentine and exposing dental pulp.
l Vitality of the pulp, size of pulp exposure, time elapsed,

Q.1. Splints. stage of development of root apex and restorability of


the fractured crown are the factors that should be con-
Ans. sidered during the management of Ellis class III frac-
l Splinting is the standard of care for stabilization of tures.
repositioned or replanted permanent teeth following l In case of closed apex, the procedure is extirpation of

trauma. pulp and conventional endodontic treatment.


l Splints can be removable or fixed. l Treatment of open or immature apex varies based on the

Removable splint: vitality of the tooth. In case of:


For example: Swed type, removable appliance with a. Vital tooth:
acrylic extending over incisal or occlusal surfaces of the Direct pulp capping or pulpotomy with calcium
teeth hydroxide is performed to allow apexogenesis.
Fixed splints: b. Nonvital tooth:
For example: Induction of root end closure or apexification is done
l Acid etch resin with/without rectangular arch with calcium hydroxide and CMCP.
wire The treatment of class III fractures can be sum-
l Orthodontic brackets with wire marized as follows:
l Interdental wiring, integral wiring i. Direct pulp capping
l Metal bars, titanium splints ii. Composite restoration
l Full arch, vacuum-moulded acrylic splint iii. Fragment reattachment
Recent advances: iv. Cvekpulpotomy or calcium hydroxide
l Composite impregnated ribbons (Ribbond) pulpotomy (apexogenesis)
l Fibber glass v. Apexification
l Synthetic fibres or tapes (Fibersplint, Kevlar) vi. Conventional root canal treatment
562 Quick Review Series for BDS 4th Year, Vol 1

Q.4. Management of intrusive luxation. l These are not accidental injuries, but they are inflicted
by people who are responsible for care of the child.
Ans.
l The child is considered to be abused if they are treated

l Intrusion is the displacement of tooth into the alveolar in a way that is unacceptable in a given culture.
bone.
Q.8. Clinical examination of child reporting to the clinic
The intrusive luxation can be managed in following
with anterior tooth trauma.
ways:
i. If tooth is with immature apex, then allow sponta- Ans.
neous re-eruption.
Clinical examination of a child should be carried out in a
ii. If tooth has closed apex either:
disciplined way to arrive at an appropriate diagnosis.
a. Surgical repositioning, splinting and endodon-
Extra oral examination:
tic treatment or
l Asymmetry of the face due to injuries
b. Orthodontic repositioning, splinting and end-
l Observation of signs and symptoms of fractures of
odontic treatment are the treatment of choice
facial bones
Q.5. Properties of an ideal splint. l Palpation of facial bones for any noticeable fractures

or deviations
Ans.
l Recording of extra oral wounds or paraesthesia of
The properties of an ideal splint are as follows: any part of the face
l It should be quick and easy to fabricate and remove. Intraoral examination:
l It should be flexible to allow functional movements of l Evaluation of bleeding sites

the teeth. l Examination of tooth crowns for any fractures, pulp

l It should be atraumatic to the teeth and adjacent soft tissues. exposures or colour changes
l It should be adequately stable.
Q.9. Mouth guards.
l It should provide access to endodontic therapy, if needed.

l It should be aesthetically pleasing. Ans.


Q.6. Materials used to store avulsed tooth. [Ref SE Q.5]
Ans. Q.10. Hanks balanced salt solution.
[Ref SE Q.4] Ans.
Q.7. Nonaccidental injury. [Ref SE Q.10]
Ans. Q.11. Ellis class V injuries.
l The nonaccidental injuries are nothing but child abuse Ans.
and neglect.
[Same as SN Q.2]

Topic 25
Local Anaesthesia and Oral Surgery
for the Child Patient
COMMONLY ASKED QUESTIONS
LONG ESSAYS:
1. What are the fundamentals of extraction techniques? Discuss special consideration and special problems en-
countered in the extractions of primary teeth.
2. What are the indications, contraindications and complications of extractions of primary teeth?
Section | I  Topic-Wise Solved Questions of Previous Years 563

SHORT ESSAYS:
1 . Mandibular nerve block in children.
2. Inferior alveolar nerve block in a 5-year-old child.
3. WAND.
4.
Topical anaesthetics.
5.
Indications and contraindications of extractions in children. [Ref LE Q.2]
6.
A 5-year-old child comes to you with a swelling of lower left 2nd deciduous molar which in indicated for extrac-
tion. What is your line of treatment? [Same as SE Q.1]
7 . Gow–Gates mandibular block anaesthesia. [Same as SE Q.1]

SHORT NOTES:
1 . Young’s formula for drug dosage.
2. Composition of local anaesthetic solution.
3. Indications for extraction of primary teeth.
4. Anaesthetic allergy.
5. Mucocoele.
6. Dentigerous cyst.
7. Topical anaesthetics.

SOLVED ANSWERS
LONG ESSAYS:
Q.1. What are the fundamentals of extraction tech- the tooth. Then the tooth is extracted with appropriate
niques? Discuss special consideration and special prob- forceps technique.
lems encountered in the extractions of primary teeth. l One of the most important principles of extraction is to

Ans. exert a firm apical pressure throughout the extraction


procedure.
Fundamentals of extraction techniques for children are as l Following the extraction, the alveolar plates are
follows: compressed together to inhibit bleeding from the
l The smaller paediatric extraction forceps such as the
socket.
nos. 150s and 151s are preferred by most paediatric Techniques of extraction of different teeth are described as
dental surgeons. follows:
l The smaller size of forceps allows easy placement in-

side the child’s oral cavity, better adaptation of smaller A. Extraction of maxillary teeth:
working ends to tooth and concealability by the opera- i. Maxillary anterior teeth:
tor’s hand. l A no. 1 forceps is useful for extracting maxillary

l The patient should be explained about what is expected anterior teeth.


of him/her. The principles of tell show and do are very l The operator or the dentist should stand in front of

important during extraction procedures. the patient during the extraction procedure.
l Just prior to extraction procedure, demonstrate the types l As maxillary anterior teeth have single conical

of pressures and movements that he/she will encounter roots, they are less susceptible for fracture and are
during the extraction. This is done by placing the index removed with rotational movement.
finger and thumb in the area of extraction and exerting l Once the tooth is held with the forceps, a firm

firm pressure. rotational movement is exerted which loosens the


l The dentist should be placed in a position in which he/ tooth from its socket.
she has good control over the instruments. ii. Maxillary posterior teeth:
l The nondominant hand is used to retract the cheeks, l The operator should stand in front of the patient

stabilize the head and to support the jaw. during the extraction procedure.
l Once the dentist and child patient are ready, a periosteal l The extraction is carried out with a maxillary uni-

elevator is used to separate the epithelial attachment of versal forceps (no. 150s). The forceps is seated
564 Quick Review Series for BDS 4th Year, Vol 1

with palatal beak first and rotating the facial beak l Radiographically, there is absence of periodontal
into proper position. ligament space.
l The palatal movement is initiated first, followed by l When fusion of cementum to bone does not occur

alternating buccal and palatal motions with slow throughout the periphery of the ankylosed tooth,
continuous force applied to the forceps. This facili- it can be removed with forceps as other extrac-
tates expansion of the alveolar bone and easy re- tions. If the forceps extraction is unsuccessful,
moval of primary molar with its divergent roots. splitting the tooth into two halves needs to be
l Because of the root structure and potential weaken- done.
ing due to the pressure from permanent tooth, root l In the presence of an underlying tooth bud, it is

fracture is not uncommon. important to make sure that the entire ankylosed
l It is vital to remember the relationship of the primary root is removed. Otherwise, fragments of anky-
molar roots to the succeeding premolar crown. losed roots may inhibit the normal eruption of the
Extraction of mandibular teeth: bicuspid.
Extraction of anterior teeth: l If succedaneous tooth is absent, the ankylosed

l All the mandibular anterior teeth are single tooth can remain in function for around 15–20
rooted teeth. years.
l Extraction of these teeth is carried out with Postextraction instructions:
rotational movement and alternating labial and l After extraction of primary teeth, clear and spe-
lingual movements. cific post extraction instructions are given to the
l Great care should be taken to avoid injury to child as well as parents, they are as follows:
adjacent tooth. i. Bite on the cotton or gauze for 15–30 min and
Extraction of mandibular posterior teeth: do not chew.
l During extraction of mandibular molars, the ii. Do not eat or drink for 30 min after the ex-
mandible should be supported properly to pre- traction.
vent any injury to the TMJ. iii. Do not use a straw to drink up to 24 h.
l Mandibular molars are extracted with no. 151s iv. Do not rinse your mouth on the day of
forceps. extraction.
l The forceps is seated with the lingual beak first v. Do not spit repeatedly. Spitting can increase
and rotating the facial beak into proper posi- the bleeding.
tion. vi. Take medications as recommended by the
l The initial luxation is towards the buccal side dentist.
and then with alternating buccal and lingual vii. If bleeding starts, keep a roll of cotton or
movements until the tooth is removed. gauze pad and bite it tightly for 30 min.
Remaining roots of primary teeth: viii. On the day of extraction take lot of cold liq-
l Root fractures can occur occasionally during ex- uids and soft or semisolid foods (e.g. soft
traction of primary teeth. bread, idly or steamed rice).
l Vigorous attempts to remove the remaining roots ix. The most important postextraction instruction
should be avoided as it can lead to damage to the in younger children is, ask them not to bite the
unerupted permanent tooth bud. lips, cheeks or tongue when it is numb. The
l If the root tip is seen clearly, use of an elevator or numbness can persist for 1–2 h after the ex-
a root tip pick can help. If several attempts to re- traction. Inadvertent biting of soft tissues can
move the root tip fail, it is best to leave it to be lead to ulcer formation.
resorbed. x. If pain or bleeding persists ask them to call
l Either the remnant resorbs or erupts alongside the dental office.
erupting premolars. When the root fragment is left
It is recommended to give a written sheet of post extrac-
inside, parents should be notified and an entry in
tion instructions rather than verbal instructions.
the case sheet is made.
Removal of ankylosed primary teeth: Q.2. What are the indications, contraindications and
l Ankylosis results due to the fusion of cementum complications of extractions of primary teeth?
with alveolar bone.
Ans.
l An ankylosed primary molar will remain sub-

merged and below the occlusal plane. l There are many reasons why both deciduous and perma-
l Clinically, it is confirmed by the infraocclusal nent teeth have to be extracted.
position and percussion test where ankylosed l Sometimes, normal teeth needed to be sacrificed to im-

tooth produces a metallic sound. prove mastication and prevent malocclusion.


Section | I  Topic-Wise Solved Questions of Previous Years 565

l In most of the instances, teeth are extracted because Mandibular nerve block (Gow–Gates mandibular block
they are affected by disease or can cause ill health due technique):
to spread of the infection. l In 1973, Gow–Gates introduced a new method of

obtaining mandibular anaesthesia, which he referred


[SE Q.5] to as mandibular conduction anaesthesia.
{Following are the indications for extraction of teeth: l In this approach the external anatomic landmarks

l Teeth affected by deep dental caries and its sequelae were used to align the needle so that anaesthetic so-
l Teeth affected by periodontal diseases lution is deposited at the base of the neck of the
l Therapeutic extractions to correct malocclusion mandibular condyle.
l Over-retained deciduous teeth l This technique is a nerve block procedure that anaes-

l Extraction of teeth for aesthetic reasons thetizes virtually the entire distribution of the fifth
l Extraction of teeth for prosthodontic reasons cranial nerve in the mandibular area, including the
l Impacted and supernumerary teeth inferior alveolar, lingual, buccal, mental, incisive,
l Teeth involved in fracture line and pathologies like auriculotemporal and mylohyoid nerves.
tumours or cysts l With a single injection the entire right or left half of

l Tooth acting as foci of infection the mandibular teeth and soft tissues can be anaes-
l Teeth affected by abrasion, attrition or hypoplasia of thetized.
the crowns l The technique has become increasingly popular and

l Teeth affected by pulpal lesions, e.g. pulpitis, pink is often referred to as the Gow–Gates technique.
spot or pulp polyp Technique:
l Teeth in the area of direct therapeutic irradiation l The external landmarks to help align the needle

Contraindications: for this injection are the tragus of the ear and the
l It is necessary for the well-being of the patient to corner of the mouth.
delay extraction until certain local or systemic l The needle is inserted just medial to the tendon of

conditions can be corrected or modified. the temporal muscle and considerably superior
There are few absolute contraindications to the to the insertion point for conventional mandibular
removal of teeth: block anaesthesia.
l Presence of acute oral infections, e.g. necrotiz- l The needle is inclined upward and parallel to a

ing ulcerative gingivitis or herpetic gingival line from the corner of the patient’s mouth to the
stomatitis and pericornitis. lower border of the tragus known as intertragic
l Extraction of teeth in previously irradiated notch.
areas for at least 1 year to allow for maximal l The needle and the barrel of the syringe should be

recovery of circulation to the bone. directed towards the injection site from the corner
l There are number of relative systemic contra indi- of the mouth on the opposite side.
cations to the tooth extraction Q.2. Inferior alveolar nerve block in a 5-year-old child.
For example: Uncontrolled diabetes acute blood
dyscrasias, untreated coagulopathies, adrenal in- Ans.
sufficiency, general debilitation for any reason, When deep operative or surgical procedures are under-
myocardial infarction.}
l

taken for the mandibular primary or permanent teeth,


Complications of extractions: the inferior alveolar nerve must be blocked.
l Fracture of tooth or alveolar bone.
l Olsen reported that the mandibular foramen is situated
l Injuries to adjacent teeth.
at a level lower than the occlusal plane of the primary
l Gingival and mucosal lacerations.
teeth of the paediatric patient. Therefore, the injection
l Root displaced into maxillary sinus.
must be made slightly lower and more posteriorly than
l Maxillary sinus perforation.
for an adult patient.
l If a permanent tooth bud is removed or slightly
l An accepted technique is one in which the thumb is laid
moved, it should be carefully pushed into the on the occlusal surface of the molars, with the tip of the
socket to its position and sutures are placed. thumb resting on the internal oblique ridge and the ball
l Extraction of wrong tooth.
of the thumb resting in the retromolar fossa.
l Haemorrhage and haematoma.
l Firm support during the injection procedure can be
l TMJ trauma.
given when the ball of the middle finger is resting on the
posterior border of the mandible.
SHORT ESSAYS: l The barrel of the syringe should be directed on a plane

between the two primary molars on the opposite side of


Q.1. Mandibular nerve block in children. the arch. It is advisable to inject a small amount of the
solution as soon as the tissue is penetrated and to continue
Ans.
566 Quick Review Series for BDS 4th Year, Vol 1

to inject minute quantities as the needle is directed to- None of the preschool-aged children exposed to the
l

wards the mandibular foramen. WAND required restraint during the initial interval
l The depth of insertion averages about 15 mm but varies while nearly half of the children receiving a tradi-
with the size of the mandible and its changing propor- tional injection required some type of immediate
tions depending on the age of the patient. Approxi- restraint.
mately 1 mL of the solution should be deposited around
Q.4. Topical anaesthetics.
the inferior alveolar nerve.
l One can block the lingual nerve by bringing the syringe Ans.
to the opposite side with the injection of a small quan-
l Topical anaesthetics renders free nerve endings in ac-
tity of the solution as the needle is withdrawn.
cessible structures like mucous membrane, abraded skin
l If small amounts of anaesthetic are injected during in-
or cornea of eye incapable of stimulation by the applica-
sertion and withdrawal of the needle for the inferior
tion of suitable agent directly to the surface.
alveolar nerve block, the lingual nerve will invariably be
l Topical anaesthetics are available in gel, liquid, oint-
anaesthetized as well.
ment and pressurized spray forms.
l For the removal of mandibular permanent molars or
l These agents are applied to the oral mucous membranes
sometimes for the placement of a rubber dam clamp on
with a cotton tipped applicator.
these teeth, it is necessary to anaesthetize the long buc-
l A variety of anaesthetic agents have been used in topical
cal nerve.
anaesthetic preparations, including ethyl aminobenzo-
l A small quantity of the solution may be deposited in the
ate, butacaine sulphate, cocaine, dyclonin, lidocaine and
mucobuccal fold at a point distal and buccal to the indi-
tetracaine.
cated tooth.
l Ethyl aminobenzoate (benzocaine) liquid, ointment or
l All facial mandibular gingival tissue on the side that has
gel preparations are probably best suited for topical
been injected will be anaesthetized for operative proce-
anaesthesia in dentistry. They offer a more rapid onset
dures.
and longer duration of anaesthesia than other topical
agents.
Q.3. WAND.
l They are not known to produce systemic toxicity as oral
Ans. topical anaesthetics, but a few localized allergic reac-
tions have been reported from prolonged or repeated use.
Computer-controlled local anaesthetic delivery system
l The pleasant tasting and quick-acting liquid, gel or oint-
(WAND):
ment preparations seem to be preferred by most dentists.
l WAND is a computerized local anaesthetic delivery
l Some topical anaesthetics however present a disadvan-
system (the WAND or CompuDent and STA system).
tage if they have a disagreeable taste to the child. Also,
l The name ‘WAND’ is synonymous with ‘magic
the additional time require to apply them may allow the
wand’ for needle phobia and STA for single tooth
child to become apprehensive concerning the approach-
anaesthesia.
ing procedure.
l The system includes a conventional local anaes-
l They reduce the slight discomfort that may be associ-
thetic needle and a disposable wand-like syringe
ated with the insertion of the needle before the injection
held by a pen grasp when used for oral local anaes-
of the local anaesthetic.
thetic injections.
l Examples of commercially available products are
l A microprocessor controls the flow rate, constant
Hurricaine, Topicale and Gingicaine.
pressure and controlled volume. A foot control regu-
All three products are available in gel form.
lates the delivery of anaesthetic solution.
l Gingicaine is also available in liquid and spray
l The system includes an aspiration cycle for use when
forms, and Hurricaine is available as a liquid.
necessary.
Topicale is available in ointment form.
l This can be used for all possible local anaesthetic
Topical anaesthesia should be produced in approxi-
techniques like infiltration, nerve block and intraliga-
mately 30 s.
mentary injections.
l A more recently developed product for achieving
l Injections may take more time because of the
topical anaesthesia is known as Denti Patch (No-
reduced anaesthetic flow rate. The controlled flow of
ven Pharmaceuticals, Inc.), a lidocaine transoral
anaesthetic is thought to reduce pain, and thus,
delivery system.
patient’s fear and anxiety.
l Block, infiltration, palatal and periodontal ligament This system seems to be designed primarily for situa-
injections are all reported to be more comfortable for tions in which superficial oral tissue anaesthesia is desired
the patient with the WAND than with conventional for several minutes rather than the shorter time required for
injection techniques. local anaesthetic injections.
Section | I  Topic-Wise Solved Questions of Previous Years 567

Q.5. Indications and contraindications of extractions in l Removal of primary teeth should be carried out only
children. when indicated.
The following are indications for removal of primary teeth:
Ans.
i. Unrestorable decayed tooth
[Ref LE Q.2] ii. The retained primary tooth if it is interfering with
the permanent tooth eruption
Q.6. A 5-year-old child comes to you with a swelling of
iii. In cases of acute dentoalveolar abscess or cellulitis
lower left 2nd deciduous molar which in indicated for
iv. Persistence of periapical infection after pulp therapy
extraction. What is your line of treatment?
v. Infected teeth in the jaw or surrounding tissues
Ans.
Q.4. Anaesthetic allergy.
[Same as SE Q.1]
Ans.
Q.7. Gow–Gates mandibular block anaesthesia.
l Allergy is defined as a hypersensitive state acquired
Ans. through exposure to particular allergen.
l Clinical manifestations of allergy to local anaesthetic
[Same as SE Q.1]
solution include angioedema, urticaria, dermatitis,
fever, asthma, rhinitis and anaphylaxis. They may vary
SHORT NOTES: from case to case.
Q.5. Mucocoele.
Q.1. Young’s formula for drug dosage.
Ans.
Ans.
l Mucocoele is a lesion involving salivary glands and
l In children the drug dosage requirements constantly
their ducts.
change as a function of age.
l It usually occurs in the first and second decades of life
l Dosage based on body weight, age is practical but not
and has no gender predilection.
ideal concept. Current dosage recommendations are
l This is the most common lip swelling in children and is
usually based on basal metabolism of child.
associated with trauma due to various reasons. It may be
Young’s formula for drug dosage is as follows:
associated with trauma and orthodontic appliances.
l This rule is based upon the age of the child, regard-
l They are more common on the lower lip, but may also
less of the weight.
occur on the palate, cheek, tongue and floor of the
l It is to divide the age of the child by the age plus 12,
mouth.
and the resulting fraction is the portion of the adult
l If the contents of the cyst are liberated, they usually are
dose, which is to be used.
found to consist of a thick mucinous material. Some le-
Age of child × Adult dose sions regress and enlarge periodically and may disappear
Child dose = after traumatic injury which result in their evacuation.
Age + 12
Treatment of choice:
l Surgical excision with removal of the underlying
Q.2. Composition of local anaesthetic solution.
minor salivary glands.
Ans. l Recurrence is common with incomplete removal or

l Local anaesthetic produces loss of sensation to pain in repeated trauma.


a specific area of the body without the loss of con- Q.6. Dentigerous cyst.
sciousness.
l Composition of local anaesthetic solution is as follows: Ans.
i. Lidocaine hydrochloride – local anaesthetic agent l Dentigerous cyst or follicular cyst is one of the common
ii. Epinephrine – vasocontrictor cysts in children and originates after the crown of the
iii. Sodium metabisulphite – oxidizing agent tooth has been completely formed.
iv. Methyl paraben – preservative l It results in accumulation of fluid between the crown
v. Thymol – antifungal agent and the reduced enamel epithelium.
vi. Distilled water/ringer lactate solution – solvent or l It may grow rapidly and is usually associated with an
vehicle impacted, embedded or an unerupted tooth.
l It commonly occurs in the mandibular and maxillary
Q.3. Indications for extraction of primary teeth.
third molar and the maxillary canine areas as these are
Ans. the common sites for impacted teeth.
568 Quick Review Series for BDS 4th Year, Vol 1

l This cyst has the potential to expand aggressively caus- l Topical anaesthetics are available in gel, liquid, oint-
ing facial asymmetry, displacement of teeth, severe root ment and pressurized spray forms.
resorption of adjacent teeth and pain. l A variety of anaesthetic agents have been used in topical

Treatment: anaesthetic preparations, e.g. ethyl aminobenzoate


l Smaller lesions can be surgically removed by enucle- (benzocaine) and lidocaine, liquid, ointment or gel.
ation. l Examples of commercially available products are Hur-

l The larger cysts which involve serious loss of bone ricaine, Topicale and Gingicaine. All three products are
and thin mandibular border can be treated by inser- available in gel form. Gingicaine is also available in
tion of a surgical drain or marsupialization. liquid and spray forms, and hurricaine is available as a
l Recurrence is uncommon. liquid. Topicale is available in ointment form.
l Topical anaesthesia will be produced in approximately
Q.7. Topical anaesthetics.
30 s following application.
Ans. l A more recently developed product for achieving topi-

cal anaesthesia is known as Denti Patch, a lidocaine


l Topical anaesthesia is that form of anaesthesia obtained
transoral delivery system.
by the direct application of the drug to skin or to the
mucous membrane surface.
l Topical anaesthetics reduce the slight discomfort that

may be associated with the insertion of the needle


before the injection of the local anaesthetic.

Topic 26
NSAIDs, Antimicrobial Drugs and Miscellaneous
COMMONLY ASKED QUESTIONS
SHORT ESSAYS:
1 . Drug dosage calculations for children.
2. Classification and guidelines for use of NSAIDs in paediatric dental patients.
3. Fundamentals and guidelines for antibiotic use in paediatric dentistry.

SHORT NOTES:
1 . Young’s rule. [Ref SE Q.1]
2. Vitamins A and D.
3. Amoxicillin in paediatric dentistry.
4. Antibiotic resistance.
5. Metronidazole.
6. Young’s formula for drug dosage. [Same as SN Q.1]

SOLVED ANSWERS
SHORT ESSAYS:
Q.1. Drug dosage calculations for children. Various rules for calculating drug doses in children are
Ans. as follows:
l In children the drug dosage requirements constantly i. Clark’s rule
change as a function of age. ii. Young’s rule
l Dosage based on body weight, age is practical but not iii. Fried’s rule
ideal concept. Current dosage recommendations are iv. Gabiu’s rule
usually based on basal metabolism of child. v. Dilling’s rule
Section | I  Topic-Wise Solved Questions of Previous Years 569

vi. Cowling’s rule l Thus dosages to be used for a child of different


vii. Body surface area rule, etc. ages are as follows:
i. Clark’s rule:
Age of the child Fraction of adult dose to be used
l This is based upon the relative weight of the

child as compared with the weight of the aver- 1 year 1/12th


age adult (150 pounds). 2 years 1/8th
l The rule is to divide the weight of the child in
3 years 1/6th
pounds, by the average weight of the adult, i.e.
150, and to take this fraction of the adult dose. 4 years 1/4th

Weight (pounds) × Adult dose 7 years 1/3rd


Child dose = 14 years 1/2
150
20 years 2/3rd

21 years Adult dose


{SN Q.1}
ii. Young’s rule: vi. Bastedo’s rule:
l This rule is based upon the age of the child,

regardless of the weight. Age of child + 3 of the fraction of the


Child dose =
l It is to divide the age of the child by the age 30 adult dose
plus 12, and the resulting fraction is the portion vii. Fried’s rule:
of the adult dose, which is to be used. l For infants under 1 year:

Age of child × Adult dose Age of infant (in months) of the fraction of
Child dose = Child dose =
Age + 12 150 the adult dose
viii. Catzel’s rule:
l If offers a safe guide based on surface area and ex-
iii. Cowling’s rule: pressed as a percentage of adult dose for a patient.
l It is also based upon the age of the child.

l The fraction of the adult dose, which is to be Age Percentage of adult dose
used, is obtained by dividing the age at the next 1 25
birthday by 24. 3 35
Age at next birthday × Adult dose 7 50
Child dose =
24
12 75
iv. Dilling’s rule:
l He analysed extensive weight statistics of chil- ix. Body surface area (BSA):
dren and found that Young’s formula is suffi- l It is determined from a nomogram using the

ciently accurate up to the eleventh year and child’s height and weight.
Cowling’s is accurate until the 15th year, but l The BSA of an individual can also be calculated

that thereafter it is very inaccurate and wholly from Dubois formula:


unsatisfactory.
BSA (m)2 = BW (kg)0.425 × Height (cm)0.725 × 0.007184
l Based on his analysis, Dilling has worked out a

new formula as follows: l The formula for calculating child’s drug dosage is
Age of child × Adult dose Child’s BSA × Adult dose
Child dose =
20 1.7
v. Gabius’ rule: l As a paediatric dentist a better understanding of
l He stated a series of fractions of the adult dose, physiologic and anatomic changes and pharma-
which were to be used at different ages until the cological considerations in children are impor-
individual attains 21 years. tant to render safe and effective drug therapy.
570 Quick Review Series for BDS 4th Year, Vol 1

Q.2. Classification and guidelines for use of NSAIDs in Fundamentals of appropriate use of antibiotics:
paediatric dental patients. l The indications for the use of antibiotics in paediatric

dental patient are to control infections and as prophy-


Ans.
laxis for the medically compromised child to prevent
Classification of analgesics: serious systemic problems.
I. Nonselective COX inhibitors (conventional NSAIDs): l The general principles to be followed while choosing

l Salicylates an antibiotic are as follows:


Aspirin and difunisal i. Identify the causative organism and determine
l Pyrazolone derivatives sensitivity.
Phenylbutazone and oxyphenbutazone ii. Use a specific narrow spectrum antibiotic.
l Indole derivatives iii. Use the least toxic antibiotic.
Indomethacin and sulindac iv. Know patient’s drug history.
l Propionic acid derivatives v. Use a bactericidal rather than a bacteriostatic agent
Ibuprofen, naproxen, ketoprofen and furbiprofen vi. Use an antibiotic with a proven history of
l Anthranilic acid derivatives success.
Mephenamic acid l The organisms most often responsible for dental in-

l Aryl acetic acid derivative fections are gram-positive streptococci and staphylo-
Diclofenac cocci.
l Oxicam derivatives l In most instances, it is not possible to isolate the or-

Piroxicam and tenoxicam ganism responsible for infection. Hence, it is wise to


l Pyrrolo-pyrrole derivatives start the treatment with penicillinase-sensitive peni-
Ketorolac cillin. They have a wide margin of safety, bacteri-
II. Preferential COX-2 inhibitors: cidal, narrow spectrum and effective against gram-
Nimesulide, meloxicam and nabumetone positive bacteria.
III. Selective COX-2 inhibitors: l If the patient does not respond positively within 36–

Celecoxib, rofecoxib and valdecoxib 48 h, change to a penicillinase-resistant antibiotic


IV. Analgesic, antipyretic with poor anti-inflammatory should be considered.
action: l If a history of penicillin sensitivity is reported, eryth-

l Para-aminophenol derivative romycin or a cephalosporin should be prescribed.


Paracetamol (acetaminophen) l The length of therapy with antibiotics should be de-

l Pyrazolone derivative termined by the severity or course of infection. In


Metamizol (dipyrone) and Propiphenazone uncomplicated dental infections, improvement
l Benzoxazine derivatives should be observed by the second day and marked
Nefopam resolution by the fourth day.
Clinical guidelines on appropriate use of analgesics: l According to Peterson, eradication of infection occurs

l Pain has sensory, emotional, cognitive and behav- by the third day after the patient becomes relatively
ioral components that are inter-related with environ- asymptomatic, i.e. no fever and little or no swelling.
mental, developmental and sociocultural factors. l According to McCallum, antibiotics should be con-

l It is a subjective experience and should be as- tinued for 24 h after the symptoms have subsided.
sessed and treated as such. Therefore, depending on the severity and course of
l Relief of pain in paediatric patients can be accom- the infection, antibiotic therapy should be considered
plished by non-narcotic analgesics. for a 5–7 days period of time.
l Surgical procedures such as removal of an im- l The oral route is preferred for the commonly seen

pacted tooth or a mesiodens may require the use dental infections in paediatric patients.
of additional narcotic analgesics. A liquid form is available for most preparations and
l A common finding in the paediatric patients under should be used, if the young patient is unable to
the age of 8 years is the inability to swallow a tab- swallow the tablet or capsule.
let. Under such circumstances, the use of suspen- AAPD guidelines on appropriate use of antibiotics:
sions, syrups or suppositories are recommended. i. Antibiotic therapy is usually not indicated, if the
dental infection is contained within the pulpal tis-
Q.3. Fundamentals and guidelines for antibiotic use in
sue or the immediately surrounding tissue. In this
paediatric dentistry.
case, the child will have no systemic signs of an
Ans. infection (i.e. no fever and no facial swelling).
Section | I  Topic-Wise Solved Questions of Previous Years 571

ii. The drug should be administered as soon as pos- l Cholecalciferol: controls Ca11 ions concentration. It
sible for the best result. exerts its major influence by combining with nonhis-
iii. If the infection is not responsive to the initial tone proteins in the nuclei of intestinal epithelial cells.
drug selection, a culture and sensitivity testing of l Hypervitaminosis D: hypercalcaemia.

isolates from the infective site may be indicated. l Hypovitaminosis D: secondary hyperparathyroidism.

iv. The minimal duration of drug therapy should be l Deficiency of vitamin D results in rickets in children

limited to 7 days and usually can be a course of and osteomalacia in adults.


10–14 days. Oral manifestations of deficiency:
v. The importance of completing a full course of l Delayed eruption of primary and permanent teeth

antibiotic must be emphasized. If the patient


Q.3. Amoxicillin in paediatric dentistry.
stops consuming the antibiotics early, the surviv-
ing bacteria can restart an infection that may be Ans.
resistant to the original antibiotic.
Amoxicillin is one of the commonly used antibiotics in
vi. Prolonged antibiotic therapy may be needed in
dentistry.
paediatric periodontal diseases.
Amoxicillin:
vii. For viral infections, antibiotics are not indicated
Dosage: Children: 25–50 mg/kg/(div 8–12 h) (maxi-
unless there is a secondary bacterial infection.
mum 1500 mg/day)
Side effects: Rash and diarrhoea
SHORT NOTES: Available in: Tablets, capsules, syrups and suspension
and injection.
Q.1. Young’s rule. Availability and trade name:
Capsules: Novamox, Mox, Wymox, Blumox, Lamoxy
Ans.
(250 mg, 500 mg)
[Ref SE Q.1] Kid tab: Wymox (125 mg), Lamoxy (125 mg)
Syrups: Novamox (125 mg, 250 mg/5 mL), Mox (125 mg,
Q.2. Vitamins A and D.
250 mg/5 mL. Wymox, Lamoxy, Blumox (125 mg/5 mL);
Ans. Drops: Novamox (100 mg/mL)
Injections: Mox, Hipen (250 mg, 500 mg)
l Vitamins A and D are fat-soluble vitamins.
Caution: Increase dose interval in renal impairment cases.
Vitamin A:
l Regulates epithelial differentiation Q.4. Antibiotic resistance.
General manifestations of deficiency:
Ans.
l Night blindness

l Xeropthalmia characterized by dryness in l Bacterial resistance to antibiotics is one of the most


conjunctiva and cornea significant challenges in the management of infectious
l Bitot spots in forms of triangular plaques in diseases.
conjunctiva l The antibiotic resistance has tremendously increased by

Oral manifestations of deficiency: widespread, indiscriminate use of antibiotics in medical


l Keratinising metaplasia of epithelium resulting in and dental applications.
increased keratin formation l Current issues in bacterial resistance include:

l Enamel hypoplasia and atypical dentin formation i. Increasing resistance of staphylococci to penicillinase-
Uses: resistant penicillin (methicillin and oxacillin)
i. Prophylaxis of vitamin A deficiency during in- ii. Resistance of pneumococci to penicillins by altera-
fancy, pregnancy, lactation, hepatobiliary dis- tion of bacterial penicillin-binding proteins
eases, etc.: 3000–5000 IU/day in presence of in- iii. Multidrug resistance among Gram-negative bacte-
creased requirement ria due to bacterial modifying enzymes and ex-
ii. In skin diseases like acne, psoriasis, icthyosis – tended spectrum lactamases
Retinoc acid or synthetic analogues of vitamin A The development of bacterial resistance can be mini-
like tretinoin or isotretinoin are used mized by:
Vitamin D: l Consistently using an appropriate antibiotic for an ade-

l Vitamin D is commonly referred to as the antirachitic quate period of time.


vitamin. l Planning combination drug therapy.
572 Quick Review Series for BDS 4th Year, Vol 1

Q.5. Metronidazole. Availability and trade name:


Tablets: Metrogyl, Aristogyl (200 mg, 400 mg)
Ans.
Suspension: Flagyl, Metrogyl susp (200 mg/5 mL,
Dosage and availability of metronidazole for use in chil- Aristogyl susp (100 mg/5 mL)
dren are as follows: Injection: Metrogyl, Metronidazole i.v. 1500 mg/100 mL
Dosage: infusion)
Children: 30–50 mg/kg/day div q6h p.o./i.v.; route: Caution: Severe liver or renal disease, reduce dose and
orally and i.v. monitor regularly
Precaution:
Q.6. Young’s formula for drug dosage.
Give i.v. infusion slowly over 30–60 min
Side effects: Ans.
Nausea, headache, metallic taste, paraesthesia, transient
[Same as SN Q.1]
leucopenia and rarely seizures
Section I

Topic-Wise Solved Questions


of Previous Years

PART III COMMUNITY DENTISTRY


Topic 1 Introduction  575
Topic 2 Public Health  584
Topic 3 Epidemiology  603
Topic 4 Dental Public Health  628
Topic 5 Preventive Dentistry  649
Topic 6 Fluorides  671
Topic 7 Indices Used in Preventive Dentistry  685
Topic 8 Health Statistics  710
Topic 9 Social Sciences  723
This page intentionally left blank
Section I

Topic-Wise Solved Questions


of Previous Years
Part III
Community Dentistry

Topic 1
Introduction
COMMONLY ASKED QUESTIONS
LONG ESSAYS:
1 . Who are public health dentists and how do they differ from clinical dentists?
2. Define public health and discuss in detail the eventual differences between community dentistry and private
practice at an individual level.
3. Define community dentistry. What are its objectives? Discuss the various tools used in public health dentistry.
4. What are the similarities between personal and community health care workers? [Same as LE Q.1]
5. List the differences between clinical practice and dental public health. [Same as LE Q.2]
6. Define health, public health, dental public health and community dentistry. Describe the tools of community
dentistry. [Same as LE Q.3]

SHORT ESSAYS:
1 . OSHA regulation of infection control.
2. Define community dentistry. Discuss the various community programmes by which oral diseases can be prevented.

SHORT NOTES:
1 . What are types of oral examination?
2. Define community dentistry. [Ref SE Q.2]
3. Complete examination. [Same as SN Q.1]
4. Dental practice management.
5. Importance of dental records.
6. Dentist population ratio.
575
576 Quick Review Series for BDS 4th Year, Vol 1

7. Dental manpower planning in India.


8. Tools of public health. [Ref LE Q.3]
9. Setting up of dental clinic. [Same as SN Q.4]
10. Advantages of computerized dental records. [Same as SN Q.5]

SOLVED ANSWERS
LONG ESSAYS:
Q.1. Who are public health dentists and how do they Procedures performed Procedures performed by
differ from clinical dentists? by clinicians community health workers
Ans. iv. Treatment iv. Programme operation

l Public health dentists/professionals are usually salaried v. Payment for service v. Finance
employees who are accountable to both their immediate vi. Evaluation vi. Approval
supervisors and the taxpayers.
l The public health professionals focus on communities
Knutson summarized the procedures employed by clinician
with special characteristics of culture, socioeconomic and public health worker as follows:
status and values, which make these groups more diffi- Similarities
cult and expensive to reach. A. Examination/survey
l Clinical dentists/clinicians or private practitioners are
l When a patient first comes to a dental office, the dental
relatively independent health care providers and the professional carries out a careful examination. The ex-
decisions made by them are in the context of their train- amination ordinarily starts with health and personal
ing, and they abide by legal framework of dental prac- history, and then goes on to clinical assessment.
tice and the dentist–patient relationship. l An understanding of the patient’s general health and
l Private practitioners usually serve middle-class patients
personal background also forms a necessary context
who have at least some concern for their oral health. for determining the dental treatment plan.
Public health dentists differ from clinical dentists in the l The first step in public health practice parallels that

following ways: used by the clinician. But here, it is the community


l Private practitioners deal with one patient at a time, while that must be examined. It is called a survey instead
community health workers deal with groups of people. of an examination, and the parallel to the general
l Public health dentists are salaried employees with fringe health history taking is a situation analysis.
benefits like pension plan, sick leave and paid leave, l In dental public health, the word survey without fur-

while clinical dentists take home a higher pay with less ther clarification usually means clinical assessments
fringe benefits. of the extent and severity of disease in a population.
l Goals of public health dentists are socially determined. l Purpose of survey is to determine the nature and the ex-

l The public health workers go to the community, while tent of the problem, just as an examination is done when
the patients come to dental practitioners. a patient comes to a dental clinic with a complaint.
l The work of public health dentists is visible and l Hence, basically, examination is called a survey in a

publicly accountable, while clinicians are independent community setting, and the parallel of the general
health care providers. health history is called a situation analysis.
l WHO defines situation analysis as one comprising
Similarities between personal and community health care assessment of population demographics, mobility,
workers are as follows: economic resources and infrastructure.
l According to Knutson – It is the nomenclature of the activi-
B. Diagnosis/analysis
ties carried out by both personal and community health work- l Following the examination of a patient, the next step
ers that varies, while the procedure is basically the same. is diagnosis. It is the procedure of converting the
l Knutson has outlined the following differences between the
information or raw data collected in the survey by
procedures employed by a clinician in treating a patient and meaningful figures or statistics in order to define the
a public health worker providing community health care. characteristics of specific community health prob-
lems, just as dental clinicians uses their examination
Procedures performed Procedures performed by
data to guide them to an accurate diagnosis.
by clinicians community health workers
l Most of the times a professional statistician or a
i. Examination i. Survey
special-purpose computer is employed to process
ii. Diagnosis ii. Analysis and analyse the survey data.
iii. Treatment planning iii. Programme planning l Computers have made survey analysis much less tax-

ing and more efficient.


Section | I  Topic-Wise Solved Questions of Previous Years 577

C. Treatment planning/programme planning l Similarly, data collected in the initial survey serve as
l Once diagnosis is made, one can proceed to make the baseline against which a programme appraisal
plans for effective treatment. can be made to assess the effectiveness of the public
l Treatment planning is often complex because many health programme.
factors have to be balanced. In addition, to the dentist’s l Public health workers are accountable to the commu-

professional judgement of what is required, there is the nity for a periodic appraisal of their performance just
patient’s interest in his or her oral health, the cost of as dental clinicians are accountable to their patients.
treatment, the patient’s reluctance to undergo certain
types of treatment. Alternative methods of treatment Q.2. Define public health and discuss in detail the even-
need to be considered. Final outcome; be it acceptance tual differences between community dentistry and pri-
or total rejection, vary from patient to patient. vate practice at an individual level.
l Public health professionals like the clinician would
Ans.
like to have the ideal programme plan accepted with
enthusiasm. However, the community’s reaction to l The American Board of Public Health Dentistry and the
such a plan may be to reject it or to carry out only a American Dental Association modified the Winslow
part of it or to adopt a less costly alternative. It is the definition of public health and defined Dental Public
community that makes the ultimate decision. Health as ‘the science and art of preventing and control-
D. Treatment/payment operations ling dental disease and promoting dental health through
l Execution of the programme once the plan has been organized community efforts’.
accepted by all the parties involved. l A suggested modification of Knutson’s definition is as

l When a treatment plan has been accepted, the clini- follows: ‘Dental public health is concerned for and ac-
cian arranges a schedule to carry out the treatments. tivity directed towards the improvement and promotion
l Similarly, when a specific community public health of the dental health of the population as a whole as well
programme has been adopted, a public health team as of the individuals within that population’.
with varied disciplines may be called on for pro-
gramme operation. Differences between private practice and community
E. Payment/programme funding health work
l Mutually agreed methods of payment for dental ser- i. Private care seeks to minimize the chance that the best pos-
vices are arranged between the patient and the dentist. sible outcome will occur, often unlimited by resource re-
l Private practitioner and the patient usually mutually straints. Public health work, conversely, seeks to minimize
agree on the payment of the dental service before the the chance that the worst possible outcome will occur.
treatment plan is undertaken, and it can be in the ii. Private practitioners are relatively independent health
form of cash down payment or monthly billing. care providers and the decisions made by them are in
l Programme funding is often a complicated mix of local, the context of their training, legal framework of dental
state and federal funds, which the dental public health practice and the dentist–patient relationship. By con-
professional must know how to secure and manage. trast, the public health professionals are usually sala-
l By and large, community health programmes are ried employees who are accountable to both their im-
financed by the local governing body of the commu- mediate supervisors and the taxpayers.
nity, state or central government or by several volun- iii. Private practitioners by and large serve middle-class
tary organizations. patients who have at least some concern for their oral
l Grant proposals for dental programmes, submitted to health. However, focus of the public health professionals
local service clubs and local foundations, have is often in communities with special characteristics of
proved successful in many cases. culture, socioeconomic status and values, which make
l However, the continuance of the programme depends these groups more difficult and expensive to reach.
on the support or the felt needs of the communities; iv. Private practitioners on an average take home higher pay,
lack of funds initially or at subsequent appropriations have their own choice of location based on personal/pro-
may cause abortment of the plan. fessional reasons and have the option of partnership with
F. Evaluation/programme appraisal other dentists or to work under someone on a commission/
l The dentist’s or hygienist’s evaluation of progress consultancy basis. All this is not possible for public health
begins during the course of treatment and is repeated employees. Although public health professionals take
at each visit. home a lower amount of pay, they enjoy fringe benefits
l Observations made during the initial examination, such as retirement, sick leave, insurance and paid vacation.
such as extent of plaque and calculus deposits, are v. Employees of institutions get rewards and challenges of
evaluated from time to time on recall. being on the cutting edge of new developments, of in-
l Evaluation can be objective, only if careful examination teracting with talented fellow faculty members and of
of records of initial conditions were made and are avail- relating to students who represent the future. Such atti-
able for study and comparison with later observations. tude is difficult to maintain for private practitioners.
578 Quick Review Series for BDS 4th Year, Vol 1

vi. Public health professional have several career options, detail. Only after this is the programme planning done
such as the armed forces and public health services. and the search for funds is started.
Private practitioners have to most of the time stay xi. Most of the treatment part is done by the private prac-
confined to their personal clinics. titioner alone, with only a few referrals for every
vii. In private practice, the individual dental practitioner complex part of it. However, a public health team
usually deals with one patient at a time. However, in a comprises several disciplines and individuals.
public health set-up the health professional deals with xii. Private practice undertakes both curative and preven-
an entire group/population. tive aspects of very complex diseases, while public
viii. In private practice, it is the patient who comes to the health programmes usually work only with the pre-
practitioner. In a community health care, it is the pub- ventive aspect and refer the cases to bigger health
lic health practitioner who goes to the patient. centres if the disease is in full-blown stage.
ix. In private practice, the patients coming to receive treat- xiii. Treatment done by private practitioners is usually
ment pay for the services rendered to them by the prac- immediate and visible. However, public health pro-
titioner. However, this might not be true in a community grammes deal with issues that usually prove to be
set-up, as the public health programmer usually has to beneficial only in the long run, e.g. fluoridation pro-
depend on outside sources for funding of the expenses. grammes and epidemiological studies.
x. Private practitioners usually take a rapid medical his- xiv. Private practitioners usually require large capital and
tory, examination and reach a final diagnosis. Then a money to run daily expenses. Also, there are certain
treatment is planned and done within a few appoint- legal frameworks of rules that they have to be careful
ments. However, in a public health programme, first about, e.g. COPRA. The community health workers
the need to actually undertake a particular programme are by and large free from such tensions.
depends on whether it fulfils all the requirements of a xv. Private practitioners earn on a day-to-day basis. The
public health problem. Once this is done, next comes public health professionals get their perks at the end of
the need to survey the population and the problem in the month.

Differences between private dental practice and public health dentistry


Characteristics Private dental practice Public health dentistry
Target group Individual patient A community or a group of individuals
Collection of information Taking proper history and oral clinical examination Analysis of health and morbidity records available
Special investigations Radiography, blood test, biopsies, exfoliative Epidemiological surveys
cytology
Basis of conclusion Diagnosis Situational analysis of oral health status and needs
and utilization of services
Major emphasis Curative and restorative care Promotive and preventive care
Requirements for success Patient’s consent and cooperation Community participation
Service provider Dentist alone Health team professionals and paraprofessionals,
community volunteers
Intervention Appropriate dental procedure Promotive and preventive measure at individual
and community level
Supportive disciplines Psychology Social psychology, education, epidemiology and
biostatistics
Organization and management Not relevant Highly relevant
Perspective Immediate Long term
Evaluation and results Relief of symptoms and restoration of function Formal programme evaluation
Aftercare Recall, further sittings Continuing core, self-core

Funding Generally by the patient themselves Generally by government or local authorities


concerned
Section | I  Topic-Wise Solved Questions of Previous Years 579

Q.3. Define community dentistry. What are its objectives? l Epidemiology is also defined as the study of the dis-
Discuss the various tools used in public health dentistry. tribution and determinants of health-related events in
population.
Ans.
Aims of epidemiology
l The American Board of Public Health Dentistry and the l To minimize or eradicate the disease or health

American Dental Association modified the Winslow problem and its consequences
definition of public health and defined Dental Public l To minimize the chances of its occurrence in future

Health as ‘the science and art of preventing and control- Objectives of epidemiology
ling dental diseases and promoting dental health through l To define the magnitude and occurrence of dis-

organized community efforts’. ease conditions in man


l A suggested modification of Knutson’s definition is as l To identify the aetiologic factors responsible for

follows: ‘Dental public health is concerned for and ac- the above conditions in man
tivity directed towards the improvement and promotion l To provide data necessary for planning, imple-

of the dental health of the population as a whole as well mentation and evaluation of programmes aimed
as of the individuals within that population’. at preventing, controlling and treating diseases
l It is that form of dental practice which serves the ii. Biostatistics
community as a patient rather than an individual. It is l Biostatistics is the branch of statistics concerned

concerned with the dental education of the public with with mathematical facts and data related to biologi-
applied dental research and with the administration of cal events.
group dental care programme as well as the prevention l Medical statistics is a further speciality of biostatis-
and control of dental diseases on a community basis. tics when the mathematical facts and data are related
to health, preventive medicine and disease.
Objectives of Public Health Dentistry
Uses of biostatistics
l To provide health education in community, motivation
l To define normalcy
to the individual or motivation to the community for
l To test whether the difference between two popu-
receiving dental services
lations, regarding a particular attribute, is real or a
l To deliver dental treatments to the community through
chance occurrence
organized dental health camps, especially in rural areas
l To study correlation or association between two
l To provide dental prophylaxis and other oral hygiene mea-
or more attributes within the same population
sures, including instructions in home care of the mouth
l To evaluate the efficacy of vaccines, sera, etc. by
l To deliver dental care to children through school health
control studies
programmes
l To locate, define and measure the extent of mor-
l To implement application of caries preventive measures such
bidity and mortality in the community
as topical applications of fluorides, pit and fissure sealants
l To evaluate the achievements of public health
l To demonstrate new dental preventive methods and pro-
programmes
cedures to the community
iii. Social sciences
l To provide dental treatments like extractions, oral pro-
l Social sciences usually include sociology, cultural an-
phylaxis and fluoride applications through dental camps
thropology and psychology. They frequently include
or through school health programmes
economics, government and, sometimes, history.
l To conduct dental public health activities and field ex-
l The public health workers, when they embark upon orga-
periences for dental students and dental student nurses
nized community effort, are very dependent on the group
behaviour of individuals determined by their culture.
{SN Q.8} l One of the important developments in public health

Tools of Dental Public Health during the last decade is that the social scientists
i. Epidemiology have been called in to aid in adapting new health
ii. Biostatistics programmes to existing cultural patterns.
l The social scientists become necessary when effort
iii. Social sciences
iv. Principles of administration. and effect do not match each other, and we want to
know why.
l They help us in the assessment of the process our

i. Epidemiology programme is using or plans to find out how well


l Epidemiology is defined as a scientific study of fac- this process fits with the sociocultural system of the
tors and conditions related to disease as they occur group with which we are working.
in people. The word epidemiology is derived from iv. Principles of administration
epi (in, on or upon), demos (people) and logos (sci- l The dentist with a leadership role in public health

ence). Parkin in 1873 defined epidemiology as the programme needs to know many of the principles by
branch of medical science dealing with epidemics. which large enterprises are administered.
580 Quick Review Series for BDS 4th Year, Vol 1

l There are two main areas into which administrative l Prohibit eating, drinking, handling contact lenses, etc.
work may be divided – organization and management. in contaminated environments.
l Organization deals with the structure of an agency l Ban storage of food and drinks in refrigeration or other

and the way people are arranged into working spaces where blood or infectious materials are stored.
groups within it. l Place blood and contaminated specimen to be shipped,

l Management is concerned with the handling of per- transported or stored into suitable closed containers that
sonnel and operations in such a way that the work of prevent leakage.
the agency gets done. l At no cost to employees, provide them with necessary

personal protective equipment (PPE) and clear direc-


Q.4. What are the similarities between personal and
tions for use of appropriate universal barrier protection
community health care workers?
in treating all patients (PPE includes gloves and gowns).
Ans. l As soon as feasible after treatment, attend to housekeep-

ing requirements, including floors and sinks, that are


[Same as LE Q.1]
subject to contamination.
Q.5. List the differences between clinical practice and l Provide a written schedule for cleaning.

dental public health. l Contaminated sharps are regulated waste; discard them

in hard-walled containers.
Ans.
l Contaminated equipment that require service must first
[Same as LE Q.2] be decontaminated, or a biohazard label must be used to
indicate contaminated parts.
Q.6. Define health, public health, dental public health
l Ensure that employees correctly use and discard PPE or
and community dentistry. Describe the tools of commu-
properly prepare them for reuse.
nity dentistry.
l Provide laundering of protective garments used for uni-
Ans. versal precautions at no cost to employees.
[Same as LE Q.3] Q.2. Define community dentistry. Discuss the various
community programmes by which oral diseases can be
prevented.
SHORT ESSAYS:
Ans.
Q.1. OSHA regulation of infection control.
Ans. {SN Q.2}
The OSHA regulations of infection control are summa- Winslow has defined public health as ‘the science and
rized as follows: art of preventing disease, prolonging life and promoting
l Provide hepatitis B immunization to employees without
physical and mental efficiency through organized com-
charge within 10 days of employment. munity effort for the sanitation of the environment, the
l Require that universal precautions be observed to prevent
control of communicable infections, the education of the
contact with blood and other potentially infectious mate- individual in personal hygiene, the organization of med-
rial. Saliva is considered to be a blood-contaminated ical and nursing services for the early diagnosis and
body fluid in relation to dental treatments. preventive treatment of disease and the development of
l Implement engineering controls to reduce production of
the social machinery to ensure everyone a standard of
contaminated spatter, mists and aerosols. living adequate for the maintenance of health, so orga-
l Provide facilities and instructions for washing hands
nizing these benefits as to enable every citizen to realize
after removing gloves and for washing skin immedi- his birthright of health and longevity’.
ately or as soon as feasible after contact with blood or
potentially infectious materials.
l Needles and other sharp items are prescribed to be The various community programmes which can prevent
handled safely. oral diseases are as follows:
l Prescribe disposable or single-use needles, wires, carpules l Public health programmes are usually identified with

and sharps as close to the place of use as possible, as soon the subsets of population other than the affluent. There
as feasible, in hard walled, leak-proof containers that are is a great need for methods that will permit better target-
closable. Containers must be red or bear a biohazard label. ing of individuals at high risk to oral disease.
Teeth must be discarded into the same containers. l Additional emphasis is needed on periodontal disease

l Implement work practice control precautions to mini- detection and treatment programmes. Probably the two
mize splashing, spatter or contact of bare hands with most cost-effective dental-preventive procedures are the
contaminated surfaces. combined use of fluorides and pit and fissure sealants.
Section | I  Topic-Wise Solved Questions of Previous Years 581

l Educational and political methods should be used to b. Oral cancer screening programme
ensure universal fluoridation, and greater use of auxilia- l Probably the programme in which most health

ries is necessary in the delivery of preventive dentistry professionals participate as a public service is the
health services. oral cancer screening programme.
l Removal of the barrier to care and the improved l Oral cancer is found more frequently in persons

oral health result in an effective dental public health over 50 years of age, in heavy smokers or in alco-
programme. holics. To begin the process of programme plan-
l Projects in this category may include: ning, the first step is to look at the population
a. A caries bottle-mouth syndrome education programme served to discern their need for an oral cancer
for young mothers screening programme.
b. An oral cancer screening programme for older l Communications with the target population and

adults with health professional peers should begin after


c. Dental health education and prevention obtaining answers to the following questions:
d. Proving dental services to special population groups What is the age makeup of the population?
e. A mouth-guard fabrication programme for high What are the mortality, morbidity, incidence and
school football players prevalence of oral cancer in this population?
f. A fluoride therapy programme for school children, etc. Who will sponsor the programme, and how will it
be staffed and funded?
a. Caries bottle-mouth syndrome education programme What existing resources are available?
for young mothers l It is important that lay individuals should be inter-

l The principal dental diseases, caries and peri- viewed to discern their knowledge of oral cancer
odontal diseases, are both infectious and chronic, as well as their receptivity to such a cancer screen-
and deserve their share of attention towards their ing programme.
prevention. l Professional societies and organizations with

l The majority of available resources have been the prime interest in the programme should be
allocated to caries control for several reasons: consulted.
i. Caries was considered a disease of children for l Arrangements need to be made for establishing

whom a great amount of sympathy existed; the screening sites. Their locations should be as
periodontal disease was considered a disease of convenient as possible for the adult population,
adults, who could take care of themselves. especially the geriatric groups. Neighbourhood
ii. It was easier to target survey and preventive health centres, family clinics and nursing homes
programmes for children in conventional school are ideal settings for such programmes.
settings than for adults scattered throughout the l If possible, the cancer screening should be ar-

work place. ranged as part of a health fair, at which other


iii. The current caries pattern in children indi- screening activities are simultaneously con-
cates that the fissured tooth surfaces experi- ducted. Such scheduling greatly increases par-
ence the vast majority of caries attacks, while ticipation.
a decline has occurred on smooth surfaces. l Arrangements need to be made with appropriate

iv. Pit and fissure sealants offer a highly effective private, public health or hospital-based histopa-
means to protect these vulnerable occlusal thology laboratories for processing cytology
surfaces. Sealants can be used in innovative smears or biopsies arising from the programme.
ways in public health programmes. They can l The programme should be funded properly and

be used effectively in conjunction with mini- supported. The short-range goal is to induce as
mum restorations as well as over incipient many people as possible to enter the pro-
lesions where complete sealing is ensured to gramme; the long-term goal is to reduce the
avoid the loss of tooth tissue. number of deaths resulting from undetected
v. The preventive benefit provided to a commu- oral cancer.
nity by the initiation of fluoridation or a changed l Once the screening phase is completed, it is then

practice act that permits hygienists to apply necessary to ensure that all referred persons are
sealants may be greater than the benefit attained sent for a definitive diagnosis and the confirmed
from a lifetime of practice by a dozen dentists. neoplastic lesions are treated.
vi. As caries-control measures are increasing in c. Dental health education and prevention
effectiveness, emphasis has shifted to devel- l Dental health education and prevention pro-

oping a nationwide periodontal disease con- grammes have traditionally comprised a signifi-
trol initiative. cant portion of dental public health activities.
582 Quick Review Series for BDS 4th Year, Vol 1

l Programmes devoted to school children have been Type 2: Limited Examination


particularly popular because children have been l This is a more limited examination, using mouth mir-

highly susceptible to dental caries. ror and explorer, good illumination, posterior bitewing
l Enthusiasm for dental health education in many radiographs or periapical radiographs, when indicated.
school programmes needs tempering because it l This is used in intensive clinical studies of special

promotes unrealistic expectations in caries control groups.


that are often not realized when programmes are Type 3: Inspection
carefully evaluated. When expectations are not l This is indicated in public health surveys/epidemio-

realized, it may result in disappointment and the logical surveys and is done by using mouth mirror
attitude that no school-based preventive pro- and explorer and available illumination.
grammes are successful. Type 4: Screening
l Rather than eliminating school-based preventive l This is done by using tongue depressor and available

programmes that some might think are ineffective, it illumination. It is used in the inspection of school
is important that they be maintained by employing children to discover quickly the need of treatment.
careful evaluation methods designed to determine
Q.2. Define community dentistry.
the impact of the programme on the oral health of
target population in terms of reduced disease. Ans.
d. Special population groups
[Ref SE Q.2]
l Traditionally, dental public health programmes

have consisted of a number of projects designed Q.3. Complete examination.


for special subgroups in the population.
Ans.
l Certain groups, because of their occupation, posi-

tion or location, do not have access to private [Same as SN Q.1]


practitioners and therefore must be cared for in
Q.4. Dental practice management.
special clinics.
l Thus, military populations, inhabitants of Indian Ans.
reservations, prisoners, institutionalized people,
Setting up of a dental clinic and dental practice manage-
veterans administration hospital patients, handi-
ment is a complex task that depends on various aspects of
capped, nursing home residents, geriatric popula-
dentistry.
tions and the indigent have traditionally received
Management of dental office depends on
care in clinics supported by public or private
l Dentist
funds.
l Dental assistant
l The provision of restorative and preventive care to
l Receptionist
many of these groups requires special training and
l Cleaning personnel
particular skills.
l Business manager

l Material usage
l Lab technician
SHORT NOTES: l Disposal of waste

Q.1. What are the types of oral examination? Dentist


l The dentists should have a good communication with
Ans.
their patients. If the ideal qualities like punctuality,
American Dental Association (1970) has classified the personality, patience and perfection are developed by
types of inspection and oral examination as follows: identities, they will definitely be successful in their
a. Type 1: complete examination private practice.
b. Type 2: limited examination Dental assistance
c. Type 3: inspection l The assistant should keep the working area and all

d. Type 4: screening the necessary equipment ready for the work.


Type 1: Complete Examination Dental receptionist
l It involves a complete examination, using mouth mir- l Dentists should appoint a full-time receptionist, who

ror and explorer, adequate illumination through full should be trained to keep records in files or to feed
mouth radiographs and additional diagnostic methods, them to computer, and should be able to handle the
such as percussion, pulp-vitality tests, transillumina- patient crowd by giving appointments after consult-
tion, study models and other laboratory tests. ing the dentist.
Section | I  Topic-Wise Solved Questions of Previous Years 583

Cleaning personnel l As per an estimate in 1995, the total number of dentists


l Appoint part-time personnel to clean dental office in India was 31,694.
floor, equipment, etc. Registered ‘A’ class: 27,720 (‘A’ class – degree from a
Dental laboratory recognized dental institute)
l Dentist should have business association with a Registered ‘B’ class: 3974 (‘B’ class – no bachelor’s
good dental laboratory which delivers the work on degree; practicing because of experience or technical
time. qualifications)
Business manager All India dentist to population ratio – 1:25,000 (A and
l Business manager should look after all the financial B combined)
matters related to dental office. Dentists serving in rural India – 10–12%
Material usage Dentists serving urban India – 90%
l Care should be taken to have sufficient amount of l Considering the number of dental graduates passing out

material in the stock so that the routine work does each year, it was estimated that in the year 2000, there
not get disturbed. were about 50,000 dentists that bring the dentist popula-
Disposal of waste tion ratio to 1:20,000 (Source: 4th Conference of Central
l Every dental clinic should have its own arrangement Council of Health and Family Welfare – Proceedings of
for the proper disposal of waste. National Oral Health Policy).
Q.5. Importance of dental records. Q.7. Dental manpower planning in India.
Ans. Ans.
Dental patient record is the data storage system, which con- l Manpower planning is part of a general plan for provid-
sists of patient’s history, examination, diagnosis, treatment ing health services.
plan and treatment, follow-up and prognostic information. l Health manpower planning has been defined as the pro-
Advantages of computerized dental records are as cess of estimating the number of persons and the kind
follows: of knowledge, skills and attitudes they need to achieve
l Record duplication can be performed in office. predetermined health targets and attaining health status
l Minimal amount of time and materials is required. objectives.
l Stored records are extremely portable and durable. l Such planning also involves specifying who is going to

l Records can be archived as needed, considering space do what, when, where, how, with what resources, and
and timeliness. for which population groups or individuals so that the
l Compactness and rapid accessing of records is simple, knowledge and skills necessary for adequate perfor-
quick and accurate. mance can be made available according to predeter-
l With the inclusion of emails in dental records, dentists mined policies and time schedules.
can create information links with other dentists, insur- l This planning requires continuous monitoring and eval-

ers, dental associations, academies, biomedical librar- uation, and it must be a continuing and not a sporadic
ies, dental supplies and speciality centres. process.
l The computer-based oral health records can be pro-

grammed to display and describe a patient’s condition, Q.8. Tools of public health.
diagnosis and treatment options to facilitate education Ans.
and informed consent.
[Ref LE Q.3]
Q.6. Dentist population ratio.
Q.9. Setting up of dental clinic.
Ans.
Ans.
l The dental health manpower in the country depends on
various factors, e.g. total population, literacy and the [Same as SN Q.4]
economic status of the country.
l The dentist to population ratio in the year 1978 was Q.10. Advantages of computerized dental records.
1:68,400 and the total number of dentists was 8750. Ans.
l In 1985, the total number of registered dentists (quali-

fied and unqualified) was stated as 10,000. [Same as SN Q.5]


584 Quick Review Series for BDS 4th Year, Vol 1

Topic 2
Public Health
COMMONLY ASKED QUESTIONS

LONG ESSAYS:
1. Define potable water. Write in detail the steps taken in large-scale water purification.
2. Define health education. Write in detail the steps of health education.
3. Define mass education and describe in detail the various methods and means of mass education.
4. Describe and discuss various methods used in disposal of biowaste.
5. Discuss the aims and objectives of school dental health programme.
6. What is potable water? Explain the steps involved in the purification of water on a large scale. [Same as LE Q.1]
7. Describe in detail the large-scale purification of water. [Same as LE Q.1]
8. Explain purification of water by rapid sand filtration. [Same as LE Q.1]
9. Describe briefly the water purification method on large-scale in a city. [Same as LE Q.1]
10. Define health education. Add a note on the principles of health education. [Same as LE Q.2]
11. What is health education? Describe in detail the principles of health education. [Same as LE Q.2]
12. Give the WHO definition of health education. Enumerate the different principles of health education. Explain
different methods for imparting group dental health education. [Same as LE Q.2]
13. Define health education. Enumerate the approaches of health education. Discuss in detail the principles of
health education. [Same as LE Q.2]
14. Define heath education. Discuss the principles of health education. What are the barriers of health education?
How to overcome those barriers? [Same as LE Q.2]
15. Define health education. Classify the educational aids used in health education and add a note on the role of
mass media in health education. Explain the principles of health education in detail. [Same as LE Q.3]
16. Define health education and describe the role of mass media in health education programmes. [Same as LE Q.3]
17. Describe the role of mass media in dental awareness programme. [Same as LE Q.3]
18. Describe the objectives of school dental health programme and add a note on the concept of incremental dental
care. [Same as LE Q.5]
19. What are the objectives of school dental service? What are the advantages of a school-based programme?
Which are different school dental health programmes? What is incremental dental care? [Same as LE Q.5]
20. Write in detail the steps taken in planning a dental health education programme for controlling diseases in a
school having 3000 students. [Same as LE Q.5]

SHORT ESSAYS:
1. Define health. Enumerate the various environmental factors necessary for the maintenance of general health
of an individual.
2. Define health. Describe ‘Health for All by 2000 AD’.
3. Write in detail about a comprehensive preventive dentistry programme for a primary school in rural area.
4. What is hydrologic cycle? Enumerate the techniques for the purification of water. Write in detail about small-
scale purification of water.
5. What are the different aids used towards dental health education in a community?
6. Describe principles of chlorination.
7. Health care waste and its significance. [Ref LE Q.4]
8. Vitamin B complex.
9. Iceberg of disease.
10. Approaches to health education.
11. Elements of primary health care.
12. Personal barrier technique for infection control.
Section | I  Topic-Wise Solved Questions of Previous Years 585

1 3. Waterborne infections.
14. Define preventive dentistry. Describe incremental dental care and comprehensive dental care. [Same as SE Q.3]
15. Aids to health education. [Same as SE Q.5]
16. Disposal of hospital waste. [Same as SE Q.7]
17. Disposal of dental clinic wastes. [Same as SE Q.7]
18. Incinerator. [Same as SE Q.7]
19. Composting. [Same as SE Q.7]
20. Disposal of waste. [Same as SE Q.7]
21. Iceberg phenomenon. [Same as SE Q.9]
22. Panel discussion. [Same as SE Q.10]

SHORT NOTES:
1. Steps taken in large-scale purification of water. [Ref LEQ.1]
2. Dimensions of health.
3. Composting.
4. THETA. [Ref LE Q.5]
5. Incremental dental care.
6. Break-point chlorination.
7. Panel discussion.
8. Askov dental health education.
9. Tattletooth programme. [Ref LE Q.5]
10. Group discussion.
12. Manure pits.
13. Back washing.
14. Rapid sand filtration.
15. Vitamin D.
16. Soil, seed and sower.
17. School dental health programmes.
18. Bangalore method of composting.
19. Barriers of health education.
20. Safe water.
21. Mass media. [Ref LE Q.3]
22. Audiovisual aids.
23. Vital layer.
24. Contents of health education.
25. Workshop.

SOLVED ANSWERS
LONG ESSAYS:
Q.1. Define potable water. Write in detail the steps taken Purification of water consists of the following two types:
in large-scale water purification. . Large-scale purification of water
A
B. Small-scale purification of water
Ans.
Potable water {SN Q.1}
Potable water is the type of water that cannot harm the
consumer, even when ingested over prolonged periods. Large-Scale Purification of Water
Potable water, also called safe or wholesome water, is It is of three types:
defined as the one that is free from pathogenic agents and A. Storage
harmful chemical substances, pleasant to taste and usable B. Filtration
for domestic purposes. C. Chlorination
586 Quick Review Series for BDS 4th Year, Vol 1

A. Storage l The formation of vital layer is known as ‘rip-


l Here the water is drawn out from the source and ening’ of filter. It may take several days for the
impounded in natural or artificial reservoirs. vital layer to form. It extends 2–3 cm into the
l A reserve of water is provided by the storage from top portion of the sand bed.
which further pollution is excluded. This is the natu- l It helps in removing organic matter and free-

ral method of purification. ing water from bacteria, and holds back bacte-
(a) Physical: 90% of suspended impurities settle ria and oxidizes ammonical nitrogen into
down in 24 h by gravity. The water becomes nitrates.
clearer. l Until the vital layer is formed fully, for the

(b) Chemical: Certain chemical changes take place. first few days, filtrate is usually run to waste.
Storage results in the reduction of ammonia At the bottom of the filter bed is the under-
content and rise in nitrates. drainage system. It consists of porous or per-
(c) Biological forated pipes that serve the dual purpose of
l There is reduction in bacterial count. providing an outlet for filtered water and sup-
l Degradation of pathogenic organisms. porting the filter medium above.

l If the water is stored for a long period, there iii. Filter box
is likelihood of growth of algae that imparts The first three elements are contained in the filter
bad smell and colour to the water. box. The filter box is an open box, usually rect-
B. Filtration angular in shape, 2.5 to 4-m deep and is built
Two types of filters are used: the ‘biological’ or ‘slow wholly or partly below ground. The walls may be
sand’ filters; and the ‘rapid sand’ or ‘mechanical’ filters. made of stone, brick or cement. The filter box
Slow sand or biological filters consists from top to bottom:
Slow sand filters essentially consist of the following:
i. Supernatant water Supernatant water 1–1.5 m
ii. A bed of graded sand
Sand bed 1.2 m
iii. An under-drainage system
iv. A system of filter-control valves Gravel support 0.30 m

Filter bottom 0.16 m


i. Supernatant Water
The supernatant water above the sand bed
serves two important purposes: It provides a iv. Filter control
constant head of water, and second, it pro- The purpose of this device is to maintain a steady
vides waiting period of some hours for the rate of filtration.
raw water to undergo partial purification by Following are the advantages of slow sand filter:
sedimentation, oxidation and particle agglom- l It is easy to construct and operate.

eration. l Low construction cost.

ii. Sand Bed l High physical, chemical and bacteriological qual-

l The most important part of the filter is the ity of filtered water.
sand bed, which presents a vast surface l 99.9% reduction in bacterial count.

area. The sand should be clean and free Rapid sand or mechanical filters
from clay and organic matter. Rapid sand filters are of two types: the gravity type, and
l The water is subjected to mechanical the pressure type. The following steps are involved in
straining, sedimentation, adsorption, oxi- the purification of water by rapid sand filters:
dation and bacterial action. l Coagulation: First, the raw water should be

l Vital layer: The filter which is newly laid treated with chemical coagulant, e.g. alum, the
acts merely as a mechanical strainer, and dose of which varies from 5 to 40 mg or more per
cannot truly be considered as ‘biological’. litre, depending upon the turbidity and colour,
As time passes, a slimy growth covers the temperature and pH of water.
surface of the sand bed, which is known as l Rapid mixing: The treated water is then subjected

‘schmutzdecke’, vital layer, zoogleal layer to violent agitation for a few minutes in a ‘mixing
or biological layer. This layer consists of chamber’.
thread-like algae and plankton, diatoms and l Flocculation: Next, treated water is slowly and

bacteria, and it is slimy and gelatinous in gently stirred for 30 min in a ‘flocculation
nature. chamber’.
Section | I  Topic-Wise Solved Questions of Previous Years 587

l Sedimentation: The coagulated water is then iii. Perchloron


led into sedimentation tanks and kept for about l Chlorine gas is more advantageous because it

2–6 h when the flocculent precipitates together is cheap, quick in action, efficient and easy to
with impurities and bacteria settle down in the apply. A special equipment known as ‘chlori-
tank. nating equipment’ is required for applying
l Filtration: The partly clarified water is now chlorine gas to water supplies because chlo-
subjected to rapid sand filtration. rine gas is an irritant to eyes and is poisonous.
Filtration l Chloramines, which are loose compounds of

During the phase of filtration, the filters are chlorine and ammonia, have fewer tendencies
clogged by suspended impurities and bacteria. to produce chlorinous taste and give more
The filters soon become dirty and begin to lose persistent type of residual chlorine.
their efficiency. When the ‘loss of head’ ap- l The disadvantage of chloramines is that they

proaches 7–8 feet, filtration is stopped and the have a slower action than chlorine and so they
filters are subjected to a washing process known are not being used to any great extent in water
as ‘back washing’. treatment.
Back washing l Solutions prepared from high-test hypo-

Washing accomplished by reversing the flow of chlorite (HTH) are also used for water dis-
water through the sand bed is called back wash- infection.
ing. This dislodges the impurities and cleans the Break-point chlorination
sand bed. Compressed air is used as part of the When chlorine dose in water is increased, a reduction
back-washing process. occurs in residual chlorine. The end products do not
Advantages represent any residual chlorine. This fall in residual
l Rapid sand filter can deal with raw water chlorine continues with further increase in chlorine dose
directly. until after a stage residual chlorine begins to increase in
l Less space is occupied by the filter bed. proportion. The point at which it appears is called
l Rapid filtration is 40–50 times that of slow break-point chlorination or dosage.
sand filters. Other Agents
l Easy washing of the filter. l Ozonation

l More flexibility in operation. l Ultraviolet radiation

C. Chlorination
l Chlorination is a supplement, not a substitute to Q.2. Define health education. Write in detail the steps of
sand filtration. health education.
l Chlorine kills pathogenic bacteria but it has no
Ans.
effect on spores and certain viruses, except in high
doses. Health education is the process that informs, motivates and
l It oxidizes iron, manganese and hydrogen sulphide; helps people to adopt and maintain healthy practices and
it destroys some taste and odour-producing constitu- lifestyles, advocates environmental changes as needed to
ents; it reduces algae and slime organisms, and aids facilitate the goal and conducts professional training and
coagulation. research to the same end.
Action of chlorine Principles of health education
When chlorine is added to water, there is a formation (a) Interest
of hydrochloric and hydrochlorous acids. The alka- (b) Participation
linity of the water neutralizes hydrochloric acid. (c) Known to unknown
Hypochlorous acid ionizes to form hydrogen ions (d) Comprehension
and hypochlorite ions. (e) Reinforcement
Hypochlorous acid plays a major role in the disin- (f) Motivation
fecting action of chlorine. Hypochlorite ions contrib- (g) Learning by doing
ute to a small extent. Hypochlorous acid is 70–80 (h) Soil, seed and sower
times more effective than hypochlorite ions. (i) Good human relations
Method of chlorination (j) Community leaders
Chlorine is applied as one of the following means: (a) Interest
i. Chlorine gas l It is a psychological principle that people are un-

ii. Chloramine likely to listen to things that are not of their interest.
588 Quick Review Series for BDS 4th Year, Vol 1

l Health teaching should relate to the interest of the (i) Good human relations
people. If a health programme is based on the needs l Good human relations are of utmost importance in

of the people, they will be willing to participate in learning. The health educator must be kind and sym-
the programme and only then it will be a people’s pathetic.
programme. (j) Leaders
(b) Participation l In health education work, we try to penetrate a com-

l It is very important in health education. Participation munity through local leaders – the village headman,
is based on psychological principle of active learn- the schoolteacher or the political worker – because
ing; it is better than passive learning. they are the people who the community respects. If the
l Group discussion, panel discussion and workshop leaders are convinced first about the programme, the
provide opportunities for active learning. task of implementing the programme will be easy.
(c) Known to unknown Barriers of communication
l Health education work should be started from the a. Physiological – difficulties in hearing, expression
known to unknown, i.e. start with where the people b. Psychological – emotional disturbances and neurosis
are and what they understand and proceed to new c. Environmental – noise, invisibility and congestion
knowledge. d. Cultural – levels of knowledge and understand-
l New knowledge will bring about a new, enlarged ing, customs, beliefs and attitudes
understanding, which can give rise to an insight into The barriers should be identified and removed for
the problem. achieving effective communication.
(d) Comprehension The practice of health education involves a num-
l The level of understanding, education and literacy of ber of health education materials.
people to whom the teachings are directed to should Audiovisual aids
be known. Words that cannot be understood should These help to simplify unfamiliar concepts and bring
not be used. about understanding where words fail. Audiovisual
l Communication should always be in the language aids are categorized as follows:
that people understand, and never use words that are (a) Auditory aids: radio, tape recorder, micro-
strange and new to them. Teaching should be within phones, amplifiers and earphones
the mental capacity of audience. (b) Visual aids:
(e) Reinforcement l Not requiring projection: chalkboard, leaf-

l Only few people can learn all new things in a single lets, posters, charts, flannel graphs, exhib-
period. Repetition at intervals is needed. Every health its, models, specimens, etc.
campaign needs reinforcement; we may call it a l Requiring projection: slides, filmstrips

‘booster dose’. (c) Combined audiovisual aids: television, sound


(f) Motivation films and slides-tape combination
l Awakening the desire of a person to learn is called Methods in health communication
motivation. There are two types of motives – Primary The methods in health communication may be
and secondary. grouped as follows:
l Primary motives are driving forces initiating (a) Individual approach
people into actions, these motives are inborn Health education may be given in personal inter-
desires. views in the consultation room or in the health centre
l Secondary motives are based on desires created by or in the homes of the people. Public health nurses,
outside forces or incentives. health visitors and health inspectors have plenty of
(g) Learning by doing opportunities for individual health teaching.
l Learning is an action process, not a memorizing one Advantages
in a narrow sense. l Can discuss, argue and persuade the indi-

l The Chinese proverb ‘if I hear, I forget; if I see, vidual to change his or her behaviour.
I remember; if I do; I know’ illustrates the impor- l Provides opportunities to ask questions in

tance of learning by doing. terms of specific interests.


(h) Soil, seed and sower Disadvantage
l The people are the soil, the health facts are the seeds l Numbers reached are small.

and the transmitting media is the sower. (b) Group approach


l The seeds or health facts must be truthful and based Group teaching is an effective way of educating
on scientific knowledge. The transmitting media a community. The subject must relate directly to
should be attractive, palatable and acceptable. the interest of people.
Section | I  Topic-Wise Solved Questions of Previous Years 589

Chalk and talk (lecture): A lecture may be defined topic briefly and invites the panel speakers to present
as carefully prepared oral presentation of facts, or- their points of view. The chairman has to keep the
ganized thoughts and ideas by a qualified person. discussion going and develop the train of thought.
l The group should not be more than 30 l After the main aspects of the subject are explored

members and the talk should not exceed by the panel speakers, the audience are invited to
15–20 min. The lecture method can be take part. Panel discussion can be an extremely
made more effective by combining with effective method of education, provided it is prop-
suitable audiovisual aids, such as flipcharts, erly planned and guided.
flannel graphs, exhibits, films, and charts. Symposium
Disadvantage: Learning is passive and it does not l A symposium is a series of speeches on a selected

stimulate thinking and problem-solving capacity. subject. Each person or expert presents an aspect of
Demonstrations the subject briefly. There is no discussion among
l A demonstration is a carefully prepared presenta- the symposium members in the panel discussion.
tion to show how to perform a skill or procedure. In the end, the audience may raise questions. A
l It is carried out step by step before an audience or chairman makes a comprehensive summary at the
target group. The demonstrator involves the audi- end of the entire section.
ence in discussion. l Workshop: It consists of series of meetings, usu-

Group discussion ally four or more, with emphasis on individual


l Group discussion permits individuals to learn by freely work; it may be divided into small groups and
exchanging their knowledge, ideas and opinions. each group will choose a chairman and a recorder.
l Group discussion is considered valuable where The individuals work, solve a part of the problem
long-term compliance is involved (e.g. cessation through their personal effort with the help of con-
of smoking). sultants, contribute to group work and group dis-
l The group should comprise not less than 6 and not cussion and leave the workshop with the plan of
more than 12 members. The participants should action on the problem. The workshop provides
be visible to each other. each participant opportunity to improve his effec-
l A group leader initiates the subject, helps the dis- tiveness as a professional worker.
cussion in a proper manner, prevents side conver- l Role-playing: Role-playing or sociodrama is

sations, encourages everyone to participate and based on the assumption that many values in a
sums up the discussion in the end. situation cannot be expressed in words and the
l In a group discussion, the work is discussed communication can be more effective if the situa-
within the group with the help of consultants and tion is dramatized by the group. The audience are
resource personnel. not passive but actively concerned with drama. It
l The total workshop members should observe the is particularly a useful educational device for
following rules: schoolchildren.
(a) Idea should be expressed clearly and concisely. Mass approach – education of the general public
(b) Should listen to what others say. l An effective health education programme for the

(c) Should not interrupt when others are speaking. whole community can be carried out through
(d) Only relevant remarks should be made. mass media communication. Mass media are
(e) Should accept criticism gracefully. ‘one-way’ communication.
(f) Should help to reach conclusions. l The mass media include television, radio, health

Limitations magazine, newspaper and printed material.


Those who are shy may not take part in the discus- l They are useful in transmitting messages to peo-

sions. Some members may dominate the discussion. ple even in remote places.
Some members may deviate from the subject and l Their effectiveness can give high returns for the

make the discussion irrelevant or unprofitable. time and money involved.


Panel discussion
l Persons who are qualified to talk about the topic Q.3. Define mass education and describe in detail the
sit and discuss a given problem, or the topic, in various methods and means of mass education.
front of a large group or audience.
Ans.
l Panel discussion comprises a chairman and about

four to eight speakers. Health education is a process that informs, motivates and
l The chairman opens the meeting, welcomes the group helps people to adopt and maintain healthy practices and
and introduces the panel speakers. He introduces the lifestyles, advocates environmental changes as needed to
590 Quick Review Series for BDS 4th Year, Vol 1

facilitate the goal and conducts professional training and l Primary motives are driving forces initiating people
research to the same end. into actions, these motives are inborn desires. Secondary
motives are based on desires created by outside forces
Principles of health education
or incentives.
(a) Interest
(g) Learning by doing
(b) Participation
l Learning is an action process, not a memorizing one
(c) Known to unknown
in a narrow sense.
(d) Comprehension
l The Chinese proverb ‘if I hear, I forget; if I see,
(e) Reinforcement
I remember; if I do; I know’ illustrates the impor-
(f) Motivation
tance of learning by doing.
(g) Learning by doing
(h) Soil, seed and sower
(h) Soil, seed and sower
l The people are the soil, the health facts are the seeds
(i) Good human relations
and the transmitting media is the sower.
(j) Community leaders
l The seeds or health facts must be truthful and based

(a) Interest on scientific knowledge. The transmitting media


l It is a psychological principle that people are un- should be attractive, palatable and acceptable.
likely to listen to things that are not of their interest. (i) Good human relations
l Health teaching should relate to the interest of the l Good human relations are of utmost importance in learn-

people. If a health programme is based on the needs ing. The health educator must be kind and sympathetic.
of the people, they will be willing to participate in (j) Leaders
the programme and only then it will be a people’s l In health education work, we try to penetrate the com-

programme. munity through local leaders – the village headman,


(b) Participation the schoolteacher or the political worker – because
l It is very important in health education. Participation they are the people whom the community respects.
is based on psychological principle of active learn- If the leaders are convinced first about the programme,
ing; it is better than passive learning. the task of implementing the programme will be easy.
l Group discussion, panel discussion and workshop
Educational aids used in health education
provide opportunities for active learning.
They can be basically classified into three categories:
(c) Known to unknown
(A) Audio aids
l Health education work should be started from the
(B) Visual aids
known to unknown, i.e. start where the people are
(C) A combination of audio and visual aids
and with what they understand and then proceed to
new knowledge. (A) Audio aids
l New knowledge will bring about a new and enlarged The audio aids are based on the principles of sound,
understanding, which can give rise to an insight into electricity and magnetism. The modern world makes use
the problem. of different types of audio aids. The most commonly
(d) Comprehension used audio aids in health education are as follows:
l The level of understanding, education and literacy of l Megaphones

people to whom the teachings are directed should be l Public addressing systems or microphones

known. Words that cannot be understood should not l Gramophone records and discs

be used. l Tape recorders

l Communication should always be in the language l Radios

that people understand, and never use words that are l Sound amplifiers

strange and new to them. Teaching should be within (B) Visual aids
the mental capacity of the audience. The visual aids are based on the principle of projec-
(e) Reinforcement tion. Visualization helps individuals to understand
l Only few people can learn all new things in a single better than any other media. The visual aids can be
period. Repetition at intervals is needed. Every health classified as (a) projected aids, (b) nonprojected aids
campaign needs reinforcement; we may call it a and (c) other aids.
‘booster dose’. (a) Projected aids
(f) Motivation These are the visual aids which needs projection
l Awakening the desire of a person to learn is called from a source on to a screen. The various projected
motivation. There are two types of motives – Primary aids include the following:
and secondary. l Films or cinemas
Section | I  Topic-Wise Solved Questions of Previous Years 591

l Film strips [SE Q.7]


Slides
l

l Overhead projectors
{Various methods of waste disposal are
( a) Dumping
l Epidiascopes
(b) Controlled tipping or sanitary landfill
l Transparencies
(c) Incineration
l Bioscopes
(d) Composting
l Video cassettes
(e) Manure pits
l Silent films
(f) Burial
(b) Nonprojected aids
These are the visual aids which do not require any (a) Dumping
projection. The commonly used aids without projec- In this method, refuse is dumped in low-lying areas
tion are (i) blackboard, (ii) pictures, (iii) cartoons, partly as a method of reclamation of land but mainly as
(iv) photographs, (v) posters, (vi) flashcards, (vii) an easy method of disposal of dry refuse. Due to the
charts, (viii) graphs, (ix) flannel boards and (x) leaflets. bacterial action, refuse decreases considerably in vol-
(c) Other aids ume and is converted gradually into humus.
These include the traditional media which makes use of Drawback of open dumping
light and sound. Following are the different methods: l The refuse is exposed to flies and rodents.

Folk dances l It is a source of nuisance from the site of smell


Folk songs and unsightly appearance.
Puppet shows l The loose refuse is dispersed by wind.
Dramas l Drainage from dumps leads to the pollution of

(C) Combination of audio and visual aids surface and ground water.
These are the modern media available. The advantage (b) Controlled tipping
of this type of media is that sound and sight can be Controlled tipping or sanitary landfill is one of the best
combined together to create a better presentation. This methods of refuse disposal where suitable land is avail-
includes the following: able. A trench is prepared and the material is placed in
(a) Television it, adequately compacted and covered with earth at the
(b) Tape and slide combinations end of working day.
(c) Videocassette player and recorder The term ‘modified sanitary landfill’ has been applied to
(d) Motion pictures or cinemas those operations where compaction and covering are
Mass approach – education of the general public accomplished once or twice a week.
Mass media are ‘one-way’ communication. They The following three methods are used in this operation:
are useful in transmitting messages to people even l The trench method: This method is used where

in remote places. levelled ground is available. A 2–3-m deep and


4–12-m wide trench is dug out. The refuse is com-
pacted and covered with excavated earth.
{SN Q.21} l The ramp method: This method is well suited

l Their effectiveness can give high returns for the time where the terrain is moderately sloping. Some ex-
and money involved. The mass media include the cavation is done to secure the covering material.
following: l The area method: This method is used for land de-

l Television pressions, disused quarries and clay pits. The refuse


l Radio is deposited, packed and consolidated in 2–2.5-m
l Health magazine deep uniform layers. Each layer is sealed on its ex-
l Newspapers posed surface with at least 30-cm thick mud cover.
l Printed materials Such sealing prevents infestation by flies and ro-
l Direct mailing dents and suppresses the nuisance of smell and dust.
l Posters, billboards and signs This method often has the disadvantage of requiring
l Health museums and exhibitions supplemental earth from outside sources.
l Films (c) Incineration
It is the best method where suitable land is not available.
Hospital refuse is disposed by this method. The refuse
Q.4. Describe and discuss various methods used in dis-
is reduced to one-fourth of its original weight and the
posal of biowaste.
residue is called ‘clinker’, which is mixed with lime and
Ans. used for road making.
592 Quick Review Series for BDS 4th Year, Vol 1

The incinerator consists of the following: rotating machine and incubated. The entire pro-
l A furnace or combustion chamber maintained at cess completes in 4–6 weeks.
over 1200°C (e) Manure pits
l A platform for tipping the refuse The problem of refuse disposal in rural areas can be solved
l Stokers for raking the refuse forward by digging ‘manure pits’ by individual householders. Gar-
l Baffle plate to drive out all fumes bage, cattle dung, straw and leaves should be dumped into
Types of incinerators manure pits and covered with earth after each day’s dump-
(a) Double-cell meldrum ing. Within 5–6 months, the refuse is converted into ma-
(b) Single-cell destructors nure, which can be returned to fields. This method of refuse
The chief advantage is that the cost of carting is minimized. disposal is best suited in rural communities.
Disadvantages (f) Burial
(i) Expensive This method is suitable for small camps. A 1.5-m wide
(ii) Moisture presence during rainy season and 2-m deep trench is prepared, and at the end of each
affects burning of refuse day, the refuse is covered with 20–30 cm of earth. When
(d) Composting the level in the trench is 40 cm from the ground level,
Composting is a method of combined disposal of refuse the trench is filled with earth and compacted, and a new
and night soil and sludge. The following methods of trench is dug out. The contents may be taken after 4–6
composting are now used: months and used in fields.}
l Bangalore method (anaerobic method): It is also

called the hot fermentation process. It has been Q.5. Discuss the aims and objectives of school dental
recommended as a satisfactory method of dis- health programme.
posal of town wastes and night soil. Trenches, Ans.
3-feet deep, 5–8-feet broad and 15–30-feet long,
are dug. It should be located not less than half a Aims of school dental service
mile from city limits. First, about 6-inches thick l To help every schoolchild appreciate the importance

layer of refuse is spread at the bottom of the of a healthy mouth


trench. Over this, a 2-inch thick night soil is l To help every schoolchild appreciate the relationship

added. Then alternate layers of refuse and night of dental health, general health and appearance
soil are added in the proportion of 6 inches and l To encourage the observance of dental health prac-
2 inches, respectively, till the heap rises to 1 foot tices, including personal care, professional care,
above the ground level. The top layer should be of proper diet and oral habits
at least 9-inch thick refuse. Then the heap is cov- l To correlate dental health activities with the total
ered with excavated earth. Within 7 days, as a re- school health programme
sult of bacterial action, considerable heat is gener- l To stimulate the development of resources to make
ated in the compost mass. This heat persists for dental care available to all children and youths
2 or 3 weeks, and serves to decompose the refuse l To stimulate dentists to perform adequate health ser-
and night soil and to destroy all pathogenic and vices for children
parasitic organisms. At the end of 4–6 months, Objectives of school dental health
decomposition is completed and the resulting l To help every schoolchild appreciate the importance

manure is well decomposed, odourless and in- of a healthy mouth


nocuous material of high manurial value is ready l To help every schoolchild appreciate the relationship

for application to the land. between dental health and general health and appearance
l Mechanical composting (aerobic method): An- l To encourage the observance of dental health prac-

other method of composting becoming popular is tices, including personal care, professional care,
mechanical composting. In this method, compost proper diet and oral habits
is literally manufactured on a large scale by pro- l To enlist all groups and agencies interested in the

cessing raw materials and turning out into a fin- promotion of school health
ished product. Salvable materials, such as rags, l To correlate dental health activities with the total

bones, metal glass and items, which are rendered school health programme
to interfere in grinding, are first cleared from the Advantages of school-based programme
refuse. These are then pulverized in pulverizing l Prevention and maintenance, as the programme starts

equipment in order to reduce the size of particles at an early age. It provides a complete oral examina-
to less than 2 inches. The pulverized refuse is tion during early childhood.
then mixed with sewage, sludge or night soil in a l Initial cost is less.
Section | I  Topic-Wise Solved Questions of Previous Years 593

l Man hours for initial care are less.


Programme philosophy and goals
l Early lesions of dental caries are treated before pulp
l Tattletooth programme relates to the dental health
involvement.
of individuals and focuses on a total person.
l Topical and other preventive measures can be main-
l The goal of the programme is to reduce dental
tained on periodic basis.
diseases and to develop dental habits to last for
l Periodontal disease is interrupted at the beginning
lifetimes.
itself.
Programme implementation
l Service bills are equalized and spaced regularly.
l Statewide implementation plan
l Child develops the habit of visiting dentist periodically.
l Teachers are trained to present dental health infor-
l The programme helps the community to obtain a
mation
favourable impression on the dentist.
School dental health programmes
A. Askov Dental Health Education Cost of materials
l Askov is a small farming community in Minnesota. l It is less than $1.00 per child.

l It includes caries prevention and control, dental E. Tooth keeper programme


health education and dental care. Philosophy and goals
l All recognized methods for preventing dental car- (a) This programme capitalizes on reinforcement
ies are used during demonstration, except com- activities and helps children to develop per-
munal water fluoridation. sonal responsibility for health care.
B. School Dental Health Nurse Programme (b) Primary goal of tooth keeper programme is
l The New Zealand School Dental Nurse Plan was education, rather than oral hygiene, with em-
introduced in 1921. phasis on establishing positive health values.
l This plan has attracted tremendous attention in Implementation
dental circles all over the world. l Teachers are trained each year by dental health

l When the service began, care was offered only to consultants.


younger school-age children but eligibility now l All necessary information and teaching re-

extends to all children in primary and intermedi- sources are provided by this system.
ate schools (2½- to 13½-year old). l The teachers are requested to carry out the

C. Learning about Your Oral Health programme for 16 consecutive weeks.


l ‘Learning about your oral health’ was developed F. THETA programme (teenage health education
by American Dental Association (ADA), Bureau teaching assistants)
of Dental Health Education and its consultants in
response to a request from 1971 ADA’s house of
{SN Q.4}
delegates.
Programme philosophy and goals Philosophy and goals
(a) This is a comprehensive programme covering l Utilizes qualified dental personnel to train inter-
current dental concepts. ested high school students to teach preventive
(b) The objective of this programme is to develop dentistry to elementary students.
the knowledge, skills, and attitudes needed for l The programme goal is to give young children the
the prevention of dental disease. knowledge and skills to start them on the way to
(c) The priority of the programme is to de- lifetime of preventive dentistry practice.
velop effective plaque control knowledge
and skills.
Implementation
l Suggested guidelines and a THETA teacher’s

manual are forwarded to interested parties.


Cost of materials
{SN Q.9}
l Training manual costs $0.50.
D. Tattletooth programme development l Plaque control kit for a class of 35 costs $8.00.
The Tattletooth programme was developed as a co-
Q.6. What is potable water? Explain the steps involved
operative effort between
in the purification of water on a large scale.
l Texas Dental Health Professional Organization

l Texas department of health Ans.


l Texas education agency
[Same as LE Q.1]
594 Quick Review Series for BDS 4th Year, Vol 1

Q.7. Describe in detail the large-scale purification of water. Q.16. Define health education and describe the role of
mass media in health education programmes.
Ans.
Ans.
[Same as LE Q.1]
[Same as LE Q.3]
Q.8. Explain purification of water by rapid sand filtration.
Q.17. Describe the role of mass media in dental aware-
Ans. ness programme.
[Same as LE Q.1] Ans.
Q.9. Describe briefly the water purification method on [Same as LE Q.3]
large-scale in a city.
Q.18. Describe the objectives of school dental health
Ans. programme and add a note on the concept of incremen-
[Same as LE Q.1] tal dental care.

Q.10. Define health education. Add a note on the prin- Ans.


ciples of health education. [Same as LE Q.5]
Ans. Q.19. What are the objectives of school dental service?
[Same as LE Q.2] What are the advantages of a school-based programme?
Which are different school dental health programmes?
Q.11. What is health education? Describe in detail the What is incremental dental care?
principles of health education.
Ans.
Ans.
[Same as LE Q.5]
[Same as LE Q.2]
Q.20. Write in detail the steps taken in planning a dental
Q.12. Give the WHO definition of health education. health education programme for controlling diseases in
Enumerate the different principles of health education. a school having 3000 students.
Explain different methods for imparting group dental
Ans.
health education.
[Same as LE Q.5]
Ans.
[Same as LE Q.2]
SHORT ESSAYS:
Q.13. Define health education. Enumerate the ap- Q.1. Define health. Enumerate the various environmen-
proaches of health education. Discuss in detail the prin- tal factors necessary for the maintenance of general
ciples of health education. health of an individual.
Ans. Ans.
[Same as LE Q.2] WHO defined health as ‘a state of complete physical, men-
Q.14. Define heath education. Discuss the principles of tal and social well-being and not merely the absence of
health education. What are the barriers of health educa- disease or infirmity’.
tion? How to overcome those barriers? Environmental factors
Environment is defined as man’s external surroundings.
Ans. It is divided into three components:
[Same as LE Q.2] (a) Physical environment
l This is applied to nonliving things and physi-
Q.15. Define health education. Classify the educational cal factors with which man is in constant in-
aids used in health education and add a note on the role teraction (e.g. air, water, soil, housing, cli-
of mass media in health education. Explain the princi- mate, heat, light, noise, debris and radiation).
ples of health education in detail. (b) Biological environment
l The biological environment is the universe of
Ans.
living things, which surround man, including
[Same as LE Q.3] man himself.
Section | I  Topic-Wise Solved Questions of Previous Years 595

(c) Psychosocial environment Comprehensive health service criteria


l It is difficult to define psychosocial environ- a. To make provision of adequate preventive, curative
ment due to the varied social, economic and and promotive health services.
cultural contexts. b. To be as close to the beneficiaries as possible.
l It includes a complex of psychosocial factors, c. To have the widest cooperation between the people,
which are defined as ‘those factors affecting the service and the profession.
personal health, health care and community d. To be available for all, irrespective of their ability to
well-being that stem from the psychosocial pay for it.
make-up of an individual and the structure e. To look after more specifically the vulnerable and
and functions of social groups’. weaker sections of the community.
l A stable and harmonious equilibrium be- f. To create and maintain a healthy environment in both
tween man and his environment is necessary home and working places. Such a care needs to be
to reduce man’s vulnerability to diseases and complete, competent and continuous, and coordinate
to permit him to lead a more productive and compassion for the community.
satisfactory life. Advantages
l Less money spent on permanent teeth.
Q.2. Define health. Describe ‘Health for All by 2000 AD’.
l Interruptions are less.

Ans. l Psychologically better.

Disadvantages
Health is defined as ‘a state of complete physical, mental and
l Disease has already occurred.
social well-being and not merely the absence of disease’.
l Initial cost of controlling the disease is more.
Health for All by 2000
l More dental man hours required.
l Most of the people in the developed countries enjoy
Incremental dental care
all the determinants of good health: adequate in-
l It is defined as a periodic care so spaced that incre-
come, nutrition and education, sanitation, safe drink-
ments of dental diseases are treated at the earliest
ing water and comprehensive health care. In contrast,
time, consistent with proper diagnosis and operating
only 10%–20% of the population in developing
efficiency in such a way that there is no accumula-
countries enjoy ready access to health services of any
tion of dental needs.
kind. Death claims 60–250 of every 1000 live births
l In schoolchildren, incremental dental care repre-
within the first year of life, and the life expectancy is
sents ideal pattern for the care and appreciable in-
30% lower than that in the developed countries.
cidence of new dental disease is to be expected
l It is concluded that the neglected 80% of world’s
each year.
population too have an equal claim to health care,
Disadvantages
protection from the killer diseases of childhood and
l Time consuming, e.g. multiple fillings.
to primary health care for mothers and children.
l Psychological: Young people develop their own hab-
l Public health in its present form is a combination of
its, so habits learned in childhood would not neces-
scientific disciplines (e.g. epidemiology, biostatis-
sarily be carried to adulthood.
tics, laboratory sciences, social sciences and de-
l There is exhaustion of financial resources as it is a
mography) and skills and strategies (e.g. epidemio-
long and periodic programme.
logic investigations, planning and management, and
l Interruptions in dental health care programme may
interventions and surveillance evaluation) that are
occur due to migration.
directed to the maintenance and improvement of
health of people. Q.4. What is hydrologic cycle? Enumerate the tech-
niques for purification of water. Write in detail about
Q.3. Write in detail about a comprehensive preventive
small-scale purification of water.
dentistry programme for a primary school in rural
area. Ans.
Ans. Purification of water can be done under two techniques:
(A) Purification of water on large scale
Comprehensive dental care
(B) Purification of water on small scale
It is defined as the meeting of accumulated dental
needs at the time a population group is taken into the (A) Purification of water on large scale
programmes, and the detection and correction of new l Storage

increments of dental disease on a semi-annual or other l Filtration

periodic basis. l Chlorination


596 Quick Review Series for BDS 4th Year, Vol 1

(B) Purification of water on small scale Q.5. What are the different aids used towards dental
Household purification of water health education for the community?
(i) Boiling
Ans.
(ii) Chemical disinfection
(a) Bleaching powder Educational aids can be basically classified into the follow-
(b) Chlorine solution ing three categories:
(c) Chlorine tablets (A) Audio aids
(d) High test hypochlorite (HTH) (B) Visual aids
(e) Potassium permanganate (C) A combination of audio and visual aids
(f) Iodine
(iii) Filtration A. Audio aids
Purification of water on small scale is described below. The audio aids are based on the principles of sound,
i. Boiling of water electricity and magnetism. The modern world makes
l For routine drinking purposes (to disinfect), use of different types of audio aids. The most com-
candle-filtered water need to be boiled for 1 or monly used audio aids in health education are as
2 min. follows:
l This will destroy the viruses of diseases like l Megaphones

hepatitis A and poliomyelitis. If the purpose is l Public addressing systems or microphones

to get rid of spores (sterilization), then the l Gramophone records and discs

unfiltered water should be put on rolling boil l Tape recorders

for 20 min. This applies to sterilization of cut- l Radios

ting and piercing instruments. l Sound amplifiers

ii. Chemical disinfectants B. Visual aids


(a) Bleaching powder: 2.5 g of good quality bleach- l Visual aids are based on the principles of projection.

ing powder is enough for 1000 L of water. This Visualization helps individuals to understand better
can be applicable in routine situations as well. than any other media.
(b) Chlorine solution: For wells, bleaching pow- l The visual aids can be classified as follows:

der should not be directly added but should be (a) Projected aids
dissolved in a sample of water and the super- (b) Nonprojected aids
natant chlorine solution should be mixed in (c) Other aids
well water by lowering it in a drawing bucket. (a) Projected aids
(c) Chlorine tablets: Ideal for use by travellers These are visual aids which need projection from
and in emergencies. a source on to a screen. The various projected
(d) Iodine: This is biologically active and can aids include the following:
interfere with thyroid function, hence should l Films or cinemas

be used carefully. l Film strips

(e) Potassium permanganate: Not used because l Slides

of lack of antiviral action. l Overhead projectors

iii. Filtration – candle (ceramic) filters, which are l Epidiascopes

only bacterial filters l Transparencies

Filtration is described as follows: l Bioscopes

(a) Slow sand or biological filters l Video cassettes

Elements of slow sand filters: l Silent films

l Supernatant water (b) Nonprojected aids


l A bed of graded sand These are visual aids which do not require any
l An underdrainage system projection. The commonly used aids without pro-
l A system of filter control valves jection are (i) blackboards, (ii) pictures, (iii) car-
(b) Rapid sand or mechanical filters toons, (iv) photographs, (v) posters, (vi) flash-
Rapid sand filters are of two types: gravity cards (vii) charts, (viii) graphs, (ix) flannel boards
type, and pressure type. and (x) leaflets.
Steps of rapid sand filters: (c) Other aids
l Coagulation These include the traditional media, which
l Rapid mixing makes use of light and sound. The different
l Flocculation methods are as follows:
l Sedimentation l Folk dances

l Filtration l Folk songs


Section | I  Topic-Wise Solved Questions of Previous Years 597

l Puppet shows l Meat, fish, eggs, vegetables and fruits contain smaller
l Dramas amount.
C. Combination of audio and visual aids Functions
These are the modern media available. The advantage l It is essential for utilization of carbohydrates.

of this type of media is that sound and sight can be l It is involved in direct oxidative pathway for glucose.

combined together to create a better presentation. This Deficiency


includes the following: l Deficiency of vitamin B1 causes beriberi and

(a) Television Wernicke encephalopathy.


(b) Tape and slide combinations l Dry beriberi is characterized by nerve involvement.

(c) Videocassette player and recorder l Wet beriberi is characterized by heart involvement.

(d) Motion pictures or cinemas l Infantile beriberi is seen in infants between 2 and

4 months of life.
Q.6. Principles of chlorination. l Wernicke encephalopathy is characterized by

Ans. ophthalmoplegia, polyneuritis, ataxia and mental


deterioration.
l Chlorination is one of the greatest advances in water Daily requirement
purification. It is a supplement, not a substitute to sand l The average daily requirement is 1.5 mg.

filtration. II Riboflavin or vitamin B2


l Chlorine kills pathogenic bacteria, but has no effect on Source
spores and certain viruses except in high doses. It oxi- l Milk, eggs, liver, kidney and green leafy vegetables.

dizes iron, manganese and hydrogen sulphide; it de- l Meat and fish contain small amounts of vitamin B12.

stroys some taste and odour-producing constituents; Functions


and it controls algae and slime organisms, and aids l It has a fundamental role in cellular oxidation.

coagulation. l It is a cofactor in a number of enzymes involved

Action of chlorine in energy metabolism.


l When chlorine is added to water, it reacts to form Deficiency
hydrochloric and hydrochlorous acids. Hydrochloric Deficiency of vitamin B2 causes the following:
acid is neutralized by the alkalinity of water. l Angular stomatitis

l Hypochlorous acid ionizes and forms hydrogen ions l Cheilosis

and hypochlorite ions. l Glossitis

l The disinfecting action of chlorine is mainly due to Daily requirement


hypochlorous acid, and to small extent due to hypo- l The average daily requirement of vitamin B2 is

chlorite ions. 1.5 mg.


l Hypochlorous acid is more effective (70–80 times) III Niacin
than hypochlorite ions. Source
Methods of chlorination l Liver, kidney, meat, poultry, fish, legumes and

For disinfecting large bodies of water, chlorine is applied groundnut.


as one of the following: Functions
l Chlorine gas l Essential for the metabolism of carbohydrate, fat

l Chloramine and protein.


l Perchloron l Essential for normal functioning of skin, intesti-

nal system and nervous system.


Q.7. Health care waste and its significance. Deficiency
Deficiency of niacin causes the following:
Ans.
l Pellagra.

[Ref LE Q.4] l It is characterized by three Ds – diarrhoea, derma-

titis and dementia.


Q.8. Vitamin B complex. l Glossitis.

l Stomatitis.
Ans.
Daily requirement
I Thiamine or vitamin B1 l The average daily requirement is 18 mg.
l Vitamin B1 is a water-soluble vitamin. IV Vitamin B6
l Source – whole grain cereals, wheat, gram, yeast, This exists in three forms: pyridoxine, pyridoxal and
pulses, oilseeds and nuts. pyridoxamine.
598 Quick Review Series for BDS 4th Year, Vol 1

Source submerged portion of the iceberg) far exceeds the


l Milk, liver, meat, egg yolk, fish, whole grain cereals. known morbidity.
Functions l The hidden part of the iceberg thus constitutes an im-

l It plays an important role in the metabolism of portant, undiagnosed reservoir of infection or disease in
amino acids, fats and carbohydrates. the community, and its detection and control is a chal-
Deficiency lenge to modern techniques in preventive medicine.
l Deficiency of vitamin B 6 causes peripheral
Q.10. Approaches to health education.
neuritis.
Daily requirement Ans.
l Adults – 2 mg/day
Approaches to health education
l Pregnancy and lactation – 2.5 mg/day
( a) Individual approach
V Folate
(b) Group approach
Source
l Chalk and talk
l Liver, meat, dairy products, eggs, milk, fruits and
l Group discussion
cereals.
l Panel discussion
Functions
l Symposium
l Plays a role in the synthesis of the nucleic acids.
l Demonstration
l It is also needed for the normal development of
l Workshop
blood cells in the marrow.
l Role-playing
Deficiency
(c) Mass approach
Deficiency of folate causes the following:
l Television
l Megaloblastic anaemia
l Radio
l Glossitis
l Health magazine
l Cheolosis
l Newspapers
l Gastrointestinal disturbances
l Printed materials
VI Vitamin B12
Panel discussion
Source
l In a panel discussion, persons who are qualified to
l Liver, kidney, meat, fish, egg, milk and cheese.
talk about the topic sit and discuss a given problem,
Functions
or the topic, in front of a large group or audience.
l It has a separate biochemical role in the synthesis
l The panel comprises a chairman and about 4–8
of fatty acids in myelin.
speakers.
l It cooperates with folate in the synthesis of DNA.
l The chairman opens the meeting, welcomes the
Deficiency
group and introduces the panel speakers.
Deficiency of vitamin B12 causes the following:
l The chairman introduces the topic briefly and invites
l Megaloblastic anaemia
the panel speakers to present their points of view.
l Demyelinating neurological lesions in the spi-
l The chairman has to keep the discussion going
nal cord
and develop the train of thoughts.
l Infertility
l After the main aspects of the subject are explored

Q.9. Iceberg of disease. by the panel speakers, the audience is invited to


take part.
Ans. l Panel discussion can be an extremely effective

method of education, provided it is properly


l According to this concept, disease in a community may
planned and guided.
be compared with an iceberg.
l The tip of the iceberg represents what the physician sees Q.11. Elements of primary health care.
in the community, i.e. clinical cases.
Ans.
l The vast submerged portion of the iceberg represents

the hidden mass of disease, i.e. latent, inapparent, pres- Elements of primary health care. The Alma-Ata declara-
ymptomatic and undiagnosed cases and carriers in the tion has outlined eight essential components of primary
community. health care.
l The ‘water line’ represents the demarcation between i. Education concerning prevailing health problems
apparent and inapparent disease. In some diseases and the methods of preventing and controlling them
(e.g. hypertension, diabetes, anaemia, malnutrition ii. Promotion of food supply and proper nutrition
and mental illness) the unknown morbidity (i.e. the iii. An adequate supply of safe water and basic sanitation
Section | I  Topic-Wise Solved Questions of Previous Years 599

iv. Maternal and child health care, including family iii. Hand care
planning l Hands, through direct contact with a patient’s saliva,

v. Immunization against major infectious diseases become contaminated and therefore are sources for
vi. Prevention and control of locally endemic diseases cross-infection.
vii. Appropriate treatment of common diseases and injuries l Cross-infection can be at least partially controlled by

viii. Provision of essential drugs making a conscious effort to keep away the gloved
hands from touching objects other than the instru-
Q.12. Personal barrier technique for infection control. ments and disinfected parts of equipments prepared
for immediate patient.
Ans.
iv. Gloves
Personal barrier for infection control includes the l Wearing gloves is a part of the total plan for control

following: of cross-contamination and protection of clinician.


i. Clinical attire l Unseen blood from a patient can be impacted,

l The wearing apparels of clinicians and their assis- retained under nails of five or more fingers after ex-
tants are vulnerable to contamination from splash, posure during an appointment.
splatter, aerosols and patient contact. l Hepatitis virus is known to be resistant to drying, might

l The gown or uniform should be designed and cared for be retained under a fingernail with blood or saliva.
in a manner that will minimize cross-contamination. l Even scrubbing cannot remove all microorganisms from

Gown, uniform or scrub suit nails. Tiny cuts and abrasions cannot be seen or felt.
l Gowns, uniforms or scrub suits are expected to be l Protective gloves are needed at all times.

clean and maintained as free as possible from


Q.13. Waterborne infections.
contamination.
l Wearing clinic coats over street clothes cannot be Ans.
recommended because of the exposure of street
Waterborne diseases are caused by the presence of infective
clothes to infectious materials.
agent or an aquatic host in water.
Solid, closed front
By the presence of infective agent:
l The garment should be closed at the neck.
(a) Viral: viral hepatitis, poliomyelitis, hepatitis E
l The fabric should be able to be washed commer-
(b) Bacterial: cholera, typhoid, bacillary dysentery
cially and withstand washing with bleach.
(c) Protozoal: amoebiasis, giardiasis
Hair and head covering
(d) Helminthic: round worm, whipworm, threadworm
l Hair must be worn off the shoulders and back.
(e) Leptospiral: Weil disease
When longer, it must be held within a head cover.
l Since hair is exposed to much contamination, an By the presence of an aquatic host:
appropriate head cover is advised when using (a) Cyclops: guinea worm, fish tape worm
hand pieces, ultrasonic or abrasive instruments. (b) Snail: schistosomiasis
Use of face mask
Q.14. Define preventive dentistry. Describe incremental
l Face masks should be worn to prevent airborne
dental care and comprehensive dental care.
infections, when either the patient or the operator
is known to have acute respiratory infection. Ans.
l Tie on the mask before a scrub or hand wash.
[Same as SE Q.3]
l Use fresh mask for each patient.

l When mask becomes wet, it should be changed, Q.15. Aids to health education.
because a wet mask is not longer an effective barrier.
Ans.
l Keep the mask on after completing a procedure,

while still in the presence of aerosols. [Same as SE Q.5]


ii. Eye protection
Q.16. Disposal of hospital waste.
l This is necessary during dental and dental hygiene

appointments for dental team members and patients. Ans.


l It is necessary to prevent physical injuries and infec-
[Same as SE Q.7]
tions of the eyes.
l Glasses should be worn at all times. Q.17. Disposal of dental clinic wastes.
l For dental personnel who do not require corrective
Ans.
lens for vision, protective glasses with clear lens
should become a routine part of clinical dress. [Same as SE Q.7]
600 Quick Review Series for BDS 4th Year, Vol 1

Q.18. Incinerator. l It is a process of nature whereby organic matter breaks


down under bacterial action, resulting in formation of
Ans.
relatively stable humus-like material, called the com-
[Same as SE Q.7] post, which has considerable manurial value for soil.
Q.19. Composting. There are two methods of commercial importance:
i. Bangalore method (anaerobic method)
Ans.
l It is also called the hot fermentation process.

[Same as SE Q.7] l It has been recommended as a satisfactory method of

disposal of town waste and night soil.


Q.20. Disposal of waste.
ii. Mechanical composting (aerobic method)
Ans. l This is another method of composting.

l In this, compost is literally manufactured on a large


[Same as SE Q.7]
scale by processing raw materials and turning out
Q.21. Iceberg phenomenon. into a finished product.
l The entire process of composting is completed in
Ans.
4–6 weeks.
[Same as SE Q.9]
Q.4. THETA.
Q.22. Panel discussion.
Ans.
Ans.
[Ref LE Q.5]
[Same as SE Q.10]
Q.5. Incremental dental care.
Ans.
SHORT NOTES:
l Incremental dental care is defined as a periodic care so
Q.1. Steps in large-scale purification of water.
spaced that increments of dental diseases are treated at
Ans. the earliest time, consistent with proper diagnosis and
operating efficiency in such a way that there is no
[Ref LE Q.1]
accumulation of dental needs.
Q.2. Dimensions of health. l In schoolchildren, incremental dental care represents

ideal pattern for the care and appreciable incidence of


Ans.
new dental disease to be expected each year.
The dimensions of health are as follows: l In private practice, 6 months is the commonest, al-

i. Physical dimension though not the only interval between two visits.
ii. Mental dimension l In public health programmes, 1-year intervals are usu-

iii. Social dimension ally employed.


iv. Spiritual dimension l Aim is basically to have a rational approach towards the

v. Emotional dimension dental problems on annual basis, and a plan for lifelong
A few other dimensions are as under: dental care.
l Vocational dimension
Q.6. Break point chlorination.
l Philosophical dimension

l Cultural dimension Ans.


l Socioeconomic dimension
l Break point chlorination is defined as a point at which
l Environmental dimension
chlorine starts appearing in water sample after meeting
l Educational dimension
the chlorine demand of sample.
l Nutritional dimension
l When chlorine dose in water is increased, a reduction in
l Preventive dimension
residual chlorine occurs.
Q.3. Composting. l The end products do not represent any residual chlorine.

l This fall in residual chlorine continues with further in-


Ans.
crease in chlorine dose, until after stage residual chlo-
l Composting is a method where excrement or sludge is rine begins to increase in proportion.
combined with solid waste to produce manure quality l The point at which it appears is called break point chlo-

material. rination or dosage.


Section | I  Topic-Wise Solved Questions of Previous Years 601

Q.7. Panel discussion. ‘A process with intellectual, psychological and social dimen-
sions relating to activity that increases the abilities of people
Ans.
to make informed decisions affecting their personal family
l Panel discussion is the type of discussion where persons and community well-being. This process, based on scientific
who are qualified to talk about the topic sit and discuss principles, facilitates learning and behavioural change in
a given problem, or the topic, in front of a large group both health and consumers, including children and youth’.
or audience.
Q.12. Manure pits.
l This panel comprises a chairman and about 4–8 speakers.

l The chairman opens the meeting, welcomes the group Ans.


and introduces the panel speakers. l In this method, the garbage, cattle dung, straw and
l He introduces the topic briefly and invites the panel leaves should be dumped into manure pits and covered
speakers to present their points of view. with earth after each day’s dumping.
l The chairman has to keep the discussion going and l Within 5–6 months, the refuse is converted into manure,
develop the train of thoughts. which can be returned to the field.
l After the main aspects of the subject are explored by the l This method of refuse disposal is effective and rela-
panel speakers, the audience is invited to take part. tively simple in rural communities.
Q.8. Askov dental health education. Q.13. Back washing.
Ans. Ans.
l Askov dental health education is a classic example of a l In rapid sand filtration, washing is accomplished by
school dental health programme. reversing the flow of water through sand bed, which is
l Dental findings are available through a 10-year period, called back washing.
which includes 28% reduction in dental caries in decidu- l Back washing dislodges the impurities and cleans the

ous teeth of children aged 3–5 years; 34% reduction in sand bed.
caries in permanent teeth of children aged 6–12 years; and l Compressed air is used as part of the back-washing

14% reduction in caries in children aged 13–14 years. process.


Q.9. Tattletooth programme. Advantages
l Rapid sand filter can deal with raw water directly.
Ans.
l The filter bed occupies less space.

[Ref LE Q.5] l Filtration is rapid. It is 40–50 times that of slow sand

filters.
Q.10. Group discussion.
l Washing of the filter is easy.

Ans.
Q.14. Rapid sand filtration.
l Group discussion is a type of discussion where indi-
Ans.
viduals are allowed to learn by freely exchanging their
knowledge, ideas and opinion. l In rapid sand filtration, washing is accomplished by
l This is considered valuable where long-term compli- reversing the flow of water through the sand bed, which
ance is involved. is called back washing.
l The group should comprise not less than 6 and not more l Back washing dislodges the impurities and cleans the

than 12 members. sand bed.


l There should be a group leader who initiates the subject, l Compressed air is used as part of the back-washing
helps the discussion in proper manner, prevents side process.
conversations, encourages everyone to participate, and l The following steps are involved in the purification of

sums up the discussion at the end. water by rapid sand filters:


i. Coagulation
Q.11. Dental health education.
ii. Rapid mixing
Ans. iii. Flocculation
iv. Sedimentation
l Dental health education is defined as the provision of
v. Filtration
health information to people in such a way that they
apply it to everyday living. Q.15. Vitamin D.
l In order to standardize a comprehensive definition of
Ans.
health education, including dental health education,
several national health organizations drafted the follow- The important forms of vitamin D in man are calciferol
ing definition. (vitamin D2) and cholecalciferol (vitamin D3).
602 Quick Review Series for BDS 4th Year, Vol 1

Source l The top layer should be at least 9-inch-thick layer of


i. Sunlight: Vitamin D is synthesized in the body by the refuse.
action of ultraviolet rays of sunlight on 7-dehydrocho- l Then the heap is covered with excavated earth. Within

lesterol, which is stored in abundance in the skin. 7 days, as a result of bacterial action, considerable heat is
ii. Foods: Liver, egg yolk, butter, cheese, milk, fish fat. generated in the compost mass. This heat persists for 2 or
3 weeks, and serves to decompose the refuse and night soil
Functions
and to destroy all pathogenic and parasitic organisms.
l It helps in the absorption of calcium.
l At the end of 4–6 months, decomposition is completed
l Used in maintenance of calcium homeostasis and skel-
and the resulting manure is well decomposed, odour-
etal integrity.
less, innocuous material of high manurial value ready
Deficiency for application to the land.
l Rickets – observed in young children Q.19. Barriers of health education.
l Osteomalacia – observed in adults
Ans.
Daily requirement
Adults – 2.5 mcg (100 ID) The barriers of health education are as follows:
l Physiological – difficulties in hearing and expression
Infants and children – 5.0 mcg (200 ID)
l Psychological – emotional disturbances and neurosis
Pregnancy and lactation – 10.0 mcg (400 ID)
l Environmental – noise, invisibility and congestion
Q.16. Soil, seed and sower. l Cultural – levels of knowledge and understanding, cus-

Ans. toms, beliefs and attitudes

l The people are the soil, the health facts are the seeds and Q.20. Safe water.
the transmitting media is the sower. Ans.
l The seeds or health facts must be truthful and based on
l Water intended for human consumption should be not
scientific knowledge.
only ‘safe’ but also ‘wholesome’.
l The transmitting media should be attractive, palatable
l Safe water is one that cannot harm the consumer, even
and acceptable.
when ingested over prolonged periods.
Q.17. School dental health programmes.
Safe water, potable water or wholesome water is defined as
Ans. follows:
l Free from pathogenic agents
Following are the school dental health programmes:
l Free from harmful chemical substances
l Askov Dental Health Education
l Pleasant to taste
l School Dental Health Nurse programme
l Usable for domestic purposes
l Learning About Your Oral Dental Health

l Tattle Tooth programme Q.21. Mass media.


l Tooth Keeper programme
Ans.
l THETA programme (teenage health education teaching

assistants). [Ref LE Q.3]


Q.18. Bangalore method of composting. Q.22. Audiovisual aids.
Ans. Ans.
l Bangalore method of composting is also called hot l Audiovisual aids help to simplify unfamiliar concepts
fermentation process. and bring about understanding where words fail.
l It has been recommended as a satisfactory method of They can be classified into following categories:
disposal of town wastes and night soil. (a) Auditory aids: radio, tape recorder, microphones,
l Trenches, 3-feet deep, 5–8-feet broad and 15–30-feet amplifiers, and earphones
long, are dug. (b) Visual aids:
l First, about 6-inch thick layer of refuse is spread at the i. Not requiring projection: chalkboard, leaflets,
bottom of the trench. Over this, 2-inch thick night soil posters, charts, flannel graphs, exhibits, models,
is added. specimens, etc.
l Then alternate layers of refuse and night soil are added ii. Requiring projection: slides, filmstrips
in the proportion of 6 inches and 2 inches, respectively, (c) Combined audiovisual aids: television, sound films
till the heap rises to 1 foot above the ground level. and slides-tape combination
Section | I  Topic-Wise Solved Questions of Previous Years 603

Q.23. Vital layer. iii. Hygiene


iv. Family health care
Ans.
v. Control of communicable and noncommunicable
l A slimy growth that covers the surface of sand bed is diseases
known as ‘schmutzdecke’, vital layer, zoogleal layer or vi. Mental health
biological layer. vii. Prevention of accidents
l This layer is slimy and gelatinous and consists of viii. Use of health services
thread-like algae and plankton, diatoms and bacteria.
Q.25. Workshop.
l The formation of vital layer is known as ‘ripening’ of filter.

l It may take several days for the vital layer to form. Ans.
l It extends 2–3 cm into the top portion of the sand bed.
l Workshop consists of series of meetings, usually four or
It is the ‘heart’ of the slow sand filter.
more, with emphasis on individual work, within the group,
l It removes organic matter, holds back bacteria and oxi-
with the help of consultants and resource personnel.
dizes ammonical nitrogen into nitrates and helps in
l The total workshop may be divided into small groups,
yielding bacteria-free water.
and each group will choose a chairman and a recorder.
Q.24. Contents of health education. l The individuals work, solve a part of the problem

through their personal effort with the help of consul-


Ans.
tants, contribute to group work and group discussion
The contents of health education can be categorized into the and leave the workshop with a plan of action on the
following eight main divisions: problem.
i. Human biology l The workshop provides each participant opportunity to

ii. Nutrition improve his effectiveness as a professional worker.

Topic 3
Epidemiology
COMMONLY ASKED QUESTIONS

LONG ESSAYS:
1. Define epidemiology. Mention the different epidemiological studies. Describe in detail descriptive epidemiology.
2. Discuss the aetiology and epidemiology of periodontal diseases. Describe the preventive strategies of periodon-
tal diseases in Indian population.
3. What is epidemiological triad? Discuss the role of components of epidemiological triad in the causation of dental caries.
4. Define epidemiology. What are different epidemiological study designs? Discuss different types of bias.
5. Define epidemiology. Describe its aims, objectives and various terminologies used in the field of epidemiology.
6. Define epidemiology. Describe the role of host and environmental factors in the epidemiology of oral cancer.
7. Discuss the scientific method of conducting dental epidemiological investigation. Give importance of each step.
8. Define prevention in relation to dental and oral diagnosis and different levels of prevention. Describe the pro-
cedure to control and prevent periodontal disease.
9. Explain in detail about descriptive epidemiology. [Same as LE Q.1]
10. Describe steps of a descriptive study. [Same as LE Q.1]
11. Define epidemiology and dental epidemiology. Discuss significance of health trends and describe the role of
descriptive epidemiology in epidemiological investigation. [Same as LE Q.1]
12. What are aims of epidemiology? Explain case–control studies in detail. [Same as LE Q.1]
13. Describe steps in conducting a randomized controlled trial. [Same as LE Q.1]
14. Define epidemiology. Describe in detail the various epidemiological studies. [Same as LE Q.1]
15. Define epidemiology. Describe in brief the types of epidemiological studies. [Same as LE Q.1]
16. Define epidemiology. What are the different types of epidemiological studies? Describe prospective and retro-
spective studies. Mention the merits and demerits of each. [Same as LE Q.1]
17. Define epidemiology. Write in detail the epidemiology of periodontal diseases. [Same as LE Q.2]
18. Describe the role of agent and host factors in the epidemiology of periodontal diseases. [Same as LE Q.2]
604 Quick Review Series for BDS 4th Year, Vol 1

1 9. Define epidemiology. Discuss environmental factors affecting dental caries prevalence. [Same as LE Q.3]
20. Define epidemiology. Discuss the role of environmental factors in the epidemiology of dental caries. [Same as LE Q.3]
21. Define prevention. What are the objectives of different levels of prevention? Discuss the measures used for
primary prevention of dental caries at individual and community levels. [Same as LE Q.3]
22. Write preventive dentistry. What are the various levels of prevention? Write in detail about the primary and
secondary levels of prevention of oral cancer. [Same as LE Q.6]
23. Define a tumour. Write in detail the epidemiological triad of oral cancer. [Same as LE Q.6]
24. Define epidemiology. Describe in brief the epidemiology of oral cancer. [Same as LE Q.6]
25. Define epidemiology. Describe the epidemiological factors of oral cancer. [Same as LE Q.6]
26. Mention different oral habits contributing to the development of such lesions in India, and describe any two
conditions in detail. [Same as LE Q.6]

SHORT ESSAYS:
1 . Epidemiological triad. [Ref LE Q.3]
2. Advantages and disadvantages of case–control study.
3. Cohort studies.
4. Comparison between case–control and cohort studies.
5. Tools of measurements in epidemiology. [Ref LE Q.5]
6. Vipeholm study. [Ref LE Q.3]
7. Turku sugar studies. [Ref LE Q.3]
8. Elements of a cohort study. [Same as SE Q.3]
9. Analysis in case–control and cohort studies. [Same as SE Q.3]

SHORT NOTES:
1. Write a note on screening.
2. Enumerate the various steps of cohort study.
3. Blinding.
4. Define and discuss pandemic.
5. Rate, ratio and proportion. [Ref LE Q.5]
6. Principles of epidemiology. [Ref LE Q.5]
7. Odds ratio.
8. Cross-infection.
9. Smokeless forms of tobacco. [Ref LE Q.6]
10. Zoonosis.
11. Long-term/secular trends. [Ref LE Q.1]
12. Bias. [Ref LE Q.4]
13. Descriptive epidemiology. [Ref LE Q.1]
14. Prospective studies.
15. Matching.
16. Case–control study.
17. True positive.
18. Cross-sectional studies.
19. Analytical epidemiology.
20. Define epidemiology. [Ref LE Q.2]
21. Cohort study.
22. Randomized trial.
23. Mention the types of descriptive studies. [Same as SN Q.13]
24. Methods of collection of data.
25. Tobacco-related cancer.
26. Morbidity and mortality.
27. Blind study. [Same as SN Q.3]
28. Pandemic. [Same as SN Q.4]
29. Ratio. [Same as SN Q.5]
30. Selection bias. [Same as SN Q.12]
31. Berkesonian bias. [Same as SN Q.12]
Section | I  Topic-Wise Solved Questions of Previous Years 605

SOLVED ANSWERS
LONG ESSAYS:
Q.1. Define epidemiology. Mention different epidemiologi- l The population selected for the study has to be de-
cal studies. Describe in detail descriptive epidemiology. fined in terms of the total number and the composi-
tion of individuals within the population in terms of
Ans.
characteristics such as age, sex, occupation, culture,
Epidemiology is defined as the study of the distribution and socioeconomic characters.
determinants of health-related states and events in popula- l The ‘defined population’ can either be the total
tions, and the application of this study to control health population in a geographic area or a representative
problems (John Last, 1988). sample taken from that population.
Parkin (1873) defined epidemiology as ‘the branch of l It can also be special groups, such as age and sex
medical science which deals with the treatment of epidemics’. groups, schoolchildren, factory workers and hospi-
MacMahon (1960) defined epidemiology as ‘the study of tal patients. The population thus defined should be
the distribution and determinants of disease frequency in man’. large enough to make characteristics like age and
M. Last (1988) defined epidemiology as ‘the study of sex meaningful.
the distribution and determinants of health-related states or II Defining the disease under study
events in specified populations, and the application of this l This is the second step in descriptive epidemiologi-
study to the control of health problems’. cal study.
l The disease needs to be defined with an ‘operational
Different epidemiological studies are as follows:
definition’, which is a definition with which the disease
A. Descriptive epidemiology
or condition can be identified and measured in the de-
B. Analytical epidemiology
fined population with a degree of accuracy.
a. Case–control study
l This type of definition will be precise and valid for
b. Cohort study
epidemiologist and will help in identifying people
C. Experimental epidemiology
with the disease from those who do not have the
disease.
III Describing the disease under study
{SN Q.13} l This step is used to describe the occurrence and

A. Descriptive epidemiology distribution of a disease by the time of its occur-


l Descriptive studies are usually the first phase of rence, the place of occurrence and the persons who
an epidemiological investigation. These studies are affected with the disease.
are concerned with the observation of the distribu- A. Time distribution
tion of the disease or any health-related events in Time can be measured in terms of hours, days,
human populations and the identification of the weeks, months, years, etc. Three types of time fluc-
characteristics with which the disease or condi- tuations or time trends have been observed in the
tion under study seems to be associated. occurrence of disease, which are as follows:
Following are the various steps involved in a descriptive i. Short-term fluctuations
study: ii. Periodic fluctuations
I. Defining the population to be studied iii. Long-term fluctuations or secular trends
II. Defining the disease under study i. Short-term fluctuations
III. Describing the disease in terms of: Example of a short-term fluctuation is an ‘epi-
(a) Time demic’. An epidemic can be defined as ‘the occur-
(b) Place rence in a community or region of cases of an ill-
(c) Person ness or other health-related events clearly in excess
IV. Measurement of disease of normal expectancy’.
V. Comparing with known indices There are three major types of epidemics,
VI. Formulating an aetiological hypothesis which can be classified as follows:
A. Common source epidemics
(a) Single exposure or ‘point source’ epi-
I Defining the population to be studied demics
l This is the first phase of descriptive epidemiological (b) Continuous or multiple exposure epi-
study. demics
606 Quick Review Series for BDS 4th Year, Vol 1

B. Propagated epidemics l The propagated type of epidemic usually


(a) Person to person occurs in places where a large number of
(b) Arthropod vector susceptible individuals are aggregated or
(c) Animal reservoir where there is irregular supply of new sus-
C. Slow or modern epidemics ceptible persons.
C. Slow or modern epidemics
A. Common source epidemics
l The concept of an epidemic in the past was to
(a) Single exposure or ‘point source epidemics’
describe an acute outbreak of infectious dis-
l ‘Point source epidemics’ are the response
eases. More recently, importance has been
of a group of people to a source of infection
given to ‘excessive prevalence’ as the basic
or contamination to which they were ex-
implication in an epidemic.
posed to almost simultaneously. All resul-
l This characteristic is exemplified by many non-
tant cases develop within one incubation
infectious diseases as well as diseases known to
period of disease. An example is food poi-
be associated with microorganisms.
soning.
l The time distribution of epidemic cases can be
l The contaminated foodstuff results in an
represented graphically as an ‘epidemic curve’.
outbreak of food poisoning in those people
The epidemic curve is suggestive of a time re-
who have consumed it from the single
lationship with exposure to a suspected source
source.
and a cyclical or seasonal pattern, which indi-
l The common-source epidemics need not be
cates that polio and diphtheria have shown a
always due to exposure to an infectious
downward trend.
agent. It can also result from other causes
like the environmental pollution, for which ii. Periodic fluctuations
the ‘Bhopal gas tragedy’ in India is a classic Periodic fluctuations in the time distribution are of two
example of epidemic, continued over one types:
incubation period. There are chances of ei- (a) Seasonal variations/trends
ther a continuous or multiple exposures to a A prominent feature of infectious disease is oc-
common source or a propagated spread. currence in seasonal variations. For instance,
(b) Continuous or multiple exposure epidemics measles and varicella are usually found with
l These are the common source epidemics in their peak incidence during the early spring sea-
which the exposure from the same source son; similarly, upper respiratory tract infections
might be prolonged – can be continuous, are seen more during the winter season, and the
repeated or intermittent – need not be at the gastrointestinal tract infections have a seasonal
same place or at the same time (e.g. a well rise during summer months.
of contaminated water). In this case, the (b) Cyclic trends
resulting epidemic tends to be more ex- Certain diseases appear in cycles, which may be
tended or irregular. spread over short periods of time, like days, weeks,
l An epidemic may also be initiated from a months or years (e.g. incidence of measles ap-
common source, and then continues as a peared in cycles of major peaks every 2–3 years
propagated epidemic. before its vaccines were invented). Noninfectious
B. Propagated epidemics conditions also show periodic fluctuations, e.g.
(a) Person to person automobile accidents are known to be more fre-
(b) Arthropod vector quent on weekends, especially on Saturdays.
(c) Animal reservoir
l These are of infectious origin and usually

result due to transmission of infectious


{SN Q.11}
agent from person to person (e.g. epidem-
ics of hepatitis A and poliomyelitis). iii. Long-term or secular trends
l This type of epidemic usually exhibits a l Secular changes are the changes in disease frequency

gradual rise and then tails off over a much that occur gradually over long periods of time.
longer period of time. l Oral cancer is an example of a disease that has

l The process of transmission of infectious agent shown a consistent upward trend during the past 50
continues until the number of susceptible per- years in many of the developed countries whereas
sons is reduced or the susceptible persons are diseases like tuberculosis, typhoid fever, polio and
no longer exposed to infected persons. diphtheria have shown a downward trend.
Section | I  Topic-Wise Solved Questions of Previous Years 607

B. Place distribution Local distributions


l The study of the pattern of disease distribution l Distribution of disease can also vary within

among different populations reveals variations in cities or towns. Inner and outer city variations
disease pattern not only between countries but also in disease frequency are best studied with the
within the same country. aid of ‘spot maps’ or ‘shaded maps’. These
l The geographic pattern of disease provides clues maps show areas of high or low frequency, the
about the aetiology of disease. boundaries and patterns of disease distribution
The distribution of disease according to place can be at a glance.
classified as follows: l For example, if the map shows ‘clustering’ of

International variations cases, it may suggest a common source of


l The pattern of disease occurrence varies from infection or a common risk factor shared by all
country to country. It is a known fact that can- the cases.
cer prevails in every part of this world. How- Migrant studies
ever, there exist differences in the incidence of l If it is observed that a disease is associated

various types of cancer in different parts of the with a particular geographic area or region, it
world. Certain types of cancers are more com- is of interest to know what duration of resi-
monly seen in some parts of the world, while dency in the area is necessary for the acquisi-
some other parts have fewer incidences or no tion of susceptibility to the disease, and how
incidence of particular types at all. long the susceptibility is maintained by resi-
l Oral cancer has the highest incidence in coun- dents after leaving the area.
tries like India, Bangladesh, Sri Lanka and l Migration of human populations on a large

Pakistan whereas it is relatively rare in the scale from one country to another provides the
Western world. The World Health Organiza- opportunity to evaluate the role of possible
tion (WHO) has played a major role in the genetic and environmental factors in the occur-
improvement and dissemination of interna- rence of disease in a population.
tional statistics related to health. Migrant studies are usually of two types
National variations (a) Comparing the rate of occurrence of disease
l Variations in the occurrence of disease exist and the death rate for migrants with those of
within the same country or national boundar- their kin who have stayed at home. Such
ies. type of a migrant study permits the com-
l In India, a wide range of conditions, like parison of genetically similar groups living
endemic goitre, malaria and fluorosis, show under different environmental conditions or
national variation with some parts of the coun- exposures. If the rate of occurrence of a
try affected more and others less affected or disease and death rate in migrants are simi-
not affected at all. lar to that of the country of adoption over a
Rural–urban variations period of time, it can be attributed to the
l The health and disease are not equally distrib- environment.
uted in urban and rural populations. (b) Comparing the migrants with the local popu-
l Diseases like chronic bronchitis, lung cancer, lation of the host country provides valuable
cardiovascular diseases, drug addiction and information about the genetically different
psychological problems, and accidents, are groups living in a similar environment. If the
usually found to be more in urban areas rates of occurrence of a disease and death
whereas skin and zoonotic infections and soil- among the migrants are similar to their coun-
transmitted helminth infections are found more try of origin, it can be attributed to genetic
commonly in rural areas. factors.
l These variations in the disease pattern can be C. Person distribution
attributed to the factors like differences in so- l In descriptive epidemiology, the disease is further

cial classes, population density, levels of sani- described by defining the persons affected by the
tation, deficiencies in medical care, levels of disease in terms of their age, sex, occupation, ethnic-
education and other environmental factors that ity, origin, marital status, social class, habits, family
influence the occurrence of disease. history and other host factors.
608 Quick Review Series for BDS 4th Year, Vol 1

l These factors help in understanding the natural his- India, religion has been used as an index of ethnic
tory of disease. group. The examples for diseases exhibiting vari-
The host factors that influence the disease pattern in an ations in their frequency with regard to ethnicity
individual are as follows: include tuberculosis, coronary heart disease, can-
● Age cer and sickle cell anaemia.
Knowledge of age is important for two reasons: l Occupation

first, it may assist in understanding the factors Occupation is of greater usefulness in formulating
responsible for the development of disease, and causal hypothesis than age, gender or ethnic
second, age may produce indirect effects that group. Occupation can be used in a number of
must be taken account of. Certain diseases are ways as follows:
more prevalent in specific age groups. For exam- (a) As a measure of socioeconomic status
ple, measles usually occurs in childhood, cancer (b) For identification of risks associated with
in the middle age and atherosclerosis in old age. exposure to agents peculiar to certain oc-
Dental caries is generally believed to be a ‘disease cupations
of childhood’. Knowledge of age associations is (c) To identify groups whose general patterns
also useful for administrative purposes like help- of life vary because of the different de-
ing assign services to needy parts of population or mands made by their occupations
community. Certain diseases are known by the names
l Bimodality that are associated with their occupational
It is the occurrence of two separate peaks in the origin like Chimney Sweepers’ cancer,
age incidence of a disease. It indicates, first of all, Woolsorters’ disease and Farmer’s lung.
that the material is not homogenous – that the Occupation determines the special circum-
entity under examination might probably be di- stances such as the working environment
vided into two. Bimodality even suggests the ex- peculiar to particular occupations. This is
istence of causal differences other than that on well supported by the fact that certain oc-
which the classification of disease is based. An cupations influence or alter the habit pat-
example of a disease exhibiting ‘bimodality’ is terns of people engaged in that particular
dental caries, which is usually found in children occupation, e.g. patterns of sleep, and hab-
as pit and fissure caries, and in the older age group its like smoking and alcoholism. Sedentary
as root caries. lifestyles are also associated with certain
l Gender diseases like heart diseases. Occupational
Dental diseases have shown variation in their fre- habits, such as holding of nails in the
quency between males and females. In the case of mouth as done by carpenters or upholstery
oral cancer, studies have reported that females workers, thread biting among tailors and
have fewer incidences than males. the pressure of reed or other mouthpiece on
l Ethnic group/ethnicity the teeth of players of musical instruments,
This term is usually used to designate subgroups have been found to be conducive to peri-
of a population that, because of a common ances- odontal disease.
tral or cultural background, have a similar genetic l Socioeconomic status

or environmental milieu, or both, and is more The association of disease with socioeconomic
homogenous than that of the population at large. status varies according to the measure of socio-
The homogenicity may be expressed in a disease economic status used. Various epidemiological
pattern that differs from the usual illness pattern studies have shown that frequency and distribu-
of the general population. The term ethnic group tion of disease is different in the upper and lower
usually implies some degree of common ancestry social classes. Individuals belonging to upper
in the group, but it may not necessarily account social class exhibit longer life expectancy and bet-
for a particular group’s distinctive disease pattern. ter health and nutritional states than the individu-
The ethnic group can be identified in terms of als belonging to lower social class. Diseases such
race, religion, place of birth or combinations of as dental caries, coronary heart disease, hyperten-
the three. In some countries like the USA, the sion and diabetes mellitus are more prevalent
most frequently used criterion of ethnic group is among the individuals of upper class whereas
their race, whereas in some other countries like periodontal diseases and skin and zoonotic
Section | I  Topic-Wise Solved Questions of Previous Years 609

diseases are more prevalent among the individuals done for a longer duration by repeating peri-
of lower social class. odically.
l Marital status ● In a longitudinal study, the same individu-

Many diseases exhibit association with marital als are examined on repeated occasions
status. Married persons are found to have lower and the changes within the group are re-
mortality rates as compared to single, widowed or corded in terms of elapsed time between
divorced persons. The reasons attributed to this observations.
can be that married persons are healthier and are Uses of longitudinal studies
generally more secure and protected. However, l Used for studying the natural history of

marital status can be a risk factor for certain dis- disease and its outcome
eases such as carcinoma of the cervix, which is l Used for identifying the risk factors associ-

relatively rare among nuns. Another use of infor- ated with the disease
mation about marital status in epidemiological l Used for calculating the incidence rate of

studies is in investigating the possible influence of disease.


a common environment on the health of marital Longitudinal studies provide valuable infor-
partners. mation but are difficult to organize and are
l Behaviour more time-consuming.
The focus of research in this particular field has V Comparing with known indices
been on habits like cigarette smoking, usage of The essence of any epidemiological study is to
other forms of tobacco, alcoholism, drug abuse, make comparisons and to ask questions. By making
sedentary lifestyle and overeating. The diseases comparison between different populations and sub-
which can be attributed mainly due to the behav- groups of the same population, it is often possible
iour of individuals (with regard to the habits) are to reach a conclusion with regard to the disease
coronary heart disease, cancer, hypertension, obe- aetiology and also to identify groups or subgroups
sity, etc. Certain infectious diseases such as that are potentially at high risk of acquiring certain
typhoid, cholera and diarrhoeal disorders are diseases.
spread through the movement of people in masses VI Formulation of an aetiological hypothesis
as in the case of pilgrimages. This is the final step in a descriptive epidemiologi-
IV Measurement of disease cal study. By studying the determinants and distri-
l The measurement of disease is done in terms of bution of a disease, it is possible to formulate a hy-
mortality and morbidity indicators. pothesis related to the aetiology of disease.
l The morbidity can be expressed in terms of ‘in- Hypothesis can be defined as ‘a supposition arrived
cidence’ and ‘prevalence’. The incidence of a at by observation or by reflection’. The hypothesis
disease can be obtained by using a ‘longitudinal can be tested using the techniques of analytical
study’, and the prevalence can be obtained from epidemiology after which it may be accepted or
a ‘cross-sectional study’. rejected.
a. Cross-sectional studies (prevalence studies) An epidemiologic hypothesis should specify the
l The simplest in concept and execution is the following:
cross-sectional study. In this, a set of individu- l The population – the characteristics of the per-

als are chosen who may be a representative sons to whom hypothesis applies
sample of the general population or of people l The cause being considered – the environmental

in a particular community or a sample of exposure


members of some special subgroup – school l The expected effect – the disease

children, the armed forces and workers in a l The dose–response relationship – the amount of

particular industry, etc. cause needed to lead to the stated incidence of


l The measurements of exposure and effect are effect
made at the same time, providing information l The time–response relationship – the time period

about the relationship between a disease and that will elapse between exposure to the cause
others; it is relatively easy and economical to and observation of the effect
carry out. The uses of descriptive epidemiology are as
b. Longitudinal studies follows:
● In simpler words, it can be said that longitu- l Provides data with regard to the types of disease

dinal studies are cross-sectional studies problems and their magnitude in the community.
610 Quick Review Series for BDS 4th Year, Vol 1

l Provides information about the aetiology of a The following studies have been done by different profes-
disease, and helps in the formulation of an aetio- sionals in India on the prevalence of periodontal diseases:
logical hypothesis. l School-going children
l Provides data required for planning, organiz- l Handicapped children
ing and evaluating preventive and curative l Pregnant women
services. l Adults
l Leads the path for further research with regard to A. School-going children
a particular disease problem. Important studies conducted among school-going chil-
Q.2. Discuss the aetiology and epidemiology of peri- dren are as follows:
odontal diseases. Describe the preventive strategies of Rao et al. (1980) conducted a study among 500
periodontal diseases in Indian population. schoolchildren aged 5–10 years in Udupi and re-
ported that oral hygiene status was poor in all chil-
Ans. dren. Twenty-eight per cent of the study population
had marginal gingivitis and 72% had chronic general-
ized gingivitis. Poor oral hygiene was found to be the
{SN Q.20}
major factor for increase in prevalence of gingivitis.
Epidemiology is defined as the study of the distribution Rao (1985) studied oral hygiene status of school-going
and determinants of health-related states and events in children aged 4–14 years in Mysore. Results showed
populations, and the application of this study to control that oral hygiene was better in girls than in boys.
health problems (John Last, 1988). Pandit et al. (1986) examined 480 boys and girls, aged
Parkin (1873) defines epidemiology as ‘the branch 8–18 years, in Delhi using papillary marginal attach-
of medical science which deals with the treatment of ment index, and found prevalence of periodontal dis-
epidemics’. ease to be 41.7%. The prevalence was found to be
MacMahon (1960) defines epidemiology as ‘the 42.2% in the age group of 8–10 years, while in the age
study of the distribution and determinants of disease group of 11–13 years, it was 44.2% and above 14 years
frequency in man’. of age, the prevalence was 54.6%. This study showed
M Last (1988) defines epidemiology as ‘the study of an increase in prevalence with age.
the distribution and determinants of health-related states Srivastava (1989) examined 690 children aged 6–17
or events in specified populations, and the application of years in Jhansi, and reported that 6–8 years age
this study to the control of health problems’. group had a prevalence of 42% and 15–17 years age
group had a prevalence of 94.02%, indicating an in-
crease in prevalence with age.
Epidemiology of Periodontal Disease Bode et al. (1990) examined 1240 tribal students in
Progress was slower in the field of periodontal diseases the Wardha district of Maharashtra, and found that
compared with dental caries because unlike dental caries, raw food decreased the prevalence of periodontal
periodontal diseases do not lend itself easily to objective disease.
measurement. B. Among handicapped children
Changing perceptions of periodontal diseases Mehrotra et al. (1982) examined 61 physically handi-
In 1961, the expert committee of WHO said that ‘gingi- capped and 66 mentally retarded individuals using the
vitis invariably developed to periodontitis’. Russell’s Index. Among physically handicapped chil-
Research in 1990s reached at the following findings: dren, the prevalence of periodontal disease was 88.5%
l Severe periodontitis was seen only in a small proportion as compared to 100% in mentally retarded subjects.
of population, whereas mild and mild-to-moderate gin- They reported that the greater prevalence was due to
givitis were more common. lack of awareness of oral hygiene.
l Gingivitis and periodontitis are associated with bacterial In 1991, in Mumbai, a total of 466 children, aged
flora that have some similarities but also some differences. 11–14 years, comprising mentally subnormal, physi-
Gingivitis precedes periodontitis. Fraction of sites and not cally handicapped, juvenile delinquents and normal
all sites with gingivitis later develop periodontitis. children, were examined; 100% periodontal disease was
l Although periodontal disease is usually related to age, it found in mentally subnormal, 95.95% in juvenile delin-
is not a natural consequence of aging. quents and 97.38% in physically handicapped children.
l Periodontal disease is not a major cause of tooth loss in The periodontal status of mentally abnormal children
adults. revealed that 7.45% had 4–5-mm-deep pockets requir-
Epidemiological studies on prevalence and incidence of ing surgical intervention. Sixty per cent of the children
periodontal diseases (India) had calculus deposits requiring oral prophylaxis.
Section | I  Topic-Wise Solved Questions of Previous Years 611

C. Among pregnant women bone loss is higher than the facial and lingual loss.
Dixit et al. (1980) studied in Lucknow the occurrence Severely affected are lower centrals and laterals and
and severity of gingivitis in 80 pregnant and 40 nonpreg- upper molars.
nant women in the age group of 20–40 years. The au- l Endocrine changes: Puberty, menstruation and preg-

thors found a significantly higher severity of gingivitis in nancy and hyperthyroidism increase the chances of
pregnant as compared with nonpregnant women. They gingivitis.
also noted increase in its severity in the second trimester. l Traumatic occlusion: Sharp cusp acts as ‘plungers’

D. Among adults and lead to periodontitis.


Ramachandra et al. (1973) surveyed 6647 rural and l Food impaction: Food impaction causes chronic gin-

1536 urban population of Tamil Nadu, and found that givitis, which if let to continue leads to periodontal
periodontal disease was high in both the cases (95.3% diseases.
and 95.5% respectively). Periodontal index score was l Tooth position: Irregular alignment makes it difficult

found to increase with age. to keep these areas clean. Sometimes the roots ap-
In 1990, Anil, Hari and Vijay Kumar T assessed the peri- proach each other too closely. This may allow for
odontal conditions of 2756 subjects, aged 15–44 years, insufficient intervening alveolar support, resulting in
from urban and rural areas of Trivandrum using the com- early pocket formation.
munity periodontal index of treatment needs (CPITN). The l Occupational habits: Occupational habits, such as

authors reported that calculus and bleeding was more fre- thread biting and holding nails between teeth, can
quent (860/86%) in the age group of 15–19 years, shallow have traumatic effects on the periodontium.
pocketing in 80% of the subjects in the age group of l Neuroses: Bruxism, lip, cheek and nail-biting also
5–29 years and deep pockets of more than 6 mm in 33% of have traumatic effects on the periodontium.
subjects in the age group of 35–44 years. l Use of tobacco: Components present in tobacco
Maily, Banarjee and Pal (1994) examined 5960 subjects lower tissue resistance and increase susceptibility to
aged 15–65 years in a rural population of West Bengal gingivitis and periodontal diseases.
using CPITN. The subjects selected were mostly poor l Misuse of toothbrush: Improper use of toothbrush
farmers. Remarkable finding was that there was relatively not only causes abrasion and recession of gingival
low percentage of people with deep periodontal pockets. tissues but also irritates already inflamed tissues.
They found the CPITN score of 4 in only two to three l Concomitant disease: There is a tendency towards
subjects. On the other hand, calculus was widespread. alveolar bone destruction in patients with uncon-
trolled diabetes. Heavy metal poisoning may accen-
Aetiology of Periodontal Diseases
tuate gingivitis. Acute monocytic leukaemia and
Epidemiological triad
pernicious anaemia cause gingivitis.
1. Host factors
l Income: Periodontal diseases increase with decrease
2. Agent factors
in income.
3. Environmental factors
l Education: Severity of periodontal diseases and level

1. Host factors of education are inversely related.


l Age: Older age groups. 2 . Agent factors
l Sex: More common in males. A. Dental plaque
l Race: Blacks are more affected. B. Calculus
l Intraoral variations: Gingivitis is more seen on the

interproximal areas than in the buccal and lingual A. Dental plaque


areas. The upper arch shows more gingivitis than the l It is defined as soft deposits that form a biofilm ad-

lower arch (except lingual). On the facial aspect, up- hering to tooth surface or other hard surfaces of oral
per first and second molars are more prone to gingi- cavity, including removable and fixed restorations.
vitis followed by lower anteriors, and the least prone l Disruption of balance between plaque bacteria

is the lower second premolar. On the lingual aspect, and host results in periodontal diseases.
lower first and second molars are the most prone to l Types of plaque

gingivitis, followed by lower premolar, and the least i. Supragingival


prone are the upper anteriors. Gingivitis is more of- ii. Subgingival plaque
ten seen on the right arch than on the left arch. iii. Marginal plaque
l Severity of bone loss: Incisor and molar areas are l Marginal plaque is responsible for gingivitis,

more severely involved than canine and premolar whereas supragingival and tooth-associated
areas. Maxillary teeth experience more bone loss subgingival plaque is responsible for calculus
than the mandible (except anterior). Interproximally and root caries.
612 Quick Review Series for BDS 4th Year, Vol 1

B. Calculus 3. Environmental factors


Calculus is an adherent calcified mass that forms on l Geographic variations: Periodontal diseases are

the surface of natural teeth and dental prosthesis. It found to be high in Chile, Jordan, India, Malaysia
consists of mineralized plaque. and Ceylon. Underdeveloped and dentist-deprived
Two types of calculus areas have increased scores of periodontal diseases.
i. Supragingival calculus: It is white or whitish- l Nutrition: Avitaminosis C and niacin deficiency

yellow in colour. show higher prevalence for periodontal diseases.


ii. Subgingival calculus: It is dark brown or l Degree of urbanization: Rural population seems to

greenish-black in colour. suffer more from periodontal diseases compared to


Calculus provides a fixed nidus for the con- urban population.
tinued accumulation of plaque. l Stress: Stress is said to predispose acute necrotizing

ulcerative gingivitis and is often seen in military


groups and in exam-going students.

Prevention of periodontal disease

Levels of
prevention Primary Secondary Tertiary
Preservative Health promotion Specific protection Early diagnosis Disability limitation Rehabilitation
service and treatment
promotion
Service provided by Periodic visit to dentist, Oral hygiene Self-examination and Use of dental Use of dental
individuals demand for preventive practice referral, use of dental service service
service service
Service provided by Dental health education Supervised Periodic scaling and Provision of dental Provision of
community promotion of research school brushing referral, provision of caries dental caries
and lobby efforts provi- programme dental service
sion oral hygiene aids

Service provided by Patient education, Correction of tooth Complete examina- Deep curettage, root Removable of
dental profession plaque control malalignment and tion, splint, curettage planning, splinting, fixed prosthesis
programme, recall oral prophylaxis corrective restorative periodontal surgery,
reinforcement and occlusal service selective extraction

Q.3. What is an epidemiological triad? Discuss the role . Host factors


A
of components of epidemiological triad in the causation The host is the man himself.
of dental caries. l Tooth

Composition: The enamel consists of 96% inorganic mat-


Ans.
ter and 4% water and organic matter. The dentine consists
Epidemiology is defined as ‘the study of the distribution of 35% of organic matter and water and 65% of inorganic
and determinants of health-related states and events in matter. The cementum consists of 45%–50% of inorganic
populations, and the application of this study to control matter and 50%–55% of organic matter and water.
health problems’ (John Last, 1988). Morphology: Food, bacteria and debris tend to get en-
Parkin (1873) defines epidemiology as ‘the branch of trapped due to the presence of deep, narrow, occlusal
medical science which deals with the treatment of epidemics’. fissures or buccal and lingual pits.
Prevention is defined as ‘the efforts, which are made Caries may develop rapidly on the base of fissures
to maintain normal development, physiological function since defects are common in these areas.
and to prevent diseases of the mouth and adjacent parts’ l Saliva
(Blackerby). By its flushing action, it helps in the removal of bacteria
It is defined as the perception of the disease process and food debris.
(Learell and Clark). Calcium and phosphorus
The quantity of calcium and phosphorus is inversely
[SE Q.1]
related to the rate of flow. Its quantity is less in the
{Epidemiological triad in the causation of dental caries case of caries-active individuals.
is as follows: Ammonia
. Host factors
A The quantity of ammonia is inversely related to car-
B. Agent factors ies activity because ammonia retards plaque forma-
C. Environmental factor tion and neutralizes acid.
Section | I  Topic-Wise Solved Questions of Previous Years 613

Urea Properties of cariogenic plaque


Urea gets hydrolyzed to ammonium carbonate by l The rate of sucrose consumption is higher.

urease, which has a neutralizing effect. l Synthesize more intracellular polysaccharides.

Ptyalin and amylase l More lactic acid is formed twice as much as extra-

Help in the degradation of starch cellular polysaccharide is produced.


pH of saliva l Higher levels of Streptococcus mutans.

The pH of saliva depends on its bicarbonate content. l Lower levels of Streptococcus sanguis and Acti-

As the flow rate of saliva increases, the pH also in- nomyces.


creases. As the viscosity of saliva increases, the car- C. Environmental Factor
ies activity also increases. The saliva also contains a Diet
number of antibacterial substances or enzymes like Diet is defined as the types and amounts of food
lactoperoxidase, lysozyme, lactoferrin and IgA. eaten daily by an individual. Difference in caries in-
l Sex cidence was noted among populations with dissimi-
Dental caries is more common in females. lar diets.}
l Race Some dietary studies are described below:
Dental caries is more common in the white population
[SE Q.6]
compared to the black population.
l Age {a. Vipeholm Study
It is more commonly seen in childhood, although dental l It was conducted by Gustafsson et al. in 1954.
caries is considered to be the disease with universal preva- l Investigation was done among 436 adult
lence. After the age of 60 years, root caries is seen, which inmates for five years in a mental institution
is mainly due to gingival recession and deterioration of at the Vipeholm Hospital, Sweden.
oral clearance ability. It shows three peaks: 4–8 years, l The diet provided in the institution was

11–19 years and 55–65 years. nutritious but contained little sugar, with no
l Familial heredity provisions for between-meal snacks.
‘Good or bad teeth run in the family’. Offspring have l There was a relatively low rate of dental

the same score as parents. Inheritance of a characteristic caries in the inmates.


tooth structure has lesser influence than environmental l The inmates were divided into seven groups,

factors. i.e. a control group, a sucrose group, a bread


l Developmental disturbances group, a chocolate group, a caramel group,
Presence of deep pits and fissures, enamel hypoplasia an 8-toffee group and a 24-toffee group.
and enamel defects make the tooth more prone to dental Conclusions of the study are as follows:
caries. (a) Increase in carbohydrate increased the
l Economic status level of dental caries.
In young primary school children, dental caries decreases (b) Risk of caries was greater if food was
with increase in income. Among adults, as income de- in a form that would be retained on
creases, there is a decrease in dental caries. tooth surfaces.
l Concomitant disease (c) Risk of caries was greater if sugar was
Dental caries is found to be less in subjects with consumed between meals.
controlled diabetes. (d) Increase in caries activity varied be-
l Oral hygiene habits tween individuals.
Dental caries is found to be less among those who main- (e) Increased caries activity rapidly disap-
tain good oral hygiene. peared upon withdrawal of sugar-rich
B. Agent Factors foods.
Consists of dental plaque forming streptococci (Strepto- (f) Increased caries activity was noticed
coccus mutans). because of a high concentration of
Role of microorganisms in caries sugar in solution and its prolonged re-
l They are a prerequisite for caries initiation. tention on tooth surfaces.
l A single type of microorganism is capable of (g) Caries activity is correlated with the
inducing caries. clearance time of sugar. Physical form
l Acid production is a prerequisite for caries but not and frequency of intake were impor-
all acidogenic organisms are cariogenic. tant in cariogenicity than total amount
l Streptococcus strains produce extracellular dex- ingested.
trans or levans. b. Hopewood House Study
l Organisms vary greatly in their capacity to induce l This study was conducted by Sullivan in

caries. 1958. The dental status of children aged


614 Quick Review Series for BDS 4th Year, Vol 1

between 3 and 14 years, residing at Hopewood House, e. Hereditary Fructose Intolerance


New South Wales, for 10 years, was studied longitudi- Persons having this rare metabolic disorder had learned
nally. to avoid any food that contains fructose or sucrose be-
l Absence of meat and a rigid restriction on refined carbo- cause consuming these foods causes symptoms of nau-
hydrate were the two principal features of the Hopewood sea, vomiting, tremor, etc. It was noted that the level of
House diet. It was found that 53% of the children at the dental caries was lower in this group compared with
Hopewood House were caries-free, whereas only 0.4% of others.
the 13 years old, state schoolchildren were free from car- Vitamins like A, D, K and B complex (B6), calcium,
ies. Children’s oral hygiene was poor and gingivitis was phosphorus, fluoride and amino acids like lysine and
prevalent in 75% of them. fats have an inhibitory effect on dental caries.
l This work shows that, in institutionalized children, at Geographic variations: Decayed missing filled teeth
least, dental caries can be reduced by diet, without (DMFT) is found to be decreasing in developed coun-
the beneficial effects of fluoride and in the presence tries, and increasing in developing countries. The use of
of unfavourable oral hygiene.} fluorides, oral hygiene practice and diet play a major
role as a cause for this difference.
[SE Q.7] Soil: Selenium is said to increase dental caries whereas
{c. Turku Sugar Study molybdenum and vanadium are said to decrease dental
l This study was conducted by Scheinin, and Makinen caries.
et al. in 1975. Urbanization: Dental caries is said to increase with ur-
l In a 2-year feeding study, 125 young adults, divided banization.
into three groups, consumed the entire dietary intake Climate: Sunlight is said to decrease caries whereas
using these sugars exclusively: sucrose group – 35 rainfall is said to increase dental caries. Ultraviolet
people, fructose group – 38 people and xylitol group – (UV) light from the Sun promotes synthesis of vitamin
52 people. D and along with other factors it might account for
l There was a dramatic reduction found in the inci- lower caries experience.
dence of dental caries after 2 years of xylitol con-
sumption. Prevention of Dental Caries
l For the first 12 months, fructose was as cariogenic as l The objective of different levels of prevention

sucrose but became less at the end of 24 months. is that every oral health activity implemented
l It was also found that frequent between-meal chew- by the individual, the community or the dental
ing of a xylitol gum produced an anticariogenic professional is targeted towards the preven-
effect.} tion of some aspects of the health–disease
d. Seventh-Day Adventist Children Study continuum.
l The Seventh-Day Adventist had certain restrictions l Coordinated efforts by the individual, the com-

in their diet, which included the limitation of munity and the dental professional are needed to
a. Sugar sticky elements attain and maintain optimum oral health because
b. Highly refined starches of the complexity of disease aetiology.
c. Between-meal snacking l A multitude of preventive dentistry services tar-

These studies show that the level of dental caries was geted towards dental caries, periodontal disease,
much lower in the Seventh-Day Adventist group com- oral cancer and orofacial defects are presented in
pared to the other children. the following tables:

Prevention of dental caries

Levels of
prevention Primary Secondary Tertiary
Preservative Health promotion Specific protection Early diagnosis and Disability Rehabilitation
service promote treatment limitation
Service provided Diet planning, demand for Oral hygiene practice. Self-examination Use of dental Use of dental
by individuals preventive service, periodic Appropriate use of and referral, use of service service
visit to the fluoride. Use of fluo- dental services
dental office ride dentifrices
Section | I  Topic-Wise Solved Questions of Previous Years 615

Levels of
prevention Primary Secondary Tertiary
Service provided Dental health education, pro- Supervised school Periodic scaling Provision of Provision of
by community motion of research and lobby brushing programme, and referral, provi- dental caries dental caries
efforts provision, oral hygiene pits and fissure seal- sion of dental
aids, diet counselling, caries ant, topical fluoride service
activity test application

Service provided Patient education, plaque Correction of tooth Complete examina- Complex restora- Removal of
by dental control programme, recall malalignment and tion, PRR, treatment tion, pulpotomy, fixed prosthe-
profession reinforcement oral prophylaxis of incipient lesion, RCT extraction sis implants
pulp capping

PRR, preventive resin restoration; RCT, root canal treatment and extraction.

Q.4. Define epidemiology. What are the different epidemio- Biases in case–control study are as follows:
logical study designs? Discuss the different types of bias. i. Memory or recall bias
When cases and controls are asked questions about their
Ans. past history, it may be more likely for the cases to recall
Epidemiology is defined as ‘the study of the distribution the existence of certain events or factors than the con-
and determinants of health-related states and events in trols who are healthy persons.
populations, and the application of this study to control
health problems’ (John Last, 1988). {SN Q.12}
Parkin (1873) defines epidemiology as ‘the branch of
medical science which deals with the treatment of epidemics’. ii. Selection bias
Essential steps in an epidemiological investigation are This is called diagnostic bias wherein diagnosis it-
as follows: self is more likely be biased if the exposure is pres-
i. First, acquire the basic knowledge about the disease ent in history. The cases and controls may not be
accumulated through clinical studies. representative of cases and controls in the general
ii. Define the problem. The first thing is to identify the population. There may be systematic differences in
disease clinically and see the persons who became sick characteristics between cases and controls. The se-
most recently. It is not desirable that one finds a disease lection bias can be best controlled by prevention.
different from what it really is, or what the person had
seen before. iii. Interviewer’s bias
iii. Collect the available data and formulate hypothesis – l The interviewer/tests/investigation, etc. may lack depth
not conclusion. This is intended to guide in planning in controls whereas the cases are thoroughly worked up.
the inquiry, which is necessary, if it is to be fruitful. It l When the interviewer knows the hypothesis and also
involves the knowledge of local conditions. knows who the cases are, the bias may occur.
iv. Plan to answer the following questions: l This prior information may lead the interviewer to
l Regarding collection of data – What to collect, how question the cases more thoroughly than controls re-
much and for how long to collect and how to record; what garding a positive history of the suspected causal factor.
degree of accuracy to aim at in making observations? This type of bias can be eliminated by double blinding.
l How to analyse? iv. Bias due to confounding
l How to interpret and present the report? l Confounding is an important source of bias.
v. Investigate each individual unit of the universe of the l This is seen especially when one has multiple risk
disease in respect of characteristics involved so that the factors that are related to each other. This bias can be
conclusions arrived could be applied with confidence to removed by matching in case–control studies.
the community as a whole. For practical purposes, sam-
pling techniques may be applied, which should be repre- {SN Q.12}
sentative and adequate for drawing up valid conclusions.
vi. Arrange for special investigation needed to establish v. Berkesonian bias
collateral circumstances using laboratory facilities, l This bias arises because of the different rates of ad-

engineering and other expert consultations. Bias is any mission to hospitals for people with different diseases.
systematic error in the determination of the association l It is termed after Dr Joseph Berkesonian, who

between the exposure and the disease. recognized this problem.


616 Quick Review Series for BDS 4th Year, Vol 1

vi. Problems due to overmatching l Likewise, smoking increases the risk of acquiring a
l By matching the common ones, such as age and sex, wide range of diseases.
one tries to reduce confounding. At times, investiga- v. Post hoc bias
tions land up in trouble by overmatching, wherein a l This is another potential bias in a cohort study. This arises

potential confounder is matched among cases and due to the use of data from a cohort study to make obser-
controls (e.g. religion, which has important role in vations that were not part of the original study intent.
substance abuse pattern). l Thus, interesting relationships are often observed in

Bias in analysis cohort studies, which were not originally anticipated.


It is due to the presence of confounder at the time of
Q.5. Define epidemiology. Describe its aims, objectives and
analysis.
various terminologies used in the field of epidemiology.
Bias in cohort studies
Factors related to the selection of population, re- Ans.
sponse rate, collection of information, methodolo-
Epidemiology is defined as ‘the study of the distribution
gies used and analytical strategies employed often
and determinants of health-related states and events in
introduce bias, which, if not anticipated, can lead to
populations, and the application of this study to control
incorrect conclusion concerning a possible relation-
health problems’ (John Last, 1988).
ship between an exposure and a disease.
Parkin (1873) defines epidemiology as ‘the branch of
medical science which deals with the treatment of epidemics’.
There are five different types of biases
Prevention is defined as ‘the efforts, which are made to
i. Selection bias
maintain normal development, physiological function and to
l This occurs when the group actually studied does
prevent diseases of the mouth and adjacent parts’ (Blackerby).
not reflect the same distribution of characteristics
Aims of epidemiology
like age, sex, occupation and race as occurring in the
The International Epidemiological Association has
general population.
listed three main aims of epidemiology, which were put
ii. Follow-up bias
forward in 1973 by Lowe and Kostrzewski as follows:
l To accomplish the successful follow-up of all mem-
l To describe the distribution and size of disease prob-
bers of the cohort is one of the major problems in a
lems in human populations
cohort study.
l To identify aetiological factors
l If the loss to follow-up occurs equally in the exposed
l To provide the data essential to the planning, implemen-
and unexposed groups, the internal validity should
tation and evaluation of services for the prevention,
not be affected, assuming, of course, that the rate of
control and treatment, and setting up priorities among
disease occurrence is the same among those lost to
those services
follow-up as among those not lost to follow-up
Ultimate Aim
within each exposure group.
To eliminate or reduce health problems or its conse-
l If the rate of disease is different among those who
quences and promote health and well-being
are lost to follow-up, then internal validity of the
study may be affected, i.e. the relationship between
exposure and outcome may be changed. {SN Q.6}
iii. Information bias
l This is also called ‘misclassification bias’ as it oc-
Principles of epidemiology
curs due to misclassification in the subgroups of the The four important principles are as follows:
l Exact observation (strict, vigorous, accurate, precise)
study population.
l Correct interpretation (free from error)
l Information bias occurs when there is an error in the
l Rational explanation (intelligent, sensible, reasonable)
classification of individuals with respect to the outcome
l Scientific construction (by expert knowledge and
variable. This may result from measurement errors,
imprecise measurements and misdiagnosis of cases. technical skill)
iv. Confounding bias
l This is the bias arising due to the ‘confounding factor’. Tools of measurement in epidemiology
Confounding factor can be defined as the factor that is The most commonly used measurements in epidemiol-
associated with both the exposure and the disease. ogy are as follows:
l Smoking and age are the two common confounders a. Measurements of mortality
in cohort. b. Measurements of morbidity
l The risk of disease varies with age for almost all c. Measurements of disability
diseases. d. Measurements of natality
Section | I  Topic-Wise Solved Questions of Previous Years 617

[SE Q.5] Use of mortality data


a. Explaining trends
{The most commonly used tools of measurement in epide- b. Indicating priorities for health action
miology are as follows:
c. Designing intervention programmes
i. Rates
ii. Ratio Mortality rates and ratio
iii. Proportions} i. Crude death rate 5 Number of deaths during the year
in mid-year population
(SE Q.5 and SN Q.5) ii. Specific death rate: It may be
(a) cause or disease specific
i. {(Rate: Measures the occurrence of some particular (b) related to specific groups
event in a population during a given time period. Specific death rates due to tuberculosis 5 number of
deaths from tuberculosis during a calendar year in a
Number of death in one year  1 000 mid-year population.
Death rate 
Mid-year population Specific death rates for males 5 number of deaths
among males during a calendar year in a mid-year
ii. Ratio: It expresses a relation in size between two population.
random quantities. Numerator is not a component of iii.
the denominator.
Number of children with scabies at a certain time Total Number of deaths
Number of children with malnutrition at a certain time d ue to a particular disease
Case fatality rate 5
iii. Proportion: It is a ratio that indicates the relation in Total number of cases
magnitude of a part of the whole. due to the same disease
Numerator is included in the denominator. It is usu-
iv. Proportional mortality rate: It expresses the ‘number of
ally expressed as percentage.)}
deaths due to a particular cause (or in a specific age
group) per 100 (or 1000) total deaths’.
[SE Q.5]
{The number of children with scabies at a certain No. of deaths from
time 3 100 the specific disease in a year
Pr oportional mortality rate 5
The total number of children in the village at the same time Total d eaths
from all causes in that year
Concept of numerator and denominator
a. Numerator: Refers to the number of times an event has v. Survival rate (e.g. 5-year survival rate)
occurred. In case of rate, the numerator is a component
of the denominator. Total number of patients
b. Denominator: It would be: Total number of patients diagnosed or treated
5
(I) Related to the population and would comprise alive after 5 yearss Measurement of morbidity
(i) Mid-year population
(ii) Population at risk Morbidity is defined as any departure, subjective
(iii) Person time or objective, from a state of physiological well-
(iv) Person distance being.
(v) Subgroup of the population Three aspects of morbidity are commonly measured by
(II) Related to the total events and comprise the num- morbidity rates or morbidity ratio, namely frequency,
ber of accidents ‘per 1000 vehicles’.} duration and severity. Disease frequency is measured
by incidence and prevalence.
Measure of mortality
The duration of illness is measured as average duration/
Mortality data provide the starting point for many epide-
cases. Case fatality rate may be used as an index of severity.
miological studies. The basis of mortality data is the Death
Certificate. Uses
Limitations of mortality data l They describe the extent and nature of disease in the
a. Incomplete reporting community.
b. Lack of accuracy l They usually provide more accurate and clinically rele-

c. Lack of uniformity vant information about patient characteristics.


d. Choosing a single cause of death l They serve as a starting point for aetiological studies

e. Diseases with low fatality and play crucial role in prevention.


618 Quick Review Series for BDS 4th Year, Vol 1

l They are needed for monitoring and evaluation of dis- I 5 10 cases/1000 population/year
ease control activities. D 5 5 years
P 5 10 3 5 5 50/1000 population
Incidence
Longer the duration of the disease, greater is its preva-
The number of new cases occurring in a defined popula-
lence (e.g. tuberculosis).
tion during a specified period of time.
Q.6. Define epidemiology. Describe the role of host
Number of new cases of specific
and environmental factors in the epidemiology of oral
disease during a given time period
Incidence rate 5 cancer.
Popp ulation at risk
during that period Ans.

For example, 500/30,000 3 1000 5 16.7/1000/year One of the major threats to public health in the developed
Uses of incidence rate world and increasingly in the developing world is cancer.
(a) To control disease In India, it is estimated that among the 400 million indi-
(b) For research of aetiology and pathogenesis, distribution viduals aged 15 years and above, 47% use tobacco.
of diseases and efficacy of preventive and therapeutic Aetiology and risk factors
measures l The cause of oral cancer is almost certainly multifacto-
Prevalence rial and involves many alterations in host immunity and
Prevalence refers specifically to all current cases (old 1 metabolism, angiogenesis and exposure to chronic in-
new) existing at a given point in time, or over a period flammation in a genetically susceptible individual.
of time in a given population. l The carcinogenic changes may be influenced by onco-
Two types genes, viruses, irradiation, drugs (tobacco and alcohol),
i. Point prevalence hormones, nutrients or physical irritants.
l Point prevalence is defined as the number of

all current cases (old 1 new) of a disease Established risk factors for development of oral cancer
at one point in time in relation to a defined l Smoking tobacco
l Chewing tobacco/oral snuff
population.
l Chewing betel quid (paan), especially if tobacco is
l The ‘point’ in point prevalence for all prac-

tical purposes may consist of day, several included


l Heavy consumption of alcohol
days or even a few weeks depending upon
l Presence of a potentially malignant oral lesion or condition
the time it takes to examine the population
sample. Other contributory or predisposing factors
l Number of all current cases of a specified dis- l Dietary deficiencies, particularly vitamins A, C and E
ease existing at a given point in time of esti- and iron
mated population at the same point in time. l Familial or genetic predisposition
ii. Period prevalence l Viral infections, particularly certain types of human
l Period prevalence measures the frequency of papilloma viruses
all current cases existing during a period of l Sunlight (for lip cancer)
time (annual prevalence) expressed in relation l Candida albicans infection
to a defined population. l Immune deficiency diseases or immune suppression

l Environmental exposure to the burning fossil fuels


Total number of existin g cases
l Dental trauma or chronic oral sepsis
of specific disease during a
given peeriod of time interval Life cycle
Period prevalence   100
Estimated mid-intervaal Most cases of oral cancer can be attributed to certain life-
polulation at risk style risk factors and are thus preventable. In a minority of
cases, particularly among younger patients, these are ab-
Uses of prevalence
sent, producing a challenge for research in their aetiology.
l Helps to estimate the magnitude of health/disease prob-
l A wide variety of risk factors have been implicated in
lems in a community and identify high-risk populations
oral cancer aetiology. These include use of tobacco, al-
l Useful for administrative and planning purposes
cohol, irradiation, oral sepsis, poor diet and nutrition,
Relationship between prevalence (P) and incidence (I) poor oral hygiene, chronic irritation from a sharp tooth
P 5 I 3 DI 5 P/D or broken restoration, syphilis, genetic predisposition,
D 5 PII altered immunity, etc.
Section | I  Topic-Wise Solved Questions of Previous Years 619

Tobacco
vine). Quid contains areca nut, which may be used raw
l There is an excellent evidence from many sources
or baked, boiled lime obtained from limestone or sea-
around the world that use of tobacco is by far the most
shells, and according to local customs may include ani-
important risk factor for oral cancer.
seed, catechu, cardamom, cinnamon, coconut, cloves,
l The use of tobacco has declined in some high-income
sugar and tobacco.
countries but is still increasing in low- and middle-in-
Mainpuri tobacco: Includes tobacco, slaked lime,
come countries, especially among young people and
finely cut areca nut, camphor and cloves. About 7% of
women.
people in Uttar Pradesh use this. A high prevalence of
l According to WHO (1984), in developing countries,
oral cancer and leukoplakia among persons who use
including India, the chewing of tobacco, and association
Mainpuri tobacco has been found.
with areca nut in the form of betel quid (paan), is the
Mawa: A preparation containing thin shavings of
most important cause of oral cancer.
areca nut with addition of some tobacco and slaked lime.
Tobacco use can be broadly classified as follows: It is usually sold wrapped in cellophane papers and tide
Smoking and smokeless forms in shape of small ball. Before consumption, the packet is
Smoking tobacco rubbed vigorously to mix the contents. This is most
l Manufactured cigarettes: These consist of shred or re- commonly seen in Gujarat.
constituted tobacco processed with hundreds of chemi- Mishrilmasheri: Prepared by roasting tobacco on a
cals. They are the predominant form of tobacco used hot metal plate until it is uniformly black. It is then pow-
worldwide. dered and used with catechu. Catechu is a residual ex-
l Bidis: These contain small amount of tobacco hand- tract obtained by soaking hard wood of the tree Acacia
wrapped in dried temburni leaf and tied with string. catechu in boiling water. It is used in Maharashtra, espe-
Bidis are found throughout Southeast Asia, and are the cially by women.
most used type of tobacco in India. Zarda: Tobacco leaf boiled in water along with lime
l Cigars: These are made of air-cured and fermented to- and spices until evaporation. The residual tobacco is
bacco with a tobacco wrapper, and come in all shapes then dried and coloured with dyes.
and sizes, from cigarette-sized cigarillos, cheroots, Gurakhu: It is a paste of powdered tobacco, molasses
stumpen, chuttas and dhumtis. and other ingredients; primarily used to clean teeth. It is
mostly used in Bihar.
In reverse chutta and dhumti smoking:
Gutkha: It is prepared by crushing the betel nut, to-
The end of cigar is placed in the mouth. The use of this
bacco and adding some sweet or savoury flavour. Gutkha
form of tobacco is commonly seen in coastal areas.
has originated from India but due to its sweetness, fla-
l Pipes:
vours and nuts, it has been taken up by young people, in
l These are made of briar, slate, clay or other inert
particular, outside India.
substance.
The snuff consists of two varieties:
l The tobacco is placed in the bowl and inhaled
Dry snuff: It is powdered tobacco that is inhaled
through the stem, sometimes through water.
through the nose or taken by the mouth. Its use is now in
l Water pipe, also known as shisha or hubbly bubbly,
decline.
is commonly used in northern Africa, the Mediter-
Moist snuff: A small amount of ground tobacco is
ranean region and parts of Asia. In Southeast Asia,
held in the mouth between the cheek and the gum.
clay pipes, known as suipa, chilum and hookli, are
Other products include khaini, shammah and nass or
used widely.
naswa. Areca nut alone can be carcinogenic and appears
l Kreteks: These are clove-flavoured cigarettes. They are
to be responsible for a high incidence of oral cancer in
widely smoked in Indonesia.
Melanesia, where it is often taken along with smoking.
l Sticks: These are made from sun-cured tobacco, known
A major cause of the distressing condition is areca nut.
as brus, and wrapped in cigarette paper.

l Oral submucous fibrosis, which has a high rate of ma-


{SN Q.9}
lignant transformation (up to 6% over 10–15 years).
Smokeless tobacco l The risk of oral cancer increases with amount of to-

The two types of smokeless tobacco are as follows: bacco consumed per day and the number of years of
Chewing tobacco and snuff consumption.
Types of chewing tobacco are as follows: l The case against tobacco is further strengthened by the

Paan: Most common form dating back to more than findings that the cancers almost always occur on the
2000 years. Paan means betel leaf (from Piper betle side of the mouth where the tobacco quid is kept, and
620 Quick Review Series for BDS 4th Year, Vol 1

the probability of developing cancer is directly related Diet


with the duration and intensity of use. l Diet is probably the next most powerful factor. A healthy

diet protects against cancer. The antioxidant vitamins A,


Alcohol
C and E scavenge potentially mutagenic radicals from
l The second most important risk factor for oral cancer is
damaged cells.
excessive consumption of alcohol.
l Trace elements like zinc and selenium and adequate
l Alcohol with tobacco acts synergistically so that the
supply and absorption of iron to prevent anaemia (with
combined damage is more than multiplied. There has
consequent mucosal atrophy) are also important.
been a doubling of the alcohol consumption per head of
the population in the last few decades in many western Other factors
countries, and this is thought to be the main reason for White plaques, red plaques and submucous fibrosis, white
their rising incidence of oral cancer. patches, yeasts or hyphae of the fungus, Candida albicans,
and human papilloma viruses (HPV types 16 and 18).
Prevention of oral cancer at different levels

Levels of
prevention Primary Secondary Tertiary
Preservative service Health promotion Specific Early diagnosis and Disability Rehabilitation
protection promotion treat- limitation
ment
Service provided by Diet planning, demand for Avoidance of Self-examination Use of dental Use of dental
individuals preventive service, periodic irritants and referral, use of service service
visit to dental office dental services
Service provided by Dental health education, Periodic scaling Provision of Provision of dental
community promotion of research and and referral, dental caries caries
lobby efforts provision, oral provision of dental
hygiene aids, diet counsel- service
ling, caries activity test

Service provided by Patient education Avoidance of Complete examina- Chemotherapy, Maxillo facial
dental profession known irritations tion, biopsy, oral radio therapy, surgery, removal
in oral cavity cytology, complete surgery of fixed prosthesis.
excision Plastic surgery,
speech counselling

Q.7. Discuss the scientific method for conducting dental . Conducting examinations
D
epidemiological investigation. Give importance of each E. Analysing the data
step. F. Drawing the conclusions
G. Publishing the results
Ans.
A. Establishing the objectives
Epidemiology is defined as ‘the study of the distribution The investigator must be absolutely clear about the objec-
and determinants of health-related states and events in tives of the investigation before considering its design. In
populations, and the application of this study to control many scientific experiments, the objectives can be stated
health problems’ (John Last, 1988). in the form of a hypothesis is to be tested. The starting
Parkin (1873) defined epidemiology as ‘the branch of point of a study is frequently an expression of a null hy-
medical science which deals with the treatment of epidemics’. pothesis; that is the assumption, for example, there is no
MacMahon (1960) defined epidemiology as ‘the study of difference in the extent of dental disease between the
the distribution and determinants of disease frequency in man’. groups to be investigated, or in the circumstances of a
M Last (1988) defines epidemiology as ‘the study of the clinical trial, and that one method is no better than an-
distribution and determinants of health-related states or other in preventing or treating a disease or condition. The
events in specified populations, and the application of this objective of the study is then to test this hypothesis.
study to the control of health problems’. B. Designing the investigation
An epidemiology should follow a scientific method for Types of study
conducting dental epidemiological investigation. Example: There are two main subdivisions of both descriptive
A. Establishing the objectives and analytical epidemiology.
B. Designing the investigation i. The prevalence study or point prevalence or
C. Selecting the sample cross-sectional study: Here the occurrence of a
Section | I  Topic-Wise Solved Questions of Previous Years 621

disease or condition in a population is ex- dentifrice will enable the effects of the fluoride
pressed at a given point in time. The preva- toothpaste to be assessed.
lence study is commonly used for comparing C. Selecting the sample
between two or more populations or the same In a selected sample, a criterion is set for inclusion in
populations. the study and each individual satisfying the criterion
ii. The incidence or longitudinal study: Here the forms part of the sample. This type of sample may be
number of new diseases in a population is mea- self-selected or it may be selected by the research
sured over a period of time. worker. The self-selected sample involves volunteers for
Clinical trials examination, and they may differ greatly from the rest
When a new treatment has been developed, it is im- of the population. In a dental survey, subjects might
portant to provide answers to two questions: volunteer for an oral examination because they are
(a) ‘Does it work’? proved of having good dentition or, alternatively, be-
(b) ‘Is it better than the existing treatment’? cause they think that they need treatment and hope that
l The first stage is to conduct an experimental this will be done for them free of cost. Self-selection
clinical trial in which the effectiveness of inevitably leads to bias in one direction or another.
the treatment is measured under ideal con- Selection by epidemiologist may take many forms, such
ditions. A fissure sealant, for example, may as by selecting each patient whose registration number
be tested in older children using premolar ends in 3, whose birthday is on the 13th of month or
teeth and the clinical operation performed whose surname begins with a certain letter. These meth-
by the researcher or another experienced ods may lead to bias.
clinician. Sometimes a stratified random sample is taken. If the
l However, even if the results of an experimen- condition under investigation is known to be related to
tal clinical trial are encouraging, these are various factors such as age, sex or area of residence, the
only applicable under these ideal conditions. population is first divided into these groups or strata and
l Under normal conditions, the fissure seal- a random sample is taken in each stratum; for example,
ant would often be applied to the molars of dental caries is an age-specific disease, so any popula-
younger children, a technique performed tion in which a survey is to be made into the prevalence
by a variety of clinicians. or extent of the disease should be stratified by age.
l It is therefore necessary to carry out a D. Conducting examinations
community clinical trial or field trial, Three aspects are important for the scientific epidemio-
which is designed to measure the efficacy logical study of dental disease and conditions:
of agent under real-life conditions. l The examination methods and diagnostic aids.

l This enables the investigator to decide l The diagnostic criteria.

whether the new form of treatment is a l The indices used for measurement and reporting

more efficient use of resources than the Basic requirements for the mouth examination are a
existing treatment. chair, preferably with a headrest, to seat the subject;
Controls a source of illumination, a lamp attached to the head
The control group must be as similar as possible to of the examiner or a fibre optic light source; and
the test group (exposed individuals), except in re- some method of cleaning the teeth to remove loose
spect of the factor under investigation, when the in- debris, where necessary.
vestigation is to be carried out. The control group is Two principles should be considered:
necessary both when the factor under investigation is First, the examination for epidemiological purposes
naturally occurring and when it is under the control should be as automatic as possible to obviate exces-
of an epidemiologist. sive intrusion of subjective thought, and for this rea-
Methods of study son, it is probably desirable to perform it quickly.
Some investigations can be carried out as controlled Second, the objective of epidemiological study is to
experiments by giving or withholding a specific fac- examine in fairly large numbers.
tor and measuring the effect on the prevention or Excessive time spent on each individual necessitates
cure of a disease. For example, a clinical trial may be a reduction in the number of individuals examined.
designed to investigate the efficacy of a fluoride A recorder, live or tape, is necessary for receiving
toothpaste. Here, a series of prevalence studies may the information called by the examiner; even if a hu-
be made at intervals on the same group of children to man recorder is available, some workers use a tape
ascertain the incidence of new caries in each child. recorder as well to check possible errors in the
Comparison with a control group using a placebo transcription.
622 Quick Review Series for BDS 4th Year, Vol 1

Before any disease or condition can be studied, it is Levels of prevention


necessary to decide on well-defined criteria for its There are three levels of prevention:
diagnosis and classification. I. Primary prevention
If more than one examiner is involved in a study, they It is defined as the action taken prior to the onset of
must be carefully standardized in their diagnosis. disease, which removes the possibility that a disease
They should undergo training. will ever occur.
E. Analysing the data WHO recommended the following approaches for the
The methods of analysis must be considered at the begin- primary prevention of chronic diseases where the risk
ning of the investigation, not after it has been completed. factors are established:
F. Drawing the conclusions and publishing the report l Primordial prevention

The conventional pattern is summarized as follows: l Population (mass) strategy

Introduction: The introduction should include the l High-risk strategy

review of literature, reasons for conducting the pres- To have an impact on the population, all the above-
ent investigation, objectives of the present investiga- mentioned approaches should be implemented.
tion, and the hypothesis to be tested. Primordial prevention
Materials and methods: This deals with the selection In this level, efforts are directed towards discourag-
and description of the samples and the methods used ing children from adopting harmful lifestyles. The
in diagnosis, together with diagnostic criteria. The main intervention is through individual and mass
whole method and technique of investigation should education.
be set out clearly. Population (mass) strategy
G. Publishing the report This approach is directed at the whole population
Results: These should be tabulated and illustrated ap- irrespective of individual risk levels (e.g. water fluo-
propriately with relevant implication in the text. ridation).
Discussion and conclusions: The investigation, its find- High-risk strategy
ings and its conclusions are discussed at the discretion The aim of this level is to bring preventive care to
of the author. individuals at special risk. This requires detection of
individuals at high risk.
Q.8. Define prevention in relation to dental and oral di-
II. Secondary prevention
agnosis and different levels of prevention. Describe the
It employs routine treatment methods to terminate a
procedure to control and prevent periodontal disease.
disease process and to restore tissues to as near normal
Ans. as possible.
III. Tertiary prevention
Blackerby defined it as the efforts made to maintain normal
It employs measures necessary to replace lost tissues
development and physiological function and to prevent
and to rehabilitate patients to the point that functions is
diseases of the mouth and adjacent parts.
as near normal as possible after the failure of second-
Learell and Clark defined it as the interception of the
ary prevention.
disease process.

Control and prevention of periodontal disease

Levels of
prevention Primary Secondary Tertiary
Preservative Health promotion Specific protection Early diagnosis and Disability Rehabilitation
service treatment promotion limitation
Service provided Periodic visit to dentist, Oral hygiene Self-examination and Use of dental Use of dental
by individuals demand for preventive practice referral, use of dental service service
service services
Service provided Dental health education, Supervised Periodic scaling and Provision of dental Provision of
by community promotion of research school brushing referral, provision of caries dental caries
and lobby efforts provi- programme dental service
sion, oral hygiene aids

Service Patient education, Correction of tooth Complete examina- Deep curettage, root Removal of
provided by plaque control misalignment and tion, splint, curettage planning, splinting, fixed prosthesis
dental profession programme, recall oral prophylaxis corrective restorative periodontal surgery,
reinforcement and occlusal service selective extraction
Section | I  Topic-Wise Solved Questions of Previous Years 623

Q.9. Explain in detail about descriptive epidemiology. Q.19. Define epidemiology. Discuss environmental fac-
tors affecting dental caries prevalence.
Ans.
Ans.
[Same as LE Q.1]
[Same as LE Q.3]
Q.10. Describe steps of a descriptive study.
Q.20. Define epidemiology. Discuss the role of environ-
Ans.
mental factors in the epidemiology of dental caries.
[Same as LE Q.1]
Ans.
Q.11. Define epidemiology and dental epidemiology. Dis-
[Same as LE Q.3]
cuss significance of health trends and describe the role of
descriptive epidemiology in epidemiological investigation. Q.21. Define prevention. What are the objectives of dif-
ferent levels of prevention?
Ans.
Discuss the measures used for primary prevention of
[Same as LE Q.1]
dental caries at individual and community levels.
Q.12. What are aims of epidemiology? Explain case–
Ans.
control studies in detail.
[Same as LE Q.3]
Ans.
Q.22. Write preventive dentistry. What are the various
[Same as LE Q.1]
levels of prevention? Write in detail about the primary
Q.13. Describe steps in conducting a randomized con- and secondary levels of prevention of oral cancer.
trolled trial.
Ans.
Ans.
[Same as LE Q.6]
[Same as LE Q.1]
Q.23. Define a tumour. Write in detail the epidemiologi-
Q.14. Define epidemiology. Describe in detail the vari- cal triad of oral cancer.
ous epidemiological studies.
Ans.
Ans.
[Same as LE Q.6]
[Same as LE Q.1]
Q.24. Define epidemiology. Describe in brief the epide-
Q.15. Define epidemiology. Describe in brief the types of miology of oral cancer.
epidemiological studies.
Ans.
Ans.
[Same as LE Q.6]
[Same as LE Q.1]
Q.25. Define epidemiology. Describe the epidemiological
Q.16. Define epidemiology. What are the different types of factors of oral cancer.
epidemiological studies? Describe prospective and retro-
Ans.
spective studies. Mention the merits and demerits of each.
[Same as LE Q.6]
Ans.
Q.26. Mention different oral habits contributing to the
[Same as LE Q.1]
development of such lesions in India, and describe any
Q.17. Define epidemiology. Write in detail the epidemi- two conditions in detail.
ology of periodontal diseases.
Ans.
Ans.
[Same as LE Q.6]
[Same as LE Q.2]
Q.18. Describe the role of agent and host factors in the
SHORT ESSAYS:
epidemiology of periodontal diseases. Q.1. Epidemiological triad.
Ans. Ans.
[Same as LE Q.2] [Ref LE Q.3]
624 Quick Review Series for BDS 4th Year, Vol 1

Q.2. Advantages and disadvantages of case–control Exposure rates


study. a. Cases 5 a/(a 1 c) 5 33/35 5 94.2%
b. Controls 5 b/(b 1 d) 5 55/82 5 67.0%
Ans.
The next step will be to ascertain whether there is a
Advantages of case–control studies statistical association between exposure status and
a. Can be carried out easily. occurrence of oral cancer.
b. Rapid and less cost (compared with cohort studies). (b) Estimation of risk
c. Few subjects are required. ‘Relative risk’ (RR) or ‘risk ratio’:
d. These studies are suitable to investigate rare diseases Relative risk is defined as the ratio between the inci-
or diseases about which little is known. dence of disease among exposed persons and incidence
e. No risk to subjects. among nonexposed persons.
f. Allows the study of several different aetiological fac- It is given by the formula:
tors (e.g. tobacco chewing and alcohol, and genetic
factors in oral cancer). Incidence among exposed
Re lative risk 5
g. Risk factors can be identified. Rational prevention Incid ence among a nonexposed
and control programmes can be established. Analysis and interpretation in case cohort study.
h. No attrition problems, because case–control studies The data are analysed in terms of
do not require follow-up of individuals in the future. Incidence rates of outcome among exposed and nonex-
i. Minimal ethical problems. posed.
Disadvantages of case–control studies In a cohort study, we can determine incidence rates di-
a. Since it relies on memory or past records, there is a rectly in exposed and nonexposed cohorts. A hypothetical
problem of bias, the accuracy of which may be un- example shows how incidence rates may be calculated.
certain; validation of information obtained is difficult
or sometimes impossible. Example of a cohort study on tobacco chewing and oral
b. Difficulty in selection of an appropriate control group. cancers
c. Incidence cannot be measured, and can only estimate
the relative risk. Did not
d. The cause and associated factors cannot be distin- Risk factor Developed developed
guished. In many case–control studies, it is a seri- (chewing tobacco) oral cancer oral cancer Total
ous problem where it is not possible to determine Present (chewers) 45 9955 10,000
whether the attribute led to the disease/condition or Absent (nonchewers) 5 9995 14,000
vice versa.
e. The evaluation of therapy or prophylaxis of disease
cannot be done. Incidence rates:
f. The representativeness of cases and controls is an- (a) Among tobacco chewers5 45/10,000 5 4.5 per
other major concern. 1000
(b) Among nonchewers5 5/10,000 5 0.5 per 1000
Q.3. Cohort studies. Estimation of risk
Ans. Having calculated the incidence rates, the next step
is to estimate the risk of outcome (e.g. disease or
Analysis and interpretation in case–control study. death) in the exposed and nonexposed cohorts. This
This is the final step in the analysis of case–control is done in terms of relative risk or ‘risk ratio’.
study. It is used to find out: It is the ratio of the incidence of disease (or death)
(a) Exposure rates among cases and controls to suspected among exposed and nonexposed groups.
factor RR 5 Incidence of disease (or death) among
(b) Estimation of disease risk associated with exposure exposed and
(odds ratio) Incidence of disease (or death) among nonexposed
(a) Exposure rates 4 .5
9
A case–control study provides a direct estimation of  0 .5
exposure rates (frequency of exposure) to a suspected This implies nine times higher risk of developing
factor in disease and non-disease groups (e.g. in a case– oral carcinoma in tobacco chewers compared with
control study of tobacco chewers and oral cancer): nonchewers.
Cases (without oral cancer) 5 a Estimation of relative risk is important in aetiologi-
Cases (with oral cancer) 5 b cal enquiries.
Section | I  Topic-Wise Solved Questions of Previous Years 625

Attributable risk (AR) or ‘risk difference’ SHORT NOTES:


It is the difference in the incidence rates of dis-
ease (or death) between exposed and nonexposed Q.1. Write a note on screening.
groups. Ans.
Q.4. Comparison of case–control and cohort studies. l Screening is also known as case finding.
Ans. l It is the process of searching apparently healthy popula-
tions for cases of early disease. The object is to cover as
Case–control study large a population as possible with as simple a test as it
l Case–control study is retrospective. will yield helpful results.
l It proceeds from effect to cause; here the disease has l There are some public health officials who feel that because
already occurred. dental disease is almost universal, all school children should
l Presence of exposure in cases and controls is compared. be routinely referred to as sources of dental treatment.
l Relatively easy to carry out.

l Useful for rare cases with smaller numbers.


Q.2. Enumerate the various steps in a cohort study.
l Can only have one outcome but can have multiple Ans.
‘exposures’.
l Only derives odds ratio.
The following are the various steps in a cohort study:
l Substantial biases can occur.
i. Selection of study subjects
l Relatively less costly and no dropouts.
ii. Obtaining data on exposure
Cohort study iii. Selection of comparison groups
l Cohort study is prospective.
iv. Follow-up
l Proceeds from cause to effect; here the disease is
v. Analysis
expected to occur in the future. Q.3. Blinding.
l Development of disease in exposed and nonexposed
Ans.
compared.
l Time-consuming and difficult to carry out. In order to reduce problems, a technique known as ‘blinding’ is
l Suitable for common diseases with common exposure. adopted in cohort studies. Blinding can be done in three ways:
l Can have multiple outcomes. (a) Single-blind trial: The trial is so planned that the par-
l Derives relative risk, attributable risk, etc. ticipant is not aware whether he or she belongs to a
l Biases are generally lower. study group or control group.
l Relatively expensive, and dropout rate is higher. (b) Double-blind trial: The trial is so planned that neither
the investigator nor the participant is aware of the group
Q.5. Tools of measurements on epidemiology.
allocation and the treatment received.
Ans. (c) Triple-blind trial: The participant, the investigator and
the person analysing the data are all ‘blind’. Ideally, of
[Ref LE Q.5]
course, triple blinding should be used but double blind-
Q.6. Vipeholm study. ing is the most frequently used method.
Ans. Q.4. Define and discuss pandemic.
[Ref LE Q.3] Ans.
Q.7. Turku sugar studies. Pandemic is defined as an epidemic usually affecting a large
proportion of population, occurring over a wide geographic
Ans.
area such as a section of a nation, the entire nation, a continent
[Ref LE Q.3] or the world (e.g. influenza pandemics of 1918 and 1957).
Q.8. Elements of a cohort study. Q.5. Rate, ratio and proportion.
Ans. Ans.
[Same as SE Q.3] [Ref LE Q.5]
Q.9. Analysis in case–control and cohort studies. Q.6. Principles of epidemiology.
Ans. Ans.
[Same as SE Q.3] [Ref LE Q.5]
626 Quick Review Series for BDS 4th Year, Vol 1

Q.7. Odds ratio. Types of cohort studies:


1. Prospective cohort studies
Ans.
2. Retrospective cohort studies
Odds ratio is the measure of the strength of association 3. A combination of retrospective and prospective cohort
between risk factor and outcome. studies (ambispective)
The derivation of odds ratio is based on three Prospective cohort studies
assumptions: A prospective cohort study (or ‘current’ cohort study)
(a) The disease being investigated must be relatively rare. is one in which the outcome, for example, disease
In fact, the majority of chronic diseases have a low in- has not yet occurred at the time the investigation
cidence in the general population. begins. Most prospective studies begin in the present
(b) The cases must be representative of those with a disease. and continue into the future.
(c) The controls must be representative of those without a Q.15. Matching.
disease.
Odds ratio 5 ad/bc Ans.
If the odds ratio is estimated at 8.1, this implies that the l Matching is a step in case–control studies.
risk of oral cancer was 8.1 times higher in individuals l The controls may differ from the cases in a number of fac-
who chewed tobacco than in those who did not chew tors such as age, sex, occupation and social status. An impor-
tobacco. tant consideration is to ensure comparability between cases
and controls. This involves what is known as ‘matching’.
Q.8. Cross-infection. l Matching is defined as ‘the process by which selection

Ans. of controls is done in such a way that they are similar to


cases with regard to certain pertinent selected variables
Cross-infection is defined as the presence of an infectious (e.g. age) which are known to influence the outcome of
agent on a body surface, also on or in clothes, beddings, disease and which, if not adequately matched for com-
toys, surgical instruments (or) dressings or other inanimate parability, could distort or confound the results’.
articles (or) substances, including water milk and food. It is
also called contamination. Q.16. Case–control study.
Ans.
Q.9. Smokeless forms of tobacco.
Case–control study is a common first approach to test
Ans. causal hypothesis. The case–control method has following
distinct features:
[Ref LE Q.6] l Both exposure and outcome (disease) have occurred

before the start of the study.


Q.10. Zoonosis. l The study proceeds backwards from effect to cause.

Ans. l It uses a control or comparison group to support or

refute an inference.
An infection or infectious disease transmissible under natu-
ral conditions from vertebrate animals to man. There are four basic steps in conducting a case–control study:
l Selection of cases and controls
Q.11. Long-term/secular trends. l Matching

l Measurement of exposure
Ans.
l Analysis and interpretation
[Ref LE Q.1] l Cohort study

Q.12. Bias. Q.17. True positive.


Ans. Ans.
[Ref LE Q.4] True positive is a measure used to evaluate a screening test
which denotes those individuals found positive on the test,
Q.13. Descriptive epidemiology.
who have the condition or disorder being studied.
Ans.
Q.18. Cross-sectional studies.
[Ref LE Q.1]
Ans.
Q.14. Prospective studies.
l Cross-sectional studies (prevalence studies) are simplest
Ans. in concept and execution.
Section | I  Topic-Wise Solved Questions of Previous Years 627

l In this, a set of individuals are chosen who may be a Randomized controlled trials (RCT)
representative sample of the general population or of The basic steps in conducting an RCT include the
people in a particular community or a sample of mem- following:
bers of some special subgroup – school children, the l Drawing up a protocol

armed forces, workers in a particular industry, etc. l Selecting reference and experimental populations

l The measurements of exposure and effect are made at l Randomization

the same time, providing information about the relation- l Manipulation or intervention

ship between a disease and others; it is relatively easy l Follow-up

and economical to carry out. l Assessment of outcome

Q.19. Analytical epidemiology. Q.23. Mention the types of descriptive studies.


Ans. Ans.
l Analytical epidemiology is the second major type among [Same as SN Q.13]
epidemiological studies.
Q.24. Methods of collection of data.
l Here the focus is the individual within a population

unlike descriptive epidemiology, which focuses on the Ans.


entire population. It is designed primarily to establish the
Following are the methods of collection of data:
causes of disease by investigating association between
l Questionnaires
exposure to a risk factor and the occurrence of disease.
l Surveys
l The objective is to test the hypothesis. They are of two
l Records
types:
l Interviews
a. Case–control study
b. Cohort study Q.25. Tobacco-related cancer.
Q.20. Define epidemiology. Ans.
Ans. Tobacco-related cancer is the established risk factor for
development of oral cancer.
[Ref LE Q.2]
Various forms of tobaccos are as follows:
Q.21. Cohort study. l Smoking tobacco

l Chewing tobacco/oral snuff


Ans.
l Chewing betel quid (paan), especially if tobacco is

l Cohort is defined as a group of people who share a com- included


mon characteristic or experience within a defined time
Q.26. Morbidity and mortality.
period. (Prospective study, longitudinal study, incidence
study and forward-looking study.) Ans.
l Three types of cohort studies have been established on
l Morbidity is defined as any departure, subjective or
the basis of the time of occurrence of disease in relation
objective, from a state of physiological well-being.
to the time at which the investigation is initiated and
l Three aspects of morbidity are commonly measured by
continued:
morbidity rates or morbidity ratio, namely frequency,
i. Prospective cohort studies
duration and severity.
ii. Retrospective cohort studies
l Mortality data provide a starting point for many epide-
iii. A combination of retrospective and prospective co-
miological studies.
hort studies (ambispective)
l The basis of mortality data is the death certificate.

Elements of a cohort study


Use of mortality data
l Selection of study subjects
l Explaining trends
l Obtaining data on exposure
l Indicating priorities for health action
l Selection of comparison groups
l Designing intervention programmes
l Follow-up

l Analysis Q.27. Blind study.


Q.22. Randomized trial. Ans.
Ans. [Same as SN Q.3]
628 Quick Review Series for BDS 4th Year, Vol 1

Q.28. Pandemic. Q.30. Selection bias.


Ans. Ans.
[Same as SN Q.4] [Same as SN Q.12]
Q.29. Ratio. Q.31. Berkesonian bias.
Ans. Ans.
[Same as SN Q.5] [Same as SN Q.12]

Topic 4
Dental Public Health
COMMONLY ASKED QUESTIONS
LONG ESSAYS:
1. Write in detail about the health care delivery systems in India.
2. Describe in detail about finance in dental diseases.
3. Define dental auxiliaries. Write in detail about various dental auxiliaries.
4. Discuss in detail the principles of ethics to be followed by a dentist. Add a note on unethical practices by dentists.
5. Enumerate the different chapters of Dentists Act of India. Write composition and functions of Dental Council
of India.
6. Write in detail various modes of payment for dental care. [Same as LE Q.2]
7. Describe the method of payment for dental care. [Same as LE Q.2]
8. Describe payment plans for dental care. [Same as LE Q.2]
9. Discuss in detail the various mechanisms of payment of dental care. [Same as LE Q.2]
10. What are the various methods by which payment for dental health services are carried out? [Same as LE Q.2]
11. What is meant by Dental Service Corporation? Describe the various dental prepayment plans. [Same as LE Q.2]
12. Define and classify dental auxiliaries. Write about the functions of a school dental nurse and a dental hygienist.
[Same as LE Q.3]
1 3. Define dental auxiliary and describe the functions of various dental ancillaries. [Same as LE Q.3]
14. Define auxiliary. Describe the operative auxiliary. [Same as LE Q.3]
15. Define dental auxiliary. Describe the functions of dental auxiliary. [Same as LE Q.3]
16. Define dental auxiliary. Write in detail the various operating and nonoperating auxiliaries. [Same as LE Q.3]
17. Define dental auxiliary. Classify dental auxiliary and describe the functions of expanded function dental auxil-
iary. [Same as LE Q.3]
1 8. Discuss the various ethical considerations in dental practice. [Same as LE Q.4]
19. Discuss responsibilities of dentist in professional relationship with patient and employment and supervision of
auxiliary staff. [Same as LE Q.4]

SHORT ESSAYS:
1. UCR fees.
2. Indian Dental Association or IDA.
3. Principles of ethics. [Ref LE Q.4]
4. New Zealand dental nurse. [Ref LE Q.3]
5. Dental Council of India. [Ref LE Q.5]
6. Expanded function dental auxiliaries. [Ref LE Q.3]
7. Public programmes in financing dental care. [Ref LE Q.2]
8. 90th Percentile. [Ref LE Q.2]
9. The work of WHO.
10. Delta dental plan. [Ref LE Q.2]
Section | I  Topic-Wise Solved Questions of Previous Years 629

1 1. Private fee for service. [Ref LE Q.2]


12. Capitation fees. [Ref LE Q.2]
13. Voluntary health agencies in India. [Ref LE Q.1]
14. UNICEF.
15. Ethical principles. [Same as SE Q.3]
16. Functions of Dental Council of India (DCI). [Same as SE Q.5]
17. Expanded function dental auxiliary. [Same as SE Q.6]
18. World Health Organization (WHO). [Same as SE Q.9]
19. Voluntary organization. [Same as SE Q.13]

SHORT NOTES:
1. What are various voluntary health agencies? [Ref LE Q.1]
2. Dental auxiliary. [Ref LE Q.3]
3. Fee schedules. [Ref LE Q.2]
4. Primary health care. [Ref LE Q.1]
5. Denturist.
6. Indian Dental Association. [Ref SE Q.2]
7. Consumer Protection Act and the dentist.
8. Red Cross. [Ref LE Q.1]
9. Dental ethics. [Ref LE Q.4]
10. Classification of members of Indian Dental Association.
11. Office bearers of Central Council of Indian Dental Association.
12. Bhore Committee.
13. WHO oral health unit.
14. Appropriate technology. [Ref LE Q.1]
15. Tools of dental pubic health.
16. Health guides. [Ref LE Q.1]
17. Capitation fee. [Ref LE Q.2]
18. Mention any four extended duties of dental hygienists. [Ref LE Q.3]
19. Comprehensive dental care.
20. Closed panel of payment. [Ref LE Q.2]
21. Open panel of dental practice. [Ref LE Q.2]
22. Jurisprudence.
23. Coinsurance. [Ref LE Q.2]
24. Consent.
25. International Red Cross.
26. Voluntary health organization. [Same as SN Q.1]
27. COPRA. [Same as SN Q.7]

SOLVED ANSWERS
LONG ESSAYS:
Q.1. Write in detail about the health care delivery sys- A. Public Health Sector
tems in India. It consists of the following:
(a) Primary health care
Ans.
l Primary health centres
In India, health care delivery system is represented by five l Subcentres
major sectors or agencies, as follows: (b) Hospitals/health centres
A. Public Health Sector l Community health centres
B. Private Sector l Rural hospitals
C. Indigenous Systems of Medicine l District hospitals/health centres
D. Voluntary Health Agencies l Specialist hospitals
E. National Health Programmes l Teaching hospitals
630 Quick Review Series for BDS 4th Year, Vol 1

(c) Health insurance schemes Principles of primary health care are as follows:
l Employees State Insurance i. Equitable distribution
l Central Government Health Scheme It is the first key principle in the primary health
(d) Other agencies care strategy, i.e. health services should be
l Defence services equally shared by all people; whether rich or
l Railways poor, urban or rural, all must have access to
B. Private Sector health services.
It consists of the following: ii. Community participation
(a) Private hospitals, polyclinics, nursing homes and All individuals, families and communities should
dispensaries be involved in the promotion of their own health
(b) General practitioners and clinics and welfare. This is an essential ingredient of
C. Indigenous systems of medicine primary health care.
It consists of the following: iii. Intersectoral coordination
l Ayurveda and Siddha Planning with other sectors to avoid unnecessary
l Unani and Tibbi duplication of activities is an important element
l Homeopathy of intersectoral approach. To achieve such coop-
l Unregistered practitioners eration, countries should review their administra-
D. Voluntary health agencies tive system, relocate their resources and intro-
E. National health programmes duce suitable legislation for coordination to take
place.
. Public health sector
A
(a) Primary health care
{SN Q.14}
{SN Q.4}
iv. Appropriate technology
Primary health care is defined as essential health care Appropriate technology has been defined as the
made universally accessible to individuals and accept- ‘technology that is scientifically sound, adaptable to
able to them through their full participation and at a cost local needs and acceptable to those who apply it and
the community and country can afford. those for whom it is used, and that can be maintained
by the people themselves in keeping with the prin-
The characteristics of primary health care are as follows: ciples of self-reliance with the resources the com-
i. It is an essential health care based on practical, munity and country can afford’.
scientifically sound and socially acceptable
methods and technology. Primary health care in India
ii. It should be rendered universally acceptable to Village level
individuals and the families in the community. The following schemes are in operation to implement
iii. It should be affordable. the National Health Policy at the village level:
iv. It requires joint efforts of health sector and other (a) Village Health Guide Scheme
health-related factors like education, food and (b) Training of local dais
agriculture, social welfare, animal husbandry, (c) ICDS scheme
housing and rural reconstruction.

{SN Q.4} {SN Q.16}


The elements of primary health care are as follows: (a) Village Health Guide Scheme
l Education concerning prevailing health problems This scheme was introduced on 2 October 1977 with
and the methods of preventing and controlling them the idea of securing people’s participation in the
l Promotion of food supply and proper nutrition care of their own health.
l An adequate supply of safe water and basic sanitation A village health guide is a person with an aptitude
l Maternal and child health care, including family of social service and is not a full-time government
planning functionary.
l Immunization against major infectious diseases One village health guide is present for each
l Prevention and control of locally endemic diseases village/1000 rural population. As of date, there are
l Appropriate treatment of common diseases and injuries 4.10 lakh village health guides functioning in the
l Provision of essential drugs country.
Section | I  Topic-Wise Solved Questions of Previous Years 631

They are selected based on the following criteria: l This worker is selected from the community
l They must be permanent residents of local she is expected to serve and undergoes training
community, preferably women. for 4 months in various aspects of health, nutri-
l They must be able to read and write, having tion and child development.
formal education of at least up to l Her payment is Rs 200–250 per month.

VI standard. l Her duties include health check-up, immuniza-

l They should be acceptable to all sections of the tion, supplementary nutrition, health educa-
community. tion, nonformal preschool education and sev-
l They should be able to spare at least 2–3 h eral services.
every day for community health work. Primary health centre level
l They should undergo a short training in pri- l The concept of Primary Health Centre was

mary health care at the nearest primary health given in 1946 by the Bhore Committee. In
centre for a duration of 200 h, spread over a January 1953, the Central Council of Health
period of 3 months and receive Rs 200 per had recommended the establishment of health
month as stipend. centres.
l After completion, they receive a working man- l In 1962, the Mudaliar Committee had recom-

ual and a kit of simple medicines belonging to mended that the existing primary health cen-
modern and traditional systems of medicine in tres should be strengthened and the population
vogue in that part of country to which they to be served by them to be scaled down to
belong. 40,000.
Their duties include the following: l The National Health Plan in 1983 proposed the

l They should treat simple diseases and activi- reorganization of primary health centres on the
ties in first aid. basis of population for more effective coverage,
l Care of mother and child health, including with one primary health centre for population
family planning, health education and sanita- of every 30,000 in rural areas in the plains and
tion. one for population of every 20,000 in hilly,
l They do community health work of about 2–4 tribal and backward areas.
h and get paid an honorarium of Rs 50 per Following are the functions of primary health
month and drugs worth Rs 600 per centres:
annum. l Providing medical care

i. (b) Local dais l Maternal and child health, including family

l Under the Rural Health Scheme, the local dais planning


are trained for 2 days in a week for 30 working l Safe water supply and basic sanitation

days at a primary health centre to gain knowl- l Prevention and control of locally endemic dis-

edge of the elementary concepts of maternal ease


and child health care and sterilization and ob- l Collecting and reporting of vital statistics
stetric skills. l Education about health

l Their duty on the remaining 4 days of l National health programmes

the week is to accompany health worker (fe- l Referral services

male) to the villages, preferably in the area l Training of health workers, health guides, local

where the dai stays. dais and health assistants


l During training, each dai is required to con- l Basic laboratory health services

duct at least two deliveries under the guidance The staffing pattern of primary health centre is as
of a health worker. follows:
l After successful completion of training, each One medical officer, one pharmacist, one nurse
dai is provided with a delivery kit and a cer- midwife, one female health worker, one block
tificate. extension educator, two health assistants (one
ii. (c) Anganwadi workers (‘angan’ means a court- male and one female), one VDC, one LDC, one
yard) lab technician, one driver and four class IV
l Under the Integrated Child Development Ser- workers.
vices (ICDS) scheme, there is an anganwadi Subcentre level
worker for a population of 1000. l The subcentre is the peripheral outpost of the ex-

l About 100 anganwadi workers are there in isting health delivery system in rural areas. One
each ICDS project. subcentre should be present for a population of
632 Quick Review Series for BDS 4th Year, Vol 1

every 3000 in hilly, tribal and backward areas, and Central Government Health Scheme
for a population of every 5000 in general. This scheme was introduced in 1954 in New
l The functions of a subcentre are limited to mother Delhi to provide comprehensive medical care to
and child health care, family planning and immuni- the employees of central government.
zation. One male and one female health worker The facilities under the scheme include the following:
attends each subcentre. l Outpatient care through a network of dispen-
l One health assistant supervises the work of six saries
health workers. l Supply of necessary drugs
l Six subcentres are located in each primary health l Laboratory and X-ray investigations
centre area. l Domiciliary visits
(b) Hospitals/health centres l Hospitalization facilities at government as well
Community health centre as private hospitals recognized for the purpose
l The community health centres were established l Specialist consultations
by upgrading primary health centres. l Paediatric services, including immunization
l One community health centre covers a popula- l Antenatal, natal and postnatal services
tion of 80,000–120,000 and has 30 beds and l Emergency treatment
specialists in surgery, medicine, obstetrics and l Supply of optical and dental aids at reason-
gynaecology and paediatrics with X-ray and able rates
laboratory facilities. l Family welfare services
l A new nonmedical post, called community health (d) Other health agencies
officer, was created at each health centre. Defence Medical Services
Staffing pattern of community health centre l Defence services have their own organization for
Four medical officers, seven nurse midwives, one providing medical care to defence personal un-
dresser, one compounder/pharmacist, one lab tech- der the banner ‘Armed Forces Medical Services’.
nician, one radiographer, two ward boys, one dhobi, l The services provided are integrated and
three sweepers, one mali, one chowkidar, one aya comprehensive, embracing preventive, pro-
and one peon. motive and curative services.
Rural hospitals Health care for railway employees
At present, a number of primary health centres also l The railways provide comprehensive health
have hospitals. There are plans to have an epidemio- care services through railway hospitals, health
logical wing attached to them. units and clinics.
District hospitals l The railway employees are provided health
A hospital differs from a health centre in the follow- check-up at the time of entry into service, and
ing respects: thereafter on yearly basis.
l In a hospital, services provided are mostly cu- l There are lady medical officers, health visi-
rative, while in a health centre, services are tors and midwives who look after maternal
preventive, promotive and curative. and child health and school health services.
l A hospital has no catchment area, while a l Specialist services are also available at divi-
health centre is responsible for a definite area sional hospitals.
and population. B. Private sector
l In hospital, the team consists of only the cura- l A large share of health services are provided by pri-
tive staff, while the health team in a health vate sector.
centre is a mix of medical and paramedical l General practitioners constitute 70% of medical profes-
workers. sion, and most of them tend to congregate in urban areas,
(c) Health insurance schemes providing curative services to those who are able to pay.
Employees State Insurance (ESI) Scheme C. Indigenous systems of medicine
l The ESI scheme introduced by an act of Parlia- l The practitioners of indigenous systems of medicine
ment in 1948 is a unique piece of social legisla- (Ayurveda, Siddha, Homeopathy, etc.) provide bulk
tion in India. of medical care to rural people.
l The act provides for medical care in cash and
(SE Q.13 and SN Q.1)
kind, benefits in the contingency of sickness,
maternity and employment injury and pension {D.  Voluntary Health Agencies
for dependents in case of death of worker be- l A voluntary health agency may be defined as an or-
cause of employment injury. ganization administered by an autonomous board,
Section | I  Topic-Wise Solved Questions of Previous Years 633

which holds meetings, collects funds for its support, chiefly doctors, health visitors and social workers in
from private sources, and expends money, whether with or anti-tuberculosis work, promotion of health
without paid workers, to conduct programmes primarily di- education and promotion of consultations and
rected towards furthering public health by providing health conferences.
services and health education, by advancing research and v. Bharat Sevak Samaj
legislation for health and by a combination of these activities. l It is a non-political and nonofficial organiza-

tion formed in 1952. One of the prime objec-


Functions of voluntary health agencies are as follows: tives of Bharat Sevak Samaj (BSS) is to help
(a) Supplementing the work of government agencies people to achieve health by their own actions
(b) Pioneering and efforts.
(c) Education vi. Central Social Welfare Board
(d) Demonstration l The Central Social Welfare Board is an autono-

(e) Guarding the work of government agencies mous organization under the general adminis-
(f) Advancing health legislation trative control of Ministry of Education. It was
Voluntary health agencies in India} set up by the Government of India in August
1953.
(SE Q.13 and SN Q.8)
vii. The Kasturba Memorial Funds
i. {(Indian Red Cross Society l It was created in the memory of Kasturba Gan-

The Indian Red Cross Society was established in 1920. dhi after her death in 1944. The main object of
Its activities are as follows: the fund was to improve the lot of Indian
(a) Relief work women, especially in villages, through gram
(b) Milk and medical supplies sevikas.
(c) Armed forces viii. Family Planning Association of India
l The Family Planning Association was formed
(d) Maternal and child welfare services
(e) Family planning in 1949 with its headquarters in Bombay. The
(f) Blood bank and first aid)} association has trained several hundred doc-
tors, health visitors and social workers in prop-
[SE Q.13] agating family planning in India.
ix. All India Women’s Conference
ii. {Hind Kusht Nivaran Sangh
l It was established in 1926. This is the only
l It was founded in 1950 with its headquarters in
women’s voluntary welfare organization in the
New Delhi.
country.
l Its functions included rendering of financial
x. The All-India Blind Relief Society
assistance to various leprosy homes and
l It was established in 1946 with a view to coor-
clinics, health education through publica-
dinate different institutions working for the
tions and posters, training of medical work-
blind.
ers and physiotherapists, conducting re-
xi. Professional bodies
search and field investigations, organizing
l Indian Medical Association, All India
All India Leprosy Workers Conferences and
Licentiates Association, All India Dental As-
publication of ‘Leprosy in India’, a quar-
sociation and Trained Nurses Association of
terly journal.
India are voluntary agencies qualified in their
iii. Indian Council for Child Welfare
respective specialities and possess registerable
l This council was established in 1952.
qualifications.}
l The services are devoted to secure for India’s

children the opportunities and facilities, by law E. National Health Programmes


and other means, which are necessary to enable Some of the national health programmes conducted in
them to develop physically, mentally, morally, India are as follows:
spiritually and socially in healthy and normal i. National Malaria Eradication Programme
manner and under the conditions of freedom ii. National Filaria Control Programme
and dignity. iii. National Tuberculosis Programme
iv. Tuberculosis Association of India iv. National Leprosy Eradication Programme
l It was formed in 1939. It has branches in all the v. Diarrhoeal Diseases Control Programme
states. Its activities include organizing annual vi. STD Control Programme
TB Seal campaign to raise funds, training of vii. National Programme for Control of Blindness
634 Quick Review Series for BDS 4th Year, Vol 1

viii. Iodine Deficiency Disorder Programme primarily used by middle-income population rather
ix. Universal Immunization Programme than the low income. It was primarily meant to finance
x. National Family Welfare Programme prosthetic treatment.
xi. National Water Supply and Sanitation Pro- C. Private third party prepayment plans
gramme l Third party payment for dental services is defined as

xii. National Diabetes Control Programme ‘payment for services by some agency rather than
xiii. National Mental Health Programme directly by the beneficiary of those services’.
xiv. National Cancer Control Programme l The dentist and the patient are the first and the sec-

xv. National AIDS Control Programme ond parties, and the administrator of finances is the
xvi. Child Survival and Safe Motherhood Programme. third party. The third party is also known as the car-
rier, insurer, underwriter or administrative agent.
Q.2. Describe in detail the finance in dental diseases.
Principles of insurance and dental care
Ans. To be insurable, a risk must
l Be precisely definable
[SE Q.7] l Be of sufficient magnitude that if it occurs, it

{Different methods for payment of dental health services constitutes a major loss
are as follows: l Be infrequent

A. Private fee for service l Be of unwanted nature

B. Post-payment plans l Be beyond the control of individuals

C. Private third party prepayment plans l Do not constitute a moral hazard

D. Salary All health insurance schemes violate some of these


E. Public programmes} principles because illness is unpredictable and health
care is usually a wanted service. Due to these prob-
[SE Q.11] lems, different types of payments are offered by in-
{A.  Private fee for service surance carriers:
Private fee for service, a two-party arrangement, is a (a) Deductable
traditional form of reimbursement for dental services in (b) Coinsurance
most countries. (c) Group insurance
Advantages (a) Deductible
l It is culturally acceptable. l It is also called ‘front-end payment’. It is a
l It has flexible fees and can be charged in accor- stipulated flat sum that a patient must pay
dance with market conditions. towards the cost of treatment before the
l It is administratively simple. benefits of the programme goes into effect.
l It is the only system under which some form of

dental care is likely to be provided.


Disadvantage {SN Q.23}
l Some patients cannot afford the dental care
(b) Coinsurance
offered and thus are unable to receive dental care
l It is also called as ‘copayment’. A patient pays a
if private fee for service were the only financing
percentage of the total cost of treatment.
mechanism for dental care.}
l Coinsurance is defined as an arrangement un-
B. Post-payment plans
der which carrier and beneficiary are liable for
l It was first introduced in the late 1930s by local den-
a share of the cost of dental services being
tal societies of Pennsylvania and Michigan. It is also
provided.
known as budget payment plan.
l Under this payment plan, the patient borrows money

from a bank or finance company to pay dentist fee at


the time the agreement is made to receive care. (c) Group insurance
l After the application is approved by the lending insti- l Health insurance was at first offered only

tution, the dentist is paid the entire fees, less a discount to groups. This was because illness is rea-
charge. The patient then repays the loan (with interest sonably predictable in groups. The proba-
of course) to the bank in budgeted amounts. bility of adverse selection was reduced by
l It was developed with the intention of providing den- the use of waiting periods after enrolment
tal care to large segments of population, and it was before any benefits become available.
Section | I  Topic-Wise Solved Questions of Previous Years 635

Reimbursement of dentists in prepayment plans (c) Prepaid group practice, including health maintenance
The preferred method for reimbursement for dentist in organization and independent practice associations
prepayment plans is of three types: (d) Capitation plans
l Usual

l Customary (a) Commercial insurance companies


l Reasonable fee (UCR) l The fundamental difference between commercial in-

Usual fee: The fee usually charged for a given ser- surance carriers and others is that commercial insur-
vice by an individual dentist to private patients is his ance carriers operate for profit.
or her own usual fee. l Commercial insurance companies have become

Customary fee: A fee is customary when it is in the competitive through a variety of other mechanisms.
range of usual fee charged by a dentist of similar l They can be more selective about the group to

training and experience for the same service within which it chooses to offer dental insurance.
the specific and limited geographic area. l They claim no obligation towards the dental

Reasonable fee: A fee is reasonable if it meets cer- health of the community.


tain stipulated criteria. l They do not conduct fee audits and post-treat-

Table of allowances ment dental examinations to assess the quality of


l A table of allowances is defined as a list of covered care delivered.
services assigned to each service, a sum that rep- Disadvantages
resents the total obligation of a plan with respect l They do not encourage utilization of professional

to payment for such service but that does not nec- services.
essarily represent a dentist’s full fee for that ser- l In order to allow the profit margin, they need to

vice. For example, if a third-party plan permits a charge higher premiums.


fee of Rs 80 for a particular service for which the (b) Nonprofit health service corporation
dentist’s usual fee is Rs 100, the dentist will carry
out the service, pick up the Rs 80 from the carrier [SE Q.10]
and may charge the patient Rs 20 to make up the {1.  Delta Dental Plans
usual fee. In June 1954, the Seattle District Dental Soci-
ety in Washington State was approached with a
request for a comprehensive dental care pro-
gramme for children up to 14 years of age.
{SN Q.17} A dental service corporation is a legally consti-
Capitation tuted nonprofit organization incorporated on a
A capitation fee is defined as a fixed monthly or yearly state-by-state basis and sponsored by a constituent
payment paid by a carrier to a dentist in a closed panel, dental society to negotiate and administer con-
based on the number of patients assigned to the dentist tracts for dental care.
for treatment. Accordingly, the National Association of Den-
The money is paid regardless of whether the patient tal Service Plans (NADSP) was formed in June
participates in the plan, receives no care, a little care or 1966. In April 1969, the NADSP changed its
great deal of care. name to Delta Dental Plans Association. The
majority of board directors in most of Delta
{SN Q.3} plans are dentists, who are elected by a state-
Fee schedules approved dental society. Other board members
A fee schedule is defined as maximum dollar allowance represent the worlds of finance, insurance, la-
for dental procedures that apply under a specific contract. bour and consumer groups.}
Characteristics of a dental service corporation
as per Mitchelle and Hoggard are as follows:
1. Professional sponsorship
Private third party prepayment plan can be divided 2. Nonprofit operation
into the following: 3. Participation permitted by all licenced
(a) Commercial insurance companies dentists with the state
( b) Nonprofit health service corporations such as 4. Benefits provided on service basis
Delta Dental Plans and Blue Cross Blue 5. Freedom of choice allowed for both pa-
Shield tients and dentists
636 Quick Review Series for BDS 4th Year, Vol 1

[SE Q.10] median fee, which is equivalent to 50th percentile,


would be Rs 90. The key point to note is that if
{Reimbursement of dentists in Delta plans payment is made at the 90th percentile (Rs 120),
Delta plans use the UCR concept almost exclusively.
90% of the participating dentists receive their full
Reimbursement of dentists under the fee for service
fee for the service and only 10% of the participat-
programme depends on whether a dentist is partici-
ing dentists would be less than their usual fee. In a
pating (par dentist) or nonparticipating (non-par den-
similar way, nonparticipating dentists are paid 50th
tist) with Delta.
percentile.}
A participating dentist has a contractual agree-
2. Blue Cross Blue Shield
ment with Delta plan to render care to covered
The health service corporations, of which Blue Cross
subscribers.
Blue Shield is the most important corporation, have
Delta plans encourage all dentists to participate who
for years offered limited dental coverage as a part of
do agree to the following conditions:
hospital/surgical/medical policies. They have ad-
1. Pre-filing of their usual and customary fees
opted many of the cost control features pioneered by
2. Acceptance of payment for their services at
Delta plans.
the 90th percentile of fees as payment in
(c) Prepaid group practice
full, which means they will not assess the
It is defined by ADA as ‘Group practice is that type
patient for further charges
of dental practice in which dentists, sometimes in
3. Fee audits by auditors from Delta
association with the members of other health profes-
4. Post-treatment inspection of randomly cho-
sions, agree formally between themselves on certain
sen patients
central arrangements designed to provide efficient
5. Withholding of a small amount of each
dental health service’. Net income in a group prac-
payment by Delta, usually to build up in-
tice can be divided equally or prorated according to
surance reserves}
patient load, years of service, speciality, etc., whereas
Nonparticipating dentists can also treat patients
majority of patients who receive care through group
covered under Delta plans and are reimbursed by
practices do so on the basis of usual private patient
Delta. They do not need to prefile their fees and
fee for services.
are not subject to fee audits; however, they are
Advantages
paid at a lower percentile than the 90th, often at
1. Better ways of organizing one’s life vacations,
the median, or the 50th percentile.
and continuing education leaves can be
planned more readily.
[SE Q.8]
2. Less disruption in practice caused by illness of
{90th Percentile dentist.
The percentiles of a set of data divide the total 3. Improved quality of care.
frequency into hundredths so that the 90th percen- 4. Financial fringe benefits.
tile is the value below which 90% of the observa- 5. Economies of scale through the sharing of
tions exist. equipments, personnel and other resources.
When the payment is made at the 90th percentile, 90% Disadvantages

of the participating dentists will receive their full fee 1. Personality – It is essential that dentist in a
for the service, and only 10% will be paid at less than group be temperamentally compatible.
their usual fee. This method helps to control payment 2. Dentists, unlike physicians, are taught to work
at the end of the scale while paying the majority their independently and this desire for independence
full fee. is one reason suggested for the slow growth of
For example, suppose in a given area there are dental group practice.
100 participating dentists who have filed their fee Health-Maintenance Organization (HMO)
for a particular service. The fee filed will vary An HMO is defined as ‘a legal entity, which pro-
from one dentist to another. In this instance, the vides a prescribed range of health services to each
fees charged vary from Rs 50 to Rs 150. individual who has enrolled in the organization in
Ten dentists charge Rs 50, 20 of them charge Rs return for a prepaid, fixed and uniform payment’.
60, 30 of them charge Rs 70 or less, and so on. HMO’s five essential elements:
There are a few dentists who charge considerably 1. A managing organization
higher for the service. The 95th percentile is 2. A delivery system
Rs 140 and the 100th percentile is at Rs 150. The 3. An enrolled population
Section | I  Topic-Wise Solved Questions of Previous Years 637

4. A benefit package Disadvantage


5. A system of financing and prepayment 1. Lack of financial incentives.
Dental personnel in HMO
1. Staff model: Dentists, dental hygienists and [SE Q.7]
dental assistants are salaried employees of
HMO. {E. Public programmes
2. Group model: HMO contracts directly with ( a) Medicare
a group practice, partnership or corporation (b) Medicaid
for providing dental services. (a) Medicare
3. Direct contract model: HMO contracts di- l Title XVIII of the Social Security Amendments
rectly with the individual provider for pro- of 1965 is the programme known as Medicare.
vision of services. l This programme removed all financial barriers
Independent practice association for hospital and physician services for all per-
It is an association of independent dentists that sons over the age of 65 years, regardless of their
develops its own management and fiscal struc- financial means.
ture for the treatment of patients enrolled in an l The expenditure of the programme was consider-
HMO. ably higher than estimated in the first few years of
(d) Capitation plans operation. By the mid-1970s, it had two parts:
Part A: Hospital insurance
[SE Q.12]
Part B: Supplemental medical insurance
{A capitation fee is defined as a fixed monthly or l Both parts contain a highly complex series of

yearly payment paid by a carrier to a dentist in a service benefits and require some payment by
closed panel, based on the number of patients as- the individual.
signed to the dentist for treatment. l Medicare addresses the problems of old age, which

The money is paid regardless of whether the patients have high health care needs and high income. It
utilize care or not.} was brought into action because voluntary health
insurance system was unable to provide adequate
(SE Q.12 and SN Q.20) coverage above the age of 65 years.
{(Closed panel (b) Medicaid
l Closed panel practice exists when a group of l Title XIX of the Social Security Amendments
dentists who share office facilities provide of 1965 established the programme known as
stipulated services to an eligible group for a set Medicaid. The original intent of the programme
premium. was to provide funds to meet the health care
l For beneficiaries of plans using closed panels, needs of all indigent and medically indigent
choice of dentist is limited to panel members, persons.
and dentist must accept any beneficiary as a l It provides medical and health-related services to
patient.)} the poorest people of the USA under the cate-
(SE Q.12 and SN Q.21) gory of parents and children, the disabled and the
elderly.
{(Open panel Inpatient hospital care
Any licenced dentist can participate, the benefi- The following services can be availed, pro-
ciary has a choice from all licenced dentists, and vided they are considered to be medically
the dentist may accept or refuse any beneficiary.)} necessary:
D. Salary l Outpatient hospital care

Dentists in some group practices, especially closed l Physician services

panel clinics, and those in the armed forces and those l Nursing home care

employed by public agencies, are salaried. l Nurse midwife and nurse practitioner
Advantages services
1. Dentist will be largely free from business con- l Laboratory and X-ray services

cerns of running a practice, and this allows the l Early and periodic screening, diagnostic

dentist to concentrate on clinical matters. and treatment services for individuals


2. Fringe benefits are also often attractive. under the age of 21 years
638 Quick Review Series for BDS 4th Year, Vol 1

l Federally qualified health centre (FQHC) and The duties of dental surgery assistants are as follows:
rural health clinic (RHC) service (a) Reception of patient
l Home health services for individuals aged (b) To prepare patient for any treatment he or she
21 years or older may need
According to the Federal law, the amount, duration (c) To provide all necessary facilities such as
and scope of each service provided must be suffi- mouthwash and napkins
cient to reasonably achieve its purpose.} (d) Sterilization care and preparation of instruments
(e) Preparation and mixing of restorative materi-
Q.3. Define dental auxiliaries. Write in detail about als, including both fillings and impression
various dental auxiliaries. materials
Ans. (f) Care of patient after treatment until he or she
leaves, including clearing away of instruments
and preparation of instruments for reuse
(g) Preparation of surgery for next patient
{SN Q.2}
(h) Presentation of documents to dental surgeon
A dental auxiliary or ancillary is defined as a person for his or her completion and filing of the same
who is given responsibility by a dentist so that he or she (i) Assistance with X-ray work, and the process-
can help the dentist render dental care but is not qualified ing and mounting of X-rays
with a dental degree. (j) To instruct patient, where necessary, for the
correct use of toothbrush
Classification
(k) After-care of patients who have had general
According to World Health Organization (WHO), they
anaesthesia
are classified as follows:
b. Dental secretary/receptionist
i. Nonoperating auxiliary
This is a person who assists the dentist with secre-
(a) Clinical: This is a person who assists profes-
tarial work and patient reception duties.
sional in clinical work but does not carry out any
c. Dental laboratory technician
independent procedures in oral cavity.
l A nonoperating auxiliary who fulfils the prescriptions
(b) Laboratory: This is a person who assists profes-
provided by dentists regarding extra oral con-
sional by carrying out certain technical labora-
struction and repair of oral appliances and bridge-
tory procedures.
work. They are also known as dental mechanics.
ii. Operating auxiliary
l The functions of dental technician, in addition to
This is a person who, not being a professional, is
the casting of models from impression made by
permitted to carry out certain treatment procedures in
dentists, include the fabrication of dentures,
the mouth under the direction and supervision of a
splints, orthodontic appliances, inlays, crowns
professional.
and special trays.
l ‘Denturist’ is a term applied to those dental lab

Revised classification technicians who are permitted in some states in


i. Nonoperating auxiliaries are classified as follows: the USA and elsewhere to fabricate dentures di-
l Dental surgery assistant
rectly for patients without a dentist’s prescription.
l Dental secretary/receptionist
They may be licenced and registered. Their craft
l Dental laboratory technician
is called ‘denturism’.
l Several countries have allowed laboratory techni-
l Dental health educator

ii. Operating auxiliaries are classified as follows: cians to work directly with the public.
l The ADA has vigorously opposed the denturists’
l School dental nurse

l Dental therapist
movement at political level, as denturists are un-
l Dental hygienist
qualified to treat patients, and the poor quality of
l Expanded function dental ancillaries
care and even actual harm could be the result.
d. Dental health educator
i. Nonoperating auxiliaries l This is a person who instructs in the prevention of
a. Dental surgery assistant dental disease and may also be permitted to apply
This was started in the USA more than a century ago in preventive agents intraorally.
1885 when Dr C. Edmund Kells of New Orleans em- l In a few countries, the duties of some dental sur-
ployed a woman as a ‘lady in attendance’ so that unac- gery assistants have been extended to carry out
companied female patients could come to his clinic. certain preventive procedures.
Section | I  Topic-Wise Solved Questions of Previous Years 639

ii. Operating auxiliaries Their training includes the following:


l Clinical caries diagnosis
[SE Q.4] l Technique of cavity preparation in deciduous and

{a.  School dental nurse permanent teeth


l School dental nurse is a person who is permitted to l Material handling and restorative skills

diagnose dental disease and to plan and carry out l Vital pulpotomies under rubber dam in deciduous

certain specified preventive and treatment mea- teeth and extraction of deciduous teeth under local
sures, including some operative procedures in the anaesthesia
treatment of dental caries and periodontal disease in l They have minimal training in interpretation of

defined groups of people, usually schoolchildren. X-rays


l Due to severe dental diseases found in army re- c. Dental hygienist
cruits during World War I (1914–1918), in 1921, l As per the Indian Dentist Act of 1948, a dental

the Dental Nurse Scheme was established in hygienist means a person, not being a dentist or
Wellington, New Zealand. medical practitioner, scales, cleans or polishes
l The time period of training is 2 years. teeth, or gives instructions in dental hygiene.
l After completion of training, school dental nurse l A dental hygienist is an operating auxiliary li-

is assigned a school where she is employed by cenced and registered to practice dental hygiene
government to provide regular dental care to 450 under the laws of appropriate state, province, terri-
to 700 children. School dental nurse is accepted tory or nation. Dental hygienists work under the
as a member of school. supervision of dentists.
l They are expected to provide care to children at The usual functions of dental hygienists are as
nearly 6-month intervals. They are under general follows:
supervision of a district principal dental officer. l Cleaning of mouths and teeth with particular at-

Duties of school dental nurses as listed by the tention to calculus and stains
New Zealand Department of Health Division l Topical application of fluorides, sealants and

includes the following: other prophylactic solutions


l Oral examination l Screening or preliminary examination of patients

l Prophylaxis as individuals or in groups (schoolchildren or in-


l Topical fluoride application dustrial employees) so that they may be referred
l Advice on dietary fluoride supplements to dentists for treatment
l Administration of local anaesthesia l Instruction in oral hygiene

l Cavity preparation and placement of amalgam l Resource work in the field of dental health

filling in primary and permanent teeth


l Pulp capping
[SE Q.6]
l Extraction of primary teeth

l Individual patient instruction in tooth brushing {d.  Expanded function dental auxiliary
and oral hygiene l They have been referred to as expanded func-
l Classroom and parent–teacher dental health tion dental assistants, expanded function
education dental hygienist, expanded function auxil-
l Referral of patient to private practitioners for iary, technotherapist or expanded duty dental
more complex services, such as extraction of auxiliary.
permanent teeth, restoration of fractured perma- l EFDA is a dental assistant, or a dental hygienist
nent incisors and orthodontic treatment} in some cases, who has received further training
b. Dental therapist in the duties related to the direct treatment of
l This is a person who is permitted to carry out to patients, although still working under direct su-
the prescription of a supervising dentist, certain pervision of a dentist.
specified preventive and treatment measures, Duties of dental assistant in extended function
including the preparation of cavities and restora- l Retraction of gingiva
tion of teeth. l Taking impressions for cast restoration, space
l They are likened to New Zealand-type school maintainers and orthodontic appliances
dental nurse but are not permitted to diagnose l Etching of teeth
and plan dental care. The training of dental thera- l Determining root length and fitting of trial end-
pists is for about a period of 2 years and involves odontic filling points
both reversible and irreversible procedures. l Applying pit and fissure sealants}
640 Quick Review Series for BDS 4th Year, Vol 1

(SE Q.6 and SN Q.18) the closer the better, particularly in the begin-
ning. The formal training extends from 4 to
{(Duties of dental hygienist in extended function 6 months, followed by a period of field training
Retraction of gingiva
under direct and constant supervision.}
l

l Taking impressions for cast restoration, space

maintainers and orthodontic appliances Q.4. Discuss in detail the principles of ethics to be followed
l Temporary stabilization procedures by a dentist. Add a note on unethical practices by dentists.
l Debridement of the periodontal surgical site
Ans.
l Suture placement)}

(SE Q.3 and SN Q.9)


[SE Q.6]
{Frontier auxiliaries {(Ethics is defined as the part of philosophy that deals
l In developed countries, dentists are confined in with moral conduct and judgement.
urban centres, and the areas that are too distant
from public or private dental offices have a de- The major principles of ethics are as follows:
mand for persons providing comprehensive care i. To do no harm (nonmaleficence)
or emergency pain relief. ii. To do good (beneficence)
l Frontier auxiliaries include capable common peo-
iii. Autonomy
ple, in particular nurses and former dental assis- iv. Justice
tants, with minimum training. v. Truthfulness
Their functions include: vi. Confidentiality)}
l Dental prophylaxis

l Dental health education [SE Q.3]


l Providing relief from pain
{i. To do no harm (nonmaleficence)
l Referral, and fluoride rinse programme
l This principle is attributed to Hippocrates and is
l Simple denture repair
considered to be the foundation of social morality.
In 1958, WHO introduced two new types of auxiliaries:
l As per this principle, the first and foremost duty of a
i. Dental licentiate
health care professional is to ensure that his or her
ii. Dental aide
actions do not harm the patient in any way.
i. Dental licentiate l Use of unsterilized instruments, underfilling or over-

Dental licentiate is a semi-independent operator filling and carelessness in handling hard and soft
trained for 2 years. Their dental knowledge and skills tissues of the mouth are some of the instances that
are equivalent to those of New Zealand dental nurses. can harm the patient. So these should be avoided by
Their functions include careful thought and implemented proper care.
l Dental prophylaxis ii. To do good (beneficence)
l Cavity preparation l This principle is also attributed to Hippocrates.

l Fillings of primary and permanent teeth l The health care professional should place the interest

l Extraction under local anaesthesia of patient above his or her own interest.
l Drainage of dental abscesses l He or she has to plan a treatment or order an investi-

l Treatment of prevalent diseases of support- gation only if necessary.


ing tissues l It should be the role of dentists and dental hygienists

l Early recognition of serious conditions to benefit patients, and not to cause any harm.
They are responsible to the chief of regional or l The patient expects that the care provider will initiate

local health services. Their service would proba- beneficial action, and that there is an agreement between
bly occur in rural or frontier areas, and so supervi- the doctor and the patient that some good will occur.
sion and control would probably be remote. l At a community level, one has to balance harms and

ii. Dental aide benefits of programmes, and select the one that pro-
l Among native populations, the dental aides vides the greatest balance of good or evil.
provide elementary first aid procedures for the l One should make use of cost–benefit analysis to de-

relief of pain, including extraction of teeth termine appropriate programme for the community.
under local anaesthesia, control of haemor- iii. Autonomy
rhage and recognition of dental disease. l This principle dictates that health care professionals

l They would operate only within a salaried respect the patients’ right to make decisions concern-
health organization and be under supervision, ing the treatment plan.
Section | I  Topic-Wise Solved Questions of Previous Years 641

l Patients should not be bystanders in their treatment on the skill and prompt attention shown by him or
but be active participants. her, always remembering that his or her personal
l The health care professional has to ensure that con- reputation, professional ability and fidelity remain
sent is obtained before any care is instituted. his or her best recommendation.
l Patients should be informed about different modes of l They should be mindful of the high character of

treatment, their risks and benefits, consequences of their mission and the responsibilities they hold in
not availing the treatment, etc. the discharge of their professional duties.
l Information given should be easily understood, l The welfare of the patient should be conserved to

facilitating the patient to make a voluntary consent. the utmost of practitioner’s ability.
l Informed consent, both a legal and an ethical concept, l They should be courteous, sympathetic, friendly

is an essential component of a patient’s right to au- and helpful to and always ready to respond to the
tonomy. The following elements should be present: call of their patients, and should be polite and
l Disclosure of appropriate information dignified.
l Comprehension of information by the patient l They should be punctual and fulfil their appoint-

l Voluntary consent ments.


l Competence to consent l They should adhere to the remuneration for pro-

iv. Justice fessional services with as much uniformity as the


l It is described as fairness or equal treatment, giving varying circumstances may admit.
to each his or her right or due. l They should not make any discrimination among

l It directs health care professionals to provide care to all. religion, nationality, race, caste and breed, party
l The dentists have responsibilities to treat children or politics or social standing.
mentally retarded, and they cannot shy away from l They should keep all the personal information

the responsibilities bestowed on them by the society. about patients that they come to know directly or
l When the resources are limited, especially at the indirectly in the course of professional practice in
community level, one has to choose a group of pa- utmost confidence. The dentists should also see
tients eligible for treatment as well as type of care. that their auxiliary staff has observed this rule.
l Whether it is possible to give the same or equal treat- Duties of dentists towards their colleagues
ment for all, equitable treatment for the group con- l They should cherish a proper pride in their col-

cerned is to be considered. leagues and shall not disparage them by actions,


v. Truthfulness deeds or words.
l Patient–doctor relationship is based on mutual trust. l They should not contemplate or do anything harm-

l Patient expects the dentist to be truthful about the in- ful to the interest of the members of the fraternity.
formation given, treatment rendered and the prognosis. l They should honour mutual arrangements made

l Lying or concealing or manipulation of the informa- regarding remuneration.


tion fails to show respect for persons and their l They should retire in favour of regular dentist

autonomy, violates explicit agreements and threatens after the emergency is over.
relationships based on trust. l If a dentist is consulted by the patient of another

vi. Confidentiality dentist and the former finds out that a patient is
l This principle figures in the Hippocratic oath and suffering due to some faults from previous treat-
also in the International Code of Medical Ethics. ment, it is his or her duty to institute correct treat-
l Each and every patient has the right to expect that all ment at once with the least comment.
communications and records pertaining to their care l They should regard it as a pleasure and privilege

will be treated as confidential. to render gratuitous service to another dentist, his


l It is very natural to gossip about a patient, particu- or her spouse and family members.
larly if it is someone famous or possibly a neighbour, Unethical practice
but to do so would break a bond of trust between the The following practices are considered unethical:
dental professional and the patient. l If a dentist employs in his or her professional

l Confidentiality must be maintained at all times. practice any professional assistant who is not a
The Dental Council of India makes the following regula- registered dental hygienist, or a dental me-
tions for laying down standards of professional conduct.} chanic whose name is not registered in the
Duties and obligations of dentists towards patients state dentists register.
and public l Styling by any dentist or a group of dentists his

l Each and every dentist should remember that care or her/their ‘dental clinic’ or chambers by the
of the patient and treatment of the disease depend name of dental hospitals.
642 Quick Review Series for BDS 4th Year, Vol 1

l Any contravention of drugs. explain and, after giving him or her reasonable
l Signing under his or her name and authority opportunity of being heard and after making such
any certificate that is untrue, misleading or enquires, determine the action to be taken against
improper, or giving false certificates or testi- the dentist under Section 41 of the Act.
monials directly (or) indirectly concerning the Legal vulnerability in dental practice
supposed virtues of secret therapeutic agents Legal vulnerability in dental practice may be divided
or medicines. into two broad categories.
l Immorally involving abuse of professional (a) Criminal
relationship. (b) Civil
l Aiding in any kind of illegal practice.

l Promise of radical cure by the employment of (a) Criminal


secret methods of treatment. i. Violations of statutory law are termed crimes.
l Advertising, whether directly or indirectly, for They constitute acts that are deemed by the
the purpose of obtaining patients for promot- government to be against public interest.
ing his or her professional advantage. The penalties include the following:
l Employing any agent or canvasser for the pur- l Loss or suspension of licence

pose of obtaining patients. l Mandatory psychiatric counselling

l Using of signboards larger than 0.9 metric by l Drug rehabilitation

0.6 metric and the use of such words as ‘Teeth, l Mandatory continuing education

and Painless Extraction’, other than his or her l Fines

name and qualifications as defined under Clause l Jail term

(i) of Section 2 of the Act. ii. Violations of administrative laws (State Board,
l Insertion of any paragraphs and notice in the State Education Department) are termed quasi
press. crimes. The penalties include all actions under
l Allowing the dentist’s name to be used to des- crimes except the jail term.
ignate commercial articles such as toothpaste, (b) Civil
toothbrush and tooth powder. Civil vulnerability includes TORT and Contract.
l Mentioning after dentist’s name any other Contract
abbreviation except those indicating dental l A contract is defined as ‘an agreement

qualifications, such as: between parties’ and can be either ver-


RDP – Registered Dental Practitioner bal or written.
MIDA – Member Indian Dental Association l In law, both are equally binding, but as

FICD – Fellow of International College of the parties may have differing recollec-
Dentists tions of what was said, the advantages
MICD – Master of International College of of a written agreement are apparent.
Dentists Doctor–patient contract
FACD – Fellow of American College of A written treatment plan and charge esti-
Dentists mate, which is signed by the patient, puts
MRSH – Member of Royal Society of Hygiene the legal foundation of doctor–patient rela-
Announcement for the change of address: tionship.
i. A notice for the change of address shall be in- Duties owed by the doctor
timate to the concerned State Dental Council. l Use reasonable care in the provision of services as

ii. A dentist may issue a formal announcement in measured against acceptable standards set by
the press, one insertion per paper, regarding other practitioners with similar training in a simi-
the following: lar community
l On starting practice l Be properly licenced and registered and meet all

l On change of type of practice other legal requirements to engage in the practice


l On changing address of dentistry
l On temporary absence from duty l Employ competent personnel and provide for

l On resumption to practice their proper supervision


Action of unethical conduct: l Maintain a level of knowledge in keeping with

l When complaint or information is received by current advances in the profession


the State Dental Council, the concerned State l Obtain informed consent from the patient before

Dental Council may call upon the dentist to instituting an examination or treatment
Section | I  Topic-Wise Solved Questions of Previous Years 643

Charge a reasonable fee for services based on


l 8. Staff, remuneration and allowances
community standards 9. The executive committee
l Keep the patient informed of his or her progress 10. Recognition of dental qualification
l Keep accurate records of the treatment ren- 11. Qualification of dental hygienist
dered to the patient 12. Qualification of dental mechanics
l Maintain confidentiality of information 13. Effect of recognition
l Make appropriate referrals, and request neces- 14. Power to acquire information as to courses of
sary consultations study and training and examination
l Comply with all laws regulating the practice of 15. Inspections
dentistry 15a. Appointment of visitors
Duties owed by the patient 16. Withdrawal of recognition
In accepting the care, the patient should 16a. Withdrawal of recognition of recognized dental
l Follow home care instructions qualification
l Stick to appointments 17. Mode of declarations
l Cooperate in case 17a. Professional conduct
l Consent 18. The Indian register
The general principle is that a doctor who treats a 19. Information to be furnished
patient without the patient’s consent is guilty of 20. Power to make regulations
unauthorized touching, for which the doctor can Chapter III: State Dental Council
be held liable to the patient in damage. Treatment 21. Constitution and composition of State council
without consent is assault. 22. Inter-State agreements
Therefore, to be valid, consent should be informed. 23. Composition of joint State councils
l Description of the proposed treatment 24. Incorporation of State councils
l Material or foreseeable risks 25. President and vice president of State council
l Benefits and prognosis of proposed treatment 26. Mode of elections
l Alternatives to proposed treatment 27. Term of office and casual vacancies
l Risks, benefits and prognosis of alternative 28. Staff, remuneration and allowances
treatments 29. Executive committee
All these factors must be described to the patient 30. Information to be furnished
in language the patient understands, and the pa- Chapter IV
tient must be given an opportunity to ask ques- 31. Preparation and maintenance of register
tions and have questions answered. 32. First preparation of register
33. Qualification for entry on first preparation of register
Q.5. Enumerate the different chapters of Dentists Act
34. Qualification for subsequent registration
of India. Write composition and functions of Dental
35. Scrutiny of applications for registration
Council of India.
35a. Special provision for amending the register of
Ans. dentists
36. Register of dental hygienist and dental mechanics
The Dentists Act 1948 (Act 16 of 1948) is an act to regulate
37. Qualification for registration as a dental hygienist
the profession of dentistry. It was introduced on 29 March
38. Qualification for registration as a dental mechanic
1948. The Act contains the following chapters with their
39. Renewal fees
subsections as contents.
40. Entry of additional qualifications
Dentists Act
41. Removal from register
Chapter I: Introductory
42. Restoration to register
1. Short title and extent
43. Bar of jurisdiction
2. Interpretation
44. Issue of duplicate certificate
2a. Construction of reference to laws not in force in
45. Printing of register
Jammu and Kashmir
46. Effect of registration
Chapter II: Dental Council of India
46a. Transfer of registration
3. Constitution and composition of council
Chapter V: Miscellaneous
4. Incorporation of council
47. Penalty for falsely claiming to be registered
5. Mode of elections
48. Misuse of titles
6. Term of office and casual vacancies
49. Practice by unregistered persons
7. President and vice president of council
50. Failure to surrender certification of registration
644 Quick Review Series for BDS 4th Year, Vol 1

5 1. Companies not to engage in dentistry l The council has formed some rules and regulations
52. Cognisance of offences to serve the above functions in which the following
53. Payment of part of fees to the council are specified.
53a. Accounts and audit l Basic principles for the maintenance of minimum

54. Appointment of commission of enquiry education standard for the BDS degree.
55. Power to make rules l Minimum physical requirements of dental college.

l Minimum staff pattern for undergraduate dental


[SE Q.5] studies in colleges with 40, 60 and 100 admissions.
{Composition of the Dental Council of India l Basic qualifications and teaching experience required

to teach BDS and MDS students.


The Dental Council of India was formed on 12 April
l
l General establishment of dental facilities, its dura-
1949. As per the Dentists Act 1948, the representa-
tives of following groups contribute to the composi- tion of course and selection of students.
l Migration and transfer rules for students.
tion of the Dental Council of India.
l Regulations of scheme of exam for BDS and MDS.
l One registered dentist possessing a recognized dental
l Dental curriculum: time and subject specifications for
qualification elected by the dentists registered in Part
A of each State register clinical programmes, field programme, syllabus, etc.}
l One member elected from amongst themselves by Q.6. Write in detail various modes of payment for den-
the members of the Medical Council of India tal care.
l Not more than four members elected from among

themselves, by principals, deans, directors and Ans.


vice principals of dental colleges in the States [Same as LE Q.2]
training students for recognized dental qualifica-
tions, provided that not more than one member Q.7. Describe the method of payment for dental care.
shall be elected from the same dental college; and Ans.
heads of dental wings of medical colleges in the
States training students for recognized dental [Same as LE Q.2]
qualifications Q.8. Describe payment plans for dental care.
l One member from each university established by

law in the State, which grants as recognized dental Ans.


qualification, to be elected by the members of the [Same as LE Q.2]
senate of the university, or in case the university has
no senate, by the members of the court, from Q.9. Discuss in detail the various mechanisms of pay-
amongst the members of the dental faculty of the ment of dental care.
university or in case the university has no dental
Ans.
faculty, from amongst the members of the medical
faculty thereof [Same as LE Q.2]
l One member to represent each State, nominated by
Q.10. What are the various methods by which payment
the government of each such State from among per-
for dental health services are carried out?
sons registered either in a medical register or a dental
register of the State Ans.
l Six members nominated by the Central government,

of whom at least one shall be a registered dentist pos- [Same as LE Q.2]


sessing a recognized dental qualification and practic- Q.11. What is meant by Dental Service Corporation?
ing or holding an appointment in an institution for Describe the various dental prepayment plans.
the training of dentists in a union territory, and at
least two shall be dentists registered in the Part B of Ans.
State register [Same as LE Q.2]
l The Director General of Health Services, ex officio

Functions of Dental Council of India Q.12. Define and classify dental auxiliaries. Write
l The Dental Council of India is mainly concerned
about the functions of a school dental nurse and a dental
with the maintenance of standard of dental educa- hygienist.
tion, and further it is the duty of the council to regis- Ans.
ter qualified dentist and eliminate quacks from the
field. [Same as LE Q.3]
Section | I  Topic-Wise Solved Questions of Previous Years 645

Q.13. Define dental auxiliary and describe the functions experience for the same service within the specific
of various dental ancillaries. and limited geographic area.
l The fee level is determined by the administrator of a
Ans.
dental benefit plan from actual submitted fees for a spe-
[Same as LE Q.3] cific dental procedure to establish the maximum benefit
payable under a given plan for a given procedure.
Q.14. Define auxiliary. Describe the operative auxiliary.
Reasonable Fee
Ans. l A fee is reasonable if it meets the above two

criteria.
[Same as LE Q.3]
l The fee charged by a dentist for a specific dental

Q.15. procedure that has been modified by the nature and


severity of the condition being medical and dental
Define dental auxiliary. Describe the functions of dental
complications or unusual circumstances, and there-
auxiliary.
fore may differ from the dentist’s usual fee or the
Ans. benefit administrator’s customary fee.
[Same as LE Q.3] Q.2. Indian Dental Association or IDA.
Q.16. Define dental auxiliary. Write in detail the various Ans.
operating and nonoperating auxiliaries.
Ans. {SN Q.6}
[Same as LE Q.3] Indian Dental Association
Q.17. Define dental auxiliary. Classify dental auxiliary and The Indian Dental Association was formed in the year
describe the functions of expanded function dental auxiliary. 1949 soon after the Dentists Act 1948 was passed by
Indian Parliament, before which it was known as All
Ans. India Dental Association. The association was registered
[Same as LE Q.3] in Delhi in 1967 with register number S/26S.

Q.18. Discuss the various ethical considerations in den- Composition


tal practice. l The Central Council

l State branches
Ans. l Local branches

[Same as LE Q.4] l Defence branches

Q.19. Discuss responsibilities of dentist in professional


relationship with patient and employment and supervi-
Official Relations
sion of auxiliary staff.
l World Dental Federation

Ans. l Commonwealth Dental Association

l Asia Pacific Dental Association


[Same as LE Q.4]
l International Association of Dental Research

l Society for Research on Nicotine and Tobacco

SHORT ESSAYS: l Registered head office

l Registered health office shall be where Honourable

Q.1. UCR fees. Secretary General resides or practices


Objectives
Ans.
The main objectives of the association are as follows:
The preferred method of reimbursement for dentist in pre- l To promote and encourage advancement of dental

payment plans is usual, customary and reasonable fee (UCR). and allied sciences
Usual fee l Encouragement of members to undertake mea-

The fee usually charged for a given service by an individ- sures for improvement of public health and educa-
ual dentist to private patients is his or her usual fee. tion in India
Customary fee l To maintain the honour and dignity and uphold

l A fee is customary when it is in the range of the interests of dental profession and cooperation be-
usual fee charged by dentist of similar training and tween the members thereof
646 Quick Review Series for BDS 4th Year, Vol 1

Functions World Health Organization (WHO)


l Conducting periodical meetings and conferences of It is a specialized, largest, non-political, most prominent,
the members of association and that of dental profes- self-governing, influential and multilateral health agency
sion in general of United Nations with headquarters in Geneva, Switzer-
l Publishing and circulating journals land. The constitution came into force on 7 April 1948,
l Opening of libraries should be encouraged which is celebrated every year as ‘World Health Day’.
l Publishing the papers from time to time related to Membership
dental researches Membership of WHO is opened to all countries. Each
l To encourage research in dental and allied sciences State contributes yearly to the budget, and each is entitled
with grants out of funds of association by the estab- to the services and aids that the organization provides.
lishment of scholar and maintenance of international Structure
contacts with dental associations a. The World Health Assembly
l To encourage educational campaign in matter of oral b. The Executive Board
hygiene c. The Secretariat
l To consider and express its views or questions per- Work of WHO
taining to the Indian legislation affecting public i. Prevention and control of specific diseases
health, the dental profession and dental education; to ii. Development of comprehensive health services
take such step time to time regarding the same as iii. Family health
shall deemed expedient or necessary iv. Environmental health
l To protect public from unethical treatment by un- v. Health statistics
qualified practitioners vi. Biomedical research
l Trying to set exemption from custom duty essential vii. Health literature and information
dental materials and instruments viii. Cooperation with other organizations
Q.3. Principles of ethics. Q.10. Delta dental plan.
Ans. Ans.
[Ref LE Q.4] [Ref LE Q.2]
Q.4. New Zealand dental nurse. Q.11. Private fee for service.
Ans. Ans.
[Ref LE Q.3] [Ref LE Q.2]

Q.5. Dental Council of India. Q.12. Capitation fees.

Ans. Ans.

[Ref LE Q.5] [Ref LE Q.2]

Q.6. Expanded function dental auxiliaries. Q.13. Voluntary health agencies in India.

Ans. Ans.

[Ref LE Q.3] [Ref LE Q.1]

Q.7. Public programmes in financing dental care. Q.14. UNICEF.

Ans. Ans.

[Ref LE Q.2] The United Nations International Children’s Emergency


Fund (UNICEF) was created by the UN General Assembly
Q.8. 90th Percentile. during its first session in 1946. The main purpose was to
meet the emergency needs of children around the world.
Ans.
Functions of UNICEF
[Ref LE Q.2] l Combining humanitarian and development objec-

tives, UNICEF cooperates with countries in their


Q.9. The work of WHO.
efforts to protect their children and to enable them to
Ans. develop their full potential.
Section | I  Topic-Wise Solved Questions of Previous Years 647

UNICEF has its goal as the realization of every child


l SHORT NOTES:
of the opportunity to enjoy the basic rights and
privileges, and to contribute to their country’s prog- Q.1. What are various voluntary health agencies?
ress and well-being. Ans.
UNICEF cooperates with over 137 developing coun-
[Ref LE Q.1]
tries in several ways:
i. It assists in the planning and extension of ser- Q.2. Dental auxiliary.
vices benefitting children, in consultation Ans.
with the countries concerned.
ii. It provides support to strengthen the training [Ref LE Q.3]
and orientation of national personnel, in- Q.3. Fee schedules.
cluding health and hygiene functionaries,
Ans.
teachers and nutritionists and child-welfare
specialists. [Ref LE Q.2]
iii. It supports activities related to women’s em- Q.4. Primary health care.
powerment and community participation in
implementation and monitoring of services Ans.
benefitting children and women. [Ref LE Q.1]
iv. It helps in the development of appropriate
communication material for advocacy and in- Q.5. Denturist.
formation dissemination and education. Ans.
v. It delivers technical supplies, equipment and
Denturist is a term applied to those dental lab technicians
other aids.
who are permitted to fabricate dentures directly for patients
Organization of UNICEF
without a dentist’s prescription in some states in the USA
l The headquarters of UNICEF are located in New
and elsewhere. They may be licenced or registered. Their
York, USA.
craft is called ‘denturism’.
l It has nine regional offices in different parts of world.

l The regional office for south-central Asia is in Q.6. Indian Dental Association.
New Delhi, India, which includes India, Sri Lanka, Ans.
Afghanistan, Maldives, Nepal and Mongolia.
[Ref SE Q.2]
Q.15. Ethical principles.
Q.7. Consumer Protection Act and the dentist.
Ans.
Ans.
[Same as SE Q.3]
Consumer Protection Act (COPRA)
Q.16. Functions of Dental Council of India (DCI). The Consumer Protection Act was passed by the Parliament
in 1986.
Ans.
It was passed with a view to
[Same as SE Q.5] l Provide better protection of the interests of consumers

l Establish consumer councils to educate public


Q.17. Expanded function dental auxiliary.
l Create authorities for the settlement of consumer

Ans. disputes
l Provide quicker and cheaper remedy, when there
[Same as SE Q.6]
is deficiency in services and claims for damages
Q.18. World Health Organization (WHO). Q.8. Red Cross.
Ans. Ans.
[Same as SE Q.9] [Ref LE Q.1]
Q.19. Voluntary organization. Q.9. Dental ethics.
Ans. Ans.
[Same as SE Q.10] [Ref LE Q.4]
648 Quick Review Series for BDS 4th Year, Vol 1

Q.10. Classification of members of Indian Dental Asso- The tools of dental public health are as follows:
ciation (IDA). l Epidemiology
l Biostatistics
Ans.
l Social sciences

The members of IDA are classified as follows: l Principle of administration

l Honorary members l Preventive dentistry

l Ordinary members
Q.16. Health guides.
l Director members

l Student members Ans.


l Affiliated members

l Associate members
[Ref LE Q.1]

Q.11. Office bearers of central council of Indian Dental Q.17. Capitation fee.
Association. Ans.
Ans. [Ref LE Q.2]
Following are the office bearers of IDA: Q.18. Mention any four extended duties of dental
l One president
hygienists.
l One president-elect

l Three vice presidents Ans.


l One honorary general secretary
[Ref LE Q.3]
l One honorary joint secretary

l One honorary assistant secretary Q.19. Comprehensive dental care.


l One honorary treasurer
Ans.
l One editor of the journal of IDA

l One chairman of the Council on Dental Health (CDH) Comprehensive dental care is the meeting of accumulated den-
l One honorary secretary of CDH tal needs at the time a population group is taken into the pro-
gramme, and the detection and correction of new increments of
Q.12. Bhore Committee.
dental diseases on semi-annual or other periodic basis.
Ans. Preventive measures aimed to minimize disease are a
part of comprehensive dental care.
The Bhore Committee was appointed in 1943 by the British
Government to survey situation in India and to suggest recom- Q.20. Closed panel of payment.
mendations to the government that was to be run by Indians.
Ans.
Q.13. WHO Oral Health unit.
[Ref LE Q.2]
Ans.
Q.21. Open panel dental practice.
l The WHO Oral Health unit was established in 1956.
Ans.
l WHO plays an important role in the advancement of

oral epidemiology. WHO has published a manual titled [Ref LE Q.2]


‘Oral Health Surveys: Basic Methods’.
Q.22. Jurisprudence.
l Another manual titled ‘Application of the International

Classification of Diseases to Dentistry and Stomatology’. Ans.


l The WHO Oral Disease Preventive Programme was es-
Jurisprudence is the philosophy of law or the science that
tablished in 1976 and one of its main objectives is to
treats the principles of law and legal regulations.
develop model preventive programmes that can be rec-
ommended for use in the countries or areas having dif- Q.23. Coinsurance.
ferent disease levels and progression patterns.
Ans.
Q.14. Appropriate technology.
[Ref LE Q.2]
Ans.
Q.24. Consent.
[Ref LE Q.1]
Ans.
Q.15. Tools of dental pubic health.
The term Consent is defined as follows: ‘When two or more
Ans. persons agree upon the same thing in the same sense they
Section | I  Topic-Wise Solved Questions of Previous Years 649

are said to consent’ (as per the definition given in Section 13 The International Red Cross is the largest humanitarian
of the Indian Contract Act 1872). network in the world.
The International Committee of Red Cross includes the
Following are the types of consent:
following:
Implied consent
l Providing help and relief during natural disasters
Express consent
l Services to armed forces
Informed consent
l Services to war veterans
Proxy consent
l Programmes on first aid and nursing

Q.25. International Red Cross. l Programmes on health education

l Providing maternity and child welfare services


Ans.

Topic 5
Preventive Dentistry
COMMONLY ASKED QUESTIONS
LONG ESSAYS:
1. What is prevention in dentistry? Describe the different levels of prevention in dentistry.
2. Define preventive dentistry. Describe incremental dental care and comprehensive dental care.
3. Define preventive dentistry. How could dental caries be prevented at different levels of prevention?
4. Enumerate various steps undertaken to prevent dental caries in a community.
5. Describe the methods of prevention and control of periodontal diseases.
6. Discuss the role of dental plaque in the initiation of different dental diseases. Briefly explain plaque control programmes.
7. What are fissure sealants? Enumerate in detail their importance, indications, contraindications and their meth-
ods of application.
8. Define the caries activity test. Describe the ideal requirements of a caries activity test. Enumerate and explain
caries activity tests.
9. Describe and discuss in detail various levels of prevention. [Same as LE Q.1]
10. Define prevention in dentistry. What are the objectives of different levels of prevention? [Same as LE Q.1]
11. Define prevention. What are the different levels of prevention of dental and oral diseases? Discuss the various
aspects of treatment available under those levels. [Same as LE Q.1]
1 2. Describe and discuss the levels of prevention of dental caries. [Same as LE Q.3]
13. Define preventive dentistry. Describe the primary level of prevention as applied to dental caries. [Same as LE Q.3]
14. Describe the levels of prevention as applied to the dental caries. [Same as LE Q.3]
15. Define preventive dentistry. Describe the primary level of prevention as applied to dental caries? [Same as LE Q.3]
16. Define preventive dentistry. Discuss the various methods of secondary level of prevention in dental caries. [Same as LE Q.3]
17. Write about the various levels of prevention of oral disease. Describe in detail the prevention services rendered
by a dentist for the prevention of dental caries at various levels. [Same as LE Q.3]
18. Define preventive dentistry. Discuss the various methods of secondary level of prevention in dental caries. [Same as LE Q.3]
19. Define water fluoridation. Describe the various systemic methods used for the prevention of dental caries. [Same as LE Q.4]
20. Give a detailed account of the role of fluoride in preventive dentistry. [Same as LE Q.4]
21. Define community dentistry. Describe the measures undertaken to prevent periodontal diseases at community
level. [Same as LE Q.5]
22. What is prevention? What are the different levels of prevention as related to dentistry? How could a community
dental programme be implemented for the control of periodontal diseases? [Same as LE Q.5]
23. Describe the measures used in primary prevention of periodontal disease at individual and at community level.
[Same as LE Q.5]
24. Define dental plaque. Enumerate different methods of plaque control. Write in detail about chlorhexidine as a
plaque control agent. [Same as LE Q.6]
25. Define plaque control. Describe in detail the mechanical plaque control aids and chemical plaque control
agents. [Same as LE Q.6]
650 Quick Review Series for BDS 4th Year, Vol 1

SHORT ESSAYS:
1. Diet counselling.
2. ART.
3. Primary prevention. [Ref LE Q.1]
4. Anticaries mouth rinses. [Ref LE Q.6]
5. Disclosing solution.
6. Dentifrice.
7. Brushing methods for children.
8. Mechanical plaque control. [Ref LE Q.6]
9. Fissure sealants. [Ref LE Q.7]
10. Preventive resin restoration.
11. Hyatt prophylactic odontotomy.
12. Vipeholm study.
13. Bisbiguanides.
14. Plaque control. [Ref LE Q.6]
15. Bass technique.
16. Stillman method.
17. Levels of prevention. [Same as SE Q.3]
18. Secondary level of prevention. [Same as SE Q.3]
19. Chemical control of dental plaque. [Same as SE Q.4]
20. Antiplaque mouth rinses. [Same as SE Q.4]
21. Erythrosine dye. [Same as SE Q.5]
22. Disclosing agents. [Same as SE Q.5]
23. Brushing and flossing for children. [Same as SE Q.7]
24. Fones method of brushing. [Same as SE Q.7]
25. Pit and fissure sealants. [Same as SE Q.9]
26. Chemical antiplaque agents. [Same as SE Q.14]
27. Chemical plaque control. [Same as SE Q.14]

SHORT NOTES:
1. Tertiary level of prevention. [Ref LE Q.1]
2. Critical pH.
3. Dentifrices. [Ref SE Q.6]
4. Plaque control.
5. Powered toothbrush.
6. Caries vaccine.
7. Name dental caries activity tests.
8. Sugar substitutes.
9. Atraumatic restorative treatment.
10. Dental plaque.
11. Define primary prevention.
12. Classification of toothbrushing techniques.
13. Disclosing agents. [Ref SE Q.5]
14. Toothbrush. [Ref LE Q.6]
15. Charters method of toothbrushing.
16. Hopewood house study.
17. Classify pit and fissure sealants.
18. Salivary reductase test.
19. Dental floss. [Ref LE Q.6]
20. Preventive resin restorations.
21. Alban’s test.
22. Modified Bass technique.
23. Interdental cleansing agents.
24. Trace elements and dental caries.
Section | I  Topic-Wise Solved Questions of Previous Years 651

2 5. Substantivity.
26. Caries vaccine. [Same as SN Q.6]
27. Mechanical plaque control aids. [Same as SN Q.4]
28. Xylitol. [Same as SN Q.8]
29. Primordial prevention. [Same as SN Q.11]
30. Indications of Charters method of toothbrushing. [Same as SN Q.15]

SOLVED ANSWERS
LONG ESSAYS:
Q.1. What is prevention in dentistry? Describe the dif- strengthen the host through a variety of inter-
ferent levels of prevention in dentistry. ventions like health education, environmental
modifications, nutritional interventions, life-
Ans.
style and behavioural changes.
[SE Q.3] ii. Specific protection
It is the provision of conditions for normal mental
l {Preventive dentistry is defined as ‘the procedure em-
and physical functioning of human being indi-
ployed in practice of dentistry and community dental vidually and in group.
health programmes which prevent the occurrence of It includes the promotion of health, the prevention
oral diseases and oral abnormalities’. of sickness and the curative and restorative medi-
l The prevention is to promote health, to restore health
cine in all its aspects.
when it is impaired and to minimize suffering and distress. Examples: Fluorides and pit and fissure seal-
l Prevention can be discussed under three levels:
ants for caries prevention.}
i. Primary prevention ii. Secondary prevention
ii. Secondary prevention l It is defined as the ‘action which halts the prog-
iii. Tertiary prevention ress of a disease at its incipient stage and prevents
i. Primary prevention complications’.
l Primary prevention is defined as ‘the action taken l The specific interventions are early diagnosis and

prior to the onset of disease, which removes the adequate treatment.


possibility that a disease will ever occur’. l The earlier a disease is diagnosed and treated, the

l It is a ‘holistic’ approach designed to promote better it is from the point of view of prognosis and
health or to protect against specific disease preventing further complications or long-term
‘agents’ and hazards in environment. disability.
l Primary prevention is far more than preventing l Early diagnosis and treatment cannot be consid-

the occurrence of disease and prolonging life; it ered as prevention since the disease process has
includes the concept of ‘positive health’ – a con- already started. As this mode of intervention will
cept that encourages achievement and mainte- intercept the disease and prevent further compli-
nance of ‘an acceptable level of health that will cations, it has been included under prevention.
enable every individual to lead a socially and Advantage
economically productive life’. Important in reducing the high mortality and mor-
Advantages of primary prevention are as follows: bidity of certain diseases like hypertension.
l Economical Disadvantages
l Safe Expensive.
l The individual is not yet exposed to disease symp- Patient is already subjected to pain and suffering,
toms like pain and suffering and loss of productivity for the community.
Primary prevention involves intervention of disease, i.e.
{SN Q.1}
any attempt to intervene or interrupt the usual sequence
in the development of disease in man. iii.  Tertiary prevention
There are two modes of intervention in primary prevention: l It is the intervention in the late pathogenesis
i. Health promotion phase. It includes all measures available to re-
l ‘It is the process of enabling people to increase duce or limit impairments and disabilities, mini-
control over, and to improve health.’ mize suffering caused by existing departures
l This mode of intervention is not directed from good health and to promote the patient’s
against any particular disease but is intended to adjustment to irremediable conditions.
652 Quick Review Series for BDS 4th Year, Vol 1

can be maintained on a periodic basis. Bills for dental ser-


l Treatment, even in the late stages of the disease,
vices are equalized and regularly spaced.
may prevent sequelae and limit disability.
The concept of incremental care at the earliest available
The modes of intervention under tertiary prevention age has been common at least for two generations in dental
are as follows: sciences for school children in the USA. Treatment pro-
a. Disability limitation grammes are ‘gotten off the ground’ by taking the youngest
l Disability is any restriction or lack of ability
available group, the first year, and carrying it forward in
to perform an activity in the manner or subsequent years as far as funds permit, each year adding a
within the range considered normal for a new class of children at the next earliest available age, until
human being. For example, inability to talk an entire child population is being served to as high an age
or pronounce certain words clearly. as permitted by available resources. In economic terms, the
l The objective of disability limitation is to
programme is supposed to avoid high expenditure for initial
prevent or halt the transition of the disease dental care. In terms of dental health, it is supposed to con-
process from impairment to handicap. fine dental disease to small early increments, thus reducing
l Impairment is any loss or abnormality of
loss of teeth, and to inculcate a habit of periodic return to
psychological, physiological or anatomical dental office in subsequent years.
structure or function. For example, loss of Disadvantages to the principle of incremental care
teeth due to disease. 1. Operative dentistry is more time-consuming on a piece-
Handicap is a disadvantage for a given individual, meal basis than upon a wholesale basis. A large opera-
resulting from impairment or disability that limits tive programme can be handled on the quadrant basis
or prevents the fulfilment of a role that is normal under local anaesthesia. This makes for rapid cavity
for that individual. For example, due to loss of preparation and easy isolation of teeth for filling proce-
teeth, the teacher loses his job and becomes dures. Five or six tooth surfaces can thus be filled in the
unemployed. time required for only two or three surfaces if these are
b. Rehabilitation scattered in various parts of the mouth.
It is the combined and coordinated use of 2. It relates to the effort usually made to implement it at
medical, vocational, social and educational the earliest available age, which coincides with the entry
measures for training and retraining the indi- of a child into some public health or public school pro-
vidual to the highest possible level of func- gramme. The result of this timing is that financial re-
tional ability. sources are usually exhausted even before the elemen-
It reduces the impact of the disabling condition tary school population has been cared for and the
and helps the person to participate actively and highschool child receives no maintenance care at all.
join the mainstream. This concept of incremental care needs re-examination,
since adolescents receive no aid in stemming the mas-
Q.2. Define preventive dentistry. Describe incremental sive onslaught of dental caries on permanent teeth.
dental care and comprehensive dental care.
Studies conducted indicate that teenagers need the heavy load
Ans.
of restorative services. One is the large increase in decayed
l Preventive dentistry is defined as the ‘procedure em- missing and filled (DMF) tooth. DMF surfaces on DMF tooth,
ployed in the practice of dentistry and community den- which has been found to occur during this period. This leads
tal health programmes which prevent occurrence of oral to increased operating time per person per year as the age of
disease and oral abnormalities’. the patient has advanced. There is also an actual increase in
chair hours per year needed for dental maintenance care be-
Incremental care may be defined as periodic home care, so
yond the heavy load of initial care in the first year of a new
spaced that increments of dental disease are treated at the
patient.
earliest time consistent with such a way that there is no ac-
The disadvantages of incremental care at earliest avail-
cumulation of dental needs beyond the minimum. In private
able age are as follows:
practice, 6 months is the commonest, although not the only,
i. Attention to deciduous teeth
interval between visits. In public health programmes, 1-year
ii. Psychology and changing patterns of modern family life
intervals are usually implemented.
iii. Increasing likelihood of interruption in children’s den-
This represents the ideal pattern for care where applicable
tal health programmes
incidence of new dental disease is to be expected each year.
iv. Inertia towards the seeking of private dental care
Lesions of dental caries are treated before there is a chance
of pulpal involvement. Periodontal disease is intercepted at Comprehensive dental care is the meeting of accumulated
or near the beginning. Topical and other preventive measures dental needs at the time a population group is taken into the
Section | I  Topic-Wise Solved Questions of Previous Years 653

programme (initial care) and the detection and correction of quality work, using the best of modern restorative tech-
new increments of dental disease on a semi-annual or other niques, is postulated. This is comprehensive dental care
periodic basis (maintenance care). in the true sense.
Preventive measures aimed to minimize disease are a
part of comprehensive dental care. We shall be thinking Q.3. Define preventive dentistry. How could dental car-
not only in the terms of the elimination of pain and infec- ies be prevented at different levels of prevention?
tion but also in terms of the restoration of serviceable
Ans.
teeth to good functional form, the replacement of missing
teeth, maintenance care for the control of early lesions Preventive dentistry is defined as the ‘procedure employed
of dental disease and, most important of all, preventive in practice of dentistry and community dental health pro-
measures, educational and otherwise, so that the popula- grammes which prevent occurrence of oral disease and oral
tion may experience a lower prevalence of disease. Good abnormalities’.

Levels of
prevention Primary Secondary Tertiary
Preventive health Health promotion Specific protection Early diagnosis and Disability Rehabilitation
promotion specific prompt treatment limitation
Services provided Diet planning, de- Appropriate use of fluoride; Self-examination and Use of dental Use of dental
by individual mand for preventive intake of sufficient fluori- referral, use of dental services services
services, periodic dated water. Appropriate use services
visits to the dental of fluoride prescriptions; use
office of fluoride dentifrice. Oral
hygiene practices
Services provided Dental health educa- Community school water Periodic screening and Provision of Provision
by the community tion programme fluoridation referral dental of dental
Promotion of re- School fluoride mouth rinse Provision of dental services services
search efforts programme services
Lobby efforts School fluoride tablet
programme
School sealant programme

Services provided Plaque control Topical fluoride application Complete examination Complex Removable
by dental profes- programme, diet Fluoride supplement or rinse Prompt treatment of restorative and fixed
sional counselling, recall Pit and fissure sealants incipient lesion dentistry prosthesis
reinforcement, den- Caries activity tests Preventive resin Pulpotomy
tal caries activity restorations RCT
tests Simple restorative Extraction
dentistry
Pulp capping

RCT, root canal treatment.

Q.4. Enumerate various steps undertaken to prevent that a very strong inverse correlation exists between
dental caries in a community. decreasing caries and increasing fluoride content of the
water supply up to about 1–1.5 mg of F/L in both de-
Ans.
ciduous and permanent dentitions. Further, when the
Water fluoridation content of fluoride in water increases above 1 mg/L, the
It is defined as ‘an upward adjustment of fluoride ion prevalence of dental fluorosis increases markedly.
concentration in a public drinking water supply so that Water fluoridation is defined as the controlled adjust-
the level of fluoride is maintained at the normal physi- ment of the concentration of fluoride in a communal
ological level of 1 ppm to prevent dental caries with water supply so as to achieve maximum caries reduction
minimum possibility of causing dental fluorosis’. and a clinically insignificant level of fluorosis.
The studies carried out by Dean et al. in the USA Salt fluoridation
showed that a high degree of reduction in caries oc- For almost four decades, fluoridation of drinking water
curred in temperate climates when the fluoride content has been regarded as the most effective single fluorida-
was approximately 1 mg/L. Studies have also shown tion method of preventing dental caries on a mass scale.
654 Quick Review Series for BDS 4th Year, Vol 1

Fluoridation of salt as an alternative method was sug- fluoridated milk involves the addition of a fluoride compound
gested more than 30 years ago and the successful use of in milk in appropriate quantity such that the resulting product
salt enriched with iodine to prevent goitre has led to contains the required fluoride concentration. The concentra-
studies in several countries using fluoridated salt for the tion of fluoride required in the product is dictated by the fluo-
prevention of dental caries. ride dose to be delivered to recipient children so as to provide
Fluoridated salt has been on sale in Switzerland since them with the optimum amount in line with the recommen-
1955, and by 1967, more than three-quarters of domes- dations of the WHO Expert Committee (1994), i.e. ranging
tic salt sold in Switzerland was fluoridated with 90 mg from 0 to 1.0 mg fluoride per day depending on child’s age,
of fluoride per kilogram of salt. In the more recent in- and fluoride concentration in the local water supply.
vestigations, the level of fluoride has been raised to 200, Sodium fluoride is generally added to milk in the form
250 and 350 mg of fluoride per kilogram of salt. Despite of concentrated aqueous solution using a fixed volume
the widespread use of fluoridated salt in Switzerland, its ratio to obtain the required product. When disodium
effectiveness cannot be easily measured, since, in many monofluorophosphate is used as a fluoridating agent, it
Swiss communities, other preventive programmes, in- too is added to pasteurized milk in the form of concen-
cluding topical fluoride therapy, have been introduced in trated aqueous solution.
addition to fluoridated salt. Monofluorophosphate also reacts with calcium to form
Marthaler et al. has proved that preventive effectiveness a neutral complex, i.e. calcium monofluorophosphate,
of 250 mg of fluoride per kilogram of salt in caries was which is more soluble than calcium fluoride.
more than reduction in caries brought out by the addi-
tion of 90 mg of fluoride per kilogram of salt. Q.5. Describe the methods of prevention and control of
Tooth reported the effectiveness of 250 mg of fluoride periodontal diseases.
per kilogram of salt in Hungary after 8 years of use. The Ans.
results of his study indicated a reduction of 39% in deft
in 6-year-old children in the test community, whereas Community dentistry
the caries increased by 7% in the children of control It is the branch of dentistry practised in relation to
community over the same period. population and group, and derives from epidemiology
The caries preventive effectiveness of fluoridated salt is an awareness of service required to organize the appli-
impressive, although based on a smaller number of stud- cation of these services for the benefit of population.
ies (as compared with water fluoridation) over a maxi- Primary prevention
mum period of 10 years. It is defined as the ‘action taken prior to the onset of
The main advantages of salt as a vehicle for fluorides disease, which removes the possibility that a disease will
are as follows: ever occur’. Intervention is in the prepathogenesis phase.
l It does not require a community water supply as Primary prevention is far more than preventing the oc-
in the case of water fluoridation. currence of disease and prolonging life, and includes the
l It permits individuals to accept or reject it. concept of ‘positive health’ – A concept that encourages
l Nonfluoridated salt, like noniodized salt, can be achievement and maintenance of ‘an acceptable level of
available to the population. health that will enable every individual to lead a socially
The main disadvantages of salt fluoridation are as follows: and economically productive life’.
l Fluoridated salt consumption is lowest when the Advantages of primary prevention:
need for fluorides is high – in the early years of life. l Low cost

The current view is that a high salt intake may contrib- l Safe

ute to hypertension. Intervention is an attempt to intervene or interrupt the usual


Among the countries that have used salt as a vehicle of sequence in the development of disease in man. The modes
fluorides are Colombia, Hungary, Mexico and Switzer- of intervention in primary prevention are as follows:
land. The experience of salt fluoridation has been lon- 1. Health promotion
gest and most widespread in Switzerland. It is the process of enabling people to increase
Milk fluoridation control over, and to improve health. It is not di-
Milk is an essential component of human diet, both as a rected against any particular disease but is in-
source of micro- and macro-nutrients, as well as being tended to strengthen the host through a variety of
a carrier for undesirable contaminants, e.g. drugs and interventions like:
powder, each containing a variety of fluoridating agents. l Health education

Compounds that have been used to fluoridate milk include l Environmental modifications

sodium fluoride, calcium fluoride, disodium monofluoro- l Nutritional interventions

phosphate and disodium silicofluoride. The manufacture of l Lifestyle and behavioural changes
Section | I  Topic-Wise Solved Questions of Previous Years 655

2. Specific protection 2. Rehabilitation


It is the provision of conditions for normal mental It is the combined and coordinated use of medical,
and physical functioning of a human being, indi- vocational, social and educational measures for
vidually and in group. It includes the promotion training and retraining the individual to the highest
of health, the prevention of sickness and the cura- possible level of functional ability. It reduces the
tive and restorative medicine in all its aspects. impact of the disabling condition and helps the per-
Example: Fluorides and pit and fissure sealants son to participate actively and join the mainstream.
for caries prevention.
Q.6. Discuss the role of dental plaque in the initiation of
Secondary prevention
different dental diseases. Briefly explain plaque control
It is defined as the ‘action which halts the progress of a
programmes.
disease at its incipient stage and prevents complications’.
The specific interventions are early diagnosis and ade- Ans.
quate treatment.
Plaque control is the removal of microbial plaque and the
The earlier a disease is diagnosed and treated, the better
prevention of its accumulation on teeth and adjacent gingi-
it is from the point of view of prognosis, thus preventing
val tissues. Besides, it also deals with the prevention of
further complications or long-term disability. Strictly
calculus formation.
speaking, early diagnosis and treatment cannot be con-
sidered prevention, since the disease process has al- [SE Q.8]
ready started. However, it has been included under pre-
vention because this mode of intervention will intercept {Mechanical Plaque Control Aids
the disease and prevent further complications. The various aids used for mechanical plaque control can be
Advantage listed as follows:
It is important in reducing the high mortality and 1. Toothbrushes
l Manual toothbrush
morbidity of certain diseases like hypertension.
l Electrical toothbrush
Disadvantages
l It is expensive.
2. Interdental aids
l Dental floss
l The patient is already subjected to pain and suffer-
l Triangular toothpicks
ing, and loss of productivity for the community.
l Handheld triangular toothpicks
Tertiary prevention
l Proxa-pic
It is the intervention in the late pathogenesis phase. It is
defined as all the measures available to reduce or limit 3. Interdental brushes
impairments and disabilities, minimize suffering caused Proxabrush system
by existing departures from good health and to promote Bottle brushes
the patient’s adjustment to irremediable conditions. Single-tufted brushes (flat or tapered)
Treatment, even in the late stages of a disease, may 4. Yarn
prevent sequelae, and limit disability. 5. Superfloss
The modes of intervention are as follows: 6. Perio-aid
1. Disability limitation 7. Aids for gingival stimulation
l Rubber tip stimulator
The objective of disability limitation is to prevent
l Balsa wood edge
or halt the transition of disease process from im-
pairment to handicap. 8. Others
l Gauze strips
Impairment is any loss or abnormality of psycho-
l Pipe cleansers
logical, physiological or anatomical structure or
l Water irrigation device
function. For example, loss of teeth due to disease
in a teacher. Aids for edentulous or partially edentulous patients:
l Denture and partial clasp brushes
Disability is any restriction or lack of ability to per-
l Cleansing solutions
form an activity in the manner or within the range
considered normal for a human being. For example,
inability to talk or pronounce certain words clearly. Toothbrushes
Handicap is a disadvantage for a given individual,
resulting from an impairment or a disability that {SN Q.14}
limits or prevents the fulfilment of a role that is Toothbrushes are the most widely used oral hygiene
normal for that individual. For example, the aids. It is the principal instrument, in general used for
teacher loses his job and becomes unemployed. accomplishing the goals of plaque control.
656 Quick Review Series for BDS 4th Year, Vol 1

these brushes may also be used to carry antimicrobial


The toothbrush has been described as ‘the most classic
agents (chlorhexidine [CHX]) to interproximal areas.
and principal method employed in oral hygiene’. Accord-
Irrigation devices
ing to the American Dental Association’s (ADA) Council
Irrigation devices have been proven to be a valuable
on Dental Therapeutics, ‘The toothbrush is designed pri-
supplement for mechanical plaque control measures. It is
marily to promote cleanliness of teeth and oral cavity’.
mainly beneficial in the removal of unattached plaque
and debris. Irrigation devices are mainly composed of a
Types of toothbrush: built-in pump and a reservoir. These devices may also be
l Manual toothbrush used to deliver antimicrobial agents, e.g. chlorhexidine,
l Powdered toothbrush subgingivally. When used as an adjunct to toothbrushing,
l Sonic and ultrasonic toothbrush these devices have a beneficial effect on periodontal
l Ionic toothbrush health by retarding plaque and calculus formation. How-
Interdental cleaning aids ever, transient bacteremia has been reported following
The toothbrush is an excellent device for plaque re- the use of subgingival irrigation devices, especially in
moval from surfaces, which are accessible. However, it the presence of inflammation.
is not adequate for interproximal cleaning. Special Gingival physiotherapy
efforts have to be made and special devices have to be The rationale behind gingival physiotherapy is the fact
used for the removal of plaque from interdental areas, that mechanical stimulation of the gingivae, by either
especially from posterior molars. Interdental aids are toothbrushing or interdental cleansing with different
adjunctive devices which are used to remove plaque aids, or by simple finger massage, leads to:
from interproximal tooth surfaces. i. Increased keratinization
ii. Increased blood flow
iii. Increased flow of gingival crevicular fluid (GCF)
{SN Q.19} within the gingival sulcus
Dental floss Tongue scraping
These types of interdental cleaning aids are indicated to It is a relatively new phenomenon, which is simply de-
remove plaque from interproximal surfaces with type 1 fined as ‘the process of removing debris from the sur-
gingival embrasures. Dental floss may be available in face of the tongue with some form of scraper designed
various forms: for this purpose’. Recent studies have shown a correla-
l Multifilament – Twisted/nontwisted
tion between tongue scraping and reduction of halitosis,
l Bonded/nonbonded
gingival disease and tooth decay. Most tongue scrapers
l Thick/thin
are made of soft flexible plastic. However, a soft tooth-
brush may also be used for this purpose.
Technique
l Waxed/nonwaxed i. Brushing
The degree of plaque control achieved by any type of ii. Tongue cleaning devices}
floss is similar. Therefore, the choice of floss is based on
individual preference.
[SE Q.14]
Interproximal/interdental brushes
In the case of type 2 gingival embrasure, interdental {Chemical plaque control
brushes have proven to be the best choice for plaque re- Chemical plaque-control agents have proven to be an
moval from interproximal tooth surfaces. Their design is ideal adjunct to mechanical plaque-control procedures,
similar to that of a bottle brush, which may be mounted especially in individuals with a defective host-defence
on specifically designed handles to make its use for the mechanism, mentally or physically handicapped per-
posterior areas of the mouth more comfortable. They may sons and postoperatively in patients who have under-
also be used to clean furcation areas and root concavities. gone surgical procedures.
Powered interdental brushes Ideal requisites of an antiplaque agent
This device has shown to be as efficient as any other l Should significantly reduce plaque and gingivitis

interdental aid in achieving plaque removal and reduc- l Should prevent growth of pathogenic bacteria

ing inflammation. Besides, it requires less dexterity and l Should prevent development of resistant bacteria

makes access to gingival areas easier. l Should be compatible with oral tissues

Unitufted/single-tufted brushes l Should not stain teeth/alter taste

These types of interdental aids are basically employed l Should exhibit good retentive properties (substantivity)

in type 3 gingival embrasures. Besides plaque removal, l Should be inexpensive and easy to use}
Section | I  Topic-Wise Solved Questions of Previous Years 657

[SE Q.4 and Q.14] iii. Tooth not fully erupted and where isolation of tooth is
not possible.
{Classification of chemical plaque control agents iv. Life expectancy of tooth is limited.
First-generation antiplaque agents
v. Occlusal surfaces that are already carious and require
They are capable of reducing plaque scores by about
restoration.
20%–50%. They exhibit poor retention within the
vi. Presence of rampant caries.}
mouth. Examples: antibiotics, phenols, quaternary
ammonium compounds and sanguinarine.
Second-generation antiplaque agents
They produce an overall plaque reduction of around
Q.8. Define the caries activity test. Describe the ideal
70%–90% and are better retained by oral tissues and
requirements of a caries activity test. Enumerate and
exhibit slow-release properties. Example: bisbigua-
explain caries activity tests.
nides (chlorhexidine).
Third-generation antiplaque agents Ans.
They block binding of microorganisms to tooth or
Caries activity tests have been used in dental research for
to each other. As compared to chlorhexidine, these
many years, and some tests have been adapted for routine
do not exhibit good retentive properties. Example:
use in the dental office. ‘To run a caries preventive pro-
delmopinol.}
gramme without using microbiological methods is like run-
Q.7. What are fissure sealants? Enumerate in detail ning a weight-control programme without a scale’.
their importance, indications, contraindications and Caries activity refers to the increment of active lesions
their methods of application. (new and recurrent lesions) over a stated period of time.
Caries activity is a measure of the speed of progression
Ans.
of a carious lesion.
Caries susceptibility refers to the inherent tendency of
[SE Q.9] the host and target tissue, the tooth, to be afflicted by the
caries process. This is the susceptibility (or resistance) of a
{
l Pit is defined as a small pinpoint depression located at tooth to a caries-producing environment.
the junction of developmental grooves or at terminals Caries activity tests measure the degree to which the
of those grooves. local environmental challenge (e.g. dietary effect on micro-
l Fissures are defined as deep clefts between adjoining
bial growth and metabolism) favours the probability of oc-
cusps. currence of carious lesions.
l Pit and fissure sealant are defined as ‘a cement or a
Some caries activity and susceptibility tests:
resin introduced into unprepared occlusal pits and fis- i. Lactobacillus colony count
sures of caries susceptible teeth forming a mechanical ii. Snyder test
and physical protective layer against the action of acid- iii. Salivary reductase test
producing bacteria and their substances’. iv. Saliva tongue blade method
Indications for the placement of pit and fissure sealant v. Rapid caries activity test by Resazurin
i. Precarious deep and narrow retentive pits and fissures, Bacterial caries activity tests
which may cause wedging of an explorer. i. Lactobacilli count test
ii. Stained pits and fissures with minimum decalcifi- This test was put forward in 1933 by Hadley. This
cation. was the first microbiological caries activity test used
iii. No radiographic or clinical evidence of proximal caries extensively by practitioners.
and possibility of adequate isolation. Principle involved
iv. In caries-free pit and fissures, if the patient desires. This test estimates the number of acidogenic and
v. Caries pattern indicative of more than one lesion aciduric bacteria in the patient’s saliva.
per year. A selective medium of pH 5.0, which favours the
vi. Morphology of pit at risk of caries and other factors growth of Lactobacillus, is the basis of the test.
associated with increased incidence of caries. Improved selective medium (LBS agar) with an
vii. Community-based sealant programme. acidic pH, a high content of acetate and other salts
and low surface tension that is highly selective for
Contraindications for sealant usage the growth of lactobacilli are used.
i. Well-coalesced and self-cleansing pits and fissures. The number of colonies that grow on the medium
ii. Radiographic or clinical evidence of presence of inter- upon incubation is an index of the aciduric flora
proximal carious lesions. present in saliva.
658 Quick Review Series for BDS 4th Year, Vol 1

Procedure ii. Snyder Test


A stimulated, whole saliva specimen is collected Marshall L Snyder in the early 1940s proposed a
over a 5-min period, preferably before breakfast, by lactobacilli test that was much easier to accomplish
chewing 1-g paraffin wafer or a sterilized rubber than lactobacilli counts.
band. The laboratory procedure begins with the lin- Principle involved
ing up of seven test tubes, each with 9 mL of saline. This test was based on the fact that lactobacilli are
One millilitre of saliva is placed in the first test tube acidogenic and aciduric. Since the amount of acid
and the contents are shaken. One millilitre from this produced is directly proportional to the number of
tube is transferred to the second tube, which is also lactobacilli, both the counting method and the Sny-
shaken before another serial transfer is made to the der method measure the same cariogenic potential.
third tube. The serial transfers and shaking are re- The selective medium used for the Snyder test
peated until all tubes have been inoculated. In this has a pH of approximately 5, which is optimum
way, test tube one has 10–1 bacteria, test tube two has for growth of lactobacilli but extremely restric-
10–2 bacteria, etc. tive for other organisms that are not aciduric.
A 1-mL aliquot is taken from each of these tubes and In order to visually evaluate the rapidity and the
placed in a series of Petri dishes. Then approximately extent of acid production, bromocresol green is
10 mL of Ragosa lactobacilli selective medium, incorporated into the medium to indicate pH
which has been allowed to cool to 45°C, is added. change. The medium is blue at pH 5 and green at
After 4 days, the number of colonies on the plates is pH 4.6, yellowish at pH 4.2 and yellow at pH 3.8.
counted. The most accurate counting can be obtained A standardized colour chart is used as an aid in
on plates containing 35–100 colonies. Thus, if the determining the colour changes.
plate selected had the 10–3 dilution and 50 colonies The medium is initially prepared by adding 1 L of
were counted, the total count of bacteria from the boiling water to 61 g of the powdered Snyder’s
saliva would be 50 3 100 or 50,000. Counts can be medium and adjusting the pH with glacial acetic
scored from 1 to 4, depending on whether they fall acid. Approximately 5 mL of medium is placed in
within the range as follows: sterile test tubes that are stored in the refrigerator.
a. Score 1 5 0–1000 (light or no caries activity) Procedure
b. Score 2 5 1000–10,000 (slight caries activity) To perform the test, a specimen of stimulated whole
c. Score 3 5 10,000–100,000 (moderate caries saliva is secured with paraffin stimulation. One tube
activity) from the refrigerator is heated to 100°C to liquefy
d. Score 4 5 100,000 and above (marked caries the agar. It is then cooled to 45°C before 0.1 mL of
activity) saliva specimen is added and the tube is shaken be-
If the score increases for a group of individuals, so fore being placed in the incubator for 72 h at 37°C.
does the caries score. There is usually an excellent At the end of 24 h, and again at 48 and 72 h, the
correlation between zero count and caries resistance. colour of the medium is recorded from 1 to 4 on the
In spite of the well-established direct relationship basis of whether the colour remains the same or
between lactobacilli counts and DMFS scores, this changes to light green, light yellow or definite yellow.
method for evaluation of caries activity fell into dis- If the colour changes to definite yellow in 24 h,
use, partially because of its demerits and the intro- the individual is considered as caries-suscepti-
duction of the Snyder, Alban and dip-slide tests that ble. If no changes occur in 72 h, the individual is
are easier to perform. caries-resistant. In-between scores are less infor-
Demerits of this test mative but can be used along with clinical judg-
l Lactobacilli may be responsible for the progres- ment as an aid in evaluating caries status.
sion of lesions, as they are probably not essential Interpretation of results is given in the table below:
for the initiation of lesions; their levels in saliva Snyder test
reflect the number of existing lesions and acidic
conditions in the mouth. Time (h)
l Test results are not available for several days. 24 h 48 h 72 h
l Counting of the colonies is a very tedious process. Colour Yellow Yellow Yellow
l The test is not simple, as it requires relatively
Caries activity Marked Definite Limited
complex equipment and personnel with bacterio-
logical training. Colour Green Green Green
l The cost is relatively high. Caries activity Continue test Continue test Inactive
Section | I  Topic-Wise Solved Questions of Previous Years 659

Advantages of Snyder test Procedure


Simple The buccal surfaces of the teeth are swabbed
Less armamentarium with a cotton applicator incubated in the
Cost is moderate medium.
iii. Alban Test (Modified Snyder Test) Change in pH following 48-h incubation is read
Arthur L. Alban, a paedodontist from California, in on pH meter, or the colour change is read by the
1970, modified the Snyder test to make available an use of colour comparator.
easy-to-accomplish caries activity test for routine Results of swab test with pH meter
dental office use.
This method uses the same medium with the excep- pH Caries activity
tion that less agar is used, which allows easier .4.6 Inactive
permeation of bacteria and end products through
4.6–4.5 Mildly active
the agar column.
Procedure 4.4–4.2 Active
A 5-mL tube of agar is removed from the refrig- #4.1 Very active
erator, but not heated.
The patient is asked to drool or spit unstimulated The swab test is advantageous over the Snyder test
saliva directly into the tube until there is a thin as no collection of saliva is necessary. Therefore, it
layer of salvia covering the agar. is particularly valuable in evaluating caries activity
A small funnel can be used for sample collec- in very young children. However, the swab test is
tion. With small children, a cotton swab can be not widely used now.
rubbed across the tooth surface and it is inserted v. Dip Slide Methods
beneath the surface of agar. The tube is then in- l A specially designed dip slide of plastic is coated
cubated for 4 days and the colour change is with LBS agar.
noted every day. l Undiluted, paraffin-stimulated saliva is flowed
The colour change is scored from 0 to 4, with over the agar surface. The amount of saliva in-
the score being based on the amount of colour oculated on the dip slide is relatively constant in
change occurring from top to bottom in the tube. spite of the method of inoculation.
a. A zero score indicates no colour change. l The plastic slide holders are positioned vertically
b. 1 1 score is a colour change to yellow in with a slight tilt to assure both agar surfaces are
the top one-fourth of the tube. wetted with saliva. Excessive saliva is allowed to
c. 2 1 to the halfway mark. drain on to a clean absorbent paper.
d. 3 1 to the three-fourth mark. l The slide is then placed in a sterile tube, which is
e. 4 1 when the entire length of the agar col- tightly closed and incubated at 35°C for 4 days.
umn has changed to yellow. l It is then removed and the colony density is de-
Alban’s test is probably most predictive when termined by comparing it with an accompanied
the scores are at 0 or 4 1 levels at the end of 24 and model chart.
96 h, respectively. l The lactobacilli will form transparent or white
Like other lactobacilli tests, the Alban test is out- colonies.
standing for indicating caries inactivity. Alban test l Reading of more than 10,000 colonies per millili-
is ideal for educating patients as they can under- tre of saliva is considered high whereas reading of
stand the role of acids in caries process. less than 1000 colony counts is considered low.
Tests can be repeated and favourable changes in l Any result between 1000 and 10,000 is consid-
diet and plaque-control procedures are reflected ered medium.
within few weeks by corresponding changes in the Tests based on Streptococcus mutans
Alban test scores. Principle
iv. Swab Test l The tests measure the number of S. mutans
This test was developed by Grainger et al. in 1965. colony-forming units per unit volume of saliva.
Principle involved l Culturing plaque samples from discrete sites,
It is based on the same principle as the Snyder test. such as occlusal surfaces and proximal areas,
It measures the aciduric–acidogenic component is an ideal method for the purpose of quanti-
of oral flora after a suitable incubation period by fying S. mutans that have colonized on teeth.
employing a colour indicator in the test medium l However, this is not practical, and hence salivary
or by directly reading pH on a pH meter. samples may be used as a workable alternative.
660 Quick Review Series for BDS 4th Year, Vol 1

l Incubation on a selective streptococcus me- l The tests are economical and suitable for
dium, known as Mitis Salivarius agar (MSA), mass screening.
with a high concentration of sucrose (20%) l To identify low-risk population not requir-

and 0.2% J-L bacitracin/millilitre (MSB), ing preventive treatment.


suppresses the growth of most non-S. mutans Based on both S. mutans and lactobacilli
colonies. Advanced dip slide methods of S. mutans and lactoba-
a. Streptococcus mutans count tests cilli (Dentocult® [registered symbol] and Strip mutans)
l The number of S. mutans in human saliva S. mutans in saliva
has been proposed as a reliable indicator l S. mutans level in saliva is done using

of caries activity. Dentocult® SM (Orion Diagnostica, Finland)


l In the quantitative evaluation of the num- following manufacturer’s instructions.
ber of S. mutans colony-forming units, a l A disc impregnated with bacitracin is

serial dilution is accomplished using 1 mL dropped inside the tube that contains selec-
of saliva specimen. One millilitre aliquots tive culture media. For S. mutans to sample
of these serial dilutions are then plated us- saliva, the strip is rotated for 10 times on
ing conventional Mitis Salivarius agar with the surface of the tongue and put in the tube
the addition of sucrose and bacitracin. with media and incubated at 37°C for 48 h.
l This is followed by an incubation period The results of the strip were compared with manufac-
of 4 days, in which the CFUs are counted. turer’s chart, and the data were coded as follows:
A threshold value of 2.5 3 106 CFU/mL l Code 0 and 1: ,l05

of saliva has been suggested to select chil- l Code 2: .105,106

dren considered to be at a high caries risk. l Code 3: .106

b. Dip slide method for S. mutans Lactobacillus levels in saliva – Dentocult® LB


®
l This method is very similar to that of dip l The Dentocult method is simple and highly prac-

slide method for lactobacilli. tical for estimating salivary levels of Lactobacil-
l Undiluted paraffin-stimulated saliva is lus and other aciduric microorganisms.
poured on a special plastic slide coated l It was introduced by Larmas in 1975. This method

with Mitis Salivarius agar containing 20% makes use of a self-contained kit with a shelf life
sucrose. of at least 1 year.
l The agar surface is thoroughly moistened l This simplified, prepackaged selective culture

and the excess saliva is allowed to drain- system is easily adapted for office use and does
off from two discs containing 5-mcg baci- not require special equipment.
tracin placed on the agar, 20-mm apart. Method
l The slide is then tightly screwed into a l Undiluted paraffin-stimulated saliva is poured over

cover tube and incubated at 37°C for 48 h a plastic slide that is coated with LBS agar on both
in a scaled jar. sides. Excess saliva is allowed to drain-off and the
Scoring is done as follows: slide is placed in a sterile tube. The tube that is
i. Low: Colonies are discrete and could be read- tightly closed is incubated at 35–37°C for 4 days.
ily counted at 15 3 magnification, with the l At the end of 4 days, the colony density on the

total counted CFU inside the inhibition zone slide is not counted but compared with a model
being less than 200. chart and classified as 1000, 10,000, 100,000 or
ii. Medium: The colonies are discrete, and the 1,000,000 aciduric organisms/millilitre of saliva.
3
number in the zone of inhibition is more than l Code 0: 10 mfc/mL
4
200 at 32 3 magnification. l Code 1: 10 mfc/mL

iii. High: The colonies are tiny, and almost com- 5


l Code 2: 10 mfc/mL
6
pletely or totally cover the inhibition zone l Code 3: 10 mfc/mL

with the number of uncountable colonies Older methods


when using the 32 3 magnification. Enamel solubility test
l S. mutans test using Mitis Salivarius l It is the same as the Fosdick dissolution test.

medium with bacitracin is very good in This test is not generally suited for office
identifying children with 1-mm caries procedures.
increments. l It is based on the fact that when glucose is added

l Positive scores do not correlate well with to saliva containing powdered enamel, organic
high caries increments. acids are formed. These, in turn, decalcify
Section | I  Topic-Wise Solved Questions of Previous Years 661

the enamel, resulting in an increase in the amount l Lactic acid is then added to the sample until a pH
of soluble calcium in the saliva–glucose–enamel of 6.0 is reached. The amount of lactic acid
mixture. needed to reduce pH from 7.0 to 6.0 is the mea-
l The extent of increased calcium is supposedly sure of buffer capacity.
a direct measure of the degree of caries suscep- l This number can be converted to milli equivalents

tibility. per litre.


Salivary reductase test l There is an inverse relationship between buffering

Rapp in 1962 claimed that this test measures the ac- capacity of saliva and caries activity.
tivity of the reductase enzyme present in salivary l The saliva of individuals whose mouth contains a

bacteria. considerable number of carious lesions frequently


Principle involved has a lower acid-buffering capacity. This test,
l The reductase enzyme is involved in the formation however, does not correlate adequately with car-
of products dangerous for tooth surface. ies activity.
l The test measures the rate at which an indicator Fosdick calcium dissolution test
molecule diazoresorcinol changes from blue to red Principle
to colourless or leukoform on reduction by mixed l This test measures the milligrams of powdered

salivary flora. enamel dissolved in 4 h by acid formed when the


Procedure patient’s saliva is mixed with glucose and pow-
®
l A kit is available under the trade name of Treatex . dered enamel.
l A 5 mL of stimulated saliva is collected in a plas- Procedure
tic container with paraffin wax stimulation. The l A 25 mL of stimulated saliva is collected, part of

sample is then mixed with diazoresorcinol dye, which is analysed for calcium content. The rest is
which colours the saliva blue. placed in an 8-inch sterile test tube with about 0.1 g
l As the dye is reduced, the colour changes and the of powdered human enamel.
caries conduciveness readings are taken after 15 l The tube is sealed and shaken for 4 h at body

min. No incubation is needed. temperature with test tube agitation equipment.


l The results are interpreted as given in the table After agitation, it is again analysed for calcium
below. content.
Interpretation of salivary reductase test results l The amount of enamel dissolution increases with

increase in caries activity.


Colour change Caries conductiveness Q.9. Describe and discuss in detail various levels of
Blue in 15 min Nonconducive prevention.
Orchid in 15 min Slightly conducive Ans.
Red in 15 min Moderately conducive [Same as LE Q.1]
Red immediately on mixing Highly conducive
Q.10. Define prevention in dentistry. What are the
Colourless in 15 min Extremely conducive objectives of different levels of prevention?
Ans.
Buffer capacity test
l Buffer capacity can be quantitated using either a [Same as LE Q.1]
pH meter or a colour indicator.
Q.11. Define prevention. What are the different levels
l The test measures the quantity of acid required to
of prevention of dental and oral diseases? Discuss
lower the pH of saliva through an arbitrary pH
the various aspects of treatment available under those
interval, such as from pH 7.0 to 6.0 or the amount
levels.
of acid or base required to bring colour indicators
to their end point. Ans.
Procedure
[Same as LE Q.1]
l A 10 mL of stimulated saliva is collected under

oil at least 1 h after eating; 5 mL of this is taken Q.12. Describe and discuss the levels of prevention of
in a beaker. dental caries.
l The pH of the saliva is adjusted to 7.0 by addition
Ans.
of lactic acid or base. The level of lactic acid in
the graduated cylinder is re-recorded. [Same as LE Q.3]
662 Quick Review Series for BDS 4th Year, Vol 1

Q.13. Define preventive dentistry. Describe the primary Q.22. What is prevention? What are the different levels
level of prevention as applied to dental caries. of prevention as related to dentistry? How could a com-
munity dental programme be implemented for the con-
Ans.
trol of periodontal diseases?
[Same as LE Q.3]
Ans.
Q.14. Describe the levels of prevention as applied to the
[Same as LE Q.5]
dental caries.
Q.23. Describe the measures used in primary prevention
Ans.
of periodontal disease at individual and at community
[Same as LE Q.3] level.
Q.15. Define preventive dentistry. Describe the primary Ans.
level of prevention as applied to dental caries?
[Same as LE Q.5]
Ans.
Q.24. Define dental plaque. Enumerate different meth-
[Same as LE Q.3] ods of plaque control. Write in detail about chlorhexi-
dine as a plaque control agent.
Q.16. Define preventive dentistry. Discuss the various
methods of secondary level of prevention in dental caries. Ans.
Ans. [Same as LE Q.6]
[Same as LE Q.3] Q.25. Define plaque control. Describe in detail the mechani-
cal plaque control aids and chemical plaque control agents.
Q.17. Write about the various levels of prevention of
oral disease. Describe in detail the prevention services Ans.
rendered by a dentist for the prevention of dental caries
[Same as LE Q.6]
at various levels.
Ans.
SHORT ESSAYS:
[Same as LE Q.3]
Q.1. Diet counselling.
Q.18. Define preventive dentistry. Discuss the various
Ans.
methods of secondary level of prevention in dental
caries. Dietary control for caries: Most important dietary modifi-
cation is to decrease the frequency of eating sugars.
Ans.
Diet counselling
[Same as LE Q.3] i. Restrict the number of eating periods to three main
meals.
Q.19. Define water fluoridation. Describe the various
ii. Avoid carbohydrate snacks in between meals.
systemic methods used for the prevention of dental
iii. If required, take low-carbohydrate and high-protein
caries.
snacks and fruits in between meals.
Ans. iv. Increase eating of high-protein foods, e.g. meat, fish
and milk.
[Same as LE Q.4]
v. Restrict carbohydrate eating so that only 30%–50%
Q.20. Give a detailed account of the role of fluoride in of total calories requirement is provided to the body.
preventive dentistry. vi. As far as possible, eliminate eating sticky sweets
like chocolates, toffees, candies, cakes and pastries.
Ans.
vii. Eat firm detersive food like raw vegetables and
[Same as LE Q.4] fruits liberally. This reduces dental plaque forma-
tion and increases salivary flow.
Q.21. Define community dentistry. Describe the mea-
sures undertaken to prevent periodontal diseases at Q.2. ART.
community level.
Ans.
Ans.
l The atraumatic restorative treatment (ART) is also
[Same as LE Q.5] known as alternate restorative treatment.
Section | I  Topic-Wise Solved Questions of Previous Years 663

l ART is a procedure based on removing carious tooth


{SN Q.13}
tissues using hand instruments alone, and restoring the
cavity with an adhesive restorative material. l A disclosing agent is a preparation in liquid, tablet or
l The results of ART were so encouraging that the system lozenge form, which contains a dye or other contour-
has been adopted by WHO, and is being promoted ing agent. A disclosing agent is used for the identifi-
worldwide as a useful technique for communities that cation of bacterial plaque, which might otherwise be
lack regular dental facilities. invisible to the naked eye.
Principles of ART l When applied to the teeth, the agent imparts its co-

The two main principles of ART are as follows: lour to soft deposits but can be rinsed easily from
i. Removing carious tooth substance using hand clean tooth surfaces. After staining, the deposits that
instruments only can be distinctly seen provide a valuable visual aid in
ii. Restoring the cavity with a restorative material patient instruction.
that adheres to tooth, e.g. glass ionomer cement l Hence, such a procedure can demonstrate dramatically

Advantages of ART to the patients the presence of deposits and the areas
l Easily available, inexpensive hand instruments are that need special attention during personal oral care.
used rather than expensive electrically driven dental
equipment.
l As it is almost a painless procedure, the need for Utility of disclosing agents
local anaesthesia is eliminated or minimized. l Personalized patient instruction and motivation

l ART involves the removal of only decalcified tooth l Self-evaluation by the patient

tissues, which results in relatively small cavities and l To evaluate the effectiveness of oral hygiene mainte-

conserves sound tooth tissue as much as possible. nances


l Sound tooth tissue need not be cut for retention of l Preparation of plaque indices

filling material. The retention is obtained by the mi- l In research studies with regard to effectiveness of

crotags produced due to etching and also due to the plaque control devices like toothbrushes and denti-
chemical adhesion of glass ionomer restorative mate- frices
rial with cavity walls. Properties of an acceptable disclosing agent
l A practice of straightforward and simple infection A. Intensity of colour: A distinct staining of deposits
control is used without using autoclaved handpieces. should be evident. The colour should contrast with
l The leaching of fluoride from glass ionomer proba- the normal colours of oral cavity.
bly remineralizes sterile demineralized dentine and B. Duration of intensity: The colour should not rinse
prevents the development of secondary caries. off by using ordinary rinsing methods, or removed
l The combined preventive and curative treatment can by saliva for the period required to complete in-
be done in one appointment. structions or clinical service. It is desirable to re-
l Restorations can be easily repaired. move colour from the gingival tissue and the lips
l It is economical and a less time-consuming proce- after the appointment, as the patient might react to
dure as in one sitting several fillings can be done. colour if it is retained for a long period.
l One of the greatest advantages of ART is that it C. Taste: The patients should not be made uncomfort-
enables oral health workers to reach people who able by an unpleasant or highly flavoured substance.
otherwise would have never received any oral health The main reason for using disclosant is to motivate
service. the patient; therefore, the use of agent should be
pleasant and lead to cooperation.
Q.3. Primary prevention.
D. Irritation to mucous membrane: The patient should
Ans. be questioned concerning the possibility of idiosyn-
crasy to an ingredient. When this information is
[Ref LE Q.1]
obtained, it should be entered on patient’s perma-
Q.4. Anticaries mouth rinses. nent history record. Because of the possibility of
allergy, more than one type of disclosing agent
Ans.
should be available for use.
[Ref LE Q.6] E. Diffusibility: A solution should be thin enough so
that it can be readily applied to exposed surface of
Q.5. Disclosing solution.
the teeth, yet thick enough to impart an intensive
Ans. colour to bacterial plaque.
664 Quick Review Series for BDS 4th Year, Vol 1

F. Astringent and antiseptic properties: These proper- A dentifrice contains a number of ingredients that serve a
ties may be highly desirable because the disclosing definite purpose in providing adequate plaque control, thus
agent may contribute other factors to the technique. preventing caries and periodontal diseases.
Agents used for disclosing plaque: Application of dentifrices
a. Iodine preparations The amount of toothpaste or gel needed for effective
Skinners iodine solution cleaning is a pea-sized dab on the top half of the tooth-
Diluted tincture of iodine brush. The dentifrice should preferably be dispersed in
b. Mercurochrome preparations between the bristles rather than on the tips. Children
Mercurochrome solution 5% under 6 years of age should only be given half the
Flavoured mercurochrome disclosing solutions amount of dentifrice as compared to that of an adult.
c. Bismark brown (Easlick’s disclosing solution)
Q.7. Brushing methods for children.
d. Merbromin
e. Erythrosine Ans.
FD&C No. 3/No. 28
Fones method or circular scrub method (1934)
f. Fast green
Indication
FD&C green No. 3
Indicated for young children who want to do their own
g. Fluorescein
brushing but do not have the muscle development for
FD&C yellow No. 8 (used with special ultraviolet
techniques that requires more coordination.
source to make the agent visible)
Technique
h. Two-tone solutions
The child is asked to stretch his or her arms such that
FD&C green No. 3
they are parallel to the floor. The child is then asked to
FD&C red No. 3
make big circles using the whole arm in the air. The
i. Basic fuchsin
circles are reduced in diameter until very small circles
Application of disclosing agent
are made in front of the mouth. The child is now ready
The disclosing solution may be directly applied onto the
to make circles on the teeth with a toothbrush, making
tooth surface using cotton pellet or it may be rinsed after
sure that the teeth and gums are covered.
proper dilution. The tablets and wafers may be chewed
Advantages
or swished around the mouth for 30–60 s and rinsed.
l This technique has equal or better potential than the

Q.6. Dentifrice. Bass technique for plaque removal and prevention of


gingivitis.
Ans.
l It is easy to learn.

l Shorter time.

{SN Q.3} l Good for physically and emotionally handicapped

individuals.
l According to the ADA’s Council on Dental Thera-
l Patients who lack dexterity for more technical brush-
peutics, ‘A dentifrice is a substance used with a
ing method.
toothbrush for the purpose of cleaning the accessible
l Gingiva is provided with good stimulation.
surfaces of the teeth’.
Disadvantages
l Webster described the term dentifrice as derived
l Possible trauma to gingivae.
from dens (tooth) and fricare (to rub).
l Inter-dental areas are not properly cleaned.
l Dorland described it as a preparation for cleaning
l Detrimental for adults, especially who use brush
and polishing the teeth. These are aids for cleaning
vigorously.
and polishing tooth surfaces.
It may contain the following: Q.8. Mechanical plaque control.
a. The therapeutic agent such as fluoride to in-
Ans.
hibit dental caries
b. Antimicrobial agent, such as chlorhexidine and [Ref LE Q.6]
cetrimides, to reduce microorganism
Q.9. Fissure sealants.
c. An anticalculus agent, such as zinc chloride, to
dissolve calculus Ans.
The functions of toothpaste in conjunction with
[Ref LE Q.7]
toothbrushing are as follows:
l Minimizing plaque build-up Q.10. Preventive resin restoration.
l Anticaries action
Ans.
l Removal of stains

l Mouth freshener Preventive resin restoration (PRR)


Section | I  Topic-Wise Solved Questions of Previous Years 665

l A logical extension of preventive sealant strategy The decision on what is best for the patient involves
l

involves the use of resin restorative materials plus in a complex mix of clinical diagnostic skills, estimates
discrete areas of caries attack on a fissured surface. of overall caries risk for individual patient and as-
First reported by Simonsen and Stallard (1978), this sessment of patient’s interest and likely compliance
20-year-old concept has gained wide approval. with preventive behaviour (Elderton, 1994).
l The procedure involves use of a dental handpiece to
Q.11. Hyatt prophylactic odontotomy.
remove only those areas of the tooth affected by car-
ies, followed by bonding resin restorative material Ans.
into them, and finally covering all restorative material
l Hyatt in 1923, in his famous paper ‘Prophylactic Odon-
and any remaining fissured anatomy with sealant.
totomy’, advocated filling the fissures of teeth with silver
l Obviously, saving of tooth structure is significant. By
or copper oxyphosphate cement as soon as the teeth
avoiding the old philosophy of ‘extension for preven-
erupted, and later preparing a small cavity and filling it
tion’ of tooth preparation and replacing it with an
with amalgam. In this way, these caries-susceptible pit and
idea of discrete removal of caries, there is a major
fissures would be less susceptible to subsequent caries.
reduction in intra-coronal preparation and tooth
l The technique, called prophylactic odontotomy, was
structure loss.
proposed by Hyatt (1923).
l The bonded one-surface restoration with sealant
l He advocated the placement of small amalgam restora-
overlay has proven a very effective long-term method
tions in pits and fissures of newly erupted teeth before
to treat Class I carious lesions. These restorations
the appearance of clinical signs of decay. Later, when
have equivalent or better success than amalgam
the teeth are fully erupted, a small occlusal cavity is
restorations.
prepared to fill with amalgam.
l The longevity of PRR is dependent, to a great extent,
l He recommended eliminating all susceptible fissures
on the retention and repair of the overlying sealant.
by cutting a shallow, minimal width Class I cavity in
l Preventive resin restorations are an extension of the
enamel and then filling it with amalgam.
sealant technique that allows for caries control with
l Prophylactic odontotomy is no more recommended due
minimal loss of tooth structure. This method is indi-
to the encouraging results with fissure sealants.
cated where caries within a fissure has just reached
Advantages
the dentine.
l Small filling, minimum pulpal irritation and exten-
l Under ideal circumstances, the fissure sealants can
sion for prevention are not required.
successfully prevent progression of caries, therefore
l Serious injury with deep decay can be prevented.
sealing of very superficial lesion is a viable option
l The procedure is relatively painless.
compared to the more destructive conventional re-
l The surfaces treated do not force accumulation of
storative approach, which involves the removal of
carbohydrates and food debris and may be remineral-
healthy tooth structure to gain good access.
ized by salivary constituents.
l For early decay, where space allows, glass ionomer

veneered with unfilled resin should be used. The Q.12. Vipeholm study.
main difficulty in determining the optimal form of
Ans.
management for an early decay lesion is the diagno-
sis of the state of the fissure. Purpose of the Study
l Laser fluorescent diagnostic probes and digital imag- l Ingestion of sugar at meals in a sticky form

ing equipment may eventually improve the practitio- l Ingestion of sugar between meals in a sticky form

ner’s diagnostic accuracy on the presence and depth Method


of caries in occlusal pits, fissures and developmental The institutional diet was nutritious, but contained little
grooves. At this stage, it is not clear whether these sugar, with no provision for in-between meal snacks.
methods differentiate between infected and affected Four meals were eaten daily. The dental caries rate in
dentine, and lack of differentiation can lead to inap- the inmates was relatively low. The experimental design
propriate cavity management. divided the inmates into one control and six experimen-
l The management of these doubtful occlusal fissures tal groups.
has now become a dilemma for dental practitioners i. A control group
in every day clinical practice. Recommendations to It consisted of 60 males with an average age
use air abrasion techniques to open up all suspecting of 34.9 years, who for 2 years received a low-
fissures and grooves do not fit well with the current carbohydrate, high-fat diet, practically free from
minimal intervention philosophy, even though it may refined sugar. Caries activity was completely
be the easiest solution for practitioner. suppressed.
666 Quick Review Series for BDS 4th Year, Vol 1

After 2 years, this diet was replaced by an ordi- ii. Risk of caries is greater if the sugar is consumed in
nary diet, to which was added 110 g of sugar per a form that will be retained on teeth surfaces.
day at meal times, which was accompanied by a iii. The risk of sugar increasing caries activity is great-
small but significant rise in caries. est if the sugar is consumed in between the meals.
ii. A sucrose group iv. Upon withdrawal of sugar-rich foods, the increased
It consisted of 57 males who received 300 g of caries activity disappears rapidly.
sucrose given in solution at meal times but was v. Caries lesions might continue to appear despite the
reduced to 75 g during the last 2 years. No sig- avoidance of refined sugar and maximum restric-
nificant increase in caries was found. tions of natural sugars and dietary carbohydrates.
iii. A bread group vi. Increase in the clearance time of sugar increases
This group was subdivided into 41 males and caries activity.
42 females, who during the first 2 years received vii. This study showed that the physical form of carbo-
345 g of sweet bread containing 50 g of sugar hydrates is much more important in cariogenicity
once daily with their afternoon coffee, which than the total amount of sugar ingested.
did not produce a demonstrable increase in car-
ies. During the next 2 years, four portions of Q.13. Bisbiguanides.
sweet bread were given daily with all meals, Ans.
resulting in a significant increase in caries, more
in males than in females. Chlorhexidine gluconate (0.2%)
iv. A chocolate group Chlorhexidine gluconate is a cationic bisbiguanide ef-
It consisted of 47 males, who received 300 g of fective against an array of microorganisms, including
sucrose given in solution at meal times during Gram-positive and Gram-negative organisms, fungi,
the first 2 years, which was reduced to 110 g, yeasts and viruses. Chlorhexidine (CHX) exhibits both
supplemented with 65 g of milk chocolate daily antiplaque and antibacterial properties.
between meals during the next 2 years. Caries Mechanism of action
increment was low in the initial period but in- i. Antiplaque action of CHX
creased significantly during the second period. The superior antiplaque activity of CHX is due to
v. A caramel group its property of sustained availability – ‘substantiv-
After 2 control years, this group of 62 males ity’. This involves a reservoir of CHX, slowly
received 22 caramels daily in two portions be- dissolving from all oral surfaces, resulting in
tween meals during the third year, changed to 22 ‘bacteriostatic milieu’ in oral cavities.
caramels in four portions between meals in the CHX desorbed from the oral mucosa has three
fourth year. In the fifth year, the caramels were mechanisms of plaque inhibition:
withdrawn and replaced with an isocaloric l Prevents pellicle formation by blocking

quantity of fat with meals. There was a signifi- acidic groups on salivary glycoproteins,
cant increase in caries, which resulted in with- thereby reducing glycoprotein adsorption
drawal of caramels, resulting in fall of caries onto the tooth surface.
increment. l Prevents adsorption of bacterial cell wall onto

vi. An 8-Toffee group the tooth surface by binding to the bacteria.


It consisted of 40 males, who got a low- l Prevents binding of mature plaque by precipi-

carbohydrate, high-fat diet in the first year. tating agglutination factors in the saliva and
Then they received eight toffees a day in the displacing calcium from the plaque matrix.
second year during breakfast and lunch, which ii. Antibacterial action of CHX
was later changed to in-between meals. l CHX is a dicationicbisbiguanide with broad

vii. A 24-Toffee group antibacterial activity. It exhibits a wide spec-


The group consisted of 48 males, who received trum of activity, encompassing Gram-positive
24 toffees in between meals during the third and and Gram-negative bacteria, yeast, dermato-
fourth year, followed by the withdrawal of toffees phytes and some lipophilic viruses. CHX has
in the fifth year. This group showed the greatest strong affinity for binding to skin and mucous
increase in caries during the third and fourth year membrane.
followed by a sharp drop in the fifth year. l CHX shows different effects at different con-

The Main Conclusions of the Vipeholm Study centrations, i.e.:


i. An increase in carbohydrate (mainly sugar) defi- (a) Bacteriostatic at low concentrations
nitely increased caries activity. (b) Bactericidal at high concentrations
Section | I  Topic-Wise Solved Questions of Previous Years 667

These concentrations vary between bacterial Q.16. Stillman method.


species.
Ans.
l After a single rinse with CHX, saliva itself

exhibits antibacterial activity for about 5 h, and Indications


suppresses salivary bacterial counts for over l Dental plaque removal form cervical areas below the

12 h. Following several rinses of CHX, number height of contour of enamel and from exposed
of aerobic and anaerobic species in saliva can proximal surfaces.
be reduced by 80%–90%. CHX has also been l General application for cleaning tooth surfaces and

found to be a potent antifungal agent of oral massage of the gingiva.


cavity. l Recommended for cleaning the areas with progress-

ing gingival recession and root exposure to prevent


Q.14. Plaque control.
abrasive tissue destruction.
Ans. Technique
l The bristles are pointed apically with an oblique
[Ref LE Q.6]
angle to the long axis of tooth.
Q.15. Bass technique. l The bristles are positioned partly on the cervical

aspects of teeth and partly on the adjacent gingivae.


Ans.
l The bristles are activated by short back and forth
Bass technique is the most widely accepted and most effec- motion and simultaneously moved in a coronal
tive method for removal of dental plaque, adjacent and direction.
directly beneath the gingival margin. l Following 20 strokes, the procedure is repeated sys-
Indications tematically on adjacent teeth. A soft toothbrush is
a. For dental plaque removal adjacent to and directly indicated for this technique.
beneath the gingival margins Disadvantages
b. Particularly adaptable for open interproximal areas, l Time-consuming

cervical areas beneath the height of contour of l Improper brushing can damage the epithelial attachment

enamel and exposed root surfaces


Q.17. Levels of prevention.
c. Recommended for routine patients with or without
periodontal involvement Ans.
Technique
[Same as SE Q.3]
l The bristles are placed at an angle of 45° to the gin-

giva and moved in small circular motions. Strokes are Q.18. Secondary level of prevention.
repeated for almost 20 times on three teeth at a time.
Ans.
l On the lingual aspect of anterior teeth, the brush is

inserted vertically and the heel of the brush is [Same as SE Q.3]


pressed in the gingival sulci and proximal surfaces at
Q.19. Chemical control of dental plaque.
an angle of 45°. Bristles are then activated.
l Occlusal surfaces are cleansed by firmly pressing the Ans.
bristles against pits and fissures and activating them.
[Same as SE Q.4]
Advantages
l Effective method for removing plaque adjacent to Q.20. Antiplaque mouth rinses.
and directly beneath the gingival margin, cervical
Ans.
areas and sulcus
l Provides good gingival stimulation [Same as SE Q.4]
l Easy to learn
Q.21. Erythrosine dye.
l Recommended for routine patient with or without

periodontal involvement Ans.


Disadvantages
[Same as SE Q.5]
l Overzealous brushing may convert ‘very short

strokes’ into a scrub brush technique and cause in- Q.22. Disclosing agents.
jury to gingival margin.
Ans.
l Time-consuming.

l Dexterity requirement is too high for certain patients. [Same as SE Q.5]


668 Quick Review Series for BDS 4th Year, Vol 1

Q.23. Brushing and flossing for children. (c) Interdental brushes


l Proxabrush system
Ans.
l Bottle brushes

[Same as SE Q.7] l Single-tufted brushes (flat or tapered)

(d) Yarn
Q.24. Fones method of brushing.
(e) Superfloss
Ans. (f) Perio-aid
iii. Aids for gingival stimulation
[Same as SE Q.7]
(a) Rubber tip stimulator
Q.25. Pit and fissure sealants. (b) Balsa wood edge
iv. Others
Ans.
(a) Gauze strips
[Same as SE Q.9] (b) Pipe cleansers
(c) Water irrigation device
Q.26. Chemical antiplaque agents.
v. Aids for edentulous or partially edentulous patients
Ans. (a) Denture and partial clasp brushes
(b) Cleansing solutions
[Same as SE Q.14]
Q.5. Powered toothbrush.
Q.27. Chemical plaque control.
Ans.
Ans.
l Powered toothbrushes are also known as automatic,
[Same as SE Q.14]
mechanical or electric toothbrushes. These brushes
mimic the action of manual toothbrushes and also make
toothbrushing faster and efficient.
SHORT NOTES: l The head of these toothbrushes oscillate in a side-

to-side motion, or in a rotary motion. The frequency of


Q.1. Tertiary level of prevention.
oscillation is around 40 Hz in an ordinary powered
Ans. toothbrush.
[Ref LE Q.1] Q.6. Caries vaccine.
Q.2. Critical pH. Ans.
Ans. Caries vaccine is a vaccine to prevent and protect against
tooth decay.
l Critical pH is the pH at which any particular saliva
Routes of administration:
ceases to be saturated with calcium and phosphate.
i. Oral
l Below this value, the organic material of the tooth may
ii. Systemic
dissolve.
iii. Active gingivo-salivary
Q.3. Dentifrices. iv. Passive dental immunization
Ans. Q.7. Name dental caries activity tests.
[Ref SE Q.6] Ans.
Q.4. Plaque control. a . Lactobacillus colony count test
b. Calorimetric Snyder’s test
Ans.
c. Swab test
Mechanical plaque control aids d. S. mutans level in saliva
The various aids used for mechanical plaque control can e. Dip-slide method for S. mutans count
be listed as follows: f. Salivary buffer capacity test
i. Toothbrushes g. Salivary reductase test
(a) Manual toothbrush h. Alban’s test
(b) Electrical toothbrush i. Streptococcus mutans screening test
ii. Interdental aids j. Fosdick calcium dissolution test
(a) Dental floss k. ORA test
(b) Triangular toothpicks
Q.8. Sugar substitutes.
l Handheld triangular toothpicks

l Proxapic Ans.
Section | I  Topic-Wise Solved Questions of Previous Years 669

Various sugar substitutes are of two types: Q.14. Toothbrush.


i. Noncaloric sweeteners
Ans.
Examples: saccharin, cyclamates, aspartame
ii. Caloric sweeteners [Ref LE Q.6]
Examples: sorbitol, xylitol, fructose, glucose
Q.15. Charters method of toothbrushing.
Q.9. Atraumatic restorative treatment. Ans.
Ans. Indications
l Individuals having open interdental spaces with
l The atraumatic restorative treatment (ART) is also
known as alternate restorative treatment. missing papilla and exposed root surfaces
l Those wearing fixed partial denture (FPD) or orthodon-
l ART is a procedure based on removing carious tooth

tissues using hand instruments alone and restoring the tic appliances
l For patients who have had periodontal surgery.
cavity with an adhesive restorative material.
l Patients with moderate gingival recession, particu-
Q.10. Dental plaque. larly interproximally
l Massage and stimulation for marginal and interden-
Ans. tal gingiva
l Dental plaque is a complex, metabolically intercon- Technique
nected and highly organized bacterial ecosystem. l A soft/medium multi-tufted toothbrush is indicated

l It is a structure of vital significance as a contributing for this technique. Bristles, directed coronally, are
factor to the initiation of carious lesion. placed at an angle of 45° to the gingiva.
l Bristles are activated by mild vibratory strokes with
Q.11. Define primary prevention. the bristle ends lying interproximally.
Disadvantages
Ans.
l Brush ends do not engage the gingival sulcus to

l Primary prevention is defined as the ‘action taken prior remove subgingival bacterial accumulations.
to the onset of disease, which removes the possibility l In some areas, the correct brush placement is limited

that a disease never occurs’. or impossible, therefore modifications become nec-


l Primary prevention is far more than preventing the oc- essary, which add to the complexity of the procedure.
currence of disease and prolonging life, it includes the l Requirements in digital dexterity are high.

concept of ‘positive health’ – A concept that encourages Q.16. Hopewood house study.
achievement and maintenance of ‘an acceptable level of
health that will enable every individual to lead a socially Ans.
and economically productive life’. The dental status of children aged between 7 and 14 years,
l Primary prevention is a ‘holistic’ approach designed to
residing at Hopewood house, Bowrel, New South Wales,
promote health or to protect against specific disease was studied longitudinally for 10 years.
‘agents’ and hazards in the environment. The absence of meat and a rigid restriction on refined
Q.12. Classification of toothbrushing techniques. carbohydrate were the two principle features of the Hope-
wood house diet.
Ans.
Q.17. Classify pit and fissure sealants.
Brushing methods, broadly classified by J.C. Greene
(1966), are as follows: Ans.
The roll technique: Modified Stillman/Rolling Stroke Classification of pit and fissure sealants
The vibratory technique: Stillman, Charter and Bass Mitchell and Gordon (1990) stated that sealants could
method be differentiated in the following ways:
The circular technique: Fones method i. Cyanoacrylates
The vertical technique: Leonard’s method ii. Polyurethanes, e.g. epoxylite
The horizontal technique: ‘Scrub brush’ method iii. Dimethacrylates, e.g. BisGMA (diluted with MMA,
The physiological technique: Smith’s method ratio 3:1)
iv. Glass ionomer
Q.13. Disclosing agents.
Classification of resin sealants
Ans. A. Based on curing method
l First-generation sealants – polymerized with UV
[Ref SE Q.5] light of 350 microns
670 Quick Review Series for BDS 4th Year, Vol 1

l Second-generation sealants – self-cured or chemi- Procedure


cally cured A portion of 60 g of Snyder test agar is placed in 1 L of
l Third-generation sealants – visible light cured at water, and the suspension is brought to a boil on a low
430–490-microns wavelength flame. When thoroughly melted, the agar is distributed
l Fourth-generation sealants – fluoride-releasing seal- using about 5 mL per tube.
ants, addition of fluoride for added benefit These tubes should be autoclaved for 15 min, allowed to
B. Based on presence of filler cool and stored in a refrigerator.
l Unfilled – better flow Two tubes of Alban medium are taken and the patient is
l Semi-filled – strong and resistant to wear asked to expectorate a small amount of saliva directly
C. Based on colour into the tube.
l Tinted – for easy identification The tubes are labelled and incubated at 98.6°F (37°C)
l Clear – difficult to detect for up to 4 days.
l Opaque – for easy identification The tubes are observed daily for
l Coloured – e.g. pink (better fluoride release) Change of colour from bluish-green (pH 5) to definite
yellow (pH 4 or below).
Q.18. Salivary reductase test.
Depth of the medium in which the change has occurred.
Ans. Daily results collected over a 4-day period should be
recorded on patient’s chart.
Salivary reductase test
The following method is used for final recordings, after
Principle
n or 96 h of incubation:
This test measures the activity of reductase enzyme
Negative readings for the entire incubation period are
present in salivary bacteria.
labelled ‘negative’.
Procedure
All other readings are labelled ‘positive’ whether 1,
l Saliva is collected by chewing paraffin and expecto-
11, 111 or 1111.
rated directly into the collection tube.
Slower change or less colour change (compared to pre-
l The sample is then mixed with diazoresorcinol dye.
vious test) is labelled as ‘improved’.
l The ‘caries conduciveness’ reading or colour change is
Faster change or more pronounced colour change (com-
done after 15 min. No incubation procedures are required.
pared to previous test) is labelled as ‘worse’.
Advantages
When consecutive readings are nearly identical, they are
l No incubation required
labelled with ‘no change’.
l Quick results
Advantages
Disadvantage
l Simple
l Test results vary with time after food intake and after
l Low cost
brushing
l Diagnostic value when negative results are obtained

Q.19. Dental floss. l Its motivational value (ideal for education)

Disadvantages
Ans.
l More armamentaria required

[Ref LE Q.6] l Based on subjective evaluation of a colour change

that may not be clear-cut


Q.20. Preventive resin restorations.
Ans. Q.22. Modified Bass technique.

l Preventive resin restorations are an extension of sealant Ans.


technique that allows caries control with minimal loss
Indications
of tooth structure. This method is indicated where caries
l As a routine oral hygiene measure
within a fissure has just reached the dentine.
l Intra-sulcus cleansing
l Under ideal circumstances, the fissure sealants can suc-
Technique
cessfully prevent progression of caries, therefore seal-
l This technique combines the vibratory and circular
ing of a very superficial lesion is a viable option com-
movements of Bass technique with the sweeping mo-
pared to the more destructive conventional restorative
tion of Roll technique.
approach, which involves the removal of healthy tooth
l The toothbrush is held so that the bristles are at 45°
structure to gain good access.
to the gingivae. Bristles are gently vibrated by mov-
Q.21. Alban’s test. ing brush handle in a back and forth motion.
l In a single motion, the bristles are then swept
Ans.
over the sides of teeth towards their occlusal
It is a simplified substitute for the Snyder’s test. surfaces.
Section | I  Topic-Wise Solved Questions of Previous Years 671

Advantages Q.25. Substantivity.


l Excellent sulcus cleaning
Ans.
l Good interproximal and gingival cleaning

l Good gingival stimulation The superior antiplaque activity of chlorhexidine is due to


its property of sustained availability – ‘substantivity’. This
Q.23. Interdental cleansing agents.
involves a reservoir of chlorhexidine, slowly dissolving
Ans. from all oral surfaces, resulting in the ‘bacteriostatic milieu’
of oral cavity.
Interdental cleaning aids
i. Dental floss Q.26. Caries vaccine.
ii. Triangular toothpicks
l Handheld triangular toothpicks
Ans.
l Proxapic [Same as SN Q.6]
iii. Interdental brushes
l Proxabrush system Q.27. Mechanical plaque control aids.
l Bottle brushes
Ans.
l Single-tufted brushes (flat or tapered)

iv. Yarn [Same as SN Q.4]


Q.24. Trace elements and dental caries. Q.28. Xylitol.
Ans. Ans.
l Trace elements are defined depending on chemical, [Same as SN Q.8]
physical or biological field being discussed.
l In the field of biology, elements that are present only in Q.29. Primordial prevention.
minute quantities in animal tissues are called trace elements. Ans.
Trace elements can be grouped as follows:
[Same as SN Q.11]
l Cariostatic: F, P

l Mildly cariostatic: Mo, V, Cu, Sr Q.30. Indications of Charters method of toothbrushing.


l Doubtfully cariostatic: Be, Co, Mn, Sn, Zn

l Caries inert: Ba, Al, Ni, Pd


Ans.
l Caries promoting: Se, Mg, Cd, Pt [Same as SN Q.15]

Topic 6
Fluorides
COMMONLY ASKED QUESTIONS
LONG ESSAYS:
1. Describe the mechanism of action of fluorides in preventing dental caries. Mention the various professionally
applied topical fluorides and their strength.
2. Define water fluoridation. Describe in detail the procedural steps taken in planning a water fluoridation
programme in a city.
3. Describe the role of systemic fluorides in prevention of dental caries.
4. What is fluoridation and defluoridation? Write scoring criteria of Dean’s index.
5. Classify the various fluoride delivery methods in dentistry. Write in detail about the preparation application and
recommended age groups in Knutson’s technique.
6. Describe in detail about the topical fluorides used in preventive dentistry. [Same as LE Q.1]
7. Describe the role of topical fluoride in prevention of dental caries. [Same as LE Q.1]
672 Quick Review Series for BDS 4th Year, Vol 1

8. Describe the different uses of fluoride in dentistry. What is the mode of action of fluoride in the prevention of
dental caries? Which are the common fluorides used in dentistry? Mention advantages and disadvantages of
each. [Same as LE Q.1]
9. Describe briefly the importance of fluoride in dentistry. What are the different types of fluorides used for local
application in children? Write merits and demerits of each. [Same as LE Q.1]
10. Define water fluoridation. Discuss its feasibility in India. [Same as LE Q.2]
11. Define water fluoridation. Discuss the feasibility of community water fluoridation in India. [Same as LE Q.2]
12. Define water fluoridation. Describe the various systemic methods used for the prevention of dental caries. [Same as LE Q.2]
13. Discuss the importance and benefits of community water fluoridation for the prevention of dental diseases in a
community. [Same as LE Q.2]
14. Discuss the role of fluorides in community and school water fluoridation. Describe the ill effects of fluorides in
drinking water. [Same as LE Q.2]
15. Discuss in detail the uses of systemic fluorides for the prevention of dental caries in children. [Same as LE Q.3]
16. Discuss the importance of systemic fluorides in the prevention of teeth decay. State the mechanism of its action
in short. [Same as LE Q.3]
17. Classify systemic fluoride and write in detail about systemic fluoridation. [Same as LE Q.3]
18. Define fluoridation of water. Describe the procedures of water defluoridation. [Same as LE Q.4]
19. Define dental fluorosis. What are the various methods of supplementation and defluoridation? [Same as LE Q.4]

SHORT ESSAYS:
1. Mechanism of action of fluorides. [Ref LE Q.1]
2. APF gel.
3. Salt fluoridation. [Ref LE Q.3]
4. Defluoridation. [Ref LE Q.4]
5. Shoe leather survey.
6. Topical fluorides.
7. Fluoride varnishes.
8. Fluoride mouthwashes.
9. Milk fluoridation.
10. Acute bifluoride toxicity.
11. Mechanism of actions of fluoride in preventing dental caries. [Same as SE Q.1]
12. APF. [Same as SE Q.2]
13. APF solution. [Same as SE Q.2]
14. Brudevold’s solution. [Same as SE Q.2]
15. Fluoridated salt. [Same as SE Q.3]
16. Nalgonda technique. [Same as SE Q.4]
17. Tropical fluoride application. [Same as SE Q.6]

SHORT NOTES:
1. Fluorosis index. [Ref LE Q.4]
2. Acute fluoride poisoning. [Ref SE Q.10]
3. Chocking phenomenon.
4. Certain lethal dose of fluoride.
5. Indications of dietary fluoride supplements.
6. Fluoride mouth rinses. [Ref SE Q.8]
7. Fluoride tablets.
8. School water fluoridation.
9. Salt fluoridation. [Ref SE Q.3]
10. Optimum levels of fluoride.
11. Fluor protector.
12. Milk fluoridation.
13. Mention methods of systemic fluoridation. [Ref LE Q.3]
14. Dental fluorosis.
Section | I  Topic-Wise Solved Questions of Previous Years 673

1 5. Endemic fluorosis.
16. Dean’s index. [Same as SN Q.1]
17. Trendley H. Dean. [Same as SN Q.1]
18. Details about acute fluoride toxicity and management. [Same as SN Q.2]
19. Lethal dose of fluoride. [Same as SN Q.2]
20. What are the signs and symptoms of acute fluoride toxicity? [Same as SN Q.2]
21. Dosage of fluoride tablets. [Same as SN Q.7]

SOLVED ANSWERS
LONG ESSAYS:
Q.1. Describe the mechanism of action of fluorides in viii. Desorption of protein and bacteria
preventing dental caries. Mention the various profes- l Hydroxyapatite crystals have both positive and
sionally applied topical fluorides and their strength. negative receptor sites. Acidic protein group binds
to calcium site and basic to phosphate site. Fluoride
Ans.
inhibits the binding of acidic protein to hydroxy-
[SE Q.1] apatite, thereby displaying its beneficial effects.
ix. Alteration in tooth morphology
{Mechanism of action of fluorides l Dentition in fluoridated communities showed a
i. Improved crystallinity tendency towards rounded cusps shallow fissures
l Fluoride helps in increasing the crystal size and thus
due to selective inhibition of ameloblast.}
less strain is produced in crystal lattice.
l This can be achieved by converting the amorphous Topical fluorides
calcium phosphate into crystalline hydroxyphosphate. Definition
ii. Void theory Topical fluoride therapy refers to the use of systems
l Voids are filled by fluoride in the hydroxyapatite containing relatively large concentrations of fluoride that
crystal, which helps in attaining a stable form by are applied locally or topically to erupted tooth surfaces to
forming more and stronger hydrogen bonds. prevent the formation of dental caries.
l Lower solubility is obtained through greater stabil- Classification
ity, and thus there is greater resistance to dissolution I. Operator administered
in acids. Fluoride solutions
iii. Acid solubility Sodium fluoride 2%
l Fluorapatite or fluoridated hydroxyapatite is less Stannous fluoride 8%
soluble than hydroxyapatite, therefore it has greater Fluoride gels
stability. Acidulated phosphate fluoride 1.23%
iv. Enzyme inhibition Fluoride varnishes
l Fluoride forms a complex with enolase enzyme to Duraphat
inhibit glucose transport. It also binds with phospha- Fluor protector
tases to reduce acid production. II. Self-administered
v. Suppressing the flora Fluoride dentifrices
l Bacterial metabolism is inhibited by stannous Sodium fluoride
fluoride, which oxidizes the thiol group present in Fluoride mouth rinses
bacteria. Dentifrices containing monofluorophosphate
vi. Antibacterial action A. Sodium fluoride 2%
l The transport of uptake of glucose into cells of oral Method of preparation
streptococci is decreased when the concentration of Sodium fluoride solution is prepared by dissolving
fluoride is above 2 parts per million (ppm) in solu- 20 g of sodium fluoride powder in 1 L of distilled
tion and it also reduces ATP synthesis. water in a plastic bottle.
vii. Lowering free surface energy Advantages
l Fluoride substitutes the hydroxyl ions to reduce l It has an accepted taste.

the free surface energy and thus indirectly reduces l It is stable if stored in plastic containers.

the deposition of pellicle and subsequent plaque l Once applied to the teeth, the solution is allowed

formation. to dry for 3 min, so the clinician can take up


multiple-chair procedure.
674 Quick Review Series for BDS 4th Year, Vol 1

l The series of treatments must be repeated only Duraphat


four times in the general age range of 3–17 years, l Duraphat is a sodium fluoride in varnish form

rather than at annual or semi-annual intervals. containing 22.6 mg F/mL suspended in an alco-
Disadvantage holic solution of natural organic varnishes.
l Four visits are required at short periods of 3, 7, 11 l It is available in bottles of 30 mL suspension con-

and 13 years. taining 50 mg NaF/mL.


B. Stannous fluoride 8% l The active fluoride available is 22,600 ppm.

Method of preparation (Muhler’s solution) E. Fluoride mouth rinses


l Stannous fluoride solution should be prepared Mouth rinsing is a practical and effective means of self-
before use each time, as it has no shelf life. application of fluoride.
l It is prepared by dissolving 0.8 g stannous fluo- Advantage
ride in 10 mL of diluted water in a plastic con- It has 30%–40% average reduction in dental caries
tainer, and the solution prepared is shaken briefly. incidence.
l The solution is applied immediately to the teeth. Disadvantage
Advantage It requires community participation.
l It is applied only once a year.

Disadvantages Q.2. Define water fluoridation. Describe in detail the


l It has to be prepared freshly each time before use.
procedural steps taken in planning a water fluoridation
l It has a metallic taste.
programme in a city.
C. Acidulated phosphate fluoride 1.23% Ans.
Method of preparation (Brudevold’s solution)
l It is prepared by dissolving 20 g of sodium fluo- Water Fluoridation
ride in 1 L of 0.1 M phosphoric acid. To this is Water fluoridation is defined as the upward adjustment
added 50% hydrofluoric acid to adjust the pH at of the concentration of fluoride ions in public water supply
3.0 and fluoride concentration at 1.23%. in such a way that the concentration of fluoride ions in
APF gel water will be consistently maintained at 1 ppm by weight.
l For the preparation of APF gel, a gelling agent Feasibility
methylcellulose or hydroxyethyl cellulose is Water fluoridation procedure is feasible only if:
added to the solution and the pH is adjusted i. There is a reasonable number of homes receiving the
between 4 and 5. municipal water supply.
Advantages ii. People avoid drinking water from individual wells or
l There is no staining of tooth structure. rainwater tanks.
l It is stable when kept in polyethylene bottle. iii. There is availability of suitable equipment.
l In case of gel, self-application is possible. iv. There is adequate supply of fluoride.
Disadvantages v. Workers are available in the water treatment plant to
l It is sour and bitter in taste. maintain the system and keep records.
l Repeated applications necessitate the use of suc- vi. Adequate money is available for initial installation and
tion, thereby minimizing its use in the field. running costs.
l It cannot be stored in glass containers because it
Various systemic methods used in prevention of dental
may remove mineral from (etch) the glass. caries
l The repeated or prolonged exposure of porcelain or
A. Water fluoridation
composite restorations to APF could result in the a. Community water fluoridation
loss of materials and could cause surface roughen- b. School water fluoridation
ing and possible cosmetic changes. B. Salt fluoridation
D. Fluoride varnish C. Milk fluoridation
The two most commonly used varnishes are as follows: D. Fluoride tablets
l Duraphat (NaF varnish)
E. Fluoride drops
l Fluor protector (silane fluoride)

Fluor protector (Silane fluoride) Community water fluoridation


Composition l The first community water fluoridation scheme began in

l Fluor protector is a colourless polyurethane Grand Rapids, USA, in 1945. This was quickly fol-
lacquer. lowed by other large-scale studies in North America and
l The fluoride compound is a difluorosilane- other countries throughout the world.
ethyl-difluorohydroxysilane. l The reduction in caries prevalence in primary and

l The active fluoride available is 7000 ppm. permanent dentitions has also been associated with an
Section | I  Topic-Wise Solved Questions of Previous Years 675

increasing number of subjects with caries-free denti- Disadvantages of water fluoridation


tions, a slower rate of progression of dental caries and l It interferes with human rights.

with less frequent involvement of dental pulp. l Other modes are not considered.

l Common source of water supply may not be present.


The studies were done on different cities, one as experi-
mental city and other as a control, and the results were School water fluoridation
compared. l It was first initiated as a pilot study in 1954 at St. Thomas,

Virgin Islands, USA.


Studies on water fluoridation l It is one of the several effective alternatives for preven-

tion of dental caries in children of communities where


Experiment (city) Control
water fluoridation is not feasible.
i. Grand Rapids (Michigan) Muskegon l It can be used only if the surrounding areas from which

ii. Newburg (New York) Kingston the students come have a low fluoride content.
l The concentration of fluoride in the school water system
iii. Brantford (Ontario, Canada) Sarnia
is 4.5 ppm in contrast to 1 ppm of community water
iv. Evanston (Illinois) Oak Park
supply. This adjustment is done to compensate for the
v. Tiel (The Netherlands) Culemborg reduced water intake, since the school day and year are
shorter and hence the time spent at school.
Grand Rapids–Muskegon study Advantages
In this study, sodium fluoride was added to the water of l There is no effort required by the recipients.

Grand Rapids. The effects of 6½ years of fluoridation in l Good results in reducing caries.

Grand Rapids were reported by Arnold et al. in 1953. The l Minimal equipment is required.

results showed that the caries experience was almost half l It is not expensive.

that of 6-year Muskegon study.


Disadvantages
Newburgh–Kingston study l Children do not receive the benefit until they go to school.

Here, sodium fluoride was added to the drinking water of It allows only fewer benefits in the primary dentitions.
Newburg on the Hudson River. Kingston town was the con- l All children do not go to school.

trol. After 10 years of fluoridation, it was reported that the l Amount of water drunk cannot be regulated.

DMF rate had considerably reduced, thus confirming the l Continuous careful monitoring is required.

caries inhibitory property of fluoride drinking water.


Q.3. Describe the role of systemic fluorides in preven-
Evanston–Oak Park study tion of dental caries.
Fluoridation experiment also began in Evanston, Illinois, and
Ans.
the nearby community of Oak Park acted as the control town.
After 14 years of fluoridation in Evanston, there was a reduc-
tion of 49% in DMF values. The Evanston–Oak Park study {SN Q.13}
presented the most detailed data of all the fluoridation studies.
Systemic fluorides
Dutch study (Tiel–Culemborg) A. Water fluoridation
The drinking water in Tiel was fluoridated at a level of a. School water fluoridation
1.1 ppm. Culemborg with fluoridation concentration of b. Community water fluoridation
0.1 ppm was the control. After 13 years of fluoridation in B. Salt fluoridation
Tiel, the number of tooth affected by dental caries was 58% C. Milk fluoridation
lower in Tiel than in Culemborg. D. Dietary fluoride supplements
Benefits of community water fluoridation a. Fluoride tablets
l A large number of people are benefitted.
b. Fluoride drops
l Consumption of water is regular.

l Fluoridated drinking water makes dental enamel more Mechanism of action of systemic fluorides
resistant to dental decay. The cariostatic mechanism of systemic fluorides is ex-
l Fluoridated water has topical effect through the release plained under the following headings:
in saliva. l Rendering enamel more resistant to dissolution

l Fluoridation of community water is the least expensive l Inhibition of bacterial enzyme systems enzymatic action

and the most effective way to provide fluoride to a large l Reducing tendency of the enamel surface to absorb proteins

group of people. l Modification in the size and shape of teeth


676 Quick Review Series for BDS 4th Year, Vol 1

Enamel contains millions of rods that run from dentino- Disadvantages of water fluoridation
enamel junction to the tooth surface. The enamel rod is l It interferes with human rights.

hexagonal in shape, which is flattened on two opposite l Other modes are not considered.

sides. The crystal has three axes. The central position is oc- l Common source of water supply may not be

cupied by the hydroxyl ions surrounded by calcium periph- present.


erally by phosphate ions.
[SE Q.3]
Different systemic fluoridation
A. Water fluoridation {B. Salt fluoridation
l Salt fluoridation appears to be the most effective
a. School water fluoridation
l It was first initiated as a pilot study in 1954 at
method to deliver fluoride to a target population where
St. Thomas, Virgin Islands, USA. water fluoridation is not possible, thus it also provides
l One of the several effective alternatives for pre-
a better mode for reaching a huge number of people.
l This programme was first introduced in Switzer-
vention of dental caries in children of communi-
ties where water fluoridation is not feasible is the land in 1955, with 5 mg of potassium iodine and
fluoridation of the school water supply. 90 mg of sodium fluoride per kilogram.
l It can be used only if the surrounding areas from
Advantages
l The possibility of fluorosis is minimal.
which the students come have low fluoride contents.
l It is safe.
l The concentration of fluoride in the school water sys-
l It requires low cost.
tem is 4.5 ppm in contrast to 1 ppm of community
l Individual monitoring is not required.
water supply. This upward adjustment is to compen-
l It is freely available.
sate for the reduced water intake, since the school day
l Its distribution can be easily monitored.
and year are shorter and hence the time spent at school.
l Supply for salt fluoridation can be effectively
Advantages
l No effort is required by the recipients.
controlled.
l It is readily accepted, as the addition of fluoride
l It has good results in reducing caries.

l Minimal equipment is required.


does not alter colour.
l The process is not expensive.
Method of preparation
Disadvantages Type 1: Fluoride is added to salt by spraying con-
l Children do not receive the benefit until they
centrated solutions of sodium fluoride and potas-
go to school. It allows only fewer benefits in sium fluoride on salt on a conveyor belt.
the primary dentitions. Type 2: Sodium fluoride and calcium fluoride are
l Not all children go to school.
first mixed with slightly moist salt or mixed with a
l Amount of water drunk cannot be regulated.
flow conditioner, such as tricalcium phosphate, and
l Continuous careful monitoring is required.
these premixed granules are added to the dry salt.}
b. Community water fluoridation [SE Q.9]
l The first community water fluoridation scheme began
in Grand Rapids, USA, in 1945. This was quickly {C. Milk fluoridation
followed by other large-scale studies in North Amer- l Milk is a reasonable vehicle for fluoride since it is

ica and other countries throughout the world. a food used universally by infants, pregnant women
l The reduction in caries prevalence in primary and and children.
permanent dentitions has also been associated with l Milk is an excellent source of calcium and phospho-

an increasing number of subjects with caries-free rus, and when fortified with vitamin D, it contains all
dentitions, a slower rate of progression of dental car- essentials for the development of bones and teeth.
ies and with less frequent involvement of dental pulp. l Fluoridation of milk was first mentioned by Ziegler in

Benefits of community water fluoridation 1956. The rate of absorption of fluoride from milk and
l Large number of people are benefitted. water has a significant reduction in fluoride absorption
l Consumption of water is regular. from milk during the first hour, but thereafter absorption
l Fluoridated drinking water makes dental enamel continues at higher levels for longer periods of time.
more resistant to dental decay and also has topi- Advantage
cal effect through the release in saliva. l Milk is a staple food for children and infants.

l Fluoridation of community water is the least Disadvantages


expensive and most effective way to provide l Cost of fluoridated milk would be considerably

fluoride to a large group of people. higher.


Section | I  Topic-Wise Solved Questions of Previous Years 677

l There should be a centralized milk supply. providing optimal protection from dental caries and reduc-
l There are variations in intake and quantity of milk.} ing fluorosis.
D. Dietary fluoride supplements Optimum limit of fluoride in drinking water for the pre-
l Fluoride supplements were first introduced in the vention of dental caries is 0.7–1.2 ppm.
late 1940s and were intended as a substitute for Procedures of water defluoridation are based on the
fluoridated water for children in nonfluoridated following:
areas. These were introduced at a time when it was A. Ion exchange process
assumed that fluoride’s cariostatic effects were i. Anion exchange resins
principally pre-eruptive. These include polystyrene anion exchange resins and
l The most common method of delivering systemic, or basic quaternary ammonium-type resins (e.g. Tulsion
dietary, supplements are as follows: A27, Deacedite, FF-IP, Lewatit, MIH-59 and Amberlite
i. Fluoride drops with/without vitamins IRA-400).
ii. Fluoride tablets with/without vitamins ii. Cation exchange resins
iii. Lozenges For example:
iv. Oral rinse supplements (swished and swallowed) (a) Defluoron-1: Sulphonated saw dust impreg-
Dietary fluoride supplements are available as follows: nated with 2% alum solution.
l 0.125, 0.25 and 0.50 mg drops (b) Carbion: It is a cation exchange resin of good
l 0.25, 0.50 and 1.0 mg tablets/lozenges durability and it can be used on sodium and
l 1 mg, 5 mL oral rinse supplements hydrogen cycles.
Supplements contain fluoride, typically 0.25, 0.5 or (c) Magnesia: It is used to remove the excess
1.0 mg, usually as sodium fluoride, but sometimes as fluoride.
acidulated phosphate fluoride, potassium fluoride or (d) Defluoron-2: This was developed in 1968. It is
calcium fluoride. a sulphonated coal and works on the alumin-
Indications for use ium cycles.
(a) It is useful in the areas where there is no central B. Addition of chemicals to water
water supply or where the fluoride concentration a. Lime
is low in well water or where parental motivation b. Lime with magnesium salts
is very high. c. Aluminium salts
(b) As an interim measure in those communities d. Magnesia
with a central water system that have not yet e. Calcium phosphate
implemented community water fluoridation. f. Bentonite
(c) Useful in the areas where water fluoridation or g. Fuller’s earth
salt fluoridation schemes cannot be implemented. h. Diatomaceous earth
Nalgonda technique
Q.4. What is fluoridation and defluoridation? Write
l This technique was introduced in the town of
scoring criteria of Dean’s index.
Kadiri in Andhra Pradesh in 1980. This is a more
Ans. economical method of defluoridation.
l In this method, the fluoridated water is defluori-
Water fluoridation
dated by addition of lime and aluminium sul-
It is defined as the upward adjustment of the concentra-
phate, and aluminium chloride can also be added.
tion of fluoride ion in a public water supply in such a way
l This technique can be used for both domestic and
that the concentration of fluoride ion in water is maintained
community water supplies.
at 1 ppm by weight, thus preventing dental caries along
Advantages
with minimum possibility of causing dental fluorosis.
l Its design, construction and maintenance are

[SE Q.4] simple.


l Adaptable to domestic usage.
{Water defluoridation l Readily available chemicals are used.
Defluoridation is the process of removing excess, natu- l No regeneration of media.
rally occurring fluorides from drinking water to reduce the l No handling of caustic acids and alkalis.
prevalence and severity of dental fluorosis. l Less wastage of water.

Or C. Reverse osmosis
l Reverse osmosis (RO) is a water filtration process in
It is a downward adjustment of fluoride ion in high fluoride- which drinking water, which contains dissolved sol-
containing drinking water to the optimum level, thereby ids, is run through a membrane and the water then
678 Quick Review Series for BDS 4th Year, Vol 1

goes to a storage tank and 99.9% of unwanted inor- 2 . Milk fluoridation


ganic compounds, sediment and other contaminants 3. Salt fluoridation
are flushed down the drain. 4. Fluoride drops
l This form of water treatment is one of the most effec- 5. Fluoride tablets
tive methods for producing high-quality drinking water. 6. Fluoride lozenges
D. Electrolysis B. Topical
l Electrochemically generated aluminium can remove 1. Sodium fluoride
most contaminants present in water by precipitation 2. Stannous fluoride
and adsorption. 3. Acidulated phosphate fluoride
l Through the process of electrolysis, coagulating 4. Fluoride varnish
agents, such as metal hydroxides, are produced.} 5. Fluoride dentifrice
Dental fluorosis 6. Fluoride mouth rinse.
l It can be defined as hypoplasia or hypomaturation of
Knutson’s technique
tooth enamel or dentine produced by the chronic inges-
Method of preparation
tion of excessive amounts of fluoride during the period
l A 2% NaF solution can be prepared by dissolving 20 g
when teeth are developing.
of NaF powder in 1 L of distilled water in a plastic bottle.
l There is direct inhibitory effect on enzymatic action of
l It is essential to store fluoride in plastic bottles be-
ameloblasts, leading to defective matrix formation and
cause if stored in glass containers, the fluoride ion of
subsequent hypomineralization.
solution can react with silica of glass, forming SiF,
thus reducing the availability of free active fluoride
{SN Q.1} for proposed anticaries action.
Dean’s Index Method of application (Knutson’s technique)
l Initially, cleaning and polishing of the teeth is done.
Dean score Criteria l An upper and opposing lower quadrant is isolated

0 Normal enamel. with cotton rolls.


l Teeth are dried thoroughly.
0.5 Questionable mottling: Normal translucency
l A 2% NaF is applied with cotton applicators and is
is varied by a few white flecks or white spots.
permitted to dry in the teeth for about 4 min.
1.0 Very mild mottling: Small opaque paper
l Procedure is repeated for the remaining quadrants.
white areas are scattered over the teeth,
l After completion, patient is instructed to avoid eat-
involving less than 25% of the surfaces.
ing, drinking or rinsing for 30 min.
2.0 Mild mottling: The white opaque areas are
l Second, third and fourth applications are done at
more extensive but do not involve more
than 50% of the surface. weekly intervals.
3.0 Moderate mottling: All enamel surfaces are Recommended ages
affected, and those subjected to attrition Full series of four treatments is recommended at 3, 7, 11
show marked wear. Brown stains are fre- and 13 years of age.
quent disfiguring features.

4.0 Severe mottling: All enamel surfaces are


Q.6. Describe in detail about the topical fluorides used
affected, and hypoplasia is so marked that in preventive dentistry.
tooth form may be altered. A major diag-
nostic sign is discrete or confluent pitting.
Ans.
Brown stains are widespread and the teeth [Same as LE Q.1]
often present a corroded appearance.
Q.7. Describe the role of topical fluoride in prevention of
dental caries.
Q.5. Classify the various fluoride delivery methods in den- Ans.
tistry. Write in detail about the preparation application
and recommended age groups in Knutson’s technique. [Same as LE Q.1]
Ans. Q.8. Describe the different uses of fluoride in dentistry.
What is the mode of action of fluoride in the prevention of
Various fluoride delivery methods are divided into two dental caries? Which are the common fluorides used in
categories: dentistry? Mention advantages and disadvantages of each.
A. Systemic
1. Water fluoridation Ans.
(a) Community [Same as LE Q.1]
(b) School
Section | I  Topic-Wise Solved Questions of Previous Years 679

Q.9. Describe briefly the importance of fluoride in den- Q.18. Define fluoridation of water. Describe the proce-
tistry. What are the different types of fluorides used for dures of water defluoridation.
local application in children? Write merits and demerits
Ans.
of each.
[Same as LE Q.4]
Ans.
Q.19. Define dental fluorosis. What are the various
[Same as LE Q.1]
methods of supplementation and defluoridation?
Q.10. Define water fluoridation. Discuss its feasibility in
Ans.
India.
[Same as LE Q.4]
Ans.
[Same as LE Q.2]
SHORT ESSAYS:
Q.11. Define water fluoridation. Discuss the feasibility
Q.1. Mechanism of action of fluorides.
of community water fluoridation in India.
Ans.
Ans.
[Ref LE Q.1]
[Same as LE Q.2]
Q.2. APF gel.
Q.12. Define water fluoridation. Describe the various sys-
temic methods used for the prevention of dental caries. Ans.
Ans. Acidulated phosphate fluoride 1.23% is also known as
Brudevold’s solution.
[Same as LE Q.2]
Method of preparation (Brudevold’s solution)
Q.13. Discuss the importance and benefits of community l It is prepared by dissolving 20 g of sodium fluoride in
water fluoridation for the prevention of dental diseases 1 L of 0.1 M phosphoric acid. To this is added 50%
in a community. hydrofluoride acid to adjust the pH at 3.0 and fluoride
concentration at 1.23%.
Ans.
APF Gel
[Same as LE Q.2]
l For the preparation of APF gel, a gelling agent, methyl-
Q.14. Discuss the role of fluorides in community and cellulose or hydroxyl ethyl cellulose, is added to the
school water fluoridation. Describe the ill effects of fluo- solution and the pH is adjusted between 4 and 5.
rides in drinking water.
Method of application
Ans. i. Oral prophylaxis.
ii. Teeth are isolated with cotton rolls on both lingual and
[Same as LE Q.2]
buccal sides.
Q.15. Discuss in detail the uses of systemic fluorides for iii. Teeth are dried.
the prevention of dental caries in children. iv. APF solution is continuously and repeatedly applied
with cotton applicators.
Ans.
v. Teeth are kept moist for 4 min.
[Same as LE Q.3]
Recommended frequency
Q.16. Discuss the importance of systemic fluorides in the The recommended frequency of APF topical application
prevention of teeth decay. State the mechanism of its is twice a year.
action in short. Mechanism of action
l When APF is applied on the teeth, it initially leads to de-
Ans.
hydration and shrinkage in the volume of hydroxyapatite
[Same as LE Q.3] crystals, which further on hydrolysis forms an intermedi-
ate product called dicalcium phosphate dihydrate (DCPD).
Q.17. Classify systemic fluoride and write in detail
l This DCPD is highly reactive with fluoride and starts
about systemic fluoridation.
forming immediately when APF is applied and fluoride
Ans. penetrates into the crystals more deeply through the
openings produced by shrinkage and leads to formation
[Same as LE Q.3]
of fluorapatite.
680 Quick Review Series for BDS 4th Year, Vol 1

Advantages
Disadvantages
l No staining of tooth structure.
l There is no precise control, as the salt intake var-
l Stable when kept in polyethylene bottle.
ies greatly among people.
l In case of gel, self-application is possible.
l There are now international efforts to reduce so-

Disadvantages dium intake to help control hypertension.


l Sour and bitter in taste.

l Repeated applications necessitate the use of suction,

thereby minimizing its use in the field. Q.4. Defluoridation.


Ans.
APF gel APF solution
i. Relatively costly Economical [Ref LE Q.4]
ii. Readily available Prepared easily Q.5. Shoe leather survey.
iii. Self-application Applied by dentist Ans.
l Shoe leather survey is a study of relationship between
Q.3. Salt fluoridation.
fluoride concentration in drinking water, mottled enamel
Ans. and dental caries.
l Dr H. Trendley Dean pursued full-time research on mottled
[Ref LE Q.3]
enamel. He continued McKay’s work to find the extent and
geographical distribution of mottled enamel in the USA.
{SN Q.9} l His aim was to find out the minimal threshold of fluoride,

l Salt fluoridation appears to be the most effective the level at which fluorine began to blemish the teeth.
l He showed conclusively that the severity of mottling
method to deliver fluoride to a target population
where water fluoridation is not possible, thus it also increased with increasing fluoride concentrations in the
provides a better mode for reaching a huge number drinking water.
l He gave the following observations according to water
of people.
l This programme was first introduced in Switzerland
concentration:
in 1955, with 5 mg of potassium iodine and 90 mg of When water concentration was
sodium fluoride per kilogram. i. 4 ppm or more – signs of discrete pitting.
ii. 3 ppm or more – mottling was widespread.
iii. 2–3 ppm – teeth had dull chalky appearance.
Method of preparation iv. 1 ppm or less – no mottling of any aesthetic
Type 1: Fluoride is added to salt by spraying concen- significance.
trated solution of sodium fluoride and potassium fluo- v. He also reported that the incidence of caries in
ride on salt on a conveyor belt. these teeth was less as compared to nonfluori-
Type 2: Sodium fluoride and calcium fluoride are first dated teeth.
mixed with slightly moist salt or mixed with a flow l With the excitement of the early results, water fluorida-
conditioner, such as tricalcium phosphate, and these tion started in the USA.
premixed granules are added to the dry salt. l It was then believed that the addition of 0.1 ppm fluo-

ride to the drinking water would result in a maximal


{SN Q.9} reduction in the permanent tooth caries experience of
6- to 8-year-old children.
Advantages l With similar conditions, a significant but not a complete
l The possibility of fluorosis is minimal. reduction in the permanent tooth dental caries experi-
l It is safe. ence had been achieved in those aged 12–14 years.
l It requires low cost.

l Individual monitoring is not required.


Q.6. Topical fluorides.
l It is freely available. Ans.
l Its distribution can be easily monitored.

l Supply for salt fluoridation can be effectively


Topical fluoride is the most effective of anticaries agents.
controlled. Topical fluorides can be divided into:
l It is readily accepted, as the addition of fluoride
A. Professionally applied
does not alter colour. Neutral sodium fluoride (2%)
Stannous fluoride (8%)
Section | I  Topic-Wise Solved Questions of Previous Years 681

Acidulated phosphate fluoride (1.23%) APF gel


Amine fluoride For the preparation of APF gel, a gelling agent meth-
Fluoride gels ylcellulose or hydroxyethyl cellulose is added to the
Fluoride varnishes solution and the pH is adjusted between 4 and 5.
B. Self-applied fluorides Method of application
Toothbrushing dentifrices l Oral prophylaxis.

Toothbrushing solutions or gels l Teeth are isolated with cotton rolls on both lin-

Toothbrushing prophylaxis pastes gual and buccal sides.


Applying gels in trays l Teeth are dried.

l APF solution is continuously and repeatedly


Mouth rinses
applied with cotton applicators.
i. Sodium fluoride application
l Teeth are kept moist for 4 min.
Method of application (Knutson’s technique)
Recommended frequency
l Initially, cleaning and polishing of the teeth is
The recommended frequency of APF topical appli-
done.
cation is twice a year.
l An upper and opposing lower quadrant is iso-
iv. Fluoride varnish
lated with cotton rolls.
Method of varnish application
l Teeth are dried thoroughly.
l Oral prophylaxis.
l 2% NaF is applied with cotton applicators and is
l Teeth are dried.
permitted to dry in the teeth for about 4 min.
l Teeth are not isolated with cotton rolls as varnish,
l Procedure is repeated for the remaining quadrants.
being sticky, has a tendency to stick to cotton.
l After completion, patient is instructed to avoid
l The application is done first on lower arch as
eating, drinking or rinsing for 30 min.
saliva collects more rapidly around it, and then
l Second, third and fourth applications are done at
on the upper arch.
weekly intervals.
l Application of varnish is done with single tufted
Recommended ages
small brush.
Full series of four treatments is recommended at the
l After application, patient is made to sit with
age of 3, 7, 11 and 13 years.
mouth open for 4 min.
ii. Stannous fluoride
l Patient is instructed not to rinse or drink anything
l A 2% NaF solution can be prepared by dissolv-
at all for 4 h and not to eat anything solid but take
ing 20 g of NaF powder in 1 L of distilled water
only liquids and semisolids till next morning.
in a plastic bottle.
l It is essential to store fluoride in plastic bottles Q.7. Fluoride varnishes.
because if stored in glass containers, the fluoride
Ans.
ion of solution can react with silica of glass,
forming SiF, thus reducing the availability of free l Fluoride varnishes are developed in order to increase the re-
active fluoride for proposed anticaries action. tention of topical fluoride on the enamel for a longer period.
Method of application (Muhler’s technique) l To enhance the caries inhibitory property of topical

l The recommended procedure for application of fluorides, a new coating method was developed in
SnF2 begins with thorough prophylaxis. which the teeth was coated with a lacquer-containing
l The teeth are then isolated with cotton rolls and fluoride called F-lacquer, which released fluoride ions
dried with compressed air. to the dental enamel in high concentrations for several
l Either a quadrant or half of the mouth can be hours in the atmosphere of the mouth.
treated at one time.
The two most commonly used varnishes:
l A freshly prepared 8% solution of SnF2 is ap-
l Duraphat (NaF varnish)
plied continuously to the teeth with cotton ap-
l Fluor protector (silane fluoride)
plicator and reapplication of the solution to a
particular tooth is done every 15–30 s so that the Composition
teeth are kept wet for 4 min. l Fluor protector is a colourless polyurethane lacquer.

iii. Acidulated phosphate fluoride – 1.23% l The fluoride compound is a difluorosilane-ethyl-

Method of preparation (Brudevold’s solution) difluorohydroxysilane.


It is prepared by dissolving 20 g of sodium fluoride l The active fluoride available is 7000 ppm.

in 1 L of 0.1 M phosphoric acid. To this is added l Duraphat is a sodium fluoride in varnish form contain-

50% hydrofluoride acid to adjust the pH at 3.0 and ing 22.6 mg F/mL suspended in an alcoholic solution of
fluoride concentration at 1.23%. natural organic varnishes.
682 Quick Review Series for BDS 4th Year, Vol 1

l It is available in bottles of 30 mL suspension contain- Q.9. Milk fluoridation.


ing 50 mg NaF/mL. The active fluoride available is
Ans.
22,600 ppm.
l Milk is a reasonable vehicle for fluoride since it is a
Method of varnish application
food used universally by infants, pregnant women and
l Oral prophylaxis.
children.
l Teeth are dried
l Milk is an excellent source of calcium and phosphorus,
l Teeth are not isolated with cotton rolls as varnish being
and when fortified with vitamin D, it contains all essen-
sticky has a tendency to stick to cotton.
tials for the development of bones and teeth.
l The application is done first on lower arch as saliva col-
l Fluoridation of milk was first mentioned by Ziegler in
lects more rapidly around it, and then on the upper arch.
1956. The rate of absorption of fluoride from milk
l Application of varnish is done with single tufted small
and water has a significant reduction in fluoride ab-
brush.
sorption from milk during the first hour, but thereaf-
l After application, patient is made to sit with mouth open
ter absorption continues at higher levels for longer
for 4 min.
periods of time.
l Patient is instructed not to rinse or drink anything at all

for 1 h and not to eat anything solid but take only liquids Compounds used for milk fluoridation
and semisolids till next morning. l Calcium fluoride
Recommended dose l Sodium fluoride

l The recommended dose of 0.5 mL of duraphat for sin- l Disodium monofluorophosphate

gle application contains 11.3 mg F, and 0.5 mL of fluor l Disodium silicofluoride

protector contains 3.1 mg F.


Feasibility of milk fluoridation in India
Q.8. Fluorides mouthwashes. Although milk fluoridation is advantageous, this method
does not seem to be viable and feasible because of variable
Ans.
supply and consumption of milk.
Advantage
{SN Q.6}
l Milk is a staple food for children and infants.
Fluoride mouth rinses
Disadvantages
l Mouth rinsing is a practical and effective means
l Cost of fluoridated milk would be considerably higher.
of self-application of fluoride.
l There should be a centralized milk supply.
l The only persons excluded from the practice of
l There are variations in intake and quantity of milk.
this method are children under 6 years of age and
those who cannot rinse because of oral-facial Q.10. Acute bifluoride toxicity.
musculature problems or any other handicap.
Ans.
Method of preparation
The procedure of making a rinse everyday is by dis-
solving 200 mg NaF tablet (10 mg NaF rest the filler {SN Q.2}
as lactose) in 5 teaspoons of fresh water (25 mL
approx.), which is sufficient for mouth rinse of a Acute fluoride toxicity
family of about four members. l Ingestion of large doses of fluoride at one time.

l Probably toxic dose (PTD) is defined as the

threshold dose that could cause serious or life-


Method of use threatening systemic signs and symptoms.
l Rinse daily with 1 teaspoonful (5 mL) brushing before bed. l Safely tolerated dose: 8–16 mg/kg body weight.
l Swish between teeth with lips tightly close for 60 s and l Toxic dose: 16–32 mg/kg body weight.
expectorate. l Lethal dose: 32–64 mg/kg body weight.
l Fluoride rinses can be used as daily mouthwash by com-

munity, and fortnightly in schools.

Factors affecting acute toxicity


{SN Q.6}
l Bioavailability

Advantage l Route of administration

A 30%–40% average reduction in dental caries incidence. l Age

Disadvantage l Rate of absorption

Requires community participation. l Acid–base status


Section | I  Topic-Wise Solved Questions of Previous Years 683

Q.12. APF.
{SN Q.2}
Ans.
Signs and symptoms
l Nausea, vomiting, abdominal pain, diarrhoea [Same as SE Q.2]
l Excess salivation and mucosal discharge
Q.13. APF solution.
l Generalized weakness and carpopedal spasms

l Weak thready pulse, fall in blood pressure Ans.


l Depression of respiratory centre
[Same as SE Q.2]
l Decreased plasma calcium level, increased plasma

potassium level Q.14. Brudevold’s solution.


l Cardiac arrhythmia
Ans.
l Coma and death

[Same as SE Q.2]
Management Q.15. Fluoridated salt.
Immediate treatment aimed at reducing fluoride absorption:
l Inducing vomiting
Ans.
l Fluid replacement [Same as SE Q.3]
l Monitoring levels of plasma calcium and potassium
Q.16. Nalgonda technique.
Emergency treatment for fluoride overdose Ans.
Amount of [Same as SE Q.4]
fluoride (mg/kg
body weight of
Q.17. Tropical fluoride application.
fluoride ion) Treatment Ans.
Less than i. Give calcium orally (milk) to relive GI
[Same as SE Q.6]
5.0 mg/kg symptoms. Observe for a few hours.
ii. Induced vomiting not required.
More than i. Empty stomach by induced vomiting with
SHORT NOTES:
5 mg/kg emetic. For patients with depressed gag Q.1. Fluorosis index.
reflex, Down syndrome or severe mental
retardation, induced vomiting is contrain- Ans.
dicated and endotracheal intubation
should be performed before gastric lavage. [Ref LE Q.4]
ii. Give orally soluble calcium in any form
(e.g. milk, 5% calcium gluconate, cal- Q.2. Acute fluoride poisoning.
cium lactate solution). Admit to hospital
and observe for a few hours.
Ans.

More than 15 l Admit to hospital immediately. [Ref SE Q.10]


mg/kg l Induce vomiting.
l Begin cardiac monitoring and be pre-
Q.3. Chocking phenomenon.
pared for cardiac arrhythmia. Ans.
l Slowly administer intravenously 10 mL
of 10% calcium gluconate solution. Ad- Choking phenomenon occurs during application of sodium
ditional doses may be given if clinical fluoride.
signs of tetany or Q-T interval prolon-
gation develop; electrolytes, especially
It can be explained as follows:
calcium and potassium, should be l When NaF is applied topically, it reacts with hydroxy-

monitored using diuretics if necessary. apatite crystals to form CaF2.


l Adequate urine output should be main- l Due to high concentration of fluoride, there is initial
tained using diuretics if necessary. formation of CaF2 on the tooth surface, which prevents
l General supportive measures for shock.
further entry of fluoride into the tooth. This is called
choking-off effect, as it blocks the diffusion.
Q.11. Mechanism of actions of fluoride in preventing Q.4. Certain lethal dose of fluoride.
dental caries.
Ans.
Ans.
l A lethal dose is the amount of drug likely to cause death.
[Same as SE Q.1] l Lethal dose: 32–64 mg/kg body weight.
684 Quick Review Series for BDS 4th Year, Vol 1

Signs and symptoms: l One of several effective alternatives for prevention of


l Nausea, vomiting, abdominal pain, diarrhoea dental caries in children of communities where water
l Excess salivation and mucosal discharge fluoridation is not feasible is the fluoridation of the
l Generalized weakness and carpopedal spasms school water supply.
l Weak thready pulse, fall in blood pressure Advantages
l Depression of respiratory centre l No effort required by the recipients

l Decreased plasma calcium level, increased plasma l Good results in reducing caries

potassium level l Minimal equipment

l Cardiac arrhythmia, coma and death l Not expensive

Disadvantage
Q.5. Indications of dietary fluoride supplements.
l Children do not receive the benefit until they go to

Ans. school. It allows only fewer benefits in the primary


dentitions.
Dietary fluoride supplements are available in the following
forms: Q.9. Salt fluoridation.
0.125, 0.25 and 0.50 mg drops
Ans.
0.25, 0.50 and 1.0 mg tablets/lozenges
1 mg, 5 mL oral rinse supplements [Ref SE Q.3]
Indications of dietary fluoride supplements:
i. In areas where there are no central water supplies Q.10. Optimum levels of fluoride.
ii. As an interim measure in those communities with a Ans.
central water system that have not yet implemented
community water fluoridation l The optimum level of fluoride in the water is in the
iii. In areas where water fluoridation or salt fluoridation range of 0.7–1.2 ppm.
l This range effectively reduces tooth decay, while mini-
schemes cannot be implemented
mizing the occurrence of dental fluorosis.
Q.6. Fluoride mouth rinses.
WHO-recommended levels of fluoride in water
Ans. l According to WHO (1971), the recommended level of
[Ref SE Q.8] fluoride in water is as follows:
i. The concentration of fluoride is 0.7–1.2 ppm.
Q.7. Fluoride tablets. ii. Depending on the community’s mean, maximum
Ans. daily temperature, the concentration of fluoride is
cold climate – 1.2 ppm
l Fluoride tablets provide systemic effect before mineral- temperate climate – 0.7 ppm
ization of primary and permanent teeth, and a topical l According to WHO (1994), the recommended optimum
effect thereafter. level of fluoride in drinking water is 0.5–1.0 ppm.
l Caries reduction in the range of 50%–80% has been
reported when fluoride administration in the form of Q.11. Fluor protector.
tablets is started before 2 years of age and continued for Ans.
a minimum of 3–4 years.
l Fluor protector is a colourless polyurethane lacquer
Availability dissolved in chloroform and dispensed in 1 mL
l Fluoride tablets are commercially available as NaF
ampoules.
tablets of 2.2 mg, 1.1 mg and 0.55 mg, yielding 1 mg, l This product contains 2% difluorosilane.
0.5 mg and 0.25 mg fluoride, respectively. l Silane fluoride of fluor protector reacts with water to
l Sodium fluoride tablets with vitamin combinations are
produce considerable amount of hydrofluoric acid (HF),
also available. which penetrates into enamel more readily than fluoride.
Recommended dose Q.12. Milk fluoridation.
l The daily recommended doses of fluoride for children

below 2 years is 0.5 mg, between 2 and 3 years is Ans.


0.5–0.7 mg and above 3 years is 1–1.5 mg.
l Milk is a reasonable vehicle for fluoride since it is a food
Q.8. School water fluoridation. used universally by infants, pregnant women and children.
l Milk is an excellent source of calcium and phosphorus,
Ans.
and when fortified with vitamin D it contains all essen-
l School water fluoridation was first initiated as a pilot tials for the development of bones and teeth.
study in 1954 at St. Thomas, Virgin Islands, USA. l Fluoridation of milk was first mentioned by Ziegler in 1956.
Section | I  Topic-Wise Solved Questions of Previous Years 685

Advantage excessive amounts of fluoride during the period when


l Staple food for children and infants teeth are developing.
Disadvantages . Skeletal fluorosis
b
l Cost of fluoridated milk would be considerably higher. There is heavy deposition of fluoride in the skeleton.
l Centralized milk supply should exist. When a concentration of 10 mg/L is exceeded, crippling
fluorosis can ensue, leading to permanent disability.
Q.13. Mention methods of systemic fluoridation.
c. Genuvalgum
Ans. A new form of fluorosis is characterized by genuvalgum
and osteoporosis of lower extremities.
[Ref LE Q.3]
Q.16. Dean’s index.
Q.14. Dental fluorosis.
Ans.
Ans.
[Same as SN Q.1]
l Dental fluorosis can be defined as hypoplasia or hypo-
maturation of tooth enamel or dentine produced by the Q.17. Trendley H. Dean.
chronic ingestion of excessive amounts of fluoride dur-
Ans.
ing the period when teeth are developing.
l There is a direct inhibitory effect on enzymatic action of [Same as SN Q.1]
ameloblasts, leading to defective matrix formation and
Q.18. Details about acute fluoride toxicity and management.
subsequent hypomineralization.
l Tooth absorbs stains from oral cavity due to its porous Ans.
structure. This discolouration can be reduced to certain
[Same as SN Q.2]
degree by bleaching the teeth externally.
l Major cause is consumption of water containing high Q.19. Lethal dose of fluoride.
levels of fluoride during the first 6 years of life.
Ans.
l Both primary and permanent teeth will be affected, but

greater fluorosis in permanent teeth is seen because much [Same as SN Q.2]


of the mineralization of primary teeth occurs before birth.
Q.20. What are the signs and symptoms of acute fluo-
Q.15. Endemic fluorosis. ride toxicity?
Ans. Ans.
Endemic fluorosis has been observed in many parts of the world [Same as SN Q.2]
where drinking water contains excessive amounts of fluorine.
Q.21. Dosage of fluoride tablets.
Endemic fluorosis results in
a. Dental fluorosis Ans.
It can be defined as hypoplasia or hypomaturation of tooth
[Same as SN Q.7]
enamel or dentine produced by the chronic ingestion of

Topic 7
Indices Used in Preventive Dentistry
COMMONLY ASKED QUESTIONS
LONG ESSAYS:
1 . Define index. Classify them. Add a note on DMFT and DMFS indices and their limitations.
2. Define index and classify them. Write about the ideal requisites of an index.
3. Define an index. What are the objectives of an index? Describe the Russell’s periodontal index.
4. Describe the index. Describe the ideal requirements of an index. Write briefly about CPITN index.
5. What is an epidemiological index? What are the ideas requisite of an index? How indices are classified? Write
scoring criteria of Dean’s fluorosis index.
686 Quick Review Series for BDS 4th Year, Vol 1

6. What is an index? Give broad classification of indices used in dental epidemiologic investigations and describe
in detail Silness and Loe dental plaque index.
7. Discuss in detail the Ramfjord periodontal diseases index, and also mention the advantages and limitations of
the same.
8. Define an index and describe the various indices used for the survey of gingival diseases.
9. Write about development, objectives, scoring system, advantages and limitations of Greene and Vermillion’s
simplified oral hygiene index.
10. Define an index. Describe the indices used for dental caries. [Same as LE Q.1]
11. Discuss in detail ideal requisites of an index and evaluate critically advantages and disadvantages of DMFT and
DMFS indices. [Same as LE Q.1]
12. Enumerate different indices used for the assessment of dental caries in the community, and describe any one
index in detail. [Same as LE Q.1]
13. Define index, give the classification of indices. What are the ideal requisites of an index? [Same as LE Q.2]
14. Define and describe ideal characteristics of indices. Write in detail about Russell’s periodontal index. 
[Same as LE Q.3]
15. Define index. Which are the ideal requisites of an index? Write in detail about community periodontal index of
treatment needs. [Same as LE Q.4]
16. What is CPITN index? Give its criteria of scoring system and mention its advantages and limitations. [Same as LE Q.4]

SHORT ESSAYS:
1. CPITN index. [Ref LE Q.4]
2. Debris index simplified. [Ref LE Q.9]
3. Gingival index of Loe and Silness. [Ref LE Q.8]
4. Russell’s index. [Ref LE Q.3]
5. Dean’s fluorosis index. [Ref LE Q.5]
6. Uses of an index.
7. Irreversible indices. [Ref LE Q.1]
8. PMA index. [Ref LE Q.8]
9. CPI index.
10. Rules governing DMF index. [Ref LE Q.1]
11. Caries index for primary dentition.
12. Oral hygiene index simplified. [Ref LE Q.9]
13. CPITN probe.
14. Sulcus bleeding index. [Ref LE Q.8]
15. Moller index.
16. Silness and Loe plaque index. [Ref LE Q.6]
17. Plaque and calculus components of periodontal disease index. [Ref LE Q.7]
18. Scoring criteria of CPITN. [Same as SE Q.1]
19. Loe and Silness index. [Same as SE Q.3]
20. Periodontal index Russell. [Same as SE Q.4]
21. DMFT index. [Same as SE Q.7]
22. Rules governing caries indices. [Same as SE Q.10]

SHORT NOTES:
1 . DMFT index. [Ref LE Q.1]
2. Advantages and limitations of CPITN index. [Ref LE Q.4]
3. Classify indices in dentistry. [Ref LE Q.5]
4. Ideal properties of an index. [Ref LE Q.5]
5. Russell’s index. [Ref LE Q.3]
6. OHI-S. [Ref LE Q.9]
7. Dental caries index for mixed dentition.
8. Write a short note on the CPITN probe.
9. Scoring codes in CPITN index. [Ref LE Q.4]
Section | I  Topic-Wise Solved Questions of Previous Years 687

1 0. Community fluorosis index. [Ref LE Q.5]


11. Scoring criteria of Russell’s index. [Ref LE Q.3]
12. Community periodontal index (CPI). [Ref SE Q.9]
13. Reversible index. [Ref LE Q.1]
14. Mention any four requisites for ideal index designing in epidemiological survey. [Same as SN Q.4]
15. Mention the ideal requirements of index system in epidemiology. [Same as SN Q.4]
16. Russell’s periodontal index. [Same as SN Q.5]
17. CPITN probe. [Same as SN Q.8]

SOLVED ANSWERS
LONG ESSAYS:
Q.1. Define index. Classify them. Add a note on DMFT ii. Simplified indices
and DMFS indices and their limitations. Simplified indices measure only a representative
Ans. sample of the dental apparatus. For example,
Greene and Vermillion’s Oral Hygiene Index-
An index is defined as a numerical value describing the rela-
Simplified (OHI-S).
tive status of a population on a graduated scale with definite
Indices may be classified in certain general catego-
upper and lower limits, which is designed to permit and facili-
ries according to the entity which they measure, like:
tate comparison with other populations classified by the same
(a) Disease index
criteria and methods (A.L. Russell).
(b) Symptom index
Dental index is an abbreviated measurement of the amount or
(c) Treatment index
condition of disease in a population; a numerical scale with
l The ‘D’ (decay) portion of the DMF index
defined upper and lower limits designed to permit and facili-
best exemplifies a disease index.
tate comparison with other populations classified by the same
l The indices measuring gingival/sulcular
criteria and method (Pamela Zarkowski).
bleeding are essentially symptom indices.
Depending on the direction in which their scores can l The ‘F’ (filled) portion of the DMF index
fluctuate, indices are classified as either reversible or irre- best exemplifies a treatment index.
versible. In general, there are two types of dental indices.
i. Irreversible index i. The first type of index measures the ‘num-
The index that measures conditions that will not change ber’ or ‘proportion’ of people in a popula-
is called irreversible index. In irreversible index, scores, tion with or without a specific condition at
once established, cannot be decreased in value on sub- a specific point in time or interval of time.
sequent examinations. For example, irreversible index ii. The second type of dental index measures
measures dental caries. the ‘number’ of people affected and the
‘severity’ of the specific condition at a spe-
cific time or interval of time.
{SN Q.13} Dental indices can also be classified under spe-
ii. Reversible index cial categories as follows:
The index that measures conditions that can be l Simple Index

changed is called reversible index. Reversible index Simple index measures the presence or absence
scores can increase or decrease on subsequent ex- of a condition. For example, an index that
aminations. For example, reversible index measures would measure the presence of dental plaque
periodontal conditions. without an evaluation of its effect on gingiva.
l Cumulative Index

Cumulative index measures all the evidence


Depending on the extent to which areas of oral cavity of a condition, past and present. For exam-
are measured, indices are classified into ‘full mouth’ or ple, DMF index for dental caries.
‘simplified’. Decayed-missing-filled teeth index (DMFT index)
i. Full mouth indices
{(SE Q.7 and SN Q.1)}
Full mouth indices measure the patient’s entire
periodontium or dentition. For example, Russell’s l {(The decayed-missing-filled teeth index (DMFT
periodontal index (PI). index) was developed in 1938 by Henry T. Klein,
688 Quick Review Series for BDS 4th Year, Vol 1

Carrole E Palmer and J.W. Knutson to determine the (SE Q.7 and SE Q.10)
prevalence of coronal caries.
l The DMFT index is a simple, rapid, versatile and
{(Principles and rules in recording DMFT
universally accepted and applicable measurement i. A tooth must not be counted more than once. It
that has been used widely for several decades. is counted as a decayed, missing, filled or
l This index is based on the fact that the dental hard
sound tooth.
tissues are not self-healing; established caries leaves ii. Decayed, missing and filled teeth should be
a scar of some sort. The tooth either remains decayed recorded separately and be counted because the
or, if treated, is extracted or filled. components of DMF are of great interest.
l The DMFT index is therefore an irreversible in-
iii. When counting the number of decayed teeth,
dex, meaning that it measures total lifetime caries the teeth that have restorations with recur-
experience. rent decay must also be counted as decayed
Procedure/method teeth.
The DMFT index, applied only to permanent teeth, is iv. Care must be taken to list as missing only the
composed of three components: the D-component for teeth that have been lost due to decay. Also,
‘Decayed’, the M-component for ‘Missing’ and the included should be the teeth that are so badly
F-component for ‘Filled’. As described by the authors, decayed that they are indicated for extraction.
‘D’ – used to describe decayed teeth The following teeth should not be counted as
‘M’ – used to describe missing teeth due to caries missing teeth:
‘F’ – used to describe teeth that have been previously filled (a) Unerupted teeth
To obtain the DMFT index, the examiner, under favour- (b) Teeth that have been extracted for orth-
able lighting conditions and using a No. 3 plain mirror and odontic purpose
a fine pointed pig tail explorer, will determine the sum of (c) Teeth missing due to accident
how many teeth are ‘Decayed’, ‘Missing’ or extracted due (d) Teeth that are congenitally missing
to decay, and ‘Filled’ with either a permanent or a tempo- v. A tooth may have several restorations but it is
rary restoration as a result of caries involvement.)} counted as one tooth.
vi. Deciduous teeth are not included in DMF count.
[SE Q.7] vii. A tooth is considered to be erupted when the
occlusal surface or incisal edge is totally ex-
{All 28 permanent teeth are examined. The teeth not posed or can be exposed by gently reflecting
included are
the overlying gingival tissue with a mirror or
l Third molars
explorer.
l Unerupted teeth
viii. A tooth is considered to be present even though
l Congenitally missing and supernumerary teeth
the crown has been destroyed and only the roots
l Teeth removed for reasons other than dental caries,
are left.
such as for orthodontic treatment or impaction
ix. The following criteria should be considered
l Teeth restored for reasons other than dental caries
when determining the ‘care needed’:
such as trauma (fracture), cosmetic purposes or
l Tooth decay
for use as a bridge abutment
l Gingival tissue
l Primary tooth retained with permanent successor
l Malocclusion
erupted; the permanent tooth is evaluated since a
l Abscess
primary tooth is never included in this index
l Retained root tip/root tips
The instruments used in the examination procedure are
l Prophylaxis needed
a mouth mirror and an explorer.
l Appliances needed

Following are the criteria for identification of dental WHO modification of DMF index (1986)
caries: i. All third molars are included here.
a. The lesion is clinically visible and obvious. ii. All the temporary restorations are considered as
b. The explorer tip gives a catch; it can penetrate deep into decayed teeth, ‘D’.
soft demineralized enamel. iii. Only carious cavities are considered as ‘D’, the
c. There is blackish discolouration or loss of translucency initial lesions (chalky spots, stained fissures
typical of undermined or demineralized enamel. etc.) are not considered as decayed teeth ‘D’.
d. The explorer tip in a pit or fissure catches or resists The DMF index can be applied to denote the
removal after moderate to firm pressure on insertion, number of affected teeth (DMFT) or to measure
and when there is softness at the base of the area.} the surfaces affected by dental caries (DMFS).)}
Section | I  Topic-Wise Solved Questions of Previous Years 689

(SE Q.7 and SE Q.11) (B) Group average


Total the D, M and F for each individual. Then
{(Examination method for DMF (permanent teeth only) divide the total ‘DMF’ by the number of indi-
‘D’ – Decayed
viduals in the group, i.e.
Indicates the number of permanent teeth that are
decayed. In counting the number of decayed perma- Total DMF
nent teeth, remember that a tooth can only be Average DMF 5
Total number of the subjects examined
counted once. It cannot be counted as decayed and
filled. If it has been restored and caries are detected, (C) Per cent needing care
count it as decayed. Be sure that the explorer falls To calculate the per cent of teeth needing resto-
into carious tooth substance and not just in a deep rations, divide the total ‘D’ component by the
groove before counting occlusal caries. total number examined, i.e.
‘M’ – Missing
Indicates the number of missing permanent teeth Total number of decayed tooth
Per cent needing care 5
due to decay. Teeth that are so badly decayed that Total number examined
they are indicated for extraction are counted as
missing. Teeth that have been extracted for orth- (D) Per cent of teeth lost
odontic reasons, unerupted permanent teeth, miss- To calculate the per cent of teeth lost, divide the total
ing teeth because of accident or any congenitally ‘M’ component by the total number examined, i.e.
missing teeth are not included. When possible, Total number of missing teeth
histories should be taken when it is suspected that Per cent of teeth lost 5
Total number examined
teeth have been lost for reasons other than caries.
‘F’ – Filled (E) Per cent of filled teeth
Indicates the number of permanent teeth that have To calculate the per cent of filled teeth, divide the
been attacked by caries but have been restored to total ‘F’ component by the total ‘DMFT’, i.e.
keep them in a healthy condition in mouth. A tooth
may have several fillings but it is counted as one Total number of filled teeth
Per cent of filled teeth 5
tooth. If a tooth has a filling but shows evidence of Total 'DMFT'
recurrent decay, it is counted as a decayed tooth.
To calculate the missing permanent teeth per
Coding Criteria for DMF Index 100 children (MPT/100), divide the total num-
ber of missing teeth by the total number exam-
Code Criteria ined and then multiply by 100, i.e.
E Excluded tooth or tooth space
Total number of missing teeth  1 0 0
1 Sound permanent tooth MPT / 100 
Total number examined
2 Filled permanent tooth
3 Decayed permanent tooth The maximum possible DMFT scores are 32 (if
third molars are included) and 28 (if third mo-
In case tooth is absent:
lars are excluded).
‘0’ Missing tooth – unerupted, impacted and congenitally Limitations of DMFT index
missing
i. DMFT values are not related to the number of teeth
‘X’ Extracted permanent tooth) } at risk.
ii. DMFT index can be invalid in older adults because
teeth can become lost for reasons other than caries.
[SE Q.7]
iii. DMFT index can be misleading in children whose
{Calculation of the index teeth have been lost due to orthodontic reasons.
The maximum individual DMFT score is 28 or 32 if the iv. DMFT index can overestimate caries experience in
wisdom teeth are included. For example, a DMFT score teeth in which ‘preventive fillings’ have been placed.
of 312155 10 for an individual means that three teeth v. DMFT index is of little use in studies of root caries.
are decayed, two teeth are missing and five teeth have Decayed-missing-filled-tooth surface index (DMFS)
fillings. Furthermore, it also means that 18 (i.e. 28 – 10 The decayed-missing-filled permanent tooth surfaces
5 18) teeth are intact.} (DMFS) index was developed by Henry T. Klein,
(A) Individual DMFT Carrole E. Palmer and J.W. Knutson in 1938 along with
Total each component, i.e. D, M and F sepa- the decayed-missing-filled permanent teeth (DMFT)
rately, then total D 1 M 1 F 5 DMF. index to assess the prevalence of coronal caries.
690 Quick Review Series for BDS 4th Year, Vol 1

Procedure/method An index is defined as a numerical value describing the


l The DMFS index has three components: the D- relative status of a population on a graduated scale with
component for ‘Decayed’, the M-component for definite upper and lower limits. It is designed to permit and
‘Missing’, and the F-component for ‘Filled’. DMFS facilitate comparison with other populations classified by
is a more detailed index than the DMFT by summing the same criteria and methods (A.L. Russell).
the total number of decayed, missing and filled per- Dental index is an abbreviated measurement of the
manent tooth surfaces. amount or condition of a disease in a population; a numerical
l The DMFS index is more simple and versatile than scale with defined upper and lower limits designed to permit
DMFT index. DMFS index has practically universal and facilitate comparison with other populations classified
acceptance and is one of the best-known dental indi- by the same criteria and method (Pamela Zarkowski).
ces today. It is calculated for each subject and can be Depending on the direction in which their scores can
averaged over subsets of the population. fluctuate, indices are classified as either reversible or irre-
l The principles, rules and criteria for DMFS index are versible.
similar to that for DMFT index, which has been described i. Irreversible index
previously along with description of DMFT index. The index that measures conditions that will not change
To obtain the DMFS index, the examiner requires a is called irreversible index. In irreversible index, scores,
proper lighting. Using a No. 3 plain mirror and a fine- once established, cannot be decreased in value on sub-
pointed pigtail explorer, all the teeth are examined to sequent examinations. For example, irreversible index
find the index: ‘Decayed’, ‘Missing’ or Extracted due measures dental caries.
to decay, and ‘Filled’ with either a permanent or a tem- ii. Reversible index
porary restoration as a result of caries involvement. The index that measures conditions that can be changed is
Surfaces examined called reversible index. Reversible index scores can in-
l For posterior teeth: Five surfaces are examined and crease or decrease on subsequent examinations. For ex-
recorded – facial, lingual, mesial, distal and occlusal. ample, reversible index measures periodontal conditions.
l For anterior teeth: Four surfaces are examined and
Depending on the extent to which areas of oral cavity are
recorded – facial, lingual, mesial and distal.
measured, indices are classified into ‘full mouth’ or ‘simplified’.
For permanent dentition, the maximum DMFS score
i. Full mouth indices
is 128 (i.e. surfaces) for 28 teeth, or 148 for 32 teeth.
Full mouth indices measure the patient’s entire peri-
Total surface count for a DMFS index
odontium or dentition. For example, Russell’s periodon-
If 28 teeth are examined (i.e. third molars are excluded)
tal index (PI).
16 posterior teeth (16 3 5) 5 80 surfaces
ii. Simplified indices
12 anterior teeth (12 3 4) 5 48 surfaces
Simplified indices measure only a representative sample
Total 5 128 surfaces.
of the dental apparatus. For example, Greene and
If third molars are included (4 3 5) 5 20 surfaces
Vermillion’s oral hygiene index simplified (OHI-S).
Total 5 148 surfaces.
Individual DMFS index Indices may be classified in certain general categories
Total number of decayed surfaces 5 D according to the entity that they measure, such as:
Total number of missing surfaces 5 M (a) Disease index
Total number of filled surfaces 5 F (b) Symptom index
DMFS score 5 D 1 M 1 F (c) Treatment index
l The DMFS index is more sensitive. It is usually l The ‘D’ (decay) portion of the DMF index best ex-

the index of choice in a clinical trial of a caries emplifies a disease index.


preventive agent. l The indices measuring gingival/sulcular bleeding are
l This is because relative incidence is more likely to essentially symptom indices.
be detected over the limited period of clinical trial. l The ‘F’ (filled) portion of the DMF index best exem-

l But DMFS examination takes longer, and is more plifies a treatment index.
likely to produce inconsistencies in diagnosis In general, there are two types of dental indices.
and may require the use of radiographs to be fully i. The first type of index measures the ‘number’
accurate. or ‘proportion’ of people in a population with
or without a specific condition at a specific
Q.2. Define index and classify them. Write about the point in time or interval of time.
ideal requisites of an index. ii. The second type of dental index measures the
Ans. ‘number’ of people affected and the ‘severity’
Section | I  Topic-Wise Solved Questions of Previous Years 691

of the specific condition at a specific time or inter- completely meets all of these conditions, but choice
val of time. of an index in any given situation should be made
Dental indices can also be classified under special on the basis of how closely the index approximates
categories as follows: them and by the requirements of the study in which
l Simple index the index is being used.
Simple index measures the presence or absence of There is often a trade-off between sensitivity and
a condition. For example, an index that would specificity and between precision and reliability.
measure the presence of dental plaque without an For example, caries used to be diagnosed using a
evaluation of its effect on gingiva. sharpened probe. While this technique was very
l Cumulative index good at finding cavities (sensitivity), it often over-
Cumulative index measures all the evidences of a diagnosed caries when the probe became wedged
condition – past and present. For example, DMF in a fissure pattern (low specificity). Likewise,
index for dental caries. periodontal probes with fewer gradations (e.g.
Ideally, an index should possess the following CPI probe) are less precise but provide more reli-
properties: able data than those with finer gradations.
l Clarity, simplicity and objectivity

The examiner should be able to carry the rules of Q.3. Define an index. What are objectives of an index?
the index in his or her head, and the index should Describe the Russell’s periodontal index.
be reasonably easy to apply so that there is no
Ans.
undue time lost during field examination. The
criteria for the index should be clear and unam- Index is defined as a numerical value describing the relative
biguous with mutually exclusive categories. status of a population on a graduated scale with definite
l Validity upper and lower limits, and is designed to permit and
The index must measure what it is intended to facilitate comparison with other populations classified by
measure, so it should correspond with clinical the same criteria and methods (A.L. Russell).
stages of the disease under study at each point. To Dental index is an abbreviated measurement of the
illustrate this, the number of bacteria in a sample amount or condition of disease in a population; a numerical
of saliva is not a valid measure of bacteria in den- scale with defined upper and lower limits designed to per-
tal plaque, nor is the number of missing teeth in mit and facilitate comparison with other populations classi-
an adult necessarily a valid measure of caries ac- fied by the same criteria and method (Pamela Zarkowski).
tivity, because the teeth can be lost for reasons The main purpose or objective of using indices in dental
other than caries. epidemiology is to increase understanding of the disease
l Reliability process, thereby leading to methods of control and preven-
The index should measure consistently at differ- tion. In addition, it attempts to discover populations at high
ent times and under a variety of conditions. The and low risk and to define a specific problem under investi-
term ‘reliability’ is virtually synonymous with gation.
reproducibility, which means the ability of the The simplest form of measuring any disease is by the
same or different to interpret and use the index in count of the number of cases of its occurrence. But with
the same way. oral diseases, simple counts of cases are often of limited
l Quantifiability use because of high prevalence of these conditions in many
The index should be amenable to statistical populations.
analysis so that the status of a group can be ex- A simple count can be made more useful by turning it
pressed by a number that corresponds to a rela- into a proportion, which is done by providing a denomina-
tive position on a scale from zero to the upper tor to the count. In oral diseases, proportions are useful
limit. when looking at the conditions that occur relatively uncom-
l Sensitivity monly, such as cleft lip and palate or oral cancers. But with
The index should be able to detect reasonably more prevalent conditions of dental caries and periodontal
small shifts in either direction in the group condi- diseases, proportions are of limited use except in those
tion. populations or subgroups in which the conditions occur less
l Acceptability frequently.
The use of index should not be painful or demean- As an example, a measure of the proportion of the
ing to the subject. population suffering from dental caries in many parts of
In practice, no index or measure is wholly accurate, India is a useful measure, because caries is still relatively
and probably no index used in oral epidemiology infrequent there.
692 Quick Review Series for BDS 4th Year, Vol 1

(SE Q.4 and SN Q.5) tooth are considered a scoring or gingival unit) are
assessed for gingival inflammation and periodontal
{(The periodontal index (PI) was developed in 1956 by involvement.
A.L. Russell. This index was developed over a trial period
Scoring criteria
of 10 years, because of the lack of sophisticated method-
Russell chose the scoring values (0, 1, 2, 6, 8) to relate
ologies to assess prevalence and severity of gingivitis and
the stages of the disease in an epidemiological survey to
destructive periodontal diseases.)}
the clinical conditions observed.
[SE Q.4] Russell’s rule
The Russell’s rule states that ‘when in doubt, assign the
{Initially, epidemiological studies of gingival and periodontal lower score’.)}
diseases in large populations were directed simply towards seg-
regating individuals into subgroups on the basis of the clinical [SE Q.4]
appearance of tissues of anterior teeth. These observations only
permitted the evaluation of the relative proportions of affected
{Calculation of the index
The PI score per individual is obtained by adding all
and unaffected individuals in the populations under consider-
individual scores and dividing by the number of teeth
ation. So, according to WHO (1999), the PI has made great
present or examined, i.e.
strides to the epidemiology of periodontal disease due to its defi-
nition that very quickly achieved wide international acceptance. Sum of individual sco res
PI Score per person 5
However, today this index is not used much in epidemiological Number of teeth present
surveys because of the introduction of new periodontal indices
and refinement of criteria amid increasing periodontal research. Drawbacks of a periodontal index
The PI was intended to estimate deeper periodontal Since only a mouth mirror, and no calibrated probe or
disease by measuring the presence or absence of gingival radiographs, is used when performing PI examination,
inflammation and its severity, pocket formation and masti- the results tend to underestimate the true level of peri-
catory function. PI values on a scale range from 0 to 8 with odontal disease, especially early bone loss, in a popu-
increasing prevalence and severity of disease. The PI is re- lation. The number of periodontal pockets without
ported to be useful among large populations, but it is only obvious supragingival calculus is also underestimated
of limited use for individuals or small groups.} in the PI.}

(SE Q.4 and SN Q.5) [SE Q.4]


{(The PI is probably the most widely used periodontal index {(Uses of periodontal index
in epidemiological surveys around the world. The PI is a l Used in the National Health Survey (NHS), the larg-
composite index because it records both reversible changes est ongoing health survey in the USA.
due to gingivitis and more destructive and presumably irre- l Used in epidemiological surveys.

versible changes brought by deeper periodontal diseases. l More data on periodontal disease can be assembled

Method using PI than any other index.


All teeth present are examined. All gingival tissues cir-
cumscribing each tooth (i.e. all tissues circumscribing a
{SN Q.11}
Scoring criteria for field studies
Score Criteria for field studies Additional radiographic criteria for clinical studies
0 Negative. There is neither overt inflammation in the investing tissue Radiographic appearance is essentially normal
nor loss of function due to destruction of supporting tissue.
1 Mild gingivitis. An overt area of inflammation in free gingival does not
subscribe the tooth.
2 Gingivitis. Inflammation completely circumscribes the tooth but there
is no apparent break in the epithelial attachment.
4 Used only when radiographs are available. There is early notch-like resorption of alveolar crest.
6 Gingivitis with pocket formation. The epithelial attachment has been There is horizontal bone loss involving the entire
broken and there is a pocket (not merely a deepened gingival crevice alveolar crest, up to half of the length of tooth root.
due to swelling in free gingiva). There is no inference with normal
masticatory function; the tooth is firm in its socket and has not drifted.

8 Advanced destruction with loss of masticatory function. The tooth There is advanced bone loss involving more than half
may be loose, drifted, sound dull on percussion with metallic of the tooth root, or a definite infra bony pocket with
instrument or is depressible in its socket. widening of periodontal ligament. There may be root
resorption or rarefaction at the apex.)}
Section | I  Topic-Wise Solved Questions of Previous Years 693

[SE Q.4] WHO and the ‘Federation Dentaire Internationale’


(WHO/FDI).
{Interpretation. l This index was developed primarily to survey and

Clinical condition Individual PI score evaluate periodontal treatment needs rather than de-
termining past and present periodontal status, i.e. the
Clinically normal supportive tissue 0–0.2
recession of gingival margin and alveolar bone. The
Simple gingivitis 0.3–0.9 CPITN is an evolution of the ‘621’ method, named
Beginning destructive periodontal disease 1.0–1.9 for the WHO technical report series publication
number in which this method first featured.
Established destructive periodontal disease 2.0–4.9
l CPITN is the index used frequently for the last decade
Terminal disease 5.0–8.0 } in periodontal epidemiology. Although originally in-
tended as a screening procedure for epidemiological
Q.4. Describe the index. Describe the ideal requirements purposes, the CPITN has been adapted and adopted
of an index. Write briefly about CPITN index. for other purposes: in a promotional role in developing
periodontal health awareness programmes, for initial
Ans.
screening and for monitoring changes in periodontal
Index is defined as a numerical value describing the relative needs of individuals in clinical practice.}
status of a population on a graduated scale with definite
upper and lower limits, and is designed to permit and (SE Q.1 and SN Q.2)
facilitate comparison with other populations classified by {(Major advantages of CPITN
the same criteria and methods (A.L. Russell). i. It is simple.
Dental index is an abbreviated measurement of the ii. Procedure is not time-consuming.
amount or condition of disease in a population; a numerical iii. International uniformity
scale with defined upper and lower limits designed to per- Its limitations include partial recording, exclusion of
mit and facilitate comparison with other populations classi- some important signs of past periodontal breakdown –
fied by the same criteria and method (Pamela Zarkowski). notably attachment loss – and absence of any marker of
Ideally, an index should possess the following properties: disease activity or susceptibility. CPITN, therefore, is
l Clarity, simplicity and objectivity
not a diagnostic tool and should not be used for planning
l Validity
of specific clinical treatment of an individual patient.)}
l Reliability
[SE Q.1]
l Quantifiability

l Sensitivity {Codes and criteria


l Acceptability The appropriate code for each sextant is determined with
respect to the following criteria. The codes are listed in
No index or measure is wholly accurate, and probably no descending order of treatment complexity as follows:
index used in oral epidemiology completely meets all of ‘Code X’
these conditions, but choice of an index in any given situa- When only one tooth or no tooth is present in a sextant
tion should be made on the basis of how closely the index (third molars are excluded unless they function in place
approximates them and by the requirements of the study in of second molars).
which the index is being used. ‘Code 4’
There is often a trade-off between sensitivity and specific- Pathological pocket of 6 mm or more present, i.e. the
ity and between precision and reliability. For example, caries black area of CPITN probe is not visible.
used to be diagnosed using a sharpened probe. While this Note: If the designated tooth or teeth are found to have a
technique was very good at finding cavities (sensitivity), it 6 mm or deeper pocket in the sextant being examined, a
often overdiagnosed caries when the probe became wedged in code of 4 is given to the sextant. Recording of code 4
a fissure pattern (low specificity). Likewise, periodontal probes makes further examination of that sextant unnecessary –
with fewer gradations (e.g. CPI probe) are less precise but There is no need to record presence or absence of patho-
provide more reliable data than those with finer gradations. logical pockets of 4 or 5 mm, calculus or bleeding.
‘Code 3’
[SE Q.1]
Pathological pocket of 4 or 5 mm present, i.e. when the
{Community Periodontal Index of Treatment Needs gingival margin is on the black area of probe.
(CPITN) Note: If the deepest pocket found at the designated
l The Community Periodontal Index of Treatment tooth or teeth in a sextant is 4 or 5 mm, a code of 3
Needs (CPITN) was developed in 1982 by Jukka is recorded – There is no need to examine for calcu-
Ainamo et al. of the Joint Working Committee of lus or gingival bleeding.
694 Quick Review Series for BDS 4th Year, Vol 1

‘Code 2’ reduce inflammation and bring 4- or 5-mm pock-


Calculus or other plaque-retentive factors, such as ill- ets to values of 3 mm or below 3 mm. Thus,
fitting crowns or poorly adapted edges of restorations, sextants with these pockets are placed in the
are either seen or felt during probing. same treatment category as scaling and root
Note: If no pocket depth is observed, which involved planning, i.e. ‘Treatment Need 2’ (TN-2).
or exceeded the coloured area of CPITN probe, but TN-3: A sextant scoring code 4 (6 mm or deeper pock-
supra- or subgingival calculus or other plaque- ets) may or may not be successfully treated by means of
retentive factors are detected, code of 2 is assigned deep scaling and efficient personal oral hygiene mea-
– it is unnecessary to examine for gingival bleeding. sures. Code 4 is therefore assigned to ‘complex treat-
‘Code 1’ ment’, which can involve deep scaling, root planning
Bleeding observed during or after probing. and more complex surgical procedures.}
Note: If neither pathological pocketing nor calculus is
Q.5. What is an epidemiological index? What are the
observed but bleeding occurs after gentle probing, Code 1
ideas requisites of an index? How indices are classified?
is recorded for the sextant. The gingiva of the desig-
Write scoring criteria for Dean’s fluorosis index.
nated tooth or teeth should be inspected for presence or
absence of bleeding before the subject is allowed to Ans.
swallow or close the mouth. At times, bleeding may
Epidemiological indices are attempts to quantitate clinical con-
become evident only 10–30 seconds after probing. Any
ditions on a graduated scale, thereby facilitating comparison
gingival bleeding is scored as Code 1.
among populations examined by the same criteria and methods.
‘Code 0’
Healthy tissue; no signs of disease.
Note: If no findings of treatment are involved (i.e. no {SN Q.4}
pockets, calculus or bleeding), then Code 0 (zero) is
Ideally, an index should possess the following properties:
recorded for the sextant.}
l Clarity, simplicity and objectivity

(SE Q.1 and SN Q.9) l Validity

l Reliability

l Quantifiability
{(Code 0 No periodontal disease (healthy periodontium)
l Sensitivity
Code 1 Bleeding observed during or after probing l Acceptability
Code 2 Calculus or other plaque-retentive factors seen or
felt during probing
Code 3 Pathological pocket 4–5 mm in depth; gingival {SN Q.3}
margin situated on black band of the probe
Depending on the direction in which their scores can
Code 4 Pathological pocket, 6 mm or more in depth; black fluctuate, indices are classified as either reversible or
band of probe is not visible)}
irreversible.
i. Irreversible index
[SE Q.1] The index measuring conditions that will not change
is called Irreversible index. In irreversible index,
{Classification of treatment needs scores, once established, cannot be decreased in value
Population groups or individuals are allocated to appro-
on subsequent examinations. For example, irreversible
priate ‘Treatment Need’ (TN) category on the following
index measures dental caries.
basis:
ii. Reversible index
TN-0: A recording of Code 0 (healthy) or Code X (miss-
The index measuring conditions that can be changed is
ing) for all six sextants indicates that there is no need of
called Reversible index. Reversible index scores can in-
treatment.
crease or decrease on subsequent examinations. For ex-
TN-1: A code of 1 or more indicates a need for improv-
ample, reversible index measures periodontal conditions.
ing personal oral hygiene of that individual.
TN-2: Depending on the extent to which areas of oral cavity
(a) A code of 2 or more indicates need for profes- are measured, indices are classified into ‘Full mouth’
sional cleaning of teeth and removal of plaque- or ‘Simplified’.
retentive factors. In addition, the patient, obvi- i. Full mouth indices
ously, requires oral hygiene instruction. Full mouth indices measure the patient’s entire peri-
(b) Shallow to moderate pocketing (4 or 5 mm – odontium or dentition. For example, Russell’s peri-
code 3): Oral hygiene and scaling will usually odontal index (PI).
Section | I  Topic-Wise Solved Questions of Previous Years 695

ii. Simplified indices Classification Criteria


Simplified indices measure only a representative Very mild (1) Small, opaque and paper white areas scattered
sample of dental apparatus. For example, Greene and irregularly over the tooth but not involving as
Vermillion’s Oral Hygiene Index-Simplified (OHI-S). much as approximately 25% of tooth surface.
Frequently included in this classification are
Indices may be classified in certain general categories teeth showing no more than about 1–2 mm of
according to the entity that they measure, such as: white opacity at the tip of the summit of the
cusps of bicuspids or second molars.
(a) Disease index
(b) Symptom index Mild (2) White opaque areas in the enamel of teeth are
(c) Treatment index more extensive but do not involve as much as
50% of tooth.
l The ‘D’ (decay) portion of the DMF index best

exemplifies a disease index. Moderate (3) All enamel surfaces of the teeth are affected and
surfaces subjected to attrition show wear. Brown
l The indices measuring gingival/sulcular bleeding
stains are frequently a disfiguring feature.
are essentially symptom indices.
l The ‘F’ (filled) portion of the DMF index best Severe (4) All enamel surfaces of the tooth are affected and
hypoplasia is so marked that the general form
exemplifies a treatment index.
of tooth may be affected. The major diagnostic
In general, there are two types of dental indices: sign of this classification is discrete or confluent
pitting. Brown stains are widespread and teeth
i. The first type of index measures the ‘number’ or
‘proportion’ of people in a population with or
often present a corroded-like appearance.) }
without a specific condition at a specific point in
time or interval of time. Q.6. What is an index? Give broad classification of indi-
ii. The second type of dental index measures the ‘num- ces used in dental epidemiologic investigations and de-
ber’ of people affected and the ‘severity’ of specific scribe in detail Silness and Loe dental plaque index.
condition at a specific time or interval of time.
Ans.
Dental indices can also be classified under special cat-
Index is defined as a numerical value describing the relative
egories as follows:
status of a population on a graduated scale with definite
l Simple index
upper and lower limits, and is designed to permit and
Simple index measures the presence or absence of a
facilitate comparison with other populations classified by
condition. For example, an index that would measure
the same criteria and methods (A.L. Russell).
the presence of dental plaque without an evaluation
Dental index is an abbreviated measurement of the
of its effect on gingiva.
amount or condition of disease in a population; a numerical
l Cumulative index
scale with defined upper and lower limits designed to per-
Cumulative index measures all the evidences of a
mit and facilitate comparison with other populations classi-
condition, past and present. For example, DMF index
fied by the same criteria and method (Pamela Zarkowski).
for dental caries.
l Based on the direction in which their scores can fluctuate,

indices are classified as either reversible or irreversible.


l Indices may be classified in certain general categories
(SE Q.5 and SN Q.10) according to the entity that they measure, such as:
{(Dean’s fluorosis index – modified criteria (1942) i. Disease index
ii. Symptom index
iii. Treatment index
Classification Criteria
Dental indices can also be classified under special catego-
Normal (0) The enamel represents the usual translucent ries as follows:
semi-vitriform type of structure. The surface is
l Simple index
smooth, glossy and usually of pale, creamy
white colour. Index that measures the presence or absence of a condi-
tion. For example, an index that would measure the
Questionable The enamel discloses slight aberrations from
(0.5) the translucency of normal enamel, ranging
presence of dental plaque without an evaluation of its
from a few white flecks to occasional white effect on gingiva.
spots. This classification is used in those in- l Cumulative index
stances where a definite diagnosis of the mild- Index that measures all the evidences of a condition,
est form of fluorosis is not warranted, and the past and present. For example, DMF index for dental
classification of ‘normal’ is not justified.
caries.
696 Quick Review Series for BDS 4th Year, Vol 1

[SE Q.16] When no plaque adheres to the point of the explorer,


the area is considered to have a ‘0’ score.
{Silness and Loe dental plaque index l When plaque adheres, a score of ‘1’ is assigned.

The plaque index was described in 1964 by P. Silness


l Plaque that is on the surface of calculus deposits and
and H. Loe, and more fully described in 1967 by H. Loe. on dental restorations of all types in the cervical third
l The plaque index is unique among the indices used for is evaluated and included.
assessment of plaque because it ignores the coronal ex-
tent of plaque on the tooth surface area and assesses only Scoring criteria (Silness and Loe, 1964)
the thickness of plaque at the gingival area of the tooth.
l This index is one of the most widely used and recog-
Score Criteria
nized among the plaque indices that have demon- 0 No plaque
strated good validity and reliability. 1 A film of plaque adhering to free gingival margin and
l It can be used on all surfaces of all or selected teeth, adjacent area of the tooth. The plaque may be seen in
or for selected surfaces of all or selected teeth. How- situ only after application of disclosing solution or by
using the probe on tooth surface.
ever, one criticism is the subjectivity in estimating
plaque. Therefore, it is recommended that a single 2 Moderate accumulation of soft deposits within the
examiner be trained and used with each group of gingival pocket, or the tooth and gingival margin that
can be seen with naked eye.
patients throughout a clinical trial.
l The plaque index (PlI) may be used in large-scale epi- 3 Abundance of soft matter within the gingival pocket
demiological studies or for smaller groups or within and/or on the tooth and gingival margin. }
the dentition of an individual. It has been applied to
studies involving children and adults, and is consid- Q.7. Discuss in detail the Ramfjord periodontal diseases
ered a reliable technique for evaluating both mechani- index, and also mention the advantages and limitations
cal anti-plaque procedures and chemical agents. of the same.
Method
Ans.
l The evaluation or scoring is done on the entire

dentition (whole mouth basis) or on selected teeth Sigurd P. Ramfjord in 1959 developed the plaque compo-
(selected mouth basis). nent of periodontal disease index (PDI).
l Only plaque on the cervical third of the tooth is evalu- This index was the first one that attempted to use a
ated with no attention to plaque that has extended to numerical scale to assess the extent of plaque covering the
the middle or incisal thirds. surface area of a tooth.
l The surfaces examined are the four gingival areas of Selection of teeth and surface
the tooth, i.e. the distal-facial, facial, mesial-facial The scoring is done on the six teeth selected by Ramfjord:
and lingual surfaces. Unlike the buccal (facial) sur- i. 16 – maxillary right first molar
face, the lingual surface is considered as one unit. ii. 21 – maxillary left central incisor
l The mouth mirror, a light source, a dental explorer iii. 24 – maxillary left first premolar
and air-drying of the teeth and gingiva are used in the iv. 36 – mandibular left first molar
scoring of this index. v. 41 – mandibular right central incisor
Six teeth that are evaluated when this index is done vi. 44 – mandibular right first premolar
on selected teeth basis are The PDI is a clinician’s modification of Russell’s peri-
i. Maxillary right first molar (16) odontal index (PI) for epidemiological surveys of peri-
ii. Maxillary right lateral incisor (12) odontal disease. As in the case of the PI, the PDI was
iii. Maxillary left first bicuspid (24) developed due to a lack of methodologies to determine
iv. Mandibular left first molar (36) prevalence and severity and with the intent to be a more
v. Mandibular left lateral incisor (32) sensitive version of PI for use in clinical trials. The PDI
vi. Mandibular right first bicuspid (44) was devised for use in large populations as well as in
If any one of the above-mentioned index teeth is individuals and small groups unlike PI. The PDI has
missing, there is no substitution and the index has to been purported to be sufficiently accurate for use in
be done on a whole mouth basis. longitudinal studies. It is primarily concerned with an
Procedure accurate assessment of the periodontal status of an in-
l The tooth is dried and examined visually. When no dividual person. Emphasis is placed on recording the
plaque is visible, an explorer is used to test the surface. attachment level of periodontal tissues relative to the
The explorer is passed across the tooth surface in the cemento–enamel junction (CEJ). Such accurate and
cervical third and near the entrance to the sulcus. measurable assessments are essential for longitudinal
Section | I  Topic-Wise Solved Questions of Previous Years 697

studies of periodontal disease and as a scientific basis Scoring criteria


of clinical trials.
Score Criteria
Objectives of PDI
The following objectives are incorporated into the de- 0 No plaque present
sign of the index: 1 Plaque present on some but not on all interproximal,
l To assess the prevalence and severity of gingivitis buccal and lingual surfaces of tooth
and periodontitis within individual dentitions and 2 Plaque present on all interproximal, buccal and lin-
in population groups. gual surfaces but covering less than one-half of these
l To provide an accurate basis for incidence and surfaces
longitudinal studies of periodontal disease. 3 Plaque extending over all interproximal, buccal and
l To provide a meaningful basis for estimating the lingual surfaces, and covering more than one-half of
need for periodontal therapy in selected popula- these surfaces
tion groups.
l To provide accurate recordings of clinical trials
Only fully erupted teeth should be scored. Missing teeth
of preventive and therapeutic procedures in peri-
should not be substituted.
odontics.
Calculations
l To provide measurable reference data for as-
The plaque score per person is obtained by summing all
sessment of correlations with factors of poten-
individual tooth scores and dividing by the number of
tial significance in the aetiology of periodontal
teeth examined, i.e.
disease.
Assessment of degree of periodontal disease includes a Total score
subjective assessment of colour, form, density and Plaque score of an individual 5
No. of teeth examined
bleeding tendency of gingival tissues. The most impor-
tant feature of PDI is the measurement of the level of Uses
periodontal attachment related to the cemento–enamel 1 . Suitable for longitudinal studies of periodontal
junction of the teeth. disease
Components of periodontal disease index 2. Helpful in the total assessment of periodontal
The PDI comprises three components: status
l Plaque component 3. In epidemiological surveys
l Calculus component 4. Suitable for the clinical trials of preventive or thera-
l Gingival and periodontal component peutic agents
All the three components will be detailed out separately Calculus component of periodontal disease index
in the following sections. In 1959, a calculus index was described by Sigurd
P. Ramfjord as one of the components of his periodontal
[SE Q.17]
disease index (PDI).
{Plaque component of the periodontal disease index The calculus component of PDI assesses the presence
The surfaces scored are facial, lingual, mesial and distal. and extent of calculus of six index teeth.
Method Teeth and surfaces examined
l The tooth should be stained with Bismarck brown The six index teeth selected for scoring calculus are as
solution. Bismarck brown solution is placed in a dap- follows:
pen dish and two Richmond cotton pellets are placed i. 16 – maxillary right first molar
in the dish until they appear completely saturated with ii. 21 – maxillary left first molar
the solution. Scoring of plaque is done after staining. iii. 24 – maxillary left first bicuspid
l One cotton pellet is removed with a cotton plier and iv. 36 – mandibular left first molar
touched gently on to the lingual and buccal surfaces v. 41 – mandibular right central incisor
of mandibular teeth. vi. 44 – mandibular right first bicuspid
l The second pellet is touched on to the palatal and The facial (buccal/labial) and lingual surfaces of the six
buccal surfaces of maxillary teeth. The occlusal sur- index teeth are examined.
faces are also rubbed with the pellet. So, the disclos- Method
ing solution flows over all teeth surfaces. The presence and extent of calculus on the facial and
l The patient is then instructed to spit and rinse the lingual surfaces of the six index teeth are evaluated us-
mouth twice thoroughly. The scoring is now done by ing a mouth mirror and a dental explorer and/or a peri-
noticing the stained surfaces. odontal probe.
698 Quick Review Series for BDS 4th Year, Vol 1

The scoring of calculus is done according to the follow- l Although numerical indices are easier to use in clini-
ing criteria: cal trials and epidemiological surveys, they are lim-
ited in that they do not constitute a ratio scale, i.e. a
Score Criteria ‘two’ is not necessarily twice as much as inflamma-
0 Absence of calculus tion as ‘one’.
l Another consideration with gingival indices is that
1 Supragingival calculus extending only slightly below
the free gingival margin (not more than 1 mm) they only evaluate the condition of soft tissue and
may or may not be directly or proportionally related
2 Moderate amount of supragingival and subgingival
calculus or subgingival calculus alone to underlying periodontal destruction.
Indices used to assess gingival inflammation.
3 An abundance of supragingival and subgingival
calculus} [SE Q.8]

Calculation of the index


{Papillary marginal attachment index (PMA index)
The papillary marginal attachment index (PMA index),
The calculus score per tooth is summed and divided by
developed in 1944 by Maury Mossier and I Schour, is
the number of teeth examined to yield the calculus score
probably the first successful attempt to design a nu-
per person.
merical system for recording gingival health.
The calculus component of PDI also has a high degree of
The primary impetus that led to the development of PMA
examiner reproducibility and can be performed quickly.
index during 1944–1947 was the need for some quantita-
Uses
tive method of recording readily observable inflamma-
l Epidemiologic surveys
tory conditions of gingivae to replace the current gross
l Longitudinal studies in periodontal diseases
assessment gingivitis in both children and adults as mild,
Q.8. Define an index and describe the various indices moderate or severe. The basic philosophy used in the
used for the survey of gingival diseases. development of PMA index was very similar to that of
DMF index, i.e. the number of gingival units affected was
Ans.
counted, rather than the severity of inflammation.
Index is defined as a numerical value describing the relative A gingival unit is divided into three components:
status of a population on a graduated scale with definite i. Papillary gingiva (P) – the gingival portion between
upper and lower limits, and is designed to permit and the teeth
facilitate comparison with other populations classified by ii. Marginal gingiva (M) – the marginal collar sur-
the same criteria and methods (A.L. Russell). rounding the teeth
Dental index is an abbreviated measurement of the iii. Attached gingiva (A) – the gingival portion overly-
amount or condition of disease in a population; a numerical ing the bony alveolar process
scale with defined upper and lower limits designed to per- Method
mit and facilitate comparison with other populations classi- l The facial surface of the gingiva around a tooth is
fied by the same criteria and method (Pamela Zarkowski). divided into three gingival units: papillary gingiva
The process of quantifying the prevalence, incidence and (P), marginal gingiva (M) and attached gingiva (A).
related factors of gingival diseases is an important operation – The presence or absence of inflammation on each
one that is necessary to the eventual improvement of dental gingival unit is recorded.
public health. Prevention of gingival diseases rests on the l Although all the facial tissues surrounding all the
knowledge of their aetiology as well as an understanding of the teeth could be assessed in this manner, usually only
occurrence and distribution of related factors and conditions. the maxillary and mandibular incisors, canines and
Measuring gingivitis in quantitative terms allows one to premolars are examined.
assess whether conditions are improving or relapsing, and l The routine examination procedure from the maxil-
whether prevention programmes are accomplishing or fail- lary left second molar to the maxillary right second
ing their objectives. All the indices used to assess gingival molar and then to the mandibular right second molar
diseases have relied on one or more of the following criteria: to the mandibular left second molar is followed.
(a) Gingival colour l Third molars are excluded. The degree of gingivitis
(b) Gingival contour for each gingival unit is as follows:
(c) Gingival bleeding P – involvement of papillary gingiva and its severity
(d) Extent of gingival inflammation expressed in scores from 0 to 5.
(e) Gingival crevicular fluid flow M – involvement of marginal gingiva and its severity
l Also, most indices have assigned a number to each expressed in scores from 0 to 5.
of the criteria evaluated with the exception of A – involvement of attached gingiva and its severity
gingival crevicular fluid. expressed in scores from 0 to 3.
Section | I  Topic-Wise Solved Questions of Previous Years 699

Thus, in general, mild gingivitis is confined to the papil- and marginal areas of the gingival are scored. Scoring of
lary area (P); moderate gingivitis means spread to mar- the attached gingival was eliminated as it was found to be
ginal gingivae (M); and severe gingivitis is identified by rarely involved.
its spread to the attached gingivae (A). Scoring criteria
Scoring criteria The gingival areas are assessed as follows:
The degree of gingivitis for each gingival unit is scored
as follows: Score Criteria
Scoring of PM index 0 No inflammation
‘P’ 1 Bleeding from gingival sulcus on gentle probing;
tissue otherwise appears normal
0 Normal; no inflammation
2 Bleeding on probing plus a change in colour due to
11 Mild papillary engorgement; slight increase in size
inflammation; no oedema
21 Obvious increase in size of gingival papilla; bleeding on
3 Bleeding plus a change in colour and oedematous
pressure
swelling
31 Excessive increase in size with spontaneous bleeding
4 Ulceration or additional symptoms
41 Necrotic papilla
Calculation of the index
51 Atrophy and loss of papilla (through inflammation)
Sum of all scores
‘M’ The average PM index 5
Numb er of areas scored
0 Normal; no inflammation visible
Gingival index (GI)
11 Engorgement; slight increase in size, no bleeding
21 Obvious engorgement; bleeding upon pressure
[SE Q.3]
31 Swollen collar; spontaneous bleeding; beginning infiltration {The gingival index (GI) was developed by H. Loe and
into attached gingivae J. Silness in 1963. It was developed solely for the pur-
41 Necrotic gingivitis pose of assessing the severity of gingivitis and its loca-
tion in four possible areas by examining only the quali-
51 Recession of free marginal gingivae below the cemento–
tative changes (i.e. severity of the lesion) of the gingival
enamel junction due to inflammatory changes
soft tissue. The GI does not take into account periodon-
‘A’ tal pocket depth, degrees of bone loss or any other
quantitative change of the periodontium.
0 Normal; pale rose; stippled The GI is one of the most widely accepted and used
11 Slight engorgement with loss of stippling; changes in gingival indices due to its documented validity, reliabil-
colour may or may not be present ity and ease of use. However, even though the GI has
21 Obvious engorgement of attached gingivae with marked demonstrated sufficient sensitivity to distinguish be-
increase in redness; pocket formation present tween groups with mild and severe gingivitis, it may not
discriminate between the middle ranges.
31 Advanced periodontitis; deep pockets evident
Method
Calculation of the index The severity of gingivitis is scored on all surfaces of all
The number of affected papillary, marginal and teeth or selected teeth, or on selected surfaces of all
attached units is counted and the P, M and A nu- teeth or selected teeth. It can also be used on individual
merical values are summed separately, then added or large population groups.
together and expressed numerically as the PMA The teeth selected as index teeth are as follows:
index score per person. i. 16 – maxillary right first molar
Uses ii. 12 – maxillary left lateral incisor
i. In clinical trials iii. 24 – maxillary left first premolar
ii. On individual patients iv. 36 – mandibular left first molar
iii. For epidemiologic surveys} v. 32 – mandibular left lateral incisor
Papillary marginal index (PM index) vi. 44 – mandibular right first premolar
The papillary marginal index was developed in 1958 by l To obtain the GI, the examiner will first need

H.R. Mühlemann and Z.S. Mazor as a modification of sufficient lighting, a mouth mirror and a
Schour and Massler’s PMA index. This index was intro- probe. The teeth and gingiva should be dried
duced in a study on school children, where only papillary lightly with a blast of air and/or cotton rolls.
700 Quick Review Series for BDS 4th Year, Vol 1

l The tissues surrounding each tooth are divided the lingual surfaces of all the upper or maxillary teeth are
into four gingival scoring units: distal-facial examined beginning with the upper left second molar.
papilla, facial margin, mesial-facial papilla For the lower or mandibular arch, the examination be-
and the entire lingual gingival margin. Unlike gins with the lower left second molar through to the
the facial surface, the lingual surface is not right second molar. On the left side of the midline, the
subdivided in an effort to minimize examiner’s examination sequence is distal, buccal/labial and me-
variability in scoring, since it will most likely sial, and on the right side, it is mesial, buccal/labial and
be viewed indirectly with a mouth mirror. distal. Afterwards, all lingual surfaces are scored begin-
l A blunt instrument, such as a periodontal ning with the left second molar.
pocket probe, is used to assess the bleeding Third molars or wisdom teeth are not examined or scored
potential of tissues. in the upper or lower arch. According to the developers,
Each of the four gingival units is assessed according to the scoring for GI takes approximately 2–5 min under
the following criteria: optimal conditions and with chair side assistance.
The calculation of GI of an individual remained the
Score Criteria same, except the maximum number of teeth examined
0 Absence of inflammation/normal gingiva in the denominator is 28 instead of 6.
1 Mild inflammation; slight change in colour, slight
Uses
oedema; no bleeding on probing 1. To determine the prevalence and severity of gingivi-
tis in epidemiologic surveys
2 Moderate inflammation; moderate glazing, redness,
oedema and hypertrophy. Bleeding on probing 2. To assess severity of gingivitis in individual dentition
3. Used in controlled clinical trials of preventive or
3 Severe inflammation; marked redness and hypertrophy
therapeutic agents}
ulceration; tendency to spontaneous bleeding
Modified gingival index (MGI)
The modified gingival index (MGI) was developed in
Calculation of the index 1986 by R. RLobene, T. Weatherford, N.M. Ross, R.A.
The gingival index score for the area is obtained by Lamm and L.A. Menaker as a modification of the Loe
summing the scores around each tooth. If the scores and Silness’s gingival index to assess the prevalence and
around each tooth are summed and divided by four, the severity of gingivitis.
gingival index score for the tooth is obtained. Unlike GI, the MGI has a noninvasive approach, meaning
Summing all the scores per tooth and dividing by the num- there is no gentle probing to possibly provoke bleeding
ber of teeth examined provides the gingival index score per on pressure, which was one of the main reasons for its
person. The gingival index may be used to evaluate a seg- development. The other reason for its development was to
ment of mouth or a group of teeth in the same way. increase sensitivity in the low region of scoring scale. For
The numerical scores of the gingival index may be associ- MGI, determining the severity of gingivitis is strictly
ated with varying degrees of clinical gingivitis as follows: based on visual observation, which has maintained a high
visual sensitivity, especially with incipient gingivitis.
Gingival scores Condition Since its development, the MGI has been used widely,
0.1–1.0 Mild gingivitis especially in clinical trials of therapeutic agents.
Method
1.1–2.0 Moderate gingivitis
To obtain MGI, the labial/facial and lingual surfaces of
2.1–3.0 Severe gingivitis gingival margins and interdental papillae of all erupted teeth
or selected teeth (the same as in the case of gingival index)
Modifications of gingival index are examined and scored using the following criteria.
In 1967, Loe detailed the sequence of the examination
procedure and slightly modified the procedure to include Score Criteria
the entire dentition instead of six teeth (Marks et al., 0 Normal (absence of inflammation)
1993). This detailed examination is as follows: 1 Mild inflammation (slight change in colour, little
The examination of all erupted teeth typically starts with change in texture) of any portion of gingival unit
the upper right second molar and continues over the mid- 2 Mild inflammation of entire gingival unit
line to the upper left second molar using gentle probing
pressure. For teeth on the right side of the midline, the 3 Moderate inflammation (moderate glazing, redness,
oedema and/or hypertrophy) of gingival unit
examination sequence is distal, buccal/labial and mesial.
On the left side, the examination sequence is mesial, buc- 4 Severe inflammation (marked redness and oedema/
cal/labial and distal. When the three surfaces (i.e. distal, hypertrophy, spontaneous bleeding or ulceration) of
gingival unit
buccal/labial and mesial) of all teeth have been scored,
Section | I  Topic-Wise Solved Questions of Previous Years 701

The third molars are excluded. For a full mouth exami- Calculation
nation with 28 teeth, a maximum number of 108 gingi- Each of the four gingival units (M and P) is scored from
val units (i.e. marginal and papillary) are examined and 0 to 5 to obtain the SBI of area. The scores for the four
scored for gingivitis (i.e. 56 marginal and 52 papillary). units are summed and divided by 4 to obtain the SBI of
Again, for MGI, the examination of gingivitis is strictly tooth. SBI is determined by summing the scores of indi-
based on visual observation. There is no gentle probing or vidual teeth and the number of teeth.}
pressure to observe the presence or absence of bleeding. Papillary bleeding index (PBI)
Calculation The papillary bleeding index (PBI) was developed by H.
To calculate the MGI for an individual, the papillary and R. Mühlemann in 1977 as a modification of the sulcus
marginal scores are added and divided by the total num- bleeding index (SBI) developed by Mühlemann and
ber of sites (i.e. gingival units) examined. Son. The PBI is based on the bleeding following gentle
Indices used for assessment of gingival bleeding probing of interdental papilla.
Method
[SE Q.14] The PBI is performed by sweeping the papillary sulcus
{Sulcus bleeding index (SBI) on the mesial and distal aspects with a periodontal
The sulcus bleeding index (SBI) is an index for assess- probe. The mouth is divided into quadrants, with the
ment of gingival bleeding. It was developed in 1971 by maxillary right and mandibular left quadrants probed
H.R. Mühlemann and S. Son. This index system is a lingually, and the maxillary left and mandibular right
modification of the papillary marginal index (PMI) of quadrants probed buccally. The blunt periodontal probe
Mühlemann and Mazor (1958). The purpose of this in- is carefully inserted into the gingival sulcus at the base
dex is to locate areas of gingival sulcus bleeding upon of the papilla on the mesial aspect, then moved coro-
gentle probing, and thus recognizing and recording the nally to the papilla tip. This is repeated on the distal
presence of early inflammatory gingival disease. aspect of the same papilla. The intensity of any bleeding
Method thus provoked is recorded on a scale of 0–4.
The SBI is based on the evaluation of gingival bleeding Scoring criteria.
on probing, gingival contour and gingival colour change.
Four gingival units are scored systematically for each
Score Criteria
tooth: the labial and lingual marginal gingivae (M units),
and the mesial and distal papillary gingivae (P units). 0 No bleeding after probing.
The probing of the four areas should be carried out un- 1 A single discreet bleeding point appears after probing.
der proper illumination. The probe should be held paral- 2 Several isolated bleeding points or a single fine line of
lel to the long axis of the tooth for M units and direct the blood appears.
probe towards the col area for P units. After the probing
3 The interdental triangle fills with blood shortly after
is done, wait for 30 seconds before scoring apparently probing.
healthy gingival units. The gingivae should be dried
gently to observe colour changes clearly. 4 Profuse bleeding occurs after probing; blood flows
immediately into the marginal sulcus.
Scoring criteria
The assessment of gingival bleeding is done on a scale
of 0–5, according to the following criteria. Calculation
Each papilla is scored according to the criteria. The
Score Criteria scores are totalled and divided by the number of papilla
0 Healthy appearance of P and M. No bleeding upon examined.
sulcus probing Gingival bleeding index (GBI)
1 Apparently healthy P and M showing no colour or
The gingival bleeding index was developed by H.G.
contour changes and no swelling but bleeding from Carter and G.P. Barnes in 1974 to record the presence or
sulcus on probing absence of gingival inflammation as determined by
2 Bleeding on probing and colour change caused by in-
bleeding from interproximal gingival sulci.
flammation (reddening). No swelling or macroscopic Method
oedema All interproximal areas having a mesial and distal sul-
3 Bleeding on probing, change in colour, slight oedema-
cus component are considered to be susceptible to gin-
tous swelling gival inflammation, and these areas are recorded as total
areas at risk. Each interproximal area has two sulci,
4 (1) Bleeding on probing, colour change, obvious swelling
(2) Bleeding on probing and obvious swelling which are scored either as one interdental unit or indi-
vidually. Certain areas may be excluded from scoring
5 Spontaneous bleeding on probing, colour change,
because of accessibility, tooth position, diastemata or
marked swelling with or without ulceration
other factors, and if exclusions are made, a consistent
702 Quick Review Series for BDS 4th Year, Vol 1

procedure should be followed for an individual and for inflammation in the interdental area by the presence or
a group if a study is to be made. absence of bleeding.
A full complement of teeth has 30 proximal areas. Third Method
molars are usually excluded and 26 interdental units are The examination is done on interdental area around the
scored. entire dentition. A triangular wooden interdental cleaner
Procedure is used for this index. The wooden interdental cleaner is
Unwaxed dental floss is used. Floss has the advantages of inserted gently into each interdental area and removed
being readily available, disposable and usable by the in- immediately in such a way as to depress the papilla by
structed patient. The floss is first passed interproximally about 1–2 mm. The path of insertion of interdental
on one side of the papilla and then on the other. The floss cleaner should be parallel to the occlusal surface, taking
is then curved around the adjacent tooth and brought be- care not to angle the point in an apical direction. The
low the gingival margin. The floss is moved up and down insertion and removal of interdental cleaner is done for
for one stroke, with care not to lacerate the gingival. Fin- about four times before moving on to the next inter-
ger rests are adapted to provide controlled and consistent proximal area. The presence or absence of bleeding
pressure. A new length of clean floss is used for each should be recorded within 15 seconds for each area.
area. Retract for visibility of bleeding from both facial Scoring criteria
and lingual aspects. A gap of 30 seconds should be The number of bleeding sites is totalled for an individ-
allowed for re-inspection of an area that does not show ual score for comparison with scores over a series of
blood either immediately in the area or on the floss. appointments.
Scoring criteria Calculation
Bleeding indicates the presence of disease. No attempt The index score is expressed as the percentage of the
is made in this index to quantify the severity of bleeding total number of sites evaluated. The calculations can be
because no bleeding represents healthy tissues. The made for total mouth, quadrants or maxillary versus
number of bleeding areas and scoring units are re- mandibular. The EIBI for an individual can be calcu-
corded. Patient participation in observing and recording lated by using the following formula:
over a series of appointments can increase motivation.
Modified sulcular bleeding index (mSBI) Number of bleeding areas  100
EIBI 
The modified Sulcular bleeding index (mSBI), also Total n umber of areas
known as the modified sulcus bleeding index, was
developed in 1987 by A. Mombelli, M.A. Van Oosten, Q.9. Write about development, objectives, scoring system,
E. Schurch, Jr, and N.P. Land to determine the severity advantages and limitations of Greene and Vermillion’s
of gingival bleeding, a sign of inflammation associated simplified oral hygiene index.
with periodontal disease. Ans.
The mSBI is one of the several modified versions of PBI
that originated from the PM index of 1958 (Mühlemann
[SE Q.12]
and Mazor, 1958), later named as the Sulcus bleeding
index (Mühlemann and Son, 1971) to avoid confusion {The simplified oral hygiene index (OHI-S) was developed
with the PMA index. in 1964 by John C. Greene and Jack R. Vermillion, the de-
A periodontal probe is used and passed along the gingival velopers of the original oral hygiene index (OHI). Even
margin to provoke bleeding, and the clinical findings are though the original OHI was determined to be simple, sen-
recorded according to the following scores and criteria. sitive and useful, it was time-consuming and required more
decision-making. So, an effort was made to develop a more
Score Criteria
simplified version with equal sensitivity.
The simplified oral hygiene index (OHI-S) differs from
0 No bleeding when a periodontal probe is passed
along the gingival margin
the original OHI (the oral hygiene index) in the number of
tooth surfaces scored (6 rather than 12), the method of se-
1 Isolated bleeding spots visible lecting the surfaces to be scored, and the scores which can
2 Blood forms a confluent red line on margin be obtained. The criteria used for assigning scores to the
3 Heavy or profuse bleeding
tooth surfaces are the same as those used for the OHI. How-
ever, the criteria and scoring for tooth surfaces remained the
same.
Eastman interdental bleeding index (EIBI) Similar to OHI, the OHI-S has two components: the
The Eastman interdental bleeding index was developed simplified debris index (DI-S) and the simplified calculus
in 1984 by K. Abrams, J. Caton and A. Polson to assess index (CI-S). Each of these indices, in turn, is based on
Section | I  Topic-Wise Solved Questions of Previous Years 703

numerical determinations representing the amount of debris and the surfaces reduced in height by caries or trauma are
or calculus found on preselected tooth surfaces. not scored. Instead, an alternate tooth is examined.
Selection of tooth surfaces After the six possible tooth surfaces are selected, the
The six surfaces examined for OHI-S are selected from scores are determined, recorded and computed for DI-S
four posterior and two anterior teeth. In the posterior and CI-S, respectively.
portion of dentition, the first fully erupted tooth distal to The following definitions and criteria are used to deter-
the second bicuspid, usually the first molar but some- mine the scores for each of the surfaces examined.}
times the second or third molar is examined. The buccal
surfaces of the selected upper molars and the lingual (SE Q.2 and Q.12)
surfaces of the selected lower molars are inspected. {(Debris Index Simplified (DI-S)
In the anterior portion of the mouth, the labial surfaces of ‘Oral Debris’: Oral debris is a soft foreign matter loosely
the upper right and the lower left central incisors are scored. attached to the teeth. It consists of mucin, bacteria and
In the absence of either of these anterior teeth, the central food, and varies in colour from greyish-white to green or
incisor on the opposite side of the midline is substituted. orange. The surface area covered by debris is estimated
Only fully erupted permanent teeth are taken for scoring. by running the side of an explorer (Shepard’s hook) along
Natural teeth with full crown restorations and surfaces the tooth surface being examined. The occlusal or incisal
reduced in height by caries or trauma are not scored. extent of the debris is noted as it is removed.
Surfaces and teeth to be examined The mouth is examined first for debris (i.e. DI-S). The
surface area covered by debris is estimated by running the
Tooth Surfaces side of No. 5 explorer (Shepard -b hook) along the tooth
16 (upper right first molar) Buccal
surface being examined. The occlusal or incisal extent of
the debris is determined and recorded as it is removed.
11 (upper right central incisor) Labial
For DI-S, the following scoring criteria are used:
26 (upper left first molar) Buccal Debris index simplified.
36 (lower left first molar) Lingual
31 (lower left central incisor) Labial Score Criteria
46 (lower right first molar) lingual 0 No debris or stain present
1 Soft debris covering not more than one-third of the
tooth surface, or presence of extrinsic stains without
If a designated tooth is not a fully erupted permanent other debris regardless of surface area covered
tooth or has a full crown restoration or has surfaces re-
2 Soft debris covering more than one-third but not
duced in height by caries or trauma, a substitution is
more than two-thirds of the exposed tooth surface
made, which is as follows:
3 Soft debris covering more than two-thirds of the
exposed tooth surface
For tooth 16 Tooth 17 – upper right second molar
Tooth 18 – upper right third molar
After the six possible debris scores are recorded, the DI-S
For tooth 11 Tooth 21 – upper left central incisor
value is calculated. After the debris score is obtained for
For tooth 26 Tooth 27 – upper left second molar each of the six possible preselected tooth surfaces for DI-S,
For tooth 36 Tooth 37 – lower left second molar four from the posterior region and two from the anterior
region, the calculus score is examined.)}
Tooth 38 – lower left third molar
For tooth 31 Tooth 41 – lower right central incisor [SE Q.12]
For tooth 46 Tooth 47 – lower right second molar {Calculus index simplified (CI-S)
Tooth 48 – lower right third molar ‘Oral calculus’: Calculus is defined as a deposit of inor-
ganic salts composed primarily of calcium carbonate
and phosphate mixed with food debris, bacteria and
Examination methods and scoring system desquamated epithelial cells. There are two main types
For OHI-S, each surface, buccal or lingual, is considered of dental calculus, which are differentiated primarily by
half of the tooth circumference. Also, only fully erupted location on the tooth in relation to free gingival margin.
permanent teeth are scored. A tooth is considered fully i. Supragingival calculus – Denotes deposits, usu-
erupted if the occlusal or incisal surface has reached the ally white to yellowish-brown in colour, occlusal
occlusal plane. Natural teeth with full crown restorations to free gingival margin.
704 Quick Review Series for BDS 4th Year, Vol 1

ii. Subgingival calculus – Denotes deposits located


{SN Q.6}
at the apical of free gingival margin. These depos-
its are usually light brown to black in colour. Uses of OHI-S index
The No. 5 explorer (Shepard’s hook) is used to estimate l The OHI-S has been widely used in the studies of the

the surface area covered by the supragingival calculus epidemiology of periodontal disease.
and to probe for subgingival calculus. The following l OHI-S has been used as a standard companion of peri-

scoring codes and criteria are used for CI-S: odontal index in the studies of the Interdepartmental
Committee on Nutrition for National Defence (ICNND).
Calculus index simplified. l OHI-S is useful in the evaluation of dental health

Score Criteria education programmes in public school systems.


l OHI-S is used in evaluating the cleansing efficiency
0 No calculus present
of toothbrushes.
1 Supragingival calculus covering not more than one-
l OHI-S is used to evaluate an individual’s level of oral
third of the exposed tooth surface
cleanliness and, to a more limited extent, in clinical trials.
2 Supragingival calculus covering more than one-third l OHI-S is easy to use because the criteria are objec-
but not more than two-thirds of the exposed tooth sur-
face, or the presence of individual flecks of subgingi-
tive, the examination may be performed quickly and
val calculus around the cervical portion of tooth a high level of reproducibility is possible with a
minimum of training sessions.
3 Supragingival calculus covering more than two-thirds of
the exposed tooth surface or a continuous heavy band of
subgingival calculus around the cervical portion of tooth Q.10. Define an index. Describe the indices used for
dental caries.
Calculation of the index Ans.
For each individual, the debris and calculus scores are
[Same as LE Q.1]
totalled and divided by the number of tooth surfaces
scored. For an individual score to be calculated, at least Q.11. Discuss in detail ideal requisites of an index and
two of the six possible tooth surfaces are examined. For evaluate critically advantages and disadvantages of
a group of individuals, the debris and calculus scores are DMFT and DMFS indices.
obtained by calculating the average of individual scores.
Ans.
The average of individual or group scores is DI-S or
CI-S. Individual scores are calculated to one decimal [Same as LE Q.1]
place, and group scores may be calculated to one or two
Q.12. Enumerate different indices used for the assess-
decimal places, depending on the sample size and the
ment of dental caries in the community, and describe
use of data (Greene and Vermillion, 1964).
any one index in detail.
Once DI-S and CI-S are calculated separately, they are
combined or added together for OHI-S. The DI-S and Ans.
CI-S values range from 0 to 3, which can be interpreted
[Same as LE Q.1]
as follows:
Good: 0.0–0.6 Q.13. Define index, give the classification of indices.
Fair: 0.7–1.8 What are the ideal requisites of an index?
Poor: 1.9–3.0
Ans.
The OHI-S value ranges from 0 to 6, which can be in-
terpreted as follows: [Same as LE Q.2]
Good: 0.0–1.2
Q.14. Define and describe ideal characteristics of indi-
Fair: 1.3–3.0
ces. Write in detail about Russell’s periodontal index.
Poor: 3.1–6.0}
Calculation example Ans.
The following are the examples to calculate the index.
[Same as LE Q.3]
The scores for debris and calculus should be tabulated
separately and index for each is calculated indepen- Q.15. Define index. Which are the ideal requisites of an
dently but in the same manner. index? Write in detail about community periodontal
The average individual or group debris and calculus index of treatment needs.
scores are combined to obtain a simplified oral hygiene
Ans.
index, as follows:
Oral hygiene index 5 debris index 1 calculus index [Same as LE Q.4]
Section | I  Topic-Wise Solved Questions of Previous Years 705

Q.16. What is CPITN index? Give its criteria of scoring iii. Measure the effectiveness of mechanical devices
system and mention its advantages and limitations. for personal care, such as toothbrushes, interden-
tal cleaning devices or water irrigators.
Ans.
In community health
[Same as LE Q.4] An index can
i. show the prevalence and trends of incidence of a
particular condition occurring within a given
SHORT ESSAYS: population;
Q.1. CPITN index. ii. provide baseline data to show existing dental
health practices
Ans.
iii. assess needs of a community
[Ref LE Q.4]
Q.7. Irreversible indices.
Q.2. Debris index simplified.
Ans.
Ans.
[Ref LE Q.1]
[Ref LE Q.9]
Q.8. PMA index.
Q.3. Gingival index of Loe and Silness.
Ans.
Ans.
[Ref LE Q.8]
[Ref LE Q.8]
Q.9. CPI index.
Q.4. Russell’s index.
Ans.
Ans.
[Ref LE Q.3] {SN Q.12}
Q.5. Dean’s fluorosis index. This index is based on the modifications of the earlier
Ans. used Community Periodontal Index of Treatment Needs
(CPITN). The modifications were done by the inclusion
[Ref LE Q.5] of measurement of ‘Loss of Attachment’ and the elimi-
Q.6. Uses of an index. nation of the ‘Treatment Needs’ category.
Indicators: Three indicators of periodontal status are
Ans. used for this assessment: gingival bleeding, calculus
For individual patients and periodontal pockets.
An index can: A specially designed lightweight CPI probe with 0.5-mm
l Provide individual assessment to help a patient
ball tip is used, with a block band between 3.5 and 5.5 mm
recognize an oral problem. and rings at 8.5 and 11.5 mm from the ball tip. (Earlier this
l Reveal the degree of effectiveness of present oral probe was known as ‘CPITN–C’ probe.)
hygiene practices.
l Motivate the person in preventive and professional
Procedure
care for the elimination and control of oral disease.
Sextants: The mouth is divided into sextants as in the
l Evaluate the success of individual and profes-
case of CPITN defined by tooth numbers 18-14, 13-23,
sional treatment over a period of time by compar-
24-28, 38-34, 33-43 and 44-48. A sextant should be
ing index scores.
examined only if there are two or more teeth present
l Provide a means for the personal assessment
which are not indicated for extraction. (Note: This re-
of abilities to educate and motivate individual
places the former instruction to include single remain-
patients by dental hygienist.
ing teeth in the adjacent sextant.)
In research
Index teeth: For adults aged 20 years and over, the teeth
An index is used to:
to be examined are as follows:
i. Determine baseline data before experimental fac-
tors are introduced.
17 16 11 26 27
ii. Measure the effectiveness of specific agents for the
prevention, control or treatment of oral conditions. 47 46 31 36 37
706 Quick Review Series for BDS 4th Year, Vol 1

Two molars in each posterior sextant are paired for recording Loss of attachment
but if one is missing, there is no replacement. If no index Information about the loss of attachment may be col-
teeth or tooth are present in a sextant qualifying for examina- lected from index teeth to obtain an estimate of the
tion, all the remaining teeth in that sextant are examined and lifetime accumulated destruction of periodontal attach-
the highest score is recorded as the score for sextant. In this ment. This permits comparisons between population
case, distal surfaces of third molars should not be scored. groups, but is not intended to describe the full extent of
For subjects under the age of 20 years, only six index loss of attachment in an individual.
teeth, 16, 11, 26, 46, 31 and 36, are examined. This modi- The most reliable way of examining for the loss of attach-
fication is made to avoid scoring the deepened sulci as- ment in each sextant is to record this immediately after re-
sociated with eruption as periodontal pockets. For the cording the CPI score for that particular sextant. The highest
same reason, when children under the age of 15 are exam- scores for CPI and loss of attachment may not necessarily be
ined, pockets should not be recorded, i.e. only bleeding found on the same tooth in a sextant. Loss of attachment
and calculus should be considered. should not be recorded for children younger than 15 years.
Sensing gingival pockets and calculus: An index tooth Probing pocket depth gives some indication of the extent
should be probed, using the probe as a ‘sensing’ instrument, of loss of attachment. This measurement is unreliable
to determine pocket depth and to detect subgingival calcu- when there is gingival recession, i.e. when the cemento–
lus and bleeding response. The sensing force used should enamel junction is visible. When the cemento–enamel
be no more than 20 g. A practical test for establishing this junction is not visible and the highest CPI score for a
force is to place the probe point under the thumbnail and sextant is less than 4 (probing depth less than 6 mm), any
press until blanching occurs. For sensing subgingival calcu- loss of attachment for that sextant is estimated to be less
lus, the lightest possible force that will allow movement of than 4 mm (loss of attachment score 5 0).
the probe ball tip along the tooth surface should be used. The extent of loss of attachment (LA) recorded using
When the probe is inserted, the ball tip should follow the following codes is shown in the table below:
the anatomical configuration of the surface of tooth
root. If the patient feels pain during probing, this is in- Score Criteria
dicative of the use of too much force. 0 Loss of attachment 0–3 mm (CEJ not visible and CPI
The probe tip should be inserted gently into the gingival score 0–3)
sulcus or pocket, and the total extent of the sulcus of pocket 1 Loss of attachment 4–5 mm (CEJ within the black band)
is explored. For example, the probe is placed in the pocket
2 Loss of attachment 6–8 mm (CEJ between the upper
at the distobuccal surface of the second molar, as close as limit of the black band and the 8.5-mm ring)
possible to the contact point with the third molar, keeping
3 Loss of attachment 9–12 mm (CEJ between 8.5-mm
the probe parallel to the long axis of tooth. The probe is
and 11.5-mm rings)
then moved gently, with short upward and downward
movements, along the buccal sulcus or pocket to the mesial 4 Loss of attachment, 12 mm or more (CEJ beyond
11.5-mm rings)
surface of the second molar and from the distobuccal sur-
face of the first molar towards the contact area of premolar. x Excluded sextant (less than two teeth present)
A similar procedure is carried out for lingual surfaces, 9 Not recorded (CEJ neither visible nor detectable)
starting disto-lingually to the second molar.
Examination and recording Q.10. Rules governing DMF index.
The index teeth, or all remaining teeth in a sextant
where there is no index tooth, should be probed and the Ans.
highest score recorded in the appropriate box. [Ref LE Q.1]
The scoring criteria are as follows:
Q.11. Caries index for primary dentition.
Score Criteria
Ans.
0 Healthy
‘Def’ Index
1 Bleeding observed, directly or by using a mouth
mirror, after probing The ‘Def index’ was described in 1944 by A.O. Gruebbel
as an equivalent index of DMF index for measuring
2 Calculus detected during probing but all the black
bands on the probe visible
dental caries in primary dentition. The caries indices
used for primary dentition are ‘deft’ index and ‘defs’
3 Pocket 4–5 mm (gingival margin within the black band index equivalent to the DMFT and DMFS indices, re-
on the probe)
spectively, used for permanent dentition.
4 Pocket 6 mm or more (black band on the probe not visible) As defined by Gruebbel: d, decayed tooth; e, extracted
x Excluded sextant (less than two teeth present) tooth; and f, filled tooth.
The basic rules and principles for def index are the
9 Not recorded
same as that for DMF index.
Section | I  Topic-Wise Solved Questions of Previous Years 707

Examination method for Def index (deciduous teeth only) (C) Per cent needing care
‘d’ – Decayed: Indicates the number of deciduous teeth To calculate the per cent of teeth needing resto-
that are decayed. In counting the number of decayed rations, divide the total ‘d’ component by the
deciduous teeth, remember that tooth can only be total numbers examined, i.e.
counted once. It cannot be counted as filled and de-
cayed. If it has been restored, and caries are detected, Total number of decayed tooth
Per cent needing care 5
count it as decayed. Be sure, the explorer falls into cari- Total number examined
ous tooth substance and not just in a deep groove before
(D) Per cent affected
counting occlusal caries.
To calculate the per cent affected, divide the total
‘e’ – Extracted: Indicates the deciduous teeth that have
number affected by the number examined, i.e.
been extracted due to caries or are so badly decayed that
they are indicated for extraction. Because of wide varia- Total number of affected
tions in the time of exfoliation of deciduous teeth, it is Per cent affected 5
Total number examined
difficult to determine whether a tooth missing from the
deciduous dentition was normally exfoliated or was ex- (E) Per cent filled
tracted because of advanced caries. If it is established ac- To calculate the per cent of filled teeth, divide
curately that a missing deciduous tooth has been lost due the total ‘f’ component by the total ‘def’, i.e.
to caries, include it with those indicated for extraction.
‘f’– Filled: Indicates the number of deciduous teeth at- Total number of filled teeth
Per cent filled 5
tacked by caries but have been restored without any re- Total 'deft'
current decay present. A tooth may have several fillings
(F) Extracted deciduous teeth per 100 children
but it is counted as one-tooth. If a tooth has a filling but
To calculate the extracted deciduous teeth per
shows evidence of recurrent decay, it is counted as a
100 children (EDT/100), divide the total num-
decayed tooth.
ber of extracted teeth by the total numbers
Coding criteria for primary tooth dentition
examined and multiply by 100, i.e.
Code Criteria Total number of extracted teeth  100
EDT / 100 
E Excluded tooth or tooth space Total number examined
P1 Sound deciduous tooth
Q.12. Oral hygiene index simplified.
P2 Filled deciduous tooth
P3 Decayed deciduous tooth Ans.
When tooth is absent [Ref LE Q.9]
‘0’ Missing tooth – unerupted, congenitally missing or Q.13. CPITN probe.
missing for any other reason
Ans.
‘X’ Extracted deciduous tooth
l The recommended periodontal probe for use with CPITN
Calculation of Def index was first described by WHO (TRS 621-1978). This
For deciduous or primary teeth, the maximum ‘deft’ probe was designed for two purposes: measurement of
score for an individual would be 20, since primary den- pocket depth, and detection of subgingival calculus.
tition has a maximum of 20 teeth. The maximum score The CPITN probe is thin at the handle and is of light-
for ‘defs’ will be 88, since the primary dentition has a weight (5 g). This probe is particularly designed for
maximum number of 20 teeth. gentle manipulation of often very sensitive soft tissues
(A) Individual def score around the teeth.
Total each component, i.e. d, e and f separately, l The pocket depth is measured through colour coding with

then total d 1 e 1 f 5 def a black mark starting at 3.5 mm and ending at 5.5 mm.
(B) Group average The probe has a ‘ball tip’ of 0.5-mm diameter that allows
Total d, e and f for each child examined. easy detection of subgingival calculus. This feature com-
Then divide the total ‘def’ by the number of bined with lightweight probe facilitates the identification
children examined, i.e. of the base of the pocket, thus decreasing the tendency of
false reading by over measurement.
Total def l A variant of this basic probe has two additional lines
Average def 5
Total number of teeth children examined at 8.5 mm and 11.5 mm from the working tip. The
708 Quick Review Series for BDS 4th Year, Vol 1

additional lines may be of use when performing a l For examination procedure, the tooth is probed with a
detailed assessment and recording of deep pockets for force of no more than 20 g, described as a force in which
the purpose of preparing a treatment plan for complex a probe point can be inserted under the fingernail without
periodontal therapy. causing pain or discomfort, to determine pocket depth.
l The Joint Working Committee of WHO/FDI has advised l When gently inserting the probe into the gingival

the manufactures of CPITN probes to identify the instru- pocket, the ball tip should follow the anatomical con-
ments as either ‘CPITN-E’ for epidemiological probe figuration of the tooth root surface.
with 3.5- and 5.5-mm markings, or ‘CPITN-C’ for clini- l As stated earlier, the total extent of the pocket should be

cal probe with additional 8.5- and 11.5-mm markings. examined for at least six points on each tooth: mesio-
buccal, midbuccal, distobuccal and the corresponding
Probing procedure lingual sites. For detecting subgingival calculus, the
l A tooth is probed to determine pocket depth and to lightest possible force should be used to allow move-
detect subgingival calculus and bleeding response. ment of probe’s ball tip along the tooth surface.
l The probing force can be divided into ‘working compo- l If the subject feels pain during the probing procedure,

nent’ – to determine pocket depth, and ‘sensing compo- this is an indication of too much force. The probing may
nent’ – to detect subgingival calculus. be done by withdrawing probe between each probing, or
l The working force should not be more than 20 g – A by probe tip remaining in the sulcus or pocket in order
practical test for establishing this force is to gently in- to walk the probe around each surface (i.e. buccal and
sert the probe point under the fingernail without causing lingual) of the tooth (Cutress, Ainamo and Sardo-Infirri,
pain or discomfort. 1987). ‘Walking’ the probe should be done with short
l The probe is inserted between the tooth and the gingiva, upward and downward movements.
and the sulcus depth or pocket depth is noted against the l After probing, the gingiva or gum of the examined tooth

colour code or measuring lines. The ball end of the probe should be inspected for the presence or absence of
should be kept in contact with the root surface. The direc- bleeding before the subjects are allowed to swallow or
tion of the probe during insertion should, whenever pos- close their mouths. Bleeding may be delayed for up to
sible, be in the same plane as the long axis of the tooth. 10 to 30 seconds after probing.
l For ‘sensing’ subgingival calculus, the lightest possible
Q.14. Sulcus bleeding index.
force that will allow movement of the probe ballpoint
along the tooth surface is used. Ans.
l Pain to the patient during probing is, in most cases, in-
[Ref LE Q.8]
dicative of the use of a too heavy probing force.
l Recommended sites for probing are mesial, midline and Q.15. Moller index.
distal on both facial and lingual/palatal surfaces.
Ans.
l The probing may be done by withdrawing the probe

between each probing, or alternatively, with the probe This indexing system was developed by I.J. Moller and
tip remaining in the sulcus, the probe may be ‘walked’ S. Poulsen in 1966, as a standardized system for diagnos-
around the tooth. ing, recording and analysing dental caries data.
l Sites, in addition to the recommended ones, should be The basis for the development of this system is to make
probed if there is suspicion that a higher scoring condi- available a system that can be used in different situations.
tion is present. The advantage of the system seems to be its flexibility in
l When CPITN was first described, there was no specific meeting the varying needs of different types of clinical
rule for the number of times a tooth should be probed studies on dental caries.
for examination procedure. It was only stated that the A standardized system for diagnosing, recording and
number of probings would depend on the condition of analysing dental caries data includes:
the surrounding tissue, and that exceeding four probings (1) Standardization of diagnostic criteria
per sextant would be rare (Ainamo et al., 1982). (2) Standardization of the equipment used for examination
l According to later literature, a tooth should be probed (including the circumstances under which the examina-
for at least six points: mesiobuccal, midbuccal, disto- tion was carried out)
buccal and the corresponding sites on the lingual sur- (3) Standardization of the recording procedures and field
face. Also, when first described, it was stated that the records
probing force should be no more than 25 g. Now, the Diagnostic criteria
probing force should be no more than 20 g. Further- The diagnostic criteria on which this system is based
more, there was initially no differentiation with the were originally defined by Moller (1966). The criteria
procedural method based on age. are specified for (i) pit and fissure surfaces, (ii) smooth
Section | I  Topic-Wise Solved Questions of Previous Years 709

surfaces and (iii) radiographic evaluation of proximal Q.16. Silness and Loe plaque index.
surfaces.
Ans.
Untreated carious lesions are divided into four types:
Type 1, Type 2, Type 3 and Type 4, which make it pos- [Ref LE Q.6]
sible to exclude certain types of carious lesions either in
Q.17. Plaque and calculus components of periodontal
the diagnosis or during the analysis.
disease index.
Equipment used
The instruments used in this system include unscratched, Ans.
plane mouth mirrors and standardized dental probes
[Ref LE Q.7]
(‘Hoist’ probe, C.G. Brincker and Copenhagen). Each
probe is used only once after which it is sent to the Q.18. Scoring criteria of CPITN.
manufacturer for restandardization.
Ans.
The dental examination should be carried out with the
patient seated in a dental chair under the light of a high [Same as SE Q.1]
quality operating lamp. Precleaning of teeth, isolation
Q.19. Loe and Silness index.
with cotton rolls and saliva ejector and drying with
compressed air is highly recommended. Radiographic Ans.
examinations of proximal surfaces are carried out wher-
[Same as SE Q.3]
ever possible.
Recording procedure Q.20. Periodontal index Russell.
In the clinical procedure, all teeth in the dentition
Ans.
(excluding third molars) are examined. A tooth is re-
corded as erupted when any part of it projects through [Same as SE Q.4]
the gingiva.
Q.21. DMFT index.
For each permanent molar and premolar, five surfaces
are examined. For each canine and incisor, four surfaces Ans.
are examined.
[Same as SE Q.7]
For primary dentition, the examination is performed ac-
cordingly. Examination of the teeth is done in routine Q.22. Rules governing caries indices.
order, from maxillary right to maxillary left, and from
Ans.
mandibular right to mandibular left. For each tooth, the
examination is performed by examining each surface in [Same as SE Q.10]
the following sequence: (1) occlusal surface, (2) mesial
surface, (3) vestibular surface, (4) distal surface and
(5) lingual or vestibular surface.
SHORT NOTES:
The recordings are done on a field record form, which Q.1. DMFT index.
is designed in such a way that it serves as a punch guide.
Ans.
The tooth surfaces are indicated as follows: ‘O’, occlu-
sal; ‘M’, mesial; ‘B’, vestibular; ‘D’, distal; and ‘L’, [Ref LE Q.1]
lingual.
Q.2. Advantages and limitations of CPITN index.
The coding done on the recording form is as follows:
Ans.
Code Criteria [Ref LE Q.4]
0 Sound tooth Q.3. Classify indices in dentistry.
1 Type 1 caries
Ans.
2 Type 2 caries
[Ref LE Q.5]
3 Type 3 caries
4 Type 4 caries
Q.4. Ideal properties of an index.

5 Filled tooth Ans.


6 Missing tooth due to caries [Ref LE Q.5]
7 Tooth or tooth surface not erupted Q.5. Russell’s index.
8 Tooth missing for reasons other than caries
Ans.
9 Congenitally missing, and not recordable
[Ref LE Q.3]
710 Quick Review Series for BDS 4th Year, Vol 1

Q.6. OHI-S. Q.10. Community fluorosis index.


Ans. Ans.
[Ref LE Q.9] [Ref LE Q.5]
Q.7. Dental caries index for mixed dentition. Q.11. Scoring criteria of Russell’s index.
Ans. Ans.
l In case of mixed dentitions, the caries indices for perma- [Ref LE Q.3]
nent teeth and deciduous teeth have to be done separately.
Q.12. Community periodontal index (CPI).
l A DMFT or DMFS and a ‘deft’ or ‘defs’ are never

added together. Each child is given a separate index for Ans.


permanent and primary teeth.
[Ref SE Q.9]
l The index for permanent teeth is usually determined

first and then the index for primary teeth. Q.13. Reversible index.
Q.8. Write a short note on CPITN probe. Ans.
Ans. [Ref LE Q.1]
l The recommended periodontal probe for use with CPITN Q.14. Mention any four requisites for ideal index de-
was first described by WHO (TRS 621-1978). This signing in epidemiological survey.
probe was designed for two purposes: measurement of
Ans.
pocket depth, and detection of subgingival calculus.
l The CPITN probe is both thin at the handle and is of [Same as SN Q.4]
very lightweight (5 g). This probe is particularly de-
Q.15. Mention the ideal requirements of index system in
signed for gentle manipulation of often very sensitive
epidemiology.
soft tissues around the teeth.
l The pocket depth is measured through colour coding with Ans.
a black mark starting at 3.5 mm and ending at 5.5 mm.
[Same as SN Q.4]
l The probe has a ‘ball tip’ of 0.5-mm diameter that

allows easy detection of subgingival calculus. Q.16. Russell’s periodontal index.


l This feature combined with the light probe weight facili-
Ans.
tates the identification of the base of pocket, thus decreas-
ing the tendency for false reading by overmeasurement. [Same as SN Q.5]
Q.9. Scoring codes in CPITN index. Q.17. CPITN probe.
Ans. Ans.
[Ref LE Q.4] [Same as SN Q.8]

Topic 8
Health Statistics
COMMONLY ASKED QUESTIONS
LONG ESSAYS:
1 . Define data. Give its types and describe the various ways of presentation.
2. Define sample. Describe sampling designs.
3. Enumerate various measures of dispersion, and describe in detail the tests of significance.
4. Define planning. Write in detail the various steps in planning.
5. What is a survey? Explain the steps of survey in detail.
6. Define evaluation. Write briefly the importance of evaluation in a public health programme.
7. What is statistics and biostatistics? Give the importance of statistics in epidemiological investigations and give
the meaning of the terms ‘standard deviation’, ‘median’ and ‘mean’.
Section | I  Topic-Wise Solved Questions of Previous Years 711

8. Define sample and describe different methods of sampling techniques. [Same as LE Q.2]
9. What is sample in epidemiological investigation? Give broad classification of sample. State, with reasons, which
is an ideal sample and describe the method of selection of the same. [Same as LE Q.2]
10. Define sample in epidemiological survey. Discuss various types of samples in epidemiological survey and
mention their importance. [Same as LE Q.2]
11. Define plan. Describe a plan for controlling oral diseases among schoolchildren in a town. [Same as LE Q.4]
12. Steps in planning. [Same as LE Q.4]
13. Define survey. Give its aims and objectives. Describe the scientific methods of dental survey procedures.
[Same as LE Q.5]
14. Define survey. What are different types of oral health surveys? Write in detail the procedure of conducting an
oral health survey in rural areas. [Same as LE Q.5]
15. Steps in conducting a survey. [Same as LE Q.5]
16. Define survey. Describe the step-by-step procedure of conducting survey of oral disease in Andhra Pradesh.
[Same as LE Q.5]
17. Define evaluation. Enumerate the different methods of evaluation. [Same as LE Q.6]

SHORT ESSAYS:
1. Probability sampling methods. [Ref LE Q.2]
2. Chi-square test. [Ref LE Q.3]
3. Steps of evaluation. [Ref LE Q.6]
4. Mean, median and mode. [Ref LE Q.7]
5. Gaussian curve.
6. Pathfinder surveys.
7. Types of bar diagrams. [Ref LE Q.1]
8. Index age groups recommended by WHO for survey.
9. Measures of central tendency. [Same as SE Q.4]
10. Statistical average. [Same as SE Q.4]
11. Normal curve. [Same as SE Q.5]

SHORT NOTES:
1. Cluster sampling.
2. Uses of statistics. [Ref LE Q.7]
3. Steps in planning oral health care. [Ref LE Q.4]
4. Stratified random sampling. [Ref LE Q.2]
5. Normal curve and its properties.
6. Standard deviation. [Ref LE Q.3]
7. Multistage sampling.
8. Pilot survey.
9. Systematic random sampling.
10. Mode.
11. Network sampling. [Ref LE Q.2]
12. Frequency polygon. [Ref LE Q.1]
13. Measures of central tendencies.
14. Evaluation. [Ref LE Q.6]
15. Pie chart. [Ref LE Q.1]
16. Name criteria for evaluation.
17. Bar chart. [Ref LE Q.1]
18. Survey. [Ref LE Q.5]
19. Primary source data.
20. Planning. [Ref LE Q.4]
21. Histogram.
22. Method of collection of data. [Ref LE Q.1]
23. Questionnaire survey.
24. Impact evaluation.
25. Importance of pilot survey. [Same as SN Q.8]
712 Quick Review Series for BDS 4th Year, Vol 1

SOLVED ANSWERS
LONG ESSAYS:
Q.1. Define data. Give its types and describe the various Presentation of data
ways of presentation. Once data are collected, the first object is to express it
in a sample form, which will permit, directly or by means of
Ans.
further calculations, conclusions to be drawn.
l Data are a set of values of one or more variables re- The collected data can be represented as follows:
corded on one or more individuals. I Tabulation
l Data consist of discrete observations of attributes or l Simple table
events that carry little meaning when considered alone. l Association table
l Data are the raw materials of statistics. From data, l Correlation table
analysis is made. Principles are formulated and conclu- l Master table
sions are drawn. l Frequency distribution table
l Data are asset of isolated raw facts, figures and statistics II Charts and diagrams
that are interpreted and can be processed, manipulated l Bar charts
and maintained. (a) Simple bar chart
(b) Multiple bar chart
Types of data
(c) Component bar chart
(a) Based on characteristics
l Histograms
i. Qualitative data
(a) Frequency polygon
l Nominal data
(b) Frequency curve
l Ordered data
l Pie chart
l Interval data
l Pictogram
l Ratio data
III Line diagrams
ii. Quantitative data
IV Statistical maps
l Discrete data

l Continuous data I. Tabulation


iii. Quantitative continuous data l It is the process of systematically presenting the
(b) Based on sources numerical facts in such a form that the information
i. Primary data thus displayed is more readily understood.
ii. Secondary data l This representation usually takes place in the form
(c) Based on field of arranging numerical data in columns and rows.
i. Character type i. Simple table
ii. Numeric type l Brings out a specific point or answers a specific
iii. Logic type question. For example, population of India.
iv. Date type ii. Association table
l When we have to show the association between
i. Primary data
two attributes, we use the association table. It is
These are the data obtained directly from an individual.
also called as 232 table, because it consists of
Advantages
two rows and two columns.
l Precise information
iii. Correlation table
l Reliable
l This type of table is used for showing the rela-
Disadvantages
tionship between two quantitative varieties of
l Time-consuming
continuous type.
l Expensive
l The distribution of objects/persons gives an idea
ii. Secondary data
whether there is a positive or negative correla-
These are obtained from outside source, e.g. hospital
tion between varieties.
records and school registers.
iv. Master table
l Sometimes, the data that can be presented in
{SN Q.22}
numerous smaller tables are presented in one
Methods of collecting data table only. This type of table gives maximum
l Questionnaires information at a glance.
l Surveys v. Frequency distribution table
l Records l In a frequency distribution table, the data are first

l Interviews split up into convenient groups (class intervals)


Section | I  Topic-Wise Solved Questions of Previous Years 713

and the number of items (frequency) occurring in III. Line diagrams


each group. This diagram is one of the simplest types of diagrams
used to study the changes of values of variables over
[SE Q.7] time.
{II. Charts and diagrams} IV. Statistical maps
Statistical maps are used when statistical data refer to
{SN Q.17} geographic or administrative areas.
l {(Bar charts are a way of presenting a set of num- Q.2. Define sample. Describe sampling designs.
bers by the length of a bar – The length of the bar is
proportional to the magnitude to be represented. Ans.
l Bar charts are a popular medium of presenting statis- l A sample is a part of a population called the ‘Universe’,
tical data because they are easy to prepare and enable ‘Reference’ or ‘Parent’ population.
values to be compared visually. l Sampling can be defined as an investigation of part of a
The following are some examples of bar charts:)} population and collection of information which can then
be generalized to cover the whole population.
(a) Simple bar chart
Methods of sampling technique
Bars may be vertical or horizontal. The bars are
It can be divided into the following types:
usually separated by appropriate spaces with an
eye to neatness and clear presentation. [SE Q.1]
(b) Multiple bar chart
Two or more bars can be grouped together. (A) {Probability sampling
l Simple random
(c) Component bar chart
l Stratified random
The bars may be divided into two or more parts,
l Systematic random
each part representing a certain item and propor-
l Area/cluster sampling
tional to the magnitude of that particular item.
l Histograms
(B) Nonprobability sampling
l Accidental/convenience
A histogram is a special type of bar chart. The
l Judgement/purposive
successive groups of data are linked in a definite
l Network/snowball
numerical order. It is a pictorial diagram of
l Quota sampling
frequency distribution. For example, age-wise
l Dimensional sampling
prevalence of dental caries.
l Mixed sampling

(A) Probability sampling


{SN Q.12} It is a method of sampling, the distinctive feature of
which is that each individual unit in the total popula-
Frequency polygons
l
tion has a known probability of being selected.
A frequency polygon can be used as a diagrammatic (a) Simple random sampling
method to represent the frequency distribution. It is ob- l Every member of the population has an equal
tained by joining the midpoints of the histogram blocks. chance of being included in the sample.
For example, age-wise prevalence of dental caries. l This type of sampling is used when the popula-

tion is homogenous.
Pictograms
l The basic procedure is as follows:
It is a pictorial or diagrammatical representation l Preparing a sampling frame
of data by a pictorial symbol. For example, popu- l Deciding on the size of sample
lation per physician. l Selecting the required number of units

l Several methods of achieving random selec-

{SN Q.15} tion, e.g. lottery method}


Pie diagram/charts
l (SE Q.1 and SN Q.4)
Here the entire graph looks like a pie and its compo-
{((b)  Stratified random sampling
nents represent slices cut from a pie. The pie chart
l This sampling is drawn in a systematic way
has different sectors corresponding to the frequen-
so that each portion of the sample represents
cies of the variables in distribution.
a corresponding stratum of the universe.
714 Quick Review Series for BDS 4th Year, Vol 1

A stratified random sample is obtained using the (e) Dimensional sampling


following procedure: In this technique only a small sample is needed,
l The population to be sampled is subdivided each case will be examined in detail.
into groups known as strata so that characteris- (f) Mixed sampling design
tics of each group are homogenous. It constitutes the combination of both probability
l A simple random sample is then chosen from and nonprobability sampling procedures.
each stratum. This method provides greater ac-
Q.3. Enumerate various measures of dispersion, and
curacy and can concentrate on a wider geo-
describe in detail tests of significance.
graphical area.)}
Ans.
[SE Q.1]
Measures of dispersion helps us to know the spread of
{(c) Systematic random sampling
l
observation on each side of the average.
This process involves the selection of certain
l
l Dispersion is a degree of spread or variation of the vari-
elements in a series according to predetermined able about a central value.
sequence.
l This method can be adopted as long as there is Methods of dispersion
no periodicity of occurrence of any particular ( a) Range
event in the population. (b) Mean deviation
(d) Area or cluster sampling (c) Standard deviation
l The cluster or area sample is particularly appro-
(a) Range
priate when a simple random sample proves to l Range is defined as the difference between the high-
be prohibitive in terms of travel, time and cost. est and the lowest figures in a given sample.
l Here the sampling units are clusters, and the
l It is the simplest measure of dispersion.
sampling frame is a list of these clusters. l This measure gives no information about the values
l In a multistage random sampling, a large scale
that are between extreme values.
of survey is used. For example, selected school, Advantage
roll numbers and classes. l It is easy to calculate.
Advantages Disadvantages
l Economical
l It is unstable.
l Administratively simple}
l It is affected by one extremely high or low score.
(B) Nonprobability sampling l It is of no practical importance.
(a) Accidental or convenience sampling (b) Mean deviation
One will not always be able to randomly sample l It is the average of deviation from arithmetic mean:
from the population of interest. They are inexpen-
sive and less time-consuming. M. D.   X  X i / n
(b) Judgement or purposive sampling
This involves the selection of elements, which rep- where o (sigma) is the sum of, X is the arithmetic mean
resent a typical sample from a target population. and Xi is the value of each observation in the data.
When this approach is used, the quality of sample
selected depends on the accuracy of researcher’s
judgement of what constitutes a typical sample.
{SN Q.6}

{SN Q.11} (c) Standard deviation


l It is the most important and widely used measures of
(c) Network or snowball sampling dispersion.
This involves a multistage technique that utilizes l It is also known as root mean square deviation.
social network of individuals who tend to share com- l Greater the standard deviation, greater will be the
mon characteristics. magnitude of dispersion from the mean.
l The standard deviation is the square root of variance,

(d) Quota sampling 2


This procedure involves the selection of propor- (X  X i )
tional samples of subgroups within a target popula- SD 
n
tion to ensure generalization of findings.
Section | I  Topic-Wise Solved Questions of Previous Years 715

Test of significance (d) The standard error of this mean difference is


It is a test done mainly to find out whether the differ- calculated by the formula S/√n.
ences observed between the two groups, whenever (e) It is calculated by the following formula:
two sets of observations are compared, is because of
sampling or any other factors. Mean difference
t5
Standard error test for large samples Standard error of the mean difference
If a sample has more than 30 observations, then it is
(f) The degree of freedom (df) for this calcula-
considered to be a large sample.
tion t is n – 1, where n is the number of pairs
The formula used to test difference between any two large
of observation.
samples in terms of mean or proportion is as follows:
(g) From t distribution table, P is noted down cor-
l Standard error of difference between two means
responding to (n – 1) df and then calculated
SE  (S12 / n1 )  (S22 /n2 ) value of t.
(h) If P is more than 0.05, the mean difference is
where S1 and S2 are standard deviation of two sam- insignificant, and if it is less than 0.05, the
ples, and n1 and n2 are the respective sample sizes. mean difference is significant.
[SE Q.2] ii. The unpaired t-test
(a) Set up the null hypothesis that the difference
{Chi-square test in two means is zero.
It is an alternate method of testing the significance of dif- (b) Calculate the means and standard deviations
ference between two proportions. for the two groups separately.
(a) Test the null hypothesis (c) Calculate the standard error of difference of
Set up a null hypothesis that ‘there is no difference be- means.
tween the two’ and then proceed to test the hypothesis. (d) The standard error of the difference between
(b) Applying the x2 test the two means is calculated by the following
formula:
(O  E )2
2 
E n1  n2  (n1  1) S12  (n1  2 ) S22 
SE  [ ]  
where O is the observed value, and E is the expected n1n2  n1  n2  2 
value.
(c) Finding the degree of freedom where S1 and S2 are the standard deviations of
It depends on the number of columns and rows, and is the two groups, and n1 and n2 are the respec-
given by the following formula: tive number of observations in the two groups.
(e) Calculate t by the following formula:
df  (c  1) (r 1)
where c 5 number of columns, and r 5 number of rows. Difference between the means of two samp les
t5
(d) Probability tables Standard error of the difference betweeen the two means
Depending on the value of ‘P’, the conclusion is
(f) Compute the pooled degrees of freedom as
drawn.}
n11n1 – 2.
Standard error test for small samples
(g) Refer to table of ‘t distribution’ and find out
l A sample is considered to be small if it has less than
the probability level P corresponding to the
30 observations. The test applied is called the t-test.
above degrees of freedom and calculated t.
l When the investigation is in terms of comparing
(h) conclusions are made on the basis of this P.
the observations carried out on the same individu-
als, say before and after certain experiment, such Q.4. Define planning. Write in detail the various steps in
comparisons are called paired comparisons; when planning.
the observations are carried out in two indepen-
Ans.
dent samples and their values are compared, it is
known as unpaired comparison.
i. t-test for paired comparison {SN Q.20}
(a) First, the null hypothesis that ‘the two sets of l Planning is defined as a process of preparing a set of
observations are not different’ is set up. decisions for action in the future, and it must precede
(b) Difference between the readings before and development and change in any organization.
after experimentation is calculated for each l It is also defined as a systemic approach to defining
individual. the problem, setting priorities, developing specific
(c) Mean and standard deviation of these differ- goals and objectives and determining alternative
ences are calculated. strategies and a method of implementation.
716 Quick Review Series for BDS 4th Year, Vol 1

l If priorities are not set up accordingly, the pro-


{SN Q.3}
gramme may not serve those individuals for
Steps in a planning process whom care was needed the most.
l Conducting a needs assessment l Once the priorities are set up and the target group

l Determining priorities is identified, the type of programme should be


l Development of goals, objectives and activities established.
l Identification of resources and constraints (c) Development of programme goals, objectives and
l Alternative strategies activities
l Implementation, supervision, evaluation and revision l Programme goals are broad statements on the overall

purpose of a programme to meet a defined problem.


l Programme objective should be more specific.

(a) Conducting a needs assessment It should specify the following:


The first step in the planning process is to conduct l The nature of the situation or condition to be

needs assessment. The main reasons are as follows: attained


l To define the problem and to identify its extent l Extent or magnitude of situation

and severity l The area where the programme will be conducted

l To obtain the profile of the community to ascer- After the goals and objectives are specified, the pro-
tain causes of the problem gramme activities are developed for achieving the
l To evaluate the effectiveness of programme goals and objectives.
The information gathered consists of the following: (d) Identification of resources and constraints
l General information: the number of individuals, Resources to be considered are personnel, equipment
geographic distribution, diet, socioeconomic sta- and supplies, facilities and financial resources needed
tus, ethnic background, public service available for the programme.
and school systems The criteria that are used to determine the resources to
l Pattern of dental disease and its distribution be used include the following:
based on clinical examination records l Appropriateness: The most suitable resources to

l Status of dental health programmes get the job done should be selected.
l Funds, facilities and labour available l Adequacy: It refers to the extent or degree to

l Preventive dental programmes existing in the which the resources would complete the job.
community l Effectiveness: It refers to how capable the re-

Analysis of the data collected sources are at completing the job.


l Socioeconomic status: It tells whether the popu- l Efficiency: It refers to the cost and the amount of

lation can afford dental care. time expended to complete the job.
l Population breakdown: It tells the cultural and Constraints
language issues that should be considered. l Constraints may result from organizational poli-

l Age distribution: It tells about the target groups cies, resource limitations or characteristics of
and helps in setting priorities for planning. community.
l Educational status: It tells about the educational l If constraints are identified earlier, it helps in

level of community. modifying the design of the programme.


l Public transport system: It gives an idea regarding Constraints that commonly occur in dental programmes
a population’s ability to get health care service. include the following:
l Labour data: It gives us information as to the l Lack of funding

number of dentist providing care. l Inadequate transport system

l Fluoride status: It is essential for dental planning. l Labour shortages

(b) Determining priorities l Lack of or inadequate facilities

l It is a method of imposing people’s value and l Negative community attitudes towards dentistry

judgements of what is important. l Lack of time

l When resources are limited, it becomes neces- (e) Alternative strategies


sary to set up priorities of the community and l It is important to generate a sufficient number of

community should be involved in establishing alterations so that out of all options at least one
the priorities. option may be considered to be acceptable.
l Hence, a health advisory committee is formed, (f) Implementation, supervision, evaluation and revision
which represents consumers, community, leaders The process of putting a plan into operation is known as
and providers. implementation phase.
Section | I  Topic-Wise Solved Questions of Previous Years 717

Rules for implementation: incidence study. In prevalence study, the occurrence of


l The activity should be clearly specified. a disease is expressed at a given point in time, whereas
l Someone must be held responsible for the whole in incidence study, the amount of new disease in a
activity. given population is measured over a period of time.
l The preparatory steps should be identified before B. Controls: It is a parallel group formed according to
doing the activity. the cases, and it should not be exposed to the fac-
l The steps should be listed in order and checked so tors under study.
that no step is missed out. C. Method of study: It could be case–control study or co-
l Beginning and ending of each step should be hort study. In the case–control study, the starting point
determined. is a group with the disease under investigation, whereas
l The organization affected by the activity should in the cohort study, the population of individuals is clas-
be consulted, and the potential problems should sified according to various factors of interest.
be identified. iii. Selecting the sample
l The resources needed should be specified along Since it is not possible to study the whole population,
with their sources. only a sample or only a part of study population is
l The constraints need to be specified and addressed. taken for investigation.
l The people involved in the programme should be iv. Conducting the examination
aware of what is expected of them and by when. Before conducting the examination, one has to orga-
l Monitoring, evaluating and revising the pro- nize the survey, which includes the following:
gramme. (a) Obtaining approval from the authorities: Per-
l Once the programme is implemented, it requires mission to examine population groups must be
continuous surveillance of all activities. obtained from a local, regional or national au-
thority. In case of schoolchildren, permission
Q.5. What is a survey? Explain the steps of survey in
must be obtained from the school authority.
detail.
(b) Budgeting: A budget should be prepared, which
Ans. should include all the resources required, in-
cluding personnel to carry out the survey.
{SN Q.18} (c) Scheduling: Basic oral health examination of
child takes 5–10 min, and for adult it takes 15–
l A survey is an investigation in which information is 20 min. Using this as the guideline, daily and
systematically collected but in which experimental weekly schedules can then be prepared and
method is not used. handed over to the survey personnel, school as
l Surveys are the methods of collecting data, and ana- well as health authorities.
lysing and evaluating them to determine the amount (d) Emergency care and referral: A list of referral
of disease problems in a community. facilities and addresses should be prepared be-
fore the survey so that the emergency conditions
can be referred to appropriate care.
Objectives
(e) Validity and reliability of the data: In any survey,
l To provide overall health status and needs of a population
the diagnostic method chosen should be both
l To monitor changes in disease levels and patterns
valid and reliable. Validity of a test is its ability to
Steps in survey measure what it is intended to measure. The reli-
i. Establishing the objective ability of a test is its ability to give the same re-
ii. Designing the investigation sults if repeated. Each examiner should diagnose
iii. Selecting the sample the condition in the same way on every occasion
iv. Conducting the examination as should other examiner using the same criteria.
v. Analysing the data The following precautions are usually taken:
vi. Drawing the conclusion i. Keep the number of examiners to a minimum.
vii. Publishing the reports ii. Discuss interpretation of borderline problems
i. Establishing the objective carefully in advance.
The investigator must be clear about the objective of the iii. Use only one make and design of explorer.
investigation before considering its design. iv. Divergences of opinion or of observation can
ii. Designing the investigation be discussed and minimized.
A. Types of study: Surveys may be descriptive or analyti- v. Circulate among examiners any rules or sys-
cal. It is again subdivided into prevalence study and tems which may seem pertinent.
718 Quick Review Series for BDS 4th Year, Vol 1

(f) Personnel and organization i. Formative evaluation


i. Recording clerk: The recording clerk is one l Formative evaluation refers to the internal evaluation

who is able to follow the instructions exactly of a programme. It is the examination of the process
and to print numbers and letters clearly as or activities of a programme as taking place.
instructed by the examiner. l It is usually carried out to aid in the development of

ii. Organizing clerk: The organizing clerk helps a programme in its early phases. For example, a fluo-
to maintain a constant flow of subjects and ride rinse programme is initiated in the neighbour-
to enter general descriptive information on hood, in which paraprofessionals are trained to ad-
the recorded forms. He or she is also re- minister three types of fluoride rinses.
sponsible for the adequate supply of sterile l After 3 days of operation, the work of paraprofes-

instruments. sionals is observed to determine the extent to which


(g) Instruments and supplies the sequence is being maintained.
l Plane mouth mirrors – 30 per examiner l The observation and determination of correct or incor-

l Periodontal probes – 30 per examiner rect procedure sequence provide an example of exam-
l Several pair of tweezers ining the activities of a programme as occurring.
l Containers and concentrated sterilizing l If the sequence is incorrect, formative evaluation

solution allows the programme to make remedial changes at


l A washbasin that point, and thereby improve the performance.
l Cloth or paper hand towels l It is used by programme developers and programme

l Gauze staff members concerned with, whether various com-


(h) Examination area: The area for conducting ex- ponents of a programme are workable or changes
amination should be planned and arranged for should be made to improve programme activities.
maximum efficiency and ease of operation. ii. Summative evaluation
v. Analysing the data l Summative evaluation judges the merit or worth of a

Then the data are analysed. Different tables and graph- programme after it has been in operation.
ical presentation of the data enable meaningful conclu- l It is an attempt to determine whether a fully opera-

sion to be drawn. tional programme is meeting the goals for which it


vi. Drawing the conclusion was developed.
The conclusion given should be specifically related to l Summative evaluation is aimed at programme

the investigation that has been carried out. decision-makers, who will decide whether to con-
vii. Publishing the reports tinue or terminate a programme.
The report of the survey should contain the following:
l Statement and purpose of the survey
[SE Q.3]
l Materials and methods {Basic steps in evaluation
l Results i. Determine what is to be evaluated
l Discussion and conclusions A. Evaluation of structure: This is an evaluation of
l Summary or abstract whether facilities, equipment, manpower and orga-
Q.6. Define evaluation. Write briefly the importance of nization meet the standards.
evaluation in a public health programme. B. Evaluation of process: The process of dental care
includes the problems of recognition, diagnostic
Ans. procedures, treatment and clinical management,
care and prevention. The way in which the activities
of a programme are carried out is evaluated by com-
paring with predetermined standards.
{SN Q.14} C. Evaluation of outcome: This is concerned with the
end results of a programme. The end results should
l Evaluation is defined as the judgement of merit or
improve dental health in a community. Then oppor-
worth about a particular person, place or thing.
tunities should be provided for discussing evaluation
l It is the collection and analysis of information to
results.
determine the programme performance.
ii. Planning the methodology
Types of evaluation
l Certain standards and criteria must be included in the
According to Scriven, the two types of evaluation are
planning stage.
i. Formative evaluation
l A format should be prepared for gathering informa-
ii. Summative evaluation
tion in keeping with the purpose of evaluation.
Section | I  Topic-Wise Solved Questions of Previous Years 719

i. Gathering information
ii. For comparing health status of one country with that
l Evaluation requires collection of data information.
of another, and for comparing the present status with
The type of information may include political,
that of the past
cultural, economic, environmental and administra-
iii. For planning and administration of dental health
tive factors.
services
l The above factors influence the health situation
iv. For prediction of health trends
as well as the morbidity and mortality statistics.
v. To evaluate progress and development of disease
ii. Analysis of results
vi. To help dentist to think quantitatively and to be able
l Once information has been gathered, analysis
to assess probabilities
and interpretation of data should take place
within a short time.
iii. Taking action Standard deviation
l For evaluation to be truly productive, emphasis l It is the most important and widely used measure of

should be placed on actions (actions designed to dispersion.


support, strengthen or modify the services involved). l It is also known as root mean square deviation.

l This may also call for shifting priorities, revising l Greater the standard deviation, greater will be the mag-

objectives or development of new programmes or nitude of dispersion from the mean.


services to meet previously unidentified needs. l Standard deviation is the square root of variance.

vi. Evaluation of quality of dental care programmes


Schonfeld suggested the following four levels of S. D.  √ ( X  Xi )2 / n
evaluation:
i. Evaluate the individual restoration, proce- [SE Q.4]
dure or service. {Mean
ii. Evaluate the mouth that is the relationship of l This measure implies the arithmetic average or arithme-
one dental procedure to another. tic mean.
iii. Consider the patient’s total oral health and l It is obtained by summing up all the observations and
the influence that dental care has had on the dividing the total by the number of observations.
attitude towards dentistry and on dentally For example, the fasting glucose levels of a sample of 10
related behaviour. children are as follows:
iv. A look at the family and community, evalu-
ate the dental services provided for groups 1 2 3 4 5 6 7 8 9 10
and communities and determine the number 56 62 63 65 65 65 65 68 70 71
of and social distribution of persons receiv-
ing adequate dental care. Total 5 650
vii. Re-evaluation Mean 5 650/10 5 65
Evaluation is an ongoing process and the health Mean is denoted by the sign ‘xˉ’
activities should be more relevant, efficient and Advantages
effective.} l Easy to calculate

Q.7. What is statistics and biostatistics? Give the impor- l Easily understood

tance of statistics in epidemiological investigations, and l Utilizes entire data

give the meaning of the terms ‘standard deviation’, ‘me- l Amenable to algebraic manipulation

dian’ and ‘mean’. l Affords good comparison

Ans. Disadvantage
l Mean is affected by extreme values. In such cases, it
l Statistics can be defined as the science of collecting, sum- leads to bad interpretation.
marizing, presenting, analysing and interpreting the data.
l Biostatistics deals with the statistical methodologies Median
involved in biological sciences. l Here the data are arranged in an ascending or descend-

ing order of magnitude and the value of middle observa-


tion is located.
{SN Q.2}
Advantages
Uses of statistics
l It is more representative than mean.
i. To measure the state of health of community and to
l It does not depend on every observation.
identify its health problems, their nature etc.
l It is not affected by extreme values.}
720 Quick Review Series for BDS 4th Year, Vol 1

Q.8. Define sample and describe different methods of SHORT ESSAYS:


sampling techniques.
Q.1. Probability sampling methods.
Ans.
Ans.
[Same as LE Q.2]
[Ref LE Q.2]
Q.9. What is sample in epidemiological investigation?
Give broad classification of sample. State, with reasons, Q.2. Chi-square test.
which is an ideal sample and describe the method of Ans.
selection of the same.
[Ref LE Q.3]
Ans.
Q.3. Steps of evaluation.
[Same as LE Q.2]
Ans.
Q.10. Define sample in epidemiological survey. Discuss
various types of samples in epidemiological survey and [Ref LE Q.6]
mention their importance. Q.4. Mean, median and mode.
Ans. Ans.
[Same as LE Q.2] [Ref LE Q.7]
Q.11. Define plan. Describe a plan for controlling oral Q.5. Gaussian curve.
diseases among schoolchildren in a town.
Ans.
Ans.
When a large number of observations of any variable char-
[Same as LE Q.4] acteristic are taken at random to make it a representative
Q.12. Steps in planning. sample, it is framed in a frequency distribution table.
The following will be observed:
Ans. l Some observations are above the mean, while others are

[Same as LE Q.4] below it.


l When they are arranged in orders, maximum number of
Q.13. Define survey. Give its aims and objectives. De- frequencies will be seen in the middle around the mean and
scribe the scientific methods of dental survey procedures. fewer at the extremes, decreasing smoothly on both sides.
l Almost half of the observations lie above and the rest
Ans.
below the mean, and all observations are symmetrically
[Same as LE Q.5] distributed on each side of the mean.
Q.14. Define survey. What are different types of oral This type of distribution is called a normal distribution or
health surveys? Write in detail the procedure of con- Gaussian distribution.
ducting an oral health survey in rural areas. Characteristics of a normal curve
l It is a symmetrical bell-shaped curve.
Ans.
l Total area of the curve is 1. Its mean is zero and stan-
[Same as LE Q.5] dard deviation is 1.
l The parameters of mean, median and mode coincide at
Q.15. Steps in conducting a survey.
the centre.
Ans. l The maximum observations are in the centre and gradu-

[Same as LE Q.5] ally decrease towards the extremities on either side.

Q.16. Define survey. Describe the step-by-step procedure Q.6. Pathfinder surveys.
of conducting survey of oral disease in Andhra Pradesh. Ans.
Ans. l Pathfinder survey is a practical and economic survey
sampling methodology. The method used is a stratified
[Same as LE Q.5]
cluster sampling technique, which aims to include the
Q.17. Define evaluation. Enumerate the different meth- most important population subgroups likely to have dif-
ods of evaluation. fering disease levels and to cover a standard number of
subjects in specific index age groups at any one location.
Ans.
l In this way, statistically significant and clinically relevant
[Same as LE Q.6] information for planning is obtained at minimum expense.
Section | I  Topic-Wise Solved Questions of Previous Years 721

This methodology is suitable for obtaining the following effect of dental caries, the level of severe periodontal
information: involvement and the general effects of care provided
i. Prevalence of various oral diseases affecting the can be monitored in this age group.
population. l 65–74 years: This age group has become more im-

ii. Important variations in disease level, severity and portant with changes in age distribution and increase
need for treatment in the subgroups of population. in life span. Data for this group are needed for both
iii. A picture of age profiles of oral diseases in a popu- planning appropriate care for the elderly and moni-
lation is to provide information about severity and toring overall effects of oral care services.
progression of disease, and to give an indication as
Q.9. Measures of central tendency.
to whether the levels are increasing or decreasing.
Pathfinder survey is classified as follows: Ans.
l Pilot survey
[Same as SE Q.4]
It includes only the most important subgroups in
population and only one or two index ages, usu- Q.10. Statistical average.
ally 12 years and one other age group.
Ans.
l National pathfinder survey

It incorporates sufficient examination sites to cover [Same as SE Q.4]


all important subgroups that may have differing
Q.11. Normal curve.
disease levels or treatment needs and at least three
of index ages. This type of survey design is suit- Ans.
able for the collection of data for the planning and
[Same as SE Q.5]
monitoring of services in all countries.
Q.7. Types of bar diagram.
SHORT NOTES:
Ans.
Q.1. Cluster sampling.
[Ref LE Q.1]
Ans.
Q.8. Index age groups recommended by WHO for survey.
l Cluster sampling is used when the population forms
Ans. natural groups or clusters.
l This sampling is appropriate when a simple random
The index age groups are as follows:
sample proves to be prohibitive.
l 5 years for primary teeth.
l First, a sample of clusters is selected and then all the
l 12, 15, 35–44 and 65–74 years for permanent teeth.
units in each of the clusters are surveyed.
l In each age group, minimum 25–50 subjects are to be

considered. Advantages
l 5 years: This age is of interest in relation to the levels l It is administratively simple.

of caries in primary dentition, which may exhibit l It is less expensive.

changes over a shorter time span than the permanent


Disadvantage
dentition at other index ages.
l The findings cannot be generalized to the parent popu-
l 12 years: This is the age at which children leave pri-
lation.
mary school, and is also the last age at which reliable
sample may be obtained easily through the school Q.2. Uses of statistics.
system. At this age, all permanent molars would have
Ans.
erupted except the third molars. This age is also cho-
sen as the global monitoring age for caries for interna- [Ref LE Q.7]
tional comparisons and monitoring of disease trends.
Q.3. Steps in planning oral health care.
l 15 years: At this age, the permanent teeth have been

exposed to oral environment for 3–9 years. The as- Ans.


sessment of prevalence of caries at this age is there-
[Ref LE Q.4]
fore often more meaningful than at 12 years of age.
This age is also important for the assessment of peri- Q.4. Stratified random sampling.
odontal disease indicators in adolescents.
Ans.
l 35–44 years: This age group is the standard monitor-

ing group for the health conditions of adults. The full [Ref LE Q.2]
722 Quick Review Series for BDS 4th Year, Vol 1

Q.5. Normal curve and its properties. l By this method, each unit in the sampling frame would
have the same chance of being selected but the number
Ans.
of possible samples is greatly reduced.
When a large number of observations of any variable char- Q.10. Mode.
acteristic are taken at random to make it a representative
sample, it is framed in a frequency distribution table. This Ans.
type of distribution is called the normal distribution or the Mode is the value that occurs with the greatest frequency. A
Gaussian distribution. distribution may have more than one mode.
Characteristics of a normal curve For example, diastolic blood pressure of 10 individuals:
l It is a symmetrical bell-shaped curve. 85, 75, 81, 79, 71, 60, 75, 78, 72 and 73.
l Total area of the curve is 1. Its mean is zero, and stan- Here, Mode 5 75
dard deviation is 1. Advantages
l The parameters of mean, median and mode coincide in l It eliminates extreme variation.
the centre. l It is easily located by mean inspection.
l The maximum observations are in the centre and gradu- l It is easy to understand.
ally decrease towards extremities on either sides. Disadvantages
l Its exact location is uncertain.
Q.6. Standard deviation.
l It is not exactly defined.
Ans.
Q.11. Network sampling.
[Ref LE Q.3]
Ans.
Q.7. Multistage sampling.
[Ref LE Q.2]
Ans.
Q.12. Frequency polygon.
l In multistage sampling, the first stage is to select groups Ans.
or clusters.
l Then subsamples are taken in as many subsequent [Ref LE Q.1]
stages as necessary to obtain the desired sample size. Q.13. Measures of central tendencies.
For example:
Ans.
First stage: choice of states within countries.
Second stage: choice of towns within each state. Mean, median and mode are called measures of central
Third stage: choice of neighbourhood within each tendencies.
town. Mean
This measure implies the arithmetic average or arithmetic
Q.8. Pilot survey.
mean. It is obtained by summing up all the observations
Ans. and dividing the total by the number of observations.
Median
l Pilot survey includes the most important subgroups of
Here the data are arranged in an ascending or descend-
a population and only one or two index ages, usually
ing order of magnitude and the value of middle observa-
12 years and one other age group.
tion is located.
l Such a survey provides the minimum amount of data
Advantages
needed to commence planning.
l It is more representative than mean.
l Additional data should then be collected to provide a
l It does not depend on every observation.
reliable baseline for the implementation and monitoring
l It is not affected by extreme values.
of services.
Mode
Q.9. Systematic random sampling. Mode is the value that occurs with the greatest fre-
quency. A distribution may have more than one mode.
Ans.
Q.14. Evaluation.
l A systemic sample is obtained by selecting one unit at
random and then selecting additional units at evenly Ans.
spaced intervals till the sample of required size has been
[Ref LE Q.6]
obtained.
l This method can be adopted as long as there is no peri- Q.15. Pie chart.
odicity of occurrence of any particular event in the
Ans.
population.
l The population can be homogenous or heterogenous. [Ref LE Q.1]
Section | I  Topic-Wise Solved Questions of Previous Years 723

Q.16. Name criteria for evaluation. l The successive groups of data are linked in a definite
numerical order.
Ans.
l It is a pictorial diagram of frequency distribution.

Criteria for evaluation are as follows: For example, age-wise prevalence of dental caries.
l Effectiveness

l Efficiency Q.22. Method of collection of data.


l Appropriateness Ans.
l Adequacy
[Ref LE Q.1]
Q.17. Bar chart.
Q.23. Questionnaire survey.
Ans.
Ans.
[Ref LE Q.1]
l Questionnaire survey is a standard method of data col-
Q.18. Survey. lection in clinical and epidemiological, psychological
and demographic research.
Ans.
l It is used for measuring subjective phenomenon.

[Ref LE Q.5]
Types
Q.19. Primary source data. l Mailed questionnaires.

l Telephone interviews: It is easy to conduct in urban


Ans.
areas.
Primary source data are the data obtained directly from an l Face-to-face interviews: It is used in community sur-
individual. veys and clinical research.
Advantages
Q.24. Impact evaluation.
l Precise information

l Reliable Ans.
Disadvantages
Impact is an expression of overall effect of a programme.
l Time-consuming
For example, a dental health education programme for a rural
l Expensive
population, wherein the use of toothbrush and paste is being
Q.20. Planning. stressed. Immediately after the programme, they will not buy
brush and paste. On the second visit, probably many of them
Ans.
will be using them. Hence, it shows what impact you have,
[Ref LE Q.4] and how effective was your impact on them.
Q.21. Histogram. Q.25. Importance of pilot survey.
Ans. Ans.
l A histogram is a special sort of bar chart. [Same as SN Q.8]

Topic 9
Social Sciences
COMMONLY ASKED QUESTIONS
LONG ESSAYS:
1. What do you understand by social sciences? Write a note on social scientist. Name the various forms of hard and
soft tissue mutilation practices, and write a note on soft tissue mutilation.
2. Classify types of child behaviour and discuss the various behaviour management techniques.
3. Define habit. Classify the habits. Discuss the role of habits in initiation and progress of dental and oral diseases.
4. Describe psychological development of a child from birth through adolescence.
5. Describe the methods for the management of child in dental office. [Same as LE Q.2]
724 Quick Review Series for BDS 4th Year, Vol 1

6 . Describe the various methods available for the management of child in dental office. [Same as LE Q.2]
7. Define and classify oral habits. Discuss the following: [Same as LE Q.3]
a. Thumbsucking
b. Mouth breathing

SHORT ESSAYS:
1. Social stratification.
2. Freud’s development stages. [Ref LE Q.4]
3. Occupational hazards for dentist.
4. Superego. [Ref LE Q.4]
5. Operant conditioning.
6. Management of mentally challenged patient.
7. Taboos in dentistry.
8. Fear.
9. HOME. [Ref LE Q.2]
10. Modelling. [Ref LE Q.2]
11. Prevention of AIDS in dental practice.
12. Wright’s classification of behaviour of children in dental office.
13. Physical restrainer. [Ref LE Q.2]
14. TSD. [Ref LE Q.2]

SHORT NOTES:
1. Oedipus complex.
2. Write a brief note on sociology.
3. Define modelling. [Ref LE Q.2]
4. Reinforcement. [Ref LE Q.2]
5. Taboos related to oral health.
6. Autoclave.
7. Tell-show-do. [Ref LE Q.2]
8. Nuclear family. 9. Enumerate basic behaviour management techniques. [Ref LE Q.2]
10. Define behaviour management. [Ref LE Q.2]
11. Oral manifestations of AIDS.
12. Tongue thrusting.
13. Taboos related to dentistry in India. [Same as SN Q.5]
14. Culture and oral health. [Same as SN Q.5]

SOLVED ANSWERS
LONG ESSAYS:
Q.1. What do you understand by social sciences? Write l Economics
a note on the social scientist. Name the various forms of l Political science
hard and soft tissue mutilation practices and write a Social scientist
note on soft tissue mutilation. Social scientists are the ones who are called upon to aid
in adapting new health programmes to existing cultural
Ans.
patterns.
Social science is defined as a discipline committed to the Their functions are as follows:
scientific examination of human behaviour. l Programme planning and evaluation
Social sciences includes: l Public health experiments
l Sociology l Estimation of indigence
l Social anthropology Various forms of hard and soft tissue mutilations
l Social psychology l Nontherapeutic tooth extraction (avulsion)
Section | I  Topic-Wise Solved Questions of Previous Years 725

l The breaking off of tooth crowns ii. Temporary piercing of orofacial soft tissues for
l Alteration in the shape of tooth crowns by filing and ceremonial purposes.
chipping iii. Uvulectomy.
l Dental inlay work iv. Facial scarring.
l Lacquering and staining of teeth l The best-known example of temporary mutila-

l Practices such as the placement of gold crowns for tion of soft and perioral soft tissues is the one
adornment purposes practised by Hindu males in India.
l Tattooing of gingival l During the ceremony of Thapasyam (Penace-

l Piercing of lips and perioral soft tissues white; Thaipusam), men, in a state of apparently
l Piercing of orofacial soft tissues self-induced trance, pierce the skin with a variety
l Uvulectomy of sharp weight-bearing hoops and lances.
l Facial scarring l The person shows no signs of pain and the

Some of the reasons for tooth mutilations are listed wounds do not bleed.
below: l Facial soft tissues are often subjected to scarification.

l Tribal identification l Scarification may be carried out for a variety of

l Initiation rite reasons, including tribal identity aesthetics, to en-


l Sign of manhood or bravery hance sexual appeal, to indicate status and to sig-
l Differentiation of sexes nify events such as puberty, marriage or childbirth.
l Sign of marriageable age in females
Q.2. Classify types of child behaviour and discuss the
l Sign of ceremonial rebirth
various behaviour management techniques.
l To ensure a life after death

l Aesthetics and fashion Ans.


l Therapeutic purposes l Behaviour in paedodontics is defined as a discipline that fo-
l Sign of mourning cuses on the psychological, social and learning problems of
l Sign of subjugation children and adolescents, as they relate to dental situations.
l Form of punishment l Behaviour is defined as any change in the functioning of
l Cultural mimicry an organism.
l To enable an individual to spit properly

l Local superstition associated with phenomena


Various classifications of child behaviour
such as rain I. Frankl’s classification (1962)
Mutilations of soft tissues II. Lampshire’s classification
Tattooing III. Kopel’s classification
l The gingiva may be tattooed when females reach
IV. Wright’s classification
puberty, become betrothed or get married. V. Modified Wright’s classification (addition of symbolic
l It is practised by men to relieve the pain associ-
modifications to the Frankl’s rating scale)
ated with ‘diseased gums’. I. Frankl’s behaviour rating scale (1962)
l The technique of gingival tattooing involves l Rating No. 1 – Definitely negative
painting the gingiva with a layer of pigmented l Refuses treatment
material, usually carbon, which is then impreg- l Immature, uncontrollable
nated into gingival mucosa by means of sharp l Defiant behaviour
thorns or needles that pierce the mucosa. l Crying forcefully
l A blue-black colouration is the usual hue achieved l Rating No. 2 – Negative
with gingival tattoos. l Reluctance to accept treatment
l The material used to tattoo the gingiva may be ob- l Immature, timid and whining
tained from calcified peanuts, burned wood or lamp- l Some evidence of negative attitude but not pro-
black (the soot obtained from burning oil lamp). nounced.
l A tattooed lower lip in a Sudanese woman signi- l Rating No. 3 – Positive
fies that the woman is married. l Accepts treatment
l Facial tattoos may incorporate a triangular-shaped l Tense cooperative
tattoo on the skin surface at the angle of mouth. l Willingness to comply with dentist
l It has its basis in ritual of warding off the ‘evil eye’. l Rating No. 4 – Definitely positive
Other mutilation practices l Good rapport with the dentist
i. Piercing of lips and perioral soft tissues, and the l Understanding and interested in dental procedures
insertion of materials such as wood, ivory or metal. l Laughs and enjoys the situation
726 Quick Review Series for BDS 4th Year, Vol 1

II. Lampshire’s classification I. Basic behaviour management


i. Cooperative 1. Communicative management
Physically and emotionally relaxed. a. Voice control
ii. Tense cooperative b. Nonverbal communication
Tensed and cooperative. c. Tell-show-do
iii. Outwardly apprehensive d. Positive reinforcement
Avoids treatment initially. e. Distraction
Hides behind mother and avoids looking or 2. Parental presence or absence
talking to dentist but eventually accepts the 3. Nitrous oxide/oxygen inhalation sedation
treatment. II. Advanced behaviour management
iv. Fearful 1. Hand-over-mouth exercise
Requires considerable support so as to overcome 2. Medical immobilization
the fears of dental treatment. 3. Sedation
v. Stubborn or defiant 4. General anaesthesia
Passively resists treatment by using techniques. In general, behaviour management methods are classi-
vi. Hypermotive fied as follows:
Child is acutely agitated and resorts to screaming A. Nonpharmacological methods
and kicking. B. Pharmacological methods
vii. Handicapped
A. Nonpharmacological methods
Physically or mentally.
a. Preappointment behaviour modifications
viii. Emotionally immature
b. Communication
Emotionally handicapped.
c. Behaviour-shaping techniques
III. Kopel’s classification
i. Tell-show-do
l Very young patient
ii. Desensitization
l Emotionally disturbed patient
iii. Modelling
l Child from a broken or poor family
iv. Contingency
l Pampered or spoilt child

l Neurotic child

l Excessively fearful child


{SN Q.9}
l Hyperactive child . Behavioural management techniques
d
l Physically handicapped child i. Audio analgesia
l Child with previous untouched medical or dental ii. Biofeedback
experience iii. Voice control
IV. Wright’s modification of Frankl’s behaviour iv. Hypnodontics
l Rating No. 1: definitely negative (– –) v. Coping
l Rating No. 2: negative (–) vi. Relaxation
l Rating No. 3: positive (1) vii. Aversive conditioning
l Rating No. 4: definitely positive (11) viii. Implosion therapy
ix. Retraining
{SN Q.10}
B. Pharmacological methods
Behaviour Management I. Premedication
l Behaviour management: It is defined as the means by a. Sedatives and hypnotics
which the dental health team effectively and effi- b. Antianxiety drugs
ciently performs treatment for a child, and at the c. Antihistaminics
same time installs a positive dental attitude. II. Conscious sedation
III. General anaesthesia
l Behaviour shaping: It is a procedure that very . Nonpharmacological methods
A
slowly develops behaviour by reinforcing succes- The nonpharmacological methods of behaviour man-
sive approximations of the desired behaviour until agement are explained in detail below.
the desired behaviour comes to be. It is sometimes a. Preappointment behaviour modification
called ‘stimulus response theory’. Preappointment behaviour modification in-
According to AAPD guidelines, 2002–2003: cludes everything that is to be done to influence
Section | I  Topic-Wise Solved Questions of Previous Years 727

positively the child’s behaviour before the child (SE Q.14 and SN Q.7)
enters the dental clinic.
Various methods used for this purpose are as follows:
l {(The TSD method of introducing dental tech-
niques is extremely effective for shaping children’s
i. Films or videotapes showing a model of a
behaviour and conditioning them to accept treat-
cooperative child patient.
ment.
ii. It can also be performed on live models,
Objectives of TSD technique are as follows:
such as other children or parents.
l Teach the patient about important aspects of dental
b. Communication
visit.
l Effective communication, i.e. imparting or in-
l Familiarization of dental settings.
terchange of thoughts, opinions or information
l Shape the patient’s response to procedures through
is an important factor in dealing with children.
desensitization and well-described expectations.
l The first objective of successful management
l TSD is a series of successive approximations; the
of a young dental patient is to establish com-
steps followed in TSD are as follows:
munication.
A. Tell children about the treatment to be carried out.
l Based on the age of children, there are many
B. Show them a part of it, and how it will be done.
ways of initiating a verbal communication, but
C. Then do it.)}
in dental setups, it is affected primarily through
dialogue, tone of voice, facial expression and
body language. [SE Q.14]
l Involving children in conversation will relax l {TSD technique includes verbal explanations at the
them and enable the dentists to learn about the patient’s understanding level (tell) appropriately us-
patients. ing second language or word substitutes wherever
l Verbal communication with young children is best necessary.
initiated with complementary comments about l Demonstration of visual, auditory, olfactory and tac-
their dress or interests followed by some questions tile aspects of the procedure in a nonthreatening
that elicit an answer other than yes or no. fashion (show).
l For the successful communication to take l Completion of the procedure without deviating from
place, all the four elements of communication, what was explained and demonstrated (do).
i.e. sender, message, context and receiver must l While working intra-orally, children should be shown
be present and consistent. as much of the procedure as possible. One should be
c. Behaviour-shaping techniques truthful with children and yet should not frighten
l Behaviour shaping is the procedure by which them.
the desired behaviour is instilled and incul- l Any deviation from that of originally explained or
cated in children. demonstrated procedure can affect the relation
l Behaviour shaping is based on a planned intro- between children and dentist. So, honesty is re-
duction of treatment procedures so that children quired to achieve positive attitude of children in
are gradually trained to accept treatment in a future.
relaxed and cooperative manner. Indications: All patients who can communicate regard-
Various behaviour-shaping techniques are as follows: less of the level or the method of communication.}
i. Tell-show-do ii. Desensitization
ii. Desensitization Desensitization technique involves three stages:
iii. Modelling a. Training the patients to relax.
iv. Contingency b. Constructing a hierarchy of fear-producing
stimuli.
[SE Q.14]
c. Introducing each stimulus in hierarchy to relaxed
i. {Tell-show-do technique patient, starting with the stimulus that causes the
l Almost five decades ago, Addelston (1959) for- least fear.
malized a technique that encompasses several For example, if the child is afraid of dental clinic, the
concepts from the social learning theory. It was general desensitization should include gradual introduc-
called Tell-show-do (TSD) technique. tion of child to the following:
l The TSD is a behaviour-shaping technique. It l Reception, waiting room and receptionist

gives good results and is recommended to be fol- l Paedodontist, hygienist and nurse

lowed in routine practice.} l Dental surgery


728 Quick Review Series for BDS 4th Year, Vol 1

l Dental chair
{SN Q.4}
l Oral examination
l Prophylaxis a. Reinforcement
At each stage, the child’s fears are allayed by the kind, l It is a method of increasing the probability (fre-

friendly and reassuring manner of receptionist, nurse, quency) of a desired behaviour by presentation of
hygienist and paedodontist to reinforce positive ap- a pleasant stimulus or withdrawal of an aversive or
proach in child. unpleasant stimulus. Here the stimulus is termed
as reinforcer.
(SE Q.10 and SN Q.3) l Positive reinforcers are presented. For example, a

pat on the back or shoulder, shaking hand or ver-


iii.  {(Modelling (imitation) bal praise in the presence of parent for which the
l Children are capable of acquiring almost any behav-
child will be happy. In the form of gifts like tooth-
iour that they observe closely and that is not too brush kits, drawing kits, favourite cartoon stickers
complex for them to perform at their level of physi- or toys appropriate for their age.
cal development. l Negative reinforcers are withdrawn. For example,
l Modelling is learning by imitation based on the
withdrawal of hand piece if the child is afraid of
observational learning theory of Albert Bandura noise of that and usage of hand instruments so
(1969). that the child will accept dental treatment in the
l Goal of modelling is to make patient reproduce the
next appointment.
behaviour exhibited by models.
Stages and requirements of modelling
l Two stages of modelling are acquisition and perfor- b. Omission or time-out
mance, and the four requirements are attention, re- l It is the means of increasing the probability

tention, motoric reproduction and reinforcement (frequency) of a desired behaviour by with-


and motivation.)} drawal of or threatening to withdraw a
pleasant stimulus. For example, warning or
[SE Q.10] threatening child that mother will be sent
outside the operatory if the child is not co-
{Advantages of modelling procedures are as follows: operating for the procedure.
l Stimulation of good behaviour.
c. Punishment
l Facilitation of behaviour in more appropriate man-
l It is the means of increasing the frequency
ner.
of a desired behaviour by the presentation
l Extinction of fears and apprehensions.
of an aversive stimulus. For example, use of
l Modelling is effective particularly in the following
voice control, protective stabilization or
conditions:
hand over mouth.
l When the patient is in a state of arousal.
d. Behavioural management techniques
l When model’s behaviour has positive conse-
i. Audioanalgesia
quences.
ii. Biofeedback
l When modelling is performed on models having
iii. Voice control
higher status and prestige.}
iv. Coping
iv. Contingency management
v. Relaxation
l This technique is based on the operant conditioning
vi. Aversive conditioning
theory of B.F. Skinner.
vii. Implosion therapy
l It is the method of modifying behaviour by presen-
viii. Retraining
tation or withdrawal of reinforcers. These reinforc-
ers are the pleasant or unpleasant stimuli mentioned i. Audioanalgesia
in the Operant Conditioning Theory of child psy- l It is also called ‘white noise’. This

chology. consists of providing a sound stimulus


Contingency management includes the following: intensity so that the patient finds it dif-
a. Reinforcement: It can be either positive reinforce- ficult to attend to anything else. The
ment or negative reinforcement. effect is due to distraction, displace-
b. Omission/time-out. ment of attention and a positive feeling
c. Punishment. on the part of dentist that it can help.
Section | I  Topic-Wise Solved Questions of Previous Years 729

ii. Biofeedback l Verbalizing fears to others is another well-


l In this method, certain physiologic reactions known way of coping. Expressive communica-
of the body are detected that may indicate tion serves to release tension.
fear, e.g. checking heart rate that may in- l Another best way is facing the threat of stress or

crease when the patient is under stress. pain enjoying affiliative behaviour.
iii. Voice control v. Relaxation
l Voice control is a controlled alteration of Jacobson and others used specific relaxation tech-
volume, pace or tone of voice to control nique.
child’s disruptive behaviour. l Relaxation usually involves a series of basic

l Voice control is done to establish authority, exercise that may take several months to learn,
gain child’s attention and compliance and and requires the patient to practice at home for
avert negative behaviour. at least 15 min each day.
l Voice control is most effective when used l Telling oneself to relax is another mechanism

in conjunction with other communica- of personal coping. This technique apparently


tion. A sudden command ‘to stop crying works by reducing tension, a well-known po-
and pay attention’ may be a necessary tentiator of pain.
preliminary measure for the future com-
munication. [SE Q.9]
l Sudden and firm commands can be used to

get child’s attention or stop from whatever is
{vi. Aversive conditioning
Aversive conditioning includes the following two
being done.
techniques:
l This is an abrupt and emphatic change in
a. Hand-over-mouth exercise (HOME)
dentists’ tone of voice to emphasize their
b. Physical restraints
displeasure with child’s inattention.
l As soon as the child complies, we should a. Hand-over-mouth exercise (HOME)
thank and compliment him/her for the resul- l This technique was first described in the

tant excellent behaviour. 1920s by Dr Evangeline Jordan.


l If used properly in correct situations, voice l Levitas described hand-over-mouth tech-

control is an effective management tool. nique (HOMT) as hand-over-mouth exercise


Objectives (HOME).
l To gain patient’s attention and compli- l A hand is placed over the child’s mouth and

ance behavioural expectations are calmly ex-


l To avert negative or avoidance behaviour plained. The child is told that the hand will
l To establish authority be removed as soon as appropriate behav-
Indications: Voice control is indicated for iour begins. When the child responds, the
uncooperative, inattentive and communica- hand is removed and the child’s appropriate
tive child. behaviour is reinforced. The method may
Contraindications: In children who due to require reapplication.
age disability, medication or emotional im- l It was called emotional surprise therapy by

maturity are unable to understand and coop- Lampshire, and aversive conditioning by
erate. Kramer.
iv. Coping Objectives of HOME therapy are as follows:
l Patients differ not only in their perception and in l To gain the child’s attention, enabling

response to pain but also in their ways of dealing communication with the dentist so that
or coping with the stress associated with painful appropriate behavioural expectations can
experiences. be explained.
l The same can be used to modify child’s l To ensure the child’s safety in the deliv-

behaviour in dental clinic. ery of quality dental treatment.


Different coping mechanisms are as Indications of HOME
follows: l Used for normal children who are

l Distraction or displacement of attention momentarily hysterical, belligerent or


away from threat. defiant.
730 Quick Review Series for BDS 4th Year, Vol 1

l to
Used for children with sufficient maturity viii. Retraining
understand simple verbal commands. l It is required in children displaying consider-

Contraindication of HOME able apprehension or negative or uncooperative


l Immature, frightened or the child with a behaviour, which may be due to a previous den-
serious physical, mental or emotional tal visit.
handicap.} l The objective of retraining is to build a new

series of images and associations in child’s


[SE Q.13] mind.
{b. Physical restraints l If child’s expectancy of getting hurt does not

Protective stabilization or restraining is


l come true, a new series of expectancies are
defined as the restriction on patient’s free- learned so that paedodontists and their auxilia-
dom of movement, with or without the ries could be trusted.
patient’s permission, to decrease risk of l Child develops a new perception of dental clinic

injury while allowing safe completion of and a new relationship to dentistry.


treatment.
l Parental or guardian consent must be ob- Q.3. Define habit. Classify the habits. Discuss the role
tained prior to use of restraints. of habits in initiation and progress of dental and oral
Objectives of using restraints are as follows: diseases.
l To reduce or eliminate untoward move-

ment Ans.
l To protect the patient and dental staff l According to Dorland (1957), habit can be defined as a
from injury fixed or constant practice established by frequent repetition.
l To facilitate delivery of quality dental l Buttersworth (1961) defined a habit as a frequent or
treatment constant practice or acquired tendency, which has been
Indications fixed by frequent repetition.
l A patient who requires immediate diag- l According to Mathewson (1982), oral habits are learned
nosis or treatment and cannot cooperate patterns of muscular contractions.
due to lack of maturity or mental or Classification
physical disability. Based on causative factors
The following are some commonly used l Obsessive (deep rooted)
physical restraints: l Nonobsessive (easily learned and dropped)
l Oral: Mouth props, padded wrapped l Obsessive (deep rooted)
tongue blades, a finger guard or an intra- a. Intentional or meaningful, e.g. nail biting, digit
occlusal thimble and rubber/plastic bite sucking and lip biting
blocks b. Masochistic or self-inflicting injurious habit,
l Body: Papoose board, triangular sheet, e.g. gingival stripping
pediwrap, beanbag dental chair insert, l Nonobsessive (easily learned and dropped) –
safety belt, extra assistant unintentional or empty, e.g. abnormal pillowing
l Extremities: Posey strap, Velcro straps, and chin propping
towel/tape, extra assistant l Functional habits, e.g. mouth breathing and
l Head: Forearm support, head positioner, tongue thrusting or bruxism
plastic bowl and extra assistant Classification of habits
Physical restraints should never be used for I. According to James
punishment. An explanation of their benefits James has classified habits into the following types:
should be presented if communication is pos- a. Useful habits
sible with child and parents.} The habits that are essential for normal functions.
vii. Implosion therapy For example:
In this technique, the patients are flooded with many l Proper positioning of tongue
stimuli that have affected them adversely. l Respiration
l The child has no other choice but to face it until l Normal deglutition
negative behaviour disappears. b. Harmful habits
l It comprises HOME technique, voice control and The habits that have deleterious effect on teeth
physical restraints together. and their supporting structures.
Section | I  Topic-Wise Solved Questions of Previous Years 731

For example: Sucking habits can also be classified as follows:


l Thumbsucking a. Nutritive sucking habits
l Tongue thrusting For example, breast/bottle feeding
II. According to Morris and Bohanna b. Nonnutritive sucking habits
Morris and Bohanna have classified habits into the For example, thumb/finger sucking and pacifier sucking
following types: Theories associated with thumbsucking
l Pressure habits i. Classical Freudian theory (1919)
i. Sucking habits, e.g. thumbsucking, lip suck- ii. Sucking reflex (Ergel, 1962)
ing, finger sucking and tongue thrusting iii. Rooting reflex (Benjamin, 1962)
ii. Biting habits, e.g. nail biting, pencil biting iv. Learning theory (Davidson, 1967)
and lip biting v. Oral drive theory (Sears and Wise, 1982)
l Nonpressure habits vi. Johnson and Lasson (1993) – combination of psy-
Habits that do not apply direct force on teeth or choanalytic and learning theories
its supporting structures, e.g. mouth breathing. Aetiology of sucking habits
l Postural habits, e.g. chin rest l Sucking urge – unrestricted breast feeding, bottle or

l Miscellaneous habits, e.g. bruxism cup feeding


III. According to Klein l Surplus sucking urge – nonnutritive sucking, e.g.

Klein has classified habits into the following types: thumb or dummy sucking for satisfaction
l Empty habits – habits not associated with any Causative factors
deep-rooted psychological problems, e.g. abnor- Various causative factors are as follows:
mal pillowing and chin propping. i. Parents’ occupation
l Meaningful habits – habits that have a psycho- Low socioeconomic groups are more prone to thumb-
logical bearing, e.g. nail biting, lip biting and digit sucking, as the mother is unable to provide sufficient
sucking breast milk to infants; hence in the process the infant
IV. According to Finn suckles intensively for a long time, thereby exhaust-
Finn has classified habits into the following types: ing the sucking urge. Whereas in high socioeconomic
l Compulsive habits – These are deep-rooted habits status, the mother is in a better position to feed child
that have acquired a fixation in children to the and in a short time the baby’s hunger is satisfied.
extent that they indulge in that habit whenever ii. Working mother
their security is threatened by events that occur Child cultivates thumb-sucking habit due to insecure
around them. feeling.
Children tend to suffer increased anxiety when an iii. Number of siblings
attempt is made to correct the habits. More number of children, so neglect of child leads
l Noncompulsive habits – These include habits to development of this habit.
that are easily learned and dropped as chil- iv. Order of birth of child
dren mature. Later the sibling rank of a child, greater chances of
Thumbsucking habit oral habit.
Thumbsucking can be defined as placing the thumb in v. Social adjustment and stress
various depths of mouth. Digit sucking develops as an emotional-based
Classification behaviour.
A. Normal thumbsucking vi. Age of the child
During the first and second year of life, thumbsucking Age at which the digit sucking habit appears has a
is considered normal; it usually disappears as child significance.
matures and does not generate any malocclusion. Clinical effects of thumbsucking
B. Abnormal thumbsucking l Dentofacial changes associated with thumbsucking

Habit persists beyond preschool period and has del- can affect maxilla, mandible, interarch relationship,
eterious effects on dentofacial structures. lip placement and function, etc.
C. Psychological l The severity of malocclusion caused by thumbsuck-

Habit may have a deep-rooted emotional factor in- ing depends on the following factors:
volved, such as insecurity, neglect or loneliness of a. Duration: Amount of time spent indulging in the habit.
child. b. Frequency: The number of times the habit is acti-
D. Habitual vated/day.
No psychological bearing just as habit. c. Intensity: Vigour with which the habit is performed.
732 Quick Review Series for BDS 4th Year, Vol 1

Effects of thumbsucking on dental arch and its sup-


l Two types of habit breakers are as follows:
porting structures are as follows: a. Removable – These are passive and remov-
i. Proclination or labial tipping of maxillary ante- able appliances consisting of a crib placed
rior teeth palatal to 21,12 and anchored to oral cavity
ii. Increasing overjet – due to proclination of max- by means of clasps on posterior teeth, e.g.
illary anterior and lingual tipping of mandibular tongue spikes and tongue guards.
incisors b. Fixed – These appliances consist of molar
iii. Anterior open bite (goverbite) – restriction of in- bands/crowns on first permanent molars
cisor eruption and supraeruption of buccal teeth with palatal assembly and soldered spurs
iv. Posterior crossbite – narrow maxillary arch pre- made of either nickel chrome or stainless
disposing to crossbite due to contraction of steel.
cheek muscles during thumbsucking Other mechanical aids used to intercept the habit
v. Tongue thrust – develops as a result of open bite include the following:
vi. Effects on lips – hypotonic upper lip, hyperactive l Bandaging of the thumb
lower lip, h lip incompetence and hyperactive l Bandaging of the elbow
mentalis activity Fixed intraoral anti-thumbsucking appliance
vii. Other effects – psychological health, risk of mal- l A lingual arch forms the base of the appli-
position of jaws, speech defects and digit defects ance to which are added interlocking
Diagnosis wires; the anterior position in area of an-
i. History – frequency and duration of habit terior part of hard palate disturbs contact
ii. Child’s emotional status – assessed by enquiring: of thumb to palate.
l Feeding habits Bluegrass appliance by Haskell (1991)
l Parental care of child l It consists of modified six-sided roller ma-
l Working parents chined from teflon to prevent thumb place-
iii. Examination of child’s fingers – presence of clean ment. This is slipped over 0.045 SS wire
nails and callus on finger soldered to molar orthodontic bands of
iv. Intraoral clinical examination – proclination, open 3–6-month duration. Instruct the patient to
bite, etc. turn roller instead of sucking digit. Digit
Treatment sucking is often seen to stop immediately.
The child, parent and dentist form a team to assist child Quad helix: Prevents thumb from being in-
in stopping this habit. serted and also corrects malocclusion by ex-
Treatment is mainly of three categories: panding the arch.
A. Psychological therapy C. Chemical approach
B. Reminder therapy or mechanotherapy – remov- l Placing bitter-tasting or foul-smelling prepa-
able habit breakers and fixed habit breakers rations on thumb so that if sucked can make
C. Chemical approach the habit distasteful.
A. Psychological treatment l Commonly used medicaments are as follows:
a. Screening patients for underlying psycho- a. Pepper dissolved in a volatile medium
logical disturbances and referring to profes- b. Quinine
sionals for counselling. c. Asafoetida
b. Children aged between 4 and 8 years need Current strategies
only reassurance, positive reinforcement and Current strategies in treating thumb-sucking habit are as
friendly reminders to divert child’s attention follows:
to other things such as play and toys. l Increasing the arm length of night suit
c. Dunlop’s beta-hypothesis: l Thumb home concept
l Forced purposeful repetition of a habit l Use of hand puppets
eventually associated with unpleasant re- l Thumbsucking books
actions, and the habit is abandoned. l My special shirt
l Dunlop’s beta-hypothesis is especially Mouth breathing habit
practiced in older children, i.e. aged 8 years Sassouni (1971) defined mouth breathing as habitual
and above. respiration through mouth instead of nose.
B. Reminder therapy or mechanotherapy l Mouth breathing is an altered way of breathing
This is basically reminding appliances that assist through mouth and is an adaptation to obstruction
to quit the habit. in nasal passages.
Section | I  Topic-Wise Solved Questions of Previous Years 733

l The obstruction may be temporary or recurrent. Nasal breathers – demonstrate good control of
While more often it is partial than complete, the alar muscles
airway resistance may be enough to force the iii. Mirror test (fog test): Two-surface mirror is
subject to breathe through mouth. placed on patient’s upper lip. If air condenses on
l Mouth breathing results in altered jaw and tongue upper side of mirror, the patient is a nasal
posture, which in turn alters orofacial equilib- breather, and if it does sooner on the opposite
rium, leading to malocclusion. side, then the patient is a mouth breather.
Classification of mouth breathing iv. Massier water holding test: Patient is asked to
According to Sim and Finn, mouth breathing can be hold mouth full of water for few minutes without
categorized into three types: swallowing. Mouth breathers cannot retain water
i. Obstructive – complete or partial obstruction of for more than 2–3 min.
nasal passage results in mouth breathing v. Zwemer butterfly test: Take a few fibres of cotton
ii. Habitual – due to deep-rooted habit that is un- and place them just below nasal openings. On
consciously performed exhalation, if cotton fibres flutter downwards, pa-
iii. Anatomic – patients with short upper lip that tient is a nasal breather, and if fibres flutter up-
does not permit complete mouth closure ward, patient is a mouth breather. Children should
Aetiology close their eyes before the cotton is held to their
A. Obstructive causes nostrils and mouths so that breathing will be en-
l Nasal polyps tirely natural and not forced if they are deliber-
l Obstructive adenoids ately instructed to breathe through their nose.
l Congenital enlargement of nasal turbinates vi. Rhinometry (inductive plethysmography): The
l Chronic inflammation of nasal mucosa total airflow through nose and mouth can be
l Benign tumours quantified using inductive plethysmography.
l Deviated nasal septum vii. Cephalometrics: Can be used to calculate amount
B. Anatomic causes of nasopharyngeal space.
l Short upper lip Treatment
l Underdeveloped nasal cavity The major aspect of treating mouth-breathing pa-
C. Obstructive sleep apnoea tients is to treat and eliminate underlying pathology
Clinical features that has created the habit.
i. The type of malocclusion associated with mouth l Removal of nasal or pharyngeal obstruction by

breathing is called ‘long face syndrome’ or classical ENT surgeon.


adenoid facies or vertical maxillary excess. l Interception of habit – vestibular screen adhe-

ii. Dolichofacial skeletal pattern. Long and narrow sive tapes for lip seal.
face with short and flaccid upper lip. l Rapid maxillary expansion – causes widening

iii. Expressionless and blank face. of arch, resulting in h nasal flow and g nasal air
iv. Anterior open bite. resistance.
v. Contraction of upper arch, narrow V-shaped upper l Symptomatic treatment like deep breathing

jaw with a high narrow palate and posterior crossbite. exercises and lip exercises.
vi. hOverjet due to labial flaring of maxillary anteriors.
Q.4. Describe psychological development of child from
vii. Anterior marginal gingivitis and gingival hypertrophy
birth through adolescence?
are seen in mouth breathers and hcaries incidence.
viii. Narrow nose and nasal passage, widely flared ex- Ans.
ternal nares.
Psychology is the science dealing with human nature, func-
ix. Excessive appearance of maxillary anterior teeth
tion and phenomenon of their soul in the main.
with a ‘gummy smile’.
Child psychology is the science that deals with mental
x. Nasal tone in voice.
power or an interaction between conscious and subcon-
Examination of a child for mouth breathing habit
scious elements in a child.
Diagnosis
Sigmund Freud gave two theories:
i. Observe the patient
A. Psychoanalytical theory
l Mouth breathers – Lips will be apart.
B. Psychosexual theory
l Nasal breathers – Lips will be touching.

ii. Ask the patients to take a deep breath through noses A. Psychoanalytical theory
Mouth breathers – no change in shape or size of l This theory helps in understanding intrapsychic pro-

external nares cess and personality development.


734 Quick Review Series for BDS 4th Year, Vol 1

This theory was proposed by Sigmund Freud (1905).


l iii. In infants, oral cavity is the site for identifying
He compared human mind to an iceberg. needs. Children will put their thumb or anything
Elements of psychic triad else that they can reach to in their mouths.
i. According to Freud, personality is composed of iv. This is a dependent stage, since the infant is
three parts and is known as psychic triad – Id, ego dependent on parents for their oral needs.
and superego. v. If the child does not receive sufficient gratifica-
ii. Each system has its own functions but the three tion of pleasure at this age, fixation to this stage
intact are required to govern the behaviour. occurs as the individual grows older. For exam-
Id ple, oral dependency in the form of digit sucking
l It is governed by ‘the pleasure principle’. in older individuals.
l It is present at birth, is impulse-ridden and vi. If the child’s needs are not adequately met in this
strives for immediate pleasure and gratifica- age, the following traits may develop: pessimism,
tion without regard to rules, realities of life demanding, frustration and jealousy.
or morals of any kind. vii. Excessive oral gratification leads to excessive
Ego optimism or narcissism.
l Ego is governed by ‘the reality principle’. b. Anal stage
l Ego tries to satisfy the Id’s urge of pleasure i. This stage is also known as ‘terrible twos’ that
but only in the realistic ways that take ac- occurs between the age of 1 and 3 years and is
count of what is possible in the real world. marked by the egocentric behaviour.
l It is the mediator between Id and Superego. ii. During this stage, the anal zone becomes the prin-
It channelizes Id into a socially acceptable cipal zone of pleasure. Gratification is derived
way. from expelling or with hoarding faeces.
[SE Q.4] iii. During this stage, neuromuscular control oc-
curs. Control over the sphincter results in in-
{Superego creased voluntary activity.
l It is governed by ‘the moral principle’. iv. This stage is characterized by development of
l It develops around the age of 5 years. It es- personal autonomy and independence. Children
tablishes and maintains the person’s moral realize the increased voluntary control that pro-
consciousness on the basis of a complex of vides them with the sense of independence and
ideas and values internalized by parents. autonomy.
l It stems from internalization of feeling good
v. Children realize their control over their needs
and bad, love and hate, praising and forbid- and practices with a sense of shame.
ding, rewards and punishment. vi. Overemphasis by adults on toilet training will
l It is linked to a social consciousness derived
result in the compulsive, obstinate and perfec-
in part from the familial cultural restrictions tionist behaviour in later life called ‘anal per-
placed on the growing child.} sonality’.
B. Psychosexual theory vii. Anal personality is characterized by abnormal be-
Freud believed that an individual progresses through haviour, like disorderliness, abstinence, stubborn-
several developmental stages that affect personality. ness, willfulness and frugality.
[SE Q.2] viii. Less controlled toilet training results in an im-
pulsive personality in later life. The transition
{According to Freud, development stages are classified between the anal and phallic stages is known as
into following: ‘urethral stage’. It is also characterized by com-
a. Oral stage (0–1 year)
petitiveness. Children derive pleasure from ex-
b. Anal stage (1–3 years)
ercising control over the urinary sphincter.
c. Phallic/Oedipal stage (3–7 years) c. Phallic stage
d. Latency period (7–12 years) i. Sex identification, which occurs between 3 and
e. Genital stage (12–18 years) 6 years of age, is an important feature of this stage.
a. Oral stage ii. During this stage, children explore and experience
i. The first year of a child’s life is called oral their genital organs as pleasurable.
stage and is the earliest stage of development Phallic stage is characterized by:
characterized by passiveness and dependency. Oedipus complex
ii. The oral cavity is the primary zone of pleasure Electra complex
because hunger is satisfied by oral stimulation. Oedipus complex
Section | I  Topic-Wise Solved Questions of Previous Years 735

l The child begins to direct their awakened ii. Children have a material personality. Sense of
sexual impulses towards the parents of op- identity develops and helps them to separate
posite sex. from the dependence of parents.
l Attachment between young boys and iii. They can satisfy genital potency and realize their
mother, and they consider father as their goals for reproduction and survival.
enemy. iv. Fluctuating extremities in emotional behaviour
l The name ‘Oedipal complex’ comes from and preoccupation with philosophical and ab-
Greek mythology. Oedipus, the king of stract thoughts predominate due to struggle to
Thebes, unwillingly slew his father and attain a firm sense of self.
married his mother. v. The individual becomes more matured and will
Little boys strive to imitate their father to gain be able to make right decision and prepare to
affection of their mother. Freud also described face the world.}
Oedipal complex in boys as a desire to have a
sexual relation with mother.
SHORT ESSAYS:
Electra complex Q.1. Social stratification.
l Young girls develop attraction towards their
Ans.
father and resent the mother being close to
the father. Social stratification
l The child realizes sexual qualities without People in a community are differentiated by different
embarrassment. characteristics, which they bear. Social scientists have
l In a phobic child, these unconscious and used occupation as a means of determining the levels of
unacceptable wishes and feelings associ- social standings of an individual in a community.
ated with Oedipal situation does not Occupational classification
enter consciousness. All occupations are classified in five groups:
l If the characteristic features of this stage are l Professional occupation
not resolved, the balance between male and l Intermediate occupation
female roles does not develop. l Non-manual skilled occupation
Clinical significance l Manual skilled occupation
l Keep the opposite sex during the child man- l Unskilled occupation
agement in the dental office/clinic. Other measures of social differentiation
l Keep mother in case of boy and father in case of l Education
baby girl during the child management. l Income
d. Latency stage l Purchasing power
i. This stage begins with resolution of Oedipus l Religion
complex around the age of 5–6 years and ends l Rural and urban
with the onset of puberty at 6–12 years, and is a Attitude to disease
period of consolidation of sex roles. The attitude of people towards health and disease varies
ii. Maturation of ego takes place and the superego in different social classes.
becomes firmly internalized. This stage is quiet as Upper middle class
compared to the stages before and after. l The members of this class include the professionals,
iii. There develops a greater degree of control over business and executive groups living in preferred ar-
instinctual impulses. eas and well-maintained homes.
iv. Child gains better sense of initiative and starts l They value their teeth, are interested in preventive
adapting to the adverse environment. dentistry and pursue various types of dental care.
v. The goal of this phase is the further development l The dentists are visualized as professionals who not
of personality. only repair teeth but also prevent decay and loss of
vi. Lack of inner control or excessive inner control teeth and make a person more attractive and beautiful.
results in an immature behaviour and decreased Lower middle class
development of skill. l Members include owners of small businesses, minor
e. Genital stage executives, teachers, salesmen and white-collar
i. It begins with puberty, extends to young adult- workers.
hood and is characterized by reopening of ego l The dentists are regarded as authorities who fix teeth
struggle to gain mastery and control over the and give directions as to how teeth should be taken
impulses of Id and superego. care of and who are useful for preventive dentistry.
736 Quick Review Series for BDS 4th Year, Vol 1

They are the most compulsive people in their dental


l (c) Noise
attitudes. Auditory effects consist of temporary or permanent
Upper lower class hearing loss. Nonauditory effects consist of ner-
l This includes skilled and semi-skilled blue-collar vousness, fatigue and interference with speech.
workers. (d) Vibration
l They are people of limited education and are law- Vibration affects hands and arms. After some months
abiding, respectable and hard-working citizens. or years of exposure, the fine blood vessels of the
l They feel that there is a little they can do to stance fingers may be increasingly sensitive to spasm.
the inevitable, including the loss of their teeth. (e) Ultraviolet radiation and computer lasers
l They receive artificial dentures at a relatively low age l Effects – Eyes are affected, causing intensive

and are happy with them. conjunctivitis and keratitis.


l They instruct their children how to take care of their l Radiation X-rays effects – The radiation effects

teeth but the children are more or less on their own are totally painless yet life threatening.
after that. (f) Sharps
l They acquire confidence in the reputation of clinic l Glasswares and sharp needles, lancets, BP blades,

because it was started by a well-known agency and broken ampoules and test tubes are hazardous.
in part, because they see their friends there. This can l Cuts, scratches and abrasions are potential loca-

be called clinic habit. tions of infections.


Lower class Chemical hazards
l It consists of unskilled labourers and people who l Dentists are exposed to various types of chemicals

skip from job to job, have limited education, live in that are hazardous while providing care. These in-
slum areas and exhibit no stable life pattern. clude mercury, beryllium, armlets, silica and latex.
l They reveal the most consistent neglect of teeth and l Mercury – Average daily intake of mercury from

require careful understanding whether they could amalgam restoration is 1.2–1.3 mg in subjects
receive adequate care in public health facilities. with seven to eight restorations. Effects – mercury
poisoning.
Q.2. Freud’s development stages.
l Methacrylates – Effects include irritation to skin, eyes

Ans. or mucous membrane, allergic dermatitis, asthma and


paraesthesia in fingers.
[Ref LE Q.4]
l Silica – Inhalation of dust containing free silica leads

Q.3. Occupational hazards for dentist. to silicosis.


l Beryllium – Some of the dental alloys contain beryl-
Ans.
lium, and when working on items such as dental
Occupational hazards for dentists crowns and bridges and partial denture framework
Dentists may be exposed to different types of hazards: and they develop chronic beryllium disease.
l Physical hazards l Latex gloves – Most of the professionals are allergic
l Chemical hazards to this product. Effects – urticaria.
l Biological hazards Biological hazards
l Mechanical hazards l Transmissible diseases that are of great concern to

l Psychosocial hazards the dentists nowadays are HBV, HIV, HCV, HSV and
Physical hazards Mycobacterium tuberculosis.
(a) Heat electrical hazards – Exposure may occur l Hepatitis B – Causative agent: Dane particle in virus;

when there is lack of maintenance to any electrical 0.00000001 mL of blood can transmit the disease.
equipment, lack of understanding of the equipment l Hepatitis C – Transmission is similar to HBV through

and its controls. parenteral routes identified, and is mostly associated


(b) Light with IV drugs or administration of blood products,
l The acute effects of poor illumination lead to less with sexual or vertical transmission.
eye strain, headache, eye pain, lachrymation l Tuberculosis – It has not been demonstrated that oral

and congestion around the cornea, and the health care procedures generate TB droplet nuclei;
chronic effects on health includes ‘miner’s therefore, the risk of transmission is probably quite low.
nystagmus’. Mechanical hazards
l Exposure to excessive brightness is associated l These are not alarming, and can be reduced or avoided

with discomfort, annoyance and visual fatigue. by being careful while operating. For example,
Section | I  Topic-Wise Solved Questions of Previous Years 737

traumatic injuries due to carelessness while using l Many dentists, for instance, negatively reinforce fear
sharp instruments, etc. behaviour.
Psychosocial hazards l When patients become nervous, a typical response is

l Stressful situations form an inherent part of a den- to stop and reassure them.
tist’s everyday work. l The main effect in this instance is that the fear is

l Establishment of good relation with patient forms an negatively reinforced because it is followed by the
important aspect of practice by which patient com- cessation of aversive procedure and is positively re-
pliance can be achieved. inforced by dentist’s soothing words.
l Dentists with their busy schedules will be deprived
Q.6. Management of mentally challenged patient.
of social interactions and spend less time with fam-
ily, leading to burnout syndrome. Ans.
Q.4. Superego. Management of mentally challenged patient
l A short attention span, restlessness, hyperactivity and
Ans.
erratic emotional behaviour may characterize patients
[Ref LE Q.4] with mental retardation undergoing dental care.
l Providing dental treatment to a person with mental re-
Q.5. Operant conditioning.
tardation requires adjusting to social, intellectual and
Ans. emotional delays.
Operant conditioning by B.F. Skinner: The following procedures have proved beneficial in estab-
l The main concept of this theory is that an individual lishing dentist–patient rapport and reducing the patient’s
learns to produce a voluntary response where the anxiety about dental care:
consequences or the outcomes are instrumental in l Give the family a brief tour of the office before attempt-

bringing about the reoccurrence of the stimulus. ing treatment and introduce the office staff to patient
l The individual’s response is changed as a result of and their family in order to reduce the patient’s fear of
previous responses. the unknown.
l Behaviours that operate or control the environment l Allow the patient to bring a favourite item, e.g. stuffed

are called ‘operants’. Relationship between the oper- animal or toy to hold for the visit.
ants or behaviours and the consequences that follow l Keep the parents inside the operatory.

them is called ‘contingency’. l Be repetitive, and speak slowly and in simple terms.

l Operant conditioning is classified into four contin- l Give only one instruction at a time.

gency arrangements: positive reinforcements, nega- l Actively and carefully, listen to the patient.

tive reinforcements, punishment and response cost or l Reward the patient with compliments after the success-

time out. ful completion of each procedure.


l Positive reinforcement: Occurs when behaviour, l Invite the parent into the operatory for assistance and to

good or bad, is followed by a rewarding event. aid in communication with the patient.
Many times, just praising patients for maintaining l Ask the parents not to communicate when the dentist is

proper oral hygiene can stimulate them to con- communicating.


tinue their meticulous oral hygiene behaviour at l Keep appointment short.

home. l Gradually progress to more difficult and lengthy procedures.

l Negative reinforcement: Occurs when behaviour is l Schedule the patient’s visit early in the day.

followed by the termination of an aversive event, l Generally, patients with mild retardation can be treated

thus increasing the likelihood of past behaviour. as regular patients. If the extent of work is more, N2O
l Punishment and time out, on the other hand, reduce sedation or general anaesthesia may be needed.
the likelihood of recurrence of behaviour. l However, sedation or general anaesthesia may be invari-

l Punishment exists when behaviour is followed by the ably needed to provide dental treatment for children
onset of an aversive event. with moderate and severe retardation.
l Time out or response cost: Refers to behaviour being

followed by the termination of a positive event. Q.7. Taboos in dentistry.


l In a dental setting, the dentist and auxiliaries often
Ans.
unknowingly arrange incorrect reward contingencies
that result in reinforcement of fear responses and the l Taboo is a strong social prohibition (or ban) against
extinction of non-fear responses. words, objects, actions or discussions that are considered
738 Quick Review Series for BDS 4th Year, Vol 1

undesirable or offensive by a group, culture, society or l Finger pressure on the socket, use of a hot twig
community. applied to the wound and application of a variety
l Many oral and written beliefs, superstitions, traditions of plant materials such as twigs and leaves that are
and the object of a wide range of decorative and mutila- believed to have styptic properties.
tory practices have been the cause of considerable suf- Mutilations of tooth crown
fering for many people. l Mutilations of tooth crown include alteration in the

l Some of these traditional practices are mentioned below. shape or appearance of teeth.
Tooth mutilations l This is done by chipping and filing, dyeing and lac-

Tooth mutilation practices have been recorded for in- quering of teeth, decoration of the tooth crown by
habitants of nontropical environments, but most of these inlays, overlays, etc.
customs are observed among the people living in world’s l Reasons for altering the shape of teeth – aesthetics,

tropical regions. tribal identity, initiation rituals, religious motives and


These practices include the following: identification with animals.
l Nontherapeutic tooth extraction (avulsion) Lacquering and dyeing of tooth
l Breaking off of tooth crowns l The blackening of teeth using an iron-containing

l Alteration in the shape of tooth crowns by filing mixture applied on tooth surface was custom-
and chipping practiced in ancient Japan.
l Dental inlay work l It was primarily used to signify marriage, fidelity in

l Lacquering and staining of teeth marriage and for aesthetic reasons.


l Miscellaneous practices such as placement of l Staining of teeth is usually accomplished by chewing

gold crowns for adornment purposes the leaves or bark of specific plant species.
Reasons for tooth mutilations Decorative dental inlays and crowns
Basic themes such as initiation, identification and l Use of dental inlays and crowns for adornment pur-

aesthetics underlie many of the mutilation customs poses is a form of nontherapeutic tooth mutilation
encountered throughout the tropics. occasionally encountered among contemporary peo-
Beliefs and associations ascribed to the practice of ple within and outside the tropics.
tooth avulsions are tribal identification, initiation l Usually carried out for the purpose of beautification,

rite, sign of manhood or bravery, differentiation of to signify wealth or some other events.
sexes, sign of marriageable age in females, etc. l The nature of inlay materials used are haematite,

Tooth avulsion jade, pyrite, turquoise, obsidian and gold.


l Tooth avulsion describes the deliberate removal of a Mutilations of soft tissues
tooth for ritual or traditional purposes. Tattooing
l Reasons for tooth avulsion: Ritual tooth avulsion is l The gingiva may be tattooed when females reach

carried out for reasons of identification, religious, puberty, become betrothed or get married.
spiritual reasons, to signify some life event such as l It is practised by men to relieve pain associated

the transition from childhood to manhood or woman- with ‘diseased gums’.


hood or for aesthetics and fashion. l A blue-black coloration is the usual hue achieved

Methods of tooth avulsion with gingival tattoos.


l In majority of the cultures where tooth avulsion l The material used to tattoo gingiva may be ob-

is performed, tooth is knocked out rather than tained from calcified peanuts, burned wood or
extracted. lampblack.
l This is usually carried out by placing a piece of Other mutilation practices
wood or metal against the labial aspect of the i. Piercing of lips and perioral soft tissues and the in-
tooth crown and then the striking the end of this sertion of materials such as wood, ivory or metal
object, from an appropriate direction, with some ii. Temporary piercing of orofacial soft tissues for cer-
form of mallet. emonial purposes
l Teeth may be completely avulsed by this method iii. Uvulectomy
or loosened sufficiently to allow removal using iv. Facial scarring
the fingers.
l Pain relieving or anaesthetic measures are not Q.8. Fear.
routinely prescribed preoperatively, during the
Ans.
operation or postoperatively.
l Following tooth avulsion, attempts to control l Fear is a physio-psychological response to a realistic
postoperative haemorrhage may be employed. threat or danger to one’s existence.
Section | I  Topic-Wise Solved Questions of Previous Years 739

l It is the primary emotion for survival against danger, of past experiences may fade entirely from their
which is acquired soon after birth. consciousness but the emotion associated with the
Types of fear forgotten experience determines, to a large extent,
a. Innate fear their reaction to a similar event in the future.
b. Objective fear
Q.9. HOME.
c. Subjective fear
d. Suggestive fear Ans.
e. Imitative fear
[Ref LE Q.2]
f. Imaginative fear
Innate fear Q.10. Modelling.
l Innate fear is the fear without stimuli or previous
Ans.
experience and is thus also dependent on the vul-
nerability of the individual. [Ref LE Q.2]
Objective fear
Q.11. Prevention of AIDS in dental practice.
l Objective fear is based on the child’s own experi-

ence. It is produced by direct physical stimulation. Ans.


It is the response to stimuli that is felt, seen, heard,
l Acquired immunodeficiency syndrome (AIDS) is a
smelt or tasted and is not linked or accepted.
clinically defined condition caused by infection with
Subjective fear
HIV type I or much less commonly type II.
l Children develop subjective fear based on some-
l The incubation period from the time of infection to
body else’s experience without actually undergo-
the appearance of symptoms of AIDS is approximately
ing dental treatment themselves.
11 years in adults.
l Parents may tell the child about an unpleasant or
l Therefore, HIV-infected individuals can unknowingly
pain-producing situation undergone by them, and
spread the virus to sexual or needle sharing partners,
this fear may be retained in the child’s mind.
and in the case of infected mothers, it is spread to their
Suggestive fear
children.
l It may be acquired by observing fear in others,

and the child develops a fear for the same object Preventive measures to be followed in dental practice are
as real and genuine. as follows:
l Child’s anxiety is closely correlated with parental l Barrier techniques and proper sterilization.

anxiety. If the parent is sad, the child feels sad, and l HIV is sensitive to autoclaving at 121°C for 15 min at

if the parent displays fear, then the child is fearful. 1 atmospheric pressure.
Imitative fear l Dry heat sterilization of instruments up to 170°C.

l Imitative fears may be transmitted subtly and may l Virus can be inactivated by heating lyophilized factor at

be displayed by parent and acquired by child 68°C for 72 h.


without either being aware of it. l Disinfectants for innate objects:
l These are generally recurrent fears and therefore l Calcium hypochlorite 0.2%, sodium hypochlorite

are more deep-seated and difficult to eradicate. l 6% hydrogen peroxide for more than 30 min

Displayed emotion, such as anxiety observed in l 2% glutaraldehyde and 6% hydrogen peroxide

the parent’s face, may create more of an impres- l Sodium dichloroisocyanurate treatment for 10 min at

sion than verbal suggestion. room temperature with 10% household bleach, 50%
For example, a mother who fears going to a den- ethanol and 3% hydrogen peroxide.
tist and goes only under great emotional stress l Gloves may be disinfected by immersing them in boil-

transmits this fear unconsciously to her child who ing water for 20 min or alternatively soaking in 1% so-
is observing her. dium hypochlorite overnight.
Imaginative fear
l As the imaginative capacities of children develop, Q.12. Wright’s classification of behaviour of children in
imaginary fears become more intense. Imaginary dental office.
fears, therefore, become greater with age and
Ans.
mental development, up to a certain age, when
reason shows them to be ill founded. I. Cooperative behaviour
l Fears may be irrational in the sense that children l Child is cooperative.

may not know why they are frightened. Memories l Reasonably relaxed.
740 Quick Review Series for BDS 4th Year, Vol 1

l Develop good rapport with dentist. Oedipus complex


l Laughs and enjoys the situation. l The child begins to direct their awakened sexual

II. Lacking cooperative behaviour impulses towards the parent of opposite sex.
l This behaviour is in contrast to cooperative child. l Attachment between young boys and mother, and

l Includes young (0–3 years), disabled and physi- they consider father as their enemy.
cally and mentally handicapped children. l The name Oedipal complex comes from Greek my-

l They can have major behavioural problems. thology. Oedipus, the king of Thebes, unwillingly
III. Potentially cooperative behaviour slew his father and married his mother.
l Child is cooperative. l The little boys strive to imitate their father to gain

l Physically and mentally fit. affection of their mother. Freud also described Oedi-
l Child’s behaviour can be modified. pal complex in boys as a desire to have a sexual rela-
l Group of children require behavioural modification tion with mother.
procedures.
Q.2. Write a brief note on sociology.
Potentially cooperative behaviour is subclassified into
the following types: Ans.
Uncontrolled
Sociology is defined as the study of groups of individuals
l Age group 3–6 years
that form a society, how they interact and behave within
l Also called incorrigible behaviour
themselves, and the outcome of these interactions.
l Tears, loud crying, physical lashing out and
Sociological factors governing dental health care ser-
flailing of hands
vices are as follows:
Defiant behaviour
l Affordability of dental health services
l Found in all ages.
l Acceptability of dental health services
l Also referred to as stubborn and spoilt.
l Community participation
l They do not like to go to dental clinic.
l Sustainability of programme
Timid behaviour
l Seen in overprotective child Q.3. Define modelling.
l Milder but highly anxious
Ans.
l Is shy but cooperative

Tense cooperative [Ref LE Q.2]


l Borderline behaviour.
Q.4. Reinforcement.
l Child is tensed in mind.

Whining behaviour Ans.


l Whining throughout the process.
[Ref LE Q.2]
l Cry is controlled, constant and not loud.

Q.5. Taboos related to oral health.


Q.13. Physical restrainer.
Ans. Ans.

[Ref LE Q.2] l Taboo is a strong social prohibition (or ban) against


words, objects, actions or discussions that are consid-
Q.14. TSD. ered undesirable or offensive by a group, culture, soci-
Ans. ety or community.
l Examples of taboos related to dentistry are as follows:
[Ref LE Q.2] l Nontherapeutic tooth extraction (avulsion)
l Breaking of tooth crowns

SHORT NOTES: l Alteration in the shape of tooth crowns by filing and

chipping
Q.1. Oedipus complex.
l Dental inlay work

Ans. l Lacquering and staining of teeth

According to Freud, phallic stage of development of child l Practices such as the placement of gold crowns for

is characterized by: adornment purposes


l Oedipus complex l Tattooing of gingival

l Electra complex l Piercing of lips and perioral soft tissues


Section | I  Topic-Wise Solved Questions of Previous Years 741

Piercing of orofacial soft tissues


l l Acquired immunodeficiency syndrome (AIDS) is a
Uvulectomy
l clinically defined condition caused by infection with
l Facial scarring HIV type I or much less commonly type II.
l Facial soft tissues are often subjected to scarification. l Oral manifestations of AIDS patients include fungal,

l Scarification may be carried out for a variety of rea- bacterial and viral infections, linear gingival erythema
sons, including tribal identity aesthetics, to enhance and pyogenic bacterial infection such as otitis media,
sexual appeal, indicate status and signify events such as hepatosplenomegaly, chronic pneumonitis and progres-
puberty, marriage or childbirth. sive encephalopathy.
Q.6. Autoclave. Q.12. Tongue thrusting.
Ans. Ans.
l An autoclave is a device used for sterilization using According to Schneider (1982), tongue thrust is defined as
steam under pressure technique. a forward placement of tongue between the anterior teeth
l A basic principle of chemistry is that when the pressure and the lower lip during swallowing.
of a gas increases, temperature of the gas increases pro- Aetiology
portionally. The factors considered as a cause of tongue thrusting
l Sterilization by steam under pressure is carried out habit, according to Fletcher, are as follows:
at temperatures between 108 and 147°C for 15 min at i. Genetic factors
15 pounds pressure. ii. Learned behaviour (habit)
iii. Maturational factors
Q.7. Tell-show-do.
iv. Mechanical restrictions
Ans. v. Neurological disturbance
vi. Psychogenic factors
[Ref LE Q.2]
Clinical features
Q.8. Nuclear family. Some common clinical features of tongue thrust habit
are as follows:
Ans.
l Proclination of anterior teeth

l Nuclear family consists of husband, wife and children. l Bimaxillary protrusion

l This is common in urban areas and it is most easy to l Anterior open bite

advise them to change their behaviour since the family l In case of lateral tongue thrust, posterior open bite

is small. and posterior crossbite


l It is universal in all human societies.
Q.13. Taboos related to dentistry in India.
Q.9. Enumerate basic behaviour management tech-
Ans.
niques.
[Same as SN Q.5]
Ans.
Q.14. Culture and oral health.
[Ref LE Q.2]
Ans.
Q.10. Define behaviour management.
[Same as SN Q.5]
Ans.
[Ref LE Q.2]
Q.11. Oral manifestations of AIDS.
Ans.
This page intentionally left blank
Section I

Topic-Wise Solved Questions


of Previous Years

PART IV PERIODONTICS
Topic 1 Gingiva 745
Topic 2 Tooth-Supporting Structures (Periodontal
Ligament, Alveolar Bone, Cementum) 757
Topic 3 Age-Related Changes in the Periodontium 766
Topic 4 Classification of Diseases of the Periodontium 769
Topic 5 Epidemiology of Gingival and Periodontal Diseases 772
Topic 6 Periodontal Microbiology 780
Topic 7 Dental Calculus, Iatrogenic and Other Local
Predisposing Aetiological Factors 789
Topic 8 Smoking and Periodontium 794
Topic 9 Host Response: Basic Concepts 795
Topic 10 Host–Microbial Interactions in Periodontal Diseases 800
Topic 11 Trauma from Occlusion 802
Topic 12 Influence of Systemic Diseases on the Periodontium
and Periodontal Medicine 813
Topic 13 Dental Implants 822
Topic 14 Defence Mechanisms of the Gingiva 826
Topic 15 Gingival Inflammation and Clinical Features of Gingivitis 834
Topic 16 Gingival Enlargements 843
Topic 17 Acute Gingival Infections 856
Topic 18 Desquamative Gingivitis 864
Topic 19 Gingival and Periodontal Diseases in Children
and Young Adolescents 868
Topic 20 Periodontal Pocket 873
Topic 21 Bone Loss in Periodontal Diseases 884
Topic 22 Periodontitis: Chronic, Refractory and Necrotizing Ulcerative 892
Topic 23 Aggressive Periodontitis 897
Topic 24 Periodontal Abscess 904
Topic 25 Halitosis 908
Topic 26 Clinical Diagnosis and Advanced Diagnostic Methods 911
Topic 27 Determination of Prognosis 926
Topic 28 Periodontal Treatment Plan 931
Topic 29 Periodontal Instrumentation 934
Topic 30 Principles of Periodontal Instrumentation 944
Topic 31 Sonic and Ultrasonic Instrumentation 950
Topic 32 General Principles and Concepts of Growth 953
Topic 33 Plaque Control 956
Topic 34 Chemotherapeutic Agents 973
Topic 35 Periodontal Splints 981
Topic 36 General Principles of Periodontal Surgery 984
Topic 37 Gingival Surgical Procedures 997
Topic 38 Periodontal Flap Surgery 1006
Topic 39 Resective Osseous Surgery 1010
Topic 40 Regenerative Osseous Surgery 1015
Topic 41 Furcation Involvement and Its Management 1028
Topic 42 Endodontic Periodontal Lesions and Their Management 1033
Topic 43 Orthodontic Periodontal Inter-Relationship 1038
Topic 44 Perioprosthodontics/Occlusal Evaluation 1040
Topic 45 Supportive Periodontal Treatment (Maintenance Phase) 1044
Section I

Topic-Wise Solved Questions


of Previous Years
Part IV
Periodontics

Topic 1
Gingiva
COMMONLY ASKED QUESTIONS
LONG ESSAYS:
1. Define gingiva. Describe its macroscopic and microscopic appearances and functions. Add a note on importance
of gingival fluid.
2. Describe the normal structure of gingiva. Write in detail about the electron microscopic structure of gingival
epithelium. [Same as LE Q.1]
3. Define oral mucosa. Describe the clinical and microscopic features of normal gingiva. [Same as LE Q.1]
4. Describe the clinical and histological features of normal healthy gingiva. [Same as LE Q.1]
5. Define gingiva. What are the parts of normal gingiva? Describe the microscopic picture of normal gingiva. 
[Same as LE Q.1]

SHORT ESSAYS:
1 . Compare attached gingiva and alveolar mucosa.
2. Histology of gingival surface epithelium. [Ref LE Q.1]
3. Gingival fibres.
4. Functions of attached gingiva.
5. Sulcular epithelium.
6. Gingival fluid.

745
746 Quick Review Series for BDS 4th Year, Vol 1

7. Junctional epithelium. [Ref LE Q.1]


8. Dentogingival junction.
9. Gingival pigmentation.
10. Describe the various concepts of formation of the gingival sulcus and give its significance.
11. Differences between attached gingiva and alveolar mucosa. [Same as SE Q.1]
12. Microscopic features of healthy gingiva. [Same as SE Q.2]
13. Importance of attached gingiva. [Same as SE Q.4]
14. Functions of gingival crevicular fluid. [Same as SE Q.6]
15. Methods of collecting gingival fluid. [Same as SE Q.6]
16. Factors affecting gingival crevicular fluid flow. [Same as SE Q.6]

SHORT NOTES:
1. Col.
2. Gingiva.
3. Gingival fibres.
4. Classification of gingival fibres. [Ref SE Q.8]
5. Gingival sulcus.
6. Mast cells in gingival.
7. Enumerate gingival blood supply, lymphatics and nerves.
8. Saliva.
9. Long junctional epithelium.
10. Functions of gingival fibre system.
11. Clinical significance of keratinized gingiva.
12. Interdental papilla.
13. Microscopic features of healthy gingiva.
14. Gingival crevicular fluid.
15. Langerhans cells.
16. Palatogingival groove.
17. Define and classify embrasures.
18. Orogranulocytes.
19. Attached gingiva. [Ref SE Q.4]
20. Mucogingival junction.
21. Gingival stippling.
22. Biological width.
23. Transgingival probing.
24. Dentogingival unit.
25. Stillman’s clefts.
26. McCall’s festoons.
27. Gingival col. [Same as SN Q.1]
28. Free gingiva. [Same as SN Q.2]
29. Gingival fibres and their importance. [Same as SN Q.3]
30. Role of mast cells in gingival. [Same as SN Q.6]
31. Role of saliva as a defence mechanism of gingiva. [Same as SN Q.8]
32. Importance of attached gingiva. [Same as SN Q.19]
Section | I  Topic-Wise Solved Questions of Previous Years 747

SOLVED ANSWERS
LONG ESSAYS:
Q.1. Define gingiva. Describe its macroscopic and mi- l Since the interdental papilla has a shape in
croscopic appearances and functions. Add a note on conformity with the outline of the interdental
importance of gingival fluid. contact surfaces, a concavity – a col – is es-
tablished in the premolar and molar regions.
Ans.
Col
Gingiva is defined as a part of the oral mucosa that covers l It is a valley-like depression that connects
the alveolar process of the jaws and surrounds the necks of the facial and lingual papillae.
the teeth. l It conforms to the shape of interproximal
The clinical or macroscopic features of the gingiva are as contact areas.
follows: l It is prone to inflammation due to its non-

l The colour of attached and marginal gingiva is usually keratinized epithelium.


‘coral pink’. l The interdental papilla is absent in the

l It varies according to the degree of vascularity, epithelial presence of diastema.


keratinization and pigmentation.
l Gingiva is divided anatomically into:
[SE Q.2]
i. Marginal gingiva or free or unattached gingiva {Microscopic features of gingiva are as follows:
ii. Attached gingiva 1 . Gingival epithelium
iii. Interdental gingiva or papillary gingiva 2. Gingival connective tissue
i. Marginal gingiva Gingival epithelium
l The gingival epithelium comprises the epithelial tis-
l The terminal border or edge of the gingiva surround-

ing the neck of the teeth in a collar-like fashion. sue that covers the external surface of the gingiva as
l Usually 1-mm width of marginal gingiva forms
well as the epithelium lining the gingival sulcus and
the soft tissue wall of the gingival sulcus. the junctional epithelium.
l Keratinocyte is the principal cell type of the gingival
l The free gingival margin is located on the enamel

surface, approximately 1.5–2-mm coronal to the epithelium.


l In addition to the keratin-producing cells, the epithe-
cementoenamel junction (CEJ), and is demar-
cated from the attached gingiva by a shallow de- lium also contains the nonkeratinocytes like:
pression called the free gingival groove. i. Melanocytes
ii. Attached gingiva ii. Langerhans cells
Importance of attached gingiva iii. Merkel cells
l It is continuous with marginal gingiva and
iv. Inflammatory cells
extends to the mucogingival junction on the These are also called ‘clear cells’ since in histo-
facial and lingual aspects of mandible and fa- logic sections the zone around their nuclei ap-
cial aspect of maxilla. pears lighter than that in the surrounding keratin-
l It is firm, resilient and tightly bound to the
producing cells.
periosteum of the alveolar bone. i. Melanocytes
l Stippling is seen in 40% of the attached gingiva. l Melanocytes are pigment-synthesizing spe-
Functions cialized cells which produce melanin and are
l It braces the marginal gingiva. responsible for the melanin pigmentation oc-
l It allows for proper deflection of food. casionally seen on the gingiva.
l It provides room for proper placement of l These are embryologically derived from neu-
toothbrush. ral crest cells that eventually migrate into the
l It has an aesthetic value and is critical for epithelium.
overall maintenance of gingival health. l The melanocytes are located in the basal layer
iii. Interdental gingiva of the epithelium.
l The interdental gingiva occupies the gingival l The melanin is produced as granules, known
embrasures apical to the contact areas. as the melanosomes, stored within the cyto-
l The shape of the interdental gingiva is deter- plasm of the melanocytes as well as the adja-
mined by the contact relationships between the cent keratinocytes.
teeth, the width of the proximal tooth surfaces l In both lightly and darkly pigmented individu-
and the course of the CEJ. als, melanocytes are present in the epithelium.
748 Quick Review Series for BDS 4th Year, Vol 1

ii. Langerhans cells l The HDs are involved in the attachment of


l Langerhans cells are dendritic cells located among the epithelium to the underlying basement
keratinocytes at suprabasal levels containing large membrane.
granules called Birbeck granules. Functions of basal cell layer
l These are derived from the cellular differentiation l The important function is protecting

of monocytes. the underlying structures.


l These are believed to play a role in the defence l Producing new epithelial cells.

mechanism of the oral mucosa. l They synthesize and secrete the macro-

l In the presence of an infection, Langerhans cells molecules that constitute the basal lamina.
will take up and process microbial antigen to be- ii. Prickle cell layer (stratum spinosum)
come fully functional antigen-presenting cells, l The cells are polyhedral shaped and are

thereby inhibiting or preventing further antigen pen- characterized at the light microscopic level
etration of the tissue. by apparent intercellular bridges.
iii. Merkel cells l These cells possess decreased numbers of

Merkel cells have been suggested to have a sensory mitochondria equipped with short cytoplas-
function. mic processes resembling spines. The cyto-
The gingival epithelium is subdivided into three sec- plasmic processes occur at regular intervals
tions based on its function and some of its histological and give the cells a prickly appearance.
characteristics. l Together with intercellular protein–carbohy-

a. Oral epithelium (OE) drate complexes, cohesion between the cells


b. Sulcular epithelium is provided by numerous ‘desmosomes’, i.e.
c. Junctional epithelium} pairs of HDs, which are located between the
cytoplasmic processes of adjacent cells.
a. Oral epithelium iii. Granular cell layer (SG)
l OE is a stratified, squamous, keratinizing epithe-
l The epithelial cells of the SG are flattened
lium that lines the vestibular and oral surfaces of and tonofibrils take up an increasing volume
the gingiva. of the cytoplasmic contents.
l It extends from the mucogingival junction to the
l The flattened cellular elements of the SG
gingival margin, while on the palatal surface it contain relatively sparse, round cytoplasmic
blends with the palatal epithelium. granules with keratohyalin in its cytoplasm
l OE can be divided into the following cell layers:
and enzyme containing Odland bodies.
i. Basal cell layer (stratum basale or stratum l Odland bodies or keratinosomes are modi-
germinativum) fied lysosomes present in the layers between
ii. Prickle cell layer (stratum spinosum) the stratum spinosum and SG.
iii. Granular cell layer (stratum granulosum
[SG]) iv. Keratinized cell layer (stratum corneum)
iv. Keratinized cell layer (stratum corneum) l The stratum corneum consists of tightly

i. Basal cell layer (stratum basale) packed cornified cells.


l The basal cell layer represents the germi- l The cells contain densely packed tonofila-

native layer. ments. No nuclei or cytoplasmic organelles


l The cells of basal layer are mainly cylin- are detectable. This form of keratinization is
drical or cuboidal in appearance in con- referred to as orthokeratinization, i.e. com-
tact with the basal lamina. plete keratinization of the epithelial cells.
l The basal lamina or basement membrane b. Sulcular epithelium
under the electron microscope reveals an l The sulcular epithelium is a stratified, squamous

electron-dense layer, the lamina densa and nonkeratinized epithelium which lines the gin-
(LD) in contact with the connective tis- gival sulcus.
sue compartment, and an electron lucent l It is apically bounded by the junctional epithelium

layer, the lamina lucida (LL) in contact and meets the epithelium of the oral cavity at the
with the epithelial cells. height of the free gingival margin.
l The cell membrane of the epithelial cells l It is the epithelium which is continuous with the OE

facing the LL harbours a number of and lines the lateral surface of the sulcus apically,
electron-dense and thicker zones ap- and overlaps the coronal border of the junctional
pearing at various intervals along the epithelium.
cell membrane known as hemidesmo- l The epithelium shares many of the characteristics of

somes (HDs). the OE, including good resistance to mechanical


Section | I  Topic-Wise Solved Questions of Previous Years 749

forces and relative impermeability to fluid and cells, l The cellular and extracellular dynamics of the junc-
except for the surface layer that is nonkeratinized. tional epithelium are essential for its protective and
l The tightly sealed intercellular spaces (ICS) con- regenerative functions.
tribute to the low permeability of sulcular epithe- l While cell mitosis occurs in the basal and possibly
lium unlike the junctional epithelium, which is also in some of the tooth-facing junctional epithe-
heavily infiltrated by PMNs. lial cells, exfoliation of daughter cells takes place at
the free surface of the junctional epithelium, i.e. at
[SE Q.7] the bottom of the sulcus and the interdental col.
{c. Junctional epithelium l Junctional epithelial cells migrate in the coronal
l Junctional epithelium is a stratified, squamous non- direction towards the free surface, where they des-
keratinizing epithelium that surrounds the tooth like quamate. Since the surface area occupied by the
a collar, facing both the gingival connective tissue basal cells is much greater than that of bottom of
and the tooth surface. the sulcus, exfoliation must occur at an extremely
l Junctional epithelium is formed by the confluence high rate.
of the OE and the reduced enamel epithelium dur- l The junctional epithelium is more permeable than
ing tooth eruption. the oral or sulcular epithelium due to wide intercel-
l The attachment of the junctional epithelium to the lular spaces.
tooth is reinforced by the gingival fibres. The junc- l Soluble substances can diffuse from the oral cavity
tional epithelium and the gingival fibres are consid- into the underlying gingival connective tissue epi-
ered as functional unit, also known as dentogingival thelia, while both fluids and cells can travel through
unit. the junctional epithelium from the connective tissue
l The junctional epithelium tapers off in the apical into the gingival sulcus on their way to the oral cav-
direction, and it consists of 15–30 cell layers coro- ity. Thus, it has a bidirectional flow.
nally, and only 1–3 cell layers at its apical termina- l The connective tissue adjacent to the junctional
tion. The length of the junctional epithelium ranges epithelium tends to show an increased level of cell
from 0.25 to 1.35 mm. turnover, because of its permeability to bacterial
l The cells of junctional epithelium can be grouped products and other assorted antigens originating in
into two strata, i.e. a basal layer called the stratum the oral cavity.
basale, and a suprabasal layer called the stratum l While cell mitosis occurs in basal layer and possi-
suprabasale. bly also in some areas adjacent to it, the zone be-
l The proliferative cell layer responsible for most cell comes infiltrated with chronic inflammatory cells,
divisions is located in contact with the connective primarily lymphocytes and plasma cells.
tissue. The shedding surface of the junctional epi- l The intercellular spaces of the junctional epithe-
thelium is located at its coronal end, which also lium provide a pathway for fluid and transmigrating
forms the bottom of the gingival sulcus. leukocytes. In the absence of clinical signs of in-
l Compared with other epithelia, junctional epithe- flammation, approximately 30,000 PMNs migrate
lial cells are interconnected by a few desmosomes per minute through the junctional epithelia of all
only and occasionally by gap junctions. human teeth into the oral cavity.}
l The junctional epithelium, particularly its basal cell Importance of gingival fluid (sulcular fluid)
layers, is well innervated by sensory nerve fibres. l The gingival fluid can be represented as either

l The junctional epithelium is divided into three a transudate or an exudate.


zones: (i) coronal, (ii) middle and (iii) apical. l The gingival fluid contains a vast array of bio-

l The junctional epithelium has two basal laminae, one chemical factors, offering potential use as a
that faces the tooth (internal basal lamina) and one that diagnostic or prognostic biomarker of the bio-
faces the connective tissue (external basal lamina). logical state of the periodontium in health and
l The external basal lamina has the same structure and disease.
composition as other basement membranes, else- l The gingival fluid contains components of con-

where in the body, while the internal basal lamina nective tissue, epithelium, inflammatory cells,
has distinctively different structural and molecular serum, and microbial flora inhabiting the gin-
characteristics. It lacks most of the common base- gival margin or the sulcus (pocket).
ment membrane components such as collagen types l In the healthy sulcus, the amount of the gingi-

IV and VII, most laminin isoforms. val fluid is very small, while during inflamma-
l The internal basal lamina comprises two zones: tion, the gingival fluid flow increases, and its
i. Electron-lucent zone – the LL composition resembles that of an inflammatory
ii. Electron-dense zone – the LD exudate.
750 Quick Review Series for BDS 4th Year, Vol 1

l The functions of gingival fluid are as follows: SHORT ESSAYS:


i. Cleanses material from the sulcus.
ii. Improves adhesion of the epithelium to the Q.1. Compare attached gingiva and alveolar mucosa.
tooth by plasma proteins. Ans.
iii. Possesses antimicrobial properties.
iv. Exerts antibody activity to defend the gingiva. l Alveolar mucosa is red in colour but attached gingiva is
v. The tissue fluid transports a variety of mol- coral pink.
l Use of Schiller’s potassium iodide staining: This solu-
ecules, these together with the leukocytes,
represent a host defence system against the tion stains the glycogen content of tissues and is more
bacterial challenge. in alveolar mucosa than in attached gingiva. It stains
dark brown to alveolar mucosa compared to the at-
[SE Q.2] tached gingiva, the differentiating line between them
{Gingival connective tissue being mucogingival junction.
The major components of gingival connective tissue l Alveolar mucosa gets blown out but attached gingiva is

are as follows: firm when anaesthetized.


i. Collagen fibre bundles 60%–65% l As a probe is moved from movable tissue to firm tissue,

ii. Cellular elements (fibroblasts 5%, various leuko- blanching of tissue occurs at mucogingival junction.
cytes, mast cells, tissue macrophages, etc. 3%)
iii. Vascular elements (blood and lymph vessels), Q.2. Histology of gingival surface epithelium.
nerves and ground substance about 35% Ans.
The connective tissue of the gingiva is known as the
lamina propria and consists of two layers: [Ref LE Q.1]
i. Papillary layer
ii. Reticular layer Q.3. Gingival fibres.
l Connective tissue has a cellular and an Ans.
extracellular compartment composed of
fibres and ground substance. The connective tissue of the marginal gingiva is densely
l The ground substance fills the space
collagenous containing a prominent system of collagen
between fibres and cells. Its major con- fibre bundles called gingival fibres.
stituents are water, glycoproteins and Following are the groups of principal fibres of gingiva:
proteoglycans.} i. Alveologingival
ii. Dentoperiosteal
Q.2. Describe the normal structure of gingiva. Write in iii. Gingivodental group
detail about the electron microscopic structure of gingi- iv. Circular group
val epithelium. v. Trans-septal group
vi. Dentoperiosteal
Ans. Following are the secondary fibre bundles of gingiva:
i. Periosteogingival
[Same as LE Q.1] ii. Interpapillary
iii. Transgingival
Q.3. Define oral mucosa. Describe the clinical and mi-
iv. Intercircular
croscopic features of normal gingiva.
v. Intergingival semicircular
Ans.
Most of the fibres are composed of collagen. Type 1
[Same as LE Q.1] collagen forms the main bulk of the collagen.
Elastic and oxytalan fibres are confined to perivascular
Q.4. Describe the clinical and histological features of regions, although oxytalan fibres are also found as thin fibre
normal healthy gingiva. bundles within collagen-rich connective tissue.
Ans.
Q.4. Functions of attached gingiva.
[Same as LE Q.1]
Ans.
Q.5. Define gingiva. What are the parts of normal gin- Gingiva is divided anatomically into:
giva? Describe the microscopic picture of normal gingiva. i. Marginal gingiva or free or unattached gingiva
Ans. ii. Attached gingiva
iii. Interdental gingiva or papillary gingiva
[Same as LE Q.1]
Section | I  Topic-Wise Solved Questions of Previous Years 751

l It is the epithelium which is continuous with the


{SN Q.19}
OE and lines the lateral surface of the sulcus and api-
Attached Gingiva cally overlaps the coronal border of the junctional
Importance of attached gingiva epithelium.
l It is continuous with marginal gingiva and l The epithelium shares many of the characteristics

extends to the mucogingival junction on the of the OE, including good resistance to mechanical
facial and lingual aspects of mandible and forces and relative impermeability to fluid and
facial aspect of maxilla. cells, except for the surface layer that is nonkera-
l It is firm, resilient and tightly bound to the tinized.
periosteum of alveolar bone. l The tightly sealed intercellular spaces (ICS) contrib-

l Stippling is seen in 40% of attached gingiva. ute to the low permeability of sulcular epithelium.
l When pocket or recession extends beyond the l If it is reflected and exposed to the oral cavity, it has

mucogingival junction, there is absence of at- the potential to keratinize and bacterial flora of the
tached gingiva. sulcus is totally eliminated. These findings suggest
Functions that the local irritation of sulcus prevents sulcular
l It braces the marginal gingiva. keratinization.
l It allows for proper deflection of food. l The overall structure of the sulcular epithelium re-

l It provides room for proper placement of sembles that of the OE, except for the surface layer
toothbrush. that is nonkeratinized than its counterpart in the OE.
l It has an aesthetic value and is critical for over-

all maintenance of gingival health. Q.6. Gingival fluid.


Ans.
Measurement of attached gingiva is as follows:
l The gingival fluid can be represented as either a transu-
i. Clinically: Attached gingiva is coral pink in colour.
date or an exudate.
ii. Using Schiller’s potassium iodide solution to
l The gingival fluid contains a vast array of biochemical
stain mucosa and attached gingiva. Glycogen
factors, offering potential use as a diagnostic or prog-
content in alveolar mucosa is more than at-
nostic biomarker of the biologic state of the periodon-
tached gingiva; hence, it gets stained dark
tium in health and disease.
brown compared to attached gingiva.
l The gingival fluid contains components of connective
iii. Passing the probe horizontally from alveolar
tissue, epithelium, inflammatory cells, serum and mi-
mucosa to attached gingiva. The probe is moved
crobial flora inhabiting the gingival margin or the sulcus
from the movable tissue to firm tissue, blanch-
(pocket).
ing occurs at the site of mucogingival junction.
l In the healthy sulcus, the amount of the gingival fluid is
iv. Anaesthetizing the area which blows out the
very small, while during inflammation, the gingival
loose alveolar mucosa and leaves the firm at-
fluid flow increases, and its composition resembles that
tached gingiva and hence easily differentiates
of an inflammatory exudate.
the mucogingival junction.
The functions of gingival fluid are as follows:
Q.5. Sulcular epithelium. i. It cleanses material from the sulcus.
ii. It improves adhesion of the epithelium to the tooth
Ans.
by plasma proteins.
The gingival epithelium is subdivided into three sections iii. It possesses antimicrobial properties.
based on its function and some of its histological character- iv. It exerts antibody activity to defend the gingiva.
istics. v. The tissue fluid transports a variety of molecules.
a. OE These together with the leukocytes represent a
b. Sulcular epithelium host defence system against the bacterial chal-
c. Junctional epithelium lenge.
Sulcular epithelium Methods of collecting gingival fluid:
l The sulcular epithelium is a stratified, squamous and l Fluid is derived with filter papers from gingival
nonkeratinized epithelium which lines the gingival sulcus intrasulcular (within the gingival sulcus)
sulcus. and extrasulcular (at its entrance) according to
l It is apically bounded by the junctional epithelium filter strips.
and meets the epithelium of the oral cavity at the l Filter paper strips – used in enzyme assays, no-
height of the free gingival margin. tably protease enzyme.
752 Quick Review Series for BDS 4th Year, Vol 1

l Microcapillary tubes are used which are better l It provides support to gingiva and attaches it to bone,
from filter paper as it allows for complete anchors tooth to bone, maintains relationship of adja-
collection. cent teeth and secures alignment of teeth in the arch and
l Crevicular washing methods of various types of stabilizes them in the arch.
acrylic plate, four collection tubes, peristaltic
Q.9. Gingival pigmentation.
pump. Modification is ejection collecting needle
from which sample is drained into a tube by con- Ans.
tinuous suction.
l Melanin, a nonhaemoglobin-derived brown pigment, is
l Twisted threads into gingival sulcus.
responsible for the normal pigmentation of the skin,
l The Gingival crevicular fluid (GCF) samples col-
gingiva and remainder of the oral mucous membrane.
lected may be measured on a perio paper or blot-
l It is present in all normal individuals often not in suffi-
ter, employing an electronic transducer, i.e. peri-
cient quantities to be detected clinically, but it is absent
otron. The samples may also be analysed by
or severely diminished in albinos.
ninhydrin staining method or isotope dilution
l Distribution in black individuals is as follows: gingiva
method.
60%, hard palate 61%, mucous membrane 22% and
Q.7. Junctional epithelium. tongue 15%.
Ans. l Gingival pigmentation occurs as a diffused deep purple

or irregularly shaped brown and light brown patches. It


[Ref LE Q.1] appears in gingiva 3 h after birth.
Q.8. Dentogingival junction. l Oral repigmentation refers to clinical reappearance of

Ans. melanin pigment after a period of depigmentation of


oral mucosa resulting from chemical, thermal, surgical,
l The attachment of junctional epithelium to the tooth is pharmacological or idiopathic. Information on the
reinforced by gingival fibres, which brace the marginal repigmentation of oral tissues after surgical procedure is
gingiva against the tooth surface. For this reason, the limited.
junctional epithelium and gingival fibres are considered
as a functional unit, also called dentogingival unit. Q.10. Describe the various concepts of formation of the
gingival sulcus and give its significance.
{SN Q.4} Ans.
l The connective tissue of the marginal gingiva is l The gingival sulcus is a shallow fissure between the
densely collagenous, containing a prominent system marginal gingiva and the enamel or cementum.
of collagen fibre bundles called gingival fibres, which l It is V-shaped and barely permits the entry of periodon-
consist of type 1 collagen. tal probe. Under normal or ideal condition it is about
Following are the groups of principal fibres of gingi- 0 mm; its probing depth being 1–3 mm.
val connective tissue: l It is bounded to tooth on one side and the sulcular epi-
i. Dentogingival group thelium on the other side and the coronal end of junc-
ii. Alveologingival group tional epithelium at its most apical point.
iii. Dentoperiosteal group l Schroeder and Page have summarized the events of
iv. Circular group continued differentiation that occur as a new basal cell
v. Trans-septal group derives from a mitotic activity in the stratum basale and
Following are secondary fibres of gingival connec- makes its way towards the intraoral epithelial surface:
tive tissue: i. Cells lose the ability to multiply by mitotic division.
i. Periosteogingival group ii. Cells produce elevated amounts of protein and ac-
ii. Interpapillary group cumulate keratohyalin granules, keratin filaments
iii. Transgingival group and macromolecular matrix in their cytoplasm.
iv. Intercircular group iii. Cells lose their cytoplasmic organelles responsible
v. Intergingival group for protein synthesis and energy production.
vi. Semicircular group l Cells eventually degenerate into a cornified layer due to

the process of intracellular keratinization but without


l These collagen fibre groups originate from cemental loss of cell–cell attachment.
surface of tooth and get embedded into lamina propria l Cells are finally sloughed away from the epithelial sur-
of gingiva and periosteum of alveolar crest or cemental face and into the oral cavity as the cell–cell attachment
surface of adjacent root forming dentogingival junction. (HD and gap junctions ultimately disintegrate).
Section | I  Topic-Wise Solved Questions of Previous Years 753

Q.11. Differences between attached gingiva and alveolar l The colour of gingiva is coral pink and varies according
mucosa. to the degree of vascularity, epithelial keratinization and
pigmentation.
Ans.
l Gingiva is demarcated into:

[Same as SE Q.1] a. Marginal or free gingiva or unattached


b. Attached gingiva
Q.12. Microscopic features of healthy gingiva.
c. Interdental gingiva (papillary)
Ans.
Q.3. Gingival fibres.
[Same as SE Q.2]
Ans.
Q.13. Importance of attached gingiva.
The connective tissue of the marginal gingiva is densely
Ans. collagenous, containing a prominent system of collagen
fibre bundles called gingival fibres, which consist of type 1
[Same as SE Q.4]
collagen.
Q.14. Functions of gingival crevicular fluid. Importance of gingival fibres is as follows:
i. To brace the marginal gingiva firmly against the tooth
Ans.
ii. To provide the rigidity necessary to withstand the
[Same as SE Q.6] forces of mastication without being deflected away
from the tooth surface
Q.15. Methods of collecting gingival fluid.
iii. To unite the free marginal gingiva with cementum of
Ans. the root and the adjacent attached gingiva
[Same as SE Q.6] Q.4. Classification of gingival fibres.
Q.16. Factors affecting gingival crevicular fluid flow. Ans.
Ans. [Ref SE Q.8]
[Same as SE Q.6] Q.5. Gingival sulcus
Ans.
SHORT NOTES: l It is defined as the space or shallow crevice between the
Q.1. Col. tooth and the free gingiva, and extends apical to the
junctional epithelium.
Ans.
l It is bounded by tooth surface on one side, sulcular epi-

l Col is a valley-like depression that connects the facial thelium on the other and coronal end of junctional epi-
and lingual papillae of interdental gingiva in the poste- thelium at its most apical point.
rior teeth. l It is V-shaped and barely permits the entrance of the
l The interdental gingiva occupies the gingival embra- periodontal probe.
sures apical to the contact area. A concavity known as l Under normal or ideal conditions it is about 0 mm.

col is established in the premolar and molar regions. It l Probing depth of a clinically normal gingival sulcus in

conforms to the shape of interproximal contact area. humans is 2–3 mm.


l The epithelium of the col is nonkeratinized; hence, it is
Q.6. Mast cells in gingiva.
prone to inflammation and represents the most frequent
site for initiation of the disease. Ans.
Q.2. Gingiva. Mast cells characteristics are as follows:
i. Oval or round in shape
Ans.
ii. Contain cytoplasmic granules
l The part of the oral mucosa that covers the alveolar iii. Granules contain
process of the jaw and surrounds the neck of the teeth is a. mediators of inflammation such as histamine and
known as gingiva. serotonin
l The gingiva extends from gingival margin to the mu- b. anticoagulants such as heparin
cogingival junction except on the palatal surfaces of iv. When stimulated they degranulate, disseminating their
maxillary molar, where it merges with the palatal granular contents into the surrounding connective
mucosa. tissue
754 Quick Review Series for BDS 4th Year, Vol 1

Q.7. Enumerate gingival blood supply, lymphatics and The functions of gingival fibres are as follows:
nerves. l Trans-septal fibres: They maintain teeth in arch.

l Alveolar crest fibres: They secure teeth in socket and


Ans.
resist lateral forces applied to the tooth.
Blood supply of gingiva l Horizontal fibres: They prevent lateral movement.
i. Supraperiosteal arterioles, overlying the alveolar l Oblique fibres: They resist apically directed masticatory
bone along the facial and lingual surfaces send forces.
branches to the surrounding tissues. l Apical fibres: They prevent tooth tipping, resist forces
ii. Vessels of the periodontium extend into the gingiva of luxation and protect blood and nerve supply.
and anastomose with capillaries in the sulcus area. l Inter-radicular fibres: They resist tipping of tooth forces
iii. Arterioles emerging from the crest of the interdental of luxation and rotation.
septa.
Lymphatic drainage of gingiva Q.11. Clinical significance of keratinized gingiva.
Gingiva brings in the lymphatic vessels of connec- Ans.
tive tissue papillae. It progresses to the periosteum of
the alveolar process and then to regional lymph l Keratinized gingiva includes marginal gingiva. They
nodes, mainly submaxillary group. have flattened keratinocytes.
l The terminal border or edge of the gingiva surrounds
Nerve supply of the gingiva
It is derived from the fibres arising from the nerves the neck of the teeth in a collar-like fashion.
l Usually 1-mm width of marginal gingiva forms the soft
in the periodontal ligament and from the labial buc-
cal and palatal nerves. tissue wall of the gingival sulcus.
l The free gingival margin is located on the enamel sur-
Q.8. Saliva. face approximately 1.5–2 mm coronal to the CEJ and is
Ans. demarcated from the attached gingiva by a shallow de-
pression called the free gingival groove.
Saliva provides its antimicrobial defence mechanism be-
sides catering to various other functions such as lubrication, Q.12. Interdental papilla.
physical protection, cleansing, buffering and maintenance Ans.
of tooth integrity.
The functions of saliva are as follows: i. Interdental papilla usually occupies the gingival embra-
i. Protection: It provides washing action that clears non- sures. There are three parts of interdental gingiva –
adherent, potentially harmful substances in oral cavity. facial papilla, lingual papilla and col, which is a valley-
ii. Digestion: It contains two digestive enzymes, amylase like depression that connects the facial and lingual
that breaks down complex carbohydrates, and lipase papilla.
that digests fats. ii. The lateral borders and tips of interdental papilla are
iii. Antibacterial properties: It plays a major bacteriostatic formed by the continuation of marginal gingiva and the
role in the oral cavity. intervening portion by attached gingiva.
iii. In the presence of diastema, the interdental papilla will
Q.9. Long junctional epithelium. be absent.
Ans.
Q.13. Microscopic features of healthy gingiva.
l Junctional epithelium consists of a collar-like band of
Ans.
stratified squamous nonkeratinizing epithelium.
l It has three or four layers but increases later. l The gingival epithelium comprises the epithelial tissue
l Junctional epithelium is formed by confluence of OE that covers the external surface of gingiva as well as the
and reduced enamel epithelium during tooth eruption. epithelial lining, the gingival sulcus and the junctional
l Cell layers not juxtaposed to the tooth exhibit numerous epithelium.
free ribosomes and prominent membrane bound struc- l The principle cell type of the gingival epithelium is ke-
tures such as Golgi complexes, cytoplasmic vacuoles, ratinocyte. In addition to the keratin producing cells,
presumably phagocytic, lysosome-like bodies are present. which comprise about 90% of the total cell population,
l Different keratin polypeptides of junctional epithelium the epithelium contains nonkeratinocytes like:
have a particular histochemical pattern. i. Melanocytes
ii. Langerhans cells
Q.10. Functions of gingival fibre system.
iii. Merkel cells
Ans. iv. Inflammatory cells
Section | I  Topic-Wise Solved Questions of Previous Years 755

l They are also known as clear cells, since in histo- l Encroachment of embrasure and interdental spaces will
logical sections in the zone around the nuclei appears interfere with normal escape of food debris and oral
lighter than that in surrounding keratin-producing hygiene measures.
cells. l Embrasure are of three types:

i. Interproximal embrasure
Q.14. Gingival crevicular fluid.
ii. Embrasure with no gingival recession
Ans. iii. Larger spaces with exposed root surfaces
l In gingiva there is a fluid transudate that flows from the Q.18. Orogranulocytes.
site of the seal presumably as a mechanical factor in
Ans.
minimizing bacterial accumulation.
l This fluid contains a variety of macromolecular compo- l Living PMNs in the cell saliva are called orogranulo-
nents that are derived from the serum and the interstitia cyte.
of gingiva. l In a normal individual, it has been estimated that 30,000

l As inflammation of the gingiva increases, the transudate neutrophil per minute enter the oral cavity via the gingi-
changes to an inflammatory exudate containing higher val sulcus through the junctional epithelium surround-
level of serum-derived molecules, vascular-derived cel- ing the teeth.
lular components of inflammation and locally derived l This flow of neutrophils is required for periodontal
molecules from the gingival tissue. health and protection of caries.
l Defects in neutrophil function and chemotaxis are
Q.15. Langerhans cells.
associated with early-onset periodontal disease in
Ans. children.
l Oral leukocyte migratory rate index has been sug-
l Langerhans cells are dendritic cells located among kera-
gested as a reproducible index of periodontal health
tinocytes at suprabasal levels containing large granules
and allows an objective assessment of periodontal
called Birbeck granules.
health.
l They are derived from cellular differentiation of mono-
l Orogranulocytic migratory rate has been shown to be a
cytes.
nonsubjective laboratory index for inflammatory peri-
l The Langerhans cells are believed to play a role in the
odontal diseases.
defence mechanism of oral mucosa.
l In the presence of infection, Langerhans cell will take Q.19. Attached gingiva.
up and process microbial antigen to become fully func-
Ans.
tional antigen presenting cells, thereby inhibiting fur-
ther antigen penetration of the tissues. [Ref SE Q.4]
Q.16. Palatogingival groove. Q.20. Mucogingival junction.
Ans. Ans.
l Palatogingival groove found in primarily on maxillary l The junction between attached gingiva and alveolar
incisors is observed in 8.5% of individuals. mucosa is called mucogingival line or junction.
l It is associated with increased plaque accumulation, l Its importance in periodontal disease is that its position

clinical attachment and bone loss. remains consistent.


l These are developmental grooves which sometimes ap-
Q.21. Gingival stippling.
pear in maxillary lateral incisors.
l Presents accessibility problems. Ans.
Q.17. Define and classify embrasures. i. The gingiva presents a textured surface similar to an
orange peel and is referred to as being stippled.
Ans.
ii. It is best viewed by drying the gingiva, and the attached
l Establishment of good embrasure form will either pre- gingiva is stippled.
vent debris retention or allow the lip, cheek and tongue iii. The stippling is a form of adaptive specialization or
action to render the area clean. reinforcement for function.
l At times, the interproximal embrasure is obliterated by iv. Reduction or loss of stippling is a common sign of gin-
bulky solder joints or by crown contours, the resultant gival disease.
pressure brings about a catering of the interdental pa- v. When gingival health is restored after treatment, the
pilla which allows entrapment of plaque. stippled appearance returns.
756 Quick Review Series for BDS 4th Year, Vol 1

Q.22. Biological width. Q.25. Stillman’s clefts.


Ans. Ans.
l Ingber et al. defined biological width as ‘the minimal l Gingival contour is affected by Stillman’s cleft.
width at the gingival sulcus required to maintain a nor- l It is a narrow, triangular shaped gingival recession
mal gingival attachment’. which exposes cementum as the recession progresses
l Biological width is commonly represented as the sum of apically.
epithelial and connective tissue measurements, which is l According to Carranza, these are defined as apostrophe-

approximately 2.04 mm. shaped indentations extending from and into gingival
l Biological width acts as a clinical guide for the place- margin for varying distances on facial surface.
ment of restoration margins, since a minimum of 3 mm
Q.26. McCall’s festoons.
should exist from restorative margin to the alveolar
bone, allowing for 2 mm of the biological width space Ans.
and 1 mm of sulcus depth.
l These are noted on oral examination of gingiva as rolled
l Periodontium will react by crestal bone resorption and
thickened margins of gingiva seen near canine when
apical migration of dentogingival complex to re-establish
recession progresses till the mucogingival junction.
the inherent biological width.
l These are life preserver–shaped enlargements of mar-
l Violating the biological width by placing subgingival
ginal gingiva that occur most frequently in the canine
margin causes physical trauma, subgingivally placed
and premolar areas on the facial surface.
restoration act as a plaque trap.
l McCall’s festoons and Stillman’s cleft were earlier con-

Q.23. Transgingival probing. sidered to be due to trauma from occlusion but presently
they are considered to represent the inflammatory
Ans. changes seen in marginal gingiva.
l Transgingival probing or transcircular probing is the Q.27. Gingival col.
sounding technique for plotting the morphological out-
line of alveolar bone. Ans.
l This procedure is performed under local anaesthesia,
[Same as SN Q.1]
utilizing the periodontal probe pushed through the gin-
gival tissue as a sounding devise to determine the shape Q.28. Free gingiva.
of the infrabony defects. Ans.
l Osseous craters are found to make up to two-thirds of

all mandibular defects. It can also be diagnosed by [Same as SN Q.2]


transgingival probing. Q.29. Gingival fibres and their importance.
l This is also used to determine biological width.

l The factors affecting transgingival probing are as Ans.


follows: [Same as SN Q.3]
i. Size of the probe
ii. Angulation of the probe Q.30. Role of mast cells in gingival
iii. Probing force Ans.
iv. Inflammatory status of the tissue
[Same as SN Q.6]
Q.24. Dentogingival unit.
Q.31. Role of saliva as a defence mechanism of gingiva.
Ans.
Ans.
l The attachment of junctional epithelium to the tooth is
reinforced by gingival fibres, which brace the marginal [Same as SN Q.8]
gingiva against the tooth surface. Q.32. Importance of attached gingiva.
l Hence, the junctional epithelium and gingiva fibres are

together considered as functional unit named as dento- Ans.


gingival unit. [Same as SN Q.19]
Section | I  Topic-Wise Solved Questions of Previous Years 757

Topic 2
Tooth-Supporting Structures (Periodontal
Ligament, Alveolar Bone, Cementum)
COMMONLY ASKED QUESTIONS
LONG ESSAYS:
1 . Describe the structure and functions of periodontal ligament.
2. Describe the structure of cementum and add a note on functions of cementum.
3.
Describe in detail the role of alveolar bone in health and periodontal disease.
4.
Discuss in detail about the anatomy, histology and functions of periodontal ligament. [Same as LE Q.1]
5.
Define periodontal ligament. Describe the microscopic and macroscopic features of periodontal ligament. [Same as LE Q.1]
6.
Enumerate the components of periodontium. Describe the structure of periodontal ligament. [Same as LE Q.1]
7.
Define cementum. Describe structure, composition and clinical significance of cementum. [Same as LE Q.2]
8.
Describe the normal structure and function of cementum. Discuss the histological importance of cementum in
periodontal therapy. [Same as LE Q.2]
9. Define cementum. Classify and give its microscopic structure. Add a note on functions of cementum. [Same as LE Q.2]

SHORT ESSAYS:
1. Alveolar bone. [Ref LE Q.3]
2. Functions of periodontal ligament. [Ref LE Q.1]
3. Enumerate the various groups of principal fibre bundles. What are the periodontal ligament changes in trauma
from occlusion? [Ref LE Q.1]
4. Oxytalan fibres.
5. Composition of cementum.
6. Cementoenamel junction.
7. Fenestration and dehiscence.
8. Describe the mechanisms by which periodontal ligament resists occlusal forces.
9. Hypercementosis.
10. Write briefly about alveolar bone. [Same as SE Q.1]
11. Define periodontal ligament and describe its functions. [Same as SE Q.2]
12. Cementum and its biological importance. [Same as SE Q.5]
13. Cementoenamel junction relationships. [Same as SE Q.6]

SHORT NOTES:
1. Name physical functions of periodontal ligament.
2. Oxytalan fibres. [Ref SE Q.4]
3. Sharpey’s fibres.
4. Functions of cementum. [Ref LE Q.2]
5. Types of cementoenamel junction. [Ref SE Q.6]
6. Classification of cementum by Schroeder and Page. [Ref LE Q.2]
7. Causes of hypercementosis. [Ref SE Q.9]
8. Blood supply to periodontal ligament.
9. Principal fibres of periodontal ligament. [Ref LE Q.1]
10. Fenestration and dehiscence. [Ref SE Q.7]
11. Define periodontium. What does it comprise?
12. Define intermediate plexus.
13. Lamina dura.
14. Acellular cementum.
15. Cellular cementum.
16. Periodontal fibres. [Same as SN Q.9]
17. Histology of periodontal fibres. [Same as SN Q.9]
758 Quick Review Series for BDS 4th Year, Vol 1

SOLVED ANSWERS
LONG ESSAYS:
Q.1. Describe the structure and functions of periodontal l These are reconstructed even after destruction of the
ligament. alveolar bone resulting from periodontal disease.
Ans. l These fibres may be considered as belonging to

the gingiva because they do not have osseous


The periodontal ligament (PDL) is a complex vascular and
attachment.
highly cellular connective tissue that surrounds the tooth
l They maintain the teeth in the arch.
root and connects it to the inner wall of the alveolar bone.
ii. Alveolar crest group
Structure of periodontal ligament
l Alveolar crest fibres extend obliquely from the ce-
l It is continuous with the connective tissue of the
mentum just beneath the junctional epithelium to the
gingiva and communicates with the marrow spaces
alveolar crest.
through vascular channels in the bone.
l Fibres also run from the cementum over the alveo-
l Average width of the PDL is 0.2 mm.
lar crest and to the fibrous layer of the periosteum
l The periodontal space is diminished around teeth
covering the alveolar bone.
that are not in function or in unerupted teeth, but it is
l The alveolar crest fibres prevent the extrusion of
increased in teeth subjected to hyperfunction.
the tooth and resist lateral tooth movements.
l The PDL space has the shape of an hourglass.
l The incision of these fibres during periodontal
Composition
surgery does not increase tooth mobility unless
The PDL consists of the following:
significant attachment loss has occurred.
i. Periodontal fibres
l They secure teeth in the socket by resisting lateral
a. Principal fibres
forces applied to the tooth.
b. Secondary fibres
iii. Horizontal group
ii. Cellular elements
l Horizontal fibres extend at right angles to the long axis
iii. Ground substance
of the tooth from the cementum to the alveolar bone.
[SE Q.3] l They prevent lateral tooth movement.
{Periodontal fibres iv. Oblique group
(SE Q.3 and SN Q.9) l Oblique fibres extend from cementum obliquely to

the bone in a coronal direction.


{(Principal fibres l They are the most numerous fibres in the PDL.
l The most important elements of the PDL are the They are believed to be primarily responsible for
principal fibres. These are collagenous and arranged absorbing the chewing forces on the tooth and are
in bundles and follow a wavy course when viewed hence the main support of the tooth.
in longitudinal section. l They resist apically directed masticatory forces.
l The terminal portions of the principal fibres that are l They do not occur on incompletely formed roots.
inserted into cementum and bone are termed as v. Apical fibres
Sharpey’s fibres. They radiate from cementum in a rather irregular man-
l The principal fibre bundles consist of individual ner to the bone at the apical region of the socket. They
fibres that form a continuous anastomosing network prevent tooth tipping, and resist forces of luxation.
between tooth and bone. vi. Inter-radicular group
l These fibres are remodelled by the PDL cells to l The inter-radicular fibres fan out from the cemen-
adapt to the physiological needs and in response to tum to the tooth in the furcation areas of multi-
different stimuli. rooted teeth.
l These fibres are arranged in six groups that develop se- l They resist tipping of tooth, forces of luxation and
quentially in the developing root. They are as follows: rotation.}
i. Trans-septal group
ii. Alveolar crest group Secondary fibres
iii. Horizontal group l Two immature forms are found: oxytalan and elua-
iv. Oblique group nin.
v. Apical group l The so-called oxytalan fibres run parallel to the root
vi. Inter-radicular group)} surface in a vertical direction and bend to attach to
the cementum in the cervical third of the root. They
[SE Q.3] are thought to regulate vascular flow.
{i. Trans-septal group l In addition to these fibre types, small collagen fibres
l Trans-septal fibres extend interproximally associated with the larger principal collagen fibres
over the alveolar bone crest and are embed- have been described. These fibres run in all directions,
ded in the cementum of adjacent teeth. forming a plexus called the indifferent fibre plexus.
Section | I  Topic-Wise Solved Questions of Previous Years 759

Cellular elements of periodontal ligament ii. Transmission of occlusal forces to the bone.

Four types of cells have been identified in the PDL: iii. Attachment of the teeth to the bone.
i. Connective tissue cells iv. Maintenance of the gingival tissues in their proper
ii. Epithelial rest cells relationship to the teeth.
iii. Immune system cells v. Resistance to the impact of occlusal forces (shock
iv. Cells associated with neurovascular elements absorption).}

Connective tissue cells i. Provision of a soft tissue ‘casing’


l The connective tissue cells include fibroblasts, In order to protect the vessels and nerves from injury
cementoblasts and osteoblasts. by mechanical forces, the PDL provides a soft tissue
l The fibroblasts are the most common cells in casing around them.
the PDL and appear as ovoid or elongated cells ii. Transmission of occlusal forces to bone
oriented along the principal fibres, exhibiting l When an axial force is applied to a tooth, a tendency

pseudopodia-like processes. towards displacement of the root into the alveolus


l These cells synthesize collagen and possess the occurs. The oblique fibres alter their wavy, untensed
capacity to phagocytose ‘old’ collagen fibres pattern and by assuming their full length, they sus-
and degrade them by enzyme hydrolysis. Thus, tain the major part of the axial force.
collagen turnover appears to be regulated by l When a horizontal or tipping force is applied, two

fibroblasts. phases of tooth movement occur. The first is within


l Osteoblasts and cementoblasts, as well as os- the confines of the PDL, and the second produces a
teoclasts and odontoclasts, are also seen in the displacement of the facial and lingual bony plates.
cemental and osseous surfaces of the PDL. l The apical portion of the root moves in a direction

Epithelial rests of Malassez opposite to the coronal portion. In areas of tension,


l They form a lattice work in the PDL and appear the principal fibre bundles are taut rather than wavy.
as either isolated clusters of cells or interlacing In areas of pressure, the fibres are compressed, the
strands depending on the plane in which the tooth is displaced, and a corresponding distortion of
microscopic section is cut. bone exists in the direction of root movements.
l The epithelial rests are considered as remnants In single-rooted teeth, the axis of rotation is located
of Hertwig’s root sheath. They are distributed in the area between the apical third and the middle
close to the cementum throughout the PDL of third of the root. The root apex and the coronal half
most teeth and are most numerous in the apical of the clinical root have been suggested as other
and cervical areas. locations of the axis of rotation. The PDL, which
l They diminish in number with age by degener- has an hourglass shape, is narrowest in the region of
ating and disappearing or by undergoing calci- the axis of rotation.
fication to become cementicles. l In multirooted teeth, the axis of rotation is located

Ground substance in the bone between the roots.


l The PDL also contains a large proportion of ground iii. Resistance to impact of occlusal forces (shock absorp-
substance, filling the spaces between fibres and cells. tion)
l It consists of two main components: Two theories pertaining to the mechanism of tooth sup-
a. Glycosaminoglycans, such as hyaluronic acid and port have been considered:
proteoglycans (i) Tensional theory

b. Glycoproteins, such as fibronectin and laminin (ii) Viscoelastic system theory
l Ground substance also has high water content (70%). a. Tensional theory
l The cell surface proteoglycans participate in several l This theory states that the principal fibres of

biological functions, including cell adhesion, cell– the PDL are the major factors in supporting
cell and cell–matrix interactions, binding to various the tooth and transmitting forces to the bone.
growth factors as coreceptors, and cell repair. l When a force is applied to the crown, the

principal fibres first unfold and straighten


[SE Q.2]
and then transmit forces to the alveolar
{Functions of periodontal ligament bone, causing an elastic deformation of the
The functions of the PDL are categorized into: bony socket.
i. Physical l Finally, when the alveolar bone has reached its
ii. Formative and remodelling limit, the load is transmitted to the basal bone.
iii. Nutritional and sensory b. Viscoelastic system theory
Physical functions l This theory states that the displacement of
The physical functions of the PDL are as follows: the tooth is largely controlled by fluid
i. Provision of a soft tissue ‘casing’ to protect the ves- movements, with fibres having only a sec-
sels and nerves from injury by mechanical forces. ondary role.
760 Quick Review Series for BDS 4th Year, Vol 1

l When forces are transmitted to the tooth, the c. Coiled Meissner’s corpuscles, also mecha-
extracellular fluid passes from the PDL into noreceptors, found in the midroot region.
the marrow spaces of bone through foram- d. Spindle-like pressure and vibration end-
ina in the cribriform plate. ings, which are surrounded by a fibrous
l After depletion of tissue fluids, the fibre bun- capsule and located mainly in the apex.}
dles absorb the slack and tighten. This leads to
Q.2. Describe the structure of cementum and add a note
a blood vessel stenosis. Arterial back pressure
on functions of cementum.
causes ballooning of the vessels and passage
of the blood ultrafiltrates into the tissues, Ans.
thereby replenishing the tissue fluids.
l Cementum is a calcified avascular mesenchymal tissue
that forms the outer covering of the anatomic root.
[SE Q.2] l The two main types of cementum are as follows:

a. Acellular cementum/primary cementum


{Formative and remodelling function b. Cellular cementum/secondary cementum
l PDL and alveolar bone cells are exposed to physical
forces in response to mastication, parafunction, speech Both the types consist of a calcified interfibrillar matrix
and orthodontic tooth movement. and collagen fibrils.
l Cells of the PDL participate in the formation and

resorption of cementum and bone, which occur in The differences between acellular and cellular cementum
physiological tooth movement, in the accommodation are as follows:
of the periodontium to occlusal forces and in the repair Acellular cementum Cellular cementum
of injuries. (primary cementum) (secondary cementum)
l The PDL is constantly undergoing remodelling. Old
i. It forms during root forma- i. It forms after the eruption
cells and fibres are broken down and replaced by new tion before the tooth of the tooth and in re-
ones, and mitotic activity can be observed in the fibro- reaches occlusion. sponse to functional
blasts and endothelial cells. demands.
l Fibroblasts form the collagen fibres, and the residual ii. It does not contain any ii. It contains cementocytes
mesenchymal cells develop into osteoblasts and ce- cells. in lacunae.
mentoblasts. iii. Sharpey’s fibres make up iii. Sharpey’s fibres occupy
l Therefore, the rate of formation, and the differentia- most of it. smaller portion and are
tion of osteoblasts, cementoblasts and fibroblasts af- parallel to the root surface.
fect the rate of formation of collagen, cementum and iv. It is seen at the coronal iv. It is seen at a more apical
bone. portion of root. portion of root.
Nutritional and sensory functions v. Formation is slow. v. Formation is more rapid.
l The PDL supplies nutrients to the cementum, bone and

gingiva by way of blood vessels and also provides lym- vi. Arrangement of collagen vi. Collagen fibres are irregu-
fibres are more organized. larly arranged.
phatic drainage. In relation to other ligaments and ten-
dons, the PDL is a highly vascularized tissue. This rela-
tively high blood vessel content may provide Classification
hydrodynamic damping to applied forces as well as
high perfusion rates to the PDL. {SN Q.6}
l The PDL is abundantly supplied with sensory nerve fi-
Schroeder has classified cementum as follows:
bres capable of transmitting tactile, pressure and pain a. Acellular afibrillar cementum (AAC)
sensations by the trigeminal pathways. Nerve bundles l It contains neither cells nor extrinsic or intrin-
pass into the PDL from the periapical area and through sic collagen fibres, except for the mineralized
channels from the alveolar bone that follow the course ground substance.
of the blood vessels. l It is a product of cementoblasts and is found
l The bundles divide into single myelinated fibres, which
almost exclusively on the enamel near the ce-
ultimately lose their myelin sheaths and end in one of mentoenamel junction with a thickness of 1–15
the four types of neural termination: microns.
a. Free endings, which have a tree-like configuration b. Acellular extrinsic fibre cementum (AEFC)
and carry pain sensation. l It is composed primarily of Sharpey’s fibres of
b. Ruffini-like mechanoreceptors, located primarily in PDL but does not contain cementocytes.
the apical area.
Section | I  Topic-Wise Solved Questions of Previous Years 761

l It is a product of cementoblasts and fibroblasts. {SN Q.4}


l Developmentally, it occupies the coronal one-
half of the root surface. Its thickness is be- Functions
l Primary function of cementum is to provide an-
tween 30 and 230 microns.
c. Cellular mixed stratified cementum (CMSC) chorage to the tooth in its alveolus. This is
l It harbours both intrinsic and extrinsic Sharpey’s
achieved through the collagen fibre bundles of the
fibres and may contain cells. PDL, whose ends are embedded in cementum.
l Cementum also plays an important role in main-
l In humans, it is seen in the apical third of the

roots, apices and furcation areas. taining occlusal relationships whenever the inci-
l Its thickness varies from 100 to 1000 microns.
sal and occlusal surfaces are abraded due to attri-
d. Cellular intrinsic fibre cementum (CIFC) tion, the tooth supraerupts in order to compensate
l It contains only intrinsic fibres secreted by
for the loss, and deposition of new cementum oc-
cementoblasts and not by the PDL fibroblasts. curs at the apical root area.
l It also serves as a major reparative tissue for root
l In humans, it fills the resorption lacunae.

e. Intermediate cementum or the hyaline layer of surfaces, thereby maintaining integrity of the root
Hopewell–Smith surface.
l It is a poorly defined zone near cementoenamel

junction.
l It appears to contain cellular remnants of Q.3. Describe in detail the role of alveolar bone in health
Hertwig’s sheath embedded in calcified ground and periodontal disease.
substance. Ans.
l The significance of this layer is that it contains

enamel like proteins, which help in the attach- Alveolar bone in health
ment of cementum to dentine.
[SE Q.1]
{ l Alveolar bone is that portion of the maxilla and man-
dible that forms and supports the tooth sockets.
Composition l The alveolar process is the thickened ridge of bone

l The cementum is composed of both inorganic (46%) that contains the tooth sockets on bones that bear
and organic matter (54%). teeth.
l The inorganic content of cementum is less than l Alveolar bone proper is a thin layer of bone lining

that of bone (65%), enamel (97%) and dentine the roots of the teeth and provides attachment to
(70%). principal fibres of PDL. Radiographically, this bun-
l Inorganic salts are present in the form of hydroxy- dle bone appears as a thin radiopaque line surround-
apatite crystals. They have the highest fluoride ing the roots of teeth, called the lamina dura.
content. l The alveolar bone is perforated with numerous open-

l The organic matrix is chiefly composed of 90% ings by branches of intra-alveolar nerves and blood
type I collagen, 5% type III collagen and 5% non- vessels and therefore called the cribriform plate.
collagenous proteins like enamel proteins, adhesion l Mature bones, whether compact or cancellous, con-

molecules like tenascin and fibronectin, glycosami- sist of layers or lamellae. Three different types of
noglycans like chondroitin sulphate, dermatan sul- lamellae are present: circumferential lamellae, con-
phate and heparan sulphate, which constitute the centric lamellae and interstitial lamellae.
remaining organic matrix. l Periosteum: A connective tissue membrane
l Two main sources of collagen fibres in the cemen- surrounding each compact bone is known as peri-
tum are: osteum.
i. Sharpey’s (extrinsic) fibres, which are embedded l Endosteum: The inner surface of both compact and

portion of the principal fibres of PDL and are cancellous bone is covered by endosteum, a single
formed by the fibroblasts. layer of bone cells that separates the bone surface
ii. Fibres belonging to the cementum matrix (intrin- from the bone marrow within.
sic) and are produced by cementoblasts. l It has two basic constituents:

l The noncollagenous components of the interfibrillar i. Cells like osteoblasts, osteoclasts and osteocytes
ground substance are proteoglycans, glycoproteins ii. Extracellular matrix made up of 65% inorganic
and phosphoproteins. and 35% organic content
762 Quick Review Series for BDS 4th Year, Vol 1

Inorganic substance is composed of calcium, phos-


l Q.6. Enumerate the components of periodontium.
phate along with some trace elements. Organic matrix Describe the structure of periodontal ligament.
consists of 90% type I collagen with small amounts
Ans.
of noncollagenous proteins such as osteonectin, os-
teocalcin, bone morphogenetic proteins, etc. [Same as LE Q.1]
l Osteoclasts occupy shallow, hollowed-out depres-
Q.7. Define cementum. Describe structure, composition
sions on the bone surface created by themselves,
and clinical significance of cementum.
called the Howship’s lacunae.
l Parathyroid hormone (PTH) functions as a major Ans.
mediator of bone remodelling and as an essential
[Same as LE Q.2]
regulator of calcium homeostasis, producing several
distinct and independent effects on the bone remod- Q.8. Describe the normal structure and function of ce-
elling process, resulting in both bone formation and mentum. Discuss the histological importance of cemen-
bone resorption, i.e. anabolic and catabolic activity.} tum in periodontal therapy.
Alveolar bone in disease
Ans.
Fenestrations and dehiscence are seen during disease in
relation to alveolar bone proper: [Same as LE Q.2]
l Fenestrations are isolated areas in which the root
Q.9. Define cementum. Classify and give its microscopic
surface is covered only by periosteum and gingiva
structure. Add a note on functions of cementum.
but the marginal bone remains intact. The root is
denuded of the bone. Ans.
l When marginal bone is also involved, the defect is
[Same as LE Q.2]
called dehiscence.
l The reasons could be prominent root contours,

tooth malposition and labial protrusion of the root SHORT ESSAYS:


combined with a thin bony plate.
Q.1. Alveolar bone.
l Bone resorption occurs by three processes: (i) de-

calcification, (ii) degradation of matrix and (iii) Ans.


transport of soluble factors to the extracellular fluid.
[Ref LE Q.3]
l Periodontal bone loss may also occur in general-

ized skeletal disturbances (e.g. hyperthyroidism, Q.2. Functions of periodontal ligament.


leukaemia, Langerhans cell histiocytosis) by
Ans.
mechanisms that may be totally unrelated to the
usual periodontal problems. [Ref LE Q.1]
Bone destruction patterns in periodontal diseases are as
Q.3. Enumerate the various groups of principal fibre
follows:
bundles. What are the periodontal ligament changes in
l Horizontal bone loss
trauma from occlusion?
l Bone deformities (osseous defects)

l Vertical or angular defects Ans.


l Osseous craters
[Ref LE Q.1]
l Bulbous bone contours

l Reversed architecture The PDL changes in trauma from occlusion are as follows:
l Ledges l When occlusal forces exceed the adaptive capacity of
l Furcation involvement the tissues, tissue injury results, which is known as
Q.4. Discuss in detail about the anatomy, histology and trauma from occlusion.
l The ligament is widened at the expense of the bone,
functions of periodontal ligament.
resulting in angular bone defects without periodontal
Ans.
pockets and the tooth becomes loose.
[Same as LE Q.1] l Severe tension causes widening of the PDL, thrombosis,

Q.5. Define periodontal ligament. Describe the micro- haemorrhage, tearing of the PDL and resorption of the
scopic and macroscopic features of periodontal ligament. alveolar bone.
l The bone is resorbed from viable PDL adjacent to ne-
Ans.
crotic areas and from marrow spaces, a process called
[Same as LE Q.1] undermining resorption.
Section | I  Topic-Wise Solved Questions of Previous Years 763

Q.4. Oxytalan fibres. l Whenever the incisal and occlusal surfaces are abraded
due to attrition, the tooth supraerupts in order to com-
Ans.
pensate for the loss, and deposition of new cementum
occurs at the apical root area.
{SN Q.2}
Q.6. Cementoenamel junction.
l The PDL contains two immature forms of fibres,
Ans.
namely oxytalan and eluanin fibres.
l In addition to the principal fibre groups, PDL con-

tains other well-formed fibre bundles that run at right {SN Q.5}
angles to the regular fibre bundles.
l The oxytalan fibres run parallel to the root surface in (The relation between cementum and enamel at cervical
a vertical direction and bend to attach to the cemen- region of teeth is variable.
tum in the cervical third of the root. There are three types of cementoenamel junction (CEJ):
l These fibres are associated with blood vessels and i. In about 60% of teeth, cementum overlaps cervi-
nerves of the PDL and are thought to regulate vascu- cal end of enamel for a short distance. This oc-
lar flow. curs when the enamel epithelium degenerates at
l An elastic meshwork has been described in the PDL its cervical termination, permitting connective
as being composed of many elastin lamellae with tissue to come in direct contact with the enamel
peripheral oxytalan fibres and eluanin fibres. surface.
l Oxytalan fibres have been shown to develop de novo ii. In approximately 30% of all teeth, the cementum
in the regenerated PDL. and enamel meet as an edge-to-edge butt joint,
l It was suggested that they provide elastic properties forming a distinct delineation at the cervical
to PDL. There are also reticulate fibres, which are margin.
fine and immature collagen fibres with a lattice-like iii. 10% of all teeth have a gap between the cemen-
arrangement. tum and the enamel, exposing root which can
lead to sensitivity at the site. This occurs when
enamel epithelium in the cervical portion of the
root is delayed in its separation from dentine.
Q.5. Composition of cementum. Because of exposed dentine in this case, gingival
Ans. recession may result in accentuate sensitivity.

Cementum is a calcified avascular mesenchymal tissue that


forms the outer covering of the anatomic root. Q.7. Fenestration and dehiscence.
The composition of cementum is as follows:
l Cementum comprises 45%–50% inorganic salts in the
Ans.
form of hydroxyapatite crystals.
l The remaining matrix is made up of collagen fibres and

noncollagenous matrix protein. Type I collagen is the {SN Q.10}


predominant form and constitutes 90% of its organic l Fenestrations and dehiscence are seen during disease
components. in relation to alveolar bone proper.
l The noncollagenous components of the interfibrillar
l Fenestrations are isolated areas in which the root
ground substance are proteoglycans, glycoproteins and surface is denuded of bone and is covered only by
phosphoproteins. periosteum and gingiva but the marginal bone re-
l The inorganic content of cementum is less than that of
mains intact.
bone, enamel and dentine. It has the highest fluoride l When denuded area includes marginal bone, the de-
content. fect is called dehiscence.
Biological importance of cementum is as follows: l These defects occur in approximately 20% of the
l A gap between the cementum and enamel, exposing
teeth; more commonly seen on facial bone and ante-
root, can lead to sensitivity at the site. This occurs when rior teeth, and are frequently bilateral.
enamel epithelium in the cervical portion of the root is l The causes of these defects are still not clear.
delayed in its separation from dentine. l The reasons could be prominent root contours, tooth
l If the cementum and enamel fail to meet, gingival reces-
malposition and labial protrusion of the root com-
sion may result in accentuated sensitivity because of bined with a thin bony plate.
exposed dentine.
764 Quick Review Series for BDS 4th Year, Vol 1

l Bone resorption occurs by three processes:


l It is largely an age-related phenomenon, and it may be
i. Decalcification
localized to one tooth or affects the entire dentition.
ii. Degradation of matrix
l It is not possible to distinguish between physiologi-
iii. Transport of soluble factors to the extracellular fluid
cal thickening of cementum that occurs with age and
l Periodontal bone loss may also occur in generalized
hypercementosis.
skeletal disturbances, e.g. hyperthyroidism, leukaemia,
l It occurs as a generalized thickening of root cemen-
Langerhans cell histiocytosis by mechanisms that may
tum with a nodular enlargement at the apical one-
be totally unrelated to the usual periodontal problems.
third or as spike-like excrescences due to fusion of
Q.8. Describe the mechanism by which periodontal liga- cementicles to the roots or due to calcification of
ment resists occlusal forces. PDL fibres which are inserted into cementum.
Ans.
Pertaining to the mechanism of tooth support or the shock Aetiology
absorption, three theories have been put forward. When l Localized hypercementosis is seen in teeth without

force is applied to the crown, the principal fibres unfold and antagonists to keep pace with active eruption or due
straighten and transmit force to the underling bone. The to low-grade periapical irritation as a result of pulpal
bone undergoes elastic deformity till the limit is reached disease. It is believed to be a compensatory mecha-
and then the load is transmitted to the basal bone. nism to counteract the destruction of fibrous attach-
Tensional theory ment of the tooth. It is also attributed to the excessive
l This theory states that the principal fibres of the PDL occlusal and orthodontic forces.
are the major factors in supporting the tooth and l Generalized hypercementosis may be hereditary or

transmitting forces to the underlying bone. observed in patients with Paget disease. Other systemic
l When a force is applied to the crown, the principal disturbances that may lead to or may be associated
fibres first unfold and straighten and then transmit with hypercementosis include acromegaly, arthritis,
forces to the alveolar bone, causing an elastic defor- calcinosis, rheumatic fever and thyroid goitre.
mation of the bony socket. Radiographic features
l When the alveolar bone has reached its limit, the l The radiolucent shadow of the PDL and the radi-

load is transmitted to the basal bone. opaque lamina dura are always seen on the outer
Viscoelastic system theory border of an area of hypercementosis enveloping it as
l This theory states that the displacement of the tooth it would in normal cementum.
is largely controlled by fluid movements, with fibres l On the other hand, periapical cemental dysplasia,

having only a secondary role. condensing osteitis and focal periapical osteopetrosis
l When forces are transmitted to the tooth, the extracel- may be differentiated from hypercementosis by the
lular fluid passes from the PDL into the marrow spaces fact that all these entities are located outside the
of bone through foramina in the cribriform plate. shadow of the PDL and lamina dura.
l After depletion of tissue fluids, the fibre bundles ab- Clinical significance
sorb the slack and tighten. This leads to a blood ves- l Hypercementosis itself does not require treatment.

sel stenosis. Arterial back pressure causes ballooning l It could pose a problem if an affected tooth requires

of the vessels and passage of the blood ultrafiltrates extraction. In multirooted tooth, sectioning of the
into the tissues, thereby replenishing the tissue fluids. tooth may be required before extraction.
Thixotropic theory Q.10. Write briefly about alveolar bone.
l According to this theory, the PDL has the rheological
Ans.
behaviour of a thixotropic gel.
l The shock absorption concept can be also explained [Same as SE Q.1]
by this theory. Q.11. Define periodontal ligament and describe its
Q.9. Hypercementosis. functions.

Ans. Ans.
[Same as SE Q.2]
{SN Q.7} Q.12. Cementum and its biological importance.
l The term ‘hypercementosis’ or ‘cemental hyperpla- Ans.
sia’ refers to a prominent thickening of the cementum.
[Same as SE Q.5]
Section | I  Topic-Wise Solved Questions of Previous Years 765

Q.13. Cementoenamel junction relationships. Q.7. Causes of hypercementosis.


Ans. Ans.
[Same as SE Q.6] [Ref SE Q.9]
Q.8. Blood supply to periodontal ligament.
SHORT NOTES: Ans.
Q.1. Name physical functions of periodontal ligament. l The main blood supply to PDL is from superior and
inferior alveolar arteries.
Ans.
l The blood supply of PDL can be divided into three

The physical functions of PDL are as follows: groups:


l Protection of the underlying vessels and nerves by pro- i. Apical group of arteries: From vessels supplying
viding a soft tissue casing the pulp
l Transmission of occlusal forces to the bone ii. Alveolar group of arteries: Also called perforating
l Attachment of teeth to the bone arteries
l Maintenance of gingival tissues in their proper relation- iii. Gingival group of arteries: From gingival blood
ship to the teeth supply
l Resistance to impact of occlusal forces, i.e. shock
Q.9. Principal fibres of periodontal ligament.
absorption
Ans.
Q.2. Oxytalan fibres.
[Ref LE Q.1]
Ans.
Q.10. Fenestration and dehiscence.
[Ref SE Q.4] Ans.
Q.3. Sharpey’s fibres. [Ref SE Q.7]
Ans. Q.11. Define periodontium. What does it comprise?
l The terminal portions of the principal fibres of peri- Ans.
odontal ligamentum that are inserted and embedded into
l PDL is a specialized connective tissue that
cementum and bone are called Sharpey’s fibres.
forms the interface between the tooth and the alveolar
l They constitute a considerable proportion of the bulk of
bone.
cementum, and are composed mainly of collagen type I.
l It comprises the following components:
l Type III collagen appears to coat the type I collagen of

Sharpey’s fibres.
l ‘Bundle bone’ is the term given to the bone adjacent
Connective
to the PDL that contains a great number of Sharpey’s
Cellular Extracellular tissue
fibres.
components components components
l Some Sharpey’s fibres are completely calcified but most

contain an uncalcified central core within a calcified Synthesizing cells Fibres l Blood
l Fibroblasts Collagen fibres vessels
outer layer.
l Cementoblasts l Principal l Nerves

l Osteoblasts group l Lymphatics


Q.4. Functions of cementum.
Resorptive cells l Secondary

Ans. l Fibroblasts group


l Cementoclasts Elastic fibres
[Ref LE Q.2] l Osteoclasts l Oxytalan

Epithelial cells l Elaunin


Q.5. Types of cementoenamel junction. l Cell rests of Ground substance
Malassez l Protein poly-
Ans.
Progenitor cells saccharides
l Undifferenti- l Glycosamino-
[Ref SE Q.6]
ated mesen- glycans
Q.6. Classification of cementum by Schroeder and Page. chymal cells l Proteoglycans

Defence cells l Glycoproteins,


Ans. l Mast cells especially
l Macrophages fibronectin
[Ref LE Q.2]
766 Quick Review Series for BDS 4th Year, Vol 1

Q.12. Define intermediate plexus. The features of acellular cementum are as follows:
l No cellular elements.
Ans.
l Collagen fibres are more organized.
l In addition to principal fibre types, small collagen fibres l Rate of development is relatively slow.
associated with the larger principal collagen fibres have l Forms during root formation.
also been described. These fibres run in all directions,
Q.15. Cellular cementum.
forming a plexus called the indifferent fibre plexus or
intermediate plexus. Ans.
l They are reticulate fibres, which are fine, immature col-
Depending on cellularity, the cementum has been classified
lagen fibres with a lattice-like arrangement.
as follows:
Q.13. Lamina dura. i. Acellular or primary cementum
ii. Cellular or secondary cementum
Ans.
The features of cellular cementum are as follows:
l The interdental septum normally presents a thin, radi- l Lacunae and canaliculi containing cementocytes and

opaque border that is adjacent to the PDL and at the their processes are present.
alveolar crest, referred to as the lamina dura. l Collagen fibres are irregularly organized.

l This appears radiographically as a continuous white l Rate of development is relatively fast.

line, but in reality it is perforated by numerous small l Forms after root formation and in response to functional

foramina and traversed by blood vessels, lymphatics demands.


and nerves, which pass between the PDL and the bone.
Q.16. Periodontal fibres.
l Radiographically, the lamina dura is prominent in the

deciduous dentition, with a wider periodontal space Ans.


than in the permanent dentition.
[Same as SN Q.9]
l Because the lamina dura represents the bone surface

lining the tooth socket, the shape and position of the Q.17. Histology of periodontal fibres.
root and changes in the angulation of the X-ray beam
Ans.
produce considerable variations in its appearance.
[Same as SN Q.9]
Q.14. Acellular cementum.
Ans.
Depending on cellularity, the cementum has been classified
as follows:
i. Acellular or primary cementum
ii. Cellular or secondary cementum

Topic 3
Age-Related Changes in the Periodontium
COMMONLY ASKED QUESTIONS
LONG ESSAYS:
1 . Describe the effects of ageing upon the periodontal tissues.
2. Describe the changes in the periodontal tissues due to ageing. [Same as LE Q.1]
Section | I  Topic-Wise Solved Questions of Previous Years 767

SHORT ESSAYS:
1 . Age changes in the periodontium.
2. Nature of periodontal diseases in older age group.
3. Age changes in the periodontium. [Same as SE Q.2]

SHORT NOTES:
1 . Age changes in gingiva. [Ref SE Q.1]
2. Bruxism.

SOLVED ANSWERS
LONG ESSAYS:
Q.1. Describe the effects of ageing upon the periodontal deterioration, like loss of elasticity and increased
tissues. resistance of the tissue, may cause decreased per-
meability, decreased nutrient flow and the accu-
Ans.
mulation of wastes in the cell.
Ageing of the periodontium is a result of cellular ageing. In l Thus, vascular peripheral resistance, i.e. decreased
general, cellular ageing is the basis for the intrinsic changes blood supply may secondarily decrease cellular
seen in oral tissues over time. The ageing process does not function.
affect every tissue in the same way. For example, muscle Physiological changes
tissue and nerve tissue undergo minimal renewal, whereas l Decrease in the number of collagen fibres – reduction
epithelial tissue, which is one of the primary components of or loss in tissue elasticity.
the periodontium, always renews itself. l Decrease in vascularity – decreased production of
Intrinsic changes mucopolysaccharides.
l In the ageing process, cell renewal takes place at a l With ageing, the alveolar bone shows a decrease in
slower rate and with fewer cells, so the effect is to bone density, an increase in bone resorption, and a
slow down the regenerative processes. decrease in vascularity also occurs. In contrast, how-
l As the progenitor cells wear out and die, there are ever, cementum shows cemental thickness.
fewer and fewer of these cells to renew the dead Functional changes
ones. l With ageing, the cells of the oral epithelium and
l This effect is a characteristic of the age-related periodontal ligament have reduced mitotic activity,
changes and biological changes that occur with and all cells experience a reduction in metabolic rate.
ageing. l These changes also affect the immune system and
l The decreased cellular component has a concomitant healing in the periodontium. There is a reduction in
effect to decrease cellular reserves and protein syn- healing capacity and rate.
thesis. This affects the oral epithelium in that the l Inflammation, when present, develops more rapidly
tissue becomes thin, with reduced keratinization. and severely.
Stochastic changes l Individuals are highly susceptible to viral and fungal
l Stochastic changes occurring within cells affect tis- infections because of abnormalities in T-cell
sues. For example, glycosylation and cross-linking function.
produce morphological and physiological changes. Compensatory changes
i. Loss of elasticity and increased mineralization l Compensatory changes occur as a result of ageing or
(fossilization) – structures become stiffer disease.
ii. Loss of regenerative power – structures become l These changes affect the tooth or periodontium and
less soluble and more thermally stable present clinically as follows:
iii. Somatic mutations – decreased protein synthesis i. Gingival recession and reduction in bone height
and structurally altered proteins are common conditions.
iv. Free radicals – accumulation of waste in the cell ii. Attrition is a compensatory change that acts as a sta-
l All these things produce a decline in the physiologic bilizer between loss of bony support and excessive
processes of tissue. leveraging from occlusal forces imposed on the teeth.
l Most changes are primarily a result of ageing, iii. A reduction in ‘overjet’ of the teeth is seen, mani-
although some are secondary to physiologic festing as an increase in the edge-to-edge contact
768 Quick Review Series for BDS 4th Year, Vol 1

of the anterior teeth. Typically, this is related to the l The periodontal ligament width is increased as a
approximal wear of the posterior teeth. result of less number of teeth supporting the entire
iv. An increase is seen in the food table area, with functional load.
loss of ‘sluiceways’, and in mesial migration. l Decrease in its width is associated with reduced

Functional changes are associated with reduced effi- strength of the masticatory musculature and contin-
ciency of mastication. Although effectiveness of mastica- ues deposition of cementum and bone.
tion may remain, efficiency is reduced because of missing Cementum
teeth, loose teeth, poorly fitting prostheses or noncompli- l An increase in cemental width is a common find-

ance of the patient, who may refuse to wear prosthetic ing because deposition continues after tooth
appliances. eruption.
l Increase in surface irregularity because of accu-
Q.2. Describe the changes in the periodontal tissues due
mulation of resorption bays.
to ageing.
Alveolar bone
Ans. l Osteoporosis.

l Decreased vascularity.
[Same as LE Q.1]
l Reduction in metabolic rate and healing capacity.

l Resorption activity is increased and rate of bone

SHORT ESSAYS: formation is decreased.


l Irregularities on alveolar bone facing periodontal
Q.1. Age changes in the periodontium.
ligament is seen. More irregular periodontal sur-
Ans. face of bone and less regular insertion of collagen
fibres.
Effect of ageing on the periodontium
Gingiva
Q.2. Nature of periodontal diseases in older age group.
Ans.
{SN Q.1}
l Periodontal disease in older adults is usually referred to
i. Gingival epithelium
as chronic periodontitis.
l Thinning and decreased keratinization of gingi-
l Since periodontitis is a chronic disease, much of the
val epithelium which leads to epithelial permea-
ravages of the disease detected in older adults result
bility to bacterial antigens and decreased resis-
from an accumulation of the disease over time.
tance to functional trauma
l One theory states that many sites of advanced periodon-
l Flattening of rete pegs and altered cell density
tal disease have resulted in tooth loss earlier in life,
l Reduced or unchanged amount of stippling
suggesting that older age is not a risk factor for peri-
l Increased width of attached gingiva
odontal disease.
l Greater amount of intercellular substances
l Evidence is limited on whether the risk factors for peri-
ii. Gingival connective tissue
odontal disease differ with age.
l Coarser and denser connective tissue which is
l General health status, immune status, diabetes, nutri-
due to increased rate of conversion of insoluble
tion, smoking, genetics, medications, mental health
collagen into soluble collagen, increased me-
status, salivary flow, functional deficits and finances
chanical strength and increased denaturating
may modify the relationship between periodontal dis-
temperature.
ease and age.
l Some frequently prescribed medications for older
adults can alter the gingival tissues. Steroid-induced
gingivitis has been associated with postmenopausal
iii. Periodontal ligament women receiving steroid therapy. Gingival over-
l Decreased number of fibroblasts and more irregular growth can be induced by such medications as cyclo-
structure. sporines, calcium channel blockers and anticonvul-
l Decreased organic matrix production and epithelial sants (e.g. nifedipine and phenytoin) in the presence
cell rests and increased amount of elastic fibres. of poor oral hygiene. This gingival overgrowth fur-
l Both increase and decrease in the width of the peri- ther decreases a person’s ability to maintain good
odontal ligament is seen. oral hygiene.
Section | I  Topic-Wise Solved Questions of Previous Years 769

Q.3. Age changes in the periodontium. Bruxism is defined as the diurnal or nocturnal parafunc-
tional activity that includes clenching, bracing, gnashing
Ans.
and grinding of the teeth.
[Same as SE Q.1] Effects of bruxism on periodontium
l No causative association exists between bruxism and

gingival inflammation or periodontitis.


SHORT NOTES: l Bruxism can cause tooth mobility, tooth fracture,

tooth wear, periodontal and muscle pain.


Q.1. Age changes in gingiva.
Management
Ans. A maxillary and mandibular stabilization appliance is
generally considered the most effective means of man-
[Ref SE Q.1]
aging bruxism.
Q.2. Bruxism.
Ans.

Topic 4
Classification of Diseases of the Periodontium
COMMONLY ASKED QUESTIONS
LONG ESSAYS:
1. Classify periodontal diseases and conditions as per International Workshop for Classification of Periodontal
Diseases and Conditions.

SHORT ESSAYS:
1. Classify periodontitis. Describe the clinical and radiographic features of chronic periodontitis.

SHORT NOTES:
1 . Classify periodontal diseases.
2. Enumerate inflammatory conditions of periodontium.

SOLVED ANSWERS
LONG ESSAYS:
Q.1. Classify periodontal diseases and conditions as per
1. Associated with dental plaque only with/without
International Workshop for Classification of Periodon-
other local contributing factors
tal Diseases and Conditions.
2. Gingival diseases modified by systemic factors
Ans. a. Associated with the endocrine system:
International Workshop for Classification of Periodon- Puberty-associated gingivitis, menstrual cycle-
tal Diseases and Conditions (1999) associated gingivitis, pregnancy-associated
I. Gingival diseases gingivitis, pyogenic granuloma and diabetes
A. Dental plaque-induced mellitus–associated gingivitis.
770 Quick Review Series for BDS 4th Year, Vol 1

b. Associated with blood dyscrasias: Leukaemia- V. Necrotizing periodontal diseases


associated gingivitis and others. Necrotizing ulcerative gingivitis (NUG), Necrotizing
3. Gingival diseases modified by medications ulcerative periodontitis (NUP)
a. Drug-induced gingival enlargements VI. Abscesses of the periodontium
b. Drug-influenced gingivitis, e.g. oral contraceptives Gingival, periodontal, pericoronal abscesses
4. Gingival diseases modified by malnutrition, e.g. VII. Periodontitis associated with endodontic lesions
vitamin C and others. Combined periodontal–endodontic lesions
B. Nonplaque-induced gingival lesions VIII. Developmental or acquired deformities and
1. Specific bacterial origin: Neisseria gonorrhoeae, conditions
Treponema pallidum, streptococcal species and others a. Localized tooth-related factors that modify or
2. Viral origin: herpes virus infections and others predispose to plaque-induced gingival diseases/
3. Fungal origin: candida species infections, linear periodontitis
gingival erythema, histoplasmosis and others i. Tooth anatomic factors
4. Genetic origin: hereditary gingival fibromatosis ii. Dental restorations/appliances
and others iii. Root fractures
5. Manifestations of systemic conditions: iv. Cervical root resorption and cemental tears
a. Mucocutaneous disorders: lichen planus, pem- b. Mucogingival deformities and conditions around
phigus, pemphigoid, etc. teeth
b. Allergic reactions: i. Gingival/soft tissues recession on facial/
i. Dental restorative materials – mercury, lingual/interproximal /papillary
acrylic, etc. ii. Lack of keratinized gingiva
ii. Reactions attributed to toothpastes/denti- iii. Decreased vestibular depth
frices, mouth rinses/washes, chewing gum iv. Aberrant frenum/muscle position
additives, food and additives c. Gingival excess
iii. Others d. Pseudo-pockets, inconsistent gingival margin,
6. Traumatic lesions: Chemical, physical, thermal, excessive gingival display, gingival enlargement,
factitious, iatrogenic, accidental abnormal colour, mucogingival deformities and
7. Foreign body reactions conditions on edentulous ridges
8. Not otherwise specified (NOS) i. Vertical and/or horizontal ridges deficiency
II. Chronic periodontitis ii. Lack of gingival/keratinized tissue
(Based on clinical, radiographic, historical and labora- iii. Gingival/soft tissue enlargement
tory characteristics) iv. Aberrant frenum/muscle position
A. Localized (,30% of sites involved) v. Decreased vestibular depth
B. Generalized (.30% of sites involved) vi. Abnormal colour
Both groups can be categorized as slight: 1–2 mm; e. Occlusal trauma
moderate: 3–4 mm; severe: 5 mm CAL i. Primary occlusal trauma
III. Aggressive periodontitis ii. Secondary occlusal trauma
(Otherwise clinically healthy individuals, rapid attach-
ment and bone loss, not consistent with local deposits,
familial aggregation) SHORT ESSAYS:
a. Localized (circumpubertal onset; first molar or
incisor has proximal attachment loss; robust se- Q.1. Classify periodontitis. Describe the clinical and
rum antibody response to infective agents) radiographic features of chronic periodontitis.
b. Generalized (affects below 30 years of age; gen- Ans.
eralized proximal attachment loss; poor serum
antibody response to infective agents; episodic The diseases of periodontitis can be classified into the fol-
nature of periodontal disease) lowing three major types based on clinical, radiographic,
IV. Periodontitis as a manifestation of systemic diseases historical and laboratory characteristics.
a. Associated with haematological disorders: Ac- 1. Chronic periodontitis
quired neutropaenia, leukaemia and others l Localized form

b. Associated with genetic disorders: Leukocyte Ad- l Generalized form

hesion deficiency syndromes (LAD) deficiency l Slight form

syndromes and others l Moderate form


Section | I  Topic-Wise Solved Questions of Previous Years 771

2. Aggressive periodontitis marginal tissues may obscure the underly-


l Localized form ing inflammatory changes.
l Generalized form iv. Pocket formation of variable depth.
Periodontitis as a manifestation of systemic diseases: v. Loss of periodontal attachment.
i. Haematologic disorders vi. Loss of alveolar bone both horizontal and
a. Acquired neutropaenia vertical bone loss.
b. Leukaemia vii. Tooth mobility often appears in advanced
c. Others cases.
ii. Genetic disorders Symptoms
a. Familial and cyclic neutropaenia l Bleeding gums – on brushing or eating.

b. Down syndrome l Spaces occur between the teeth as a result of

c. Leukocyte adhesion deficiency syndromes tooth movement.


d. Papillon–Lefevre syndrome l Teeth are mobile – usually painless, sometimes

e. Chediak–Higashi syndrome pain may be present.


f. Histiocytosis syndromes l Exposed roots – sensitivity to heat, cold or

g. Glycogen storage disease both.


h. Infantile genetic agranulocytosis l Areas of dull pain radiating deep into the jaw.

i. Cohen syndrome l Areas of food impaction add to patient discom-

j. Ehlers–Danlos syndrome fort.


k. Hypophosphatasia l Gingival tenderness or itchiness may be present.

l. Others Radiographic features


iii. NOS l Two patterns of bone loss are observed in

Chronic periodontitis chronic periodontitis.


It is defined as ‘an infectious disease resulting in in- a. Vertical
flammation within the supporting tissues of the teeth, b. Horizontal
progressive attachment loss and bone loss’. Vertical bone loss
Clinical features l Attachment loss and bone loss on the

General characteristics toot surface is greater than that on an


i. Age of onset is usually 30–35 years. adjacent.
ii. Supragingival and subgingival plaque accu- l Usually associated with angular bony

mulation is frequently associated with calcu- defects and infrabony pockets.


lus formation. Horizontal bone loss
iii. Gingival inflammation. l Attachment loss and bone loss occur at a

l In patients with poor oral hygiene – slightly to uniform rate on majority of tooth sur-
moderately swollen gingiva and exhibits faces.
alteration in colour ranging from pale red to l Generally associated with suprabony

magenta. pockets.
l Loss of stippling is seen.

l Changes in surface topography – blunted

or rolled gingival margins are flattened or


SHORT NOTES:
cratered papillae. Q.1. Classify periodontal diseases.
l In patients with regular home care –
Ans.
changes in colour, contour and consistency
may not be visible on inspection and may Classification of periodontal diseases and conditions
be detected only as bleeding on probing of Gingival diseases
gingiva. l Plaque-induced gingival diseases

l Gingival bleeding either spontaneous or in l Nonplaque-induced gingival diseases

response to probing is common and in- Chronic periodontitis


flammation-related exudates of crevicular l Localized

fluid and suppuration from pocket may l Generalized

also be found. Aggressive periodontitis


l Some cases as a result of long-standing l Localized

low-grade inflammation thickened, fibrotic, l Generalized


772 Quick Review Series for BDS 4th Year, Vol 1

Periodontitis as a manifestation of systemic diseases Q.2. Enumerate inflammatory conditions of peri-


1. Necrotizing periodontal diseases odontium.
l NUG
Ans.
l NUP

2. Abscesses of periodontium Following are some of the inflammatory conditions of peri-


l Gingival abscess odontium:
l Periodontal abscess i. NUG
l Pericoronal abscess ii. Acute herpetic gingivostomatitis
3. Periodontitis associated with endodontic lesions iii. Recurrent aphthous stomatitis (RAS)
l Endodontic-periodontal lesion iv. Pericoronitis
l Periodontal-endodontic lesion v. Desquamative gingivitis
l Combined lesion vi. Chronic periodontitis
4. Developmental or acquired deformities and vii. Aggressive periodontitis
conditions viii. NUP
l Localized tooth-related factors that predispose to ix. Refractory periodontitis
plaque-induced gingival disease or periodontitis
l Mucogingival deformities and conditions
around teeth
l Mucogingival deformities and conditions on

edentulous ridges
l Occlusal trauma

Topic 5
Epidemiology of Gingival and Periodontal Diseases
COMMONLY ASKED QUESTIONS
LONG ESSAYS:
1 . Define dental epidemiology and write in detail about the indices used in assessing gingival inflammation.
2. Describe the possible causes as to why the incidence and prevalence of periodontal diseases are very high in
India.

SHORT ESSAYS:
1. Indices used to measure periodontal destruction.
2. CPITN probe.
3. Silness and Loe index.
4. Bleeding point index. [Ref LE Q.1]
5. Oral hygiene simplified.
6. Periodontal disease index.
7. Define index. State uses and ideal requirements of an index.
8. Community periodontal index.
9. Russell’s periodontal index. [Same as SE Q.1]
10. Name any two plaque indices describing in detail any one of them. [Same as SE Q.5]
Section | I  Topic-Wise Solved Questions of Previous Years 773

SHORT NOTES:
1 . OHI-S. [Ref SE Q.5]
2. Silness and Loe index. [Ref SE Q.3]
3. Define risk factor and risk indicator.
4. CPITN index.
5. Indices used to measure calculus.

SOLVED ANSWERS
LONG ESSAYS:
Q.1. Define dental epidemiology and write in detail l The PDI does so by combining the assessments of
about the indices used in assessing gingival inflam- gingivitis and gingival sulcus depth on six selected
mation. teeth (numbers 3, 9, 12, 19, 25 and 28).
l This group of teeth, frequently referred to as the
Ans.
Ramfjord teeth, has been tested as reliable indicators
John M. Last (1988) defined epidemiology as ‘the study of for the various regions of the oral cavity.
distribution and determinants of health-related states or l Calculus and plaque are also examined to assist in
events in specified populations, and the application of this formulating a comprehensive assessment of peri-
study to the control of health problems’. odontal status.
The following are the various indices used to assess l A numerical score for the gingival status component
gingival inflammation: of the PDI is obtained by adding the values for all the
l Papillary marginal attachment (PMA) index by Schour
gingival units and dividing it by the number of teeth
and Massler (1944) present. This index has been used in epidemiological
l Gingivitis component of the periodontal disease
surveys, longitudinal studies and clinical trials.
l Gingival index by Loe and Silness (1963)
Gingival index by Loe and Silness (1963)
l Sulcular bleeding index by Muhlemann and Son (1971)
This index was solely developed for the purpose of as-
l Papillary bleeding index by Muhlemann and Son (1977)
sessing the severity of gingivitis and its location in four
l Bleeding points index by Lennox and Kopczyk
possible areas.
l Interdental bleeding index by Caton and Polson
Method
l Gingival bleeding index by Ainamo and Bay
l The severity of gingivitis is scored on all surfaces
PMA index by Schour and Massler (1944) of all teeth or selected teeth or on selected sur-
l In this index, the number of gingival units affected
faces of all teeth or selected teeth. The tissues
were counted rather than the severity of the inflam- surrounding each tooth are divided into four gin-
mation. gival scoring units:
l A gingival unit is divided into three component parts:
i. Distal facial papillae
i. Papillary gingiva (P) ii. Facial margin
ii. Marginal gingiva (M) iii. Mesial facial papillae
iii. Attached gingiva (A) iv. Entire lingual gingival margin
l The presence or absence of inflammation on each
l A blunt instrument is used for recording the
gingival unit is recorded as 1 or 0, respectively. scores based on the following criteria:
l The P, M and A numerical values for all the teeth are
0 – No inflammation
added separately and then added together to express 1 – Mild inflammation, no bleeding elicited on
the PMA index score per person. probing
Calculation of PMA score: 2 – Moderate inflammation, bleeding on probing
PMA score 5 P 1 M 1 A 3 – Severe inflammation
l The developers of this index eventually added a se-
l The scores around each tooth are added and di-
verity component for assessing gingivitis, the papil- vided by four to arrive at the score for that par-
lary units (P) were scored on a scale of 0–5 and the ticular tooth.
marginal (M) and attached gingiva were scored on a l Then total all the teeth scores and divide it by the
scale of 0–3. number of teeth.
Gingivitis component of the periodontal disease l This provides the gingival index score per person.
l The periodontal disease index (PDI) is used to mea-
The numerical values are correlated as follows:
sure the presence and severity of periodontal disease. 0.1–1.0: mild gingivitis
774 Quick Review Series for BDS 4th Year, Vol 1

1.1–2.0: moderate gingivitis 4 – Profuse bleeding occurs after probing, blood


2.1–3.0: severe gingivitis flows immediately into the marginal sulcus.
Bleeding points index by Lennox and Kopczyk
[SE Q.4]
l The index was developed to assess a patient’s oral

{Indices of gingival bleeding hygiene performance.


Sulcular bleeding index by Muhlemann and Son l It determines the presence or absence of gingival

(1971) bleeding interproximally and on the facial and lin-


l The purpose of the index is to locate areas of the gual surfaces of each tooth.
gingival sulcus that bleed upon gentle probing, l A periodontal probe is drawn horizontally through

and thus recognize and record the presence of the gingival crevice of a gradient, and the gingiva is
early inflammatory gingival disease. examined for bleeding after 30 s.
l Four gingival units are scored systemically for Interdental bleeding index by Caton and Polson
each tooth: the labial and lingual marginal gingiva l The index utilizes a triangle-shaped toothpick made

(M units) and the mesial and distal papillary gin- up of soft, pliable wood to stimulate the interproxi-
giva (P units). mal gingival tissue.
l The probe is held parallel with the long axis of the l The interproximal cleaner is inserted horizontally

tooth and 30 s after probing; scoring is done based between the teeth from the facial surface, depressing
on the criteria, which ranges from 0 to 5. the interproximal papillae by up to 2 mm.
l Each of the four gingival units is scored 0–5. l The wooden cleaner is inserted and removed four

Scores for the four units are added and divided by times, arid the presence or absence of bleeding
four. Adding the scores of the undivided teeth and within 15 s is noted.
dividing them by the number of teeth can deter- l The score is determined by dividing the number of

mine the sulcus bleeding index. bleeding sites by the number of sites evaluated.
Scoring criteria Gingival bleeding index by Ainamo and Bay
0 – Normal appearing gingiva, no bleeding upon l The index was developed as an easy and suitable way

probing for the practitioner to assess a patient’s progress in


1 – Apparently healthy with no colour or contour plaque control.
changes, but bleeding on probing l The presence or absence of gingival bleeding is de-

2 – Bleeding on probing, colour change due to in- termined by gentle probing of the gingival crevice
flammation, no oedema and contour changes with a periodontal probe.
3 – Bleeding on probing, colour change, mild inflam- l The appearance of bleeding within 10 s indicates a
matory oedema positive score, which is expressed as a percentage of
4 – Bleeding on probing, colour change, severe in- the total number of gingival margins examined.}
flammatory oedema Q.2. Describe the possible causes as to why the incidence
S – Spontaneous bleeding on probing, colour change, and prevalence of periodontal diseases are very high in
very severe inflammatory oedema with or without India.
ulceration.
Papillary bleeding index by Muhlemann and Son (1977) Ans.
l It is based on bleeding elicited following gentle Periodontal diseases in India are still the greatest single
probing of the interdental papilla. cause that lead to tooth loss.
l A blunt periodontal probe is carefully inserted into l Low socioeconomic group showed greater severity due

the gingival sulcus at the base of the interdental pa- to poor oral hygiene practice.
pilla on the mesial aspects and then moved coronally l Rural areas had higher periodontal diseases due to poor

to the papilla tip. oral hygiene. Severity of disease increased with age.
l This is repeated on the distal aspect of the same pa- l Poor oral hygiene was found to be the main factor for

pilla. The intensity of any bleeding thus provoked increase in the prevalence of gingivitis.
will be recorded on a scale of 0–4. l Greater prevalence of periodontal diseases in mentally

Scoring criteria retarded children is due to lack of awareness of oral


0 – No bleeding. hygiene.
1 – A single discrete bleeding point appears. l The possible factors contributing to periodontal diseases

2 – Several isolated bleeding points or a single fine in such children could be attributed to a nutritional defi-
line of blood appears. ciency, malocclusion and oral health habits like brux-
3 – The interdental triangle fills with blood shortly ism, tongue thrusting and mouth breathing. Also, there
after probing. is difficulty in the maintenance of oral hygiene due to
Section | I  Topic-Wise Solved Questions of Previous Years 775

low power of concentration and low neuromuscular plaque and calculus and are much less concerned about
coordination. their dental health.
l One of the investigations revealed that individuals l In India, it has been shown that beedi smokers have a

affected with periodontosis belonged to the middle or higher periodontal index and oral hygiene index score
lower income families from the western and northern than cigarette smokers.
parts of the country and consumed a diet that was typi- l Epidemiological studies in India have further shown

cally vegetarian in character. that people with systemic disease and malnutrition have
l A clinical impression was gained that periodontosis more periodontal diseases.
sufferers, besides being rated as having clean mouths, l It is recognized that chronic inflammatory periodontal

presented hereditary histories on the existence of diseases and their sequelae are rarely life-threatening
periodontosis among the female members of the and many other health problems are of more serious
family. concern (Miyazaki et al., 1984; Pilot and Miyazaki,
l To summarize the data, 95%–100% population has been 1994).
shown to be suffering from periodontal diseases in vari-
ous population groups, only differing in severity from
one to another.
SHORT ESSAYS:
l In the younger age group, i.e. till the age of 20 years, it Q.1. Indices used to measure periodontal destruction.
is gingivitis in its various severities, and above that age,
Ans.
specially above the age of 30 years, 99%–100% of the
population has been reported to be suffering from peri- Periodontal index (PI) (A.L. Russell, 1956)
odontal diseases and as high as 70% from periodontal l Its purpose is to assess and score the periodontal

pockets or bone resorption. disease status of population and in epidemiologic


l It seems evident from a number of descriptive epide- studies.
miological studies that habits such as smoking and betel l Instruments used are mouth mirror and light source.

leaf chewing are associated with an above average se- l Teeth are selected on whole mouth basis for entire

verity of periodontal diseases. Such groups have more dentition.

Scoring criteria for PI

Score Criteria
1 Negative: There is neither overt inflammation in the investing tissues nor loss of function due to destruction of supporting tissue.
2 Mild gingivitis: There is an overt area of inflammation in the free gingivae, which does not circumscribe the tooth.
4 Used only when radiographs are available.
6 Gingivitis with pocket formation: The epithelial attachment has been broken and there is a pocket. There is no interference
with normal masticatory function; the tooth is firm in its socket and has not drifted.

8 Advanced destruction with loss of masticatory function: The tooth may be loose, may have drifted, may sound dull on per-
cussion with a metallic instrument, or may be depressible in its socket.

Recording format for PI


18 17 16 15 14 13 12 11 21 22 23 24 25 26 27 28

48 47 46 45 44 43 42 41 31 32 33 34 35 36 37 38

PI score is given for each individual tooth based on ‘Russell’s rule: When in doubt, assign the lesser score’
the above data, and thus the clinical condition is as- Calculation
sessed for the given patient. Sum of individual scores
PI score per person =
Number of teeth present
776 Quick Review Series for BDS 4th Year, Vol 1

The value obtained is interpreted as follows: Surfaces examined


Individual PI score Clinical condition i. Only plaque at the cervical third of the tooth is
evaluated with no attention to plaque that has ex-
0–0.2 Clinically normal supportive tissues
tended to middle or incisal thirds.
0.3–0.9 Simple gingivitis ii. Four gingival areas of the tooth:
1.0–1.9 Beginning destructive periodontal l Distofacial
disease l Facial

2.0–4.9 Established destructive periodontal l Mesiofacial

disease l Lingual

5.0–8.0 Terminal disease


Scoring Criteria (Silness and Loe)
Score Criteria
Q.2. CPITN probe.
0 No plaque in the gingival area.
Ans.
1 A film of plaque adhering to the free gingival margin
l For the purpose of a new index called Community Peri- and adjacent area of tooth. The plaque may be recog-
nized only after application of disclosing agent or by
odontal Index of Treatment Needs (CPITN), a special
running a probe across the tooth surface.
probe called CPITN probe was designed for examining
patients. 2 Moderate accumulation of soft deposits within the gin-
gival pocket, on the gingival margin and/or adjacent
CPITN probe or WHO probe
tooth surface, which can be seen by the naked eye.
l In 1978, it was described by an expert committee

appointed by WHO. 3 Abundance of soft matter within the gingival pocket


and/or on the gingival margin and adjacent tooth
l It is designed for the following three purposes:
surface.
a. To measure the pocket depth
b. To detect the subgingival calculus
c. To manipulate the sensitive soft tissues around
the teeth {SN Q.2}
l Probe has a thin handle and is of very light weight
Calculation
(5 g).
l It has a ball-end tip of 0.5-mm diameter that
Total score of each tooth
PII of each tooth =
allows easy detection of subgingival calculus and 4
gingival bleeding without traumatizing the tissue.
l It has a black band starting at 3.5 mm and ending Total of PII of each tooth
PII (Plaque index for individual) =
at 5.5 mm (CPITN-E). Number of teeth examined
l The probe has two additional markings when
The index is interpreted as follows:
used in clinical work of 8.5 mm and 11.5 mm
l Excellent: 0
(CPITN-C).
l Good: 0.1–0.9
Q.3. Silness and Loe index. l Fair: 1.0–1.9

l Poor: 2.0–3.0
Ans.

{SN Q.2}
l Silness and Loe index is also known as plaque index Q.4. Bleeding point index.
(1964). Ans.
l Its purpose is to assess the thickness of plaque only

at gingival surface. [Ref LE Q.1]


l Instruments used are mouth mirror, dental explorer Q.5. Oral hygiene simplified.
and light source.
Ans.
Selection of teeth The plaque indices commonly used are as follows:
Teeth are selected on the following basis: i. Plaque component of PDI by Ramfjord
l Whole mouth basis – entire dentition ii. Simplified oral hygiene index (OHI-S) by Greene and
l Selected mouth basis – 16, 12, 24, 36, 32, 44 Vermillion (1964).
Section | I  Topic-Wise Solved Questions of Previous Years 777

Plaque component of PDI by Ramfjord 2 – Supragingival calculus covering more than one-third
l The index is used on the six teeth selected by Ramfjord but not more than two-thirds of the exposed tooth sur-
(teeth number 3, 9, 12, 19, 25 and 28) after staining face or the presence of the individual flecks of subgin-
with Bismarck brown solution. gival calculus around the cervical portion of the tooth or
l The criterion is to measure the presence and extent of both.
plaque on a scale of 0–3, looking specifically at all 3 – Supragingival calculus covering more than two-
interproximal facial and lingual surfaces of the index thirds of the exposed tooth surface or a continuous
teeth. heavy band of subgingival calculus around the cervical
Scoring criteria portion of the tooth or both.
0 – No plaque present
1 – Plaque present on some but not all interproximal, {SN Q.1}
buccal and lingual surfaces
2 – Plaque present on all interproximal buccal and Method
lingual surfaces but covering less than one-half of the l Each component is assessed on a scale of 0–3.

surfaces l Only a mouth mirror and a Shepherd’s Crook or

3 – Plaque extending over all interproximal, buccal sickle-type dental explorer, and no disclosing
and lingual surfaces, and covering more than one- agents are used for examination.
half of these surfaces l The six tooth surfaces examined are numbers 3, 8,

Only fully erupted teeth are scored and missing 14, 24 (facial surface) and numbers 19, 30 (lin-
teeth should not be substituted. gual surfaces).
l Each tooth surface is divided horizontally into
Total score
Plaque score of an individual = gingival, middle and incisal thirds.
Number of teeth examined l For the DI-S, a dental explorer is placed on the

Uses incisal third and moved towards the gingival third


It can be used in the following: and scores are awarded according to the criteria.
a. Longitudinal studies of periodontal disease The DI-S score per person is obtained by totalling
b. Epidemiological surveys the debris score per the tooth surface and dividing
c. Clinical trials of preventive or therapeutic it by the number of surfaces examined.
l The CI-S assessment is performed by gently plac-
agents
ing a dental explorer into the distal gingival crev-
ice and drawing it subgingivally from the distal
{SN Q.1} contact area to the mesial contact area.
l Scoring is done according to the criteria.
OHI-S by Greene and Vermillion (1964) l The CI-S score per person is obtained by totalling
The OHI-S measures the surface area of the tooth the calculus scores per tooth surface and dividing
that is covered by debris and calculus. it by the number of surfaces examined.
It consists of two components: l The OHI-S score per person is the total of DI-S
a. Debris index — simplified (DI-S) and CI-S scores per person. The clinical levels of
b. Calculus index — simplified (CI-S) oral cleanliness for debris that can be associated
with groups (DI-S, CI-S).
Scores are as follows:
Scoring criteria for DI-S Good: 0.0–0.6
0 – No debris or stain present Fair: 0.7–1.8
1 – Soft debris covering not more than one-third of the Poor: 1.9–3.0
tooth surface or the presence of extrinsic stains without
other debris regardless of surface area covered
2 – Soft debris covering more than one-third but not The clinical levels of oral hygiene that can be associ-
more than two-thirds of the exposed tooth surface ated with group OHI-S scores are as follows:
3 – Soft debris covering more than two-thirds of the Good: 0.0–1.2
exposed tooth surface Fair: 1.3–3.0
Scoring criteria for CI-S Poor: 3.1–6.0
0 – No calculus present
Q.6. Periodontal disease index.
1 – Supragingival calculus covering not more than one-
third of the exposed tooth surface Ans.
778 Quick Review Series for BDS 4th Year, Vol 1

PDI (Siguard P. Ramfjord, 1959) ii. Reveals the degree of effectiveness of present
l The PDI is a clinician’s modification of the Russell’s oral hygiene practices.
PI for epidemiologic surveys of periodontal disease. iii. Motivates the person in preventive and profes-
l Teeth are selected on the following basis: sional care for the elimination or control of oral
i. Whole mouth basis – entire dentition diseases.
ii. Selected mouth basis – 16, 21, 24, 36, 41, 44 iv. Over a period of time the success of an individual
Components of PDI and professional treatment are evaluated by com-
i. Plaque component paring index scores.
ii. Calculus component In the case of research studies
iii. Gingival and periodontal component: It helps in the following:
l The scoring of plaque is restricted to the i. To determine the baseline data before the in-
gingival half of the facial and lingual troduction of experimental factors
surfaces of the index teeth. ii. To measure the effectiveness of specific
l Mouth mirror, dental explorer and light agents for the prevention control and treat-
source are the instruments used. ment of oral conditions
iii. To measure the effectiveness of devices for
Scoring criteria for PDI personal care, e.g. toothbrushes, interdental
cleaning devices
Score Criteria In community health
0 Absence of dental plaque. i. Shows the prevalence and trends of incidence of
1 Dental plaque in the interproximal areas or at the a particular condition seen in a given population
gingival margin covering less than one-third of the ii. Provides a baseline data to show the existing
gingival half of the facial or lingual surface of the dental health practices
tooth.
iii. Assesses the needs of a community and com-
2 Dental plaque covering more than one-third but pares the effects of a community programme and
less than two-thirds of the gingival half of the fa- evaluates the results
cial or lingual surface of the tooth.
Ideal characteristics of an index
3 Dental plaque covering two-thirds or more of the 1. It should be simple to use and accurate.
gingival half of the facial or lingual surface of the 2. It should require minimal equipment and ex-
tooth.
penses.
3. It should have clear-cut criteria, which are readily
understandable.
Calculation 4. It should be as free as possible from subjective
Total score interpretation.
Plaque score of an individual =
Number of teeth examined 5. It should be reproducible by the same examiner or
different examiners.
6. Be amenable to statistical analysis, have validity
Q.7. Define index. State uses and ideal requirements of and reliability.
an index. 7. It should not require an excessive amount of time
to complete.
Ans.
8. It should not cause patient discomfort or other-
According to A.L. Russell, the numerical value describing wise be unacceptable to a patient.
the relative status of the population on a graduated scale with
definite upper and lower limits, which is designed to permit Q.8. Community periodontal index.
and facilitate comparisons with other populations and classi-
Ans.
fied by the same criteria and methods is known as an index.
According to Glickman, epidemiologic indices are the Community periodontal index (CPI)
attempts to quantitate clinical conditions on a graduated l This is a modification of the CPITN index by the
scale, thereby facilitating comparison among populations inclusion of measurement of ‘loss of attachment’ and
examined by the same criteria and methods. elimination of the ‘treatment needs’ category.
Uses of an index l A mouth mirror, CPI probe (earlier called CPITN-C
In the case of individual patients probe) and light source are the instruments used.
i. Provides individual assessment to help a patient l Teeth selected are:
recognize an oral problem. i. Index teeth (up to 19 years) – 16, 11, 26, 36, 31, 46
Section | I  Topic-Wise Solved Questions of Previous Years 779

ii. Index teeth (aged 20 years or more) – 17, 16, 11, Q.10. Name any two plaque indices describing in detail
26, 27, 37, 36, 31, 46, 47 any one of them.

Codes and criteria for CPI are as follows: Ans.

Code Criteria [Same as SE Q.5]


0 Healthy
1 Bleeding observed, directly or by using a mouth mirror, SHORT NOTES
after probing
Q.1. OHI-S.
2 Calculus detected during probing but all of the black
band on the probe visible Ans.
3 Pocket 4–5 mm (gingival margin within the black band [Ref SE Q.5]
on the probe)
Q.2. Silness and Loe index.
4 Pocket 6 mm or more (black band on the probe not
visible) Ans.
X Excluded sextant (less than two teeth present)
[Ref SE Q.3]
9 Not recorded
Q.3. Define risk factor and risk indicator.
Ans.
Loss of attachment
l Information on the loss of attachment may be col- Risk is the probability that an individual will get a specific
lected from index teeth. disease in a given period.
l The most reliable way of examining the loss of Risk factor
attachment in each sextant is to record this im- It is defined as an aspect of personal behaviour or life-
mediately after recording the CPI score for that style, an environmental exposure or an inborn or inher-
particular sextant. ited characteristic that, on the basis of epidemiologic
l Loss of attachment should not be recorded for evidence, is known to be associated with health condi-
children under the age of 15 years. tions considered important to be prevented.
When the Cemento enamel Junction (CEJ) is not Risk indicator
visible and the highest CPI score for a sextant is A probable or putative risk factor that has been associ-
less than 4 (probing depth less than 6 mm), any ated with the disease through cross-sectional studies.
loss of attachment for that sextant is estimated to Risk marker
be less than 4 mm (loss of attachment score 5 0). A factor that is associated with increased probability of
The extent of loss of attachment is recorded using the the future disease.
following codes:
Q.4. CPITN index.
0 – Loss of attachment 0–3 mm (CEJ not visible and
CPI score 0–3). Ans.
If the CEJ is not visible and the CPI score is 4, or if
CPITN by Ainamo and Associates
the CEJ is visible:
l The CPITN was developed for ‘Joint Working Com-
1 – Loss of attachment 4–5 mm (CEJ within the
mittee’ of ‘WHO’ and ‘FDI’ by Jukka Ainamo, David
black band)
Barmes, George Beagrie, Terry Cutress, Jean Martin
2 – Loss of attachment 6–8 mm (CEJ between the
and Jennifer Sardo-Infirri in 1982.
upper limit of the black band and the 8.5-mm ring)
l This index was developed primarily to survey and
3 – Loss of attachment 9–11 mm (CEJ between the
evaluate periodontal treatment needs rather than de-
8.5- and 11.5-mm rings)
termining past and present periodontal status, i.e. the
4 – Loss of attachment 12 mm or more (CEJ beyond
recession of gingival margin and alveolar bone.
the 11.5-mm ring)
l This can be used in both community and the
X – Excluded sextant (less than two teeth present)
individual.
9 – Not recorded (CEJ neither visible nor detectable)
Advantages
Q.9. Russell’s periodontal index. l Simplicity

l International uniformity
Ans.
l Permits rapid examination of a population to de-
[Same as SE Q.1] termine periodontal treatment needs
780 Quick Review Series for BDS 4th Year, Vol 1

Disadvantages l Calculus surface severity index (CSSI)


l The position of the gingival margin is not l Marginal line calculus index (MLCI) by Muhlemann
recorded. and Villa
l It does not provide assessment of past periodontal l Volpe–Manhold index (VMI)

breakdown. l PDI (calculus component)

l A great deal of useful information is lost when l Calculus component of OHI-S by Greene and
only the worst score per sextant is recorded. Vermillion
l CPITN underestimates the pockets greater than l Calculus component of the PDI by Ramfjord

6 mm in older age groups and overestimates the l Probe method of calculus assessment by Volpe and

need for scaling in younger age groups. associates


l Calculus surface index by Ennever and coworkers
Q.5. Indices used to measure calculus.
Ans.
The following are the various indices used to measure
calculus:
l Calculus surface index (CSI)

Topic 6
Periodontal Microbiology
COMMONLY ASKED QUESTIONS
LONG ESSAYS:
1. Define and classify dental plaque. Write in detail about its composition.
2. Define dental plaque. Describe its role in aetiology of periodontal diseases.
3. Describe in detail the steps in the formation of dental plaque.
4. Define dental plaque. Describe the characteristics of the gel-like matrix of ‘biofilm’.
5. Discuss supragingival and subgingival plaque. [Same as LE Q.1]
6. Describe the role of microorganisms in the aetiology of periodontal diseases. [Same as LE Q.2]
7. Discuss the role of plaque in the aetiology of periodontal diseases. [Same as LE Q.2]
8. What is dental plaque? Describe its role in the initiation and progression of gingival and periodontal diseases.
[Same as LE Q.2]
9. Describe in detail the plaque retention as aetiological factor of periodontal diseases. [Same as LE Q.2]
1 0. Define dental plaque. Write in detail about the formation of plaque. Add a note on specific plaque hypothesis.
[Same as LE Q.3]
1 1. Define plaque. Describe in detail the composition, structure and formation of dental plaque. [Same as LE Q.3]
12. Define dental biofilm and highlight the properties of the same. [Same as LE Q.4]

SHORT ESSAYS:
1 . Structure and composition of plaque. [Ref LE Q.1]
2. Compare supragingival and subgingival plaque.
3. Composition of dental plaque in adult periodontitis.
4. Subgingival plaque. [Ref LE Q.1]
5. Specific plaque hypothesis. [Ref LE Q.2]
6. Mechanism of formation of plaque. [Ref LE Q.3]
Section | I  Topic-Wise Solved Questions of Previous Years 781

7. Socransky’s modification of Koch’s postulates.


8. Differences between supra- and subgingival plaque. [Same as SE Q.2]
9. Define plaque and add a note on its mechanism of formation. [Same as SE Q.6]
10. Socransky’s criteria for identification of periodontal pathogens. [Same as SE Q.7]

SHORT NOTES:
1. Dental pellicle.
2. Acquired pellicle. [Ref LE Q.3]
3. Aetiological significance of microbial plaque.
4. Specific plaque hypothesis. [Ref LE Q.2]
5. Define and classify dental plaque. [Ref LE Q.1]
6. Name the stages of plaque formation. [Ref SE Q.6]
7. Materia alba.
8. Normal oral bacterial flora.
9. Coaggregation.
10. Spirochaetes.
11. Prevotella intermedia.
12. Subgingival plaque. [Ref LE Q.1]

SOLVED ANSWERS
LONG ESSAYS:
Q.1. Define and classify dental plaque. Write in detail intermicrobial matrix which accounts for 25% of
about its composition. plaque volume consisting of organic and inorganic
materials derived from bacterial products, saliva and
Ans.
gingival crevicular fluid.
l Organic matter mainly consists of polysaccharide pro-
{SN Q.5} tein complex produced by plaque microorganisms.
l Proteins such as albumin present in plaque are de-
l Dental plaque is defined clinically as a structured
rived from crevicular fluid.
resilient, yellowish-grey substance consisting of bac-
l Levans, glucans, galactose and methylpentose are some
terial aggregations that adhere tenaciously to teeth
of the carbohydrates produced by microorganisms.
and other intraoral hard surfaces such as restorations.
l Small amount of lipids found in plaque are derived
from the disrupted cell walls of Gram-negative bacteria.
l The main inorganic constituents of plaque are cal-
[SE Q.1] cium and phosphorus along with small amounts of
magnesium, potassium and sodium.}
{Structure and composition of plaque
l Plaque is primarily composed of microorganisms. (SE Q.4 and SN Q.5)
l One gram of plaque (wet weight) contains approxi- {(Types of dental plaque
mately 1011 bacteria. The number of bacteria in supra- l Plaque is broadly differentiated into two categories,
gingival plaque on a single tooth surface can exceed based on its relationship to the gingival margin:
109. In a periodontal pocket, counts can range from A. Supragingival plaque
103 bacteria in a healthy crevice to . l05 bacteria in a B. Subgingival plaque
deep pocket. l Supragingival plaque is further differentiated
l The nonbacterial organisms such as yeasts, myco-
into:
plasma, protozoa and viruses are also present in i. Coronal plaque, which is in contact with
small amounts along with host cells such as epithe- only the tooth surface.
lial cells, macrophages and leukocytes. ii. Marginal plaque, which is associated with
l These proliferating microorganisms along with host
the tooth surface at the gingival margin.
cells exist on an adherent intercellular matrix or
782 Quick Review Series for BDS 4th Year, Vol 1

or Gram-negative rods embedded in an amorphous


Subgingival plaque can be further differenti-
matrix.}
l

ated into:
i. Attached plaque Q.2. Define dental plaque. Describe its role in aetiology
ii. Unattached plaque of periodontal diseases.
Attached plaque can be tooth, epithelium
Ans.
or connective tissue associated.)}
l Dental plaque is defined clinically as a structured resil-
Supragingival plaque
ient, yellowish-grey substance consisting of bacterial
i. It is usually adherent to the tooth surface. It contains
aggregations that adhere tenaciously to teeth and other
Gram-positive cocci and Gram-negative rods and
intraoral hard surfaces such as restorations.
filaments.
Aetiological role of plaque in periodontal diseases:
ii. It is found at or above the gingival margin; when in
l It was recognized and well known that dental
direct contact with the gingival margin, it is termed
plaque plays an important role in the initiation
as marginal plaque.
and progression of periodontal diseases.
iii. Supragingival plaque typically demonstrates a strat-
ified organization of a multilayered accumulation of Nonspecific plaque hypothesis
bacterial morpho types. Gram-positive cocci and l First it was thought that the severity of periodontal

short rods predominate at the tooth surface, whereas diseases was associated with the quantity of plaque
Gram-negative rods and filaments, as well as spiro- formed. This is known as the nonspecific plaque
chaetes, predominate on the outer surface of the hypothesis.
mature plaque mass. l This hypothesis was proposed in 1976 by Walter

iv. The morphological arrangement of the flora in supra- Loesche. He delineated this from the specific plaque
gingival plaque is described as ‘corncob’ formation. hypothesis.
v. Corncob formation is characterized by central core l According to this hypothesis, when large amounts of

consisting of rod-shaped bacterial cells, e.g. Fusobac- plaque are allowed to accumulate, the toxic and viru-
terium nucleatum and coccal cells (streptococci) lent factors produced by this increased mass of
which attach along the surface of the rod-shaped cells. plaque leads to periodontal disease. When only small
amounts of plaque are present; the noxious products
(SE Q.4 and SN Q.12) are neutralized by the host.
l Inherent concept of nonspecific plaque hypothesis
{(Subgingival plaque states that control of periodontal diseases depends on
i. Subgingival plaque is found below the gingival
control of the amount of plaque accumulation.
margin, between the tooth and the gingival pocket
epithelium. (SE Q.5 and SN Q.4)
ii. The subgingival plaque differs from supragingival
plaque, primarily because of the local availability {(Specific plaque hypothesis
of blood products and a low oxidation-reduction l According to this hypothesis, only certain micro-
(redox) potential, which characterize the anaerobic organisms in plaque are pathogenic. An increase
environment. It contains many large filaments with in the number of these specific bacteria will pro-
flagella and is rich in spirochaetes. duce periodontal disease due to the release of
iii. Both morphologic and microbiologic studies of virulent factors by these bacteria.
subgingival plaque reveal distinctions between the l Presence of specific bacterial pathogens in plaque
tooth-associated and tissue-associated regions of produces substances that mediate the destruction of
subgingival plaque. host tissue.)}
Tooth-associated plaque is similar to supragingi-
val plaque whereas tissue-associated plaque [SE Q.5]
is covered with flagellated bacteria without a
well-defined extracellular matrix and numerous {
l  In order to identify the periodontal pathogens re-
bristle-brush formations. This arrangement is also sponsible for causing a particular form of periodon-
called ‘test tube–brush’ formation.)} tal disease, Sigmund Socransky put forward the
following criteria.
l Socransky’s criteria based on Koch’s postulates are
[SE Q.4]
as follows:
{iv. Test tube–brush formation is characterized by large
i. A potential pathogen associated with disease
filaments that form the long axis, and short filaments should be increased in number at diseased sites.
Section | I  Topic-Wise Solved Questions of Previous Years 783

ii. After treatment it should be decreased in number complements through alternative pathway. They can
at sites that show clinical improvement. cause intravascular coagulation and tissue necrosis
iii. It should produce some form of cellular or hu- and have cytotoxic effects on fibroblasts and other
moural immune response in the host. cells.
iv. When experimentally inoculated into animal l Endotoxins also have the ability to produce leukope-
models, it should be capable of causing the same nia and may play a pivotal role in the pathogenesis of
disease. periodontal diseases.
v. The pathogen should possess virulence factors l The endotoxins present in the cemental wall of peri-
capable of causing periodontal tissue de- odontal pockets prevent the attachment of gingival
struction. fibroblasts and also induce irreversible morphologi-
Putative periodontal pathogens such as A. actino- cal changes in the cells of the culture.
mycetemcomitans and P. gingivalis seem to fulfil l Peptidoglycan, which is a cell wall compound found
Socransky’s criteria.} in many bacteria, is capable of stimulating bone re-
Role of plaque and virulence mechanisms of periodon- sorption and inducing macrophages to produce pros-
tal pathogens in periodontal diseases taglandin and collagenases.
l Pellicle on the nonshedding hard surface provides a l Tissue destruction is also brought about by capsular
substrate on which bacteria progressively accumu- substances found on the outermost surface of many
late to form dental plaque. bacterial cells and by toxic products such as volatile
l In clinically healthy periodontium, more than 65% of sulphur compounds, butyric and propionic acid, in-
the microbial population within the gingival sulci is dole and ammonia.
made up of Gram-positive cocci. Fusiform bacilli,
Q.3. Describe in detail the steps in the formation of den-
motile rods, filaments and spirochaetes are also pres-
tal plaque.
ent. Gram-negative organisms are less frequently
present. Ans.
l Many periodontal pathogens after invading the peri-

odontal tissues successfully evade host defences, [SE Q.6]


which allow them to infect and damage the peri- {Dental plaque is defined clinically as a structured, resilient,
odontal tissues. Having entered into the host tis- yellowish-grey substance consisting of bacterial aggrega-
sues, pathogens such as A. actinomycetemcomitans tions that adheres tenaciously to teeth and other intraoral
and P. gingivalis successfully suppress host defence hard surfaces such as restorations.}
cells. A. actinomycetemcomitans produces an exo-
toxin known as leukotoxin, which inhibits PMN (SN Q.6 and SE Q.6)
function and is also capable of killing mature B and {(Formation of dental plaque
T cells. IgA, IgG proteases, fibrinolysin catalase l The process of plaque formation can be divided into
and superoxide dismutase are some of the bacterial the following three major phases which occur in a
factors that are important in the evasion of host sequence:
defences. i. Formation of organic pellicle on the tooth surface
l Pathogenic bacteria also release a number of enzymes
ii. Initial adhesion and attachment of bacteria
such as collagenases, hyaluronidase, gelatinase, ami- iii. Colonization and plaque maturation
nopeptidases, acid and alkaline phosphatases, which
lead to tissue destruction and manifestation of peri- i. Formation of organic pellicle)}
odontal disease.
l The lipopolysaccharides (LPS) of Gram-negative (SE Q.6 and SN Q.2)
bacterial cell walls consist of a complex lipid (lipid
A) attached to a polysaccharide. This LPS is called {(l  ll oral surfaces both hard and soft tissues of
A
endotoxin because it is firmly bound to the cell the oral cavity are coated with a pellicle.
surface and is released only when the cells are l A thin, saliva-derived layer, called the acquired
lysed. pellicle, covers the tooth surface and consists
l The presence of abundant Gram-negative bacteria in of numerous components, including glycopro-
periodontal pockets lead to release of endotoxins that teins (mucins), proline-rich proteins, phospho-
interact with receptors on macrophages and mono- proteins (e.g. statherin), histidine-rich proteins,
cytes leading to release of cytokines such as IL 1 and enzymes (e.g. a-amylase) and other molecules
tumour necrosis factor (TNF). They also activate that can function as adhesion sites for bacteria.
784 Quick Review Series for BDS 4th Year, Vol 1

l This involves the adsorption of positively charged specific interactions, i.e. covalent, ionic or hy-
salivary, crevicular fluid and other environmental drogen bonding. This follows direct contact or
macromolecules to negatively charged hydroxyapa- bridging through extracellular filamentous ap-
tite surfaces of teeth through electrostatic, van der pendages.
Waals and hydrophobic forces.)} l On a rough surface, bacteria are better pro-

tected against shear forces so that a change


[SE Q.6] from reversible to irreversible bonding occurs
{  Although this pellicle is protective in nature, provid-
l
more easily and more frequently.
ing lubrication and preventing tissue desiccation, its Phase 4: Colonization of the surface and biofilm
formation on the teeth surfaces forms the substrate formation
l When the firmly attached microorganisms start
for colonization and subsequent proliferation of
microorganisms. growing and the newly formed bacterial clus-
ii. Initial adhesion and attachment of bacteria ters remain attached, microcolonies or a bio-
l Formation of the organic pellicle aids in the adher-
film can develop.
ence of certain bacteria to the tooth surface. These [SE Q.6]
are the initial colonizers and are mainly Gram-
positive facultative microorganisms such as Strep- {iii. Colonization and plaque maturation
l The initial colonizers make use of the available oxy-
tococcus sanguis, Streptococcus mitis and Actino-
myces species. gen, leading to reduced oxygen levels and redox
l Bacterial adherence occurs through specific attach-
potential favouring the growth of Gram-negative
ments, such as extracellular polymeric substances anaerobic organisms such as Fusobacterium nu-
and specific molecules known as adhesins, which cleatum and Capnocytophaga species.
l These organisms adhere to cell surface receptors
attach to the receptors in the dental pellicle.
l Bacteria such as Actinomyces viscosus get attached
of the initial colonizers as they have poor ability
through adhesins present on their surface append- to get attached directly to the pellicle, and hence
ages known as fimbriae. as the plaque matures there is considerable in-
l Evidence suggests that each oral bacterium pos-
crease in the number of Gram-negative bacteria.
l Bacterial adherence to one another is termed as
sesses different cell surface binding sites for attach-
ment and multiplication. coaggregation and it plays an important role in the
l With the multiplication and growth of the primary
process of secondary colonization.
l In the early stages of plaque formation, there is
colonizers, the extracellular matrix also increases
through accumulation of bacterial products such as coaggregation between Gram-negative and Gram-
extracellular polymers.} positive organisms such as F. nucleatum and
The following sequence of events also clarifies the im- A. viscosus followed by coaggregation in the later
portance of hard-surface characteristics in plaque for- stages between Gram-negative organisms, e.g.
mation. F. nucleatum and P. gingivalis.
l Nutrients are required for growth of the ever-
Phase 1: Transport to the surface
l The first stage involves the initial transport of the
multiplying bacteria within the plaque matrix.
bacterium to the tooth surface. Random contacts The main source of nutrients in supragingival
may occur, e.g. through Brownian motion, sedi- plaque is saliva.
l In established plaque, substances produced
mentation of microorganisms, liquid flow or ac-
tive bacterial movement, i.e. chemotactic activity. by certain species become nutrients that are es-
Phase 2: Initial adhesion sential for the growth of other organisms.
l The second stage results in an initial, reversible
Examples:
adhesion of the bacterium, initiated by the inter- a. Lactate and formate produced by streptococci
action between the bacterium and the surface, and Actinomyces species are used by Veillon-
from a certain distance through long- and short- ella as an energy source.
range forces, including van der Waals attractive b. Hydrogen produced by Veillonella is made use
forces and electrostatic repulsive forces. by other organisms such as Campylobacter.
Phase 3: Attachment c. Some Gram-positive rods are able to produce vita-
l After initial adhesion, a firm anchorage between
min K, which along with hemin serves as growth
bacterium and surface will be established by factor for black pigmenting bacterial species.
Section | I  Topic-Wise Solved Questions of Previous Years 785

l Microbial population in plaque is not only one l The fluid layer bordering the biofilm has a rather sta-
of interdependence but also of competition tionary sublayer and a fluid layer in motion. Nutrient
where one species prevents the colonization of components penetrate this fluid medium by molecular
another species, e.g. streptococci, while mak- diffusion.
ing use of available oxygen, produces superox- l Especially for oxygen, a steep diffusion gradient exists

ide anions (O2), hydrogen peroxide (H2O2) and in the more compact lower regions of biofilm, which
hydroxyl radicals, which are bactericidal for further explains changes in microbial composition.
many other oral bacteria such as Aggregati- l The dental plaque biofilm has a similar structure. It is

bacter actinomycetemcomitans. heterogeneous in structure, with clear evidence of open


l Recently, an analysis of 13,000 plaque samples fluid-filled channels running through the plaque mass.
showed microorganisms to exist in com- These water channels permit the passage of nutrients
plexes.} and other agents throughout the biofilm, acting as a
l The early colonizers were in the yellow primitive ‘circulatory’ system.
(S. sanguis, S. mitis and S. oralis) and purple l Nutrients make contact with the attached microcolonies

complexes (A. odontolyticus and V. parvula). by diffusion from the water channels to the microcolony
l The secondary colonizers were in the green rather than from the matrix.
(E. corrodens, Capnocytophaga, and A. acti- l The bacteria exist and proliferate within the intercellu-

nomycetemcomitans), orange (Prevotella, lar matrix through which the channels run. The matrix
Campylobacter and F. nucleatum) and red confers a specialized environment, which distinguishes
complexes (P. gingivalis, T. forsythia and bacteria that exist within the biofilm from those that are
T. denticola). free floating, the so-called planktonic state in solutions
These red complex organisms are associated such as saliva or crevicular fluid. The biofilm matrix
with bleeding on probing. functions as a barrier.
l Substances produced by bacteria within the biofilm are

Q.4. Define dental plaque. Describe the characteristics retained and essentially concentrated, which fosters
of the gel-like matrix of ‘biofilm’. metabolic interactions among different bacteria.
l The intercellular matrix consists of organic and inor-
Ans. ganic materials derived from saliva, gingival crevicular
l Dental plaque is defined clinically as a structured, resil- fluid, and bacterial products.
ient, yellowish-grey substance consisting of bacterial l Organic constituents of the matrix include polysaccha-

aggregations that adheres tenaciously to teeth and other rides, proteins, glycoproteins and lipid material.
intraoral hard surfaces such as removable and fixed res- l Albumin, probably originating from crevicular fluid,

torations. has been identified as a component of the plaque matrix.


l Biofilms are defined as matrix-enclosed bacterial popu- The lipid material consists of debris from the mem-
lation adherent to each other and to surface or interfaces branes of disrupted bacterial and host cells and possibly
(by Costerton). food debris. Glycoproteins from saliva are an important
l According to the recent data (Widerer and Charaklis, component of the pellicle, which initially coats a clean
1989), biofilm is defined as the relatively undefinable tooth surface but they also become incorporated into the
microbial community associated with a tooth surface or developing plaque biofilm.
any other hard and nonshedding material. l The predominant form of polysaccharides produced by

l Plaque is primarily composed of bacteria in a matrix of bacteria, called dextran, also contribute to the organic
salivary glycoproteins and extracellular polysaccha- portion of the matrix. It plays a major role in maintain-
rides. ing the integrity of the biofilm.
l In general, biofilms have an organized structure. They l The inorganic components of plaque are predominantly

are composed of microcolonies of bacterial cells, non- calcium and phosphorus, with trace amounts of other
randomly distributed in a shaped matrix or glycocalyx. minerals, including sodium, potassium and fluoride.
l In the lower plaque layers, which are dense, microbes l The source of inorganic constituents of supragingival

are bound together in a polysaccharide matrix with plaque is primarily saliva. As the mineral content in-
other organic and inorganic materials. On top of the creases, the plaque mass becomes calcified to form
lower layer, a looser layer appears which is often irregu- calculus.
lar in appearance; it can extend into the surrounding l Calculus is frequently found in areas of the dentition adja-

medium. cent to salivary ducts (e.g. lingual surface of mandibular


786 Quick Review Series for BDS 4th Year, Vol 1

incisors and canines, buccal surface of maxillary first mo- Q.11. Define plaque. Describe in detail the composition,
lars), reflecting the high concentration of minerals avail- structure and formation of dental plaque.
able from saliva in those regions.
Ans.
l The inorganic components of subgingival plaque are

derived from crevicular fluid. Calcification of subgingi- [Same as LE Q.3]


val plaque also results in calculus formation. Subgingi-
Q.12. Define dental biofilm and highlight the properties
val calculus is typically dark green or dark brown, prob-
of the same.
ably reflecting the presence of blood products associated
with subgingival haemorrhage. Ans.
l The fluoride component of plaque is largely derived
[Same as LE Q.4]
from external sources such as fluoridated toothpastes,
rinses and fluoridated drinking water. Fluoride is used
therapeutically to aid in remineralization of tooth struc- SHORT ESSAYS:
tures, prevention of demineralization of tooth structures,
and inhibition of the growth of many plaque microor- Q.1. Structure and composition of plaque.
ganisms.
Ans.
Q.5. Discuss supragingival and subgingival plaque. [Ref LE Q.1]
Ans. Q.2. Compare supragingival and subgingival plaque.
[Same as LE Q.1] Ans.

Comparison between supragingival and subgingival plaque


Q.6. Describe the role of microorganisms in the aetiol-
is depicted in the following table:
ogy of periodontal diseases.
Features Supragingival plaque Subgingival plaque
Ans.
Location Present coronal to the Present apical to the
[Same as LE Q.2] margin of free gingiva free gingival
Distribution Areas left uncleaned, Attached plaque cov-
Q.7. Discuss the role of plaque in the aetiology of peri- e.g. cervical third and ers the calculus and
odontal diseases. proximal areas unattached plaque
extends to the peri-
Ans. odontal attachment

[Same as LE Q.2] Retentive Rough surface of the Overhanging margins


areas teeth or restoration, and periodontal
malposed teeth and pockets
Q.8. What is dental plaque? Describe its role in the carious lesions
initiation and progression of gingival and periodontal
Structure Adherent, densely Unattached plaque,
diseases.
packed microbial layer tooth surface attached
Ans. over pellicle on the plaque and epithelial
tooth surface attached plaque
[Same as LE Q.2] Composition Early plaque consists Early plaque has
of Gram-positive cocci anaerobic population
Q.9. Describe in detail the plaque retention as aetiologi-
Older plaque has Older plaque consists
cal factor of periodontal diseases. increased number of of Gram2ve, motile,
filaments, fusiform ba- rods and spirochaetes
Ans. cilli and spirochaetes
[Same as LE Q.2] Metabolites Predominantly Predominantly
carbohydrates proteins
Q.10. Define dental plaque. Write in detail about the Nutrition for Saliva and ingested GCF, exudate and
formation of plaque. Add a note on specific plaque bacteria food leukocytes
hypothesis.
Significance Causes gingivitis, Causes gingivitis,
Ans. supragingival calculus periodontal infection
and dental caries and subgingival
[Same as LE Q.3] calculus
Section | I  Topic-Wise Solved Questions of Previous Years 787

Q.3. Composition of dental plaque in adult periodontitis. Q.6. Mechanism of formation of plaque.
Ans. Ans.
Numerous types of periodontal diseases are found in adult [Ref LE Q.3]
population. Composition of plaque in various types of peri-
Q.7. Socransky’s modification of Koch’s postulates.
odontitis is as follows:
Chronic periodontitis Ans.
l In chronic periodontitis, a striking increase in
l Sigmund Socransky, a researcher at the Forsyth Dental
the number of Gram-negative bacteria such as
Center in Boston, proposed criteria by which periodontal
P. gingivalis, P. intermedia, A. actinomycetem-
microorganisms may be judged to be potential pathogens.
comitans, Bacteroides species, Campylobacter
l Socransky’s criteria based on Koch’s postulates are as
rectus, Eikenella species and spirochaetes are
follows:
seen in the active sites.
i. A potential pathogen associated with disease should
l Viruses such as Epstein–Barr virus-1 (EBV-1)
be increased in number at diseased sites.
(HCMV) may also have a role in the pathogenesis of
ii. After treatment it should be decreased in number at
chronic periodontitis.
sites that show clinical improvement.
Localized aggressive periodontitis
iii. It should produce some form of cellular or
l In localized aggressive periodontitis the most fre-
humoural immune response in the host.
quently detected microorganisms are A. actinomy-
iv. When experimentally inoculated into animal mod-
cetemcomitans, Capnocytophaga species, Eikenella
els, it should be capable of causing the same disease.
corrodens, P. intermedia and Campylobacter rectus.
v. The pathogen should possess virulence factors,
Of these, A. actinomycetemcomitans has been re-
capable of causing periodontal tissue destruction.
garded as a key aetiological agent due to the follow-
l Putative periodontal pathogens such as A. actinomy-
ing findings:
cetemcomitans and P. gingivalis seem to fulfil Socran-
i. From 90% of the lesions of localized aggressive
sky’s criteria.
periodontitis, A. actinomycetemcomitans was
isolated. Q.8. Differences between supra- and subgingival plaque.
ii. Elevated levels of the organism were observed in
Ans.
sites showing evidence of ongoing periodontal
destruction. [Same as SE Q.2]
iii. A significantly elevated levels of serum antibod-
Q.9. Define plaque and add a note on its mechanism of
ies to A. actinomycetemcomitans in LAP pa-
formation.
tients were reported.
iv. Clinical studies have shown a significant corre- Ans.
lation between successful treatment and elimi-
[Same as SE Q.6]
nation of A. actinomycetemcomitans and vice
versa. Q.10. Socransky’s criteria for identification of periodon-
v. A. actinomycetemcomitans produce several viru- tal pathogens.
lence factors, including a leukotoxin capable of
Ans.
inducing disease.
Generalized aggressive periodontitis [Same as SE Q.7]
l Generalized aggressive periodontitis is frequently

associated with Porphyromonas gingivalis, Bacteroi-


des forsythus and A. actinomycetemcomitans.
SHORT NOTES:
Q.1. Dental pellicle.
Q.4. Subgingival plaque.
Ans.
Ans.
l All oral surfaces, both hard and soft tissues, of the oral
[Ref LE Q.1] cavity are coated with a pellicle within few seconds fol-
lowing thorough cleaning and polishing of teeth.
Q.5. Specific plaque hypothesis. l Pellicle is the initial organic structure that is formed on

the surfaces of teeth and artificial prosthesis.


Ans.
l The first stage in pellicle formation involves adsorption

[Ref LE Q.2] of salivary proteins to apatite surfaces.


788 Quick Review Series for BDS 4th Year, Vol 1

l This results from the electrostatic ionic interaction be- l It is an ideal growth medium for the bacteria which can
tween hydroxyapatite surface which has negative charge induce gingival inflammation.
that interacts with opposite charged groups in the sali-
vary macromolecules. Q.8. Normal oral bacterial flora.
l The mean pellicle thickness varies from 100 nm at 2 h Ans.
to 500 to 1000 nm.
l The transition from pellicle to dental plaque is ex-
l Normal bacterial flora, associated with clinically healthy
tremely rapid. periodontium, comprises 65% of microbial population
made up of Gram-positive cocci ; Fusiform bacilli, mo-
Q.2. Acquired pellicle. tile rods, filaments and spirochaetes are also present.
l Gram-negative organisms and motile rods are seen less
Ans.
frequently.
[Ref LE Q.3]
Q.9. Coaggregation.
Q.3. Aetiological significance of microbial plaque.
Ans.
Ans.
l In the process of plaque formation, the bacterial adher-
Aetiological role of plaque in periodontal diseases: ence to one another is termed as coaggregation and it
l It was recognized and well known that dental plaque
plays an important role in the process of secondary
plays an important role in the initiation and progres- colonization.
sion of periodontal diseases. l In the early stages of plaque formation there is coaggre-
l First it was thought that the severity of periodontal
gation between Gram-negative and Gram-positive organ-
disease was associated with the quantity of plaque isms such as F. nucleatum and A. viscosus followed by
formed. This is known as the nonspecific plaque coaggregation in the later stages between Gram-negative
hypothesis. organisms, e.g. F. nucleatum and P. gingivalis.
l According to this hypothesis, when large amounts of
l Nutrients are required for the growth of the ever-multi-
plaque are allowed to accumulate, the toxic and viru- plying bacteria within the plaque matrix.
lent factors produced by this increased mass of
plaque leads to periodontal diseases. Q.10. Spirochaetes.
l Inherent concept of nonspecific plaque hypothesis
Ans.
states that control of periodontal diseases depends on
control of the amount of plaque accumulation. l Spirochaetes are corkscrew-like Gram-negative anaero-
bic bacteria that can be readily demonstrated by dark
Q.4. Specific plaque hypothesis.
field and phase contrast microscopic examination.
Ans. l They contain endotoxin that contribute to their pathoge-

nicity. They are capable of immunosuppression.


[Ref LE Q.2]
l Spirochaetes are ubiquitous to the subgingival plaque
Q.5. Define and classify dental plaque. and are part of oral flora.
l They probably contribute to gingival pathology when
Ans.
their numbers increase beyond a certain threshold
[Ref LE Q.1] level.
l Treponema vincentii, which is usually referred to as
Q.6. Name the stages of plaque formation.
Borrelia vincentii, is implicated in necrotizing ulcer-
Ans. ative gingivitis.
l Many studies have shown increased levels and propor-
[Ref LE Q.3]
tions of spirochaetes in chronic periodontitis and ag-
Q.7. Materia alba. gressive forms of periodontitis.
l An uncultivable spirochaete that reacts with monoclo-
Ans.
nal antibodies specific for Treponema pallidum, re-
l Materia alba is a white and soft deposit that occurs ferred to as pathogen-related oral spirochaete (PROS),
around the necks of the teeth in the unhygienic condi- is the dominant species in some plaque samples and
tions. could be important in the pathogenesis of periodontal
l Composition: Food debris, dead tissue elements, puru- diseases.
lent material and mixture of salivary proteins and lipids. l A decrease in the levels of spirochaetes, such as

l It is soft and sticky in consistency. T. Denticola, results in improvement in periodontal health.


Section | I  Topic-Wise Solved Questions of Previous Years 789

Q.11. Prevotella intermedia. l They produce superoxide dismutase and resist phagocy-
tosis and intracellular killing by immunocompetent
Ans.
cells. Of special interest is the fact that P. intermedia
l Prevotella intermedia are a set of black-pigmented bac- commonly produces beta-lactamase and may be resis-
teria which form black colonies, produce indole and tant to penicillin and other antibiotics.
ferment sucrose. l P. intermedia is associated with pregnancy gingivitis,

l The colonies of P. intermedia, when exposed to ultravio- necrotizing ulcerative gingivitis and chronic peri-
let light, give off a bright red fluorescence that is helpful odontitis.
in differentiating them from other black colonies.
Q.12. Subgingival plaque.
l P. intermedia release various toxic factors such as endo-

toxin, epitheliotoxin, gelatinase, acid and alkaline phos- Ans.


phatase, trypsin-like enzyme and also IgA and IgG
[Ref LE Q.1]
proteases.

Topic 7
Dental Calculus, Iatrogenic and Other Local
Predisposing Aetiological Factors
COMMONLY ASKED QUESTIONS
LONG ESSAYS:
1 . Define calculus. Mention its structure and composition. Discuss various theories of calculus formation.
2. What is calculus? What are the theories of calculus formation? Write about the composition and types of calcu-
lus. [Same as LE Q.1]

SHORT ESSAYS:
1 . Define calculus and mention its composition. [Ref LE Q.1]
2. Theories of calculus formation. [Ref LE Q.1]
3. Subgingival calculus.
4. Mode of attachment of calculus.
5. Describe briefly the formation of calculus.
6. Difference between supra- and subgingival calculus.
7. Define dental calculus. Discuss theories of mineralization of dental calculus. [Same as SE Q.2]

SHORT NOTES:
1 . Aetiological significance of calculus in periodontitis.
2. Define calculus and enumerate various theories of mineralization of calculus.
3. Inorganic composition of dental calculus. [Ref LE Q.1]
4. Epitactic concept in calculus formation.
5. Supragingival calculus.
6. Attachment of calculus. [Ref SE Q.4]
7. What is the role of calculus in periodontitis? [Same as SN Q.1]
8. Name theories regarding mineralization of calculus. [Same as SN Q.2]
9. State the inorganic composition of dental calculus. [Same as SN Q.3]
790 Quick Review Series for BDS 4th Year, Vol 1

SOLVED ANSWERS
LONG ESSAYS:
Q.1. Define calculus. Mention its structure and composi- [SE Q.1]
tion. Discuss various theories of calculus formation.
{   These crystal forms do not occur with the same
l
Ans. frequency in all calculus samples. Their incidence
varies with the age and the location of the calculus.
[SE Q.1]
l In about 90%–100% samples of all supragingival
{Dental calculus is defined as an adherent calcified or cal- calculus, hydroxyapatite and octacalcium phosphate
cifying amorphous mass that forms on the surface of natu- are the most common crystal forms. While brushite
ral teeth, restorations and dental prosthesis.} is more common in the mandibular anterior region,
Structure magnesium whitlockite is common in the posterior
l A very significant feature of calculus is that its sur- areas.
face is covered by a layer of unmineralized plaque. Organic components
l Calculus is a very porous substance that contains nu- l Much of the organic content of calculus is a mix-
merous spaces within the calcified matrix. These spaces ture of protein polysaccharide complexes, minor
may represent uncalcified bacteria, or they may be seen fraction of lipids, desquamated epithelial cells,
around individual calcified organisms themselves. leukocytes and various types of microorganisms.
l Microscopically, the supragingival calculus is often a l Salivary protein: 5.9%–8.2%, includes most of
layered structure in which the degree of calcification var- the amino acids.
ies from layer to layer, i.e. it is heterogeneous in nature. l Polysaccharides: 1.9%–9.1%.
l Subgingival calculus appears to be more homoge- These are derived largely from proteoglycans of
neous since it is built in layers with almost equal bacteria and salivary glycoproteins, e.g. galactose,
density of minerals. glucose, rhamnose, mannose and galactosamine.
l Lipids: 0.2% of neutral fats, free fatty acids, cho-
(SE Q.1 and SN Q.3)
lesterol, cholesterol esters and phospholipids.
{(Composition These are probably derived from the cell walls of
Dental calculus consists of 70%–90% of inorganic bacteria that are enveloped within the calculus
salts and 10%–30% organic components. during mineralization.}
Inorganic components
[SE Q.2]
The principal inorganic components are as follows:
Compounds {Various theories of calculus formation are as follows:
l Calcium phosphate – 75.9% A. Precipitation therapy – booster mechanism
l Calcium carbonate – 3.1 % B. Epitactic concept/heterogenous nucleation concept
l Magnesium phosphate and other metals – C. Inhibition theory
trace amounts A. Precipitation theory
Elements Precipitation of minerals can occur from a local rise
l Calcium – 39%
in degree of saturation of calcium and phosphate
l Phosphorus – 19%
ions.
l Magnesium – 0.8%
i. Booster mechanism
l Sodium, zinc, bromine, copper, silicon,
This is the main mechanism of precipitation
iron, and fluorine – trace amounts which states that precipitation of calcium phos-
Crystal forms phate salts results from a local rise in the degree
About two-thirds of the inorganic component of saturation of calcium and phosphate ions be-
is crystalline in structure and the four main cause of the following ways:
crystal forms in which it exists are as follows: a. Increase in pH of saliva due to:
l Hydroxyapatite – 58%, appears as sand
l loss of carbon dioxide, or
grain or rod-like crystals l production of ammonia by dental plaque
l Magnesium whitlockite – 21%, hexago-
bacteria, or
nal (cuboidal, rhomboidal) crystals l protein degradation during stagnation re-
l Octacalcium phosphate – 21%, platelet-
sults in precipitation of calcium and
like crystals phosphate salts by lowering the precipi-
l Brushite – 9%)}
tation constant
Section | I  Topic-Wise Solved Questions of Previous Years 791

b. Colloidal proteins in saliva bind calcium and l The seeding agents induce small foci of calcifica-
phosphate ions and maintain a supersaturated tion. These foci enlarge and coalesce to form
solution. When saliva stagnates in the oral cav- calculus. Hence, it is more appropriately called
ity, colloids settle down and supersaturated heterogenous nucleation.
stage is no longer maintained. It results in the C. Inhibition theory
precipitation of calcium and phosphorus salts. l According to this theory, the possibility of calci-

c. Role of the enzymes in precipitation of cal- fied mass occurring only at specific sites is be-
cium phosphate salts: cause of existence of an inhibiting mechanism at
l Phosphatases non-calcifying site.
l When calcification occurs, the inhibitors are ap-
The phosphatases liberated from plaque,
desquamated epithelial cells and bacteria parently removed or altered.
l Pyrophosphate is one possible inhibiting substance

thought to inhibit calcification by preventing the


Hydrolyse organic phosphates in saliva initial nucleus from growing by ‘poisoning’ the
growth centre of the crystal.}

Q.2. What is calculus? What are the theories of calculus


Increased concentration of free phosphate
formation? Write about the composition and types of
ions
calculus.
Ans.
Initiate mineralization of plaque
[Same as LE Q.1]
l Esterases
The esterases present in dental plaque, bacteria,
leukocytes, macrophages, desquamated SHORT ESSAYS:
epithelial cells
Q.1. Define calculus and mention its composition.
Ans.
Hydrolyse fatty esters into free fatty acids
[Ref LE Q.1]

Q.2. Theories of calculus formation.


Fatty acids form soap with calcium and
magnesium Ans.
[Ref LE Q.1]
Convert to less soluble calcium phosphate salts Q.3. Subgingival calculus.
Ans.
B. Epitactic concept/heterogenous nucleation concept l Dental calculus is classified according to its relation to
l According to this concept, calculus formation the gingival margin as supragingival or subgingival
may be initiated through epitaxis by organic com- calculus.
plexes in the matrix. l Subgingival calculus is formed by the gingival exudate,

l Epitaxis means crystal formation of a compound hence it is also called serumal calculus or submarginal
through seeding. The formation of the initial crys- calculus.
tal or nucleus is called nucleation. Clinical features
l The intercellular matrix provides the architectural l It is seen as dark brown, green calcified deposit on

template for the initial hydroxyapatite crystal. the root surface of a tooth below the free marginal
l This focus of calcification enlarges and coalesces gingiva.
to form a calcified mass. l It is found particularly within the gingival sulcus or

l Seeding or nucleating agents in calculus are not periodontal pockets.


clearly known. They are suspected to be the l It appears first either independently or on the inter-

intercellular matrix of plaque, plaque bacteria or proximal areas where supragingival calculus already
lipid component of organic matrix. exists.
792 Quick Review Series for BDS 4th Year, Vol 1

l It is occasionally seen on the dentures when it forms Q.5. Describe briefly the formation of calculus.
in narrow grooves, such as those at the tooth–acrylic
Ans.
interface.
l It is flint-like in consistency, and is firmly adherent to l Dental plaque that has undergone mineralization is
the surface and difficult to remove. known as calculus.
l Not visible on routine oral examination, may be l It is formed by the precipitation of mineral salts which

found by tactile exploration. can start between 1st and 14th day of plaque formation.
l It is diagnosed by the following: l Within first two days plaque can be 50% mineralized

Direct vision and within 12 days 60%–90% mineralization occurs.


It is seen directly: l On the inner surface of the plaque, calcification starts in

a. By using a gentle stream of air to reflect the separate foci, they gradually increase in size and co-
gingival margin alesce to form a solid mass of calculus.
b. By following gingival surgery l Calculus formation continues until it reaches maximum

c. During periodontal surgery levels in about 10 weeks and 6 months, after which
Indirect vision there is a decline in its formation due to mechanical
It is seen indirectly: wear from food and from the lips, cheeks and tongue.
a. When gingival margin is dried, the dark This decline is referred to as reversal phenomenon.
colour of subgingival calculus may be seen Formation of calculus is summarized as follows:
through the marginal tissue.
b. By probing: Attached plaque
l A fine calculus probe (WHO 621 probe)

can be used to detect the subgingival


calculus. Between the 1st and 14th day of plaque formation
l The ball end of the CPITN probe with

light touch helps determining the location


of subgingival calculus, which appears as Precipitation of mineral salts (saliva/GCF)
a rough area or catch on the root surface.
c. Exploring: The use of the subgingival ex-
plorer No. TU-17 can detect the subgingival Early plaque of heavy calculus formers contain more
calculus. calcium, thrice more phosphorus and less potassium
d. Radiographs and transillumination. (phosphorus may be more critical than calcium in
plaque mineralization)
Q.4. Mode of attachment of calculus.
Ans.
Crystal formation initially in the intercellular matrix
{SN Q.6} and on the bacterial surfaces and then within
the bacteria
The four modes of attachment of calculus to tooth sur-
Q.6. Difference between supra- and subgingival calculus.
face are described as follows:
i. Attachment by means of an organic pellicle or Ans.
cuticle that seems to predominate on the enamel
According to its relation to the gingival margin, the dental
but it occurs only infrequently on the cementum.
calculus is classified as supragingival and subgingival
ii. Although not accepted by some investigators,
calculus.
penetration of calculus bacteria into the cemen-
tum is also a mode of attachment. Supragingival Subgingival
iii. Mechanical interlocking into surface irregulari- Features calculus calculus
ties such as resorption lacunae and caries. Location Above the free gin- Below the crest
iv. Close adaptation of calculus undersurface de- gival margin of free gingival
pressions to the gently sloping mounds of the margin
unaltered cementum surface. Source Derived from the Derived from the
components of sal- gingival fluid
ivary secretions, exudate, hence
v. Calculocementum refers to calculus embedded deeply hence known as known as serumal
in cementum, which may appear morphologically simi- salivary calculus calculus
lar to cementum.
Section | I  Topic-Wise Solved Questions of Previous Years 793

Supragingival Subgingival ii. It brings plaque bacteria close to the supporting


Features calculus calculus tissues.
iii. Interferes with local self-cleansing defence
Distribution Symmetrical Related to pocket
arrangement on depth, heavier on
mechanisms.
teeth, more on proximal surfaces iv. Acts as a reservoir for irritating substances such
facial surfaces of as endotoxins, antigenic material and bone-re-
maxillary molars sorbing factors because of its permeability and
and lingual porous nature.
surfaces of
mandibular
iv. Makes plaque removal more difficult for the
anterior teeth patient.
Visibility Clinically visible in Not visible on Q.2. Define calculus and enumerate various theories of
the oral cavity routine oral mineralization of calculus.
examination
Detected by tactile Ans.
exploration
Dental calculus is defined as an adherent calcified or calci-
Colour It is white, yellowish- Dark brown or
fying amorphous mass that forms on the surface of natural
white in colour, greenish black in
stained by tobacco colour teeth, restorations and dental prosthesis.
and food pigments Various theories of calculus formation are as follows:
A. Precipitation theory – booster mechanism
Consistency Hard and clay-like Hard and firm/flint
or glass-like B. Epitactic concept/heterogenous nucleation concept
C. Inhibition theory
Composition Calcium phosphate Calcium phosphate
ratio less than sub- ratio is more than Q.3. Inorganic composition of dental calculus.
gingival calculus. supragingival
Less sodium calculus. Sodium Ans.
content content increases
with the depth of [Ref LE Q.1]
pocket, more mag-
nesium whitlockite
Q.4. Epitactic concept in calculus formation.
and less brushite Ans.
Ease of removal Easily detached Firmly attached to
from the tooth the tooth surface
Epitactic concept is also called heterogenous nucleation
concept in calculus formation and is as follows:
Q.7. Define dental calculus. Discuss theories of mineral- Cellular matrix of plaque
ization of dental calculus.
Ans.
Acts as seeding agent
[Same as SE Q.2]

Induction of formation of small foci of calcification


SHORT NOTES:
Q.1. Aetiological significance of calculus in periodontitis.
Enlargement of foci
Ans.
Aetiological significance of calculus in periodontitis is as
follows: Coalesce of foci
l Before 1960, it was believed that calculus was the

principal aetiological factor in periodontal diseases.


l According to the current view, the initial damage to
Formation of calcified mass
the gingival margin in periodontal diseases is due to
pathological effects of microorganisms in plaque. Calculus formation
The essential role of calculus in periodontal disease
may be summarized as follows:
Q.5. Supragingival calculus.
i. Calculus provides a fixed nidus for continuous
plaque accumulation. Ans.
794 Quick Review Series for BDS 4th Year, Vol 1

l Supragingival calculus is located above the free gingival Q.7. What is the role of calculus in periodontitis?
margin.
Ans.
l It is derived from the components of salivary secretions,

hence known as salivary calculus. [Same as SN Q.1]


l Clinically visible in the oral cavity.
Q.8. Name theories regarding mineralization of calculus.
l It is white and yellowish-white in colour, stained by

tobacco and food pigments. Ans.


l Hard and clay-like in consistency.
[Same as SN Q.2]
l Symmetrical arrangement on teeth, more on facial sur-

faces of maxillary molars and lingual surfaces of man- Q.9. State the inorganic composition of dental calculus.
dibular anterior teeth.
Ans.
l Easily detached from the tooth.

[Same as SN Q.3]
Q.6. Attachment of calculus.
Ans.
[Ref SE Q.4]

Topic 8
Smoking and Periodontium
COMMONLY ASKED QUESTION
SHORT ESSAY:
1. Smoking and periodontal disease.

SOLVED ANSWER
SHORT ESSAY: Impact of smoking and periodontal disease is as follows:
Q.1. Smoking and periodontal disease. Periodontal
Ans. disease Impact of smoking
Gingivitis Decreased gingival inflammation and bleeding
Effects of smoking on healthy gingival and periodontal on probing
tissues:
Periodontitis Increased pocket depth, attachment loss and
Physiology Decreased GCF flow bone loss
Bleeding on probing Increased prevalence and severity of periodontal
destruction
Decreased subgingival temperature
Increased rate of periodontal destruction
Microbiology Increased levels of periodontal pathogens in
deep pockets Increased prevalence of severe periodontitis
Immunology Altered neutrophil chemotaxis, phagocytosis Increased tooth loss
and oxidative burst
Increased prevalence with increased number of
Increased TNF-a and PGE2 cigarette smoking per day
Increased neutrophil collagenase and Decrease prevalence and severity with smoking
elastase cessation
Section | I  Topic-Wise Solved Questions of Previous Years 795

Topic 9
Host Response: Basic Concepts
COMMONLY ASKED QUESTIONS
SHORT ESSAYS:
1 . Cytokines.
2. Mast cells.
3. Role of macrophages in periodontal disease.
4. Complement system.
5. Lymphocytes.
6. Immunoglobulins.
7. Arthus reaction.

SHORT NOTES:
1. Type I hypersensitivity.
2. Functions of leukocytes.
3. Cytokines.
4. Lymphokines.
5. Active immunity.
6. Neutrophils.
7. Periodontal diseases associated with acute neutrophil disorders.
8. Functions of IgG.
9. Name the functional defects of leukocytes.
10. Anaphylaxis. [Same as SN Q.1]
11. Define cytokines. [Same as SN Q.3]

SOLVED ANSWERS
SHORT ESSAYS:
Q.1. Cytokines. l IL-1, IL-6 and tumour necrosis factor (TNF) ap-
Ans. pear to play a central role in periodontal tissue
destruction.
l Cytokines are soluble proteins secreted by cells which Important cytokines associated with periodontal dis-
act as messenger molecules that transmit signals to eases are as follows:
other cells. l IL-l: It is produced predominantly by macro-
l The family of cytokines includes interleukins, growth
phages and lymphocytes. Fibroblasts, platelets,
factors, chemokines and interferons. They act on fibro- keratinocytes and endothelial cells also release
blasts, macrophages, keratinocytes and Polymorphonu- IL-1. It triggers the release of large quantities of
clear leukocytes (PMNLs) to release matrix metallopro- prostaglandins E2 and stimulates secretion of
teinases (MMPs) that degrade connective tissue matrix. MMPs.
Actions of cytokines are as follows: l IL-2: It is produced by monocytes and T lympho-
l These initiate and maintain immune and inflam-
cytes. It stimulates T cells and enhances clonal
matory responses. expansion of beta cells into plasma cells.
l These regulate growth and differentiation of cells.
l IL-4, IL-5 and IL-10: These are produced by TH2
l Interleukins are involved in communication be-
cells and help in the activation of beta cells
tween leukocytes and other cells, such as epithe- into plasma cells and down-regulate monocytic
lia, endothelia and fibroblasts. response.
796 Quick Review Series for BDS 4th Year, Vol 1

l IL-6: It is released by lymphocytes, fibroblasts l The activated macrophages release a variety of products
and monocytes. It is responsible for conversion of that have both protective and destructive functions.
blood monocytes into osteoclasts. Protective functions of macrophages are as follows:
l IL-8: It is secreted by monocytes, keratinocytes l Phagocytosis, microbial killing and destroy dam-

and fibroblasts. It is a strong chemoattractant of aged tissues


PMNLs at low concentrations. l Stimulate stem cell growth in the bone marrow

l TNF: It is produced by macrophages and release l Antigens processed and presented to lymphocytes

lymphotoxins. It stimulates the proliferation of l Regulate growth of fibroblasts

osteoclast precursor cells and also activates ma- l Synthesize and secrete interferon, complement

ture osteoclasts to resorb bone. components, pyrogen and PMN chemotactic


l Prostaglandin E2: Macrophages and fibroblasts factors
are main sources while IL-1 induces its produc- Destructive functions of macrophages are as follows:
tion. It is a potent mediator of osteoclastic l Secrete prostaglandins and cyclic nucleotides

resorption. l Cytotoxic factors: cytokines

l Acid hydrolases
Q.2. Mast cells.
l Neutral proteases, including collagenase, elastase

Ans. and plasminogen activators


l Mast cells are widely distributed in the connective tis-
Q.4. Complement system.
sue mainly around blood vessels and are also present in
the gingival connective tissue and junctional epithelium. Ans.
l Mast cells’ prominent feature is the presence of cytoplas-
l Complement ‘C’ refers to a series of factors which oc-
mic granules, called lysosomes, which store inflamma-
cur in normal serum and are activated by characteristic
tory mediators like histamine, eosinophil chemotactic
antigen–antibody interaction and subsequently mediate
factor, neutrophil chemotactic factor and heparin.
a number of biologically significant consequences.
l Mast cells can synthesize other inflammatory mediators
l The complement system consists of the following:
like the slow-reacting substances of anaphylaxis (SRS-
i. Complement components, which are nothing but
A), tumour necrosis factor alpha (TNF-a), interleukin-6
chemically and immunologically distinct serum
(IL-6) and leukotriene C4.
proteins, at least 22 in number.
l They contain receptors for IgE antibodies which are
ii. Properdin system.
found in the gingival environment.
iii. Control proteins.
l IgE-carrying mast cells 1 antigen sensitization 0 Degra­
l When the activity of complement is induced by
dation of cell and release of heparin, histamine, SRS-A 1
antigen–antibody interaction or any other stimuli,
proteases, which mediate inflammatory responses.
C components react in a specific sequence as a
l Mast cells are important in immediate inflammation,
cascade.
and more recently mast cells have been shown to ex-
l The C cascade can be triggered off by two parallel but
press toll-like receptors, which allow the innate immune
independent mechanisms or pathways named as fol-
system to adapt which is transitory.
lows:
l The stimulation of these receptors can result in the acti-
i. The classical C pathway
vation and secretion of vasoactive substances that in-
ii. The alternative or properdin pathway
crease vascular permeability and dilation, which are two
l Once activated, a series of biologically active fragments
important signs of anaphylaxis.
are formed by cleavage.
l Localized anaphylaxis is important in initiating inflam-
l Some of the fragments, e.g. C3a and C5a, potentiate
matory responses against local microbial invasion.
inflammatory response and cause an increase in vascu-
Q.3. Role of macrophages in periodontal disease. lar permeability.
l C5a is a potent chemotactic factor for PMNs and
Ans.
monocytes, while C3b facilitates phagocytosis by op-
l Macrophages originate from peripheral blood mono- sonization.
cytes. The monocytes migrate from the blood vessels The functions of complement are as follows:
into the interstitial spaces and differentiate into macro- l It mediates immunological membrane damage

phages. like cytolysis and bacteriolysis.


l The lymphocytes release certain potent agents, which l It participates in the pathogenesis of certain hy-

affect the newly differentiated macrophages and convert persensitivity reactions by amplifying the inflam-
them into activated macrophages. matory response.
Section | I  Topic-Wise Solved Questions of Previous Years 797

l It exhibits antiviral activity and promotes phago- l Before antigen exposure, B cells express im-
cytosis and immune adherence. munoglobulin M (IgM) as part of the BCR.
l It interacts with coagulation, fibrinolytic and After antigen exposure, some B cells differen-
kininogenic systems of blood. tiate to form plasma cells dedicated to the
production and secretion of antibodies of the
Q.5. Lymphocytes.
IgM isotype.
Ans. l Others in the presence of T cells may differen-

tiate along the memory pathway, forming


l Lymphocytes are synthesized in the thymus and bone
memory B cells. They give rise to plasma cells
marrow.
on secondary exposure to antigen and produce
l The three main types of lymphocytes are distinguished
high-affinity antibodies of appropriate isotype.
on the basis of their receptors for antigens:
Natural killer (NK) cells
i. T lymphocytes
l NK cells recognize and kill certain tumour cells
ii. B lymphocytes
and virally infected cells. They possess several
iii. Natural killer (NK) cells
classes of antigen receptors, including killer-
inhibitory receptor (KIR) and killer-activating
The source of origin and function of each type of
receptor (KAR).
lymphocyte are as follows:
l Normal cells possess MHC class I molecules
T lymphocytes B lymphocytes Natural killer cells that present antigens recognized as ‘self’; these
Thymus Liver, spleen, bone Activated by Ag1 interact with KIRs and protect the cells from
marrow macrophages NK cell-mediated killing.
Cell-mediated Humoural – l Alterations in antigens presented by the MHC

immunity immunity class I molecules, occurring in tumour-infected


and virally infected cells, may result in NK-
cell activation. Even the cells can present self-
T cells antigens in response to stress or other altera-
l T cells recognize diverse antigens using a low- tion, which are recognized by the KARs. KAR
affinity transmembranous complex, the T cell activation can override KIR inhibition and
antigen receptor (TCR). cause the NK cell to kill the target cell.
l Based on whether they possess the co-receptors

CD4 or CD8, the T cells are subdivided. The Q.6. Immunoglobulins.


CD4 co-receptor reversibly binds with MHC
Ans.
class II molecules (HLA-DR, HLA-DP, HLA-
DQ) that are found on DCs, macrophages and l The immunoglobulins (Igs) are gamma globulins pro-
B cells. duced by plasma cells in response to antigens with
l CD41 T cells initiate and help with immune which they can react in a specific way.
responses by providing proliferation and dif- l Immunoglobulins are found in blood, tissues and secre-
ferentiation signals. The CD8 co-receptor tions, and are effectors of the humoral response.
scans for MHC class I molecules, which are l Immunoglobulin molecules are composed of two light
found on all cells. chains (k and l) and one of the five types of heavy
l The CD81 T cells are predominantly cyto- chains (g, a, µ, e, d). The class of the Ig molecules is
toxic T cells involved in controlling intracel- denoted by the heavy chain.
lular antigens (e.g. certain bacteria, hyphal l Basic structure of the Ig molecules resembles the letter
fungi, viruses). ‘Y’, where the tail of the Y contains the ends of the
B cells heavy chains ‘Fc fragment’ and complement binding
l The B cells recognize diverse antigens using site. The remaining area of the ‘Y’ contains the light
the B cell antigen receptor (BCR), which is a chains and the remainder of the heavy chains ‘fab/anti-
high-affinity antigen receptor. Extracellular body binding site’.
antigens, such as bacteria, fungal yeast and l The number of binding sites is called the valence of the
virions, are controlled by B cells. molecule.
l Due to the high-affinity interaction between Functions of immunoglobulins
BCR and antigen, the antigen is tightly bound, i. IgG
not scanned. Ingested antigen is degraded and l Complement fixation
presented to T cells. l Delayed antibody response
798 Quick Review Series for BDS 4th Year, Vol 1

l Opsonization The mechanism of type III hypersensitivity is briefly


l Cross-placental barrier summarized as follows:
l Increased concentration in GCF
Immune complex deposited on the basement
ii. IgA membrane of blood vessels causes
l There are two types of IgA: serum IgA and secre-
Activation of complements
tory IgA.
l Serum IgA helps in complement fixation by alter-

nate pathway. Activation of neutrophil


l Secretory IgA has increased concentration in

saliva.
l Secretory IgA protects mucosal surfaces and pre- Liberation of lysozymal enzymes
vents adhesion of bacteria to tissue surfaces, espe-
cially in the early stages of periodontal diseases. causes
iii. IgM Destruction of basement membrane
It helps in early antibody response and complement
fixation.
iv. IgE SHORT NOTES:
It causes immediate hypersensitivity reactions.
v. IgD Q.1. Type I hypersensitivity
The functions of IgD are unknown. Ans.
Q.7. Arthus reaction. i. Type I hypersensitivity is a B cell–mediated immediate
Ans. type of hypersensitivity. The antibody involved is IgE.
ii. Antibodies are fixed on the surface of tissue cells, i.e.
There are two typical type III hypersensitivity reactions: mast cells and basophils in a sensitized individual.
i. Arthus reaction (localized) iii. The antigen combines with the cell-fixed antibody,
ii. Serum sickness (generalized) leading to the release of pharmacologically active sub-
Arthus reaction stances, i.e. vasoactive amines which produce clinical
It is a localized form of type III hypersensitivity. When reaction.
an antigen is injected subcutaneously or intradermally iv. It occurs in two forms:
l The acute, potentially fatal systemic form, called
in an animal in which there were repeated administra-
tions of the same antigen previously, there occurs in- anaphylaxis.
l The chronic or recurrent, nonfatal typically local-
tense local oedema and haemorrhage which reaches
peak in 3–6 h, this is called Arthus reaction. ized form, called atopy.
Mechanism of immune complex hypersensitivity/type III Immediate hypersensitivity reaction/anaphylaxis/
hypersensitivity: type I hypersensitivity reaction:
l Bacterial antigens, antibodies to the bacteria and
Mediated by reaginic Ab–IgE mechanism is as
complement for activation are present in gingival follows:
tissues, and it is observed that immune complexes IgE + mast cells (sensitization)
commonly occur in gingival tissues derived from
patients with periodontal diseases.
l When high levels of antigens are present and per- (+) Allergen
sist without being eliminated, antigen–antibody
(IgG or IgM) complexes precipitate in and around
small blood vessels. Binds to fab portion of IgE
l The subsequent complement activation causes

tissue damage at the site of the local reaction.


Inflammation, haemorrhage and necrosis may Stimulation of mast cells
occur.
l Tissue damage appears to be due to the release of

lysozymal enzymes from various cells such as Release of mediators


neutrophils, mast cells. This reaction is referred to Histamine, SRS-A, bradykinin, platelet-activating
as immune complex or Arthus reaction. factor, prostaglandins, etc.
Section | I  Topic-Wise Solved Questions of Previous Years 799

The role of anaphylactic reactions in the pathogenesis of i. Biologically active substances released by activated
gingival and periodontal disease has not been shown, since T lymphocytes responsible for manifestation of cell-
IgE containing cells are not predominant in the gingival mediated immunity (CMI) are called lymphokines.
tissues. ii. Macrophages under the effect of lymphokines cause
destruction of microorganisms involved in CMI.
Q.2. Functions of leukocytes.
iii. They participate in many functions of T cells and trans-
Ans. mit various growth, differentiation and behavioural
signals between the cells of immune system.
Functions of leukocytes are as follows:
iv. Examples of lymphokines affecting:
i. Phagocytosis: It is a process by which leukocytes
a. Lymphocytes:
engulf bacteria and foreign material of a size visible
l Blastogenic factor (BF)
to light microscopy in an attempt to eliminate infec-
l T cells growth factor (TGF)
tion. Neutrophils and monocytes/macrophages have
l B cells growth factor (BGF)
this ability, and are considered as professional
b. Macrophages:
phagocytes.
l Migration inhibiting factor (MIF)
ii. Chemotaxis: It refers to directed motility that en-
l Macrophage chemotactic factor (MCF)
ables the leukocyte to locate its target. C5a is a
c. Granulocytes:
chemotaxin, which may be generated by any target
l Colony stimulating factor (CSF)
that activates complement.
l Chemotactic factor
iii. Antiallergic effect: Eosinophils inhibit histamine
d. Cultured cells:
release during allergic conditions; hence, their count
l Lymphotoxin (LT)
increases in allergy.
l Interferons (IFs)
iv. Antibody formation: Lymphocytes are mainly re-
sponsible for antibody formation, giving immunity Q.5. Active immunity.
to the body.
Ans.
v. Heparin production: Basophils produce heparin,
which prevents intravascular clotting. i. Active immunity is a type of acquired immunity devel-
vi. Trephone formation: Leukocytes help in the forma- oped during the lifetime of the individual.
tion of trephones from plasma proteins, which are ii. Active immunity is the resistance developed by immu-
needed for the growth and repair of tissues. nity as the result of antigenic stimulus.
iii. Active immunity is of two types: natural and artificial.
Q.3. Cytokines.
iv. Natural active immunity: This is acquired after one in-
Ans. fection or recovery from disease or subclinical infec-
tion after repeated exposures to small doses of infecting
i. Cytokines are peptide mediators or intercellular mes-
organism.
sengers which regulate immunological, inflammatory
Example: A person who has recovered from an attack of
and reparative host responses.
measles develops natural active immunity.
ii. They are produced by widely distributed cells like lym-
v. Majority of adults in developing countries possess natu-
phocytes, macrophages, platelets and fibroblasts.
ral active immunity to poliomyelitis.
iii. They are highly potent hormones, like substances active
vi. Artificial active immunity: It may be acquired artificially
even at femtomolar (10–15 M) concentration.
by inoculation of bacteria, viruses or other products.
iv. In general, they are pleiotropic with multiple effects on
Examples:
growth and differentiation of various cells.
l Killed organisms without changing antigenic
Examples:
structure of bacteria, e.g. typhoid vaccine, cholera
Some of the important cytokines are
vaccine.
i. Interleukins 1–6
l Living organism after proper attenuation, e.g.
ii. Colony stimulating factors (CSF)
small pox and BCG.
iii. TNF
iv. Interferons (IFN) Q.6. Neutrophils
v. Others, like transforming growth factor-b
Ans.
(TGF-b) and leukaemia-inhibitory factor
(LIF). i. Neutrophils constitute 60%–70% of the total leukocyte
count. They are produced in the bone marrow.
Q.4. Lymphokines
ii. They have a multilobed nucleus and the number of
Ans. lobes usually ranges from 3 to 5.
800 Quick Review Series for BDS 4th Year, Vol 1

iii. Cytoplasm contains violet or pink coloured fine Q.9. Name the functional defects of leukocytes.
granules.
Ans.
iv. Neutrophils exhibit phagocytosis and their count in-
creases in acute infections. They form the first line of
defence in the body. Type of Neutrophil Periodontal
Mechanism of action of neutrophils leukocyte defect abnormality manifestations
Protective mechanisms Leukocyte adhe- l Leukocyte func- l In homozygous
i. Pavementing/margination sion deficiency tional defects individuals with
ii. Chemotaxis type 1 (LAD-1) caused by lack defective gene
of integrin b-2 l Aggressive peri-
iii. Phagocytosis: Engulfing and destroying nox- subunit (CD18) odontitis occurs
ious substances such as immune complexes 1 l Neutrophil de- at an early age
damaged tissues, apart from various microor- fects include and affects both
ganisms and antigenic substrates within pha- impaired migra- primary and
golysosomes tion and phago- permanent
cytosis dentition
Q.7. Periodontal diseases associated with acute neutro- Leukocyte adhe- Neutrophils fail to Aggressive
phil disorders. sion deficiency express the ligand periodontitis at
type 2 (LAD-2) (CD15) for P- and young age
Ans. E-selectins, result-
Periodontal diseases associated with neutrophil disorders ing in impaired
transendothelial
are as follows: migration in re-
l Acute necrotizing ulcerative gingivitis (ANUG) sponse to inflam-
l Localized juvenile periodontitis (LJP) mation
l Prepubertal periodontitis (PPP)

l Rapidly progressive periodontitis (RPP)


Q.10. Anaphylaxis.
l Refractory periodontitis (RP)
Ans.
Q.8. Functions of IgG.
[Same as SN Q.1]
Ans.
Q.11. Define cytokines.
Functions of IgG are as follows:
l Complement fixation Ans.
l Delayed antibody response
[Same as SN Q.3]
l Opsonization

l Cross-placental barrier

l Increased concentration in GCF

Topic 10
Host–Microbial Interactions in Periodontal Diseases
COMMONLY ASKED QUESTIONS
SHORT ESSAYS:
1 . Host modulation therapy.
2. Proinflammatory cytokines.

SHORT NOTES:
1 . Prostaglandins.
2. Interleukin-1.
Section | I  Topic-Wise Solved Questions of Previous Years 801

SOLVED ANSWERS
SHORT ESSAYS:
Q.1. Host modulation therapy. IL-1 is produced primarily by activated macrophages
or lymphocytes but also may be released by other
Ans.
cells, including mast cells, fibroblasts, keratinocytes
i. The concept of host modulation was first introduced by and endothelial cells.
William (1990) and Golub et al. (1992). William intro- iii. The IL-1 family also includes the IL-1 receptor an-
duced the concept of NSAIDs, and host modulation tagonist (IL-1ra), which will bind the IL-1 receptor
with tetracycline, and chemically modified analogues without stimulation of the host cell.
was introduced by Golub et al. iv. IL-6, another inflammatory cytokine, leads to bone
ii. The term ‘modulation’ refers to the alteration of func- remodelling.
tion or status of something in response to a stimulus or v. The two forms of TNF are TNF-a and TNF-b. TNF-a
an altered physical or chemical environment. shares many of the same biological activities as IL-1,
iii. ‘Host modulation’ concept in periodontics is the treat- including the stimulation of bone resorption.
ment concept that aims to reduce tissue destruction and vi. The proinflammatory effects of IL-1 and TNF-a in-
even regeneration of the periodontium by modifying clude the following:
the destructive aspects of the host response and upregu- a. Stimulation of endothelial cells to express selectins
lating protective or regenerative process. that facilitate recruitment of leukocytes
iv. To fight against the pathogenic bacteria, normal defen- b. Activation of macrophage IL-1 production
sive immune–inflammatory reaction is essential but c. Induction of prostaglandin E2 (PGE2) by macro-
ameliorating the excessively elevated inflammatory phages and gingival fibroblasts
process to enhance the opportunities for wound healing vii. The properties of these cytokines that relate to tissue
is the host modulation therapy. destruction involve stimulation of bone resorption and
v. Matrix metalloproteinases (MMPs) are a family of induction of tissue-degrading proteinases.
proteolytic enzymes involved in the physiological viii. IL-1 is a potent stimulant of osteoclast proliferation,
degradation of extracellular matrix and basement differentiation and activation. TNF-a has similar ef-
membrane. fects on osteoclasts but is much less potent than IL-1.
Other stimulators of bone resorption are parathyroid ix. Both IL-1 and TNF-a induce production of protein-
hormone (PTH), macrophage colony stimulating factor ases in mesenchymal cells, including MMPs, which
(M-CSF), receptor activator of NFab (RANK), RANK may contribute to connective tissue destruction.
ligand and 1,25-dihydroxy vitamin D3. x. According to substantial data from in vivo studies, the
vi. There are several inhibitors too to combat the stimula- IL-1 and TNF-a are key molecules in the pathogenesis
tors of bone resorption; they are interferon gamma of periodontitis. IL-1, IL-6 and TNF-a are found in
(IFN-g), osteoprotegerin, oestrogens, androgens, calci- significant concentrations in GCF from periodontally
tonin and IL-1ra (receptor antagonist). diseased sites.
vii. Modulation of these host responses is possible and ben- Increasing severity of periodontitis is associated with
eficial. Basically, there are three potential approaches increased concentrations of IL-1 and decreasing con-
to the host modulation: centrations of IL-1ra.
l Use of anti-inflammatory drugs for blocking prosta- xi. Reduction in IL-1 concentration is associated with
glandins and proinflammatory cytokines successful treatment, and elevated levels of IL-6 in
l Inhibiting MMPs with antiproteinases GCF are associated with sites that do not respond well
l Inhibiting activation of osteoclast with sparing in initial nonsurgical phases of therapy.
agents xii. In a primate model of experimental periodontitis, ap-
plication of antagonists to IL-1 and TNF resulted in an
Q.2. Proinflammatory cytokines.
80% reduction in recruitment of inflammatory cells in
Ans. proximity to the alveolar bone and a 60% reduction in
bone loss.
i. The proinflammatory cytokines, interleukin-1 (IL-1),
IL-6 and tumour necrosis factor (TNF), appear to have
a central role in periodontal tissue destruction. SHORT NOTES:
ii. IL-1a and IL-1b are two active forms of IL-1 and both
Q.1. Prostaglandins.
are potent proinflammatory molecules and the main
constituents of ‘osteoclast-activating factor’. Ans.
802 Quick Review Series for BDS 4th Year, Vol 1

i. Major source of prostaglandins are activated macrophages, Q.2. Interleukin-1.


fibroblasts and platelets. IL-1 induces its production.
Ans.
ii. Leukotrienes, prostaglandins and related molecules are
short range hormones that are produced by many cells; i. IL-1 is produced predominantly by macrophages and
they exert their effect locally and are destroyed rapidly lymphocytes; even the fibroblasts, platelets, keratino-
and spontaneously. cytes and endothelial cells also release IL-1.
iii. Prostaglandins and leukotrienes have been detected in ii. It up-regulates adhesion molecules on endothelial cells,
biologically active concentrations in inflammatory exu- lymphocytes, neutrophils and monocytes, and also acti-
dates. Leukotrienes C4, D4 and E4 known as slow- vates T and B lymphocytes and promotes antibody
reacting substances of anaphylaxis (SRS-A) are production.
released from mast cells and basophils. i ii. IL-1 a and b are potent stimulators of connective
iv. Arachidonic acid activation through cyclooxygenase tissue destruction. They trigger the release of large
pathway results in the production of thromboxane, quantities of prostaglandins E2 from fibroblasts
prostaglandins and prostacyclines. Lipoxygenase path- and monocytes, and stimulate secretion of matrix
way results in production of leukotrienes and also metalloproteinases (MMPs), which are a family of
lipoxins (A4 and B4) and 15 epilipoxins. enzymes capable of degrading connective tissue
v. All these comprise primary pathway of alveolar bone matrix.
destruction, which begins with vasodilation, increasing
capillary permeability and bone destruction.

Topic 11
Trauma from Occlusion
COMMONLY ASKED QUESTIONS
LONG ESSAYS:
1 . What is trauma from occlusion? Give the signs, symptoms and treatment of traumatic occlusion.
2. Define and classify trauma from occlusion. Write the various stages of trauma from occlusion.
3. Describe the concepts in the role of trauma from occlusion in case of periodontal diseases. Describe the physi-
ological and pathological tooth mobility seen in the teeth involved in trauma from occlusion.
4. Define ‘trauma from occlusion’ and discuss its aetiology, clinical features and management. [Same as LE Q.1]
5. Define trauma from occlusion. Discuss the pathology, clinical and radiographic features of trauma from occlu-
sion. [Same as LE Q.1]
6. What is trauma from occlusion? Give the signs, symptoms and histopathological features of trauma from occlu-
sion. [Same as LE Q.1]

SHORT ESSAYS:
1. What is adaptive remodelling of the periodontium in response to external force? List various changes produced
in the periodontium due to remodelling.
2. Pathological migration of teeth. [Ref LE Q.3]
3. Define and classify trauma from occlusion. [Ref LE Q.2]
4. Concepts of trauma from occlusion. [Ref LE Q.3]
5. Bruxism.
6. Injury phase in trauma from occlusion. [Ref LE Q.2]
7. Management of dentinal hypersensitivity.
Section | I  Topic-Wise Solved Questions of Previous Years 803

8. Tissue response to increased occlusal forces. [Ref LE Q.2]


9. Pathological tooth migration. [Same as SE Q.2]
10. Classification and diagnosis of trauma from occlusion. [Same as SE Q.3]
11. Trauma from occlusion. [Same as SE Q.3]
12. Clinical features and diagnosis of bruxism. [Same as SE Q.5]

SHORT NOTES:
1. Define trauma from occlusion. Name the various stages of tissue response in trauma from occlusion. [Same as SE Q.1]
2. Pathological migration of teeth.
3. Clinical and radiological changes in trauma from occlusion.
4. Causes of and changes produced by primary trauma from occlusion.
5. Define primary and secondary trauma from occlusion. [Ref LE Q.2]
6. Trauma from occlusion. [Ref LE Q.2]
7. Define acute and chronic trauma from occlusion. [Ref LE Q.2]
8. Facets.
9. Parafunctional condition and periodontium.
10. Therapeutic occlusion.
11. Management of supracontacts.
12. Bruxism.
13. Passive eruption.
14. Fremitus test.
15. Night guard.
16. Occlusal appliances.
17. Define secondary occlusal trauma. [Same as SN Q.5]
18. Define acute trauma from occlusion. [Same as SN Q.6]
19. Prematurities. [Same as SN Q.11]

SOLVED ANSWERS
LONG ESSAYS:
Q.1. What is trauma from occlusion? Give the signs, l Bone loss
symptoms and treatment of traumatic occlusion. l Root resorption
Ans. Signs of TFO
i. Visual signs
When occlusal forces exceed the adaptive capacity of the l Signs that can be seen in wear registration, as
tissues, tissue injury results. The resultant injury is termed transparencies are points of contact and not
as trauma from occlusion (TFO). broad areas.
Clinical indicators of TFO may include one or more of the l Excessively large areas of contact can also be
following: detected by using a variety of marketing mate-
l Mobility rials such as carton paper, tape, ribbon or a dye.
l Fremitus ii. Auditory sign: In centric relation, there is a dis-
l Occlusion tinct ringing sound; in maximum tooth contact
l Prematurities with deflections present, the sound is dull or per-
l Wear facets in presence of other clinical features ceptible.
l Tooth migration iii. Tactile sign: In centric relation and in normal
l Fractured tooth/teeth excessive movements, response to finger contact
l Thermal sensitivity is smooth, with deflection present, a roughness
Radiographic indicators of TFO may include one or more can be detected.
of the following: iv. Migration of teeth
l Discontinuity and thickening of lamina dura l Loss of interproximal contacts and migration
l Widened periodontal ligament space (funnelling or of teeth may be the sequela of traumatic oc-
saucerization) clusal relations.
804 Quick Review Series for BDS 4th Year, Vol 1

l Unusual habit pattern may cause tooth migra- venous hyperesia, increased blood pressure in the
tion and TFO beyond the function range of pulp and pain.
normal occlusal contacts. l The jaws are tired at the end of the day, more

v. Mobility so on rising in the morning.


l The most common clinical sign of trauma to l When the teeth are pushed together, a scratch-

periodontium is increased tooth mobility. ing or squeaky sound can be heard.


l In injury stage of TFO, there is destruction of ii. Habits
periodontal fibres, which will increase the l Prematurities may initiate or perpetuate a

mobility of the tooth. habit pattern that is injurious to the periodon-


l In the final stage, the accommodation of the tium.
periodontium to increased forces entails a l Because of psychic tension, a patient contin-

widening of periodontal ligament, which also ues to grind and clench his/her teeth.
leads to increased tooth mobility. iii. Muscle hypertonicity
l Although this tooth mobility is greater than l Occlusal interferences are the main cause of

the so-called normal mobility, it cannot be muscle spasm and discomfort.


considered pathologic because it is an adapta- l The muscles of mastication, i.e. the masseter

tion and not a disease process. and temporalis, but particularly pterygoids,
l When it progressively gets worse, it can be should be inspected for hypertonicity and
considered pathologic. tenderness.
vi. Fremitus and mobility l This detects tooth-to-tooth or TMJ dysfunc-

l The most arresting and obvious sign of trauma tion or both.


are facets, fremitus and mobility. l Subluxation and crepitus in the TMJs associ-

l Fremitus is an important tool. ated with muscle spasm are frequently caused
l Palpation and percussion are two methods by cuspal interferences.
used routinely to check mobility. iv. Loosening teeth
l Fremitus means palpable vibration or move- l In advanced conditions, the patient becomes

ments. aware of the loosening teeth and may com-


l A tooth with fremitus has excess contact, pos- plain of soreness and tenderness in some
sibly related to a premature contact. areas.
vii. Wear patterns l Pain in the jaw and TMJ regions are usually

l Facets and abnormal wear patterns must be diagnostic of condyle displacement induced
differentiated from attrition caused by a coarse by occlusal interferences.
diet. v. Effects of excessive occlusal forces on dental
l Shiny and irregular facets indicate tooth-to- pulp: Complete calcification of the pulpal canal
tooth wear that is associated with bruxism. may occur in cases of long-standing severe
l These worn and abraded teeth are invariably trauma or following a single serious traumatic
firm with no sign of mobility. accident to the tooth resulting in gross distur-
viii. Gingival recession: Gingival recession may be bances of the circulation.
provoked by direct contact of the teeth with the vi. Effects of insufficient occlusal forces:
gingiva, as in severe overbite, where the upper l Insufficient occlusal forces may also be injuri-

incisors damage the buccal gingiva of the lower ous to the supporting periodontal tissue.
incisors. l Insufficient stimulation causes thinning of the

ix. Gingival clefts periodontal ligament, atrophy of the fibres,


l Gingival clefts are a combination of condi- osteoporosis of the alveolar bone and reduc-
tions caused by toothbrush abrasion. tion in bone height.
l This initially produces a linear act injury that l Hypofunction can result from an open bite

eventually develops into a gingival cleft. relationship, or absence of functional antago-


l Depending on the security of the conditions, nists or unilateral chewing habits that neglect
clefts are treated by curettage. one side of the mouth.
Symptoms of TFO Treatment
i. Thermal sensitivity: Trauma from occlusion pro- l Occlusal adjustment

duces pressure on venous return leading to l Coronoplasty


Section | I  Topic-Wise Solved Questions of Previous Years 805

l Occlusal bite planes l It most often develops from gradual


l Orthodontics changes in occlusion produced by tooth
l Permanent or temporary splint
wear, drifting movement and extrusion of
The treatment for periodontal TFO depends on teeth, combined with a parafunctional
the aetiology. habit such as bruxism and clenching, rather
Primary TFO is usually treated by one or more of than as a sequela of acute periodontal
the above procedures. trauma.
Q.2. Define and classify trauma from occlusion. Write l Chronic TFO can be primary or secondary.)}
the various stages of trauma from occlusion.
(SE Q.3 and SN Q.5)
Ans.
{(i. Primary trauma from occlusion
(SE Q.3 and SN Q.6) l Trauma from occlusion may be caused by altera-

tions in occlusal forces, reduced capacity of the


{(When occlusal forces exceed the adaptive capacity of periodontium to withstand occlusal forces or both.
the tissues, tissue injury results. The resultant injury is l When TFO is the result of alteration in occlusal
termed as trauma from occlusion.)} forces, it is called primary TFO.
[SE Q.3] l In primary TFO, the main aetiologic factors are lo-

cal alterations in the occlusion due to the following:


{Classification l The insertion of a high filling or a prosthetic re-
Types: placement that creates excessive forces on abut-
i. Depending on the onset and duration ment and antagonist teeth.
a. Acute l The drifting movement or extrusion of teeth into
b. Chronic spaces created by unreplaced missing teeth or the
ii. Depending on the cause orthodontic movement of teeth into functionally
a. Due to the alterations in occlusal forces unacceptable position.
b. Reduced capacity of the periodontium} l Changes produced by primary trauma do not alter

(SE Q.3 and SN Q.7) level of connective tissue attachment and do not
initiate pocket formation because supracrestal
{(a. Acute trauma from occlusion gingival fibres are not affected and, therefore
l Acute TFO results from an abrupt occlusal prevent apical migration of the junctional epithe-
impact such as that produced by biting a lium (JE).
hard object (e.g. stone, olive pip). ii. Secondary trauma from occlusion
l Restorations or prosthetic appliances that l Secondary TFO occurs when the adaptive capacity
interfere with or alter the direction of oc- of the tissues to withstand occlusal forces is im-
clusal forces on the teeth may induce acute paired by bone loss resulting from marginal in-
trauma. flammation.
l This reduces the periodontal attachment area and
l The results of acute TFO are
l Tooth pain
alters the leverage on the remaining tissues.
l The periodontium becomes more vulnerable to
l Sensitivity to percussion

l Increased tooth mobility


injury and previously well-tolerated occlusal
l If the force is dissipated by shift in the
forces become traumatic.)}
position of the tooth or by wearing away
[SE Q.3]
or correction of the restoration, the injury
heals and the symptoms subside. Other- {iii. Combined occlusal trauma
wise, periodontal injury may worsen and It refers to injury resulting from abnormal occlusal
develop into necrosis, accompanied by forces applied to a tooth or teeth with inadequate peri-
periodontal abscess formation or persist odontal support.}
as a symptom free chronic condition. Stages of trauma from occlusion
l Acute trauma can also produce cemental
[SE Q.6 and SE Q.8]
tear
{Tissue response occurs in three stages:
b. Chronic trauma from occlusion l Stage I: injury
l Chronic TFO is more common than l Stage II: repair
the acute form and is of greater clinical l Stage III: adaptive remodelling of the
significance. periodontium
806 Quick Review Series for BDS 4th Year, Vol 1

Stage I: injury proliferation and differentiation of fibroblast in


l This stage involves tissue injury produced by ex- collagen and bone formation.
cessive occlusal forces. l These return to normal levels after dissipation of
l The body attempts to repair the injury and restore the forces.}
the periodontium.
[SE Q.8]
l This can occur if the forces are diminished or

tooth drifts away from them. {Stage II: repair


l If the offending force is chronic, the periodontium l Repair is constantly occurring in the normal peri-
is remodelled to cushion its impact. This ligament odontium, and TFO stimulates increased repara-
is widened at the expense of the bone, resulting in tive activity.
angular bone defects without periodontal pockets l The damaged tissues are removed, and new con-

and tooth becomes loose. nective tissue cells and fibres, bone and cemen-
l Slightly excessive pressure stimulates resorption tum are formed in an attempt to restore the injured
of alveolar bone, with a resultant widening of periodontium.
periodontal ligament space. l Forces remain traumatic only as long as the dam-

l Slightly excessive tension causes elongation of age produced exceeds the reparative capacity of
the periodontal ligament fibres and apposition of the tissues.
alveolar bone. l When bone is resorbed by excessive occlusal

l In the areas of increased pressure, the blood ves- forces, the body attempts to reinforce the thinned
sels are numerous and reduced in size, while in bony trabecula with new bone. This attempt to
the areas of increased tension, they are enlarged. compensate for lost bone is called buttressing
l Greater pressure produces a gradation of changes bone formation and is an important feature of the
in the periodontal ligament, starting with com- reparative process associated with TFO.
pression of fibres, which produces areas of hyalin- l Buttressing bone formation occurs within the

ization. jaws (central buttressing) and on the bone surface


l Subsequently, injury to the fibroblast and other (peripheral buttressing).
connective tissue cells leads to necrosis of the l In central buttressing, the endosteal cells deposit

areas of ligament, vascular changes are also new bone, which restores the bony trabecula and
produced: reduces the size of the marrow spaces.
l Within 30 min, retardation and stasis of blood l Peripheral buttressing occurs on the facial and

flow occurs. lingual surfaces of the alveolar plate.


l At 2–3 h, blood vessels appear to be packed with l Depending on its severity, peripheral buttressing

erythrocytes that start to fragment. may produce a shelf-like thickening of the alveo-
l Between 1 and 7 days, disintegration of blood lar margin, referred to as lipping or a pronounced
vessel walls and release of contents to surround- bulge in the contour of the facial and lingual
ing tissue. bone.
l In addition, increased resorption of alveolar bone Stage III: adaptive remodelling of the periodontium
and resorption of tooth surface occur. l If the repair process cannot keep pace with the

l Severe tension causes widening of periodontal destruction caused by occlusion, the periodon-
ligament, thrombosis, haemorrhage, tearing of the tium is remodelled in an effort to create a struc-
periodontal ligament and resorption of alveolar tural relationship in which the forces are no lon-
bone. ger injurious to the tissues.
l Pressure severe enough to force the root against l This results in a thickened periodontal ligament,

bone causes necrosis of the periodontal ligament which is funnel shaped at the crest, and angular
and bone. defects in the bone, with no pocket formation.
l The bone is resorbed from viable periodontal liga- l The involved teeth become loose. Increased vas-

ment adjacent to necrotic areas and from marrow cularization is noticed.


spaces, a process called undermining resorption. l The injury phase shows an increase in the areas of

l The areas of the periodontium most susceptible resorption and a decrease in bone formation,
to injury from excessive occlusal forces are the whereas the repair phase demonstrates decreased
furcations. resorption and increased bone formation after
l Injury to the periodontium produces a tempo- adaptive remodelling of the periodontium, resorp-
rary depression in mitotic activity and rate of tion and formation return to normal.}
Section | I  Topic-Wise Solved Questions of Previous Years 807

Q.3. Describe the concepts in the role of trauma from l The fibre bundles that separate the zone of
occlusion in case of periodontal diseases. Describe the codestruction from the zone of irritation can
physiological and pathological tooth mobility seen in the be affected from two different directions:
teeth involved in trauma from occlusion. i. From the inflammatory lesion main-
tained by plaque in the zone of irri-
Ans.
tation
Role of trauma from occlusion in the progression of ii. From trauma-induced changes in the
periodontal diseases zone of codestruction
l Due to this exposure from two different direc-
[SE Q.4]
tions, the fibre bundles may become dissolved
{Various concepts of trauma from occlusion in progres- and/or oriented into a direction parallel to the
sion of periodontal diseases are as follows: root surface.
Glickman’s concept Waerhaug’s concept
l As claimed by Glickman and Smulow (1965, 1967), l This was also supported by Prichard (1965) and

the pathway of the spread of a plaque-associated Manson (1976).


gingival lesion can be changed if forces of an abnor- l From his similar studies he concluded that angular

mal magnitude are acting on teeth harbouring sub- defects and infrabony pockets occur often at peri-
gingival plaque. odontal sites of teeth not affected by trauma from
l According to Glickman, instead of an even destruc- occlusion.
tion of the periodontium and alveolar bone, i.e. su- l In other words, he refuted the hypothesis that trauma

prabone pockets and horizontal bone loss, which from occlusion played a role in the spread of a gingi-
occurs at sites with uncomplicated plaque-associated val lesion into the zone of codestruction.
lesions, sites which are also exposed to abnormal oc- l According to Waerhaug, the loss of periodontium

clusal force will develop angular bony defects and was the result of inflammatory lesions associ-
infrabony pockets. ated with subgingival plaque. He concluded that
l The periodontal structure can be divided into two angular defects occur when the subgingival
zones: plaque of one tooth has reached a more apical
i. The zone of irritation level than the microbiota on the neighbouring
ii. The zone of codestruction tooth, and when the volume of the alveolar bone
surrounding the roots is comparatively large.
i. Zone of irritation
This was also supported by Prichard (1965) and
l The zone of irritation includes the marginal
Manson (1976). }
and interdental gingiva. The soft tissue of
l In conclusion, when a tooth with gingival inflam-
this zone is bordered by hard tissue, i.e. the
mation is exposed to trauma, four possibilities can
tooth only on one side and is not affected by
occur:
forces of occlusion. This means that gingival
i. Trauma from occlusion may alter the pathway of
inflammation cannot be induced by TFO but
extension of gingival inflammation to the under-
is the result of irritation from microbial
lying tissues. Inflammation may proceed to the
plaque.
periodontal ligament rather than to the alveolar
l The plaque-associated lesion at a ‘nontrauma-
bone and the resulting bone loss would be angu-
tized’ tooth propagates in apical direction by
lar with infrabony pockets.
first involving the alveolar bone and only later
ii. It may favour the environment for the formation
the periodontal area. The progression of this
and attachment of plaque and calculus and
lesion results in an even (horizontal) bone
may be responsible for development of deeper
destruction.
lesions.
ii. Zone of codestruction
iii. Supragingival plaque can become subgingival if
l The zone of codestruction includes the peri-
the tooth is tilted orthodontically or migrates into
odontal ligament, the root cementum and the
an edentulous area, resulting in the transforma-
alveolar bone, and is coronally demarcated by
tion of a suprabony pocket into an infrabony
the trans-septal, i.e. interdental and the dento-
pocket.
alveolar collagen.
iv. Increased tooth mobility associated with trauma
l The tissue in this zone may become the seat of
to the periodontium may have a pumping
lesion caused by TFO.
808 Quick Review Series for BDS 4th Year, Vol 1

effect on plaque metabolites increasing their dif- . Failure to replace first molars
b
fusion. c. Other causes
Physiologic adaptive capacity of the periodontium to If the periodontium is sufficiently weakened,
occlusal forces these forces do not have to be abnormal to
l The dynamics of the periodontium to accommodate cause pathologic migration.
the forces exerted on the crown is called adaptive
a. Unreplaced missing teeth
capacity.
l The spaces created by unreplaced missing
l This varies in different persons and in the same per-
teeth lead to drifting of adjacent teeth.
son at different times. This is mainly explained by
l Drifting does not result from destruction of
four factors, which mainly influence the effect of oc-
the periodontal tissues but it usually creates
clusal forces on the periodontium.
conditions that lead to periodontal diseases,
1. Magnitude (the amount): When it is increased, the
and thus the initial tooth movement is aggra-
periodontium shows:
vated by loss of periodontal support.
a. Thickening of the periodontal ligament
b. Failure to replace first molars
b. An increase in the number and width of peri-
This results in the following:
odontal ligament fibres
l The tilting of second and third molars re-
c. An increase in the density of the alveolar bone
sulting in decreased vertical dimension
2. Direction: Changes in the direction cause a reori-
l Movement of premolars distally and the
entation of the stresses and strains within the
mandibular incisors tilting lingually
periodontium.
l Increased anterior overbite
3. Duration: Constant pressure on the bone is more
l Pushing the maxillary incisors labially and
injurious than intermittent forces.
laterally
4. Frequency: The more frequent the application of
l Extrusion of the anterior teeth due to disap-
an intermittent force, the more injurious is the
pearance of incisal apposition
periodontium.
l Diastema of the anterior teeth

c. Other causes
[SE Q.2]
l Pressure from the tongue may cause drifting of

{Pathologic migration teeth in normal conditions.


Pathologic migration refers to tooth displacement that l When the periodontium is sufficiently weak-

results when the balance among the factors that main- ened, these forces cause pathologic migration.
tain physiologic tooth position is disturbed by a peri- l Pressure from the granulation tissue of peri-

odontal disease. odontal pocket also causes pathologic migra-


Pathogenesis tion.}
The normal position of the teeth is maintained by two
Q.4. Define ‘trauma from occlusion’ and discuss its aeti-
major factors:
ology, clinical features and management.
i. The health and normal height of the periodon-
tium Ans.
ii. The forces exerted on the teeth
[Same as LE Q.1]
i. Health and normal height of the periodontium
l A tooth with weakened periodontal support is Q.5. Define trauma from occlusion. Discuss the pathol-
unable to withstand the forces and moves ogy, clinical and radiographic features of trauma from
away from the opposing force. occlusion.
l When periodontal support is reduced, forces
Ans.
that are acceptable to an intact periodontium
become injurious. Pathologic migration may [Same as LE Q.1]
continue even after a tooth no longer contacts Q.6. What is trauma from occlusion? Give the signs,
its antagonist. symptoms and histopathological features of trauma
ii. Forces exerted on the teeth from occlusion.
The changes in the forces may occur as a Ans.
result of:
a. Unreplaced missing teeth [Same as LE Q.1]
Section | I  Topic-Wise Solved Questions of Previous Years 809

SHORT ESSAYS: Widened periodontal ligament spaces seen in radio-


l

graphs
Q.1. What is adaptive remodelling of the periodontium l Hypertonicity of the muscles of mastication
in response to external force? List various changes pro- l Temporomandibular joint discomfort
duced in the periodontium due to remodelling. Diagnosis
Ans. l History and clinical examination.

l Occlusal prematurities can be diagnosed by use of


l Tissue response to trauma from occlusion occurs in articulating papers.
three stages: l Electromyographic examination can be carried
i. Stage I: injury out to check for hyperactivity of the muscles of
ii. Stage II: repair mastication.
iii. Stage III: adaptive remodelling of the periodontium
l Adaptive remodelling of the periodontium is stage III Q.6. Injury phase in trauma from occlusion.
among the stages of tissue response to trauma from Ans.
occlusion.
l If the repair process cannot keep pace with the destruc- [Ref LE Q.2]
tion caused by occlusion, the periodontium is remod- Q.7. Management of dentinal hypersensitivity.
elled in an effort to create a structural relationship in
which the forces are no longer injurious to the tissues. Ans.
l This results in a thickened periodontal ligament, which
l A number of agents have been proposed to treat dentinal
is funnel shaped at the crest, and angular defects in the hypersensitivity. Among them, desensitizing agents are
bone, with no pocket formation. best used in cases of dentinal hypersensitivity.
l Increased vascularization is noticed.
l Desensitizing agents can be applied by the patient at
l The involved teeth become loose.
home or by the dentist or hygienist in the dental office.
l The most likely mechanism of action is the reduction in
Q.2. Pathological migration of teeth.
the diameter of the dentinal tubules so as to limit the
Ans. displacement of fluid in them.
l This can be attained by the following:
[Ref LE Q.3]
i. Formation of a smear layer produced by burnishing
Q.3. Define and classify trauma from occlusion. the exposed surface
ii. Topical application of agents that form insoluble
Ans.
precipitates within the tubules
[Ref LE Q.2] iii. Impregnation of tubules with plastic resins, or seal-
ing of tubules with plastic resins
Q.4. Concepts of trauma from occlusion.
l Agents used by the patient:

Ans. l The American Dental Association (ADA) has ap-


proved the following dentifrices for desensitizing
[Ref LE Q.3]
purposes:
Q.5. Bruxism. 1. Sensodyne and thermodent, which contain stron-
tium chloride.
Ans.
2. Crest sensitivity protection.
l Bruxism is clenching or grinding of the teeth when the 3. Denquel and Promise, which contain potassium
individual is not chewing or swallowing, which results nitrate and Protect, which contains sodium citrate.
in attrition of teeth. 4. Fluoride rinsing solutions and gels can also be
l During clenching or grinding, the individual might used after the usual plaque control procedures.
impose a load of over 20 kg on a tooth over a period Agents used for dentinal hypersensitivity in office
of 2.5 s each time. This is far in excess of normal treatments:
functional stress and causes increased flow within the l Cavity varnishes

viscoelastic periodontal ligament and distortion of l Anti-inflammatory agents

alveolar bone. Treatments that partially obturate dentinal tubules


Clinical features i. Burnishing of dentine
l Advanced attrition, presence of wear facets ii. Silver nitrate
l Increased tooth mobility iii. Zinc chloride
810 Quick Review Series for BDS 4th Year, Vol 1

iv. Potassium ferrocyanide l Pathological migration refers to tooth displacement that


v. Formalin results when the balance among the factors that main-
l Calcium compounds tain physiologic tooth position is disturbed by periodon-
a. Calcium hydroxide tal disease.
b. Dibasic calcium phosphate l It mostly occurs in anterior region but also occurs in

l Fluoride compounds posterior region.


a. Sodium fluoride l The tooth may move in any direction and accompanied

b. Stannous fluoride by rotation.


l Iontophoresis l Pathological migration in occlusal or incisal direction is

l Strontium chloride known as extrusion.


l Potassium oxalate l Weakened by loss of periodontal support, the maxillary

l Restorative resins and mandibular anterior teeth drift labially and extrude,
l Dentine-bonding agents creating diastema between the teeth.
l Lasers introduced recently for the treatment of den- l Pathological migration is also an early sign of localized

tinal hypersensitivity aggressive periodontitis. It also occurs due to increased


l Desensitizing agents act by the precipitation of crys- or modified forces exerted on the teeth like unreplaced
talline salts on the dentinal surface, which block missing teeth and failure to replace first molars.
dentinal tubules l Other causes like trauma from occlusion, pressure from

l Prove to be effective only when used for at least tongue and granulation tissue of periodontal pocket also
2 weeks contribute to pathological migration.
Q.3. Clinical and radiological changes in trauma from
Q.8. Tissue response to increased occlusal forces. occlusion.
Ans. Ans.
[Ref LE Q.2] Clinical changes in trauma from occlusion:
l The most common clinical sign is increased tooth
Q.9. Pathological tooth migration.
mobility.
Ans. Radiological changes in trauma from occlusion:
[Same as SE Q.2] l Widening of periodontal ligament space

l Thickening of lamina dura along the lateral aspect,


Q.10. Classification and diagnosis of trauma from
occlusion. apical region of root and in bifurcation areas
l A vertical rather than horizontal destruction of
Ans. interdental septum
[Same as SE Q.3] l Root resorption

Q.11. Trauma from occlusion. Q.4. Causes of and changes produced by primary
Ans. trauma from occlusion.
Ans.
[Same as SE Q.3]
Q.12. Clinical features and diagnosis of bruxism. Causes of primary trauma from occlusion
Periodontal injury produced around the teeth following:
Ans. l The insertion of a high filling or a prosthetic re-

[Same as SE Q.5] placement that creates excessive forces on abut-


ment and antagonist teeth.
l The drifting movement or extrusion of teeth into
SHORT NOTES: spaces created by unreplaced missing teeth.
Q.1. Define trauma from occlusion. Name the various l The orthodontic movement of the teeth into func-

stages of tissue response in trauma from occlusion. tionally unacceptable positions.


Changes produced by primary trauma from occlusion:
Ans. l Primary trauma from occlusion neither alters the

[Same as SE Q.1] level of connective tissue attachment nor initiates


pocket formation.
Q.2. Pathological migration of teeth. l The supracrestal gingival fibres are not affected and

Ans. therefore prevent the apical migration of the JE.


Section | I  Topic-Wise Solved Questions of Previous Years 811

Q.5. Define primary and secondary trauma from l It is a treated occlusion employed to counteract struc-
occlusion. tural inter-relationship related to traumatic occlusion.
Ans. Q.11. Management of supracontacts.

[Ref LE Q.2] Ans.

Q.6. Trauma from occlusion. Following are the various procedures used to correct pre-
maturities or supracontacts:
Ans. i. Grooving: consists of restoring depth of developmental
[Ref LE Q.2] grooves made shallow by occlusal wears done by ta-
pered cutting tools.
Q.7. Define acute and chronic trauma from occlusion. ii. Spheroiding: reducing prematurity and restoring origi-
Ans. nal tooth contour. Its purpose is not simply to narrow
occlusal surface but also used to restore buccolingual
[Ref LE Q.2] width of occlusal surface as it was before.
Q.8. Facets. iii. Pointing: consists of restoring cusp points, contours
done with cutting rotating tools, usually a tapered stone
Ans. is for pointing.
l Facets and abnormal wear patterns must be differenti-
Q.12. Bruxism.
ated from attrition caused by a coarse diet.
l Shiny and irregular facets indicate tooth-to-tooth wear Ans.
that is associated with bruxism.
l Bruxism is defined as the diurnal or nocturnal parafunc-
l These worn and abraded teeth are invariably firm, with
tional activity that includes clenching, bracing, gnash-
no sign of mobility.
ing and grinding of the teeth.
Q.9. Parafunctional condition and periodontium. Effects of bruxism on periodontium
l No causative association exists between bruxism and
Ans.
gingival inflammation or periodontitis.
l Harmful effects of parafunctional conditions that result l Bruxism can cause tooth mobility, tooth fracture,

from psychic influences on the organic control of tissues tooth wear, periodontal and muscle pain.
are known as psychosomatic disorders. l The muscle pain may contribute to masticatory sys-

l These disorders are induced in the oral cavity by the tem disorders.
following ways: l The selective serotonin reuptake inhibitor (SSRI)

i. Through the development of habits that are injurious medications have been reported to encourage bruxism.
to the periodontium Management
ii. By the direct effect of the autonomic nervous system A maxillary and mandibular stabilization appliance is
on the physiologic tissue balance generally considered the most effective means of man-
l Under conditions of mental and emotional stress, the aging bruxism.
mouth may subconsciously become an outlet for the
Q.13. Passive eruption.
gratification of basic drives in the adult.
l Gratification may be derived from neurotic habits like Ans.
grinding or clenching the teeth, nibbing on foreign ob-
l Passive eruption is exposure of the teeth by apical mi-
jects (pencils or pipes), nail biting or excessive use of
gration of the gingiva.
tobacco, which are all potentially injurious to the peri-
l It is different from active eruption where the movement
odontium.
of the teeth is in the direction of occlusal plane.
Q.10. Therapeutic occlusion. l Recently, passive eruption is considered as a pathologic

process.
Ans.
Passive eruption is divided into following four stages:
l Therapeutic occlusion is obtained by altering existing Stage 1: The teeth reach line of occlusion. The JE
occlusion to get an occlusion which is consistent with and base of the gingival sulcus are on the enamel.
the health. Stage 2: The JE proliferates so that part is on the
l A therapeutic occlusion is the result of specific interven- cementum and part is on the enamel. The base of the
tions designed to treat dysfunction or disease. sulcus is still on the enamel.
812 Quick Review Series for BDS 4th Year, Vol 1

Stage 3: The entire JE is on the cementum, and the Q.15. Night guard.
base of the sulcus is at the cementoenamel junction
Ans.
(CEJ). As JE proliferates from the crown onto the
root, it does not remain at the CEJ any longer than at Night guards are most commonly used for long-term noc-
any other area of the tooth. turnal habit control therapy and long-term mandibular repo-
Stage 4: The JE has proliferated farther on the cemen- sitioning.
tum. The base of the sulcus is on the cementum, a portion Bite planes/occlusal splints/bite guards
of which is exposed. Proliferation of the JE onto the root l An occlusal guard is usually made of acrylic resin

is accompanied by the degeneration of gingival and peri- and is designed to cover the occlusal surface and/or
odontal ligament fibres and their detachment from the incisal edges of the teeth.
tooth. At present, it is believed to be the result of chronic l They are therapeutic because of their ‘bite rising’ ef-

inflammation, and therefore a pathologic process. fects, i.e. passive stretching of painful muscle fibres.
l In bruxers, it may minimize tooth wear and reduce
Q.14. Fremitus test.
tooth contacts that act as trigger factors eliciting
Ans. bruxism; it may decrease after the pain is relieved
with the wearing of occlusal guard.
l Fremitus test is a measurement of the vibratory pattern
of the teeth when the teeth are placed in the contacting Q.16. Occlusal appliances.
positions and movements.
Ans.
l This is limited to the maxillary teeth. But in cases of

edge-to-edge occlusion or when there is little overlap of Occlusal appliances are used in dentistry for the diagnosis
teeth, mandibular teeth can also be assessed. and treatment of a variety of dental disorders like tooth
l Procedure: Index finger is placed on each maxillary hypermobility, myofascial pain dysfunction syndrome
tooth at about the cervical third, and the patient is asked (MPDS), disorders of the condyle disc assembly.
to click the back teeth repeatedly. Classification
l The following classification system is used to grade i. Sectional appliances: These appliances contact a
fremitus: sector or group of teeth as opposed to all of the teeth.
Class I fremitus: Mild vibration or movements detected. Example: modified Hawley bite plane
Class II fremitus: Easily palpable vibration but no visi- ii. Full-coverage appliances: These appliances contact
ble movements. all of the teeth. Example: maxillary full occlusal
Class III fremitus: Movements visible with naked eye. splint (night guard)
l The grade/degree recorded may be subjective but the

following range has been suggested: Q.17. Define secondary occlusal trauma.
l N 0 Normal (without vibration or movement). Ans.
l 1 0 One-degree fremitus, only slight vibration can

be felt. [Same as SN Q.5]


l 11 0 Two-degree fremitus, the tooth is clearly
Q.18. Define acute trauma from occlusion.
palpable but movement is barely visible.
l 111 0 Three-degree fremitus, movement is Ans.
clearly observed visually. [Same as SN Q.4]
l Fremitus differs from mobility in which fremitus is the

tooth displacement created by the patient’s own occlusal Q.19. Prematurities.


force; therefore, the amount of force varies greatly from
Ans.
patient to patient unlike mobility, where the force with
which it is measured tends to be the same for each examiner. [Same as SN Q.11]
Section | I  Topic-Wise Solved Questions of Previous Years 813

Topic 12
Influence of Systemic Diseases on the Periodontium
and Periodontal Medicine
COMMONLY ASKED QUESTIONS
LONG ESSAYS:
1 . Antibiotic prophylaxis for the medically compromised patient.
2. Describe periodontal therapy during pregnancy.
3. Relationship between diabetes and periodontal disease and management of a diabetic patient with periodontal
disease.
4. Mention the precautions to be taken during pregnancy in periodontal therapy. [Same as LE Q.2]
5. Management of periodontal diseases in a pregnant patient. [Same as LE Q.2]

SHORT ESSAYS:
1. Describe in short oral manifestations in scurvy.
2. Chediak–Higashi syndrome.
3. Periodontal manifestations of leukaemia.
4. Effects of diabetes and periodontal health. [Ref LE Q.3]
5. AIDS and periodontium.
6. Kaposi sarcoma.
7. Risk factors associated with periodontal diseases.
8. Periodontal manifestations of HIV infection.
9. Antibiotic prophylaxis for infective endocarditis.
10. Discuss management of a diabetic patient. [Ref LE Q.3]
11. Periodontal infection a cause for atherosclerosis.
12. Possible role of ascorbic acid deficiency in aetiology of periodontal disease. [Same as SE Q.1]
13. Diabetes as a risk factor in periodontal diseases. [Same as SE Q.4]
14. AIDS and periodontal diseases. [Same as SE Q.5]
15. Discuss periodontal findings in AIDS. [Same as SE Q.5]

SHORT NOTES:
1. Oral lesions in diabetes mellitus. [Ref LE Q.3]
2. Scorbutic gingivitis.
3. Stress and periodontal diseases.
4. Periodontal medicine.
5. Mention any five neutrophil disorders causing periodontitis.
6. Periodontal care of patients with tuberculosis.
7. Periodontal treatment planning of a dental transplant patient.
8. Gingiva in leukaemia. [Ref SE Q.3]
9. Periodontal manifestations of diabetes mellitus. [Same as SN Q.1]
10. Influence of diabetes on the periodontium. [Same as SN Q.1]
11. Vitamin C in gingival disease. [Same as SN Q.2]
12. Scurvy. [Same as SN Q.2]
814 Quick Review Series for BDS 4th Year, Vol 1

SOLVED ANSWERS
LONG ESSAYS:
Q.1. Antibiotic prophylaxis for the medically compro- which may cause supine hypotensive syndrome with
mised patient. decreased placental perfusion.
l Decreasing blood pressure, syncope and loss of con-
Ans.
sciousness may occur.
The prophylactic regimens for susceptible patients under- l These consequences may be prevented by avoiding a
going dental, oral, respiratory tract or oesophageal proce- fully reclined position if possible and placing the
dures are as follows: patients on their left side or simply by elevating the right
hip 5–6 inches during treatment.
Condition Agents used Regimen dosage l Appointments should be short, and the patient should be
A standard Amoxicillin Adults: 2.0 g allowed to change positions frequently.
prophylaxis in Children: 50 mg/kg orally l Other precautions during pregnancy relate to the poten-
general 1 h before procedure tial toxic or teratogenic effects of therapy on the foetus.
Patients unable Ampicillin Adults: 2.0 g intramuscularly i. Ideally, no medications should be prescribed, but in
to take oral (i.m.) or intravenously (i.v.) situations where analgesics, antibiotics, local anaes-
medications Children: 50 mg/kg i.m.or thetics and other drugs are required during preg-
i.v. within 30 min before
procedure
nancy, based on the needs of the patient, they should
be reviewed for potential adverse effects on the foe-
Patients allergic Clindamycin Adults: 600 mg
tus before prescribing.
to penicillin or Children: 20 mg/kg orally
Cephalexin or 1 h before procedure ii. Use of dental radiographs during pregnancy should
cefadroxil Adults: 2.0 g be kept to a minimum. The American Dental Asso-
or Children: 50 mg/kg orally ciation has stated that ‘normal radiographic guide-
Azithromycin 1 h before procedure lines do not need to be altered because of preg-
or clarithro- Adults: 500 mg
nancy’. Use of a properly positioned lead apron is an
mycin Children: 15 mg/kg orally
1 h before procedure absolute requirement.
Patients allergic Clindamycin Adults: 600 mg; Q.3. Relationship between diabetes and periodontal
to penicillin and or Children: 20 mg/kg i.v. disease and management of a diabetic patient with peri-
unable to take Cefazolin within 30 min before odontal disease.
oral medications procedure
Adults: 1.0 g; Ans.
Children: 25 mg/kg i.m. or
i.v. within 30 min before The two major types of diabetes are as follows:
procedure i. Type 1 (insulin-dependent diabetes)
ii. Type 2 (noninsulin-dependent diabetes)
Q.2. Describe periodontal therapy during pregnancy. The classic signs of diabetes include the following:
i. Polydipsia (excessive thirst)
Ans. ii. Polyuria (excessive urination)
l The main aim of periodontal therapy in pregnant iii. Polyphagia (excessive hunger, often associated with
patients is to minimize the potential exaggerated inflam- weight loss)
matory response related to pregnancy-associated hor-
monal alterations.
l The only nonemergency periodontal procedures per-

formed in pregnant patients are meticulous plaque con- {SN Q.1}


trol, scaling, root planing and polishing. Oral manifestations of diabetes mellitus:
l The second trimester of pregnancy is the safest period
l Cheilosis
to perform periodontal treatment, but still long, stressful l Mucosal drying and cracking
appointments and surgical procedures should be de- l Burning mouth and tongue
layed until the postpartum period. l Diminished salivary flow
l During the second and third trimesters, as the uterus
l Alterations in the flora of the oral cavity with
increases in size, obstruction of the vena cava and aorta greater predominance of Candida albicans, hae-
may occur if the patient is placed in a supine position molytic streptococci and staphylococci
resulting in reduction of return cardiac blood supply
Section | I  Topic-Wise Solved Questions of Previous Years 815

l If the patient has any of these signs or symptoms, or Laboratory evaluation of diabetes control
if the clinician’s index of suspicion is high, further Diabetes mellitus may be diagnosed by any one of
investigation with laboratory studies and physician the following different laboratory methods:
consultation is indicated. i. Random blood glucose: ,200 mg/dL
ii. Fasting plasma glucose: normal fasting glu-
[SE Q.4]
cose is 70–100 mg/dL
{Pathogenesis iii. Two-hour postprandial glucose: normal 2-h
The underlying factors that accompany diabetes postprandial glucose is ,140 mg/dL
mellitus, which may account for the apparent in- iv. Glycosylated haemoglobin assay (HbA1c):
creased prevalence of periodontal disease in these 4%–6% normal
patients can be considered under the following ,7%: good diabetes control
headings: 7%–8%: moderate diabetes control
i. Vascular changes .8%: action suggested to improve diabetes
l Thickening and hyalinization of vascular control
walls, diastase resistant thickening of cap-
[SE Q.10]
illary basement membranes, swelling and
occasional proliferation of endothelial {Treatment
cells and splitting of capillary basement i. If suspected to be a diabetic, following proce-
membrane. dures should be performed:
l Diabetics-induced changes in the capillary l Consult the patient’s physician

basement membrane may have an inhibi- l Analyse laboratory tests, fasting blood glu-

tory effect on the transport of oxygen, white cose, postprandial blood glucose, glycated
blood cells (WBCs), immune factors and haemoglobin and urinary glucose
waste products, all of these could affect tis- ii. Periodontal treatment in a patient with uncon-
sue repair and regeneration. trolled diabetes is contraindicated.
ii. Functions of PMNs iii. Diabetic patients with periodontitis should re-
l Impairment of PMN function is a charac- ceive oral hygiene instructions, mechanical de-
teristic feature of diabetes mellitus. bridement to remove local factors and regular
l Disorders include reduced phagocytosis maintenance.
and intracellular killing, impaired adher- iv. If there is a periodontal condition that requires
ence and impaired chemotactic response. immediate care, prophylactic antibiotics should
l Inhibition of the glycolytic pathway with be given.
the PMNs, abnormal cyclic nucleotide v. If patient is a ‘brittle’ diabetic, optimal periodon-
metabolism, which disrupts the organiza- tal health is a necessity. Glucose levels should be
tion of microtubules and microfilaments, continuously monitored and periodontal treat-
or a reduction in leukocyte membrane ment should be performed when the disease is in
receptors. a well-controlled state. Prophylactic antibiotics
iii. Biochemistry of crevicular fluid should be started 2 days preoperatively. Penicil-
l Alterations in the constituents and flow rate lin is the drug of first choice.
of crevicular fluid have been shown to be Guidelines
associated with diabetes. i. Early morning appointments are ideal because of
l Cyclic AMP levels seem to be reduced in less stress and optimal insulin levels. Clinician
the diabetes group when compared with should make certain that the prescribed insulin
control. has been taken followed by a meal.
iv. Changes in plaque microflora ii. After any surgical procedures, postoperative in-
l The hyaluronidase activity is lower in sulin dose should be altered.
plaque of diabetes individuals. iii. Tissues should be handled atraumatically and as
l Due to above causes, it has been sug- minimally as possible.
gested that diabetic patient is more sus- iv. In case of anxious patients, preoperative sedation
ceptible to periodontal breakdown, which is required.
is characterized by extensive bone loss, v. Concentration of epinephrine used should not be
increased tooth mobility, widening of greater than 1:100,000.
periodontal ligament, suppuration and ab- vi. Antibiotic prophylaxis is recommended for
scess formation.} extensive therapy.
816 Quick Review Series for BDS 4th Year, Vol 1

vii. Recall appointments and fastidious home oral l Vitamin C deficiency is also characterized by
care should be stressed.} increased capillary permeability, susceptibility to
traumatic haemorrhages, hyporeactivity of the
Q.4. Mention the precautions to be taken during preg-
contractile elements of the peripheral blood ves-
nancy in periodontal therapy.
sels and sluggishness of blood flow.
Ans. Clinical manifestations of scurvy
l Bleeding, swollen gingiva and loosened teeth are
[Same as LE Q.2]
common features of scurvy.
Q.5. Management of periodontal diseases in a pregnant l Haemorrhagic lesions into the muscles of the ex-

patient. tremities, the joints and sometimes the nail beds,


petechial haemorrhages, often around hair follicles.
Ans.
l Increased susceptibility to infections.

[Same as LE Q.2] l Impaired wound healing.

Gingivitis
l Gingivitis with enlarged, haemorrhagic, bluish
SHORT ESSAYS: red gingiva is described as one of the classical
Q.1. Describe in short oral manifestations in scurvy. signs of vitamin C deficiency, but gingivitis is not
caused by vitamin C deficiency.
Ans.
l Vitamin C–deficient patients do not necessarily

l Severe vitamin C deficiency in humans results in scurvy, have gingivitis.


a disease characterized by haemorrhagic diathesis and l Acute vitamin C deficiency does not cause or in-

delayed wound healing. crease the incidence of gingival inflammation but


l It may occur in infants in their first year of life if formu- it does increase its severity.
lae are not fortified with vitamins and in older individu- l Vitamin C deficiency may aggravate the gingival

als with restricted diets. Malnutrition associated with response to plaque and worsen the oedema, en-
alcoholism may predispose an individual to scurvy. largement and bleeding.
Role of ascorbic acid in aetiology of periodontal disease Correcting the deficiency may reduce the severity
Ascorbic acid plays a role in periodontal disease through of the disorder; gingivitis will remain as long as
one or more of the following mechanisms: bacterial factors are present.
l Optimal levels of vitamin C would maintain the Periodontitis
epithelium’s barrier function to bacterial products l Acute vitamin C deficiency results in oedema

and is apparently required to maintain the integ- and haemorrhage in the periodontal ligament,
rity of the periodontal microvasculature as well as osteoporosis of the alveolar bone and tooth
the vascular response to bacterial plaque and mobility.
wound healing. l Vitamin C deficiency alone does not cause peri-

l Low levels of ascorbic acid influence the metabo- odontal destruction; local bacterial factors are re-
lism of collagen within the periodontium, affect- quired for increased probing depth and attach-
ing the ability of the tissue to regenerate and repair ment loss to occur.
itself. l The vitamin C deficiency has its greatest impact

l Deficiency of ascorbic acid interferes with bone on periodontal disease when pre-existing disease
formation, leading to loss of periodontal bone. and other codestructive factors are present.
l Changes that do occur in alveolar bone and other
Q.2. Chediak–Higashi syndrome.
bones because of failure of the osteoblasts to form
osteoid take place very late in the deficiency state. Ans.
l Increasing levels of ascorbic acid enhance both the
l Chediak–Higashi syndrome is a rare hereditary disease
chemotactic and the migratory action of leuko-
that affects the production of organelles found in almost
cytes without influencing their phagocytic activity.
every cell and is characterized by recurrent bacterial
l Depletion of vitamin C may interfere with the
infections.
ecologic equilibrium of bacteria in plaque and
l It exhibits oral ulcerations and rapidly destructive peri-
thus increase its pathogenicity.
odontitis.
l Scurvy results in defective formation and mainte-
l In this condition, loss of teeth occurs at a very young
nance of collagen, impairment or cessation of
age due to periodontal attachment loss. The teeth are
osteoid formation and impaired osteoblastic
lost before adulthood.
function.
Section | I  Topic-Wise Solved Questions of Previous Years 817

l Chemotaxis of neutrophils and monocytes is defective


infiltrates. A more diffused submucosal bleeding
in this condition. Phagocytosis is also affected because
manifests as ecchymosis.
of the reduced ability of lysosomes to fuse with phago-
l Bleeding may also be a side effect of the chemo-
somes.
therapeutic agents used to treat leukaemia.
l It affects mostly the melanocytes, platelets and phago-

cytes.
l It causes partial albinism, mild bleeding disorders and

recurrent bacterial infections. iii. Periodontal infections


l Neutrophils contain abnormal, giant lysosomes that can l Infections of the periodontal tissues secondary to

fuse with the phagosome but their ability to release their leukaemia can be of two types:
contents is impaired. As a result, killing of ingested a. An exacerbation of an existing periodontal disease
microorganisms is delayed. b. Through an increased susceptibility of the peri-
odontium to fungal, viral or bacterial infections
Q.3. Periodontal manifestations of leukaemia.
iv. Oral ulceration and infection
Ans. l In leukaemia the response to bacterial plaque and

other local irritation is altered; the cellular component


The leukaemias are malignant neoplasias of WBC precur-
of the inflammatory exudates differs both quantita-
sors characterized by:
tively and qualitatively from that in nonleukaemic
l Diffuse replacement of the bone marrow with proliferat-
individuals.
ing leukaemic cells
l Granulocytopenia results from the displacement of
l Abnormal numbers and forms of immature WBCs in the
normal bone marrow cells by leukaemic cells, and
circulating blood
increases the host susceptibility to opportunistic
l Widespread infiltrates in the liver, spleen, lymph nodes
microorganisms and leads to ulcerations and
and other body sites
infections.
Periodontal manifestations in leukaemic patients
l These lesions occur in sites of trauma such as the
l Oral and periodontal manifestations of leukaemia con-
buccal mucosa in relation to the line of occlusion or
sist of leukaemic infiltration, bleeding, oral ulcerations
on the palate.
and infections.
l Discrete, punched-out ulcers penetrating deeply into

the submucosa and covered by a firmly attached


{SN Q.8} white slough can be found on the oral mucosa.
l The gingival appearance is peculiar bluish red. It is
i. Gingival enlargement
l The enlargement is primarily the result of a mas-
sponge-like and friable, and bleeds persistently on
sive leukaemic cell infiltration into the gingival slightest provocation or even spontaneously in leu-
connective tissue. When present, it is usually a kaemic patients.
l The epithelium presents a variety of changes, and
feature of acute monocytic leukaemia, although
it has been reported as a feature of other forms, may be thinned or hyperplastic. Common findings
including chronic lymphocytic leukaemia. include degeneration associated with intercellular
l The enlarged gingiva hinders mechanical plaque
and intracellular oedema and leukocytic infiltration
removal; hence, there is an inflammatory compo- with diminished surface keratinization.
l A gingival bacterial infection in leukaemic patients
nent enhancing this enlargement.
ii. Gingival bleeding can be the result of an exogenous bacterial infection
l The gingival bleeding can be an early sign of
or an existing bacterial infection.
leukaemia and is secondary to thrombocytopenia Q.4. Effects of diabetes and periodontal health.
that accompanies the leukaemia.
l It is especially marked when the platelet count
Ans.
drops below 10,000/mL and is compounded by [Ref LE Q.3]
poor oral hygiene.
l Gingival haemorrhage is a common finding in
Q.5. AIDS and periodontium.
leukaemic patients, even in the absence of clini- Ans.
cally detectable gingivitis.
l This bleeding tendency can also manifest in the
l All HIV-infected patients may not know that they are
skin and throughout the oral mucosa, where pete- infected when they report for dental treatment. Individ-
chiae are often found, with or without leukaemic uals with known HIV infection may not admit their
status on the medical history.
818 Quick Review Series for BDS 4th Year, Vol 1

l Therefore, every patient receiving dental treatment iii. Red complex pathogens
should be managed as a potentially infected person, us- l Pathogens consisting of Porphyromonas gingivalis,

ing universal precautions for all therapy. Treponema denticola and Tannerella forsythensis
l Extensive periodontal treatment plans must be consid- are implicated in progression of periodontitis along
ered with regard to the patient’s systemic health, prog- with few organisms like Fusobacterium nucleatum
nosis and survival time. and Campylobacter rectus.
l Large variations in progression of HIV disease exist iv. Diabetes
among individuals, and selection of an appropriate l Diabetes is one of the most prevalent chronic dis-

treatment plan depends on the state of the patient’s over- eases in the world. Landmark study by Genco et al.
all health. has demonstrated a high prevalence of periodontitis
l Although there appear to be few contraindications to in the prima Indian community.
routine dental treatment for many HIV-infected patients, l Of late, a two-way relationship has been demon-

the periodontal treatment plan is influenced by the pa- strated with the pro-inflammatory cytokine TNF-a
tient’s overall systemic health and coincident oral infec- implicated in the pathogenesis of periodontitis and
tions or diseases. insulin resistance in type II diabetes.
l An awareness of oral disorders associated with HIV v. The major risk determinants of periodontal disease include
infection allows the clinician to recognize previously a. Age
undiagnosed disease or to modify treatment protocols b. Gender
appropriately. c. Genetic factors
Q.6. Kaposi sarcoma. a. Age
l Early evidence demonstrates that both prevalence
Ans.
and severity of periodontal disease increase with
l Kaposi sarcoma is a malignancy associated with HIV. old age.
l The common locations within the oral cavity are the l However, the age effect most likely represents

palate, maxillary gingiva and dorsal surface of tongue. the cumulative effect of prolonged exposure to
The lesions are reddish to deep purple and may be true risk factors.
macular or nodular. Large gingival lesions often inter- l When good oral hygiene is maintained, effects of

fere with mastication. age on periodontal disease progression could be


l Oral Kaposi sarcoma typically presents as red-bluish considered negligible.
swellings with or without ulceration. b. Gender
l The tumour initially manifests as a red purple or bluish l Multiple studies have demonstrated poor periodon-

patch, which later presents as nodules resembling hae- tal health in men as compared with women. Gen-
mangiomas or a peripheral giant cell granuloma. der association with periodontitis is related to pre-
l The lesions arise initially in the subepithelial or submu- ventive practices rather than any genetic factors.
cosal connective tissue. c. Genetic factors
l Pathologic changes of the bone or the periodontal tissue l Genetic alterations as a risk factor have been
can be seen only after some growth of the tumour. reported by various studies.
l Early-onset periodontitis has been found to have
Q.7. Risk factors associated with periodontal diseases.
a familial aggregation.
Ans. l The second line of evidence has come from twin

studies, which have demonstrated a 50% herita-


Various risk factors associated with periodontal diseases are
bility component for chronic periodontitis.
as follows:
l The third important evidence is the existence of
i. Anatomic factors
gene polymorphisms in a population. Some of
The developmental grooves, furcation areas and pros-
the well-studied gene polymorphisms in peri-
thetic restorations favour accumulation of plaque and
odontal diseases are on immunoglobulin recep-
thereby act as a risk factor.
tors, TNF-a, IL-1b, etc. Individuals with poly-
ii. Smoking
morphisms in the genes coding for these
l Smoking is one of the well-established risk factors
mediators/cytokines/receptors have a great risk
for several of the periodontal diseases.
of suffering from severe forms of periodontitis.
l It affects the progression through its effect on mi-

crobiological profile (increase in anaerobic organ-


Q.8. Periodontal manifestations of HIV infection.
isms), physiology (decrease in pocket temperature)
and immune system (decreased chemotaxis). Ans.
Section | I  Topic-Wise Solved Questions of Previous Years 819

The periodontal manifestations of HIV infection include Regimen Antibiotic Dosage


the following three conditions:
Patients unable Ampicillin 2.0 g intramuscularly
a. Linear gingival erythema to take oral medi- or intravenously
b. Necrotizing ulcerative gingivitis cations within 30 min before
c. Necrotizing ulcerative periodontitis (NUP) procedure

a. Linear gingival erythema Patients unable to Clindamycin 600 mg intrave-


l This is characterized by red to bluish-red oedema-
take oral medica- or nously within
tions and allergic cefazolin 30 min before
tous gingival tissue. to penicillin procedure (must be
l It usually appears with swollen interdental papillae diluted and
and increased tendency to bleeding. injected slowly)
l Significantly more bleeding sites and destruction of in- 1.0 g intramuscularly
terdental papillae are found in HIV-seropositive patients. or intravenously
within 30 min before
b. Necrotizing ulcerative gingivitis procedure
l In HIV-infected patients, the condition has been de-

scribed as red and swollen gingiva with yellowish-


grey marginal areas of necrosis with loss of interden- Q.10. Discuss management of a diabetic patient.
tal papillae.
Ans.
l It is characterized by its extremely rapid progression

and destruction of the periodontal and bony sub- [Ref LE Q.1 and Q.3]
stances usually accompanied by severe pain.
Q.11. Periodontal infection a cause for atherosclerosis.
c. Necrotizing ulcerative periodontitis
l NUP is characterized by generalized aggressive alveo- Ans.
lar bone loss and attachment apparatus destruction.
Atherosclerosis comes from the Greek words ‘athero’ (gruel
l There is also deep-seated pain, spontaneous gingival
or paste) and ‘sclerosis’ (hardness) in which deposits of
bleeding, halitosis and tooth mobility.
fatty substances, like cholesterol, cellular waste products,
All the above three conditions may be collectively
calcium and other substances, accumulate in the blood ves-
referred to as necrotizing gingivostomatitis.
sel wall forming a plaque that elevates into the blood vessel
Staging of necrotizing gingivostomatitis
blocking the blood flow.
Staging of necrotizing infections was proposed by
The ruptured plaque fragments can release certain pro-
Pindborg, who described four stages:
coagulant chemicals that may cause platelet aggregation
I. Only the tip of the interdental papilla was
and in turn cause thrombus formation.
affected.
There are many risk factors for atherosclerosis like diabe-
II. Marginal gingiva was affected, with punched-
tes mellitus, smoking, intake of fatty diet and hypertension.
out papilla.
In addition, microbes and viruses have also been impli-
III. Attached gingiva was also affected.
cated in producing inflammation, leading to atherosclero-
IV. Bone was exposed.
sis. Some of the bacteria cited are Chlamydia pneumoniae
Q.9. Antibiotic prophylaxis for infective endocarditis. and Helicobacter pylori along with viruses like herpes
simplex.
Ans.
Since microbes have been implicated in the initiation of
Recommended antibiotic prophylaxis regimens for peri- atherosclerosis, researchers started to associate periodontal
odontal procedures in adults at risk of infective endocarditis pathogens with atheroma formation, especially Porphy-
are as follows: romonas gingivalis. Deshpande et al. (1998) demonstrated
periodontal pathogens in atheromatous plaques and aortic
endothelial cells. On the other side, different studies have
Regimen Antibiotic Dosage confirmed the strong association between periodontitis and
Standard oral Amoxicillin 2.0 g, 1 h before coronary artery disease.
regimen procedure When putative periodontal microbes forming the sub-
Alternate regimen Clindamycin or 600 mg, 1 h before
gingival flora induce an inflammatory immune response in
for patients allergic Azithromycin/ procedure the gingival tissues, resident cells and inflammatory cells
to amoxicillin/ clarithromycin or 500 mg, 1 h before produce inflammatory cytokines and mediators.
penicillin Cephalexin/ procedure If interventional treatment is not carried out, this gingi-
cefadroxil 2.0 g, 1 h before vitis progresses to periodontitis when there will be in-
procedure
creased cytokines, enzymes and mediators.
820 Quick Review Series for BDS 4th Year, Vol 1

When there is pressure on the periodontium during Q.13. Diabetes as a risk factor in periodontal diseases.
brushing/mastication, tooth movement, microbes, toxins,
Ans.
cytokines and mediators are pushed into the systemic blood
circulation. They travel to distant sites and initiate a pathol- [Same as SE Q.4]
ogy specific to that organ/system.
Q.14. AIDS and periodontal diseases.
Proposed mechanisms by which periodontal infections
may affect the onset and progression of atherosclerosis and Ans.
CHD/stroke:
[Same as SE Q.5]
l Certain microbes like Streptococcus sanguis and Por-

phyromonas gingivalis express a protein called platelet Q.15. Discuss periodontal findings in AIDS.
aggregation-associated protein (PAAP) and aggregate
Ans.
platelets, leading to thromboembolic events.
l Periodontitis influences atherosclerosis in more than [Same as SE Q.5]
one way:
a. Direct invasion of periodontal pathogens
l Periodontal pathogen directly invades endothelial
SHORT NOTES:
cells and induces cytokines, chemokines and ad- Q.1. Oral lesions in diabetes mellitus.
hesion molecules which may initiate/accelerate
Ans.
atherosclerosis.
b. Long-standing infections (e.g. periodontitis) [Same as LE Q.3]
l They may significantly up-regulate inflammatory
Q.2. Scorbutic gingivitis.
cascade that through systemic circulation reach the
vulnerable artery and shift the status of endothelial Ans.
cells from antithrombotic to prothrombotic one.
l Gingivitis seen because of vitamin C deficiency is also
l In addition, periodontitis increases the production
called scorbutic gingivitis.
of coagulation factor VIII/von Willebrand factor
l Severe vitamin C deficiency in humans results in scurvy,
(vWF) antigen, increasing the risk of thrombus
a disease characterized by haemorrhagic diathesis and
formation. Also, periodontitis increases the fi-
retardation of wound healing.
brinogen level and WBC counts, leading to in-
l Vitamin C deficiency (scurvy) results in defective for-
creased blood viscosity.
mation and maintenance of collagen, retardation or ces-
c. Action of macrophages
sation of osteoid formation and impaired osteoblastic
l Phagocytic cells like macrophages ingest peri-
function.
odontal pathogens. Resistant to phagocytosis,
l The classic clinical signs of scurvy describe the gingiva
pathogens survive within the macrophages and
as being bright red, swollen, ulcerated and susceptible
travel to distant sites.
to haemorrhage.
l At certain areas these macrophages with ingested
l Vitamin C deficiency is also characterized by increased
live cells get attached to immunologically com-
capillary permeability, susceptibility to traumatic haem-
promised endothelial cells and initiate the athero-
orrhages, hyporeactivity of the contractile elements of the
sclerotic changes.
peripheral blood vessels and sluggishness of blood flow.
d. Porphyromonas gingivalis
l Porphyromonas gingivalis has certain proteins Q.3. Stress and periodontal diseases.
like heat shock proteins (HSP) that are similar to
Ans.
corresponding human proteins.
l When these proteins travel to sites, antibody di- l Harmful effects that result from psychic influences on
rected towards these bacterial HSP, because of the organic control of tissues are known as psychoso-
structural homology react with human HSP, in- matic disorders.
ducing an autoimmune reaction. This is called l Psychological conditions, particularly psychological

‘molecular mimicry’. stress, have been implicated as risk indicators for peri-
odontal disease.
Q.12. Possible role of ascorbic acid deficiency in aetiol- l The various ways in which psychosomatic disorders may

ogy of periodontal disease. induce periodontal disease in the oral cavity are as follows:
i. Through the development of habits that are injuri-
Ans.
ous to the periodontium, e.g. smoking and alcohol
[Same as SE Q.1] consumption
Section | I  Topic-Wise Solved Questions of Previous Years 821

ii. Poor oral hygiene l Periodontal manifestations include oral ulcer-


iii. Poor nutritional intake ation, inflamed gingiva, rapid periodontal break-
iv. Overeating of high fat diet due to stress and by the down and alveolar bone loss.
direct effect of autonomic nervous system on the l Treatment: Plaque control and supportive
physiologic tissue balance releases cortisol, leading measures.
to depressed immunity b. Chronic benign neutropenia of childhood
All of the above increase susceptibility to bacterial in- l The onset is usually between 6 and 20 months of

fection and lead to periodontal disease. age, and in most patients, the condition is self-
Following are the periodontal manifestations associ- limiting.
ated with stress. l The main periodontal feature is a bright red,

i. Osteoporosis of alveolar bone hyperplastic, oedematous gingiva confined to the


ii. Degeneration of periodontal ligament width of attached gingiva.
iii. Epithelial soughing l Treatment: Appropriate antimicrobial agents.

iv. Reduced osteoblastic activity and formation of peri- c. Benign familial neutropenia
odontal pockets l It is transmitted as an autosomal dominant trait.

v. Delayed wound healing l The periodontal manifestations include hyper-

plastic gingivitis exhibiting oedematous and


Q.4. Periodontal medicine. bright red appearance. The gingival tissues bleed
profusely on probing.
Ans.
l Treatment: Plaque control and use of antimicro-

l The concept of periodontal diseases as localized entities bial mouthwashes.


affecting only the teeth and supporting apparatus is d. Benign familial neutropenia
oversimplified and needs to be revised. l A severe form of benign familial neutropenia

l Periodontal diseases may have wide-ranging sys- e. Chronic idiopathic neutropenia


temic effects. In most persons, these effects may be l There is a persistent neutropenia from birth.

relatively inconsequential or at least not evident l Clinical symptoms include persistent recurrent

clinically. infections throughout the patient’s life.


l However, in susceptible individuals, periodontal infec- l Periodontal manifestations include persistent

tion may act as an independent risk factor for systemic severe bleeding gingivitis.
disease and may be involved in the basic pathogenic l Treatment: strict oral hygiene programme. Anti-

mechanism of these conditions or it may exacerbate septic irrigation and antibiotic prophylaxis are
existing systemic disorders. advisable before tissue manipulation.
l Proper use of knowledge of relationship between peri-

odontal disease and systemic health requires the profes- Q.6. Periodontal care of patients with tuberculosis.
sionals to expand their horizons and to recognize the
Ans.
oral cavity as one of the many interrelated organ sys-
tems. Patient education is also very important in this l The patient with tuberculosis should receive only emer-
regard. gency care.
l If the patient has completed chemotherapy, his/her
Q.5. Mention any five neutrophil disorders causing peri-
physician should be consulted regarding infectivity
odontitis.
and the results of sputum cultures for Mycobacterium
Ans. tuberculosis.
l When medical clearance has been given and the sputum
The WBC disorders that affect the periodontium are as
culture results are negative, these patients may be
follows:
treated normally.
Neutropenias
l Any patient who gives a history of poor medical follow-
a. Cyclic neutropenia
up (e.g. lack of yearly chest radiographs) or shows signs
b. Chronic benign neutropenia of childhood
or symptoms indicative of tuberculosis should be
c. Benign familial neutropenia
referred for evaluation.
d. Severe familial neutropenia
l Adequate treatment of tuberculosis requires a minimum
e. Chronic idiopathic neutropenia
of 18 months, and thorough post-treatment follow-up
a. Cyclic neutropenia should include chest radiographs, sputum cultures and a
l It is characterized by a cyclic depression of the review of the patient’s symptoms by the physician for at
PMN count in peripheral blood. least every 12 months.
822 Quick Review Series for BDS 4th Year, Vol 1

Q.7. Periodontal treatment planning of a dental trans- Q.8. Gingiva in leukaemia.


plant patient.
Ans.
Ans.
[Ref SE Q.3]
l The transplant patients are usually kept on immunosup-
Q.9. Periodontal manifestations of diabetes mellitus.
pressive drugs, which result in decreased resistance of
body to infections. Ans.
l Due to drug-induced thrombocytopenia or anticoagula-
[Same as SN Q.1]
tion or both of them, excessive bleeding may occur dur-
ing or after periodontal therapy. Q.10. Influence of diabetes on the periodontium.
l Prior to transplantation, a dental team approach should be
Ans.
used to determine which teeth can be easily maintained.
l As periodontal abscess is a life-threatening situation in [Same as SN Q.1]
these patients, teeth with severe bone loss, furcation in-
Q.11. Vitamin C in gingival disease.
volvement or those requiring extensive surgical treat-
ment should be extracted before transplantation. Ans.
l In renal transplant patients, the following things should
[Same as SN Q.2]
be considered prior to treatment planning:
i. Screening for hepatitis B and C Q.12. Scurvy.
ii. Prophylactic antibiotics
Ans.
iii. Determination of immune system compromises
levels [Same as SN Q.2]

Topic 13
Dental Implants
COMMONLY ASKED QUESTIONS
SHORT ESSAYS:
1 . Indications and contraindications for implant therapy.
2. Failure of implants.
3. Osteoinduction, osteoconduction and osseointegration.
4. Maintenance of dental implants.

SHORT NOTES:
1 . Peri-implantitis.
2. Home care methods for maintenance of dental implants. [Ref SE Q.4]
3. Osseointegration.
4. Implant-bone interface.
5. Peri-implant diseases. [Same as SN Q.1]
6. Aetiology of peri-implant diseases. [Same as SN Q.1]
Section | I  Topic-Wise Solved Questions of Previous Years 823

SOLVED ANSWERS
SHORT ESSAYS:
Q.1. Indications and contraindications for implant ii. Surgical complications include
therapy. l Oversized osteotomy site preparation

l Broken burs
Ans.
l Improper angulations
A. Indications for dental implants are extensive and are l Inappropriate instrumentation
as follows: l Perforation
l Patients with edentulous mandible, maxilla or both l Haemorrhage
l Edentulous frontal regions l Inadequate amount of soft tissue
l Patients with single tooth loss iii. Complications in early stages:
l Patients with loss of multiple teeth (Kennedy classes l Postoperative infections
2 and 3) l Dysaesthesia
B. Contraindications for implants are described under l Dehiscence
general, medical and dental conditions as follows: l Sinusitis
i. General contraindications l Radiolucencies
l Economic/cost factor l Mobility
l Nonmotivated patient iv. Late failures
ii. Medical contraindications The late failures are often the results of excessive load
l Chronic smoking or infection. These failures are summed up into two
l Uncontrolled diabetes mellitus groups:
l Uncontrolled hypertension a. Mechanical complications
l Patients undergoing radiation therapy b. Biological complications
l Patients on steroid therapy
a. Mechanical complications
l Psychological problems
l Mechanical complications arise as a result of
iii. Dental contraindications
occlusal overloading.
l Inadequate bone
l Forces resulting from functional or parafunc-
l Close proximity to the vital structures
tional occlusal contacts of the natural teeth re-
l Poor oral hygiene
sult in physiological adaptation of the tissues
l Local pathology
of the attachment apparatus.
All the above-mentioned contraindications are not abso-
l As osseointegrated implants have no periodon-
lute contraindications but indicate a less than optimal suc-
tal ligament; the adverse forces generated by
cess rate for the implants.
occlusal activity may lead to high stress and
Q.2. Failure of implants. microfractures in the coronal bone to implant
contact and thus lead to loss of osseointegra-
Ans.
tion around the neck of the implant.
Implant failures do happen in implant therapy in spite of the l Unfavourable forces may also lead to mechan-
long-term predictability of osseointegration. ical complications such as screw loosening,
Failure of implants could be due to: screw fracture or fixture fracture.
i. Improper patient selection l Abutment screw fracture is another frequent
ii. Surgical complications problem of major concern.
iii. Complications during early stages following implant b. Biological complications
placement l These complications are bacterial in origin and
iv. Late failures are characterized by bone loss combined with
a soft tissue inflammatory response that dem-
i. Improper patient selection onstrates suppuration with probing depth of
l The patients who are unmotivated to control plaque more than 6 mm.
l Chronic smokers l Clinically unhealthy implants are classified as
l Patients with systemic conditions like uncontrolled ‘ailing’ or ‘failing’.
diabetes l An implant exhibiting peri-implant mucositis
l Patients with insufficient quality and quantity of is an ailing implant, as it involves only inflam-
bone to support the implant fixture matory changes confined to the soft tissue
824 Quick Review Series for BDS 4th Year, Vol 1

surrounding an implant. Sometimes, they may facilitate treatment of disease processes after successful
exhibit early bone loss along with soft tissue implant therapy.
pocketing but this bone loss tends to become l Before starting a maintenance programme for a patient,
static at 3- to 4-month maintenance checks. we should ensure that baseline data have been established.
l A failing implant may show evidence of pock- Probing pocket depth and mucosal margins are noted and
eting, bleeding on probing, purulence and indi- radiographic crestal bone levels are established.
cations of bone loss that are progressing de- l The evaluation is done initially after 1 day, 1 month,
spite previous therapy. 3 months, 6 months and then at yearly intervals.
l An implant that is progressively losing its bone l During every visit, a detailed examination of the im-
anchorage but is clinically stable can be de- plant should be done prior to instrumentation. This in-
fined as failing implant. Failing implants have cludes evaluation of soft tissue around the implant, im-
a poor prognosis. plant mobility, prosthesis, radiographs and plaque and
l Microflora associated with failing dental im- calculus scores.
plants is identical to those found in chronic l Debridement of implant/removal of the deposits on the
adult periodontitis. implant should be accomplished with instruments that
l Clinically, a failed implant exhibits clinical do not scratch the implant surface, e.g. plastic, resin,
mobility, peri-implant radiolucency and a dull graphite and gold-tipped scalers.
sound when percussed. The failed implant l Instruments such as stainless steel curettes and standard
must be removed because it is nonfunctional ultrasonic and sonic instruments are not recommended
and will only lead to further deterioration of to be used on implants.
the implant site. l A rubber cup can be used to polish the implant surface with
a nonabrasive toothpaste, fine polishing paste or tin oxide.
Q.3. Osteoinduction, osteoconduction and osseointegra-
tion.
{SN Q.2}
Ans.
Home care methods
Osteoinduction
Various home care methods for maintenance of den-
l It involves new bone formation from osteoprogenitor
tal implants are as follows:
cells derived from primitive mesenchymal cells un-
l A soft sulcular toothbrush is the primary
der the influence of one or more inducing agents that
plaque control device around an implant resto-
emanate from bone matrix.
ration.
l During remodelling process, osteoinductive materi-
l Powered and sonic toothbrushes are also safe
als are more contributory to bone formation.
to use around titanium implants.
l Bone allografts and autografts are most commonly
l Foam tips, interproximal brushes and dispos-
used osteoinductive materials in implant dentistry.
able wooden picks are certain auxiliary devices
Osteoconduction
that can assist in plaque removal from the em-
l It characterizes bone growth by apposition from the
brasures.
surrounding bone. This process must occur in the
l Chemical antiplaque agents can be delivered
presence of bone or differentiated mesenchymal cells.
with interdental aids to enhance their effective-
l Alloplasts and xenografts are the most common
ness.
osteoconductive materials used in implant dentistry.
l Every patient with dental implants requires a
Osseointegration
lifetime careful maintenance to ensure implant
l It is a histologic term defined as bone in direct
health and longevity.
contact with an implant at a magnification of light
microscope.
l The direct bone to implant interface without inter- SHORT NOTES:
vening connective tissue was described as osseoin-
Q.1. Peri-implantitis.
tegration earlier by Strock and more recently by
Branemark et al. Ans.
Q.4. Maintenance of dental implants. l As soon as an implant is exposed to the oral cavity,
plaque will form on its surface. This process is similar
Ans.
to that seen on the natural teeth.
l The patient should be recalled at regular intervals l Pathologic changes of the peri-implant tissues are cate-

in order to provide optimal preventive services and gorized as peri-implant diseases or peri-implantitis.
Section | I  Topic-Wise Solved Questions of Previous Years 825

l Peri-implant infections are classified as peri-implant Q.2. Home care methods for maintenance of dental
mucositis and peri-implantitis depending on the severity. implants.
l Peri-implant mucositis is a term used to describe revers-
Ans.
ible inflammatory reactions in the mucosa adjacent to
an implant. These inflammatory reactions subsequently [Ref SE Q.4]
progress and lead to loss of supporting bone, resulting
Q.3. Osseointegration.
in an irreversible condition known as peri-implantitis.
l The organism associated with peri-implantitis is similar Ans.
to that of periodontitis.
l It is defined as a direct structural and functional connection
Signs and symptoms of peri-implantitis
between bone and the surface of a load-carrying implant.
l Pockets, bleeding, suppuration, swelling of peri-
l In 1960, the concept of osseointegration was put for-
implant tissues and vertical destruction of the
ward by Branemark et al.
crestal bone are associated with peri-implantitis.
l This concept of a direct bone-to-implant contact was
l The defect is usually saucer shaped, with osseoin-
proved by Schroeder et al.
tegration only at the apical part of the fixture.
l However, it has been shown that 100% bone connection
l Pain is an unusual factor but when present is
with the implant does not occur, and the exact degree of
associated with acute infection.
bone attachment with the implant, to be termed as os-
l Peri-implantitis if left untreated can progress deep
seointegration, has led to a definition of osseointegra-
into supporting bone and lead to implant loss. The
tion based on clinical stability of the implant instead of
overall goal of therapy is to establish a functional
histological criteria.
restoration and acceptable aesthetics. Therefore,
any therapy provided should arrest further loss of Q.4. Implant–bone interface.
bone support and re-establish a healthy peri-implant
Ans.
mucosal seal.
Nonsurgical therapy The relationship between implant and bone involves mech-
l The most conservative approach to treatment in- anisms like the following:
volves nonsurgical therapy. This treatment modal- i. Fibro-osseous integration
ity includes pharmacological therapy, occlusal ii. Osseointegration
therapy and mechanical debridement. iii. Bioactive integration
l Pharmacological therapy for patients presenting

with an ailing implant involves subgingival irriga- Fibro-osseous integration


tion for 10 days to 3 weeks (2–3 times per 24-h When soft tissues are interposed between the implant
time period). This may be completed at home fol- surface and the bone, it is known as fibro-osseous inte-
lowing careful instructions from the clinician or gration. This means that there is encapsulation of the
dental hygienist. implant and rejection of the implant by the body.
l Chlorhexidine is most often prescribed because of Osseointegration
its antimicrobial effect and substantivity at the It is defined as a direct structural and functional connec-
affected site. tion between bone and the surface of a load-carrying
l Other pharmacological therapies include local appli- implant.
cation of tetracycline fibres and systemic antibiotics. Biointegration
l A third nonsurgical therapy recommended for It is achieved in cases where the implant is coated with
treating the ailing or failing implant is mechanical bioactive materials like hydroxyapatite. These materials
debridement. stimulate bone formation, leading to a physicochemical
l Preparation of implant surface can be achieved bond. Such implants have been found to attain rapid
with a supersaturated solution of citric acid for 30–60 s integration.
in order to remove the endotoxins from the implant
Q.5. Peri-implant diseases.
surfaces. Irradiation with soft laser for elimination of
bacteria associated with peri-implantitis has also Ans.
shown promising results in the destruction of bacte-
[Same as SN Q.1]
rial cell.
Surgical techniques Q.6. Aetiology of peri-implant diseases.
l When nonsurgical therapies are ineffective, surgi-
Ans.
cal techniques are indicated. They include resec-
tive and regenerative techniques. [Same as SN Q.1]
826 Quick Review Series for BDS 4th Year, Vol 1

Topic 14
Defence Mechanisms of the Gingiva
COMMONLY ASKED QUESTIONS
LONG ESSAYS:
1 . Describe in detail the defence mechanism of gingiva.
2. Discuss in detail the defence mechanisms of the gingiva. [Same as LE Q.1]

SHORT ESSAYS:
1 . Role of saliva in oral health. [Ref LE Q.1]
2. Methods of collection of GCF.
3. Gingival fluid.
4. Role of saliva in oral defence mechanism. [Ref LE Q.1]
5. Saliva in oral defence. [Same as SE Q.1]
6. GCF. [Same as SE Q.3]
7. Enzymes in gingival fluid. [Same as SE Q.3]
8. Factors affecting GCF flow. [Same as SE Q.3]
9. Composition of gingival cervical fluid. [Same as SE Q.3]

SHORT NOTES:
1. Gingival fluid.
2. Functions of saliva.
3. Intrasulcular drug delivery.
4. Antibacterial factors of saliva.
5. Clinical significance of gingival fluid.
6. Name defence mechanisms of gingiva. [Ref LE Q.1]
7. Orogranulocytes.
8. Saliva.
9. GCF. [Same as SN Q.1]
10. Clinical significance of crevicular fluid. [Same as SN Q.5]
11. Enumerate the protective component of saliva in periodontal disease process. [Same as SN Q.5]
12. Role of saliva as a defence mechanism of gingiva. [Same as SN Q.8]

SOLVED ANSWERS
LONG ESSAYS:
Q.1. Describe in detail the defence mechanism of {SN Q.6}
gingiva.
The various mechanisms that play an important role in
Ans.
the defence of the periodontal tissues against infection
are as follows:
i. Epithelial barrier
Section | I  Topic-Wise Solved Questions of Previous Years 827

in turn cause further activation of the junctional


ii. Gingival crevicular fluid (GCF)
epithelial cells.
iii. Saliva
l Osteopontin, a phosphorylated glycoprotein, has a
iv. Orogranulocytes )
pivotal role in the development of immune re-
sponses and subsequent repair process that occurs
I. Epithelial barrier following tissue destruction in various inflamma-
l The stratified epithelia of the skin and oral mucosa are tory diseases.
examples of the toughest and the most protective epi- II. GCF
thelia and provide a barrier between an organism and Composition of GCF
its environment and perform many additional special- a. Cellular elements
ized functions depending on the site of the body. i. Desquamated epithelial cells
l The protective barrier is made of tough epithelial ii. Leukocytes
layer. Epithelial cell adhesion involves desmosomes iii. Bacteria
that maintain intercellular adhesion, hemidesmosomes b. Electrolytes
that maintain keratinocyte basement membrane adhe- i. Sodium
sion and intercellular adherent junctions that connect ii. Potassium
to the actin cytoskeleton in virtually all cell types. iii. Calcium
l The keratinized epithelia with the cornified cell enve- iv. Fluoride
lope function as a critical part of the protective bar- v. Magnesium
rier provided by these tissues. vi. Phosphates
l The disruption of any of the above leads to c. Organic compounds
genetic and autoimmune diseases that result from i. Carbohydrates
altered expression or function of these structures. ii. Proteins
l Acute, chronic and neonatal stress affects the muco- iii. Lipids
sal barrier by increasing the permeability of the epi- iv. Immunoglobulins
thelial tight junctions. v. Complement components
l This increase in paracellular permeability depends upon d. Metabolic and bacterial products
the release of interferons from CD41 lymphocytes and l Lactic acid
involves mast cell activation of the actinomyosin ring l Hydroxyproline
and contraction of epithelial cells. This altered perme- l Prostaglandins
ability favours the permeation to antigens and toxins, l Urea
and also to bacteria inducing a local immune activation. l Endotoxins
Junctional epithelium in antimicrobial defence l Cytotoxic substances
l The active populations of cells and antimicrobial l Antibacterial factors
functions of junctional epithelium together form e. Enzyme and enzyme inhibitors
the first line of defence against microbial invasion l Acid phosphatase
into the tissues. l Alkaline phosphatase
l The internal and external basal laminas act as bar- l Pyrophosphatase
riers against infective agents. l b-glucuronidase
l Constant and rapid cell turnover that occurs within l Lysozyme
the junctional epithelium is an important factor in l Hyaluronidase
the microbial defence. l Proteinases (mammalian proteinases/bacterial
l Rapid shedding of epithelial cells and effective proteinases, e.g. cathepsin D and G, elastase
removal of bacteria adhering to the epithelial cells and collagenase
is an important part of the antimicrobial defence l Lactic dehydrogenase
mechanisms at the dentogingival junction.
l Active antimicrobial substances produced by the

junctional epithelium, such as defensins, lyso- The action of some of the important components is
somal enzymes, chemokines (IL-8) and cytokines tabulated as follows:
(IL-1, IL-6 and TNF-a), which are secreted as a A. Cellular elements Action
response to microbial challenge by the epithelial
i. Epithelial cells Form a physical barrier
cells, attract and activate professional defence
cells such as lymphocytes and polymorphonu- ii. Leukocytes Phagocytosis and killing of
microorganisms
clear leukocytes (PMNs). These secreted products
828 Quick Review Series for BDS 4th Year, Vol 1

B. Electrolytes b. Elastase l Acts upon elastin, proteogly-


cans, fibrinogen and
i. Sodium l Genesis of plaque
collagen
ii. Potassium l Precipitation of proteins l Widening of epithelial

l Precipitation of mucoproteins intercellular spaces, results


along enamel surface in partial destruction of basal
membrane and loss of
C. Organic compounds collagen
i. Immunoglobulins Immune function c. Cathepsin-G l Hydrolyses haemoglobin, fi-
l Activation of cells brinogen, collagen and pro-
l Control of inflammatory
teoglycans
reaction
l Elimination of antigen d. Plasminogen activator l Fibrinolysis
l Preparation of microbes (streptokinase and uroki- l Plays a role in inflammation
and foreign particles for nase) l Essential for wound healing

phagocytosis
e. Collagenase l Collagenolytic activity
l Play a role in immune

response f. Bacterial proteases l Tissue damage


D. Metabolic and bacterial products g. Lactic dehydrogenase l Catalyses the reversible re-
duction of pyruvate to lactate
i. Prostaglandins l Vasodilation and inhibition of
collagen synthesis
l Bone resorption

ii. Urea l Elevates the pH of supragingi-


val plaque in presence of gin-
III. Saliva
givitis and periodontitis due
to production of ammonia by
[SE Q.1]
microorganisms
iii. Endotoxins l Highly toxic to gingival tissue {Protection
l A major protective function of saliva results from
iv. Cytotoxic substances l Highly toxic metabolite (cyto-
(like H2S) toxic effect)
its role in maintenance of ecological balance in
the oral cavity via:
v. Antibacterial factors Prevents growth of bacteria
l
a. Debridement/lavage
E. Enzyme and enzyme inhibitors b. Aggregation and reduced adherence by both
Acid phosphatase l Associated with connective immunologic and nonimmunologic means
tissue catabolism c. Direct antibacterial activity
l Attacks teichoic acid, one of d. Also possesses antifungal and antiviral sys-
the components of bacterial tems
cell wall
l Saliva effectively maintains pH in the oral cavity,
Alkaline phosphatase l Plays a role in calcification contributes to the regulation of plaque pH and
Pyrophosphatase l Plays a role in calculus for- helps neutralize reflux acids in the oesophagus.
mation l The most important salivary buffer in saliva is

b-Glucuronidase l Used as lysosomal marker bicarbonate–carbonic acid system.


l Bicarbonate and phosphate ions as well as basic
Lysozyme l Bactericidal properties and
also some detrimental effect
proteins in saliva maintain the near neutral pH in
upon epithelial cells the oral cavity, which prevents demineralization
l Lytic effect of connective tis- of enamel that would otherwise occur due to acids
sue, thereby contributing to produced by sugar-metabolizing bacteria.
formation of pocket l The fluid nature and the components of saliva

Hyaluronidase l Widening of intercellular protect the oral cavity in several ways. Saliva
spaces in the junctional provides a washing action that clears nonadher-
epithelium ent, potentially harmful substances in the oral
Mammalian proteases cavity.
l Viscous components, e.g. mucins lubricate oral tis-
a. Cathepsin-D Attacks various components of
epithelium and connective tissue sues and form a barrier against microbial products.
Section | I  Topic-Wise Solved Questions of Previous Years 829

l Salivary proteins form a thin coating on tooth The role of saliva in oral health is summarized as follows:
surfaces, the salivary pellicle that contributes to Salivary Probable
protecting these surfaces. Function components mechanism
l Saliva is supersaturated with calcium and phos-
Lubrication Glycoproteins, Coating similar to
phate ions, and this state is maintained by certain mucoids gastric mucin
calcium-binding proteins, notably acidic proline-
Physical Glycoproteins, Coating similar to
rich proteins and statherin. This leads to post-
protection mucoids gastric mucin
eruptive maturation of enamel surfaces, which
increases their hardness and resistance to demin- Cleansing Physical flow Clearance of debris and
bacteria
eralization.
Such environment also favours remineraliza- Buffering Bicarbonate and Antacids
tion of beginning carious lesions (white spots), phosphate
provided that cavitation does not occur. Tooth integrity Minerals Maturation, remineral-
Antibacterial properties of saliva maintenance Glycoprotein ization
l Saliva plays a major bacteriostatic role in the oral
pellicle Mechanical protection
cavity by interfering with microbial colonization. Antibacterial Immunoglobulin A Control of bacterial
l Mucins form a physical antimicrobial barrier. action Lysozyme colonization
l Salivary IgA (sIgA) is an important factor in oral
Lactoperoxidase Breaks bacterial cell
walls
immune defence; together with salivary aggluti- Oxidation of suscepti-
nins (glycoproteins), sIgA causes clumping of ble bacteria
certain microorganisms, thus preventing them
from adhering to oral and dental surfaces.
l Salivary antibodies appear to be synthesized lo-
IV. Orogranulocytes
cally, since they react to strains of bacteria that are
l The viable neutrophils present in the saliva are
indigenous to the oral cavity.
termed as orogranulocytes or salivary corpuscles.
l Other components, namely histatins, lysozyme,
l In normal individuals, 30,000 neutrophils per min-
lactoferrin and myeloperoxidase inhibit bacterial
ute enter the oral cavity via the gingival sulcus. This
growth.
flow of neutrophils is required for periodontal
l Lysozyme is a hydrolytic enzyme which is effec-
health.
tive against both Gram-negative and Gram-positive
l Any defect in neutrophil function and chemotaxis is
organisms, specifically acting against the Actinoba-
associated with early-onset periodontal disease in
cillus and Veillonella species.
children.
l The Lactobacillus thiocyanate system is another
l This flow of neutrophils may also be important for
effective defence mechanism inherent in the sa-
protection from caries. These viable neutrophils
liva that has bactericidal properties to some strains
present in the saliva are termed as orogranulocytes
of Lactobacillus and Streptococcus species. It
or salivary corpuscles.
prevents the accumulation of lysine and glutamic
l The oral leukocyte migratory rate (OMR) allows an
acid, both of which are essential for growth of
objective assessment of periodontal health.
various bacterial species.
The orogranulocytic migratory rate (OMR) has
l Saliva contains another antibacterial agent called
been shown to be a nonsubjective laboratory index
lactoferrin that has a potent bactericidal action
for inflammatory periodontal disease.
against the Actinobacillus species.
l Myeloperoxidase, an enzyme released by leuko- Q.2. Discuss in detail the defence mechanisms of the
cytes, has potent bactericidal activity against the gingiva.
Actinobacillus species besides preventing the at-
Ans.
tachment of certain initial colonizers, namely Ac-
tinomyces strains, to the hydroxyapatite in the [Same as LE Q.1]
tooth surface.
l Tissue inhibitors of matrix metalloproteinases
SHORT ESSAYS:
are the components in saliva with the ability to
inhibit the activity of collagen-degrading en- Q.1. Role of saliva in oral health.
zymes (collagenase), which may be associated
Ans.
with bacterial activity or host-mediated cellular
responses.} [Ref LE Q.1]
830 Quick Review Series for BDS 4th Year, Vol 1

Q.2. Methods of collection of GCF. volumes of fluids as compared with the former
two models.
Ans.
Advantages of using absorbing paper strips
The various methods of collecting GCF are as follows: l It is a simple procedure that can be visual-

i. Use of absorbing paper strips ized directly.


ii. Twisted threads placed around and into the sulcus l Quantitative assessment of the fluid can be

iii. Sampling by means of micropipettes obtained.


iv. Crevicular washing methods l It seems to be compatible with subsequent

The most difficult hurdle to overcome when collecting chemical analysis.


GCF is the scarcity of material that can be obtained l Evaporation is kept to a minimum by using

from the sulcus. Periotron.


Methods of collection depending upon the nature of the Disadvantages of using absorbing paper strips
analyte under investigation l The nonspecific attachment of the analyte to

It is usually made from the anterior teeth (least con- the fibres of paper strip reduces the quantity
tamination). of the detectable sample.
l The intrasulcular technique/Brill’s tech-

i. Use of absorbing paper strips nique causes irritation of the sulcular epi-
The placement of filter paper strip in relation to the thelium that can itself trigger the flow of
sulcus or pocket is important. GCF.
Two techniques are followed: l To minimize this irritation, Loe and Holm-
A. Intracrevicular Pedersen placed the filter paper strip just at
The end of the paper strip is gently inserted into the the entrance of the pocket or over the pocket
pocket until minimum resistance is felt. entrance.
B. Extracrevicular l In this way, fluid seeping out is picked up
In this technique, the strip is placed at the by the strip but the sulcular epithelium is
entrance of the gingival crevice. not in contact with the paper.
Evaluation of amount of fluid collected: l Contamination can occur. In case of evapo-
a. Appreciation by direct viewing and staining, ration of sample, it has to be repeated many
proposed by Egelberg and Attstrom times. It is not very reliable (ninhydrin tech-
l The strip stained with an alcoholic solution of nique).
ninhydrin at a concentration of 0. 2% gives l Dislocation of the paper strip thereby
blue or purple colour. disturbes the integrity of the marginal
l The stained area can then be measured with a tissues.
transparent scale, calipers or calibrated mag- l When Periotron is used, daily check on the
nifying glass. accuracy of reading should be performed;
b. By weighing the strip care should be taken to insert paper strips
l The strip is weighed before collection of the into the machine in a standardized position
sample within a sealed micro-centrifugation for correct reading.
plastic tube, and is also weighed immediately ii. Twisted threads placed around and into the gingival
after the collection of the sample. sulcus
c. Use of Periotron l Twisted threads have also been used in the collection
l This is the latest and standard method for of GCF from gingival sulcus.
measuring gingival fluid absorbed on paper l Pre-weighed twisted threads were used by Weinstein
strips. It was developed by Harco Electronics. et al.
l HAR-600 is an electronic device whose func- l The threads were placed in the gingival crevice
tioning units are a pair of upper and lower around the tooth, and the amount of fluid collected
counterparts which can be opened and closed in was estimated by weighing the sample thread.
order to insert or remove the strip of filter paper. iii. Sampling by means of micropipettes
l A moistened strip of paper when inserted be- l Krasse and Egelberg were the first to utilize capil-
tween the two jaws will give a reading on the lary tubing, which permits absorption by capillarity.
screen. l Capillary tubes of standardized length and diameter
l HAR-6000 is the latest technique which was are placed in the pockets, their content is centrifuged
found to be sensitive in detecting small and analysed.
Section | I  Topic-Wise Solved Questions of Previous Years 831

Advantages l Total and differential leukocyte counts can be


l This method allows for complete collection and obtained.
assessment of the collected sample. Disadvantages
Disadvantages l The technique does not permit absolute quan-

l Due to viscosity of the fluid, aspiration through a titative assessments, as the dilution factor can-
pipette is very difficult, hence collection of fluid not be determined.
is difficult. Other methods
l The recovery of the sample can also be very l Isotope dilution method

demanding. l Plastic strips

iv. Crevicular washing methods l Platinum loops

Crevicular washings can be used to study GCF from Q.3. Gingival fluid.
clinically normal gingiva.
Ans.
There are two techniques that are available for the study
of gingival fluid components. l GCF is one of the defence mechanisms of the gingiva.
First method The role of GCF in maintaining health of the tissues is
l This was proposed by Tokamoli and Oppenheim extremely important.
and is based on individual acrylic appliances. l Even in gingival health, there is a fluid transudate

l This method uses an appliance consisting of a that flows from the site of this seal, presumably as a
hard acrylic plate covering the maxilla with soft mechanical factor in minimizing bacterial accumu-
borders and a groove following the gingival mar- lation.
gins. It is connected to four collection tubes. l This fluid also contains a variety of macromolecular

l The washings are obtained by rinsing the crevicu- components that are derived from the serum and the
lar areas from one side to the other, using a peri- interstitia of the gingiva.
staltic pump. l GCF is a fluid occurring in minute amounts in the gin-

l A modification of the first method uses two injec- gival crevice. It can be collected from the gingival crev-
tion needles fitted one within the other such that ice surrounding the teeth.
during sampling the inside (ejection) needle is at l It cleanses material from the crevice, containing sticky

the bottom of the pocket, and the outside (collect- plasma proteins that improve adhesions of the epithe-
ing) needle is at the gingival margin. lial attachment, has antimicrobial properties, and exerts
l The collection needle is drained into a sample antibody activity.
tube by continuous suction. l As such, the fluid reflects the constituents of serum, the

Advantages cellular response in the periodontium and contributions


l It is useful for longitudinal studies. from the gingival crevice.
l Concentrations of various enzymes and the num- l GCF is an inflammatory exudate. Its presence in clini-

ber of cells like PMNs and epithelial cells at the cally normal sulci can be explained because gingiva that
marginal area could be followed by this technique. appears clinically normal invariably exhibits inflamma-
l It permits collection of gingival fluid without tion when examined microscopically.
disturbing the integrity of the marginal tissue. l The amount of GCF is greater when inflammation is

l Contamination is least by this technique. present and is sometimes proportional to the severity of
Disadvantages inflammation.
l It is a complex procedure. l GCF production is not increased by trauma from occlu-

l It represents dilution of crevicular fluid. sion but by mastication of coarse food, toothbrushing
Second method and gingival massage, ovulation, hormonal contracep-
l This was proposed by Skapski and Lehner. tives and smoking.
l The procedure involves the ejection and re-aspiration l Other factors that influence the amount of GCF are cir-

of a known amount of solution into a given interden- cadian periodicity and periodontal therapy.
tal crevice. l Circadian periodicity: There is a gradual increase in

Advantages GCF amount from 6 a.m. to 10 p.m. and a decrease


l It has an advantage of being useful for cases of afterwards.
clinically normal gingiva. l Sex hormones: Female sex hormones increase GCF

l It is useful for studying the number and func- flow, probably because they enhance vascular permea-
tional state of cells and bacteria from the cre- bility. Pregnancy, ovulation and hormonal contracep-
vicular area. tives increase gingival fluid production.
832 Quick Review Series for BDS 4th Year, Vol 1

Q.4. Role of saliva in oral defence mechanism. coating similar to gastric mucin due to salivary compo-
nents like glycoproteins and mucoids.
Ans.
ii. Cleansing: By its physical flow, it permits clearance of
[Ref LE Q.1] debris and bacteria.
iii. Buffering: Carbonates and phosphates present in saliva
Q.5. Saliva in oral defence.
cause buffering action by forming antacids.
Ans. iv. Tooth integrity maintenance: Mineral content of saliva
causes glycoprotein pellicle maturation, remineraliza-
[Same as SE Q.1]
tion and provides mechanical protection.
Q.6. GCF. v. Antibacterial action: Salivary components like immu-
noglobulin A, lysozyme and lactoperoxidase control
Ans.
bacterial colonization, break bacterial cell walls and
[Same as SE Q.3] cause oxidation of susceptible bacteria, respectively.
Q.7. Enzymes in gingival fluid. Q.3. Intrasulcular drug delivery.
Ans. Ans.
[Same as SE Q.3] Drugs in GCF
l Drugs that are excreted through GCF may be used
Q.8. Factors affecting GCF flow.
advantageously in periodontal therapy.
Ans. l Tetracyclines and metronidazole are good examples

of intrasulcular drugs. These drugs are excreted in


[Same as SE Q.3]
crevicular fluid when given systemically.
Q.9. Composition of gingival cervical fluid. l When these drugs are given locally, higher concen-

trations are achieved in the desired area leading to


Ans.
faster action as well as sparing morbidity of systemi-
[Same as SE Q.3] cally given antibiotics.
Q.4. Antibacterial factors of saliva.
SHORT NOTES: Ans.
Q.1. Gingival fluid.
l Saliva plays a major bacteriostatic role in the oral cavity.
Ans. l It interferes with microbial colonization, and mucins
form a physical antimicrobial barrier.
i. GCF occurs in minute amounts in the gingival crevice.
l sIgA is an important factor in oral immune defence,
ii. It is one of the defence mechanisms of the gingiva. As
together with salivary agglutinins (glycoproteins) sIgA
such, the fluid reflects the constituents of serum, the
causes clumping of certain microorganisms, thus pre-
cellular response in the periodontium and contributions
venting them from adhering to oral and dental surfaces.
from the gingival crevice.
l Other components, namely histatins, lysozyme, lacto-
iii. GCF can be collected from the gingival crevice sur-
ferrin and myeloperoxidase inhibit bacterial growth.
rounding the teeth.
l Lysozyme, a hydrolytic enzyme, is effective against both
iv. Even in gingival health, there is a fluid transudate that
Gram-negative and Gram-positive organisms. They spe-
flows from the site of this seal, presumably as a me-
cifically act against the Actinobacillus and Veillonella sp.
chanical factor in minimizing bacterial accumulation.
l The Lactobacillus thiocyanate system is another effec-
v. The role of GCF in maintaining health of the tissues is
tive defence mechanism inherent in the saliva that has
extremely important and is as follows:
bactericidal properties to some strains of Lactobacillus
a. It cleanses material from the crevice.
and Streptococcus species.
b. It has antimicrobial properties and exerts antibody
l Lactoferrin is another antibacterial agent present in
activity.
saliva that has a potent bactericidal action against the
Q.2. Functions of saliva. Actinobacillus species.
l Myeloperoxidase, an enzyme released by leukocytes,
Ans.
has potent bactericidal activity against the Actinobacil-
The functions of saliva are as follows: lus species, besides preventing the attachment of certain
i. Lubrication and physical protection: Saliva helps initial colonizers, namely Actinomyces strains on the
in lubrication and physical protection by forming a tooth surface.
Section | I  Topic-Wise Solved Questions of Previous Years 833

Q.5. Clinical significance of gingival fluid. Q.8. Saliva.


Ans. Ans.
l GCF is an inflammatory exudate. l Saliva forms an important aspect in defence against
l Its presence in clinically normal sulci can be explained microbial invasion.
because gingiva that appears clinically normal invariably l The fluid nature and the components of saliva protect

exhibits inflammation when examined microscopically. the oral cavity by washing action that clears nonadher-
l The amount of GCF is greater when inflammation is ent, potentially harmful substances in the oral cavity.
present and is sometimes proportional to the severity of l Viscous components, e.g. mucins, lubricate oral tissues

inflammation. and form a barrier against microbial products.


l GCF is increased by mastication of coarse food, tooth- l Bicarbonate, and phosphate ions as well as basic pro-

brushing and gingival massage, ovulation, hormonal teins in saliva maintain the near neutral pH in the oral
contraceptives and smoking. cavity, which prevents demineralization of enamel.
l Other factors that influence the amount of GCF are cir- l Salivary proteins form a thin coating on tooth surfaces,

cadian periodicity and periodontal therapy. the salivary pellicle that contributes to protecting these
a. Circadian periodicity: There is a gradual increase in surfaces.
GCF amount from 6 a.m. to 10 p.m. and a decrease l Saliva is supersaturated with calcium and phosphate

afterwards. ions, and this state is maintained by certain calcium-


b. Sex hormones: Female sex hormones increase GCF binding proteins, notably acidic proline-rich proteins
flow. Pregnancy, ovulation and hormonal contracep- and statherin. This leads to posteruptive maturation of
tives increase gingival fluid production. enamel surfaces, which increases their hardness and
resistance to demineralization. Furthermore, such envi-
Q.6. Name defence mechanisms of gingiva.
ronment also favours remineralization of beginning
Ans. carious lesions.
[Ref LE Q.1] Q.9. GCF.
Q.7. Orogranulocytes. Ans.
Ans. [Same as SN Q.1]
l The viable neutrophils present in the saliva are termed Q.10. Clinical significance of crevicular fluid.
as orogranulocytes or salivary corpuscles.
Ans.
l In normal individuals 30,000 neutrophils per minute

enter the oral cavity via the gingival sulcus. This flow of [Same as SN Q.5]
neutrophils is required for periodontal health.
Q.11. Enumerate the protective component of saliva in
l Any defect in neutrophil function and chemotaxis is as-
periodontal disease process.
sociated with early-onset periodontal disease in children.
l This flow of neutrophils may also be important for Ans.
protection from caries. These viable neutrophils present
[Same as SN Q.5]
in the saliva are termed as orogranulocytes or salivary
corpuscles. Q.12. Role of saliva as a defence mechanism of gingiva.
l The orogranulocytic migratory rate (OMR) has been
Ans.
shown to be a nonsubjective laboratory index for in-
flammatory periodontal disease. [Same as SN Q.8]
834 Quick Review Series for BDS 4th Year, Vol 1

Topic 15
Gingival Inflammation and Clinical Features
of Gingivitis
COMMONLY ASKED QUESTIONS
LONG ESSAYS:
1. Define gingival recession. Enumerate its causes.
2. Describe correlation of clinical and underlying microscopic features of acute and chronic gingivitis.
3. Define gingival bleeding. Write about causes and management of gingival bleeding.
4. Define gingival abscess. Write in detail about aetiology and treatment of the same.
5. Enumerate the stages of gingivitis and treatment for advanced stage of gingivitis.
6. What is gingival recession? How will you treat a case of localized gingival recession on mandibular left central
incisor? [Same as LE Q.1]
7. Discuss in detail the aetiology, classification and management of gingival recession. [Same as LE Q.1]
8. Discuss the causes of gingival bleeding. [Same as LE Q.3]
9. Discuss gingival bleeding on probing. [Same as LE Q.3]
10. Enumerate stages of gingivitis. Discuss established lesion. [Same as LE Q.5]
11. Established lesion of chronic gingivitis. [Same as LE Q.5]

SHORT ESSAYS:
1 . Gingival recession. [Ref LE Q.1]
2. Describe the clinical features and management of acute pericoronitis.
3. Gingival pigmentations.
4. Management of localized gingival bleeding.
5. Gingival bleeding on probing.
6. Treatment for localized gingival recession. [Same as SE Q.1]

SHORT NOTES:
1. Stillman cleft.
2. McCall festoons.
3. Gingival recession.
4. Causes of gingival bleeding on probing. [Ref SE Q.5]
5. Causes of gingival recession. [Ref LE Q.1]
6. Enumerate the stages of gingivitis. [Ref LE Q.5]
7. Plasma cell gingivitis.
8. Define gingival abscess. [Ref LE Q.4]
9. Transgingival probing.
10. Pericoronitis.
11. Classify the gingival recession. [Same as SN Q.3]
12. Aetiology of gingival recession. [Same as SN Q.5]
13. Acute pericoronitis. [Same as SN Q.10]
Section | I  Topic-Wise Solved Questions of Previous Years 835

SOLVED ANSWERS
LONG ESSAYS:
Q.1. Define gingival recession. Enumerate its causes. I. Sullivan and Atkins classification
Ans. a. Shallow narrow
b. Shallow wide
[SE Q.1] c. Deep narrow
{The exposure of the root surface by an apical shift in the d. Deep wide}
position of the gingiva is known as gingival recession.} II. Miller’s classification
Class 1: Includes marginal tissue recession that
(SE Q.1 and SN Q.5)
does not extend to the mucogingival junction. There
{(Aetiology of gingival recession is no loss of bone or soft tissue in the interdental
area. This type of recession can be narrow or wide.
l Gingival inflammation: Plaque-induced gingival in-
flammation is the primary aetiological factor re- Class 2: Marginal tissue recession that extends to or
sponsible for gingival recession. beyond the mucogingival junction. There is no loss of
l Next most common cause is faulty brushing technique.
bone or soft tissue in the interdental area. This type
l Surgical treatment of inflammatory periodontal disease.
of recession can be classified into wide and narrow.
l Other secondary/contributing factors of gingival
Class 3: Marginal tissue recession that extends to or
recession are categorized as follows: beyond the mucogingival junction; in addition,
a. Anatomic factors there is bone and/or soft tissue loss interdentally or
b. Habits malpositioning of the tooth.
c. Iatrogenic factors Class 4: Marginal tissue recession that extends to or
d. Physiologic factors beyond the mucogingival junction with severe bone
loss and soft tissue loss interdentally and/or severe
a. Anatomic factors tooth malpositioning.
i. Tooth malposition or abnormal position of
the tooth in the arch, e.g. the periodontium [SE Q.1]
on the labial aspect of the tooth will be in-
variably thin when it is placed labially. {Clinical significance
ii. Presence of dehiscence and fenestrations. Exposed root surfaces are susceptible to caries.
l

iii. Gingival ablation from soft tissues like cheek Wearing away of the cementum exposed by reces-
l

and lips. sion leaves an underlying dentinal surface that is


b. Habits extremely sensitive.
l Hyperaemia of the pulp and associated symptoms
i. Faulty toothbrushing technique or brushing with
hard bristles may lead to gingival recession. may also result from exposure of the root surface.
l Interproximal recession creates spaces in which
ii. Recent concept is that there may be a positive
relationship between smoking and recession. plaque and food debris can accumulate.
l Finally, it is aesthetically unacceptable.
But the exact mechanism is not reported.
c. Iatrogenic factors Treatment
i. Primary trauma from occlusion has been Denuded root surfaces are covered for two purposes:
reported to cause gingival recession. i. To solve aesthetic problem in anterior teeth
ii. Orthodontic movement in a labial direction ii. To widen zone of attached gingiva, thereby
and improper restorations can lead to gingi- solving a possible mucogingival problem
val recession. Several procedures are as follows:
iii. Clasps of ill-fitting denture and mandibular I. Treatment based on width of attached
overdenture bars or aprons in partial dentures. gingiva
d. Physiologic factors
Gingival recession was thought to be a physio- Adequate amount of Inadequate amount of
logic process related to ageing but there was no attached gingival attached gingiva
convincing evidence for a physiologic shift of
i. Pedicle graft i. Free soft tissue autograft
the gingival attachment.)} l Double papillae ii. Subepithelial connective
l Laterally displaced tissue graft
[SE Q.1] ii. Coronally repositioned flap
{Classification of gingival recession with semilunar incision
836 Quick Review Series for BDS 4th Year, Vol 1

II. Treatment based on distribution of Acute forms of gingivitis


recession
Underlying microscopic
Generalized recession Clinical changes features
Localized recession (involving few teeth) i. Diffuse puffiness and i. Diffuse oedema of acute
i. Pedicle graft i. Free soft tissue graft softening inflammatory origin,
l Double papillae ii. Subepithelial connective fatty infiltration in
l Laterally displaced tissue graft xanthomatosis
ii. Coronally repositioned flap iii. Coronally repositioned ii. Sloughing with greyish, ii. Necrosis with formation
with semilunar incision }
flap flake-like particles of debris of pseudomembrane
iii. Guided tissue regeneration adhering to eroded surface composed of bacteria,
polymorphonuclear leu-
Prognosis kocytes and degener-
ated epithelial cells in
l Prognosis of classes 1 and 2 is good to excellent.
fibrinous meshwork
l Class 3 – only partial coverage can be

expected. iii. Formation of vesicle iii. Intercellular and intra-


cellular oedema with
l Class 4 – poor prognosis, and finally it is aes-
degeneration of nucleus
thetically unacceptable. and cytoplasm and rup-
ture of cell wall
Q.2. Describe correlation of clinical and underlying mi-
croscopic features of acute and chronic gingivitis.
Ans.

Correlation between clinical and histopathologic features Q.3. Define gingival bleeding. Write about causes and
in gingival inflammation is as follows: management of gingival bleeding.
Chronic gingivitis Ans.
Underlying microscopic Gingival bleeding on probing
Clinical changes features l Gingival bleeding on probing indicates an inflamma-

Soggy puffiness of gingiva that Infiltration by fluid and cells tory lesion in both epithelium and connective tissue
pits on pressure of inflammatory exudate that exhibits specific histologic differences compared
Gingiva exhibiting marked l Degeneration of connec- with healthy gingiva.
softness and friability, with tive tissue and epithe- l The two earliest signs of gingival inflammation pre-
ready fragmentation on explora- lium associated with in- ceding established gingivitis are:
tion with probe and pinpoint jurious substances that a. increased gingival crevicular fluid production rate
surface areas of redness and provoke the inflamma-
desquamation tion and inflammatory
b. bleeding from the gingival sulcus on gentle
exudate probing
l Change in connective l Gingival bleeding on probing is an important diag-
tissue–epithelium rela- nostic factor for clinicians to use in planning peri-
tionship, with inflamed odontal therapy. It is associated with inflammation
and engorged connective
tissue expanding to
and ulceration of the epithelium lining of gingival
within a few epithelial sulcus.
cells of surface, thinning l The presence of plaque for only 2 days can initiate
of epithelium and degen- gingival bleeding on probing, whereas once estab-
eration associated with lished, it may take 7 days or more after continued
oedema and leukocyte
invasion, separated by
plaque control and treatment to eliminate gingival
areas in which rete pegs bleeding.
are elongated to connec- l The presence of bleeding is an indication of active
tive tissue gingival inflammation, and until it is controlled, the
Firm, leathery consistency of Fibrosis and epithelial patient is at a risk of continuing periodontal disease
gingiva proliferation associated with and tissue destruction.
long-standing chronic l Even though gingival bleeding on probing may not
inflammation
be a good diagnostic indicator for clinical attachment
Section | I  Topic-Wise Solved Questions of Previous Years 837

loss, its absence is an excellent negative predictor of the Acute gingival bleeding
future attachment loss. l Acute episodes of gingival bleeding are caused by

Gingival bleeding injury and can occur spontaneously in a gingival


i. Local factors disease.
Contributing factors to plaque retention that may lead to l Laceration of the gingiva by toothbrush bristles

gingivitis are as follows: during aggressive toothbrushing or by sharp


l Anatomic and developmental tooth variations pieces of hard food can cause gingival bleeding
l Caries even in the absence of a gingival disease.
l Frenum pull l Spontaneous bleeding or bleeding on slight

l Iatrogenic factors provocation can occur in acute necrotizing ul-


l Malpositioned teeth cerative gingivitis. In this condition, engorged
l Mouth breathing blood vessels in the inflamed connective tissue
l Overhanging restorations are exposed by ulceration of the necrotic surface
l Partial dentures epithelium.
l Lack of attached gingiva ii. Gingival bleeding associated with systemic changes
l Recession l Gingival haemorrhage occurs spontaneously or after

Chronic and recurrent bleeding irritation, and is excessive and difficult to control in
l The most common cause of abnormal gingi- patients with systemic changes.
val bleeding on probing is chronic inflam- l All these conditions have a common feature of hae-

mation. mostatic mechanism failure and result in abnormal


l The chronic or recurrent bleeding is provoked bleeding in the skin, internal organs and other tis-
by: sues, including the oral mucosa.
a. Mechanical trauma (e.g. from toothbrush- l Haemorrhagic disorders in which abnormal gingival

ing, toothpicks or food impaction) bleeding is encountered are as follows:


b. Biting into solid foods such as apples i. Vascular abnormalities (vitamin C deficiency or
l Gingival bleeding includes dilation and en- allergy, e.g. Schonlein–Henoch purpura)
gorgement of the capillaries and thinning or ii. Platelet disorders (thrombocytopenic purpura)
ulceration of the sulcular epithelium. iii. Hypoprothrombinemia (vitamin K deficiency)
As the degenerated epithelium is less protec- iv. Other coagulation defects (haemophilia, leukae-
tive, stimuli that are normally innocuous cause mia and Christmas disease), deficient platelet
rupture of the capillaries and gingival bleeding. thromboplastic factor (PF3) resulting from urae-
l Sites that bleed on probing have a greater area mia, multiple myeloma and postrubella purpura
of inflamed connective tissue, e.g. cell-rich, l The effects of hormones:

collagen-poor tissue than sites that do not The hormonal replacement therapy, oral contra-
bleed. ceptives, pregnancy and the menstrual cycle are
l In most cases, the cellular infiltrate of sites that also reported to affect gingival bleeding. Changes
bleed on probing is predominantly lymphocytic, in androgenic hormones are significant, modify-
a characteristic of stage II, or early gingivitis. ing factors in gingivitis, especially among ado-
l The severity of the bleeding and the ease of lescents.
its provocation depend on the intensity of the l Among pathologic endocrine changes, diabetes is an

inflammation. endocrine condition with a well-characterized effect


l After the vessels are damaged and ruptured, on gingivitis.
interrelated mechanisms induce haemostasis. l Several medications have also been found to have

The vessel walls contract, and blood flow is adverse effects on the gingiva. They are as follows:
diminished; blood platelets adhere to the anticonvulsants, antihypertensive calcium channel
edges of the tissue and a fibrous clot is formed, blockers, immunosuppressant drugs.
which contracts and results in approximation They are known to cause gingival enlargement,
of the edges of the injured area. Bleeding re- which secondarily can cause gingival bleeding.
curs when the area is irritated. l The American Heart Association has recommended

l In cases of moderate or advanced periodonti- over the counter aspirin as a therapeutic agent for
tis, the presence of bleeding on probing is cardiovascular disease, and aspirin is often pre-
considered a sign of active tissue destruction. scribed for rheumatoid arthritis, osteoarthritis,
838 Quick Review Series for BDS 4th Year, Vol 1

rheumatic fever and other inflammatory joint If residual lesion is large or poorly accessible, surgi-
l

diseases. cal access may be required.


l Thus, it is important to consider aspirin’s effect on
Q.5. Enumerate the stages of gingivitis and treatment
bleeding during a routine dental examination to
for advanced stage of gingivitis.
avoid false-positive readings.
Ans.
Q.4. Define gingival abscess. Write in detail about aeti-
ology and treatment of the same.
Ans. {SN Q.6}
l Gingivitis or the inflammation of gingiva is the most
{SN Q.8} common form of gingival diseases.
l Page and Schroeder in 1976 classified the progres-
A gingival abscess is a localized, acute inflammatory sion of gingival and periodontal inflammation on the
lesion that may arise from a variety of sources, including basis of clinical and histopathological evidences.
microbial plaque infection, trauma, and foreign body They divided progressing lesions into the following
impaction. stages:
Aetiology i. Stage I gingivitis, or the initial lesion
Common aetiologic factor is the bacteria carried ii. Stage II gingivitis, or the early lesion
deep into the tissues when a foreign substance, e.g. iii. Stage III gingivitis, or the established lesion
toothbrush bristle, piece of apple core, lobster shell iv. Stage IV gingivitis, or the advanced lesion
fragment, is forcefully embedded into the gingiva. The advanced lesion was considered to reflect the pro-
gression of gingivitis to periodontitis or a state of transi-
tion of gingivitis to periodontitis.
Clinical features
l It is generally limited to the marginal gingiva or

interdental papilla. Stage I gingivitis: initial lesion


l It appears as a red swelling with a smooth shiny sur- l The characteristic feature of the initial lesion is due
face in early stages. to the enhanced levels of activity of the host defence
l It is usually a sudden onset with a rapidly expanding mechanisms that are normally operative within the
lesion. gingival tissues.
l Within 24–48 h, the lesion usually becomes fluctuant l This initial response of the gingiva is also called
and pointed with a surface orifice from which puru- subclinical gingivitis. The initial lesion is localized
lent exudate may be expressed. to the region of the gingival sulcus. The tissues af-
l The lesion generally ruptures spontaneously if fected include a portion of the junctional epithelium,
permitted to progress. the oral sulcular epithelium and the most coronal
l The adjacent teeth are often sensitive to percussion. portion of the connective tissue.
Treatment Clinical features
l Topical or local anaesthesia is administered through Initial response of the gingiva to bacterial plaque is
infiltration. not obvious clinically.
l When possible, scaling and root planing are completed Histological features
to establish drainage and removal of microbial deposits. l When a normal, infiltrate-free gingiva is subjected
l In acute situations, the fluctuant area is incised with to the accumulation of microbial plaque, the ini-
a No. 15 scalpel blade, and exudate may be ex- tial lesion emerges within 2–4 days.
pressed by gentle digital pressure. l The initial lesion is a response to the generation of
l Any foreign material (e.g. dental floss, impression chemotactic and antigenic substances in the
material) is removed. region of the gingival sulcus.
l The area is irrigated with water and covered with l There is alteration of the most coronal portion
moist gauge under light pressure. of the junctional epithelium, within 24 h the
l Once bleeding has stopped, the patient is dismissed marked changes are evident in the gingival mi-
with instructions to rinse with warm salt water every crovasculature.
2 h for the remainder of the day. l Classic vasculitis of vessels, subjacent to the junc-
l After 24 h, the area is reassessed and if resolution is tional epithelium.
sufficient, scaling not previously completed is under- The vessels of the gingival plexus become en-
taken. gorged and dilated, and many polymorphonuclear
Section | I  Topic-Wise Solved Questions of Previous Years 839

leukocytes migrate into the junctional epithelium l Collagen fibre content of the affected tissue is re-
and the gingival sulcus. duced by about 70%. This alteration, which oc-
l Exudation of fluid from the gingival sulcus, pres- curs at an early stage of the disease, especially
ence of serum proteins, especially fibrin extravas- affects the dentogingival and circular fibre groups
cularly. that normally support the junctional epithelium.
l Loss of perivascular collagen occurs and the re- The loss of collagen, therefore, may be a major
sultant space is occupied by fluid, serum proteins factor in continuing loss of tissue integrity and
and inflammatory cells. normal gingival function as the disease pro-
l A few macrophages and blast-transforming lym- gresses.
phocytes may appear within the junctional epithe- l The leukocytes infiltrate between the epithelial

lium, and in the connective tissue the lympho- cells and may be present in sufficiently large
cytes are almost exclusively T cells. numbers to disrupt the continuity of epithelial
l The gingival sulcus contains migrating leuko- barrier. The area of affected connective tissue can
cytes, sloughed epithelial cells and bacteria. be distinguished clearly from the surrounding
Stage II gingivitis: early lesion normal tissue by the presence of inflammatory
The early lesion overlaps and evolves from the initial cells and the decreased collagen content.
lesion with no clear-cut dividing line. Stage III gingivitis: established lesion
Clinical features Clinical features
l Owing to the proliferation of capillaries and in- l Features of the early lesion are presented in an

creased formation of capillary loops between rete accentuated form.


pegs and ridges, the signs of erythema and bleed- l This conversion is probably the result of subgin-

ing may also be evident on probing. gival plaque extension, which allows the forma-
Histological features tion of a shallow gingival pocket, where the tooth
l The early lesion appears at the site of initial lesion surface is covered by bacterial plaque and the
within 4–7 days of plaque accumulation. soft-tissue wall is lined by a pocket epithelium.
l The lesion is the result of the formation and main- l In some cases, the pocket epithelium may be thick

tenance of a dense lymphoid-cell infiltrate within and resemble a long oral sulcular epithelium;
the gingival connective tissues. more frequently, however, the pocket epithelium
l Here, the features of the initial lesion are accentu- becomes thin and ulcerated.
ated. As mild gingivitis appears, a two- to three- l The blood vessels become engorged and congested,

fold increase occurs in the number of inflamma- and therefore the venous return is impaired and the
tory cells. The infiltrate continues to be comprised blood flow becomes sluggish. All this results in
predominantly lymphocytes and a few plasma localized gingival anoxaemia, which superimposes
cells. a bluish hue on the reddened gingiva.
l Accumulation of lymphoid cells immediately Established lesions of two types exist. They are as
subjacent to the junctional epithelium at the site follows:
of acute inflammation. a. Some lesions remain stable and do not
l Most of these lymphocytes are T cells; only about progress for months or years.
6% are identified as B cells. They are diffusely ar- b. Other groups of lesions apparently become
ranged immediately under the epithelium at the zone more active and convert to progressive de-
of injury subjacent to the junctional epithelium. structive lesions.
l The lymphocytic infiltration remains localized l The nature of this conversion has been studied but

and does not extend deeply into the tissues. is not understood.
l The flow of gingival fluid and the number of cre- l The proportion of T cells decreases and that of B

vicular leukocytes reach their maximum and level cells and plasma cells increases as gingivitis
off between 6 and 12 days after the onset of clini- appears and becomes more severe.
cal gingivitis. l It has been suggested that a conversion from a

l Although the oral sulcular epithelium and the oral predominantly T cell infiltrate to a B cell infil-
epithelium generally do not become infiltrated, trate is the major event in the conversion of stable
the junctional epithelium contains a variably in- established lesions into aggressive destructive
creased number of transmigrating neutrophilic lesions.
granulocytes and infiltrating mononuclear cells, l The destructive lesions are characterized by acute

including lymphocytes, macrophages, plasma inflammation with a great deal of exudation and
cells and mast cells. by the presence of many neutrophils
840 Quick Review Series for BDS 4th Year, Vol 1

l As destruction slows, the acute inflammation is l The junctional epithelium is no longer closely at-
resolved and the infiltrate becomes predominated tached to the tooth surface. The pocket epithelium
by lymphoid and other mononuclear cells. that has now formed has a heavy leukocyte infil-
l A likely cause of the conversion of a stable estab- trate, predominantly of PMNs, which eventually
lished lesion to an aggressive lesion is a change migrate across the epithelium into the gingival
in the microbial flora or infection of the gingival pocket.
tissues. l The pocket epithelium is more permeable to the

Histological features passage of substances into and out of the underly-


l The distinguishing feature of the established le- ing connective tissues and is temporarily ulcer-
sion is a predominance of plasma cells within the ated in many places.
affected connective tissues at a stage prior to ex- l Alveolar bone loss occurs.

tensive bone loss. l There are widespread manifestations of inflam-

l Plasma cells appear along the blood vessels and matory and immunopathologic tissue reactions.
between collagen fibre bundles and deep within l Plasma cells predominate in the lesion, although

the connective tissue. lymphocytes and macrophages are also present.


l Junctional epithelium shows widening of the in- l The lesion is no longer localized; it may extend

tercellular spaces filled with cell debris and lyso- apically, as well as laterally, to form a variably
somes derived from lymphocytes, monocytes and broad band around the necks and roots of the
neutrophils. teeth.
l The junctional and oral sulcular epithelium may l The highly organized fibre bundles of the mar-

proliferate and migrate into the infiltrated connec- ginal gingiva lose their characteristic orientation
tive tissue and along the root surface, with conver- and architecture.
sion to pocket epithelium.
Q.6. What is gingival recession? How will you treat a
l The basal lamina may be destroyed in some areas.
case of localized gingival recession on mandibular left
l Continuing loss of collagen is apparent in the
central incisor?
zone of infiltration in other more distant regions;
fibrosis and scarring may begin to occur. Ans.
l Enzyme histochemistry.

Chronically inflamed gingiva has elevated levels [Same as LE Q.1]


of acid phosphatase, alkaline phosphatase, gluc- Q.7. Discuss in detail the aetiology, classification and
uronidase, glucosidase, esterases, aminopeptidase management of gingival recession.
and cytochrome oxidase.
l Neutral mucopolysaccharide levels are decreased. Ans.
l Collagenase is present in the gingival tissues,
[Same as LE Q.1]
which can be produced by periopathogens and
neutrophils. Q.8. Discuss the causes of gingival bleeding.
Stage IV gingivitis: advanced lesion
Ans.
l This stage is also known as the phase of periodontal
breakdown. Here, there is persistence of features [Same as LE Q.3]
described for the established lesion but there is ex-
Q.9. Discuss gingival bleeding on probing.
tension of the lesion into the alveolar bone and peri-
odontal ligament. Ans.
Clinical features
l Formation of periodontal pocket, suppuration,
[Same as LE Q.3]
mobility, migration and eventually tooth exfolia- Q.10. Enumerate stages of gingivitis. Discuss estab-
tion. lished lesions.
Histological features
l The advanced lesion represents frank and overt Ans.
periodontitis. [Same as LE Q.5]
l Continued loss of collagen subjacent to the pocket

epithelium with fibrosis at more distant sites can Q.11. Established lesion of chronic gingivitis.
be seen.
Ans.
l The junctional epithelium migrates apically from

the cementoenamel junction. [Same as LE Q.5]


Section | I  Topic-Wise Solved Questions of Previous Years 841

SHORT ESSAYS: The surgical procedure to remove the operculum is


called operculectomy. The operculum can also be
Q.1. Gingival recession. removed by electrosurgery.
Ans. l The removal of the flap should result in a site that can

be easily maintained by the patient. To ensure this,


[Ref LE Q.1]
the flap covering the occlusal surface as well as the
Q.2. Describe the clinical features and management of tissue distal to the tooth has to be removed.
acute pericoronitis.
Q.3. Gingival pigmentations.
Ans.
Ans.
l Pericoronitis refers to the inflammation of the gingiva or
l The colour of the gingiva is determined by several fac-
a pericoronal flap of an incompletely erupted tooth.
tors, in which pigments within the epithelium are one of
l The most commonly involved tooth is the mandibular
the factors.
third molar.
l Types of pigmentation observed in gingiva are catego-
l The major factor responsible for the involvement of the
rized as follows:
mandibular third molar is the space between the erupt-
i. Endogenous pigmentation
ing tooth and the overlying flap.
ii. Exogenous pigmentation
Clinical features
l The pericoronal flap will be red and swollen with the
Endogenous pigmentation Exogenous pigmentation
presence of exudate.
l Melanin pigmentation is l From atmospheric irritants,
l Patient complains of the pain radiating to the ear,
physiologic, seen in e.g. coal and metal dust, co-
throat and floor of the mouth. certain ethnic groups like louring agents added in
l Patient complains of foul taste and inability to open Africans foods, tobacco and amalgam
the jaws. l Haemochromatosis: iron tattoo
l Lymphadenitis may be present.
deposition – blue grey
pigmentation
l Systemic symptoms such as fever and malaise are
l Jaundice: bile pigments –
also present. yellowish colour
Complications
l Pathological: Addison l Heavy metals like bismuth,
l Pericoronal abscess
disease, Peutz–Jeghers mercury, lead and silver pro-
l Peritonsillar abscess syndrome, Albright syn- duce a line in the marginal
l Cellulitis drome (polyostotic fibrous gingiva or are seen as patches
l Ludwig angina dysplasia), von Reckling- in the attached gingiva
hausen disease (neurofi- l Metallic pigmentation: only
Treatment of acute pericoronitis
bromatosis) in areas of gingival inflam-
l Treatment depends to a large extent on the following
mation due to increased ves-
factors: sel permeability, systemically
i. Severity of inflammation absorbed metal sulphides get
ii. Systemic involvement precipitated in the perivascu-
iii. Possibility of retaining the tooth lar area in the subepithelial
connective tissue
l The acute pericoronitis is treated to relieve the acute

symptoms that consist of gentle irrigation of the area


with warm saline to remove debris and exudates. Systemic causes for colour changes
l Occlusion is evaluated to determine if an opposing
The causes can be endogenous or exogenous. They are
tooth is occluding with the pericoronal flap. Occlusal
either
adjustment is necessary in case of opposing tooth
A. Localized
traumatizing the flap.
B. Generalized
l Administration of antibiotics in severe cases with

involvement of lymph nodes. A. Localized pigmentation


l Drainage can be established if the swelling is fluctu- Examples are amalgam tattoo, graphite or other tattoos,
ant by using a No. 15 blade. naevus, melanotic macules, malignant melanoma and
l After resolution of acute symptoms, the tooth should Kaposi sarcoma.
be evaluated for possibility of erupting to a func- B. Multiple or generalized pigmentation
tional position. i. Genetics: Idiopathic melanin pigmentation (racial
l If the tooth is to be retained, the pericoronal flap should or physiological pigmentation), Peutz–Jeghers
be surgically excised using the periodontal knives. syndrome
842 Quick Review Series for BDS 4th Year, Vol 1

ii. Habits: Smoking, betel chewing


l Self-inflicted oral trauma, such as toothbrush
iii. Drugs: Antimalarials, antimicrobials, minocycline,
abrasion, improper flossing
chlorpromazine, ACTH, zidovudine, ketoconazole,
l Infection, which can be either tooth- or gum-
methyldopa, busulphan, menthol, contraceptive
related
pills and heavy metals exposure (gold, bismuth,
l Vitamin C or K deficiency
mercury, silver, lead and copper)
l Hormonal changes during pregnancy
iv. Endocrine: Addison disease, Albright syndrome,
l Chemical irritants such as aspirin
pregnancy and hyperthyroidism
l Leukaemia
v. Postinflammatory: Periodontal disease, postsurgi-
l Placement of new dentures can lead to denture
cal gingival repigmentation
sores/irritations
vi. Others: Haemochromatosis, generalized neurofi-
l Idiopathic thrombocytopenic purpura
bromatosis, incontinentia pigmenti, Whipple dis-
l Coagulation disorders – haemophilia, Christmas
ease, Wilson disease, Gaucher disease, HIV dis-
disease
ease, thalassaemia, pigmented gingival cyst and
nutritional deficiencies
Clinical significance
Q.4. Management of localized gingival bleeding. l Bleeding on probing is an objective method for ex-

amining gingival connective tissue inflammation.


Ans.
l Ulceration of the periodontal pocket wall epithelium
Localized gingival bleeding is mainly due to the accumula- causes bleeding but bleeding in and of itself is not
tion of plaque and calculus as well as poor oral hygiene. diagnostic for a specific form of disease.
Management
Q.6. Treatment for localized gingival recession.
i. First visit
l Supragingival scaling after taking a thorough case Ans.
history to rule out systemic conditions.
[Same as SE Q.1]
l It will reduce the inflammation and hence size of

the gingiva.
l The patient is given oral hygiene instructions. SHORT NOTES:
ii. Second visit
Q.1. Stillman cleft.
l All the sources of plaque accumulation are re-

moved, e.g. overhang restorations are corrected. Ans.


l Carious lesions are restored.
Stillman cleft is a narrow, triangular-shaped gingival reces-
l Partial dentures are evaluated and corrected.
sion, which exposes cementum as the recession progresses
Q.5. Gingival bleeding on probing. apically.
Ans. Q.2. McCall festoons.
l The first clinical sign of gingivitis is bleeding subse- Ans.
quent to gentle probing.
McCall festoons are rolled, thickened margins of gingiva
l The site is considered inflamed if bleeding is provoked
seen near canine when recession progresses till the muco-
by a blunt periodontal probe inserted to the bottom of
gingival junction.
the gingival pocket and moved gently along the lateral
wall of gingiva. Q.3. Gingival recession.
Ans.
l Gingival recession is the apical shift in the position of
gingiva exposing the root surface.
{SN Q.4}
l Actual position of gingiva is more important than the

Causes of bleeding on probing are as follows: apparent position as it shows the loss of attachment that
l Gingivitis: Poor oral hygiene, inadequate plaque has taken place.
removal, calculus accumulation l During certain times the denuded root surface is hidden

l Periodontitis by the inflamed pocket wall. This part of root surface is


l Anticoagulants, e.g. coumarin and heparin known as the hidden recession and the part that is visi-
ble clinically is called visible recession.
Section | I  Topic-Wise Solved Questions of Previous Years 843

Q.4. Causes of gingival bleeding on probing. l It is an accurate method of evaluation of bone architec-
ture and provides additional information of the same. It
Ans.
is performed under local anaesthesia.
[Ref SE Q.5] l The height and contour of the facial and lingual bones

obscure on the radiograph by the dense roots is ob-


Q.5. Causes of gingival recession.
tained.
Ans. l The architecture of the interdental bone, the number of

bony walls and the presence of furcation defects are


[Ref LE Q.1]
obtained.
Q.6. Enumerate the stages of gingivitis.
Q.10. Pericoronitis.
Ans.
Ans.
[Ref LE Q.5]
l Pericoronitis refers to the inflammation of the gingiva or
Q.7. Plasma cell gingivitis. a pericoronal flap of an incompletely erupted tooth.
l The most commonly involved tooth is the mandibular
Ans.
third molar.
l Plasma cell gingivitis is also referred to as atypical and l The major factor responsible for the involvement of the

plasma cell gingivostomatitis. mandibular third molar is the space between the erupt-
l Frequently it consists of a mild marginal gingival en- ing tooth and the overlying flap.
largement that extends to attached gingiva.
Q.11. Classify the gingival recession.
l Clinically, gingiva appears red, friable and bleeds easily.

l It is thought to be allergic in origin, possibly related to Ans.


the components of chewing gum or dentifrices.
[Same as SN Q.3]
l Microscopically, the connective tissue contains a dense in-

filtrate of plasma cells that also extends to oral epithelium. Q.12. Aetiology of gingival recession.
Q.8. Define gingival abscess. Ans.
Ans. [Same as SN Q.5]
[Ref LE Q.4] Q.13. Acute pericoronitis.
Q.9. Transgingival probing. Ans.
Ans. [Same as SN Q.10]
l Transgingival probing is also known as sounding.

Topic 16
Gingival Enlargements
COMMONLY ASKED QUESTIONS
LONG ESSAYS:
1 . Classify gingival enlargements. Write in detail the drug-induced gingival enlargement.
2. Classify gingival enlargements. Describe clinical features and histopathology of leukaemic enlargement.
3. Classify gingival enlargement. What is conditioned enlargement in pregnancy?
4. Define gingival enlargement. Write briefly the differences between chronic and acute inflammatory gingival
enlargements.
844 Quick Review Series for BDS 4th Year, Vol 1

5. Classify gingival enlargements. Give indications, contraindications and method of gingivectomy.


6. How will you differentiate between scorbutic gingival enlargement and leukaemic gingival enlargement?
7. Discuss briefly about chronic and acute inflammatory gingival enlargements.
8. Classify gingival enlargement. Discuss in detail signs, symptoms and treatment of Dilantin sodium gingival
enlargement. [Same as LE Q.1]
9. Classify gingival enlargement. Discuss the histopathology and clinical features of drug-induced gingival
enlargement. [Same as LE Q.1]
1 0. Define and classify gingival enlargements. Describe aetiology, clinical features and management of any one type
of gingival enlargement. [Same as LE Q.1]
11. Classify gingival enlargement. Discuss phenytoin-induced gingival enlargement in detail. [Same as LE Q.1]

SHORT ESSAYS:
1 . Classify gingival enlargements. Add a note on idiopathic enlargement. [Ref LE Q.1]
2. Compare drug-induced gingival enlargement and leukaemic gingival enlargement.
3. Discuss noninflammatory gingival enlargement.
4. Benign tumours of gingiva.
5. Pyogenic granuloma (granuloma pyogenicum).
6. Peripheral giant cell granuloma.
7. Compare drug-induced and idiopathic gingival enlargements. [Same as SE Q.3]

SHORT NOTES:
1. Conditioned gingival enlargement.
2. Angiogranuloma.
3. Classify gingival enlargements.
4. Drug-induced gingival enlargement.
5. What is the difference between gingival abscess and periodontal abscess?
6. Pregnancy tumour.
7. Wegener granulomatosis.
8. Developmental gingival enlargements.
9. Give differential diagnosis of ‘epulis’.
10. Leukaemic gingival enlargement.
11. What is drug-induced gingival hyperplasia? [Same as SN Q.4]
12. Pathogenesis of phenytoin sodium gingival enlargement. [Same as SN Q.4]

SOLVED ANSWERS
LONG ESSAYS:
Q.1. Classify gingival enlargements. Write in detail the
drug-induced gingival enlargement. B. Fibrotic enlargement
a. Drug-induced gingival enlargement (DIGE)
Ans.
b. Hereditary or idiopathic gingival enlargement
C. Combined enlargement (inflammatory 1 fibrosis)
[SE Q.1]
D. Enlargements associated with systemic diseases/
{Gingival enlargement is increase in the size of the gingiva. conditions
It is a result of interaction between the host and a variety of a. Conditioned enlargement
other factors. i. Pregnancy
On the basis of aetiology and pathogenesis, gingival en- ii. Puberty
largements are classified as follows: iii. Vitamin C deficiency
A. Inflammatory enlargement iv. Diabetes-induced gingival enlargement
a. Acute v. Nonspecific conditioned enlargement (granu-
b. Chronic loma pyogenicum)
Section | I  Topic-Wise Solved Questions of Previous Years 845

. Systemic diseases causing gingival enlargement


b Drugs associated with gingival enlargement.
i. Leukaemia Group of drug Commonly used agents
ii. Granulomatous diseases (Wegener granulo-
Anticonvulsants
matosis, sarcoidosis, etc.)
E. Neoplastic enlargement (gingival tumours) Hydantoin derivatives Mephenytoin, phenytoin,
ethotoin
a. Benign tumours
b. Malignant tumours Succinimides Ethosuximide
F. False enlargements} Methsuximide
On the basis of location and distribution, gingival en-
Phensuximide
largement is classified as follows:
A. Localized Valproic acid derivatives Valproic acid
Limited to the gingiva adjacent to a single tooth or Immunosuppressants Cyclosporine, tacrolimus
group of teeth. Calcium channel blockers
B. Generalized
Dihydropyridine derivatives Amlodipine
Involving the gingiva throughout the mouth.
C. Marginal Felodipine
Confined to the marginal gingiva. Nicardipine
D. Papillary
Nifedipine
Confined to the interdental papilla.
E. Diffused Nimodipine
Involving the marginal and attached gingiva as well Nisoldipine
as papillae.
Nitrendipine
F. Discrete
An isolated sessile or pedunculated tumour-like Benzothiazine derivatives Diltiazem
enlargement. Phenyl alkylamine derivatives Verapamil
The degree of gingival enlargement can be scored
as follows:
l Grade 0: No signs of gingival enlargement. Anticonvulsants
l Grade I: Enlargement confined to interdental l In patients with epilepsy and in the management of

papilla. neuralgia, anticonvulsants like phenytoin have been


l Grade II: Enlargement involves papilla and used to control seizure disorders. Within a year of its
marginal gingiva. clinical use, gingival overgrowth can be seen in these
l Grade III: Enlargement covers three quarters patients.
or more of the crown. l Valproic acid, phenobarbitone, primidone, mephe-

Drug-induced gingival enlargement (DIGE) nytoin and ethosuximide are anticonvulsant drugs,
l The drugs that are reported to be associated other than phenytoin that have been associated with
with gingival overgrowth are anticonvulsants, gingival overgrowth.
immunosuppressants and calcium channel l Valproic acid derivatives like sodium valproate car-

blockers. ries a relatively low risk for developing gingival


l Despite their pharmacological diversity, all overgrowth and may be a reasonable treatment alter-
these drugs have a similar mechanism of action native to phenytoin.
at cellular level. Clinical features
l They are known to inhibit intracellular cal- l Phenytoin-induced gingival overgrowth is character-

cium ion influx; hence the action of these ized by initial enlargement of the interdental papillae
drugs on calcium and sodium influx may be and is less frequently accompanied by increased thick-
the reason for these dissimilar drugs to have ening of the marginal tissue.
a common side effect upon a secondary l Affected tissues typically present a granular or

target tissue like gingiva in susceptible pebbly surface, with the enlarged papilla extend-
individuals. ing facially and/or lingually covering the tooth
l The clinical features and histological appear- surfaces.
ance of gingival overgrowth induced by these l Affected papillae may become enlarged to the

drugs are reported to coincide. point that they contact resulting in the clinical
846 Quick Review Series for BDS 4th Year, Vol 1

presence of pseudo-clefts. The facial gingiva of l Lamina propria is characterized by proliferation


the anterior sextants is more commonly affected. of fibroblasts and an increase in collagen forma-
Immunosuppressants tion accompanied by an increase in noncollage-
l Cyclosporine is a widely used immunosuppressant nous matrix proteins.
drug for treating a variety of autoimmune diseases. l Focal accumulation of infiltrative inflammatory

l Its exact mechanism of action is not clearly un- cells within connective tissue has been demon-
derstood but it appears to selectively and revers- strated.
ibly inhibit helper T cells and NK cells, which Pathogenesis
play a role in cellular and humoral immune l Pathogenesis has not been clearly understood.

responses. l Available evidence suggests the direct effect of

l Inhibition of these cells is believed to be based on the the involved drug on specific subpopulation of fi-
blocking of calcium channels of the cells. broblasts in gingival connective tissue, genetic
l Overgrowth is greater in patients who are medicated predisposition, intercellular calcium metabolism
with cyclosporine along with calcium channel and exchange, molecular mechanisms (various
blockers. cytokines such as EGF, PDGF), inactivation of
l Cyclosporine-associated gingival overgrowth resem- collagenases and inflammation induced by plaque.
bles phenytoin-induced gingival overgrowth, and it is l These dynamic variables may act on gingival mi-

reported to be more vascularized than phenytoin-in- lieu individually or collectively to alter the hae-
duced gingival overgrowth. mostatic steady state.
l Tacrolimusis is another immunosuppressant drug Prevention and treatment
used because of its less severe side effects and is re- l There is a significant correlation between the oc-

ported to cause lesser gingival overgrowth compared currence and/or severity of drug-induced gingival
with cyclosporine. overgrowth and the presence of plaque and
Calcium channel blockers plaque-retentive factors.
l Calcium channel blockers associated with gingival l Patients should be encouraged to perform meticu-

overgrowth are amlodipine, felodipine, nicardip- lous oral hygiene and frequent recall visits for
ine, nifedipine, nimodipine, nisoldipine and ni- removal of local deposits and professional assis-
trendipine. tance.
l Nifedipine is the most commonly used and widely l Physician should be advised to refer the patient

studied drug. Nifedipine-induced gingival over- scheduled to receive a drug associated with gingi-
growth is reported to be dose-dependent. val overgrowth to a dentist for baseline evaluation
l These are the drugs used in the management of a and control of local aetiological factors.
variety of cardiovascular diseases like hypertension, l If small areas without attachment loss and ade-

angina pectoris and coronary artery spasm. quate keratinized gingiva are involved, gingivec-
l They act by inhibiting calcium ion influx across the tomy is indicated.
cell membrane of heart and smooth muscle cells, l If larger areas are involved with attachment and
blocking its cellular mobilization of calcium. bone loss, surgical intervention may be necessary,
l Clinical and histological features of nifedipine- like flap surgery.
induced gingival overgrowth are comparable with l By consultation with patient’s physician, if pos-

that of phenytoin-induced gingival overgrowth. sible, the causative drug may be substituted with
Other Drugs a suitable noninductive drug. Placement of pres-
l The ability of male and female sex hormones to in- sure appliances after surgical correction of over-
duce hyperplastic and oedematous gingivitis has growth to prevent or reduce tendency for recur-
been established. rence has been reported.
l Association of gingival overgrowth with the use l Management of drug-induced gingival over-

of contraceptives (oestrogen/progesterone) has been growth should be a team approach involving


reported. physician, dentist, and health care professionals,
l The antibiotic erythromycin is also reported to cause for prevention, early diagnosis and effective
gingival overgrowth. treatment.
Histological features
Q.2. Classify gingival enlargements. Describe clinical
l It consists of connective tissue with an overlying
features and histopathology of leukaemic enlargement.
multilayered parakeratinized epithelium with
varying thickness and long rete pegs. Ans.
Section | I  Topic-Wise Solved Questions of Previous Years 847

Gingival enlargement is increase in the size of the gingiva. occurring either spontaneously or on slight
It is a result of interaction between the host and a variety of irritation.
other factors. l Patients with leukaemia may also have a sim-

On the basis of aetiology and pathogenesis, gingival ple chronic inflammation without the involve-
enlargements are classified as follows: ment of leukaemic cells and may present with
A. Inflammatory enlargement the same clinical and microscopic features
a. Acute seen in patients without the disease.
b. Chronic l Most cases reveal features of both simple

B. Fibrotic enlargement chronic inflammation and leukaemic infiltrate.


a. Drug-induced enlargement l True leukaemic enlargement often occurs in

b. Hereditary or idiopathic gingival enlargement acute leukaemia but also may be seen in sub-
C. Combined enlargement (inflammatory 1 fibrosis) acute leukaemia. It seldom occurs in chronic
D. Enlargements associated with systemic diseases/conditions leukaemia.
a. Conditioned enlargement Histological features
i. Pregnancy l In leukaemic patients, gingival enlargements ex-

ii. Puberty hibit various degrees of chronic inflammation.


iii. Vitamin C deficiency l Biopsy of enlarged gingivae will disclose the

iv. Diabetes-induced gingival enlargement presence of leukaemic infiltrate.


v. Nonspecific conditioned enlargement (granu- l It is differentiated from inflammatory gingival

loma pyogenicum) hyperplasia by the presence of mononuclear


b. Systemic diseases causing gingival enlargement blast cells that show cytologic atypia.
i. Leukaemia l Mature leukocytes and areas of connective tissue

ii. Granulomatous diseases (Wegener granulomato- are infiltrated with a dense mass of immature
sis, sarcoidosis, etc.) and proliferating leukocytes, the specific nature
E. Neoplastic enlargement (gingival tumours) of which varies with the type of leukaemia.
a. Benign tumours l Engorged capillaries, oedematous and degen-

b. Malignant tumours erated connective tissue and epithelium with


F. False enlargements various degrees of leukocytic infiltration and
Leukaemic gingival enlargement oedema are found.
Leukaemia is a disorder of the leukocytes which l Isolated surface areas of acute necrotizing in-

demonstrate an abnormality in the formation and flammation with a pseudo-membranous mesh-


functioning of the leukocytes. It manifests itself in work of fibrin, necrotic epithelial cells, poly-
the gingiva as an associated gingival enlargement. morphonuclear leukocytes (PMNs) and bacteria
Clinical features are often seen.
l Oral lesions occur in about 10% of the patients

with both acute and chronic forms of all types Q.3. Classify gingival enlargement. What is conditioned
of leukaemia, i.e. myeloid, lymphoid and enlargement in pregnancy?
monocytic type.
Ans.
l In both acute and chronic leukaemias, the gin-

gival overgrowth occurs and the gingiva is On the basis of aetiology and pathogenesis, gingival en-
boggy, oedematous and deep red. largements are classified as follows:
l The gingival enlargement is due to the leukae- I. Inflammatory enlargement
mic cell infiltration in the areas of mild chronic a. Acute
irritation. b. Chronic
l Leukaemic enlargement may be diffused or II. Fibrotic enlargement
marginal and localized or generalized. In severe a. Drug-induced enlargement
cases, the teeth may be completely hidden. b. Hereditary or idiopathic gingival enlargement
l In leukaemic enlargement, the gingiva is III. Combined enlargement (inflammatory 1 fibrosis)
boggy, oedematous and appears generally blu- IV. Enlargements associated with systemic diseases
ish-red and has a shiny surface. a. Conditioned enlargement
l The consistency is moderately firm but there is i. Pregnancy
a tendency towards friability and haemorrhage, ii. Vitamin C deficiency
848 Quick Review Series for BDS 4th Year, Vol 1

iii. Diabetes-induced gingival enlargement The gingival enlargement does not occur without
l

iv. Nonspecific conditioned enlargement (granu- the presence of bacterial plaque.


loma pyogenicum) l The enlargement is usually generalized and tends

b. Systemic diseases causing gingival enlargement to be more prominent interproximally than on the
i. Leukaemia facial and lingual surfaces.
ii. Granulomatous diseases (Wegener granuloma- l The enlarged gingiva is bright red or magenta, soft

tosis, sarcoidosis, etc.) and friable and has a smooth and shiny surface.
V. Neoplastic enlargement (gingival tumours) l Bleeding occurs spontaneously or on slight prov-

a. Benign tumours ocation.


b. Malignant tumours Tumour-like gingival enlargement
VI. False enlargements l It is an inflammatory response to bacterial plaque

Conditioned enlargement and is modified by the patient’s condition.


l Conditioned enlargement occurs when the sys- l It usually appears after the first trimester of preg-

temic condition of the patient exaggerates or nancy but may occur earlier.
distorts the usual gingival response to dental l The lesion appears as a discrete, mushroom-like,

plaque. flattened spherical mass that protrudes from the


l The nature of the modifying systemic influence gingival margin or more often from the inter-
makes it different from chronic gingivitis. proximal space and is attached by a sessile or
l Bacterial plaque is necessary for the initiation of pedunculated base and tends to expand laterally.
this type of enlargement, but plaque is not the sole l Generally dusky red or magenta in colour; it has a

determinant of the nature of the clinical features. smooth and glistening surface that often exhibits
l The three types of conditioned gingival enlarge- numerous deep red, pinpoint markings.
ment are as follows: l It is a superficial lesion and usually does not in-

a. Hormonal (pregnancy, puberty) vade the underlying bone.


b. Nutritional (associated with vitamin C defi- l The consistency varies; the mass is usually semi-

ciency) firm but it may have various degrees of softness


c. Allergic and friability.
l Nonspecific conditioned enlargement is also seen. l It is usually painless unless its size and shape

Enlargement in pregnancy foster accumulation of debris under its margin or


l Pregnancy gingival enlargement may be marginal interfere with occlusion, in which case painful
and generalized or may occur as single or multi- ulceration may occur.
ple tumour-like masses. Histopathology
l During pregnancy there is an increase in the levels l Gingival enlargement in pregnancy is called

of both progesterone and oestrogen, which by the angiogranuloma.


end of the third trimester reach 10 and 30 times, l Both marginal and tumour-like enlargements

respectively, the levels during the menstrual cycle. consist of a central mass of connective tissue,
Hormonal changes with numerous diffusely arranged, newly
formed and engorged capillaries lined by
cuboid endothelial cells as well as a moder-
Changes in vascular permeability ately fibrous stroma with varying degrees of
oedema and chronic inflammatory infiltrate.
l The stratified squamous epithelium is thickened,

Gingival oedema with prominent rete pegs and some degree of in-
tracellular and extracellular oedema, prominent
intercellular bridges and leukocytic infiltration.
Increased inflammatory response to dental plaque l Although the microscopic findings are characteris-

tic of gingival enlargement in pregnancy, they are


The subgingival microbiota may also undergo
l not pathognomonic because they cannot be used to
changes, including an increase in Prevotella inter- differentiate pregnant and nonpregnant patients.
media. Treatment
Marginal enlargement l Most gingival diseases during pregnancy can
l Marginal gingival enlargement during pregnancy be prevented by the removal of plaque and
results from the aggravation of previous inflam- calculus as well as the institution of fastidious
mation. oral hygiene at the outset.
Section | I  Topic-Wise Solved Questions of Previous Years 849

l In pregnancy, treatment of the gingiva, which is lim- Q.5. Classify gingival enlargements. Give indications,
ited to the removal of tissue without complete elimi- contraindications and method of gingivectomy.
nation of local irritants, is followed by recurrence of
Ans.
gingival enlargement.
l Although spontaneous reduction in the size of gin- On the basis of aetiology and pathogenesis, gingival en-
gival enlargement typically follows the termina- largements are classified as follows:
tion of pregnancy, complete elimination of the re- I. Inflammatory enlargement
sidual inflammatory lesion requires the removal of a. Acute
all plaque deposits and factors that favour its ac- b. Chronic
cumulation. II. Fibrotic enlargement
a. Drug-induced enlargement
Q.4. Define gingival enlargement. Write briefly the dif-
b. Hereditary or idiopathic gingival enlargement
ferences between chronic and acute inflammatory gingi-
III. Combined enlargement (inflammatory 1 fibrosis)
val enlargements.
IV. Enlargements associated with systemic diseases
Ans. a. Conditioned enlargement
i. Pregnancy
ii. Vitamin C deficiency
Gingival enlargement is increase in the size of the gingiva. iii. Diabetes-induced gingival enlargement
It is a result of interaction between the host and a variety iv. Nonspecific conditioned enlargement (granu-
of other factors. loma pyogenicum)
Chronic b. Systemic diseases causing gingival enlargement
inflammatory Acute inflammatory i. Leukaemia
enlargement enlargement ii. Granulomatous diseases (Wegener granuloma-
Aetiology i. Factors favouring i. Bacteria carried
tosis, sarcoidosis, etc.)
plaque retention. deep into the V. Neoplastic enlargement (gingival tumours)
ii. Prolonged expo- tissues. a. Benign tumours
sure to plaque. ii. When a foreign b. Malignant tumours
body like a tooth- VI. False enlargements
brush bristle or a
lobster shell frag-
Gingivectomy means excision of the gingiva or surgi-
ment is forcefully cal removal of gingival tissue.
embedded into the Rationale
gingiva. l To improve visibility and accessibility for

Location and i. Generally located i. Localized to mar- complete calculus removal and thorough
distribution on papillary or ginal or papillary smoothening of roots.
marginal gingiva. gingiva, e.g. gingi- l To create a favourable environment for gingi-
ii. May be localized val periodontal
val healing.
or generalized. abscess.
l To restore a physiological gingival contour.
Clinical i. Life preserver- Painful, rapidly Types of gingivectomy
features shaped enlarge- expanding lesion of
i. Surgical gingivectomy
ment or as a dis- sudden onset.
crete mass which Within 24–48 h, it be- ii. Gingivectomy by electrosurgery
is sessile or pe- comes fluctuant and iii. Laser gingivectomy
dunculated or as a pointed with a surface iv. Gingivectomy by chemosurgery
painful ulceration orifice through which Indications
between marginal purulent exudate
Gingivectomy is indicated for elimination of:
and adjacent comes out.
l Supragingival fibrotic pockets irrespec-
gingiva.
tive of the pocket depth
Histopathol- Shows preponder- Gingival abscess con-
l Gingival enlargements
ogy ance of inflammatory sists of a purulent focus
l Suprabony periodontal abscesses
cells and fluid with in the connective tissue
vascular engorge- surrounded by a dif- Contraindications
ment, capillary for- fused infiltration of l When bone surgery is indicated
mation and degener- polymorphonuclear l Where the bottom of the pocket is apical to
ative changes. neutrophils, oedema-
mucogingival junction
tous tissue and vascular
l When aesthetics is the primary concern,
engorgement.
especially in the anterior maxilla
850 Quick Review Series for BDS 4th Year, Vol 1

Techniques for surgical gingivectomy Scorbutic gingival Leukaemic gingival


i. External bevel gingivectomy enlargement enlargement
When the face of the blade is directed coronally.
Clinical l Gingiva is bluish- l Bluish-red, sponge
ii. Internal bevel gingivectomy features red, soft, friable like and friable,
When the face of the blade is directed apically. and has smooth bleeds persistently
Procedure and shiny surface. on slight provoca-
i. The pockets are recorded and marked with l Tissue is spongy, tion or spontane-
a Crane–Kaplan pocket marker to create hyperaemic and ously.
bleeds spontane- l Gingival necrosis
bleeding points. ously. and pseudo-mem-
ii. With a Kirkland periodontal knife BP l Surface necrosis brane formation is
blade (No. 15), start the incision such that with pseudo- seen.
it is bevelled at approximately 45° to the membrane
tooth surface about 1 mm apical to the formation.
bleeding points on both facial and lingual Histopathology l Epithelium be- l Chronic inflamma-
surfaces. comes thin and tion with mature
iii. In case of external bevel gingivectomy, shows spongiosis leukocytes, and
and may show connective tissue
the incision is directed coronally to a severe atrophy. infiltrated with im-
point between the base of the pocket and l Connective tissue mature and prolif-
the crest of the bone, as close as possi- shows poorly erating leukocytes.
ble to the bone without exposing it, to formed collagen l Isolated areas of

remove the soft tissue coronal to the fibres and many acute necrotizing
thin-walled and inflammation
bone. leaking blood shows covering of
iv. In case of internal bevel gingivectomy, the vessels. pseudo-membrane,
incision is directed apically to the level meshwork of
of alveolar crest to incorporate the pocket fibrin, necrotic
lining. epithelial cells,
polymorphonu-
v. Surgical excision of pocket wall is done, clear leukocytes
and the area is cleaned. and bacteria.
vi. The granulation tissue is curetted, and any
remaining calculus and necrotic cementum
are removed and the area is covered with a Q.7. Discuss briefly about chronic and acute inflamma-
periodontal pack. tory gingival enlargements.
Ans.
Q.6. How will you differentiate between scorbutic
gingival enlargement and leukaemic gingival en- Inflammatory enlargements
largement? l Gingival enlargements may result from chronic or
acute inflammatory changes but chronic changes are
Ans.
much more common.
The differences between scorbutic gingival enlargement l Inflammatory enlargements usually are a secondary
and leukaemic gingival enlargement are as follows: complication to any of the other types of enlarge-
ments, creating combined gingival enlargements. It
is important to understand the double aetiology and
treat them adequately.
Scorbutic gingival Leukaemic gingival Chronic inflammatory enlargement
enlargement enlargement Aetiology
l Prolonged exposure to dental plaque.
Aetiology l Deficiency l Malignant
of vitamin C neoplasms of leu- l Factors that favour plaque accumulation and re-

kocyte precursors tention like poor oral hygiene.


l Irritants l Irritation by anatomic abnormalities and improper

Distribution l Marginal gingiva l Diffuse or restorative and orthodontic appliances.


and location marginal l Gingival changes associated with mouth breathing
l Localized or like gingivitis and gingival enlargement. The gin-
generalized giva appears red and oedematous, with a diffused
Section | I  Topic-Wise Solved Questions of Previous Years 851

surface shininess of the exposed area. The maxil- foreign substance, e.g. toothbrush bristle, piece of
lary anterior region is the common site of such in- apple core, lobster shell fragment, is forcefully
volvement. Its harmful effect is generally attributed embedded into the gingiva.
to irritation from surface dehydration. l The lesion is confined to the gingiva and should

Clinical features not be confused with periodontal or lateral


l Chronic inflammatory gingival enlargement origi- abscesses.
nates as a slight ballooning of the interdental pa- Histopathology
pilla and marginal gingiva. In the early stages, it l The gingival abscess consists of a purulent focus

produces a life preserver-shaped bulge around the in the connective tissue, surrounded by a diffused
involved teeth. infiltration of PMNs, oedematous tissue and vas-
l This bulge can increase in size until it covers part cular engorgement.
of the crowns. The enlargement may be localized l The surface epithelium has varying degrees of

or generalized and progresses slowly and pain- intracellular and extracellular oedema, invasion
lessly unless it is complicated by acute infection by leukocytes and sometimes ulceration.
or trauma.
l Occasionally, chronic inflammatory gingival en- Q.8. Classify gingival enlargement. Discuss in detail
largement occurs as a discrete sessile or peduncu- signs, symptoms and treatment of Dilantin sodium gin-
lated mass resembling a tumour. It may be inter- gival enlargement.
proximal or on the marginal or attached gingiva.
l The lesions are slow-growing masses and usually Ans.
painless. They may undergo spontaneous reduc- [Same as LE Q.1]
tion in size, followed by exacerbation and contin-
ued enlargement. Q.9. Classify gingival enlargement. Discuss the histopa-
l Painful ulceration sometimes occurs in the fold thology and clinical features of drug-induced gingival
between the mass and the adjacent gingiva. enlargement.
Histopathology
l Chronic inflammatory gingival enlargements Ans.
show the exudative and proliferative features of [Same as LE Q.1]
chronic inflammation.
l Lesions that are clinically deep red or bluish-red Q.10. Define and classify gingival enlargements. De-
are soft and friable with a smooth and shiny sur- scribe aetiology, clinical features and management of
face, and they bleed easily. any one type of gingival enlargement.
l They also have a preponderance of inflammatory
Ans.
cells and fluid, with vascular engorgement, new
capillary formation and associated degenerative [Same as LE Q.1]
changes.
l Lesions that are relatively firm, resilient and pink
Q.11. Classify gingival enlargement. Discuss phenytoin-
have a greater fibrotic component with an abun- induced gingival enlargement in detail.
dance of fibroblasts and collagen fibres. Ans.
Acute inflammatory enlargement
l A gingival abscess is a localized, painful and rapidly [Same as LE Q.1]
expanding lesion that is usually of sudden onset.
l It is generally limited to the marginal gingiva or in-

terdental papilla. In its early stages it appears as a red SHORT ESSAYS:


swelling with a smooth and shiny surface. Q.1. Classify gingival enlargements. Add a note on idio-
l Within 24–48 h, the lesion usually becomes fluctuant
pathic enlargement.
and pointed with a surface orifice, from which a pu-
rulent exudate may be expressed. Ans.
l The adjacent teeth are often sensitive to percussion.
[Ref LE Q.1]
If permitted to progress, the lesion generally ruptures
spontaneously. Q.2. Compare drug-induced gingival enlargement and
Aetiology leukaemic gingival enlargement.
l Acute inflammatory gingival enlargement results

from bacteria carried deep into the tissues when a Ans.


852 Quick Review Series for BDS 4th Year, Vol 1

Drug-induced fibrotic enlargement (phenytoin,


cyclosporine, nifedipine) Leukaemia
Aetiology l Long-term usage of the respective drug l Malignant neoplasms of leukocyte precursors.
Location l Both generalized marginal and papillary gingiva are l Diffused or marginal.
involved l Localized or generalized.
Clinical features l Bead-like enlargement of facial and lingual gingival margins. l Bluish-red, sponge-like and friable, bleeds sponta-
l Massive tissue folds covering the crowns of teeth interfer- neously or persistently on slight provocation.
ing with occlusion. l Gingival necrosis and pseudo-membrane forma-

l Appears to project from beneath the gingival margin. tion is seen.


l Does not occur in edentulous spaces.

l More severe in maxillary and mandibular anterior

regions.
l It may occur in mouths with little or no plaque and may

be absent in mouths with abundant deposits.

Histopathology l Hyperplasia of connective tissue and epithelium. l Chronic inflammation with mature leukocytes and
l Abundance of amorphous ground substance. connective tissue infiltrated with immature and
l Fibroblast to collagen ratio is equal to that of normal proliferating leukocytes.
gingiva. l Isolated areas of acute necrotizing inflammation

l The connective tissue appears highly vascularized in with pseudo-membranous meshwork of fibrin, ne-
cyclosporine-induced enlargement. crotic epithelial cells, polymorphonuclear leuko-
cytes and bacteria.

Q.3. Discuss noninflammatory gingival enlargement.


Ans.

Drug-induced fibrotic enlargement (phenytoin,


cyclosporine, nifedipine) Idiopathic gingival enlargement
Aetiology l Long-term therapy of the respective drug. l Unknown, possible aetiology may be hereditary.
Location l Marginal and papillary, generalized. l Diffused enlargement and generalized.
Clinical features l Bead-like enlargement of facial and lingual gingival l Facial and lingual surface of maxillary and man-
margins. dibular teeth are affected but involvement limited
l Massive tissue folds covering the crowns of teeth to either jaw.
interfering with occlusion. l Enlarged gingiva is pink in colour, firm and leath-

l Appears to project from beneath the gingival margin. ery in consistency and has a characteristic peb-
l Does not occur in edentulous spaces. bled surface.
l More severe in maxillary and mandibular anterior l Enlargement projects into the oral vestibule, and

regions. jaw appears distorted.


l It may occur in mouths with little or no plaque and may

be absent in mouths with abundant deposits.

Histopathology l Hyperplasia of connective tissue and epithelium. l Increase in the amount of connective tissue and
l Abundance of amorphous ground substance. consists of densely arranged collagen bundles and
l Fibroblast to collagen ratio is equal to that of normal numerous fibroblasts.
gingiva.
l The connective tissue appears highly vascularized in

cyclosporine-induced enlargement.

Q.4. Benign tumours of gingiva. Focal fibrous hyperplasia (irritation fibroma)


l Focal fibrous hyperplasia is most often encountered
Ans.
in adults and is primarily located on the gingiva.
The various benign tumours of gingiva are as follows: l It is a nodular lesion and usually has a dome-like

l Focal fibrous hyperplasia (irritation fibroma) growth with a smooth surface of normal colouration.
l Peripheral ossifying fibroma l Surface hyperkeratosis is sometimes encountered.

l Peripheral giant cell granuloma It is a slowly progressing lesion and may remain
l Gingival cyst of the same size for many years.
Section | I  Topic-Wise Solved Questions of Previous Years 853

l It is sometimes referred to as peripheral fibroma. l The giant cell tissue consists of a mixture of
Histological features mononuclear giant cells with background of
l The surface epithelium may be intact, exhibits RBCs. Capillary vessels and sinusoidal spaces
hyperorthokeratosis or shows foci of ulceration. are usually present.
l This epithelium overlies an underlying mass of l Heavy deposits of haemosiderin are common.

dense fibrous connective tissue composed of Gingival cyst


significant amount of soft mature collagen in a Gingival cyst develops in the gingival soft tissues
scar-like pattern. outside bone and is derived from the rest of the den-
Peripheral ossifying fibroma tal lamina.
l It is a gingival nodule consisting of a reactive Clinical features
hyperplasia of connective tissue containing focal l Gingival cyst occurs as a firm, compressible

areas of bone. and fluid-filled swelling on the facial gingiva


l The peripheral ossifying fibroma is a reactive usually in the anterior or premolar region.
fibrous proliferation, probably of periosteal or l It usually develops as a solitary lesion and

periodontal ligament origin. rarely results in a cluster of lesions.


Clinical features l The lesion is of the same colour as the adjacent

l It arises from the interdental papillae, although normal mucosa.


occasionally it is seen to arise from the facial/ l It can occur in free or attached gingival or in-
lingual attached gingiva. terdental papilla.
l The mass originates from within the periodon- Histological features
tal ligament. l Lesions are often small with the epithelial lin-

l The overlying mucosa may be smooth and of ing of two to five cells in thickness.
normal colouration, or there may be foci of l They often exhibit neural thickening. Some-

surface ulceration. times, clear cells may be seen.


Histological features
l Within the connective tissue are diffused sheets Q.5. Pyogenic granuloma (granuloma pyogenicum).
of fibroblasts with plump monomorphic nuclei.
Ans.
l The overall picture is one of hypercellularity

with hyalinized collagenous component. In fo- l It is a distinctive clinical entity originating as a response
cal areas, osteoid deposits can be identified. of the tissues to a nonspecific infection.
l Rarely mature trabeculae of bone are seen. l It is a tumour-like growth that is considered as an exag-

Peripheral giant cell granuloma. gerated and conditioned response to minor trauma.
The peripheral giant cell granuloma is a hyper- l It arises as a result of minor trauma to the tissues, which

plastic reaction of the gingival connective tis- provides a pathway for the invasion of nonspecific types
sue in which the histiocytic and endothelial of microorganisms.
cellular component predominate. l The tissues respond in a characteristic manner to organ-
Clinical features isms of low virulence by overzealous proliferation of a
l Peripheral giant cell granuloma is found in all vascular type of connective tissue.
age groups with peak incidence in adults Clinical features
around 30 years of age. l Occurs most frequently on the gingiva.

l It is common in females and equally distrib- l More common in anterior region of maxilla.

uted between maxilla and mandible. It is most l They develop rapidly, reach full size.

commonly found anterior to the molars. l Lesion is usually an elevated, pedunculated or sessile
l Lesions begin as a reddish or purplish dome- vascular mass with a smooth, lobulated or even a warty
shaped swelling of the interdental papilla or surface, which commonly is ulcerated and shows a ten-
alveolar ridge. dency to bleed either spontaneously or upon slight trauma.
l Larger lesions usually encircle one or more l It is deep red or reddish-purple depending upon its

teeth, often involving the periodontal ligament. vascularity, painless and rather soft in consistency.
l These lesions produce loosening and move- Histological features
ment of the teeth. l The epithelium is thin and atrophic but it may be

Histological features hyperplastic also.


l Microscopic appearance reveals a nodular l Vast numbers of endothelium lined vascular spaces

arrangement of giant cell tissue separated by and the extreme proliferation of fibroblasts and bud-
fibrous septa. ding endothelial cells.
854 Quick Review Series for BDS 4th Year, Vol 1

Moderately intense infiltration of PMNLs, lympho-


l l This type of gingival enlargements initiate in the pres-
cytes and plasma cells. ence of bacterial plaque.
l The connective tissue stroma is typically delicate. l There are three types of conditioned enlargements:

Treatment i. Hormonal
l They are treated by surgical excision. ii. Nutritional
l During excision, care should be taken to scale the iii. Allergic
adjacent tooth and make certain that it is free of cal- Q.2. Angiogranuloma.
culus, since it may be irritating which leads to recur-
rence of the lesion. Ans.
Q.6. Peripheral giant cell granuloma. l Gingival enlargement in pregnancy is also termed as
Ans. angiogranuloma in order to avoid implication of neo-
plasm.
Clinical features l It is not a neoplasm but an inflammatory response
l Giant cell lesions of the gingiva-like peripheral giant
to local irritation and is modified by the patient’s
cell granuloma arise interdentally or from the gingi- condition.
val margin, occur most frequently on the labial sur- l It usually appears after the first trimester but may also
face and may be sessile or pedunculated. occur earlier.
l They vary in appearance from smooth and regularly

outlined masses to irregularly shaped and multilobu- Q.3. Classify gingival enlargements.
lated protuberances with surface indentations. Ans.
l Ulceration of the margin is occasionally seen.

l The lesions are painless, vary in size and may cover


Gingival enlargement can be classified based on aetiologi-
several teeth. cal factors and pathological changes as follows:
l Inflammatory gingival enlargement
l They may be firm or spongy, and the colour varies
l Drug-induced gingival enlargement
from pink to deep red or purplish-blue.
l Enlargement associated with systemic factors
l In some cases, the peripheral giant cell granuloma of

the gingiva is locally invasive and causes destruction a. Conditioned enlargement


of the underlying bone. b. Enlargements due to systemic diseases
l Idiopathic gingival enlargement
l Complete removal leads to uneventful recovery.
l Neoplastic gingival enlargement
Histopathology
l False enlargement
l The giant cell granuloma has numerous foci of mul-

tinuclear giant cells and haemosiderin particles in a Q.4. Drug-induced gingival enlargement.
connective tissue stroma.
Ans.
l Areas of chronic inflammation are scattered throughout

the lesion, with acute involvement occurring at the surface. l The use of the following drugs is associated with gingi-
l The overlying epithelium is usually hyperplastic, val enlargement.
with ulceration at the base. Bone formation occa- a. Anticonvulsants
sionally occurs within the lesion. b. Immunosuppressants
Treatment c. Calcium channel blockers
Surgical excision. l Phenytoin (Dilantin sodium) is an anticonvulsant that

Q.7. Compare drug-induced and idiopathic gingival has been used to control seizure disorders in patients
enlargements. with epilepsy and in the management of neuralgias.
l Phenytoin stimulates proliferation of fibroblast-like
Ans.
cell in tissue culture and also decreases the collagen
[Same as SE Q.3] degradation.
l It occurs most commonly in younger individuals.

l Phenytoin-induced gingival overgrowth is characterized


SHORT NOTES: by initial enlargement of the interdental papillae and is
Q.1. Conditioned gingival enlargement. less frequently accompanied by increased thickening of
the marginal tissue.
Ans.
l Management of drug-induced gingival overgrowth

l Conditioned enlargements are caused by the systemic should be a team approach involving the physician, the
conditions of the patient, which exaggerates the usual dentist and the health care professionals for prevention,
gingival response to dental plaque. early diagnosis and effective treatment.
Section | I  Topic-Wise Solved Questions of Previous Years 855

Q.5. What is the difference between gingival abscess and Consistency


periodontal abscess? l Semifirm but may have varying degrees of

softness and friability.


Ans.
l It is usually painless.
The differences between gingival abscess and periodontal Histopathology
abscess can be tabulated as follows: l Marginal and tumour-like enlargement con-

Gingival abscess Periodontal abscess


sists of a central mass of connective tissue, the
periphery of which is outlined with stratified
Location Localized swell- Usually affects the
l l
squamous epithelium.
ing affecting the deeper periodontal
marginal and in- structures, including Treatment
terdental gingiva. deep pockets, furca- l Meticulous plaque control, scaling and root
tions and vertical osse- planning, polishing should be the only none-
ous defects and located mergent periodontal procedure performed.
beyond the mucogingi-
l The second trimester is the safest time in
val junction.
which treatment may be performed.
Aetiology l Impaction of l Periodontal pocket l During pregnancy, the lesion should be removed
foreign objects related to destruction
surgically only when it interferes with mastica-
in previously by periodontitis.
healthy sites. tion and causes severe disfigurement and if the
patient willingly wants to get it removed.
Clinical l Gingiva appears l Associated with a peri-
findings to be red, swol- odontal pocket which Q.7. Wegener granulomatosis.
len and ex- may be either su-
tremely painful, prabony or infrabony. Ans.
and sometimes l Tooth elevation and

impacted foreign mobility may be seen; Wegener granulomatosis is a disease of unknown aetiology,
object may still tooth is tender on which basically involves the vascular, renal and respiratory
be embedded lateral percussion. systems.
into the gingiva. l Pain is localized and
Clinical features
patient can identify the
l Occurs at any age but majority of cases are in fourth
offending tooth.
l Affected tooth may
and fifth decades of life.
be vital or sometimes l Slight male predilection.
nonvital. l It is a multisystem disease which is usually first char-
l May be associated with
acterized clinically by the development of rhinitis,
a fistula.
sinusitis and otitis or ocular symptoms.
Radio- No bone loss is Bone loss is seen. l The patient soon develops cough and haemoptysis,
graphic evident. Radiolucency along the fever and joints pain.
features lateral aspect of the root.
l Haemorrhagic or vesicular skin lesions are also pres-

ent commonly.
Oral manifestations
Q.6. Pregnancy tumour.
l Involvement of gingiva is the most common and
Ans. characteristic manifestation and termed as straw-
berry gingivitis.
Pregnancy tumour or angiogranuloma
l Gingival lesions may be ulcerations, friable granular
l It is an inflammatory response to local irritation and
lesions or simply enlargements of gingiva.
is modified by the patient’s systemic condition.
l The inflammatory process starts in the interdental
Clinical features
papilla and spreads rapidly to the periodontal struc-
Appearance
ture and leads to bone loss and tooth mobility.
l The lesion appears as a discrete mushroom-
Treatment
like flattened spherical mass that protrudes
l Cytotoxic agents like cyclophosphamide and predni-
from the interdental papilla or the gingival
sone have provided good prognosis.
margin and is attached by a sessile or peduncu-
lated base. Q.8. Developmental gingival enlargements.
Colour
Ans.
l Dusky red or magenta with smooth glistening

surface that frequently exhibits numerous deep l Developmental enlargements are physiological and usu-
red, pinpoint markings. ally present no problems.
856 Quick Review Series for BDS 4th Year, Vol 1

l During the various stages of eruption, particularly of the l The gingival enlargement is due to the leukaemic cell
primary dentition, the labial gingiva may show a bul- infiltration in areas of mild chronic irritation.
bous marginal distortion caused by superimposition of l In severe cases, the teeth may be completely hidden.

the bulk of the gingiva on the normal prominence of the l Leukaemic enlargement may be diffused or marginal

enamel in the gingival half of the crown. and localized or generalized.


l This enlargement is known as developmental en- l It may appear as a diffused enlargement of the gingival

largement. mucosa, an oversized extension of the marginal gingival


or a discrete tumour-like interproximal mass.
Q.9. Give differential diagnosis of ‘epulis’.
l In leukaemic enlargement, the gingiva is generally blu-

Ans. ish-red and has a shiny surface.


l Epulis is a generic term used clinically to designate all Q.11. What is drug-induced gingival hyperplasia?
discrete tumours and tumour-like masses of the gingiva.
Ans.
l It serves to locate the tumour but not to describe it.

l Most lesions referred to as ‘epulis’ are inflammatory [Same as SN Q.4]


rather than neoplastic.
Q.12. Pathogenesis of phenytoin sodium gingival en-
l Differential diagnosis of epulis includes oral fibroma.
largement.
Q.10. Leukaemic gingival enlargement.
Ans.
Ans.
[Same as SN Q.4]
l In leukaemic gingival enlargement, the gingiva is boggy,
oedematous and deep red in colour.

Topic 17
Acute Gingival Infections
COMMONLY ASKED QUESTIONS
LONG ESSAYS:
1 . Describe clinical features and treatment of acute necrotizing ulcerative gingivitis.
2. What are acute infections of gingiva? Describe in detail any one of them.
3. Describe aetiology, clinical features, differential diagnosis and treatment of acute necrotizing ulcerative gingivi-
tis in detail. [Same as LE Q.1]
4. Classify gingival lesions and discuss in detail acute necrotizing ulcerative gingivitis and its management.
[Same as LE Q.1]
5. Give the signs, symptoms, differential diagnosis and treatment of acute necrotizing ulcerative gingivitis.
[Same as LE Q.1]
6. What is a Vincent infection? Write its aetiology, clinical features and management. [Same as LE Q.1]
7. Enumerate acute gingival infection. Discuss the aetiopathogenesis, clinical features and treatment of acute nec-
rotizing ulcerative gingivitis. [Same as LE Q.1]
8. Give the signs, symptoms, differential diagnosis and treatment of acute herpetic gingivostomatitis. [Same as LE Q.2]
9. Enumerate acute gingival lesions. Discuss in detail aetiology, clinical features, histopathology and differential
diagnosis of acute herpetic gingivostomatitis. [Same as LE Q.2]

SHORT ESSAYS:
1 . Pericoronitis.
2. Compare acute herpetic gingivostomatitis and necrotizing ulcerative gingivitis.
Section | I  Topic-Wise Solved Questions of Previous Years 857

3 . Clinical features of acute herpetic gingivostomatitis. [Ref LE Q.2]


4. Treatment of ANUG. [Ref LE Q.1]
5. Management of acute pericoronitis. [Same as SE Q.1]
6. Aetiology of ANUG. [Same as SE Q.4]
7. NUG treatment. [Same as SE Q.4]

SHORT NOTES:
1. Aphthous ulcers.
2. Enumerate the acute infections/acute lesions of the gingiva. [Ref LE Q.2]
3. Primary herpetic gingivostomatitis.
4. Pericoronitis.
5. EM findings of NUG. [Ref LE Q.1]
6. Management of acute herpetic gingivostomatitis. [Ref LE Q.2]
7. Diagnosis of acute herpetic gingivostomatitis.
8. Gingival abscess.
9. Differential diagnosis of acute herpetic gingivostomatitis. [Ref LE Q.2]
10. Bacterial microflora of ANUG.
11. Acute herpetic gingivostomatitis. [Same as SN Q.3]
12. Management of pericoronitis. [Same as SN Q.4]
13. Treatment of acute pericoronitis. [Same as SN Q.4]

SOLVED ANSWERS
LONG ESSAYS:
Q.1. Describe clinical features and treatment of acute l The causative Fusobacterium species are Gram-
necrotizing ulcerative gingivitis. negative, obligate, anaerobic and rod-shaped
Ans. organisms.
l It is postulated that the temporary establishment
l Acute necrotizing ulcerative gingivitis (ANUG) is a of a specific anaerobic environment allows Fuso-
rapid onset, painful microbial disease of the gingiva bacterium and the normal oral spirochaetes to
caused primarily by the Fusobacterium species, proba- multiply synergistically, producing this infection.
bly in combination with oral spirochaetes. l Borrelia, Gram-positive cocci, b-haemolytic
l ANUG is renamed as necrotizing ulcerative gingivitis streptococci and Candida albicans have been iso-
(NUG). It is also known as trench mouth because of its lated from the lesions of HIV-associated NUG.
prevalence in the soldiers working in trenches during Predisposing factors
World War I. Vincent first described the bacteria associ- They are divided as local predisposing factors and sys-
ated with these infections; hence the disease was also temic predisposing factors:
known as Vincent angina. a. Local predisposing factors
l It is characterized by the death and sloughing of gingi- l Pre-existing gingivitis, deep periodontal pock-
val tissues and presents with characteristic signs and ets and pericoronal flaps which favour the
symptoms. proliferation of anaerobic fusiform bacilli and
Aetiology spirochaetes.
l It is a fusospirochaetal infection caused by a fuso- l Frequency of NUG increases with smoking
spirochaetal complex. and tobacco use.
This complex consists of the following bacteria: b. Systemic predisposing factors
Treponema microdentium, intermediate spiro- l Immunodeficient patients are most commonly
chaetes, Vibrios, fusiform bacilli and filamentous affected by NUG.
organisms, Borrelia species. l Nutritional deficiencies (e.g. vitamins C and B2).
l Although there are 16 known species of Fusobac- l Fatigue caused by chronic sleep deficiency.
terium, F. nucleatum is the main organism found l Habits such as alcohol or drug abuse and sys-
in the oral cavity. temic diseases, e.g. diabetes.
858 Quick Review Series for BDS 4th Year, Vol 1

l Other debilitating diseases that may predispose Stage 3: Necrosis of the marginal gingiva (NUP)
patients to NUG include chronic diseases like Stage 4: Necrosis extending to the marginal gingiva
syphilis, cancer, severe gastrointestinal disor- (NUP)
der, leukaemia, anaemia and acquired immu- Stage 5: Necrosis involving the buccal and labial mu-
nodeficiency syndrome (AIDS). cosa (necrotizing stomatitis)
c. Psychosomatic factors Stage 6: Necrosis exposing alveolar bone (necrotizing
The disease is often associated with stressful situ- stomatitis)
ations, e.g. school examination, induction into Stage 7: Necrosis perforating skin of the cheek (NOMA)
armed forces, patients with depression or other Relation of bacteria to the characteristic lesion
emotional disorders, and patients feeling inade-
quate at handling life situations.
Psychiatric Disturbances {SN Q.5}
Clinical features The characteristic lesion of NUG is studied under light
l It is usually present as an acute disease, and and electron microscopy to study the relation of the bac-
symptoms are sudden in onset. teria in the lesion.
Sometimes, it resolves on its own and has milder Listgarten described the following zones in the le-
symptoms leading to a subacute stage. sion, which may overlap with one another, all zones may
l Patients may have an episode of debilitating dis- not be present at the same time.
ease, or acute respiratory tract infection, continu- Zone 1 (bacterial zone)
ous work without rest, poor nutrition, tobacco use The most superficial zone comprises various types of
and psychological stress are frequent features. bacteria and few spirochaetes of the small, medium
The characteristic signs are as follows: and large types.
i. There are punched-out, crater-like depres- Zone 2 (neutrophil-rich zone)
sions at the crest of the interdental papillae Contains numerous leukocytes, predominantly leu-
and it may involve the marginal gingiva, kocytes with bacteria, including spirochaetes of vari-
whereas attached gingiva and oral mucosa ous types interspersed in between the leukocytes.
are rarely involved. Zone 3 (necrotic zone)
ii. Grey, pseudo-membranous slough covers Consists of disintegrated tissue cells, fibrillar mate-
the gingival craters. It is demarcated from rial, remnants of collagen fibres and numerous inter-
the healthy gingiva by a pronounced linear mediate and large type of spirochaetes, with few
erythema. other bacteria.
iii. In some cases, lesions may be denuded of the Zone 4 (zone of spirochaetal infiltration)
pseudo-membrane, exposing red, shiny and Consists of well-preserved tissue infiltrated with in-
haemorrhagic gingival surface. termediate and large spirochaetes without other or-
iv. The lesion will bleed on slight provocation. ganisms.
v. There is fetid odour and increased salivation.
vi. Patient complains of a constant radiating and
gnawing pain that is aggravated by eating Diagnosis
spicy and hot food and on chewing. l Diagnosis is primarily made based on the

vii. There is metallic foul taste and pasty saliva. clinical findings of gingival pain, ulceration
viii. Extraoral and systemic signs and symptoms and bleeding.
include local lymphadenopathy and mild fever. l Microscopic examination of a bacterial smear

ix. In severe cases, high fever, increased pulse or a biopsy specimen does not give specific
rate, leukocytosis, loss of appetite and gen- picture.
eral lassitude are the most common systemic l Biopsy specimen can be used to differentiate

signs, and they are more severe in children. NUG from specific infections, such as tubercu-
Clinical course losis, or neoplastic disease.
NUG may lead to necrotizing ulcerative periodontitis Treatment
(NUP) if untreated. Objectives of treatment of NUG include the
Horning and Cohen extended the staging as follows: following:
Stage 1: Necrosis of the tip of the interdental papilla i. Resolution of acute phase.
(NUG) ii. Treatment of the chronic disease either
Stage 2: Necrosis of the entire papilla (either NUG or underlying the acute involvement or else-
NUP) where in the oral cavity.
Section | I  Topic-Wise Solved Questions of Previous Years 859

iii. Alleviation of generalized symptoms such Patient has to be checked for resolution of sys-
l

as fever and malaise. temic signs and symptoms.


iv. Correction of the systemic aetiological fac- l The area of the lesion will be erythematous but

tors, e.g. smoking and stress. with marked reduction of necrotic tissue, scaling
may be reperformed.
[SE Q.4]
l Patient should follow the same instructions as

{NUG is managed in a systematic sequence of three given during the first visit.
clinical visits. Third visit
First visit l Usually following 5 days after the second visit,

l Treatment of acute lesion is the primary goal. patient is evaluated for the resolution of symp-
l Complete evaluation of the patient regarding toms and a complete protocol for the periodontal
medical history and history of recent illness. management is planned.
l Evaluation of dietary history, history of smoking, l The patient is asked to discontinue hydrogen per-

risk factors for HIV and psychological factors. oxide rinse and continue with chlorhexidine
l General examination should be performed to mouthwash for 2 or 3 weeks.
examine the vital signs and palpation of lymph l If required, scaling and root planing can be re-

nodes, especially submaxillary and submental peated. A patient should be reinstructed to follow
nodes. Patient should also be examined for the the appropriate plaque-control measures.
skin lesions. l To prevent possible recurrence, the patient is fur-

l The pseudo-membrane and nonattached surface ther counselled on nutrition, smoking cessation
debris should be removed with a moist cotton and other associated habits.
swab and a topical anaesthetic is applied to the l Appointments should be scheduled for the treat-

affected area. ment of chronic gingivitis, periodontal pockets as


l Supragingival scaling with ultrasonic instruments well as for the elimination of all local irritants,
can be performed. including defective restorations.
l Subgingival scaling and curettage is contraindi- l Patient is re-evaluated after 1 month for mainte-

cated as it may lead to extension of infection and nance of oral hygiene, health habits, psychosocial
can cause bacteraemia. factors and determination of the need for recon-
l Usually 4 weeks time is required for a patient to structive or aesthetic surgery.
be symptom-free. Periodontal surgery and extrac- Additional treatment considerations
tions are postponed until the patient is symptom- l Contouring of gingival margin

free. l Nutritional supplements}

l Patient has to be placed on the following antibi- Contouring of gingival margin


otic regimen: l In cases of NUG when there is a severe loss of

i. Amoxicillin 500 mg orally every 6 h for 10 days. interdental gingiva as well as bone, the normal
ii. If allergic to amoxicillin, erythromycin 500 mg gingival architecture is restored by a periodon-
every 6 h or metronidazole 500 mg twice daily tal plastic surgery or by reshaping the gingiva
for 7 days. surgically.
iii. To control the pain, an analgesic such as non- Nutritional supplements
steroidal anti-inflammatory drug (NSAID) l When the patient is unable to take food due to

can be prescribed. pain, nutritional supplements may be indicated


The following instructions are given to the patient: along with local treatment.
a. Avoid tobacco, alcohol and condiments. l Patient may be given a standard multivitamin
b. Patient is advised to rinse with 3% hydrogen peroxide preparation along with therapeutic dose of
mixed with equal amount of warm water every 2 h or vitamins B and C.
twice daily with 0.2% chlorhexidine mouthwash. Q.2. What are acute infections of gingiva? Describe in
c. Patient should be advised to avoid overzealous tooth detail any one of them.
brushing and use of interdental cleaning devices.
d. An ultrasoft toothbrush may be used to clean the Ans.
surface debris.
e. Patient is advised to take adequate rest. {SN Q.2}
Second visit The acute gingival infections are the infections that occur with
l It should be scheduled usually 1 or 2 days after
sudden onset, are of short duration and also very painful.
the first visit.
860 Quick Review Series for BDS 4th Year, Vol 1

latent HSV in the neuronal ganglia that innervate


The common conditions that can be included under
the site.
this category are explained below.
l HSV-1 most commonly resides in the trigeminal
According to Manson, classification of various acute
ganglion.
gingival lesions is as follows:
l Secondary manifestation occurs as a result of
a. Traumatic lesions of gingiva
various stimuli such as sunlight, trauma, fever and
l Physical injury
stress.
l Chemical injury
l Secondary manifestations include herpes labialis,
b. Viral infections
herpetic stomatitis, herpetic genitalis, ocular her-
l Acute herpetic gingivostomatitis
pes and herpetic encephalitis.}
l Herpangina
Oral signs
l Measles
l It involves both the gingiva and the oral mucosa.
l Herpes varicella/zoster virus infections
l In the initial stage, it is characterized by the pres-
l Glandular fever
ence of discrete, spherical, grey vesicles on the
c. Bacterial infections
gingiva, labial and buccal mucosa, soft palate,
l ANUG
pharynx, sublingual mucosa and the tongue.
l Tuberculosis
l After approximately 24 h, the vesicles rupture and
l Syphilis
form painful, small ulcers with a red, elevated,
d. Fungal diseases
halo-like margin and a depressed, yellowish or
l Candidiasis
greyish-white central portion. The ulcers may oc-
e. Gingival abscess
cur in clusters or can be widely separated.
f. Aphthous ulcers
l In some cases, the vesicles may not be present.
g. Erythema multiforme
The lesion may appear as diffused, erythematous,
h. Drug allergy and contact hypersensitivity
shiny discolouration and oedematous enlargement
of the gingiva with a tendency to bleed.
l The lesion resolves on its own by 7–10 days. It
[SE Q.3]
heals without scarring.
{Primary herpetic gingivostomatitis Oral symptoms
l Primary herpetic gingivostomatitis is an infection of l Patient complains of soreness of mouth associated
the oral cavity affecting mostly infants and children with difficulty in eating and drinking.
younger than 6 years of age. l The lesions are painful and sensitive to touch,
The causative organism is herpes simplex virus type changes in temperature and food and beverages,
1 (HSV-1). especially spicy foods and fruit juices and to the
Clinical features action of coarse foods.
l Primary herpetic gingivostomatitis develops These oral signs and symptoms are accompanied
mostly in children and young adults. by the following constitutional signs and symp-
l Males and females are equally affected. toms like:
l Painful vesicular lesions develop on all mucosal l Cervical adenitis
surfaces and rupture to produce foul-smelling ulcers. l High-grade fever
l The patient is usually febrile, drools, has signifi- l Generalized malaise
cant malaise and will have tender cervical lymph- Diagnosis
adenopathy. l The diagnosis in most cases can be established by
l The lesions and acute illness last for about 10 days history and clinical examination.
and resolve with scar formation. l Confirmatory tests include virus culture and im-
The HSV-1 gains access to the patient via direct munologic tests using monoclonal antibodies or
or airborne water droplet transmission from an DNA hybridization techniques.
infected individual.
l The mucous membrane lesions represent direct

viral infection at the site of inoculation. {SN Q.9}


l The clinical course is limited by the synthesis of
Differential diagnosis
viral-specific antibodies (IgM: 3–5 days, IgG:
The lesion resembles the following conditions:
5–21 days).
l NUG
l After primary infection, the virus ascends through
l Desquamative gingivitis
sensory and autonomic nerves and persists as
Section | I  Topic-Wise Solved Questions of Previous Years 861

Q.8. Give the signs, symptoms, differential diagnosis


l Recurrent aphthous stomatitis (RAS)
and treatment of acute herpetic gingivostomatitis.
l Erythema multiforme
l Stevens–Johnson syndrome Ans.
l Bullous lichen planus
[Same as LE Q.2]
Q.9. Enumerate acute gingival lesions. Discuss in detail
Treatment aetiology, clinical features, histopathology and differen-
tial diagnosis of acute herpetic gingivostomatitis.
{SN Q.6}
Ans.
l Previously, treatment consisted of only supportive
[Same as LE Q.2]
care; recently, an antiviral therapy with 15 mg/kg of
an acyclovir suspension given five times daily for
7 days has been implicated to reduce the course of SHORT ESSAYS:
disease.
Q.1. Pericoronitis.
l Antiviral therapy is found to be effective only if the

lesions are diagnosed within 3 days of onset, and it Ans.


reduces symptoms like fever, difficulty in eating and
Pericoronitis is an acute infection which refers to inflamma-
also the appearance of new extraoral lesions.
tion of gingiva and surrounding soft tissues of an incom-
l The patients reporting after 3 days of onset should be
pletely erupted tooth. It occurs most frequently in the
given a palliative care which includes removal of
mandibular third molar region.
plaque and food debris, administration of NSAIDs
Types
and nutritional supplements.
a. Acute
l To prevent exacerbation, periodontal therapy should
b. Subacute
be postponed until the acute symptoms subside.
c. Chronic
Clinical features
Signs and symptoms
Q.3. Describe aetiology, clinical features, differential
l The pericoronal flap will be markedly red and
diagnosis and treatment of acute necrotizing ulcerative
swollen with presence of exudate.
gingivitis in detail.
l Patient complains of pain radiating to the ear,
Ans. throat and floor of the mouth.
l Patient is extremely uncomfortable because of
[Same as LE Q.1]
inability to open the jaws and will complain of
Q.4. Classify gingival lesions and discuss in detail acute foul taste.
necrotizing ulcerative gingivitis and its management. l In addition to pain, swelling of the cheek in the

Ans. region of the angle of the jaw is seen.


l Lymphadenitis may be present.
[Same as LE Q.1] l Toxic systemic symptoms such as fever, mal-
Q.5. Give the signs, symptoms, differential diagnosis aise and leukocytosis are also present.
and treatment of acute necrotizing ulcerative gingivitis. Complications
l Pericoronal abscess.
Ans.
l Peritonsillar abscess, cellulitis and Ludwig an-
[Same as LE Q.1] gina are infrequent but nevertheless potential
Q.6. What is a Vincent infection? Write its aetiology, sequelae of acute pericoronitis.
clinical features and management. Treatment
The treatment of pericoronitis depends on:
Ans.
l Severity of the inflammation
[Same as LE Q.1] l Systemic involvement

l The possibility of retaining the tooth


Q.7. Enumerate acute gingival infection. Discuss the
aetiopathogenesis, clinical features and treatment of First visit
l Gentle irrigation of the area with warm saline
acute necrotizing ulcerative gingivitis.
to remove debris and exudates.
Ans. l Occlusion is evaluated to determine whether an

[Same as LE Q.1] opposing tooth is occluding with the pericoronal


862 Quick Review Series for BDS 4th Year, Vol 1

flap. Occlusal adjustment is necessary in case of Q.3. Clinical features of acute herpetic gingivostomatitis.
opposing tooth traumatizing the flap.
Ans.
l Administration of antibiotics is indicated in se-

vere cases with involvement of lymph nodes. [Ref LE Q.2]


l Drainage can be established if the swelling is
Q.4. Treatment of ANUG.
fluctuant by using a No. 15 blade.
l Instructions to the patient include hourly rinses Ans.
with warm saline gargles. l ANUG is a rapid onset, painful microbial disease of the
l The acute pericoronitis is treated to relieve the
gingiva caused primarily by the Fusobacterium species,
acute symptoms. probably in combination with oral spirochaetes.
Next visit l ANUG is renamed as NUG.
l After resolution of acute symptoms, the tooth
It is also known as trench mouth.
should be evaluated for possibility of erupting to Aetiology
a functional position. l It is a fusospirochaetal infection caused by a
l If the tooth is to be retained, the pericoronal flap
fusospirochaetal complex.
should be surgically excised using the periodontal l Although there are 16 known species of Fuso-
knives. bacterium, F. nucleatum is the main organism
l The surgical procedure to remove the operculum
found in the oral cavity.
is called operculectomy. It can also be performed l The causative Fusobacterium species are
by electrosurgery. Gram-negative, obligate, anaerobic, rod-shaped
l The removal of the flap should result in a site that
organisms.
can be easily maintained by the patient. l It is postulated that the temporary establish-
To ensure this, the flap covering the occlusal surface as ment of a specific anaerobic environment
well as the tissue distal to the tooth has to be removed. allows Fusobacterium and the normal oral
spirochaetes to multiply synergistically, pro-
Q.2. Compare acute herpetic gingivostomatitis and nec- ducing this infection.
rotizing ulcerative gingivitis. [Ref LE Q.1]
Ans. Q.5. Management of acute pericoronitis.
Ans.
Primary herpetic
gingivostomatitis NUG [Same as SE Q.1]
Aetiology HSV-1 Occurs due to interac- Q.6. Aetiology of ANUG.
tion between the host Ans.
and bacteria (fusospi-
rochaetal complex). [Same as SE Q.4]
Clinical l Diffuse erythema l A necrotizing Q.7. NUG treatment.
features and vesicular condition with Ans.
lesions which punched-out gingi-
rupture leaving val margins covered [Same as SE Q.4]
slightly depressed by a pseudo-
area of ulcers. membrane that
l Diffused involve- peels off.
SHORT NOTES:
ment of the l Only marginal gin-
Q.1. Aphthous ulcers.
gingiva. giva is involved.
Age Children and Occurrence in chil- Ans.
adolescents are more dren is a rare case.
l Aphthous ulcers are characterized by the appearance of
commonly affected.
discrete spherical vesicles that rupture after 1 or 2 days
Course It has a course of No definite duration, and form depressed spherical ulcers.
7–10 days. of course.
l The ulcers consist of a saucer-like red or greyish-red
Immunity Acute episodes lead It does not develop an central portion and an elevated rim at the periphery.
and to development of immunity and is not l The lesions may occur anywhere in the oral cavity, the
contagion immunity and it is contagious.
mucobuccal fold, the floor of the mouth being the most
contagious.
common sites.
Section | I  Topic-Wise Solved Questions of Previous Years 863

l It is a painful lesion and may occur as a single lesion or not have circulatory antibodies against virus, e.g. her-
lesions scattered throughout the mouth. The duration of petic whitlows on the hands of hospital staff and dis-
each lesion is 7–10 days. seminated infection in newborn.
Aphthous stomatitis may occur in the following forms: l Yellowish fluid-filled vesicles are found in oral cavity at
a. Occasional aphthae any place. They rupture to form painful ulcers covered
They are single lesions that occur at intervals that by grey membrane and surrounded by red halo.
vary from months to years. l Healing occurs in 7–14 days with no scarring.
b. Acute aphthae l Sometimes, acute generalized gingivitis is also present.
There is an acute episode of aphthous ulcer which l After primary infection, virus may travel along the
may persist for weeks. It is often seen in children nerves and remain in dormant phase in trigeminal or
and adults with acute gastrointestinal disorders. lumbosacral ganglion.
c. Chronic recurrent aphthae Histological features
It is a condition in which one or more oral lesions Both primary and secondary lesions show:
are always present and are recurrent. l Ballooning degeneration in cells.

Aetiology l Lipschultz bodies in some cells within the

It is unknown. nuclei causing perinuclear halo.


Predisposing factors Treatment
l Hormonal disturbances, allergic phenomena, l Antiviral drugs, e.g. acyclovir, vidarabine, idoxu-

gastrointestinal disorders and psychosomatic ridine.


factors. l Suppurative treatment for fever.

Treatment l Aspirin or acetaminophen.

Various medications have been used in the l Dicyclomine hydrochloride is used topically.

treatment of this condition as follows:


Q.4. Pericoronitis.
a. Local applications:
Using 8% zinc chloride, Talbot’s iodine, Ans.
phenol, riboflavin, thiamine, etc.
l The soft tissue infection around crown of partially im-
b. Chlortetracycline (aureomycin) has been
pacted tooth is termed as ‘pericoronitis’.
successfully used as a mouthwash or ap-
Clinical features
plied topically as a 3% ointment or ad-
l The pericoronal flap will be markedly red and
ministered systemically in the form of
swollen with presence of exudate.
250-mg capsules.
l A pocket is formed between operculum and the
c. Palliative treatment:
crown harbouring microorganisms.
l To make the patient comfortable dur-
l Streptococci and anaerobic bacteria predominate
ing the disease period.
in this pocket.
l Relief in pain is obtained with 0.5%
l Initially mild inflammation occurs in soft tissues.
solution of dyclonine hydrochloride –
But repeated trauma due to mastication further
a topical anaesthetic mouthwash that
increases the inflammation and swelling.
may be diluted 1:1 with water.
l In severe cases, local and extraoral swelling is
l Systemically administered aspirin is
present along with pain and fever.
usually sufficient for relief of pain.
l In more severe or serious cases, infection may
d. Supportive treatment:
spread to surrounding fascial planes.
l Copious fluid intake and systemic
l Trismus and fever is also present along with facial
antibiotic therapy for management of
swelling.
toxic systemic complications.
Treatment
Q.2. Enumerate acute infections/acute lesions of the l Scaling and irrigation with saline and chlorhexi-
gingiva. dine in mild case.
Ans. l Operculectomy.

l Extraction of impacted tooth could be done.


[Ref LE Q.2]
l Antibiotic should also be prescribed.
Q.3. Primary herpetic gingivostomatitis.
Q.5. EM findings of NUG.
Ans.
Ans.
l Primary herpetic stomatitis occurs in those persons who
are not infected previously with herpes virus or they do [Ref LE Q.1]
864 Quick Review Series for BDS 4th Year, Vol 1

Q.6. Management of acute herpetic gingivostomatitis. Q.9. Differential diagnosis of acute herpetic gingivosto-
matitis.
Ans.
Ans.
[Ref LE Q.2]
[Ref LE Q.2]
Q.7. Diagnosis of acute herpetic gingivostomatitis.
Q.10. Bacterial microflora of ANUG.
Ans.
Ans.
Diagnosis of acute herpetic gingivostomatitis:
l The diagnosis in most cases can be established by his- l ANUG is a fusospirochaetal infection caused by a fuso-
tory and clinical examination. spirochaetal complex.
l Confirmatory tests include: This complex consists of the following bacteria:
i. Virus culture Treponema microdentium, intermediate spirochaetes,
ii. Immunological tests using monoclonal antibodies or Vibrios, fusiform bacilli and filamentous organisms,
DNA hybridization techniques Borrelia species.
l Although there are 16 known species of Fusobac-
Q.8. Gingival abscess.
terium, F. nucleatum is the main organism found
Ans. in the oral cavity.
l The causative Fusobacterium species are Gram-
Gingival abscess is a localized, acute inflammatory lesion
negative, obligate, anaerobic, rod-shaped organisms.
that may arise from a variety of sources, including microbial
l Borrelia, Gram-positive cocci, b-haemolytic
plaque accumulation, trauma and foreign body impaction.
streptococci and Candida albicans have been iso-
Aetiology
lated from the lesions of HIV-associated NUG.
Impaction of foreign objects (food particles, dental
floss, impression material) in previously healthy sites. Q.11. Acute herpetic gingivostomatitis.
Clinical findings
Ans.
Gingiva appears to be red, swollen and extremely pain-
ful, and sometimes impacted foreign object may still be [Same as SN Q.3]
embedded into the gingiva.
Q.12. Management of pericoronitis.
Treatment
l Under topical or local anaesthesia, when possible, Ans.
scaling and root planing are completed to establish
[Same as SN Q.4]
drainage and any foreign material is removed.
l The area is irrigated with warm water and covered Q.13. Treatment of acute pericoronitis.
with moist gauze under light pressure.
Ans.
l Once bleeding has stopped, the patient is dismissed

with instructions to rinse with warm saline gargles [Same as SN Q.4]


every 2 h for the rest of the day.
l After 24 h, the area is reassessed.

Topic 18
Desquamative Gingivitis
COMMONLY ASKED QUESTIONS
LONG ESSAYS:
1. Describe about aetiology, clinical features, pathogenesis and management of chronic desquamative gingivitis.
2 . Classify desquamative gingivitis lesions and describe in detail the candidiasis lesions.
Section | I  Topic-Wise Solved Questions of Previous Years 865

3. Describe the aetiopathogenesis, histopathology, clinical features and treatment of chronic desquamative gingivi-
tis. [Same as LE Q.1]
4. What is desquamative gingivitis? Give the clinical and histopathological features and treatment of severe form
of desquamative gingivitis. [Same as LE Q.1]

SHORT ESSAYS:
1 . Treatment of chronic desquamative gingivitis.
2. Mucous membrane pemphigoid.
3. Define and classify chronic desquamative gingivitis lesions. [Ref LE Q.2]
4. Management of chronic desquamative gingivitis. [Same as SE Q.1]

SHORT NOTES:
1. Define chronic desquamative gingivitis. [Ref LE Q.1]

SOLVED ANSWERS
LONG ESSAYS:
Q.1. Describe about aetiology, clinical features, patho- a. Mild desquamative gingivitis
genesis and management of chronic desquamative l Females between the age of 17 and 23 years
ingivitis. are usually affected.
Ans. l It is characterized by diffused erythema of the

marginal, interdental and attached gingiva.


l It is painless, and the patient generally com-
{SN Q.1} plains of the discolouration of the gingiva.
l Desquamative gingivitis is a clinical term used to de- b. Moderate desquamative gingivitis
scribe red, painful, glazed and friable gingiva, which l Persons between the age of 30 and 40 years are

may be a manifestation of some mucocutaneous con- usually affected.


ditions such as lichen planus or the vesiculobullous l Moderate form presents as a patchy distribu-

disorders. tion of bright red and grey areas involving the


l Chronic desquamative gingivitis is characterized by marginal and attached gingiva.
intense redness and desquamation of the surface epi- l Gingival surface appears soft, smooth and

thelium of the attached gingiva. shiny, exhibits pitting on pressure.


l Initially, it was considered to be a degenerative con- l Epithelium is not firmly adherent to the under-

dition and was termed as ‘gingivosis’. lying connective tissue, and it peels off on
l For many years, desquamative gingivitis was consid- massaging the gingiva with finger, exposing
ered as a specific disease entity until McCarthy et al. the connective tissue.
(1960) reported that it is nothing but nonspecific l Oral mucosa in general appears smooth and

gingival manifestations of a variety of systemic dis- shiny. Patients complain of burning sensation
turbances. in the mouth. Exposure to thermal changes and
inhalation of air is painful, and spicy food is
intolerable.
Clinical features l Tooth brushing results in denudation of the
l Chronic desquamative gingivitis is now believed to gingival surface causing pain. Accumulation of
be the oral manifestation of dermatosis-like lichen deposits on teeth increases the gingival inflam-
planus, mucous membrane pemphigoid, bullous mation.
pemphigoid or pemphigus. c. Severe desquamative gingivitis
l Based on the clinical manifestations, desquamative l It is characterized by irregularly shaped de-
gingivitis is divided as: nuded areas on the gingiva. These areas are
a. Mild bright red in colour and scattered all over the
b. Moderate gingival surface. Other regions of the gingiva
c. Severe appear greyish-blue.
866 Quick Review Series for BDS 4th Year, Vol 1

l Condition is extremely painful and there is constant l Topical corticosteroid ointment may be used for
dry and burning sensation all over the oral cavity. several times a day. Before application, gingiva
l The surface epithelium can be easily peeled off; this may be gently dried with sterile sponge. Triam-
sign is termed as Nikolsky sign. A blast of air cinolone acetonide (0.1%), fluocinonide (0.05%)
directed at the gingiva results in elevation of the or desonide (0.05%) may be used for topical ap-
epithelium. plication.
l The oral mucosa is smooth and shiny. l In severe cases, systemic administration of corti-
l Patients having the severe form of desquamative costeroids may be considered. It should be started
gingivitis cannot tolerate coarse food, condiments or only after evaluation of the general health of the
changes in temperature. patient and physician’s consent.
Histopathology l Mucous membrane pemphigoid responds fa-
l The lesions in desquamative gingivitis may be of vourably to systemic steroid therapy. Prednisone
bullous- or lichenoid-type. 30–40 mg daily or on alternate days to begin
l Histopathologically, the bullous type has the fea- with and gradually reduced to 5–10 mg daily or
tures of mucous membrane pemphigoid and the on alternate days as a maintenance dose may be
lichenoid type resembles lichen planus. used.
l Separation of epithelium from the underlying l Nutritional supplements may also be recom-
connective tissue begins with separation of colla- mended.
gen fibrils and a decrease in the number of an- l It takes a long time for the lesions to heal; hence
choring fibrils. the patient should be told not to discontinue the
l The epithelium appears atrophic with reduced treatment before the lesions heal and to wait pa-
keratinization and infiltration of the connective tiently for the results.
tissue with inflammatory cells. Q.2. Classify desquamative gingivitis lesions and de-
Diagnosis scribe in detail the candidiasis lesions.
l Proper diagnosis is important for deciding the line

of treatment. Oral cavity should be examined Ans.


carefully to see whether any other lesions are [SE Q.3]
present.
l In the case of lichen planus, gingiva as well as {Classification of diseases that clinically present as des-
other parts of the oral mucosa may be affected, quamative gingivitis is as follows:
usually. Papular skin lesions, especially in the l Dermatological

regions of wrists and ankles are commonly seen i. Lichen planus


in lichen planus. ii. Mucous membrane pemphigoid
l A detailed history would reveal whether any iii. Bullous pemphigoid
extraoral lesions are present. Conjunctivitis, iv. Pemphigus vulgaris
burning sensation on urination, vaginal irrita- v. Chronic ulcerative stomatitis
tion, etc. are suggestive of mucous membrane vi. Linear IgA disease
pemphigoid. vii. Lupus erythematosus
l A possible hormonal aetiology may be thought of l Allergic reaction

in cases having history of menopause or hysterec- i. Dental restorative materials


tomy. Biopsy will help to confirm the diagnosis of ii. Food (reaction to oral hygiene products, chew-
lichen planus, mucous membrane pemphigoid or ing gum, etc.)
some of the rare chronic bacterial infections such iii. Drug-induced mucocutaneous disorder
as candidiasis. l Traumatic lesions

Treatment i. Physical injury


l Improvement of oral hygiene is essential to ii. Chemical injury
reduce the gingival inflammation. iii. Thermal injury
l Patient must be instructed to use soft toothbrush l Miscellaneous conditions}

for routine plaque control. Care should be taken Candidiasis


not to traumatize the gingival tissue during l Moniliasis or thrush is a common mycotic in-

brushing. fection of the oral mucosa.


l Hydrogen peroxide (3%) diluted with two parts of l Causative organism is Candida albicans,

warm water may be used as mouthwash twice which is a common inhabitant of the oral
daily. cavity.
Section | I  Topic-Wise Solved Questions of Previous Years 867

l In normal circumstances, they are non-pathogenic, Management of chronic desquamative gingivitis must be
but the organism may become pathogenic in de- based on an understanding of the basic disease process
bilitated immunosuppressed adults, and in infants causing the gingival reaction.
and adults who have been on antibiotic therapy for It consists of two phases:
long. a. Local treatment
l Oral lesions may involve any part of oral mucosa. b. Systemic treatment
They appear as creamy white adherent patches, Local treatment
which bleed when removed forcibly. i. Improvement of oral hygiene is essential to reduce
l Candidiasis is an oral manifestation of AIDS. the gingival inflammation.
l On the basis of clinical manifestations, four types l Oral hygiene instructions.

have been described as follows: l Patient must be instructed to use soft toothbrush for

i. Pseudo membranous routine plaque control. Care should be taken not to


ii. Atrophic traumatize the gingival tissue during brushing.
iii. Hyperplastic l Oxidizing mouthwashes, e.g. H2O2 (3%) diluted.

iv. Epidermal and perioral type ii. Hydrogen peroxide (3%) diluted with two parts of
warm water may be used as mouthwash twice daily.
i. The pseudo membranous type appears as white,
l Topical corticosteroid ointment or cream may be
curd-like plaque on the mucosa with erythema
used for several times a day. Before application,
and ulceration. The organisms tend to invade the
gingiva may be gently dried with sterile sponge,
epithelium.
e.g. triamcinolone acetonide (0.1%), fluocinonide
ii. The atrophic type is usually seen on the dorsum
(0.05%) or desonide (0.05%).
of the tongue with erythema and papillary
Systemic treatment
atrophy.
l In severe cases, systemic administration of cortico-
iii. The hyperplastic type is characterized by hyper-
steroids may be considered. It should be started only
keratosis of the epithelium with white plaques
after evaluation of the general health of the patient
that are difficult to remove.
and by physician’s consent.
iv. The epidermal and perioral type show scaly
l Systemic corticosteroids in moderate doses.
patches at the angle of the mouth.
l Mucous membrane pemphigoid responds favourably
Diagnosis
to systemic steroid therapy. Prednisone 30–40 mg
l It can be confirmed by examining the scrapings
daily or on alternate days to begin with and gradually
for the spores and mycelia of C. albicans.
reduced to 5–10 mg daily or on alternate days as a
Treatment
maintenance dose may be used.
l Antimycotic drugs like nystatin or clotrima-
l Nutritional supplements may also be recommended.
zole are quite effective in controlling the in-
l It takes a long time for the lesions to heal; hence the
fection.
patient should be told not to discontinue the treat-
Q.3. Describe the aetiopathogenesis, histopathology, ment before the lesions heal and to patiently wait for
clinical features and treatment of chronic desquamative the results.
gingivitis. Q.2. Mucous membrane pemphigoid.
Ans. Ans.
[Same as LE Q.1] l Mucous membrane pemphigoid, also known as cicatri-
Q.4. What is desquamative gingivitis? Give the clinical cial pemphigoid, is a group of putative autoimmune,
and histopathological features and treatment of severe chronic inflammatory, subepithelial blistering diseases
form of desquamative gingivitis. predominantly affecting mucous membrane with or
without clinically observable scarring.
Ans. l It predominantly affects women in the fifth decade of

[Same as LE Q.1] life. It has been reported rarely in young children.


l Mucous membrane pemphigoid is characterized by an

immune reaction involving autoantibodies directed


SHORT ESSAYS: against basement membrane zone, followed by comple-
Q.1. Treatment of chronic desquamative gingivitis. ment activation and subsequent leukocyte recruitment.
Proteolytic enzymes release and dissolve the basement
Ans. membrane zone.
868 Quick Review Series for BDS 4th Year, Vol 1

l Cicatricial pemphigoid involves the oral cavity conjunc- Oral hygiene should be improved.
l

tiva and mucosa of the nose, vagina, rectum, oesopha- Regular brushing with soft brushes and use of
l

gus and urethra. hydrogen peroxide mouthwash.


l Oral lesions are characterized by erosive or desquama-
Q.3. Define and classify chronic desquamative gingivitis
tive gingivitis. Vesicles and ulcerations also may be seen
lesions.
on the gingiva. Attached gingiva is erythematous. Bul-
lae rupture in about 2–3 days forming irregular-shaped Ans.
ulcers. Healing of the ulcers is generally delayed and
[Ref LE Q.2]
takes up to 3 weeks.
Histopathology Q.4. Management of chronic desquamative gingivitis.
l The lesions show subepithelial vesiculation.
Ans.
l Epithelium is separated from the connective tissue

at the region of basement membrane. Under elec- [Same as SE Q.1]


tron microscope, the basement membrane shows
a split.
l Inflammatory infiltration is also present.
SHORT NOTES:
Treatment
Q.1. Define chronic desquamative gingivitis.
l Mucous membrane pemphigoid generally treated

with systemic corticosteroids. Ans.


l Topical application of corticosteroids has only
[Ref LE Q.1]
limited value.

Topic 19
Gingival and Periodontal Diseases in Children
and Young Adolescents
COMMONLY ASKED QUESTIONS
LONG ESSAYS:
1. Classify gingival and periodontal diseases in children. Discuss in detail the gingival diseases in children.

SHORT ESSAYS:
1 . Physiologic gingival changes associated with tooth eruption.
2. Prepubertal periodontitis.

SHORT NOTES:
1 . Hypophosphatasia.
2. Papillon–Lefevre syndrome.
3. Aetiology of gingival recession.
4. Linear gingival erythema.
5. Chediak–Higashi syndrome.
6. Eruption cyst.
7. Localized gingival recession. [Same as SN Q.3]
Section | I  Topic-Wise Solved Questions of Previous Years 869

SOLVED ANSWERS
LONG ESSAYS:
Q.1. Classify gingival and periodontal diseases in chil- l Management: No treatment is required; it is
dren. Discuss in detail the gingival diseases in children. self-limiting and includes improving oral hy-
giene. Only severe cases may require antibiotic
Ans. therapy.
In 1999, the American Academy of Periodontology con- b. Gingivitis associated with poor oral hygiene
ducted an International Workshop for the classification of l It can be grouped as early, moderate and ad-

periodontal diseases and conditions, which resulted in a vanced gingivitis.


new classification. l Early gingivitis is quickly reversible and

Gingival and periodontal diseases in children are treated with good tooth brushing and flossing.
classified as follows: l Moderate and severe gingivitis requires more

I. Classification of gingival diseases elaborate measures.


i. Simple gingivitis l Oral hygiene and gingivitis are directly related.

l Eruption gingivitis Adequate oral hygiene practice has beneficial


l Gingivitis associated with poor oral hygiene effect on reducing gingivitis.
ii. Acute gingival inflammation ii. Acute gingival inflammation
l Herpes simplex virus (HSV) infection a. HSV infection
l Recurrent aphthous ulcer b. Recurrent aphthous ulcer
l Acute necrotizing ulcerative gingivitis (ANUG) c. ANUG
l Acute candidiasis d. Acute candidiasis
l Acute bacterial infections e. Acute bacterial infections
iii. Chronic marginal nonspecific gingivitis a. Gingivitis associated with HSV 1 infection
l Plaque induced
l It is caused by the HSV.
iv. Conditioned gingival enlargement l Herpetic stomatitis is a common oral disease
l Puberty gingivitis
which develops in both children and young
l Fibromatosis
adults.
l Phenytoin-induced gingival overgrowth
l It rarely occurs before the age of 6 months, ap-
v. Scorbutic gingivitis parently because of the presence of circulating
II. Classification of periodontal diseases antibodies in the infant, derived from the
i. Prepubertal periodontitis mother. The disease occurring in children is
l Localized early-onset periodontitis
frequently the primary attack.
l Generalized early-onset periodontitis
l It is characterized by the development of fever,
ii. Periodontitis associated with systemic diseases irritability, headache, pain upon swallowing
l Papillon–Lefevre syndrome
and regional lymphadenopathy.
l Ehlers–Danlos syndrome
l Within a few days, the mouth becomes painful
l Hypophosphatasia
and the gingiva is intensely inflamed. The lips,
l Chediak–Higashi syndrome
tongue, buccal mucosa, palate, pharynx and
l Leukocyte adhesion deficiency
tonsils may also be involved.
l Neutropenia
l Shortly yellowish, fluid-filled vesicles develop.
l Down syndrome
These rupture and form shallow, ragged, ex-
i. Simple gingivitis tremely painful ulcers covered by a grey mem-
a. Eruption gingivitis brane and surrounded by an erythematous halo.
b. Gingivitis associated with poor oral hygiene l They heal spontaneously within 7–14 days and

leave no scar.
a. Eruption gingivitis l Treatment is symptomatic such as application
l It is seen during the eruption of teeth and sub-
of topical anaesthetic agents on the ulcers, soft
sides without any treatment soon after eruption. diet and adequate fluids.
l More likely seen in 6- to 7-year-old patients
b. Gingivitis associated with recurrent aphthous
when permanent teeth begin to erupt. stomatitis
l It may be painful and develop into pericoroni-
l Aphthous stomatitis is triggered due to any
tis or pericoronal abscess. kind of stress, gastrointestinal disturbance,
870 Quick Review Series for BDS 4th Year, Vol 1

nutritional deficiency, hormonal imbalance, l It may be localized or may be more generalized.


infection, allergy, etc. l It persists for a longer period of time but is rarely
l Prodromal phase of paraesthesia at the site of ulcer- painful.
ation is observed. The ulcers appear in crops of two l Treatment is limited to maintaining the oral hygiene

or three and are less than 10 mm in diameter. They and regular professional prophylaxis.
are painful and discrete or confluent. iv. Conditioned gingival enlargement
l It persists for about 10–12 days and heals with no a. Puberty gingivitis
scar formation. b. Fibromatosis
l Treatment is symptomatic. Chlorhexidine or ny- c. Drug-induced gingival overgrowth
statin or tetracycline can be prescribed if they are
infected. a. Puberty gingivitis
c. ANUG l It occurs in the prepubertal and pubertal periods

l ANUG is caused by Borrelia vincentii and spiro- in young children.


chaetes. l Gingival inflammation is confined to the anterior

l Severe ulcerating gingivitis involving the inter- segment and may be limited to single arch only.
proximal papillae, covered by a pseudo-membrane, l Gingiva on the lingual aspect is relatively unin-

is the characteristic. volved.


l It is associated with fetid odour. l Resolves with maintenance of adequate oral hy-

l After penicillin therapy and application of hydro- giene, removal of local irritants and restoration
gen peroxide, recovery promptly occurs within of caries.
36 h. b. Hereditary fibromatosis gingival enlargement
d. Acute candidiasis l It is characterized by slow, progressive and be-

l It is the most common mycotic infection of the nign enlargement of the gingiva.
oral mucosa caused by Candida albicans. l The surface is normal appearing, but fibrous

l It is seen in three types of individuals – debilitated tending to displace the teeth and is nonpainful.
or immunosuppressed adults, infants and adults l It appears as soon as the deciduous teeth erupt in

who have been on antibiotic therapy for a long the oral cavity and covers the teeth completely.
time. l It regresses only if the teeth are extracted.

l Oral lesions are described as four clinical types: l Although surgical excision is the treatment of

a. Pseudo-membranous type: white curd-like choice, it has tendency to recur.


plaques c. Drug-induced hyperplasia
b. Atrophic type: usually seen on the dorsum l Phenytoin, an anticonvulsant drug, is the com-

of the tongue with erythema and papillary mon cause of hyperplasia of the gingiva.
atrophy Other drugs like cyclosporine and nifedipine
c. Hyperplastic type: hyperkeratosis of the epi- also induce hyperplasia of gingiva.
thelium with white plaques l It is asymptomatic, i.e. painless enlargement of
d. Epidermal and perioral type: scaling patches at the gingiva, especially on the interproximal as-
the corner of the lips pect.
Treatment l Gingiva appears pink and firm unless infected.

Current treatment is the use of antimycotic l Buccal and anterior segment are more often af-

agent like cotrimoxazole in the form of oral fected than the lingual and posterior segment.
troches every 3 h for a week to 10 days. l There is a formation of pseudo-pockets.

e. Gingivitis associated with acute bacterial infection l Management includes adequate oral hygiene
l It is caused by streptococci group of bacteria. maintenance, change of drug or dosage and sur-
l The gingiva is painful and bleeds easily. gical excision.
l Papilla is enlarged with associated gingival v. Scorbutic gingivitis
abscess. l Aetiology: Vitamin C deficiency.

l Treatment includes broad-spectrum antibiotics, l Distribution: Limited to marginal tissue and papil-

improvement of oral hygiene and chlorhexidine lae.


mouthwash. l It is associated with severe pain and spontaneous

iii. Chronic marginal nonspecific gingivitis haemorrhage.


l It has no specific aetiology, may be triggered by l Management includes ascorbic acid supplements and

hormonal imbalance. oral hygiene maintenance.


Section | I  Topic-Wise Solved Questions of Previous Years 871

SHORT ESSAYS: molars. By the age of 15 years, most of the permanent


teeth are lost.
Q.1. Physiologic gingival changes associated with tooth l Prepubertal periodontal disease may occur as:
eruption. i. A generalized form
Ans. ii. A localized form
l Normally, it is generalized but when it occurs in decidu-
l Significant changes occur in the periodontium as the ous dentition, it is localized.
dentition changes from the deciduous to the permanent Generalized form
teeth. Most of the changes are associated with eruption l It is usually associated with systemic conditions
and are physiologic in nature. like Papillon–Lefevre syndrome, Down syn-
l Physiologic gingival changes that occur during the
drome, neutropenias, Chediak–Higashi syndrome,
eruption of tooth can be studied in three stages as hypophosphatasia, leukaemias and leukocyte ad-
follows: hesion deficiency.
a. Pre-eruptive stage l Clinical features of generalized prepubertal peri-
b. Eruptive stage odontitis include involvement of primary and
c. Posteruptive stage permanent dentition, rapid destruction of alveolar
a. Pre-eruptive stage bone, severe inflammation of the gingiva along
l Before eruption of the permanent tooth crown, the with gingival proliferation, involvement of all the
gingiva presents a bulge that is firm and pink, may teeth except third molars, tendency for the disease
be slightly blanched and conforms to the contour to be refractory to antibiotic therapy, frequent re-
of the underlying crown. spiratory infections and destruction of the roots of
b. Eruptive stage teeth.
l This stage is associated with formation of gingival l Normally, children do not present alveolar bone

margin. resorption characteristic of periodontitis. They


l As the tooth erupts, the gingival margin and sul- may at the most have gingivitis, especially during
cus develop. At this point, the margin is rounded, puberty.
oedematous and reddened. Localized form
l During the period of active tooth eruption, it is l The localized form is not associated with any of

normal for the marginal gingiva surrounding par- the syndromes.


tially erupted teeth to appear prominent. This is l In the localized form, the periodontal destruction

most evident in the maxillary anterior region. occurs in relation to a single tooth or a group of
l The prominence is caused by the height from the teeth.
contour of the erupting tooth and the mild inflam- l Plaque accumulation is minimum. Gingival in-

mation from mastication. flammation may be present. Patients do not have


c. Posteruptive stage the history of frequent infections.
l The gingiva reduces in bulk and becomes more l Periodontal destruction is not as rapid as in the

thin, tight and firmly attached around the cervical generalized form.
portion of the tooth. Histology
l Prepubertal periodontitis shows an absence of
Q.2. Prepubertal periodontitis. neutrophils in the gingiva but numerous lym-
Ans. phoid cells, especially plasma cells, are pres-
ent. Severe alveolar bone loss may occur in the
l Prepubertal periodontitis has its onset early in life and absence of neutrophils.
affects deciduous as well as permanent dentition. l Actinobacillus actinomycetemcomitans, Bacte-
l Prepubertal periodontitis is rather rare. There is an ab- roides, and Fusobacterium are found in the
normality in chemotaxis of either neutrophils or mono- gingival pocket.
cytes or both. Treatment
l It generally begins by the age of 4 years or it is seen l Management includes early diagnosis, den-
soon after the eruption of the primary teeth. tal curettage, prophylaxis, removal of se-
l Premature loss of teeth is common, and all primary verely mobile teeth and broad-spectrum an-
teeth may be lost by 3 years of age. The permanent teeth tibiotics.
then erupt normally but within few years destructive l Lesions generally respond to mechanical ther-
periodontal disease affects all the teeth except the third apy along with systemic antibiotics.
872 Quick Review Series for BDS 4th Year, Vol 1

SHORT NOTES: l The position of the tooth in the arch is the most impor-

You might also like